You are on page 1of 1092

MATHEMATICS-XII (Volume I) CONTENTS

1. RELATIONS 1.1-1.34

2. FUNCTIONS 2.1-2.80

3. BINARY OPERATIONS 3.1-3.40

w
4. INVERSE TRIGONOMETRIC FUNCTIONS 4.1-4.126

5. ALGEBRA OF MATRICES
Flo 5.1-5.71

ee
Fr
6. DETERMINANTS 6.1-6.100
or
ur
7. ADJOINT AND INVERSE OF A MATRIX 7.1-7.40
sf

8. SOLUTION OF SIMULTANEOUS LINEAR EQUATIONS 8.1-8.23


k
Yo
oo
B

9. CONTINUITY 9.1-9.48
re

10. DIFFERENTIABILITY 10.1-10.20


ou
ad
Y

11. DIFFERENTIATION 11.1-11.125


nd
Re

12. HIGHER ORDER DERIVATIVES 12.1-12.25


Fi

13. DERIVATIVE AS A RATE MEASURER 13.1-13.26

14. DIFFERENTIALS, ERRORS AND APPROXIMATIONS 14.1-14.14

15. MEAN VALUE THEOREMS 15.1-15.20

16. TANGENTS AND NORMALS 16.1-16.44

1' INCREASING AND DECREASING FUNCTIONS 17.1-17.42

18. MAXIMA AND MINIMA 18.1-18.85

19. INDEFINITE INTEGRALS 19.1-19.208

ReadYourFlow.COM
CHAPTER ]
RELATION

1.1 INTRODUCTION
In Class XI, we have introduced the notion of a relation, its domain, co-domain and range. Let us
recall that a relation from a set A to a set B is a subset of A x B. If B is a relation from a set A to a set
B and (a, b) e R, then we say that a is related to b under relation R and we write as a R b. If

low
(a, b) <£ R, then we say that a is not related to b under R and we write as aRb. A relation can be
represented in roster form or tabular form. Sometimes, we also describe a relation by describing
the common property between the elements of ordered pairs in it. For example, a relation R on
the set A = {1, 2, 3, 4, 5} defined by R = {(a, b) :b = a + 2} can also be expressed as: a R b if and

ee
only ifb=a + 2.
rF
Fr
In this chapter, we will study different types of relations.

1.2 RECAPITULATION
or
In this section, we will recall some definitions which have been discussed in the earlier class.
sf
u
CARTESIAN PRODUCT OF SETS Let A and B be two non-empty sets. The set of all ordered pairs (a, b)
k
Yo

such that a eA and b eBis called the cartesian product of set A with set B and is denoted by Ax B.
oo

Thus, Ax B = {(a, b) :a eA and b e B}


B

Note that Bx A = {(b, a) :b e B and a eA}


re

Also, AxB = <|), if A=<t> orB = <j>


ou

If A = {1, 2, 3} and B = {x, y], then


ad

A x B = {(1, x), (by), (2, x), (2, y), (3, x), (3, y)}
Y

B x A = {(x, 1), (x, 2), (x, 3), (y, 1), (y, 2), (y, 3)}
nd

A x A = {(1,1), (1,2), (1, 3), (2,1), (2, 2), (2, 3), (3,1), (3, 2), (3, 3)}
Re

and, B x B = {(x, x), (x, y), (y, x), (y, y)}


Fi

RELATION Let A and B be two sets. Then a relation R from set A to set B is a subset of A x B.
Thus, R is a relation from AtoB<^>R^AxB.
If B is a relation from a non-void set A to a non-void set B and if (a, b) e R, then we write aRb
which is read as "a is related tofr by the relation B". If (tf, b) <z R, then we write aRb and we say
that a is not related to b by the relation B.
If A and B are finite sets consisting of m and n elements respectively, then Ax B has mn ordered
pairs. Therefore, total number of relations from A to B is 2mn.
DOMAIN Let Rbea relation from a set A to a set B. Then the set of allfirst components or coordinates of
the ordered pairs belonging to R is called the domain ofR.
Thus, domain of B = {a: (a, b) e R}
Clearly, domain of B c A.
If A = {1, 3,5, 7}, B = {2, 4, 6, 8,10} and B = {(1, 8), (3, 6), (5, 2), (1, 4)} is a relation from A to B,
then
Domain (B) = {1, 3,5}

ReadYourFlow.COM
1.2 MATHEMATICS-XII

RANGE Let Rbea relation from a set A to a set B. Then the set of all second components or coordinates of
the ordered pairs belonging to R is called the range of R.
Thus, Range of R = {b :(a,b) e R}.
Clearly, range of R c B.
If A = {1,3,5, 7}, B = {2, 4, 6, 8,10} and R = {(1, 8), (3, 6), (5, 2), (1, 4)} is a relation from A to B,
then
Range (R) = {8, 6, 2,4}
RELATION ON A SET Let A be a non-void set. Then a relation from A to itself i.e. a subset of Ax A is
called a relation on set A.
INVERSE OF A RELATION Let A, B be two sets and let Rbea relation from a set A to a set B. Then, the
inverse of R, denoted by R ~ \ is a relation from B to A and is defined by
R" 1 = {(b, a) :(a, b)eR).

low
Clearly, (a, b) eR o (b, a) eR'1.
Also, Domain (R) = Range (R_ 2) and. Range (R) = Domain (R~ l)
Let A = {1, 2, 3}, B = {a, b, c, d}be two sets and R = {(1, a), (1, c), (2, d), (2, c)} be a relation from A

ee
to B. Then, R~ 1 = {(a, 1), (c, 1), (d, 2), (c, 2)} is a relation from B to A.
rF
Fr
1.3 TYPES OF RELATIONS or
In this section, we intend to discuss various types of relations on a set A.
sf
u
1.3.1 VOID, UNIVERSAL AND IDENTITY RELATIONS
k

VOID RELATION Let A be a set. Then, <j> e A x A and so it is a relation on A. This relation is called the
Yo
oo

void or empty relation on set A.


B

In other words, a relation R on a set A is called void or empty relation, if no element of A is


related to any element of A.
re

Consider the relation R on the set A = {1, 2, 3, 4,5} defined by R = {(a, b): a -b = 12}.
ou
ad

We observe that a - & ^ 12 for any two elements of A.


Y

(a, b) <£ R for any a,b eA.


=> R does not contain any element of A x A
nd
Re

=> R is empty set


Fi

=> R is the void relation on A.


UNIVERSAL RELATION Let A be a set. Then, Ax Ac: Ax A and so it is a relation on A. This relation is
called the universal relation on A.
In other words, a relation R on a set is called universal relation, if each element of A is related to
every element of A.
Consider the relation R on the set A = {1, 2, 3, 4, 5, 6} defined by R = {(a, b) e R : \ a - b \ > 0}.
We observe that
| fl - b | > 0 for all a, b e A
=> (a, b) eR for all (a,b) eAx A
=> Each element of set A is related to every element of set A
=> R =A x A
=> R is universal relation on set A
NOTE It is to note here that the void relation and the universal relation on a set A are respectively the
smallest and the largest relations on set A. Both the empty (or void) relation and the universal relation are
sometimes called trivial relations.

k
ReadYourFlow.COM
RELATIONS 1.3

ILLUSTRATION Let A be the set of all students of a boys school. Show that the relation Ron A given by
R = {(a, b): a is sister ofb) is empty relation and R' = {(a, b): the difference between the heights of a and b
is less than 5 meters} is the universal relation.
SOLUTION Since the school is boys school. Therefore, no student of the school can be sister of
any student of the school. Thus,
{a, b) &R for any a,b eA.
Hence, R = cfii.e. R is the empty or void relation on A.
It is obvious that the difference between the heights of any two students of the school has to be
less than 5 meters.
(a, b) eR for all a,b eA.
=> R=Ax A
=> R is the universal relation on set A.
IDENTITY RELATION Let A be a set. Then, the relation IA = {(a, a) :a e A} on A is called the identity

low
relation on A.
In other words, a relation on A is called the identity relation if every element of A is related to
itself only.

ee
If A = {1, 2, 3}, then the relation IA = {(1,1), (2, 2), (3, 3)} is the identity relation on set A. But,
rF
Fr
relations R^ - {(1,1), (2, 2)} and R2 = {(1,1), (2, 2), (3, 3), (1, 3)} are not identity relations on A,
because (3, 3) e R-[ and in R2 element 1 is related to elements 1 and 3. or
1.3.2 REFLEXIVE RELATION
sf
u
DEFINITION A relation R on a set A is said to be reflexive if every element of A is related to itself.
k
Yo

Thus, R is reflexive <=> (a, a) e R for all a eA.


oo

A relation Ron a set A is not reflexive if there exists an element a e A such that (a, a) <£R.
B

ILLUSTRATION 1 Let A = {1, 2, 3) be a set. Then R = {(1,1), (2, 2), (3, 3), (1, 3), (2,1)} is a reflexive
re

relation on A. But, R^ = ((1,1), (3, 3), (2,1), (3, 2)} is not a reflexive relation on A, because 2 e A but
(2,2)*%
ou
ad

ILLUSTRATION 2 The identity relation on a non-void set A is always reflexive relation on A. However,
Y

a reflexive relation on A is not necessarily the identity relation on A. For example, the relation R-{(a, a),
nd

(b, b), (c, c), (a, b)} is a reflexive relation on set A= {a, b, c} but it is not the identity relation on A.
Re

ILLUSTRATION 3 The universal relation on a non-void set A is reflexive.


Fi

ILLUSTRATION 4 A relation R on N defined by (x, y) e R <=> x >y is a reflexive relation on N,


because every natural number is greater than or equal to itself
ILLUSTRATION 5 Let Xbea non-void set and P(X) be the power set ofX. A relation R on P(X) defined
by (A, B) <= R o A ^ B is a reflexive relation since every set is subset of itself.
ILLUSTRATION 6 Let L be the set of all lines in a plane. Then relation R on L defined by
(f, l2) e R <=> f is parallel to l2 is reflexive, since every line is parallel to itself.
1.3.3 SYMMETRIC RELATION
DEFINITION A relation R on a set A is said to be a symmetric relation iff
(a, b) eR => (b, a) eR for all a,b e A
i.e. aRb => bRa for all a,b eA.
ILLUSTRATION l The identity and the universal relations on a non-void set are symmetric relations.
ILLUSTRATION 2 Let L be the set of all lines in a plane and let Rbe a relation defined on L by the rule
(*, y) e R <=> x is perpendicular to y. Then, R is a symmetric relation on L, because L-j 1 L2 => L2 1 Lj
i.e. (Lj, L2) e R => (L2, L{) e R.

ReadYourFlow.COM
1.4 MATHEMATICS-XII

ILLUSTRATION 3 Let S be a non-void set ami R be a relation defined on power set P(S) by(A, B) <=R
<=> A e Bfor all A,B e P(S). Then, R is not a symmetric relation.
NOTE A relation R on a set A is not a symmetric relation if there are at least two elements a,b e A such
that (a, b) eR but (b, a) <2 R.
ILLUSTRATION 4 Let A = {1,2,3,4j and let R^ and be relations on A given by = {(1,3), (1,4),
(3, 1), (2, 2) (4, 1)} and R2 = {(1, 1), (2, 2), (3, 3), (1, 3)}. Clearly, R-^ is a symmetric relation on A.
However, R2 is not so, because (1, 3) e R2 but (3,1) 0 R2.
NOTE A reflexive relation on a set A is not necessarily symmetric. For example, the relation R = {(1,1),
(2, 2), (3, 3), (1, 3)} is a reflexive relation on set A = {1, 2, 3} but it is not symmetric.
ILLUSTRATION 5 Prove that a relation R on a set A is symmetric iffR =R~1.
SOLUTION First, let R be a symmetric relation on set A. Then, we have to prove that R = R_1. In
order to prove this we have to prove that R c R_1 and R-1 c R.

w
Now, (a, b) eR
=> (b, a) e R [v R is symmetric]
(a, b) eR -l
=>
Tims, {a, b) eR
Flo
{a, b) e R-1 for all a,b eA.
[By def. of inverse relation]

ee
R ^R -1

Fr
So,
Now, let (x, y) be an arbitrary element of R-1. Then, or
ur
(*, y) e R -l
sf

=> (y, x) e R [By def. of inverse relation]


k
Yo

=> (x, y) e R [v R is symmetric]


oo

Thus, (x, y) e R-1 => (x, y) eR for all x, y e A.


B

So, R-1 c R ...(ii)


re

Thus, from (i) and (ii), we get R = R-1.


ou
ad
Y

Conversely, let R be a relation on set A such that R =R-1. Then we have to prove that R is a
symmetric relation on set A. Let {a, b) e R. Then,
nd
Re

{a, b) eR
(b, a) eR -1
Fi

=> [by def. of inverse relation]


=> (b, a) eR [■.• R = R-1]
Thus, {a, b) e R => (b, a) e R for alla,b e A.
So, R is a symmetric relation on A. Hence, R is symmetric iff R = R_1.
1.3.4 TRANSITIVE RELATION
DEFINITION Let A be any set. A relation R on A is said to be a transitive relation iff
{a, b) eR and {b, c) e R =^> {a, c) e R for all a,b, c e A.
i.e.. aRb and bRc => aRc for all a,b c e A.
ILLUSTRATION l The identity and the universal relations on a non-void set are transitive.
ILLUSTRATION 2 The relation R on the set N of all natural numbers defined by
(x, y) e R <=> x divides y, for all x, y e N is transitive.
SOLUTION Let x, y, 2 s N be such that (x, y) e R and (y, z) e R. Then,
(x, y) eR and (y,2) e R
=> x divides y and, y divides 2

ReadYourFlow.COM
RELATIONS 1.5

=> There exist p, q eN such that y = xp and z = ijq


=> z = (xp) q
=> z = x (pq)
=> x divides z [•• pqzN]
=> (x, z) e R
Thus, (x, y) e K, (y, z) e R => (x, z) e R for all x, y,z eN.
Hence, R is a transitive relation on N.
ILLUSTRATION 3 On the set N of natural numbers, the relation R defined by xRy => x is less than y is
transitive, because for any x, y,z eN
x < y and y <z =5- x <z i.e., xRy and yRz => xRz
ILLUSTRATION 4 Let S be a non-void set and Rbe a relation defined on power set P(S) by (A, B) eR
o Ac Bfor all A, B e P(S). Then, R is a transitive relation on P(S), becausefor any A, B,C e P(S)
=> (A, B) eR and (B,C) eR => Ac B and BcC=>AcC=> (A, C) eR

w
ILLUSTRATION 5 Let L be the set ofall straight lines in a plane. Then the relation "is parallel to " on L is
a transitive relation, because for any Z2,1% e L.
h\\h and h\\h ^ h! I h
ILLUSTRATION 6
Flo
The relation "is congruent to" on the set T of all triangles in a plane is a transitive

ee
relation.

Fr
1.3.5 ANTISYMMETRIC RELATION
DEFINITION Let A be any set. A relation R on set A is said to be an antisymmetric relation iff
for
ur
(a, b) eR and (b, a) e R=> a = b for all a,b e A
NOTE It follows from this definition that if (a, b) e R but (b, a) g R, then also R is an antisymmetric
ks

relation.
Yo
oo

ILLUSTRATION l The identity relation on a set A is an antisymmetric relation.


ILLUSTRATION 2 The universal relation on a set A containing at least two elements is not
eB

antisymmetric, because ifa*b are in A, then a is related to b and b is related to a under the universal
relation will imply that a = b but a^b.
r

ILLUSTRATION 3 Let R be a relation on the set N of natural numbers defined by


ou
ad

xRy <=> 'x divides y' for all x, y eN


Y

This relation is an antisymmetric relation on N. Since for any two numbers a,b eN.
nd
Re

a | b and b \ a => a = b i.e. aRb and bRa => a = b


It should be noted that this relation is not antisymmetric on the set Z of integers, because wefind that for
Fi

any non-zero integer a, aR (-a) and (-a) R a but a* - a.


ILLUSTRATION 4 Let S be a non-void set and R be a relation on the power set P(S) defined by
(A, B) e R o Ac B for all A, B e P(S)
Then, R is an antisymmetric relation on P(S), because
(A, B) eR and (B, A) eR => Ac B and Be A=> A - B
ILLUSTRATION 5 The relation < ("less than or equal to") on the set R of real numbers is antisymmetric,
because a <b and b <a => a = b for all a, b eR.
ILLUSTRATIVE EXAMPLES
LEVEL-1
EXAMPLE 1 Three relations Ry, R2 and R3 are defined on set A = [a,b,c} as follows:
(i) Ri ={(a, a), (a, b), (a, c), (b, b), (b, c), (c, a), (c, b), (c, c)},
(ii) R2 = {(a, b), (b, a), (a, c), (c, a)}
(hi) R3 = {(a,b),(b,c),(c,a)}.
Find whether each o/R1, R2 and R3 is reflexive, symmetric and transitive.

ReadYourFlow.COM
1.6 MATHEMATICS-XII

SOLUTION (i) Reflexive: Clearly (a, a), {b, b), {c, c) e Rj. So, is reflexive on A.
Symmetric: We observe that (a, b) e R-j but (b, a) g So, R j is not a symmetric relation on A.
Transitive: We find that (b, c) e Rj and (c, a) e R^ but (b, a) <£ Rj. So, R-j is not a transitive
relation on A.
(ii) Reflexive: Since (a, a), (b, b) and (c, c) are not in R2. So, it is not a reflexive relation on A.
Symmetric: We find that the ordered pairs obtained by interchanging the components of
ordered pairs in R2 are also in R2. So, R2 is a symmetric relation on A.
Transitive: Clearly (a, b) e R2 and (b, a) e R2 but (a, a) g R2.
So, it is not a transitive relation on R2.
(iii) Reflexive: Since none of (<7, fr)and(c, c) is an element of R3. So, R3 is not reflexive on A
Symmetric: Clearly, (b, c) e R3 but (c, t) g R3. So, R3 is not a symmetric relation on A.
Transitive: Clearly, {a, b) e R3 and {b, c) e R3 but (a, c) g R3. So, R3 is not a transitive relation

low
on A.
EXAMPLE 2 Show that the relation R on the set A = {1,2,3} given by
R - {(1,1), (2, 2), (3, 3), (1, 2), (2, 3)} is reflexive but rieither symmetric nor transitive.
[NCERT]

ee
SOLUTION Since 1, 2, 3 e A and (1,1), (2, 2), (3, 3) e R i.e. for each a eA, (a, a) e R. So, R is
rF
Fr
reflexive.
We observe that (1, 2) e R but (2,1) g R. So, R is not symmetric.or
Also, (1, 2) e R and (2, 3) e R but (1, 3) g R. So, R is not transitive.
sf
u
EXAMPLE 3 Show that the relation R on the set A = {1, 2, 3}given by R = {(1, 2), (2,1)} is symmetric
but neither reflexive nor transitive. [NCERT]
k
Yo
oo

SOLUTION We observe that (1,1), (2, 2) and (3, 3) do not belong to R. So, R is not reflexive.
B

Clearly, (1,2) e R and (2,1) e R. So, R is symmetric.


re

As (1, 2) e R and (2,1) e R but (1,1) g R. So, R is not transitive.


EXAMPLE 4 Check the following relations R and S for reflexivity, symmetry and transitivity:
ou
ad

(i) aRb iff b is divisible by a, a, b eN


Y

(ii) /| S /2 iffli _L Z2, where l^ and /2 are straight lines in a plane.


nd
Re

SOLUTION (i) We have, [NCERT]


aRb a\b for all a,b eN.
Fi

Reflexivity: For any a eNrwe have


a \ a => aRa.
Thus, aRa for all a eN. So, R is reflexive on N.
Symmetry: R is not symmetric because ifa\b, then b may not divide a. For example, 21 6 but 6\2.
Transitivity: Let a, b, c eN such that aRb and bRc. Then,
aRb and bRc => a\b and b \ c => a| c => aRc.
So, R is a transitive relation on N.
(ii) Let L be the set of all lines in a plane. We are given that
/| S /2 <=> /} _L /2 for all /j, /2 e L.
Reflexivity: S is not reflexive because a line cannot be perpendicular to itself i.e. / _L / is not true.
Symmetry: Let f, I2 e L such that f S l2. Then,
/l S /2 => /-j -L /2 => /2 -L /i => /2 S /}.
So, S is symmetric on L.
Transitive: S is not transitive, because ^ 1 l2 and l2 _L /3 does not imply that /x _L /3.

L
ReadYourFlow.COM
RELATIONS 1.7

EXAMPLE 5 Let a relation R-y on the set R of real numbers be defined as (a, b) e R-yO l + ab > Ofor all
a,b e R. Show that R-y is reflexive and symmetric but not transitive.
SOLUTION We observe the following properties:
Reflexivity: Let a be an arbitrary element of R. Then,
a eR
=> l+a.a=l+a2>0 [v a2 >0 for all fl e R]
=> (a, a) e Ry [By definition of Ry]
Thus, (a, a) e Ry for alia e R. So, Ry is reflexive on R.
Symmetry: Let(a,b) e R. Then,
(a, b)eRy
1 + ab >0
=> 1 +ba>0 [v ab = ba for all a, b e R]

w
=> (b, a) e Ry [By definition of Ry]
Thus, (a, b) e => (b, a) e Ry for all a, b e R. So, Ry is symmetric on R.

Flo
Transitivity: We observe that (l,l/2)eR1 and (1/2,-l)sR1 but (1, -1) g Rj because
1 + 1 x (-1) = 0 O.So,Ry is not transitive on R.

ee
EXAMPLE 6 Determine whether each of thefollowing relations are reflexive, symmetric and transitive:

Fr
(i) Relation R on the set A = {1, 2, 3,... ,13,14} defined as R = {(x, y): 3x -1/ = 0}
(ii) Relation R on the set N of all natural numbers defined as R = {(x, y):y = x + 5 and x < 4}
or
ur
(iii) Relation R on the set A = {1, 2, 3, 4,5, 6} defined as R = {(x, y): y is divisible by x}
sf
(iv) Relation R on the set Z of all integer defined asR = {(x, y): x - y is an integer}
k
Yo

SOLUTION (i) R = {(x, y): 3x - y = 0}, where x, y e A = {1, 2, 3,..., 13,14} [NCERT]
oo

R = {(1/ 3), (2, 6), (3, 9), (4,12)}


B

Reflexivity: Clearly, (1,1) g R. So, R is not a reflexive relation on A.


re

Symmetry: We observe that (1, 3) eR but (3,1) g R. So, R is not a symmetric relation A.
ou

Transitivity: We observe that (1, 3) eR and (3, 9) e R but (1, 9) g R. So, R is not a transitive
ad

relation A.
Y

(ii) R = {(x, y): y = x + 5 and x < 4}, where x, i/ e N.


R4l,6),(2,7),(3,8)}
nd
Re

Reflexivity. Clearly, (1,1), (2, 2) etc. are not in R. So, R is not reflexive.
Fi

Symmetry: We find that (1, 6) eR but (6,1) g R. So, R is not symmetric.


Transitivity: Since (1, 6) e R and there is no order pair in R which has 6 as the first element. Same
is the case for (2, 7) and (3, 8). So, R is transitive.
(iii) R = {(x, y): y is divisible by x}, where x, y e A = {1, 2, 3, 4,5, 6}.
Reflexivity: We know that
x is divisible by x for all x e A
(x, x) e R for all x e A
=> R is reflexive on set A.
Symmetry: We observe that 6 is divisible by 2 but 2 is not divisible by 6. This means that
(2, 6) e R but (6, 2) g R.
So, R is not symmetric on set A.
Transitivity: Let (x, y) e R and (y, z) e R. Then,
(x, y) e R and (y, z) e R.
=> y is divisible by x and, 2 is divisible by y
=> 2 is divisible by x

ReadYourFlow.COM
1.8 MATHEMATICS-XII

=> (x, z) e R
So, R is transitive relation on A.
(iv) R = {(x, y):x - y is an integer}, where x,y & Z
Reflexivity: We have,
x -x = 0, which is an integer for all x e Z.
=> (x, x) e R for all x s Z
=> R is reflexive on Z.
Symmetry: Let (x, y) eR. Then,
(x, y) eR
=> x -1/ is an integer, say, A.
=> y-x = -X
=> y - x is an integer [v XeZ=> -XeZ]
=> (y, x) e R

low
Thus, (x, y) e R => (y, x) e R for all x, y e Z.
So, R is symmetric on Z.
Transivity: Let (x, y) eR and (y, z) e R. Then,
(x, y) eR and (y,z) eR

ee
=> x-y and y -2 are integers
rF
Fr
=> (x - y) + (y - z) is an integer [v Sum of two integers is an integer]
=> x -z is an integer or
=> (x, z) e R
sf
u
So, R is transitive on Z.
k
Yo

EXAMPLE 7 Show that the relation R on R defined as R = {(a, b):a < b), is reflexive and transitive but
oo

not symmetric. [NCERT]


B

SOLUTION We have, R = {(a, b):a< b}, where a,b eR


re

Reflexivity: For any fl e R


a < a
ou
ad

=> (fl, a) eR for all fl e R


Y

=> R is reflexive.
nd

Symmetry: We observe that (2, 3) e R but (3, 2) g R. So, R is not symmetric.


Re

Transitivity: Let (a, b) eR and (b, c) e R. Then,


Fi

(a, b) eR and (b, c) eR


=> a < b and b < c
=> a < c
=> (a, c) eR
So, R is transitive.
EXAMPLE 8 Let S be the set of all points in a plane and R be a relation on S defined as
R = {(P, Q): Distance between P and Q is less than 2 units}.
Show that R is reflexive and symmetric but not transitive.
SOLUTION We observe the following properties of relation R:
Reflexivity: For any point P in set S, we find that
Distance between P and itself is 0 which is less than 2 units.
=> (P, P) e R
Thus, (P, P) e R for all P e S.
So, R is reflexive on S.

ReadYourFlow.COM
RELATIONS 1.9

Symmetry: Let P and Q be two points in S such that (P, Q) e R. Then,


{P'Q)*K
=> Distance between P and Q is less than 2 units.
=> Distance between Q and P is less than 2 units
=> (Q, P)eR
So, R is symmetric on S.
Transitivity: Consider points P, Q and R having coordinates (0, 0), (1.5, 0) and (3.2, 0). We
observe that the distance between P and Q is 1.5 units which is less than 2 units and the distance
between Q and R is 1.7 units which is also less than 2 units. But, the distance between P and R is
3.2 which is not less than 2 units. This means that (P, Q) eR and (Q, R) eRbut(P, R) gR.So,R
is not transitive on S.
LEVEL-2
EXAMPLE 9 Letx = {1, 2, 3, 4,5, 6, 7, 8, 9}. Let R-j be a relation on X given by R1 = {(x, y) :x-y

low
is divisible by 3} and R2 be another relation on X given by R2 = {(x, y): {x, y\ c {1, 4,7} or
{x, y} c {2,5, 8}or {x, y} cz {3, 6, 9}}. Shoiv that R-^ = R2.
[NCERT1
SOLUTION Clearly, R^ and R2 are subsets of X x X. In order to prove that Rj =R2, it is

ee
sufficient to show that R^ c R2 and R2 c R^
rF
Fr
We observe that the difference between any two elements of each of the sets {1, 4,7}, {2,5, 8 J and
{3, 6, 9} is a multiple of 3. or
Let (x, y) be an arbitrary element of R^ Then,
sf
u
(x, y) e R]
k

=> x - y is divisible by 3.
Yo
oo

x - y is a multiple of 3.
B

=> {x, y} c (1, 4,7} or {x, y} c {2,5, 8} or {x, y} c {3, 6, 9}


re

=> (x, y) e R2
Thus, (x, y) e Rx => (x, y) e R2.
ou
ad

So, R^ c R2 ...(i)
Y

Now, let {a, b) be an arbitrary element of R2. Then,


nd

{a, b) g R2
Re

{a, &} c {1, 4, 7} or {c, b} e {2,5, 8} or {fl, 1?} e {3, 6, 9}


Fi

=> a- bis divisible by 3


=> (a, b) gR1
Thus, (a, b) eR2 => {a, b) g R1
So, R2 c R1 ...(ii)
From (i) and (ii), we get: Ri = R2
EXAMPLE 10 Show that the relation R on the set R ofall real numbers, defined as R - {{a, b):a <b } is
neither reflexive nor symmetric nor transitive. [NCERT1
SOLUTION We have, R = {{a, b): a <b2}, where a, bsR.

i < m2
Reflexivity: We observe that —
1 1
I — I is not true. Therefore, — G R.
2'2
So, R is not reflexive.
Symmetry: We observe that -1 < 32 but 3 ^ (-1)2 i.e. (-1, 3) g R but(3,-1) G R.
So, R is not symmetric.

ReadYourFlow.COM
1.10 MATHEMATICS-XII

Transivity: We observe that


2 <(- 3)2 and -3<l2but2^12 i.e. (2, - 3) e R and (- 3, -1) e K but(2,1) g R.
So, K is not transitive.
EXAMPLE 11 Let A = {1, 2, 3}. Then, show that the number of relations containing (1, 2) and (2, 3)
which are reflexive and transitive but not symmetric is three. [NCERT]
SOLUTION The smallest reflexive relation on set A containing (1, 2) and (2, 3) is
K={(M),(2, 2), (3, 3), (1,2), (2, 3)}
Since(l, 2) e R and(2, 3) e Rbut(l, 3) g R.So,R is not transitive. To make it transitive we have
to include (1, 3) in R. Including (1, 3) in R, we get
R1 = {(1,1), (2, 2), (3, 3), (1, 2), (2, 3), (1, 3)}
This is reflexive and transitive but not symmetric as (1, 3) e R^ but (3,1) ^ R1.
Now, if we add the pair (2, 1) to Rl to get R2 ={(1,1), (2, 2), (3, 3), (1, 2), (2, 3), (1, 3), (2,1)).

w
The relation R2 is reflexive and transitive but not symmetric. Similarly, by adding (3,2) and (3,1)
respectively to Ry we get

Flo
R3 = {(1,1), (2, 2), (3, 3), (1, 2), (2, 3), (1, 3),(3, 2)},
R3 = {(1,1), (2, 2), (3, 3), (1, 2), (2, 3), (1, 3), (3,1), (3, 2)}

ee
These relations are reflexive and transitive but not symmetric.

Fr
We observe that out of ordered pairs (2,1), (3, 2) and (3,1) at a time if we add any two ordered
pairs at a time to R-j, then to maintain the transitivity we will be forced to add the remaining
or
ur
third pair and in this process the relation will become symmetric also which is not required.
sf
Hence, the total number of reflexive, transitive but not symmetric relations containing (1,2) and
(2, 3) is three.
k
Yo
oo

EXERCISE 1.1
B

LEVEL-1
re

1. Let A be the set of all human beings in a town at a particular time. Determine whether
ou

each of the following relations are reflexive, symmetric and transitive:


ad

(i) R = {(x, y): x and y work at the same place }


Y

(ii) R = {{x, y): x and y live in the same locality)


nd

(iii) R = {(x, y): x is wife of y}


Re

(iv) R = {(x, y): x is father of y} [NCERT]


Fi

2. Relations Rly R2 , R3 and R4 are defined on a set A = {a, b, c} as follows:


= {{a,a)/(a,b)l{afc)/(b/b)/(b,c)/(c,a)l(c/b)r(c/c)}
K2 = {(fl/fl)}
R3 = {(b,c)}
R4 = {(a,b),(b, €),(€, a)}.
Find whether or not each of the relations R 1' R2, R3, R4 on A is (i) reflexive (ii)
symmetric (iii) transitive.
3. Test whether the following relations R-j, R2, and R3 are (i) reflexive (ii) symmetric and
(iii) transitive:
(i) R-l onQo defined by {a,b) e Rl <^> a =l/b
(ii) R2 on Z defined by(a,b)eR2<^>\a-b\<5
(iii) R3 on R defined by(a,b)eR3<=>a2 -4 ab + 3b2 = 0.
4. Let A = {1,2,3(, and let Ra = {(1,1), (1,3), (3,1), (2,2), (2,1), (3,3)), R2 = {(2,2), (3,1), (1,3)},
R3 = {(1,3), (3,3)). Find whether or not each of the relations R-j, R2, R3 on A is (i) reflexive
(ii) symmetric (iii) transitive.

ReadYourFlow.COM
RELATIONS

5. The following relations are defined on the set of real numbers:


(i) aRb if« - b > 0 (ii) aRb iff 1 + flb > 0 (iii) aRb if | rt | <b.
Find whether these relations are reflexive, symmetric or transitive.
6. Check whether the relation R defined on the set A = {1, 2, 3, 4,5, 6}as R = {(a, b) :b = a + 1}
is reflexive, symmetric or transitive. [NCERT]
o
7. Check whether the relation R on R defined by R = {(a, b): n <b }is reflexive, symmetric or
transitive. [NCERT, CBSE 2010]
8. Prove that every identity relation on a set is reflexive, but the converse is not necessarily
true.
9. If A = { 1, 2, 3, 4}, define relations on A which have properties of being
(i) reflexive, transitive but not symmetric.
(ii) symmetric but neither reflexive nor transitive.
(iii) reflexive, symmetric and transitive. [NCERT EXEMPLAR]

low
10. Let R be a relation defined on the set of natural numbers N as
R = {{x, y): x, y eNr2x + y = 41}
Find the domain and range of R. Also, verify whether R is (i) reflexive, (ii) symmetric
(iii) transitive. [CBSE 2014]
11. Is it true that every relation which is symmetric and transitive is also reflexive? Give

ee
reasons.
rF
Fr
12. An integer m is said to be related to another integer n if m is a multiple of n. Check if the
relation is symmetric, reflexive and transitive.
for
13. Show that the relation">" on the set R of all real numbers is reflexive and transitive but not
u
symmetric.
ks

14. Give an example of a relation which is


Yo
oo

(i) reflexive and symmetric but not transitive.


B

(ii) reflexive and transitive but not symmetric.


(iii) symmetric and transitive but not reflexive.
re

(iv) symmetric but neither reflexive nor transitive.


ou
ad

(v) transitive but neither reflexive nor symmetric. [NCERT]


Y

LEVEL-2
nd
Re

15. Given the relation R = {(1, 2), (2, 3)1 on the set A = [1, 2, 3), add a minimum number
Fi

ordered pairs so that the enlarged relation is symmetric, transitive and reflexive.
16. Let A = {1, 2, 3) and R = {(1, 2), (1,1), (2, 3)} be a relation on A. What minimum number of
ordered pairs may be added to R so that it may become a transitive relation on A.
17. Let A = {a, b, c] and the relation R be defined on A as follows: R = {(<?, a), (b, c), (a, b)}. Then,
write minimum number of ordered pairs to be added in R to make it reflexive and
transitive. [NCERT EXEMPLAR]
18. Each of the following defines a relation on N:
(i) x>y,x,yeN (ii) x + y = 10, x, y e N
(iii) xy is square of an integer, x,y sN (iv) x + 4i/ = 10, x, y € N
Determine which of the above relations are reflexive, symmetric and transitive.
[NCERT EXEMPLAR]
ANSWERS
1. (i) Reflexive, symmetric and transitive
(ii) Reflexive, symmetric and transitive
(iii) Neither reflexive, nor symmetric but transitive

ReadYourFlow.COM
MATHEMATICS-XII

(iv) neither reflexive nor symmetric nor transitive


2. R-i is reflexive but neither symmetric nor transitive.
is symmetric and transitive but not reflexive.
#3 is transitive but neither reflexive nor symmetric.
R4 is neither reflexive nor symmetric nor transitive.
3. (i) Ri is symmetric but it is neither reflexive nor transitive
(ii) R2 is reflexive and symmetric but it is not transitive
(hi) R 3 is reflexive but it is neither symmetric nor transitive.
4. Ri is reflexive but neither symmetric nor transitive
R2 is symmetric but neither reflexive nor transitive.
R3 is transitive but neither reflexive nor symmetric.
5. (i) Transitive (ii) Reflexive and symmetric but not transitive (iii) Transitive neither
reflexive nor symmetric.
6. Neither reflexive nor symmetric nor transitive

w
7. Neither reflexive nor symmetric nor transitive.
9. (i) R = {(1,1), (2, 2), (3, 3), (4, 4), (1, 2)) (ii) R = {(1,2), (2,1)}

Flo
(iii) R = { (1,1), (2, 2), (3, 3), (4, 4), (1, 2), (2,1)}
10. Domain R = (1, 2, 3,19, 20), Range R = {39, 37, 35,..., 7, 5, 3,1).

ee
R is neither reflexive nor symmetric and is not transitive.

Fr
11. No. Relation R = {(1,1), (1, 2), (2,1), (2, 2)} on = {1, 2, 3) is symmetric and transitive but
not reflexive.
for
ur
12. Reflexive and transitive but not symmetric.
14. (i) R = {(1,1), (2, 2), (3, 3), (1, 3), (3,1) (2,3), (3,2)) on A = {1,2, 3}
ks

(ii) R = Kb 1), (2, 2), (3, 3), (1, 3)) on A = {1, 2, 3)


Yo
oo

(iii) R = {(1, 3), (3,1), (1,1), (3, 3)) on A = {1,2,3}


eB

(iv) R = {(1,1), (1, 3), (3,1), (2, 3), (3, 2)) on A = {1, 2, 3) (v) R = {(1,1)| on A = {1, 2, 3)
15. (1,1), (2, 2), (3, 3), (1, 3), (2,1), (3, 2), (3,1) 16. (1, 3), One 17. (b, b), (c, c), (a, c)
r

18. (i) transitive (ii) symmetric (iii) reflexive, symmetric and transitive (iv) transitive.
ou
ad

__________________________________ HINTS TO NCERT& SELECTED PROBLEMS


Y

1. (iv) The relation R on the set A of all human beings in a town is given by (x, y) e R iff x is
nd

father of y.
Re

Reflexivity: Since a person x cannot be father of himself. So, (x, x) g A. Consequently, R is


Fi

not reflexive.
Symmetry. Let x, y e A be such that (x, y) e R. Then,
(x, y) e R
=> x is father of y
=> y cannot be father of x
=> (y, x) g R
So, R is not symmetric.
Transitivity: Let x, y, 2 e A be such that (x, y) e R and (y, z) e R. Then,
(x, y) e R and (y, 2) e R
=> x is father of y and y is father of 2
=> x is grandfather of 2
=> (x, z) g R
6. The relation R on set A = {1, 2, 3, 4,5, 6} is defined as (a, fr) g R iff b = a + 1. Therefore,
R = {(1, 2), (2, 3), (3, 4), (4,5), (5, 6)}.

ReadYourFlow.COM
RELATIONS 1.13

Clearly, (a, a) &R for any a &A. So, R is not reflective on A.


We observe that (1,2) eR but (2,1) g R. So, R is not symmetric.
We also observe that(1, 2) eR and (2, 3) e R but(l, 3) g R. So, R is not transitive.
7_ The relation R on R is defined by R = {(a, b):a<b3}
We observe that (-2) e R is such that (-2) < (-2)3 is not true. So, R is not reflexive.
Since 1 ^(S1^3)'7 but 31/3 £1 i.e.(l, 31,/3) e Rbut (S1^3,1) ^ R. So, R is not symmetric.
R is not transitive because (5, 2) e R and (2, 21 / 3) eR but (5, 23// 3) ^ R.
8. Let I be the identity relation on a set A. Then,
(a, a) el for all e A => / is reflexive.
Converse: The relation ((1, 1), (2, 2), (3, 3), (1,3)} is a reflexive relation on set A = |1,2,3}
but it is not the identity relation on A.
11. A relation R on the set Z of integers defined by (a,b) eR o a and b are both odd, is

low
symmetric and transitive but it is not reflexive. Because no even integer is related to itself.
16. For reflexivity, we must add (1,1), (2, 2) and (3,3). For symmetry and transitivity we must
add (2,1), (3,2), (1,3), (3,1) in R.

ee
1.3.5 EQUIVALENCE RELATION
rF
Fr
DEFINITION A relation R on a set A is said to be an equivalence relation on A iff it is
(i) reflexive i.e. (a, a) eRfor alia e A. or
(ii) symmetric i.e. (a, b) eR^> (b, a) e Rfor all a,b eA.
and, (Hi) transitive i.e. (a, b) e R and (b, c) eR=> (a, c) e Rfor all a,b, c e A.
sf
u
An equivalence relation R defined on a set A partitions the set A into pair wise disjoint subsets.
k
Yo

These subsets are called equivalence classes determined by relation R.The set of all elements of
oo

A related to an element a e A is denoted by [fl] i.e. [rt] = {x eA: (x, a) e R). This is an equivalence
eB

class. Corresponding to every element in A there is an equivalence class. Any two equivalence
classes are either identical or disjoint. The collection of all equivalence classes forms a partition
r

of set A.
ou
ad
Y

ILLUSTRATIVE EXAMPLES
LEVEL-1
nd
Re

EXAMPLE i Let Rbe a relation on the set of all lines in a plane defined by (/-j , /2) e R <=> line f is
Fi

parallel to line l2. Show that R is an equivalence relation.


SOLUTION Let L be the given set of all lines in a plane. Then, we observe the following
properties.
Reflexive: For each line / e L, we have
/||/ => (/,/) eR forall/eL
=> R is reflexive
Symmetric: Let f, l2 e L such that (f, l2) eR. Then,
(h' h) e ^ W ^2 ^ h W h ^ (h' h) s
So, R is symmetric on L.
Transitive: Let l1, l2, l3 eL such that (Zj, l2) e R and (l2, If) e R. Then,
(/•j, l2) e R and (Z2, /g) eR => f || Z2 and Z2 IU3 => ^ II ^3 => ih' h) e ^
So, R is transitive on L.
Hence, R being reflexive, symmetric and transitive is an equivalence relation on L.

ReadYourFlow.COM
r 1.14 MATHEMATICS-XII

EXAMPLE 2 Shozu that the relation 'is congruent to' on the set ofall triangles in a plane is an equivalence
relation.
SOLUTION Let S be the set of all triangles in a plane and let R be the relation on S defined by
(A-], A2) e R <=> triangle Aj is congruent to triangle A2.
We observe the following properties of relation R\
Reflexivity: For each triangle A e S, we have
A = A => (A, A) e K for all A e S => R is reflexive on S
Symmetry. Let A-j, A2 e S such that (Aj, A2) e R. Then,
(A-j, A2) e R A^ = A2 => A2 s A^ => (A2, A^) e R.
So, R is symmetric on S
Transitivity: Let Alx A2, A3 e S sucli tliat (Alf A2) e R and (A2, A3) e R. Tlien,
(Aj, A2) e R and (A2, A3) e R => Aa = A2 and A2 = A3 => Aj = A3 => (A2, A3) e R
So, R is transitive on S.

w
Hence, R being reflexive, symmetric and transitive, is an equivalence relation on S.
EXAMPLE 3 Show that the relation R defined on the set A of all triangles in a plane asR = {(Tj, T2): Tj is
similar toT2}is an equivalence relation.

Flo
ee
Consider three right angle triangles Tj zvith sides 3,4,5; T2 with sides 5,12,13 andT^ zvith sides 6,8,10.
Which triangles amongT-^, T2 andT3 are related? [NCERT]

Fr
SOLUTION We observe the following properties of relation R. or
Reflexivity: We know that every triangle is similar to itself.
ur
(T,T) eR forallTeA
sf

=> R is reflexive.
k
Yo

Symmetry. Let (Tlr T2) e R. Then,


oo

(TvT2) eR
B

=> Ti is similar to T2
re

=> T2 is similar to Tj
ou
ad

=> (T2, T{) eR


Y

So, R is symmetric.
Transitivity: Let T1, T2, T3 e A such that (Tt, T2) e R and (T2, T3) e R. Then,
nd
Re

(Tj, T2) e R and (T2,T3)eR


Fi

=> Ti is similar to T2 and T2 is similar to T3


=> Tj is similar to T3
=> (Jl, T3) e R
So, R is transitive.
Hence, R is an equivalence relation on set A.
In triangles Tj and T3, we observe that the corresponding angles are equal and the
corresponding sides are proportional i.e. ^ Hence, Tj and T3 are related.

EXAMPLE 4 Let n be a positive integer. Prove that the relation R on the set Z of all integers numbers
defined by(x, y) e R <=> x-y is divisible by n, is an equivalence relation on Z.
SOLUTION We observe the following properties of relation R. [NCERT EXEMPLAR1
Reflexivity: For any a eN
a - a = 0 = Oxn
a -ais divisible by n
=> {a, a) <eR

ReadYourFlow.COM
RELATIONS 1.15

Thus, (a, a) eR for all a e Z. So, R is reflexive on Z


Symmetry: Let {a, b) eR. Then,
(a, b) eR
(a - b) is divisible by n
=> {(i -b) = np for somep eZ
b -a = n{-p)
=> b - a is divisible by n [v peZ=>-peZ]
=> (b, a) eR
Thus, (a, b) e R=> (b, a) e R for all a,b eZ.
So, R is symmetric on Z.
Transitivity: Leta,b, c eZ such that (a,b) eR and(b, c) e R. Then,
(a, b) eR

w
=> (a - b) is divisible by n
=> a-b =np for some p eZ
and.
=>
(b, c) eR
(b - c) is divisible by n
Flo
ee
=> b - c = nq for some q eZ

Fr
{a, b) eR and(b, c) eR
=> a- b = np and b - c = nq
or
ur
(a -b) + (b - c) = np + nq
sf

=> a-c = n(p + q)


k
Yo

=> a - c is divisible by n [••• p,qeZ^>p + qeZ]


oo

=> (a, c) eR
B

Thus, (a, b) eR and (b, c) e R=> (a, c) e R for all a,b,c eZ.
re

So, R is transitive relation on Z.


ou
ad

Thus, R being reflexive, symmetric and transitive, is an equivalence relation on Z.


Y

REMARK In the above example, if we take n = 2, then R can be described as


(x, y) e R <^> x-y is divisible by 2
nd
Re

Clearly, R is an equivalence relation on Z.


Fi

Let us now find the equivalence classes determined by R.


[0] ={x e Z : (x, 0) e R] =[x eZ : x — 0 is divisible by 2} = {x e Z : a: is divisible by 2}
= {0, ± 2, ± 4, ± 6,
[1] = {x e Z : (x, 1) e R} = {x e Z : x -1 is divisible by 2}
=> [l] = {xeZ:x-l =2X, XeZ]
=> [l] = [x eZ:x = 2X + l,XeZ)
=> [1] = {±1,± 3, ±5, ±5, ±7,...)
[2] = {x e Z : (x, 2) e R} = {x e Z : x - 2 is divisible by 2}
=> [2] = {x e Z :x-2 = 2X, XeZ) ={xeZ :x = 2 + 2X, XeZ)
=> [2] = |0,± 2, ± 4, ± 6,.. which is same as the equivalence class [01
[3] = {x e Z : (x, 3) e R} = {x e Z : x - 3 is divisible by 2}
=> [3] = {x e Z : x - 3 = 2X, XeZ) ={x eZ :x = 3 + 2X, XeZ]
=> [3] = (± 1, ± 3, ± 5 ± 7,..which is sam as the equivalence class [1]
Continuing in this manner, we find that
[0] = [2] = [4] = [6] =...

ReadYourFlow.COM
r 1.16 MATHEMATICS-XII

[1] =[3] = [5] = [7] =...


and. [0] n [1] = (jx Also, Z = [0] u [1].
Thus, R partitions the set Z into two pair wise disjoint sets known as equivalence classes.
Similarly, the relation R onZ given by
(x, y) <e R <» x -1/ is divisible by 3
partitions Z into 3 pair wise disjoints sets i.e. equivalence classes given by
[0] = {-6,-3,0, 3,6,9,..j
[1] = {•••/ -8, -5, -2,1, 4,7,10,.. J
[2] = {...,7,-4,-1,2,5, 8,11,..j
such that Z = [0] u [1] u [2].
EXAMPLE 5 Show that the relation R on the set A of all the books in a library of a college given by
R = {(x, y): x and y have the same number of pages}, is an equivalence relation.
SOLUTION We observe the following properties of relation R. [NCERT]

low
Reflexivity: For any book x in set A, we observe that
x and x have the same number of pages.
(x, x) e R
Thus, (x, x) e R for all x e A.

ee
So, R is reflexive.
rF
Fr
Symmetry. Let (x, y) e R. Then,
(x, y) e R
or
=> x and y have the same number of pages
sf
u
y and x have the same number of pages
k
Yo

=> (y, x) e R
oo

Thus, (x, y) e R => (y, x) e R


B

So, R is symmetric.
re

Transitivity: Let (x, y) e R and (y, z) e R. Then,


ou
ad

(x, y) e R and (y, z) e R


Y

(x and y have the same number of pages) and (y and z have the same number of
pages)
nd
Re

=> x and z have the same number of pages.


Fi

=> (x, z) e R
So, R is transitive.
Thus, R is reflexive, symmetric and transitive.
Hence, R is an equivalence relation.
EXAMPLE 6 Shozo that the relation R on the set A = {1, 2, 3, 4,5}, given by
R = {(a,b): \ a - b\ is even }, is an equivalence relation.
Show that all the elements of {l, 3, 5| are related to each other and all the elements of {2,4} are related to
each other. But, no element of{l, 3,5} is related to any element of [2,4).
SOLUTION We have, [NCERT, CBSE 2009]
R = {(fl, fr) :| a-fr | is even}, where a,b e A = {1,2, 3, 4,5}.
We observe the following properties of relation R.
Reflexivity: For any a e A, we have
\a -a\ = 0, which is even
(a, a) eR for all a e A
So, R is reflexive.

ReadYourFlow.COM
RELATIONS 1.17

Symmetry: Let (a, b) e R. Then,


(a, b) eR
=> | a - fr | is even
=> | & - | is even
=> (b, a) eR
Thus, (a,b) eR => (b, a) eR
So, R is symmetric.
Transitivity: Let (a, b) e R and (b, c) eR. Then,
(a,b)eR and (b,c)eR
=> | | is even and | & - c | is even
=> (a and b both are even or both are odd) and (b and c both are even or both are odd)
Now two cases arise:

low
CASE I When b is even
In this case.
(a, b) eR and (b, c) e R
=> | fl - b | is even and | fr - c | is even

ee
=> a is even and c is even [v b is even]
rF
Fr
=> | a - c | is even
=?- (a, c) eR or
CASE II When b is odd
sf
u
In this case,
(a, b) eR and (br c) eR
k
Yo

=> | a - b | is even and 1- c | is even


oo

=> a is odd and c is odd [v b is odd]


B

=> | « - c | is even
re

(a, c) e R
ou

Thus, (a, b) e R and (b, c) eR => (a, c) eR


ad

So, R is transitive.
Y

Hence, R is an equivalence relation.


nd
Re

We know that the difference of any two odd (even) natural numbers is always an even natural
number. Therefore, all the elements of set {1, 3, 5} are related to each other and all the elements
Fi

of {2, 4} are related to each other.


We know that the difference of an even natural number and an odd natural number is an odd
natural number. Therefore, no element of {1, 3, 5} is related to any element of {2, 4}.
EXAMPLE 7 Show that the relation R on the set A = {x e Z : 0 < x < 12}, given by R = {(a, b) : \ a - b\
is a multiple of 4} is an equivalence relation. Find the set of all elements related to 1 i.e. equivalence
class [1]. [NCERT, CBSE 2010]
SOLUTION We have,
R = {{a, b) : \ a - b\is a multiple of 4], where a, b e A = {x e Z : 0 < x <12} = {0,1, 2,..., 12}.
We observe the following properties of relation R.
Reflexivity: For any a e A, we have
| rt - a | = 0, which is a multiple of 4.
=> (a, a) eR
Thus, (a, a) e R for all a eA.
So, R is reflexive.

ReadYourFlow.COM
r
1.18 MATHEMATICS-XII

Symmetry: Let (a, b) e R. Then,


(a, b) eR
=> | a - & | is a multiple of 4
=> \n - b\ = 4A. for some X e N
=> \b - a \ = 4.X for some XeN [v \a-b\ = \b-a\]
(b, a) eR
So, R is symmetric.
Transitivity: Let (a, b) e R and (b, c) e R. Then,
(a, b) e R and (b, c) e R
=> | - ft | is a multiple of 4 and | b - c | is a multiple of 4
=> \a - b\ = 4X and \b -c\ = 4p for some X,\j.eN
a-b = ± 4X and b-c = ± 4p
=> a — c = ± 4X ± 4p

low
=> « - c is a multiple of 4
=> | (? - c | is a multiple of 4
=> {a, c) e R
Thus, {a, b) e R and (b, c) eR => (a, c) eR

ee
So, R is transitive.
rF
Fr
Hence, R is an equivalence relation.
Let x be an element of A such that (.t, 1) e R. Then,
for
| x -11 is a multiple of 4
ou
=> l*-l| = 0,4,8,12
ks

=> x-1 = 0,4,8,12


oo

=> * = 1,5,9 [v 13 eA]


Y
B

Hence, the set of all elements of A which are related to 1 is {1, 5, 9} i.e. [1] = {1,5, 9}.
re

EXAMPLE 8 Show that the relation R on the set A of points in a plane, given by
ou
ad

R={(P,Q)'- Distance of the point P from the origin is same as the distance of the point Qfrom
the origin},
Y

is an equivalence relation. Further show that the set of all points related to a point P * (0, 0) is the circle
nd
Re

passing through P with origin as centre. [NCERT]


Fi

SOLUTION Let O denote the origin in the given plane. Then,


R={(P,Q):OP=OQ)
We observe the following properties of relation R.
Reflexivity: For any point P in set A, we have
OP = OP
=> (P, P) eR
Thus, (R, P) e R for all P e /i
So, R is reflexive.
Symmetry: Let P and Q be two points in set A such that
(P,Q)eR
=> OP = OQ
=> OQ = OP
=> (Q, P) e R
Thus, (P, Q) e R => (Q, P) e R for all P, Q e
So, R is symmetric.

ReadYourFlow.COM
RELATIONS 1.19

Transitivity: Let P, Q and S be three points in set A such that


(P,Q) e R and (Q, S) g R
OP =OQ and OQ=OS
=> OP=OS
=> {P,S)eR
Thus, (P,Q) e R and (Q, S) g R => (P, S) g R for all P,Q, S eA
So, R is transitive.
Hence, R is an equivalence relation.
Let P be a fixed point in set A and Q be any point in set A such that (P, Q) g R. Then,
(P,Q)eR
=> OP = OQ
=> Q moves in the plane in such a way that its distance from the origin
O (0, 0) is always same and is equal to OP.

low
=> Locus of Q is a circle with centre at the origin and radius OP.
Hence, the set of all points related to P is the circle passing through P with origin O as centre.
LEVEL-2

ee
EXAMPLE 9 Prove that the relation R on the set N x N defined by
rF
Fr
(a,b) R (c, d) <z> a + d =b + c for all (a, b), (c, d) eN x N
is an equivalence relation. or
Also, find the equivalence classes [(2, 3)]and[{l, 3)].
[CBSE 2010]
sf
u
SOLUTION We observe the following properties of relation R.
Reflexivity: Let (a, b) be an arbitrary element of N x N. Then,
k
Yo
oo

(a, b) eN x N
B

=> a,b eN
re

=> a+ b = b + a [By commutativity of addition on N]


=> (a, b) R (a, b)
ou
ad

Thus, (a, b) R (a, b) for all (a, b) eN x N. So, R is reflexive on N x N.


Y

Symmetry: Let (a, b), (c, d) eN x N be such that (a, b) R (c, d). Then,
(a, b) R (c, d)
nd
Re

a + d =b + c
Fi

=> c +b =d + a [By commutativity of addition on N]


=> (c, d) R (a, b) [By definition of R]
Thus, {a, b) R(c,d)=> (c, d) R (a, b) for all (a, b), (c, d) eN xN.
So, R is symmetric on N x N.
Transitivity: Let (a, b), (c, d), (e,f) eN xN such that (a, b) R (c, d) and (c, d) R (e, /). Then,
(a, b) R (c, d) => a + d = b + c
=> (a + d) + (c + f) = (b + c) + (d + e)
(c, d) P (^/ /) => c + / = d + e
=> a+f=b + e=> (a,b) R (e, f)
Thus, {a, b) R (c, d) and (c, d) R (e, f) => (a, b) R (e, f) for all (a, b), (c, d), (e, f) eN xN.
So, R is transitive onN x N.
Hence, R being reflexive, symmetric and transitive, is an equivalence relation on N x N.
[(2, 3)] = {(x, y) eN xN : (x, y) R (2, 3)}
=> [(2, 3)] = {(.v, y)eNxN:x+3=y + 2} = [{x, y) eNxN :x-y =1}
= {(x, y) eNxN :y = x +1}

ReadYourFlow.COM
1.20 MATHEMATICS-XM

= [(x,x + 1):xgN}
= {(1/ 2), (2, 3), (3, 4), (4,5),..J
[(7, 3)] = {(.r, y) eNxN : {x, y) R (7, 3))
= {(x, i/) eNxN:x+3 = i/ + 7}
= {(x, y) e Nx N : y = x -^}
= {(x, x - 4) eNxN :x eN]
= {(5,1), (6, 2), (7, 3), (8, 4), (9, 5),..|
EXAMPLE 10 Let A ={1, 2, 3,..., 9) and R be the relation on Ax A defined by (a,b) R (c, d) if
a + d = b + c for all {a,b), (c, d) e Ax A. Prove that R is an equivalence relation and also obtain the
equivalence class [(2,5)]. [NCERT EXEMPLAR, CBSE 2014]
SOLUTION We observe the following properties of relation R.
Reflexivity: Let (a, b) be an arbitrary element ot Ax A. Then,
(a,b) e A x A

w
=> a,b e A
=> a + b = b +a [By commutativity of addition on N]
=> (a, b) R (a, b)

Flo
Thus, (a, b) R (a, b) for all (a, b) e Ax A. So, R is reflexive on A x A

ee
Symmetry: Let {a, b), (c, d) eAx A be such that (a, b) R (c, d). Then,

Fr
{a, b) R (c, d)
=> a+d=b+c
for
ur
=> c+b=d+a [By commutativity of addition on N]
(c, d) R (a, b)
ks

Thus, (a, b) R (c, d) => (c, d) R (a, b) for all (a, b), (c, d) eAx A.
Yo
oo

So, R is symmetric on A x A
eB

Transitivity: Let (a, b), (c, d), (e,f) eAx A such that (a, b) R (c, d) and (c, d) R (e, /). Then,
(a, b) R (c, d) => a + d = b + c
r

=> {a + d)+(c+f) = {b + c)+(d + e)


ou

(c, d) R (e,f)=> c+f = d + e


ad
Y

=> a+f =b + e
nd

=>(«, b) R (e,f)
Re

Thus, (a, b) R (c, d) and (c, d) R (e, f) => {a, b) R (e, f) for all {a, b), (c, d), (e, f) eAx A.
Fi

So, R is a transitive relation on A x A.


Hence, R is an equivalence relation on Ax A.
Now,
[(2,5)] = {(x, y) e A x A : (x, y) R (2,5)}
= {(x, y) e A x A : x + 5 = y + 2} = {(x, y) e A x A : y = x + 3}
= {(x, x + 3): x e A and x + 3 e A] = {(1, 4), (2,5), (3, 6), (4,7), (5, 8), (6, 9)}
EXAMPLE ll Let N be the set of all natural numbers and let R be a relation on N x N, defined by
(a, b) R (c, d) o ad = be for all {a, b), (c, d) eN x N.
Show that R is an equivalence relation on N x N. Also, find the equivalence class [(2, 6)].
SOLUTION We observe the following properties of relation R.
Reflexivity: Let (a, b) be an arbitrary element of N x N. Then,
(a,b) eN x N
=> a,b eN
=> ab -ba [By commutativity of multiplication on N]

ReadYourFlow.COM
RELATIONS 1.21

=> (a, b) R (a, b)


Thus, (a, b) R (a, b) for all {a, b) eN x N.
So, R is reflexive onN x N.
Symmetry: Let (a, b), (c, d) e N x jV be such that {a, b) R (c, d). Then,
(a,b)R(c,d)
ad =bc
=> cb = da [By commutativity of multiplication on N]
=> (c, d) R (a, b)
Thus, (a, b) R (c, d)=> (c, d) R (a, b) for all {a, b), (c, d) <=N x N.
So, R is symmetric on N x TV.
Transitivity: Let (a, b), (c, d), {e, f) eN x N such that (a, b) R (c, d) and (c, d) R (e, /). Then,
(a, b) R (c, d) => ad - be

w
=> {ad) {cf) = {be) {de) => af = be => {a, b) R {e, f)
(c, d) R (e, f)=> cf = de
Thus, (a, b) R (c, d) and (c, d) R (e, /) => (a, b) R (e, f) for all {a, b), (c, d), (e, f) eN x N.
So, R is transitive on TV x N.
Flo
ee
Hence, R being reflexive, symmetric and transitive, is an equivalence relation on N x TV.

Fr
[(2, 6)] = {(*, y)eNxN:(x,y)R(2f 6)}
= [(x,y)eNxN:3x = y\
for
ur
= {(*, 3x):x eN] ={(1, 3), (2, 6), (3, 9), (4,12),.. J
EXAMPLE 12 Let N denote the set of all natural numbers and R be the relation on N x N defined by
ks

(a, b) R (c, d) <=> ad (b + c) = be (a + if). Check whether R is an equivalence relation on N x N.


Yo
oo

SOLUTION We observe the following properties of relation K. [CBSE2015]


eB

Reflexivity: Let (a, b) be an arbitrary element of N x TV. Then,


{a, b) eN x N
r
ou

=> a,b eN
ad

=> ab (b + a) = ba (a + b) [By commutativity of addition and multiplication on TV]


Y

=> (a, b) R (a, b)


nd
Re

Thus, {a, b) R {a, b) for all (a, b) eN x N. So, R is reflexive on N x N.


Fi

Symmetry:Let (a, b), (c, d) eN x N be such that (a, b) R (c, d). Then,
(a,b)R(c,d)
=> ad {b + c) - bc(a + d)
=> cb {d + a) = da(c + b) [By commutativity of addition and multiplication on N]
(c, d) R {a, b)
Thus, (a, b) R (c, d) => (c, d) R (a, b) for all (a, b), (c, d) eN x N.
So, R is symmetric on N x TV.
Transitivity: Let (a, b), (c, d), (e, f) eN xN such that (a, b) R (c, d) and (c, d) R (e, /). Then,
b+c a+d 1 1 1 1
{a, b) R {c, d) ad{b + c) - bc{a + d) => ------ - ------ =>_ + _ = + _
be ad bead
d+e c+f 11 11
and. (c, d) R {e, f)=> cf{d + e) = de{c + f) => ---- - => - + - = - + - ...(ii)
de cf d e c f
Adding (i) and (ii), we get

ReadYourFlow.COM
1.22 MATHEMATICS-XII

1 1 1 1 1 1
b c d e = |-+-|+U+7
a d
1 1 1 1 b+e = il/
=> - +- - + — => —■—- => af (b + e) =be(a + /) => (a, b) R (e, f)
b e a f be af
Thus, {a, b) R (c, d) and (c, d) R (e, f) => {a, b) R (e, f) for all {a, b), (c, d), {e, f) eN x N.
So, R is transitive on N x N.
Hence, R being reflexive, symmetric and transitive, is an equivalence relation on N x N.
REMARK Let m be an arbitrary but fixed integer. Two integers a and b are said to be congruence modulo
m if a - b is divisible by m and we write a = b (mod m).
Thus, a =b (mod m) <=> a -b is divisible by m.
For example, 18 = 3 (mod 5) because 18 - 3 =15 which is divisible by 5. Similarly, 3 =13 (mod 2)
because 3 -13 = -10 which is divisible by 2. But 25 £ 2 (mod 4) because 4 is not a divisor of25 - 2 = 23.

low
EXAMPLE 13 Prove that the relation 'congruence modulo m' on the set Z of all integers is an equivalence
relation.
SOLUTION We observe the following properties of the given relation.

ee
Reflexivity: Let a be an arbitrary integer. Then,
rF
Fr
a - a = 0 = Oxm => a - is divisible by m => a = a (mod;//)
Thus, a = a (mod m) for all r? e Z.
or
So, "congruence modulo m" is reflexive.
sf
u
Symmetry: Let a,b eZ such that a =b (mod m). Then,
k
Yo

a = b (mod m)
oo

a-b is divisible by m
B
re

=> a - b = X m for A. e Z
=> b-a = (-X) m
ou
ad

=> b - a is divisible by m [v X eZ => - XeZ]


Y

=> b = a (mod m)
nd
Re

So, "congruence modulo m" is symmetric on Z.


Fi

Transitivity: Let a,b,ceZ such that a = b (mod m) and b = c (mod ;;/). Then,
a=b (mod m) => a - b is divisible by m => a - b =X-i m for some ^ e Z
b = c (mod m) => fr - c is divisible by m=> b - c = ?v2 m for some X2 eZ
(a -b) + (b -c) = A.-i m + X2 m = (A.| + X2) m
=> a -c = X2 m, where A.3 = A^ + A.2 e Z.
=> a = c (mod m)
Thus, a=b (mod m) and b = c (mod m) => a = c (mod ;;/).
So, "congruence modulo m" is transitive on Z.
Hence, "congruence modulo m" is an equivalence relation on Z.
EXAMPLE 14 Show that the number of equivalence relations on the set {1, 2, 3} containing (1,2) and
(2,1) is two. [NCERT]
SOLUTION The smallest equivalence relation R-^ containing (1, 2) and (2,1) is
^={(1,1), (2, 2), (3, 3), (1,2), (2,1)}

ReadYourFlow.COM
RELATIONS 1.23

Now, we are left with four ordered pairs namely (2,3) (3,2), (1, 3) and (3,1). If we add any one,
say (2,3) to Rp then for symmetry we must add (3, 2) and then for transitivity we are forced to
add (1,3) and (3,1). Thus, the only equivalence relation other than R1 is the universal relation.
Hence, the total number of equivalence relations containing (1, 2) and (2,1) is two.
EXAMPLE 15 Given a non-empty set X, consider P (X) which is the set of all subsets of X. Define a
relation in P (X) as follows:
For subsets A, B in P (X), A R B if A c. BAs R an equivalence relation on P (X)? Justify your answer.
[NCERT]
SOLUTION It is given that for any A, B in P (X): ARB o AcB
We observe the following properties of R.
Reflexivity: For any A in P (X), we have
^ c2\ => ARA

low
So, R is reflexive on P (X).
Symmetry: Let A, Bin P (X) be such that ARB. Then,
ARB => A aB
This need not imply that B c A. In fact it is possible only when A = B.

ee
Also, we know that {1, 2} c {1, 2, 3}, but {1, 2, 3} <2 {1, 2}.
rF
Fr
So, R is not a symmetric relation on P (X).
Transitivity: Let A, B, C be in P (X) such that
or
ARB and B R C => A a B and BczC => A a C => ARC
sf
u
So, R is a transitive relation on P (X).
k
Yo

Thus, R is reflexive and transitive relation on P (X) but it is not symmetric.


oo

Hence, R is not an equivalence relation on P (X).


B
re

EXAMPLE 16 Let R be the equivalence relation in the set A =(0,1,2, 3,4,5} given by
R ={(a, b): 2 divides (a-b)}. Write the equivalence class [0]. [CBSE2014J
ou
ad

SOLUTION Clearly, the equivalence class [0] is the set of those elements in A which are related
Y

to 0 under the relation R. i.e. [0] = {(«, 0) e P : a e A}.


Now, (a, 0) eR
nd
Re

=> rt - 0 is divisible by 2 and a e A


Fi

=> a e A such that 2 divides a


=> rt = 0, 2, 4
Thus, [0] = (0, 2, 4).
EXAMPLE 17 On the set N of all natural numbers, a relation R is defined as follows:
nRm o Each of the natural numbers n and m leaves the same remainder less than 5 when divided by 5.
Show that R is an equivalence relation. Also, obtain the pairwise disjoint subsets determined by R.
SOLUTION We observe the following properties of relation R.
Reflexivity: Let a be an arbitrary element of N. Then, either a is less than 5 and if >5, then on
dividing a by 5 we obtain a remainder as one of the numbers 0,1, 2, 3,4.
Thus, aRa for all a eN. So, R is reflexive on N.
Symmetry: Let a,b eN such that aRb. Then,
aRb => Each of a and b leaves the same remainder less than 5 when divided by 5
=> Each of b and a leave the same remainder less than 5 when divided by 5
=> bRa
Thus, aRb => bRa for all a,b eN. So, R is symmetric.

ReadYourFlow.COM
1.24 MATHEMATICS-XII

Transitivity : Leta,b, c eNbe such thataRb andbRc.Then,


aRb => Each of a and b leaves the same remainder less than 5 when divided by 5
bRc => Each of b and c leaves the same remainder less than 5 when divided by 5
Each of a and c leaves the same remainder less than 5 when divided by 5
=> aRc
Thus, aRb and bRc => aRc for all a, fr, c e N.
So, R is a transitive relation on N.
Hence, R is an equivalence relation on N.
Let us now find the equivalence classes.
[1] = {x eN : x R 1}
= {x eN :x and 1 leave the remainder less than 5 when divided by 5}
= {x sN :x leaves the remainder 1 when divided by 5}

low
= {1,6,11,16, 21,..}
[2] = {x eN :x R 2)
= {* e N : Each of x and 2 leave the remainder less than 5 when divided by 5}

ee
= [x eN :x leaves the remainder 2 when divided by 5}
rF
Fr
= {2,7,12,17, 22,..|
[3] = [x eN -.x R 3} or
= [x eN: Each of x and 3 leave the remainder less than 5 when divided by 5}
sf
u
= {x eN :x leaves the remainder 3 when divided by 5}
k

= {3, 8,13,18, 23,..J


Yo
oo

[4] = {x eN : x R 4}
B

= [x eN: Each of x and 4 leave the remainder less than 5 when divided by 5}
re

= [x eN :x leaves the remainder 4 when divided by 5}


= {4, 9,14,19,.. J
ou
ad

[5]={x eN :x R 5}
Y

= [x eN: Each of x and 5 leave the remainder less than 5 when divided by 5|
nd
Re

= {x eN :x leaves the remainder 0 when divided by 5}


Fi

= {5,10,15,.. J
Proceeding in this manner we find that
[1] = [6] = [11]...
[2] = [7] = [12]...
[3] =[8] =[13]...
[4] = [9] = [14]...
and. [5] = [10] = [15] =...
Thus, we obtain the following disjoint equivalence classes:
[1], [2], [3], [4], [5] such that N = [1] u[2] u[3] u[4] u[5]

1.4 SOME USEFUL RESULTS ON RELATIONS


In this section, we shall discuss some useful results on relations as theorems.
THEOREM 1 If R and S are two equivalence relations on a set A, then R nS is also an equivalence
relation on A.

ReadYourFlow.COM
RELATIONS 1.25

OR
The intersection of two equivalence relations on a set is an equivalence relation on the set.
PROOF It is given that R and S are relations on set A. [NCERT1
Rcz Ax A and Sc Ax A
=> R n Sc A x A
=> R n Sis also a relation on A.
Now, we shall show that it is an equivalence relation on /l.
We observe the following properties of relation R n S.
Reflexivity: Let a be an arbitrary element of A. Then,
a eA
=> (a, a) eR and (a, a) eS [v R and S are reflexive]
=> (a, a) e R n S
Thus, (a, a) e R n S for all a e A So, R n S is a reflexive relation on A.

w
Symmetry: Let arb e A such that (a, b) e R n S. Then,
(«, &) e R n S
=> (a, b) e R and (a, b) e S

Flo
ee
=> (b, a) eR and (b, a) e S [v R and S are symmetric]

Fr
=> (b, a) eR r\S
Thus, (a, b) e R n S=^> (b, a) e R n> S for all (a, b) eR nS. for
So, R n S is symmetric on A.
ur
Transitivity: Let a,b,c eA such that (a,b)eRnS and (b, c) eR nS. Then,
ks

(a,b) e R nS and (b, c) eR nS


Yo

=> {(a, b) eR and (a, b) e S)} and {(b, c) eR and (b, c) e S)}
oo

=> {(a, b) e R, (1?, c) e R}and {(a, b) e S, (b, c) e S}


eB

v R and S are transitive.


=> (a, c) e R and (a, c) e S (a, b) eR and (b, c) e R => {a, c) e R
r

(a, b) eS and (b, c) e S=> (a, c) e S


ou
ad
Y

=> (a, c) e R n S
Thus, (a,b) e R n S and (b, c) e R n S=> (a, c) e R n S.
nd
Re

So, R n Sis transitive on A.


Fi

Hence, R n S is an equivalence relation on A.


THEOREM 2 The union of two equivalence relations on a set is not necessarily an equivalence relation
on the set.
PROOF Let A = {a, b, c} and let R and S be two relations on A, given by
R = {(a, a), (b, b), (c, c), (a, b), (b, a)} and, S = {(a, a), (b, b), (c, c), (b, c), (c, b)}
It can be easily seen that each one of R and S is an equivalence relation on A. But, R u S is not
transitive, because (a,b) e R S and (b, c) eR o S but (a, c) g R u S.
Hence, R u S is not an equivalence relation on A.
THEOREM 3 IfR is an equivalence relation on a set A, then R~ 1 is also an equivalence relation on A.
OR
The inverse of an equivalence relation is an equivalence relation.
PROOF Since R is a relation on A.
RcAxA=>R~1cAxA=>R~l is also a relation on A.
Now, we shall show that R_1 is an equivalence relation on A.

ReadYourFlow.COM
r
1.26 MATHEMATICS-XII

We observe the following properties of relation R 1.


Reflexivity: Let a be an arbitrary element of A. Then,
a eA
=> (a, a) e R [y R is reflexive]
=> (a, a) eR -1 [By definition of R-1]
Thus, (a, a) eR-1 for all a e A.
So, R-1 is reflexive on A.
Symmetry: Let (a, b) eR~^. Then,
(a, b) e R -1
=> (b, a) eR [By definition of R ’]
=> (a, b) eR [•.• R is symmetric]
(b, a) eR -l

low
=> [By definition of R~l]
Thus, (fl, b) e R-1 => (b, a) eR~l for all a,b eA.
So, R-1 is symmetric on A.
Transitivity: Let (a, b) eR -1 and (fr,c)eR

ee
1. Then,
rF
Fr
(a, b) e R-1 and (b, c) eR -1
=> (b, a) eR and (c, b) eR or [By definition of R ^
=> (c, b) eR and(b, a) e R
sf
u
=> (c, a) eR [•.• R is transitive]
(a, c)e R -1
k

=> [By definition of R ~l]


Yo
oo

Thus, (a, b) eR 1 and (b, c) eR -1 => (a, c) eR 1 for all a,b,c e A.


B

So, R 1 is transitive on A.
re

Hence, R-1 is an equivalence relation on A


ou
ad
Y

EXERCISE 1.2
LEVEL-1
nd
Re

1. Show that the relation R defined by R = {(a, i?): rt-bis divisible by 3; rt, fr e Z} is an
Fi

equivalence relation. [CBSE 2008]


2. Show that the relation R on the set Z of integers, given by
R = {(a, b): 2 divides a -b}, is an equivalence relation. [NCERT]
3. Prove that the relation R on Z defined by
(arb) e R a - b is divisible by 5
is an equivalence relation on Z. [CBSE 2010]
4. Let« be a fixed positive integer. Define a relation R on Z as follows:
(a,b) eR <=> 0 - fr is divisible by n.
Show that R is an equivalence relation on Z.
5. Let Z be the set of integers. Show that the relation R = {(a, b):a,b eZ and a + b is even] is an
equivalence relation on Z.
6. m is said to be related to n if m and n are integers and m - n is divisible by 13. Does this define
an equivalence relation?
7. Let R be a relation on the set A of ordered pairs of non-zero integers defined by
(x, y) R (u, v) iff xv = yu. Show that R is an equivalence relation. [NCERT]
8. Show that the relation R on the set A = (x e Z ; 0 < x < 12}, given by R = {(a, b): a = b}, is an
equivalence relation. Find the set of all elements related to 1.

ReadYourFlow.COM
RELATIONS 1.27

9. Let L be the set of all lines in XY-plane and R be the relation in L defined as R = {(L-j, L2):
is parallel to L2}. Show that R is an equivalence relation. Find the set of all lines related to
the line y = 2x + 4.
10. Show that the relation R, defined on the set A of all polygons as
R = {(Pj, P2): Pj and P2 have same number of sides),
is an equivalence relation. What is the set of all elements in A related to the right angle
triangle T with sides 3,4 and 5? [NCERT1
11. Let O be the origin. We define a relation between two points P and Q in a plane if OP = OQ.
Show that the relation, so defined is an equivalence relation.
12. Let R be the relation defined on the set A = (1, 2, 3, 4, 5, 6, 7 }by R = {(a, b): both n and b are
either odd or even). Show that R is an equivalence relation. Further, show that all the
elements of the subset jl, 3,5, 7} are related to each other and all the elements of the subset
{2, 4, 6} are related to each other, but no element of the subset {1, 3,5, 7} is related to any
element of the subset [2, 4, 6). [NCERT1

w
LEVEL-2
13. Let S be a relation on the set R of all real numbers defined by S = {((?,/?) e R x R:a 2 +b~2 =1}.

Flo
Prove that S is not an equivalence relation on R.
14. Let Z be the set of all integers and Z0 be the set of all non-zero integers. Let a relation R on

ee
Z x Z0 be defined as follows:

Fr
(a, b) R (c, d) <=> ad = be for all (a, b), (c, tf) e Z x Zq
Prove that R is an equivalence relation on Z x Zq.
for
ur
15. If R and S are relations on a set A, then prove the following:
(i) R and S are symmetric => R n S and R u S are symmetric
ks

(ii) R is reflexive and S is any relation => R u S is reflexive.


Yo

16. If R and S are transitive relations on a set A, then prove that R u S may not be a transitive
oo

relation on A.
B

17. Let C be the set of all complex numbers and C0 be the set of all non-zero complex numbers.
re

Let a relation R on C0 be defined as


Z} R z2 « 21 ~ z2
ou

is real for allz-],z2 sCq.


ad

21 +22
Y

Show that R is an equivalence relation.


nd
Re

ANSWERS
8. ID 9. {y = 2a- -i- c: c e R} 10. Set of all triangles
Fi

HINTS TO NCERT& SELECTED PROBLEMS


2. The relation R on Z is given by R = {(a, b): 2 divides a -b}.
We observe the following properties of relation R.
Reflexivity: For any a eZ
a-a = 0 = 0x2
=> 2 divides a-a => {a, a) e R
So, R is a reflexive relation on Z.
Symmetry: Leta,b eZ be such that
(a, b) e R
=> 2 divides a - b
=> <7 - fr = 2 Z for some A. e Z
=> b - a = 2(-X), where -XeZ
=> 2 divides b - a => (b, a) eR
Thus, (a, b) a R => (/’,(?) e R. So, R is a s\anmetric relation on Z.
Transitivity: Let a,b, c eZbe such that (a, b) a R and (b, c) a R. Then,

ReadYourFlow.COM
1.28 MATHEMATICS-XII

(a, b) eR => 2 divides b - a => b - a = 2X for some X e Z


and, (b, c) eR => 2 divides c - b => c-b = 2|.i for some ja e Z
b - a + c -b = 2 (X + (i)
=s c — ci = 2 (A. + (.i), where + |.i s Z
=> 2 divides c - rt
=> (rt, c) eR
Thus, (a, b) e R and (b, c) e R=> (a, c) e R.
So, R is a transitive relation on Z.
Hence, R is an equivalence relation on Z.
7. The relation R on Z x Z is defined by
(.v, y) R (u, v) <=> xv = yu for all (x, y), (u, v) eZx Z
We observe the following properties of R on Z x Z.
Reflexivily: For any (x, y) eZ x Z

low
•vy = yx [•.• Multiplication is commutative on Z]
=> (x, y) R (x, y)
Thus, (x, y) R (x, y) for all (x, y) eZ x Z.
So, R is a reflexive relation on Z x Z.

ee
Symmetry. Let (x, y), [u, v) eZx Z such tlrat (x, y) R ((u, v). Then,
rF
Fr
(x, i/) R (u, v) => xv = yu => uy = vx => (u, v) R (x, y)
Thus, (x, y) R (it, v) => (u, v) R (x, y) for all (x, y), (u, v) eZx Z.
or
So, R is a symmetric relation on Z.
sf
u
Transitivity: Let (x, y), (u, v), (a, b) eZx Zbe such that (x, y) R (it, v) and (u, v) R (a, b)
k

Then,
Yo
oo

(x, y) R (it, v) -=> xv = yu


=> (xv) (ub) =(yv) (va) => xb = ya => (x,y) R (a,b)
B

and, (u, v) R (a, b) => ub = va


re

So, R is a transitive relation on Z x Z.


Hence, R is an equivalence relation on Z x Z.
ou
ad

10. The relation R on the set of A of all polygons is defined as


Y

R = {(Pi, P2): P\ and P2 have same number of sides}


nd
Re

We observe the following properties of R on A.


Reflexivity: Let P be any polygon in A. Then,
Fi

P and P have same number of sides.


=> (P, P) e R
Thus, (P, P) e R for all P eA. So, R is a reflexive relation on A.
Symmetry: Let P|, P2 be two polygons in A such that(P1, P2) e R. Then,
(P,,P2)gR
=> P| and P2 have same niunber of sides
=> P2 and Pj have same number of sides
=> (P2' Pi) 6R
So, R is symmetric on A.
Transitivity: Let P-,, P2, P3 be three polygons in A such that(P1, P2) e R and (P2, P3) e R.
Then,
(P], P2) e R => P^ and P2 have same number of sides
and, (P2, P3) e R P2 and P3 have seune number of sides
P| and P3 have same number of sides.

ReadYourFlow.COM
RELATIONS 1.29

=> (Pi,P3)eR
Thus, (Pj, P2) e R and (P2/ P3) e P => (P^, P3) e P
So, P is a transitive relation on A.
Hence, P is an equivalence relation on A.
Let P be a polygon in A such that (P, T) e P, where T is a right angled triangle with sides 3,
4 and 5. Then,
(P, T) e R
=> Polygon P and triangle T have same number of sides
=> P is any triangle in A
Hence, the set of all elements in A related to T is the set of all triangles in A.
12. The relation P on set A = {1,2, 3, 4, 5, 6, 7} is defined by
P = {{n, b): both a and b are either odd or even}
We observe the following properties of P on A\

low
Reflexivity: Clearly, (1,1), (2, 2), (3, 3), (4, 4), (5,5), (6, 6), (7,7) eP. So, P is a reflexive
relation on A.
Symmetry: Let a,b e A be such that {a, b) e R. Then,
{a, b) eR

ee
=> Both a and b are either odd or even
rF
Fr
=> Both b and a are either odd or even
=> (b, a) gR or
Thus, {a, b) eR=> (b, a) e R for all n, b e A.
sf
u
So, P is a symmetric relation on A.
k

Transitivity: Leta,b, c eZbe such that(<?,fc) eR,(b, c) e P. Then,


Yo
oo

{a, b) e R => Both a and b are either odd or even


B

(b, c) eR => Both b and c are either odd or even


re

If both a and b are even, then


(b, c) eR => Both b and c are even
ou
ad

Both a and c are even


Y

If both a and b are odd, then


nd

(b, c) eR => Both b and c are odd


Re

Both a and c are odd


Fi

Thus, both a and c are even or odd. Therefore, {a, c) eR.


So, {a, b) eR and (b, c) eR => (a, c) e R.
Consequently, P is a transitive relation on A.
Hence, P is an equivalence relation on A.
We observe that two numbers in A are related if both are odd or both are even. Since
{1, 3, 5,7} has all odd numbers of A. So, all the numbers of {1, 3,5, 7} are related to each
other. Similarly, all the numbers of {2, 4, 6} are related to each other as it contains all even
numbers of set A. An even odd number in A is related to an even (odd) number in A. So, no
number of the subset {1, 3,5, 7} is related to any number of the subset {2, 4, 6}.
_____________________________________VERY SHORT ANSWER QUESTIONS (VSAQs)
Answer each of the following questions in one word or one sentence or as per exact requirement of the
question:
1. Write the domain of the relation P defined on the set Z of integers as follows:
{a,b)eR <=> a2+b2 =25

ReadYourFlow.COM
1.30 MATHEMATICS-XII

2. If R = {(.t, y): x2 + i/2 < 4; x, 1/ e Z} is a relation on Z, write the domain of R.


3. Write the identity relation on set A = {a,b, c).
4. Write the smallest reflexive relation on set ,4 = {1, 2, 3, 4}.
5. If R = {(x, y)\x + 2y- 8} is a relation on N, then write the range of R. [CBSE2014]
6. If R is a symmetric relation on a set A, then write a relation between R and R-1.
7. Let R = {(x, y) : \ x2 -y2 | <1} be a relation on set A = {1, 2, 3, 4,5}. Write R as a set of
ordered pairs.
8. 11A = {2, 3, 4}, B = {1, 3, 7} and R = {(x, y): x e A, y e B and x < y} is a relation from A to B,
then write R-1.
9. Let A = {3,5, 7}, B = {2, 6,10} and R be a relation from A to B defined by R = {(x, y): x and y
are relatively prime). Then, write R and R A.
10. Define a reflexive relation.

low
11. Define a symmetric relation.
12. Define a transitive relation.
13. Define an equivalence relation.
14. If A = {3,5, 7} and B = {2, 4, 9} and R is a relation given by "is less than", write R as a set

ee
ordered pairs.
rF
Fr
15. A = {1, 2, 3, 4,5, 6, 7, 8} and if R = {(x, y): y is one half of x; x, y e A}is a relation on A, then
write R as a set of ordered pairs.
16. Let A = {2, 3, 4,5} and B = {1, 3, 4}. If R is the relation from A to B given by a Rb iff "a is a
for
divisor olb Write R as a set of ordered pairs.
ou
17. State the reason for the relation R on the set {1, 2, 3} given by R = {(1, 2), (2,1)} not to be
ks

transitive. [CBSE2011]
oo

18. Let R = |(fl, rt ): a is a prime number less than 5) be a relation. Find the range of R.
Y
B

[CBSE2014]
re

19. Let R be the equivalence relation on the set Z of integers given by R = {(a, b):2 divides a - b}.
Write the equivalence class [0]. [NCERT EXEMPLAR]
ou
ad

20. For the set A = {1, 2, 3), define a relation R on the set A as follows:
Y

R =1(1/1)/ (2, 2), (3, 3), (1,3)|


Write the ordered pairs to be added to R to make the smallest equivalence relation.
nd
Re

21. Let A = {0,1, 2, 3) and R be a relation on A defined as


Fi

R = ((0, 0), (0,1), (0, 3), (1, 0), (1,1), (2, 2), (3, 0), (3, 3))
Is R reflexive? symmetric" transitive?
22. Let the relation R be defined on the set A = jl, 2, 3, 4,5) by R = {{a, b): | a2 -b2\ < 8). Write R
as a set of ordered pairs.
23. Let the relation R be defined on N by a Rb iff 2a + 3b = 30. Then write R as a set of ordered
pairs.
24. Write the smallest equivalence relation on the set A = {1, 2, 3}.
ANSWERS
1. {0, ± 3, ± 4, ± 5} 2. {0, ± 1, ± 2} 3. {{a, a), (b,b),(c,c)}
4. {(1,1), (2, 2), (3, 3), (4, 4)} 5. {1, 2, 3} 6. R =R~1
7. {(1,1), (2, 2), (3, 3), (4, 4), (5, 5)} 8. R -1 = {(3, 2), (7,2), (7, 3), (7,4)}
9. R = {(3, 2), (3,10), (5, 2), (5, 6), (7, 2), (7, 6), (7,10)}
R"1 = {(2, 3), (10, 3), (2,5), (6,5), (2, 7), (6, 7), (10, 7)}
14. R = {(3,4), (3, 9), (5, 9), (7, 9)} 15. R = {(2,1), (4, 2), (6, 3), (8, 4)}

ReadYourFlow.COM
RELATIONS 1.31

16. {(2, 4), (4, 4), (3, 3)} 17. (1, 2) e R and (2,1) e R but (1,1) £ R
18. {8, 27} 19. [0] = {0, ± 2, ± 4, ± 6,..| 20. (3,1)
21. Reflexive and symmetric
22. R = {(1,1), (1, 2), (2,1), (2, 2), (2, 3), (3, 2), (3, 3), (3, 4), (4, 3), (4, 4), (5,5)}
23. R = {(3, 8), (6, 6), (9, 4), (12, 2)} 24. {(1,1), (2, 2), (3, 3)}
HINTS TO NCERT SELECTED PROBLEMS
17. We observe that (1, 2) e R and (2,1) e R but (1,1) g R. Hence, R is not transitive.
18. We have,
R = [(a, a^) :a is prime less than 5}
=> R={(a/a3):a = 2,3}
=> R =1(2, 8), (3, 27)}
Range (R) ={8, 27}
19. We have.

w
R =[(a,b):2 divides a - b}
For any a eZ,
[rt] = {x :(x, a) eR\ = {x: 2 divides x - fl}

Flo
[0] = [x eZ :2 divides x -Q\ = {x eZ :2 divides x} = {0, ± 2, ± 4, ± 6,...}

ee
Fr
MULTIPLE CHOICE QUESTIONS (MCQs)
Mark the correct alternative in each of the following: or
ur
1. Let R be a relation on the set N given by R = {(a, b): a = b - 2, b > 6}. Then,
sf
(a) (2, 4) e R (b) (3,8) eR (c)(6,8)eR (d) (8,7) eR
k

2. Which of the following is not an equivalence relation on Z?


Yo
oo

(a) a R fr <=> a + fr is an even integer (b) a Rb <=> a -b is an even integer


(d) a R b <=> a =b
B

(c) a R b o a <b
3. R is a relation on the set Z of integers and it is given by (x, y) e R <=> | x - y | < 1.Then, R is
re

(a) reflexive and transitive (b) reflexive and symmetric


ou
ad

(c) symmetric and transitive (d) an equivalence relation


Y

4. The relation R defined on the set A ={1,2, 3, 4, 5}by R = | (a, b):\ a2-b~\ <16 L is given by
nd
Re

(a) {(1,1), (2,1), (3,1), (4,1), (2, 3)} (b) {(2, 2), (3, 2), (4, 2), (2, 4)}
Fi

(c) {(3, 3), (4, 3), (5, 4), (3, 4)} (d) none of these
5. Let R be the relation over the set of all straight lines in a plane such that R /2 <=> /] 1 12-
Then, R is
(a) symmetric (b) reflexive
(c) transitive (d) an equivalence relation
6. If A = [a, b, c], then the relation R ={(b, c)} on A is
(a) reflexive only (b) symmetric only
(c) transitive only (d) reflexive and transitive only
7. Let A = {2,3,4,5,..., 17,18}. Let be the equivalence relation on A x A, cartesian product
of A with itself, defined by (a, b)~(c,d) iff ad = be. Then, the number of ordered pairs of the
equivalence class of (3, 2) is
(a) 4 (b) 5 (c) 6 (d) 7
8. Let A = {1,2, 3}. Then, the number of relations containing (1, 2) and (1, 3) which are
reflexive and symmetric but not transitive is
(a) 1 (b) 2 (c) 3 (d) 4
9. The relation 'R' in N x N such that (a, b) R (c, d) <=> a + d = b + c is

ReadYourFlow.COM 4
1.32 MATHEMATICS-XII

(a) reflexive but not symmetric (b) reflexive and transitive but not symmetric
(c) an equivalence relation (d) none of these
10. IM = {1, 2, 3), B = {1,4, 6, 9} and K is a relation from A to B defined by 'x is greater than y
The range of R is
(a) {1,4,6,91 (b) {4,6,9} (c) {11 (d) none of these
11. A relation R is defined from {2, 3, 4, 5} to {3, 6, 7,10) by : x B t/ <=> x is relatively prime to y.
Then, domain of R is
(a) {2,3,5} (b) {3,5} (c) {2,3,4} (d) {2,3,4,51
12. A relation <{) from C to B is defined by x ((> y <=> | x | = y. Which one is correct?
(a) (2+3/) <}>13 (b) 3 <})(- 3) (c) (1+/)(()2 (d) /<t>l
13. Let B be a relation on N defined by x + 2 y = 8. The domain of B is
(a) {2,4,8} (b) {2,4,6,81 (c) {2,4,6} (d) {1,2,3,4}
14. B is a relation from {11,12,13} to {8,10,12} defined by y = x - 3. Then, B_1 is

w
(a) {(8,11), (10,13)} (b) {(11, 8), (13,10)}
(c) {(10,13), (8,11), (8,10)} (d) none of these
15.

Flo
Let B = {(a, a), (b, b), (c, c), (a, b)} be a relation on set A = [a, b, c). Then, B is

ee
(a) identity relation (b) reflexive

Fr
(c) symmetric (d) equivalence
16. Let A = {1, 2, 3} and B = {(1, 2), (2, 3), (1, 3)} be a relation on A. Then, B is
for
ur
(a) neither reflexive nor transitive (b) neither symmetric nor transitive
(c) transitive (d) none of these
ks

17. If B is the largest equivalence relation on a set A and S is any relation on A, then
Yo
oo

(a) B c S (b) ScB (c) B=S (d) none of these


B

18. If B is a relation on the set A = {1,2,3,4,5,6, 7, 8, 9} given by x B y <=> y = 3 x, then B =


re

(a) {(3,1), (6, 2), (8, 2), (9, 3)J (b) {(3,1), (6, 2), (9, 3)}
(c) {(3,1), (2, 6), (3, 9)} (d) none of these
ou
ad

19. If B is a relation on the set A = {1, 2, 3} given by B = (1,1), (2, 2), (3, 3), then B is
Y

(a) reflexive (b) symmetric (c) transitive (d) all the three options
20. If A = {a, b, c, d), then a relation B = {(//, b)r (b, a), (a, a)} on A is
nd
Re

(a) symmetric and transitive only (b) reflexive and transitive only
Fi

(c) symmetric only (d) transitive only


21. If A = (1, 2, 3), then a relation B = {(2, 3)} on A is
(a) symmetric and transitive only (b) symmetric only
(c) transitive only (d) none of these
22. Let B be the relation on the set A = {1, 2, 3, 4} given by
B = {(1, 2), (2, 2), (1,1), (4, 4), (1, 3), (3, 3), (3, 2)}. Then,
(a) B is reflexive and symmetric but not transitive
(b) B is reflexive and transitive but not symmetric
(c) B is symmetric and transitive but not reflexive
(d) B is an equivalence relation
23. Let A = {1, 2, 3}. Then, the number of equivalence relations containing (1, 2) is
(a) 1 (b) 2 (c) 3 (d) 4
24. The relation B= {(1,1), (2, 2), (3, 3)} on the set (1, 2, 3} is
(a) symmetric only (b) reflexive only
(c) an equivalence relation (d) transitive only

ReadYourFlow.COM
RELATIONS 1.33

25. S is a relation over the set R of all real numbers and it is given by (a, b) gS<^> ab > 0.
Then, Sis
(a) symmetric and transitive only (b) reflexive and symmetric only
(c) antisymmetric relation (d) an equivalence relation
26. In the set Z of all integers, which of the following relation R is not an equivalence relation?
(a) x R y : if x < y (b) xRy.ifx = y
(c) * R y: if x - y is an even integer (d) x R y: if x = y (mod 3)
27. Let A ={1,2, 3} and consider the relation R ={(1,1), (2, 2), (3, 3), (1,2), (2, 3),(1, 3)}. Then,
R is
(a) reflexive but not symmetric (b) reflexive but not transitive
(c) symmetric and transitive (d) neither symmetric nor transitive
28. The relation S defined on the set R of all reai number by the rule aSbitfa>b is

w
(a) an equivalence relation
(b) reflexive, transitive but not symmetric
(c) symmetric, transitive but not reflexive
(d) neither transitive nor reflexive but symmetric
Flo
ee
29. The maximum number of equivalence relations on the set A ={1, 2, 3) is

Fr
(a) 1 (b) 2 (c) 3 (d) 5
30. Let R be a relation on the set N of natural numbers defined by n R m iff/t divides m. Then, R is
for
ur
(a) Reflexive and symmetric (b) Transitive and symmetric
ks

(c) Equivalence (d) Reflexive, transitive but not symmetric


Yo

[NCERT EXEMPLAR]
oo

31. Let L denote the set of all straight lines in a plane. Let a relation R be defined by /R m iff I is
eB

perpendicular to m for all /, m e L. Then, R is


(a) reflexive (b) symmetric (c) transitive (d) none of these
r
ou
ad

[NCERT EXEMPLAR]
Y

32. Let T be the set of all triangles in the Euclidean plane, and let a relation R on T be defined as
aRb if a is congruent to b for all a,b & T. Then, R is
nd
Re

(a) reflexive but not symmetric (b) transitive but not symmetric
Fi

(c) equivalence (d) none of these


33. Consider a non-empty set consisting of children in a family and a relation R defined asaRb
if a is brother of b. Then, R is
(a) symmetric but not transitive (b) transitive but not symmetric
(c) neither symmetric nor transitive (d) both symmetric and transitive
34. For real numbers x and y, define x R y iff x -y + v:2 is an irrational number. Then the relation
R is
(a) reflexive (b) symmetric (c) transitive (d) none of these

____________________________________________________________________ANSWERS
1. (c) 2. (c) 3. (b) 4. (d) 5. (a) 6. (c) 7. (c) 8. (a) 9. (c)
10. (c) 11. (d) 12. (d) 13. (c) 14. (a) 15. (b) 16. (c) 17. (b) 18. (d)
19. (d) 20. (c) 21. (c) 22. (b) 23. (b) 24. (c) 25. (b) 26. (a) 27. (a)
28. (b) 29. (d) 30. (d) 31. (b) 32. (c) 33. (b) 34. (a)

ReadYourFlow.COM
1.34 MATHEMAT1CS-XII

SUMMARY
1. A relation from a set A to a set B is a subset of Ax B.
2. Total number of relations from a set consisting of w elements to a set consisting of » element
is 2mn.
3. A relation on a set A is a subset of A x A.
4. A relation R on a set A is said to be
(i) the identity relation, if every element of A is related to itself only.
(ii) reflexive, if (rt, (?) e .R for all d e A
(iii) symmetric, if (a, b) eR=> (b, a) eR for all rt, fr e A
(iv) transitive, if (a, b) eR and (b, c) e R => (a, c) e R for all £?, b, c e A
(v) an equivalence relation, if it is reflexive, symmetric and transitive
(vi) antisymmetric, if (fl, fr) e JR and (b,a)<=R => a=b
(vii) the empty relation, if R = (j>

w
(viii) the universal ration, if R = A x A.

Flo
ee
Fr
for
ur
ks
Yo
oo
B
re
ou
ad
Y
nd
Re
Fi

ReadYourFlow.COM
CHAPTER

FUNCTION

2.1 INTRODUCTION
The concept of function is of paramount importance in Mathematics and among other
disciplines as well. In earlier class we have introduced the notion of function and we have learnt
about some special functions like identity function, constant function, polynomial function,

w
rational function, modulus function, greatest integer function, signum function etc. along with
their graphs. Addition, subtraction, multiplication and division of two real functions have also

Flo
been studied in the earlier class. In this chapter, we would like to extend our study about
functions from where we finished in earlier class. We will study about various kinds of

ee
functions, composition of functions and inverse of a function. Let us first recapitulate what we

Fr
have learnt about functions in earlier class.

2.2 RECAPITULATION
for
ur
FUNCTION AS A SET OF ORDERED PAIRS Let A and B be two non-empty sets. A relation f from A to B
i.e. a sub set of A x B is called a function (or a mapping or a map) from A to B, if
ks

(i) for each a & A there exists b eB such that (a, b) e f.


Yo
oo

(ii) (a, b) e f and (a, c) e f => b = c.


B

Thus, a non-void subset of A x B is a function from A to B if each element of A appears in some


re

ordered pair in f and no two ordered pairs in / have the same first element.
If (a, b) e f, then b in called the image of a under /.
ou
ad

FUNCTION AS A CORRESPONDENCE Let A and B be two non-empty sets. Then a function f 'from set A
Y

to set B is a rule or method or correspondence which associates elements of set A to elements of set B such
nd

that:
Re

(i) all elements of set A are associated to elements in set B.


Fi

(ii) an element of set A is associated to a unique element in set B.


In other words, a function ffrom a set A to a set B associates each element of set A to a unique element of
set B.
Terms such as "map" (or "mapping"), "correspondence" are used as synonyms for "function".
/
If/is a function from a set A to a set B, then we write / : A —» B or A —> B, which is read as f is a
function from A to B or/ maps A to B.
If an element a eAis associated to an element & e B, then b is called "thef-image of a" or "image of a
underf" or" the value of thefunction fat a". Also, a is called the pre-image of b under the function /.
We write it as b =f(a).
The set A is known as the domain of / and the set B is known as the co-domain of /. The set of all
/-images of elements of A is known as the range of / or image set of A under / and is denoted by
f(A).
Thus,/(A) = [f(x): x g A} = Range of/.
A visual representation of a function is shown in Fig. 2.1.

ReadYourFlow.COM
2.2 MATHEMATICS-XII

Domain Range/(x)

A B
Fig. 2.1

FUNCTION AS A MACHINE A function can also be regarded as a machine that gives unique output in set
B corresponding to each input from the set A just as the function 'machine' shown in Fig. 2.2 which
generate an output y = 2x3 + 5for each input x.

Input value x
CUBE*
Q

low
Multiply

ee
by 2
rF
Fr
o Add5
00or
Output value ij = 2x3 + 5
sf
u
Fig. 2.2
k
Yo

Usually real functions are described by using a mathematical formula. It is traditional to let x
oo

denote the input and y the corresponding output and to describe the function we write an
eB

equation relating x and y. In such an equation x and y are called variables. Because the value of
the variable y is determined by that of the variable x, so we call y the dependent variable and x the
r

independent variable.
ou
ad

If A and B are two sets having m and n elements respectively, then total number of functions
Y

from,4 to Bisnm.
A function/: A -> B is called a real valued function if B is a subset of R (set of all real numbers).
nd
Re

If A and B both are subsets of R, then / is called a real function.


Fi

In order to represent a real function y = f(x) geometrically as a graph, we use a cartesian


coordinate system on which imits for the independent variable x are marked on the horizontal
axis i.e. x-axis and units for the dependent variable y on the vertical axis i.e. y-axis.
GRAPH OF A FUNCTION The graph of a real function f consists of points whose coordinates (x, y) satisfy
y=f (x),for all x e Domain (/).
In this section, we shall discuss graphs of Some standard real functions.
By the definition of a real function/, for a given x in its domain there is only one number y = / (x)
in its range. Geometrically, this means that any vertical line x = a crosses the graph of / (x) at
most once only. This observation leads to the following useful criterion for checking whether a
curve in a plane is the graph of a function or not.
VERTICAL LINE TEST A curve in a plane represeiits the graph of a real function ifand only if no vertical
line intersects it more than once.
The curves shown in Fig. 2.3 (a) are the graphs of function whereas the curves shown in
Fig. 2.3 (b) are not the graphs of functions as tfyere exist vertical lines which intersect them more
than once.

ReadYourFlow.COM
FUNCTIONS 2.3

Y Y

J----

1
O X O X

Function Function
(a)

w
Flo
ee
Fr
Not a function Not a function
(b)
for
ur
Fig. 2.3
ks

Following are some standard real functions which will occur very frequently in the study of
Yo
oo

calculus.
eB

CONSTANT FUNCTION Ifk is a fixed real number, then a function f (x) given by f(x) = k for all x eR is
called a constant function.
r

Sometimes we also call it the constant function k.


ou
ad

We observe that the domain of the constant function f (x) =k is the set R of all real numbers and
Y

range of / is the singleton set [k].


nd

The graph of a constant function f (x) = /c is a straight line parallel to x-axis (see Fig. 2.4) which is
Re

above or below x-axis according as k is positive or negative. If k = 0, then the straight line is
Fi

coincident to x-axis.

f(x) = k
k

X' O X

\~
I i r I
Fig. 2.4 Constant function

ReadYourFlow.COM
2.4 MATHEMATICS-XII

IDENTITY FUNCTION Thefunction that associates each real number to itselfis called the identityfunction
and is usually denoted by l.
Thus, thefunction I: R R defined by I(x) = x for all x e R is called the identity function.

low
Fig. 2.5 Identity function

Clearly, the domain and range of the identity function are both equal to R.

ee
The graph of the identity function is a straight line passing through the origin and inclined at an
rF
Fr
angle of 45° with X-axis.
MODULUS FUNCTION The function f (x) defined by
for
x , when x > 0
/(*) = I *
ou
-x , when x < 0
ks

is called the modulus function.


oo
Y

It is also called the absolute value function.


B
re

We observe that the domain of the modulus function is the set R of all real numbers and the
range is the set of all non-negative real numbers i.e. R+ ={x eR:x>0}.
ou
ad

The graph of the modulus function is as shown in Fig. 2.6 for x > 0, the graph coincides with
Y

the graph of the identity function i.e. the line y = x and for .r < 0, it is coincident to the line y = -x.
nd
Re
Fi

Fig. 2.6 Modulus function

The modulus function has the following properties:


(i) For any real number x, we have

*1

ReadYourFlow.COM
FUNCTIONS 2.5

COS X , 0<x<-
For example, Jcos2 x COS X
2
71
- COS X , - <x <n
2
(ii) If a, b are positive real numbers, then
x2 <a2 <=> |x| < a «• -a <x <a
x2 > a2 <=> x\ > a <=>x<-aor,x>a
x2 <a2 <=> | x | < fl <» — a < x < a
2 2
X > <=> x\> a <=> x <-a or, x > a
a2 <x2 <b2 o a <\x\ <b o x e[-b, - a] u [a,b]
a2 <x2 <b2 o a<\x\<b <=> e(-b, - a) v(a,b)

w
(iii) For real numbers x and y, we have
|x + y| =|x|+|y|,if(x>0 and y > 0) or, (x < 0 and y < 0)

Flo
| x - y | = | x| - | y |, if (x > 0 and| x| >| y |) or, (x < 0 , y < 0 and| x| > | y |)

ee
|x±y|<|x|+|y|

Fr
U±yl>|h'l-|y| |
GREATEST INTEGER FUNCTION (FLOOR FUNCTION) For any real number x, we use the symbol [x]
for
ur
or, [xj to denote the greatest integer less than or equal to x. For example,
[2.75] = 2, [3] =3, [0.74] = 0,[-7.45] =-8etc.
ks
Yo

The function f :R R defined by f (x) =[x] for all x e R is called the greatest integer function or the
oo

floor function.
eB

It is also called a step function.


Clearly, domain of the greatest integer function is the set R of all real numbers and the range is
r
ou

the set Z of all integers as it attains only integer values.


ad

The graph of the greatest integer function is shown in Fig. 2.7.


Y
nd
Re

y
-3
Fi

,
o--- i
X' -3 -2 -1 0 12 3 X
-o-l

Fig. 2.7 Greatest integer function


PROPERTIES OF GREATEST INTEGER FUNCTION If n is an integer and x is a real number betweenn
and n + 1, then
(i) [-n] = -[n] (ii) [x + k] - [x] + k for any integer k.
-1 , if x g Z
(iii) I-*! = -M-1 (iv) [x] + [- x] = 0 , if x e Z

ReadYourFlow.COM
2.6 MATHEMATICS-XII

2[x] + 1, if xeZ
(V) [x]-[-x] = (vi) [x] > k => x>k, wherek eZ
2[x] , ifxeZ
(vii) [x] </c => x < k + 1, where k g Z (viii) [x] > => x>k + l, were k eZ
(ix) [x] <k => x < k, where k eZ (x) [x + y] = [x] + [y + x - [x]] for all x, y eR
(xi) [x]+ x + ij + [x + ^ n-1
+ ... + x + = [nx] ,n eN.
n
SMALLEST INTEGER FUNCTION (CEILING FUNCTION) For any real number x, we use the symbol fx"|
to denote the smallest integer greater than or equal to x.
For example.
[4.71 =5' T"7-2! =-7' f5! =5' [0.751 =1 etc.
The function f :R R defined by f(x) - [xl for all x e R is called the smallest integerfunction or the
ceiling function.

low
It is also a step function.
We observe that the domain of the smallest integer function is the set R of all real numbers and
its range is the set Z of all integers.
The graph of the smallest integer function is as shown in Fig. 2.8.

ee
rF
Fr
for
ou
ks
oo
Y
B
re
ou
ad
Y

Fig. 2.8 Smallest integer function


nd
Re

PROPERTIES OF SMALLEST INTEGER FUNCTION Following are some properties of smallest


Fi

integer function:
(i) [-«! = -[«! , whereneZ (ii) [-xl= -[x'1 + 1, wherexel?-Z
(iii) [x + «l =[xl +77, where x e R - Z and?? e Z (iv) [xl + [-xl =

(v) M + f-xl=j 2[xl-l,


2 [xl ,
ifxgZ
ifxeZ
FRACTIONAL PART FUNCTION For any real number x, we use the symbol (xj to denote the
fractional part or decimal part of x. For example,
{3.45) = 0.45, {- 2.75) = 0.25, {- 055} = 0.45, {3) = 0, {- 7) = 0 etc.
The function f: R —> R defined by f (x) = {x} for all x eR is called the fractional part function.
We observe that the domain of the fractional part function is the setR of all real numbers and the
range is the set [0,1).
It is evident from the definition that
/ (x) = {x} = x - [x] for all x e R

ReadYourFlow.COM
FUNCTIONS 2.7

The graph of the fractional part function is as shown in Fig. 2.9.

Y
2

X' -1 o 1 2 X

4 -1 f
r

low
Fig. 2.9 Fractional part function

SIGNUM FUNCTION The function f defined by


I* , x*0
11
1 , x>0
or, f (x) = ■ 0 , x = 0 is called the signum function.

ee
/(*) = x
0 , x=0 -1 , x < 0
rF
Fr
The domain of the signum fimction is the set R of all real numbers and the range is the set
for
{-1,0,1}
ou
The graph of the signum function is as shown in Fig. 2.10.
ks

Y T
oo
Y
B

--- 1
re

(0,1)
ou
ad

*
Y

-*■

X' - 4 -3 -2 -1 0 1
4(0,-!)
nd
Re

t
Fi

Y'
Fig. 2.10 Signum function

EXPONENTIAL FUNCTION Ifa is a positive real number other than unity, then a function that associates
each x <= R to ax is called the exponential function.
In other words, a function /: R R defined by / (a:) = ax, where a > 0 and a * 1 is called the
exponential function.
We observe that the domain of an exponential function is R the set of all real numbers and the
range is the set (0, oo) as it attains only positive values.
As > 0 and a *1. So, we have the following cases.
CASE I When a >\
We observe that the values of y =/ (x) = ax increase as the values of x increase.

ReadYourFlow.COM
2.8 MATHEMATICS-XII

Also,
<1 for x < 0
/ (x) = ax • = 1 for x = 0
>1 for x > 0.
Thus, the graph of / (x) = ax for a >1 as shown in Fig. 2.11.

Y /
y = ax,a> 1
3
--------- L------- L 2-j-

-
(0,1)
* ♦ » ♦

X'-4 -3-2-1 ° 1 2 3 4 X
!------ ------- j-..... i-------i...... —1 .... —............ ..........

low
------ -2

- -3
i r

ee
Fig. 2.11 Exponential function
rF
Fr
We also observe that:
2X <3X <4X <... for all x>0 or
2* = 3* = 4* =... = 1 for x = 0
sf
u
2* >3* >4* >.... for x < 0
k
Yo
oo

So, the graphs of / (x) =2X, f (x) = 3*, / (x) = 4X etc. are as shown in Fig. 2.12.
B

Y
re

t 4 I
! >y = 4r
ou
ad

i 3 —- / yy = y
Y

= 2*
nd

y = 3X
Re

T
(0,1)
Fi

1
* t ♦
X'~4 -3 _2 -l O 1 2 3 4 X
Y-------

Y'
J___I.__ 1___ L....J
Fig. 2.12 Exponential functions on same scale

CASE II W/jch 0 < fl < 1


In this case, the values of y =/ (x) = ax decrease with the increase in x and y > 0 for all x eR.
Also,
>1 for x < 0
y = f (x) = ax = 1 for x = 0
<1 for x > 0
Thus, the graph of / (x) = ax for 0 < rt < 1 is as shown in Fig. 2.13.

ReadYourFlow.COM
FUNCTIONS 2.9

ny - ax,o
i < a <\ Y,
... 4
\
3

2 j-

i--- (0,1)
*
X'-4 -3 -2 -1 0 1 2 3 4 X
I 1I
-2
Y'
Fig. 2.13 Exponential Function

w
The graphs of / (,t) = ax, 0 < fl < 1 for different values of a are shown in Fig. 2.14.

Flo
Y

ee
X—
r
Fr
X
1
n-2
for
ur
x i .X

y-(i)
l y=3
i
1
ks

(0,1) y
Yo
oo

X' o x
eB

———
Y'
r

Fig. 2.14 Exponential functions on same scale


ou
ad
Y

REMARK We have, 2<e <3. Therefore, graph off (x) = ex is identical to that off (x) - ax for a>l and
the graph off{x) =e~x is identical to that of f(x) = ax for 0 <a <1.
nd
Re

LOGARITHMIC FUNCTION If a > 0 and a*l, then thefunction defined by f (x) = log„ x, x>0 is called
Fi

the logarithmic function.


In earlier classes we have learnt that the logarithmic function and the exponential function are
inverse functions.
i.e. logax=y o x = ay
We observe that the domain of the logarithmic function is the set of all positive real numbers i.e.
(0,00) and the range is the set R of all real numbers.
As a>0 and a * 1. So, we have the following cases.
CASE I When a>l
In this case, we have
<0 for 0 < x < 1
y = loga * j = 0 for x = 1
>0 for x > 1

Also, the values of y increase with the increase in x.

ReadYourFlow.COM
2.10 MATHEMATICS-XII

So, the graph of y = logfl a: is as shown in Fig. 2.15.

w
Fig2.15 Logarithmirfuncti on f(x)=logax,a>l
CASE II When 0 < fl < 1
In this case, we have
Flo
ee
>0 for 0 < x < 1

Fr
y = toga * = 0 for x = 1
<0 for x > 1
for
ur
Also, the values of y decrease with the increase in x. So, the graph of y = logfl x is as shown in
Fig. 2.16.
ks
Yo
oo

Y
eB

_ ; 3
r
ou
ad

(1,0)
Y

X' -4 -3 -2 -1
o 2 3 4 X
nd
Re

-1
Fi

----- 2
\
f -3 f- Ax)= logfl x
o<a<l
i. r
Fig2.16 Logarithmicfuncti on/(x) = log^x,0<a>1
Following are some useful properties of logarithmic function:
(i) logfl 1=0, where a>0, a*l
(ii) logfl a = 1, wherea>0,a*\
(iii) logfl (xy) = log,, | x | + logfl | y |, where a>0, a*l and xy > 0
/
(iv) log,, - = logfl | x | - logfl | y |, where a > 0, fl * 1 and - > 0
^y y
(v) logfl (x”) = n logfl | x|, wherea>0, a*l and x” > 0
(vi) tog^ xm = — log| a 11 x |, where a>0, a*l and xm > 0, a” > 0

(vii) xlogfl V = ylo8fl x, where x>0, y>0, fl>0, a*1

ReadYourFlow.COM
FUNCTIONS 2.11

(viii) If a > 1, then the values of / (x) = logfl x increase with the increase in x.
i.e. x <y <=> logrtx < log,, y
<0 for 0 < x < 1
Also, logfl x | = 0 for x=l
>0 for x > 1.
(ix) If 0 < a < 1, then the values of / (x) = logfl x decrease with the increase in x.
i.e. x < y ct> logfl x > logfl y
>0 for 0 < x < 1
Also, logfl x -j = 0 for x=l
<0 for x > 1
1
(x) logfl x = for > 0, * 1 and x > 0, x * 1.

low
logxfl
REMARK Functions f (x) = logfl x and g (x) = ax are inverse of each other. So, their graphs are mirror
images of each other in the line mirror y = x.

ee
1
RECIPROCAL FUNCTION The function that associates a real number x to its reciprocal - is called
rF
Fr
x
1
the reciprocal function. Since — is not defined for x = 0. So, we define the reciprocal function as
x
for
follows:
ou
ks

DEFINITION The function f :R - [0] R defined by /(x) = — is called the reciprocal function.
oo

Clearly, domain of the reciprocal function is R - {0} and its range is also R - {0}.
Y
B

We observe that the sign of 1/x is same as that of x and 1 /x decreases with the increase in x. So,
re

the graph of / (x) = — is as shown in Fig. 2.17.


ou
ad

x
Y

y i
......
nd

3-
Re

* -i

—----
Fi

... 2 i *

X' -8 -6 -4 -2
1 r

° 2 4 6 8 X
-1.............:

-2.

-3-
r
Fig. 2.17 Reciprocal function

SQUARE ROOT FUNCTION The function that associates a real number x to + Vx is called the
square root function. Since Vx is real for x > 0. So, we defined the square root function as follows:
DEFINITION The function f :R+ -> R defined by f (x) = + Vx is called the square root function.
Clearly, domain of the square root function is R" i.e. [0, co) and its range is also [0, co).

ReadYourFlow.COM
r

2.12 MATHEMATICS-XII

We observe that the values of / (x) = + six increase with the increase in x. So, the graph of
/ (x) = + Vx is as shown in Fig. 2.18.

Y
6
4
f 2

■ *
x'8 -6-4-2 2 4 6 8 x
-2

low
r
Fig. 2.18 Square root function

SQUARE FUNCTION The function that associates a real number x to its square i.e. x2 is called the

ee
rF
Fr
square function. Since x is defined for all x eR. So, we define the square function as follows:

DEFINITION The function f: R R defined by f (x) = x is called the square function.


for
Clearly, domain of the square function is R and its range is the set of all non-negative real
ou
ks

numbers i.e. [0, co). The graph off (x) = x is parabola as shown in Fig. 2.19.
oo
Y
B

1/ = x2
re
ou
ad
Y
nd
Re

* O
Fi

X'-8 -6 -2 2 4 6 8 X
-2

-4
Y' [
Fig. 2.19 Square function

CUBE FUNCTION The function that associate a real number x to its cube is called the cube
function. We observe that x is meaningful for all x e R. So, we define the cube function as
follows:
DEFINITION The function f :R R defined by f(x) = x3 is called the cube function.
We observe that the sign of x is same as that of x and the values of x increase with the increase
in x. So, the graph of / (x) = x is as shown in Fig. 2.20. Clearly, the graph is symmetrical in
opposite quadrants.

ReadYourFlow.COM
1

FUNCTIONS 2.13

Y 1
6
4
y = -v3
2

X'-8 -6-4-2 0 2 4 6 8 X
-2
4-4--

-6
..L Y'
Fig. 2.20 Cube function

CUBE ROOT FUNCTION The function that associates a real number x to its cube root x 1/3 is called

w
the cube root function. Clearly, 3 is defined for all x e R. So, we define the cube root function
as follows:

Flo
DEFINITION The function f :R ^ R defined by f(x) = x1^ 3 is called the cube root function.

ee
Fr
Clearly, domain and range of the cube root function are both equal to R.
Also, the sign of x1^ 3 is same as that of x and x1^ 3 increase with the increase in x. So, the graph of
for
ur
1/3 is as shown in Fig. 2.21.
/(x) = x
ks

Y
Yo
oo

3
B
re

l
ou
ad

X'_8
Y

-1 1 8 X
nd
Re

r-2
Fi

....-3- —

i Y'
Fig. 2.21 Cube root function

REMARK 1 A function f: R R is said to be a polynomial function if f (x) is a polynomial in x. For


example, f (x) = x2 - x + 4, g (x) = x3 + 3x2 + -Jl x - l etc are polynomial functions.
P(x)
REMARK 2 A function of the form f (x) = where p (x) and q (x) are polynomials and q (x) * 0, is
q(x)'
P(x)
called a rational function. The domain of a rational function f (x) = is the set of all real numbers,
q(x)
except points where q (x) = 0.
RECIPROCAL SQUARED FUNCTION The function that associates every non-zero real number x to
? 1 . .
the reciprocal of its square xz is called the reciprocal squared function. Clearly, — is defined for
xz
non-zero x. So, we define the reciprocal squared function as follows:

ReadYourFlow.COM
2.14 MATHEMATICS-XII

1
DEFINITION Thefunction f: R - {0} -> R defined by f (x) = — is called the reciprocal squaredfunction.
xz
Clearly, domain of R - {0} and range is (0, co). The graph of / (x) is shown in Fig. 2.22.

y
4
r3 I

X'-8 -6-4-2 4 6 8 X

low
-2
r
Fig. 2.22 Reciprocal squared function f(x) =
x2

ee
rF
SQUARE ROOT RECIPROCAL FUNCTION The function that associates every positive real number x

Fr
to the reciprocal of its square root Vx is called the square root reciprocal function. Clearly, —j= is
for
real for all x > 0. So, we define the square root reciprocal function as follows:
ou
1
DEFINITION The function /:(0, co) —> R defined by f (x) = —= is called the square root reciprocal
ks

function.
oo

Clearly, domain and range of / are both (0, co). The graph of / (x) is shown in Fig. 2.23.
Y
B
re

Y
ou
ad

-4- 6
Y

4-4
nd
Re

• 2 4-
Fi

* -
X'_8 -6-4-2 2 4 6 8 X
-2

-4
! r
Fig. 2.23 Square root reciprocal function

2.2.1 OPERATIONS ON REAL FUNCTIONS


In this section, we shall recall various operations, namely addition, subtraction, multiplication,
division etc. on real functions.
ADDITION Let f: R and g: D2 R be two realfunctions. Then, their sumf + g is defined as that
function from D1 nD2to R which associates each x e D-, r\D2 to the number f (x) + g (x).
In other words, if /:Dj —» R and g'.D2 -» R are two real functions, then their sum/ + g is a
function from D-j n D2 to R such that
(/ + g) (*) =/(*) + £ (*) for all x eDj nD2

ReadYourFlow.COM
FUNCTIONS 2.15

PRODUCT Let /: D-, R and g:D2-> R be two real functions. Then, their product (or pointwise
multiplication) f g is a function from D-^ r\D2to R and is defined as
(/ g) (x) = f(x)g (x) for all xeD^nD;,
DIFFERENCE (SUBTRACTION) Letf :D-± R and g:D2 R be two realfunctions. Then the difference
of gfrom f is denoted by f -g and is defined as
(f ~g)(x) = f(x)-g(x) forallxeD1nD2
QUOTIENT Letf: —>■ R and g :D2 ^ R be two real functions. Then the quotient off by g is denoted

by — and it is a function from D-j n D2 - {x : g (x) =0) to R defined by


g
T
^ (^)
/(*)
for all x nD2 -{x:g (x) = 0}
g(x)

low
Sj

MULTIPLICATION OF A FUNCTION BY A SCALAR Letf :D ^>R baa realfunction and a be a scalar (real
number). Then the product af is a function from D to R and is defined as
(a f) (x) = a f (x) forallxeD.

ee
rF
Fr
RECIPROCAL OF A FUNCTION Iff :D R is a realfunction, then its reciprocalfunction ~ is a function
for f
1 1
from D -{x:f (x) -0} to R and is defined as - (x) =
ou
f) f(x)
ks

REMARK 1 The sum, difference product and quotient are definedfor real functions only on their common
oo

domain. These operations do not make any sense for general functions even if their domains are same,
Y
B

because the sum, difference, product and quotient may or may not be meaningful for the elements in then-
re

common domain.
REMARK 2 For any real function f :D R and n e N, we define
ou
ad

(ff....f)(x) =/(x)/(x).../(x) = [f(x)}n forallxeD


Y

/i-times h-times
nd
Re

2.3 KINDS OF FUNCTIONS


Fi

If /: A B is a function, then /associates all elements of set A to elements in set B such that an
element of set A is associated to a unique element of set B. Following these two conditions we
may associate different elements of set A to different elements of set B or more than one element
of set A may be associated to the same element of set B. Similarly, there may be some elements in
B which do not have their pre-images in A or all elements in B may have their pre-images in A.
Corresponding to each of these possibilities we define a type of a function as given below.
2.3.1 ONE-ONE FUNCTION (INJECTION)
DEFINITION A function f: A Bis said to be a one-onefunction or an injection ifdifferent elements ofA
have different images in B.
Thus, /: A —» B is one-one
<=> a*b=> f(a) * f(b) for all a, b e A
<=> f(a) = f(b) => a = b for all a, e A
ILLUSTRATION l A function which associates to each country in the world, its capital, is one-one
because different countries have their different capitals.

ReadYourFlow.COM
2.16 MATHCMATICS-XN

ILLUSTRATION 2 Let f \ A —> B and g:X —>Y be two functions represented by the following
diagrams:

Fig. 2.24

Clearly, /: A -» B is a one-one function. But, g: X -» Y is not one-one because two distinct


elements and have the same image under function g.
ILLUSTRATION 3 Let A = {1, 2, 3, 4}, B = {1, 2, 3, 4,5, 6] and f: A B be a function defined by
f(x) = x + 2 for all * e A.

w
We observe that / as a set of ordered pairs can be written as / = {(1, 3), (2, 4), (3,5), (4, 6)}
Clearly, different elements in A have different images under function/.
So, / : A -» B is an injection.
Flo
ee
ILLUSTRATION 4 Let A ={1, 2, 3), B = {4,5, 6,7} and let f = {(1, 4), (2,5), (3, 6)} be a function from

Fr
A to B. Then, f (1) = 4, / (2) =5andf (3) =6. Clearly, different elements ofA have different images in B.
So, f is a one-one function. for
ur
Let /: A -» B be a function such that A is an infinite set and we wish to check the injectivity off.
In such a case it is not possible to list the images of all elements of set A to see whether different
elements of A have different images or not. The following algorithm provides a systematic
ks
Yo

procedure to check the injectivity of a function.


oo

ALGORITHM
eB

STEP I Take two arbitrary elements x, y (say) in the domain off.


STEP II Put /(x) =/(y)
r

STEP HI Solve f(x) =/(]/)• If it gives x = y only, then f: A Bis a one-one function (or an injection).
ou
ad

Otherwise not.
Y

NOTE Letf : A —> B and let x, y e A. Then, x = y => f(x) =f(y) is ahvays truefrom the definition. But,
nd

f(x) = f(y) => x - y is true only when f is one-one.


Re

ILLUSTRATION 5 Find whether thefollowing functions are one-one or not:


Fi

(i) f :R R given by f(x) = x3 + 2 for all x e R.


(ii) /: Z -> Z given by f(x) =x2 +1 for all xgZ
SOLUTION (i) Let x, y be two arbitrary elements of R (domain off) such that/(x) = /(y). Then,
/(*)=/(y)=> *3+ 2=y3+ 2=> x3=y3=>x=y
Hence,/is a one-one function from R to itself.
(ii) Let x, y be two arbitrary elements of Z such that/(x) = /(y). Then,
/(x) = /(y) => x2 + 1 = y2 + 1 => x2 = y2 => x = ± y
Here, /(x) = /(y) does not provide the unique solution x = y but it provides x = ±y. So,/is not a
one-one function.
Infact,/(2) = 22 + 1 =5 and/(- 2) =(- 2)2 + 1 = 5. So, 2 and -2 are two distinct elements having
the same image.
NOTE IfA and B are two sets having m and n elements respectively such that m < n, then total numbers
of one-one functions from A to B is nCm x m\.

ReadYourFlow.COM
FUNCTIONS 2.17

2.3.2 MANY-ONE FUNCTION


DEFINITION /l function f: A B is said to be a many-onefunction if two or more elements of set A have
the same image in B.
Thus, /: /l B is a many-one function if there exist x, y e /I such that x * y but /(x) -f(y)-
In other words, f : A -> B is a many-one function if it is not a one-one function.
ILLUSTRATION l Let f: A B and g:X -+Y be two functions represented by the following
diagrams:

low
Clearly, a2 * a4 but f(n2) =f(a4) and x-j * x2 but g (x-]) =g(x2). So, / and g are many-one
functions.
ILLUSTRATION 2 LeM = {-1,1, - 2, 2} and B = {1, 4, 9,16}. Consider f: A -> Bgiven by f(x) = x2.

ee
rF
Fr
Then, f{— 1) =1, /(1) =!,/(- 2) =4 and f(2) = 4. Thus, 1 and-1 have the same image. Similarly, 2 and
-2 also have the same image. So, f is a many-one function.
ILLUSTRATION 3 Consider a function /: Z -> Z given by /(x) = |x| for all x e Z. Then, f is a
or
many-one function because for every a eZ, a* 0, we have
sf
u
a*-a, but \a\=\-a\ => f(a) = f(-a) [••• M =M1
ILLUSTRATION 4 Show that the function f :Z Z defined by /(x) = x2 + x for all x e Z, is a
k
Yo
oo

many-one function.
B

SOLUTION Let x, i/ e Z. Then,


re

/O = /(y),
=> x +x = y +y
ou
ad

=> (x2 — i/2) + (x — y) = 0 => (x - y) (x + y + 1) = 0 => x = y or, y = -x-1.


Y

Since /(x) =/(y) does not provide the unique solutionx = ybut it also providesy = -x -l.This
means that x * y but/(x) = /(y) when y = - x -1. For example, if we put x = 1 iny = - x -1 we obtain
nd
Re

y = - 2. This shows that 1 and -2 have the same image under/ Hence,/is a many-one function.
Fi

2.3.3 ONTO FUNCTION (SURJECTION)


DEFINITION A function f : A Bis said to be an ontofunction or a surjection ifevery element ofB is the
f-image of some element of A i.e., if f{A) = B or range off is the co-domain off.
Thus,/: A -» B is a surjection iff for each b eB, there exists a e A such that f(a) = b.
INTO FUNCTION A function f: A -> B is an into function if there exists an element in B having no
pre-image in A.
In other words, / : A —> B is an into function if it is not an onto function.
ILLUSTRATION l Let f \ A -> B and y : X -> Y be two functions represented by the following
diagrams:

ReadYourFlow.COM
2.18 MATHEMATICS-XII

Clearly, b2 and b5 are two elements in B which do not have their pre-images in A. So, f: A B is an into
function.
Under function g every element in Y has its pre-image X. So,g:X ->Y is an onto function.
ILLUSTRATION 2 Let A = { -1,1, 2, - 2}, B = {1, A] and f : A B be a function defined byf(x) = x2.
Then, fis onto, because f(A) = {/(-1), /(l), f(2), /(- 2)} = {1, 4} = B.
ILLUSTRATION 3 A function f :N ^ N defined by f(x) = 2x is not an onto function, because
f(N) = {2, A, 6,...} ^ N (co-domain). In otherwords, range (/) * co-domain off.
The following algorithm can be used to check the subjectivity of a real function.
ALGORITHM
Let f : A —> B be the given function.
STEP I Choose an arbitrary element y in B.
STEP II Put f(x) = y

w
STEP III Solve the equation f(x) = y for x and obtain x in terms ofy. Let x = g(y)
STEP IV Iffor all values ofy e B, the values of x obtained from x = g(y) are in A, then fis onto.

Flo
If there are some y eB for which x, given by x = g(y), is not in A. Then, fis not onto.

ee
Following illustration will illustrate the above algorithm.

Fr
ILLUSTRATION 4 Discuss the surjectivity of the following functions:
(i) f: R R given by f(x) = x3 + 2 for all x e R. for
ur
(ii) f.R^R given byf(x) = x2 + 2 for all xeR.
(Hi) f:Z^>Z given by f(x) = 3x+2 for all x e Z.
ks
Yo

SOLUTION (i) Let y be an arbitrary element of R. Then,


oo

f(x) = y=^ x3+ 2 =y=>x= (i/ -2)1/3


B

Clearly, for all y eR,(y -2) 1/3 is a real number. Thus, for all y e R (co-domain) there exists
re

1/3 in R (domain) such that/(x) = x3 + 2- y.


x =(y - 2)
ou
ad

Hence, / : R -> R is an onto function.


Y

(ii) Clearly, f(x) =x2 + 2 > 2 for all x e R. So, negative real numbers in R(co-domain) do not
nd
Re

have their pre-images in R(domain).


Fi

Hence,/is not an onto function.


(iii) Let y be an arbitrary element of Z(co-domain). Then,
f(x) =y=> 3x + 2= y=>x= ~~

Clearly, if y = 0, then x = - 2/ 3 g Z. Thus, y = 0 e Z does not have its pre-image in Z(domain).


Hence, / is not an onto function.
ILLUSTRATION 5 Show that the function f :N -*N given by /(!)=/ (2) = 1 and / (x) = x -1 for
every x > 2, is onto but not one-one. [NCERT]
SOLUTION It is given that
=1,2
/«= h x -1, x > 2
Clearly, / (1) =/ (2) =1 i.e. 1 and 2 have the same image.
So, /: N -» N is a many-one function.
Let y be an arbitrary element in N (Co-domain). Then,
/(x) = y=> x-1 = y=> x = y + l

ReadYourFlow.COM
FUNCTIONS 2.19

Clearly, y + 1 e N (domain) for all y e N (Co-domain). Thus, for each y e N (co-domain) there
exists y + 1 e N (domain) such that / (y + 1) = y +1 -1 = y.
So, / : N -» N is an onto function.
illustration 6 Show that the Signum function f :R R, given by
1, if x>0
/(x) = 0, ifx=0
-1, ifx<0
is neither one-one nor onto. [NCERT]
SOLUTION Clearly, all positive real numbers have the same image equal to 1.
So, / is a many-one function.
We observe that the range of / is {-1, 0,1} which is not equal to the co-domain of /. So, / is not
onto.

w
Hence, / is neither one-one nor onto.
2.3.4 BIJECTION (ONE-ONE ONTO FUNCTION)

Flo
DEFINITION A function f \ A -> Bis a bijection if it is one-one as well as onto.

ee
In other words, a function f: A Bis a bijection, if it is
(i) one-one i.e.f(x) =f(y) => x=y for all x,y e A.

Fr
(ii) onto i.e.for all y eB, there exists x e A such that f(x) = y.
ILLUSTRATION! Let f: AB be a function represented by the following diagram:
or
ur
Clearly, f is a bijection since it is both injective as well as surjective.
k sf
Yo
oo
B
re
ou
ad
Y

ILLUSTRATION 2 Prove that thefunction f:Q-^Q given by f(x) = 2x - 3 for all xeQ is a bijection.
nd
Re

SOLUTION We observe the following properties of /.


Fi

Injectivity: Let x, y be two arbitrary elements in Q. Then,


/(*) = /(y) => 2x - 3 = 2y - 3 => 2x = 2y => x = y
Thus, /(x) = /(y) => x = y for all x, y e Q.
So,/is an injective map.
Surjectivity : Let y be an arbitrary element of Q. Then,
y+ 3
/(x) = y=> 2x - 3 = y => x =
2
Clearly, for all y e Q, x = ^ ^ 3 e Q. Thus, for all y e Q (co-domain) there exists x e Q (domain)

y-3 such that /(x) =f[^~^j = 2 ( y~Y^j - 3 = y. That is every element in the
given by x =

co-domain has its pre-image in x.


So,/is a surjection.
Hence, /: Q -> Q is a bijection.

ReadYourFlow.COM
2.20 MATHEMATICS-XII

ILLUSTRATION 3 Show that the function f : R R defined by f(x) = 3-t'1 + 5 for nil x e R is a
bijection.
SOLUTION We observe the following properties of/.
Injectivity: Let x, y be any two elements of R(domain). Then,
f(x) = f(y)=> 3x3 + 5 = 3y3 +5=> x3 = y3 => x = y
Thus, f(x) = f(y) => x = y for all x, y e R.
So,/is an injective map.
Surjectivity : Let y be an arbitrary element of R(co-domain). Then,
1/3
/(x) = y=>3x3 +5 = y^x3 = => x = (
3 3

Thus, we find that for all y e R (co-domain) there exists x = fy -5 I

low
I —^— e R (domain) such that

1/3] 3
y-5 1/3
/(*) = f 3 3
+ 5 = y-5+5 = y

ee
rF
This shows that every element in the co-domain has its pre-image in the domain. So, / is a

Fr
surjection. or
Hence,/is a bijection.
sf
u
ILLUSTRATION 4 Let A ={x e R :-l < x < 1} -B. Show that f: A B given by /(x) =x\x\ is a
bijection.
k
Yo
oo

SOLUTION We observe the following properties of/.


eB

Injectivity: Let x, y be any two elements in A. Then,


x*y=> x| x| *y| y| => f(x) * f(y)
So, /: A -> B is an injective map.
r
ou
ad

Surjectivity: We have.
Y

x2 , if x > 0
f(x) = x|x| =
nd

-x2 , if x <0
Re
Fi

'■y
If 0 < x < 1, then / (x) = x takes all values between 0 and 1 including these two points.
Also, if -1 < x < 0, then / (x) = - x2 takes all values between -1 and 0 including -1. Therefore,
/(x) takes every value between -1 and 1 including -1 and 1. So, range of / is same as its
co-domain.
Hence, /: A -> B is an onto function.
Thus, /: A -> B is both one-one and onto.
Hence, it is a bijection.
A LITER We have.
x2 , if x > 0
fix) = x|x| =
- x2 , if x < 0
9 9
For x > 0, / (x) = x represents a parabola opening upward and for x < 0, / (x) = - x represents
a parabola opening downward.
So, the graph off (x) is as shown in Fig. 2.31.

ReadYourFlow.COM
FUNCTIONS 2.21

X' -1 O X
-4
\j = -x2

1
___
H,-l) r

low
Fig. 2.31 Graph of/(.r) = 3:|a:|

It is evident from the graph of / (x) that / is one-one and onto.


REMARK It follows from the above discussion that if A and B are tzvo finite sets and f :A -> B is a

ee
function, then
rF
Fr
(i) / is an injection => n (A) < n (B)
(ii) / is a surjection => n(B) <n (A)
for
(iii) f is a Injection => n(A)=n(B).
ou
ks

ILLUSTRATIVE EXAMPLES
oo

LEVEL-1
Y
B

EXAMPLE 1 Let A be the set ofall 50 students ofclass XII in a central school. Let f \ A -> N be afunction
re

defined by
f (x) = Roll number of student x
ou
ad

Show that f is one-one but not onto.


Y

SOLUTION Here, / associates each students to his (her) roll number. Since no two different
nd

students of the class can have the same roll number. Therefore, / is one-one.
Re

We observe that / (A) = Range of / = {1, 2, 3, ...,50} * N i.e. range of / is not same as its
Fi

co-domain. So, / is not onto.


EXAMPLE 2 Show that the function f :N ->N, given by f (x) = 2x, is one-one but not
onto. [NCERT]
SOLUTION We observe the following properties of /.
Injectivity: Let x-j, x2 e N such that f(x{) = /(x2). Then,
/ (xa) = / (x2) => 2xa = 2x2 => Xj = x2
So, / is one-one.
Surjectivity: Clearly, / takes even values. Therefore, no odd natural number in N (co-domain)
has its pre-image in domain. So, / is not onto.
EXAMPLE 3 Prove that f : R R, given by f (x) = 2x, is one-one and onto. [NCERT]
SOLUTION We observe the following properties of /.
Injectivity: Let xlf x2 e R such that / (x^) = / (x2). Then,
/ (*i) = / (x2) => 2xl = 2x2 => X1 = x2
So, f :R —» R is one-one.

ReadYourFlow.COM1
2.22 MATHEMATICS-XII

Surjectivity: Let y be any real number in R (co-domain). Then,


/(*) = y => 2x = y => x = l
2

Clearly, ^v e R for any y e R such that / (y


— 2(1— - y. Thus, for each y e R (co-domain)

there exists x = ^ e R (domain) such that / (x) = y. This means that each element in co-domain

has its pre-image in domain. So, /: R R is onto.


Hence, /: R -» R is a bijection.
EXAMPLE 4 Show that thefunction f :R -» R, defined as f (x) = x , is neither one-one nor onto.
INCERT]
SOLUTION We observe that 1 and -1 e R such that / (-1) = / (1) i.e. there are two distinct
elements in R which have the same image. So, / is not one-one.

w
Since / (x) assumes only non-negative values. So, no negative real number in R (co-domain) has
its pre-image in domain of / i.e. R. Consequently / is not onto.

Flo
These facts are evident from the graph of / (x) as shown in Fig. 2.32.

ee
Fr
for
ur
ks
Yo
oo
r eB
ou
ad
Y

Fig. 2.32
nd
Re

EXAMPLE 5 Show that f :R R, defined as /(x) = x3, is a bijection.


Fi

SOLUTION We observe the following properties of /. [NCERT]


Injectivity: Let x,y e R such that / (x) = / (y). Then,
/(*) = /(y) => *3 = y3 => * = y
So, / : R -» R is one-one.
Surjectivity: Let y e R (co-domain). Then,
/(x) = y => x3 = y => x = y 1/3

Clearly, y 1/3 e R (domain) for all y e R (co-domain).


Thus, for each y e R (co-domain) there exists x = y 1/3 e R (domain) such that / (x) = x3 =y.
So, /: R -» R is onto.
Hence, / : R -> R is a bijection.
1
EXAMPLE 6 Show that thefunction f: Rq -> Rq , defined as f (x) = — , is one-one onto, where R0 is
x
the set of all non-zero real numbers. Is the result true, if the domain R0 is replaced by N with co-domain
being same as R0 ? (NCERT]

ReadYourFlow.COM
FUNCTIONS 2.23

SOLUTION We observe the following properties off.


Injectivity: Let x, y e R0 suc^ t^at: / (*) = / (y)- Then,
1
/W =/(y)=> -X - x = y
y
So, /: R0 is one-one.
Surjectivity: Let y be an arbitrary element of R0 (co-domain) such that / (x) = y. Then,
r/ x 1 1
/(x) = y=> - = y => x = —
x y
Clearly, x = - e R0 (domain) for all y e Rq (co-domain). Thus, for each y e Rq (co-domain)
y
1 1
there exits x = — e Rq (domain) such that / (x) = — = y.

low
So, / : R0 -> Rq is onto.
Hence, / : R0 -> Rq is one-one onto.
This is also evident from the graph of / (x) as shown in Fig. 2.33.

ee
T
rF y

Fr
3 for
2
ou
1
ks

X'
*
oo

-8-6-4 0 2 4 6 8 X
Y

-1
B
re

-2 1 ....
ou
ad

XU
Y

Fig. 2.33
nd
Re

1
Let us now consider /: N ^ R0 given by / (x) = —.
x
Fi

For any x, y e N, we find that


1 1
/(x) = /(y) => - = => x = y
x y
So, /: N -> R0 is one-one.
2 3
We find that — , — etc. in co-domain R0 do not have their pre-image in domain N. So,/: N —> R0
3 ' 5
is not onto.
Thus, /": N —> R0 is one-one but not onto.
EXAMPLE 7 Prove that the greatest integer function f: R R, given by f (x) = [x], is neither one-one
nor onto, where [x] denotes the greatest integer less than or equal to x. [NCERT]
SOLUTION We observe that
/(x) = 0 for all x e [0,1)
So, f: R -> R is not one-one.

ReadYourFlow.COM J
2.24 MATHEMATICS-XII

Also, /: R -> R does not attain non-integral values. Therefore, non-integer points in R
(co-domain) do not have their pre-images in the domain. So, / : R R is not onto.
Hence, /: R -* R is neither one-one nor onto.
This is also evident from the graph of the greatest integer function shown in Fig. 2.34.
V*
Y

... 2

■ 1

*
X' -4 -3 -2 -1 1 2 3 4 X
-1
-2

low
-Q

-------- 3
Y'
Fig. 2.34

ee
EXAMPLE 8
rF
Show that the modulus function f :R R, given by f (.t) =| x | is neither one-one nor

Fr
onto. [NCERT]
SOLUTION We observe that / (- 2) = / (2). So, / is not one-one.
for
ou
Also,/ (x) = | x | assumes only non-negative values. So, negative real numbers in R (co-domain)
ks

do not have their pre-images in R (domain).


Hence, / is neither one-one nor onto.
oo

This is also evident from the graph of / (x) = | x | shown in Fig. 2.35.
Y
B
re
ou
ad
Y
nd
Re
Fi

Y'
Fig. 2.35
EXAMPLE 9 Let C and R denote the set of all complex numbers and all real numbers respectively. Then
show thatf :C -> R given by f(z) - \z\forallzeC is neither one-one nor onto.
[NCERT EXEMPLAR]
SOLUTION Injectivity: We find thatzj =1 -i and z2 = 1 + / are two distinct complex numbers in
C such that|z|| =|z2| i.e. z1 *z2 but/(zj) = /(z2).
This shows that different elements in C may have the same image. So,/is not an injection.
Surjectivity: f is not a surjection, because negative real numbers in R do not have their
pre-images in C. In other words, for every negative real number a there is no complex number
z g C such that/(z) =|z| = a. So,/is not a surjection.

ReadYourFlow.COM
FUNCTIONS 2.25

EXAMPLE 10 Show that the function f :R R given by f{x) =ax + b, where a,b eR, a* 0 is a
bijection. [CBSE 2010]
SOLUTION Injectivity : Let x, y be any two real numbers. Then,
f(x) = f(y) => ax + b - ay + b=> ax = ay => x = y
Thus, f(x) = f(y) => x = y for all x, y e R(domain).
So,/is an injection.
Surjectivity: Let y be an arbitrary element of R(co-domain). Then,
y-b
f(x) = y => ax + b = y => x -
a
Clearly, x = -—- e R (domain) for all y e R (co-domain). Thus, for all y e R (co-domain) there
a
y-b
exists x = e R (domain) such that
a

low
'«= '("r) • -f1?)
This shows that every element in co-domain has its pre-image in domain. So,/is a surjection.

ee
Hence,/is a bijection.
rF
Fr
EXAMPLE 11 Show that the function f :R R given by f(x) = cos x for all x e R, is neither one-one
nor onto. for [NCERT EXEMPLAR]
SOLUTION Injectivity: We know that/(0) = cos 0=1 and f(2ii) - cos In = 1.
ou
So, different elements in R may have the same image. Hence,/is not an injection.
ks

Surjectivity: Since the values of cos x lie between -1 and 1, it follows that the range of/(x) is not
oo

equal to its co-domain. So,/is not a surjection.


x —1
Y
B

EXAMPLE 12 Let A = R - (2} and B = R - {!}. Iff: A -> B is a mapping defined byf(x) = , show
x -2
re

that f is bijective.
SOLUTION Injectivity: Let x, y be any two elements of A. Then,
ou
ad

m = f(y)
Y

x —1 y-1
x-2
nd

y-2
Re

=> (x -1) (y - 2) = (x - 2) (y -1) => xy - y - 2x + 2 = xy-x-2y + 2=> x = y


Fi

Thus, /(x) = /(y) => x = y for all x, y & A.


So,/is an injective map.
Surjectivity: Let y be an arbitrary element of B. Then,
x —1 l-2y
/(*) = y=> = y => (x -1) = y (x - 2) => x =
x-2 1-y
1 -2y is a real number for all y * 1.
Clearly, x = —
-y
l-2y
Also, i -2y * 2 for any y, for, if we take — = 2, then we get 1=2, which is wrong.
i-y -y

Thus, every element y in B has its pre-image x in A given by x = i -2y . So,/is a surjective map.
i-y
Hence,/is a bijective map.

ReadYourFlow.COM
2.26 MATHEMATICS-XII

EXAMPLE 13 Let A and B be two sets. Shozv that f:AxB->BxA defined by f (a, b) = (b, a) is a
bijection. [NCERT]
SOLUTION Injectivity: Let (a-^, b{) and (a2, fr2) e ^ x # such that
/(fll/kl) = f{a2'hl)
al) = 0^2' al)
b] = b2 and aj = a2
=> (*!/&!) = (“2'b2)
Thus,/ (a^, b-^ =f(a2, b2) => (av b-^ = (a2, b2) for all (alt bf), (a2,b2) e Ax B.
So, / is an injective map.
Surjectivity: Let (b, a) be an arbitrary element ofBxA. Then,
b eB and a <=A => (a,b) &Ax B.
Thus, for al\(b, a) gBx A there exists (a,b) eAx B such that / (a, b) = (b, a).
So, f: Ax B B x Ais an onto function.
Hence, / is a bijection.

low
EXAMPLE 14 Let A be any non-empty set. Then, prove that the identityfunction on set A is a bijection.
SOLUTION The identity function IA :A-*Ais defined as
IA (x) = x for all x eA.

ee
Injectivity: Let x, y be any two elements of A. Then,
rF
Fr
ia (*) = ^ x = y [By definition of IA ]
So, IA is an injective map. or
Surjectivity: Let y e A. Then, there exists x =y e A such that
sf
u
IA (x) = x = y.
k

So, IA is a surjective map.


Yo
oo

Hence, IA : A -» A is a bijection.
B

EXAMPLE 15 Let f :N - {1} -+Nbe defined by, f(n) = the highest prime factor ofn.
re

Show that f is neither one-one nor onto. Find the range off.
ou

SOLUTION We have,
ad

/ (6) = (the highest prime factor of 6) = 3, / (9) = (the highest prime factor of 9) = 3
Y

and. / (12) = (the highest prime factor of 12) = 3.


nd

So, / is a many-one function.


Re

Clearly, image of any n e N - {1} is the largest prime number that divides n. So, the range of/
Fi

consists of prime numbers only. Consequently, range of / * N (co-domain). So, / is not onto
function.
Hence, / is neither one-one nor onto. The range of / is the set of all prime numbers.
EXAMPLE 16 Let A ={1, 2). Find all one-to-one functions from A to A.
SOLUTION Let /: A -> A be a one-one function. Then, / (1) has two choices, namely, 1 or 2.
So, /(l) = 1 or /(l) = 2.
CASE I When / (1) = 1:
As /: A A is one-one. Therefore, /(2) = 2.
Thus, we have
/(1)=1 and / (2) = 2.
CASEII When / (1) = 2 :
Since /: A -> A is one-one. Therefore, / (2) = 1.
Thus, in this case, we have
/(l) =2 and /(2) =1

ReadYourFlow.COM
FUNCTIONS 2.27

So, there are two one-one functions say / and g from A to A given by
/ (1) =1/ / (2) = 2 and, g (1) = 2, g (2) =1.
ALITER All one-to-one functions from A to itself can be expressed in the following two row
notation as follows:
1 2 1 2
/ = 1 2 'Z = 2 1
First row contains elements of the domain and second row contains the corresponding images.
Clearly, each arrangement of second row provides a one-to-one function from A to itself.
EXAMPLE 17 Consider the identity function :N -> N defined as, IN (x) = x for all x e N.
Show that although is onto but IN + IN :N -+ N defined as
(JN +/N)(x) = JN (x) + IN (x) = x + x = 2x
is not onto. [NCERT]
SOLUTION We know that the identity function on a given set is always a bijection. Therefore,

w
IN :N -^N is onto.
We have.
(IN + Ift) (x) = 2x for all x e N

Flo
ee
This means that under + 1^, images of natural numbers are even natural numbers. So, odd
natural numbers in N (co-domain) do not have their pre-images in domain N. For example, 1,3,5

Fr
etc. do not have their pre-images. So, + /^ : N -» N is not onto.
EXAMPLE 18 Consider thefunction f: [0, tt/2] -> R given by f (x) = sin xandg: [0, n/2)-^ R given
for
ur
by g (x) = cos x. Show thatfand g are one-one, but f + g is notone-one.
[NCERT]
SOLUTION We observe that for any two distinct elements Xj and x2 in [0 , ti:/2]
ks

sin x-j * sin x2 and cos x-j ^ cos x2 [See graphs of / (x) = sin x & / (x) = cos x]
Yo
oo

=> / (x^ * / (x2) and g (x^ # g (x2)


=> f and g are one-one.
B

We have.
re

(/ + g) (x) =f{x) + g{x)=smx + cos x


ou
ad

=> (/ + g) (0) = sin 0 + cos 0° = 1 and (/ + g) ^ j = sin -^ + cos -^ = 1


Y

Thus, ^ ^ ^ but, (/ + g) (0) =(f + g) ^ j • So, / + g is not one-one.


nd
Re
Fi

EXAMPLE 19 Let f :X ->Y be a function. Define a relation RonX given by R = {(a, b):f (a) =f (b)}.
Show that R is an equivalence relation on X. [NCERT, CBSE 2010]
SOLUTION We observe the following properties of relation R.
Reflexivity: For any a eX, we have
f (a) -f (a) => (a, a) e R =$ R is reflexive.
Symmetry: Let a,b e X be such that {a, b) e R. Then,
(a, b) eR => f (a) = f (b) => f (b) =f (a) => (b, a)eR
So, R is symmetric.
Transitivity: Let a,b, c eXbe such that (a, b) e R and (b, c) e R. Then,
{a,b)eR and (b,c)eR
=> /(«) =f(b) and f (b) =f(c)
=> f (a) =/(c)
=> (a, c) eR
So, R is transitive.
Hence, R is an equivalence relation.

ReadYourFlow.COM
2.28 MATHEMATICS-XII

LEVEL-2
EXAMPLE 20 Show that the function f: R -» {x e R: -1 < x < 1} defined by f(x) = -—^—r , x e R is
1 +1 x\
one-one ontofunction. [NCERT]
SOLUTION We have.
———, if x>0
l+x
/w=4s 1 - .Y
, if .y < 0

So, following cases arise:


CASE I When x > 0
X
In this case, we have f (x) = —1—
1+x

w
Injectivity: Let x, y eR such that x > 0, y > 0. Then,
i/
/ M = / (y) => = ^^ => x + xy = y + xy => x = y
1 +x

Flo
ee
So, / is an injective map.

Fr
Surjectivity: When x > 0, we have
/ (x) = - A - > 0 and f (x) < 1
for
ur
l+x
Let y e [0,1) be any real number. Then,
ks

x 1/
Yo

fix) = y => = y => X =


oo

l+x i-y
B

Clearly, x > 0 for all y e [0,1). Thus, for each y e [0,1) there exists x = —> 0 such that/(x) = y.
re
ou

So, / is an onto function from [0,1) to [0,1)


ad
Y

CASEn When x < 0:


In this case, we have
nd
Re

fix) = r~
Fi

1 -x
Injectivity: Let x, y <= R such that x < 0, y < 0. Then,
f(x) = fiy) => => x-xy = y - xy => x = y
1 -x
So, / is an injective map.
Surjectivity: When x < 0, we have
fix) = <0
1 -x
Also, f (x) = —— = -1 + —— >-l
1 -x ' ' 1 -x
-1 </ (x) < 0
Let y e (-1, 0) be an arbitrary real number such that / (x) = y. Then,
= y => x = y
X
fix) =y =>
1 -X 1+y

ReadYourFlow.COM
FUNCTIONS 2.29

Clearly, x < 0 for y e (-1, 0). Thus, for each y e (-1, 0) there exists x = —< 0 such that/(x) = y.
+y
So, f is an onto function from (-1, 0) to (-1, 0).
Hence, f: R {x e R : - l < x <1} is a one-one onto function.
EXAMPLE II Show that the function f : R -> R given by f (x) = x3 + x is a bijection.
SOLUTION Injectivity: Letx, y eR such that
fix) = fiy)
=> x3 + x = y3 + y
=> x3 -y3 + (x -y) = 0
=> (x-y)(x2 + xy + y2 + 1) = 0
=> X -y = o [•.• x2 + xy + y2 > 0 for all x, y e R .-. x2 + xy + y2 +1 > 1 for all x, y e R]

w
=> x = y
Thus, / (x) = / (y) => x-y for all x, y e R.
So, / is an injective map.

Flo
ee
Surjectivity: Let y be an arbitrary element of R. Then,

Fr
f(x) = y => x3 + x = y => x3 + x - y = 0
We know that an odd degree equation has at least one real root. Therefore, for every real value
O
for
ur
of y, the equation x + x - y = 0 has a real root a such that
a3 + a- y = 0=> a3 + a = y => f (a) = y
ks
Yo

Thus, for every y eR there exists a eR such that / (a) = y. So, / is a surjective map.
oo

Hence, / : R -> R is a bijection.


B

EXAMPLE II Show that f :N —> N defined by


re

n ^ , if n is odd
ou
ad

f(n) = 2 J
n .
Y

— .if n is even
2
nd
Re

is many-one onto function. [NCERT.CBSE 20091


Fi

SOLUTION We observe that


1±i = 1 and /(2)=- = 1.
/(l) = 2 J 2
Thus, 1, 2 e N such that 1^2 but / (1) = / (2). So, / is a many-one function.
Surjectivity Let n be an arbitrary element of N.
If n is an odd natural number, then 2n -1 is also an odd natural number such that
2h -1 + 1
/ (2/7-1) = = n
2
If n is an even natural number, then 2n is also an even natural number such that
2/7
/(2«) = y n.

Thus, for every/7 e N (whether even or odd) there exists its pre-image in N. So,/is a surjection.
Hence, / is a many-one onto function.
EXAMPLE!! Shozv that thefunction f: NN given by, f (n) = nl)n for allneN is a bijection.

ReadYourFlow.COM
r

2.30 MATHEMATICS-XII

SOLUTION We have,
/(n) = n-(-l)n for all neN

f(n) = n-l, if n is even


=>
h + 1 , if /? is odd

Injectivity: Let n, m be any two even natural numbers. Then,


/ (M) = f (rri) => n - l = m - l => n = m
If n, m are any two odd natural numbers. Then,
/ (?7) = /(m) => n + 1 =m + l => n = m.
Thus in both the cases, f (n) =f (m) =z> n=m.
If 11 is even and m is odd, then n * m. Also / (ri) is odd and / (w) is even. So, f (ri) * f (m).
Thus, n * m => f {n) * f (m).
So, / is an injective map.

w
Surjectivity: Let n be an arbitrary natural number.
If ;i is an odd natural number, then there exists an even natural number n + 1 such that
f (n + 1) = n + 1 -1 = n

Flo
If n is an even natural number, then there exists an odd natural number (n -1) such that

ee
/(n-l) = n-l +1 n

Fr
Thus, every n e N has its pre-image in N. So, f :N -+ N is a surjection.
Hence, /: N -> N is a bijection.
for
ur
EXAMPLE 24 Let f: N u {0} N kj (0) be defined by
ks

n + 1, if n is even
f(n) =
Yo

n-l , if n is odd
oo

Show that f is a bijection.


eB

SOLUTION / is an injection : Letn, m e N u {0).


r

If n and m are even, then


ou
ad

/ (n) = / (m) => n + 1 =m +1 => n = m


Y

If n and m are odd, then


nd

/ (n) = f (m) ^ n-l =m-1 => n = m


Re

Thus, in both case, we have


Fi

f(n) = f (m) => n - m.


If n is odd and m is even, then/ (n) = n -1 is even and f (m) =m + 1 is odd. Therefore,
n + m => f (n) + f (m).
Similarly, if n is even and m is odd, then
n + m => f (n) + f (ni).
Hence, f is an injection.
/ is a surjection : Let n be an arbitrary element of N u {0}.
If n is an odd natural number, there exist an even natural number n -1 e N u {0} (domain) such
that / (n -1) = n -1 + 1 = n.
If n is an even natural number, then there exists an odd natural number n + 1 e N u {0} (domain)
such that f (n + 1) = « +1 -1 = n.
Also,/(l) =0.
Thus, every element of N u {0} (co-domain) has its pre-image in N u {0} (domain). So, / is an
onto function.

ReadYourFlow.COM
FUNCTIONS 2.31

EXAMPLE 25 Let A be a finite set. Iff :A^>Aisa one-one function, show that f is onto also.
SOLUTION Let A = {ai, a2, ao,,an}. In order to prove that / is onto function, we will have to
show that every element in A (co-domain) has its pre-image in the domain A. In other words,
range of / = A.
Since /: A -» A is a one-one function. Therefore, / (a^, f (a2),(an) are distinct elements of
set A. But, A has only n elements. Therefore, A -{f (af), f (a2),, f i.e. Co-domain = Range.
Hence, /: A -> A is onto.
EXAMPLE 26 Let A be a finite set. Iff: A A is an onto function, show that f is one-one also.
SOLUTION Let A =1^, fl2 / ••• / %}• In order to prove that/is a one-one function, we will have to
show that/ (fl-j), / (a2),..., f (an) are distinct elements of A.
Clearly, Range of / = [f (af), f (a2),...., f (a,,)}
Since / : A -» A is an onto function. Therefore,
Range of/= A => [f {a^, f (a2),...., f (aj = A

low
But, A is a finite set consisting of n elements. Therefore, / (af), f (a2), f {a3),..., f (an) are distinct
elements of A. Hence, / : A —> A is one-one.
EXERCISE 2.1

ee
LEVEL-1
rF
Fr
1. Give an example of a function
(i) which is one-one but not onto.
for
(ii) which is not one-one but onto.
(iii) which is neither one-one nor onto. INCERT EXEMPLAR]
ou
ks

2. Which of the following functions from A to B are one-one and onto?


(i) /j = ((1, 3), (2,5), (3,7)}; A = (1, 2, 3}, B ={3,5,7}
oo

(ii) f2 = 1(2, a), (3, b), (4, c)}; A = {2, 3, 4}, B = fa, b, c)
Y
B

(iii) /3 = {(a, x), (b, x), (c, z), (d, z)}; A = [a, b,c,d\,B = {x, y, z]
re

3. Prove that the function /: N —> N, defined by f(x) =x + j + 1 is one-one but not onto.
ou
ad

4. Let A = {-1, 0,1( and / = [(x, x2): x e A). Show that / : A -> A is neither one-one nor onto.
Y

5. Classify the following functions as injection, surjection or bijection:


(i) f:N->N given by/(j) =x2 (ii) f:Z^>Z givenby/(x) =x2
nd
Re

(iii) /: N -> N given by f(x) =x3 (iv) /: Z -> Z given by/(x) = x3


Fi

(v) f :R R, defined by f(x) =| x| (vi) /: Z -» Z, defined by /(x) = x2 + x


(vii) /: Z —> Z, defined by/(x) =x-5 (viii) f :R -> R, defined by/(x) = sin x
(ix) f:R -> R, defined by f(x) = x3 + 1 (x) /: R -> R, defined by/(x) =x3 - x
(xi) f :R R, defined by/(x) = sin2 x + cos2 x
2x+ 3
(xii) /: Q - {3) -> Q, defined by /(x) =
x-3
(xiii) /: Q Q, defined by /(x) = x3 + 1 (xiv) f: R -> R, defined by /(x) =5x3 +4
(xv) /: R —> R, defined by/(x) = 3 - 4x (xvi) / : R -> R, defined by/(x) =1 + x2
(xvii) /: R R, defined by/(x) = — [NCERT EXEMPLAR]
x2 +1
6. If /: A B is an injection such that range of / = {fl}. Determine the number of elements in A.
x-2
7. Show that the function/ : R - (3) -» R - {!) given by/(x) = is a bijection.
x- 3
[CBSE 2012, NCERT EXEMPLAR]

ReadYourFlow.COM J
2.32 MATHEMATICS-XII

8. Let A =[-1,1]. Then, discuss whether the following functions from A to itself are one-one,
onto or bijective:
(ii)s(x) =1^1 (hi) h(x)=x2 [NCERT EXEMPLAR]
9. Are the following set of ordered pairs functions? If so, examine whether the mapping is
injective or surjective:
(i) {(x, y): x is a person, y is the mother of x]
(ii) {(a,b) :flis a person,I? is an ancestor of fl] [NCERT EXEMPLAR]
10. Let A ={1,2, 3). Write all one-one from A to itself.
11. If /: R —» R be the function defined by / (x) = 4x3 +7, show that / is a bijection.
[CBSE2011]
12. Show that the exponential function /: R -> R, given by / (x) = ex, is one-one but not onto.
What happens if the co-domain is replaced by Rq (set of all positive real numebrs).

w
13. Show that the logarithmic function /: Rg ^ R given by / (x) = logfl x , rt > 0 is a bijection.

Flo
14. If A = [1, 2, 3), show that a one-one function/: A -> A must be onto.
15. If A = [1, 2, 3], show that an onto function/: A -» A must be one-one.
[NCERT]
[NCERT]

ee
16. Find the number of all onto functions from the set A = {1, 2, 3,..., n) to itself. [NCERT]

Fr
17. Give examples of two one-one functions /j and f2 from R to R such that /j + /2 : R -> R,
defined by (/j + /2) (x) = fa (x) + /2 (x) is not one-one.
for
ur
18. Give examples of two surjective function fa and /2 from Z to Z such that fa + /2 is not
surjective.
ks
Yo

19. Show that if fa and /2 are one-one maps from R to R, then the product fax f2:R R
oo

defined by (fa x /2) (x) = fa (x) /2 (x) need not be one-one.


B

20. Suppose fa and f2 are non-zero one-one functions from R to R. Is fl necessarily one-one?
re

fl
ou
ad

Justify your answer. Here, — fl.: R -> R is given by —fl (x) = fi w


for all x g R.
Y

fl fly fl (*)
21. Given A = {2, 3, 4], B - [2, 5, 6, 7). Construct an example of each of the following:
nd
Re

(i) an injective map from A to B


Fi

(ii) a mapping from A to B which is not injective


(iii) a mapping from A to B.
LEVEL-2
22. Show that /: R ^ R, given by / (x) = x - [x], is neither one-one nor onto.
23. Let /: N —» N be defined by
f(n) = nn +-11,, ifif nn isis even
odd

Show that/ is a bijection. [CBSE 2012, NCERT]


________ ANSWERS
2- fl' fl
5. (i) one-one but not onto (ii) Neither one-one not onto
(iii) Injective but not surjective (iv) Injective but not surjective.
(v) Neither an injection nor a surjection (vi) Neither Injective nor Surjective
(vii) Bijective (viii) Neither injective nor surjective

ReadYourFlow.COM
FUNCTIONS 2.33

(ix) Bijective (x) Surjective but not injective


(xi) Neither injective nor surjective (xii) Injective but not surjective
(xiii) Injective (xiv) Bijective
(xv) Bijective (xvi) Neither injective nor surjective.
(xvii) Neither one-one nor onto
6. 1
8. (i) one-one but not onto (ii) neither one-one nor onto
(iii) neither one-one nor onto
9. (i) represents a function which is surjective but not injective
(ii) does not represent a function
10. (i) / (1) =1,/ (2) =2, f (3) = 3; (ii) /(!)=!,/(2) = 3,/(3) = 2;
(iii) / (1) = 2, / (2) = 3, / (3) = 1; (iv) / (1) = 2, / (2) =1, / (3) = 3;
(v) / (1) = 3, / (2) = 2, / (3) = 1 (vi) / (1) = 3, / (2) = 1, / (3) = 2

w
HINTS TO NCERT & SELECTED PROBLEMS
1. (i) /: Z -» Z given by f(x) = 3x + 2 (ii) /: Z ->{0} given by f(x) = | * I
(iii) / : Z -» Z given by f{x) = 2x2 + 1
3. We have, f (x) = x2 + x + 1
Flo
ee
Fr
Injectivity: Let x, y eNbe such that
/(*) =/(y)
for
=> x2 + x + 1 = y2 + y + 1
ur
=> x 2 -y2 + x -y = 0
ks

=> (x-y){x + y + l) =0
Yo
oo

=> x-y = 0 [v x + y + 1 * 0 for any x, y e N]


B

=> * = y
So, / is a one-one function.
re

Clearly, / (x) = x2 + x + 1 > 3 for all x e N.


ou
ad

So, / (x) does not assume values 1 and 2. Therefore, / : N -» N is not an onto function.
Y

4. We have,/(x) =x2, x e {-1, 0,1}


nd

Clearly, /(-!)=/ (1). So, / is not one-one.


Re

Range (/) = (0,1} * A So, /: A -hv A is not onto.


Fi

6. It is given that /: A -» B is an injective map such that range of / is {a}. As / is an injective


map, therefore different elements of A have different images in B. So, A has just one
element.
x -2
7. /:R-{3}->R-{l}isgivenby/(x) =
x-3
Injectivity: Let x, y <=R - (3} be such that
/ (x) = / (y)
x -2 = y-2
=> ------
x-3 y-3
i i i
=> 1 + — = 1+ => x - 3 = y - 3 => x = y
x-3 y-3 x - 3 y-3
So, / is a one-one function.
Surjectivity: Let y be an arbitrary element of R - {1}. Then,
x-2 2-3y
/(x) =y=> = y => x =
x-3 i-y

ReadYourFlow.COM
2.34 MATHEMATICS-XII

Also, ^ = 3 => 1 = 0 which is an absurd result. Therefore, x ^ 3.


Clearly, x e R - (3) for all y g R - {1}.
Thus, for each y g R - {1} there exists x = -—^ g R - {3} such that / (x) = y.
-y
So, / is an onto function.
10. All one-one functions from A =[1, 2, 3} to itself are obtained by arranging elements of the
'1 2 3^j
second row in the two row notation
1 2 3 '
12. For any x, y g R
/ (*) = / (y) => = e'J => x = y
/: R -> R is one-one.
Clearly, range (/) = (0, cc) ^ R. So, / is not onto.
13. f :Rq ^ R is given by / (x) = log,, x, > 0

w
For any x, y e Rq

/. / is one-one.
Flo
/ W = / (y) => iog,, ^ = log,, y => x = y

ee
For each y g R, there exists x = ay & R0+ such that / (x) = logfl ay = y.

Fr
So, / is onto. Hence, / is a bijection.
14. We have, A = (1,2,3) and f: A —» A is a one-one function. Therefore, f (1), / (2), / (3) are
for
ur
distinct elements of A. But, A has three elements only. Therefore, A =[f (1), / (2), / (3)}
i.e., range (/) = A. So, / is onto.
ks

15. We have, A = (1,2,3)


Yo
oo

It is given that / : A -> A is an onto function. Therefore,


B

if (1)// (2),/ (3)) = A


re

=> / (1), / (2), / (3) are distinct elements of A.


=> / : A -» A is one-one.
ou
ad

16. Since every onto function from A to itself is one-one (See example 22). Therefore, total
Y

number of onto functions from A to itself is same as the number of bisections from A to itself,
which is equal to n!.
nd
Re

17. Let /[ : R -> R and /2 : R -> R be given by/j (x) = x and /2 (x) = - x.
Fi

Clearly, fa and /2 are one-one. But, (fa + f2) (x) = x - x = 0 for all x g R is not one-one.
18. Let fa-.Z Z and f2 : Z -> Z be given by fa (x) = x and /2 (x) = - x. Then, fa and /2 are
surjections, but /-[+ f2'.Z Z is not surjection. Because, (f1+ f2) (x) = x - x = 0 for all
x g Z.
19. Take/1(x)=x and /2(x)=x.
20. Take /j: R -> R given by fa (x) = x3 and /2 : R -> R given/2 (x) = x.
22. We have, / (x) = x - [x]
Clearly, / (x) = 0 for all x g Z.
So, /: R -> R is a many-one function.
Clearly, range (/) = [0,1) * R. So, / is an into function.

2.4 COMPOSITION OF FUNCTIONS


Let A, B and C be three non-void sets and let /: A -» B, g: B C be two functions. Since/is a
function from A to B, therefore for each x g A there exists a unique element/(x) g B. Again, since
g is a function from B to C, therefore corresponding to /(x) g B there exists a unique element
g{f x)) g C. Thus, for each x g A there exists a unique element g (f(x)) gC.

ReadYourFlow.COM
FUNCTIONS 2.35

It follows from the above discussion that / and g when considered together define a new
function from A to C. This function is called the composition of/and g and is denoted by go/ We
define it formally as follows:

w
DEFINITION Let f: A B and g:B ->C be two functions. Then a function gof :A^>C defined by
(gof){x) = g(f (x)),for all x e A
is called the composition off and g.

Flo
NOTE 1 It is evident from the definition that gof is defined only iffor each x e A, f(x) is an element of

ee
domain ofg so that we can take its g-image. Hence, for the composition gof to exist, the range off must be a

Fr
subset of the domain of gas shown in Fig. 2.37.
B C
or
ur
A f 8
D
k sf

' m'
Yo

Range of gof
oo

Range
B

x
off
re
ou
ad
Y

Domain of/
Domain of g
nd

Fig. 2.37
Re

NOTE 2 It should be noted that gof exists iffthe range offis a subset of domain ofg. Similarly,fog exists if
Fi

range ofg is a subset of domain off.


NOTE 3 In order to visualize hozvfunctional composition works, let us think gof in terms of an "assembly
line" in which f and g are arranged in series with output f (x) becoming the input ofg as shown in
Fig. 2.38.

X 8
x machine
X
loutl
i £(/■(*))

8ifM)
f
machine 8(fW)

Fig. 2.38

ReadYourFlow.COM
2.36 MATHEMATICS-XII

ILLUSTRATIVE EXAMPLES
LEVEL-1

EXAMPLE 1 Let R be the set of real numbers. Iff :R R ■, f(x) = x2 and g: R R; g(x) =2x + 1.
Then, find fog and gof Also, show that fog * gof.
SOLUTION Clearly, range of/is a subset of domain of g and range of g is a subset of domain off
So, fog and go/both exist.
Now, (go/ )(x) = g (f(x)) = g (x2) = 2 (x2) + 1 = 2x2 + 1
And, (/og)(x) = f(g(x)) = f(2x +1) = (2x + l)2
2x2 + 1 ^ (2x + l)2
gof* fog.
EXAMPLE 2 Let f: R R ; f(x) = sin x and g:R R ; g(x) = x2 find fog and gof

w
SOLUTION Clearly,/og and go/both exist.
Now, (gof)(x) = g(f(x)) = g(sinx) = (sin x)2 = sin2 x
And,
Flo
(fog) (x) = / (g(x)) = / (x2) = sin x2.

ee
EXAMPLE 3 Let f: [2, 3, 4,5} -> {3, 4,5, 9} and g: {3, 4,5, 9) —» {7,11,15} be functions defined as

Fr
/(2) = 3,/(3) =4,/(4)=/(5) =5 and, g(3)=g(4)=7 and g(5) =g(9) =11.
Find gof [NCERT]
for
ur
SOLUTION We have. Range of / = {3, 4, 5}
ks

Clearly, it is a subset of domain of g. So, gof exists and gof : {2, 3, 4,5} -> {7,11,15} such that
Yo

gof (2) =g (/ (2)) = g (3) =7; go/(3) =g (/(3)) =g (4) =7


oo

gof (4) = g (/ (4)) = g (5) = 11 and gof (5) = g (/ (5)) = g (5) = 11


eB

Hence, gof : [2, 3, 4,5} -> {7,11,15} such that gof = {(2, 7), (3, 7), (4,11), (5,11)}
r

EXAMPLE4 Let f :{1, 3, 4} -» {1, 2,5} and g: {1, 2,5} {1, 3} be given by / = {(1, 2), (3,5), (4,1)}
ou
ad

andg = {(1, 3), (2, 3), (5,1)}. Write dozen gof [NCERT]
Y

SOLUTION Clearly, co-domain of / is same as the domain of g. So, gof exists and
gof :{l, 3, 4} -» {1, 3} such that
nd
Re

gof (1) =g (/ (D) =g (2) = 3; go/ (3) =g (/ (3)) =g (5) = 1; gof (4) =g (/ (4)) =g (1) = 3
Fi

Hence, gof: [1, 3, 4} -» {1, 3} such that gof = {(1, 3), (3,1), (4, 3)}.
EXAMPLE 5 Find gofand fog, iff :R ->R andg: R ^ R are given byf(x)=\x\ andg(x) = | 5x - 21.
[NCERT]
SOLUTION Clearly,
gof (x) =g(f (x)) =g (| x|) = | 51 x| -2 | = || |5x-2| , if x >0
-5x-2|,ifx<0
and, fog (x) =f (g (x)) -f (I 5x - 21) = 11 5x - 21 | =| 5x - 21.
EXAMPLE 6 If thefunctionsf andg are given by f = {(1, 2), (3,5), (4,1)} andg = {(2, 3), (5,1), (1, 3)},
find range off and g. Also, write dozen fog and gof as sets of ordered pairs.
SOLUTION We have.
Range off = Set of second components of ordered pairs in / = {2,5,1}
Similarly, Range of g = {3,1}
We have. Domain / = {1, 3, 4} and. Domain g = (2,5,1}
Clearly, Range / c Domain g and. Range g ~ Domain/.
So, fog and go/both exist.

ReadYourFlow.COM
FUNCTIONS 2.37

Now, fog(2) =f(g (2)) =/(3) =5; fog (5) =f(g (5)) =/(l) = 2 and, fog (1) =/ (1)) =/(3) =5.
fog = 1(2,5), (5, 2), (1,5)}
We have.
(1) = g (/(I)) = g (2) = 3; go/ (3) = g (/ (3)) = g (5) = 1 and, gof (4) = g (f (A)) = g (1) = 3
go/ = {(1,3), (3,1), (4, 3)}
EXAMPLE 7 If the function f :R R be given by f(x) =x2 + 2 and g: R -[!) -* R be given by
x
g(x) = —:—. Find fog and gof [CBSE2014]
x-1
SOLUTION Clearly, range/ = domain g and, range g = domain/ So, fog and go/both exist.
A2 X2
X
Now, (fog) (x) = f(g (x)) = / — +2 = 2+2
x-1 x -1 (x-l)
x2 + 2 x2 + 2

w
and. (gof) (x) = g (/ (x)) = g (x2 + 2) =
(x2 + 2) -1 x2 +1
Hence, gof :R R and fog: R - {1} -» R are given by
x2 + 2 x2
Flo
ee
(gof)(x) = and (fog) (x) = 2+2
x2 +1 (x-1)

Fr
R -1 fco defined as /(x) 3x + 4
EXAMPLE8 Iff-R- or be
5x - 7
ur
7 x + 4 . Shoiv that gof = IA and fog = IB, where B = R ~ | ^| an^ A = R “ j^j
sf
defined as g (x) =
5x - 3
k
Yo
oo

[NCERT]
B

SOLUTION It is given that / : A B and g:B A. Therefore, gof :A->A and fog :B B.
?f 3* + 4
re

+4
3x + 4 5x -7 21 x + 28 + 20x - 28 41x
ou

gof(x) = g(f(x)) = g =x
ad

5x -7 3x + 4 15x + 20 -15x + 21 41
Y

5 -3
\ 5x -7 /
nd
Re

gof: A ^ A is such that gof (x) = x for all x e A.


So, gof = I a-
Fi

?f7x + 4> + 4
7x + 4 5x - 3/ .
____ 21x + 12 + 20x -12 41x
Now, fog (x) = f (g(x)) = f = =x
5x - 3 ( 7x + 4 35x + 20 - 35x + 21 41
5 -------- -7
1 5x-3
fog : B —> B such that fog (x) = x for all x e B.
So, fog = lB-
EXAMPLE 9 Iff ■ R -> R is defined by f(x) = x2 - 3x + 2,find f (f(x)). [NCERT]
SOLUTION We have,
/(/(*))= /(x2 -3x + 2)
=> / (/ (x)) = f W)’ Where y = x2 - 3x + 2.
=> f(fW) = y2-3y + 2 [••• f(x) = x2 - 3x + 2 f(y) = y2 - 3y + 2]
=> / (/ (x)) = (x2 - 3x + 2)2 - 3 (x2 - 3x + 2) + 2 = x4 - 6x3 + 10x2 - 3x.

ReadYourFlow.COM
2.38 MATHEMATICS-XII

EXAMPLE 10 If f, g: R —> R are defined respectively by f(x) = x2 + 3x + l, g(x) =2x - 3, find


(i)fog (ii)gof (iii)fof (iv)gog. [NCERT EXEMPLAR]
SOLUTION Clearly, Range/= Domain g and. Range g = Domain/. Therefore,/og, gof fof and
gog all exist.
(i) For any x eR, we have
{fog) (x) = f(g (x)) = f(2x-3) = (2x - 3)2 + 3 (2x - 3) + 1 = 4x2 -6x + l
fog: R -» R is defined by (fog) (x) = 4x2 - 6x + 1 for all x e R.
(ii) For any x eR, we have
(gof) (x) = g(f (x)) = g (x2 + 3x +1) = 2 (x2 + 3x + 1) - 3 = 2x2 + 6x -1
gof : R -> R is defined by (gof) (x) = 2x2 + 6x -1 for all x g R
(iii) For any x eR, we have

w
(fof) (x) = f(f (x)) = / (x2 + 3x +1) = (x2 + 3x + l)2 + 3 (x2 + 3x + 1) +1
(fof) (x) = x4 + 6x3 + 14x2 + 15x + 5
=>
fof :R
Flo
R is defined by (fof) (x) = x4 + 6x3 + 14x2 + 15x + 5 for all x g R

ee
Fr
(iv) For any x eR, we have
(Rog) (x) = g (g (x)) = g (2x - 3) = 2 (2x - 3) - 3 = 4x - 9
or
ur
gog : R -> R is defined by (gog) (x) = 4x - 9.
sf
EXAMPLE ll Let f: Z Z be defined by f (x) = x + 2. Find g :Z Z such that gof = Iz.
SOLUTION We have,
k
Yo
oo

8°f = lZ
=>
B

gof (x) = Iz (x)


for all x g Z
=> g (/ (x)) = x for all x g Z
re

=> g (x + 2) = x for all x g Z


ou
ad

=> g(y) = y-2 for all y eZ, where x + 2 = y


Y

=> g (x) = x - 2 for all x g Z.


Hence, g:Z Z defined by g (x) = x - 2 for all x g Z, is the required function.
nd
Re

EXAMPLE 12 Iff : R -> Rbe defined by f (x) = 2xfor all x eR. Find g:R -> R such that gof = IR.
Fi

SOLUTION We have,
g°f = h
=> gof (x) = IR (x) for all x eR
g (f (*)) = * for all x g R
=> g(2x) = x for all x g R
=> g{y)= \ for all y eR, where 2x = y

rw =f for all x eR.

Hence, g: R -> R given by, g (x) = - for all x e R, is the xequired function.

EXAMPLE 13 Let f, g and h be functions from R to R. Show that:


(i) (/ + g) oh =foh + goh (ii) (fg) oh = (foh) (goh) [NCERT]
SOLUTION (i) Since /, g and h are functions from R to R. Therefore,
(f + g) oh: R -> R and foh + goh: R R

ReadYourFlow.COM
FUNCTIONS 2.39

For any x eR
(if + g) oh) {x) = (f + g) (h {x)) = f(h {x)) +g(h {x)) = foh {x) + goh (x)
if + g)oh =foh + goh
(ii) Clearly, (fg) oh :R -> R and (foh) (goh) :R R such that
\(fg) oh] (x) =(fg) (h (x)) = f(h (x)) g (h (x)) = (foh) (*) (goh) (x)
=> {(fg) oh] (x) = {(foh). (goh)} (x) for all x e R
(fg) oh = (foh) .(goh).
1 ,x>0
EXAMPLE 14 Let f \ R R be the signum function defined as f (x) = - 0 , x=0
-1 ,x<0
and g:R R be the greatest integer function given by g(x) = [x]. Then, prove that fog and gof coincide
in [-1, 0). [NCERT]

low
SOLUTION For any x e [-1, 0), we have
f°8 (x) =f(8 (*)) =/ (M) =/(~ 1) = -1 and, gof (x) =g(f (x)) =g(-l) =[-!] =-1
gof (x) = fog (x) for all x e [-1, 0)
Hence, gof and fog coincide on [-1, 0).

ee
rF
Fr
LEVEL-2
EXAMPLE 15 Let A = {x e R :0 <x < 1). Iff -.A-^Ais defined by
for
x , if x eQ
fix) =
ou
1 - x ,if x e Q
ks

then prove that fof (x) = xforallx e A. [NCERT EXEMPLAR]


oo

SOLUTION Let x e A. Then, either x is rational or x is irrational. So two cases arise.


Y
B

CASE I When x e Q:
re

In this case, we have f(x) = x.


ou

fof(x) = f(f(x)) = f(x) = x [••• fix) = x]


ad
Y

CASE II When x £ Q:
In this case, we have / (x) = 1 - x.
nd
Re

fof(x) = f(f(x))
Fi

=> fof (x) = / (1 - x) [v xeQ /(x) = 1 — x]


=> fof (x) = 1 - (1 - x) = x [y xgQ=>l-xgQ=>/(l-x)=l-(l-x)]
Thus, fof (x) = x whether x e Q or, x g Q.
Hence, fof (x) = x for all x e A.
EXAMPLE 16 Let f:R->R and g:R->R be two functions such that fog (x)= sin x 2 and
gof (x) = sin2 x. Then, find f (x) and g (x).
SOLUTION We have,
fog (x) = sin x and, gof (x) = sin x
=> / (g (x)) = sin (x2) and, g (f (x)) = (sin x)2
/ (x) = sin x and, g (x) = x .
example 17 Iff :R -> R be given by
f (x) = sin2 x + sin2 (x + 7t/ 3) + cos x cos (x + tc/ 3) for all x ER,and g-.R-^R
be such that g (5/4) = 1, then prove that gof: R R is a constant function.

ReadYourFlow.COM
2.40 MATHEMATICS-XM

SOLUTION We have,
/ (x) = sin x + sin (x + ti/ 3) + cos x cos (x + ti/ 3)

=> /«4{ 2 sin 2 x + 2 sin 2 x + — + 2 cos x cos x + —


3 3

=> / (x) = ~ 1 - cos 2x +1 - cos |^2x + ^j + cos ^ 2x + ^ j + cos ^

=> / (x) — ^ ^ - cos 2x - cos f 2x + + cos f 2x + ^


3 3
1 5 | cos (2x) + cos |^2x + + cos ^2x + j
/(*) = X
2 2

=> /w = | |"2cos(2x + f) cos —3 + cos 71 . n


2x + —
3

low
=> f(x) = ^ ^ - cos f 2x + ^ + cos f 2x + -^ - for all x e R.
3 3 4
Therefore, for any x e R, we have

ee
g°f(x) = g(f(x)) = = 1
rF
Fr
Thus, go/ (x) = 1 for all x e R. Hence, go/: R -> R is a constant function.for
EXAMPLE 18 Let / :Z Z be defined by / (ri) = 3n/or cillnsZ and g:Z Z be defined by
ou
— , if n is a multiple of 3
R(h) = for alln e Z.
ks

0 , if 77 is not a multiple of 3
oo

Show that go/ = fz and fog * Iz.


Y
B

SOLUTION Since / :Z Z and g:Z -» Z. Therefore, go/ .Z -±Z and fog: Z -» Z.


re

For any neZ, we have


ou
ad

go/ (77) = g(f («))


Y

=> £0/(77) = £(377)


377
nd
Re

g°f (”) = y = 77 v 377 is a multiple of 3 / (377) = —


3
Fi

=> go/ (n) = 77 for all 77 e Z


=> £0/ = h-
For any n eZ, we have
/Og (77) = f{g (77))

=> fog (77) . Mi) •/ (0)


if 77 is a multiple of 3

, if 77 is not a multiple of 3
^ j , if ?7 is a multiple of 3
=> f°g (77) =
3x0 , if 77 is not a multiple of 3
77, if 77 is a multiple of 3
=> fogiri) =
0 , if 77 is not a multiple of 3
Clearly, fog (n) * n for all 77 e Z. In fact, fog (n) = n only for multiple of 3. So, fog * Iz.

ReadYourFlow.COM
FUNCTIONS 2.41

EXAMPLE 19 Let f :R -*R be a function given by f (x) =ax + b for all x e R. Find the constants
a and b such that fof = IR.
SOLUTION We have,
f°f = h
=> fof (x) = l[i (x) for all x e R
=> / (/ W) = x for all x e R [y Ir (x) = x for all x e R]
=> f (ax + b) = x for all x e R
=> a (ax + b) + b =x for all x e R
(a2 -1) x + ab +b = 0 for all x e R
=> a2-1=0 and ab+b=0 [v (a2 -1) x + (ab + 1?) = 0 is an identity in x]
=> a = ±1 and b (a + 1) = 0
Whena = 1

low
b(a + l) = 0 => 2b = 0 => b = 0
a = 1 and b = 0.
When a = -1
b (a + 1) = 0 for all & e R

ee
a = -1 and b can take any real value.
rF
Fr
Hence, either a=l and fr = 0, or a = -1 and b can take any real value.
EXAMPLE 20 Letf \ A Abe a function such that fof = f. Show thatfis onto ifand only iff is one-one.
or
Describe fin this case.
sf
u
SOLUTION We have, fof = f.
Let / : A -> A be onto. Then, we have to prove that / is one-one.
k
Yo
oo

Let x, y, e A. Then, asf:A —> A is onto there exist a, |3 e A such that


B

/(a) = x and / (p) = y


re

Now, fix) = f(y)


=> /(/(«)) = f if m [Using (i)]
ou
ad

fof (a) = /o/(P)


Y

=> /(a) = /(P) I'M = f]


=> * = y [Using (i)]
nd
Re

So, / is one-one.
Fi

Thus, /: A A is onto => /: A -> A is one-one.


Conversely, let / : A -> A be one-one. Then, we have to prove that / is onto.
Let y be an arbitrary element in A. Then,
fof =f
=> fof iy) = / (y)
=> /(/(y)) = /(y)
/(y) = y [y /: A -> A is one-one]
Thus, for all y e A, there exists y e A such that / (y) = y. Hence, / is onto.
Now, fof = f
=> fof (x) = / (x) for all x e A
=> f(f(x)) = f(x) for all x e A
=> /(a) = a for all a = f (x) e A
Thus, / (x) = x for all x e A
EXAMPLE 21 Letf, g: R -> R be a twofunction defined as f(x) =|x| + xandg(x) = \x\-xforall x e R.
Then, findfog and gof [NCERT EXEMPLAR]

ReadYourFlow.COM
2.42 MATHEMATICS-XII

SOLUTION We have.
x + x = 2x, if x > 0
f(x)=\x\+x = -x + x = 0, if x < 0
x-x = 0, if x>0
and. ^(x)=|x| -x =
-x-x = -2x, if x < 0
The graphs of /(x) and g(x) are shown in Fig. 2.39 (i) and 2.39 (ii) respectively. It is event from
these graphs that range (/) =[0, co) and range (g) =(-oo, 0]. Thus, range (/) c domain (g) and
range (g) e domain (/). So, fog and gof both exist.

w
Flo
ee
Fr
for
ur
Fig. 2.39 (i) Graph of/(x) Fig. 2.39(H) Graph of g(x)
Now,
ks

Mx)=fWx)>{T2x)',«x<0
Yo
oo
eB

2x0=0, if x > 0 'v -2x > 0 if x < 0


=> fog(x) =
2(-2x) =-4x, if x < 0 2x) = 2 (- 2x) = - 4x, if x < 0
r
ou

and, gof(x)=g(f(x)) = l g{2x), if x > 0


ad

g(0) , if x <0
Y

0, if x > 0
=> gof(x) =
nd
Re

0, if x < 0
Fi

=> gof{x) = 0 for all x e R.


2.4.1 PROPERTIES OF COMPOSITION OF FUNCTIONS
THEOREM 1 The composition offunctions is not commutative i.e. fog gof.
PROOF Let f: A B and g: B -> C be two functions. Then, the function gof exists because the
range of/is a subset of the domain of g. But,/og cannot exist unless the range of g is a subset of
domain of/i.e. unless C <= A. As such we find that fog does not exist if C £ A hut fog will be a
function from B to itself if A = C. Thus, if A = C.
f: A ^>B and g: B ^ A=> gof: A -> A and fog :B B
Now, we find that both fog and gof exist but they cannot be equal if A and B are two distinct sets,
which are their domains. However if A = B = C, then both gof and fog exist and both are from A to
itself, even then they may not be equal as shown in Example 1 on page 2.35.
Hence, in general the composition of functions is not necessarily commutative.
THEOREM 2 The composition offunctions is associative i.e. iff, g, h are threefunctions such that (fog)oh
andfo (goh) exist, then (Jog) oh = fo (goh). [NCERT]

ReadYourFlow.COM
FUNCTIONS 2.43

PROOF Let A, B, C, D be four non-void sets. Let h: A B, g: B -> C and /: C -> D be three
functions. Then,
h: A -> B, g: B -> C, f: C -»D
=> fog: B -+ D and h: A —> B
=> (fog) oh: A —» D
Again, h : A-> B, g: B C, f :C-> D
=> f :C and goh : A ->C
=> fo (goh) :A-*D
Thus, (fog)oh and fo(goh) are functions from set A to set D.
Now, we shall show that {(fog)oh} (x) = {fo (goh)} (.v) for all x e A
Let x be an arbitrary element of A and let 1/ e B, z e C such that h (x) = y and g(y) =z.Then,
{(fog) oh] (x) = (fog) {h (x)}
=> {(fog) oh) (x) = (fog) (y) [••• h(x)=y]

w
=> {(fog) oh) (x) = f(g (y))
=> {(fog) oh) (x) = f (z) [••• g(y) =2] ...(i)
And,
=>
{fo (goh)) (x) = f {(goh) (x)}
{fo (goh)) (x) =f{g(h(x)))
Flo
ee
=> {fo (goh)) (x) = f {g (y)} [v h(x)=y)

Fr
=> {fo (goh)) (x) = f (z) [••• g(y)=2] •••(ii)
From (i) and (ii), we have
for
ur
{(fog) oh) (x) = |/o (goh)} (x) for all x e A.
Hence, (fog) oh = fo (goh)
ks
Yo

THEOREM 4 The composition of two bijections is a bijection i.e. if f and g are two bijections, then
oo

gof is also a bijection. [NCERT]


eB

PROOF Let /: A -> B and g: B -> C be two bijections. Then, gof exists such that gof: A C.
We have to prove that gof is injective as well as surjective map.
r
ou
ad

Injectivity: Let x, y be two arbitrary elements of A. Then,


Y

=> (gof) (x) = (gof) (y)


=> g (/ (*)) = g(fiy))
nd
Re

fix) = f(y) [v g is an injective map]


Fi

=> x = y [v f is an injective map]


Thus, (gof) (x) = (gof ) (y) for all x, y e A. So, gof is an injective map.
Surjectivity: In order to prove the surjectivity of gof, we have to show that every element in C
has its pre-image in A i.e. for allz e C, there exists x e A such that (gof) (x) =z.
Let z be an arbitrary element of C. Then,
z eC => there exists y e B such that g(y) =z [y g is a surjective map]
and. y e B => there exists x e A such that / (x) = y [v / is a surjective map]
Thus, we find that for every z e C, there exists x e A such that
(gof)(x)=g(f(x))=g(i/)=z.
i.e. every element of C is the gof-image of some element of A.
So, gof is a surjective map.
Hence, gof being both injective as well as surjective, is a bijective map.
THEOREM 4 Let /: A -> B. Then, folA = 7B o/ =/ i.e. the composition of any function with the
identity function is the function itself

ReadYourFlow.COM
2.44 MATHEMATICS-XII

PROOF Since IA :A A and f: A B, therefore fo IA : A -> B. Now let x be an arbitrary


element of A. Then,
<J0IA)(X) = HIA (*» = /(*) [v IA (x) = x for all x e A]
folA = f
Again, f:A->B and IB :B B => IB of :A B.
Now, let x be an arbitrary element of B. Letf(x) = y. Then, y e B.
(IB of)(x) = IB(f(x))
=> (IBof)(x) = IB (y) [•* fix)=y]
=> (IB of){x) = y
=> (IBof)(x)=f(x) [" (y)=j/forally eB]
h°f=f .
Hence, f oIA =IB of =f
THEOREM 5 Let f: A B, g: B -> A be two functions such that gof = IA. Then, f is an injection and g

w
is a surjection.
PROOF fis an injection: Let x, y e A be such that / (x) = / (y). Then,

=>
fix) = f (y)

Flo
ee
gif(x)) = g if (y))
=> gof (X) = gof (y)

Fr
=> IA ix) = I a iy) [••• gof = IA (Given)]
=> x = y [By definition of IA ]
for
ur
Thus, f (x) =f (y) => x = y for all x, y e A.
ks

So, / is an injective map.


Yo
oo

g is a surjection: Asg:B ^ A therefore to prove that g is a surjection. It is sufficient to prove that


every element in A has its pre-image in B.
eB

Letxbeanarbitrary element of A. Then, as/:A -» Bis a fimction therefore/(x) e B. Let/(x) =y.


Then,
r
ou
ad

giy) = g if ix))
Y

=> giy) = g0fix)


=> g iy) = 1A (x) l-gof = IA1
nd
Re

=> giy) = x
Fi

Thus, for every x e A there exists y =f (x) e B such that g (y) = x. So, g is a surjection.
THEOREM 6 Let f: A -» Bandg:B Abe twofunction such that fog = lB.Then,f is a surjection and g
is an injection.
PROOF fis a surjection : In order to prove that /: A -> B is a surjection, it is sufficient to prove that
every element in B has its pre-image in A. Let b be an arbitrary element of B. Since g: B A.
Therefore, g (b) e A.
Let gib) = a.
f ia) = fig m [••• a=g(k)l
=> fid) = fog{b)
=> /(«) = l Bib) [••• fog = lB\
=> fid) = b
Thus, for every b eB there exists a e A such that / {a) = b. So, / is a surjection,
g is an injection: Let x, y be any two elements of B such that g (x) = g (y). Then,
gix) = giy)
=> f ig M) = fig iy))

ReadYourFlow.COM
FUNCTIONS 2.45

=i> fog (x) = fog (y)


=> h (*) = h (V) [••• fog = IB]
=> x = y
Thus, g (a') = g (y) => x = y for all x, y, e B.
So, g is an injection.
THEOREM 7 Let f: A B and g:B ->C be two functions. Then,
(i) gof :A->C is onto => g:B ->C is onto
(ii) gof : /I —> C is one-one => f: A B is one-one
(iii) gof: A is onto and g:B ^>C is one-one => f: A Bis onto
(iv) gof \ A ->C is one-one and f :A -> Bis onto => g:B -+C is one-one.
PROOF (i) In order to prove that g: B -» C is onto whenever gof : A C is onto, it is sufficient to
prove that for all z e C there exists y e B such that g (y) = z.
Let z be an arbitrary element of C. Since gof : A —» C is onto. Therefore, there exists x e A such

w
that
gof (x) = z
=> g (/ (x)) = Z
=> g (y) = z, where y = / (x) e B.

Flo
ee
Thus, for allz e C, there exists y =f (x) e B such that g (y) =z.

Fr
Hence, g: B C is onto.
(ii) In order to prove that /: A -» B is one-one, it is sufficient to prove that
or
ur
f(x) = f (y) => x=y for all x, y e A.
sf
Let x, y e A such that / (x) = / (y). Then,
f(x)=f(y)
k
Yo

[v g: B ^ C is a function]
oo

g(/(x)) - gif (y))


=> gof (X) = gofiy)
B

=> x = y [•.• gof : A -> C is one-one]


re

Hence, / : A —» B is one-one.
ou
ad

(iii) In order to prove that /: A B is onto, it is sufficient to prove that for all y e B there exists
x e A such that / (x) = y. Let y be an arbitrary element of B. Then,
Y

g(y)eC [v g: B —» C]
nd
Re

Since gof : A -> C is an onto function. Therefore, for any g (y) e C there exists x e A such that
go/(x) = g (y)
Fi

=> g if ix)) = g iy)


fix) = y [y g is one-one]
Thus, for all y e B there exists x e A such that / (x) = y.
Hence, /: A —> B is onto.
(iv) Let yl7 y2, e B such that g (y1) =g (y2). In order to prove that g is one-one, it is sufficient to
prove that y-j = y2.
Since /: A —> B is onto and y^, y2 & B. So, there exist x1 x2 eA such that/(x1)=y1 and
fix2)=V2-
Now, g (yi) = g(y2)
=> gif ixi)) = gif ix2))
=> gof (xj) = gof (x2)
=> X'i = x2 [•.• gof : A -» C is one-one]
=> / (xa) = / (x2) [•.• /: A -> B is a function]
=> yi = y2
Hence, g: B —> C is one-one.

ReadYourFlow.COM
r

2.46 MATHEMATICS-XII

EXERCISE 2.2
LEVEL-1
1. Find gof and fog when f: R R and g: R -> R are defined by
(i) / (x) = 2x + 3 and g (x) = x2 +5 (ii) / (x) =2x + x2 and g (x) =x3
(hi) / (x) = x2 + 8 and g (x) = 3x3 + 1 (iv) / (x) = x and g(x)=|xj
(v) /(x) =x2 + 2x-3 and g(x) = 3x-4 (vi) /(x)=8x3 and g(x) = x 1/3
2. Let / = {(3,1), (9, 3), (12, 4)) and g = {(1, 3), (3, 3), (4, 9), (5, 9)). Show that gof and fog are
both defined. Also, find fog and gof
3. Let / ={(!,-1), (4, - 2), (9, - 3), (16, 4)) and g ={(-1, - 2), (- 2, - 4), (- 3, - 6), (4, 8)}. Show
that go/is defined while fog is not defined. Also, find gof
4. Let A = [a, b, c), B = [u v, iv) and let / and g be two functions from A to B and from B to A
respectively defined as: / = {(a, v), (b, u), (c, zo)}, g = [(ur b), (v, a), {w, c)}.

w
Show that /and g both are bijections and find/og and gof.
5. Find/og (2) and gof (1) when: / : R -» R ; / (x) = x2 + 8 and g: R -» R;g(x) = 3x3 + 1.

Flo
6. Let R+ be the set of all non-negative real numbers. If /: R+ —> R+ and g: R+ —> R are
defined as / (x) = x2 and g (x) = + Vx. Find fog and gof Are they equal functions.

ee
7. Let /: R -> R and g : R -> R be defined by / (x) = x2 and g (x) = x + 1. Show that fog * gof.

Fr
8. Let /: RR and g:R->R be defined by /(x)=x + l and g(x)=x-l. Show that
fog = gof =IR.
for
ur
9. Verify associativity for the following three mappings : f: N ^ Z0 (the set of non-zero
integers), g:Z0 ->Q and /j : Q -> R given by /(x) = 2x, g (x) = 1/x and h (x) = ex.
ks

10. Consider / :N->N, g:N->N and /z: N -> R defined as / (x) = 2x, g (y) = 3y + 4 and
Yo
oo

h (z) = sin z for all x, y,z&N. Show that ho (gof) = (hog) of. [NCERT]
11. Give examples of two functions /: N -> N and g:N -> N such that gof is onto but / is not
B

onto. [NCERT]
re

12. Give examples of two functions /: N -> Z and g: Z Z such that gof is injective but g is
ou

not injective.
ad

[NCERT]
Y

LEVEL-2
nd
Re

13. If /: A -> 6 and g: B C are one-one functions, show that gof is a one-one function.
14. If /: A —» B and g: B are onto functions show that gof is an onto function.
Fi

ANSWERS
1. (i) gof (x) = 4x2 + 12x + 14, fog (x) = lx2 + 13
(ii) gof (x) = (x2 + 2x)3, fog (x) =2x3 + x6
(iii) gof (x) = 3 (x2 + 8)3 + 1, fog (x) = 9x6 +6x3 + 9
(iv) gof (x) = | x|, fog(x)= \x\
(v) g°f (*) = 3x2 + 6x -13, fog (x) = 9x2 - 18x + 5
(vi) gof (x) = 2x, fog (x) = 8x
2. gof = {(3, 3), (9, 3), (12, 9)} fog = {(1,1), (3,1), (4, 3) (5, 3)}
3. gof = {(!,- 2), (4, - 4), (9, -6), (16, 8)}
4. fog = \(u, u), (v, v), (zo, iv)) gof = {(a, a), (b, b), (c, c))
5. fog (2) = 633, gof (1) = 2188

ReadYourFlow.COM
FUNCTIONS 2.47

HINTS TO NCERT& SELECTED PROBLEMS


3. We have. Range g ={-2, - 4, - 6, 8), Domain / = {1, 4, 9,16},
Range / = {-1, - 2, - 3, 4}, Domain# ={-l, - 2, - 3, 4).
Clearly, Range / = Domain # but. Range g cz Domain /. So, /o# is not defined but gof is
defined.
10. We have,
/ (x) = 2x, g (y) = 3y + 4 and h (z) = sin z for all x, y, z e N
••• gof(x) = #(/(*)) = #(2x) = 3 (2x) + 4 = 6x + 4
=i> {ho (gof)} (x) = h{(gof)(x)} = h(6x +4) = sin (6x + 4)
(hog) (x) = h (g (x)) = h (3x + 4) = sin (3x + 4)
{(hog) of} (x) = (hog) (f (x)) = (hog) (2x) = sin 2 (3x + 4) = sin (6x + 4) ...(ii)
From (i) and (ii), we get

w
ho (gof) = (hog) of
11. If f(x) = x + 1 and #(x) = j x -1, if x >1 then / : N -> N is not onto because
1 , if x =1'
Range (f) = N - {1} ^ Co-domain off
Flo
ee
Now, gof (x) = g(f (x)) = g(x + l) = x +1 -1 = x [v x + 1 > 1]

Fr
Clearly, gof, being identity function, is onto.
12. Let f :N ->N and g:Z -» Z be given by / (x) = x and # (x) = | x |. Then, g is not injective as
for
ur
g (-2) =g (2) = 2. We observe that gof: N -» Z is given by
gof(x)=g(f(x))=g(x)=\x\=x [•/ x e N]
ks

Clearly, gof is injective but g is not injective.


Yo
oo

2.5 COMPOSITION OF REAL FUNCTIONS


B

In the previous section, we have learnt about the composition of general functions. We have
re

learnt that if f: A -» B and g \C D, then


gof : A —» D is defined as gof (x) = g (f(x)), provided that Range (f) c Domain (g)
ou
ad

and.
Y

fog : C -> B is defined as fog (x) =f (g(x)), provided that Range (g) c Domain (/)
nd
Re

In case of real functions / and g, even if range of / is not contained in domain of g, then gof is
defined for those elements in domain of / which have their images in domain of#. Similarly, if
Fi

range of # is not a subset of domain of /, then fog is defined for those elements in domain of #
which have their images in the domain off.
Thus, we may define the composition of two real functions as follows:
DEFINITION Letf : D1 -> R and g :D2 R be two real functions. Then,
gof : X = {x e D-j: / (x) e D2} R and, fog: Y = |x e D2 : # (x) e -» R are defined as
gof (x) = g (f(x)) for all x e X and fog (x) = / (#(x)) for all xeY.
REMARK 1 If Range (f) c Domain (#), then gof R and if Range (#) c Domain (f), then
fog-.Di -> R.
REMARK 2 Tor any tioo realfunctions f and #, it may be possible that gof exists butfog does not. In some
cases, even if both exist, they may not be equal.
REMARK 3 If Range (f) n Domain (#) = (j), then gof does not exist. In other words, gof exists if Range
(f) n Domain (#) z (ji Similarly, fog exists if Range (#) n Domain (/) z c(x
REMARK 4 Iff and g are bijections, then fog and gof both are bijections.
REMARK 5 Iff:R-*R and g:R R are real functions, then fog and gof both exist.

ReadYourFlow.COM
2.48 MATHEMATICS-XII

ILLUSTRATIVE EXAMPLES
LEVEL-1

EXAMPLE l Iff: R —> R and g : R R be functions defined by f (x) =x2 +1 and g (x) = sin x,
then find fog and gof
SOLUTION We have,
/ (x) = x2 + l and g (x) = sin x
Now, x2 > 0 for all x e R
=> x2 + 1 > 1 for all x eR
=> f (x) > 1 for a\\ x eR
=> Range (/) = [l,co)
Also, -1 < sin x < 1 for all x s R

w
=> Range (g) = [-1,1]
Clearly, Range (/)=[!, <») c Domain (,§•) and. Range (^) = [-1,1] c Domain (/)
So, gof :R -» R and fog: R -> R are given by

Flo
gof (x) = g (f(x)) = g(x2 + l) = sin (x2 + 1)

ee
and, fog(x) = f (g(x)) = / (sin x) = sin2 x + 1 respectively.

Fr
EXAMPLE 2 Iff : [0, co) -» R and g: R R be defined as f (x) = Vx and g (x) = - x -1, thenfind gof
or
ur
and fog.
sf
SOLUTION Clearly, Domain (/)= [0, oo). Range (/) = [0, =o), Domain (g)=R
and, Range (g) =(-co,-1] [v - x2 < 0 for all x .\ - x2 -1 < -1 for all x e R]
k
Yo
oo

Computation of gof : We observe that: Range (/) = [0, co) e Domain (g)
B

gof exists and Domain (gof) = Domain (/) = [0, co)


Also, (gof)(x) = g(f(x)) = g(Jx) = — (Vx)2 — 1 = — x — 1.
re

Thus, gof : [0, oo) -> R is given by gof (x) = - x -1.


ou
ad

Computation offog: We have. Range (g) = [- co, -1)


Y

Clearly, it is not a subset of domain of/. So,fog does not exist.


nd
Re

EXAMPLES Iff (x) = ex and g (x) = log(, x (x > 0),findfog and gof Isfog=gof?
[CBSE 2002]
Fi

SOLUTION We observe that


Domain (/) = R, Range (/) = (0, co). Domain (g) = (0, oo) and. Range (g) = R.
Computation offog: We observe that
Range (g) = Domain (/)
fog exists and fog: Domain (g) -> R i.e. fog: (0, co) -> R such that
fog (x) = f(g (x)) = / (logt, x) = eloZe x = x
Thus, fog: (0, co) -> R is defined as fog (x) = x.
Computation of gof: We have,
Range (/) = (0, co) = Domain (g)
gof exists and gof: Domain (/) -> R i.e. gof: R R such that
gof (x) = g (f(x)) = g (ex) = logt, ex = x log,, e = x
Thus, gof : R -» R is defined as gof (x) = x
We observe that Domain (gof) ^ Domain (fog),
gof* fog.

ReadYourFlow.COM
FUNCTIONS 2.49

EXAMPLE 4 If f (x) =Jx(x> 0) and g (x) =x2 -1 are two real functions, find fog and gof Is
fog=gofl [CBSE 2002]
SOLUTION We observe that
Domain (/) = [0, co), Range (/) = [0, co). Domain (g) = R
and, Range (g)=[-l/<x>) [v x1 > 0 for all x e R x1 - l > -1 for all x e R]
Computation of gof: We observe that: Range (/) = [0, co) c Domain (g).
gof exists and gof : [0, oo) _> R such that
gof(x) = g(f{x)) = g(yfx) = {yfx)2 -1 = x-l
Thus, gof: [0, co) -> R is defined as gof (x) = x-l.
Computation of fog: We observe that
Range (g) = [-1, co) g Domain (/)
Domain (fog) = (x: x e Domain (g) and g (x) g Domain (/)}

low
Domain (fog) = {x: x g R and g (x) g [0, co)}
=> Domain (fog) = (x: x g R and x -1 g [0, co)}
=> Domain (fog) = {x: x e R and x2 -1 > 0}

ee
=> Domain (fog) = (x: x g R and x < -1 or, x > 1}
rF
Fr
=> Domain (fog) = {x: x < -1 or x > 1}
=> Domain (/og) = (-co, -l]u[l,oo)
for
Also, fog(x) = f(g(x)) = f (x2 —1) = ^/x2 -1
ou
Thus, fog: (— oo, -1] u [1, co) -> R is defined as fog (x) = -Jx2 -1.
ks
oo

We find that/og and gof have distinct domains. Also, their formulas are not same.
Y
B

Hence, fog * gof.


re

EXAMPLE 5 Iff (x) = - and g (x) = 0 are two real functions, show that fog is not defined.
x
ou
ad

SOLUTION Clearly,
Y

Domain (/ ) = R - {0}, Range (/) = R - {0}, Domain (g) = R and. Range (g) = {0}.
Clearly, Range (g) n Domain (/)=<}).
nd
Re

Hence, fog is not defined.


Fi

example 6 Let f (x) = [x] and g (x) = | x |. Find

(hi) (/ + 2g) (-1)

SOLUTION We have,
/ (x) = [x] and g (x) = | x |
Clearly, Domain (/) = R and. Domain (g) = R. Therefore, each of fog, gof and/ + 2g has
domain R.

(i) = g /(f) -/Uf)


-5 5
=g 3
-/
3

*(-2)-/(!) = |-2|-[f = 2-1 = 1

ReadYourFlow.COM
2.50 MATHEMATICS-XII

(ii) W)g)-W(f) =S{/(f)}-/{g(f)}


= g = 1 -1=0

(iii) We have.
(/ + 2g)(-l)= /(-l) + (2g) (-1) = / (-1) + 2^ (-1) = [-1] + 21 -11 = -1 + 2x1= 1.

LEVEL-2
x 1
EXAMPLE 7 Let f and g be real functions defined by f (x) = —1— and g (x) = . Describe the
x+1 x+3
functions gofandfog (if they exist).
SOLUTION We have,

w
1
/(*) = —~r and g (x) =
x+1 x+3

Flo
Clearly, Domain(f)=R - {-1} and. Range(/) = R - {1}
Also, Domain (g) = R - {- 3} and, Range (g) = R - {0}.

ee
Fr
Computation ofgof: We observe that
Range (f) q: Domain (g)
for
ur
Domain (gof) = (x: x e Domain (/) and / (x) e Domain (g)}
ks

=> Domain (gof) = i x: x e R - {-1} and X e R - {- 3} •


Yo

1 x+1
oo
B

=> Domain (gof) = i x e R : x * -1 and — 9^—3


x +1
re

3 x 3
ou
ad

Domain (gof) = x e R : x * -1 and x * — = - 3=> x = —


4 x+1 4
Y

Domain (gof) = R-j--^,-lj


nd
Re

x+1
Fi

x 1
Also, gof (x) = g (f (x)) = g
I\ x + 1 / +3 4x+ 3
x+1
Hence, gof: R - j - , -l|->Ris defined as gof (x) = x+1
4x+ 3
Computation offog: We observe that: Range (g) cz Domain (/)
Domain (fog) = (x: x e Domain (g) and g (x) e Domain (f)}

Domain (fog) = ^ x: x e R - {- 3) and —L— e R _ (_ 1|


x+3
1
Domain (fog) = lx:x * - 3 and *-l
x+ 3
1
Domain (fog) = (x: x - 3 and x ^ - 4} = -l => x = -4
x+ 3
Domain (fog) = {x € R : x ^ - 3, - 4}

ReadYourFlow.COM
FUNCTIONS 2.51

=> Domain (Jog) = R -{- 3, - A)


1

Also, fog (x) = f(g (x)) = /


o =
x+ 3
1
1
x+ 3t +1 x+4
x+ 3

Hence, fog: R - {- 3, - 4} -> R is defined as fog (x) = —~-

3x - 2 prove that / (/ (x)) = x /or flZZ x e R - j ^ j.


EXAMPLE 8 J// (x) =
2x - 3 '
3x - 2 . Clearly, domain (/) = R ~ j^j-
SOLUTION We have, / (x) =
2x - 3
Lety = /(x).Then,

low
3x - 2 = 3y-2
y = => 2xy - 3y = 3x - 2 => x
2x - 3 2y-3
Clearly, x e R for all y e R, y ^ Therefore, Range (/) = ^ ~ j ^ j

ee
rF
Fr
Since, Range (/) = Domain (/). Therefore, Domain (fof) = Domain (/).
Thus, for any x e Domain (/o/) = R “ j ^ j' we ^ave
for
ou
3r3x- 2^
ks

r3x-2"|_ 5{2x-3' -2 9x - 6 - 4x + 6
(fof)(x) =/(/(x)) =/
oo

X
2x - 3 )~ 3x -2 6x - 4 - 6x + 9
Y

v y 2 -------- -3
B

2x - 3
re

Hence, (fof )(x) =f (f (x)) = x for all x e R -


ou
ad
Y

2x +1 1 3
EXAMPLE 9 Iffix) = —---- ,x*~ , then show thatf(f(x)) = provided thatx * ~ J' ~ J
JJ 2x +1 2x + 3'
nd
Re

1
SOLUTION We have, /(x) =
Fi

2x +1
Clearly, domain (/) = R - j - j

Let y = —^— . Then,


2x +1
1-y
y = ------- => 2x + 1 = — => x =
1 O 1 1

2x +1 y 2y

Since x is a real number distinct from - i . Therefore, y can take any non-zero real value.

So, Range (/) = R - {0}.


We observe that Range (/) = R - {0} g; Domain (/) = R - j - j

Domain (/o/) = {x : x e Domain (/) and / (x) e Domain (/)}

ReadYourFlow.COM i
2.52 MATHEMATICS-XII

=> Domain (fof) = lx:xeR- {-I}and/WeR-{-I}}


=> Domain (fof) = 1x ^ and f (x) * - ^ j

=> Domain (fof) = \ x:x * - - and —-— * —1


JJ [ 2 2x + 1 2
Domain (fof) = | a: : x ^ and 1 _3
=> = R-
2' 2
( 1 ^ 1 2x +1
Also, fof(x)=f(f(x))=f
2x +1 /
———1
2x +1
+1
2x+ 3

Thus, /o/: R -1 - - -^j R is defined by fof (x) = 2x + l

w
2x+ 3
2x + 1 1 _3
Hence,/(/(*)) =
2x+ 3
for all \ R,x*--
2' 2

Flo
ee
^ Then, show tfwt (fofof) (x) = x
EXAMPLE 10 Let f (x) =

Fr
+ x2 VI + 3x2
for
^ . Clearly, domain (/) = R.
ur
SOLUTION We have, / (x) =
Vi + x2
ks

In order to find the range of/, we proceed as follows:


Yo
oo

Let f (x)=y. Then,


B

X x2 y
= y2 => x =
re

y =/(*)=> = y =>
V1 + *2 l + x2
ou
ad

Since x takes real values. Therefore,


Y

1 -y2 >0 y2 -1 <0 => ye (-1,1).


nd
Re

Hence, Range (/)=(-1,1)


Fi

Clearly, Range (/) c= Domain /. Therefore, /o/: R -> R and /o/o/ : R -> R.
Now,
(/o/o/) (x) = ((/o/) 0/) (x) = (fof) (f (x))

x X
=> (fofof) (x) = (fof) = ff
■Jl + X2 •Jl + X2
V v /

X X

=> (fofof) (x) = /


x T + 2x2 X
=/
x2 x2 Vl + 3x2
T+ 1+
l + x2 1 + 2x2

EXAMPLE 11 Lef / a rco/ function defined by f (x) = /x -1. Find (fofof) (x). A/so, show that
/o/*/2

ReadYourFlow.COM
FUNCTIONS 2.53

SOLUTION We have, / (x) = ^Jx -l


Clearly, Domain (/) = [1, °o) and Range (/) = [0, oo).
We observe that range (/) is not a subset of domain of/.
Domain (fof) = {x: x e Domain (/) and / (x) e Domain (/)}
= {x: x g [1, oo) and /x-1 e [1, co)}
= {x: x g [1, oc) and /x -1 > 1)
= {x : x e [1, oc) and x > 2} = [2, oo)
Clearly, Range (/) = [0, oo) ct Domain (fof).
Domain ((fof) of) = (x: x e Domain (/) and / (x) g Domain (fof)}
= {x: x g [1, oo) and / (x) g [2, oo))
= (x: x g [1, co) and ^Jx -1 g [2, oo)}
= {x : x > 1 and ^x -1 > 2}

w
= {x: x > 1 and x -1 >4)
=> Domain ((fof) of) = {x: > 1 and x > 5} = [5, oo)
Now,

Flo
(fof )(x) = f (f (x)) = / (/x-1) = J/x-1 -1

ee
Fr
and. (fofof) (x) = ((fof) of) (x)
= (fof)(f(x)) or
ur
= (fof) Qx-1)
ksf
Yo
oo

Thus, fof : [2, co) —> R and fofof : [5, co) are defined as
B

fof(x) = ^x-1 -1 and (fofof) (x) = J^x-1 -1^1


re

2
Now, / 2 (x) = [f (x)]2 = ^x-1 'j = X — 1.
ou
ad
Y

/ 2 : [1, co) -> R is given by / 2 (x) = x -1


nd

Clearly, fof * f 2 ■
Re

x —1
Fi

EXAMPLE 12 Iff (x) = x 5* -1, then show that f (f (x)) = - — provided that x * 0,-1.
x +1 ' x — 1 x
SOLUTION We have, / (x) =
X+1
Clearly,/ (x) is defined for all x g R except x + 1 =0 i.e. x = -l.
Domain (/) = R-{-!).
Let us now find the range of /.
Let y =f(x). Then,
x—1 y+i
y = => X =
x +1 1-y
As x takes all real values other than -1. Therefore, y also takes all real values other than 1.
Range (/) = R - (1)
We observe that Range (/) c2 Domain (/).
Domain (fof) = {x: x g Domain (/) and / (x) g Domain (/))
x —1
= < x: x g R - 1) and gR-(-I)
x +1

ReadYourFlow.COM
2.54 MATHEMATICS-XII

x-1
x:x * -1 and * \ = {x:x * - l and .r ^ 0} = K -{-1, 0}
x+1
x-1
-1
x-1 x+1 -2 1
Now, fof (x) = / (/ (x)) = /
x-1
\ +1/
x +1 2x x
x+1
Thus, fof : R - {-1, 0} -> R is defined as
fof(x) = -- or, /(/(x)) = --
x X
1
Hence, /(/(x)) = —for all x * 0,-1.
x

EXERCISE 2.3

w
LEVEL-1
1. Find fog and gof, if

Flo (ii) /(x) = x2, g(x) = cos x

ee
(i) f(x)=ex, g (x) = loge x

Fr
(iii) /(x) = | x| , g (x) = sin x (iv)/(x) = x + 1, g(x) = e*
(v) /(x) = sin-1 x, g (x) = x2 (vi) / (x) = x + 1 , g (x) = sin x
for
ur
'y
(vii) / (x) = x + 1 , g (x) = 2x + 3 (viii) / (x) = c, c e R , g (x) = sin x
1
ks

(ix)/« = x2 + 2, g(x) =1-


1 -x
Yo
oo

2. Let / (x) = x + x + 1 and g (x) = sin x. Show that fog + go/.


B

3. If / (x) = | x |, prove that fof = f.


re

'y
4. If / (x) = 2x + 5 and g (x) = x + 1 be two real functions, then describe each of the following
ou
ad

functions:
Y

(i) fog (ii) gof (iii) fof (iv) f2


Also, show that fof * f2.
nd
Re

5. If / (x) = sin x and g (x) = 2x be two real functions, then describe gof and fog. Are these
Fi

equal functions?
6. Let /, g, h be real functions given by / (x) = sin x, g (x) = 2x and h (x) = cos x. Prove that
fog=go(fh).
7. Let / be any real function and let g be a function given by g (x) = 2x. Prove that gof =/ + /.
LEVEL-2 |

8. If / (x) = y]l -x and g (x) = log^ x are two real functions, then describe functions
fog and gof.
9. If /: (-tc/2 , k/2) -» R and g: [-1,1] -» R be defined as /(x) = tan x and g (x) = yl - x2
respectively. Describe fog and gof .
10. If / (x) = ^x + 3 and g (x) = x2 + 1 be two real functions, then find fog and gof.
11. Let/be a real function given by /(x) = ^x - 2 . Find each of the following:
(i) M (ii) fofof (iii) (fofof) (38) (iv) f2
Also, show that fof * f2.

ReadYourFlow.COM
FUNCTIONS 2.55

1+ x , 0 < x < 2
12. Let / (x) = . Find/o/.
3 -x , 2 <x < 3
13. If f, g:R -> R be two functions defined as /(x) = | x | + x and g(x) = | x | - x for all x e R.
Then, findfog and gof. Hence, find fog (-3), fog (5) and gof (-2). ICBSE 2016]
ANSWERS
1. (i) fog: (0, oo) -» R given by fog(x) = x, gof \R -> R given by gof (x) = x
(ii) fog :R -> Rgivenby fog (x) = cos" x, go/ : R -> R given by gof (x) = cos x
(iii) fog :R Rgivenby fog (x) = | sin x |, go/: R -> R given by gof (x) = sin | x
(iv) fog: R ^ Rgivenby fog (x) =ex +1, gof: R R given by go/ (x) = ex + 1
(v) fog: [-1,1] -» R given by fog (x) = sin _ 1 (x2),
gof: [-1,1]-> R given by gof (x) = (sin - 1 x)2

w
(vi) fog: R R givenby fog (x) = sin x + 1, gof :R R given by gof (x) = sin (x + 1)
(vii) fog: R ^ R givenby fog (x) = 2x + 4, gof : R R given by gof (x) = 2x +5
(viii) fog :R R givenby fog (x) = c, gof: R -> R given by gof (x) = sin c2

Flo
3x2 - 4x + 2

ee
(ix) fog\R- {1} -> R given by fog (x) =
a-*)2
Fr
x2 + 2
go/ : R -> R given by gof (x) =
or
ur
x2+l
sf
2
4. (i) fog: R -» R is given by fog (x) = 2x +7
k
Yo

(ii) gof : R -» R is given by gof (x) = 4x2 + 20x + 26


oo

(iii) fof: R -> R is given by fof (x) = 4x + 5


B

(iv) / 2 : R -> R is given by f 2 (x) = 4x2 + 20x + 25


re

5. (i) gof : R -» R is defined as gof (x) = 2 sin x


ou
ad

(ii) fog : R -> R is defined as fog (x) = sin 2x. No.


Y

8. fog: (0, e] -» R is given by (fog) (x) = V1 ~ [°ge x


nd
Re

go/:(- co, 1) R is given by (go/) (x) = - log (1 - x)


Fi

9. fog: [— 1,1 ] —> R is defined as fog (x) = tan /l - x2

gof : [- n/4 , k/4] ^ R is defined as go/ (x) = yl - tan2 x

10. fog: R —> R is defined as fog (x) = y x2 + 4 ,


gof : [- 3, oo) -> R is defined as gof (x) = x + 4
11. (i) fof: [6, oo) —> R is given by fof (x) = ^x -2 - 2

(ii) fofof : [38, co) ^ R is given by (fofof) (x) = ^^x^2 - 2 - 2

(iii) 0 (iv) f2 [2, co) ^ R is given by f2 (x) = x - 2


2 + x , if 0 < x < 1
12. fof (x) = j 2 - x , if 1 < x < 2
4 - x , if 2 < x < 3
0, x>0
13. fog (x) = gof (x) = 0, for all x /og(-3) = 12, /og(5) = 0, gof(-2) = 0
4x, x < 0 '

ReadYourFlow.COM
2.56 MATHEMATICS-XII

2.6 INVERSE OF AN ELEMENT


Let A and B be two sets and let/: A -» B be a mapping. As we have discussed earlier that ifaeA
is associated tob eB under the functionf then 'b' is called the/image of 'a' and we write it as
b =f {a). We also say that 'a' is the pre-image or inverse element of 'b' under/and we write
a=r\b).
It should be noted that the inverse of an element under a function may consist of a single
element, two or more elements or no element depending on whether function is injective or
many-one; onto or into. If /: A -> B is a many-one and into function, then the inverse of some
elements of B may or may not exist or the inverse of some element of B may consists of more
than one element. If/is a bijection, then for each b eB, f~ ^ (b) exists and it consists of a single
element only.

w
Flo
ee
Fr
If /is represented by Fig. 2.40 , then we find that
for
ur
r1 (&i)=<(>,/-1 (b2)=a4,rl (b3)=[ai/ a2}, r1 (fc4)=«3,r1 (%)=^
r 1 (b6)=<\> and, /_1 =<},
ks
Yo
oo

ILLUSTRATIVE EXAMPLES
B

LEVEL-1
re

EXAMPLE l If f :Q ->Q is given by f (x) = x2, then find


ou
ad

(i)/_1(9) (ii) / -1(-5) (iii) / ~ 1 (0)


Y

SOLUTION (i) Let / ~ 1 (9) = x. Then,


nd
Re

f(x) = 9 => x2 = 9 => x=± 3.


Fi

f~1(9) = {-3,3).
(ii) Let / _ 1 (-5) = x. Then,
/ (x) = - 5 => x = -5, which is not possible for any x e Q.
/ ~ 1 (_5) = <{>
(iii) Let/ 1(0)=x. Then,
/ (x) = 0 => x2 = 0 => x = 0.
So, /_1(0)={0}.
EXAMPLE 2 If the function f: R —> Rbe defined by f (x) = x2 + 5x + 9, find f -1 (8) fl;jd/_1(9).

SOLUTION Let / ~ 1 (8) = x. Then,

/ (x) = 8 => x2 + 5x + 9 = 8 => x = -5± V2l


which are in R.
2

ReadYourFlow.COM
FUNCTIONS 2.57

-5 + V21 -5-V21
/“1(8) =
2 2

Now, let / 1 (9) = x


f(x)=9
=> x2 + 5x + 9 = 9 => x2 + 5x = 0 => x (x + 5) = 0 =i> x = 0,-5, which are in R
= [0,-5\
EXAMPLE 3 If the function f :C be defined fry / (x) = x2 -1, find / _ 1 (- 5) and / " 1 (8).

SOLUTION Let / -1 (-5) =x. Then,


/(x) = -5 => x2 -1 = -5=> x2 = -4 => x = ^4 => x = ± 2i, which are in C.
/-1(-5) = {2i,-27}

w
Again, let / _ 1 (8) = x. Then,
/ (x) = 8 => x2 -1 = 8 => x2 = 9 => x = ± 3, which are in C.
/ -1 (8) = {- 3, 3}
Flo
ee
Fr
EXAMPLE 4 Let f: R R be defined as f (x) = x +1. Find:
(i) / _1(-5) (ii) / - 1 (26) (iii) 37) or
ur
SOLUTION (i) Let/ 1 (-5) = ar. Then,
sf
/(a:)=-5=>a:2 + l= -5=>a;2=-6=5a:=± ^6, which is not in R.
k
Yo
oo

So,/~1(-5)=<j).
B

(ii) Let/ 1 (26) = x. Then,


re

/ (x) = 26 => x2 + 1 = 26 => x2 = 25 => x = ± 5, which are real numbers


ou
ad

/ ~1 (26) = {-5,5}
Y

(iii) / " 1 {10, 37} = {x eR :f (x) = 10 or/(x) = 37}


nd

= {x e R : x2 +1 =10 or x2 + 1 = 37}
Re

= {x e R : x2 =9 or x2 = 36} = {3, - 3, 6, - 6}
Fi

2.7 INVERSE OF A FUNCTION


Let A and B be two sets and let/: A -» B be a function. If we follow a rule in which elements of B
are associated to their pre-images, then we find that under such a rule there may be some
elements in B which are not associated to elements in A. This happens when/is not an onto map.
Therefore all elements in B will be associated to some elements in A if/is an onto map. Also, if it
is a many-one function then under the said rule an
A B
element in B may be associated to more than one element
in A. Therefore an element in B will be associated to a f
unique element in A if/is an injective map.
It follows from the above discussion that if / : A -> B is a =/ 10/) y=/W
bijection, we can define a new function from B to A which \
associates each element y e B to its pre-image r1
-1 (y) e A. Such a function is known as the inverse of
/ Fig. 2.41
function/and is denoted by f ~l.

ReadYourFlow.COM
2.58 MATHEMATICS-XII

DEFINITION Let f: AB be a bijection. Then a function g:B -> A which associates each element y eB
to a unique element x e A such that f (x) = y is called the inverse off.
i.e., f(x) = y » g (y) = *
The inverse of/is generally denoted by / "1.
Thus, if f :A -» B is a bijection, then/ ~1: B -> A is such that / (x) = y <=> / “ 1 (y) = x.
In order to find the inverse of a bijection, we may follow the following algorithm.
ALGORITHM
Let / : A -» B be a bijection. To find the inverse of/we follow the following steps:
STEP I Put f (x) = y, where y eB and x e A.
STEP II Solve f (x) =y to obtain x in terms ofy.
STEP HI In the relation obtained in step II replace x fry / ~ 1 (y) to obtain the required inverse off
Following examples will illustrate the above algorithm.

w
ILLUSTRATIVE EXAMPLES

Flo
LEVEL-1

EXAMPLE l 7f A = (1, 2, 3, 4), B = {2, 4, 6, 8} and f: A Bis given by f (x) = 2x, then write f and

ee
/“ 1 as a set of ordered pairs.

Fr
SOLUTION Clearly, for
ur
/ (1) = 2 , / (2) = 4, / (3) = 6 and / (4) = 8.
/ = {(1, 2), (2, 4), (3, 6), (4, 8)} which is clearly a bijection.
ks

On interchanging the components of ordered pairs in /, we obtain


Yo
oo

r1 = {(2,1), (4, 2), (6, 3), (8, 4)).


B

EXAMPLE 2 Let S = {1, 2, 3). Determine whether thefunction f: S S defined as below have inverse.
re

Find f ~ \ if it exists.
ou
ad

(i) / = {(!, 1), (2, 2), (3, 3)} (ii) / = {(!, 2), (2,1), (3,1)} (iii) /={(!, 3), (3, 2), (2,1)}
Y

[NCERT]
SOLUTION (i) Clearly, / : S —» S is a bijection. So, / is invertible and its inverse is given by
nd
Re

/-1 ={(1,1),(2,2),(3, 3)).


Fi

(ii) Clearly, / (2) = / (3) = 1. Therefore, / is may-one and hence it is not invertible.
(iii) Clearly, /: S -» S is a bijection and hence invertible. The inverse off is given by
/ - * ={(3,1), (2, 3), (1, 2){.
EXAMPLES Considerf: [1,2, 3} ->{a,b, c] given byf (1) = a, f (2) = b and f (3) =c. Find the inverse
(/ “a) “1 off ~ a. Show thatfj “!) “1 =/. [NCERT]
SOLUTION We have,
/ = {(1, a), (2, fr), (3, c)} -(i)
Clearly, / is a bijection and hence invertible. The inverse off is given by
/_1= {(a,!),(&, 2),(c, 3))
=> (rl)-1= {(1, fl),(2,b),(3, c)} ...(ii)
From (i) and (ii), we have (/ *) 1 =/.
EXAMPLE 4 If f :R R is defined by f (x) = 2x + 7. Prove thatf is a bijection. Also, find the inverse
off.

ReadYourFlow.COM
FUNCTIONS 2.59

SOLUTION Injectivity : Let x, y be any two elements of R. Then,


/ (*) = / (y) => 2x + 7 = 2i/ + 7 => x = y
/is an injective map.
Surjectivity : Let y be an arbitrary element of R(co-domain). Then,
y = f (x)=> y = 2x + 7 => x = y-7
2
y-7 y-7
Clearly, x = — e R for all y g R. Thus, for all y g R (co-domain) there exists x = ^ gR

such that / (x) = y.


In other words every element in R (co-domain) has its pre-image in R (domain). Therefore,/is a
surjective map.
Hence,/is a bijection. Consequently / ~ 1 exists.
Inverse off: Let x e R (domain) and y e R (co-domain) such that / (x) = y. Then,

low
y-7 /-1(y) = -^7
/(x) = y => 2x + 7 = y=> x =
2
Thus, / 1: R -> R is defined as / 1 (x) = for all x g R.

ee
rF
Fr
EXAMPLE 5 Iff :R -> R is a bijection given by f {x) = x3 + 3, find/ - 1 (x).
SOLUTION Let / (x) = y. Then, or
/ (x) = y => x3 + 3 = y => x = (y - 3)1/3 => /"1 (y) = (y - 3) 1/3
sf
u
Thus, / “1 : R -> R is defined as /- 1 (x) = (x - S)1^ 3 for all x g R.
k
Yo
oo

EXAMPLE 6 Let f :R R be defined by f (x) = 3x - 7. Show that fis invertible and hencefind f~1.
B

SOLUTION In order to prove that / is invertible, it is sufficient to prove that / is a bijection.


re

Injectivity: Let x, y eR. Then,


/(*) = /(y) => 3x-7 = 3y-7 => x = y
ou
ad

Thus, / (x) = / (y) => x = y for all x, y g R. So,/ is an injection.


Y

Surjectivity: Let y be an arbitrary element of R. Then,


nd

y+ 7
Re

/ (x) = y => 3x -7 = y X =
3
Fi

Clearly, ^ +— g R for all y g R. Thus, for all y g y+7


R there exists x = £— g R such that

/w=/ry±i) = 3M-7 = y
So, / is a surjection.
Hence, / : R R is a bijection. Consequently, it is invertible.
Let/(x) = y. Then,
/(x) = y => 3x -7 = y => x =
3 J
/-Ly) = 3

Therefore,/ 1: R R is given by / 1 (x) = —+—.


3
3
EXAMPLE 7 Show that f: R -10) —> R - {0} giucn by f (x) = — is invertible and it is inverse of itself.

SOLUTION In order to prove that / is invertible, it is sufficient to show that it is a bijection.


fis an injection: Let x, y eR - {0}such that / (x) = / (y). Then,

ReadYourFlow.COM
2.60 MATHEMAHCS-XII

fix) =f(y)=> - = -=> x = y


xy
Thus, f {x) =f {y) => x=y for all x, y e R - {0}.So,/ is an injection.
fis a surjection: Let y be an arbitrary element of R - {0}. Then,
3
/M=y => - = y=>x = -
X y
Thus, for eachy eR - {0}, there exists x = — e R - {0}such that / (*) = /[ — | 3
=y-
y \y) 3/y
So, / is a surjection.
Hence,/ is a bijection. Consequently, it is invertible.
Let / {x) = y. Then,

w
/(*)=y=>- = y=>* = -=>/ 1 (y) = -
y y

Flo
Q

Thus, / _ 1 is given by /- 1 (x) = — for all x eR - {0}.


x

ee
Clearly, / (x) =/ - 1 (x) for all x eR - {0}. Hence, / is inverse of itself.

Fr
EXAMPLE 8 Let /: N u {0} -» N u {0} be defined by or
ur
n + 1 ,ifn is even
/(”) =
sf
n-1, if n is odd
k

Show thatfis invertible and f =f “1.


Yo

[CBSE 2014, NCERT]


oo

SOLUTION In Example 24 on page 2.31, we have proved that/ is a bijection. So, it is invertible.
B

In order to find /" \ let n, m e N u {0} such that


re

f(n) = m
ou
ad

=> n + 1 = m, if « is even
Y

n-1 = m, if n is odd
m -1 , if m is odd If n is even, then « + 1 = w is odd
nd
Re

=> n =
m + 1 , if m is even If n is odd, then m -1 = w is even
Fi

=> /_1w = { mm +1,


-1 , if m is odd
if m is even
« + 1 , if h is even
Hence, / -1 (n) =
w -1 , if n is odd
Clearly,/ = / 1.

2.7.1 PROPERTIES OF INVERSE OF A FUNCTION


THEOREM 1 The inverse ofa bijection is unique. [NCERT]
PROOF Let /: .A —» B be a bijection. If possible, let g: B —» A and /z: B —» A be two inverses of f.
We have to prove thatg = h. In order to prove this it is sufficient to show that g (y) = h (y) for all
y e B. Let y be an arbitrary element of B.
Let g (y) = xx and b (y) = x2. Then,
8 (y) = xl=>f(x1) = y [v g is inverse of/]
and h(y) = x2^f (x2) = y [v /z is inverse of/]

ReadYourFlow.COM
FUNCTIONS 2.61

/ (Xj) = / (x2)
=> X1 = x2 [v / is one-one]
=> g(y) = My)
Thus, g (y) = h (y) for all y e B. Hence, g = h
THEOREM 2 The inverse of a Injection is also a bijection.
PROOF Let / : A -> B be a bijection and let g: B ,4 be its inverse. We have to show that g is a
bijection.
Injectivity ofg: Let y:, y2, e B such thatg (yf) = xl and g (y2) = x2.
Since g is the inverse of/.
g (yi) = => / (^i) = yi and g (y2) = x2=> f (x2) = y2.
Now, g (yj) = g (y2) => x1 = x2 => f (xf) = f (x2) => y2 = y2
g is an injective map.
Surjectivity of g: In order to prove that g: B —».A is a surjection, we have to show that every

low
element in A has its pre-image in B under funtion g.
So, let x be an arbitrary element of A. Then,
x eA
=> There exists y e B such that f (x) =y [v / is a function from A to B]

ee
=> There exists y e B such that g (y) = x [v g is inverse off]
rF
Fr
Thus, for each x e A, there exists y e B such that g (y) = x. So, g is a surjective map.
Hence, g is a bijection.
or
THEOREM 3 If f: A —> B is a bijection and g:B -> A is the inverse off then fog =IB and gof = IA,
sf
u
where IA and IB are the identity functions on the sets A and B respectively.
k
Yo

PROOF In order to prove that gof -1^ and fog = Ig, we have to prove that
oo

(gof) (x) = x for all x e A and (fog) (y) = y for all y e B


B

Let x be an element of A such that/(x) = y. Then,


re

g(y) = * [y g is inverse off]


ou

Now, (go/)(x) = g(/(x)) = g(y) = x


ad

=> (gof) (x) = x for all x e A


Y

g°f = lA-
nd

We have.
Re

(fog) (y) = / (g(y)) = / (x) = y


Fi

=> fog (y) = y for all y e B


=> fog = IB-
Hence, gof - 1^ and fog = IB-
REMARK In the above property, if we have B = A. Then, we find that for every bijectionf: A -> A there
exists a bijection g: A —> A such that fog = gof = IA.
THEOREM 4 Iff : A —> B and g: B —>C are two bijections, then gof: A —> C is a bijection and
(so/)'1 =riog-\ [NCERT]
PROOF We have,
/ : A -» B is a bijection
=> gof: A -> C is a bijection => (gof) 1: C -» A exists.
g: B C is a bijection
Again,
/ : A —> B is a bijection => /_1 : B -» A is a bijection
• => f~l og-'-.C ^A
g: B -> C is a bijection => g-1: C -> B is a bijection
Let x e A, y e B and 2 e C such that/(x) = y and g(y) =z. Then,

ReadYourFlow.COM
2.62 MATHEMATICS-XII

(gof) (X) = g (f(x)) = g{y) = Z


=> (gt)/r1(z) = ^ -0)
Now, f(x) = y and g{y) = z
=> /_1(y) = ^ and, g~\z) = y
(rV1)^) = r\g-\z)) = r\y) - x ...(h)
From (i) and (ii), we have
(gof) ~ 1 (z) = (f~ log ~ T) (z) for allz e C.
Hence, (gof) -1 = /r-l°g-1 ■
THEOREM 5 Let f: A Bandg: B Abe twofunctions such that gof = IA and fog = IB. Then, fand
g are bijections and g=f
PROOF fis one-one : Let x,y e A such that/(.r) =f(y)- Then,
f(x)=f(y)

low
=> g(f(x)) = g(fiy))
=> (gof) (x) = (gof) (y)
=> ia (*) = ia (y) g°f = IA]

ee
=> x = y
rF
Fr
/is a one-one map.
/ is onto : Let y eB and let g(y) = x. Then, or
g(y)=x
=>
sf
f(g(y)) = f(x)
u
=> (fog) (y) = f(x)
k
Yo

=> h (y) = m [••• fog =


oo

=> y = /(*) [••• h (y) = y]


eB

Thus, for each y e B, there exists x eA such that/(x) = y. So,/is onto.


Hence,/is a bijection.
r
ou
ad

Similarly, it can be proved that g is a bijection.


Y

Now we shall show that g =/~1.


Since /: A -> B is a bijection. Therefore, /-1 exists.
nd
Re

Now, fog = Ig
Fi

=> f o(fog) = f~l o I B


(/ “ V) og = f~1oIB [By associativity]
^A O g = f~1 Ol B [••• f'1of = IA]
=> g=r1 lA°g = g and /-1 oIB = f ~ l]
Hence, g=/_1
THEOREMS Let f:A->B be an invertible function. Show that the inverse of f 1 isf.
i.e., (f-yi=f. [NCERT]
SOLUTION Since inverse of a bijection is also a bijection. Therefore, /“ 1 : B —> is a bijection
and hence invertible. As/“ 1 : B -* A is a bijection. Therefore, (/ _ 1)_ 1 - A -> B is also a bijection.
Let x be an arbitrary element of A such that / (x) = y. Then,
f~l(y)=x [•.• / 1 is the inverse of /]
(T1)-1 (x)=y ['•' (/ ~ 1)_1 is the inverse of / ~ !]

ReadYourFlow.COM
FUNCTIONS 2.63

=> (/'V1 (*)=/(*) ['■■ / (*) = y]


Since x is an arbitrary element of A. Therefore,
(/" V1 (x) =/ (x) for all x g A
Hence,
ALITER Since f: A Bis invertible and / _ 1: B A is inverse of/.
/" V = anc^ /°/~ 1 = ^ B
=> / is inverse off -1
=> /=(r1)-1
REMARK 1 Sometimes f: A B is one-one but not onto. In such a case f is not invertible. But,
f :A -> Range (f) is both one and onto. So, it is invertible and its inverse can be found.
REMARK 2 Theorem 5 suggests us an alternative method to prove the invertibility of a function. It states

low
that iff-.A^B and g:B A are two functions such that gof = IA and fog = IB, then fand g are inverse
of each other.
Theorem 5 suggests the following algorithm to find the inverse of an invertible fimction.
ALGORITHM

ee
STEP I
rF
Obtain the function and check its bijectivity.

Fr
STEP II Iff is a Injection, then it is invertible.
In order to find the inverse off, put fof -l (X) = x=* f(f~l(x))
1 = X
or
sf
STEP III Use the formula for f (x) and replace x by f ~ 1 (x) in it to obtain the LHS off (/ - 1 (x)) = x.
u
Solve this equation for f ~1 (x) to get / “ 1 (x).
k
Yo
oo

Following examples will illustrate the above algorithm.


B

ILLUSTRATIVE EXAMPLES
re

LEVEL-1
ou
ad

EXAMPLE l Prove that thefunction f : R R defined as f (x) = 2x - 3 is invertible. Also, find f 1.


Y

SOLUTION In order to prove that / is invertible, it is sufficient to show that / is a bijection.


nd
Re

f is one-one : Let x,y eR. Then,


/ (x) = f (y) => 2x - 3 = 2y - 3 => x = y
Fi

Thus, /(x)=/(y) => x = y for all x, y e R.


So, / is one-one.
fis onto : Let y be an arbitrary element in R (co-domain of /). Then,
y+ 3
/ (x) = y => 2x - 3 = y => x =
2
Clearly, x = ^ ^ e R (domain) for all y e R (co-domain). Thus, for each y eR there exists x eR

such that / (x) = y. So, / is onto.


Since /: R —» B is one-one and onto both. So, it is a bijection and hence invertible.
Now,
fof ~1 (x) = x
/ (/ “1 (x)) = x

=> 2 / ' 1 (x) - 3 = x [V / (x) = 2x - 3]

ReadYourFlow.COM
2.64 MATHEMATICS-XII

/-v> = ^
Thus, / _ 1: R -> R is given by / l/ \ *+ 3
(*) = -^ for all x e R.

EXAMPLE 2 Shozu that the function f: R R is given by f (x) =x +1 is not invertible.


SOLUTION We have, f (x) = x2 + 1.
Clearly, -2^2 but,/ (- 2) = / (2) =5.
So, / is not a one-one function. Hence, / is not invertible.
EXAMPLE 3 Shozv that /: R - {-1} -» R - {1} given by f (x) = —j- is invertible. Also, find f ^.

SOLUTION In order to prove the invertibility of f (x), it is sufficient to show that it is a bijection.
f is one-one: For any x, y e R - {-1}
y

w
/(*) = / (y) => — — => xy + x = xy + y => x = y.
x+l y+i
So,/is one-one.
/ is onto: Let y eR - {!}. Then,

Flo
ee
fix) = y => x = x = T1—
V

Fr
x+l i-y
Clearly, x e R for all y e R - (1). Also, x * -1. Because,
for
ur
x = -l=>^- = -l=>y = -l+y, which is not possible.
i-y
ks

Thus, for each y e R - {!) there exists x = ^ e R - {-1} such that


Yo
oo

i-y
B

y
re

/W = ^ i-y
x+l = y
^+1
ou
ad

i-y
Y

So, / is onto.
Thus, / is both one-one and onto. Consequently it is invertible.
nd
Re

Now,
Fi

-1 (x) = x for all x e R - (1}

/ ~ 1 (*)
=> /{/_1W) = X => = X => / " 1 (x) = for all x e R - {1}.
f-\x) + l
X .
EXAMPLE 4 Shozv that / : [-1,1] -» R, given by f (x) = — is one-one. Find the inverse of the
function/: [-1,1] -+ Range (/). x+2 [NCERT]
SOLUTION Let x, y be any two elements of [-1,1]. Then,
f(x)=f(y) => ^ xy + 2x=xy + 2y => x=y
So, /: [-1,1] -+ Range (/) is one-one.
Obviously, / : [-1,1] -» Range (/) is onto and hence invertible. Let / ~ 1 denote the inverse of/.
Then,
fof 1 (x) = x for all x e Range (/)
=> /tr1 (*))=*

ReadYourFlow.COM
FUNCTIONS 2.65

=> /1w =X
/ -1 (*) + 2
=> f~l(x)=xf~1 (x) + 2x
l 2x
=> /_AW = 1-x
-1 2.v
Hence,/-1: Range(/) :[-l, 1] is given by / (*) =
1-x
EXAMPLE 5 Let f: R —> R be defined as f (x) = lOx + 7. Find the function g:R R such that
gof=fog = IR. [NCERT, CBSE 2011]
SOLUTION We have.
fog = Ir
fog (x) = IR (x) for all x e R

low
=> / (g(x)) = x for all x e R
=> 10 g(x) + 7 = x for all x e R
x -7
=> g(x) = for all x e R
10

ee
A LITER We have,
rF
Fr
fog - gof = IR => g is the inverse of /
Let/(x) = y.Then,
y7 x -7
-=> rV') =
for
10x + 7 = y=> x = => / “1 (y) = ^
10 10 10
u
x -7
ks

Hence, g(x) =
Yo

10
oo
B

LEVEL-2
re

EXAMPLE 6 1/ the function /: [1, oo) -> [1, co) defined by f(x) = 2'v(v - 11 is invertible, find f -1 (x).
ou
ad

SOLUTION It is given that / is invertible with /-1 as its inverse.


Y

(fof ~ 1) (x) = x for all x e [1, oo)


nd

=> / (/ “ 1 (x)) = x
Re

2f~1(x)\rl(x)-l} = x
Fi

=>
-1 (x)-ll = log2 X
=> (f ~1 Wl2 1 (*) - l°g2 X = o
1 ± ^1 + 4 log2 x
=> /-1 (X) =
2
1 + ^1 + 4 log 2 x
=> /"V) = 2
EXAMPLE 7 Find the value of parameter a for which thefunction f (x) = 1 + a x, a * 0 is the inverse
of itself.
SOLUTION Clearly, / (x) is a bijection from R to itself.
Now,
x —1
fof 1 (x) = x => / (/ 1 (x)) = x => 1 + a / 1 (x) = x => / -1 M =
a
It is given that

ReadYourFlow.COM
2.66 MATHEMATICS-XII

/ (.t) = /
1 (x) for all x € .R
x —1
=> 1+ax = for all x e R
a
=> ax+1 = j x + ^^ j for all x e R

2
a = — and 1 = - — => a = 1 and a = -1 => a = -1.
a a
EXAMPLE 8 Let f :N -±Y be a function defined as f (x) = 4x + 3, where
Y ={y <zN :y = 4x + 3 for some x e N]. Show thatf is invertible. Find its inverse. [NCERT]
SOLUTION In order to prove that / is invertible, it is sufficient to show that it is a bijection.
fis one-one: For any x, y eN, we find that
/(*)=/(j/) => 4x + 3 = 4y + 3 => x=y
So, /: N -» Y is one-one.

w
/is onto: Let y be an arbitrary element of Y. Then, there exists x e N such that
y = 4x + 3 [By definition of Y]
y=/(*)

Flo
Thus, for each y e N there exists x e N such that / (x) = y. So, / : N -» Y is onto.

ee
Thus, / : N —> Y is both one and onto. Consequently, it is invertible. Let/ -1 be the inverse of/.

Fr
Then, for
ur
fof ~ 1 (x) = x for all x e Y
=> / (/ - 1(x)) = x for all x g Y
ks

-1 (x) + 3 = x for all x e Y


Yo

4/ [Using definition of / ]
oo

1 (x) = ——^ for all x e Y


B

/
4
re

Hence, / ~ 1: Y -> N is given by/ -1 (x) = ——— for all x e Y.


ou

4
ad

EXAMPLE 9 LetY ={n2 :n eN] N. Consider f :N ^>Y given by f (n) = n2. Show thatfis invertible.
Y

Find the inverse off. [NCERT]


nd
Re

SOLUTION In order to prove that / is invertible, it is sufficient to show that it is a bijection.


Fi

fis one-one: For any n,m eN, we find that


f(n)=f(m)
=> n2=m2
=> n=m [v n,m eN]
So, /: N -> Y is one-one.
fis onto: Let y be an arbitrary element of Y. Then there exists n eN such that
2
y=« [By definition of Y]
=> y=f(n)
Thus, for each y e Y there exists neN such that y = / (n). So, / :N -> Y is onto.
Hence, /: N -> Y is a bijection. Consequently, it is invertible.
Let /-1 denote the inverse of /. Then,
fof 1 (x) = x for all x e Y
=> / (/ “ 1 (x)) = x for all x e Y

ReadYourFlow.COM
FUNCTIONS 2.67

=> {/-'(x)}2 =x for all x eY [Using the def. off ]


=> f~l (x)=y[x for all x e y
Hence,/ -1 :Y -> N is given by/ - 1 (x) = Vx for all x eY.
EXAMPLE 10 Lef f-.N-^R be a function defined as /(x) = 4x2 + 12x +15. Shozu that
f:N Range (/) is invertible. Find the inverse off [CBSE2010]
SOLUTION In order to prove that / is invertible, it is sufficient to show that /: N —> Range (/)
is a bijection.
/ is one-one: For any x, y eN, we find that
f(x)=f(y)
=> 4x2 + 12x + 15 = 4i/2 + 12y + 15
4 (x2 -y2) + 12 (x -y) = 0

low
=> (x-y)(4x + 4y+ 3)=0
=> x -y = o [■.• 4x + 4y + 3 ^ 0 for any x, y e N]
=> x=y

ee
So, f :N Range (/) is one-one.
rF
Fr
Obviously, f :N -> Range (/) is onto. Hence, /: N -> Range (/) is invertible.
Let/ - 1 denote the inverse of/. Then,
for
fof 1 (x) = x for all x e Range (/)
u
ks

=> / (/ - 1 (x)) = x for all x e Range (/)


Yo
oo

=> 4 {/ ~ 1 (x)}2 + 12/ - 1 (x) +15 = x for all x e Range (/)


B

=> 4{f -1(x)l2 +12/-1(x) + 15-x = 0


re

-12± ^144-16(15-x)
=>
ou
ad

8
Y

-12 ± yl6x - 96 -3±yjx-6


=> /“,W =
8 2
nd
Re

r\x)=i±L4*r6 [■•■ / .•./“1W>0]


Fi

2.7.2 RELATION BETWEEN GRAPHS OF A FUNCTION AND ITS INVERSE


The graph of a bijection/and its inverse/ - 1 are closely related. If (a, b) is a point on the graph of
/, then b =/ (a) and a =f ~ 1 (b). Asb e Domain of / - \ therefore (b, f (b)) is a point on the
graph off ~ ^ But, (b, f (b)) = (b, a). Therefore, (b, a) is on the graph of / - Thus, if (a, b) is a
point on the graph of /, then {b, a) is a point on the graph of / ~ ^ But, (a, b) and {b, a) are
reflections of one another in the line y = x. Thus, the graph of / - 1 may be obtained by reflecting
the graph of / in the line mirror y = x. That is the graphs of / and / 1 are mirror images of each
other in the line mirror y - x (see Fig. 2.42). It is also evident from the above discussion that if the
graphs of / (x) and / - 1 (x) intersect each other, their points of intersection lie on the line y = x.
Consequently, solutions of the equation /(x)=/-1(x) are same as that of /(x)=x or.

ReadYourFlow.COM
2.68 MATHEMATICS-XII

^ y =m I 1
0
4- y=xi

y=f~Hx)

o X

w
Fig. 2.42

EXERCISE 2.4

Flo
LEVEL-1

ee
1. State with reasons whether following functions have inverse:

Fr
(i) /: {1, 2 ,3, 4} -> {10} with/ = {(!, 10), (2,10), (3,10), (4,10)}
(ii) g : {5, 6, 7, 8} -> {1, 2, 3, 4} with g = {(5, 4), (6, 3), (7, 4), (8, 2)}
for
ur
(hi) h : {2, 3, 4,5} -> {7, 9,11,13} with h = {(2,7), (3, 9), (4,11), (5,13)}
2. Find / _ 1 if it exists : f: A B where
ks

(i) A = {0, -1, -3, 2}; B = {-9, -3, 0, 6} and f(x) = 3 x.


Yo
oo

(ii) A = {1, 3, 5, 7, 9}; B = {0,1, 9, 25, 49, 81} and f(x) = x2.
B

3. Consider /: {1, 2, 3} -> {«, fr, c} and g: {a, b, c] -> {apple, ball, cat) defined as/(l) =a,
re

f(2) = b, f(3) = c, g(fl)= apple, g(fr)=ball and g(c)=cat. Show that f,g and gof are
invertible. Find/ ~ ^ g - 1 and(go/)-1 and show that(go/) - 1 =/ [NCERT]
ou
ad

4. Let A = {1, 2, 3, 4}; B = {3, 5, 7, 9}; C = {7, 23, 47, 79} and/: A -> B, g : B -> C be defined as
Y

/(j) = 2.r + 1 and g(x) = x - 2. Express (go/) and / og as the sets of ordered pairs and
nd
Re

verify that (go/) ~ 1 = / ~ 1 og“ 1.


Fi

5. Show that the function / : Q ^ Q defined by /(x) = 3x + 5 is invertible. Also, find/ ~ 1.


6. Consider / : R -> R given by / (x) = 4x + 3. Show that / is invertible. Find the inverse off.
[NCERT]
7. Consider / : R+ -» [4, co) given by / (x) = x2 + 4. Show that / is invertible with inverse/ -1
of / given by/~ 1 (x) = fx -4, where R+ is the set of all non-negative real numbers.
[NCERT, CBSE 2013]
4x+ 3 2 2
8. If / (x) = , x * —, show that/o/ (x) = x for all x * —. What is the inverse of/?
6x - 4 3 3
[CBSE 2012,2013, NCERT]
9. Consider / : R+ -> [-5, oo) given by / (x) = 9x2 + 6x -5. Show that / is invertible with
fx + 6 -1
[NCERT]
3
10. If /: R R be defined by /(x) = x - 3, then prove that/ exists and find a formula for
f ~ Hence, find/-1 (24) and/ - 1 (5).

ReadYourFlow.COM
FUNCTIONS 2.69

11. A function / : K -> i? is defined as f(x) =x3 + 4. Is it a bijection or not? In case it is a


bijection, find / ~ 1 (3).
12. If / : Q —> Q, g: Q —» Q are two functions defined by f(x) = 2x and g(x) = x + 2, show that/
and g are bijective maps. Verify that {gof)
x-2
13. Let A = R - {3} and B - R -{!).Consider the function/:/! -» B defined by f(x) = .Show
x-3
that f is one-one and onto and hence find / ~ [CBSE 2012,2014]
14. Consider the function/ : R+ -> [-9, co) given by/(x) =5x2 +6x-9. Prove that/is invertible
/54 + 5y-3
with/_1(y) = [CBSE 2015]
5
15. Let/ : N -» Nbe a function defined as/(x) = 9x2 + 6x -5.Show that/: N -> S, where S is the
range off, is invertible. Find the inverse of / and hence find / - l(43) and / ~ ^(IbS).
[CBSE 2016, 2017]

low
16. Let / : R - - —^ j -> R be a function defined as/(x) = 4x
. Show that
3x + 4
/:R Rang (/) is one-one and onto. Hence, find /-1. [CBSE 2017]

ee
rF
LEVEL-2

Fr
10x -10 r - is invertible, find / ^
17. If /: R ->(-1,1) defined by / (x) =
for
10* +10"
ou
cx- e_Ar -l
18. If / : R —»(0, 2) defined by / (x) = -----+ 1 is invertible, find f
ks

ex+e~x
oo

19. Let /: [-1, oo) -> [-1, oo) is given by /(x) = (x + l)2 -1. Show that/is invertible. Also, find
Y
B

the set S = {x: /(x) = / - 1 (x)}.


re

20. Let A = {x e R | -1 < x < 1) and let/ :A -> A,g: A -» A be two functions defined by /(x) = x2
ou
ad

and g(x) = sin Show thatg-1 exists but/-1 does not exist. Also, find g-1.
Y

21. Let / be a function from R to R such that f(x) = cos (x + 2). Is / invertible? Justify your
answer.
nd
Re

22. If A = [1, 2, 3,4} and B = {a, b, c, d }. Define any four bijections from A to B. Also, give their
Fi

inverse functions.
23. Let A and B be two sets each with finite number of elements. Assume that there is an
injective map from A to 6 and that there is an injective map from B to A. Prove that there is a
bijection from A to B.
24. If / : A A, g: A -» A are two bijections, then prove that
(i) fog is an injection (ii) fog is a surjection.
ANSWERS
1. (i) No, / is many-one (ii) No, g is many-one (iii) Yes, h is a bijection
2. (i) / - 1 = {(-9, - 3), (-3, -1), (0, 0), (6, 2)} (ii) / ~ 1 does not exist as/is not surjective.
3. / _1 = {(a, 1), (b, 2), (c, 3)), g_1 = {(apple , a), (ball, b), (cat, c)}
and, (gof) -1 = {(apple, 1), (ball, 2) (cat, 3)
4- (go/T1 =/ “ V"1 =1(7,1), (23, 2), (47 , 3), (79, 4)) 5. /" 1 (x) =~
4x + 3 10./-1(x)=(3 + x)1/3,/-1(24) = 3,/-1(5)=2
6x - 4

ReadYourFlow.COM
2.70 MATHEMATICS-XII

11. Bijection, / 1 (3) =-l 13./~1W = 3x-2 15. / -1(43) = 2, / “1 (163) = 4


x-1
/ \l/ 2
16./“V) = 4x 17./ 1(x)=^log10 1 x
18. / 1 (x) = log(i
4-3x 1 -x 2 -x

19. S = (0,-1} 20. g-1(x)=^sin-1x


21. Not invertible

22. /j =|(1- o), (2, f>)- (3, c), (4, rf)), /j " 1 =((a, 1), (b, 2), (c, 3), (d, 4)|
/2 = ((1, a), (2, c), (3, !>), (4, <*)), /2 - 1 = ((«, 1), (c, 2), (f>, 3), (d, 4))
-1
/3={(l,rf),(3,b),(2,a),(4,c)}, /3 = ((rf, 1), (b, 3),(fl, 2), (c, 4)} etc.
______________________________________ HINTS TO NCERT& SELECTED PROBLEMS
3. /: {1, 2, 3}-> {a, b, c} is given by f (1) =a, f (2) =br f (3) = c. Clearly, it is a bijection.
Similarly, g: {a, b, c} (apple, ball, cat} given by g (a) = apple,g (b) = ball, g (c) = cat is also

w
a bijection. Since composition of two bijection is a bijection.
So, gof : (1, 2, 3} —> (apple, ball, cat} is a bijection.
It is given that

Flo
/ = {(!, a), (2,1;), (3, c)} and g = {(a, apple), (b, ball), (c, cat)}

ee
gof = {(1, apple), (2, ball), (3, cat)}

Fr
Clearly, / “ 1 = {(a, 1), {b, 2), (c, 3)} and g~ 1 = {(apple, a), (ball, b), (cat, c)}
(gof)~ 1 = {(apple, 1), (ball, 2), (cat, 3)} ...(i)
for
ur
and, f~ 1og^ 1 = {(apple, 1), (ball, 2), (cat, 3)} ...(ii)
From (i) and (ii), we get (gof)~ 1 =f~ ]og~1.
ks
Yo

6. It is given that / : K -> R such that / (x) = 4x + 3.


oo

fis an injection: Let x, y e R be such that


B

/ (*) = / (]/) ^ 4* + 3 = 4y + 3 => x = y


re

So, / is an injection.
fis a surjection: Let y be an arbitrary element of R (Co-domain) such that f(x) = y. Then,
ou
ad

y-3
/ (x) = y => 4x + 3 = y => x =
Y

4
nd

Thus, for any y e R there exists x = g R such that


Re
Fi

+ 3=y

So, /: R -> R is a bijection and hence invertible.


Let/- 1 denote the inverse of/. Then,
for 1 (x) = x for all x e R
=> / (/“ 1 (x)) = x for all x e R
=> 4 /“ 1 (x) + 3 = x for all x e R
/- 1 (X) = for all x e R
J 4
7. We have,/: R+ -> [4, co) such that/(x) = x2 + 4.
fis an injection: Let x,y eR+ such that
/(*) =/(y)
=> x2 + 4 = y2 + 4

ReadYourFlow.COM
FUNCTIONS 2.71

=> x=y [•• X,\/^R+]


So, / is an injective map.
fis onto: Let y e [4, oo). Then,
f{x) = y => x2 + 4 = y => x = ^/y -4 [v xeR + ]

Also,
y e[4, co) => y-4 > 0 => ^y -4 > 0 => x = Jy-leR +

Thus, for each y e [4, co) there exists x = ^/y -4 e R+ such that
/ w = / (Vy-4) = Wy-4)2 +4=y-
So, / : K + -> [4, co) is onto.
Thus, / : R+ -> [4, oo) is a bijection and hence invertible.

low
Let f - 1 denote the inverse of /. Then,
fo/- 1 (x) = x for all x e [4, co)
=> / (/_ 1 (^)) = x for all x e [4, oo)

ee
rF
Fr
=> (/_ 1 (x))2 +4 = x for all x e [4, co)
=> / 1 (x) = ^/x - 4 for all x e [4, co)
for
4x+ 3 2
8. We have, /(x) = ,x^—
ou
6x -4 3
ks

4 4^3
+3
oo

4x+ 3 6x - 4 16x + 12 + 18x -12


Y
B

••• fof(x)=f(J(x))=f =X
67x - 4 6 ^ -4 24x + 18 - 24x +16
re

6x - 4 J
ou

2
ad

=> (Jof{x) = x for all x * —


Y

3
=> fof = I
nd
Re

=> /is inverse of itself


4x+ 3
Fi

Hence, / 1 (x) = / (x) =


6x - 4
9. We have, / : R+ -> [-5, co) given by / (x) = 9x2 + 6x -5
fis an injection: For any x, y e R +
f(x) = f(y)
=> 9x2 + 6x -5 = 9y2 + 6y -5
=> 9 (x2 - y2) + 6 (x - y) = 0
=> 3 (x - y) {3 (x + y) + 2} = 0
=> x-y = 0 [3 (x + y) + 2 ^ 0 as x, y e R+]
=> x = y
So, / is an injection.
/ is a surjection: Let y be an arbitrary element of [-5, =o). Then,
~ 1 + Vy + 6
/(x) =y => 9x2 + 6x-5=y=>(3x + l)2=y + 6=> 3x + l =Jy+ 6 => x =
3
Now, ye[-5, co)

ReadYourFlow.COM
2.72 MATHEMATICS-XII

=t> y>-5 =>y + 6>l => ^/y + 6 >1 => -1 + A/y + 6 > 0 => ' • V1' • ^ >0
3
=> x > 0 => x eR +
-l + Jij + 6
Thus, for each y e [-5, co) there exists x e R~ such that / (x) = y.
3
So, /: R+ -> [-5, co) is onto.
Thus, /: R+ —> [-5, co) is a bijection and hence invertible. Let/ -1 denote the inverse of f.
Then,
(f°f ]) (x) = x for all x e [-5, co)

/(r1 «)= x for all x g [- 5, co)

=> 9 /"IW | + 6 j/" 1 (x) | -5 = x for all x g [- 5, co)

low
=> js/^w + ij = 6 + x for all x g [- 5, co)

=> 3 / ~1 (x) + 1 = ^/6 + x for all x g [- 5, oo)

ee
=> /-V) | ^x +36—1
rF
Fr
for all x g [-5, co)
19. Let /(x) = y. Tlien,
or
/(x) = y => x = v^TT-i => f~\y)=y[y + i -l.
sf
u
Now, /(x)=/_1(x)
k
Yo
oo

=> (x + 1)2 -1 =A/x + l -1


B

^/x + 1 {(x + 1)3/2 -1| =0


re

x + 1 =0 or, (x + 1)3/2 =1 => x =0,-1


ou
ad

21. / is neither one-one nor onto. So, / is not a bijection. Hence, it is not invertible.
Y

_________________________________ VERY SHORT ANSWER QUESTIONS (VSAQs)


Anszver each of the following questions in one word or one sentence or as per exact requirement of the
nd
Re

question:
Fi

1. Which one of the following graphs represent a function?

X' X X
J

r r
(a) (b)
Fig. 2.43

ReadYourFlow.COM
FUNCTIONS 2.73

2. Which one of the following graphs represent a one-one function?

low
3. If A =[1, 2, 3} and B = [a, b}, write total number of functions from A to B.

ee
4. If A = {a, b, c] and B = {- 2, -1, 0,1, 2], write total number of one-one functions from A to B.
rF
Fr
5. Write total number of one-one functions from set A = {1, 2, 3, 4) to set B = [a, b, c).
6. If /: R —» R is defined by / (x) = x2, write / ~ 1 (25).
for
7. If /: C -» C is defined by / (x) = x2, write / ~ 1 (- 4). Here, C denotes the set of all complex
u
ks

numbers.
Yo
oo

8. If /: R -> R is given by / (x) = x3, write / ~ 1 (1).


B

9. Let C denote the set of all complex numbers. A function/: C -> C is defined by / (x) = x3.
re

Write/-1(1).
ou
ad

10. Let / be a function from C (set of all complex numbers) to itself given by / (x) = x . Write
Y

1
/ (-!)•
11. Let / : R ^ R be defined by / (x) = x4, write / ~ 1 (1).
nd
Re

12. If /: C -* C is defined by / (x) = x4, write / “ 1 (1).


Fi

13. If /: R -» R is defined by / (x) = x2, find / ~ 1 (- 25).


14. If /: C -> C is defined by / (x) = (x - 2)3, write / ~ 1 (-1).
15. If / : R -» R is defined by / (x) = lOx - 7, then write / “ 1 (x).

16. Let/: -> R be a function defined by / (x) = cos [x]. Write range (f)

17. Iff :R —>■ R defined by / (x) = 3x - 4is invertible then write/ ~ 1 (x). [CBSE2010]
18. If /: R -> R, g: R -> R are given by / (x) = (x + l)2 and g (x) = x2 + 1, then write the value
of fog (-3). , ,
19. Let A={xeR:-4<x<4 and x * 0} and f :A R be defined by / (x) = —. Write the
range of /. x
7U 71
20. Let / : — , — —» A be defined by / (x) = sin x. If / is a bijection, write set A.
2'2
21. Let / : R -» R + be defined by / (x) = ax, a >0 and a*l. Write / -1 (x).

ReadYourFlow.COM
2.74 MATHEMATICS-XII

22. Let/: R-{-1}-> R-{1} be givenby/(x) =—^-y . Write/ 1 (x).

23. Let f :R - j - j —> R be a function defined as / (x) =

Write / 1: Range of / -> R - j - j

24. Let f: R R, g: R R be two functions defined by / (x) = x2 + x + 1 and g (x) = 1 - x2.


Write fog (- 2).
2x - 3
25. Let / : R -> R be defined as / (x) = Write/o/ -1 (D-
26. Let /be an invertible real function. Write (f ~ 1 of) (1) + (J ~1 of) (2) + ... + (f ~ ^of) (100).
27. LetA={l/2/ 3,4} and B = {a,b] be two sets. Write total number of onto functions from A

low
to B.
28. Write the domain of the real function / (x) = ^jx - [x].
29. Write the domain of the real function / (x) = /[x] - x.
1

ee
30. Write the domain of the real function / (x) =
rF >/ \ x\-x

Fr
31. Write whether / : R —>■ R given by / (x) = x + Vx2 is one-one, many-one, onto or into.
for
32. Iff (x) = x + 7 and g(x) = x - 7, x e R, write fog (7).
u
I x —1|
ks

33. What is the range of the function/ (x) = ------ - ? [CBSE 2010]
Yo

x-1
oo

34. If/ : R -> R be defined by / (x) =(3 -x3)1^ 3, then find/o/ (x). [CBSE 2010]
B
re

35. If/: R -» R is defined by / (x) = 3x + 2, find / (/ (x)). [CBSE 2010]


36. Let A = (1, 2, 3), B ={4,5, 6, 7} and let / = {(1, 4), (2,5), (3, 6)} be a function fromA to B.State
ou
ad

whether / is one-one or not. [CBSE 2011]


Y

37. If/: {5, 6} -> {2, 3} andg: {2, 3} -> {5, 6} are given by / = {(5, 2), (6, 3)} and g = {(2,5), (3, 6)},
find/og. [NCERT EXEMPLAR]
nd
Re

38. Let /: R -» R be the function defined by/(x) = 4x - 3 for all x e R. Then write / ~ ^
Fi

[NCERT EXEMPLAR]
39. Which one the following relations on A = {1, 2, 3} is a function?
/ = {(1, 3), (2, 3), (3, 2)}, g = {(1, 2), (1, 3), (3,1)} [NCERT EXEMPLAR]
40. Write the domain of the real function / defined by /(x) = V 25 - x2. [NCERT EXEMPLAR!
41. Let A = {a, b, c, d\ and /: A -> A be given by / = {(a, b), (b, d), (c, a), (d, c)}, write/-1.
[NCERT EXEMPLAR]
42. Let f, g:R -» R be defined by/(x) = 2x +1 and g(x) = x2 - 2 for all x e R, respectively. Then,
find go/. [NCERT EXEMPLAR]
43. If the mapping /: {1, 3, 4} ->{1,2,5} and g: {1, 2,5} -> {1, 3}, given by
/ ={(1, 2), (3,5), (4,1)} andg = {(2, 3),(5,1),(1, 3)}, write/og. [NCERT EXEMPLAR]
44. If a function g = {(1,1), (2, 3), (3,5), (4, 7)} is described by g(x) = ox + (3, find the values of a
and p. [NCERT EXEMPLAR]
45. If/(x) =4-(x-7)3,write/-1(x). [NCERT EXEMPLAR]

ReadYourFlow.COM
FUNCTIONS 2.75

ANSWERS
1. (a) 2. (b) 3. 8 4. 60 5. 0 6. {-5,5}
7. {2i, - 2/} 8. {1} 9. {1, iv, zv2} 10. {-1,-w,-u>2}

11. {-1,1} 12. {-1,-z, 1,/} 13. (j) 14. {l,2-:v,2-w2}


x+7 17. /"1W=^
15. / “ 1 (x) = 16. {1, cos 1, cos 2}
J 10
18. 121 19. {-1,1} 20. A =[-1,1] 21. logflx
22. / 23. f-\x) = 3x 24. 7 25. 1
1 -A- 2 -5j
26. 5050 27. 14 28. R 29. (|) 30. (-co,0) 31. Many one-into
32. 7 33. {-1,1} 34. fof(x) = x 35. /(/(*)) =9*+ 9
36. Yes 37. fog = {(2, 2), (3, 3)} 38 f~\x) = 39. /

w
40. [-5, 5] 41. Z"1 = {(&, a), {d, b), (a, c), (c, zf)} 42. go/(x) = 4x2+ 4*-1
43. /og = {(2,5),(5, 2), (1,5)} 44. a = 2, p = -1 45. f~\x)=7 + (4-x) 1/3

Flo
ee
MULTIPLE CHOICE QUESTIONS (MCQs)

Fr
Mark the correct alternative in each of the following:
1. Let A={x e R:-l < x < 1} = B and C ={x e R:x > 0} and
let S = {(a, i/) e ,4 x B: x2 + y2 = 1} and S0 = {(x, y) e /I x C : x2 + y2 = 1}. Then
for
ur
(a) S defines a function from A to B (b) Sq defines a function from A to C
ks

(c) S0 defines a function from Ajo B (d) S defines a function from A to C


Yo

2. f :R R given by / (x) = x + Vx2 is


oo

(a) injective (b) surjective (c) bijective (d) none of these


B

3. If /: A-> B given by 3-^A) + 2- A =4is a bijection, then


re

(a) A = {x e R : -1 < x < oo}, B = {x e R : 2 < x < 4}


ou
ad

(b) A = {x e R : - 3 <x < cc}, B = {xeR:0<x<4}


Y

(c) A = {x e R : - 2 < x < co}, B = {x e R : 0 < x < 4}


(d) none of these
nd
Re

4. The function /: R -> R defined by / (x) = 2A + 2 I *1 is


Fi

(a) one-one and onto (b) many-one and onto


(c) one-one and into (d) many-one and into
x+a
5. Let the function /: R - {- fo} -> R - {1} be defined by / (x) = , a *b, then
x +b
(a) / is one-one but not onto (b) / is onto but not one-one
(c) / is both one-one and onto (d) none of these
6. The function /: A -> B defined by / (x) = - x + 6x - 8 is a bijection, if
(a) A = ( - co, 3] and B = (- co, 1] (h) A = [- 3, co) and B = (— oo, 1]
(c) A = (- oo, 3] and B = [1, co) (d) A = [ 3, co) and B = [1, co)
7. Let A = {x e R : -1 < x < 1} = B. Then, the mapping / : A -> B given by / (x) = x | x | is
(a) injective but not surjective (b) surjective but not injective
(c) bijective (d) none of these
8. Let /: R -> R be given by / (x) = [x] + [x + 1] - 3, where [x] denotes the greatest integer less
than or equal to x. Then, / (x) is
(a) many-one and onto (b) many-one and into
(c) one-one and into (d) one-one and onto

ReadYourFlow.COM
2.76 MATHEMATICS-XII

9. Let M be the set of all 2 x 2 matrices with entries from the set R of real numbers. Then the
function / : M -> R defined by / {A) = \ A \ for every ^4 e M, is
(a) one-one and onto (b) neither one-one nor onto
(c) one-one but-not onto (d) onto but not one-one
10. The function f: [0, oo) R given by f (x) = —1— is
x+1
(a) one-one and onto (b) one-one but not onto
(c) onto but not one-one (d) neither one-one nor onto
11. The range of the function / (x) = 7 ~A PY _ 3 is
(a) {1,2, 3,4,51 (b) (1, 2, 3, 4, 5, 6) (c) (1, 2, 3, 4} (d) {1,2, 31
12. A function / from the set of natural numbers to integers defined by
11— , when n is odd
f(n) = 2

w
- — , when n is even
2
is
(a) neither one-one nor onto
(c) onto but not one-one
Flo
(b) one-one but not onto

ee
(d) one-one and onto both
13. Let/ be an injective map with domain {x, y, z] and range {1, 2, 3} such that exactly one of the

Fr
following statements is correct: and the remaining are false.
/(*)=!, /(•/)*!, /(z)*2. or
ur
The value of / \l) is
sf
(a) x (t>) y (c) 2 (d) none of these
k

14. Which of the following functions from Z to itself are bijections?


Yo
oo

(a) / (-v) = x3 (b) / (x) = x + 2 (c) / (x) = 2 x + 1 (d) / (x) = x2 + x


B

15. Which of the following functions from A = {x: -1 < x <1} to itself are bijections?
re

71 x
(*)/(*)=! (b) g (x) = sin (c) k (x) =| x | (d) k (x) = x2
2
ou
ad

16. Let A = {x: -1 < x < 1} and f : A —> A such that / (x) = x | x |, then / is
Y

(a) a bijection (b) injective but not surjective


(c) surjective but not injective (d) neither injective nor surjective
nd
Re

x2
17. If the function / : R -> A given by / (x) =
Fi

is a surjection, then A =
x2 +1
(a) R (b) [0,1] (c) (0, Ij (d) [0,1)
18. If a function / : [2, x>) -> B defined by / (x) = x - 4 x + 5 is a bijection, then B =
(a) R (b) [1,qo) (c) [4, oc) (d) [5, co)
19. The function / : R -» R defined by / (x) = (x -1) (x - 2) (x - 3) is
(a) one-one but not onto (b) onto but not one-one
(c) both one and onto (d) neither one-one nor onto
20. The function /: [-1/2,1/2] -5- [- n/2, n/2] defined by / (x) = sin- 1 (3x - 4x3) is
(a) bijection (b) injection but not a surjection
(c) surjection but not an injection (d) neither an injection nor a surjection
el*l _«r v
21. Let / : R —> R be a function defined by / (x) = T -x '•Then'
e +e
(a) / is a bijection (b) / is an injection only
(c) / is surjection on only (d) / is neither an inje ction nor a surjection

ReadYourFlow.COM
FUNCTIONS 2.77

X ~~ ttl
22. Let /: R - (n) -> R be a function defined by / (.r) = —---- , where m * n. Then,

(a) / is one-one onto (b) / is one-one into


(c) / is many one onto (d) / is many one into
X 2 -8
23. Let / : K -> R be a function defined by / (x) = — . Then,/is
*2 +2
(a) one-one but not onto (b) one-one and onto
(c) onto but not one-one (d) neither one-one nor onto
-e -,2
24. /: R R is defined by / (x) = 2 2 15
rr +e-r
(a) one-one but not onto (b) many-one but onto

w
(c) one-one and onto (d) neither one-one nor onto
2,5. The function / : R —> R, / (x) = x2 is
(a) injective but not surjective
(c) injective as well as surjective
Flo
(b) surjective but not injective
(d) neither injective nor surjective

ee
26. A function / from the set of natural numbers to the set of integers defined by

Fr
H— , when n is odd
2
/(«) =
for
ur
, when n is even
2
ks

(a) neither one-one nor onto (b) one-one but not onto
Yo

(c) onto but not one-one (d) one-one and onto both
oo

27. Which of the following functions from A = {x e R : -1 < x < 1} to itself are bijections?
B

7Z X
re

(a)/(x) = | x| (b) / (x) = sin —


2
ou

KX
ad

(c) / (x) = sin — (d) none of these


4
Y

X
- , if x is even
28. Let / : Z -> Z be given by / (x) 2' . Then, / is
nd
Re

0 , if x is odd
Fi

(a) onto but not one-one (b) one-one but not onto
(c) one-one and onto (d) neither one-one nor onto
29. The function / : R -> R defined by / (x) -6X + 6^ is
(a) one-one and onto (b) many one and onto
(c) one-one and into (d) many one and into
30. Let / (x) = x2 and g (x) = 2r. Then the solution set of the equation fog (x) = gof (x) is
(a) R (b) 10} (c) 10, 2} (d) none of these
31. If /: R R is given by /(x) = 3x - 5, then / - 1 (x)
1 (b) is given by 'r +- ^
(a) is given by
3x -5 3
(c) does not exist because/is not one-one (d) does not exist because/is not onto.
32. If g (/ (x)) = | sin xj and / {g (x)) = (sin Vx)2, then
(a) /(x) = sin 2 x,g(x) = vx (b) /(x) = sin x, g (x) =|x|

ReadYourFlow.COM
2.78 MATHEMATICS-XII

(c) f(x)=x2,g{x)=sm -Jx (d) /and g cannot be determined.


ex
33. The inverse of the function f :R ~>{x eR: x <l) given by / (x) = — ,is
ex + e-x
1
(a) - log
1 + A' (b) | log 2 + .Y 1 1 -A
(d) none of these
(0 - log
1 -a 2-A 1 +A
34. Let A = {a e R : a > 1}. The inverse of the function/: A -» A given by/ (a) = 2X^X _ 1 is
x(x-l)
1
(b) - (1 + V1 + 4 log2 a)

1
(c) - U - VI + 4 log2 A} (d) not defined

35. Let A = {a e R : a < 1 j and f: A Abe defined as / (a) = a (2 - a). Then, / -1 (a) is

low
(a) 1 + V'rl - * (b) 1-VT^ (c) Vi - * (d) i ±
i
36. Let/(A) = . Then, {f o(f o /)} (a)
1 —A

ee
(a) a for all a e R
rF
(b) a for all a e R - {1)

Fr
(c) a for all a e R - {0,1} (d) none of these
37. If the function /: R -> R be such that / (a) = a - [a], where [a] denotes the greatest integer
for
less than or equal to a, then / -1 (a) is
u
1
ks

(a) (b) [a]-a (c) not defined (d) none of these


Yo

x-[x]
oo

1
38. If F : [1, co) [2, co) is given by / (a) = x + — ' then / -1 (a) equals.
B

A
re

(a)
A+
^ m A
(c)
x-J? -4 (d) 1 + /a2-4
ou
ad

2 1 + A2 2
Y

-1, a<0
39. Let g (a) —1 + a — [a] and / (a) = 0, a = 0, where [a] denotes the greatest integer less
nd
Re

1, A >0
Fi

than or equal to a. Then for all a, / (g (a)) is equal to


(a) a (b) 1 (c)/(a) (d) g(x)
aa
40. Let / (a) = , a ^ -1. Then, for what value of a is / (/ (a)) = a?
A+1
(a) V2 (b) -V2 (c) 1 (d) -1
41. The distinct linear functions which map [-1,1] onto [0, 2] are
(a) /(A) = A + 1, £ (a) = -A + l (b) f(x) = a — 1, g (a) = a + 1
(c) / (a) = - A — 1, g (a) = A — 1 (d) none of these
42. Let / : [2, co) —» X be defined by / (a) = 4a - a2. Then, / is invertible, if X =
(a) [2, co) (b) (- oo, 2] (c) (-0Q, 4] (d) [4, ex,)
~A| A|
43. If /: R (-1,1) is defined by / (a) = V, then / - 1 (a) equals
1 +A
I A| A
(a) (b) -Sgn:(A) j (c)- (d) none of these
1 —I A| 1 -| A 1 -A

ReadYourFlow.COM
FUNCTIONS 2.79
—I 9
44. Let [x] denote the greatest integer less than or equal to x. lif(x) = sin x,g (x) = [x ] and
1
h (x) = 2x, — < x < then
V2
(a) fogoh(x) = n/2 (b) fogoh(x) = n (c) hofog = hogof (d) hofog * hogof
45. If g (x) = x2 + x - 2 and ^ go/ (x) = 2 x2 -5 x + 2, then/ (x) is equal to

(a) 2 x - 3 (b) 2 x + 3 (c) 2x2 + 3x + l (d) 2 x2 - 3 x -1


46. If / (x) = sin x and the composite function g (/(x)) =| sin x |, then g (x) is equal to
(a) Jx-l (b) Vx (c) -Jx + 1 (d) -VJ
47. Iff :R Ris given by / (x) = x3 + 3, then/ ~1 (x) is equal to
(a) x1/3-3 (b) x 1/3 + 3 (c) (x - 3) 1/3 (d) x + 3 1/3
48. Let / (x) = x" be a function with domain {0,1, 2, 3}. Then domain off is

low
(a) {3,2,1,01 (b) (0,-1,-2,-3} (c) (0,1,8,27) (d) (0,-1, -8, -27}
49. Let /: K -> R be given by / (x) = x2 - 3. Then, / ~ 1 is given by
(a) fx + 3 (b) Vx + 3 (c) x + V3 (d) none of these

ee
rF
Fr
50. Let / : R -> R be given by / (x) = tan x. Then, / ~ ^(l) is
(b) jn7c + ^:nezj (c) does not exist (d) none of these
for
(a)I
u
2x, if x > 3
ks

51. Let / : R -» R be defined as /(x) =< x2, if 1 < x < 3


Yo
oo

3x, if x <1
B

Then, find/(-l) +/(2) + /(4)


re

(a) 9 (b) 14 (c) 5 (d) none of these


52. Let A ={1,2,..., n) and B = {a, b}. Then the number of subjections from A into B is
ou
ad

(a) nP2 (b) 2n-2 (c) 2n -1 (d) ”C2


Y

53. If the set A contains 5 elements and the set B contains 6 elements, then the number of
one-one and onto mappings from A to B is
nd
Re

(a) 720 (b) 120 (c) 0 (d) none of these


Fi

54. If the set A contains 7 elements and the set B contains 10 elements, then the number one-one
functions from A to B is
(a) 10C7 (b) 10C7 x 7! (c) 710 (d) 107
55. Let/: R-j-^j-> R be defined by/(x) = 3x + 2 . Then,
5x-3
(a) /_1(x) =* (b) / " ^x) = -/(x) (c) fof{x) = -x (d) /“1(x)=^-/(x)
19
______________________ ANS WERS
1. (b) 2. (d) 3. (d) 4. (c) 5. (c) 6. (a) 7. (c) 8. (b) 9. (d)
10. (b) 11. (d) 12. (d) 13. (b) 14. (b) 15. (b) 16. (a) 17. (d) 18. (b)
19. (b) 20. (a) 21. (d) 22. (b) 23. (d) 24. (d) 25. (d) 26. (d) 27. (b)
28. (a) 29. (c) 30. (c) 31. (b) 32. (a) 33. (a) 34. (b) 35. (b) 36. (c)
37. (c) 38. (a) 39. (b) 40. (d) 41. (a) 42. (c) 43. (b) 44. (c) 45. (a)
46. (b) 47. (c) 48. (c) 49. (d) 50. (b) 51. (a) 52. (b) 53. (c) 54. (b)
55. (a)

ReadYourFlow.COM
2.80 MATHEMATICS-XII

SUMMARY
1. Let A and B be two non-empty sets. Then, a subset / of A x B is a function from A to B, if
(i) for each a e A there exists b eB such that (n, b) ef
(ii) (rt, b) <=/and (rt, c) e/=> b=c.
In other words, a subset / of A x B is a function from A to B, if each element of A
appears in some ordered pair in / and no two ordered pairs in / have the same first
element.
2. Let A and B be two non-empty sets. Then, a function/ from A to B associates every element
of A to a unique element of B. The set A is called the domain of / and the set B is known as its
co-domain. The set of images of elements of set A is known as the range of f.
3. If / : A -> B is a function, then x = y => / (x) =/ (y) for all x, y e A.
4. A function /: A -> B is a one-one function or an injection, if
/(x) = / (y) => x=y for allx, y e A or, x * y => /(x) * /(y) for allx, y e A

w
Graphically, if the graph of a function does not take a turn, in other words a straight line
parallel to x-axis does not cut the curve at more than one point, then it is a one-one function.

Flo
Note that a function is one-one, if it is either strictly increasing or strictly decreasing.
5. A function /: A -> B is an onto function or a surjection, if range (/) = co-domain (/).

ee
6. Let A and B be two finite sets and /: A —» B be a function.

Fr
(i) If / is an injection, then n (A) < n{B)
(ii) If f is a surjection, then n (A) > n{B) for
ur
(iii) If/is a bijection, thenn(A) =/j(B)
7. If A and B are two non-empty finite sets containing m and n elements respectively, then
ks

(i) Number of functions from A to B = nm.


Yo
oo

(ii) Number of one-one functions from A to B = nCm x m !, if, n > m


B

0, if n < m
re

X (-l)"-r ”Cr rm, if m>n


(iii) Number of onto functions from A to B =
ou
ad

r=1
0, if m < n
Y

n\, if m=n
(iv) Number of one-one and onto functions from A to B =
nd
Re

0, if m * n
Fi

8.. If a function/ : A -> B is not an onto function, then/: A —> f (A) is always an onto function.
9. The composition of two bijections is a bijection.
10., If / : A B is a bijection, then y : B -> A is inverse of/, iff f (x) = y => g (y) = x
gof = IA and fog = IB
H. Let / : A -> B and y : B -> A be two functions.
(i) If gof = IA and / is an injection, then y is a surjection.
(ii) If fog = IB and / is a surjection, then g is an injection.
12. Let / :A->B and y: B -> C be two functions. Then
(i) gof : A -» C is onto => y: B -» C is onto.
(ii) gof : A -> C is one-one => / : A -> B is one-one.
(iii) gof : A C is onto and y: B —> C is one-one => f : A B is onto.
(iv) gof : A -» C is one-one and /: A -» B is onto => y: B -> C is one-one.

ReadYourFlow.COM
CHAPTER 3
BINARY OPERATIONS

3.1 INTRODUCTION
In earlier classes, we have come across with various operations like addition, subtraction,
multiplication and division of numbers, union and intersection of sets, composition of functions
etc. hr all these operations any two elements of the given set are operated to get a unique element

w
of the same set. In this chapter, we shall introduce such operations as functions from the
cartesian product of a set with itself to the set itself.

3.2 BINARY OPERATION

Flo
ee
Consider the operation of addition of natural numbers. We know that the addition '+' operates
airy two natural numbers a, b to give a unique natural number a + b. In other words, the

Fr
operation of addition '+' associates every ordered pair (a, b) of natural numbers a andb to a
unique natural number a + b. More rigorously, we can also say that V is a function from N x N
for
r
to N such that the image of (a, b) eN x N is a + b. Thus, we find that addition on N i.e.'+' can be
ou
considered as a function from N x N to N such that it relates every ordered pair (a, b)inN x N to
ks

a unique natural number a + b in N. Symbolically, we can write it as follows:


oo

+ :NxN -> N such that + {a, b) = a + b.


Y
B

The above discussion leads us to the following definition.


re

BINARY OPERATION Let S be a non-empty set. A function f :Sx S Sis called a binary operation on
set S.
ou
ad

It follows from the definition of a function that a binary operation on a set S associates each
Y

ordered pair {a,b) eSx S to a unique element/(fl, b) in S. Instead of writing/(«, b) for the image
of an ordered pair (a, b) e Sx S, conventionally we will prefer to write a/b, that is we write
nd
Re

/(a, b) as a f b.
Fi

Generally binary operations are denoted by the symbols *, 0, +, O etc instead of the letters
/, g, h, etc.
Thus, a binary operation * on a set S associates each ordered pair (a, fr) in S x S to a unique
element a* bin S. Since an ordered pair is made of two elements of S. So, we can say that a binary
operation * on a set S associates any two elements a,b of S to a unique element a* b in S.
ILLUSTRATIVE EXAMPLES
LEVEL-1
EXAMPLE l Addition (+) and multiplication (■) are binary operations on the set N of all natural
numbers, because the sum and product of any two natural numbers are natural numbers.
Addition and multiplication are also binary operations on Z (the set of integers), Q (the set of rational
n umbers), R (the set of real nu mbers) and C (the set of complex n umbers).
EXAMPLE 2 Subtraction (-) is not a binary operation on N, because the subtraction of any two natural
numbers is not always a natural number. For example, 3 and 7 are natural numbers. But, 3-7 = - 4 is
not a natural number. However, subtraction is a binary operation on Z,Q,R and C.

ReadYourFlow.COM
3.2 MATHEMATICS-XII

EXAMPLE 3 Division is not a binary operation on Z, because division of two integers need not be an
integer. Similarly, division is not a binary operation on Q, R and C as division by zero is not defined.
Hozvever, division is a binary operation on the set of all non-zero rational (or real or complex) numbers.
EXAMPLE 4 Let S be a 'non-empty set and P (S) be its power set. For any two subsets A and BofS, we
know that Au B a S. That is, for any two elements ofP(S), we have Au B e P (S). Therefore, 'u' is a
binary operation on P (S). Similarly, if A,B<zP (S), then A nB gP (S) and A - B eP (S). Thus, the
intersection of sets nand the difference of sets are also binary operations on P (S).
EXAMPLE 5 Let A be a non-empty set and S be the set of all functionsfrom A to itself Iff-.A^A and
g:A -> A are tzvofunctions, then we have, learnt in the previous chapter that fog: A -> A. That is fog e S
for any f, g e S. So, the composition offunctions is a binary operation on S.
EXAMPLE 6 Let S = {a + Jib : a,b eZ). Then, prove that an operation * on S defined by
(aj + V2 bf) * (#2 + V2 ^2) = (^1 4- af) + V2 (/?■] + bf) for all a-^, b-y, 02/ b2 g Z
is a binary operation on S.

w
SOLUTION We know that addition is a binary operation on Z.
a-y + a2 GZ,bl+b2 e Z for all ay, a2, by, b2 e Z.
=> (ay + #2) -t V2 (by + bf) s S.

Flo
ee
Thus, ii ay + ^2 by and + ^2 are any two elements of S, then

Fr
(ay + -JT-by) * (fl2 t ~J2. = (zzy + ^2) (^1 **" ^2) ^ ^
Hence, * is a binary operation on S. or
ur
EXAMPLE 7 Let S ~{1, 2, 3, 4} and * be an operation on S defined by
sf
a*b = r, zvhere r is the least non-negative remainder when product is divided by 5.
Prove that * is a binary operation on S.
k
Yo
oo

SOLUTION In order to prove that * is a binary operation on set S, we will have to show that
B

a* b g S for all a, b g S.
re

We have.
1*1= (Remainder when 1x1 = 1 is divided by 5) =1
ou
ad

1*2 = (Remainder when lx 2 = 2 is divided by 5) =2


Y

2*3 = (Remainder when 2 x 3 = 6 is divided by 5) =1


3*4 = (Remainder when 3 * 4 = 12 is divided by 5) =2
nd
Re

Similarly, we have
Fi

1*3=1,1*4 = 4, 2*1=2, 2*2 = 4, 2*4 = 3, 3*1 = 3, 3*2 = 1, 3*3 = 4, 4*1= 4,


4* 2 = 3,4* 3 = 2,4* 4 = 1.
Clearly, all these are elements of S. Thus, we observe that a*b e S for all fl, b e S. So, * is a binary
operation on S.
EXAMPLE 8 Let S = (0,1, 2, 3, 4) and * be an operation on S defined by a * b - r, where r is the least
non-negative remainder when a + b is divided by 5. Prove that * is a binary operation on S.
SOLUTION In order to prove that * is a binary operation on S, it is sufficient to show that
a* b g Sfor al\a,b g S.
We have,
0*0 = (Remainder when 0 + 0 = 0 is divided by 5) =0
3*4 = (Remainder when 3 + 4 = 7 is divided by 5) =2
2*3 = (Remainder when 2+ 3 = 5 is divided by 5) =0
Similarly, we have
0*1 = 1, 0 * 2 = 2, 0 * 3 = 3, 0*4=4
1*0 = 1,1*1 = 2,1 * 2 = 3,1 * 3 = 4,1 * 4 =0

ReadYourFlow.COM
BINARY OPERATIONS 3.3

2*0 = 2, 2*1 = 3, 2*2 = 4, 2*3 = 0,2*4 =1


3*0 = 3, 3*1 = 4, 3*2 = 0,3*3 = 1,3*4 = 2
4*0 =4, 4*1 =0, 4*2 =1,4*3 = 2 and 4*4 = 3
Clearly, a*b eS for all a,b e S. So, * is a binary operation on S.
EXAMPLE 9 Show that the operation v and Aon R defined as
av b = Maximum of a and b ; a Ab = Minimum of a and b
are binary operations of R.
SOLUTION We have,
a rif a >b
av b = Maximum of a and b -
b , if a <b
a, if a<b
and. a a b — Minimum of a and b =
b , if a >b
Thus, av b eR and a Ab eRfor al\a,b <=R. Hence, v and a are binary operations on R.

w
REMARK The operation 'v' is called the supremum and 'a' is called infinum.
EXAMPLE 10 On the set Q ofall rational numbers an operation * is defined by a*b = \ + ab. Show that *
is a binary operation on Q.

Flo
ee
SOLUTION Let a,b sQ. Then,

Fr
ab gQ [Multiplication is a binary operation on Q]
=> 1 + ab eQ [v Addition is a binary operation on Q 1 gQ, ab gQ => 1 + ab gQ]
a*b gQ
for
=>
ur
Thus, a * b g Q for all a, b g Q. Hence, * is a binary operation on Q.
EXAMPLE ll On the set W of all non-negative integers * is defined by a* b =ab. Prove that * is not a
ks
Yo

binary operation on W.
oo

SOLUTION We observe that a*b = ab e W for all on non-zero values of fl, b in W. For a=b =0,
B

we have
re

#*£>=0*0 = 0° which is meaningless


ou

0* 0 eW
ad

Hence, * is not a binary operation on W.


Y

EXAMPLE 12 On the set C of all complex numbers an operation 'o' is defined byz-^ oz2= ^ z2 for all
nd

Z|, z2 gC.Is o a binary operation onC?


Re

SOLUTION We know that the square root of a complex number z = a + ib has two values.
Fi

z| + Re (z) . | z | - Re (z)
± +1 ,ifb>0
2 2
i.e. Vz =^a + ib =•
+ j^jlzl + Re (z)~ _ . ^jz\ - Re (z) ,ifb<0
2
z-^oz2= fzi z2 does not have unique value.
For example, if Zj = 1 and z2 = i Then,
= (1 + 0/) o(() + 0 =7(1 + 00 (0+0 =Vf =±-^(l+i)
zloz2

Hence, o is not a binary operation onC.


X
EXAMPLE 13 Let M be the set of all singular matrices of the form ^ X , zvhere x is a non-zero real
x
number. On M, let * be an operation defined by, A* B=AB for all A,B gM.
Prove that * is a binary operation on M.

ReadYourFlow.COM
3.4 MATHEMATICS-XII

a [b bl
SOLUTION Let A =
T , a
be any two elements of M. Then, a, b are non-zero real
a b b
numbers.
Now,
A* B=AB= a al b b _ lab lab
a a b b lab lab
Since, a, b are non-zero real numbers. Therefore, 2 ab is also a non-zero real numbers.
Consequently,
lab lab
A* B=AB = eM
lab lab
Hence, * is a binary operation on M.

3.3 NUMBER OF BINARY OPERATIONS

low
Let S be a finite set consisting of n elements. Then, S x Shas n2 elements. Since a binary operation
on S is a function from S x S to S. Therefore, the total number of binary operations on S is equal to
the number of functions from S x S to S. We know that the total number of functions from a finite
2
set A to a finite set B is {n (B)}" * ^ Therefore, the total number of binary operations on S is n” .

ee
?2 4
rF
Fr
For example, if S = {a, b], then 2 =2=16 binary operations can be defined on S.
REMARK If '*' is a binary operation on a set S, then we also say that 'S' is closed with respect to .
for
Clearly, the set E of all even integers is closed with respect to addition but the set O of odd
u
integers is not closed with respect to addition as 1 e O, 5 e O but 1 + 5 £ O.
ks

EXERCISE 3.1
Yo
oo

LEVEL-1
B

1. Determine whether each of the following operations define a binary operation on the given
re

set or not:
(i) on N defined by a * b = for all a, b e N.
ou
ad

(ii) 'O' on Z defined by a O b = a11 for all a,b eZ.


Y

(iii) '*' on N defined by a * b = a + b - 2 for all a, b e N.


nd
Re

(iv) 'x6' on S = {1, 2, 3, 4, 5} defined by ax^b = Remainder when ab is divided by 6.


a+b , if (? + b < 6
Fi

(v) '+6' on S = {0,1, 2, 3, 4,5} defined by a +()b =


a + b -6 , iffl + b>6
(vi) 'O' on N defined by a O b = ab + ba for all a,b eN.
a -l
(vii) on Q defined by a *b = for all fl, b e Q.
b +1
2. Determine whether or not each of the definition of * given below gives a binary operation.
In the event that * is not a binary operation give justification of this.
(i) On Z + , defined * by a * b = a-b (ii) On Z +, defined * by a * b = ab
(iii) On R, define by a*b =ab2 (iv) On Z+ define*bya*b=\a-b\
(v) On Z +, define *by a*b = a (vi) On R, define *by a* b = a + 4b2
Here, Z + denotes the set of all non-negative integers. [NCERT]
3. Let * be a binary operation on the set I of integers, defined by a * b = 2a + b - 3. Find the
value of 3 *4. :BSE2011]
4. Is* defined on the set (1, 2, 3, 4,5} by a * b = LCMoffl and b a binary operation? Justify your
answer. [NCERT]

ReadYourFlow.COM
BINARY OPERATIONS 3.5

5. Let S = [a, b, c}. Find the total number of binary operations on S.


6. Find the total number of binary operations on [a, b).
7. Prove that the operation * on the set
'~a 0"
M= :a ,b eR - [0}\ defined by A* B = ABisa binary operation.
0 b

8. Let S be the set of all rational numbers of the form — , where m e Z and n = 1, 2, 3. Prove
n
that * on S defined by a* b -ab is not a binary operation.
9. The binary operation * \Rx R -> R is defined asa * b = 2a + b. Find (2 * 3) * 4. [CBSE 2012]
10. Let * be a binary operation on N given by a*b = LCM (a, b) for all a,b <=N. Find 5*7.
[CBSE 2012]
____________________________________________________________________ ANSWERS

low
1. (i) Yes (ii) No (iii) No (iv) No (v) Yes (vi) Yes (vii) No
2. (i) * is not a binary operation Z +, because 3*7 = -4gZ +
(ii) * is a binary operation on Z ^ (iii) * is a binary operation on R
(iv) * is a binary operation on Z + (v) * is a binary operation on Z +

ee
(vi) * is a binary operation on R
rF
Fr
3. 7 4. No 5. 39 6. 16 9. 18 10. 35
for
HINTS TO NCERT& SELECTED PROBLEMS
u
2. (vi) We have, a*b = a + 4b2 for all a,b eR
ks

Clearly, a* b = a + 4b2 e R for all a, b e R. So, * is a binary operation on R.


Yo
oo

4. We have.
B

n * b = LCM of a and b, where a,b e (1, 2, 3, 4,5}


re

1*1=1, 1 * 2 = 2,1 * 3 = 3,1 * 4 = 4,1 * 5 = 5,


2*1 =2, 2*2=2, 2*3 = 6,2*4=4,2*5=10
ou
ad

We observe that 2*3 = 6 and 2 * 5 = 10 do not belong to the set {1, 2, 3, 4,5}.
Y

So, * is not a binary operation on the given set.


nd
Re

3.4 TYPES OF BINARY OPERATIONS


Consider a binary operation ,*/ on a set S. For any two distinct elements in S, we have
Fi

(a,b) * (b,a)
Since '*'; S x S S. Therefore, * (a, b) and * (b, a) i.e. images of (a, b) and (b, a) under '*' may or
may not be same. In other words, a*b and b * a may or may not be equal. Thus, it is not necessary
that for a binary operation * on a set S, a * b = b * a must hold for all fl, b e S.lf a * b = b * a for all
a,b e S, then we say that the binary operation * possesses commutativity as defined below.
COMMUTATIVITY A binary operation '*' on a set S is said to be a commutative binary operation, if
a *b =b * afor all a,b e S
The binary operations addition (+) and multiplication (x) are commutative binary operations
on Z. However, the binary operation subtraction (-) is not a commutative binary operation on Z
as 3 - 2 * 2-3.
ILLUSTRATION 1 Let * be a binary operation onQ- {0} defined by a*b = ^- for all a,b <=Q - {0}.

Prove that * is commutative on Q - {0).


SOLUTION For any a,b eQ - {0}, we have
. ab , . ba
a* b = — and b * a= —
2 2

ReadYourFlow.COM
3.6 MATHEMATICS-XII

Clearly, ^ for all fl, b e Q - (0)


[v Multiplication is commutative on Q -{0}]
fl * fr = fr * a for all fl, fr e Q - {0}.
So, * is commutative on Q - {0}.
ILLUSTRATION 2 Let * be a binary operation on R, the set of all real numbers, defined by
a*b = ^a2 + b2 for all a,b eR. Show that * is commutative.

SOLUTION We have,
a*b =^Ja2 +b2 and b * a = ^b2 + a2 for all a,b eR.

But, ^ja2 +b2 ^b2 + a2 for all a,b eR


=> a*b=b*a for alia, b eR.
So, * is commutative on R.

w
ASSOCIATIVITY A binary operation on a set S is said to be an associative binary operation, if
(a * b) * c = a * (b * c)for all a,b eS.

Flo
The binary operations of addition (+) and multiplication (x) are associative binary operation on
Z. However, the binary operation subtraction (-) is not a associative binary operation on Z as

ee
(2 - 3) -5 * 2 -(3 -5).

Fr
If S is a non-empty set, then union (u) and intersection (n) are both commutative and associative
binary operation on P (S) (the power set of set S) as
for
ur
Au B = B u A, A n B = B r\ A
(A uB) uC = A u(BuC) and (A nB)nC = A n(BnC) for all A, B, C eP(S).
ks

ILLUSTRATION 3 Addition of vectors is commutative as well as associative on the set of all vectors
Yo

in 3-dimensional space. However, "cross-product" is neither commutative nor associative on 1/3.


oo

ILLUSTRATION 4 Addition of matrices is commutative as well as associative binary operation on


B

j^m xn
(set ofall m x n matrices over R). Multiplication of matrices is not commutative but it is associative
re

nxn
on R (set of all square matrices of order n over R).
ou
ad

ILLUSTRATION 5 Let S denote the set of all functions from a non-empty set A to itself. Clearly,
Y

composition offunctions '0' is a binary operation on S such that


fog * gof but (fog) oh =fo (goh) for all f,g, he S.
nd
Re

Hence, composition of functions'o' is associative but not a commutative binary operation on S.


Fi

ILLUSTRATION 6 If the operation * is defined on the set Q of all rational numbers by the rule
a*b = — for all a, b e Q. Shozv that * is associative on Q.
3
SOLUTION Let a,b,ce Q. Then,
r ab
ab 3 ) ^ (ab) c
(a*b)*c= — *c = ...(i)
3 9
be
a*
a (be)
and. a * (b * c)= —^ ...(h)
9 3
Since multiplication is associative onQ.
(ab) c = a (be)
(ab) c a (be)
=>
9 9
=> (a*b)* c=a*b * c) [By using (i) and (ii)]

ReadYourFlow.COM
BINARY OPERATIONS 3.7

Thus, (a* b) * c = a * (b * c) for all a, b, c e Q. Hence, * is associative on Q.


ILLUSTRATION 7 Examine whether the binary operation * defined onRbya*b=ab + lis associative
or not.
SOLUTION Let a,b, c e R. Then,
(a * b) * c ~(ab + 1) * c = (ab + 1) c + 1 = {ab) c + c + 1
and. a* (b * c) = a* (be + 1) =a(bc + 1) + 1 = a (be) + a + 1
Clearly, (ab) c + c + 1 * a (be) + a +1.
(a * b) * c * a* (b * c)
Hence, * is not associative on R.
ILLUSTRATION 8 is a binary operation defined on R, the set of all real numbers, by a* b = fa2 +b2
for all a, b e R. Show that * is associative on R.
SOLUTION Let a,b, c e R. Then,

w
(a*b) * c = ya2 +b2 *c + c 2=Ja2+b2+c2

2 = Ja2 | -Jb2 + c2 | = ija2 +b2 + c2


and, a*(b * c) =a2 * fb2 + c

Flo
ee
(a*b)* c = a* (b * c) for all arb, c <= R.

Fr
Hence, * is associative on R.
DISTRIBUTIVITY Let S be a non-empty set and * and 'O' be two binary operatins on S. Then, is said to
for
ur
be distributive over O, if for all a, b, c e S.
a*(b O c) = (a* b)Q (a* c) [Left distributivity of * over O ]
ks

and, (b O c) * a = (b * a) O (c * a) [Right distributivity of* over O ]


Yo
oo

The binary operation multiplication (•) on Z is distributive over the binary operation addition (+)
B

on Z, because
re

a .(b + c) = a .b + a .c
and. (b + c) .a = b . a + c . a for all a, b, c e Z.
ou
ad

However, addition (+) is not distributive over multiplication (•), because


Y

2 + (3 x 5) ^ (2 + 3) x (2 + 5).
nd

If S is a non-empty set, then union (u) is distributive over intersection (n) on P (S), because
Re

Avj(B nC) = (A u B) n(A ^jC) for all A, B,C eP (S).


Fi

Also, intersection (n) is distributive over union (u) on P (S).


ILLUSTRATIVE EXAMPLES
LEVEL-1
EXAMPLE l Discuss the commutativity and associativity of the binary operation '*' on R defined by
a + b =a + 1, + abfor alla,b eR, where on RHS we have usual addition, subtraction and mutiplication of
real numbers.
SOLUTION We have,
a* b = a + b + ab for all a,b e R.
Commutativity: Let a, b be any two elements of R. Then,
a* b = a + b + ab and b * a = b + a + ba
We know that the addition and multiplication of real numbers are both commutative binary
operations on R.
a + b + ab = b + a + ba for all a,b e R
=> a*b = b * a for all a,b eR.

ReadYourFlow.COM
3.8 MATHEMATICS-XII

So, is commutative on R.
Associativity: For any a,b, c eR, we have
a* (b * c) = a* (b + c + bc)
= a + (b + c + bc) + a (b + c + bc)
By commutativity, associativity of
addition and multiplication on R.
= a + b + c + bc + ab + ac + abc
Also, by distributivity of
multiplication over addition
= a + b + c + ab + be + ca + abc -(i)
and. (a*b) c = (a + b + ab) * c
= (a + b + ab) + c + (a + b + ab) c
= a + b +ab + c + ac +be + abc
= a + b + c + ab + bc + ca + abc ...(h)
From (i) and (ii), we have

low
a* (b * c) = (a* b)* c for all a,b, c eR
So, is associative on R.
EXAMPLE 2 Discuss the commutativity and associativity of the binary operation * on R defined by
a* b = ^ for all a,b eR.

ee
rF
Fr
SOLUTION We have,
ab
a*b = — foralla,b eR. or
Commutativity: For any a,b eR, we have
sf
u
, ab , , ba
a* b = — and b * a = —
k
Yo

4 4
oo

We know that the multiplication on R is a commutative binary operation.


eB

ab = ba for all a,b eR


ab = ba
— — cfor all
nuna,b eR
r

4 4
ou
ad

a*b = b * a for all a, t e K.


Y

So, '*' is a commutative binary operation on R.


nd

Associativity: Let a, b, c eR. Then,


Re

ab
Fi

c
ab 4 (ab) c
(a*b)* c = *c = -(i)
4 4 16
be
a
be 4 a (be)
and. a* (b * c) = a* ...(h)
4 4 16
Since multiplication is an associative binary operation on R.
(ab) c a (be)
(ab) c - a (be) => => (a*b)*c = a*(b * c) [Using (i) and (ii)]
16 16
Thus, (a* b) * c = a* (b * c) for all a,b, c e R.
So, * is an associative binary operation on R.
EXAMPLES Discuss the commutativity and associativity of binary operation defined on Q by the rule
a*b = a-b + ab for all a,b eQ. [NCERT EXEMPLAR]
SOLUTION We have,
a*b = a - b + ab for all a,b eQ.

ReadYourFlow.COM
BINARY OPERATIONS 3.9

Commutativity: For any a, b e Q, we have


a * b = a - b + ab and b * a = b - a + ba
Since a - b + ab * b - a + ba ior some a, b eQ.
a*b * b * a for some a,b eQ.
So, * is not commutative on S.
Associativity: Let a, b, c eQ. Then,
a*(b * c) = a * (b - c + be)
=> a* (b * c) = a -(b - c + bc) + a (b -c + bc)
=> a* (b * c) = a -b + c -be + ab -ac + abc
and, (a* b) * c = (a -b + ab) * c
=> (a*b)* c = (a -b + ab) - c + {a -b + ab) c
=> (a* b) * c = a -b + ab - c + ac -be + abc
=> (a*b)* c = a-b-c +ab + ac-be + abc •••(ii)
From (i) and (ii), we find that

low
a* (b * c) * (a* b) * c for some a,b, c eQ.
So, '*' is not associative on Q.
EXAMPLE 4 Let'*' be a binary operation on N, the set of natural numbers, defined bya*b - ab for all

ee
a,b e N. Is associative or commutative on N?
rF
Fr
SOLUTION We have,
2 * 3 = 23 = 8 and 3 * 2 = 32 = 9 or
2* 3 * 3* 2
sf
u
So, is not commutative on N.
Also, 2 * (2 * 3) = 2 * 23 = 2 * 8 = 28 = 256 and, (2 * 2) * 3 = 22 * 3 = 4* 3 = 43 = 64.
k
Yo
oo

Clearly, 2 * (2 * 3) * (2 * 2) * 3. So/*' is not associative on N.


B

Fience, '*' is neither commutative nor associative on N.


re

EXAMPLE 5 Let be a binary operation on N given by a*b = HCF (a, b) for all a,b eN.
ou

(i) Find : 12 * 4,18 * 24, 7 * 5


ad
Y

(ii) Check the commutativity and associativity of on N. [NCERT]


SOLUTION (i) Using definition of *, we obtain
nd
Re

12*4 = HCF (12, 4) = 4, 18 * 24 = HCF (18, 24) = 6 and, 7 * 5 = HCF(7, 5) =1


Fi

(ii) Commutativity: For any a,b eN, we have


a*b = HCF(a, b) = HCF(M) = b*a
So, '*' is commutative on N.
Associativity: For any a,b, c e N, we have
(a*b)*c = HCF(a, fr)*c = HCF(a,b, c)
and. a * (b * c) = a * HCF (b,c) = HCF(a, i>, c)
(a*b) * c = <? * (fr * c) for all a,b, c eN .
So, '*' is associative on N.
EXAMPLE 6 Consider the binary operations * : R x R R and o: Rx R R defined asa*b =\ a - b\
and aob =afor all a,b e R. Show that * is commutative but not associative, o is associative but not
commutative. Further, show that * is distributive over o. Dose o distribute over *? Justify your answer.
[CBSE 2012, NCERT]
SOLUTION For any a, b e R, we have
a * b =\ a -b\ and b * a = \b - a
\ a-b \ =\b -a\ for all a,b eR

ReadYourFlow.COM
3.10 MATHEMATICS-XII

a* b = b * afar all a,b eR


So, * is commutative on R.
We have.
((-2) *3) *4=|-2-3| *4=5*4=|5-4|=1
and. (- 2) * (3 * 4) = (- 2) * | 3 - 41 = (- 2) * 1 = | - 2 -11 = 3
((-2)* 3) *4 * (-2) *(3*4)
So, * is not associative on R.
We have, 2o3 = 2 and 3o2 = 3
2o3 * 3o2
So, o is not commutative on R.
For any a,b, c e R, we have
(aob) oc =aoc = a and ao (boc) =aob = a
(aob) oc = ao (boc) for all a, fc, c e R
So, o is associative on R.

w
For any a,b, c e R,we have
a * (boc) = a * b =\ a -b \, a*b=\a-b\,\a*c\=\a-c
and. (a*b) o(a* c)=\a-b\o\a-c\=\a-b\

Flo
a * (boc) =(a*b) o(a* c) for all a,b, c <=S

ee
So, * is distributive over 'o'.

Fr
Further, for any a,b, c & R, we have
ao(b * c) =ao\b -c\=a, aob =af aoc = a and (aob) * (aoc) = a*a=\a-a\=0
for
ur
ao (b * c) ^ (aob) * (aoc)
So, o is not distributive over' *'.
ks

LEVEL-2
Yo
oo

EXAMPLE 7 Let Abe a non-empty set and S be the set of all functions from A to itself. Prove that the
eB

composition of functions 'o' is a non-commutative binary operation on S. Also, prove that 'o' is an
associative binary operation on S.
r
ou

SOLUTION Let f,geS. Then,


ad

f: A A, g : A -> A
Y

=> fog: A A such that fog (x) =f(g (x)) for all x e A.
nd

fog e S.
Re

Thus, 'o' is a binary operation on S.


Fi

Commutativity: Let/, g eS be defined by / (x) = x2 for all x e A and, g (x) = sin x for all x e A.
Then,
fog (x) = f(g (x)) = / (sin x) = (sin x)2 sin2 x
and, gof (x) = g(f (x)) = g (x2) = sin x2
r\

Clearly, sin x * sin x for some x e A


=> fog (x) * gof (x) for some x e A
So, the composition of functions is not a commutative binary operation on S.
Associativity: Let f,g,he S. Then, f : A-> A, g: AA and h:A->A.
Let h(x) = y and g (y) = z. Then,
(fo (goh)) (x) = / ((goh) (x))
= fig (h (*)) [v h(x) = y]
= figiy))
= f(z) [•• giy) = z]

ReadYourFlow.COM
BINARY OPERATIONS 3.11

and. ((fog) oh) (x) = (fog) (h(x))


= (M (y) [v h(x) = y)
= f(g (!/))
= f(z) [••• g (y) = z] ...(ii)
From (i) and (ii), we have
(fo (goh)) (x) = ((fog) oh) (x) for all xeA
fo (goh) = (fog) oh
So, the composition of functions is an associative binary operation on S.
EXAMPLE 8 Let A = N x N and '*' he a binary operation on A defined by (a, b) * (c, d) = (ac, bd) for
all a,b, c, d eN. Show that is commutative and associative binary operation on A.
SOLUTION Let (a, b), (c, d) eNxN. Then, a, b,cfde N.
Now, a,b, c, d eN
=> ac, bd eN

low
=> (ac, bd) eN x N
Thus, (a, b), (c, d) eN x N
=> (ac, bd) e N x N ior all a,b, c, d eN
=> (a, b)*(c,d) eN xN

ee
(a, b) * (c, d) eA for all (a, b), (c, d) eA
=t>
rF
Fr
So, is a binary operation on A.
Commutativity: Let (a, b), (c, d) be any two elements of A. Then,
for
(a, b) * (c, d) = (ac, bd)
u
ks

and. (c, d) * (a, b) = (ca, db) =(ac, bd) [v ac = ca and bd - db for all a, b, c, d e N]
Yo
oo

(a, b) * (c, d) = (c,d)*(a,b)


B

Thus, (a, b) * (c, d) = (c, d) * (a, b) for all (a, b), (c, d) e A.
re

So, is a commutative binary operation on A.


Associativity: Let (a, b), (c, d), (e,f)e /l. Then,
ou
ad

(a, b) * {(c, d) * (e, f)) = (a, b) * (ce, df)


Y

= (a (ce), b (df))
"y Multiplication is associative on N
nd
Re

= ((ac) e, (bd) f)
a (ce) = (ac) e and b (df) = (bd) f
Fi

= (ac, bd) * (e, f)


= {(a, b) * (c, d)} * (e, f)
So,'*' is associative on A.
EXAMPLE 9 Let A be a set having more than one element. Let'*' be a binary operation on A defined by
a*b = a for all a,b eA.Is'*' commutative or associative on A?
SOLUTION Let a, b e A. Then,
a* b = a and b * a = b.
a*b * b * a
So, is not commutative on S.
Associativity: Let a,b, c e A. Then,
(a*b) * c = a* c = a and, a* (b * c) = a* b = a
(a*b) * c - a* (b * c) for all a,b, c e A.
So, is associative on A.
Thus, is associative on but it is not commutative on A.

ReadYourFlow.COM
3.12 MATHEMATICS-XII

EXERCISE 3.2
LEVEL-1
1. Let be a binary operation on N defined by a* b = l.c.m. (a, b) for all a,b eN.
(i) Find 2 * 4, 3 * 5,1 * 6.
(ii) Check the commutativity and associativity of on N.
2. Determine which of the following binary operations are associative and which are
commutative:
(i) * on N defined by a * b -1 for all a, b eN
(ii) * onQ defined by a * b = —^ for all a,b eQ [NCERT, CBSE 2008]
3. Let A be any set containing more than one element. Let be a binary operation on A
defined by a *b = b for all a, b e A. Is '*' commutative or associative on ^4 ?
4. Check the commutativity and associativity of each of the following binary operations:

low
(i) '*' on Z defined by a*b =a + b + ab for all a,b eZ.
(ii) on N defined by a*b = for all a,b eN.
(iii) '*' on Q defined by a*b = a -b for all a, b e Q.
(iv) 'O ' on Q defined by a O b = a2 +b2 for all a, b e Q.

ee
rF
Fr
(v) 'o' on Q defined by a ob = ^ for all n, b eQ.
for
(vi) '*' onQ defined by a* b = ab2 for all n,b eQ.
u
(vii) on Q defined by a* b = a + ab for all a, b e Q.
ks

(viii) on R defined by a * b = a + b - 7 for all a, b e Q.


Yo
oo

(ix) '*' on Q defined by a * b = (a - b)2 for all a, b e Q.


B

(x) '*' on Q defined by a*b = ab + 1 for all a, b e Q.


re

(xi) V on N, defined by a*b = a1’ for all a, b eN.


ou

(xii) '*' on Z a*b = a-b for al\a,b eZ.


ad

ab
Y

(xiii) on Q defined by a * b = — for all a, b e Q.


nd
Re

(xiv) '*' on Z defined by a */; =a + b -ab for all arb eZ.


(xv) on Q defined by a* b = gcd (a, b) for all a, b e N.
Fi

5. If thebinary operation o is defined by aob = a + b - ab on the set Q - {-1) of all rational


numbers other than 1, show that o is commutative on Q - [1],
6. Show that the binary operation * on Z defined by a * b = 3a + 7b is not commutative.
7. On the set Z of integers a binary operation * is defined by a* b = ab + 1 for all a, b eZ. Prove
that * is not associative on Z.
8. Let S be the set of all real numbers except -1 and let be an operation defined by
a*b = a + b + ab for all a,b e S. Determine whether is a binary operation on S. If yes,
check its commutativity and associativity. Also, solve the equation (2 * x) * 3 = 7.
a-b
9. On Q, the set of all rational numbers, * is defined by a*b = —^ , show that * is no
associative.
10. On Z, the set of all integers, a binary operation * is defined by a*b - a + 3b -4. Prove that *
is neither commutative nor associative on Z.
ab
11. On the set Q of all rational numbers if a binary operation * is defined by a * b - — , prove
5
that * is associative on Q.

ReadYourFlow.COM
BINARY OPERATIONS 3.13

ab
12. The binary operation * is defined by a*b = — on the set Q of all rational numbers. Show
that * is associative.
13. On Q, the set of all rational numbers a binary operation * is defined by a *b ------- . Show
that * is not associative on Q. ^
14. Let S be the set of all rational numbers except 1 and * be defined on S by
a*b =a + b-ab, for all /?, e S.
Prove that: (i) * is a binary operation on S
(ii) * is commutative as well as associative. [CBSE2014]
___________________________ ANSWERS
1. (i) 4,15,6 (ii) Commutative and associative both
2. (i) Both commutative and associative (ii) Commutative but not associative
3. Not-commutative but associative.
4. (i) Commutative and associative both (ii) Commutative but not associative

low
(hi) Neither commutative nor associative (iv) Commutative but not associative
(v) Commutative and associative both (vi) Neither commutative nor associative
(vii) Neither commutative nor associative (viii) Commutative and associative both
(ix) Commutative but not associative. (x) Commutative but not associative

ee
rF
(xi) Neither commutative nor associative (xii) Neither commutative nor associative

Fr
(xiii) Commutative and associative both (xiv) Neither commutative nor associative
(xv) Commutative and associative both
or
1
sf
ou
8. Yes, commutative and associative both, x = ~^
k

HINTS TO NCERT& SELECTED PROBLEMS


oo

a+b
Y
B

2.(ii) * on Q is defined by a * b = — for all a,b eQ.


re

Commutativity. For any a,b eQ


a+b b+a
ou
ad

a*b = =b * a
2 2
Y

So, * is commutative on Q.
Associatity. For any a,b, c, eQ
d
Re

a +b
n

+c
Fi

a+b a + b + 2c
(a*b)* c = — * c = 2
2 2 4
b+c
a+ 2a + b + c
and, a* {bn
* c)\ - a* ——
b +c
= 2
2 4
a + b + 2c 2a + b + c
Clearly, i.e. (a * b) * c = a * (b * c)
4 4
So, * is not associative on Q.
3.5 IDENTITY ELEMENT
IDENTITY ELEMENT Let '*' be a binary operation on a set S. If there exists an element e eS such that
a* e - a - e * a for all a eS.
Then, e is called an identity elemen t for the binary operation '*' on set S.
Consider the binary operation of addition (+) on Z. We know that 0 e Z such that
(7 + 0 = a = 0 + a for all a eZ
So, '0' is the identity element for addition on Z.

ReadYourFlow.COM
3.14 MATHEMATICS-XII

If we consider multiplication on Z, then '1' is the identity element for multiplication on Z,


because
1 x a = a = ax 1 for all a e Z.
We know that addition (+) and multiplication (x) are binary operations on N such that
h x 1 = n = 1 x n for all n e N
But, there do not exist any natural number e such that
n + e = n = e + n for all n e N.
So, 1 is the identity element for multiplication on N. But, N does not have identity element for
addition on N.
It follows from the above discussion that a set may or may not have an identity element for a
binary operation defined on it. Now, a natural question arises : If a set has an identity element
for a binary operation defined on it, how many identity elements can it have? The followig
theorem answers it.
THEOREM Let be a binary operation on a set S. If S has an identity element for then it is unique.

w
PROOF Let and e2 be two identity elements for the binary operation on S. Then,
e^ is identity element and e S => * e2 = e2
c2 is identity element and

Flo
e S => ^2=el ...(h)

ee
From (i) and (ii), we get tq = e2.

Fr
Hence, the identity element, if it exists, for a binary operation on a set is unique.
for Q.E.D.
REMARK Uptill now, we have been using article 'an' with identity element. As it is unique (if it exists).
ur
So, now onwards we shall be using article, 'the' with identity element for a binary operation in a given
set.
ks
Yo

ILLUSTRATION 1 Addition of matrices is a binary operation on the set Rmxn of all mxn matrices over
oo

R and O is the null matrix in Rm * n such that A + O = A =0 + A for all A e Rm xn. Therefore, O is the
B

identity element for addition on Rm x".


re

ILLUSTRATION 2 Multiplication of matrices is a binary operation on the set Rn xn of all n x n matrices


ou
ad

over R and In is the identity matrix in Rnx" such that AIn = A = In A for all A e Rn xn. Therefore, I
Y

is the identity element for multiplication of matrices on Rn xn.


ILLUSTRATION 3 Addition of vectors is a binary operation on set of all three dimensional vectors
nd
Re

^ ^ ^ ^ ^ —> —>
and 0 e V3 such that a+0 = a= 0 + a for all a e V3. So, 0 is the identity element for addition for
Fi

vectors on set V3.


ILLUSTRATIVE EXAMPLES
LEVEL-1

EXAMPLE 1 If* is defined on the set R0 of all non-zero real numbers by a *b =---- - ,find the identity
element in R for the binary operation *. ^ [CBSE 20121
SOLUTION Let e be the identity element in R for the binary operation * on R. Then,
a* e-a = e * a for all a eRQ
a* e = a and e* a = a for all a e R0
3ae Sea
=> ---- = a and =a for all a e Rq
7 7
7
=> e =—
3
Hence, 7 / 3 is the identity element in Rq.

ReadYourFlow.COM
BINARY OPERATIONS 3.15

EXAMPLE 2 Find the identity element in the set Q+ of all positive rational numbers for the operation *
defined by a* b = for all a,b e Q +.

SOLUTION Let e be the identity element in Q +. Then,


a*e=a=e*a for all a sQ +
=> a* e = a and e * a - a for all0 eQ +
ae ea
=> — = 0 and — = 0 for all 0 e Q +
2 2
e =2
Hence, 2 is the identity element in Q +.
EXAMPLE 3 If* is defined on the set R of all real numbers by a* b = fa2 +b2 ,find the identity element
in R with respect to*.

low
SOLUTION Let e be the identity element in R with respect to *. Then,
a*e=a = e*a for all 0 e R
a* e = a and e * a = a for all0 e R
=> 0“ + e2 = 0 and yje2 + a2 = a for all 0 e R
V«2

ee
=> 02 + e2=02 and e2 + 02 =02
rF
for all 0 e R

Fr
e=0 or
Hence, 0 is the identity element in R with respect to *.
sf
ou
LEVEL-2
k

EXAMPLE 4 Let Sbea non-empty set and P (S) be the power set of set S. Find the identity elementfor the
oo

union (u) as a binary operation on P (S).


Y
B

SOLUTION We observe that


re

Au<f) = /l=(j)Ui4 for every subset A of set S.


ou

=> A u <j) = yl =<j>Ui4 for all A eP (S)


ad

<j) is the identity element for union (u) on P (S).


Y

EXAMPLE 5 In example 4, find the identity elementfor intersection (n) as a binaiy operation onP (S).
d
Re

SOLUTION We observe that


n

AnS = A= SnA for every subset A of set S.


Fi

=> A n S = A = S n A for all A g P (S)


=> S is the identity element for intersection (n) on P (S).
EXERCISE 3.3
LEVEL-1
1. Find the identity element in the set /+ of all positive integers defined by 0 * fr = 0 + b for all
a,bel+.
2. Find the identity element in the set of all rational numbers except -1 with respect to *
defined by a * b = a + b + ab.
3. If the binary operation * on the set Z is defined by a * b =a + b -5, then find the identity
element with respect to*. [CBSE2012]
4. On the set Z of integers, if the binary operation * is defined by a*b=a + b +2, then find the
identity element. [CBSE 2012]
ANSWERS
1. 0 2. 0 3. 5 4.-2

ReadYourFlow.COM
3.16 MATHEMATICS-XII

3.6 INVERSE OF AN ELEMENT


INVERTIBLE ELEMENT Let'*' be a binary operation on a set S, and let e be the identity element in Sfor the
binary operation * on S. Then, an element a eSis called an invertible element if there exists an element
b eSsuch that a*b = e =b * a.
The element b is called an inverse of element a.
Thus, an element b e Sis called an inverse of an element aeS,if a*b = e = b * a.
Consider the binary operation addition (+) on Z. Clearly, 0 is the identity element for addition
on Z and for any integer a, we have
a + (-a) =0 = (-a) + a
So, - is the inverse of e Z.
Multiplication is also a binary operation on Z and 1 is the identity element for multiplication on
Z. But, no element, other than 1 e Z, is invertible.

w
THEOREM 1 bet be an associative binary operation on a set S with the identity element e in S. Then,
the inverse of an invertible element is unique.

respect to Then,

Flo
PROOF Let a be an invertible element in S. If possible, let b and c be two inverses of fl e S with

ee
a*b = b * a = e and, a* c = c* a = e

Fr
Now, (b * a)* c = e* c [v b* a = e\
c for[v e is the identity element]
ur
and. b * (a* c) = b * e [y a* c = e]
= b [y e is the identity element]
ks

Since '*' is an associative binary operation on S. Therefore,


Yo
oo

(b * a) * c = b* (a* c)
=> c = b.
eB

Hence, a has unique inverse.


Q.E.D
r
ou
ad

REMARK The inverse of an element is generally denoted by a~ l. The inverse of an element a (if it exists)
Y

with respect to the additive (or multiplicative) binary operations is generally called the additive (or the
multiplicative) inverse and is denoted by - a (or 1/a).
nd
Re

THEOREM 2 Let * be an associative binary operation on a set S and a be an invertible element ofS. Then,
Fi

(a-1)-1 = a.
PROOF Let e be the identity element in S for the binary operation * on S. Then,
a* a -1 = e = a- 1 * a
=> a~^ * a = e = a* a -1
=> a is inverse of a~ 1
=> «= (ct-y1 Q.E.D.
REMARK Let * be a binary operation on a set S and e be the identity element for * on S. Then,
e*e-e=e*e. This implies that e is invertible and c ~ 1 = e. Thus, the identity element (if it exists),
with respect to a given binary operation defined on a given set, is always invertible and it is inverse of
itself.
ILLUSTRATION 1 Multiplication is a binary operation on Q and 1 is the identity element in Q. For
n n m
every non-zero rational number — e Q there exists a rational number — such that — X — = 1 = -— X — .
n m n m m n
Thus, every non-zero rational number has its inversefor multiplication on N.

ReadYourFlow.COM
BINARY OPERATIONS 3.17

ILLUSTRATION 2 Addition of vectors is a binary operation on V3 zvith identity element 0 e I/3 For
—> —> —> —> —> —> —>
ei»en/ sector a e V3 i/zere ex/sfs - a eV3 such that a +(- a) = 0 =(- a) + a. Therefore, every vector
in V3 has its additive inverse.
ILLUSTRATIVE EXAMPLES
| LEVEL-1
ab
EXAMPLE 1 On Qq, the set ofall non-zero rational numbers, a binary operation * is defined by a * b = —
5
for all a,b eQq. Find the identity element for * in Q0- Also, prove that every element o/Q0 is invertible.
SOLUTION Let e be the identity element. Then,
a* e =a = e * a for all fl e Q0
ae , ea
— = a and — — = <7 for all a gQq
5 5

low
=> e =5
Thus, 5 is the identity element for the biliary operation * defined on Qq.
Let x be the inverse of an element a eQq. Then,
a*x=e=x*a=5

ee
a* x - 5 and x* a- 5
rF [v ? =5]

Fr
ax n , xa
=> — =5 and — =5
5 5
for
25
=> x = — , if « * 0.
ou
a
25
ks

Thus, every element fl e Q0 is invertible and its inverse is —.


a
oo

EXAMPLE 2 Let'*' be a binary operation on set Q - (1) defined by a* b = a + b - abfor all a, e Q - (1).
Y

Find the identity element with respect to * on Q. Also, prove that every element ofQ -11) is invertible.
B

[CBSE 2017]
re

SOLUTION Let the identity element e exists in Q - {1} with respect to * on Q - {!}. Then,
ou
ad

a* e = a = e * a for all a e Q - {1}


=> a* e a for all fl e Q - {1} [v '*' is commutative on Q - (1)]
Y

=> a + e - ae = a for all a e Q - (11


nd

e(l-a) = 0 for all a e Q - {1}


Re

=>
=> e - 0 [v e Q - (1) .\ * 1 => rt -1 ^0]
Fi

Thus, 0 is the identity element for * on Q - {1}.


Let a be an arbitrary element of Q - (1} and let b (if exists) be the inverse of a. Then,
a*b = 0 = b * a [v 0 is the identity element]
=> a*b = 0 [v is commutative]
=> a + b - ab = 0
=> b (l - a) = -a
=> [•.• a e Q - {1( .•. <?—l ^ 0]
<7-1
Since, <7 e Q - {!}. Therefore, b = ~~r e Q - {11
<7-1
Thus, every element of Q - {1} is invertible and the inverse of an element a is **—.
<?-l
EXAMPLE 3 On the set R - { -1) a binary operation * is defined by a * b = a + b + ab for all
a,b e R - {-1}. Prove that * is commutative as well as associative on R - {-1}. Find the identity element
and prove that every element ofR - {-1} is invertible. [CBSE 2015,2016]
SOLUTION We observe the following properties of * on R -{-!}.

ReadYourFlow.COM
3.18 MATHEMATICS-XII

Commutativity: For any a,b eR - {-1}, we have


a*b=a + b + ab and b * a =b + a + ba
a + b + ab = b + a + ba [By commutativity of addition and multiplication on R - 1}]
=> a*b =b * a
So, * is commutative on R -{-!}•
Associativity: For any a,b, c s R - {-l},\ve have
(a*b) * c = (a+ b + ab)* c
= (a + b + ab) + c + (a + b + ab) c = a+ b + c + ab+ bc + ac + abc
and. a* (b * c) = a * (b + c + be)
= a + (b + c + bc) + a (b + c + be) = a + b + c + ab + bc + ac + abc -(ii)
From (i) and (ii), we have
(a* b) * c = a* (b * c) for all fl, fr, c e R - {-1}.

w
So, * is associative on R -{-!}.
Existence of Identity: Let e be the identity element. Then,
a* e =a = e * a for all e R - {-1}

Flo
ee
=> a + e + ae = a and e + a + ea= a for all a eR - {-1}
=> e (1 + a) =0 for all a e R — (-1}

Fr
e =0.
Also, 0 e R - {-1}
for
ur
So, 0 is the identity element for * defined on R -{-!}.
ks

Existence of Inverse: Let a eR - {-1} and let b be the inverse of a. Then,


Yo

a*b =e =b * a
oo

a*b = e [v * is commutative]
eB

=> a + b + ab =0 [•• e=0]


-a
=> b=
r

a+1
ou
ad

Now, e R
Y

aeR -{-!}=> a*-l=>fl + l*o => b =


a+1
-a
nd
Re

Also, = -1 => -a = -a-1 => -1=0, which is absurd.


a+1
Fi

Thus, ^rsR-l-l).
a+1

Hence, every element of R - {-1} is invertible and the inverse of an element a is —


a+1
EXAMPLE 4 Let '*' be a binary operation on Q0 (set of all non-zero rational numbers) defined by
a* b = ^ for all a,b € Q0. Then, find the

(i) identity element in Q0 (ii) inverse of an element in Q0.


SOLUTION Identity element: Let e be the identity element in Q0. Then,
a* e = a = e * a for all a eQ0
a* e = a for all a eQ0 [v is commutative onQ0]
=> — = a for all a eQ^
4
=> c = 4 [ •• « ^ 0]

ReadYourFlow.COM
BINARY OPERATIONS 3.19

Thus, 4 is the identity element in Q0 for the binary operation


Inverse of an element: Let a be an invertible element in Q0 and let b be its inverse. Then,
a*b = e - b * a
=> a*b - 4: [v e - 4 and is commutative on Q0]
ab
=> — = 4
4
16
=> b = — [••• a eQ0 /.a * 0]
a
Clearly, — e Q0 for all a e Q0. Therefore, every element of Q0 is invertible and the inverse of an
a
element a eQ0 is—.
a
EXAMPLE 5 Let '*' be a binary operation on N given by a*b = L.C.M (a, b)for all a,b eN.

low
(i) Find5 *7,20* 16 (ii) Is * commutative?
(iii) Is * associative? (iv) Find the identity element in N
(v) Which elements ofN are invertible? Find them.

ee
SOLUTION (i) We have,
a*b = LCM of a and b
rF
Fr
5*7 - (LCM of 5 and 7) = 35 and, 20 * 16 = (LCM of 20 and 16) = 80
(ii) We have.
or
a * b = LCM of a and b and, b * a - LCM of b and a
sf
ou
We know that for any a,b eN
k

LCM of a and b = LCM of b and a


oo

a*b = b * a
Y
B

So, * is commutative on N
re

(iii) For any a, b, c eN, we have


ou

(a* b) * c = LCM of a*b and c


ad

= LCM of (LCM of a and b) and c


Y

= LCM of a, b and c
d
Re

Similarly, we have
n

a* (b * c) = LCM of a, b and c
Fi

(a*b) * c = a * (b * c) for all a, b, c eN


So, * is associative on N.
(iv) Let e be the identity element. Then,
a*e - a = e*a for all n e N
=> a* e = a for all a e N [v * is commuative]
=> LCM (a, e)=a for all a eN
=> e = 1
So, 1 is the identity element in N.
(v) Let a be an invertible element in N. Then, there exists b eN such that
a*b = 1
=> LCM (a, b) = 1
a - b = 1.
Thus, 1 is the only invertible element of N.

ReadYourFlow.COM
3.20 MATHEMATICS-XII

EXAMPLE6 Define a binary operation * on the set A ={0,1,2, 3, 4, 5} given by a* b = ab {mod 6).
Shoiv that 1 is the identity for *, 1 and 5 are the only invertible elements with 1“ 1 = 1 and 5 ” 1 =5.

SOLUTION We have,
1*0 = 0*1=0, 1*1=1=1*1, 1*2 = 2 = 2*1
1*3 = 3 = 3*1, 1*4 = 4 = 4*1,1*5=5=5*1
That is x * 1 = X = 1 * x for all x e A.
So, 1 is the identity element for * in A.
We have.
1*1=1=1*1
1 is invertible and 1 -1 = 1
Also 5*5 = (Remainder when 25 is divided by 6) = 1

w
5 is invertible and 5- 1 =5
x x
EXAMPLE 7 On the set M=A (x) =
x x

Flo
:x eR> of 2x 2 matrices, find the identity element for

ee
the multiplication of matrices as a binary operation. Also, find the inverse of an element ofM.

Fr
SOLUTION Let A (a) = “ ^ , a e R be the identity element in M. Then,
for
ur
A (x) A (a) = A (x) = A (a) A (x) for all x e R
x x a a "x xl , fa a x x
ks

=> = and for all x e R


X X a a x x a a x x X X
Yo
oo

2 ax 2ax * T for all x g R


B

=> 2 ax 2 ax X X
re

=> 2ax = x for all x g R


ou
ad

1
=> a =—
Y

2
a r
nd
Re

Thus, A ^ ^ is the identity element in M.


Fi

.2 2.
XX _
Let A ( i/) = y y be the inverse of an element A (x) = ^ in M. Then,
\y y\
A(x)A(y)=A^ = A(y)A(x)

i i
X X y y 2 2 y y X X
=>
* \y y_ 1 1 ly y] X X
2 2
1 1
=> ^ ^ ? 2 = y = J_,if„0
2xy 2xy 11 - 2 J 4x
2 2

ReadYourFlow.COM
BINARY OPERATIONS 3.21

4* is the inverse of (x) = * in M.


1
4x 4x

LEVEL-2

EXAMPLE 8 Let Xbe a non-empty set and let *be a binary operation on P (X) (the power set of set X)
defined by A * B = A u B for all A, B <= P (X). Prove that is both commutative and associative on
P (X). Find the identity element with respect to on P (X). Also, show that §eP (X) is the only
invertible element of P (X). [NCERT]
SOLUTION In Chapter 1 on sets in class XI, we have proved that for any three sets A, B,C
AuB = BkjA and (AuB)uC = Au(BuC)
Therefore, for any A, B,C eP (X), we have

low
A'u B = BuA and (A u B) uC = A u (B uC)
=> A * B = B * A and (A * B) * C = A * (B * C)
Thus, is both commutative and associative on P (X)

ee
We know that
rF
Fr
Au<}> = A = (J) u A for all A eP (X).
=> A * <j) = A = (j> * A for all A e P (X)
for
So, <t)is the identity element for '*' on P (X).
ou
Let A e P (X) be an invertible element. Then, there exists S e P (X) such that
ks

A*S = (|) = S*A


oo

=> AuS = (]) = So'A


Y
B

=> S = <i> = A
re

Thus, 4> is the only invertible element.


ou

EXAMPLE 9 Let X be a non-empty set and let'*' be a binary operation on P (X) (the pozver set of X)
ad

defined by A*B = AnB for A, B eP (X).


Y

(i) Find the identity element with respect to * in P (X).


nd
Re

(ii) Show thatX is the only invertible element ofP (X). [NCERT]
Fi

SOLUTION (i) Let £ be the identity element in P (X) with respect to *. Then,
A * £ = A = E* A for all A e P (X)
=> A r\E = A = E n A for all A c X
=> E = X.
Thus, X is the identity element with respect to * on P (X).
(ii) Let A be an invertible element of P (X) and let S be its inverse. Then,
A*S = X = S* A
=> AnS = X = SnA
=> A = S = X [v A <= X, S c= X]
Thus, X is the only invertible element of P (X) with respect to * and it is the inverse of itself.
EXAMPLE 10 Let X be a non-empty set and let be a binary operation on P (X)(the pozver set of set X)
defined by A* B =(A - B) (B - A) for all A, B e P (X). Show that:
(i) <J) is the identity element for * on P (X).
(ii) A is invertiblefor all A g P (X) and the inverse of A is A itself. [NCERT]

ReadYourFlow.COM
3.22 MATHEMATICS-XII

SOLUTION For any A e P (X), we have


A * (}) = (/4 - (|)) u (<}) - A)
= (A n (()') u(tj)n A') [v A-B = AnB']
= (Anl/)u<j) = Au<t) = A
and. (}>* A = (((>-A) u(A -<J>)
= ((j)n A') u(A n (()')
= c))u(AnLI)= <{)uA= A
A*<\> = A =<j)* A for all A e P (X).
Thus, <j>is the identity element in P (X) for the binary operation * on P (X).
For any A e P (X), we have
A* A =(A-A)u(A-A) =<f)u<{) = (|)
So, every element A of P (X) is invertible and is inverse of itself.
EXAMPLE 11 Let A = Q x Q and let * be a binary operation on A defined by

w
(a, b) * (c, d) = (ac, b + ad) for (a, b), (c, d) e A.
Then, with respect to * on A
(i) Find the identity element in A

Flo
SOLUTION (i) Let (x, y) be the identity element in A. Then,
(ii) Find the invertible elements of A.

ee
(a, b) * (x, y) = (a, b) = (x, y) * {a, b) for all (a, b) e A.

Fr
=> (ax, b + ay) = (a, b) = (xa, y + bx) for all a,b gQ
=> (ax, b + ay) - (a, b) and (a, b) - (xa, y + bx) for all a,b gQ
or
ur
=> ax = a and b + ay =b and, xa = a, y + bx = b for alla,b eQ
sf
=> x = l,y = 0
Clearly, (l,0)eQxQ = A.
k
Yo

So, (1, 0) is the identity element in A.


oo

(ii) Let (a, b) be an invertible element of A. Then there exists (c, d) e A such that
B

(a, b) * (c, d) = (l,0)=(c,d)*(a,b)


re

=> (ac, b + ad) = (1, 0) and (ca, d + be) =(1, 0)


ou

=> ac =l,b + ad = 0 and a? = l, d + be = 0


ad

1
Y

=> c = - and d = if a * 0.
a a
nd

Thus, (a, b) is an invertible element of A, if a * 0 and in such a case the inverse of (a, b) is
Re

fl -b'
Fi

a a
EXAMPLE 12 Let A = N u {0} x N u {0} and let'*' be a binary operation on A defined by
(a, b) * (c, d) =(a + c,b + d) for all (a, b), (c, d) g A.
Show that:
(i) is commutative on A. (ii) '*' is associative on A.
Also,find the identity element, ifany, in A. [NCERT]
SOLUTION (i) Commutativity : Let (a, b), (c, d) g A. Then,
(a, b) * (c, d) =(a + c,b + d) and (c, d) * (a,b) - (c + a, d + b)
a + c = c + a and b + d = d + b for all a, b, c, d gN
(a + c,b + d) = (c + a, d + b) for alla,b, c, d gN
=> (a, b) * (c, d) = (c, d) * (a, b) for all (a, b),(c, d) gN x N =A
=> is commutative on A.
(ii) Associativity For any (a, b), (c, d), (e, f) g A, we have
{(a, b) * (c, d)} * (e, f) - (a + c,b + d)* (e, f)
= ((a + c) + e, (b + d) + f)

ReadYourFlow.COM
BINARY OPERATIONS 3.23

= (a + (c + e),b + (d + f)) [v Addition is associative on N]


= (a,b) * (c + e, d + f)
= (a, b) * [(c, d)*(e/f)}
So, is associative on A.
Let (x, y) be the identity element in A. Then,
(a, b) * (x, y) = (a, b) for all (a, b) s A
=> (a + x,b + y) = (a, b) for all (a, b) g A
a + x = a,b + y = b for all a, b e N u {0}
x = 0,y = 0
Clearly, (0, 0) e A.
Also, (0, 0) * (a, b) = (a, b) for all (a, b) e A.
Thus, (0, 0) is the identity element in A.
EXAMPLE 13 Let A = N x N, and let * be a binary operation on A defined by

low
(a, b) * (c, d) = (ad + be, bd)for all (a, b), (c, d) sN x N.
Show that
(i) '*' is commutative on A. (ii) '*' is associative 071 A. (iii) A has no identity element.
SOLUTION (i) For any (a, b), (c, d) eN x Nr we have

ee
(a, b) * (c, d) = (ad + be, bd) and, (c, d) * (a, b) = (cb + da, db)
rF
Fr
Since addition and multiplication are commutative on N. Therefore,
ad + be = cb + da and bd = db or
(ad + be, bd) = (cb + da, db)
sf
u
=> (a,b) * (c, d) = (c, d) * (a,b)
k

So, * is commutative on A
Yo
oo

(ii) For any (a, b), (c, d), (e, f) e A, we have


B

{(a, b) * (c, d)} *(e,f)= (ad + be, bd) *(e,f)


re

= (ad + bc) f + (bd) e, (bd) f)


ou
ad

= (adf + bef + bde, bdf) -(i)


Y

and. (a, b) * l(c, d) * (e, /)} = (a, b) * (cf + de, df)


= (a (df)+b (cf + de), b (df ))
nd
Re

= (adf + bef + bde, bdf) ••■(ii)


Fi

From (i) and (ii), we get


[(a, b) * (c, d))*(e,f)= (a, b) * [(c, d)*(e,f)) for all (a, b), (c, d),(e,f) gN x N = A
So, * is associative on A.
(iii) Let (x, y) be the identity element in A. Then,
(a, b) * (x, y) = (a, b) for all a,b gN
=> (ay + bx, by) = (a, b) for all a,b gN
=> ay + bx = a and by -b for all a,b gN
=> x = 0, y = 1
But, 0 g N. Therefore, (0,1) £ N x N = A
So, there is no identity element in A with respect to binary operation 'on A.
EXAMPLE 14 Show that the number of binary operations on [1, 2) having 1 as identity and having 2 as
inverse of 2 is exactly one. [NCERT]
SOLUTION We know that a binary operation on a set S is a function from S x S to S. So, a
binary operation on set S = {1,2} is a function from SxS->S i.e., a function from
{(1,1), (1,2), (2,1), (2, 2)) to {1,2}.

ReadYourFlow.COM
3.24 MATHEMATICS-XH

Let* be the desired binary operation. If 1 is the identity element for * and 2 is the inverse of itself,
then 1*1=1, 1*2 = 2*1=2 and 2*2=1. Thus, * associates elements of S x S to elements of S
in the following manner.
*(1,1)=1, *(1,2) =2, * (2,1) = 2, * (2, 2) = 1
Sxs
(LD
(1,2)

(2,1)

(2, 2)

low
Clearly, * can be defined in a unique way as given above. Hence, the number of desired binary
operations is 1.
EXAMPLE 15 Determine the total number of binary operations on the set S = {1, 2,} having 1 as the
identity element. [NCERT]

ee
rF
SOLUTION Let * be the desired binary operation on S = {1, 2). Then, * is a function from

Fr
S x S = {(1,1), (1, 2), (2,1), (2, 2)} to S = {1, 2}. If 1 is the identity element for * on S. Then,
1 * 1 =1,1 * 2 = 2 * 1 = 2
or
sf
u
i.e.. * (1,1) =1, * (1, 2) = * (2,1) =2
k

Thus, the only choice left is to associate (2, 2) to some element of S. Clearly, (2, 2) can be
Yo
oo

associated to either 1 or 2 i.e., * (2, 2) = 1 or, * (2, 2) = 2.


B

So, there are two desired binary operations on S as given below:


re

(i) * (1,1) =1, * (1, 2) =* (2,1) =2and *(2, 2) =1


This can be represented by an arrow diagram as follows.
ou
ad
Y

SxS S

(LD
nd
Re

1
(1,2)
Fi

(2,1) 2

(2,2)

Fig. 3.2

(ii) * (1,1) = 1, * (1, 2) = * (2,1) = 2 and * (2, 2) = 2.


This can be represented by an arrow diagram as follows.
SxS ★
S
(1,1)
(1,2)

(2,1) 2
(2, 2)

Fig. 3.3

ReadYourFlow.COM
BINARY OPERATIONS 3.25

EXERCISE 3.4
LEVEL-1

1. Let * be a binary operation on Z defined by a*b=a+b- 4 for all a, & e Z.


(i) Show that is both commutative and associative.
(ii) Find the identity element in Z. (iii) Find the invertible elements in Z.
3ob
2. Let * be a binary operation on Q0 (set of non-zero rational numbers) defined by a*b =
for all a,b gQq-
Show that * is commutative as well as associative. Also, find its identity element, if it exists.
[CBSE2010]
3. Let * be a binary operation on Q - {-1) defined by a*b = a + b + ab for all rt, fr e Q - {-1).
Then,
(i) Show that is both commutative and associative on Q - (-1).

low
(ii) Find the identity element in Q - {-1)
(iii) Show that every element of Q - {-1) is invertible. Also, find the inverse of an arbitrary
element.
4. Let A = Rqx R, where R0 denote the set of all non-zero real numbers. A binary operation 'O'

ee
is defined on A as follows: (a, b) 0(c, d) = (ac, be + d) for all (a, b), (c, d) e R0x R.
rF
Fr
(i) Show that 'O' is commutative and associative on A
(ii) Find the identity element in A (iii) Find the invertible elements in A.
for
5. Let 'o' be a binary operation on the set Q0 of all non-zero rational numbers defined by
ou
a ob = —, for all a,b e Q0.
ks

(i) Show that 'o' is both commutative and associate.


oo
Y

(ii) Find the identity element in Q0. (iii) Find the invertible elements of Q0.
B

6. On R - {1(, a binary operation * is defined by a*b = a + b - ab. Prove that * is commutative


re

and associative. Find the identity element for * on R - {1}. Also, prove that every element of
ou

R - {1} is invertible.
ad

7. Let R0 denote the set of all non-zero real numbers and let A = Rqx Rq. If V is a binary
Y

operation on A defined by (a, b) * (c, d) = {ac, bd) for all {a, b), (c, d) e A.
nd

(i) Show that is both commutative and associative on A


Re

(ii) Find the identity element in A (iii) Find the invertible element in A.
Fi

8. Let * be the binary operation on N defined by a*b = ITCF of a and b.


Does there exist identity for this binary operation on N? [NCERT]
9. Let A-RxR and * be a binary operation on A defined by {a, b)* (c, d) = (a + c, b + d). Show
that * is commutative and associative. Find the binary element for * on A, if any.
[CBSE 2017]
_______ ANSWERS
1. (ii) 4 (iii) Inverse of fl in Z is 8 - fl 2. | 3. (ii) 0 (iii) a~ 1 = —« eQ-i-ll
a+1
4. (ii) (1, 0) (iii) Inverse of (a, b) e A is -j 6. e = 0, a- 1 = a
a -1
-1 4
5. (ii) 2 (iii) a — for all e Qq
a
1 1
7. (ii) (1,1) (iii) Inverse of {a, b) e A isl - 8. No 9. (0,0)
a'b

ReadYourFlow.COM
3.26 MATHEMATICS-XII

___________________________________ HINTS TO NCERT& SELECTED PROBLEMS


8. Let e be the identity element in N for the binary operation * defined by a* b = HCF of a and
b. Then,
a* e = e * a =a fora\\ a eN
=> * e =fl for all a e N [v * is commutative]
=> HCF of a and e is a
=> e is a factor of a
=> e =1 forfl=l, e =1, 2 for a = 2 and so on.
But, e must be unique for all a e N. Hence, the identity element in N does not exist.

3.4 COMPOSITION TABLE


A binary operation on finite set can be completely described by means of a table known as a

w
composition table. Let S = {fli, fl2' —/ an \ b6 a finite set and * be a binary operation on S. Then the
composition table for * is constructed in the manner indicated below.
We write the elements fl-j, ^ an °f fhe set S in the top horizontal row and the left vertical

Flo
column in the same order. Then we put down the element Oj * cij at the intersection of the row

ee
headed by ^ (1 < 1 < n) and the column headed by cij (1 < / < n) to get the following table:

Fr
al a2 “i an
for ai
ur
flj * fl} al * a2 a1 * cij al * aj a\*an
a2 a2 * ax a2 * a2 a2 * cij a2 * aj a2 * an
ks
Yo
oo

“i aj * “i * a2 <H * “i Oj * dj di * dn
B
re

dj * al dj * a2 dj * cij dj * dj dj * a,,
ai
ou
ad
Y

an an * a2 On * ^ On * dj dn * an
nd
Re

From the composition table we infer the following results.


Fi

(i) If all the entries of the table are elements of set S and each element of S appears once and
only once in each row and in each column, then the operation is a binary operation.
Sometimes we also say that the binary operation is well defined which means that the
operation * associates each pair of elements of S to a unique element of S. Many authors
say that S is closed under the operation *. But for us, this is a consequence of the definition
of binary operation.
(ii) If the entries in the table are symmetric with respect to the diagonal which starts at the
upper left corner of the table and terminates at the lower right corner, we say that the
binary operation is commutative on S, otherwise it is said to be non-commutative on S.
(iii) If the row headed by an element say dj coincides with the row at the top and the column
headed by dj coincides with the column on extreme left, then dj is the identity element in S
for the binary operation * on S.
(iv) If each row except the topmost row or each column except the left most column contains
the identity element then every element of S is invertible with respect to *. To find the
inverse of an element say dj, we consider row (or column) headed by dj. Then we

ReadYourFlow.COM
BINARY OPERATIONS 3.27

determine the position of Identity element e in this row (or column). If e appears in the
column (or row) headed by aj, then q and fly are inverse of each other.
We shall now introduce two binary operations on the set Zn of integers modulo n. These
two binary operations are called addition modulo n (written as +n or, ©„) and
multiplication modulo n (written as x „ or ® n).
3.4.1 ADDITION MODULO n
Let n be a positive integer greater than 1 and a,b e Zn , where Zn = {0,1, 2,..., (n -1)}. Then, we
define addition modulo n i.e. +„ as follows:
a+nb = Least non-negative remainder when a + b is divided by n.
For example,
(i) 7+56 = (Least non-negative remainder when 7 + 6 = 13 is divided by 5) = 3.
(ii) 6 +10 8 = (Least non-negative remainder when 6 + 8 = 14 is divided by 10) = 4.

low
(iii) II+7 9 = (Least non-negative remainder when 11 + 9 = 20 is divided by 7) = 6.
The composition table for +5 on Z5 = {0,1, 2, 3, 4} is as given below:

0 1 2 3

ee
+5
rF
Fr
0 0 1 2 3
1 1 2 3 4 0
or
2 2 3 4 0 1
sf
u
3 3 4 0 1 2
k
Yo

4 4 0 1 2 3
oo
B

We observe the following points from the composition table:


re

(i) All entries of the table are elements of Z5. So, +5 is a binary operation on Z5.
(ii) The table is symmetric with respect to the principal diagonal. Therefore, +5 is a
ou
ad

commutative binary operation on Z5.


Y

(iii) The row headed by 0 coincides with the top-most row and the column headed by 0
coincides with the left-most column. So, 0 is the identity element for+5 on Z5.
nd
Re

(iv) Each row and column consists of the identity element 0. So, every element of Z5 is
Fi

invertible. Also,
0+5=0 => 0 is inverse of itself
1 +5 4 = 0 => 1 is inverse of 4
2 +5 3 = 0 => 2 is inverse of 3
3 +5 2 = 0 => 3 is inverse of 2
4 +5 1 = 0 => 4 is inverse of 1
(v) We have,
(1+5 3) +5 4 = 4 +5 4 = 3 and 1 +5 (3 +5 4) = 1 +5 2 = 3
(1+5 3) +5 4 = 1 +5 (3 +5 4)
Similarly, it can be verified for other elements of Z5 that +5 is associative on Z5.
3.4.2 MULTIPLICATION MODULO n
Let n be a positive integer greater than 1 and a,b e Z„, where Z„ = {0,1, 2, 3,...,(«-1)}.
Then, we define multiplication modulo n i.e., xn as follows:
axnb = Least non-negative remainder when ab is divided by n.

ReadYourFlow.COM
3.28 MATHEMATICS-XII

For example,
(i) 4 x- 3 = (Least non-negative remainder when 4 x 3 = 12 is divided by 5) =2
(ii) 4xg6 = (Least non-negative remainder when 4 x 6 = 24 is divided by 8) = 0
(iii) 7 x-^ 8 = (Least non-negative remainder when 7 x 8 =56 is divided by 12) = 8.
Consider x10 on the set S = {2, 4, 6, 8}. The composition table for x-[0 on S is given below:
4 6 8
8 2 6
6 4 2
4 6 8
2 8 4
We make the following observations from the composition table:

w
(i) All entries of the table are elements of S. So, x ^ is a binary operation on S.
(ii) The table is symmetric with respect to the principal diagonal. Therefore, x10 is
commutative on S.

Flo
(iii) The row headed by 6 coincides with the top most row and the column headed by 6
coincides with the left-most column. Their intersection is 6. So, 6 is the identity element

ee
for x10 on S.

Fr
(iv) Since each row and each column consists of the identity element 6. So, each element of S
is invertible. Also,
for
ur
2 xio 8 - 6=> 2 -1 = 8, 8x1q2 = 6=>8 1=2, 4x1q4 = 6=>4 -1 = 4
6 x-^g 6 = 6 => 6 -1 = 6
ks

and,
Yo
oo

REMARK Multiplication modulo n (x;;) is associative because the remainders when the integers
(a x b) x c and ax (b x c) are divided by n are same.
eB

It should be noted that the composition table is helpless to determine associativity of the binary
operation. This has to be verified for each possible triad.
r
ou
ad

To illustrate the points discussed above we consider the following examples.


Y

ILLUSTRATIVE EXAMPLES
nd
Re

LEVEL-1
Fi

EXAMPLE l Consider the set S = (l, -1} of square roots of unity and multiplication (x) as a binary
operation on S. Construct the composition tablefor multiplication (x) on S. Also, find the identity element
for multiplication on S and the inverses of various elements.
SOLUTION The composition table for multiplication on S is as given below:

x 1 -1
1 1 -1
1 -1 1

We make the following observations from the table:


(i) All the entries of the table belong to S. So, multiplication is a binary operation onS.
(ii) The table is symmetric with respect to the principal diagonal (i.e., the diagonal that starts
from the upper left corner of the table and terminates at the lower right corner). So, the
binary operation i.e., multiplication is commutative on S.

ReadYourFlow.COM
BINARY OPERATIONS 3.29

(iii) First row of the table coincides with the top-most row and first column coincides with the
left-most column. These two intersect at 1. So, 1 is the identity element for multiplication
on S.
(iv) Every element of S is invertible with respect to multiplication, because the identity
element 1 appears in each row and each column. Also, (1)_ 1 = 1 and (-1)_ 1 = 1.
(v) Since multiplication of numbers is associative. So, multiplication is associative on S.
EXAMPLE 2 Consider the set S = {1, co, co } of all cube roots of unit}/. Construct the composition tablefor
multiplication (x) on S. Also, find the identity element for multiplication on S. Also, check its
commutativity and find the identity element. Prove that every element ofS is invertible.
SOLUTION The composition table for multiplication on S is as given below:
X 1 CO CO2

CO2
1 1 CO

low
CO CO co2 1 [v co3 = 1 and co4 = co]
CO2 CO2
1 CO

We make the following observations from the table:

ee
rF
(i) All the entries of the table belong to S. So, multiplication is a binary operation on S.

Fr
(ii) The table is symmetric with respect to the principal diagonal. Therefore, multiplication is
commutative on S. or
(iii) First row of the table coincides with the top-most row, first column coincides with the left
sf
u
most column and these two intersect at 1. So, 1 is the identity element for multiplication
on S.
k
Yo
oo

(iv) The identity element 1 occurs in each row and each column. So, every element of S is
invertible. Also,
B

1x1=1 => 1“1 =1, cox co2 =1 => (co)- 1 = co2 and, co2 x co = l => (co2)- 1 = co
re

EXAMPLES Consider the set S ={1, -1, z, - z] offourth roots of unity. Construct the composition table
ou
ad

for multiplication on S and deduce its various properties.


Y

SOLUTION The composition table for multiplication on S is as given below:


nd
Re

x 1 z -z
Fi

1 1 z -z
-1 -1 1 -z z
z z -i -1 1
-z -z z 1 -1
We make the following observations:
(i) All the entries of the table belong to S. So, multiplication is a binary operation on S.
(ii) The table is symmetrical with respect to the principal diagonal. Therefore, multiplication
is commutative on S.
(iii) 1 is the identity element, because the row headed by 1 coincides with the top most row
and column headed by 1 coincides with the left most column and these two intersect at 1.
(iv) Each row and each column consists of the identity element 1. So, every element of S is
invertible. Also,
1 x 1 =1 => 1 -1 = 1, -1 X -1 =1 => (-1) -1 = -1, z x - z = 1 => (z) 1 = - z
and, - z x z = 1 => (-z)- 1 = z

ReadYourFlow.COM
3.30 MATHEMATICS-XII

EXAMPLE 4 Consider the set S = {1,2, 3, 4}. Define a binary operation * on S asfolloius:
a*b = r, where r is the least non-negative remainder when ab is divided by 5.
Construct the composition tablefor'*' on S.
SOLUTION We have,
1*1 = 1, 1*2=2, 1*3 = 3, 1*4 = 4, 2*1=2, 2*2 = 4, 2*3 = 1,
2*4 = 3, 3*1=3, 3*2 = 1, 3*3 = 4, 3*4 = 2 etc.
So, we obtain the following table as the composition table for the binary operation * on S.
* 1 2 3 4
1 1 2 3 4
2 2 4 1 3
3 3 1 4 2
4 4 3 2 1

w
We make the following observations from the composition table :
(i) The binary operation * is commutative on S, because the composition table is symmetrical

Flo
about the diagonal starting at the upper left comer and ending at the lower right corner.

ee
(ii) 1 is the identity element for *, because the row headed by 1 coincides with the top row
and the column headed by 1 coincides with the extreme left column of the table and these

Fr
two intersect at 1.
(iii) Every element of S is invertible with respect to *, because the identity element 1 appears
for
ur
in each row (column). Since 1 is the identity element, so 1 is inverse of itself. We see that
in second row 1 appears at the intersection of row headed by 2 and column headed by 3.
ks

So, 2 and 3 are inverse of each other. Similarly, we find that 4 is inverse of itself.
Yo
oo

EXAMPLES Consider the infimum binary operation a on the set S = {1, 2, 3, 4,5} defined by
a Ab = Minimum of a and b. Write the composition table of the operation a.
B

SOLUTION We have, [NCERT, CBSE 2011]


re

1 a 1 = (Minimum of 1 and 1) = 1, 1 a 2 = (Minimum of 1 and 2) = 1


ou

4 a 3 = (Minimum of 4 and 3) = 3 etc.


ad
Y

So, we have the following composition table for a on S.


nd

1 2 3 4 5
Re

1 1 1 1 1 1
Fi

2 1 2 2 2 2
3 1 2 3 3 3
4 1 2 3 4 4
5 1 2 3 4 5
EXAMPLE 6 Consider a binary operation * on the set {1, 2, 3, 4, 5} given by the following multiplication
table
* 1 2 3 4
1 1 1 1 1
2 1 2 1 2 1
3 1 1 3 1 1
4 2 1 4
5 1 1 1

ReadYourFlow.COM
BINARY OPERATIONS 3.31

(i) Compute (2 * 3) * 4 and 2 * (3 * 4)


(ii) Is * commutative?
(iii) Compute (2 * 3) * (4 * 5) [NCERT]
SOLUTION (i) From the composition table we find that
2* 3=1 and 1*4=1
(2* 3) *4 =1*4=1
(ii) Clearly, the composition table is symmetrical about the diagonal starting at the upper left
corner and ending at the lower right comer. So, * is commutative.
(iii) From the composition table, we find that
2*3=1 and 4*5=1
(2* 3) *(4*5) =1*1=1.
EXAMPLE 7 Define a binary operation * on the set A ={0,1, 2, 3, 4:r5} as a * b -a + b {mod 6).
Show that zero is the identity for this operation and each element a of the set is invertible with 6- a being

low
the inverse of a.
OR
a+b ,\i a + b <6
A binary operation * on the set {0,1,2, 3, 4,5} is defined as a* b = . Show that
a + b-6, iffl + fr>6

ee
rF
zero is the identity for this operation and each element 'a' of the set is invertible with 6 - a, being the

Fr
inverse of'a'. [CBSE2011]
SOLUTION We have,
or
a*b = a + b (mod 6) = Remainder when a + b is divided by 6
sf
ou
0*1 = (Remainder when 0 +1 = 1 is divided by 6) = 1
k

2*3 = (Remainder when 5 is divided by 6) = 5


oo
Y

3*4 = (Remainder when 3 + 4 = 7 is divided by 6) = 1


B
re

4*5 = (Remainder when 4 + 5 = 9 is divided by 6) = 3 etc.


So, the composition table for * is as given below:
ou
ad
Y

* 0 1 2 3 4 5
d
Re

0 0 1 2 3 4 5
n
Fi

1 1 2 3 4 5 0

2 2 3 4 5 0 1

3 3 4 5 0 1 2

4 4 5 0 1 2

5 5 0 1 2 3

We observe that the first row coincides with the top-most row and first column coincides with
the left most column. At their intersection, we have 0. So, 0 is the identity element.
Each row (column) contains the identity element. So, each element of A is invertible.
Also, a* {6 - a) = (Remainder when £7+6-fl = 6is divided by 6) = 0
6 - a is the inverse of a for each a e A.

ReadYourFlow.COM
3.32 MATHEMATICS-XII

EXAMPLE 8 Define a binary operation * on the set A ={1,2, 3, 4} as a* b =ab {mod 5). Show that 1 is
the identityfor * and all elements of the set A are invertible with 2-1 = 3 and 4-1 = 4.
SOLUTION We have,
a*b = ab (mod 5) = Remainder when ab is divided by 5
2*3 = (Remainder when 2 x 3 = 6 is divided by 5) = 1
3*4 = (Remainder when 3 x 4 = 12 is divided by 5) = 2
4*4 = (Remainder when 4 x 4 = 16 is divided by 5) = 1 etc.
So, the composition table for * is as given below:

* 1 3 4

1 1 2 3 4

low
2 2 4 1 3

3 3 1 4 2

ee
4 4 3 2 1
rF
Fr
We observe that the first row of the composition table coincides with the top-most row and first
for
column coincides with the left-most column. These two intersect at 1. So, 1 is the identity
u
element. Since each row (column) of the composition table contains the identity element 1. So,
ks

each element of A is invertible.


Yo
oo

From the table, we find that


B

2* 3 =1 = 3* 2 and 4*4=1
re

2“ 1 = 3 and 4- 1 =4.
ou

LEVEL-2
ad
Y

EXAMPLE 9 Construct the composition table for the composition of functions (o) defined on the set
S = {/1, /2, /3, /4| of fourfunctions from C (the set of all complex numbers) to itself, defined by
nd
Re

fl (z) =Z, f2 (z) = -Z, /3 (2) = -, /4 (2) = - - for all zeC.


Fi

z z
SOLUTION In order to construct the composition table we write the elements ^, /2, /3, /4 in a
horizontal row as well as in a vertical column and fill up the cells with the composition given
below.
For any z e C, we have
(/l°/l)(z) = fl (A (z)) =/i(z)
f\°fl = f\
Similarly, /l0/2 =/2 =f2of1,f1of3 =/3 =f3qf1 ,/l0/4 =/4 =/4 o/j
Also, {f2 0 f2 (z) = f2 (f2 (2)) =/2 (-z) =-(-z) =z=f1 (z)
/2°/2 = /l
M3 (2) =/2(/3(2))=/2(j)=-J
-=/4(2)

/2°/3 = /4

ReadYourFlow.COM
BINARY OPERATIONS 3.33

/2°/4 (*) = h (A (Z)) =/2 = -(“) = p/3 W

/2°/4 “ /3
Similarly, we can make other computations.
Thus, we obtain the following composition table:

o fl fl h
fl fl fl /3

fi fi fi k h
h fi k fi fi

k k fz

w
fi fi

We make the following observations from the table:

Flo
(i) The table is symmetrical about the leading diagonal. So, 'o' is commutative on S.
(ii) fa is the identity element for 'o' on S.

ee
(iii) The composition of functions is associative. So, 'o‘ is associative on S.

Fr
(iv) We have,
fl °fl =fl => /i ~ 1 =fl 'fl °fl =fl => /2 -i -fi'h°f3-fi => /a 1-/3and'
for
ur
k °k =fi => k =/4~1
ks
Yo

EXERCISE 3.5
oo

LEVEL-1
eB

1. Construct the composition table for x4 on set S = {0,1, 2, 3}.


r

2. Construct the composition table for +5 on set S = (0,1, 2, 3, 4).


ou
ad

3. Construct the composition table for x6 on set S = {0,1, 2, 3, 4,5}.


Y

4. Construct the composition table for x5 on Z5 = {0,1, 2, 3, 4).


5. For the binary operation x 10 on set S = {1, 3,7, 9), find the inverse of 3.
nd
Re

6. For the binary operation x7 on the set S = {1, 2, 3, 4,5, 6}, compute 3_ 1 x7 4.
Fi

7. Find the inverse of 5 under multiplication modulo 11 on Z^.


8. Write the multiplication table for the set of integers modulo 5.
9. Consider the binary operation * and o defined by the following tables on set S = {a, b, c, d).

a b c d

a a b c d

(i) b d c

c c d a b

d d c b a

Show that both the binary operations are commutative and associative. Write down the
identities and list the inverse of elements.

ReadYourFlow.COM
3.34 MATHEMATICS-XII

10. Define a binary operation * on the set {0,1,2,3,4, 5} as


a+b , if a+ b <6
a*b =
fl + &-6,iffl + fc>6
Show that 0 is the identity for this operation and each element * 0 of the set is invertible
with 6 - a being the inverse of a. [NCERT]
[Hint: See Example 7 on page 3.31]
ANSWERS

1. x4 0 1 2 0 1

0 0 0 0 0 1

1 0 1 2 1 2

w
2 0 2 0 2 3

3 0 3 2

Flo 3 4

ee
4 0

Fr
for
ur
3. x6 0 1 x5 1
0 0 0 1 1
ks
Yo
oo

1 0 1 2 2
B

2 0 2 3 3
re

3 0 3 4 4
ou
ad

4 0 4
Y

5 0 5
d
Re
n

5. 7 6. 6
Fi

8. x5 4
0 1 2 3

0 0 0 0 0 0

1 0 1 2 3 4
0 2 4 1
0 3 1 4
0 4 3 2

9. (i) Identity = a, fl 1=b,c 1 =c, rf 1 = d

(ii) Identity =b, a -1 does not exist, b ~ ^ =b, c ~ 1 = d, d -1 = c

ReadYourFlow.COM
BINARY OPERATIONS 3.35

HINTS TO NCERT& SELECTED PROBLEMS

10. The binary operation * on set S = {0,1, 2, 3, 4,5} is defined as

a + b , if a + b <6
a* b = for all a,b <= S.
a + b - 6 , iffl + b >6

The composition table for * is as given below:

low
ee
rF
Fr
We observe that the first row of the above table coincides with the top most row and first
or
column coincides with the left most column. At their intersection, we have 0. So, 0 is the
sf
u
identity element. Since each row (column) consists of the identity element. So, each element
of S is invertible.
k
Yo

Also, a* (6-a) = a+6-a-6 = 0 [v a + (6 - a) >6]


oo

6 - a is the inverse of a for each a eS.


B
re

_________________________________ VERY SHORT ANSWER QUESTIONS (VSAQs)


Answer each of the follozving questions in one word or one sentence or as per exact requirement of the
ou
ad

question:
Y

1. Write the identity element for the binary operation * on the set Rq of all non-zero real
numbers by the rule a * b = -y- for all a, b e Rq.
nd
Re
Fi

2. On the set Z of all integers a binary operation * is defined by * b = a + b + 2 for all a, b e Z.


Write the inverse of 4.
3. Define a binary operation on a set.
4. Define a commutative binary operation on a set.
5. Define an associative binary operation on a set.
6. Write the total number of binary operations on a set consisting of two elements.
7. Write the identity element for the binary operation * defined on the set R of all real numbers
by the rule a*b = for all «, b e R.

8. Let * be a binary operation, on the set of all non-zero real numbers, given by a * b = — for all
5
a,b e R - {0). Writethe valueofxgivenby2 * (x * 5) =10. [CBSE2014]
9. Write the inverse of 5 under multiplication modulo 11 on the set {1,2,.. .,10).
10. Define identity element for a binary operation defined on a set.

ReadYourFlow.COM
3.36 MATHEMATICS-XII

11. Write the composition table for the binary operation multiplication modulo 1 q) on the
set S = {2, 4, 6, 8).
12. For the binary operation multiplication modulo 10 (x10) defined on the set S = {1, 3, 7, 9),
write the inverse of 3.
13. For the binary operation multiplication modulo 5 ljx5) defined on the set S = {1, 2, 3, 4}.
Write the value of (3 x5 4-1)-1.
14. Write the composition table for the binary operation <5 (multiplication modulo 5) on the set
5 = 10,1,2, 3,4).
15. A binary operation * is defined on the set R of all real numbers by the rule a*b =yja2 +b2
for all a, fr sR. Write the identity element for * on R.
16. Let +6 (addition modulo 6) be a binary operation on 5 = {0,1, 2, 3, 4,5). Write the value of
2+6 4-'+6 3-'.

w
17. Let * be a binary operation defined by a * fr = 3tf + 4b - 2. Find 4*5. [CBSE2008]
18. If the binary operation * on the set Z of integers is defined by a *b = a + 3b2, find the value
of 2 * 4.

Flo [CBSE 2009]


19. Let * be a binary operation on N given by a*b = HCF (a, b), a, b e N. Write the value of

ee
22*4. [CBSE 2009]

Fr
20. Let * be a binary operation on set of integers I, defined by a * b = 2ci + b - 3. Find the value of
3*4. [CBSE 2011]
for
ur
ANSWERS
1. 2 2.-8 6. 16 7. 7/3 8. 25
ks

9. 9
Yo
oo

11.
x10 2 4 6 8 12. 7 13. 3
B

2 4 8 6
re

2
ou
ad
Y
nd
Re

14.
Fi

15. 0
MULTIPLE CHOICE QUESTIONS (MCQs)
Mark the correct alternative in each of thefollozving:
1. If fl * b = a2 +b2, then the value of (4 * 5) * 3 is
(a) (42 +52) + 32 (b) (4 + 5)2 + 32 (c) 412 + 32 (d) (4 + 5+ 3)2
2. If a * b denote the bigger among a and b and if a-b = {a*b) + 3, then 4.7 =
(a) 14 (b) 31 (c) 10 (d) 8

ReadYourFlow.COM
BINARY OPERATIONS 3.37

3. On the power set P of a non-empty set A, we define an operation A by


XAY=(XnY)u(XnY)
Then which are of the following statements is true about A
(a) commutative and associative without an identity
(b) commutative but not associative with an identity
(c) associative but not commutative without an identity
(d) associative and commutative with an identity
4. If the binary operation * on Z is defined by a * b = a -b + ab + 4c, then value of (2 * 3) * 4 is
(a) 233 (b) 33 (c) 55 (d) -55
5. For the binary operation * on Z defined by a *b = a + b + 1 the identity element is
(a) 0 (b) -1 (c>l (d) 2
6. If a binary operation * is defined on the set Z of integers asa*b = 3a-b, then the value of

low
(2 * 3) * 4 is
(a) 2 (b) 3 (c) 4 (d) 5
7. Q+ denote the set of all positive rational numbers. If the binary operation O on Q+ is
ab

ee
defined as a O b - then the inverse of 3 is
2 '
rF
Fr
(a) 4/3 (b) 2 (c> 1/3 (d) 2/3
X X
for
8. If G is the set of all matrices of the form ^ , where x eR - {0}, then the identity element
ou
with respect to the multiplication of matrices as binary operation, is
ks

ri n -1/2 -1/2 1/2 l/2~ -i


oo

(a) 1 1 (b) -1/2 -1/2 (c)


1/2 1/2 -1
Y
B

9. Q+ is the set of all positive rational numbers with the binary operation * defined by
re

a*b = — for a\\ a, b e Q +. The inverse of an element a eQ+ is


ou
ad

2
Y

(a) a (b) - (c) - (d) -


a a a
nd
Re

10. If the binary operation O is defined on the set Q+of all positive rational numbers by
Fi

~ . Then, 3 O Q O ~ j is equal f°
aQ b =

.(c), —3 (d) 4
(a) 7T
160
(b) -I
160 10 40
11- Let * be a biliary operation defined on setQ -{!) by the rule a*b=a + b-ab. Then, the
identity element for * is
a-1
(a) 1 (b) (d) 0
a
12. Which of the following is true ?
(a) * defined by a*b = is a binary operation on Z

(b) * defined by a* b = is a binary operation on Q

(c) all binary commutative operations are associative


(d) subtraction is a binary operation on N

ReadYourFlow.COM
3.38 MATHEMATICS-XII

13. The binary operation * defined on N by a*b=a + b + ab for all n, b eN is


(a) commutative only (b) associative only
(c) commutative and associative both (d) none of these
14. If a binary operation * is defined by a * b = a + b^ + ab + 1, then (2 * 3) * 2 is equal to
(a) 20 (b) 40 (c) 400 (d) 445
15. Let * be a binary operation on R defined by a* b = ab + \. Then, * is
(a) commutative but not associative (b) associative but not commutative
(c) neither commutative nor associative (d) both commutative and associative
16. Subtraction of integers is
(a) commutative but not associative (b) commutative and associative
(c) associative but not commutative (d) neither commutative nor associative
17. The law a + b =b + ais called

w
(a) closure law (b) associative law
(c) commutative law (d) distributive law
18.

Flo
An operation * is defined on the set Z of non-zero integers by a*b = - for all a, b e Z. Then

ee
the property satisfied is ^

Fr
(a) closure (b) commutative (c) associative (d) none of these
19. On Z an operation * is defined by ci * b = a2 + b2 tor all a, b e Z. Tire operation * on Z is
for
ur
(a) commutative and associative (b) associative but not commutative
(c) not associative (d) not a binary operation
ks
Yo

20. A binary operation * on Z defined by a * b = 3a + b for all a, b e Z, is


oo

(a) commutative (b) associative


eB

(c) not commutative (d) commutative and associative


ab
r

21. Let* be a binary operation on Q+ defined by a * =^^forallfl/& eQ+. The inverse of 0.1 is
ou
ad
Y

(a) 105 (b) 104 (c) 106 (d) none of these


22. Let * be a binary operation on N defined by a * b = a + b + 10 for all a, b e N. The identity
nd
Re

element for * in IV is
Fi

(a) -10 (b) 0 (c) 10 (d) non-existent


23. Consider the binary operation * defined on Q - {1) by the rule a* b = a + b - ab for all
a,b eQ - {!}. The identity element in Q -{1} is
(a) 0 (b) 1 (C)! (d) -1

24. For the binary operation * defined on R - (-1) by the rule a * b = a + b + ab for all a,b e R- {1},
the inverse of a is
(a) -a (b) - — (c) 1 (d) a2
a +1 a
25. For the multiplication of matrices as a binary operation on the set of all matrices of the form
a b 2 31 .
,a,b&R the inverse of -3 2 15
-b a

(a)
-2 3~ 2 3 2/13 -3/13' fl 01
-3 -2 (b) -3 2 (c) (d)
3/13 2/13 0 1

ReadYourFlow.COM
BINARY OPERATIONS 3.39

26. On the set Q+ of all positive rational numbers a binary operation * is defined by a*b = ^-
for a\\ a, b e Q +. The inverse of 8 is
1 (b)'
(a) (c) 2 (d) 4
8
27. Let * be a binary operation defined on Q+ by the rule a * b = — for all a, b e Q+. The inverse
3
of 4 * 6 is
(a) ?8 (b)f , \ 3
(C)I (d) none of these

28. The number of binary operations that can be defined on a set of 2 elements is
(a) 8 (b) 4 (c) 16 (d) 64
29. The number of commutative binary operations that can be defined on a set of 2 elements is
(b) 6 (c) 4 (d) 2

w
(a) 8
ANSWERS

1. (c) 2. (c) 3. (d) 4. (b)

Flo
5. (b) 6. (d) 7. (a) 8. (c)

ee
9. (d) 10. (a) 11. (d) 12. (b) 13. (c) 14. (d) 15. (a) 16. (d)

Fr
17. (c) 18. (d) 19. (c) 20. (c) 21. (a) 22. (d) 23. (a) 24. (b)
25. (c) 26. (b) 27. (a) 28. (c) 29. (d)
for
ur
SUMMARY
ks

1. A binary operation on a set S is a function from S x S to S.


Yo
oo

A binary operation * on a set S associates any two elements a,b eS fo a unique element
B

ct *b eS.
re

2. A binary operation * on a set S is said to be


ou

(i) commutative, if a * b = b* a for ail a,b eS.


ad

(ii) associative, if(a*b)*c = a * (b * c) for all a,b, c eS


Y

(iii) distributive over a biliary operation o on S, if a * (b o c) = (a * b) o (a * c)


d
Re

and, (b o c) * a = (b * a) o(c * a) for all a,b eS.


n
Fi

3. Let * be a binary operation on a set S. An element c e S is said to be identity element for the
binary operation *, if a * e = a = e * a for all a eS.
4. Let * be a binary operation on a set S and e e She the identity element. An element a s Sis
said to be invertible, if there exists on element b e S such that a*b=e=b* a.
5. A binary operation on a finite set can be completely described by means of composition
table.
From the composition table, we can infer the following properties of the binary operation:
(i) The binary operation is commutative if the composition table is symmetric about the
leading diagonal.
(ii) If the row headed by an element say e coincides with row at the top and the column
headed by e coincides with the column on the extreme left, then e is the identity
element.
(iii) If each row, except the top-most row, or each column, except the left-most column,
contains the identity element. Then, every element of the set is invertible with respect
to the given binary operation.

ReadYourFlow.COM
3.40 MATHEMATICS-XII

2
6. Total number of binary operations on a set consisting of n elements is ri1 .
Total number of commutative binary operations on a set consisting of n elements is
n(H-l)
n 2

w
Flo
ee
Fr
for
ur
ks
Yo
oo
B
re
ou
ad
Y
nd
Re
Fi

ReadYourFlow.COM
INVERSE
TRIGONOMETRIC FUNCTIONS
4.1 INTRODUCTION
In chapter 3, we have learnt about functions, types of functions, composition of functions and
inverse of a function. In this chapter, we shall use these concepts to define the inverses of all
trigonometric functions and to study their properties. Let us first recall the definition of inverse

w
of a function.

4.2 INVERSE OF A FUNCTION

Flo
In the previous chapter, we have learnt that corresponding to every bijection (one-one onto
function) f: A B there exists a bijection g:B A defined by

ee
g (i/) - x if and only / (x) = \j.

Fr
The function g:B —> Ais called the inverse of function / : A -> B and is denoted by / ~ ^
or
ur
Thus, we have
sf
/(*) = yo/ -1(y) = x
Also,
k
Yo
oo

crV>w =/_1 \ / w = / 1(y) = x, for all x e A.


B
re

and. O®/-1) (y) =/(/"V = fix) = y, for all y e B.


ou
ad

4.3 INVERSES OF TRIGONOMETRIC FUNCTIONS


Y

We know that trigonometric functions are periodic functions, and hence, in general, all
nd

trigonometric functions are not bijections. Consequently, their inverses do not exist. However, if
Re

we restrict their domains and co-domains, they can be made bijections and we can obtain their
Fi

inverses. In the following sections, we shall do all these things to obtain the inverses of
trigonometric functions.
4.3.1 INVERSE OF SINE FUNCTION
Consider the function /: R -> R given by / (x) = sin x. The graph of this function is shown in
Fig. 4.1. Clearly, it is a many-one into function as it attains same value at infinitely many points
and its range [- 1,1] is not same as its co-domain. We know that any function can be made an
onto function, if we replace its co-domain by its range. Therefore, / : R —» [-1,1] is a many-one
onto function. In order to make/ a one-one function, we will have to restrict its domain in such a
way that in that domain there is no turn in the graph of the function and the function takes every
value between -1 and 1. It is evident from the graph off (x) = sin x that if we take the domain as
[- n/2, k/2] , then/ (x) becomes one-one. Thus,/ : [- n/2, n/2] -> [-1,1] given by/ (0) = sin 0
is a bijection and hence invertible.
The inverse of the sine function is denoted by sin - 1. Thus, sin -1 is a function with domain
[-1,1] and range [-n/2, tt/2] such that
sin- 1 x = 0 <=> sin 0 = x.

ReadYourFlow.COM
4.2 MATHEMATICS-XII

(0,1)

X' x
(-271, 0) (-71, 0) O (n,0) (271, 0)

(0,-D

w
r

Flo
Fig. 4.1 Graph of i/= sin x,-27t <x<2n:

ee
Also, sin -1 (sin 0) = 9 for all 0 s [- * / 2, ti / 2] [v /- lof(x) =/ - 1 (f(x)) = x for all x eD(f)]]

Fr
and, sin (sin- 1 x) = x for all x e [-1,1] [••• fof 1 (y) =/(/ 1 (y)) = y for all y e D(f
for
ur
The graph of the function /: [- n/2, n/2]-> [-1,1] given by /(x) = sin x is shown in
Fig. 4.2 and the graph of sin- 1: [-1,1] -> [- tc/2, ti:/2] is shown in Fig. 4.3.
ks

Y
Yo
oo
B

(0,1)
(f-1)
re
ou
ad
Y

X' X

0 (fo)
d
Re
n
Fi

(-M (0,-1)

y'

Fig.4.2 Graphof i/ = sinx,--^<x<-^

REMARK 1 In the above discussion, we have restricted the domain of sine function to the interval
[-71/2, tt/2] to make it a bijection. In fact, if we restrict its domain to any one of the intervals
[-7r/2,71/2], [7r/2, 3 7r/2], [3 tt:/2, 5 n/2], [-3 7r/2, - 71/2], [-5 n/2, - 3 n/2] or, in general
[nn -n/2,nn + n/2], n e Z, then also it becomes a bijection. We can, therefore, define the inverse of the
sine function in each of these intervals. Thus, sin- 1 x is a function with domain [-1,1] and range
[-n/2, n/2] or [- 3 n/2, -n/2]or [n/2, 3 n/2]andsoon. Corresponding to each such interval, zve get
a branch of the function sin- 1 x. The branch of the function sin-1: [-1,1] -» [- n/2, n/2] called the
principal value branch as shoivn in Fig. 4.3 and the value sin -1 xfor given value ofx e [-1,1] is called the
principal value.

ReadYourFlow.COM
INVERSE TRIGONOMETRIC FUNCTIONS 4.3

K) K)
X' (-1<0) X
o (1,0)

M)
K-f)

w
Y'
Fig. 4.3 Graph of i/=sin 1 x,-l<x<l

Flo
REMARK 2 By considering sin- 1 x «s a function with domain [-1,1] rmd range [ - tc/2, ti/2] or

ee
[k/2, 3 7r/2] or [371/2,5 ji/2] and so on, we get different branches. Ifall these branches are put together
and drawn on the same scale, we obtain the graph as shown in Fig 4.4. Clearly, this graph can be obtained

Fr
from the graph of sine function by interchanging the coordinate axes. The branch of
sin-1: [-1,1] -> [-ti:/2, tc/2] is the principal value branch and the value o/sin-1 a: lying in [-k/2, k/2]
for
ur
for a given value ofx e [-1,1] is called the principal value.
ks

Y
Yo

(o.f)
oo

M-^t) ’
eB

(0,71)
r
ou
ad
Y

(0'!)
M'-f)
nd
Re
Fi

X' X
(-1,0) o (1,0)

(0,-K)

^l(l,-f)
Y'
Fig. 4.4 Different branches of y=sin 1 a: on the same scale

ReadYourFlow.COM
4.4 MATHEMATICS-XII

REMARK 3 In chapter 3, we have learnt that the graphs of a function and its inverse (if it exists) are
mirror images of each other in the line mirror y = x. In the above discussion, we have learnt that
sin-1: [-1,1] —»[-[n/2, n/2] is the inverse offunction sin : [-n/2, k/2] -> [-1,1]. Their graphs that is
the curves y = sin x and y = sin-1 x are mirror images of each other in the line mirror y = x as shown in
Fig. 4.5.

(°'f)
(0,1)

X' (-■H

w
o (1,0)

Flo (0,-1)

ee
M)

Fr
r
for
ur
Fig. 4.5 Graphs of y = sinx and y = sin 1 x as mirror images of each other in line mirror y = x
ks

NOTE l sin-1 x is not equal to (sin x)~^, or 1


Yo
oo

sin x
B

SOME OBSERVATIONS From figures 4.2 and 4.3, we make the following observations :
re

(i) sin and sin-1 are increasing functions on [-n/2, tc/2] and [-1,1] respectively.
ou

0-j < 02 => sin O} < sin 02 for all 0J, 02 e [-n/2, k/2]
ad
Y

and, Xj < x2 => sin-1 Xj < sin-1 x2 for all xl7 x2 € [-1,1]
nd

(ii) The minimum and the maximum values of sin-1 x are - ^ and ^ respectively.
Re
Fi

(iii) sin-1 x attains the minimum value - — at x = -1 and the maximum value — at x = 1.
2 2
ILLUSTRATIVE EXAMPLES
LEVEL-1
EXAMPLE l Find the principal values of

1
(i) sin -1 (ii) sin -l [NCERT, CBSE 2011]
2 2
SOLUTION For x e [-1,1], sin 1 x is an angle 0 in the interval [-7t/2, re/2] whose sine is x
i.e. sin 0 = x. Therefore,
'■fS a in n K
(i) sin-1 — = An angle 0 e —, — such that sin 0 =
2 & 2 2 2 3

ReadYourFlow.COM
INVERSE TRIGONOMETRIC FUNCTIONS 4.5

TC n
(ii) sin-1
(4HAnan§leee 2 — such that sin 9 =
2J 6
EXAMPLE 2 Find the principal values of
(i) sin-1(j) 1
(ii) sin 1
V2
SOLUTION We know that sin 1 x denotes an angle in the interval [- n / 2 , k / 2] whose sine is
x for x e [-1,1]. Therefore,

(i) =|^An angle 9 e - such that sin 9 = — 1 = —


2 6

n
(ii) sin -| An angle 9 e such that sin 9 =
2 J2) 4

w
EXAMPLE 3 Find the value of sin

Flo
ee
SOLUTION sin

Fr
= sin 1 cos or
ur
3
sf
71
= sin -1 cos — = sin
3
k
Yo
oo

EXAMPLE 4 Find the domain of the function f(x) = sin 1 (2x - 3).
B

SOLUTION The domain of sin - 1 x is [-1,1]. Therefore, /(x) = sin -1 (2x - 3) is defined for all x
re

satisfying
ou
ad

-I<2x-3<1=> 3-1<2x<3 + 1 => 1 <x < 2 => xe[l,2]


Y

Hence, domain of/(x) = sin-1 (2x - 3) is [1, 2].


—1 9
nd

EXAMPLES Find the domain off(x) = sin (-x )• [NCERT EXEMPLAR]


Re
Fi

SOLUTION The domain of sin-1x is [-1,1]. Therefore, /(x) = sin-1 (—x2) is defined for all x
satisfying
-1 <-x2 <1
1 >x2 >-l => 0 <x2 <1 => x2<l => x2-l <9=>(x-l)(x + l)<9=>-1 <x<l

Hence, the domain of /(x) = sin (-x ) is [-1,1].


EXAMPLE 6 Find the domain o//(x) = sin -1 x + cos x. [NCERT EXEMPLAR]

SOLUTION The domain of sin-1x is [-1,1] and that of cosx is R. Therefore, domain of
/(x) = sin-1 x + cosx is [-1, l]nR =[-l, 1].

EXAMPLE 7 Find the domain of thefunction f(x) = sin -1 Vx-1. [NCERT EXEMPLAR]

SOLUTION The domain of sin-1x is [-1,1]. So, the domain of /(x) = sin-1 Vx-1 is the set of
values of x satisfying
-l<fx-l <1

ReadYourFlow.COM
4.6 MATHEMATICS-XII

=> o<7*-i <1 [*.■ V^T > o]


=> 0<x-l<l=> 1 <x <2 => xe[1,2]
Hence, the domain of/(x) = sin-1 Vx-1 is[1, 2].

EXAMPLES 7/ x, y,z e[-l, 1] such that sin“1x + sin_1y + sin_1z = --7t


—, find the value of
x^2 +yz2 +z^.
2

—1 71
SOLUTION We know that the minimum value of sin x for x e [-1,1] is —.
2
. _1 71 . -i
sm x > — , sin y>—- and sin-1 z >-^ for all x, y,z e[-1,1]
2 2
=> sm —1 z + sin Vsin

=> sin 1x + sin 11/4-sin -1"z >----3 71-

w
. -1 . . _i 3ti
sm x + sm y + sm z = - —

=>
. -1
sm
71 . _i
x = —, sm y = - -, sm z =----
7t . —1

Flo ti:

ee
2 * L2 2

Fr
=> x = y =z = -l
Hence, x2+y2+z2 = (-1)2 + (-1)2 + (-1)2 = 3. or
ur
EXAMPLE 9 Let x, y, 2 e [-1,1] be such that sin -1 x + sin 1 y + sin lz = Find the values of
sf

9
k

(i) *2018+y2019 +.-2020


z (ii) *2016 _j_ y2018 +. z_2020
Yo

x2016 +y 2018 +, z 2020


oo
B

—1 71
SOLUTION For any x e [-1,1], the maximum value of sin x is - and it attains this value at
re

2
x = 1.
ou
ad

7T . —1 7T . —1 . 7T , ..r-f-iT
sin !x < -,sm y<—,sm z <-for all x, y, z e [-1,1]
2 2 2
Y

sin-1 x + sin-1 y + sin-1z < -^ + ^ + ^ for all x, y, z e [-1,1]


nd
Re

=> sin-1 x + sin-1 y + sin_1z < for all x, y, z e [-1,1]


Fi

. -1 . _i . _i 3ti
sm x + sm y + sm 2 =~
71 . -1 71 . _1 71
sm = —,sm y= —,sin z= —
2 y 2 2
=> x =1, y = l,z =1
x20180‘
+ z 2020 = (1) 2018 + (1)2019+(1) 2020 = 3
2019
(i)
9 9
(ii)
X2016
+y
2018
+ z 2020 =1+1+1- =3-3=0
x2016 +y 2018 , ,2020 1+1+1
+z

EXERCISE 4.1
LEVEL-1
1. Find the principal value of each of the following:
V3 2 71 V3-1
(i) sin -1 (ii) sin -1 cos — (hi) sin -1
2 3 2V2

ReadYourFlow.COM
INVERSE TRIGONOMETRIC FUNCTIONS 4.7

3n 5ti
(iv) sin -1 (v) sin -1 cos— (vi) sin -1 tan —
2V2 J 4 4
2. (i) sin"1--2 sin-1 i (ii) sin -1 • -1
cos sm [NCERT EXEMPLAR]
2 V2
3. Find the domain of each of the following functions:
(i) /(x)=sin"1x2 (ii) f(x) = sin-1 x + sin x
(hi) /(x)=sin"1Vx2-l (iv) /(x) = sin"1 x + sin"12x
4. If sin“1x + sin"1i/ + sin~1z + sin“11 = 2n, then find the value of x2 +y2 +z2 +t2.
5. If (sin"1 x)2 + (sin-1 y)2 +(sin"1z)2 the value of x2 +y2 +z2.

ANSWERS
5n / \ n

w
1. (i) -f (ii) “7 (iii) ^ (iv) (v)--
12 12
(vi)f 2- (i) 'I (ii) ^

3. (i) [-1,1] (ii) [-1,1]


Flo
o
(hi) [-vX-ll^tl, V2] (iv) [-172,1/2]

ee
4. 4 5. 3

Fr
4.3.2 INVERSE OF COSINE FUNCTION
or
The grahp of cosine function is shown in Fig. 4.6. It is evident from the graph of y = cosx (see
ur
Fig. 4.6) that the function f:R -> R given by/(0) = cos 0 is a many-one into function. However,
sf

/: [0,71] -> [-1,1] is one-one onto i.e. a bijection and hence it is invertible. The inverse of cosine
k
Yo

function is denoted by cos"1.


oo
B

Y
re
ou
ad
Y

(0,1)
nd
Re
Fi

X' (-71, 0) (71,0) X


(-2ji, 0) o

(-7t, -1)

Fig. 4.6 Graph of 1/ = cosx

Thus, if cos: [0, tt] —> [—1,1] is such that cos0=x. Then, cos
cos"1 x = 0.
In other words.
cos0 = x <=> cos 1 x = 0 for all 0 e [0,71] and x e [-1,1].

ReadYourFlow.COM
4.8 MATHEMATICS-XII

The graphs of cos: [0, n] -» [-1,1] and its inverse cos-1!-!, 1] -> [0, k] are shown in Figures 4.7
and 4.8 respectively. The branch of cos-1: [-1,1] -> [0, tt] is called the principal value branch and
the value of cos-1 x lying in [0, tt] for a given value of x e [-1,1] is called the principal value.

(TO)

X' X

w
o
(f'°) (*,0)

Flo (0,-1)

ee
Fr
r
Fig. 4.7 Graph of y = cosx, 0 <.*<n
for
ur
Y
ks
Yo
oo

(-bn)
B
re
ou

M)
ad
Y
d
Re
n
Fi

X' X
(-1,0) O (1,0)

r
Fig. 4.8 Graph of y = cos 1 x

SOME OBSERVATIONS It is evident from the graphs of cosx and cos-1 x that
(i) the domain and range of cos-1 x are [-1,1] and [0, n] respectively.
(ii) both cos and cos-1 are decreasing functions in their respective domains.
0! < 02 => cos©! > cos02 for all 0j, 02 e [0, 7t]
and, Xj < x2 => cos-1 x^ > cos-1 x2 for all Xj, x2 e [-1,1]
(iii) The minimum and maximum values of cos-1 x are 0 and n respectively which are attained
at 1 and -1 respectively i.e. cos-1(l) = 0 and cos-1 (-1) = n.

ReadYourFlow.COM
INVERSE TRIGONOMETRIC FUNCTIONS 4.9

ILLUSTRATIVE EXAMPLES

LEVEL-1

EXAMPLE l Find the domain of cos 1 (2x -1).

SOLUTION The domain of cos-1 x is [-1,1]. So, the domain of cos-1 (2x-l)isthesetof all values
of x satisfying
-1 < 2x -1 <1 => 0<2x<2=> 0<x<l
Hence, the domain of cos-1 (2x -1) is [0,1].
EXAMPLE 2 Find the principal values of
V3
(i) cos -1 (ii) cos [NCERTJ
2

low
SOLUTION For any xe[-l,l], cos 1 x represents an angle in [0, rr] whose cosine is x.
Therefore,
\ / ^ n
-1 fV3
cos — = An angle 0 e [0, rt] such that cos 0 = —

ee
(i)
6
rF
Fr
(ii) cos 1 ^An angle 0 e [0, tc] such that cos ® = - for=

-ll
EXAMPLE 3 Find the principal value o/cos -1 sm cos
u
ks

2
Yo
oo

_1 1 _ 71
SOLUTION We know that cos
2_ 3
B
re

cos -1 sin cos -ll


2
ou
ad

V3l 7T
. 71
= cos -1 sm — = cos -1 V cos -i
Y

3 2 6 2 6
nd
Re

EXAMPLE 4 If x,y,z e [-1,1] such that cos-1 x + cos-1 y + cos_1z = 0,find x + y + z.


Fi

SOLUTION We have,
x, y,z e[-l, 1]
=> -1 <x<l,-l <y <1, -1, <z <1
=> 0 < cos-1 x < tc, 0 < cos-1 y < tc, 0 < cos-1 z < tc
—I —1 —1
cos x + cos y + cos z = 0
—1 —1 —1
=> cos x = 0, cos y = 0 and cos z = 0
=> x = y=z=\.
Hence, x + y + z = 3.
EXAMPLE 5 //x, y, z e [-1,1] such that cos-1 x + cos-1 y + cos_1z = 3n, then find the values of
(i) xy + yz + zx (ii) x (y + z) + y (z + x) + z (x + y) [NCERT EXEMPLAR]
SOLUTION We have,
x, y,ze[-l,l]

ReadYourFlow.COM
4.10 MATHEMATICS-XII

-1 <*<1,-1 < y <1, -1, <z <1


0 < cos-1 ^ < re, 0 < cos-1 y < tt, 0 < cos-1 z<n
cos-1 x + cos-1 y + cos_1z = 3n

=> cos -1 x + cos -1 y + cos -1 z = n + K+n

=> cos ~1x = n, COS


-1 y = K, COS -1 2 = Tt

x = -1, y = -1,2 = -1.


Therefore,
(i) xy + yz+zx= (-1) x (-1) + (-1) x (-1) + (-1) x (-1) =1+1 + 1 = 3.
(ii) x (y + z) + y (z + x) +z (x + y) = 2 (xy + yz + zx) = 2 x 3 = 6

w
EXERCISE 4.2
LEVEL-1

1. Find the domain of definition of/(x) = cos-1(x2 -4).

Flo
ee
2. Find the domain of /(x) = 2 cos-12x + sin-1x.

Fr
3. Find the domain of /(x) = cos'"1 x + cosx. or
ur
4. Find the principal value of each of the following:
sf
V3 1
(i) cos -l [NCERT] (ii) cos -1
k

[NCERT]
Yo

2 V2
oo

. 4rt
B

37t
(iii) cos 1 sm — (iv) cos -1 tan —
3
re

4
5. For the principal values, evaluate each of the following :
ou
ad

(i) cos-1 —+ 2 sin -ll


Y

[NCERT, CBSE 2012]


2 2
nd
Re

1
(ii) cos -ifl -2 sin -1| [CBSE 2012]
Fi

2 2

1
(iii) sin -1 — +2 cos -1
VI
2 2

(iv) sin -1
VI + cos -1
VI
2 2

ANSWERS
i. [-V5,-VI]u[VI, V5] 2. [-1/2,1/2] 3. [-1,1]
/•\ Stt
4. (i) —
o <u>f (iii) ^ (iv) Tt

2n .... 27t
5. (i) y
11 T (iii) y (iv)
o

ReadYourFlow.COM
INVERSE TRIGONOMETRIC FUNCTIONS 4.11

4.3.3 INVERSE OF TANGENT FUNCTION

Consider the function /: R - j(2n +1) : n e Z j -» R given by f(x) = tan x. The graph of this

function is shown in Fig. 4.9. It is evident from the graph that f(x) = tan x is a many-one onto
function and hence it is not invertible. However, the function tan j -h> R associating

k n j to tan x e R is bijection and so it is invertible. The inverse of this function is


each x e
2' 2
denoted by tan 1.

w
Y

Flo
ee
Fr
for
ur
ks
Yo

tM (t'°)
oo

X' X
o (11,0)
(f,o)
B
re
ou
ad
Y
nd
Re
Fi

r
Fig. 4.9 Graph of i/ = tanx

Clearly, tan 1:R ^ jis suc^

tan 1 x = 0 <=> tan 0 = x


Also,

tan 1 (tan 0) = 0 for all 0 e f 71 and, tan (tan 1 x) = x for all x e R


2' 2

ReadYourFlow.COM
4.14 MATHEMATICS-XII

EXAMPLE 4 Which is greater, tan 1 or tan 11 ? [NCERT EXEMPLAR]


SOLUTION We know that tan -11 = — and 1 > —
4 4
Now,
1>^
4
=> tan 1 > tan — [v tan 0 is an increasing function]
4
=> tan 1 > 1
=> tan 1 > 1 > - vl>*
4 4
=> tan 11 > -71
4

low
=> tan 1 > tan 11 v tan -It1 = —
4
EXAMPLE 5 Find the minimum value ofnfor which tan 1 - > -,« e N. [NCERT EXEMPLAR]
n 4

ee
SOLUTION We have,
rF
Fr
-in n
tan — >-
n 4
for
tan 1 - > tan 11 71
- = i.tan -li1
ou
TC 4
ks

tan tan -1 n > tan tan 11 [v tan 0 is an increasing function]


oo

ft
Y
eB

”>i [v tan (tan 1 x) = ^]


ft
r

n > 7i = 3.14
ou
ad

n = 4,5, 6,...
Y

Hence, the minimum value of n is 4.


d
Re

EXERCISE 4.3
n
Fi

LEVEL-1

1. Find the principal value of each of the following :


1 1
(i) tan -1 (ii) tan -1
V3 V3
(iii) tan -1 cos— 2ft
ft
(iv) tan -1 2 cos—
2 3
2. For the principal values, evaluate each of the following:
(i) tan-1 (-1) +cos -1 1
(ii) tan -1 2 sin 4 cos -1
V2
3. Evaluate each of the following :
(i) tan-1l + cos -i [NCERT]

(ii) tan -1 (■^)+tan‘1<"v5)+tan’1[sin(_f))

ReadYourFlow.COM
INVERSE TRIGONOMETRIC FUNCTIONS 4.15

Stt
(iii) tan -1 tan — + cos -1 cos [NCERT EXEMPLAR]
6
__________________________________________ ANSWERS

1. (i) ^ (ii) (iii) 0 (iv) -5 2. (i) ~ (ii) f 3. (i) ^ (U) (iii) 0


o o 4 2 3 4 4
4.3.4 INVERSE OF SECANT FUNCTION
In Class XI, we have learnt that sec0 is not defined at odd multiples of n/2. Therefore, a rule
associatingx eR - j(2n + l) ^ :n e Z j to secxis a function whose graph is shown in Fig. 4.12.

w
Flo
ee
Fr
(0,1) for
ur
X' tM M r
x
F¥l o
ks

(-7C,0) (71,0)
(3f-o)
Yo
oo

(-It, -1)
(0,-1)
B
re
ou
ad
Y
d
Re
n
Fi

y
Fig. 4.12 Graph of y = secx

We observe that the function sec: R -1(2n +1) : n e Z| -> R is neither one-one nor onto but.

sec: R - |(2« + 1) e z| —> (-oo. -1] w[l, c») is many-one onto. If we restrict the domain to

71 K
0. 7t then the function associating each x e 0, — — ,n to secure (-oo , -1) u[l, CO)
2 2 J
is a bijection as is evident from the graph of y = secx shown in Fig. 4.13. the inverse of
sec:
Nr K
-,n

sec -1 0 = x
/
-^(-oo. -1) u[l, oo) is denoted by sec-1 such that

x = sec0
Also, sec -1 (sec0)=0 for all 0 e [0,7r/2) u(7t/2, k]

ReadYourFlow.COM
r

4.16 MATHEMATICS-XII

(0,1)

X'
o
(f-°) X
(n,0)

w
(0,-1)

Flo
ee
Fr
for
ur
Y'
ks

Fig. 4.13 Graph of y = secx, 0<,x<,n,x* n/2


Yo
oo

and, sec (sec-1 x) =x for all a: e (-oo, -1] u [1, <x>)


B

The graphs of functions sec: [0,7t/2) u(7t/2, 7t]-»(-oo,-1] u[l, co) and
sec-1: (-oo, -1] u[l, oo) -> [0, n/2) u(n/2, n] are shown in Figures 4.13 and 4.14 respectively.
re

The branch of sec-1 shown in Fig. 4.14 is called the principal value branch and the value of
ou
ad

sec-1xin[0, n/2) u(7t/2, tx] for given value of x e(-oo, -l]u[l, co) is called the principal value.
Y

Y
d
Re
n
Fi

(-1,71) (0,71)

H)
X' X
(-1,0) o (1,0)

r
Fig. 4.14 Graph of y = sec lx

ReadYourFlow.COM
INVERSE TRIGONOMETRIC FUNCTIONS 4.17

SOME USEFUL OBSERVATION We make the following observations from the graphs of secx and
-1
sec x:
(i) sec* is an increasing function on the intervals [0, tc/2) and (rc/2, n] but, it is neither
increasing nor decreasing on [0, k/2) u(t:/2, tt].
(ii) sec-1* is an increasing function the intervals (-oo,-!] and [!,<») but, it is neither
increasing nor decreasing on (- co, -1] u [1, co).
(hi) The maximum value of sec-1 x is ti which it attains at x = -1.
(iv) The minimum value of sec-1 x is 0 which it attains at x = 1.
ILLUSTRATIVE EXAMPLES

LEVEL-1

EXAMPLE 1 Find the principal values of each of thefollowing:


-2
(i) sec-1 (2) (ii) sec -1

low
J3)'
SOLUTION For any x e (-co, -1] u [1, co), sec 1 x is an angle 0 e [0, k/2) u (k/2, tt] whose
secant is x i.e. sec 0 = x. Therefore,
sec 1(2) = An angle 0 e 0, - ju^,l such that sec 0 = 2j = -^

ee
(i)
L 2
rF
Fr
-1 ^--j=j=^An angle 0 e|^0, k such that sec 0 = - -5 5k
(ii) sec
6
or
sf
V3
ou
EXAMPLE 2 Find the set of values of sec -l
2
k

Vs
oo

SOLUTION We know that sec-1 x is defined for all x < -1 or x > 1 and — <1.
Y
B
re

sec -1 — is not meaningful.


ou
ad

Hence, the set of values of sec -1 is the null set <j).


Y
d
Re

EXAMPLE 3 Find the principal values of sec~ 1 —j= and sec- 1 (- 2).
n

V3
Fi

SOLUTION Since sec- 1: R - (-1,1) [0, u] -j-^J is a bijection. Therefore, sec -1 x represents

an angle in [0, j wh°se secant is x. Thus,

j = An angle 0 e [0, tc] - such that sec 0 = -^= Tt


(i) sec -1
y[3 6
2k
(ii) sec 1 (- 2) An angle 0 e [0, ~ suci1 sec ^
3
EXAMPLE 4 Find the domain of sec 1 (2x +1).
SOLUTION The domain of sec-1 x is (-oo, -1] u [1, co). Therefore, sec-1 (2x +1) is meaningful, if
2x +1 > 1 or, 2x + 1 < -1
=> 2x > 0 or, 2x < - 2
=> x > 0 or, x <-1
x g(-oo, -l]u[0, co)
Hence, the domain of sec-1(2x +1) is (-oo, -1] u [0, co).

ReadYourFlow.COM
4.18 MATHEMATICS-XII

EXERCISE 4.4

LEVEL-1

1* Find the principal values of each of the following:


(i) sec_1(-V2) (ii) sec’1 (2)
3 71 3k
(iii) sec -1 2 sin — (iv) sec -1 2 tan —
4 4
2. For the principal values, evaluate the following:
(i) tan"1 V3-sec-1 (-2) [CBSE 2012] (ii) sin -1 2 sec -1 2 tan —
6
3. Find the domain of
(i) sec-1 (3x-l) (ii) sec 1 x - tan 1x

w
ANSWERS
1. (i) ^(ii) | (iii) ^ 2k
2. (i) (U)
4 (iv)T
Flo
ee
3. (i) (—Qo, 0] {j[2/3, oo) (ii) (-co,-l]u[l,co)

Fr
4.3.5 INVERSE OF COSECANT FUNCTION
In Class XI, we have leamt that the function f(x) = cosec xhas domain R-{nn:neZ) and range
for
ur
R -(-1/ !)• The graph of this function is shown in Fig. 4.15. It is evident from the graph that
f: R-{nn-.n e Z] R defined as f(x)= cosec x is a many-one into function and
ks

f: R -{nn-.n <= Z) -» K -(-1,1) is many-one onto.


Yo
oo

y
B
re
ou
ad
Y
nd
Re

m
Fi

n (f'1)
X' ~2' °) (f'°) X

0 (-H

r
Fig. 4.15 Graph of y = cosec .r

ReadYourFlow.COM
INVERSE TRIGONOMETRIC FUNCTIONS 4.19

If we consider /: [-tc/2, 0) u(0, tu/2] -»(- co, -1] u[l, co), then its is a bijection and hence
invertible. Tire inverse of cosec is denoted by cosec-1 and is defined as
cosec-1* = 0 o cosec 0 = * for all 0 eI-Ti/2, 0) u(0/ 71/2] and x e(-oo, -1] u[l, 00)
Also, cosec 1 (cosec 0) = 0 for all 0 e - 0 j

and, cosec (cosec 1 x) = x for all x e (- co, -1] u[l, co)


The graphs of
cosec: [- 7t/2, 0) u(0, tt/2] (-co, -1] u[l, co)
Y

w
i (0, i)
Flo (i-1)

ee
Fr
x' (-f°) X
o
(!'°) or
ur
sf
(-f-1) (0,-1)
k
Yo
oo
B
re
ou
ad
Y

Y’
nd
Re

Fig. 4.16 Graph of 1/ = cosec x,-~<x<^-,x^0


Fi

M)
X' X
(-1,0) o (1,0)

h-f)
r
Fig. 4.17 Graph of y= cosec 1 x

ReadYourFlow.COM
4.20 MATHEMATICS-XII

and cosec 1:(-co-l]u[l,co) -> [-ti/2, 0)u(0/ tc/2] are shown in Fig. 4.16 and 4.17 respectively.

The branch of cosec_1x shown in Fig. 4.17 is called the principal value branch and the value of
cosec-1 x lying in [-Tt/2, 0) u(0, tt/2] are the principal values.

SOME OBSERVATIONS It is evident from the graphs of cosec x and cosec 1 x that

(i) cosec 0 is a decreasing function on [-71/2, 0) and (0,71/2]. But, it is neither decreasing nor
increasing on [-tc/2, 0) u(0, tc/2].
(ii) cosec-1x is decreasing on (-00, -1] and [1, co). But, it is neither increasing nor decreasing
on ( co, -1] u [1, co).
(iii) The maximum value of cosec- 1 x is tc/2 which it attains at x = 1.

w
(iv) The minimum value is cosec 1 x is -tc/2 which it attains at x = -1.

Flo
ILLUSTRATIVE EXAMPLES
LEVEL-1

ee
Fr
EXAMPLE 1 Find the principal values of cosec 1(2) and cosec 1 -j=J.
for
ur
SOLUTION For x e (-00, -1] u [1, co), cosec -1 x is an angle 0 e [-tc/2, 0) u(0, tc/2] such that
ks

cosec 0 = x.
Yo
oo

cosec 1 (2) = An angle 0e —^,oju(^0,^ such that cosec 0 = 2 = —


B

6
re

and. cosec -1 -j= I =[ An angle 0 e —0


2'
0,^
'2 i such that cosec 0 = -
2 ^
VsJ
TC

3
ou
ad
Y

1
EXAMPLE 2 Find the set of values of cosec -1
2
d
Re
n

SOLUTION We know that cosec 1 x is defined for all x < -1 or x > 1.


Fi

So, cosec -1 j is not meaningful. Hence, the set of values of cosec 1 j is the null set 4).

EXAMPLE 3 For the principal values, evaluate each of the following:


(i) tan-1 V3 - sec-1 (-2) + cosec-1 -~ (ii) 2 sec 1(2) -2 cosec 1 (- 2)

SOLUTION (i) tan -1V3 - sec-1 (-2) + cosec-1 -^=--— + - = 0


V3 3 3 3

(ii) 2 sec-12-2 cosec-1 (- 2) = 2 x - - 2 x — = TC


3 6
EXAMPLE 4 If cosec-1 x + cosec-1 1/ + cosec -1 z = - — , find the value of- + — +
2 y z X

SOLUTION We know that the minimum value of cosec 1 x is - vTiich is attained at x = -1.

ReadYourFlow.COM
INVERSE TRIGONOMETRIC FUNCTIONS 4.21

-1 -1 371
cosec -1 x + cosec y + cosec z=
2

n n
cos-1 x + cosec-1 + cosec-1 z = +
2 2

k -in
=> cosec -1 x = —n cosec -1 y=- cosec z=—
2' 2' 2

=> x = -1, y = —1, z = —1

- + y+i=H)+H)+(d) = 3
y z x (-1) (-1) (-1)

EXERCISE 4.5

w
LEVEL-1

1. Find the principal values of each of the following:


Flo
ee
(i) cosec-1 (- V2) [NCERT] (ii) cosec 1 (- 2)

Fr
2 271
(hi) cosec -1 (iv) cosec -1 2 cos —
for
ur
V3 3
ks

2. Find the set of values of cosec -i


Yo

2
oo
B

3. For the principal values, evaluate the following:


re

(i) sin -1
V3l + cosec -1 2
(ii) sec-1 (j2j + 2 cosec-1 (- V2)
ou
ad

2 V3
Y

1171
(hi) sin -1 cos l 2 cosec 1 (- 2) (iv) cosec -1 2 tan
d
Re

6
n
Fi

ANSWERS

i. (i) -7 (ii) (iii) f (iv) ~ 2. ()> 3. (i) (ii) -7 (Hi) f (iv) ~


4 6 3 2 3 4 6 3

4.3.6 INVERSE OF COTANGENT FUNCTION

We have learnt in earlier classes that the function f(x) = cotx has domain =R-{nn:n<=Z} and
range R. Therefore, f \ R -{nn:n <e Z] —> R is a many-one onto function as is evident from the
curve y = cot* shown in Fig. 4.18.

If we consider cot: (0,tc) -> R, then it is a bijection and hence invertible. The inverse of this
function is denoted by cot-1 and is defined as

cot -1 x = 0 <=> cot 0 = x for all 0 e (0, tt) and all x e R

ReadYourFlow.COM
4.22 MATHEMATICS-XII

X'
(~2n, 0)
m m (271,0)
X

(--H

w
Flo
ee
Fr
r
Fig. 4.18 Graph of y = cot x or
ur
Also,
sf
cot 1 (cot 0) = 9 for all 0 e (0, rc) and, cot (cot 1 x) = x for all x e .R.
k
Yo
oo

Graphs of y = cotx and y = cot-1 x are shown in Figures 4.19 and 4.20 respectively.
B
re

i
ou
ad
Y
nd
Re
Fi

X' X
o (!■“) (K 0)

Y' T

Fig. 4.19 Graph of y = cotx/0<x<7r

ReadYourFlow.COM
"

INVERSE TRIGONOMETRIC FUNCTIONS 4.23

(0,71)

M)
X' X
o

low
ee
rF
Fr
Y'
for
Fig. 4.20 Graph of y = cot 1x
u
ks

The branch of cot-1 :R -> (0, n) is called the principal value branch and the value of cot-1 x for
Yo
oo

given x is called the principal value.


B

SOME USEFUL OBSERVATION It is evident from the graphs of cotx and cot-1 x that
re

(i) cotx is a decreasing function on (0, tt).


ou
ad

i.e. 0! < 02 => cot 0-l > cot 02 for all 0l7 02 e (0, n)
Y

(ii) cot-1 x is a decreasing function on R.


nd

i.e. x1<x2=> cot-1 xa > cot-1 x2 for all xlr x2 e R.


Re
Fi

(iii) For all x e R, the values of cot-1 x lie between 0 and n.


(iv) cot-1 x does not attain its minimum value zero and maximum value k at points in R.
ILLUSTRATIVE EXAMPLES
LEVEL-1

EXAMPLE l Find the set of values of cot 1 (V) and cot 1 (-1)
SOLUTION For any x eR, cot-1 x is an angle 0 e (0, tt) such that cot 0 = x.
cot 1 (1) =|^ An angle 0 e(0,7t) such that cot 0=lj = ^

cot-1 (-1) =|^An angle 0 e(0, n) whose cot angent is equal to -1 j = -^


and.

Hence, required set /

ReadYourFlow.COM
4.24 MATHEMATICS-XII

EXAMPLE 2 Find the principal values of cot 1 and cot 1 (-1).

SOLUTION We know that for any x <=R, cot- 1 x denotes an angle in (0, tt) whose cotangent
is x.
cot" 1 ^ = ^ An angle in (0, n) whose cotangent is V3 j = ^

Similarly, cot- 1 (-1) = ^ An angle in (0, n) whose cotangent is (-1) j .

EXAMPLE 3 For the principal values, evaluate thefolloiving:


(i) cot-1 (-1) + cosec-1 (- -Jl) + sec-1 (2)
2
(ii) cot-1 (-VS) + tan-1(l) + sec -1

w
V3
SOLUTION (i) cot"1 (-1) + cosec-1 (-V2) + secs-1 (2) = — —

2 _5 n _5n
Flo
4 4 3 6

ee
k k
(ii) cot-1 (- V3) + tan-1 (1) + sec -1
7s) ~T+4 + 6 _T

Fr
or EXERCISE 4.6
ur
LEVEL-1
sf

1- Find the principal values of each of the following:


k
Yo
oo

(i) cot-1(-V3) (ii) cot-1 (V3)


B

1 3n
(hi) cot -1 (iv) cot -1 tan —
re

73 4
ou
ad

2. Find the domain of f(x) = cotx + cot 1 x.


Y

3. Evaluate each of the following:


nd

2
Re

(i) cot-14= - cosec -1 (- 2) + sec -1


73 73
Fi

(ii) cot -1 2 cos sin -1


73^
2

(hi) cosec -l

1 1 Tt
(iv) tan -1 + cot -1 + tan-1 sin [NCERT EXEMPLAR]
73 73 2

ANSWERS
1- (i) ^ (ii) ^ (hi) ^ 3ti
2. R-{nn:neZ}
(iy) ^
o o 3 4
3. (i) ^ (ii) * (hi) —
3 4 V 7 6 (iv) -4
12

ReadYourFlow.COM
INVERSE TRIGONOMETRIC FUNCTIONS 4.25

As a ready reference domains, ranges and principal value branches of all inverse trignometric
functions are tabulated below.
Function Domain Range Principal value branch

[-1,1] [- k/2, n/2) y = sin 1 x fromf-1, “Jto^l,


sm

-1 [-1,1] [0, 7C] y = cos- 1 x from (-1, k) to (1, 0)


COS

tan 1 R (-n/2, n/2) y = tan- 1 x from

y = cosec -1 x, from (- oo, 0) to


cosec -1 (-<», -1] U[l, oo) [- n/2, n/2] - (0)
to (co, 0)

low
y = sec- 1 x, from |^- oo, j to (-1, n),
sec -1 (-CO, -l]u[l, oo)
M-{f} and, from(l, 0) to ^co,

ee
rF
Fr
cot 1 R (0, Tt) y - cot- 1 x from (-co, n) to (oo, 0)
for
NOTE l If no branch of an inverse trigonometric function is mentioned, then it means the principal value
u
branch of that function.
ks
Yo
oo

4.4 PROPERTIES OF INVERSE TRIGONOMETRIC FUNCTIONS


B

In this section, we will learn about various properties of six inverse trigonometric functions
defined in the previous section. These properties are very useful in simplifying expressions and
re

solving equations involving inverse trigonometric functions.


ou
ad

4.4.1 PROPERTY-1
Y

In chpater 2, we have learnt that if /: A B is a bijection, then / -1 :B -> A exists such that
nd
Re

/-1o/(x)=x or, /-1(/(x))=x for all x e A. In the previous section, we have learnt that
Fi

sin : [-n/2, tc/2] [-1,1], cos : [0,7t] -» [-1,1], tan : (-n/2, n/2) -> R, cot (0, n) R
sec: [0, n/2) u(n/2, rc] -> (-<», -1] u[l, co) and cosec : [- n/2, 0) u(0, n/2] -4 (-co, -1] u[l, co)
are bijections. So, these functions and their inverses satisfy the following property.
PROPERTY-1 (i) sin-1 (sin 0) = 0 for all 0 e[-7t/2, n/2]
(ii) cos-1 (cos0) = 0 for all 0 e [0,7t]
(hi) tan-1 (tan 0) = 0 for all 0 e (-n/2, n/2)
(iv) cosec-1 (cosec 0) = 0 for all 0 e [-n/2, 0)u(0, n/2]
(v) sec-1 (sec0) = 0 for all 0 e [0, tt/2) yj(n/2, ji]
(vi) cot-1 (cot 0) = 0 for all 0 e(0, n).
In the above property we observe that the relations between trigonometric functions and their
inverses hold true for specific values of 0. If 0 does not lie in the domain of a trigonometric
function in which it is not a bijection, then the above relations do not hold good. For example,
• -1 (sin0)=0 holds true for 0 e [-tc/2, k/2]. If 0 g [-k/2, k/2], what is the value of
sm

ReadYourFlow.COM
4.26 MATHEMATICS-XII

sin 1 (sin 0)? To answer this, we partition real line into sub-intervals so that the sine function
with domain any sub interval and co-domain [-1,1] is a bijection. Clearly, such sub-intervals are

5rc 3n~ 3tc k n n n 3n 3n 5n


T/_T ' ~T~ /
2

2J
/
2'il'l 2/_2 I 2 2 J

T, „ n 3n . k ^ ^ 3n .
If 6 e —,— i.e. — < 0—, then
L2 2 J 2 2

3n n 3n ^ A^ n
------ < - 0 < — => n-------< 71 - 0 < re — => - — < 7T - 0 < —
2 2 2 2 2 2

sin 1 (sin 0) = sin -1 sin (ti-0) =7i-0

w
jff, 37i 5k . 3k 5k ,
If0e —,— i.e. — < 0 < — ,then
2 2 2 2 '

Flo
ee
<-0<- —=>2ti-—<2h-0 <2k-—=> -- <2k-Q <-=> -- <Q - 2k<-
2 2 2 2 2 2 2 2

Fr
sin 1 (sin 0) = sin -1 -sin (27i-0)j = sin 1 ^sin (0-27t)j = 0-271
or
ur
sf

IfOel"- —,-— i.e. -—< 0 <--, then


k
Yo

l 2 2j 2 2
oo
B

37T .71
-------+ 7t<7l+0<7l----- - -<7l+0<-=>--<-7I-0<-
re

2 2 2 2 2 2
ou
ad

sin 1 (sin 0) = sin -1 - sin (71+ 0) I = sin 1 sin (-7i - 0) = -7i - 0


Y
nd

5 7r 3ti . Stt _
3ti:
Re

If 0 e - — ,------ i.e.--------- < 0 <-------


2 2 2 2
Fi

- —+ 27i<27r+0<-—+ 2t: ^ --<2ti+0<-


2 2 2 2

sm —1 (sin 0) = sin -1 sin (27t + 0) = 27t+ 0

Thus, we have
27i+0,if-57r/2<0<-37i/2
-7i-0 , if - 3k/2 < 0 < - k/2
sin 1 (sin 0) = 0 , if -tc/2 < 0 < tt/2
ti-0 , if ti/2 < 0 < 3ti/2
0 — 2k , if 3k/2 < 0 <5k/2
3k-Q, if 57t/2 < 0 <77t/2 and so on.

The graph of y = sin 1 (sin x) is as shown below.

ReadYourFlow.COM
INVERSE TRIGONOMETRIC FUNCTIONS 4.27

It is evident from the graph of y = sin 1 (sin .r) that the function sin 1 (sin x) is a periodic

function with period 2ti.

•Y

./=!
5V*
X' (-2n, 0) X
(2n, 0)

w
y-l

Flo
ee
r

Fr
Fig. 4.21 Graph of/(>:) = sin 1 (sina-)
for
ur
Similarly, we find that
ks

-271-0 , if - 371 < 0 < - 2tc


Yo
oo

2tc+0 , if-27t < 0 < - tc


eB

-0 , if - rc < 0 < 0
cos 1(cos 0) =
0 , if 0 < 0 < 7t
r

27T-0 /if7t<0<27l
ou
ad

0-2tc , if 2tc < 0 < Sti and so on.


Y
nd

Y
Re
Fi

(-471, 0) (-27t, 0) O (271,0) (4ti, 0)

Y'
Fig. 4.22 Graph of y = cos 1 (cos x)

ReadYourFlow.COM
4.28 MATHEMATICS-XII

27C+0 , ii-Bn/l < 0 < - 3tc/2


n:+ 0 , if - 37r/2 < 0 < - tc/2
tan 1 (tan 0) = 0 ,if-rc/2<0< 7r/2
0-ti: , if n/2 < 0 < 3rc/2
0-271, if 37t/2 < 0 < 5tc/2 and so on

y= 2

X y X

w
X' % X
o l2 * 0)

Flo F=-f

ee
Fr
or
ur
Y'
sf
Fig. 4.23 Graph of y = tan 1 (tanx)
k
Yo
oo

2tt:4- 0 , if 0 e
eB

- 0 , if 0 6 -7C, Iu
r
ou
ad

sec 1(sec0)=-
Y
nd

2rc - 0 , if 0 e 7i, —
Re

2
Fi

0 - 27t , if 0 e 2tc, and so on

(0,71)

o (f,o) (2”'°> (f.o)


(-271,0)
(f'°)
Y'
Fig. 4.24 Graph of i/ = sec 1 (secx)

ReadYourFlow.COM
INVERSE TRIGONOMETRIC FUNCTIONS 4.29

5tt 3n
2k+0 , if 0 e------,-27i u -27r,------
2' 2
3 71 n
-7i-0,if0e------,-71 u -7i,—
2 ' 2

cosec 1(cosec 0) = 0 , if 0 e --,0 u 0,-


2' ' 2

71-0 •r a
, if 71
0 e —,71 I U I 71, —
^71
2' 2
3k 5n
0-27t, if 0e —, 2711 ul 271,— and so on

w
*=!

Flo
(“■f)

ee
Fr
X'
(-271, 0) for
ur
*=-!
ks
Yo
oo
B

Y'
re

Fig. 4.25 Graph of i/ = cosec 1 (cosec x)


ou
ad

2n + 0 , if 0 e (- 2ti, ti)
Y

TC+ 0 , if 0 e (-71, 0)
nd
Re

cot 1 (cot 0) = • 0 , if 0 e (0,7t)


Fi

0-ti: , if 0 e (7r,27i)
0-27t , if 0 e(27i, 3ti) and so on

(-271, 0)

Fig. 4.26 Graph of y = cot-1(cot x)

ReadYourFlow.COM
4.30 MATHEMATICS-XII

ILLUSTRATIVE EXAMPLES

LEVEL-1

EXAMPLE l Evaluate each of the following:


7t 2 71
(i) sin -1 sin — (ii) cos 1 cos —
3 3
. 2 71
(iii) tan 1 tan — (iv) sin -1 sin —
4 3
7 7T 3 TC
(v) cos 1 cos---- [CBSE 2009] (vi) tan tan----
6 4

(vii) sin -1 sin (- 600°) [NCERT EXEMPLAR]

w
(viii) cos -1 cos(- 680°) [NCERT EXEMPLAR]

Flo
ee
SOLUTION (i) We know that sin 1 (sin 0) = 0, if - - < 0 < -
2 2

Fr
TC
sin 1 sin —
3 3
or
ur
sf
(ii) We know that cos 1 (cos0) = 0 for 0 < 0 < tt
k

2ti 2n
Yo

cos -1 cos —
oo

3 3
B

(iii) We know that tan 1 (tan 0) = 0 for <0<—


re

2 2
tan 1 [ tan — 71
ou
ad

4 4
Y

. 2ti) 27t
(iv) sin 1 sin — h* — as — does not lie between - — and —
nd
Re

3 3 3 2 2
Fi

.
• — 1 sin — 2 TC TC 2 71 TC
Now, sin sin sin 7t - v sin — = sin tc -
3 3 3 3
71
sin sin — [v sin (tc - 0) = sin 0]
3
7C
3
ALITER From Fig. 4.21, we find that
sin-1 (sin 0) = tc - 0 for — < 0 < —
2 2
._lf. 2tc 2tc tc
sin I sm — = TC------- = —
3 3 3
7 TC 7 TC 7 TC
(v) cos -1 cos — I * —-, because — does not lie between 0 and tc.
6 6 6
7 TC 7tc 5 TC
Now, cos -1 cos — = cos 1 •! COS [ 2 TC - — — = 2 TC--------
6 6 6 6

ReadYourFlow.COM
INVERSE TRIGONOMETRIC FUNCTIONS 4.31

5n
= cos -1 cos — [v COS (2 7t - 0) = cos 0]
6
5ti:
6
ALITER From Fig. 4.22, it is evident that
cos-1 (cos 0) = 2Tt - 0 for 7i < 0 < 271
7 71 7rc Stt
cos -1 cos— = 2ti:------
6 6 6
371 3 71 71 ,71
(vi) tan 11 tan — I * — , because — does not lie between - — and —.
4 4 ' 4 2 2
3 71 3tc Tt
Now, tan 1 tan — = tan 1 tan f 71 - — K-----
4 4 4 4

low
TT
= tan”1 - tan — [v tan (Tt - 0) = - tan 0]
4
Tt Tt
= tan 1 tan
4 4

ee
ALITER It is evident from Fig. 4.23 that
rF
Fr
tan-1 (tan 0) = 0 -7t for — < 0 < —
2 2 for
-1 3k 3k K
tan tan —
4 4 4
u
ks

(vii) sin -1 sin (- 600°) = sin -1 sin - 600 x —


Yo

180
oo

107t . 107t
B

= sin-1 sin = sin -1 -sm


3 3
re

• -1 • —l Tt
- sm -sin 3tt + — = sm -sm —
ou
ad

3 3
Y

Tt
= sin -1 sm — =--
Tt

3 3
nd
Re

ALITER From Fig. 4.21, we observe that


Fi

sin-1 (sin 0) =- 3tc-0 for -— < 0 < -^


2 2
• -1 sin (- 600°) = sm • -1 sm 107t 107t Tt
sm = -3k + =—
3 3 3
347t
(vih) cos -1 cos (- 680°) i- = cos 1 (cos 680°) = cos -1 cos 680 x — = cos -1 cos
180 9
2tt
= cos -1 cos 4tt-
9
27t 27t
= cos -1 cos
9 9
ALITER From Fig. 4.22, we find that
i 34tt
cos 1 (cos 0) = 0 + 47t, if - 4tc < 0 < - 37t and - 680° =--------
9
347t 3477 2k
COS -1 cos (- 680°) V = cos -1 cos + 477 =----
9 9 9

ReadYourFlow.COM
4.32 MATHEMATICS-XII

EXAMPLE 2 Express each of the following in the simplest form:


1 - cos x
(i) tan - 1 -n<x <k [NCERT]
1 + cos x '

COS X n K
(ii) tan 1 — <x <- [CBSE2012]
,\ 1 + sin x /J' 2 2

COS X n
(iii) tan -1
71
,, — < .* < — [NCERT]
1 - sin x 2 2

cos x - sin x n 71
(iv) tan 1 ----- < .Y < — [NCERT]
cos y + sin y ' 4 4
SOLUTION (i) We have.

w
1 - cos Y
tan -1
1 + cos Y

2 sin 2 *
2
Flo
ee
= tan-1
2 cos 2 *

Fr
for
ur
ks
Yo
oo
B

— tan — , if - 7t < y < 0


re

, if 0 < Y < 71
ou
ad
Y

x
tan 1 l tan [ - - = -- /if-7l<Y<0
2 2
nd
Re

tan- 1 \ tan — 1 = — , if 0 < Y < 71


Fi

2 2
(ii) We have,
2 x 2 x
COS Y
cos — sin
tan -1 = tan 1 2 2
1 + sin y 2 x . 2 x _ . x Y
COS - + sin - + 2 sin - cos -
2 2 2 2
Y . Y Y Y
cos - - sin - cos — + sin
2 2 2 2
= tan -1
Y Y
,2
cos — + sin
2 2

Y Y
cos — sin
= tan 1 2 2
x x
cos + sin —
2 2

ReadYourFlow.COM
INVERSE TRIGONOMETRIC FUNCTIONS 4.33

x
1 - tan
= tan -1 2
x
1 + tan
2

n x n x tc n n x n ^ n x K
= tan 1 tan -- <x <-=> — < — <- =>0<------ < -
4 2 4 2 2 2 4 2 4 4 2 2
A LITER We have.
71
sin — + x
COS X 2
tan 1 = tan -1
\ 1 + sin x
71
1 - cos - + X
2
..in x) in x

low
2 sin I - + - I cos | — + -
= tan 1 -
2 sin
4 2 • = tan -i
cotlH
71 71 X 71 X

ee
7T + ^
= tan 1 tan = tan 1 tan
rF 2 4 2 4 2 4 2

Fr
(iii) We have.
for
2 x
cos----sin 2 *
cos x
u
tan 1 = tan 1 2 2
. 2x n .x x
ks

1 - sin x cos 2 —
x + sin — 2 sin - cos—
Yo

2 2 2 2
oo

x x
B

X X
cos — + sin cos — sin
2 2 2 2
re

= tan 1 -
,2
x x
cos — sin
ou
ad

2 2
Y

X x
cos — + sin
nd

2 2
Re

= tan 1 •
X x
cos — sin
Fi

2 2J
x
1 + tan
= tan 1 2
x
1 - tan
2
, f 7C X
= tan 1 tan — + —
U 2
7t X n n n. x n . it x n
---- 1----- — < x < - => — <-< — => 0 < —i— < —
4 2 2 2 4 2 4 4 2 2
A LITER We have.
n
sin — x
cos x 2
tan -1 = tan 1 •
1 - sin x 71
1 - COS------X
2

ReadYourFlow.COM
4.34 MATHEMATICS-XII

n x
2 sin [ — - — cos
= tan 1 • U 2 4 2
2 sin 2(*-X
U 2
= tan"1 n x
cot
4 2
K 71 X
= tan -1 tan
2 4 2
_ TC X
= tan-1
tan'1 rf ~4+2
(iv) We have,
cos x - sin x 1 - tan x

w
tan - 1 = tan -1
cos x + sin x 1 + tan x y

= tan"1

Flo
tan — -x
4

ee
7T 7t 7t n 7t _ 71 7t
----< X < — => — <-x< -=> 0 <- — x < —

Fr
-------X
4 4 4 4 4 4 2
EXAMPLE 3 Prove that: or
ur
+ cos x + ^/I^cosTx 71 X
sf
(i) tan- 1 •
^/l + COS X -^Jl - COS X = 4 + 2' 0 < x < —
2
k
Yo

+ sin x + ^/l - sin x


oo

(ii) cot 1 x A <x <-


> =-,0 71
[NCERT, CBSE 2009,2014,2016]
+ sin x - ^1 - sin x 2 2
B
re

SOLUTION (i) We have.


! + COS X + - COS X
ou
ad

tan
yl + cos x - 71 - COS X
Y

2 cos2 — + . 2 sin 2 x
nd
Re

= tan-1 • 2 V 2
Fi

2 cos 2 X
2 sin 2 X
2 2
x x
cos - + sin -
2 2 n X 7C
= tan’1 v 0 <- <- cos — > 0, sin — > 0
X X 2 4 2 2
cos----sin -
2 2

1 + tan -
71 X
= tan'1
1 - tan -
2
2 = tan- 1
tanM = —+ -
4 2
.
V 0<x<-
2
7T TC

4
71

4
X

2
7C

2

(ii) We know that


X ± sin
1 ± sin x = cos2 —+ sin2-±2sin —cos- = cos — . —xf
2 2 2 2 2 2
2 J 71 + sin x + 7l - sin x
cot
7l + sin x - 71 - sin x j

ReadYourFlow.COM
INVERSE TRIGONOMETRIC FUNCTIONS 4.35

x x']2 ,2
X X
cos — + sin — + cos — sin -
1 2 2 2 2
= cot <
2
X X
cos - + sin - cos — sin —
2 2 2 2

x X X X
cos - + sin — + cos----sin
2 2 2 2
= cot 1 V Vx2=|x|
X X X X
cos — + sm — cos----sin —
2 2 2 2

x x X X
cos - + sm — + cos----sm —
2 2 2 2 _ X 71 X . X
= cot 1 • [■.• 0 < — < —cos — > sm —

w
x X X X 2 4 2 2
cos - + sin - cos - sm -
2 2 2 2

= cot 1 cot
x
2
x

Flo n
v 0 <- <-
X 7t

ee
2 2 4.

Fr
EXAMPLE 4 Proue that:
^Jl + COS X + ^/l - COS X 71 X 371
(i) tan-1
or
ur
-------- , if 71 < X <-----
yjl + cos x - ^/l - COS X
1 4 2 2
sf

yi + Sin x + 71 - sin x 71 X 71
k
Yo

(ii) cot 1 -------- , if — < X < 71 [CBSE2011]


oo

^/l + sin x - ^/l - sin x 2 2 2


B

SOLUTION (i) We have,


re

_1 1^/1 + cos x + ^/l - cos x


ou

tan
ad

1 ^/l + COS X - - COS X


Y

2 cos 2 X
+ .2 sin 2 *
nd
Re

= tan”1 2 2
Fi

2 cos 2 X
2 sin 2 X
2 2
x
V2 cos - + V2 sin —
2 2
= tan -1
V2 cos —x - V2 sin -
2 2

x x 3ti ti x 3ti
- cos — + sm — 71 < X <---- =>—<—<-----
= tan 1 • 2 2 2 2 2 4
X X x X X X
- cos----sin cos — = - cos — , sin — = sin —
2 2 2 2 2 2
x x'
cos----sin —
= tan 1 2 2
x . x
cos - + sm -
2 2

ReadYourFlow.COM
4.36 MATHEMATICS-XII

1 - tan ^
= tan 1 2
1 + tan
2
-1 fn x 71 X 3k k k x n
= tan tan------ K<X <----- ------<-----------<-------
U 2 4 2 2 2 4 2 4.
(ii) We have.
Jl + sin x + Jl - sin x
cot" .
+ sin x - - sin x
2 2
x x x x
cos - + sin - + cos — sin -
= cot 1 2 2 \ 2 2
2 2
X . x X . X

low
cos - + sin - cos — sin -
2 2 II 2 2

x x X . X
cos — + sin — + cos — sin -
2 2 2 2
= cot 1

ee
X X X X
cos — + sin — cos — sin —
rF
Fr
2 2 2 2

x x x x
for
cos — + sin — cos — sin —
2 2 2 2 71 71 X 7t X . X
ou
= cot -1 Y —<X<7r=> — <—< — => COS - < sm -
x 4 2 2
ks

X X X 2 2 2
cos - + sin + cos-----sin
2 2 2 2
oo
Y

71 _ 71 X 71
eB

X f 71 X 71 X
= cot 1 tan — = cot 1 cot------ V — <X<7!=> 0 <--------- <-
2 12 2 2 2 2 2 2 4
r

REMARK In order to simplify trigonometrical expressions involving inverse trigonometrical functions,


ou
ad

following substitutions are very useful:


Y

Expression Substitution
a2 + X 2
d

x = a tan 0 or, x = a cot 0


Re
n

a 2 -x 2 x = a sin 0 or, x = a cos 0


Fi

2 2
x -fl x = a sec 0 or, x = a cosec 0
a-x a+x
or. x = a cos 20
a+x a-x
2 2
fl -x
x2 = fl2 cos 20
2 2
fl + X

EXAMPLE 5 Write the following functions in the simplest form:

x a-x
(i) tan 1 ■ ,-a <x <a [NCERT] (ii) tan" 1 ■, - a <x <a
-x 2 a+x

X X
(iii) sin -1 (iv) cos 1 •
v?+« 2 ■yjx2 +a 2

ReadYourFlow.COM
INVERSE TRIGONOMETRIC FUNCTIONS 4.37

SOLUTION (i) Putting x = asm 0, we obtain

tan- 1 • x

a sin 0
= tan 1
^ja2 -a2 sin2 0

fl sin 0
= tan -1
a cos 0

= tan 1 (tan 0) = 0 = sin -1 * v x = a sin 0 => sin 0 = — => 0 = sin 1 —


a a a
(ii) Putting x = a cos 0, we obtain
a -x

w
tan 1
a+x
a - a cos Q
= tan -1
a + a cos 0

Flo
ee
1 - cos 0
= tan -1

Fr
y1 + cos 0

2 sin 2 9
for
ur
= tan -1 2
2 9
1 2 cos
ks

2
Yo
oo

0
= tan 1 tan —
B

2
re

0
= tan -1 tan — v -fl<x<fl=i>O<0<7t=>O<—< — tan — = tan -
2 2 2 2 2
ou
ad

0 1. x -1 *
cos -1 * •: x=a cos 0 => cos 0 = — => 0 = cos
Y

_ 2 ~ 2 a a a
(iii) Putting x = a tan 0, we obtain
nd
Re

x
Fi

■ —1
sin
Jx2 +a 2
• —1 a tan 0
sin
^a2 tan2 Q + a2
a tan 0
= sin-1
a sec 0
= sin - 1 (sin 0)

= 0= tan -1 * v x -a tan 0 => tan 0 = - => 0 = tan 1 —


a a a
(iv) Putting x = a cot 0, we obtain
x
cos -1
y/x2 +a 2

ReadYourFlow.COM
4.38 MATHEMATICS-XII

a cot 0
= cos -1
^a2 cot2 0 + a2

a cot 0
cos -1
a cosec 0
X _i x
= cos -1 (cos 0) = 0 = cot 1 * X = fl cot 0 => cot 0 = — => cot —=0
a a a
EXAMPLE 6 Prove that:
(i) tan 1 • — - — cos 1 X , 0 < X < 1
+X+ V1 4 2
[NCERT, CBSE 2010,2011,2014,2017]

w
K 1
(ii) tan 1 • =----1---- cos 1 x2 , — 1 < X < 1 [NCERT EXEMPLAR]
yjl + x2-^jl-x2 4 2

SOLUTION (i) Putting x = cos 2 0, we obtain


-l j yJl~+~cos~2Q - yjl - cos 2 0
Flo
ee
tan | yjl + cos 2 0 + - cos 2 01

Fr
1 -y2 cos2 0-^2 sin2 0 or
ur
= tan
-^2 cos2 0 + -^2 sin2 0
k sf

cos 0 - sin 0
Yo

= tan 1 v 0 <x <1 => 0 <cos 20 <1 => O<20< —=>O<0<-


oo

cos 0 + sin 0 2 4
B

1 - tan 0
re

= tan'1
1 + tan 0
ou
ad

= tan -1 tan — -9
Y

4
nd
Re

= *-0 v o<0<-=> O<--0<-


4 4 4 4
Fi

71 1 _1 1
=-------- COS X cos 20 = x 20 = cos x => 0ri = -
1
cos- 1 x
4 2 2
2
(ii) Putting x = cos 20, we obtain
! J ^/l + cos 20 + - cos 20
tan | ^/l + cos 20 - - cos 20 J

-j ^2 cos2 0 +-\/2sin^0
= tan
^2 cos2 0 - -^2 sin2 0

cos 0 + sin 0
= tan 1
cos 0 - sin 0

1 + tan 0
= tan 1
1 - tan 0

ReadYourFlow.COM
INVERSE TRIGONOMETRIC FUNCTIONS 4.39

= tan -i tan -+e


4

v -i <*<1^ o<.t2 <i=> o<2e<-=> o<e<-


4 2 4
71 1 -1 [v x2 = cos 2 0 => 2 0 = cos
— + — COS x2
4 2
EXAMPLE 7 Simplify each of the following:
3 4 . ) r 3tz n [NCERT EXEMPLAR]
(i) cos 1 — cos X + sin x , where------< x < -
5 5 4 4
5 12 .
(ii) sin -1 — cos x + sin x
13 13
3 4 .
(i) In order to simplify cos -1

w
SOLUTION — cos x + - sin x , we will have to express
5 5
3 4 3
— cos x + — sin x in the form of cosine of some expression. For this, let — = r cos 0 and
5
4
5

Flo 5

ee
= r sin 0. Then,
5

Fr
3)2 4)2 r sin 0 4
r= + = 1 and, tan 0 =
for
5 r cos 0 3
ur
5
-1 4
=> r = 1 and, 0 = tan
ks

3
Yo

3 4 _i 4
oo

=> — = cos 0 and = sin 0, where 0 = tan —


5 5 3
B

3 4 .
cos -1 — cos x + — sm x
re

5 5
ou

-1 4
= cos 1 (cos 0 cos x + sin 0 sin x) = cos -1 {cos (x - 0)} = x - 0 = x - tan
ad

3
Y

5 cos x + ^- sin x I,
— we will have to express
(ii) In order to simplify sin
nd
Re

13 13
5 12 ^5 .
Fi

cos x + — sin x in the form of sine of some expression. For this, let — =r sin 0 and
13 13 13
12
— =r cos 0. Then,
13

r=
5f + 12 f
= 1 and, tan 0 =
r sin 0 5
13 13 r cos 0 12

r = 1 and, 0 = tan -1 5
12
5 12 = cos 0, where 0 = tan 1 —
= sin 0 and
13 13 12
5 12 .
sm cos x h---- sm x
13 13
-1 5
sm 1 (sin 0 cos x + cos 0 sin x) = sin 1 {sin (x + 0)} = x + 0 = x + tan
12

ReadYourFlow.COM
4.40 MATHEMATICS-XII

example 8 Simplify each of thefollowing:


sin x + cos x 71 71 sin x + cos x 7t 5rc
(i) sin 1 , — <x < — (ii) cos 1 — <x < —
V2 4 4 V2 '4 4
sin x + cos x
SOLUTION (i) sin-1
V2
= sin 1 1 . 1
-t= sin x + -t= cos x
V2 V2
K 71
= sin -1 sin x cos — + cos x sin -
4 4
7t 7C 7t A K K
= sin -1 sin x + - = x+- -- <x < — => 0 < x + - < -
4 4 4 4 4 2
sin x + cos x

w
(ii) cos -1
V2
1 1
= cos -1 —f= sin x + —cos x
V2 V2
Flo
ee
-1 . 71 K
sin x sin —

Fr
COS + COS X cos -
4 4
71 71 7t 571 A 7T
-1
or
ur
COS COS X----- = X----- V - < X < --- => 0 < X-------< 7T
4 4 4 4 4
sf

REMARK This example can also be solved by using the procedure given in the earlier example.
k
Yo
oo

LEVEL-2
B

EXAMPLE 9 Simplify each of the the following:


re

-if sinx + cosx 71


(ii) cos 1 sin x +cosx
5tt 5 71 971
(i) sin - <x < — ----< X <-----
ou
ad

V2 '4 4 V2 ' 4 4
Y

-if sinx + cosx


SOLUTION (i) sin
V2
nd
Re

1 1
= sin -1 -7=sinx + ^=cosx
Fi

V2 V2
71 . .71
= sin -1 cos—sin x + sin — cos x
4 4
= sin -1 sin x + -
4
71 71 5 71 71 71 3ti 71 71 71
= 71- X + - .• - <x<---- =>—<x + —<------ =>------ <71— XH----- < —
4 4 4 2 4 2 2 4 2
3 71
-------X
4
-l f sinx + cosx
(ii) cos
V2
1 . 1
= cos -1 —7=smx + —?= cosx
V2 V2
-1 71 . .71
cos cos x cos — + sin x sin —
4 4

ReadYourFlow.COM
INVERSE TRIGONOMETRIC FUNCTIONS 4.41

n
cos -1 COS X-----
4
ti: Stt 971 it 71
X .* ---- < X < => 71 < X - X-- <71
4 4 4 4 4
9n
----- x
4
EXAMPLE 10 Evaluate the following:
(i) sin-1 (sin 10) (ii) sin-1 (sin 5) (iii) cos 1 (cos 10) (iv) tan 1 {tan (-6)}

SOLUTION (i) We know that sin- 1 (sin 0) = 9' --


2-< 0 < -.
2
71 ,71
Here, 0 =10 radians which does not lie between —— and — But, 3tc - 0 i.e. 3ti -10 lies between
2 2

w
- — and . Also, sin (37t -10) = sin 10.
2
sin-1 (sin 10) = sin-1 (sin (3ti: -10)) = 371-10.
7n

Flo Stt 7 ti
ALITER We know that 37t < 10c < — and sin -1 (sin 0) = Sr: - 0 for — <0<—.

ee
2 2

Fr
• -1 (sin 10) = Stt-IO.
sin

- and —. But, lit-5 and 5 - 27rboth lie


(ii) Here, 0=5 radians. Clearly, it does not lie between - ^
for
ur
2
71 7T
between — and — such that
ks

2 2
Yo
oo

sin (5 - 2tc) = sin (- (27t -5)) = - sin (2k -5) = - (- sin 5) = sin 5
B

sin- 1 (sin 5) = sin- 1 (sin (5 - 2k)) =5 - 2k.


re

3k
ALITER We know that — <5C < ^ and sin 1 (sin 0) = 0 - 2k for ^ < 0 < ^ (see Fig. 4.21)
2
ou
ad

sin-1 (sin 5) = 5-2k.


Y

(iii) We know that cos -1 (cos 0) = 0, if 0 < 0 < ti. Here, 0=10 radians.
nd
Re

Clearly, it does not lie between 0 and k. However, (4k-10) lies between 0 and k such that
Fi

cos (4k -10) = cos 10.


.•. cos-1 (cos 10) = cos-1 (cos (4k-10)) = 4k-10
ALITER We know that 3k < 10c < 4k and cos-1 (cos0) = 4k -0 for 3k < 0 < 4k (see Fig. 4.22)
cos -1 (coslO) = 4k-10.
(iv) We know that tan 1 (tan 0) = 0, if - -^ < 0 < —
-. Here, 0 = - 6 radians which does not lie
2
between - — and -
-. However, 2k - 6 lies between - — - and - such that
2 2
tan (2k - 6) = - tan 6 = tan (- 6)
tan 1 {tan (-6)} = tan-1 {tan (2k - 6)} = 2k - 6

ALITER We know that - 2k < - 6C < - and tan 1 (tan 0) = 0 + 2k for - 2k < 0 < - ^ (see

Fig. 4.23).
tan 1 (tan (-6)) = 2k-6.

ReadYourFlow.COM
4.42 MATHEMATICS-XII

EXERCISE 4.7
LEVEL-1

1. Evaluate each of the following:


7t . 7%
(i) sin -1 sin — (ii) sin-1 sin —
6 6
. 5n . 1371
(hi) sin"1 sin — (iv) sin -1 sin
6 7
. 17n 17n
(v) sin -1 sin (vi) sin -1 sin -
8 8
(vii) sin 1 (sin 3) (viii) sin"1 (sin4)
(ix) sin"1 (sin 12) (x) sin"1 (sin2)

w
2. Evaluate each of the following:
7T 5n
(i) cos"1 -I cos (ii) cos -1 cos —

(iii) cos -1 cos


47t
4

Flo 4
137t

ee
(iv) cos -1 cos
3 6

Fr
(v) cos 1(cos3) (vi) cos 1 (cos 4)
for
ur
(vii) cos"1 (cos5) (viii) cos"1 (cos 12)
3. Evaluate each of the following:
ks

(i) tan"1 ftan — 6n


Yo

(ii) tan -1 tan —


oo

3 7
B

7tc 9 71
(iii) tan -1 tan — [NCERT] (iv) tan -1 tan —
re

6 4
(v) tan 1 (tanl) (vi) tan 1 (tan 2)
ou
ad

(vii) tan"1 (tan 4)


Y

(viii) tan_1(tanl2)
4. Evaluate each of the following:
nd
Re

n 2n
(i) sec -1 sec — (ii) sec -1 sec —
Fi

3 3
5 71 7n
(iii) sec -l sec— (iv) sec -1 sec —
4 3

sec -1
97t 7n
(v) sec— (vi) sec -1 sec
5 3
1371 25 71
(vii) sec -1 sec (viii) sec -1 sec
4 6
5. Evaluate each of the following:
71 3 71
(i) cosec -1 cosec — (ii) cosec -1 cosec —
4 4
671 1171
(iii) cosec -1 cosec — (iv) cosec -1 cosec
5 6
1371 9ti
(v) cosec -1 cosec (vi) cosec -1 cosec
6 4

ReadYourFlow.COM
INVERSE TRIGONOMETRIC FUNCTIONS 4.43

6. Evaluate each of the following:


(i) cot'1 f cot — 4k
(ii) cot -1 cot —
3 3
9 k 19 k
(iii) cot -1 cot — (iv) cot -1 cot
4 6
-1 8k -1 cot 21k
(v) cot cot (vi) cot
3 4
7. Write each of the following in the simplest form:

a (ii) tan 1 j x + + x2 j , x gR
(i) cot 1 • ■ ,\x\>a

-Jl+x2 -1
(iii) tan 1 j^/1 + x2 -x , x eR (iv) tan 1 < ■ , x *0

low
X

-Jl + x2 + 1 a-x
(v) tan 1 < ■ ,x*0 (vi) tan 1 , -a <x <a
x a+x

ee
rF
Fr
X x + tJi- x2 1 1
(vii) tan 1 • ,-a <x <a (viii) sin 1
<X<V2
a+ ^7 for ' 2
ou
,Jl + x + y]l -x
(ix) sin -1 ,0 <x <1 (x) sin 2 tan 1
ks

2
oo

ANSWERS
Y
B

h (i) ^ (h) (hi) ^ / \ n (vi)


(iv) ~ (v) -
re

6 6 7 8 8
(vii) k - 3 (viii) ft _ 4 (ix) 12-4k (x) k-2
ou
ad

2. (i) ^ (ii) ^ (iii) ^ (v) 3 (vi) 2k-4


(iv) ~
Y

o
(vii)2K-5 (viii) 4k-12
nd
Re

3- (i) f (ii) (hi) 7 (iv) ^ (v) 1 (vi) 2-k


Fi

D 4
(vii)4-K (viii) 12-4k
. ... K .... 2k 3k
4. « 3 (H) — (iii) ^
4
(iv)f (V)f (vi)f
, ..v 3n / • • .v 7C
(vn) — (vm) -
4 6
5. (i) ^ (ii) ^ ( \ 71 (vi)
(iii) (iv) 7 V 6
0 o

6‘ (i) f (ii) f (iii) ^


4
(iv) 76 (v)f (vi) l4
1 (iii) — cot 1 x
7. (i) cosec — (ii)------- cot x
a 2 2
(vi)| -1 X
(iv) - tan"1 x (v) ---
- tan 1 x cos
2 2 2 a
(vii) — sin" 1— (viii) — + sin" 1 x (ix) — + - cos 1x (x) yjl-x2
2 a 4 4 2

ReadYourFlow.COM
4.44 MATHEMATICS-XII

HINTS TO NCERT AND SELECTED PROBLEMS


7. (ii) Putting x = cot 0, we obtain
1 = tan 1 (cot 0 + cosec 0) = tan 1 1 + cos 0
tan x+
sin 0

2 cos 2 9
= tan 1 - 2 > = tan 1 cot - = tan -1 7t _ 0
tan
0 . 0 0 2 2~2
2 sin — cos —
2 2
n 0 TC 1 1
------ cot X
2 2 2 2
(ix) Putting x = sin 0, we obtain
x+ sin 0 + cos 0

low
sin sin = sin- 1 sin - + 0
2 V2 4
It „ TC . - 1
= — + 0= — + sm x
4 4

ee
REM ARK Lett, p and h denote respectively the base, perpendicular and hypotenuse of a right triangle
rF
Fr
PQR and let ZQPR = 0. Then,
R
for
ou
ks
oo
Y

P
r eB
ou
ad
Y

Q
d
Re
n
Fi

P
sin 0 = - — and cot 0 = -

0 = sec

=> sin P
h
It follows from the above result that any inverse trigonometric function can be expressed
interms of the remaining five inverse trigonometric functions. For example, if sin -1 f — |is to be
vl3 J
expressed interms of other five inverse trigonometric functions, then we construct a right
triangle with perpendicular p = 5 and hypotenuse h = 13. The base b of this triangle isb = 12.

ReadYourFlow.COM
INVERSE TRIGONOMETRIC FUNCTIONS 4.45

5 -1(12 5 13 13 12
sin-1 = cos = tan -l = cosec -1 = sec -1 = cot -l
13 13 12 5 12 5

4.4.2 PROPERTY-II
In Chpater 2, we have learnt that if /: A -» B is a bijection, then /_1 :B^A exists such that
/o/_1(x) = x or, / (/-1(x)) = x for all x e B. Applying this property on various trigonometric
functions and their inverses, we obtain the following property.
PROPERTY
(i) sin (sin - 1 x) = x for all x e [-1,1]
(ii) cos (cos“ 1 x) = x, for all x e [-1,1]
(iii) tan (tan ~ 1 x) = x for all x eR

w
(iv) cosec (cosec- 1 x) = x for all x e(co,-1] u[l, co)
(v) sec (sec- 1x) = x, for all x e(-oo,-1] cj[1, oo)
(vi) cot (cot- 1 x) = x

Flo
for allx e R.

ee
This property and the above Remark help us in finding the values of expression of the form

Fr
/(&-1(x)), where/ and g are trigonometric functions. We may use the following algorithm for the
for
same.
ur
ALGORITHM
ks

STEP 1 Obtain the expression and express it in the form /(g-1(x)), where f and g are trigonometric
Yo
oo

functions.
B

STEP II Express g-1(x) in terms off~l by using the following results:


re

b h
sin-1 f- = tan -UP = = sec -1 = cot -1
ou

= cos -l
ad

h h b b yp)
Y

, where p, b and h denote respectively the perpendicular, base and hypotenuse of a right triangle.
nd
Re

STEP m Let g~ ^x) = / ~ !(y). Replace g -1(x) by /-1(y) in /(^_1(x)) and use property-ll to get
Fi

f(g-\x)) =/(/_1(y))=y-
Following example will illustrate the above algorithm.
ILLUSTRATIVE EXAMPLES
LEVEL-1

example 1 Evaluate each of thefollowing:


(i) sin sin 1 (ii) sin ^cos 1

(iii) sin tan -1 15 (iv) sin ^cot 1


8

(v) sin sec -1 17 (vi) sin ^ cosec 1


15

ReadYourFlow.COM
4.46 MATHEMATICS-XII

SOLUTION (i) Using sin (sin 1 x) =x, x we obtain

sin f sin" 1 — 5
1313
(ii) In order to express cos -1 —4
in terms of sin , let us construct a right triangle with base
5
b = A and hypotenuse h = 5. The perpendicular of such triangle isp = 3.
-1 4 . 3 -1 b
cos — = sm — V COS - = sm —1 P
5 5 h h
Hence, sin cos 1 ^ = sm i sm ■ -1 3 3
5 5
(iii) The right triangle with base b = 15 and perpendicular p = 8 has hypotenuse h = 17.
_ 1 15 . _i 15
tan — = sm —
8 17 15

w
-1 15 _115 15
Hence, sin tan = sin sin 8
8 17 7

1 4 . _i 3
Flo
(iv) The hypotenuse of the right triangle with base b =4, perpendicular p = 3 is =5.

ee
cot — = sm
3 5

Fr
3
1 4 -1 3 3
Hence, sin cot = sin sin
3 5 5 4
for
ur
(v) The right triangle with base 17=15 and hypotenuse h-17 has perpendicular p = 8 has
ks

hypotenuse h =17.
Yo
oo

1 17 . -1 8
sec — = sm
15 17
B

8
re

-1 17 8
Hence, sin sec = sin sin -1 8 15
15 17 17
ou
ad

(vi) The right triangle with base b =15 and perpendicular p = 8 has hypotenuse h =17.
Y

sec-1 —
17 = sm -1 8
d
Re

15 17
n

8
Fi

8
Hence, sin sec -117 = sin sin -1 8 15
15 17 17
EXAMPLE 2 Evaluate each of the following:
1 5 8
(i) cos cos (ii) cos sin -1 (iii) cos tan -l 3
13 17 4
(iv) cos ^cot 1 ^ j -1 5 -1 13
(v) cos sec (vi) cos cosec
3 12
5
SOLUTION (i) cos cos -1 5
13 13
(ii) The right triangle with perpendicular p = 8 and hypotenuse h =17 has base b =15.
8 -1(15
sin -1 = cos
17 17

ReadYourFlow.COM
INVERSE TRIGONOMETRIC FUNCTIONS 4.47

• —1 8 -1 15 15
Hence, cos sin = cos cos
17 17 17
(iii) The right triangle with perpendicular p = 3 and base b = 4 has hypotenuse h =5.
3 4
tan -1 cos -1
4 5 3
-1 3 = cos cos -l4 4
Hence, cos tan
4 5 5 4

(iv) The right triangle with base b =15 and perpendicular p = 8 has hypotenuse h =17.
15 -lfl5
cot -1 = cos
8 17
8
Hence, cos f cot 1 — ) = cos -1 15 15
cos

low
8 17 17 15
(v) We find that the right triangle with hypotenuse h = 5 and base b = 3 has perpendicular p =4.
-1 5 -1 3
sec cos
3 5
4

ee
-1 5 3
1 3 ——
Hence, cos sec = cos cos
rF
Fr
3 5 5 3

(vi) Clearly, the right triangle with hypotenuse h =13 and perpendicular p =12 has base b =5.
for
-1 13 1 5
cosec = cos
u
12 13 12
ks

-113 -l 5 5
Yo

Hence, cos cosec = cos cos


oo

12 13 13 5
B

EXAMPLE 3 Eval uate each of the following:


re

(i) tan |\an 1 (ii) tan sin 1 (iii) tan j^cos 1


ou
ad

(iv) tan f cosec 1 (v) tan ^sec 1 (vi) tan ^cot 1


Y
nd
Re

SOLUTION (i) tan j^tan 1 ^j = -|


Fi

(ii) The right triangle with perpendicular p = 5 and hypotenuse h = 13 has base b = 12.
• -1 —
sin 5 = .tan -1 5
13 12 5
-1 5 = tan tan -1 5 5
Hence, tan sin 12
l 13 12 12
(iii) Tire right triangle with base b =8, hypotenuse h = 17 has perpendicular p = 15.
cos -1 —8 = ,tan -T 15
17 8 15
-l 8 = tan tan !l5 15
Hence, tan cos 8
17 8 8
(iv) We find that the right triangle with perpendicular p = 5 and hypotenuse h = 13 has its base
b = 12.
13 5
cosec -i = tan -1 5
5 12
12

ReadYourFlow.COM
4.48 MATHEMATICS-XM

5
Hence, tan j cosec 1 ~ = tan tan -1 5
12 12
(v) The right triangle with base b =12 and hypotenuse h=13 has perpendicular p =5.
-1 13 .
sec — = tan -1 5
12 12 5
_1 13 -l5 5
Hence, tan sec = tan tan 12
12 12 12

(vi) Clearly, the right triangle with base b = 8 and perpendicular p =15 has hypotenuse h = 17.
8 _115
cot -l = tan
15 8 15
-1 8 -115 15 8
Hence, tan cot = tan tan
V 15 8 8

w
EXAMPLE 4 Evaluate:
(i) sin (cot- 1 x) (ii) cos (tan 1 x)

SOLUTION

Flo
(i) We have to find the value of sin (cot-1x) = sin j^cot-1 ^ j . The right triangle

ee
Fr
with base b =x, perpendicular p =1 has hypotenuse /? = Vl + x2.

cot l x = sin
for
ur
VT77 i
i i
ks

Hence, sin (cot 1 x) = sin sin -1 x


Yo

Vl + x2 J Vl + x2
oo

(ii) cos (tan 1x)=cos^tan 1


B
re

The right triangle with perpendicular p = x and base b =1 has its hypotenuse h =xl + x2.
ou
ad
Y

tan-1x
nd
Re
Fi

Hence, cos (tan Lx) cos l

EXAMPLES Evaluate: cos sin INCERT EXEMPLAR]

SOLUTION cos sin -ll


4
-ll -1
= cos sin

-ll

v sin
-ii
— =cos -i
VTs
4 4
& cos 1 — = sin -1 V7
4 4

ReadYourFlow.COM
INVERSE TRIGONOMETRIC FUNCTIONS 4.49

_ Vl5 ^ 3 1;;V7 _^3Vl5-^


~ 4 X4 4X 4 ~ 16
EXAMPLE 6 Evaluate : sin ( cos 1 — + cosec -1 13
5 5
■ f -1 3 -1 13
SOLUTION sm cos — + cosec
5 5
-1 3 -1 13 -1 3 i .
= sin cos cos cosec + cos cos sm cosec -l 13
5 5 5 5
-l4 -1 12 -1 3 -1 5
= sin sin cos cos + cos cos sin sin
5 13 5 13
_4 12 3 _5__ 63
5 X 13 + 5 X 13 65

w
EXAMPLE 7 Find the value of the expression sin cot -1 cos (tan 11) [NCERT EXEMPLAR]

SOLUTION sin cot -1


cos (tan 11) j

Flo
ee
n
v tan -1-,1 = —

Fr
= sin < cot -1 cos-
4 for 4
ur
= sin cot -1 1
V2
V2
ks

V2 v cot -1 -?=
1 = sin -1
= sin sin -1
Yo

73 72 73
oo

72
B

2
73
re

3
EXAMPLE 8 Prove that:
ou
ad

(i) sec2 (tan- 1 2) + cosec2 (cot- 1 3) =15


Y

(ii) tan2(sec-12) + cot2(cosec-13) = 11 [NCERT EXEMPLAR]


d
Re
n

SOLUTION (i) We have,


Fi

sec2 (tan-1 2) + cosec2 (cot-1 3)

= {sec (tan 1 2))2 + {cosec (cot 1 3)}2

-1242 + | cosec cot 1 —


3 Yl 2
sec tan
1
= {sec (sec-1 TS)}2 + {cosec (cosec-1 TlO)}2 = (75)2 + (7T0)2 =15

ALITER sec2 (tan-12) + cosec2 (cot-1 3)


= 1 + tan2 (tan-12) +1 + cot2 (cot-13)
= 1 + {tan (tan-12)}2 +1 + {cot (cot-1 3)}2 = 1 + 22 +1 + 32 = 15

(ii) tan2 (sec-12) + cot2 (cosec-1 3)


= sec2(sec-12) -1 + cosec2 (cosec-1 3) -1
= {sec (sec-12)}2 -1 + {cosec (cosec-13)}2 -1 = 22 -1 + 32 -1 = 11

ReadYourFlow.COM
4.50 MATHEMATICS-XII

ALITER tan2 (sec-12) + cot2 (cosec-13)


-1 2^1 2 ON-) 2
tan sec + cot cosec -1 3 1
1 1
Vs Y|2 + < cot 2V2'jl 2
= < tan tan -1 cot -1
1 1 J
= (V3)2+(2V2)2 = 3 + 8=11
EXAMPLE 9 Prove that:
x2 + l
(i) sin cot 1 j cos (tan 1 .r) |
*2 + 2
x2+l
(ii) cos tan 1 ^ sin (cot 1 x) [CBSE 2010]

w
x2 + 2
SOLUTION (i) We have.

cos (tan 1 x) = cos

Flo = cos -1
1
+ X2

ee
Fr
sin cot 1 \ cos (tan 1
or
ur
1 *2+l
sf
= sin cot -1
x2 + 2
Vi
k
Yo
oo

(ii) We have.
B

sin (cot 1 x) = sin


re
ou
ad
Y
nd
Re

1 +x2 *2+l
Fi

2 + x2 x2 + 2
EXAMPLE 10 //sin j cot-1 (x +1) j
= cos (tan 1 x), then find x. [CBSE 2015]

SOLUTION We have.
sin | cot 1 (x + 1) j =
cos (tan 1 x)

1 1
sin sin -1 = cos cos -1
■Jx2 + 2x +2 yfl^x2 J
1 1
=>
\ x2 + 2x + 2 Vl + x2
=> ylx2 + 2x + 2 = Vl +x2 => x2 + 2x + 2 = x2 + 1 => 2x = -l => x = ~-
2
1
Hence, x = - ^ is a root of the given equation.

ReadYourFlow.COM
INVERSE TRIGONOMETRIC FUNCTIONS 4.51

EXAMPLE ll Solve the following equation for x :


(i) cos (tan ~1 x) = sin ^ cot-1 j [CBSE 2013,2014,2017, NCERT EXEMPLAR]

(ii) tan (cos_1x) = sin ^cot-1-^j

SOLUTION (i) We have,


cos (tan 1 x) = sin ^cot 1

:_-l4
=> cos cos -1 1 = sin sin
Vi + X 2 5
1 4
= — => 4 Vl + x2 =5 => 16 (1 + x2) = 25 16x2 =9 => x = ±-
Vl + x2 5 4
(ii) We have.

low
tan (cos 1x)=sin^cot

=> tan tan -1


VT7 = sin sin
-1 2

ee
x Vs
rF
Fr
Vl-X2 _ 2 1 -* ■■ = - => 4x2 =5 -5x2 => 9x2=5=> x = ± —
x "V5 ^
for
x2 5 3
u
LEVEL-2
ks
Yo
oo

EXAMPLE 12 If x = cosec tan 1 cos cot 1 sec sin ^ a


//
B
re

and, y = sec cot 1 sin tan 1 cosec cos 1 a


ou
ad
Y

where a e [0,1]. Find the relationship between x and y in terms of a.


SOLUTION We have.
nd
Re
Fi

x = cosec tan 1 cos cot 1 sec sin 1 fl

\N
1 1
cosec tan 1 - cos cot 1 sec sec -1 v sin 1 a = sec -1
v 1 - a2 ' j j

= cosec tan 1 • cos cot — i I

M
1
cosec tan 1 cos cos -i 1 v cot 1 = cos -i 1
yjl-a2 -Jl -a2 ^2-a2

= cosec tan 1 , ^ ■ cosec cosec 1 a/3-*2) “ /3-a2


^2-a2

ReadYourFlow.COM
4.52 MATHEMATICS-XII

\\
and, y = sec cot 1 sin tan -1 cosec cos -1*

•/ cos -1*
1
= sec cot -1 sin tan 1 cosec cosec -1 -1 1
=cosec
-Jl -a1 a/1 -fl2

1
= sec cot 1 sin tan 1

1 1
= sec cot 1 < sin sin
• —1 v tan -1 = sin -1 1
a/^J a/TV a/2-a2

w
sec cot 1 . 1 = sec ^sec 1 a/s-a2 j = /3-a2
\j2-a2
Flo
ee
Thus, we obtain

Fr
x = y = V3-fl2 ^ x2 = y2 = 3-a2.
EXAMPLE 13 //sin -1 x + sin _1 y + sin -1 z = 71, prove that
for
ur
(i) x-^1 -x2 + y tJi - y2 + z -Jl -z2 = 2xyz
ks

(ii) x4 + y4 +z4 + 4x2 y2 z2 =2 (x2 y2 + y2 z2 + z2 x2)


Yo
oo

SOLUTION (i) Let sin 1 x = A, sin 1 y = B and sin 1 z = C. Then, x = sin A, y = sin B
B

and z = sin C
re

Now,
sin 1 x + sin 1y + sin 1 z = 71
ou
ad

=> A + B + C =7i
Y

=> sin 2A + sin 2B + sin 2C = 4 sin A sin B sin C


nd
Re

2 sin A cos A + 2 sin B cos B + 2 sin C cos C = 4 sin A sin B sin C


=> sin A -^/l - sin2 A + sin B - sin2 B + sin C - sin2 C = 2 sin A sin B sin C
Fi

a/i - X2 + y a/i -y2 + 2 Vl -z2 = 2xyz


=> X

(ii) We have,
• -1 x + sin 1 y + sin 1 z
sin = 71

=> sin -1 x + sin — 1 y = 7t - sin 1z

=> cos sin 1 x + sin yj = cos I^ti-sin 1z

cos (sin- 1 x) cos (sin- 1 y) - sin (sin- 1 x) sin (sin- 1 y) = - cos (sin- 1 z)

=> cos ^COS 1Vl-X2 jcos^cos 1yjl -l/2 j -sin sin 1 x I sin sin -1' y = - cos ^cos-1Vl -z2J
= -a/i -z2
=> -^y
=> yjO- -x2)(l -y2) = xy --y/l -z2

ReadYourFlow.COM
INVERSE TRIGONOMETRIC FUNCTIONS 4.53

=> (1 -x2) (1 -y2) =^xy-Vl -z2 j [On squaring both sides]

=> 1 -x2 -y2 + x2 y2 = x2 y2+l-z2-2xy yjl-z2

=> x2 + y2 -z2 = 2xy -/T-z2

=> (x2 +y2 -z2)2 = 4x2 y2 (1 -z2)


=> x4 + y4 + z4 - 2x2 z2 - 2y2 z2 + 2x2 y2 = 4x2 y2 - 4x2 y2 z2
=> x4 + y4 + z4 + 4x2 y2 z2 = 2 (x2 y2 + y2 z2 + z2 x2)
yl + x2 --Jl -x2 n
EXAMPLE 14 If tan 1 • = • = a, then prove that x = sin 2 a.
■Jl + x2 +yjl-x
SOLUTION We have.
l ^/l +X2 -y/l -X2
tan = a
-Jl +x2 +Jl-x2

w
=> £-Jl ++ x2x +-Jl -x2 = tan a
Flo
+ x^-^x2 + 2+VU? /

ee
+x
tan a +1
=>
+ x2 -Vl -x2 + x2 +Vl-X2 j tan a -1 Fr
for
ur
+ x2 tan a +1
=>
2^1-x2 tan a -1
ks
Yo
oo

=>
-Jl -x2 1 - tan a
eB

+ x2 1 + tan a
r

1-x2 cos a - sin a


ou
ad

=>
+ x2 cos a + sin a
Y

1-x2 cos a - sin a


d

=>
Re
n

1 + x2 cos a + sin a
Fi

1-x2 1 - sin 2 a
=> => x2 = sin 2a
1 + x2 1 + sin 2 a
EXAMPLE 16 Prove that:
(K 1 - 1 fl] t ( TC 1 -l « 2b
tan -^ — + - cos - !> + tan < — — cos [CBSE 2017]
14 2 b 4 2 a

SOLUTION Let cos 1 J = ^ ^len, cos ® = f


LHS = tan ^ + + tan Tt 9
4 ~2
1 + tan 0/2 1 - tan 0/2
=> LHS =
1-tan 0/2 1 +tan 0/2

ReadYourFlow.COM
4.54 MATHEMATICS-XII

(1 + tan 9/2)~ + (1 - tan 9/2)2 2 (1 + tan2 9/2N _ 2 2b


=> LHS = — = RHS.
1 - tan2 0/2 1 - tan2 0/2 cos 0 (1

EXERCISE 4.8
LEVEL-1
1. Evaluate each of the following:
(i) sin ^sin 1 “jr j (ii) sin ^ cos 1

(iii) sin^tan 1
(iv) sin sec -117
8
(v) cosec cos 1 -^1 -112
(vi) sec sin

w
13

(vii) tan cos -1 8 (viii) cot ^ cos 1 j


17
(ix) cos ^ tan 1 ~ j

Flo
ee
Fr
2. Prove the following results:
(i) tan cos 1 ^ + tan -1 2 17 (ii) cos sin ^ + cot -1 3 6
for [CBSE 2012]
ur
3 6 2) sVTs
(iii) tan f sin -1 5 -1 3 63 (iv) sin^cos-1-^- + sin -1 5 63
---- b cos
ks

13 5 16 13 65
Yo
oo

1 4. Solve : cos j 2sin-1 (-x) J = 0


3. Solve: cos sin 1 x =
B

6
re

ANSWERS
7 12
ou

24 15
ad

1. (i) (ii) ttt (iii) (iv) —


25 13 25 17
Y

(v)! 13 15 (viii) |
(vii)
(vi) T
nd

8 4
Re

V35 1
(ix) L
Fi

3. ± 4. ± —
25 6 J2

HINTS TO NCERT& SELECTED PROBLEMS


2. (i) tan ^ cos 1 ^ + tan -i 2
3

tan cos -14 + tan tan -1 2


5 3
-1 4 -l2
1 - tan cos tan tan
5 3
-1 3
tan tan + tan tan -1 2 3 2
----1----
4 3 4 1 17
-1 3 i 3 2 6
1 - tan tan tan tan 2
-1 1---- X — u
4 3 4 3
(ii) cos ^ sin 1 “ + c°t-1 3
2

ReadYourFlow.COM
INVERSE TRIGONOMETRIC FUNCTIONS 4.55

:_-l3 -l 3 -1 3 sin cot -1 3


= cos sin cos cot -sin sin
5 2 5 2
-l4 -1 3 -sin sin -l3^, • —1 2
sin sin
= cos cos cos cos
5 Vl3 5 Vl3
_4 _3__3 _2_=____ 6
“ 5 X Vl3 5 X Vl3 5Vl3

4.4.3 PROPERTIES III & IV


PROPERTY-III Prove that:
(i) sin-1(-x) = - sin-1*, for all x e[-l, 1]
__ i __ i
(ii) cos (-*) = Ti-cos *, for all x e [-1,1]

(iii) tan - 1 (- x) = - tan- 1 * , for all x e R

w
(iv) cosec- 1 (- *) = - cosec for all x e (- co , -1] o [1, qo)

(v) sec- 1 (- x) = 7i — sec


(vi) COt- 1 (“ X) = 71 - cot- 1 x ,
Flo
for all x e (- co, -1] u [1, oo)

for all x e R

ee
Fr
PROOF (i) Clearly, - x € [-1,1] for all x e [-1,1]
Let sin - 1 (- x) - 0 for
ur
Then, - x = sin 0
=> x = - sin 0
ks

=> x = sin (- 0)
Yo
oo

=> - 0 = sin- 1 x [v x [-1,1] and - 0 e [-tt:/2 , tc/2] for all 0 e [-7r/2,7t/2]]


B

=> 0 = - sin- 1 x ...(ii)


re

From (i) and (ii), we get


ou
ad

sin - 1 (- x) = - sin - 1 x
Y

(ii) Clearly,-x e [-1,1] for all x e[-1,1].


d

Let cos- 1 (- x) = 0
Re
n

Then, - x = cos 0
Fi

=> x = - cos 0
x - cos (tt - 0)
=> cos- 1 x = 71 - 0 [•.• x e [-1,1] and ti - 0 e [0,7t] for all 0 e [0, ti]]

=> 0 = 7C - cos- 1 X •••(ii)


From (i) and (ii), we get
cos- 1 (- x) = 71 — cos- 1 x
Similarly, other results can be proved.
PROPERTY IV Prove that:

(i) sin -in = cosec for all x e (- oo, -1] u [1, oo)
x
(ii) cos- 1 f - sec 1 x, for all x e(- co, -1] u[l, co)
x

ReadYourFlow.COM
4.56 MATHEMATICS-XII

cot 1 a: ,forx>0
(iii) tan -if1
x -n+ cot- 1 x , for x < 0
PROOF (i) Let cosec- 1 x = 6

Then,
x = cosec 0
1
=> — = sin 0
x
-ll ~v x e (- oo, -1] u [1, oo) 1/x [-1,1]- {0}
=> 0 = sin -(ii)
x cosec- 1 x = 0 0 e [- n/2 , n/2] - {0}

From (i) and (ii), we get

^0 cosec 1 x

low
(ii) Let sec 1 x = 0
Then, x e (- oo, -1] u [1, co) and 0 € [0, ti] - {n/2}.
Now, sec- 1 x = 0

ee
=> x = sec 0
rF
Fr
=> — = cos 0
X
for
-ll 1
=> 0 = cos v x e (- oo, -1] u [1, oo) => e [-1,1] - (0) and 0 e [0,7t]
ou
-(ii)
x x
ks

From (i) and (ii), we get


oo

-ifl
cos sec 1 x
Y
B

x
re

(iii) Let cot 1 x = 0.


ou

Then, x e R, x ^ 0 and 0 e (0, n)


ad

-(i)
Now, two cases arise:
Y

CASE I When x > 0


nd
Re

In this case, 0 e (0, rc/2)


cot- 1 x = 0
Fi

=> X = cot 0
=> — = tan 0
x
1
=> 0 = tan -1 [y 0 e (0, tt/2)] -(ii)
x
From (i) and (ii), we get
tan-1 f —) = cot-1 x , for all x > 0.
x

CASE II When x < 0


In this case, 0 e (7r/2 , n) [•.• x = cot 0 < 0]
Now, —
- < 9 < n
2
=> — — <0 — 7C<0
2

ReadYourFlow.COM
INVERSE TRIGONOMETRIC FUNCTIONS 4.57

=> 0 - xc e (- ti/2, 0)
cot- 1 x = 0
=> x = cot 0
=> — = tan 0
x
— = - tan (tt - 0) [•.■ tan (rt-G) = - tan 0]
x
=> — = tan (0 - k)
x
1
=> 0 - 7t = tan -1 [••• Q -ne(-Ti/2, 0)]
x

tan -if1 = - 71 + 0 —(iii)

w
x
From (i) and (iii), we get

tan “ 1 f — I = - 7i + cot- 1 x , if x < 0.


x

Flo
ee
cot 1 X , for x > 0

Fr
Hence, tan 1 [ —
x - tt + cot- 1 x , for x < 0 for
ur
ILLUSTRATIVE EXAMPLES
ks

LEVEL-1
Yo
oo

EXAMPLE l Evaluate:
eB

5 7 8
(i) cos l sin -1 (ii) cot l sin -1 : -1
(iii) sec sm
13 25 17
r

SOLUTION We know that sin 1 (-x) = - sin -11 x for all x e [-1,1]. Therefore,
ou
ad
Y

5 :_-l5 -1 5 -ll2 12
(i) cos \ sin -l| = cos -sm =cos sin = cos cos
13 13 13 13 13
nd
Re

7 ■ —1 7 = - cot I sin -1 7 = -cot I cot -1 24 24


(ii) cot < sin -1 = cot -sm
Fi

25 25 25 7 7
8 -1 8 -1 8 -117 17
(iii) sec l sin -1 = sec -sin = sec sin = sec sec
17 17 17 15 15
EXAMPLE 2 Evaluate:
3 12
(i) sin < cos -1 (ii) tan i cos -1
5 13
12
(iii) cosec
13
SOLUTION We know that cos-1 (-x) = tc- cos-1 x for all x e [-1,1]. Therefore,
3 4
(i) sin \ cos -l = sin ^ 7i - cos -1 3 = sin cos
-1 3 = sin sin -14
5 5 5 5 5
12 -112 -112 5
(ii) tan \ cos -1 = tan 4 7i - cos = - tan cos = - tan tan -1 5
13 13 13 12 12

ReadYourFlow.COM
4.58 MATHEMATICS-XII

12 -1 12 1 12
(iii) cosec < cos -1 = cosec rc - cos =cosec cot
13 13 13
-1 13 13
= cosec cosec
5 5
EXAMPLE 3 Evaluate:
5
(i) sin < tan (ii) cos \ cot -1
12

(iii) cosec

SOLUTION We know that tan 1 (-x) = - tan 1 x and cot 1 (-x) = rc-cot -1 x for all x eR.
Therefore,
7 -1 7 -1 7 -1 7 7
tan -1

w
(i) sin = sin -tan = - sin tan = - sin sin
24 24 24 25 25
5 -l5 -1 5 -1 5 5
(ii) cos ( cot -1
12
= COS 71 — COt
12

Flo= -cos cot


12
= - cos cos
13 13

ee
4 -l4 -1 4 -l5 5
(iii) cosec ( cot -1 = cosec 7r-cot = cosec cot =cosec cosec

Fr
3 3 3 3 3
4 4 3
Prove that: sin -l = tan -1 = cos -1
for
EXAMPLE 4
ur
— 7t
5 3 5
SOLUTION We find that:
ks

4 • —1 4 4 4
Yo

sin-1 = -sin = - tan -1 = tan -1


oo

5 5 3 3
B

3 -1 3 4
= - tan -if4 = tan -l
and, cos -l
K - COS
-1 3
- 71 = - COS
re

5 5 5 3 3
-1 4 -1 4 -1 3
ou

= tan = cos
ad

Hence, sin -71


5 3 5
Y

tu/2 , if a: > 0
EXAMPLE 5 Prove that tan 1 x + tan 1 1
-n/2 ,ifx<0
nd

x
Re

SOLUTION We have,
Fi

l
-ifl cot x , for x > 0
tan
x - 7i + cot- 1 x , for x < 0

if1 tan 1 x + cot 1 X = 7l/2 , if x > 0


tan 1 x + tan
x tan- 1 x + cot- 1 x-7i = 7i/2-7r = -7r/2,ifx<0

EXERCISE 4.9

LEVEL-1
1. Evaluate :
7 5 13
(i) cos < sin -1 (ii) sec < cot -1 (iii) cot < sec -l
25 12 5
2. Evaluate:
7 12 3
(i) tan < cos -1 (ii) cosec cot -1 (iii) cos tan -1
25 5 4yJ

ReadYourFlow.COM
INVERSE TRIGONOMETRIC FUNCTIONS 4.59

3 5
3. Evaluate : sin < cos -l + cot -1
5 12
ANSWERS
24 13 5
1- (0 (ii) (iii)
25 5 12
24 13 4 56
2- 0) (ii) ^ (iii) 3. - —
7 5 5 65
4.4.4 PROPERTY V
PROPERTY V Prove that:
7t
(i) sin- 1 x + cos- 1 x for all x e[-l, 1]
2'
(ii) tan- 1 x + cot- 1 x = K for all x eR
2'

low
__ -i __ i

(iii) sec x + cosec x for all x e(-cc,-l]cj[l, oo).


PROOF (i) Let sin-1 x = 0 ...(i)
Then, 0 e [- n/2,7i/2] [’•’ X 6 [— 1, 1]

ee
=> --<0<-=>--<-0<-=>O<--0<7c=>--0e[0,7i]
rF
Fr
2 2 2 2 2 2
Now, sin- 1 x = 0 for
=> x = sin 0
ou
X = cos ^-0
ks

2
oo

=> cos- 1 X = --e [v x [-1,1] and (n/2 - 0) e [0, n]]


Y
B

2
n
re

=> 0 + cos 1 X ...(ii)


2
ou
ad

From (i) and (ii), we get


Y

sin- 1 x + cos- 1 x = n
2
nd
Re

(ii) Let tan- 1 x = 0 ...(i)


Fi

Then, Q e(-n/2 , n/2) [v x eR]


71 A 7t A TC A
— < 0 < — =>---- <-0<-=> 0 <----- 0 < % => ^-e)e(0,n)
2 2 2 2 2
Now, tan-1 x = 0
x = tan 0
x = cot f- - 0
2
cot-1 X = - - 0 v — - 0 e (0 , rt)
2 2
0 + cot- 1 n
X = ...(ii)
2
From (i) and (ii), we get
, -1 i n
tan x + cot x = —
2.
(iii) Let sec- 1 x = 0 -(i)

ReadYourFlow.COM i
4.60 MATHEMATICS-XII

Then, 0 g[0 , n] - {n/ 2] [•.■ x e (- oo, - 1] u [1, oo)]


=> O<0<K,e ^ -
2
=> -7X<-0<O, 0 ^ -
2
=> _^< --0<^, --0 * 0
2 2 2 2
TC.^fTCTi:] J^A A
=> ---- 0 I e - - , - and - - 0 * 0.
2 2 2 2
Now, sec 1 x = 0
x = sec 0
x = cosec --e
2

low
( 7C i TC 71 ,71 ^ «
=> cosec -1* = *-0 .■|---- 0|e — ,— and,----- 0*0
2 2 2 2 2
n
=> 0 + cosec 1 x = ...(h)
2

ee
From (i) and (ii), we get
rF
Fr
sec — 1 x + cosec -1 x = —
rc
2 for
ILLUSTRATIVE EXAMPLES
ou
ks

LEVEL-1
oo

EXAMPLE 1 Find the value of cot (tan 1 fl + cot 1 a). [CBSE 2012, NCERT]
Y
B

SOLUTION We know that tan-1 a + cot-1 a = —.


re

2
cot (tan-1 a + cot-1 a) = cot ^ =0
ou
ad
Y

EXAMPLE 2 //-l <x,y<l such that sin-^ x + sin-^ i/ ~ ^ ,find the value of cos-1 x + cos-^ y.
nd
Re

[NCERT EXEMPLAR]
SOLUTION We have,
Fi

■ -1 x+sm
• -1 %
sm y=^
2
n -1 X K -1y K • -1 x + cos -1 x = —
re , . _i _i n
=> ----- COS +------ COS =- v sm and sm y + cos y = —
2 2 2 2 2

=> / -1 x + cos -I
7T-(cOS n71
y)= —
2
=> -1 -1 7T
cos * x + cos y = —
J 2
EXAMPLE 3 If tan-1 x + tan-1 y = ^-,find cot-1 x + cot-1 y. [NCERT EXEMPLAR]

SOLUTION We have,
tan-1 x +tan y=-r
5
4 71 n L -i -i n
tan -1 x + cot -1 x = -, tan y + cot y = —
71
=> K cot4-1
— X + --cot -1 y = —
2 2 5 2' 2

L
ReadYourFlow.COM
INVERSE TRIGONOMETRIC FUNCTIONS 4.61

471
=> tc—(cot 1x + cot 1y)=-^

,-l 4ti
cot X + cot 1/ = 7X------
* 5
t-1 ,-1 71
=> cot X + cot V=—
^ 5
EXAMPLE 4 If tan^ x-coC1 x = tan-1-^=, find the value of x. [NCERT EXEMPLAR]
v3
SOLUTION We have.
tan 1 x - cot 1 x = tan 1
V3
, -1 K
=> tan x - cot x =— -(i)
6

low
We know that
tan 1x + cot 1x = - ...(ii)
2
Adding (i) and (ii), we obtain

ee
rj , —1 271
2 tan x =—
rF
Fr
3
=> tan 1 x = — => x = tan — = V3
3 3
or
EXAMPLE 5 If sin ^cos + sin 1 x j = l, find the value of x.
sf
ou
k

SOLUTION We have,
oo

sin {cos-1 — + sin-1 x) = 1


Y
B

13
re

-1 5 + sin. -1 K
=> cos x =—
13 2
ou
ad

. -1 71 -1 5 ■ -1 5 _1 5 _ 7T
sin x = — cos v sm — + cos
Y

2 13 13 13 _2
-l5
d

sin_1x = sin
Re

=>
n

13
Fi

5
x=—
13
EXAMPLE 6 If sin ^sin 1 ^ + cos -l x j = 1, then find the value of x. [NCERT, CBSE 2014]

SOLUTION We have,
sin f sin _ 1 - + cos- 1 x ) = 1
l 5 )
sin- 1 3 + cos- 1 x = sin- 1 1
5
sin-1 -1 + COS - 1 X = —7t
=>
5 :2
=> COS “1 x =— - sin- 1 -
2 5
-ll 1 + cos _1 1 =7l
=> cos -1 x = cos •/ sm -1 -
5 5 5 2
1
=> X =—
5

ReadYourFlow.COM
4.62 MATHEMATICS-XII

EXAMPLE 7 // COS ^ sin “ 1 ^ + COS 1 .r j = 0,find the value ofx. [NCERT EXEMPLAR]

SOLUTION We have.
cos sin 1- + cos 1xl = 0
5
. _l 2 _i _i „
=> sin - + cos x = cos 0
5
. _1 2 _1 TC
=> sin — + cos x = —
5 2
-1 ^ :_-l2
=> COS x = — sin
2 5
_1 2 _ 71
cos 1 x = cos -1 2
■ -1 2
=> v sin — + cos
5 5 5 ~2
2

low
=> x ——
5
EXAMPLES Evaluate: cos (2 cos 1 x + sin 1 x) at x

SOLUTION We have,

ree
cos (2 cos- 1 x + sin- 1 x)
rF
F
= cos (cos- 1 x + cos- 1 x + sin- 1 x)
-1 . -1 71
for
cos cos 11 X + - v cos x + sin x ——
2 2
u
= - sin (cos 1 x) = - sin j^sin 1 -^/l -x2 j = --Jl -x2
ks
Yo
oo

At x = —-
eB

5
24
cos 2cos ^x + sin 1x
r

25
ou
ad

5 k2
EXAMPLE 9 If (tan-1 x)2 + (cot-1 x)2 =
Y

then find x. [CBSE 2015]


8 '
SOLUTION We have,
nd
Re

5tt2
(tan -1 x)2 + (cot-1 x)2 =
Fi

8
5ti2
=> (tan 1 x)2 + (cot 1 x)2 + 2 tan 1 x cot 1 x - 2 tan 1 x cot 1 x =
8
=> (tan-1 x + cot-1 x)2 -2 tan-1 x cot-1 x = Sir
8
7T2
5ti2 , -1 x + cott-l x - —
11
=> - 2 tan 1 x — - tan 1 x = •: tan
4 2 8 2
712
5ti2
=> --Titan 1x + 2(tan 1x)2 =
8
o2
=> 2 (tan-1 x)2 -7i tan-1 x—— - 0

=> 16(tan-1x)2-87i(tan-1x)-37i2 =0
=> 16 (tan-1x)2 -12ti tan-1x + 47i tan-1 x-3ti2 = 0
=> 4 tan-1x (4 tan-1x - 3ti) + ti (4 tan-1x - 3tc) = 0

ReadYourFlow.COM
INVERSE TRIGONOMETRIC FUNCTIONS 4.63

=> (4 tan ‘x-3tc) (4 tan 1x + 7r)=0


_1 3tc tan-1x + -l = 0
=> 16 tan x------
4 4
n n -1 37T „
=> tan 1 x + — = 0 — < tan -1‘ x < — tan x------^ 0
4 2 2 4
, -1 11
=> tan x=— => x = ■tan —TC
x = —1.
4 4
EXAMPLE 10 Pnwe that tan (cot-1 x) = cot (tan -1 x).
State with the reason weather the equality is valid for all values ofx. [NCERT EXEMPLAR]
SOLUTION We know that tan-1x + cot-1x = - or, cot-1x = —-tan-1xfor allx e R.
2 2
tan (cot 1 x) = tan ^ - tan 1 x j for all x e R

low
= cot (tan 1 x) for all x e R

Clearly, the equality holds for all x e R as tan-1 x + cot-1 x = — hold for all x e R.
2

ee
LEVEL-2
rF
Fr
•1 o __i
EXAMPLE 11 Find the greatest and least values of (sin x) +(cos
for x) .
[NCERT EXEMPLAR]
SOLUTION (sin 1 x)2 + (cos 1x)2
u
ks

i2 ( -i 2
Yo

sin 1 x + COS X + 2 sin 1 x cos 1 x > - 2 sin 1 x cos 1 x


oo
B

2
re

= sin -1 X + cos
-1
X - 2 sin 1 x cos 1 x
ou
ad

7T2
k . -i n . -i 7t
------2 sin x — sin x v sin -1" x + cos -1 x = —
Y

4 2 2
2
nd

K2
Re

------ k sin 1 x + 2 sin 1 x


4
Fi

\2 K2
TC . _1 Tt
=2 sin 1 x — sin x + —
2 8
2 —2 71 2 71 2
71
• -1 X +----------
= 2^ sin 1 x -21 — sin +----
4 16 16 8
\2 712
• -1 x —K
=2 sin +—
4 16

Now,
— < sin 1 x < — for all x e [-1,1]
2 2

=> — - — < sin-1x-—for all xe [-1,1]


2 4 4 2 4

ReadYourFlow.COM
4.64 MATHEMATICS-XII

=> <(sin_1x--)< - for all x e [-1,1]


4 4 4
2 9n2
0 < sin 1 x - - <
4 16
7T2
7T . _1 Tt 2 + ^ 9rc2 + tc2
— < sin x —
16 4 16 l6~ 16
IT2 2 < 5ji2
ti , I . —i n
— <1 sin x — +
16 4 16 8
TT2 2 2 Sit2
. -1 Tt Tt
=> — <2 < sin x — +— <
8 4 16 4
Tt2 5Tt2
=> — ^(sin^x^+Ccos^x)2 <
8 4

w
Sit2 2
—1 9 —1 9
Hence, the greatest and the least values of (sin x) + (cos x) are and — respectively.
4 8
EXAMPLE 12 Find the maximum and minimum values o/(sin

Flo — 1
x)
^
+ (cos
— 1 ^
x) , where -1 < x < 1.

ee
SOLUTION Lety = (sin 1x)3+(cos 1x)3.Then,

Fr
y = (sin 1 x + cos 1 x)3 - 3 sin 1 x cos 1 x (sin 1 x + cos Xx) for
ur
Tt^3 3Tt . Tt
=> y=hr2 ------ sin 1 x - - sin 1 x v sin 1 x + cos -1 x = —
2 2 2
ks

Tt3 3Tt2
--------sin -1 x + — (sin“1 x)2
Yo

=> y =
oo

8 4 2
eB

Tt3
. - i x2 3ti2 (sin*1 x) + — - y
=> (sin x) - = 0
2 4 8
r

^Tt3
ou
ad

==> (sin 1 x)2 - — (sin 1 x) + — ^r-y = 0


Y

2 3it 8
nd

sin-1 x —
Re

=> + =0
4 16 12 3k
Fi

K ' ^-^ = 0
=> sin X------ +
4 48 3k
k^2 = 2y_^
=> sin 1 x----- •••(i)
4 3k 48
We know that
- — < sin 1 x < - for all x € [-1,1]
2 2
3k K K
=> < sin x-
4 4 _4
k^2 . 9k2
=> 0 < ( sin 1 x — < ...(h)
4 16
From (i) and (ii), we find that

3k 48 16

ReadYourFlow.COM
INVERSE TRIGONOMETRIC FUNCTIONS 4.65

n2 n2
==> -A9*2
48 16 + 48
7T3 7n3
=>
32 8
Hence, the maximum and minimum values of
_ 1 .3 -1x3 j 77r3
(sm x) + (cos x) are — and
8
A LITER (sin-1 x)3 + (cos-1 x)3

= (sin 1x + cos 1A:)|(sin 1x)2+(cos lx) ■ -1


- sm x cos -1 x

2 2
K
+ cos -1~x • -1
x + cos -1 x = —
TC

w
— —« sin 1 x -sin 1 x cos—1 x • v sm
2 2

2 2
= - i 2 sin 1 x
4
+ 2 cos -1'x

Flo
- 2 sin -1* x cos -1* x

ee
Fr
2 \2 ,2 f -1 2 2^1
= - 2 sin 1 x + 2 cos ax sin 1 x + cos -1* x sin x + cos 1x
4
for
ur
\2 ,2 n2 / ,2
n2 -1 X
ks

= — 2 sin 1 x + 2 cos 1x sin 1x COS


Yo

4 4
oo
B

,2 I2 "2
= * 3 sin-1 x + 3 cos 1x
re

4 4
ou
ad

,2 2 7t3
371
sin 1 x + COS ■'x ...(i)
Y

4 16
nd
Re

From Example 11, we have


Fi

2 2 2 .5rc2
sin-1x + cos 1x <
8 4
2 2 IS::3
3t^< 3n -1'X
=> sin 1x + COS . <
32 “ 4 ' 16

2 2
3ti3 713 < 371 I -il ti3 ^ 15ti3 ti3
=> sin 1x + COS X--------<
~32~~T6~Y' 16 16 16

713 3 3 7iz3
=> sin-1x + cos 1x <
32 8
3 3 7n3 n3
Hence, the maximum and minimum values of sin ^ x + cos -1 x are and
8 32
respectively.

ReadYourFlow.COM
4.66 MATHEMATICS-XII

EXERCISE 4.10
LEVEL-1
1. Evaluate :
_1 3 _i4 (ii) sin ^ tan 1x + tan 1 - j for x < 0
(i) cot sin —+ sec -
l 4 3
(iii) sin tan 1x + tan for .r > 0 (iv) cot tan 1fl + cot 1a [CBSE2012]

(v) cosj^sec 1 x + cosec ,|a:|>1

2. If cos-1 x + cos-1 y = ^' fiHd the value of sin _1 x + sin y.


__ *1 ^ __ -i __ -i __ -i

3. Ifsin x + sin
y= —and cos x - cos = —, find the values of x and y.
3 6

w
4. If cot f cos-1 — + sin-1 = 0, find the values of x.
Is ) '
5. If (sin 1x)2+(cos 1x)2 =
36 '
17 v? find x.

Flo
ee
6. Solve: sin | sin 1 ^ + cos 1 x ■ = 1

Fr
[CBSE2014, NCERT]

7. Solve: sin 1 x = - + cos 1 x


or
ur
6
sf
8. Solve : 4sin-1x = k-cos^x
k
Yo

9. Solve : tan -1 x + 2 cot-1 x = —


oo

3
B

10. Solve : 5 tan 1 x + 3 cot 1 x = 2tc


re

ANSWERS
1. (D 0 (ii) -1 (iii) 1 (iv) 0 (v) 0
ou
ad

3n V3-1 1 3 1
2. — 3. x= 4. x = — 5. x = —
Y

4 2V2 'y V2 5 2
1 73 1
nd
Re

6. x = - 7. x = -— 8. x = — 9. x =73 10. 1
5 2 2
Fi

HINTS TO NCERT & SELECTED PROBLEMS


6. We have.
sin f sin 1 - + cos Xx I =1
5
. -11 -i n _i Ti . _i 1 _i 1
=> sin - + cos x = — => cos x = — sin - => cos x = cos -ll- => X=-
5 2 2 5 5 5
7. We have.
• -l x = —
sin 71 + cos-1 x
6
• -1 -1 71
=> sin x =— x - cos
6
=> sin"1x-^-sin~1xj=^ • -1 x + cos-1 x = —71
v sin
2

=>
^ . -1
2sin
2ti
x =—=> sin
. _1 71
x = — => x =-—
73
3 3 2

ReadYourFlow.COM
INVERSE TRIGONOMETRIC FUNCTIONS 4.67

4.4.6 PROPERTY-VI
PROPERTY VI Prove that:

tan -i x + y , if xy < 1
1 -xy
-1 x+y
(i) tan -1 x + tan -1 y = n + tan , if x > 0, y > 0 and xy > 1
J-xy)
- ti + tan
if x + y if x < 0, y < 0 and xy > 1
l1 -xy)'
tan -1 x-y , if xy > -1
1 + xyj

tan 1 x - tan 1 y = • n + tan -1 ( x -y


(ii) if x > 0, y < 0 and xy < -1

w
l1 /+ *yj \
-1 x-y
- jt + tan if x < 0, y > 0 and xy < -1

PROOF (i) Let tan 1 x = A and tan -1 y = B. Then,


Flo
,1 + xy, '

ee
Fr
x = tan A and y = tan B and A, B e(-u/2 , rc/2).
tan A + tan 6 x+y
tan (A + B) =
for
ur
1 - tan A tan B 1 - xy
Now, the following cases arise.
ks
Yo

CASE I When x > 0, y > 0 and xy <1


oo

In this case, we have


eB

x > 0, y > 0 and xy <1


x+y
=> >0
r

1 - xy
ou
ad

=> tan (A + B) > 0 [Using (i)]


Y

=> A + B lies either in I quadrant or in III quadrant


nd
Re

yx>0=>0<A<-
=> 0 <A + B < — 2 =>0<A + B<7i
Fi

2 y>0=>0<B< n/2
= x+y
tan (A + B) [From (i)]
1 - xy
-1 f x + y
=> A + B = tan v 0 <A + B <—
J-xy, 2

=> tan- 1 x + tan- 1 y = tan -1 x+y


1 - xy
CASE II When x < 0, y < 0 and xy < 1
In this case, we have
x < 0, y < 0 and xy < 1
x+y
=> < 0
1 - xy
=> tan (A + B) < 0 [From (i)]

ReadYourFlow.COM
4.68 MATHEMATICS-XII

A + B lies in II quadrant or in IV quadrant.


v x< 0=>-7i/2</I<0l
=> A + B lies in IV quadrant y< 0 =>-7r/2<B<0J=> - k<A + B <0

<A + B < 0
2
x+y [From (i)]
tan {A + B) =
1 -xy
x+y
=> A + B = tan -i
l1 -xy)
tan 1 x + tan 1 y = tan -i * + y

CASE III When x > 0 and y < 0 or x <0 and y > 0

w
In this case, we have
x > 0 and y < 0
=> ^e(0,7i/2) and B e(-tt/2, 0)

Flo
ee
A + B e(- n/2 , n/2)
x+y

Fr
tan (A + B) = [From (i)]
1 -xy
or
ur
-1 x+y
=> A + B = tan
sf
i-xy
k

-l x+y
Yo

=> tan -1 x + tan- 1 y = tan


oo
B

Similarly, if x < 0 and y > 0, we have


re

tan- 1 x + tan- 1 y = tan -if x+y


ou
ad

V-xy)
Y

It follows from the above three cases that


tan- 1 x + tan- 1 y = tan -1 x + y , ifxycl.
nd
Re

1 - xy
Fi

CASE IV When x > 0, y > 0 and xy > 1


In this case, we have
x > 0, y > 0 and xy > 1
x + y <o
=>
1 -xy
x+y
=> tan (A + B) <0 From (i), tan (A + B) =
1 - xy
=> A + B lies either in II quadrant or in IV quadrant
=> A + B lies in II quadrant [v x > 0, y > 0 => A, B e (-tc/2, 0) => A + B e (- 0, rc)]

=> -< A + B < n


2

■^-n < (A + B) — 7i < 0

ReadYourFlow.COM
INVERSE TRIGONOMETRIC FUNCTIONS 4.69

=> — < (A+ B)-n<0

x+y
tan (A + B) = [From (i)]
1 - xy
x+y
=> - tan {tc - (A + B)} = [v tan {k - (/t + B)} = - tan {A + B)]
1 - xy
= x+y
=> tan {(^4 + B) - tt}
1 -xy
x + y'
A + B - % = tan 1

-1 x+y
=> A + B = 71 + tan
,^-xy

w
tan 1 x + tan 1 y = n + tan 1 x+y

CASEY When x <0,y <0 and xy > 1:


Flo
ee
In this case, we have

Fr
x < 0, y < 0 and xy > 1
x+y
=> > 0
for
ur
1 - xy
x+y
=>
ks

tan (A + B) > 0 From (i), tan (A + B) =


1 - xy
Yo
oo

=> A + B lies either in I quadrant or III quadrant


B

A + B lies in III quadrant [•.• x < 0, y < 0 => A, B <s (- tc/2,0) => A + B e (- tt, 0)]
re

—n <A + B< ——
ou

2
ad
Y

=> n-n < k+ (A + B)<k-^


d
Re

=> 0<n + (A + B)<-


n
Fi

Now, x+y
tan (A + B) = [From (i)]
1 - xy
= x+y
=> tan (7t + A + B) [•.• tan (tt + 0) = tan 0]
1 - xy
/ \
-1 x + y
=> TT + A + B = tan

-if x + y
=> A + B = - 7i + tan

=> tan- 1 x + tan- 1 y = - tt + tan -1 x+y

(ii) Let tan-!x = A and tan-1y = B. Then,


=> x = tan A, y = tan B and A, B e (- tc/2 , k/2)

ReadYourFlow.COM
4.70 MATHEMATICS-XII

tan A - tan B
tan (A -B) =
1 + tan A tan B
x-y
tan (A -B) =
1+xy
CASE I When xy >-\
If x > 0 and y > 0, then
A e (0 , rc/2), B e (0, n/2)
=> a-bJ--.-
2'2
tan (A - B) = -y-y [From (i)]
1+xy
x-y
=> A-B = tan" 1

low
1 + xy

=> tan 1 x + tan -1 y = tan 1 x~y


! + xyj

ee
tan 1 x - tan 1 y = tan -i x-y
=>
rF for all x, y with xy > -1.

Fr
1 + xy
CASE II When x > 0, y < 0 and xy < -1:
for
In this case, we have
ou
x > 0, y < 0
ks

=> A 6(0,71/2), Be (-ti/2, 0)


oo

=> A e (0 , tc/2), - B e (0 , tt/2)


Y
B

=> A - B e (0 , ti)
re

Again, x > 0, y < 0 and xy < -1


ou
ad

=> x>0,-y>0 and 1 + xy < 0


Y

=> x-y >0 and 1 + xy < 0


x-y
< 0
nd

=>
Re

1 + xy
Fi

tan (A-B) < 0


=> A-B e(7t/2, ti) [v A-B e(0,7:)]
=> —<A -B < ti
2
=> < (A-B)-rc < 0

tan (A-B) = x-y [From (i)]


1 + xy

=> -tan [t:-(A-B)} = x-y


1 + xy

=> tan {(A - B) - tt] = x-y


1 + xy
-1 x-y
=> (A - B) - t: = tan
1+xy

ReadYourFlow.COM
INVERSE TRIGONOMETRIC FUNCTIONS 4.71

=> A - B = n+ tan -i
1 + xy)

=> tan 1 x - tan 1 y = n + tan -i *-y


1+xi/J

CASE m When x < 0, y > 0 and xy <-l


In this case, we have
x <0, y> 0 and xy < -1
=> x - y < 0 and 1 + xy < 0

=> x~y > 0


1 + xy

w
=> tan (A -E) >0 [From (i)]
=> (A - B) lies either in I quadrant or in III quadrant

=t> -n < A -B <- —


2
Flo
[•.• x < 0, y > 0 => A e(- 71/2, 0), B e (0,7t/2) => - 7T < A - B < 0]

ee
Fr
=> 0<rc + (A-B) < ^

x-y
for
ur
tan (A - B) =
1 + xy
ks

x-y
=> tan (ti + (A - B)) =
Yo

1 + xy
oo
B

x-y
k+ A - B = tan -1
re

l1 + *yj
ou
ad

x-y
A - B = - 7r + tan -1
Y

x-y
d

tan 1 x - tan 1 y = -n + tan -1


Re

=>
n

1 +xyj
Fi

REMARK If x^, x2, x3,xll e R, then

tan -1 x1 + tan- 1 x2 + ... + tan- 1 x,, = tan -1


fs1-s3 + %-s7 +...
1 -^ + S4 -S6 + ...

where Sk denotes the sum of the products ofx^, x2,..., x„ taken kata time.

ILLUSTRATIVE EXAMPLES
LEVEL-1

-1 2 -1 7
EXAMPLE 1 Prove that: tan — + tan — = tan -1 1
11 24 2
SOLUTION We have,
-1 2 . -1 7
tan — + tan
11 24

ReadYourFlow.COM
4.72 MATHEMAT1C8-XII

2 7
+
= tan 1 11 24 v tan 1 x + tan 1 y = tan 1 * + ^ , If xy < 1
2 7 \}-xy)
1 - x
11 24

48 + 77 125
= tan 1 = tan 1 = tan -1 fl
264-14 250 2

EXAMPLE 2 Prove that: tan 1 2 + tan 1 3 = —


4
SOLUTION We have,
tan- 1 2 + tan- 1 3

2+3
= k + tan 1 v tan 1 x + tan 1 y = n + tan -i x+y ifxy>l
1 -2x 3 {l-xyj'

low
n 371
= 7t + tan 1 (-1) = n---- = —
4 4
EXAMPLE3 Prove that: tan-1 1 + tan" 1 2 + tan-1 3 = ti [CBSE 2010]

ee
SOLUTION
rF
tan 11 + tan 1 2 + tan 1 3 = tan 11 + (tan 1 2 + tan 1 3)

Fr
_n 3n
[See Example 2]
~4 + T
for
=n
u
ks

_ l 63
EXAMPLE 4 Prove that: sin -1 —
12
+ cos
-1 4
— + tan — =n
Yo

13 5
oo

16
SOLUTION We have,
B

-1 12 _ 1 63
— + cos -1 — 4
re

sm + tan
13 5 16
ou
ad

-1 12 -l 3 -1 63 r . _il2 ^ _1 12, -l4 -1 3


= tan — + tan — + tan — v sm — = tan — & cos - = tan
5
Y

4 16 13 5 5 4
12 3 v tan 1 x + tan 1 y
—+—
nd
Re

= 7T + tan 1 5 4 _ 1 63
+ tan x+y
12 3 16 = rc + tan- 1 , if xy>l
Fi

1 -—x
5 4 1 -xyj

63 63
= 71 + tan -1 + tan 1
-16 16
_ 1 63 _1 63
= iz- tan — + tan [v tan 1 (- x) = - tan 1 x]
16 16
= 71
EXAMPLE 5 If tan -12 + tan -1 3 + 9 = n, find the value of 0.
SOLUTION We have,
tan-12 + tan-1 3 + 0 = ti
=> tan-12 + tan-1 3 = 7r-0
2+3
7T+ tan -1 = 7t —0
1 -2x 3
7i+ tan-1(-l) = 7t-0

ReadYourFlow.COM
INVERSE TRIGONOMETRIC FUNCTIONS 4.73

=> Tt-— = 71-0


4
9=^
4
EXAMPLE 6 Prove that:
(i) tan" 1 - + tan- 1 — = tan- 1 -
7 13 9
-ll -ll
(ii) tan - + tan - + tan -1 1 [CBSE 2011,2013]
2 5 8 4
n [CBSE 2012]
(iii) tan 1 — + tan 1 — - tan 1 —
4 5 19 4
-ll- + tan -ll- + tan _ll
— + tan
_i 1 n
—=— [NCERT, CBSE 2008,2010,2016]
(iv) tan

low
5 7 3 8 4
(v) cot 17 + cot ^ 8 + cot 118= cot 1 3 [NCERT EXEMPLAR, CBSE 2014]
-ll
SOLUTION (i) LHS = tan — + tan -1 1
7 13

ee
1 1
rF
Fr
-+—
-1 7 13 -1 x + y
=> LHS = tan v tan 1 x + tan 1 y = tan if xy < 1
1
1 --x
1 or 1 -xy '
7 13
sf
u
-1 20 -1 2
=> LHS = tan = tan - = R.H.S.
90 9
k
Yo
oo

-ll— + tan -ll- + tan -1 1


(ii) LHS = tan
B

2 5 8
re

LHS = I tan 1 - + tan 1 - -ll


+ tan
2 5 8
ou
ad

1 1 v tan 1 x + tan 1 y
Y

— T —
2 5 -ll
=> LHS = tan-1
1 1
+ tan -if x + y) if xy < 1
d

8 = tan
Re

1 --x -
i - J7
n

2 5
Fi

7 1
-+— -1 r 65
-1 7 9 8 • = tan = tan- 11 = — = R.H.S.
LHS = tan - + tan “1 — = tan-1
9 8 i 7 —1
1-----X 65 4
9 8
Q O
8
(iii) LHS = tan-1 — + tan-1-----tan -l
4 5 19
LHS = {tan 1 — + tan 1 - tan -1 8
4 5 19
3 3
—+—
LHS = tan-1- 4 5 -1 8
=> • - tan
1 3
1---- x — 3 19
4 5J
-1 27
=> LHS = tan - tan -1 8
11 19

ReadYourFlow.COM
4.74 MATHEMATICS-XII

27 _ 8
=> LHS = tan-1 11 19 = tan _1 ^ = tan-1! = - = R.H.S.
27 8 425 4
1 + —x
11 19 J
(iv) LHS = tan 1 ^ + tan 1 - + tan 1 — + tan -ll
5 7 3 8
=> -ll -ll + [ tan 1 — + tan 1 -
LHS = tan - + tan
5 7 3 8
1 1 ^ ^ 1 1
+
=> LHS = tan 1 5 7 + tan 1 3 8
1 1 1 1
1 --x - l--x
5 7) 3 8
=> LHS = tan -1 -----
6
h tan -1 11
17 23

w
6 11
—+—
_ 1 f 325
=> LHS = tan -1 17 23 = tan-11 = - = R.H.S.
6
1------x
17 23 ^
11
= tan

Flo325 4

ee
LHS = cos-17 + cot-18 + cot-118

Fr
(v)

= tan 1 — + tan 1 — + tan -1 1 v cot 1(x) = tan 1—, if x >0


i
for
7
ur
8 18
= I tan 1 - + tan 1 - + tan -1 1
ks

7 8 18
Yo

1 1
oo

+
= tan -1 7 8 • + tan -1 1 1 1
B

v xy = — x — < 1
i1 —x 1 -1 18 *78
re

7 8
. -1 3
= tan — + tan -1 1
ou
ad

11 18
Y

3 1
11 +18
= tan -1 3 1 ,
nd
Re

1 v xy = — x — <1
1-—X * 11 18
Fi

11 18
= tan 1 = tan 1 — = cot 1 3.
195 3
EXAMPLE 7 Simplify each of the following:
a+ bx} b
(i) tan -1 ,x <-
b - ax a
a cos x - b sin x n n a ,
(ii) tan 1 — < x < — , - tan x > -1 [NCERT]
b cos x + a sin x , 2 2 b
r 3a2 x -x3 ^ 1 x 1
(iii) tan 1 [NCERT]
fl3 - 3ax2 -Js a -f3
a
a+ bx -+x
SOLUTION (i) tan-1 = tan-1 b = tan 1 - + tan 1 x
\ b - ax / i a
1 -- x b
b

ReadYourFlow.COM
INVERSE TRIGONOMETRIC FUNCTIONS 4.75

a
- \ ( a cos x -b sin x — tan x
(ii) tan = tan -1 b = tan -1 —
a tan 1 (tan x) = tan 1 7 - x
b cos x + a sin x y b b
1 + - tan x
b
3a2x-x3>)
(iii) tan 1
a3 - 3ax2

3 * x 3
a a 1 ( 3x -x3
= tan 1 = 3 tan -1 * v tan = 3 tan 1 x
xf a 1 - 3x2
1-3
a

1 -X y~x
EXAMPLE 9 Prove that: tan 1 - tan -11----
-y • -1
- = sin

w
1+X i +y yjl + X2 y/l+y2

SOLUTION We have,
LHS = tan -1
1 -x
- tan -1 i-y
Flo
ee
1+x i+y

Fr
=> LHS = (tan- 11 - tan- 1 x) - (tan- 11 - tan- 1 y)

=> LHS = tan -1 y - tan- 1 x


or
ur
sf
=> LHS = tan 1 y-
l1 + y^J
k
Yo
oo

LHS = sin-1 y~x = sin — 1


y-x = RHS.
=>
B

>/(i + yx)2 + (y -x)2 ^/(1 + x2) (1 + y2)


re

EXAMPLE 10 Ifa>b>c>Q, prove that


ou
ad

ab + 1 + cot 1 be + 1 Cfl +1
cot 1 + cot l
Y

=n
a -b \ b -c c -a
nd
Re

SOLUTION We know that


Fi

cot 1 x , for x > 0


tan -ifl
x -k + cot-1 x , for x < 0

tan -1 1 , for x > 0


cot 1 x = x
rc + tan ll- , for x < 0
x
It is given that a >b > c > 0. Therefore, a-b>0,b-c>0 and c-a <0.
_! fab + 1 a-b be + 1 -l( b -e
cot = tan -1 , cot -l = tan
a -b l + ab b -c 1+bc
ca + 1 c-a
and. = 7i+ tan -1
c-a \ + ac
_ 1 f ab +1 be + 1 + cot 1 'ca + 1
cot + cot 1
a-b b -c c-a

ReadYourFlow.COM
4.76 MATHEMAT1CS-XII

T \
_1 a-b b-c -1 c - a
= tan + tan -1 + 7i + tan —
\ 1 + ab / 1+bc 1 + ca
= tan- 1 a - tan- 1 b + tan- 1 b - tan- 1 c + rc+ tan-1 c - tan- 1 a = n.
EXAMPLE ll Solve the following equations:
(i) tan - 1 1 + tan _ 1 x +1 K
[NCERT, CBSE 2010]
x-2 7+2 4
(ii) tan 1 2x + tan 1 3a: = — [ NCERT, CBSE 2009,2012]
4
x —1 1 2 a:-1 -1 23
(iii) tan- 1 + tan = tan
a: +1 2a:+ 1 36
SOLUTION (i) We have,

w
1 x-1 _ 1 a: + 1 n
tan + tan
a: - 2 a: + 2 4

tan
l a:-1
+ tan
a:
_ 1 a: + 1
-2
= tan -11
a: +2
Flo
ee
1 a: -1
= tan 11 - tan _ 1 a: + 1

Fr
=> tan
a: -2 x+2
or
ur
x+1
1-
sf
a: -1 a: + 2
=> tan -1 p tan 1
x-2 x+1
k

1+
Yo

x+2
oo

_l x+ 2-x-1
B

tan -1 x-1 = tan


re

x-2 a: + 2 + a: +1
1 a: -1
= tan -1 1
ou

=> tan
ad

A--2 2a: + 3
Y

a: -1 1
=>
a: -2 2a: + 3
nd
Re

(2a: + 3) (a: -1) = a: -2 => 2x2 + a: - 3 = a: - 2 =t> 2a:2 -1 =0 => x = ± 4=


Fi

=>
V2
(ii) We have.
tan 1 2x + tan 1 3x = -
4
2x + 3x
=> tan 1 = tan 11, if 6x2 <1
1 - 2x x 3x
5x
=> = 1, if 6x2 <1
1 -6x2

=> 6x2 + 5x -1 = 0 and x2 < -


6
1 1
=> (6x -1) (x + 1) = 0 and - —f= < x < -j=
V6 v6
1 1 1
=> x = -1, — and --j= <x<-f=
6 v6 -J6

ReadYourFlow.COM
INVERSE TRIGONOMETRIC FUNCTIONS 4.77

1
x =
6
(iii) We have.
l 2x-l -1 23
tan -1 x-1 + tan = tan
x+1 2x + l 36
x--[ 2x-l
-+
x+1 2x + 1 _ .f (x-l)(2x-l)
=> tan 1 • = tan -1 23
36 ' 1 (x + 1) (2x +1) < 1
x-1 2x -1
1-----
x+1 2x +1
2x2 -1 -1 23 (x -1) (2x -1)
=> tan 1 = tan and -1 <0
3x 36 (x + 1) (2x + 1)
2x2 -1 23 - 6x

w
and < 0
3x 36 (x +1) (2x + 1)
x
=> 24x2 - 23x -12 = 0 and

Flo
(x +1) (2x +1)
>0

ee
(3x -4) (8x + 3) = 0 and x e(-l, -1/2) u(0, oo)

Fr
4
=> x
3 or
ur
—1 —1 —1
EXAMPLE 12 If a,b,c>0 such that a + b + c= abc,find the value o/tan fl + tan b + tan c.
sf
SOLUTION It is given that
k
Yo

a + b + c = abc
oo

abc a b .
— = - + - +1
B

c c c
re

i i fa b
=> ab =l+\ - + -
Vc c
ou
ad

a+b
Y

=> ab -1 =
c
a+b
nd
Re

=> ab-1>0 v a,b, c>0 >0


c
Fi

=> ab >1
Now,
tan 1a + tan 1 b + tan 1 c
/ a + b + tan 1c
= rt + tan -1 [v ab > 1]
1 -ab
abc-c + tan 1c
= 7t + tan -1 [v a + b + c = abc => a + b = abc - c]
1 -ab
-c (1 - ab) + tan 1c =Ti+tan 1(-c) + tan 1 c = n - tan 1c + tan 1c = n
= iz+ tan -1
1 -ab
EXAMPLE 13 Solve the equation:
tan tJx2 + x + sin 1 -Jx2 + x +1 = ^
1 [NCERT EXEMPLAR]

SOLUTION This equation holds, if x2 + x > 0 and 0<x2+x + l<l


Now, x2 + x > 0 and 0<x2+x + l<l

ReadYourFlow.COM
r

4.78 MATHEMATICS-XII

=> a:2 + a: > 0 and x2 + x + 1 < 1 [v x2 + x + 1 > 0 for all x]


=> x2 + x > 0 and x2 + x < 0

=> x2 + x = 0 => x = 0,-1

Clearly, these two values satisfy the given equation.


Hence, x = 0, -1 are the solutions of the given equation.

LEVEL-2

EXAMPLE 14 /ffl-j, fl2/ an are *n arithmetic progression with common differenced, then evaluate
the following expression:
d ( d d d
tan l tan -l + tan -1 + tan -l tan -l
1 + fl-| ^2 1+ 1 + fl3 fl4

w
[NCERT EXEMPLAR]
SOLUTION It is given that flj, fl3/---/«,7 are in arithmetic progression with common
difference d.
d-a2-al -a3-a2 -a4 -a3 -..=an-an_1

Flo
ee
d ^ d d d
tan -1 + tan -1 + tan -1 +...+ tan -1

Fr
/. tan
J + a1a2 l + a2a3 1 + fl3 «4 ^
or
ur
= tan • tan -1 a2 ~ai + tan "if + tan -1 fl4 ~a3 +...+ tan -1 an ~an-l
f
1 + fl2 , \yl+a3a2 1 + a4a3 1 + «n-l y.
ks
Yo

= tan tan -ixa2 - tan -1ifl1 1 + 1 tan ^-tan 1 a2 I.......+...+ [ tan lan- tan ^arI_i
oo
B

= tan tan -1 an - tan -1 a^


re
ou

^ a,,-^ _{n-\)d
ad

- tan • tan -1 [:an =a1 + (n-l) d]


l + analy
Y

1+ 1+ an
EXAMPLE 15 Prove that:
d
Re

n n 2 +n
n

2m
Z tan 1 = tan -1
Fi

m=\ m4 + m2 + 2 n2 + n + 2
SOLUTION We have,
n i 2m
L.H.S. = Z tan
m=1 m4 + m2 + 2

2m
=> L.H.S. = S tan 1 •
m=l 1 + (m4 + m2 + 1)

2m
=> L.H.S. = I tan 1
m=l 1 + (m2 + l)2 -m2

2m
=> L.H.S. = I tan -1
/n = 1 1 + (m2 + m + 1) (77T2 - m + 1)
77 (m2 + m + 1) - (m2 - m +1)
=> L.H.S. = Z tan -1
777 = 1 1 + (m2 +/77 + 1) (m2 - m + 1)

ReadYourFlow.COM
INVERSE TRIGONOMETRIC FUNCTIONS 4.79

n
=> L.H.S. = I tan 1 (m2 + w + 1) - tan l{m2 - m + l)
m=1

=> L.H.S. = (tan- 1 3 - tan- 11) + (tan- 1 7 - tan- 1 3) + (tan- 113 - tan- 1 7)

+ + ( tan -1 («2 +« + !)- tan- 1 (n2 - n + Y)

L.H.S. = tan 1 (n2 + n +1) - tan 11

n2 + n + l -1 n2 + n
=> L.H.S. = tan -1 = tan 1 = RHS
l+{n2 +ti + l) n2 +n +2 ^

EXAMPLE 16 Sum the folloiving series to infinity :


1 1 1
tan - 1 y + tan 1 2 + tan 1
1+1 + 1 1+2+2 1 + 3 + 32

w
SOLUTION Let T„ be the nth term of the given series. Then,

Tn = tan -1
1
l + n + n2
Flo • = tan
-1 (n + 1) -n
1 + (n + l)n
tan 1 (n + 1) - tan 1 n

ee
Fr
Let S be the sum of the given series to infinity. Then,
oc
S= I Tn
for
ur
«= l
n
=> S = lim S Tr
ks

n -> oo r=1
Yo
oo

=> S = lim (Tj + T2 +.... + Tn)


B

n —> co
re

|(tan-1 2-tan-11)+ (tan 1 3 - tan 12) + ... + (tan 1 (n +1) - tan 1 «)|
=> S = lim
ou

> CO
ad

Tt n
tan 1 (n + 1) - tan 11 \ = tan -1 co - tan
Y

=> S = lim 1 = -
n —> 00 2 4 4
nd
Re

EXAMPLE 17 If Cj > Ofori =1, 2,...,n, prove that


Fi

-if c2 — C1 -1 ^
tan -1 ci*-y + tan + tan -1 c3 ~ c2 + + tan -1 1 = tan
I l + c2 cr A +c3 c2 cn y
,c\y + x/
SOLUTION We have.
X 1 1 1 ^ 1 1 ^
cn-l -1 1
LHS = tan -1 y + tan -1 c\ c2 +....+ tan -1 + tan
1
1 + ^.1 i + U- 1+ Cn

y ci C1 c2 Cn-lCfi

-1 ----
* tan -1 I -i 1 -1 1 + tan 1 — - tan -1 1 +
=> LHS = tan + tan 1----- tan —
V cl) cl c2j c2 c3j

1
+ tan -1 - tan -1 1 + tan -1 1
^-1 Cn cn
/

=> LHS = tan-1— = R.H.S.


y

ReadYourFlow.COM
4.80 MATHEMATIC8-XII

EXAMPLE 17 Prove that tan " 1 ^ + tan - \ —


zx L -i xy
+ tan
n , ^ , ,
where x,y,z>Q such that
xr zr 2'
2 2 2 2
* +y +Zz =r
SOLUTION We have, x2 +y2 + z2 = r 2
yz zx z2 z2
Also, — X —=-y <1
xr yr r2 2 2 2

_ 1 yz t -i xz
tan — + tan — + tan -1 xy
xr yr zr
yz + xz
= tan- 1 xr K -i xy
■ + tan
t yz zx zr
xr yr

w
z(x2 +y2)
xyr -1 [z(x2 + y2):c r2
= tan-1 + tan -1 xy = tan • + tan -1
1
z2
r2
Flo
zr xyr 2
x +y
2

ee
Fr
-1 zr + tan-1 ( — | = COt_1 f — 1+ tan-1 f ^ ] = —
= tan
yxy zr zr zr 2
for
ur
a+b + c
EXAMPLE 18 If a, b, c >0and s = , prove that
2
ks

tan -1 -— + tan -1 1----


2bs
+ tan -1
Yo

=n
oo

be ca
B

SOLUTION We find that


re

4flfcs2 2s a+b+c
abc2 c c
ou
ad

=> a +b a+b
Y

>0
c
nd
Re

---- + tan -1
be
Fi

= n+ tan -1 + tan -1

= n+ tan -l + tan -l

= + tan -1 + tan -1
a+b+c

= 7i + tan + tan -1 = 7i-tan -1 + tan -1 = 71

ReadYourFlow.COM
INVERSE TRIGONOMETRIC FUNCTIONS 4.81

EXAMPLE 19 Evaluate:
3 sin 2 a 1
tan 1 + tan -1 — tan a , zvhere — < a < -.
5+3 cos 2 a ^ 4 2 2

1 - tan2 a 2 tan a
SOLUTION Using cos 2a = and, sin 2a = , we obtain
1 + tan2 a 1 + tan2 a

3 sin 2a + tan 1 f - tan a


tan -1
5 + 3 cos 2a 4

6 tan a 1
= tan"1 + tan -1 - tan a
8 + 2 tan2 a 4
/
3 tan a 1
= tan"1 + tan -1 — tan a

w
4 + tan2 a 4

3 tan a 1

= tan 1
---------- — + - tan a
4 + tan^ a 4

Flo
ee
3 tan2 a

Fr
1-
16 + 4 tan2 a
or
ur
(16 + tan2 a) tan a
= tan -1 2 = tan -1 (tan a) = a [v - tc/ 2 < a < rc/2]
sf
16 + tan a
k
Yo

example 20 Prove that:


oo

r, -r TC . Tl
0 , if — <A < —
tan 1 ^ tan 2A j + tan 1 (cot A) + tan 1 (cot3 A) =
B

4 2
re

n , if 0 < A < k/4

SOLUTION We know that


ou
ad

tan -iPLty
Y

[l-*yj' if xy < 1
tan 1 x + tan 1 y = / \
nd
Re

-if * + y if xy > 1
tc + tan
Fi

tc . TC
Also, cot A >1, if 0 < A <— 0<cotA<l, if — < A < -
and,
4 4 2
/ ^ A
_ i cot A + cot A •r —
71 < A
^ <-
71
tan , if
1 - cot4 A / 4 2
tan -1 (cot A) + tan 1 (cot3 A) =
l f cot A + cot3 A
tc + tan , if 0<A <-
1 - cot4 A 4

cot A TC . TC
tan 1 , if - < A < -
1 - cot2 A 4 2

cot A
tc + tan 1 , if 0 < A < —
1 - cot2 A 4

ReadYourFlow.COM
4.82 MATHEMATICS-XII

1
tan -1 — tan 2 A
. ^ 7t A TC

2 4 2
n + tan- 1 “ tan r if 0 < A <-^

-ifl . £ TC . TC
- tan — tan 2A , if- < A <-
2 4 4
Ifl
n - tan - tan 2A , if 0 <M
2
^tan 2i4 j on both sides, we get
Adding tan -1

tan -1 tan 2A j + tan- 1 (cot A) + tan- 1 (cot3 A) = ’ 0 , if tc/ 4 < A <k/2


7i/ifO<A<7t/4

w
EXERCISE 4.11
LEVEL-1
1. Prove the following results:
(i) tan-1-+ tan-1—= tan-1 ^
Flo .... . _i 12 _i 4 , _i 63

ee
(n) sin — + cos — + tan — = n
7 13 9 13 5 16

Fr
(iii) tan -1 - + tan -1 — = sin -1 * [NCERT EXEMPLAR]
4 9 V5 or
ur
2. Find the value of tan -1 * -tan -i *-y
f
[CBSE2011]
\y)
ks
Yo

3. Solve the following equations for x :


oo

(i) tan- 1 2x + tan- 1 3x = «7t + -----


B

4
re

(ii) tan 1 (x + 1) + tan 1 (x -1) = tan -1 8


31
ou
ad

(iii) tan 1 (x -1) + tan 1 x + tan 1 (x + 1) = tan 1 3x


Y

1 —x
(iv) tan 1 tan- 1 x = 0, where x > 0 [NCERT, CBSE 2008, 2010,2011]
1+x 2
d
Re
n

(v) cot 1 x - cot 1 (x + 2) = — , where x > 0


Fi

12
(vi) tan 1 (x + 2) + tan 1(x-2)=tan x>0 [CBSE 2010]

(vii) tan- 1 — + tan- ! — = -,0<x<V6 [CBSE 2010C]


2 3 4
-1 x-2 x+2
+ tan -1
K
(viii) tan [CBSE 2014]
^x-4y Va: + 47 4
(ix) tan-1(2 + x) + tan-1(2-x) =tan-1 —, where x <-V3 or, x >V3
3
1 x-2 _1 x + 2 _ 71
(x) tan + tan [CBSE 2016]
x—1 x +1 ~ 4
LEVEL-2
4. Sum the following series :
, -i 1 . _i 2 , _i 4 2«-l
tan — + tan - + tan —+...+ tan -1
3 9 33 1 + 2 2n-1

ReadYourFlow.COM
INVERSE TRIGONOMETRIC FUNCTIONS 4.83

ANSWERS
1 1 1
3- (i) A (ii) t (iii) 0, ± - (iv) -J= (v) V3 (vi)
6 4 2 a/3 4
7
(vii) 1 (viii) ± a/2 (ix) ± 3 (x) ± 4. tan-12” - —
2 4
HINTS TO NCERT& SELECTED PROBLEMS
2. tan 1--tan -i
y x+y
^ + ,tan -i y-^
, -i —
= tan
y Ay+:c
t -1 a: , i/l-A'/y^j L -i x -i. . -i x _i re
= tan — + tan 1 --------— = tan —+ tan 1 - tan — = tan 1 = —
y 1+x/yJ y y 4

w
3. (iv) We have,
-1 /1 - * -l x = 0, x > 0
tan
1 +x
tan 11 - tan 1 x - — tan- 1 x = 0
Flo
ee
2

Fr
1
=> —tan-1x = 0 => tan-1 x = — => x = tan —
4 2 6 6 a/3
(v) We have,
for
ur
1 n
tan “1 — - tan-1
x x+2 12
ks

1 _ 1
Yo
oo

x x+2 n
tan 1
B

1 2
1 +
re

x (x + 2)
2
ou

71
ad

=> tan 1
x2 + 2x + 1 12
Y

2 n
=> = tan —
d
Re

x2 + 2x + 1 12
n
Fi

__________
2 _ a/3-1
=>
x2 + 2x + 1 V3 + 1
2 2 2 => (^ + 1)2 = (V3+1)2
=> => x = a/3
x2 + 2x + 1 (a/3+1)

4.4.7 PROPERTY VII


PROPERTY VII Prove that:

sin 1 jx Vi-y2 + y V1-^2} , if-l<x,y<l and x2 + y2 < l


or
-1 x + sin 1 y = if xy < 0 and x2 + y2 > 1
(i) sin
n - sin 1 jx-y/l -y2 + y a/i -x2 J , if 0 < x, y < 1 and x2 + y2 > 1

• -1
- rt - sin -y2 + ,if-l<x, y<0 and x2 + y2 > 1

ReadYourFlow.COM
4.84 MATHEMATICS-XII

sin’1 {Wl~y2 -y Vl-^2} , y <land a:2+y2 <1


or
(ii) sin 1 x - sin 1 y = if xy > 0 and x2 + y2 > 1
n - sin 1 jx ^1 -y2 - y 1, if 0 < x < 1, -1 < y < 0 and x2 + y2 > 1

- k - sin 1 {* V1 -y2 “ 3/ -Jl-x2 |/if-l<x<0/0<y<l and x2 + y2 > 1

PROOF Let sin-1 x = /i and sin" 1 y = B. Then,

x = sin A., y = sin B and A, B e[-7c/2 , n/2]


=> cos A = -Jl -x2 , cos B = -y2 [■•’ A, B e [- it/2 , ti/2] /. cos A, cos B < 0]

low
sin (A + B) = sin A cos B + cos A sin B
=> sin (A + B) = x i/l -y2 + y ^/l -x2, ...(i)
sin (A - B) = x -y2 -y-^/l -x2,

ee
...(ii)
rF
Fr
cos (A + B) = ^\-x2 jl^y2 -•^y —(iii)
cos (A - B) = ^/l -x2 ^/l - y2 + xy
for
and. ...(iv)
u
CASE I When -1 < x, y < 1 and x2 + y2 < 1
ks
Yo

In this case, we have


oo

x2 +y2 <1
B

=> 1 - x2 > y2 and 1 - y2 > x2


re

=> (l-x2)(l-y2)>x2 y2
ou
ad

a/1-*2 ^-y2
Y

=> yl - r2 yi-y2 - xy > 0


nd
Re

=> cos (A + B) > 0


Fi

[Using (iii)]
=> A + B lies either in I quadrant or in IV quadrant
=> A + B e[-tc/2 , ti/2] [v A, B e [- rc/2 , tc/2] => -tc<A+B<tc]
sin (A + B) = x yjl -y2 + y -x2 [From (i)]
=> A + B = sin -1 -y2 + y -Jl -x2 <A + B< —
2 2
=> sin 1 x + sin 1 y = sin
1 {xfi -y2 + yV1-^2}
CASE n When xy <0 and x2 + y2 > 1:
In this case, we have
xy < 0
=> x > 0 and y < 0 or x < 0 and y > 0
=> [A e(0, tc/2] and Be[-Tc/2, 0)} or • A e and }

ReadYourFlow.COM
INVERSE TRIGONOMETRIC FUNCTIONS 4.85

-~<A+B<- -(v)
2 2
and. x1 +y2 >1
=> 1-x2 <y2 and l-y2<x2
(1 -x2) (1 -y2) < x2y2

(Jl-x2 -Jl -y2)2 < (\xy\)2 [••• xy < 0]


-\xy\<Jl-x2 yjl-y2 <\xy\
xy<^l-x2 Jl-y2 <-xy [v xy<0 | xy\=-xy]

=> JT^Jl^f-xy > 0


cos (A + B) >0

low
=> A + B lies either in I quadrant or in IV quadrant
=> A + B e[- n/2 , n/2] [Using (v)]
sin {A + B) = x yjl -y2 + y -Jl- x2

ee
=> A + B = sin [•/ A + B e[- n/2, n/2]]
rF
Fr
=> sin 1 x + sin 1 i/ = sin -1 {x^/l -y2 + for
CASE 111 When 0 <x,y<l and x2 +y2 >1
ou
ks

In this case, we have


oo

0 < x, y < 1
Y

=> A e (0 , ti/2] and B e (0 , ti/2]


B

A + B e (0 , ti] ...(vi)
re

and. xz + y^ >1
ou
ad

=> 1 - x2 < y2 and 1 - y2 < x2


Y

(l-Ar2)(l-y2) < x2 y2
nd
Re

=> J-I-X2 yjl-y2 < xy [•.• xy > 0]


Fi

=> yjl-x2 yjl-y2 -xy < 0


=> cos (A + B) <0 [Using (iii)]
=> A + B lies either in II quadrant or in III quadrant
=> -<A+B<n [•.• A + B (0 , it , from (vi)]
2
=> -7T<-(A + B)<-^

0<ti-(A + B)^^

sin (A + B) = x yjl-y2 + y y/l-x2 [From (i)]

=> sin (tt - (A + B)) = x y/l -y2 + y yjl -x2 [•.• sin (tt - 0) = sin 0]

=> n-(A + B) = sin- 1 {*^/^:y2+y>/lV?}

ReadYourFlow.COM
r

4.86 MATHEMATICS-XII

=> A + B = n-sm -y2 + y^}


sin 1 x + sin 1 y = n- sin -1

CASE IV When -1 <x ,y <0 and x2 +y2 > 1:


In this case, we have
-1<*,y <0
=> A e[~ k/2,0) and B g[- k/2,0)
=> A + B e[-n ,0) —(vii)
and, x2 +y2 >1
=> 1-x2 <y2 and 1 -y2 <x2

low
(1-x2) (1 -y2) <x2 y2

=> -Jl - x2 -y2 < xy [••• xy > o]

=> -Jl -x2 -Jl -y2 -xy <0

ee
=> cos (A + B) <0
rF [Using (hi)]

Fr
=> A + B lies either in II quadrant or in III quadrant
for
=> -k<A+B<— [Using (vii)]
2
ou
^<-(A + B)<n
ks
oo

=> -^<-k-(A + B)<0


Y
B
re

sin (A + B) = x -Jl -y2 + y yjl - x2


ou

- sin [7t + (A + B)} = x yjl-y2 + y -x2


ad

=>
Y

sin {- n - (A + B)} = x ^/l -y2 + y -x2


nd
Re

=> - k-(A + B) = sin_ 1 {xyjl-y2 +yA/l-x2j


Fi

=> A + B = -n -sin

sin 1 x + sin 1 y = - tc - sin -1

(ii) Do yourself.

ILLUSTRATIVE EXAMPLES

LEVEL-1

EXAMPLE l Prove that: sin


_1 8 . _i 3 . _i 77 -if 77
i — + sin 1 — = sin — = tan
17 5 85 36
[CBSE 2012, NCERT]
SOLUTION We have,
•_ — 1 —
sin 8 + sin -1 3
17 5

ReadYourFlow.COM
INVERSE TRIGONOMETRIC FUNCTIONS 4.87

= sin"1
8
1-
3^2 3
+— 1-
8f
17 5 5 17

= sin 1
8 4 3 15 -1 [77 -if 77
—— X 1 X sin = tan
17 5 5 17 85 36
-1 12 -1 3 _1 56
EXAMPLE 2 Prove that: cos — + sin — = sin
13 5 65
SOLUTION We have, [NCERT, CBSE 2010, 2012]
-1 12 • —1 3
cos — + sin
13 5
5 . _i 3 -1 12 . _i 5
= sin 1 — + sin — V cos — = sin —
13 5 13 13

+ — X , 1 - 5f
5 3^2 3

low
= sin — x Jl-
13 5 5 13

■ — 1 5 4 3 12 -1 56
sin ---- X — + — X ----- = sin
13 5 5 13. 65

ee
EXAMPLE 3 Prove that:
rF
Fr
... . _ i 3 . _i 8 -1 77
(i) sin — + sin — = sin [NCERT EXEMPLAR]
5 17 85
for
.... . -1 4 . _i 5 • — 1 16 71
ou
(n) sin — + sin — + sin [CBSE 2009]
5 13 65 2
ks

-1 3 8 _l 36
+ sin -1
oo

(iii) sin = cos [CBSE 2010]


5 17 85
Y
B

-1 12 _1 33 _1 56
(iv) sin - 1 — + cos
re

= cos — = sin [CBSE 2010]


5 13 65 65
ou
ad

SOLUTION Using sin 1 x ± sin 1 y = sin 1 x , we obtain


Y

-1 3 + sin 1 —
(i) sin
nd
Re

5 17
Fi

= sin 1 •
3
1-
8\2 8
+ —, 1 -
3f = sin-1
3 15 8
— X — ■+ — X —
4 • — 1 77
sin
5 17 17 5 5 17 17 5 85
. _14 . _i 5 16
(ii) sin — + sin — + sin
5 13 65
• -1 4 -1 5 _l 16
sin + sin + sin
5 13 65

-1 4 5f 5 _1 16
sin 1- + sin
5 13 13 5 65

4 12 5 3 • — 1 16
= sin 1 — X ----- + ----- X — + sin
5 13 13 5 65
. _l 63
sin + sin -1 16
65 65

ReadYourFlow.COM
4.88 MATHEMATICS-XII

_1 16 _ 1 16 _1 63 63 f
= cos — + sm v sin cos 1, 1 cos -1 16
65 65 65 65 65
n K
v sin 1 X + COS -1 X
2 2
. _1 3 . _i 8
(iii) sin — + sin —
5 17

= sin- 1 — 1- — + —Jl- 3f
8 s\2 8 3 15 8 4
sin —x —+—x—
5 17 17 5 5 17 17 5
2
• -1 —
77 = cos 1 1 36
sin 1 = cos [■.• sin 1 x = cos 1^i
85 85

w
(iv) We have,
• -1 —
sm 3 + cos -1 12
5
. _1 3
= sm — + sm —
13
. _i5

Flo cos ^x = sm 1^]

ee
5 13

Fr
3 J1 - 5 5 3\2
sm -1 -x + —X Jl- -
5 13 13 5
for
ur
_1 f 3 12 5 4 _ 1 56 56 f _1 33
cos -1
ks

sm M — x — + — X — = sin 1- = cos
15 13 13 5
Yo

65 65 65
oo

EXAMPLE 4 Solve the following equations:


B

... . _ i 3x . _ i 4x . _i
re

(i) sm — + sm — = sm x
5 5
ou
ad

(ii) sin-16x + sin-16vr3x = INCERT EXEMPLAR]


Y

2
SOLUTION (i) We have,
nd
Re

. _ 1 3x . _ 1 4x . _i
sm — + sm m — = sm x
Fi

5 5

=>

3x 16x2
5 25

=> 3x ^25 - 16x2 + 4x ^25 - 9x2 = 25x

=> x = 0 or. 3-^25 -16x2 +4-^25-9x2 = 25

Now, 3^25-16x2 + 4 -^25 - 9x2 = 25

=> 4^25-9x2 = 25 - 3 -^25 - 16x2

=> 16 (25 - 9x2) = 625 + 9 (25 - 16x2) -150 yjlB - 16x2

ReadYourFlow.COM
INVERSE TRIGONOMETRIC FUNCTIONS 4.89

=> 150^/25^16? = 450

=> 25 -16x2 = 9 => x = ±1

Hence, x = 0,1, -1 are roots of the given equation,


(ii) We have.
sin 16x + sin 16 V3x = - —
2
sin 16x = - — - sin 16^3x
2
sin (sin 1 6x) = sin ^-sin 16V3xj

6x = - sin ^ + sin ^VSxj

w
^sin-1 6V3xj
6x = - cos

=> 6x = - cos | cos-1 ^/l-(6V3x)2|


Flo sin 1 x = cos -1

ee
Fr
=> 6x = -Vl-108x2
36x2 =1 -108x2
for
ur
=> 144x2 = 1 => x — ± —
12
ks
Yo
oo

We observe that x = — does not satisfy the given equation.


12
B

1
Hence, x =-----is the only root of the given equation.
re

12
ou

EXERCISE4.12
ad
Y

LEVEL-1
nd
Re

1. Evaluate : cos f sin 1 — + sin 1 —


5 13
Fi

2. Prove the following results :


-if 63 5 3
(i) sin = sin -1 + cos -1 [CBSE 20121
65 13 5
5 _l 63
(ii) sin -1 + cos-1 —3 .
= tan [NCERT EXEMPLAR!
13 5 16
9jt 9 . -i f 1 9 .
(in)------- sin - = -sin -1 [NCERT]
8 4 3 4
3. Solve the following :
_1 x
(i) sin 1 x + sin 12x = — (ii) cos 1 x + sin -=0 [CBSE 2010]
V 3 2 6
ANSWERS
33 1 13
1. 3- (i) (ii) 1
65 2 v7

ReadYourFlow.COM
4.90 MATHEMATICS-XII

4.4.8 PROPERTY-VIII
PROPERTY VIII Prove that:
cos , ii -1 < x , y <1 and x + y >0
(i) cos 1 x + cos 1 y = <
2n - cos 1 j xy - -x2 yjl _}/2| ,if-l^x,y<l and x + y < 0

cos -1 jxy+ ^i -x2 ^1-y2} , i{ - \ <x ,y < \ and x < y


(ii) cos 1 X- cos
- cos 1 jxy +-Jl -x2 -Jl -y2 j , if-1 <y <0,0 <x<l andx>y

PROOF Let cos- 1 x = A and cos- 1 y = B. Then,


x = cos A ,y = cos B and A, B e[0 , n]

low
=> sin A = yl -x2 and sin B = -Jl -y2 sin A , sin B > 0 for A, B e [0 , tt]]
cos (A + B) = xy - yl-x2 -y2
cos (A - B) = xy + i/l -x2 -y2

ee
•••(ii)
rF
Fr
CASE I When -1 < x , y < 1 and x + y > 0:
In this case, we have for
-1 < x, y < 1
=>
ou
A, B e [0 , jr]
ks

=> 0<A + B<2n -(iii)


and, x+y>0
oo
Y

cos A + cos B > 0


eB

=> cos A > - cos B


cos A > cos (n - B)
r
ou
ad

=> A <n-B [•.• cos 0 is decreasing on [0, tc]]


=> A + B <n
Y

...(iv)
From (iii) and (iv), we get
d
Re

0 < A + B < 7i
n
Fi

cos (A + B) = xy - -x2 ^/l -y2

=> A + B = cos-1 (xy - -x2 ^/l -y2}

=t> cos- 1 x + cos- 1 y = cos 1 {xy-^jl-x2 -Jl -y2)


CASE II W/zen -1 < x , y < 0 and x + y < 0:
In this case, we have
-1<x,y <1
A , B e [0, ti]
=> 0 < A + B <2n ...(v)
and, x+y<0
cos A + cos B < 0
=> cos A < - cos B
cos A < cos (tt -B)
=> A > 7i - B [•.• cos 0 is decreasing on [0 , tt]]

ReadYourFlow.COM
INVERSE TRIGONOMETRIC FUNCTIONS 4.91

=> A + B>n •••(vi)


From (v) and (vi), we get
Ti<A + B<2n
=> -n>-{A + B)>-2n
=> tc > 2ti - (A + B) > 0
=> 0 < 271 - (A + B) < 7T
cos (A + B) = xy-yjl-x2 ^1-y2
cos {27t -(A + B)} = xy - -Jl -x2 yjl -y2

=> 27i - (A + B) = cos- 1 [xy - -Jl -x2 -y2}

=> A + B = 2k- cos -1 {xy-Jl-x2 Jl-y2}


-1
COS ~ X + cos 1y = 2n - cos- 1 (xy -^jl -x2 -Jl - y2).

low
(ii) Do yourself.

ILLUSTRATIVE EXAMPLES

ee
LEVEL-1
rF
Fr
EXAMPLE 1 Prove that: cos -1 — 12 = cos _1 33
4 + cos -1 —
5 13 65
for
SOLUTION We have, [NCERT, CBSE 2010, 2012]
-1 4 -1 12
ou
cos - + cos
ks

5 13
oo

4 12 _
= cos -1
Y

5 X 13
B
re

4 12 3 5 15_1 48 1 33
cos -1 5 X 13 5 X 13 = cos = cos
65 65 65
ou
ad

3 Q
EXAMPLE 2 Prove that: sin----- sin-1 — = cos -1 84
Y

[NCERT]
5 17 85
SOLUTION We have,
nd
Re

. _1 3 . _i 8
sm---- sm
Fi

5 17
3 1 4 . _i 8
cos
-1 ----cos
4 -1 15 v sm — = cos , sm — = cos -1 15
5 17 5 5 17 17

cos -1
4 15 L
- x — + Jl -
4f xJl- 15 f
5 17 V 5 17
4 15 3 8 _! 60 + 24 l 84
cos - 1 — X — + — X---- = cos cos
5 17 5 17 85 85 85
x2 2xy y2
EXAMPLE 3 i/cos- 1 ~ + cos- 1 - = a, prove that cos a + = sin2 a.
a b b2
SOLUTION We have. [CBSE 2016]
x

ReadYourFlow.COM
4.92 MATHEMATICS-XII

=>

— - cos a
ab
x 2 y..2
=>
ctb1
x1 | y2 2^y
=> cos a = 1 - cos 2 a
a1 b2 ab

low
x2 y2 2xy • 2 a
=> cos a sin
a2 b2 ab
EXAMPLE 3 If cos- 1 x + cos- 1 y + cos- 1 z = 7t, prove that x2 + y2 +z2 + 2xyz = 1.

ee
SOLUTION We have,
rF
Fr
cos- 1 x + cos- 1 y + cos -1 2 = 71 for
=> cos- 1 x + cos- 1 y = 71 - COS 12
u
cos- 1 x + cos- 1 y = cos [v COS 1(-2)=7t-COS
ks

=>
Yo
oo

=> COS -1 cos -1 (-z)


B
re

=> -2
ou
ad

=> (^y + z)2 = (l-x2)(l-y2)


Y

=> x2 y2 +z2 + 2xyz = 1 - x2 - y2 + x2 y'


nd
Re

=> x2 + y2 + 22 + 2xy2 = 1
Fi

EXERCISER 13

LEVEL-1

1. If cos 1 — + cos 1 — = a, then prove that 9x2 - 12xy cos a + 4y2 = 36 sin2 a.
2 3

2. Solve the equation: cos -1«— cos-1^— = cos-ll— cos -ll


x x b a
3. Solve : cos 1 V3x + cos 1 x = —
2
- „ , _i 4 -i 12 _i 33
4. Prove that: cos — + cos — = cos — [CBSE2012]
5 13 65
ANSWERS

2.x = ab 3. x = -
2

ReadYourFlow.COM
INVERSE TRIGONOMETRIC FUNCTIONS 4.93

4.4.9 PROPERTY-1 X
PROPERTY IX Prove that:
sin- 1 (2x -Jl - x2) 1 1
/ if —7= ^ x < -7=
V2 V2
(i) 2 sin 1 x = 1 (2x -x2) 1 < a: < 1
, if -^=
7i - sin [NCERT]
' -v2
- 1 (2a: yjl-x2) ,if-l<A:<- 1
- 7i - sin
V2
sin 1 (3x - 4x3)
2 2
(ii) 3 sin 1 x n - sin 1(3a:-4x3) , if ^ <x < 1

- 7i - sin - 1(3x-4x3),if-l < x < - 1

low
2
PROOF (i) Let sin 1 x = 0. Then,
x = sin 0
= yjl-x2

ee
=> cos 0 [v cos 0 > 0 for 0 e [- tc/2 , tt/2]
rF
Fr
sin 2 0 = 2 sin 0 cos 0 => sin 2 0 = 2x -Jl -x2 -(i)
1 1
CASE! When --f=<x< —
or
■Jl V2
sf
u
We have.
1 1
k
Yo

------ r= 5= X < —F= => --<0<- --<20<-


oo

V2 V2 4 4 2 2
eB

1 1 1 < 2x Jl-x2 < 1


Also, --------F=- < X < —F= =>
V2 V2
r
ou

sin 2 0 = 2x -Jl -x2


ad

[From (i)]
Y

=> 2 0 = sin
nd
Re
Fi

=> 2 sin 1 x = sin

1
CASEn When ^<x<l:
V2
We have,
-^<x<l=> -i= < sin 0 < 1 => -<6< - => -<2B<n=> -ti<-20<--=> O<k-20<-
V2 V2 4 2 2 2 2
1 < x < 1 => 0 < 2x -Jl - x2 <1
Also,
V2
sin 2 0 = 2x Jl -x2 [From (i)]

=> sin (ti - 2 0) = 2x Jl -x2

tt-20 = sin-1 f2x

ReadYourFlow.COM
4.94 MATHEMATICS-XII

=> K - 2 sin 1 ,r = sin 1

=> 2 sin 1 x = 7t - sin 1


1
CASE III When -1 < x < - -f=
V2
We have,
-l<x<-4==> -l<sin0<-4==> --<©<--=> -n<2Q<-~ => 0<n + 2Q<-
V2 V2 2 4 2 2
Also, -1 < x < - ~ => -1 < 2x Jl - x2 < 0
V2 v
sin 2 0 = 2Xyfl -x 2 [From (i)]

low
=> -sin (7i+2 0) = 2x-Jl-x2
=> sin (- n - 2 0) = 2x -Jl

=> - n - 2 0 = sin

ee
rF
Fr
=> 20 = - 7i - sin for
=> 2 sin 1 x = - 71 - sin
ou
ks

(ii) Let sin -1 x = 0. Then, x = sin 0


oo

sin 30 = 3 sin 0-4 sin 3 0


Y
B

=> sin 3 0 = 3x - 4x3


re

1 1
CASE I When -- <x <-
ou
ad

2 2
Y

We have.
— < x < — => --<sin0<—=> --<©<—=> -— < 3 0 < —
nd
Re

2 2 2 2 6 6 2 2
Fi

Also, --<x<-=> -1 < 3x - 4x3 <1


2 2
sin 3 0 = 3x -4x3
=> 30 = sin"1 (3x-4x3)
=> 3 sin- 1 x = sin- 1 (3x - 4x3)
CASEH When 1/2 < x < 1:
We have,
StT i-* 71
-<x <!=>-< sin 0<1=> — <0< —=> — <30< — => ----- <- 3 0 <-- => — <ti-30<-
2 2 6 2 2 2 2 2 2 2

Also, — <x<l=> -1 <3x-4x3 < 1


2
sin 3 0 = 3x - 4x3
=> sin(7c-3 0) = (3x - 4x3)

ReadYourFlow.COM
INVERSE TRIGONOMETRIC FUNCTIONS 4.95

K -36 = sin- 1 (3x - 4x3)


=> TC - 3 sin - 1 x = sin- 1 (3x -4x3)
=> 3sin-1x - Jr-sin-1 (3x-4x3).

CASE HI When -l <x<~-


2
We have,
-1<*<--
2
71 ^ A n
=> -1 < sin 0 < - — =>----
— < 0 <— => -— < 3 0 < - — => -—<rc + 30<9 => O<-k-0< —
2 2 6 2 2 2 2
1 1
Also, — < x < - => -1 < 3x-4;t3 <1
2 2

low
sin 3 0 = 3x - 4x3
=> -sin(7i+3 0) = 3x - 4x3 [sin (ti + 30) = - sin 30]
=> sin(-rc-3 0) = 3x-4x3

ee
=> - ti - 3 0 = sin (3x-4x3)
rF
Fr
=>-7i-3 sin 1 x = cin- 1 (3x - 4x3)
=> 3 sin - 1 x - 7t - sin- 1 (3x - 4x3)
for
ou
ILLUSTRATIVE EXAMPLES
ks

LEVEL-1
oo
Y

EXAMPLE 1 Evaluate:
eB

(i) sin (2 sin- 1 0.6) (ii) sin (2 sin 1 0.8)


r

SOLUTION (i) sin (2 sin-1 0.6)


ou
ad

1 1
sin sin 1 |2x 0.6x^l-(0.6)2 J v 2 sin 1 x = sin ^2x if- r- < X < —1=
Y

V2 V2
= sin (sin 1 0.96) = 0.96
d
Re
n

(ii) We have,
Fi

sin (2 sin -1 0.8)


sin 1 12 x 0.8 x -Jl -(0.8)2 j v 2 sin 1 x = ti - sin ^2x 1
sin 71 - if, -7= < x < 1
V2
= sin (ti - sin 1 0.96)
= sin j sin 1 (0.96) j = 0.96 [v siii (ti - 0) = sin 0]

EXAMPLE 2 Evaluate: sin (3 sin-1 0.4)


SOLUTION Using 3 sin - 1 x = sin - 1 (3x -4x3), we obtain (
sin (3 sin- 1 0.4)
= sin sin-1 J 3x 0.4-4 x (0.4) 3

|sin-1 (1.2 - 0.256) j = sin | sin-1 (0.944)J = 0.944


= sin

ReadYourFlow.COM
4.96 MATHEMATICS-XII

4.4.10 PROPERTIES X-XII


PROPERTY X Prove that
cos- 1 (2x2 -1) , if 0 < x < 1
(i) 2 cos 1 x
2n - cos- 1 (2x2 -l),if-l<x<0

cos- 1 (4x3 - 3x) , if - <x<l


2
(ii) 3 cos 1 x 2k - cos- 1 (4x3 - 3x)' if - ^x < ^
2
2n + cos- 1 (4x3 - 3x), if - l <x <-^

PROOF (i) Let cos- 1 x = 0. Then, x = cos 0


cos 2 0 = 2 cos2 0 -1 =i> cos 20 = 2x2 -1

low
CASE I When0<x<l
We have.
0 < x < 1 => 0 < cos 0 < 1 => O<0<-=> 0 < 2 0 < ji

ee
2
rF
Fr
Also, 0<x<l=> -1 <2x2 -1 <1
cos 2 0 = 2x2 -1
for
=> 20 = cos-1 (2x2 -1)
ou
ks

=> 2 cos- 1 x = cos- 1 (2x2 -1).


oo

CASEH When -1 <x<0


Y
B

We have,
re

-1 < x < 0 => -1 < cos 0 < 0 => — < 0 < k=> rt < 2 0 < 2k=> -2k<-2Q<-k => 0 <2 k-2 Q <k
2
ou
ad

Also, -1 <x<0=> -1 <2x2 -1 <1


Y

/. cos 2 0= (2x2 -1)


nd
Re

=> cos (271-2 0) = (2x2 -1)


Fi

=> 2ti - 2 0 = cos- 1 (2x2 -1)


2 0 = 271 - cos-1 (2x2 -1)
=> 2cos-1x = 2ti-cos-1 (2x2-1).
(ii) Let cos- 1 x = 0. Then, x = cos 0
cos 3 0 = 4 cos3 0-3 cos 0 => cos 30= 4x3 - 3x
1
CASE I When - <x <1
2
We have,
11
-<x<l => - < cos 0<1 => O<0< — => 0 < 3 0 < 7i
2 2 3
1 ^
Also, - <x<l=> -1 <4x - 3x <1
2
cos 3 0 = 4x3 - 3x
=>30 = cos -1 (4x3 - 3x)

ReadYourFlow.COM
INVERSE TRIGONOMETRIC FUNCTIONS 4.97

=> 3 cos 1 x = cos 1 (4x3 - 3x)

1 1
CASEn When --<x <-
2 2
We have,
2k
-I<X<U- — < cos B < — => — < 0 < —-=> 7i < 3 0 < 2 7i => -27i < - 30 < -n=> 0 <2n - 3B < n
2 2 2 2 3 3
/. cos 3 0= 4x3 - 3a:

=> cos (2 7i - 3 0) = 4a:3 - 3x

=> 2 7i - 3 0 = cos- 1 (4a:3 - 3a:)

=> 3 0 = 2 tt - cos- 1 (4a:3 - 3a:)

w
=> 3 cos- 1 a: = 2 7t - cos 1 (4a:3 - 3x)

CASEm When -l <x<--


2

Flo
ee
We have,

Fr
-1<x <--
2
or
ur
=> -1 < cos 0 < - -
2
sf
2tx
=> — < 0 < k=> 2k < 3 B < 3n => - 3n< - 3 B <-2 k => - n<2 n - 3 B <0 => 0 < 3 B -2 k <n
k
Yo

3
oo

cos 3 0 = 4a:3 - 3a:


B

=> cos (271 - 30) = 4a:3 - 3a:


re

=> cos (3 0 - 2k) = 4a:3 - 3a:


ou
ad

=> 3 B - 2 k = cos- 1 (4a:3 - 3a:)


Y

=> 3 0 = 271 + cos- 1 (4x3 - 3a:)


nd
Re

=> 3 cos- 1 x = 2tc + cos- 1 (4a:3 - 3a:).


Fi

PROPERTY XI Prove that:

2x
tan -1 , if -1 < a: <1
l-*2

2a:
(i) 2 tan 1 a: = k + tan 1 , if a: > 1
1-x2

2a:
- 7i + tan 1 , if a: <-l
1 -a: 2

ReadYourFlow.COM
4.98 MATHEMATICS-XII

1 Zx-x* ■r 1 1
tan / li “ ~r= <x < -7=
l-3x2 V3 V3
_1 3x-x 3 ^ •r 1
(ii) 3 tan 1 x = it + tan , if X > —j=
1 - 3x2 73
_1 3x - x 3 ^ 1
-n + tan , if x <—j=
v 1 - 3x2 73
PROOF (i) Let tan- 1 x = 0. Then, x = tan 0.
2 tan 0
/. tan 2 0 =
1 - tan2 0
2x

low
=> tan 2 0 =
1-x2

CASE I When -1 < x < 1


We have,

ee
— 1 < x — 1 => -1 < tan 0 < 1 => <0<—=> - — < 2 0 < —
rF
Fr
2 4 2 2
2x
/. tan 20 =
1-x2
for
ou
2x
20 = tan -1
ks

1-x2
\
oo
Y

2x
eB

=> 2 tan 1 x = tan 1


1-x2
r

CASE II When x > 1


ou
ad

We have,
Y

x >1
tan 0 > 1
d
Re
n

=> — >0> — => it > 2 0 > — => -ti<-20<- — => O<ti-20< — => -—<-ti + 20<O
Fi

2 4 2 2 2 2
2x
tan 2 0 =
1-x2
2x
=> - tan ( tc - 2 0) =
1-x2
2x
=> tan (- 7i + 2 0) =
1-x2

2x
=> - 7i + 2 0 = tan 1
1-x2

2x
=> 2 0 = 7i + tan 1
1-x2

2x
2 tan 1 x = 7i + tan 1
1-x2

ReadYourFlow.COM
INVERSE TRIGONOMETRIC FUNCTIONS 4.99

CASE III When x < -1


We have.
x < -1
tan0 < -1 => - - <0 <-—=> -ti<20 < -- => 0 <71 + 20 <-
2 4 2 2
2x
tan 2 0 =
1-x2
tan (ti+2 0) = 2x
[•.* tan (ti + a) = tan a]
1-x2

2x
=> tc + 2 0 = tan 1
2
11 "X J
2x
=> 7t + 2 tan- 1 x = tan- 1

w
1-x2
\
2x
=> 2 tan 1 x = - tc + tan 1
1-x2

Flo
ee
(ii) Let tan- 1 x = 0. Then, x = tan 0.

Fr
3 tan 0 - tan 3 0
tan 3 0 =
1-3 tan2 0 or
ur
3x - x3
sf
tan 3 0 =
1 -3x2
k
Yo

1
CASE I When -~^= <x <4=
oo

V3 V3
B

We have,
re

< x < 4= => —t= < tan 0 < -4 =>-— <0<—=> <30 < —
V3 V3 V3 6 6 2 2
ou
ad

3x -x3
tan 3 0 =
Y

1 -3x2
'Sx-x^
nd
Re

=> 3 0 = tan 1
1 - 3x2 y
Fi

l 3x-x3N|
3 tan 1 x = tan
I ! - 3x2 ,
V /
1
CASE H When x >^=
V3
We have.
1
x >

1
=> tan 0 > -p=
V3
7T A 7T
- >0 > -
2 6
71 3ti _ A n K 7C
=> - < or,
30 < — 371
=> — < - 30 < — => < 7T -39 < ^ -—<30- 7t < —
2 2 2 2 2 2 2 2
3x -x3
tan 3 0 =
1 - 3x2

ReadYourFlow.COM
4.100 MATHEMAT1CS-XII

3x-x3
=> - tan (ti - 3 0) = ['.■ tan (^ - 3 0) = - tan 3 0)]
l-3x2
3x - x3
=> tan (3 0 - k) =
l-3x2

_i 3x-x 3^1
=> 3 0 - 7i = tan
l-3x2
3x-x3
=> 3 tan 1 x - n = tan 1
l-3x2

3x -x 3 A
=> 3 tan 1 x = 7c + tan 1
l-3x2

w
1
CASEIII When x<--r=
43
We have.
x <
1
43
Flo
ee
1 <e < 30 <71+30 < -

Fr
=> tan 0 < —f=
43 2 6 2 2 2 2
3X-X3 for
ur
tan 3 0 =
l-3x2
3x-x3
ks

=> tan (ti +3 0) = [: tan (n + x) = tan x]


Yo

1 - 3x2
oo

3x -x 3 ^
B

=> 7t + 3 0 = tan 1
re

l-3x2 y
l ( 3x -x3
ou
ad

=> 71+3 tan 1 x = tan


l-3x2
Y

(3x-x3
nd

3 tan 1 x = - ti + tan 1
Re

l-3x2
Fi

PROPERTY XII Prove that:

sin”1 2x
, if -1 <x<l
1+x2

2x
(i) 2 tan 1 x tz — sin 1 , if a: >1
1 + x2

2x
-n- sin , if x < — 1
1 + x2

cos
if l-*2 , if 0 <x<oo
1 + x2
(ii) 2 tan 1 x \ /
^ 1 2 ^
- cos i 1~x
, if-oo <x <0
1 +x2y

ReadYourFlow.COM
INVERSE TRIGONOMETRIC FUNCTIONS 4.101

PROOF (i) Let tan 1 x = 0. Then, x = tan 0


2 tan 0 2x
sin 20 = => sin 2 0 =
1 + tan2 0 1 + x2
CASE I When -1 < x < 1
We have.

-1 <x<l => -1 < tan 0 < 1 =^> — — < 0 < — <2 0 < —
4 4 2 2
2x
sin 2 0 =
l + x2

2x 2x
2 0 = sin =^> 2 tan 1 x = sin 1
l + x2 l + x2

w
CASE II When x > 1
We have.

2 2
Flo
x>l=> tan 0 > 1 => - < 0 <- => — < 20 < 7t => -rc<-20<-- => O<ti-20<-
4 2 2

ee
2x

Fr
sin 2 0 =
l+x2
2x
for
ur
=> sin (rt - 2 0) =
l + x2
ks

2x
=> Ti - 2 0 = sin 1
Yo
oo

\ l + x2
B

2x 2x
=> 7r - 2 tan -1* x = sin 2 tan 1 x = 7i - sin 1
re

l + x2 l + x2
\
ou
ad

CASE III When x < -1


Y

We have.
x < -1
nd
Re

=> tan 0 < -1


n
Fi

=> - -r < 0 < - — => — 7i < 2 0 < - — => 0 < tt + 2 0 < — => < -7t-20<O
2 4 2 2 2
2x
sin 20 =
l+x2
2x 2x
-sin (ti + 2 0) = j => sin (- 71 - 2 0) =
1 +X l + x2

2x
=> - 7i - 2 0 = sin -1
l + x2

- 7i - 2 tan 1 x = sin 2x 2x
2 tan 1 x = - 7i - sin 1
IV l + x2 / l + x2

(ii) Let tan -1 x = 0. Then, x = tan 0.


1 - tan2 0 1-x2
cos 2 0 = => cos 2 0 =
1 + tan2 0 l+x2

ReadYourFlow.COM
4.102 MATHEMATICS-XN

CASE I When 0 < x < co


We have.
0<x<oo=> 0< tan 0<co=> O<0<—=> O<20<ti
2
1-x2
cos 2 0 =
1 + x2
/ 7 \

=> 2 0 = cos -1 1~x 2 => 2 tan 1 x = cos 1


ll+x-> l + x2

CASE II When - co < x < 0


We have.
- co < x < 0 => — co < tan 0 < 0 => <0<O=> - ti<20<O=> 0 < - 2 0 < rc
2

w
1-x2
cos 2 0 =
l + x2

=> cos (-2 0) =


1-x2
l + x2
Flo
ee
Fr
(l-X2)
-20 = cos -1
l + x2 for
ur
2A (l-x2)
=> - 2 tan 1 x = cos -1 1-x => 2 tan 1 x = - cos -1
l+x2 l+x2
ks

\ /
Yo
oo

ILLUSTRATIVE EXAMPLES
B

LEVEL-1
re

EXAMPLE l Prove that:


ou

3x-x3]
ad

2x 1
tan- 1 x + tan- 1 = tan -1 [NCERT, CBSE 2010]
V3
Y

1-x2 11 - 3x2 y
SOLUTION We have,
nd
Re

2x
tan-1 x + tan-1
Fi

1-x2

2x
x+
= tan 1 1-x2 -1 x+y
•: tan 1 x + tan 1 y = tan if xy < 1
2x2 i-xy)'
1-
1-x2
/ o \ / o \
x — x° + 2x 1 Sx-x'5
= tan 1 = tan
1-x2 -2x2 l\ 1 - 3x2 /
2x
ALITER LHS = tan -1‘x + tan -1|
1-x2
1 1
LHS = tan 1 x + 2 tan 1 x ------7= < X < —j=
V3 y[2>
3x-x3 \
=> LHS = 3 tan 1 x = tan -1 = RHS

ReadYourFlow.COM
INVERSE TRIGONOMETRIC FUNCTIONS 4.103

1 1 1 „ -i 31
EXAMPLE 2 Prove that: 2 tan -1 — + tan — = tan —
2 7 17
SOLUTION We have,
2 tan -1 —
1 + tan -1 1
2 7

1
2x
2x
= tan 1 ■ 2 + tan 1 1 v 2 tan 1 x = tan 1 , if -1 < x < 1
1-
if 7 1-x2
2
4 1
--- 1---
tan
-1 4
— + tan -1 1 = tan 1 • 3 7 • = tan -1 31
3 7 11----
4 x -1 17

w
3 7
EXAMPLE 3 Evaluate: tan jf tan 1 [CBSE 2013]

SOLUTION We have.
Flo
ee
1

Fr
2x
1 -1 5 5
tan 2 tan -1 = tan tan 1 5. • = tan tan
5 1 12 12
1-
for
ur
25
EXAMPLE4 Prove that: 2sin ^f-tan 1—= — [NCERT EXEMPLAR]
ks

5 31 4
Yo
oo

SOLUTION 2 sin _1 - - tan -1 17


B

5 31
re

Q
-1 17 • -l3 f -i 3
- 2 tan ---- tan v sin — = tan —
4 31 5 4
ou
ad

3
Y

2x
1 2x
= tan -1 -tan
-1 17 y 2 tan -11 x = tan -1 2 for | x | < 1
nd

S']2 31
Re

1-x
1-
4
Fi

24 17
= tan"1 —-tan 1 17 -1 7 31 = tan"1! = —
= tan
7 13 24 17 4
1 + —x —
7 31
EXAMPLE 5 Prove that:

1 x+y
tan — sin -1 2X if \ x \ <1, y > 0 and xy < 1.
2 1 + x2 1 - xy'

[NCERT, CBSE 2013]


-i i-y2
SOLUTION We know that cos = 2 tan 1 i/ for all y > 0
i + y2

and. sin -1 _^L_ = 2 tan" 1 x for all x e [-1,1]


1+x2

ReadYourFlow.COM
4.104 MATHEMATICS-XII

LHS = tan — • sin 1


2

LHS = tan - J 2 tan 1 x + 2 tan 1 y


2

=> LHS = tan tan 1 x + tan 1 y

/ \
=> LHS = tan tan i *+y
[■•• xy<\\

x+y
=> LHS = = RHS
1 -xy
5
EXAMPLE 6 Find the value of: sin < 2 cot -1 [NCERT, EXEMPLAR]

w
12
5
SOLUTION sin i 2 cot -l

-1 5
12

Flo
ee
= sin ^ 2 7T - cot [v cot 1 (-x) = 71 - cot 1 x]
12

Fr
= sin 2 7i -2 cot -1 5 [v sin (2rc -0) = - sin 0]
12
for
ur
-1 5
= - sin 2 cot
12
ks

-112
Yo

= - sin 2 tan v cot 1 x = tan 1-for x > 0


]
oo

5 x
B

12
re

2x
1 2x
= -sin - 7i-sin -1 v 2 tan 1 x = 7t - sin -1 for x > 1
ou

12 Y2 1 +x2y
ad

1+
5
Y

. _i120 _1120 120


= - sin k — sin = - sin sin
nd
Re

169 169 169


Fi

4
EXAMPLE 7 Show that: 2 tan 1 (-3) = - - + tan -1 [NCERT, EXEMPLAR]
2 3
SOLUTION LHS = 2 tan _1 (-3) = -2 tan -13 [v tan 1 (-x) = - tan 1 x]
^ 2x 3
LHS = - < 71 + tan -1 v 2 tan 1 x = 7t + tan -l ( 2x , if x > 1
vl-32 ll-x2
3
LHS = - tc - tan -1
4

=> -1 3 tan_1(-x) =-tan-1x


LHS = - K + tan y
4
TC -1 3
=> LHS = - 71 + ^ 2 ~ cot v tan 1 x = — - cot 1 x
4 2
=> LHS = - — - cot"1—
2 4

cot -1 x = tan -1
=> LHS = - — - tan -1 —
2 3 [i)forx>0]
ReadYourFlow.COM
INVERSE TRIGONOMETRIC FUNCTIONS 4.105

4
=> LHS =- —+ tan-1 = RHS
2 3
-ll -ll [NCERT EXEMPLAR]
EXAMPLES Show that: cos \ 2tan = sin 4 tan
7 3
SOLUTION We have,
-ll
LHS = cos I 2 tan
7

1-x 2 ^
=> LHS = cos ■ cos -1 v 2 tan -1* x = cos -1 for 0 < x < co
1 + x2
49
24
LHS = cos
25

w
and.
-ll -ll
RHS = sin 4 tan = sin < 2 2 tan
3
r 2x-
Y
3

Flo
ee
2x ^
=> RHS = sin 2 tan -1 3 v 2 tan 1 x = tan -1 for -1 < x < 1

Fr
1 1-x2
1--
9 or
ur
=> RHS = sin 2 tan -1 3
sf
4
3 ^
k

2x —
Yo

2x
oo

=> RHS = sin sin • -1 4 • —l


v 2 tan 1x = sin , if -1 < x < 1
9 l + x2
eB

1+ —
16 J
-1 24 24
...(ii)
r

RHS = sin sin


ou

25 25
ad

From (i) and (ii), we obtain


Y

-ll -ll
cos 2 tan = sin 4 tan
nd

7 3
Re
Fi

EXAMPLE 9 Prove that: tan 1 Vx = - cos 1 P-* x e [0,1] [NCERT, CBSE 2010]
2 \ l + x /)'
SOLUTION We have.
1
cos -1
1-x 1
cos
1 fl-(^)2l — x 2 tan 1 Vx = tan 1 Vx .
2 l+x 2 1 + (Vx)2J 2

A LITER Putting x = tan2 0, we obtain

.Yi-x 1 1 1 - tan2 0 1
RHS = -cos cos cos 1 (cos 20) = 0 = tan 1 Vx = LHS
2 \ l + x /, 2 , 1 + tan2 0 2

EXAMPLE 10 Find the value of the expression: sin ^2 tan -ll^ j + cos^tan 1 2V2 j

[NCERT EXEMPLAR]
-11
SOLUTION sin 2 tan + cos tan-1 2V2
3

ReadYourFlow.COM
4.106 MATHEMATICS-XII

1
2x
-ll 2x N
= sin ■
sm —l 3 > + cos cos v 2tan-1x = sin -1 , if -1 < x < 1
1 +I 3 , 1 + x2 ,
9
-1 3 = 3 1__14
= sin sin + cos cos -1 13j_5 + 3_15
5
3fl2 x -x3 1 x 1
EXAMPLE 11 Simplify: tan 1 [NCERT]
a3 - 3ax2 ' V3<fl<V3
f 2
l 3a x - x 3 ^
SOLUTION tan
a3 - 3ax2

X
3
3*
a a 1 T 3x - x 3 ^

w
= tan”1 • = 3 tan v tan"1 = 3 tan 1 x
2 a 1 -3x2
x
1-3
a

EXAMPLE 12 Prove the following:


Flo
ee
71
(i) 4 tan -1 —1 tan 1 -----h
1 tan -1 1

Fr
5 70 99 4
ll -l5^ 1 1 71 [CBSE 2014]
(ii) 2 tan - + sec + 2 tan
or
ur
5 7 8 4
sf
SOLUTION (i) We have,
1
4 tan - 1 — tan -1 + tan 1 1
k
Yo

5 70 99
oo

-ll -l 1
= 2 U tan - tan + tan -1 1
eB

5 70 99
2x1/5 2x
= 2 tan -1 • ► - tan - 1 —1 + L.tan - 1 —1
r

v 2 tan 1 x = tan 1 2* / if I x| < 1


ou

l-(l/5)2 70
ad

99 1 -x
Y

= 2 tan -1 5 tan 1 ------tan


1 -1 1
12 70 99
nd
Re

1 1
2x5/12 _1 120
Fi

1 29
= tan 1 • ■ - tan -1 70 99 = tan - tan
1 -(5/12)2 1 1 119 6931
1+—X
70 99
120 1
l 120 f _! 1 -1 119 239
= tan ------- tan — = tan = tan"1! = ^
119 239 120 1 4
1+ X
119 239
-ll 5V2
(ii) 2 tan - + sec -1 + 2 tan -1 1
5 7 8
-ll 5V2
= 2 \ tan - + tan -1 1 + sec -1
5 8 7
riil
-+-
5V2 A2
= 2 tan 1 < 5 8 • + tan -1 -1 v sec -1 x = tan 1
1 1 7
1 --x -
5 8
-! 13
= 2 tan + tan -1 1
39 7

ReadYourFlow.COM
INVERSE TRIGONOMETRIC FUNCTIONS 4.107

= 2 tan 1 — + tan -ll


3 7
2x1/3 2x
= tan -1 • + tan -ll v 2 tan 1 X = tan 1 , if | x\ <1
l-(l/3)2 7 l-.t2
3 1
+
_l 3 4 7 = tan"1! = ^
= tan — + tan _1- = tan"1.
4 7 3 1 4
1 --x-
4 7
EXAMPLE 13 Evaluate:
(i) tan 2 tan I_ * (ii) tan i - cos -1 V5
5 4 2 3

SOLUTION (i) tan 2 tan -1 !_^


5 4

low
1 >
2x -
5 - tan 1 1 2 tan 1 x = tan 1 2x
= tan • tan 1 , if | x | < 1
1 1-x2
1 -—
25

ee
5
rF
Fr
-1 -7
= tan tan -1 5
----- tan -1 1} = tan • tan 1 — •= tan tan-M —
12 5 17 17
1+
or
12
sf
u
V5 V5
(ii) Let cos -1 — = 0. Then, cos 0 = —
3 3
k
Yo
oo

J5
1 -—
B

fl _i V5 0 1 - cos 0 3
tan — cos — = tan
V5
re

2 3 2 1 + cos 0
1+
3
ou
ad

_____
3 - a/5 = (3-V5)2 (3-V5)2 3 - a/5
Y

^3 + ^5 ]](3 + a/5) (3 -yj5) 9-5 2


nd
Re

EXAMPLE 14 Solvefor x: 2 tan" 1 (cos x) = tan - 1 (2 cosec x).


Fi

[NCERT EXEMPLAR, CBSE 2009, 2010 C, 2014, 2016)


SOLUTION We have,
2 tan - 1 (cos x) = tan" 1 (2 cosec x)
2 cos x
=> tan -1 = tan 1 (2 cosec x)
1 - cos2 x
2 cos x
=> - = 2 cosecx => cos x = sin x => tan x = 1 => tanx = tan — => x =nii+—,n eZ
sin2 x 4 4'
EXAMPLE 15 Solve the following equations:
| cot (2 tan 1 x)j =0
(ii) sin 2 cos -1
k
(i) sin- 1 (1 - x) - 2 sin- 1 x = —
2
(iii) sin 1x + sin 1(l-x)=cos 1 x [NCERT EXEMPLAR, CBSE 2016]
SOLUTION (i) We have,
7t
sin 1 (1 -x) -2 sin- 1 x =
2

ReadYourFlow.COM
r

4.108 MATHEMATICS-XII

=> sin 1 (1 - x) = — + 2 sin 1 x


2
sin |sin 1(l-x)j=sin^ — + 2 sin-1x
2
l
=> 1 - x = cos (2 sin x)
1 - x = cos cos \l-2x2)^ [v 2 sin 1 x = cos 1 (1 - 2x2)]
=>

=> 1 -x = (1 - 2x2) => x = lx2 => x (2x -1) = 0 => x = 0, -


2
1
For, x = — , we obtain
2'
n
LHS = sin_1(l - x) - 2 sin- 1 x = sin ----2 sin -1 1 = - sin = -- * R.H.S.
2 2 2 6

w
So, x = 1/2 is not a root of the given equation.
Clearly, x = 0 satisfies the given equation. Hence, x = 0 is a root of the given equation,
(ii) We have.
11 cot (2 tan 1 x)| =0
Flo
ee
sin 2 cos

Fr
2x
=> sin 2 cos 1 cot tan 1 = 0
or v 2 tan 1 x = tan -1 2x
ur
\ 1-x2 1-x2
sf

1-x2
k

v cot 1 x = tan -11


Yo

=> sin 2 cos 1 cot cot 1 = 0


oo

2x x
' s)
B

1-x2
re

=> sin 2 cos 1 =0


2x
ou
ad
Y

?^2
• -1 2 1-x2 1-x2
=> sin sin 1- = 0 2 cos 1 x = sin 1 (2x -x2)
2x 2x
nd
Re
Fi

1-x2 2^2
-x
=0
x 2x

1-x2 1 - X2 ^ 2
=> = 0 or. 1- =0
x 2x
2^
9 1 -x
=> 1 -x = 0 or, - = 1
2x
x = ±1 or, (1-x2)2 = 4x2
Now, (1 -x2)2 =4x2
=> (1 -x2)2 -(2x)2 =0
(1 -x2 -2x)(l -x2 + 2x) = 0
=> 1-x2 -2x = 0 or, 1 -x2 + 2x = 0

ReadYourFlow.COM
INVERSE TRIGONOMETRIC FUNCTIONS 4.109

x2 + 2x-1 = Oor,*2-2x-l = 0
=> x = -l±V2or,x = 1±V2
Hence, x = ±l, -l ± 4l, 1 ± V2 are the roots of the given equation.
(hi) We have,
•_ —1 x + sin 1 (1 - x) = cos 1 x
sin
■ -1
sin-1 x + sin-1 (1 -x) = —-sin -1 x- —
K sin
■ -1 x
=> in x ■/ cos
2 2

=> sin 1 (1 - x) = — - 2 sin ^x


2
sin | sin 1 (1-x)| = sin ^-2sin 1x

=> 1 - x = cos (2 sin !x)


1 - x = cos | cos_1(l - 2x2) v 2 sin _1 x = cos-1(l - 2x2)

w
1 -x =1 -2x2 => 2x2 -x = 0 => x(2x-l) =0 => x = 0 or, x = -^
=>
Clearly, these values satisfy the given equation.
Flo
ee
Hence, x = 0, ^ are the roots of the given equation.

Fr
EXAMPLE 16 Solve for x:
for
ur
-if 2x ^ _ 1 1-X2 71
= — -1 < x < 1 [CBSE 2011]
tan ------T + cot
2x 3'
\ 1 -x2 /,
ks
Yo

SOLUTION We know that


oo

tan -if1
B

, if x > 0
cot- 11 x , if x > 0 x
tan -if1 i.e. cot 1 x =
re

x - it + cot- 1 x , if x < 0 n + tan -ifl- , if x < 0


x
ou
ad

So, following cases arise:


Y

CASE I When 0 <x <1:


nd
Re

In this case, we have


1 -x2 1 = tan -1
Fi

2x
cot 1
2x 1-x2
Given equation is
2x 1 fl-X2l 71
tan -1 + cot
1-x2 2x 3

2x 71
=> 2 tan -1 [Using (i)]
1-x2 3

=> 4 tan 1 x = — => tan- 1x=—=> x = tan — = tan 15° = —-


3 12 12 V3 +1
CAST II When -1 < x < 0:
In this case, we have
1 -x2 2x
cot 1 = 7i + tan -1 •••(ii)
2x 1-x2

ReadYourFlow.COM
4.110 MATHEMATICS-XII

Given equation is
-1 2x 1 - -v2 "l n
tan ------ - + cot 1
{l-x2
\
2x 3

2x 2x 71
=> tan -1 2 + 7r + tan -1 [Using (i)]
u -x l-*2 3

2x 2n
=> 2 tan-1
[l-x2 3
271 _ 1 K n 1
=> 4 tan -1* x = ------=> tan x = — => x = tan
3 6 6
CASE ffl When x = 0:
In this case, we have

w
2x 2^
LHS = tan - 1 -1 1 — X - tan 1 (0) + cot 1 (co) =— and, RHS = —
-y + cot
1 - xz

So, x = 0 is not a solution of the given equation.


2x

Flo 2 3

ee
V3-1

Fr
Hence, x = and x = are solutions of the given equation.
V3 +1 V3 or
ur
LEVEL-2
sf

-i r x2 -i ^ 2ti
EXAMPLE 17 Solve: cos + tan -1
k
Yo

,x2 + l; 3
oo

SOLUTION The given equation is


B

-if x2 -1
re

2x 2n
cos + tan -1
x2+l 2
vx -1 3
ou
ad

1 -x 2 2x 2n
Y

=> 71-COS
-1 -tan -1
1+x2 ll-x2 3
nd
Re

1-x2 2x 7X
=> cos -1 + tan -1 ...(i)
Fi

1+x2 1-x2 3
We know that
tt + 2 tan -1 x, x < -1
cos -1 fl-x2' 2 tan 1 x, x > 0 -1 2x
2tan-1x, -1<x<1
and, tan
1 + x2, -2 tan_1x, x < 0 1-x2
-K+2tan-1x, x>l

So, we have the following cases:


CASE I When x < -1:
In this case, we have

cos -1
ri-x2> = -2 tan i
x and tan -l
2x
= 7t + 2tan Jx
>1+*2, 1-x2
il-x2' 2x n
cos -l + tan -1
1+x2 1-x2 3
\ /

ReadYourFlow.COM
INVERSE TRIGONOMETRIC FUNCTIONS 4.111

=> -2 tan 1 x + 7r + 2 tan 1 x = —


3
Jl
=> n = —, which is absurd.
3
So, the equation (i) has no solution for x < -1.
CASEH When -1 < x < 0
In this case, we have
1-x2 2x
cos -1 = -2 tan 1 x and tan -1 = 7T; + 2tan lx
l + x2 l-x2J
1-x2 2x
cos -1 + tan -l
ji

l + x2 1-x2 3

w
=> -2 tan 1 x + 2 tan 1 x = —
3
7C
0 = —, which is also an absurd result.
3

Flo
ee
So, the equation (i) has no solution for 1 < x < 0.

Fr
CASE III When 0 < x <1.
In this case, we have
for
ur
/ 2
1 -x 2x
cos = 2 tan 1 x and tan -1 = 2 tan 1x
T7? 1-x2
ks
Yo

1-x2
oo

2x 71
cos -1 + tan -1
l+x2 1-x2 3
B
re

2 tan 1 x + 2 tan 1 x = —
3
ou
ad

, -l ft . -l 71 .ft 7^3-1
= 2-73
Y

4 tan 1 x = — => tan x = — => x = tan — =


3 12 12 73+1
nd

Clearly, x = 2 - 73 satisfies the condition 0 < x < 1.


Re

Hence, x = 2 - 73 is a solution of equation (i).


Fi

CASE IV When x > 1.


In this case, we have
1-x2 2x
cos -1 = 2 tan 1 x and tan -1 = -7i+2 tan 1 x
l + x2 1-x2

1-x 2 2x Jl
cos -1 + tan -1
l+x2 1-x2 3

=> 2tan-'1 x — 7i + 2 tan —^ x = —


3
< . -1 471 u -1 71
=> 4 tan x = — => tan x = — => x = tan — = 73 ■
3 3 3
Hence, x = 2 - V 3, 73 are solutions of the given equation.
EXAMPLE 18 If y = cot 1 yjcos X j - tan 1 ^cos x j, prove that sin y = tan 2 x
2

ReadYourFlow.COM
4.112 MATHEMATICS-XII

SOLUTION We have.
= cot 1 ^COS X j - tan 1^cosx^
y
K
=> y = - -tan 1 ^COS X j - tan -1 [Vcos ^)
2

y = - - 2 tan - 1 ^ ->JCOS X j
2
2
1- COS X
=>
K
y = —cos -1 2 tan 1 x = cos ifl-*2
2 l+x2
1+ ^Vcos ^)
% 1 - COS X
=> y = - - cos -1
2

w
\
1 + COS X y

=> n
y =----cos -if.
tan 2 -x

=> cos -1
2

tan 2 * 7T
2

Flo
ee
x-y
2 2

Fr
n
=t> tan 2 X
cos x-y
2 2
for
ur
2 X
=> tan = sin y.
2
ks
Yo

cos a + cos P = 2 tan 1 j^tan ^ tan ^ j.


EXAMPLE 19 Prove that: cos -1
oo

v 1 + cos a cos P
B

SOLUTION We have,
re

RHS = 2 tan- 1 {tan — tan —


l 2 2
ou
ad

1 - tan2 a/2 tan2 p/ 2 l-x2l


Y

=> RHS = cos -l v 2 tan -l‘ x = cos -l


1 + tan2 a/2 tan2 p/2 l + x2
nd
Re

_1 cos2 a/2 cos2 P/2 - sin2 a / 2 sin2 p/2


Fi

=> RHS = cos


cos2 a/2 cos2 P/2 + sin2 a / 2 sin2 p/2
-1 (2cos2 a/2) (2cos2 p/2) -(2sin2 a/2) (2sin2 p/2)
=> RHS = cos
(2cos2 a/2) (2cos2 p/2) + (2sin2 a/2) (2sin2 p/2)
_ 1 J (1 + cos a) (1 + cos P) - (1 - cos a) (1 - cos P)
=> RHS = cos
| (1 + cos a) (1 + cos P) + (1 - cos a) (1 - cos P)
r cos a + cos p
=> RHS = cos -1 = LHS.
v 1 + cos a cos P y
EXAMPLE 20 Show that: 2 tan -1J tan — tan 71 P sin a cosp
= tan -1
2 4 2 v cos a + sin p
[NCERT EXEMPLAR]
71 P
SOLUTION LHS = 2 tan -1 tan — tan
2 4 2

ReadYourFlow.COM
INVERSE TRIGONOMETRIC FUNCTIONS 4.113

a n P
2 tan — tan
2 4 2
=> LHS = tan-1
2a
1 - tan tan
2 U 2
1 - tan —
a 2
2tan —
2 P
1 + tan —
LHS = tan-1 *! 2;

1 - tan - Pf
2a 2
1 - tan
2 1 + tan P
2

a P

w
2 tan 1 - tan — 1 + tan-
2 2 2
=> LHS = tan -1
pn2 -tan 2 a Pf
1 + tan
2 2

Flo
1 - tan
2

ee
2tan^l -tan2^j

Fr
=> LHS = tan -1
P + tan 2pf -tan 2 ct 1-2 tan —P + tan 2 P
or
ur
1 + 2 tan —
2 2 2 2 2
sf

1 - tan 2 P
a
2 tan
k
Yo

2 2
oo

=> LHS = tan-1


1 + 2 tan - + tan2 ^ -tan 2 a 1 -2tan- + tan2 ^
B

2 2 2 2 2
re

1 - tan 2P
a
2 tan
ou
ad

2 2
=> LHS = tan -1
2a p 1 + tan 2 a + tan 2 P 1 - tan 2 «
Y

1 - tan + 2 tan —
2 2 2 2 2
nd
Re

a
2 tan — 1 - tan 2 P
2 2
LHS = tan -1
Fi

=>
1 - tan
2 a
1 + tan 2 P + 2 tan -
P 1 + tan 2 «
2 2 2 2

1 - tan 2 P 1
a
2 tan —
_____ 2_x--------- 2
1 + tan
2 «
1 + tan
2P
2 2 • = tan sin a cos P
=> LHS = tan-1 = RHS
1 - tan 2a P
2 tan —
cos a + sin p
2 + 2
1 + tan 2 « 1 + tan P
2
2 2

a3 P3
EXAMPLE 21
2 1
Prove that: — cosec — tan
-1 « + sec 2ri tan"1 P j = (a + P) (a2 + p2).
2 2 PJ 2 2 a
SOLUTION We have,
a3 2(1
LHS = — cosec — tan
p sec 2 f —1 tan 1 P
-1 » +;— 3
2 2 P 2 2 a

ReadYourFlow.COM
4.114 MATHEMATICS-XII

a3 P3 1 1
=> LHS = + , where 0 = — tan -l —
a
and (b = — tan -1 P
2 sin2 0 2 cos2 (j> 2 P 2 a
a3 P3
=> LHS = +
1 - cos 2 0 1 + cos 2 <j)
a3 P3
=> LHS =
1 - cos (tan 1 a/P) 1 + cos (tan 1 p/a)
a3 P3
LHS = T+
P a
1 - cos cos -1 1 + cos cos 1

a3 P3

low
=> LHS = +
P a
1- 1+
\/a2 +P2 xt^+P2
a3 P3

ee
LHS =
Va2+p2-p
rF
^a2 + p2 + a

Fr
a 3 | Ja2 +p2 + pj p3 • ^/a2 + p2 - a j
for
ou
LHS = ^a2 + p2
ks

a2 + P2 - P2 a2 +p2 - a2
oo
Y

LHS = (a( ^ 2+p2


B

=> + pj+p |j Va2+p2 ■a)} \/a2+p2


re

=> LHS = a(a2+p2) + p(a2+p2)=(a + p)(a2+p2)


ou
ad

EXAMPLE 22 Prove that:


Y

cos 2 a sec 2 p + cos 2 p sec 2 a


tan -1 = tan 1 ■] tan2 (a + P) tan2 (a -P) l + tan 1 1
nd
Re

2
Fi

SOLUTION We have.

RHS = tan -1 tan2(a + p) tan2(a ~P) l + tan 11

tan2 (a + P) tan2 (a ~P) + 1


=> RHS = tan -1
1 - tan2 (a + P) tan2 (a -p)

sin2 (a + p) sin2 (a ~P) + cos2 (a + P) cos2 (a -P)


=> RHS = tan"1 •
cos2 (a + P) cos2 (a -p) - sin2 (a + P) sin2 (a ~P)

{2 sin (a + p) sin (a - P)}2 + {2 cos (a + p) cos (a - p))2


=> RHS = tan"1 •
{2 cos (a + P) cos (a -p)}2 -{2 sin (a + P) sin (a ~P))2

(cos 2 p - cos 2 a)2 + (cos 2 a + cos 2 P)2


=> RHS = tan"1 •
(cos 2 a + cos 2 P)2 - (cos 2 P - cos 2 a)2

ReadYourFlow.COM
INVERSE TRIGONOMETRIC FUNCTIONS 4.115

1 cos2 2 a + cos2 2 p cos 2 a sec 2 p + cos 2 p sec 2 a


=> RHS = tan~ = tan -1 = LHS.
2 cos 2 a cos 2 P 2

EXERCISE 4.14
LEVEL-1
1. Evaluate the following:
(i) tan 12tan

(ii) tanj^sin -1 3 [CBSE 2013, NCERT EXEMPLAR]


4
(iii) sin ^ cos-1 4
5
(iv) sin ^2 tan ^j + cos^
tan"1 V3 [NCERT EXEMPLAR]

w
2. Prove the following results:
(i) 2 sin"1-= tan -1 24
5 7

Flo
ee
1 _i 3 1 . -1 4
(ii) tan"1 — + tan -1 2 = - cos — = - sin [CBSE 2010 C]

Fr
4 9 2 5 2 5
-1 2 = - 1 -1 12
(iii) tan tan
3 2 5
for
ur
(iv) tan" 1 y + 2 tan -1 1 7t
[CBSE 2010]
3 4
ks

4 n
Yo

(v) sin — + 2 tan -1 1


oo

5 3 2
B

(vi) 2 sin" 1 — - tan -1 17 7C


re

5 31 4
(vii) 2 tan"1 ^ + tan -1 — 1 = tan -1 4 [NCERT]
ou
ad

8 7
Y

(viii) 2 tan" 1 ^ - tan -1 17 _rc [CBSE 2011]


31 ~ 4
nd
Re

(ix) 2 tan" 1 - + tan -1 — 1 = tan -1 31 [CBSE 2011]


7
Fi

2 17
1 TC
(x) 4 tan"1--tan -1 [NCERT EXEMPLAR]
5 239 4
ll-E2 -1 2x a-b
3. If sin"1 — - cos tan - , then prove that x =
1+a2 1 +b2 \-x 1 + ab
4. Prove that:
9A
(i) tan"1
1-*21 + cot 1
1-X2 K
2x 2x 2
1-x2 -ll-*2
(ii) sin< tan -1 + cos =1
2x 1 + x2
5. Ifsin"1-^- -1 2b = 2 tan" 1 x, prove that x = a + b
2 + sin
1+ a 1+b2 1 - ab

6. Show that 2 tan" 1 x + sin"1 — - is constant for x > 1, find that constant.
1+x

ReadYourFlow.COM
4.116 MATHEMATICS-XII

7. Find the values of each of the following:


(i) tan - 1 2 cos 2 sin -1 1 (ii) cos (sec 1 x + cosec' 1 x),\x\>l
2
8. Solve the following equations for x :
(i) tan 1 — + 2 tan- 1 - + tan- 1 — + tan -ll 7T

4 5 6 X 4
1-x22+2tan-1-2>:
. 7t
(ii) 3 sin- 1 —- 4 cos- 1
1 + x2 1+x 2 1 -x2 3

2x _1 l-AT 271
(iii) tan 1 2 + cot x>0 [CBSE2010]
1-x 2x 3 '

(iv) 2 tan 1 (sin x) = tan 1 (2 sec x), x * ^ [CBSE2012]

w
-l at2 -1 1 2x 2ti
(v) cos + —tan -1

Flo
x2+l 2 1-x2 3
x-2 -1 f * + 2 71
(vi) tan -1

ee
+ tan [CBSE2016]
x —1 x +1 4

Fr
LEVEL-2 or
ur
a -b 0 if a cos 6 + b
sf
14. Prove that 2 tan 1 tan — = cos
a+b 2 \ a+ b cos 0
k
Yo
oo

15. Prove that:


B

2ab -1 2xy -1 2 aft


tan - 1 j + tan = tan , where a = ax -by and fi = ay + bx.
a2-b 2 2
re

a2-p2
* -y
16. For any a,b,x,y> 0, prove that:
ou
ad

2 1 / 3ab2 - a3 3xy2 -x3


Y

— tan + - tan-1 = tan -1


3 b3 -3a2b 3 U3-3x2y
nd
Re

where a = -ax + by, ft = bx + ay.


Fi

ANSWERS
1 37
1- (i) (iii) (iv) — 6.7t 7. (i) ^ (ii) 0
17 VTo 26

*■ 1,1 -f <“> i » A (iv) x=nn + -r n sZ


4
(v) x = V3

________ VERY SHORT ANSWER QUESTIONS (VSAQs)


Answer each of the folloiving questions in one word or one sentence or as per exact requirement of the
question:
1
1. Write the value of sin -l + cos -1
2 2
2. Write the difference between maximum and minimum values of sin 1 x for x e [-1,1].
— 1 — 1 1 37t
3. If sin x + sin y + sin- z = —, then write the value of x + y + z.

ReadYourFlow.COM
INVERSE TRIGONOMETRIC FUNCTIONS 4.117

2x in terms of tan 1 x.
4. If x>l, then write the value of sin 1
1 + x2
1-x2
5. If x < 0, then write the value of cos 1 in terms of tan 1 x.
1 +x2

6. Write the value of tan 1 x + tan -if1 for x > 0.


x
7. Write the value of tan 1 x + tan 1 (i)for*<0-
2n . 2n
8. What is the value of cos -1 cos — + sin 1 sin — ?
3 3
2x
9. If-1 <x<0, then write the value of sin 1 + cos 1
1 -f x2

w
10. Write the value of sin (cot 1 x).

11. Write the value of cos- 1 f - ] + 2 sin - 1 1


2

Flo
2

ee
12. Write the range of tan - 1 x.

Fr
13. Write the value of cos- 1 (cos 1540°). or
ur
14. Write the value of sin - 1 (sin (- 600°)).
sf
15. Write the value of cos ^2 sin 1 j
k
Yo
oo

16. Write the value of sin 1 (sin 1550°).


B
re

1 -1 4
17. Evaluate: sin - cos
2 5
ou
ad

18. Evaluate: sin ^ tan 1


Y
nd

19. Write the value of cos 1 [tan — ].


Re

4
Fi

20. Write the value of cos 2 sin -1 1


2
21. Write the value of cos 1 (cos 350°) - sin -1 (sin 350°)
2fl -1 3
22. Write the value of cos - cos
V2 5
23. If tan 1 x + tan 1 y = - , then write the value of x + y + xy. ICBSE2014]
4
24. Write the value of cos- 1 (cos 6).
TC
25. Write the value of sin -1 cos — .
9
26. Write the value of sin -sin [CBSE2011J

15t:
27. Write the value of tan -1 tan
4

ReadYourFlow.COM
4.118 MATHEMATICS-XII

1
28. Write the value of 2 sin 1 - + cos -1 [CBSE2014]
2 2
a -b
29. Write the value of tan -l —
n tan -1
b a+ b
Stt
30„ Write the value of cos 1 cos — .
4

31. Show that sin 1 (2.r-Jl - x2) = 2 sin 1 .r

3t:
32. Evaluate: sin -1 sin — {CBSE2009]
5
33. If tan 1 (-73) + cot 1 x = — , find x. [CBSE2010]
2

low
34. If sin - 1 — + cos - 1 x = — , then find x. [CBSE2010]
3 2 '
1 1
35. Write the value of sin -1 - cos -1
3 3

ee
36. If 4 sin -1 x + cos ' 1 x = then what is the value of x?
rF
Fr
37. If x < 0, y < 0 such that xy = 1, then write the value of tan -1 x + tan -1‘ y.
for
38. Wliat is the principal value of sin l
V3 N
? [CBSE 2010]
ou
2
ks

39. Write the principal value of sin- 11 j.


[CBSE 2011)
oo
Y
B

. 2n \
40. Write the principal value of cos- 11 cos ^ j + s*n -1 sin — .
re

3
41. Write the value of tan f 2 tan -1 1
ou

[CBSE 2013]
ad

5
Y

42. Write tire principal value of tan 1{l) + cos [CBSE 2013]
nd
Re

43. Write the value of tan -i 2 sin 2 cos -i Va'l


Fi

[CBSE 2013]
2
44. Write tire principal value of tan 1 7 3 + cot 1V3. [CBSE 21013]
45. Write the principal value of cos-] (cos 680°). [CBSE 2014]
. 37^
46. Write the value of sin -1 sin— .

47. Write the value of sec

48. Write the value of cos

49. Write the value of cos

sin 1 x + cos ~lx) y[3


50. Write the value of the expression tan - , when x = —
2 2

ReadYourFlow.COM
INVERSE TRIGONOMETRIC FUNCTIONS 4.119

51. Write the principal value of sin

52. The set of values of cosec

-1 j for x < 0 in terms of cot 1 (x).


53. Write the value of tan

54. Write the value of cot 1 (-x) for all x e R in terms of cot 1 x.

tan 1x + cot 1x 1
55. Write the value of cos , whenx = —T=.
3 -J3
56. If cos (tan 1 x + cot 1 -J3) = 0, find the value of x.
1
57. Find the value of 2 sec 12 + sin -1

low
2
58. Ifcos^sin 1 ^ + cos = 0, find the value of x.

1371

ee
59. Find the value of cos -l cos----- .
6
rF
Fr
971
60. Find the value of tan -1 tan — .
8
for
ou
__ ANSWERS
ks

1. * 2. 7t 3. 3 4. tt: - 2 tan 1 x 5.-2 tan- 1 x


3
oo

n K 1 27t
Y

6. - 7. — 8. n 9. 0 10. 11. —
B

2 2
ft + x: 3
re

n % 7 1
12. 13. 100° 14. 60° 15. 16. 70 17.
Vio
ou

2' 2 9
ad

3 1 4
Y

18. - 19. 7T 20. - 21. 20° 22. - 23. 1


5 2 5
7k K K
nd

24. 271-6 25. — 27. -- 28. n 29. -


Re

26. 1
18 4 4
2tc 1
Fi

3tt: 1 k
30. — 32. — 33. 73 34. - 35. -- 36. -
4 5 3 2 2
K — 7t 5 11 71
37. -- 38. -- 39. 40. tc 41. — 42. —
2 3 6 12 12
2k 2tt
43. - 44. - 45. 40° 46. — 47. 48. —
3 2 5 3
Tt
49. 0 50. 1 51. - 52. 4) 53. -TT + COt 1 x 54. 7i - cot 1 x
3
73 5 71 2 71 K
55. — 56. 73 57. — 58. 59. - 60. -
2 6 5 6 8

MULTIPLE CHOICE QUESTIONS (MCQs)

1. If tan -1 Vi + X Mi-*2=7
2
= a, then x2 =
yjl + x2 + yi - x
(a) sin 2 a (b) sin a (c) cos 2 a (d) cos a

ReadYourFlow.COM
4.120 MATHEMATICS-XII

1
2. The value of tan cos -1 - sin is
5 -Jl Vl7
V29
(a)-r
29
(b) T 29
(c) (d)429
3. 2 tan 1 {cosec (tan 1 x) - tan (cot 1 x)) is equal to
(a) cot 1 x (b) cot 1 1 (c) tan 1 x (d) none of these
x
-1 y x2 2 xy y2
4. If cos 1 — + cos cos a +
a b a2 nb b2
(a) sin2 a (b) cos2 a (c) tan a (d) cot2 a
3 .
5. The positive integral solution of the equation tan -1 X + cos 1-7=L==sin-1 —j= is
a/1 + y2 n/To

w
(a) x = l,y = 2 (b) x = 2, y = 1 (c) x = 3, y = 2 (d) x = -2,y = -1
6. Ifsin_1x - cos - 1 x = —
^ , then x =

(a)i
6'
V3
Flo 1

ee
(b)i (C) ~2 (d) none of these

Fr
7. sin cot 1 tan cos 1 x is equal to
for
ur
(a) x (b) ^-x2 1
(c) - (d) none of these
x
ks
Yo

^71 is
8. The number of solutions of the equation tan 1 2x + tan 1 3x = — .
oo
eB

(a) 2 (b) 3 (c) 1 (d) none of these


9. If a = tan-1 ^ tan ^ j and p = tan 1 ^ - tan 2— n
, then
r

3J
ou
ad

(a) 4 a = 3 p (b) 3a = 4 p (c) a-p = ^ (d) none of these


Y

12
10. The number of real solutions of the equation ^/l + cos 2x = V2 sin- 1 (sin x), - iz<x <n is
nd
Re

(a) 0 (b) 1 (c) 2 (d) infinite


Fi

11. If x < 0, y < 0 such that xy = 1, then tan- 1 x + tan- 1 y equals


/ X

2
71
(b)-f (c) -71 (d) none of these
12. If u = cot- 1 {^/tan 0} - tan ~ 1 {^tan 0} then, tan f ^ ~ ^ 1 =

(a) vdan 0 (b) Jcot 0 (c) tan 0 (d) cot 0


13. If cos -1 —
* + cos —, then 4x2 - 12xy cos — + 9i/2 =
2
3 2 2 '
(a) 36 (b) 36 - 36 cos 0 (c) 18 -18 cos 0 (d) 18+ 18 cos 0
14. If a = tan
l(^x] P = tan -1 2x-y
then a - P =
Uv-*/ ^yJ'
/ \ ^
6
(b)f / \ ^
<C)2 (d)-|

ReadYourFlow.COM
INVERSE TRIGONOMETRIC FUNCTIONS 4.121

cos 1 {sin (x + 7t/3)}


15. Let f (x) =e . Then, / (8 rt / 9) =
13 7t/18 - 2k
(a) e.5 n /18 (b) t? (c) <? /18 (d) none of these
1 2
16. tan -1 —^ + tan - 1 — is equal to
11 11
(a) 0 (b) 1/2 (c) -1 (d) none of these
rr -1 X
17. If cos - + cos “ 1 ^ = 0, then 9x2 - llxij cos 0 + 4i/2 is equal to
2
(a) 36 (b) - 36 sin2 0 (c) 36 sin2 0 (d) 36 cos2 0
18. If tan- 1 3 + tan-1 x = tan - 1 8, then x =
(a) 5 (b) 1/5 (c) 5/14 (d) 14/5
33 tc V
19. The value of sin 1 cos is
5

w
3 71 < \ —71 7n
(b) (c) (d)T
(a)T 10 10
5 ji: + sin 11 sin — ] is
20. The value of cos 1 cos —
3
Flo 3

ee
5 71 10 71

Fr
<a)f (b> T (c)-3 (d) 0

21. sin 12 cos 1 78 eclualto


for
ur
(a) 6/25 (b) 24/25 (c) 4/5 (d) -24/25
ks
Yo

22. If 0 = sin- 1 {sin (- 600°)}, then one of the possible values of 0 is


oo

2 7C 2 TC
( \ n3 (b)f
B

(a) (C)T (d)"T


re

2 \
2x 1 -X
23. If 3 sin- 1 - 4 cos -1 + 2 tan 1 —, then x is equal to
ou
ad

l+x2 1 +x2 3
Y

(a) i
V3
(b) -i
V3
(c) V3 <d)-f
nd
Re

24. If 4 cos 1 x + sin 1 x = tc, then the value of x is


Fi

,)f (b)
1
(C)T
J3
(d) -4
V3
X+1 - ! X —1
25. If tan 1 + tan = tan 1 ( - 7), then the value of x is
x —1 X

(a) 0 (b) -2 (c) 1 (d) 2


■ - 1
26. If cos 1 x > sin x, then
1
(c) -\<x<-p= (d) x > 0
(h) °-<72
b )
27. In a A ABC, if C is a right angle, then tan- 1 + tan 1
(b + c) c+a
/-<a)\ 713 5 7C (d)^
^ V63
c T 6

28. The value of sin — sin- 1----- is


4 8

ReadYourFlow.COM
4.122 MATHEMATICS-XII

1 1 1 (d) ^
(a) “7x (b)73 (c)
\^2 2v2 3v 3
29. cot — - 2 cot 1 3 =
4
(a) 7 (b) 6 (c)5 (d) none of these
30. If tan* 1 (cot 0) = 2 0, then 0 =

(a)±f (b) ±7
4
(c)
6
(d) none of these

^ 2a 2a-
31. If sin -1 + cos -1 = tan -1 -r- , where a, x e (0,1), then, the value of x is
si + fl2 l-*2

(b)^ 2a
(a) 0 (c) a (d)
l-*2
0.75 j j is equal to

low
32. The value of sin 2 tan -1

(a) 0.75 (b) 1.5 (c) 0.96 (d) sin-11.5


2-y N
33. If x > 1, then 2 tan 1 x + sin -1

ee
is equal to
1 + x2,
rF
Fr
(a) 4 tan * x (b) 0 / \ 71 (d) tc
2
for
34. The domain of cos 1(x2-4)is
ou
(a) [3,5] (b) [-hi]
ks

(c) [-V5,-V3]u[V3,V5] (d) [-V5,-v/3]n[-V5,V3]


oo

35. The value of tan f cos 1 — + tan -ll is


Y
B

5 4
19 8 19
re

(a) (d) |
(b) rr (c)
8 19 12 4
ou
ad

____ ANSWERS
Y

1. (a) 2. (d) 3. (c) 4. (a) 5. (a) 6. (b) 7. (a) 8. (a) 9. (a)


10. (c) 11. (b) 13. (c) 14. (a) 15. (b)
nd

12. (a) 16. (d) 17. (c) 18. (b)


Re

19. (b) 20. (d) 21. (d) 22. (a) 23. (a) 24. (c) 25. (d) 26. (a) 27. (b)
Fi

28. (c) 29. (a) 30. (c) 31. (d) 32. (c) 33. (d) 34. (c) 35. (a)

SUMMARY
1. (i) sin -1 (sin 0) = 0, for all 0 e [-7t/2, tt/2]
(ii) cos* 1 (cos 0) = 0, for all 0 e [0, tt]
(iii) tan* 1 (tan 0) = 0, for all 0 e (- n/2, n/2)
(iv) cosec- 1 (cosec 0) = 0, for all 0 e [- 7t/2, ti/2], 0*0
(v) sec- 1 (sec 0) = 0, for all 0 e [0, tt], 0 * tt/2
(vi) cot- 1 (cot 0) = 0, for all 0 e(0, rc).
2. (i) sin (sin -1 x) = x, for all x e [-1,11
(ii) cos (cos- 1 x) = x, for all x e [-1,1]
(iii) tan (tan - 1 x) = x, for all x e R

ReadYourFlow.COM
INVERSE TRiGONOMETRIC FUNCTIONS 4.123

(iv) cosec (cosec- 1 x) = x, for all x e (- oo, -1] u [1, co)

(v) sec (sec-1 x) = x, for all x e (- oo, -1] u [1, co)

(vi) cot (cot- 1 x) = x, for all x eR.

REMARK It should be noted that sin -1 (sin 0) * 0, zf 0 e [- tt/2, ti/2].

In fact, we have
tt -0
, if 0 [-3ti/2 , - rc/2]
0 , if 0 [- tc/2 , n/2]
sin-1 (sin 0) =
TC - 0 , if 0 [k/2,3ti/2]
-2ti+0 , if 0 [3tc/2,5ti:/2] and so on.

w
Similarly, we have
-0 , if 0 e [- tc , 0]

cos 1 (cos 0) = •
0
2tc - 0
,
,
if
if

Flo
0 e [0, tc]
0 € [tc , 2/tc]

ee
-2tc+0 , if 0€[2tc,3/tc] and so on.

Fr
TC - 0 , if 0 e (- 3tc/2 , - k/2)
0 , if 0 e(- tc/2 , tc/2)
for
ur
tan 1 (tan 0) =
0 - TC , if 0 s(tc/2,3tc/2)
0 - 2tc , if 0 e (3tc/2,5tc/2) and so on.
ks
Yo
oo

3. (i) sin-1(-x) = -sin-1x, for all x e[-l, 1]


B

(ii) cos- 1 (- x) = tc - cos- 1 x , for all x e [-1,1]


re

(iii) tan - 1 (- x) = - tan - 1 x. for all x e R


ou
ad

(iv) cosec 1 (- x) = - cosec 1 x, for all x e (- oo , -1] vj [1, oo)


Y

(v) sec 1 (- x) = tc - sec ‘.V. for all x e (- oo, -1] u [1, oo)
nd
Re

(vi) cot- 1 (- x) = tc - cot- 1 x , for all x e R


Fi

4. (i) sin -if1 = cosec 1 x. for all x € (- co, -1] u [1, oo)
x

(ii) cos = sec 1 x. for all x e (- co, -1] u [1, co)

cot 1 x , for x > 0


(iii) tan 1 ^j =
- tc + cot- 1 x , for x < 0

5. (i) sin 1 x + cos 1 x = ~, for all x e [-1,1]

(ii) tan - 1 x + cot- 1 x = - for all x e R


2'

(iii) sec 1 x + cosec' for all x e (- oo, -1] u [1, co).


2'

ReadYourFlow.COM
4.124 MATHEMATICS-XII

tan -! x+y , if xy < 1


l-xy
-if * + y
6. (i) tan -1 .T + tan ’H k+ tan , if .v > 0, y > 0 and xy > 1
1 xy
-if x + y^
-7T+ tan if x < 0, y < 0 and xy > 1

-1 x-y
tan , if xy > -1
1 + xy

(ii) tan 1 x - tan 1 y = • 7i + tan -1 x-y


, if a > 0, y < 0 and xy < -1
1 + xy
x-y
-k + tan -1 if x < 0, y > 0 and xy < -1
1 + xyJ '

w
REMARK If X], x2/ x3, x,, 6 R, then
Sl~ S3 + % -$7 + ■■■)
tan ~ 1 X| + tan 1 x2 + + tan -1 x,, = tan -1

Flo 1 -S2 + S4 -S6 + ... J

ee
where denotes the sum of the products of x^, x2,... , xtl taken k at a time.
7. (i) sin ’1 x +- sin- 1 y

Fr
sin lx yl -y1 +y V1 - x2 or
, if -1 < x , y < 1 and x2 + y2 < 1
ur
sf
or
k

if xy < 0 and x + y > 1


Yo
oo

71 - sin 1 {Wl-y2 +yy/l-^2} , if 0 < x, y < 1 and x2 + y2 > 1


B

1 | x -yjl -y2 + y -^1 - x21 , if -1 < x, y < 0 and x2 + y2 > 1


re

- 7i - sin’
ou
ad

(ii) sin 1 x - sin 1 y


Y

sin jx i/l -y2 - y y 1 - x21 , if -1 < x, y < 1 and x2 + y2 < 1


nd
Re

or
Fi

if xy > 0 and x2 + y2 > 1


7i - sin -l | x -Jl -y2 - y yd - x2 |, if 0 < x < 1, -1 < y < 0 and x2 + y2 > 1

- sin* I x y/l - y2 - y y/l -x2 j / if-l<x<0,0<y<l and x2 + y2

8. (i) cos -11 x + cos

COS
^xy-y/l-x2 y'l -y2J , if -1 < x , y < 1 and x + y > 0
= *
-1/ y/l -x2 -^1 -y2| , if -1 < x , y < 1 and x + y < 0
2ti - cos
l xy -
(ii) cos 1 x - cos -1 y

ReadYourFlow.COM
INVERSE TRIGONOMETRIC FUNCTIONS 4.125

cos -1 jxy + ^l-x2 -y2 J , if -1 < A', y < 1 and x < y

-cos if-l<y<0,0<A:<l and .r > y

sin-1 (2a: A/l -x2) ,if--^<A:<-^

9. (i) 2 sin 1 x 71 - sin (2x Jl-x2) , if -J= < x < 1


7t - sin (2x -Jl -x2) , if-1 < x <--j^

sin_1(3x-4x3) 1 1
, if - - < X < -

w
2 2
(ii) 3 sin -1 x = n-sin-1 (3x-4x3) ,if|<x<l

- 7c - sin

Flo
(3x -4x3), if -1 < x < — —
2

ee
Fr
cos 1 (2x2 -1) , if 0 <x <1
10. (i) 2 cos 1 x
or
ur
271 - cos -1 (2x2 -1) , if-1 <x<0
f
ks

1
Yo

cos 1 (4x3 - 3x) ,if - <x<l


oo

2
1
eB

(ii) 3 cos 1 x 2n - cos- 1 (4x3 - 3x), if - <x<-


2
2n + cos-1 (4x3 - 3x), if -1 < x < - -
r

2
ou
ad
Y

2x
tan -1 , if -1 < x < 1
nd

1-x2
Re
Fi

-1 2x
11. (i) 2 tan 1 x = ■< 7i + tan , if x > 1
1-x2
2x
- 71 + tan 1 , if x <-l
1-x2

1 3x-x 1 1
tan 'U-^<X<73
1 - 3x2 V3
3x - x 3 ^
7t + tan - 1
•f 1
(ii) 3 tan-1 x = , if X > -7=
1 - 3x2 y V3
l 3x-x3
- 7t + tan , if x < - 4=
IV 1 - 3x2 V3

ReadYourFlow.COM
4.126 MATHEMATICS-XII

sin'1

12. (i) 2 tan-1 x = Tt-sin-1

(ii) 2 tan

w
13. (i) sin 1 x

Flo
ee
1 V1

Fr
for
ur
(ii) cos 1
ks
Yo
oo
B
re
ou
ad

(iii) tan 1 x = sin 1


Y
nd
Re
Fi

14. If Xi, x2,Xfj e R, then


tan -1 x2 + tan -1 x2 + ... + tan , where
1 - S2 + S4 - S6 +...
Sk = Sum of the products of xlf x2,xn taken k at a time.

ReadYourFlow.COM
CHAPTER 5
ALGEBRA OF MATRICES

5.1 MATRIX
DEFINITION A set ofmn numbers (real or imaginary) arranged in the form of a rectangular array of m
rows and n columns is called anmxn matrix (to be read as 'm by n' matrix).
An m x n matrix is usually written as

w
an an a\3 ••• aii ••• a\n
a2\ a22 a23 ••• fl2; ... a2n
A=

Flo
ee
^2 ^3 ■■■ tyj ••• Bin

Fr
“ml “1112 am3 •" 0,nj ■■■ amn
or
ur
In compact form the above matrix is represented by A = m xn or, A =
sf
Hie numbers a^, a12/... etc. are known as the elements of the matrix A. The element ay belongs
to /th row and y column and is called the (i, j)th element of the matrix A = [ay]. Thus, in the
k
Yo
oo

element a.y the first subscript i always denotes the number of row and the second subscript j,
number of column in which the element occurs.
B

Following are some examples of matrices:


re

[2 1
(i) A = ^ is a matrix having 2 rows and 3 columns and so it is a matrix of order 2x3
ou
ad

1 3
Y

such that flj-j =2, (712 =l/fli3 =-1/ «21 =l/fl22 = 3, «23 = 2-
sm x cos x
nd
Re

(ii) B = is a matrix having 2 rows and 2 columns and so it is a matrix of order


cos x - sin x
Fi

2x2 such that bn = sin x, b12 = cos x, b21 =cos x, b22 = - siri x.
NOTE It is to note here that to define a matrix we must define its order and its elements either by a general
formula (See illustration given below) or separately.
ILLUSTRATION Construct a 3x 4 matrix A = [Ojj] whose elements are given by
(i) Ojj =i + j (ii) Ojj =i-j
SOLUTION (i) We have,
flll ^12 a13 fl14
A = rt21 fl22 fl23 fl24 , where fly = i + j.
. a31 a32 a33 a34
flj-| =1+1=2, fl42 =1 + 2 = 3, fl|3 =1 + 3=4, fli4 =1 +4=5.
Similarly, fl^ = 3, fl22 =4, fl23 =5, fl24 =6 and fl3-j =4, fl32 =5, fl33 = 6, fl34 =7.
'2345'
Hence, A= 3 4 5 6
4 5 6 7

ReadYourFlow.COM
5.2 MATHEMATICS-XII

(ii) Proceeding as above, we obtain


0 -1 -2 - 3"
A= 1 0 -1 -2 .
2 10-1

5.2 TYPES OF MATRICES


ROW MATRIX A matrix having only one row is called a rozu-matrix or a roiv-vector.
For example, A = [1 2 -1 - 2] is a row matrix of order 1x4.
COLUMN MATRIX A matrix having only one column is called a column matrix or a column-vector.
3
1
2
For example, A = 2 and B = are column-matrices of order 3x1 and 4x1 respectively.
-1
4

w
SQUARE MATRIX A matrix in which the number of rows is equal to the number of columns, say n, is
called a square matrix of order n.

Flo
A square matrix of order n is also called a n-rowed square matrix. The elements ay of a square

ee
matrix A = [cijj] n x n for which i = j i.e. the elements «ii/«22/•••/a;in are called the diagonal

Fr
elements and the line along which they lie is called the principal diagonal or leading diagonal of
the matrix.
2 1 -1
or
ur
For example, the matrix 3-2 5 is square matrix of order 3 in which the diagonal
f
1 5 -3
ks
Yo

elements are 2,-2 and - 3.


oo

DIAGONAL MATRIX A square matrix A = [Ojj]n xn is called a diagonal matrix if all the elements, except
eB

those in the leading diagonal, are zero i.e. Ojj = 0 for all i * j.
r

A diagonal matrix of order nxn having d-^, d2,..., 4? as diagonal elements is denoted by
ou
ad

diag [dl>d2,...,d„].
Y

1 0 0
For example, the matrix A = 0 2 0 is a diagonal matrix, to be denoted by A = diag [1, 2, 3].
nd
Re

0 0 3
Fi

SCALAR MATRIX A square matrix A = [ajj]n x), is called a scalar matrix if


(i) Ojj = 0 for all i * j and, (ii) Ojj = c for all i, where c * 0.
In other words, a diagonal matrix in which all the diagonal elements are equal is called the scalar
matrix.
1 - 21 0 0
2 0
For example, the matrices A = , B= 0 1-2/ 0 are scalar matrices of
0 2
0 0 1 -2/
orders 2 and 3 respectively.
IDENTITY OR UNIT MATRIX A square matrix A = [cijj]n xtt is called an identity or unit matrix if
(i) aij = 0 for all i * j and, (ii) Oji = 1 for all i
In other words, a square matrix each of whose diagonal element is unity and each of whose
non-diagonal elements is equal to zero is called an identity or unit matrix.
The identity matrix of order n is denoted by

ReadYourFlow.COM
ALGEBRA OF MATRICES 5.3

1 0 0
1 0
For example, the matrices /2 = , = 0 1 0 are identity matrices of orders 2 and 3
0 1
0 0 1
respectively.
NULL MATRIX A matrix whose all dements are zero is called a null matrix or a zero matrix.
0 0 0 0 0
For example, and are null matrices of orders 2x2 and 2x3 respectively.
0 0 0 0 0

UPPER TRIANGULAR MATRIX A square matrix A = [ay] is called an upper triangular matrix ifay = Ofor
all i > j.
Thus, in an upper triangular matrix, all elements below the main diagonal are zero,
fl 24 3
0 5 13

low
For example, A = is an upper triangular matrix.
0 02 9
0 00 5
LOWER TRIANGULAR MATRIX A square matrix A = [ay] is called a lower triangular matrix ifay = Ofor
alii < j.

ee
rF
Thus, in a lower triangular matrix, all elements above the main diagonal are zero.

Fr
'2 00 “
For example, A = 3 2 0 is a lower triangular matrix of order 3. A triangular matrix A = [Ojj]
for
[A 53
ou
ks

n x w is called a strictly triangular iff 0^ = 0 for all i = 1, 2,..., n.


oo

5.3 EQUALITY OF MATRICES


Y
B

DEFINITION Two matrices A = [Ojj]mxn and B = [by ]r xs are equal if


re

(i) m =r i.e. the number of rows in A equals the number of rows in B


ou
ad

(ii) n = s i.e. the number of columns in A equals the number of columns in B


Y

(iii) Ojj = by fori = 1,2,..., m and j = 1, 2,..., n.


If two matrices A and B are equal, we write A=B, otherwise we write A * B.
nd
Re

3 2 1 3 2 1“
Fi

The matrices A = x y 5 and B = -1 0 5 are equal if x = -l, y = 0 andz = 4.


1 -1 4 J 1 -1 z
0 0 0 0 0
Matrices and are not equal, because their orders are not same.
0 0 0 0 0
x -y 2 x + z -1 ^ , find x, y, z, w. [CBSE 2002 C, 2013]
ILLUSTRATION 1 If
2 x -y 3z +w 0 13
SOLUTION Since the corresponding elements of two equal matrices are equal. Therefore,
x -y 2 x +z -1 5
2x-y 3 z-\-w 0 13
=> x-y = 1, 2x + z=5, 2 x - y = 0, 3z + ze=13.
Solving the equations x-y=-l and 2x-y = 0 as simultaneous linear equations, we get
x =1, y = 2.
Now putting x = 1 in 2 x + z = 5, we get z = 3. Substituting z = 3 in 3 z + w = 13, we obtain w =4.
Thus, x = 1, y = 2, z = 3 and zo = 4.

ReadYourFlow.COM
5.4 MATHEMATICS-XII

ILLUSTRATION 2 Find the values ofx, y, z and a which satisfy the matrix equation
' x + 3 2y + x ~ 0 -7
z-l 4 a -6 3 2a
SOLUTION The corresponding elements of two equal matrices are equal.
~x+3 2y + x~\ TO -7"
z-l 4fl-6 3 2a
=> x+3=0, 2y + x = -7, 2-1 = 3 and 4a - 6 = 2a.
Solving these equations, we get: a = 3, x=-3, y = -2, 2 = 4.

ILLUSTRATIVE EXAMPLES

LEVEL-1
EXAMPLE l A matrix has 12 elements. What are the possible orders it can have?

low
SOLUTION We know that if a matrix is of order m x n , then it has mn elements. Therefore, to
find all possible orders of a matrix with 12 elements, we will have to find all ordered pairs (a, b)
such that a and b are factors of 12. Clearly, all possible ordered pairs of this type are :
(1,12), (12,1), (3, 4), (4, 3), (2, 6), (6, 2)

ee
Hence, possible orders of the matrix are:
rF
Fr
1 x 12,12 x 1, 3 x 4, 4 x 3,2x6 and 6x2.
EXAMPLE 2 If A = [ajj] is a matrix given by for
'4-2 1 3" •
ou
A=[aij]= 5 7 9 6
ks

21 15 18 -25
oo

write the order of A and find the elements a2^, a^ .Also, show that a32 = #23 + a24 ■
Y
B

SOLUTION We observe that there are 3 rows and 4 columns in matrix A. Therefore, it is of order
re

3x4.
The element lying at the intersection of 2nd row and fourth column is 6.
ou
ad

<724 = 6
Y

Similarly, the element lying at the intersection of third row and fourth column is - 25.
^34 = — 25.
nd
Re

fl32 := 15, a23 = 9 and a24 = 6


Fi

^32 =: 15 =9 + 6 = i?23 "t #24 •


„(i+2;)2
EXAMPLE 3 Construct #2x2 matrix A = [#,y ] whose elements are given by Ojj
2

SOLUTION Here Ojj = —^--, 1 < / < 2 and 1 </' < 2. [NCERT]

(1 + 2 x l)2 (1 + 2)2 9 (1 + 2 x 2)2 _ 25


#H - #12 -
2 2 2' 2 2
(2 + 2 x l)2 (2 + 2 x 2)2
#2i - = 8 and #22 - = 18
2 2
9 25
#11 #12
A= 2 2
#21 #22
8 18

EXAMPLE 4 Construct #2x3 matrix A = [#,y] whose elements are given by a1j l~]
i+i

ReadYourFlow.COM
ALGEBRA OF MATRICES 5.5

SOLUTION We have, cl: = ->-)-, 1 <i <2 and 1 <j <3. Therefore,
J i
i+)
n 1 1
fln-0, / rti3-_2' fl21 “ —, #22 ’ ^ and #23 ~ — ~

0 -- --
A= 3 2
1 1
0
L3 5
EXAMPLE 5 Construct a 3x2 matrix A = [ajj] whose elements are given by
-ix K .
(i) Ojj = e,x sin j x (ii) Oij = e cos 2l + 1x

SOLUTION (i) It is given that A =[Ojj] is a 3x2 matrix such that Ojj = e ' sin jx, l <i< 3 and
1 <j <2.
sin x, rt12 = ex sin 2x, #2i = e 2x sin x, #22 = e2x sin 2x, #31 = e 3x sin x and

w
#n =

a32 = e 3y sin 2x
#H #12
A = #2i #22
ex sin x
e2xsinx
ex sin 2x
c2xsin2x

Flo
ee
.#31 a32 e 3x sin x c3Asin2x

Fr
+ ;'xj, 1 <z < 3and 1 <j <2.
(ii) It is given that A = [#,y] is a 3 x 2 matrix such that fly =e~,x cosfor
ur
—r 71 7T i
fln=t’ COS - + X = -e x sin x, #i2 = e x cos - + 2x =-c“xsin2x
2 2
ks
Yo

#2i = e~2x cos (n + x) = -e 2 v cos x, #22 = ^ 2'Y cos ( tt + 2x) = -e 2x cos 2x


oo

3k -3x cos — + 2x1 =e~ 3x sin 2x


#31 =e -3x cos — + X =e 3a sin x, a32 = e
B

2 2
re

#H #12 -e Xsinx -e~x sin2x


-c“2x
ou

cosx -e~2x cos2x


ad

A = #2i #22
e~ 3a sin x e~ 3x sin2x
Y

#31 #32

x-y 2z + w' '5 3'


nd
Re

EXAMPLE 6 Find x, y, z and w such that


2x - y 2x + w 12 15 '
Fi

SOLUTION We know that the corresponding elements of two equal matrices are equal.
'*-y 22 + w 5 3
2x -y 2x + w 12 15
=> x - y =5, 2z + #’= 3, 2x-y = 12 and 2x + w=15
Solving x-y =5 and 2x - y = 12 as simultaneous linear equations, we get x = 7, y = 2.
Putting x = 7 in equation 2x + w = 15, we get w =1.
Putting t# = 1 in 2z +1# = 3, we get z = 1.
Hence, x=7,y = 2,2=1 andw =1.
EXAMPLE 7 Consider the following information regarding the number of men and women workers in
three factories I, II and III.
Men workers Women workers
I 30 25
II 25 31
III 27 26
Represent the above information in the form of 3x 2 matrix. What does the entry in the third row and
second column represent?

ReadYourFlow.COM
5.6 MATHEMATICS-XII

SOLUTION The given information can be represented in the form of a 3 x 2 matrix as follows:
Men workers Women workers
I 30 25
II 25 31
III 27 26
The entry in third row and second column represents the number of women workers in
factory III.
LEVEL-2

a+b 2 6 2
EXAMPLE 8 If ,find the values of a and b.
5 ab 5 8
SOLUTION The corresponding elements of two equal matrices are equal.
a+b 2 6 2

w
5 ab 5 8
=> a + b = 6 and ab = 8
=>
8
«+- = 6

Flo [v ab = 8=> b = 8/a]

ee
a

Fr
a2 + 8 =6a => a2 -6a+ 8 = 0 => {a-4) (a-2) = 0 => a = 2, 4.
Now, a = 2 and ab =8 => b = 4 for
ur
and, a = 4 and ab =8 => b =2.
Hence, a = 2 and b = 4, or a = 4 and b = 2.
ks

EXAMPLE 9 For what values ofx and y are the following matrices equal?
Yo
oo

' 2x + l 3y x + 3 y2 + 2
A = /B =
B

0 y2 -5y 0 -6
re

SOLUTION The corresponding elements of two equal matrices are equal. Therefore,
ou

2x + l 3y _ x + 3 y2 + 2
ad

0 y2-5yj [ 0 -6 _
Y

2x + l = x+ 3, 3y=y2 + 2 and y2-5y=-6


nd
Re

x = 2, y2 - 3y + 2 = 0 and y2 - 5y + 6 = 0
Fi

x = 2, (y-1) (y-2) = 0 and (y-2)(y = 3)=0


=> x = 2, y=l,2 and y = 2, 3
=> x = 2,y = 2 [•.• y = 1, 2 and y = 2,8 => y = 2]
________________EXERCISE 5.1
LEVEL-1
1. If a matrix has 8 elements, what are the possible orders it can have? What if it has 5
elements? [NCERT]
'2 3 -5 2 -1
2. H A = [cijj] 1 4 9 and B = [fy] = -3 4 then find
0 7-2 1 2
(i) (?22 + ^21 (ii) anbn + a22 b22
3. Let A be a matrix of order 3 x 4. If denotes the first row of A and C2 denotes its second
column, then determine the orders of matrices R-[ and C2.

ReadYourFlow.COM
ALGEBRA OF MATRICES 5.7

4. Construct a 2 x 3 matrix A = [Ojj] whose elements cijj are given by:


(i + j)1
(i) cijj = ixj (ii) otj = 2i-j (iii) Ojj = i + j (iv) Ojj
2
5. Construct a 2 x 2 matrix A = whose elements % are given by:
(i) ^
INCERT1
2

(i-2j)2
(iii) Ojj = [CBSE 2002, NCERT EXEMPLAR!
2
(2i + j)2
(iv) ciij = [CBSE 2002]
2
|2/-3/|

w
(V) Oij = [NCERT EXEMPLAR!
2
-3/ + /I [NCERT]

Flo
(vi) Ojj
2
(vii) Ojj = e2lx sin xj

ee
[NCERT EXEMPLAR]

Fr
6. Construct a 3 x 4 matrix A = [Ojj] whose elements Ojj are given by:
(i) Ojj = i + j (ii) Ojj = i-j (iii) Ojj = 2i
for
ur
(iv) Ojj = / (v) citj = ^ |-3/ + /| [NCERT]
ks

7. Construct a 4 x 3 matrix A - [cijj] whose elements cijj are given by:


Yo
oo

i i~j
(i) Ojj = 2/ + (ii) = (iii) Ojj =i
B

J i+j
" 3x + 4y 2 x - 2y 1 _ f 2 2 4'
re

8. Find x, y, a and b if
a+b 2a-b -1 5 -5 -1 •
ou
ad

9. Find x, y, a and b if
Y

2x-3y a-b 3 1 -2 3
1 x + 4y 3a + 4b 1 6 29
nd
Re

10. Find the values of a, b, c and d from the following equations:


Fi

2a+ b a-2b 4
[NCERT]
5c -d 4c + 3d 11 24
11. Find x, y and z so that A =B, where
^ x-2 3 2z " y z 6
A =
18z y + 2 6z , B = 6y a: 2y
x 3x -yl _ F3 2"
12. If , find x, y, z, co.
2x + z 3y - co _ 4 7
x-y z _ I"-1 4’
13. If find x, y, z, co. [CBSE 2014]
2x - y co_ “ L ° 5J'
x +3 z+4 2y -7 0 6 3y - 2
14. If 4x + 6 a-1 0 2x -3 2c + 2 [NCERT]
b-3 3b z + 2c 2b+ 4 -21 0
Obtain the values of a, b, c, x, y and z.

ReadYourFlow.COM
5.8 MATHEMATICS-XII

"2x + l 5x x+ 3 10
15. If , find the value of (x + y). [CBSE2012]
0 y2 +1 0 26

4 '8 w~
16. If then find the values of x, y, z and w. [NCERT EXEMPLAR!
z+ 6 x+y 0 6 '
17. Give an example of
(i) a row matrix which is also a column matrix
(ii) a diagonal matrix which is not scalar
(iii) a triangular matrix.
LEVEL-2
18. The sales figure of two car dealers during January 2013 showed that dealer A sold 5 deluxe,
3 premium and 4 standard cars, while dealer B sold 7 deluxe, 2 premium and 3 standard
cars. Total sales over the 2 month period of January-February revealed that dealer A sold 8
deluxe 7 premium and 6 standard cars. In the same 2 month period, dealer B sold 10 deluxe,

low
5 premium and 7 standard cars. Write 2x3 matrices summarizing sales data for January
and 2-month period for each dealer.
19. For what values of x and y are the following matrices equal?
2x +1 2y x+3
A =

ee
, B =
0 y2 -5y
rF
Fr
20. Find the values of x and y if
x +10 y2 + 2y 3x + 4 3
for
0 -4 0 y2-5y_
ou
21. Find the values of a and b if A =B, where
ks

<7 + 4 3b 2a + 2 b2 + 2
oo

A= /B= [NCERT EXEMPLAR]


8 -6 8 b2-l0
Y
B
re

ANSWERS
ou

1. (i) lx 8, 8xl,2x 4,4x 2 (ii) lx5,5xl


ad

2. (i) 1 (ii) 20 3. 1 x 4, 3 x 1
Y

1 2 3 1 0 -1 2 3 4' 2 9/2 8
4. (i) (ii) (Hi)
nd

(iv)
Re

2 4 6 3 2 1 3 4 5 9/2 8 25/2
Fi

2 9/2' 0 1/2' '1/2 9/2' 9/2 8'


5. (i)
9/2 8 (ii)
1/2 0
(iii)
0 2
(iv)
[ 25/2 18

'1/2 2' 1 1/2'


(v) (vi)
1/2 1 5/2 2

‘2 3 4 5' 0 -1 -2 -3 '2 2 2 2' 1 2 3 4'


6. (i) 3 4 5 6 (ii) 1 0-1-2 (iii) 4 4 4 4 (iv) 12 3 4
4 5 6 7 2 10-1 6 6 6 6 12 3 4
"1 1/2 0 1/2
(v) 5/2 2 3/2 1
4 7/2 3 5/2
'.3 5/2 7/3 ' 0 -1/3 -1/2' 1 1 1
6 5 14/3 1/3 0 -1/5 2 2 2
’ W 9 15/2 7 (ii) 1/2 1/5 (iii)
0 3 3 3
12 10 28/3 3/5 1/3 1/7 4 4 4

ReadYourFlow.COM
ALGEBRA OF MATRICES 5.9

8. x = 2, y = -1, a = 0,b =5 9. x = 2,y=l/fl = 3, &


10. a =l,b =2, c = 3, d = 4 11. x =11,y =9,2= 3
12. x = 3, y = 7,2 = -2,(0 = 14 13. x = 1, y = 2,2 = 4, co = 5
14. fl=-2,& =- 7/c=-l/x = -3,y = -5/2 = 2 15. 7,-3
16. x = 2, y = 4,2 = -6, w =4 or x = 4, y = 2,2 =-6, w = 4
'20 0 4 3 5
17. (i) [5] (ii) 0 3 0 (iii) 0 7 1
0 0-1 0 0 2
Deluxe Premium Standard Deluxe Premium Standard
Dealer A f 5 3 4 Dealer A f 8 7 6
18.
Dealer B 7 2 3 ' Dealer B 10 5 7
19. A and B cannot be equal for any value of y. 20. x = 3, y = 1 21. a = 2,b = 2

low
HINTS TO NCERT& SELECTED PROBLEMS
1. We know that an m x ti matrix has mn elements. Therefore, to find all possible orders of a
matrix with 8 elements, we will have to find all ordered pairs (a, b) such that a and b are

ee
factors of 8. Clearly, all possible ordered pairs of this type are (1, 8), (8,1), (2, 4), (4, 2).
rF
Fr
Hence, possible orders of the matrix are: 1x8, 8x1,2x4, 4x2
If a matrix has 5 elements, then its possible orders are 1x5 and 5x1.
for
(i + ;)2
u
5. (i) Let A = [fl^] be a 2 x 2 matrix such that ——. Then,
2
ks
Yo

(i+D2 (1 + 2)2 9 (2 +1)2 - 9 a _(2 + 2)2


oo

flu = — 2 , fl-^2 — = 2'*21 =^- ~ 2 ' 22 " = 8.


2 2 2
B

flu au ' 2 9/2


re

A
a21 a22 9/2 8
ou

| - 3? + ; |
ad

(vi) Let A = [fly ] be a 2 x 2 matrix such that fly . Then,


Y

2
- 3 + 11 -3 + 2 1 -64-l| 5 - 6 + 21
flU = - 1/ «i2 - = 2
nd

2 2' fl21 2 2 ' *22 ~ 2


Re

2
Fi

' 1 1/2
A = All au
a21 a22 5/2 2
1 1
6. (v) Let A = [fly ] 3 x 4 be a matrix such that cfy = — | - 3i + j |. Then, fly = — | - 3i + j \ gives

flU
= l|-3 + l|=l, a12=^|-3 + 2|=i,fl13=I|-3 + 3|=0,fl14=i|-3 + 4|=i

fl21 = ^ I-6 + 11 = ^ ||-6 + 2|=2,«23=ih6+3|=|, «24=||-6 + 4|=l


2 ' fl22 -

1 1
fl31 = 2 I ~9 + 11 = 4/ fl32 = 2 I 9 + 2| =- “33=11-9+31 = 3, <!34=|l-9 + 4|=!
2'
An fl12 fl13 fl14 ' 1 1/2 0 1/2'
A = fl21 fl22 a23 a24 => A 5/2 2 3/2 1
_fl3l fl32 fl33 aM_ 4 7/2 3 5/2
10. We have.
2a + b a -2b 4 3
5c -d 4c + 3d 11 24, => 2a + b = 4, a -2b =- 3,5c - d =11, 4c + 3d = 24

ReadYourFlow.COM
5.12 MATHEMATICS-XII

Thus,
kA = [kaij]mxn
1 2 5 3 6 15
For example, if A = -2 3 4 , then 3 A = -6 9 12
1 2 -1 3 6-3
6 2 3 1 3/2
If A = 2 3-2 —A = 1 3/2 -1
2 4 1 2 1/2

5.5.1 PROPERTIES OF SCALAR MULTIPLICATION


Various properties of scalar multiplication are stated and proved in the following theorem.
THEOREM IfA=[aij] mxn > B - \bij]m xn are two matrices and k, l are scalars, then
(i) k(A + B) = k A + k B (ii) (k + F) A = k A + l A

w
(hi) (k l) A = k (/ A) = l(kA) (iv) (-k) A = - (k A) = k(- A)
(v) 1 A = A (vi) (-1) A = - A

Flo
PROOF (i) Since A and B are matrices of the same order mxn, A + B is also a matrix of order
mxn. Therefore, k(A + B) is also of order m x n Further, kA and kB are of order m x n. Therefore,

ee
kA + kB is also of order m x n. Thus, k(A + B) and kA + kB are matrices of the same order such

Fr
that
(k (A + B))jj = k(A + B)jj [By definition of scalar multiplication]
for
[By definition of addition of matrices]
ur
=> (k (A + B)),y = k(aij+bij)
=> (k (A + B))jj = k Ojj + k bjj [By distributivity of multiplication over addition]
=> [By definition of scalar multiplication]
ks

(k (A + B))ij = (kA)ij + (kB)jj


Yo

=> (k (A + B))jj = (kA + kB)^ [By definition of matrix addition]


oo

(k (A + B))ij = (kA + kB)ij for / = 1, 2,..., m and ; = 1, 2,...,«


B

Hence, k(A + B) =kA + kB [By definition of equality of two matrices]


re

(ii) Since k and l are scalars, k + lis also a scalar. Therefore, (/c + /) A is a matrix of order mxn.
Also, kA and /A are mxn matrices. Therefore, kA + lA is also an m x n matrix.
ou
ad

Thus, (k + l) A and kA + IA are two matrices of the same order m x n such that
Y

((k + l) A),y = (k + 1) Ojj [By definition of scalar multiplication]


nd
Re

=> ((k + 0 A)ij = kajj + lajj [By distributivity of multiplication over addition]
Fi

=> ((k + l) A)jj = (kA)jj + (lA)ij [By definition of scalar multiplication]


((k + l) A){j = (kA + lA)jj [By definition of addition of matrices]
((k + l) A),y =(kA + lA)^ for / = 1, 2,..., m and /' = 1, 2,..., n
Hence, (k + l) A = kA + IA. [By definition of equality of two matrices]
(iii) Since k and / are scalars, kl is also a scalar. Therefore, (kl) A is an m x n matrix. Also, note that
/A and kA are matrices of order mxn. Therefore, k (IA) and l (kA) are matrics of order m x n.
Thus, (kl) A and k (lA) are two matrices of the same order m x n such that
((kl) A)y = (kl) aij [By definition of scalar multiplication]

=> mA)^ = k^) [By association of multiplication]

=> ((kDA)^ = k (lA)ij [By definition of scalar multiplication]


=> ((kl) A),y = (k (lA))jj [By definition of scalar multiplication]
((kDA)^ = (fc(/A))f/ for f =1, 2,..., m and / = 1,2, ...,n
Hence, (kl) A=k (IA). [By definition of equality of two matrices]

ReadYourFlow.COM
ALGEBRA OF MATRICES 5.13

Similarly, it can be proved that (kl) A =1 (kA).


Hence, (kl) A = k (IA) = l(kA).
(iv) Putting / = - 1 in (iii), we obtain
(-k) A = k(-A) = -(kA)
(v) Putting /c = -1 in (iv), we obtain 1A = A.
(vi) Putting /c = 1 in (iv), we obtain (-1) A = - A. Q.E.D.
5.6 SUBTRACTION OF MATRICES
DEFINITION For tivo matrices A and B of the same order, the subtraction of matrix B from matrix A is
denoted by A - B and is defined as A - B = A + (- B).
-3 2 1 3 5-2
For example, if A = and B = , then
1 -4 7 -1 4 -2
-3 2 1 -3 -5 2' -6 -3 3

w
A-B = A+(-B) = +
1 -4 7 1-4 2 2-8 9
2 3 4 3 0 5
ILLUSTRATION //A= 0 4 6,B=5 3 2 ,find3A-2B.
Flo
ee
5 8 9 0 4 7

Fr
SOLUTION We have, 3A-2B = 3A + (- 2) B for
"6 9 12 ' -6 0 -10 0 9 2
ur
=> 3A-2B = 0 12 18 + -10 -6 -4 -10 6 14
15 24 27 0 -8 -14 15 16 13
ks
Yo
oo

ILLUSTRATIVE EXAMPLES
B

LEVEL-1
re

'2 3"^ and B - 0 5 1'


EXAMPLE 1 If A =
ou

, find A + B and A -B.


ad

1 2 -1 -2 7 3
Y

SOLUTION Clearly, A and B both are matrices of the same order 2x3. So, A + B and A - B
both are defined.
nd
Re

Now,
Fi

'2 3 -5' 0 5 1 '2 + 0 3 + 5 -5 + 1 2 8-4'


A+B = +
1 2 -1 -2 7 3 1-2 2 + 7 -1 + 3 -19 2
'2 3 -5' ‘ 0 5 1 _ "2 3 -5 '0 -5 -1
and. A - B = A + (-B) =
1 2 -1
+ (-l)
-2 7 3j" L1 2 -1 + 2 -7 -3
'2 + 0 3-5 -5-1 2 -2 -6'
]
1 + 2 2-7 -1-3 3 -5 -4
2 } and B = 1 4
EXAMPLE 2 If A = ,find 3A - 2B.
3 1 7 2
SOLUTION Clearly,
6 -3 -2 -8
3A = and, (-2) B =
9 3 -14 -4
3A-2B = 3A + (-2) B =1" ^ -3 -2 -8 6 + (- 2) - 3 + (- 8) '
+
3 -14 -4 9+ (-14) 3 +(-4)
4 -11
-5 -1

ReadYourFlow.COM
5.16 MATHEMATICS-XII

=> Oj: + q: = bjj + Cjj for all 1=1,2,m; ;' = 1, 2,n [Adding Cjj on both sides]
=> (A + C)jj = (B + C)jj for all / = 1, 2,m; j =1, 2,n
=> A+C = B + C
1 2 3-2'
EXAMPLE 10 Find a matrix X such that 2A + B + X = 0, where A = and, B =
3 4 1 5 '
[CBSE 2000]
SOLUTION We have,
2A + B + X = O
X = -2A-B
'-1 2' '3 -2'
=^> X = -2
3 4 .1 5_
2-4' -3 2" '2-3 -4 + 2' -1 -2'
X = + -7 -13
-6 -8 -1 -5 -6-1 -8-5

low
-2 2 O'
EXAMPLE 11 Find a matrix A such that 2A-3B+5C=0, where B = and
3 1 4
'2 0 -2'
C=
7 1 6

ee
SOLUTION We have,
rF
Fr
2A-3B+5C = O
=> 2A = 3B-5C for
f-2 2
O' '2 0 -2'
=> 2A = 3
3 1 4
-5
7 1 6
ou
ks

-6 6 0 -10 0 10'
=> 2A =
9 3 12
+
-35 -5 -30
oo
Y

'-6-10 6 + 0 0 + 10
eB

=> 2A =
9- 35 3 -5 12- 30
-16 6 10'
r

=> 2A =
ou
ad

-26 -2 -18
Y

1 r-16 6 10" -8 3 5'


=> A = -
2-26 -2 -18 -13 -1 -9 '
d
Re

x2 r-2i
n

EXAMPLE 12 Solve the matrix equation -3 AV =


Fi

2.V 9 ‘
[y2J
SOLUTION We have,

= r-2i
x2
3^1
L-/2J 2y. 9
x2 3x 21
=>
b2J 9

=>
x2 - 3x r- 2i
y2 - 6y. 9

=> x2 - 3x = -2 andy2 -by = 9


=> x2 - 3x + 2 = 0 and y2 6y - 9 = 0
6±p6+ 36
=> (x -1) (x - 2) = 0 and y =
2
x =1,2 and y = 3 ± 3 V2.

ReadYourFlow.COM
ALGEBRA OF MATRICES 5.17

EXAMPLE 13 Two farmers Ram Kishan and Gurcharan Singh cultivate only three varities of rice namely
Basmati, Permal and Naura. The sale (in ?) of these varities of rice by both the fanners in the month of
September and October are given by thefollowing matrices A and B
September sales (in
Basmati Permal Naura
[10,000 20,000 30,0001 Ram Kishan
50,000 30,000 10,000 Gurcharan Singh
October sales (in
Basmati Permal Naura
'5,000 10,000 6,000 " Ram Kishan
B =
20,000 10,000 10,000 Gurcharan Singh
Find:

w
(i) What were the combined sales in September and October for each farmer in each variety.
(ii) What was the change in sales from September to October?

Flo
(iii) If both farmers receive 2% profit on gross rupees sales, compute the profit for each farmer and for
each variety sold in October.

ee
SOLUTION (i) The combined sales in September and October is given by A + B.

Fr
Clearly,
Basmati Permal Naura
or
ur
"10,000 + 5000 20,000+10,000 30,000+6000 ] Ram Kishan
sf
A + B = 50,000 + 20,000 30,000 + 10,000 10,000 + 10,000j Gurcharan Singh
k
Yo

Basmati Permal Naura


oo

"15,000 30,000 36,0001 Ram Kishan


A+B=
B

=> 70,000 40,000 20,000 Gurcharan Singh


re

(ii) The change in sales from September to October is given by A - B.


ou
ad

Clearly,
Y

Basmati Permal Naura


'10,000-5,000 20,000-10,000 30,000 - 6000 ' Ram Kishan
nd
Re

A-B =
50,000-20,000 30,000-10,000 10,000-10,000 Gurcharan Singh
Fi

Basmati Permal Naura


'5,000 10,000 24,000 Ram Kishan
A-B =
30,000 20,000 0 Gurcharan Singh
(iii) The profit for each farmer and for each variety sold in October at the rate of 2% of gross sale
is given by
Basmati Permal Naura
5,000 10,000 6,000 Ram Kishan
2% of B = x B = 0.02 B = 0.02
100 20,000 10,000 10,000 Gurcharan Singh
Basmati Permal Naura
"100 200 120 "I Ram Kishan
400 200 200 Gurcharan Singh
Thus, in October Ram Kishan receives ? 100, ? 200 and ? 120 as profit in the sale of each variety of
rice respectively, and Gurcharan Singh receives profit of ? 400, ? 200 and ^ 200 in each variety of
rice respectively.

ReadYourFlow.COM
5.18 MATHEMATICS-XII

EXERCISE 5.2
LEVEL-1
1. Compute the following sums:
2 13 1 -2 3"
("3 -21 F-2 4
(>) i 4 + 1 3 (ii)
0 3 5 + 2 6 1
-12 5 0-3 1
2 4 1 3 -2 5
2. Let A = ,B = and C = . Find each of the following:
3 2 -2 5 3 4
(i) 2A-3B (ii) B-4C (hi) 3A- C (iv) 3A-2B+3C
2 3 -10 2 -12 3
3. If/1 = /B= /C= , find
5 7 3 4 1 2 1 0
(i) A + B and B + C (ii) 2B + 3A and 3C - 4B.

low
-1 0 2' T 0 -2 5 and C- \' -5 2
4. Let A = 'B= i_ -3 1 . Compute 2A-3B + 4C.
3 1 4 6 0-4
5. If A = diag (2 - 5 9), B = diag (1 1 - 4) and C = diag (-6 3 4), find
(i) A-2B (ii) B + C-2A (hi) 2A + 3B-5C

ee
6. Given the matrices
rF
Fr
2 1 1 9 7-1 2-4 3
A = 3-10 , B = 3 5 4 and C = 1 -1 0 for
0 2 4 2 1 6 9 4 5
ou
Verify that (A + B) + C = A + {B + C).
ks

5 2 3 6
7. Find matrices X and Y, if X + Y =
0 9
and X - Y =
0 -1 } [NCERT]
oo
Y
eB

3 2 1 0
8. Find X, if Y = and 2X + Y = [NCERT]
1 4 -3 2 •
r

6-6 0 3 2 5
ou
ad

9. Find matrices X and Y, if 2X - Y = and X + 2Y =


-4 2 1 -2 1 -7
Y

1 1 1 3 5 1
d
Re

10. If X - Y = 1 1 0 andX + Y = -1 1 4 , find X and Y.


n

1 0 0 11 8 0
Fi

1 2 -1 9-14'
11. Find matrix A, if +A=
0 4 9 -2 1 3 ’
9 1 1 5
12. If A = ,B= , find matrix C such that 5A + 3B + 2C is a null matrix.
7 8 7 12
2 -2 8 O'
13. If A = 4 2 , B = 4 -2 , find matrix X such that 2 A + 3 X =5 B.
-5 1 3 6
'1 -3 2' An \2 -1 | , find the matrix C such that A + B + C is zero
14. IfA = and, B = 1 0
2 0 2
matrix.
15. Find x, y satisfying the matrix equations

(i)
x-y 2 -2
4 x 6 H 3-2
1 0 -1
21 _
"
6 0 0
5 2x + if 5

ReadYourFlow.COM
ALGEBRA OF MATRICES 5.19

(ii) [x y + 2 z - 3] + [y 4 5] = [4 9 12]
-8
(iii) +y -11
=o INCERT EXEMPLAR]

3 4 i y 7 0
16. If 2 + , find x and y.
5 x 0 1 10 5

17. Find the value of A,, a non-zero scalar, if A,


'10 2
3 4 5 I 1 2 3l_r4 4 lO]
+ 2 -1 -3 2j _[4 2 14

18. (i) Find a matrixX such that 2A + B + X =0, where A =

[CBSE 2000]
-1 2
3 4 i -2
5

'8 O' 2-2'


(ii) If A = 4 - 2 and B = 4 2 , then find the matrix X of order 3x2 such that

low
3 6 -5 1
2A + 3X =5B. [NCERT]
19. Find x, y, z and f, if
* y x 6 4 x+y ]
+ [NCERT]

ee
(i) 3 z t -1 2t z+f 3 |
rF
Fr
<*> ^7 y-3 3 4 _ ' 7 14
+ [NCERT EXEMPLAR, CBSE 2002,2012]
1 2 J “ 15 14
for
20. If X and Y are 2 x 2 matrices, then solve the following matrix equations for X and Y.
ou
'2 3" -2 2~
ks

2X+3Y = , 3X + 2Y = [NCERT EXEMPLAR]


4 0 1 -5
oo

21. In a certain city there are 30 colleges. Each college has 15 peons, 6 clerks, 1 typist and 1
Y
eB

section officer. Express the given information as a column matrix. Using scalar
multiplication, find the total number of posts of each kind in all the colleges.
22. The monthly incomes of Aryan and Babban are in the ratio 3 : 4 and their monthly
r
ou
ad

expenditures are in the ratio 5 : 7. If each saves ? 15000 per month, find their monthly
incomes using matrix method. This problem reflects which value? [CBSE 2016]
Y

ANSWERS
d
Re
n

3-16'
1 2
Fi

1. (i) (ii) 2 9 6
2 7
-1 -1 6
1 -1 9 -17' 8 7 -2 21
2. (i) (ii) (iii) (iv)
12 -11 -14 -11 6 2 22 8
-2 2 5 '
3. (i) A + B does not exist, B + C =
5 5 1 '
1 6 1 2 -14 -3"
(ii) 2B + 3A does not exit, 3C - 4B = 4.
-6 -13 -4 27 11 -11
5. (i) diag(0 -7 17) (ii) diag (-9 14 - 18) (iii) diag (37 - 22 - 14)
4 4 Y = 1 -2
7. X = 8.
0 4 '1 0 5 -2 -1

2 3 1 1 2 0
_ T 3 -2 1 0 2 2
9. X ,Y = 10. X = 0 1 2 ,Y = -10 2
= L -2 1 -1 0 0-3
6 4 0 5 4 0

ReadYourFlow.COM
5.20 MATHEMATICS-XII

12 4/3
8-3 5 -24 -10
11. 12. 13. 4 -14/3
-2 -3 -6 -28 -38
25/3 28/3
-3 4 -1 3 3
14. 15. (i)x = ^,y = - (ii) .v =1,1/= 3,2 = 10 (iii) x =1, y = 2
-3 0 -1 2' 2
-2 -10/3
-1 -2
16. x = 2, y = - 8 17. X = 2 18. (i) 4 14/3
-7 -13 (ii)
-31/3 -7/3
-2 O' '2 1'
19. (i) x = 2,y = 4,f = 3,2=1 (ii)x = 2,y = 9 20. X = ,Y =
-1 -3 2 2
[151 450'
6 180
21. A = , 30A = 22. ? 90,000, ? 120,000
1 30

w
1 30

HINTS TO NCERT& SELECTED PROBLEMS


7. We have.

Flo
ee
[5 21 3 6
X+Y = and X-Y =

Fr
0 9 0 -1
[5 21 3 6 [5 21 3 6
=> (X + Y) + (X - Y) = + 0 x and, (X + Y)-(X-Y) =
for
ur
0 9 0 9 0 -1
[8 81 '2 -4'
^ 2X = and 2Y =
ks

0 8 0 10
Yo
oo

1 rs 8“ 1 [2 -4" [4 41 1 -2'
X = - and Y = - => X = and Y =
B

2 0 8 2 0 10 0 4 0 5
re

3 2 ' 1 O'
8. We have, Y = and 2X + Y =
1 4 -3 2
ou
ad

'3 21 f 1 O'
Y

2X +
1 4 "-3 2
nd

'i oirs 2'


Re

-2 -2' '-2 -2" -1 -1


=> 2X = => X = -
-3 2 14 -4 -2 2 -4 -2 -2 -1
Fi

18. (ii) We have.


8 O' 2 -2
A = 4 -2 / B = 4 2 and, 2A + 3X = 5B.
3 6 -5 1
Now, 2A + 3X=5B
'8 O' 2 -2
=> 2 4 -2+3X = 5 4 2
3 6 -5 1
'16 0" 10 -10
=> 8 - 4 + 3X = 20 10
6 12 -25 5
10 -10' '16 0 -6 -10 -2 -10/3
=> 3X = 20 10 8 -4 12 14 => X = 4 14/3
-25 5 6 12 -31 -7 -31/3 -7/3

ReadYourFlow.COM
ALGEBRA OF MATRICES 5.21

19. (i) We have.


r* yl _ r * 6' +
'4 x + xi
z f -1 It z+t 3
~3x 3y x+4 x+y+6
^ 3z 3t 1+z + i 2.t + 3
=> 3a = a + 4, 3y = a + y + 6, 3z = -1 + z +1, 3t = 2t + 3
=> 2a = 4, a - 2y + 6 = 0, 2z - f + 1 = 0, i = 3
=> a = 2, y = 4, z=l,f = 3
(ii) We have.
"3 41 _ f 7 14
2 r*L7 y~3.
+5' 1 2 " 15 14

"2a+3 14 ' '7 14"

low
=> => 2a+ 3 = 7, 2y - 4 =14 => a = 2, y = 9
15 2y-4 15 14

5.7 MULTIPLICATION OF MATRICES


Let us first define the product of a row matrix and a column matrix.

ee
rF
Fr
Let A =[cii fl2 • • • tyil a row rnatrix and B= f be a column matrix. Then, we define
for
bn
ou
AB = flj ^2 bn '
ks

5
oo

For example, if A =[1 - 2 3 4] and B = ^ . Then,


Y
eB

1
-2
r

5
ou
ad

AB = [1 -2 3 4] ^ = 1x5+(-2) (-4) + 3xl + 4x(-2)=5 + 8+ 3-8=8


Y

-2
d
Re
n

Using the product of a row matrix and a column matrix, let us now define the multiplication
Fi

of any two matrices.


DEFINITION Two matrices A and B are conformable for the product AB if the number of columns in A
(pre-multiplier) is same as the number of rows in B (post-multiplier).
Thus, if A = x n and B = |fy]„ x p are two matrices of orders m x nand nx p respectively, then their
product AB is of orders mx p and is defined as
(AB)jj = (;'th row of A) (;th column of 8) for all i =1, 2,..., mand=1, 2,..., p.

by
b2j
=> (ABfj = [an ^ ••• %,]

bnj
n
=> (AB)jj = Oj^ by + Oj2 b2j + . .. + airbrj+...+ ainbnj = brj

NOTE If A and B are two matrices such that AB exists, then BA may or may not exist.

ReadYourFlow.COM
5.22 MATHEMATICS-XII

2 1 3 1 -2
ILLUSTRATION! IfA = 3 -2 1 and B = 2 1 , then Aisa 3x 3 matrix and Bisa 3* 2
-1 0 1 4 -3
matrix. Therefore, A ami B are conformablefor the product AB and it is of order 3x2 such that
= (First row of A) (First column of B)
1
=> (AB)n =[213] 2 = 2x1+1x2+3x4 = 16
4
(AB)i2 = (First row of A) (Second column of B)
-2'
=> (AB)12= [21 3] 1 = 2 x-2 + 1x1 + 3 x-3 =-12
-3

w
(AB)2i = (Second row of A) (First column of B)
1
=> (AB)21 = [3-21] 2 =3xl+(-2)x2 + lx4 = 3
4

Flo
ee
Similarly, we obtain

Fr
(AB)22 =-ll/(AB)31 =3 and (AB)32=-1.
'16 -12'
for
ur
AB = 3 -11
3 -1
ks
Yo

NOTE In this case BA does not exist, because the number of columns in B is not same as the number of
oo

rows in A.
eB

2 3
ILLUSTRATION 2
n -22
Let A = 3 -1 2 . Find AB and BA and show that AB* BA.
r

4 -5
ou
ad
Y

SO! UTION Here, A is a 2 x 3 matrix and B is a 3 x 2 matrix. So, AB exists and it is of order 2x2.
2 3
nd

'1-2 3 '2 + 2 + 12 3-4-151 _ [16 -16'


Re

AB = -1 2
3 2-1 6-2-4 9 + 4 + 5 “ 0 18
Fi

4 -5
Again, B is a 3 x 2 matrix and A is a 2 x 3 matrix. So, BA exists and it is or order 3x3.

BA = ^p
if32 -1
3
2+9 -4+6
-1+6
6-3
2+4 -3-2
11
5
2
6-5
3

4 -5 L 4-15 -8-10 12 + 5 -11 -18 17


Clearly, AB * BA.
5.7.1 PROPERTIES OF MATRIX MULTIPLICATION
THEOREM 1 Matrix multiplication is not commutative in general.
PROOF Let A and B be two matrices such that AB exists. Then it is quite possible that BA may not
exist. For example, if A is a 3 x 3 matrix and B is a 3 x 2 matrix, then AB exists but BA does not
exist. Similarly, if BA exists, then AB may not exist. Further, if AB and BA both exist, then they
may not be equal as shown in illustration 2 (given above).
Hence, in general, AB * BA. Q.E.D.

ReadYourFlow.COM
ALGEBRA OF MATRICES 5.23

THEOREM 2 Matrix multiplication is associative i.e. {AB) C - A (BC),whenever both sides are defined.

PROOF Let A = [oij] mxn' B = \bjj]n * p and C = [c,y] p x Then AB is anm x p matrix and so (AB) C is
amx q matrix. Clearly, BC is of order nx q and so A (BC) is of order mx q. Thus, (AB) C and
A (BC) are of the same order.
P
Now, ((AB) Qq = (AB)jr (C)rj

n
P 2 sl1
((AB) Oq = ^ s2! bsr J cr; = ^ ^ (“is bsr) ^rj
=>

=> ((AB) Oq = ^^Crj) [By associativity of multiplication of numbers]

((AB) Oij = l^s r^(bsrcrj) = 1^(60)^ =(A(BO)ij for all i, j


=>

low
Thus, (AB) C and A (BC) are matrics of the same order such that their corresponding elements
are equal.
Hence, (AB)C=A(BC).

ee
Q.E.D.
rF
Fr
THEOREM 3 Matrix multiplication is distributive over matrix addition i.e.
(i) A(B + C) = AB + AC for
(ii) (A + B)C = AC + BC whenever both sides of equality are defined.
ou
PROOF Let A = fy] mxn' B = \bjj]n x p and C = [Cjj]n x p be three matrices. Then, B + C is of order
ks

nx p and so A (B + C) is of order m x p. Since AB and AC both are of the same order m x p.


oo

Therefore, AB + AC is of order m x p.Thus, A (B + C) and AB + AC are of the same order mxp


Y

such that
B

n
re

(A(B + C))ij = ^ %(B + C)r/


ou
ad

=> (A (B + C))ij = 1 ^ 0ir (brj + crj)


Y

n n
nd
Re

=> (A (B + C))jj = ajr brj + S 0jr Cjj


r=1
Fi

=> (A (B + C))ij = (AB)ij + (AOq = (AB + AOq for all i, j


Thus, A (B + C) and AB + AC are two matrices of the same order such that their corresponding
elements are equal.
Hence, A (B + C) = AB + AC.
Similarly, (A + B) C = AC + BC.
Q.E.D.
THEOREM 4 If A is an mxn matrix, then Im A = A = A In.

PROOF Let A = [aij]m xn. Then, A and A/„ are of the same order mxn such that
m
(ImA)ij = ^ (Im)ir(A)rj

m
=> (hn (fm)ir “rj

=> (Im A)q = (Im)i\ a\j + (hn)i2 a2j + ••• + (fn)a “ij + ••• + (^m)im “mj

ReadYourFlow.COM
5.24 MATHEMATICS-XII

0 for r ^ i
=> dm Vij Ojj for all i, j dm)ir ~ 1 for r = i

Hence, lm A = A.
n n
Now, ~ r^] (A)ir dn)rj ~ 1air dn)rj

=> 4)y = tyl dn)lj + ‘Hi dn)lj + ••• + dn)jj +.. • + ain dn)nj
=> ^ Uy aij for all i, j ['•' dn)jj = 1 and (/„)r;- = 0 for r */]
Thus, A/m and A are matrices of the same order such that their corresponding elements are
equal. So, A/„ = A.
Hence, lmA = A = A/„ . Q.E.D.
REMARK 1 The product of two matrices can be the null matrix while neither of them is the null matrix.
ro 2i '1 0" ro oi
For example, ifA = and B = , then AB = while neither A nor B is the null matrix.
0 0 0 0 0 0

low
THEOREM 5 If A is m x n matrix and O is a null matrix, then
(i) Am x„ O, 1 X p = Om xp (ii) Op x m Am x n = O pxn
i.e. the product of the matrix with a null matrix is always a null matrix.

ee
rF
PROOF (h Let A =[fl,y] m xn and O, i xp = Pty'ln x p / where by = 0 for all i, j. Then, A,„ xn On x p is an

Fr
mx p matrix such that
n
for
(Am ^ On xp)ij — ^ ^ ‘Hr Kj ~ ^ ^or Fj [by = 0 for all i, j]
ou
ks

Thus, AmxnOnXp andOm xp are two matrices of the same order such that their corresponding
elements are equal.
oo

Hence, Amxn.Onxp = Onxp-


Y
B

(ii) Proceed as in (i). Q.E.D.


re

REMARK 2 In the case of matrix multiplication if AB -O, then it does not necessarily imply that
ou

BA=0.
ad

ro ii a oiro i] fo i]
Y

1 0
For example, if A = and B = Then, AB = O. But, BA = * O.
0 0 0 0 0 0 0 0 0 0
nd
Re

Thus, AB=0 while BA^O.


Fi

5.7.2 POSITIVE INTEGRAL POWERS OF A SQUARE MATRIX


For any square matrix, we define
(i) A1 = A and, (ii) A n+ 1
= An ■ A, where n e N.
It is evident from this definition that A2 = AA, A 3 = A2 A = (AA) A. etc.
It can be easily shown that
(i) Am A” = A m + n and, (ii) (Am)n =A nin for all m, n e N.
MATRIX POLYNOMIAL Letf (x) -a0 xn + a^ x” 2 x + au be a polynomial and let
Abe a square matrix of order n. Then,
f(A) = a0An + ai A n-1 + a2 A n- 2 + .. + On_■[ A + an In

is called a matrix polynomial.


For example, if / (x) = x - 3 x + 2 is a polynomial and A is a square matrix, then
f(A) = A2 - 3A + 2/isa matrix polynomial.

ReadYourFlow.COM
ALGEBRA OF MATRICES 5.25

ILLUSTRATIVE EXAMPLES

LEVEL-1

Type I ON MULTIPLICATION OF MATRICES


a h g x
EXAMPLE 1 IfA, B, Care three matrices such that A =[xyz], B = h b f , C= y , find ABC.
g f c z
SOLUTION Since the product of matrices is associative. Therefore, we can find ABC either by
computing (AB) C or by computing A (BC). Let us compute A (BC).
Since B is a 3x3 matrix and C is 3 x 1 matrix. Therefore, BC is of order 3x1.
a h g~\[x ax + hy + gz
Now, BC = h b f y = hx +by+fz
g f CJL2 gx + fy + cz

low
Clearly, A is of order 1x3 and BC is of order 3x1. Therefore, A (BC) is of order 1x1.
Now,
ax + hy + gz
A (BC) = [x y z] hx + by + fz

ee
gx + fy + cz
rF
Fr
=> A (BC) = x (ax + hy + gz) + y (hx + by +fz) + z(gx + fy + cz)
=> A (BC) = ax2 + 2hxy + by2 + cz2 + 2 fyz + 2 gzx
for
1 -1 2 1
3 'B = 1 0 , prove that (A + B)2 * A2 + 2AB + B2.
ou
EXAMPLE 2 IfA= 2
ks

SOLUTION We have,
oo

2 1
2 "3]^ =
Y

A =
B

1 0
re

1 -1 -1 -4
A2 —
2 3 8 7
ou
ad

2 1 1 1
MU
Y

1 0 7 2
nd

2 2
Re

=> 2AB =
14 4
Fi

2 1 2 1 5 2
B2 = BB =
1 0 1 0 2 1
2 1 3 0
A+B = l 3 + 1 0 3 3
3 0 3 0 9 0
=> (A + B)2 =(A + B)(A + B) = -(i)
3 3 3 3 18 9
-1 -4 2 2 r5 21 6 0
Also, A2 + 2AB+B2 = + 14 4 + 2 1 24 12 •••(ii)
8 7
From (i) and (ii), we obtain that (A + B)2 * A2 + 2AB + B2.
1 -1 , B = ^ and (A + B)2 =A2 + B2,find a and b.
EXAMPLE 3 IfA= 2
-1
SOLUTION We have, (CBSE 2015]
(A + B)2 = A2 + B2

ReadYourFlow.COM
5.26 MATHEMATICS-XII

=> (A + B) (A + B) = + B2
=> (A + B) A + (A + B) B = A2 + B2 [By distributive law]
=> A2 + BA + AB + B2 = ,42 + B2 [By distributive law]
=> BA + AB = O
a 1 1 -1 1 -1 a 1 0 0
=> +
b -1 2 -1 2 -1 b -1 0 0
a + 2 -a -1 a-b 2 [001
=> +
b-2 -b+1 2a-b 3 0 0
2<?-fr+ 2 -a + 1 TO 01
=>
2a-2 -i> + 4 0 0
=> 2a - b + 2-0, -(7 + 1=0, 2a-2 = 0 and -b + 4 = 0
=> a = l, b = 4

low
0 1
EXAMPLE 4 IfA = / find x and y such that (xl + yA)2 = A.
-1 0
SOLUTION We have,
A =
' o r

ee
-1 0
rF
Fr
xl + yA = x
ri oi o i
0 1 -1 0 for
[x 01 0 y-
=> xl + 1//1 = +
0 x
ou
-y 0
ks

x y
xl + yA =
oo

-y x
Y

Now, {xl + yA)2 = A


eB

=> y ^ y 0 r
_-y x] v-y x -1 0
r
ou
ad

rx 2 - y 2 2xy 0 1
Y

-2xy x2 -y2J L"1 °.


d

=> x2-y" = 0, 2xy = 1, -2xy = -landx2-y2 = 0


Re
n

x2 - y2 =0 and 2xy = 1
Fi

=>
x = ± y and 2xy = 1
Now two cases arise.
CASE I When x = y and 2xy = 1
In this case, we have
9 1
x = y and 2xy = 1 => 2x = 1 => x = ± -7=
V2
1 I 1 1
x = 72andi/ = 72jorr‘ = "72andl/ = ~ V2
CASE II When x = - y and 2xy = 1
In this case, we have
x = - y and 2xy = 1 => - 2x2 =1 => x = ± -J=
V2
7 7 7 7
x = -7= and y = - or, x ’Tfandy = ^
V2 v^2 V2

ReadYourFlow.COM
ALGEBRA OF MATRICES 5.27

fa 01 01 . 2
EXAMPLES lfA = and B = ^ ^ ,find the values of a for which A -B.
1 1
SOLUTION We have,
A2 = B
a 0 a 0 ri oi
=> 1 1 5 1
1 1
a2 + 0 0+0 _ '1 O'
=> ” 5 1
a +1 0 + 1
a2 o ri oi
=> " 5 i
a +1 1
=> a2 = 1 and a +1 =5
=> a = ± 1 and a = 4, which is not possible.

w
9
Hence, there is no value of a for which A = B is true.
[2 -1 f5 21 '2 5'
EXAMPLE 6 Let A = ^ 4

Flo
,B = 7 4 ,C = 3 8 '

ee
Find a matrix D such that CD - AB = O. [NCERT, CBSE 2017]

Fr
a b
SOLUTION Let D = . Then,
x y.
for
ur
CD - AB = O
=> CD = AB
ks

2 -Ilfs 2'
Yo

'2 Slffl b
oo

=> 3 4j |_7 4
3 8_|[x y}
B

’ 2a + 5x 2b + 5y r 10-7 4-4'
=>
re

3a + 8* 3b + 8y 15 + 28 6 + 16
ou

2a + 5x 2b + 5y ’ 3 O'
ad

=> 43 22
3a + 8x 3b + 8y
Y

=> 2a +5x = 3, 3a + 8.r - 43, 2b + 5y = 0 and 3b + 8y = 22


nd
Re

Solving 2a + 5x = 3 and 3a + 8x - 43, we get: a = -191 and x - 77.


Fi

Solving 2b + 5y = 0 and 3b + 8y = 22, we get: b = -110 and y = 44.


a b -191 -110'
D = 77 44
x y.
1 3 2 1
EXAMPLE? Find the value ofx such that [1 x 1] 2 5 1 2 = 0.
15 3 2 x
[CBSE 2006, NCERT EXEMPLAR]
SOLUTION We have.
13 2 1
[1 x 1] 2 5 1 2=0
15 3 2 x
7 + 2x
=> [1x1] 12+ x = 0
21 + 2x

ReadYourFlow.COM
5.28 MATHEMATICS-XII

=> 7 + 2x + 12x + x2 + 21 + 2x = 0
x2 + 16 x + 28 = 0
=> (x + 14) (x + 2) = 0 => x = - 2 or -14.
1 0 1 0
EXAMPLES 1/A = and I = , then find k so that A2 = 8y4 + kl.
-1 7 0 1
1 0
SOLUTION We have, A = [CBSE 2005]
-1 7
A2 = AA = 1 0 1 0 1 0"
=
-1 7 -1 7 -8 49
1 0 1 0 8 0 k 0 8+k 0
and. 8A + kI =8 +k + =
-1 7 0 1 -8 56 0 k - 8 56 + A:
A2 = 8A + kl

w
1 O' 8+k 0
-8 49 - 8 56+ k
=> 1 = 8 + A:

Flo
and 56 + Ac = 49 => Ac = -7.

ee
2 -1 -1 -8 -10
EXAMPLE 9 If 1 0 A = 1-2 -5 ,find A.

Fr
|_-3 4 [NCERT EXEMPLAR]
9 22 15
SOLUTION Since the product matrix is a 3 x 3 matrix and the premultiplier of A is a 3x2
for
ur
matrix. Therefore, A is 2 x 3 matrix.
Let A = ^ ^ ^ . Then, the given equation becomes
ks
Yo
oo

2 -1 -1-8 -10'
xyz
B

1 0 1 -2 -5
a b c
re

-3 4 9 22 15
ou

2x-a 2y-b 2z-c -1 -8 -10


ad

=> x y z 1 -2 -5
Y

- 3 x+ 4a -3i/ + 4A? -3z + 4c 9 22 15


nd
Re

=> 2 x - a = -1, x = 1, - 3 x + 4 fl = 9, 2 y-b =-8,y = -2,


Fi

- 3 y + 4 b =22, 2z-c=-10, z =-5, -3z + 4c=15


=> x = 1, a = 3, y = -2, & =4,z = —5 and c = 0
'1 -2 -5
Hence, A =
3 4 0
Type II ON MATRIX POLYNOMIALS AND MATRIX POLYNOMIAL EQUATIONS
[2 0 1'
EXAMPLE 10 Let f (x) =x2 -5x + 6. Find f (A), if A = 2 13.
1 -1 0
SOLUTION First, we note that by/(A) we mean the matrix polynomial A 2 - 5A + 6/3. That is, to
obtain/(A), x is replaced by A and the constant term is multiplied by the identity matrix of order
same as that of A.
2 0 1 2 0 1
Now, A2 = AA = 2 1 3 2 1 3
1 -1 0 1 -1 0

ReadYourFlow.COM
ALGEBRA OF MATRICES 5.29

'4+0+1 0+0-1 2+0+0 5-1 2


=> A1 = 4+2+3 0+1-3 2+3+0 9-2 5
2-2+0 0-1+0 1-3+0 0 -1 -2
f(-5)x2 (-5) x 0 (-5) x 1 '-10 0 -5
-5A = (-5) x 2 (-5) x 1 (-5) x 3 -10 -5 -15
(-5) x 1 (-5) x (-1) (-5) x 0 -5 5 0
'1 001 [600'
and. 6/0=6 0 1 0 0 6 0
0 0 1 0 0 6
'5 -1 2' -10 0 -5 6 0 0
f(A)=A2-5A + 613= 9-25 + -10 -5 -15 + 0 6 0
0 -1 -2 -5 5 0 0 0 6
1 -1 -3

w
=> / (A) =A2 -5A + 6/3 = -1 -1 -10
-5 4 4

EXAMPLE 11 IfA =
3 1
-1 2
Flo
,show that A2 - 5A + 7I2 =0.

ee
3 1

Fr
SOLUTION We have, A = -1 2
3 1 3 1 9-1 3+2 8 5"
for
ur
/i2 =
-1 2 -1 2 3-2 -1+4 -5 3
(-5) x 3 (-5) x 1 1 _ '-15 -5
ks

—5 A —
Yo

(-5) x (-1) (-5) x 2 5-10


oo

7 0
= 7 10 1°
B

7J2 0 7
re

8 5 -15 -5 7 0
A2-5A+7I2 = + +
-5 3 5 -10 0 7
ou
ad

8-15 + 7 5-5 + 0 0 0
Y

=> A2 -5A + 7I2 =


-5 + 5 + 0 3-10 + 7 0 0
nd
Re

Type III ON PRINCIPLE OF MATHEMATICAL INDUCTION


Fi

The Principle of Mathematical Induction:


Let P (n) be a statement involving positive integer n such that
(i) P (1) is true i.e. the statement is true for n = 1, and
(ii) P (m +1) is true whenever P (m) is true i.e. the truth of P (m) implies the truth of P (m +1).
Then, P (n) is true for all positive integer n.
EXAMPLE 12 Prove the following by the principle of mathematical induction:
-4 ’ l+ 2n - 4» " [NCERT]
, then An = for every positive integer n.
-1 1 -2n
SOLUTION We shall prove the result by mathematical induction on n.
STEP 1 When n = 1, by the definition of integral powers of a matrix, we have
-4 [1 + 2(1) -4(1)-
1 -1 1 1-2(1)
So, the result is true for n = l.
STEP 2 Let the result be true for n = m. Then,

ReadYourFlow.COM
5.30 MATHEMATICS-XII

1 + 2m - Am
Am = -(i)
m 1 - 2m
Now we will show that the result is true for n = m + 1.
Am+1 1 + 2 (m + 1) - 4 (m + 1)
i.e.
m +1 1 - 2 (m + 1)
By the definition of integral powers of a square matrix, we have
/T+1 = Am A
Am+1 1 + 2m - 4m 3 -4
=> [By supposition (i)]
m 1 - 2m 1 -1
3 + 6m - Am - 4 - 8m + Am
=>
3 m + 1 - 2m - Am -1 + 2m
3 + 2m - 4 - 4m "
A /« + 1

w
=>
m + 1 -1 - 2m
1 + 2(m + l) - 4 (m + 1)

Flo
=> Am+l
m +1 . - 2 (m + 1)

ee
This shows that the result is true for n = m + 1, whenever it is true for n = m.

Fr
Hence, by tire principle of mathematical induction, the result is valid for any positive integer n.
EXAMPLE 13 IfAaJ COSa <ma
, then prove that
for
ur
^ u- L - sm a cos a
cosn a smn a
(i) Aa Ap - Aa + p
ks

(ii) (Aaf = -smn a cosn a for every positive integer n.


Yo
oo

[NCERT, CBSE 2004]


B

SOLUTION (i) We have.


re

cos a sin a cos P sin P ’


- sin a cos a - sin p cos P
ou
ad

cos a cos P - sin a sin p sin a cos P + cos a sin p *


Y

=> - - sin a cos P - cos a sin p cos a cos P - sin a sin p


nd

cos (a + P) sin (a + P) ‘
Re

=> Aa A^ - = A a + p-
-sin(a + P) cos(a + P)
Fi

(ii) We shall prove the result by mathematical induction on n.


STEP l When n = 1, by the definition of integral powers of a matrix, we have
cos a sin a cos (1. a) sin (1. a) ’
(Aa)1 = A
-=[
- sm a cos a
So, the result is true for n = 1.
- sin (1. a) cos (1. a)

STEP 2 Let the result be true for n = m. Then,


cos ma sin ma
(A*)m = - sin ma cos ma
Now we will show that the result is true for n = m + 1.
m+ \ cos (m + 1) a sin (m + 1) a "
i.e. (^a)
- sin (m + 1) a cos (m + 1) a
By the definition of integral powers of a square matrix, we have
m+ 1
Ma) = (Ax)W

ReadYourFlow.COM
ALGEBRA OF MATRICES 5.31

w+ 1 cos m a sin m a cos a sin a


=> (^a) [By assumption (i)]
- sin m a cos m a - sin a cos a
m+ 1 cos m a cos a - sin in a sin a cos in a sin a + sin in a cos a
=> (Aa) - sin ma sin a + cos m a cos a
- sin m a cos a - cos m a sin a
m+ l cos {in a + cx) sin (in a + a) cos (in + 1) a sin (m + 1) a
=> (^a)
- sin (m a + a) cos (in a + a) - - sin (in + 1) a cos (m + 1) a
This shows that the result is true for n = m + l, whenever it is true for n = m.
Hence, by the principle of mathematical induction, the result is valid for any positive integer n.
EXAMPLE 14 Ifa is a non-zero rent or complex number. Use the principle of mathematical induction to
prove that:
a 1 -1
lfA = 0 a , then A” = for every positive integer n.
0 a"

w
a 1
SOLUTION We have, A =
0 a

Flo
STEP 1 When n = 1, by the definition of integral powers of a matrix, we have
l-l

ee
a 1 a1 U« )
A^ — A

Fr
0 a 0 a1
So, the result is true for n = 1.
for
ur
STEP 2 Let the result be true for n = m. Then,
a"' me/"-1'
ks

Am = ...(i)
Yo

0 o’"
oo

Now we will show that the result is true for n=m + 1.


B

(m +1) am
re

i.e.
Am+\
0
ou
ad

By the definition of integral powers of a square matrix, we have


Y

A in + 1 = Am A
nd
Re

A"1 + 1 a™ nia"'- 1 a 1
=>
Fi

0 am 0 a

am x a + 0 x ma"1 1 n"1 x 1 + mcV ~ 1 x a 1


=> Am + 1
ax 0 + 0xfl'" 0x1 + a'n x a

=> A in +1
^+1 cT+mT cT + l (m +1) a'"
0 a,„+t 0 am+l
This shows that the result is true for n - m + 1, whenever it is true for n = m.
Hence, by the principle of mathematical induction, the result is true for any positive integer n.
3n -1 3'1-1 3” “1 "
1 1 1
EXAMPLE 15 If A = 1 1 1 , then prove that An = 3" -l 3" -1 3” -1 for every positive
1 1 1 3" -1 3" -1 3" -1
integer n. [NCERT]

ReadYourFlow.COM
5.32 MATHEMATICS-XII

SOLUTION We shall prove the result by the principle of mathematical induction on n.


STEP! When n = 1, by the definition of integral powers of a matrix, we have
"31-1 3I-I 3I-I'
1 1 1
A1 = A = 1 1 1 31-1 31-1 3l-l
1 1 1 3l-l 3I-I 31-1

So, the result is true for n = 1.


STEP 2 Let the result be true for n = m. Then,
3'" -1 3m - 1 3W - 1 ~
Am = 3m - 1 3»n - 1 3(7) - 1
-(i)
3W - 1 3/?; -1 3?/; - l

Now we shall show that the result is true for n = m + 1.

w
3W 3'" 3m '
i.e. Am+1 3m 3'" 3m
3'" 3'" 3'"

Flo
ee
By the definition of integral powers of a matrix, we have

Fr
Am+ 1 = Am A
3'" -1 3"' -1 3"' -1 1 1 1
for
ur
=> Am+1 3'" -1 3»f - l 3m - l 1 1 1 [Using (i)]
3/;; - 1 3m - 1 3'" -1 1 1 1
ks
Yo
oo

3'" -1 + 3m -1 + 3m “1 3m -1 + 3m -1 + 3W -1 3m “ 1 + 3m “ l + 3W ” 1
B

Am+ 1 3m ” 1 + 3m “ 1 + 3m “ 1 3m -1 + 3m -1 + 3'” ~ 1 3"' _ 1 + 3'" “1 + 3m ”1


re

3m - 1 + 3m -Us'"-1 3m ~1 + 3'" - 1 + 3"' - 1 3m -1 + 3"' -1 + 3W -1


ou
ad

3x 3m” 1 3x 3m” 1 3x3'" -1 3"' 3"' 3'"


Y

=> Am+ 1 3x 3'"“1 3x3m~l 3x3'" -1 3'" 3'" 3"z


3x3 m- 1 3x 3"' -1 3x 3'" -1 3", 3m
nd
Re

3"'
Fi

This shows that the result is true for n = m + 1, whenever it is true for n = m.
Hence, by the principle of mathematical induction, the result is valid for any positive integer n.
Type IV MISCELLANEOUS PROBLEMS
EXAMPLE 16 Under what conditions is the matrix equation A2 -B2 = (A - B) (A + B) is true?
SOLUTION We have,
A2 -B2 =(A - B) (A + B)

A2 - B2 = (A - B) A + (A - B) B By disributivity of matrix muliplication'


<=>
over matrix addition
A2 B2 =A2-BA + AB-B2 By disributivity of matrix muliplication’
<=>
over matrix addition
<=> O = A2 - BA + AB - B2 - A2 + B2
<=> 0=- BA+AB
<=> AB = BA
Thus the given matrix equation is true if the matrices A and B commute with each other.

ReadYourFlow.COM
ALGEBRA OF MATRICES 5.33

EXAMPLE 17 [f A is any m x n matrix such that AB and BA are both defined show that B is an n x m
matrix.
SOLUTION Since A is an m x n matrix such that AB exists. Therefore, the number of rows in B
should be equal to the number of columns in A. Thus, B has n rows. Further, BA exists, therefore
the number of columns in B should be equal to the number of rows in A. So B has m
columns.Hence, B is an n x m matrix.
EXAMPLE 18 A, B are two matrices such that AB and A + B are both defined; show that A, B are square
matrices of the same order.
SOLUTION Let A be an m x n matrix. Since A + B is defined, therefore B is also an m x n matrix.
Further since AB exists, therefore the number of columns in A is same as the number of rows in B
i.e. n = m. Hence, A and B are square matrices of the same order.
EXAMPLE 19 IfA and B are square matrices of order n, then prove that A and B will commute iffA -XI
and B - XI commu te for every scalar X.

w
SOLUTION A - XI and B - XI commute
<=> (A -XI) (B -XI) = (B-XI) (A-XI)
AB-XIA-XIB + X2 I2 = BA-XBI -XIA + )? I2

Flo
<=>
<=> AB-XA-XB + }? I = BA-XB-XA + )? I

ee
<=> AB = BA

Fr
<=> A and B commute.
EXAMPLE 20 If AB = A and BA = B, then show that A2 = A, B2 =B.
for
ur
SOLUTION We have, AB = A and BA = B
ks

Now, AB = A
Yo

(AB) A = AA [Multiplying both sides on right by A]


oo

A (BA) = A2 [By associativity of matrix multiplication]


B

=> AB = A2 [v BA = B]
re

=> A = A2 [v AB = A]
ou
ad

Similarly, it can be proved that B =B.


Y

EXAMPLE 21 Give an example of two matrices A and B such that


nd

(i) A * O, B * O, AB = O and BA * O [NCERT EXEMPLAR]


Re

(ii) A * O, B * O, AB = BA = O.
Fi

A 01 "0 O'
SOLUTION (i) If A - and B = , then A * O, B * O.
0 0 3 0
'i oiro oi FO 0]
But, AB =
0 0
=o
0 0J 3 0
0 O' ri oi ro oi
and. BA =
3 0 0 0 3 0

(ii) If A =
ri n and B =
i -i
^ , then A * O, B * O.
1 1 -1
"i nri -i t-i -i+r _ "0 O'
But, AB = =o
1 1 JL 1 1J i-i -i+i
i -iip r ■ i-i i-i ro o'
and. BA = ; -i+i -i+i =0
-i i i i 0 0
EXAMPLE 22 Give an example of three matrices A, B, C such that AB = AC but B ^ C.
[NCERT EXEMPLAR]

ReadYourFlow.COM
5.34 MATHEMATICS-XH

ri 01 ' 0 O' 0 O'


SOLUTION Let A = ,B = and C . Then, it can be easily verified that
0 0 -1 0 0 1
AB = AC =0. But B*C.
Type V ON APPLICATIONS OF MATRICES
EXAMPLE 23 There are twofamilies A and B. There are 4 men, 6 women and 2 children in family A and 2
men, 2 women and 4 children in family B. The recommended daily allowance for calories is : Man : 2400,
woman : 1900, child : 1800 and for proteins is : Man : 55 gm, woman : 45 gm and child : 33 gm.
Represent the above information by matrices. Using matrix multiplication, calculate the total
requirement of calories and proteins for each of the tiuo families.
SOLUTION The members of the two families can be represented by a 2 x 3 matrix F given
below.
M W C

w
A r4 6 2
F =
B 2 2 4 ]
and the recommended daily allowance of calories and proteins for each member can be
represented by 3 x 2 matrix R as given below.

Flo
ee
Calories Proteins
M T 2400

Fr
55
R = W 1900 45
C [1800 33
for
ur
The total requirement of calories and proteins for each of the two families is given by the matrix
multiplication FR as given below.
ks
Yo

4 6 2 2400 55
A [ 24600 556
oo

FR = 1900 45
2 2 4 B 15800 332
B

1800 33
re

Hence, family A requires 24600 calories and 556 gm proteins and family B requires 15,800
calories and 332 gm proteins.
ou
ad

EXAMPLE 24 Use matrix multiplication to divide ? 30,000 in tiuo parts such that the total annual
Y

interest at 9% on the first part and 11% on the second part amounts 13060.
nd

SOLUTION Let the two parts be ? x and ? (30000 - x) respectively. Let A be the 1x2 matrix
Re

representing these two parts


Fi

Part I Part II
i.e. A = [x 30000 - x\
Let R denote the 2x1 matrix representing the annual interest rates of interest on two parts i.e.
Part I [ 0.09 '
R =
Part II 0.11
Tire total annual interest on the two parts is given by the matrix multiplication AR.
AR = 3060
0.09
=> [.r 30000 - x] = 3060
0.11
=> 0.09 a-+ 0.11 (30000-x) = 3060
=> — a+ — (30000-a) = 3060
100 100
9a + 330000-11 a = 306000 => x = 12,000
Hence two parts of ? 30,000 are ? 12,000 and ? 18,000 respectively.

ReadYourFlow.COM
ALGEBRA OF MATRICES 5.35

EXAMPLE 25 Three schools A, B and C organised a mela for collecting funds for helping the
rehabilitation offload victims. They sold hand made fails, mats and platesfrom recyled material at a cost of
? 25, ? 100 and ? 50 each. The number of articles sold are given below:
ichool
A B C
Article
Hand-fans 40 25 35
Mats 50 40 50
Plates 20 30 40
Find the funds collected by each school separately by sailing the above articles. Also, find the total funds
collectedfor the purpose. [CBSE2015]
SOLUTION Three items sold by three schools are represented by the following 3x3 matrix Q
as given below.

w
Hand-fans Mats Plates
A 40 50 20
Q = B
C
25
35
40
50
30
40
Flo
ee
The price matrix representing price of of three articles in ? is a 3 x 1 matrix given by

Fr
Hand-fan 25
P= Mat 100
for
ur
Plate 50
The funds collected by schools A, B and C separately by selling three articles are given by the
ks

product matrix QP.


Yo
oo

Hand-fans Mats Plates


B

A 40 50 20 25
re

QP = B 25 40 30 100
C 35 50 40 50
ou
ad

A 40 x 25+50x100 + 20x50 A ?7000


Y

=> QP = B 25 x 25 + 40 x 100 + 30 x 50 = B ^6125


nd

C ?7875
Re

C 35 x 25 + 50 x 100 + 40 x 50
Fi

Hence, the funds collected by schools A, B and C are ? 7000, ? 6125 and ?7875 respectively.
The total funds collected = ? (7000+ 6125+ 7875) =?21000.

LEVEL-2

Type I ON MULTIPLICATION OF MATRICES


EXAMPLE 26 Prove that the product of matrices
cos 0 sin 0 2 cos <(> sin <|>
cos2 9 COS (J)
and
cos 0 sin 0 sin2 0 cos (j) sin <j) sin2 4)
is the null matrix, when 0 and ^differ by an odd multiple of~.

SOLUTION We have,
cos2 0 cos 0 sin 01 cos2 <t> cos <j> sin (]>
cos 0 sin 0 sin2 0 cos <j> sin <j) sin2 <|>

ReadYourFlow.COM
5.36 MATHEMATICS-XII

cos2 9 cos2 <j) + cos 0 cos <j) sin 0 sin (|) cos2 0 cos c|> sin(J> + cos 0 sin 0 sin2 <})
cos <{> cos 0 sin 0 + sin2 0 cos <J) sin (|) cos 0 sin 0 cos<}) sin <J) +sin2 0 sin2 <j)

'cos 0 cos (j> cos (0 - <j)) cos 0 sin (|) cos (0 - (())'
sin 0 cos <j) cos (0 - <()) sin 0 sin <}) cos (0 - <|))
_ ’° °* v 0 - <|) = (2tt + 1) neZ cos (0 - (|)) = cos (2n + 1) -^ = 0
"00
0 - tan (a / 2) "
EXAMPLE 27 Let A = and I be the identity matrix of order 2. Show that
tan (a / 2) 0
cos a - sin a
I + A=(I-A) [NCERT]
sin a cos a
SOLUTION We have.
0 f a 1 a
ri oi

low
I+A =
0 1
+ * a - tan —
2 a
- tan —
2
tan — tan —
2 0 2 1
0 f a 1
ri oi tan — tan —
i. a
and. I - A =
0 1
+ a 2 a. 2
-tan- Q - tan —

ee
2 2 1
rF
Fr
1 a
cos a - sin a tan — cos a - sin a
(I-A) = a 2 sin a
sin a cos a -tan — cos a
for
2 1
ou
a
1 - tan 2 « 2 tan —
ks

2 2
1 2 cl 2 cl
oo

cos a -sin a . a 1 + tan 1 + tan


tan 2 2
=> (I-A) a
Y

2
eB

sin a cos a - tan — 2 cl


2 1 2 tan - 1 - tan
2 2
2 cl 2 cl
r

1 + tan 1 + tan
ou
ad

2 2
21
Y

1-f2
1 t l+t2
cos a -sin a
d

1 +t2 a
Re

=> (i-a) sin


: , where t = tan —.
n

a cos a 21 1-t2 2
-t 1
Fi

l+t2 1 + f2
1 -t2 + 2f2 -2t + t-t3
cos a -sin a l+t2 l+t2
=> u-a) sin
: a cos a -t + t3 + 2t 2t2 + 1 -t2
1 +£2 1 +f2
l+t2 -t(l +t2)
cos a -sin a 1+f2 1+f2 '1 -f
=> (I-A)
sin a cos a t (1 + t2) l+t2 f 1
l+t2 1+f2
1 a
cos a -sin a - tan
=> (1-A) a 2 =1 + A
sin a cos a tan —
2 1

ReadYourFlow.COM
ALGEBRA OF MATRICES 5.37

cos x - sin x 0
EXAMPLE 28 Let F (x) = sin x cos x 0 . Shozu that F (x) F (y)= F (x + y).
0 0 1
SOLUTION We have, [NCERT]
cos x - sin x 0 cos y - sin y 0
F(x)-F(y) = sin x cos x 0 sin y cos y 0
0 0 iJl. 0 ° !.
cos x cos y - sin x sin y - sin y cos x - sin x cos y 0
=> F(x)-F(y) = sin x cos y + cos x sin y - sin x sin y + cos x cos y 0
0 0 1
cos (x + y) - sin (x + y) 0
F(*)-F(y) = sin (x + y) cos (x + y) 0 = F (x + y)
0 0 1

low
Type II ON MATRIX POLYNOMIALS AND MATRIX POLYNOMIAL EQUATIONS
2 3
EXAMPLE 29 LetA = andf(x) = x2 - 4x + 7. Shou) that/(A) = O. Use this result to find A5.
-1 2
[NCERT EXEMPLAR]

ee
SOLUTION We have, / (x) = x2 - 4x + 7
rF
Fr
/ (A) =A2 -AA +7 12 for
2 3 2 3 4-3 6+6 1 12
Now, A2 =
“ -2-2 -3+4 -4 1
ou
-1 2 -1 2
ks

-8 -12 7 0
-AA = and, 712 =
oo

4 -8 0 7
Y
B

f(A) = A2 - AA + 712
re

i i2i r -8 -12 + 7 0
f(A) = +
4 -8
ou

-4 1 0 7
ad

1-8+7 12-12 + 0 0 0
Y

=> f(A) = -4+4+0 1-8+7 ]■[ 0 0


=o
nd
Re

Now, f(A)=0
Fi

=> A2 -AA+7I2=0
=> A2 =AA -7I2
=> A3 =A2 A =(AA -7I2) A=AA2-7I2 A
=> A3 =A(AA-712)-7A [Using : A2 = 4A - 7/2]
A3 = 9A - 2SI2
=> A4 =A3A=(9A -28/2) A
=> A4 = 9A2 - 28A = 9 (4A -7I2) - 28A [Using : A2 = 4A - 7/2]
=> A4 = 36A - 63/2 - 28A = 8A - 63/2
=> A5 = A4 A = (8A - 63I2) A = 8A2 - 63/2 A
A5 = 8 (4A -7/2) - 63A = - 31A -56/2 [Using : A2 = 4A - 7J2]
2 3 1 0 -62 -93 -56 0 ' _ T -118 -93 '
=> A5 = - 31 -56
31 -62
+ 0 -56 31 -118 '
-1 2 0 1

ReadYourFlow.COM
[uoi}e3ijdi4|nui x|J4gui jo AjiApeposse Xg] lua (av) =l+mav
{maa)v = (alua)v =
9Auq 9A\ 'xi-ijeui e jo si9Mod |ejS94ui jo uogruijgp 9qj Xg

l + u,av
•91
^i + w3
■l + ui =u joj gnxi si 4|ns9j gig Mogs mou geqs 9yv\
(I)- v' u,a = uMV
'uggx ut = u joj 9ruj gq jjnsgj 9qj jgq z J3IS
‘l = UXO} 911JJ SI JJI1S9J JBq4 'og

[9 = ^ •■•] vxa =
[(u9aiS) ^3 = ev va = av = xav
9AEq 9M 'XTJJBin E JO S.I9Mod {EJ§9JUT JO UOi;TlIIJ9p 9E{4 Xq 'J = W U9TJ/4 l dais
■u uo uoponpur |EDqEui9qjEiu jo 9[dpmjd gqj Xq jjnsgj 9q4 9Aoad jjEqs 9/^4 (1) NOIiniOS

w
[tfvuwaxa iaa3N] ua uv = u(av) (n) Vua = uav (!)
u AdSdiui daijisod Huv .10/idi{] moi($ •ajmuuioj Rdi[\ jinn ipns sdoujuiu omi aq g 'y pg i£ auiAlvxa

Flo
■u i9§9jui 9AqTsod Xue joj pi[EA si 4{ns9J 9q4 'uoponpui 4E9i4Eiu9q4Eui jo 9jdi9injd gqj Xq 'gDugjq

ee
‘Ill = U JOJ 911J4 SI 41 J9A9U9qM 'l + III = U JOJ 911JJ SI 4JI1S9J gqj JEqj SMOqS Siqi

Fr
vq np (l + “0 + / l + uP = l + m (yq + id) <=

0 0 0 0 0 0
][ [ 0 0 =y or
ur
0 0 L I 0 l 0 l 0
sf
° Zq l-u flu + yq Ulu (I + ill) + I l+ uP = X + in (vq + /») <=
k
Yo

zv zql.up^+v(qup + qupui) + i l + uP = X + ui
(yq + id) <=
oo

h = r rv = iv=vi •••] zvzql.up^+vq upui + vq up + i + /»)


uP = l +IU (vq <=
B

i +
(vv) Z‘l-
7q uPm + (IV) q uPUl + (VI) q up + (/•/) ( uf) = x + III (yq + iv)
<=
re

(vq) (vq i - iuvm) + (/») (vq t _ up™) + (vq) (i up) + (iv) (i luv)= x + ill (vq
+ f) <=
ou
ad

[(!) §uisn] (vq + i») (vq 1-Upiu +1 uP)= l + iu (yq + id) <=
Y

(vq +1») Jvq + iv)= 1 +Ui (vq + iv)


nd
Re

9AEq 9M 'xijxeui e jo sj9Mod jej§94Ui jo uoqiuijop gqj Xg


Fi

vq uP (i + Ul) +I I + uP = X + ui (vq + iv) 9l

‘l + in = u joj 9iijj si jjnsgj gqj jEqj Moqs qsqs 9M Mojq


(!)- + = Jyq+iv)
ugqi m = u joj 9njj 9q jjnsgj gqj jgq zaaiS
‘l = U JOJ 9I1JJ SI JJI1S9J 9qj 'og

vql.lvi +1 lv=vqQvi +1 xv=vq + iv= x(vq + iv)


9AEq 9M 'xijjeui e jo sj9Mod jEjSgjui jo uoqiuijgp gqj Xq 'j = u ugq^ Oils
■it no uoTjonput {EDijEiugqjEUJ Xq jpisgj gqj 9A0jd gsqs 9^4 NOII/llOS
[JLH3DN] ‘UdSapii aaijisod d si u puv z .iap.10 fo xujdui pun d si j axdqm
0 0
vq L _ iPu +1 ,p= u(vq + iv) mi3aoud' = v/l oe auwvxa
l 0
NouonaNi nvoiivwaHivw jo aidiONiud no in adfix

IIX-SOI1VW3H1VW ses

ReadYourFlow.COM
ALGEBRA OF MATRICES 5.39

=> AB in + 1 = (BA) Bm [•.• AB = BA (given)]


ABm + 1 = B(AB"J) [By associativity of matrix multiplication]
m+ 1 [Using induction assumption (i)]
=> AB = B(BmA)
AB”,+ 1 = (BBm) A [By associativity of matrix multiplication]
=> AB,n + 1 = Bm + 1,4 [By definition of integral powers]
This shows that the result is true for n - m + 1, whenever it is true for n = m.
Hence, by the principle of mathematical induction, the result is true for any positive integer n.
(ii) We shall prove this result also by the principle of mathematical induction on n.
STEP 1 When n = 1, by the definition of integral powers of a matrix, we have
(AB)1 = AB = A^ Bl [v A1 =A,B1 = B]
So, the result is true for n -1.

w
STEP 2 Let the result be true for n = m. Then,
(AB),n = Am Bm

Flo
Now we shall show that the result is true for n = m + 1.

ee
i.e. (AB) m +1 = A m +1 Bnm+ 1

Fr
By the definition of integral powers of a matrix, we have
(AB) m + 1 = (AB)m (AB)
for
ur
=> (AB) m + 1 = (Am Bm) (AB) [By induction assumption (i)]
ks

=> (AB) m + 1 = Am (Bm (AB)) [By associativity of matrix multiplication]


Yo
oo

in + l [v AB = BA (given)]
=> (AB) = Am (Bm (BA))
B

in + 1 = Am ((BmB) A) [By associativity of matrix multiplication]


=> (AB)
re

m+ l = Am (Bm + 1 A) [By definition of integral powers]


=> (AB)
ou
ad

m+ 1
=> (AB) m + 1 = Am (AB ) [v ABn = B" A for all n e N (proved in (i))]
Y

=> (AB) m + 1
= (Am A) B m + 1 [By associativity of matrix multiplication]
nd

in + 1 m + 1 gin + 1
Re

=> (AB) =A
This shows that the result is true for n = m +1, whenever it is true for n = m.
Fi

Hence, by the principle of mathematical induction, the result is true for every positive integer n.
9 o
EXAMPLE 32 If A is a square matrix such that A = A, show that (I + A) =7 A +1.
[NCERT EXEMPLAR]
SOLUTION Using matrix multiplication, we obtain
(I+A)2 = (I + A)(I + A)
= I(I + A) + A(I + A) [By distributivity of multiplication over addition]
= I2 + IA + AI+A2
= I + A + A + A2 [y IA=AI = A]
= I + 2A + A2
= I + 2A+A [:A2=A]
= 1 + 3A
(I + A)3 =(I+A)2(I + A)

ReadYourFlow.COM
5.40 MATHEMATICS-XII

=> (I+A)3 =(I + 3A)(I + A)


= I(I + A) + 3A(I + A)
= I2 + IA + 3(AI) + 3 (AA)
= 1 +A + 3A + 3A2 [■:IA=AI=A]
= I+A + 3A + 3A [■.■A2=A]
= 1+7 A
EXAMPLE 33 If A is a square matrix such that A2 =1, then find the simplified value of
(A -1)3 + (A +1)3 -7 A. [CBSE 2016]
SOLUTION We have. A2 = I
A3 =A2A =IA =A

low
We know that
(A + B)3 =A3 + 3A2B + 3AB2 + B3 and, (A-B)3 =A3-3A2B+3AB2-B3
, provided that AB = BA.

ee
Since AI =IA= A.
rF
Fr
(A + I)3 =A3 + 3A2I + 3A12 + I3 and (A -1)3 = A3 - 3A2I + 3AI2-I3
for
=> (A +1)3 = A3 + 3A2 + 3A +1 and(A -1)3 = A3 - 3A2 + 3A -1
ou
(A + I)3 +(A-I)3 =2(A3 + 3A)
ks

(A + I)3 +(A-I)3 =2(A + 3A) [Using (i)]


oo
Y

=> (A + I)3 +(A-I)3 =8A


eB

Hence, (A- I)3 +(A +1)3 -7A =8A -7A =A.


r
ou
ad

EXAMPLE 34 If A =[3 5], B =[7 3], then find a non-zero matrix C such that AC =BC.
Y

[NCERT EXEMPLAR]
SOLUTION Clearly, A and B are 1 x 2 matrix. Therefore, products AC and BC exist if C is of
d
Re

order 2 x n, where n e N.
n
Fi

Now, following cases arise.


CASE I When n = l: In this case, matrix C is a 2 x 1 matrix. So, let C = ^ . Then,

AC = BC
[3 5] ^j=[7 3]^
=>

=> 3a + 5b =7a+ 3b
=> Aa = 2b
=> 2a =b
a
c= , where fleR.

CASE II When n = 2\ In this case, matrix C is a 2 x 2 matrix. So, let C = . Then,


P y.
AC =BC

ReadYourFlow.COM
ALGEBRA OF MATRICES 5.41

a x a x
=> [3 5]& = [7 3]
y] b y.
=> [3n + 5b 3x+ 5y]-l7a + 3b 7x+3y]
=> 3a + 5b = 7a + 3b and 3x + 5]/ -7x + 3y
=> b = 2a and y - 2x.
a x~
c= where a, x e R.
2a 2x '
Similarly, if n = 3
a x y
C = 2a 2x 2y\ ’ where a, x,y sR and so on.

EXERCISE 5.3

low
LEVEL-1

1. Compute the indicated products:


2 3 4 1-3 5
... r a b~\[ a -b 1 -2 1 2 3
W -baba (ii) (iii) 3 4 5 0 2 4
2 3 -3 2 -1

ee
4 5 6 3 0 5
rF
Fr
2. Show that AB * BA in each of the following cases:
-1 1 0 1 2 3
'5 -1 [2 1
for
(i) A = 6 and B = (ii) 71 = 0 -1 1 and B = 0 1 0
7 3 4
ou
2 3 4 1 1 0
ks

'1 3 O' 0 1 0
oo

(iii) A = 1 1 0 and B= 1 0 0
Y

4 1 0 0 5 1
B
re

3. Compute the products AB and BA whichever exists in each of the following cases:
3 2
and B = '1 2 3' 4 5 6'
ou

(i) A =12 ~ *
ad

2 3 1 (ii) A = -1 0 and B =
0 1 2
Y

-1 1
0 a
nd
Re

c b
(iii) A=[l -1 2 3] and B= ^ (iv) [a b] d +[abcd] c
Fi

2 d
4. Show that AB * BA in each of the following cases:
'1 3 -1 -2 3 -1 10 -4 -1 1 2 1'
(i) A = 2 -1 -1 and B = -1 2 -1 (ii) A = -11 5 0 and B = 3 4 2
3 0 -1 6 9-4 9 -5 1 1 3 2
5. Evaluate the following:
1 0 2 2
1 31 3 -2 1 3 5
(1) -1 -4 + -1 1 2 4 6 ] (ii) [1 2 3] 2
0
0 1
1 2
4
6
1 -1
1 0 2 0 1 2
(iii) 0 2
2 0 1 1 0 2
2 3
1 0 1 0 0 1
6. If A = ,B = and C = , then show that 712 = B2 =C2 =I2.
0 1 0 -1 1 0

ReadYourFlow.COM
5.42 MATHEMATICS-XII

2 \ and B = 0 4
7. IfA = 3 2
, find 3A2 -2B +1. [CBSE 2005]
-1 7
4 2
8. IfA = , prove that (A -21) (A - 31) = O.
-1 1
1 1 1 2 1 3
9. = 0 1 , show that A2 - and A3 =
0 1 0 1 •
ab b2
10. If A = 2 , show that A2 =0.
-a ab
cos 2 0 sin 2 0
11. IM = - sin 2 0 cos 2 0 , find A2 . [CBSE2000C]

2 -3 -5 -13 5

w
12. IfA = -14 5 and B = 1 -3 -5 , show that AB = BA = O 3x3-
1 -3 -4 -13 5

0 c -b

Flo a2 ab ac

ee
13. IfA = - c 0 a and B = ab b2 be , show that AB = BA =0 3x3-
b -a 0

Fr
ac be c2
2-3-5 2 -2 -4
for
ur
14. If A = -1 4 5 and B = -13 4 , show that AB - A and BA = B.
1 -3 -4 1 -2 -3
ks
Yo

-1 1 -1 043
oo

15. Let A = 3-3 3 and B = 1 -3 -3 , compute A2 - B2.


B

5 5 5 -14 4
re

16. For the following matrices verify the associativity of matrix multiplication i.e.
ou
ad

(AB) C = A (BC).
Y

1 0
12 0 1
(i) ^4 = -1 0 1 , B = -1 2 and C = -1
nd
Re

0 3
Fi

4 2 3 1 -1 1 1 2 -1
(ii) A = 1 1 2 , B= 0 1 2 and C = 3 0 1
3 0 1 2 -1 1 0 0 1
17. For the following matrices verify the distributivity of matrix multiplication over matrix
addition i.e. A (B + C)=AB + AC.
1 -1 -1 0 0 1
(i) A = , B= and C = i
0 2 2 1 -1
2 -1
0 1 and C = \ -1
(ii) A = 1 1 , B=
1 1 0 1 •
-1 2
1 0 -2 0 5-4 1 5 2
18. If A= 3-1 0 , B = -2 1 3 and C = -1 1 0 , verify that
-2 1 1 -10 2 0 -1 1
A (B -C) = AB - AC.

ReadYourFlow.COM
ALGEBRA OF MATRICES 5.43

19. Compute the elements and a22 of the matrix:


fO 1 O'
2 0 2 2 ! TO 1-1 2 -2"
A = -3 2
0 3 2 3-3 4-4 0
4 3
4 0 4
0 10
20. If A = 0 0 1 and I is the identity matrix of order 3, show that A3 = pi + qA + rA2.
P V r
21. If w is a complex cube root of unity, show that
1 W w2 W
i l¥i 0
w a,2 1 + w2 1 w w 0
IV 2 1 a, w IV
2 1 iv
2 0

low
2 -3 -5
22. If A = -1 4 5 , show that A2 = A.
1 -3 -4
4 -1 -4

ee
23. If A = 3 0-4 , show that A2 = /3.
rF
Fr
3 -1 -3
1 0 2 1
for
24. (i) If [1 1 x] 0 2 1 1 = 0, find x.
u
2 1 0 1
ks
Yo

2 3 1 r 1 -3 '-4 6'
oo

(ii) If 5 7 J [ -2 4 , find x. [CBSE2012]


-9 x
B

'2 1 2' x
re

25. If [x 4 1] 1 0 2 4 = 0,findx.
ou

0 2-4 -1
ad
Y

'0 1 -1 0
26. If [1 -1 x] 2 1 3 1 = 0, find x.
nd
Re

1 1 1 1
Fi

3 -2 1 0
27. If A
-[ 4 -2
and I =
0 1
, then prove that A2 - A + 21 =0.

3 1 1 0 , then find X so that A2 =5A + ?J.


28. If A = and I =
-1 2 0 1
3 1 , show that A2 -5A + 7/2 =0.
29. If A = [CBSE 2003,2007]
-1 2
' 2 3" , show that A2 -2A + 3I2 =0.
30. If A =
-1 0
2 3 J satisfies the equation A 3 - 4A2 + A =0.
31. Show that the matrix A = [CBSE 2005]
1 2
5 3
32. Show that the matrix A = is a root of the equation A - 12A -1=0.
12 7
3 -5
33. If A = , find A2 -5A -14/. [CBSE 2004]
-4 2

ReadYourFlow.COM
5.44 MATHEMATICS-XII

3 1
34. If A = , show that A2 -5A + 71 =0. Use this to find A4. [NCERT]
-1 2
3-2'
35. IfA = , find/c such that A 2 =kA - 212- [NCERT, CBSE 2003]
4 -2
1 0
36. If A = , find k such that A2 - 8A + kl = O. [CBSE 2005]
-1 7
fl 21
37. IfA = and / {x) =x2 - 2x - 3, show that/ (A) = O. [CBSE 2005]
2 1
2 3 1 0
38. IfA = and 7 = , then find X, ja so that A2 = X A + I
1 2 0 1
39. Find the value of x for which the matrix product
2 0 7 -x 14 x 7x "

low
0 1 0 0 1 0 equal to an identity matrix.
1 -2 1 x - 4x - 2x
40. Solve the matrix equations:

ee
1 0 1 2 0 0
i5 = o
rF
Fr
(i)[* 1] -2 -3 (ii) [1 2 1] 2 0 1 2 =0 [NCERT]
for 1 0 2 x
1 0 2 x
1 21PU
(hi) [x-5-1] 0 2 1 4=0 [NCERT] (iv) [2x 3]
u
=0
-3 0 8
ks

2 0 3 1
Yo
oo

1 2 0
41. If A = 3-4 5 , compute A2 - 4A + 3/3.
B

0-13
re

0 1 2
ou

42. Iff (x) = x2 - 2x, find / (A), where A = 4 5 0


ad

0 2 3
Y

0 1 2
nd
Re

43. If / (x) = x3 + 4x2 - x, find/(A), where A = 2 - 3 0


Fi

1 -1 0
1 0 2
44. If A = 0 2 1 , then show that A is a root of the polynomial / (x) = x3 - 6x2 + 7x + 2.
2 0 3
[NCERT]
12 2
45. If A = 2 12 , then prove that A2-4A-57 =0. [CBSE 2008]
2 2 1
'3 2 0]
46. IfA =14 0 , show that A2-7A + 1073 =0.
0 0 5
47. Without using the concept of inverse of a matrix, find the matrix * ^ j
such that
5 -7 * y -16 -6"
-2 3 z u 7 2 '

ReadYourFlow.COM
ALGEBRA OF MATRICES 5.45

48. Find the matrix A such that


1 1 '3 3 5'
(i) 0 1 A = 1 0 1

4 -4 8 4
(ii) A
‘12 3
4 5 6
-1 0 -1
W-7> -8
4
1
3 [NCERT]

(iii) 1 A = -1 2 1 (iv) [2 1 3] -1 1 0 0 =A
3 -3 6 3 0 1 1 -1
[NCERT EXEMPLAR] [NCERT EXEMPLAR]
2 -1 -1 -8 -10' -7 -8 -9
'1 2 3
(v) 1 0 A = 1 -2 -5 (vi) A = 2 4 6
4 5 6
-3 4 9 22 15 11 10 9
[CBSE2017] [CBSE2017]
"1 -2

w
49. Find a 2 x 2 matrix A such that A ^ 4 =6I2-
'0 0‘ , find A16.
50. IfA =
4 0
ro -x
Flo
ro ii and x2 - -1, then show that (A + B)2 = A2 + B2.

ee
51. If A = ,B=
x 0 1 0

Fr
[NCERT EXEMPLAR]
1 0 -3'
for
ur
52. If A = 2 1 3 , then verify that A2 +A =A(A + I), where I is the identity matrix.
ks

0 1 1
Yo

[NCERT EXEMPLAR]
oo

3 -5
eB

53. If A = ^ , then find A 2 -5 A -147. Hence, obtain A 3. [NCERT EXEMPLAR]


-4
cosx smx
r

54. (i) IfP(x) = then show that P(x) P{y) = P(x + \j)= P(y) P(x).
ou
ad

-sinx cosx
Y

x 0 O' a 0 0 xa 0 0
(ii) IfP = 0 y 0 and Q = 0 b 0 , prove that PQ = 0 yb 0 = QP
nd
Re

0 0z 0 0c 0 0 zc
Fi

[NCERT EXEMPLAR]
'2 0 1
55. If A = 2 1 3 , find A2-5A + 47 and hence find a matrix X such that
1 -1 0
A2 -5A + 4/ +X =0. [CBSE 2015]
1 1 1 n
56. If A = , prove that A" = for all positive integers n.
0 1 0 1

a b an b
V-iYl
57. If A = , prove that A" = a-l for every positive integer n.
0 1
0 1
cos 0 i sin 0
58. If A = , then prove by principle of mathematical induction that
i sin 0 cos 0
cos n 0 i sin n 0
An = for all n e N. [CBSE 2005]
i sin n 0 cos n 0

ReadYourFlow.COM
5.46 MATHEMATICS-XII

cos a + sin a V2 sin a


59. IM = prove that
- V2 sin a cos a - sin a
cos n a + sin n a V2 sin n a
An = for all 11 e N.
- V2 sin n a cos n a - sin 11 a
1 1 1
60. If /4 = 0 1 1 , then use the principle of mathematical induction to show that
0 0 1
1 n n(n +1)/2
A" = 0 1 n for every positive integer n.
0 0 1
61. If B, C are n rowed square matrices and if A=B + C, BC = CB, C =0, then show that for
every 11 eN, A fi+i = B" (B + (« + 1) C).

low
62. If A = diag (a b c), show that A" = diag (o'1 bn cn) for all positive integer n.
63. If A is a square matrix, using mathematical induction prove that (A = (An)T for all/i e N.
[NCERT EXEMPLAR]

ee
64. A matrix X has a + b rows and <? + 2 columns while the matrix Y has b + 1 rows and <7+3
rF
Fr
columns. Both matrices XY and YX exist. Find a and b. Can you say XY and YX are of the
same type? Are they equal. for
65. Give examples of matrices
u
(i) A and B such that AB * BA.
ks

(ii) A and B such that AB = O but A ^ O, B * O.


Yo

(iii) A and B such that AB = O but BA + O.


oo

(iv) A, B and C such that AB = AC but B *C,A *0.


B

66. Let A and B be square matrices of the same order. Does (A + B)2 = A2 + 2AB + B2 hold?
re

If not, why?
ou
ad

67. If A and B are square matrices of the same order, explain, why in general
Y

(i) (A + B)2 * A2 + 2 AB + B2 (ii) (A-B)2 * A2 -2AB+ B2


(iii) (A + B) (A -B) * A2 - B2.
nd
Re

68. Let A and B be square matrices of the other 3 x 3. Is (AB)2 = A2 B2 ? Give reasons.
Fi

[NCERT EXEMPLAR]
69. If A and B are square matrices of the same order such that AB = BA, then show that
(A + B)2 = A2 + 2AB+ B2. [NCERT EXEMPLAR]
3 1 4 2
1 1 1
70. Let A =
3 3 3 ]'B=5 2 and C = -3 5
-2 4 5 0
Verify that AB = AC though B * C, A * O.
71. Three shopkeepers A, B and C go to a store to buy stationary. A purchases 12 dozen
notebooks, 5 dozen pens and 6 dozen pencils. B purchases 10 dozen notebooks, 6 dozen
pens and 7 dozen pencils. C purchases 11 dozen notebooks, 13 dozen pens and 8 dozen
pencils. A notebook costs 40 paise, a pen costs ? 1.25 and a pencil costs 35 paise. Use matrix
multiplication to calculate each individual's bill.
72. The cooperative stores of a particular school has 10 dozen physics books, 8 dozen chemistry
books and 5 dozen mathematics books. Their selling prices are ^ 8.30, ? 3.45 and ? 4.50 each
respectively. Find the total amount the store will receive from selling all the items.

ReadYourFlow.COM
ALGEBRA OF MATRICES 5.47

73. In a legislative assembly election, a political group hired a public relations firm to promote
its candidates in three ways: telephone, house calls and letters. The cost per contact (in
paise) is given matrix A as
Cost per contact
40 Telephone
A = 100 House call
50 Letter
The number of contacts of each type made in two cities X and Y is given in matrix B as
Telephone House call Letter
1000 500 50001 ->X
B =
3000 1000 10000 -» Y
Find the total amount spent by the group in the two cities X and Y.
74. A trust fund has ? 30000 that must be invested in two different types of bonds. The first

w
bond pays 5% interest per year, and the second bond pays 7% interest per year. Using
matrix multiplication, determine how to divide ? 30000 among the two types of bonds. If

Flo
the trust fund must obtain an annual total interest of (i) ^1800 (ii) ?2000.
75. To promote making of toilets for women, an organisation tried to generate awarness
[NCERT]

ee
through (i) house calls (ii) letters, and (iii) announcements. The cost for each mode per

Fr
attempt is given below:
(i) ?50 (ii) ?20 (iii) ? 40 or
ur
The number of attempts made in three villages X, Y and Z are given below:
sf
(i) (ii) (iii)
X 400 300 100
k
Yo

Y 300 250 75
oo

Z 500 400 150


B

Find the total cost incurred by the organisation for three villages separately, using matrices.
re

[CBSE 2015]
76. There are 2 families A and B. There are 4 men, 6 women and 2 children in family A, and 2
ou
ad

men, 2 women and 4 children in family B. The recommend daily amount of calories is 2400
Y

for men, 1900 for women, 1800 for children and 45 grams of proteins for men, 55 grams for
women and 33 grams for children. Represent the above information using matrix. Using
nd
Re

matrix multiplication, calculate the total requirement of calories and proteins for each of
the two families. What awareness can you create among people about the planned diet
Fi

from this question? [CBSE 2015]


77. In a parliament election, a political party hired a public relations firm to promote its
candidates in three ways — telephone, house calls and letters. The cost per contact (in
paisa) is given in matrix A as
"140 Telephone
A = 200 House calls
150 Letters
The number of contacts of each type made in two cities X and Y is given in the matrix B as
Telephone House call Letters
1000 500 5000 ] City X
B= lOOOoJ City Y
3000 1000
Find the total amount spent by the party in the two cities.
What should one consider before casting his/her vote — party's promotional activity or
their social activities? [CBSE 2015]

ReadYourFlow.COM
5.48 MATHEMATICS-XII

78. The monthly incomes of Aryan and Babbar are in the ratio 3 : 4 and their monthly
expenditures are in the ratio 5 : 7. If each saves ? 15000 per month, find their monthly
incomes using matrix method. This problem reflects which value? [CBSE 2016]
79. A trust invested some money in two type of bonds. The first bond pays 10% interest and
second bond pays 12% interest. The trust received ? 2800 as interest. However, if trust had
interchanged money in bonds, they would have got ? 100 less as interest. Using matrix
method, find the amount invested by the trust. [CBSE 2016]
________________________________________________________ ____________ ANSWERS
a1+b1 0 14 0 42 '
7-2 5
1. (i) (ii) (hi)
18 -1 56
0 a2 +b2 -7 10 3
22 -2 70
-3 -4 1 12 17 22 ~
1 14
3. (i) AB = , BA does not exist (ii) AB = -4 -5 -6 , BA =
8 13 9 -3 2
-4 -4 -4

w
0 0 0 0
(hi) AB = [11], BA = l -12 3 (iv) [a2 + b2 + c2 + d2 + ac + bd\
3-369
2-246

Flo
ree
0 -1 1
6 16 26 4-20
5. (i) (ii) [82] (hi) 2 0-2 7.
-8 -18 -28

F
38 -10
5 -2 -3
-2 -9 -1
or
ur
cos 4 0 sin 4 0
11. 15. 3 26 3 19. «43 - 8, a22 - 0
f
- sin 4 0 cos 4 0
35 15 34
ks
Yo

24. (i) x = -2 (ii) x=13 25. x = -2,-1 26. 2


oo

TO O' 34. (ii) ^ 55


28. -7 33. 35. k=l
B

0 0 -55 -16
re

1
36. k=7 38. X = 4,p =-l 39. -
5
ou
ad

23
40. (i) x = - 3,5 (h) x = -1 (hi) x = 4j3 (iv) x-0, - —
Y

6 -14 10' 4 7 2 ~6 -2 6'


nd
Re

41. -21 36 -25 42. 12 19 8 43. 0 4 4


-3 5-5 8 12 3 1 1 4
Fi

'1 -4 [2 3 J (ii) l “h (iii) [-1 2 1] (iv) [-4] (v) P


47 ' |_3 -2 48. (i)
1 0
' 1 -2
4 2 ' 187 -195'
(vi) 2 0 49. 50. Null matrix 53.
-1 1 -156 148
-5 4
-1 -1 -3] fl 1 3"
55. -1 -3 -10 , 1 3 10 64. a = 2,b = 3, No 68. True when AB = BA
-5 4 2j [5 -4 -2
71. ? 157.80, ? 167.40, ? 281.40 72. H597.20 73. ? 3400, ? 7200
74. (i) ? 15000 each (ii) ? 5000, ? 25000
75. X: ? 30,000 Calories Proteins
Y: ? 23,000 76. Family A : 24600 576 77. X : ? 9900
Family B: 15800 332 Y: ? 21200
Z:? 29,000
78. ? 90,000, ? 120,000 79. ? 10,000, ? 15,000

ReadYourFlow.COM
ALGEBRA OF MATRICES 5.49

HINTS TO NCERT & SELECTED PROBLEM


34. (ii) We have,
3 1
A =
-1 2
3 1 3 1 9-1 3+2 8 5
=> A2 = AA =
-1 2 -1 2 " -3-2 -1+4 -5 3
8 -15 -5 ’ 7 01J8-15 + 7 5-5 + 01 [0 0]
A2 -5A + 71 =
5 -10
+ 0 7“ -5+5 + 0 3-10 + 7 0 0
=o
=> A2 = 5A-71
A4 = A2 A2 = (5A-7I)(5A-7I) = 5A (5A-71)-71 {5A-71)
= 25 A2- 35AI - 35/A + 4911

w
= 25 A2- 35 A - 35 A + 49/
= 25A2 -70A + 49/

Flo
= 25 (5A - 71) -70 A + 49/

ee
= 125A-175/-70A + 49/

Fr
= 55A -126/
3 1 "1 0l_ri65 55 -126 0 39 55
= 55 -126
]*[
or 0 -126 -55 -16
ur
-1 2 0 1 " -55 110
sf
3 -2'
35. (i) We have, A =
4 -2
k
Yo

'9-8 -6 + 4" "1 -2"


oo

3 -2] [3 -2
A2 =
4 -2 4 -2 12-8 -8 + 4 4 -4
B

It is given that A2 = AA-2/2


re

1 -2
= fc
3 -21 n oi
ou

-2
ad

4 -4 4 -2 0 1
Y

=>
"1 -2
4 -4 H 3k -2
4k
-2k
-2k-2
nd
Re

=> 3k-2 = 1, 4k = 4, -2k = -2 and -2k-2 = -4


Fi

=> k = 1
'1 2 Ol f0 4
40. (ii) [1 2 1] 201 2 = 0 => [1 2 1] x =0 => 4 + 2x + 2* = 0 => * =-l
1 0 2_] [x 2x
'1 0 21
44. We have, A =021 and/(x) = x3 -6x2 + 7x + 2
2 0 3
.-. / (A) = A3-6A2+7A + 2/3
"1 0 2l [1 0 2 "5 0 8"
Now, A2 = AA = 0 2 1 0 2 1 2 4 5
2 0 3J 2 0 3 8 0 13
[5 0 8' 1 0 2 '21 0 34
=> A3 = A2 A = 2 4 5 0 2 1 = 12 8 23
8 0 13 2 0 3 34 0 55

ReadYourFlow.COM
5.50 MATHEMATICS-XII

f(A) = A3-6A2 +7A + 2I3


"21 0 34 "5 0 8" 7 0 14 '2 0 0" 0 0 0
=> f(A) = 12 8 23 -6 2 4 5 + 0 14 7 + 0 2 0 0 0 0 =o
34 0 55 8 0 13 14 0 21 0 0 2 0 0 0
Hence, A is a root of the polynomial / (x) = .t3 - 6x2 + 7x + 2.
1 2 3' -7 -8 -9'
48. (li) We have to find a matrix A satisfying the equation A
4 5 6 ~ 2 4 6 ’
Clearly, the product of A with a 2 x 3 matrix is a 2 x 3 matrix. Therefore, A is a 2 x 2 matrix.
a b
Let A = . Then,
c d
"1 2 3" -7 -8 -9
A
4 5 6 2 4 6

w
a b~\n 2 3" _ f-7 -8 -9]
c rfj [4 5 6 “ 2 4 6j
a + 4b 2a + 5b 3a + 6b
c + 4d 2c+5d 3c + 6d 2 4

Flo
-7 -8 -9
6

ee
=> a + 4b = -7, 2a + 5b = - 8, 3a + 6b = -9

Fr
c + 4d = 2, 2c + 5d = 4, 3c + 6rf = 6
=> a = 1, b = -2, c = 2, d = 0
for
ur
"1 -2"
A =
2 0
ks
Yo

5 0 0 0 1 0 0 0
oo

59. (ii) A = ,B = (iii) A = ,B =


0 0 -1 0 0 0 2 1
B

2 0 3 0 -1 0
re

(iv) A ■[ 0 0
,B =
0 0
, C=
0 0
ou
ad

73. Let ? x be invested in first bond and ? 1/ be invested in second bond. Let A be the investment
matrix and B be the interest per rupee matrix. Then,
Y

5
nd
Re

100
A - [x y] and B =
Fi

7
L100J
5
100 5*
Total annual interest =AB = [x y] +
7 100 100
L100J
Also, x + y = 30000 -(i)
(i) If total interest is ? 1800. Then,
—+ 1800 => 5x + 7y = 180000 ...(h)
100 100 y
Solving (i) and (ii), we get: x = y = 15000.
(ii) If total interest is 12000. Then,
5s 7y
= 2000 => 5x + 7y = 200000 ...(hi)
100 + 100

ReadYourFlow.COM
ALGEBRA OF MATRICES 5.51

Solving (i) and (iii), we get


x = 5000 and y = 25000
76. Let F be the family matrix and R be the requirement matrix. Then,
Men Women Children
Family A 4 6 21
F=
Family B 2 2 4
Calories Proteins
Men 2400 45
R = Women 1900 55
Children 1800 33
Total requirement of calories and proteins of each of the two families is given by the matrix
product
Men Women Children Calories Proteins

w
Men 2400 45
Family A |” 4 6 2
FR = Women 1900 55
Family B 2 2 4

Flo
Children 1800 33

ree
Calories Proteins
Family A "24600 576

F
FR =
Family B 15800 332 or
ur
f
5.8 TRANSPOSE OF A MATRIX
ks

DEFINITION Let A = [ajj] be an m x n matrix. Then, the transpose of A, denoted by A or A', is an n x m


Yo
oo

matrix such that


B

(AT)ij aji for all i = 1,2,...,m;j = 1,2,.


re

Thus, AT is obtained from A by changing its rows into columns and columns into rows.
1 2 3"
ou
ad

12 3 4
2 3 2
Y

For example, if A = 2 3 4 1 , then A T = 3 4 1 ■


3 2 14 4 1 4
nd
Re

TP rV

The first row of A is the first column of A. The second row of A is the second column of A ana
Fi

so on.
5.8.1 PROPERTIES OF TRANSPOSE
We shall now state and prove some properties of transpose of a matrix as theorems given below.
THEOREM 1 For any matrix A, (A7)T =A.
T T T
PROOF Let A = [ajj] be an /n x n matrix. Then, A is an ?7 x m matrix and so (A ) is an m x n
T T
matrix. Thus, the matrices A and (A ) are of the same order such that
((^) T ^ = (A\ [By the definition of transpose]

=>
H' | = (A)jj for all 1=1,2,... , 777 and

Hence, by the definition of equality of two matrices, we obtain


;=1,2,...,77

(A1)1' = A. Q.E.D.

ReadYourFlow.COM
5.52 MATHEMATICS-XII

THEOREM 2 For any tzvo matrices A and B of the same order, (A + B)T = AT + Br.
PROOF Let /l = K;]w x„ and B = [bij]m xn. Then, A + B will be a matrix of the order mxn and so
rT J 1 t T
(A + B) will be a matrix of order n x m. Since A and B are both nxm matrices. Therefore,
AT + BT will be a matrix of the order n x m. Thus, the matrices (A + B)T and AT + BT are of the
same order such that
((/l + B)T)),y = (A + B)/f [By the definition of transpose]
=> ((A + B)T)ij = Ojt + b [By the definition of addition]
P
((A + B)7)^ = (AT)ij+(BT)ij
=> ((A + B)T),y = (AT + BT)jj for all i,; [By the definition of addition]
Hence, by the definition of equality of two matrices, we obtain

w
(A + B)t = AT + Bt
THEOREM 3 If A is a matrix and k is a scalar, then (kA)T = k(AT). Q.E.D.

1 T

Flo
PROOF Let A = [a,-.] be an m x n matrix. Then, for any scalar k, kA is also an m x n matrix and so
T T

ree
(kA) isan«x mmatrix. Againis ann x wmatrix and so L4 is annx mmatrix. Thus, the two
T T
matrices (kA) and kA are of the same order such that

F
mfhj = mp or [By the definition of transpose]
ur
f
=> ((kA)'r)ij = k aji [By the definition of scalar multiplication]
ks
Yo

=> {(kA)7)jj = k (AT)ij [By the definition of transpose]


oo

=> ((iA)r),y = (MT),y [By the definition of scalar multiplication]


B
re

Hence, by the definition of equality of two matrices, we obtain


(kA)T = kAT
ou
ad

Q.E.D.
THEOREM 4 If A and B are two matrices such that AB is defined, then (AB )T = BT AT.
Y

PROOF Let A r-p=[aij]mxn and B=[bijlIxp be tworp matrices. Then AB is an m x p matrix^p and
nd
Re

J J I rp —
therefore (AB) is a p x w matrix. Since A and B are n x m and p x n matrices, therefore B ^ AJ
Fi

is a p x m matrix. Thus, the two matrices (AB) T and B T A T are of the same order such that
((AB)T)i;. = (AB)jj [By the definition of transpose]

=> ((AB)7),] =f airb, [By the definition of matrix multiplication]

=> ((AB)7), = Z bri ajr [By commutativity of multiplication of numbers]

=> ((AB)7)ij = 2 (B7)ir (A7) ■ [By definition of transpose]


J r=1 1
=> ((AB)T)jj = (BT AT)jj [By definition of multiplication of matrices]
Hence, by the definition of equality of two matrices, we obtain (AB)T = BT Ar.
Q.E.D.

ReadYourFlow.COM
ALGEBRA OF MATRICES 5.53

GENERALISATION If A, B, C are three matrices confirmable for the products (AB) C and A (BC ), then
(ABC)7 = CT B7 A7.
REMARK The above lazv is called the reversal law for transposes i.e. the transpose of the product is the
product of the transposes taken in the reverse order.
ILLUSTRATIVE EXAMPLES
LEVEL-1

-1
EXAMPLE 1 IfA = 2 andB = [-2 -1 - 4], verify that {AB) r = B7 A7.
3
SOLUTION We have. [CBSE 2002, 2005]
-1
A = 2 and B = [-2 -1 -4]

w
3
-1 2 1 4
AB = 2 [-2 -1-4] =-4-2 -8

Flo
ee
3 -6 -3 -12

Fr
"2-4 -6"
=> (AB)7 1 -2 -3
for -(i)
4 -8 -12
ur
-i r-2i "2-4 - 6"
ks

Also, B7 A7 = [-2 -1 -4]7 2 -1 [-1 2 3] = 1 -2 -3 -.(ii)


Yo

4 -8 -12
oo

3 -4
B

From (i) and (ii), we observe that (AB)1 = B7 A7.


re

cos 0 sin 0"


EXAMPLE 2 IfA = , then find the values ofQ satisfying the equation A T + A = 72.
sin 0 cos 0
ou
ad

SOLUTION We have,
Y

cos 0 - sin 0 cos 0 sin 0


A = => Ar =
nd

sin 0 cos 0 sin 0 cos 0


Re
Fi

Now, A7 +A = I2
cos 0 sin 0" cos 0 - sin 0 fl 01
=> + sin 0 cos 0 0 1
- sin 0 cos 0
'2 cos 0 0 ri oi
=>
0 2 cos 0 0 1

=> 2 cos 0 = 1 => cos 0 = — => cos 0 = cos — => 0 = 277 Jt ± — neZ
2 3 3'
1 2 2
EXAMPLE 3 If A - 2 1 - 2 is a matrix satisfying AA7 = 913, thenfind the values ofa and b.
a 2 b
SOLUTION We have.
1 2 2 1 2 a
A = 2 1 -2 => A7 = 2 12
a 2 b 2 -2 b

ReadYourFlow.COM
5.54 MATHEMATICS-XII

AAT = 9/3

1 2 2 1 2 a 0 O'
=> 2 1 -2 2 1 2 = 9 0 1 0
a 2 b 2 -2 b 0 0 1

9 0 a + 2b + 4 '9 0 O'
=> 0 9 2a+ 2-2b 0 9 0
<7 + 2b + 4 2a+ 2-2b a2 + 4 + b2 0 0 9
=> a + 2b + 4 = 0, 2a + 2- 2b = 0 and a2 + 4 + b2 =9
=> rt + 2b + 4 = 0, a - b + 1 = 0 and a2 +b2 =5
Solving a + 2b + 4 = 0 and a-b + 1 = 0, we get: a = -2 and b = -1.
0 2y z

low
EXAMPLE 4 Find the values of x,y,z if the matrix A = x y -z satisfy the equation
x -y z
A'T a=/3. [NCERT]

ee
SOLUTION We have,
' 0 2y z'
rF
Ox*

Fr
A = X y -z => AT = 2y y -y
x -y z z -z z
for
It is given that
u
ata=i3
ks
Yo

0 X x’ 0 2y z '10 0'
oo

=> 2y y -y x y -z 0 1 0
B

z -z z x -y z_ 0 0 1
re

2x2 0 0 '10 0
ou

=> 0 6y2 0 = 010


ad

0 0 3z2 0 0 1
Y

=?> 2x2 =1, 6y2 =1, 3z2 =1


nd
Re

1 1
=> x = ±i
Fi

V2'y-±V6'2_±V3
EXERCISE 5.4
LEVEL-1

'2-3
1. Let A = andB= 2 -4 , verify that
-7 5
(i) (2A)T = 2At (ii) (A + B) T =AT + BT
(hi) (A - B)t = A7 -Bt (iv) (AB)t = Bt A7
'3'
2. lfA = 5 and B = [1 0 4], verify that(AB)T =BT AT. [CBSE2002]
2

1 -1 0 12 3
3. Let A = 2 1 3 and B = 2 13 . Find A7, B2^ and verify that
1 2 1 Oil

ReadYourFlow.COM
ALGEBRA OF MATRICES 5.55

(i) {A + B)t =AT + Bt (ii) {AB)t =bt at (hi) (2A)t =2At


-2
4. If A = 4 , B = [1 3 - 6], verify that (AB)T =BT AT.
5
3 4
2 4-1 , find {AB)t.
5. lfA = 2 'B = -1 2
-1 0
2 1
1 -1
'21 3 '
6- (i) For two matrices A and B, A = / B= 0 2 verify that (AB)T = BT AT.
4 1 0
5 0
1 3 1 4
(ii) For the matrices A and B, verify that (AB)t =BT A T, where A = , B=
2 4 2 5

3 4
-1 2 1

w
7. IfAT = -1 2 and B = /findAT-BT. [CBSE2012]
1 2 3
0 1
8. If A =
cos a sin a
- sm a cos a
T
, then verify that A A = l^-

Flo [NCERT]

ree
sin a cos a , verify that A ^ A = 12- [NCERT]
9. If A =

F
- cos a sin a
LEVEL-2 or
ur
f
10. If ; i =1, 2, 3 denote the direction cosines of three mutually perpendicular vectors
pi mj «i'
ks
Yo

in space, prove that AAJ =/,whereA = l2 rn2 n2 .


oo

[h m3 n3
B

ANSWERS
re

4 3
0 1
ou
ad

5. 7. -3 0
15-2
Y

-1 -2
nd
Re

5.9 SYMMETRIC AND SKEW-SYMMETRIC MATRICES


SYMMETRIC MATRIX A square matrix A = [a(j] is called a symmetric matrix, if Ojj = a^ for all i, j.
Fi

■3-1 r
For example, the matrix A = -1 2 5 is symmetric, because
15-2
^12 = -1 = a21, a13=l= a31, a23 =5 = a32 i.e. = fly; for allj.
It follows from the definition of a symmetric matrix that A is symmetric, iff
% = aji for all i, j
<=> (A)ij = (AT),y for all i, j
C5> A = AT.
T
Thus, a square matrix A is a symmetric matrix iff A = A.
'a h g 2+7 1 3
Matrices A = h b f , B = 1 2 3 + 27 are symmetric matrices because A T = A
g f c 3 3 + 27 4
and BT = B.

ReadYourFlow.COM
5.56 MATHEMATICS-XII

SKEW-SYMMETRIC MATRIX A square matrix A = [aq] is a skew-symmetric matrix ifOjj = -a^ for all i, j.
'0 2 -3 '
For example, the matrix A = -2 0 5 is skew-symmetric, because
3-5 0
au - 2, 02i - “ 2 => «i2 - - a2l' a\3 - - 3, fl31 - 3 => a13-~ fl31;
and, #23 = 5, a 32 = — 5 =s> #23 — — ^ 32
It follows from the definition of a skew-symmetric matrix that A is skew-symmetric iff
<=> Ojj = - ajj for all /, j
<=> Wij = {AT)ij for aU/,;'
<=> A = — A^
<=> AT = - A
T

low
Thus, a square matrix A is a skew-symmetric matrix iff A --A.
0 2/3' 0-3 5
T*
Matrices A = - 2 / 0 4 ,B = 3 0 2 are skew-symmetric matrices because A =-A
-3 -4 0 -5 -2 0

ee
and BT =-B.
rF
Fr
ILLUSTRATIVE EXAMPLES for
LEVEL-1
u
ks

EXAMPLE 1 Show that the elements on the main diagonal of a skew-symmetric matrix are all zero.
Yo

[CBSE 2017]
oo

SOLUTION Let A = [ajj] be a skew-symmetric matrix. Then,


B

“ij = ~ aji for all /, [By definition]


re

=> Oil = - an for all values of /


ou

=>
ad

2‘Hi = 0
=> aii = 0 for all values of i
Y

=> au = #22 = #33 = ... = ann = 0.


nd
Re

'0 # 3'
EXAMPLE 2 If the matrix A = 2 b -1 is skew-symmetric, find the values of a, b and c.
Fi

c 1 0
[NCERT EXEMPLAR]
SOLUTION For a skew-symmetric A = [#,y], we have
#y = - ajj for all i * j and Ojj = 0 for all i
TO # 3'
Thus, if A = 2 b -1 is skew-symmetric, then A22 = 0, A12 = - A2i and A31 = -A13.
c 1 0
b =0, # = -2 and c = - 3
'0 # 3
ALITER If A = 2 b -1 is skew-symmetric, then
c 1 0
At=-A

ReadYourFlow.COM
ALGEBRA OF MATRICES 5.57

0 a 3l7 0 a 3
=> 2 b -1 2 b -1
c 1 0 c 1 0
0 2c 0 -a -3
=> a b 1 -2 -b 1
3-10 -c -1 0
=> 2 = -a, c = -3 and b =-b => a = -2, c =-3 and 2b =0 => a--2, b - 0 and c = - 3
EXAMPLE 3 Let Abe a square matrix. Then,
(i) A + A r is a symmetric matrix [NCERT]
T*
(ii) A - A is a skew-symmetric matrix. [NCERT1
(iii) AA r and AT A are symmetric matrices.

low
SOLUTION (i) Let P = A + A7. Then,
PT = (A+A7)7 = A7 + (A7)7 [v (A + B)7 =A7 + B7]
=> P7 = A7 + A [v (A7)7=A]

ee
=> P7 = A + A7 = P
rF [By commutativity of matrix addition]

Fr
P is a symmetric matrix. for
(ii) Let Q = A - AT.Then,
u
Q7 = (A - A7)7 = A7 - (A7)7 [v (A + B)7 = A7 + B7]
ks

=> Q7 = A7 - A [v (A7)7 = A]
Yo
oo

=> qT = - (A-A7) = - Q
B

=> Q is skew-symmetric
re

(iii) We have,
ou

(AA7)7 = (A7)7 A7
ad

[By reversal law]


Y

=> (AA7)7 = AA7 [••• (,AT)T = A]


nd

=> AA is symmetric
Re

Similarly, it can be proved that A7 A is symmetric.


Fi

EXAMPLE 4 Prove that every square matrix can be uniquely expressed as the sum ofa symmetric matrix
and a skew-symmetric matrix. [NCERT]
SOLUTION Let be a square matrix. Then,
A= + AT) + ^(A - AT) = P + Q (say), where P = ^{A + AT) and Q = ^(A -AT).

PT = ^(A + A^J (A+ AT)


Now, ['■• (kA)T=kAT]

=> P7 = ±(AT+(AT)T) [v (A + B)t =Ar + BrJ

=> P7 = ^(AT+A) [■•• (Ar)r =A]

=> PT = ^(A+At) = P [By commutativity of matrix addition]

P is a symmetric matrix.

ReadYourFlow.COM
5.58 MATHEMATICS-XII

Also, Qt
=(I<^T))t \(A-Ar) ^At-(At)t)

=> qt = ^(At-A) = - j(.a-at) = ~ Q


Q is a skew-symmetric matrix.
Thus, A = P + Q, where P is a symmetric matrix and Q is a skew-symmetric matrix.
Hence, A is expressible as the sum of a symmetric and a skew-symmetric matrix.
Uniqueness: If possible, let A = R + S, where R is symmetric and S is skew-symmetric. Then,
At = (R + S)T =Rt + ST
=> A7 = R - S [v RT =R andST = - S]
Now, A = R + S and AT = R - S

=> R = ~(A + A7) = P, S = - (A -Ar) = Q-

w
2 2
Hence, A is uniquely expressible as the sum of a symmetric and a skew-symmetric matrix.

B commute.
Flo
EXAMPLE 5 IfA and B are symmetric matrices, then show that AB is symmetric ijfAB = BA i.e. A and
[NCERT]

ee
SOLUTION AB is symmetric

Fr
<=> (AB)T = AB
<=> BT A7 = AB [v (AB)T =Bt A7]
for
ur
o BA = AB [v A and B are symmetric matrices .\ AT =A,BT= B]
ks
Yo

EXAMPLE 6 Show that the matrix B AB is symmetric or skew-symmetric according as A is symmetric


oo

or skew-symmetric. [NCERT EXEMPLAR]


B

SOLUTION CASE i Let be a symmetric matrix. Then, A^ = A.


re

Now, (Bt AB)t = BJ AT (Bt)t [By reversal law]


ou
ad

=> (Bt AB)T = Bt At B [■■■ (Bt)T=B]


Y

=> (Bt AB)J = Bt AB [••• At=A]


nd
Re

B T AB
rt
is a symmetric matrix.
Fi

OASEJI Let A be a skew-symmetric matrix. Then, Ar =- A.


Now,
(Bt AB)t = Bt At (Bt)t [By reversal law]
=> (BT AB)t = BT A7 B [•■• (B7')T=B]
=> (Bt AB)T = Bt (-A) B [■■■ At=-A]
=> (BT AB)T = - BT AB
B AB is a skew-symmetric matrix.
EXAMPLE 7 Let A and B be symmetric matrices of the same order. Then, show that
(i) A + B is a symmetric matrix.
(ii) AB - BA is a skew-symmetric matrix. [NCERT]
(iii) AB + BA is a symmetric matrix.

ReadYourFlow.COM
ALGEBRA OF MATRICES 5.59

T T
SOLUTION Since A and B are symmetric matrices. Therefore, A =A and B = B.
(i) We have,
(A + B)t = AT + Bt = A + B [v At =A,Bt =B]
A + B is symmetric
(ii) We have,
(AB - BA)t = (AB) T - (BA) T
=> (AB-BA)t = Bt At - At Bt [By reversal law]
=> (AB - BA)T = BA - AB [v Bt =B, At =A]

=> (AB-BA)T = - (AB-BA)


AB - BA is skew-symmetric.
(iii) We have.

w
(AB + BA)7 =(AB)7 + (BA)7
= BTAT +atb7 [By reversal law]
= BA + AB

Flo [■A7 =A, B7 = B]

ee
=AB+BA

Fr
AB + BA is symmetric matrix.
3 2 3'
for
ur
EXAMPLE 8 Express the matrix A= A 5 3 as the sum of a symmetric and a skew-symmetric
2 4 5
ks
Yo

matrix.
oo

SOLUTION We have,
B

3 2 3 3 4 2
re

A = 4 5 3 => A‘T = 2 5 4
3 3 5
ou

2 4 5
ad
Y

3 2 3 3 4 2 6 6 5
A + A7 = 4 5 3 + 254 6 10 7
nd
Re

2 4 5 3 3 5 5 7 10
Fi

3 2 3 ' 3 4 2' 0 -2 1
and. = 4 5 3 2 5 4 = 2 0-1
2 4 5 3 3 5 -1 1 0
3 3 5/2' 0 -1 1/2
Let P = t(A + AT) = 3 5 7/2 and, Q=UA-A7) = 1 0 -1/2
5/2 7/22 5 2 -1/2 1/2 0

3 3 5/2 3 3 5/2
Then, P7 = 3 5 7/2 3 5 7/2 = P
5/2 7/2 5 5/2 7/2 5

0 -1 1/2 0 1 -1/2 0 -1 1/2'


and, Q7 = 1 0 -1/2 -1 0 1/2 1 0 -1/2 =-Q
-1/2 1/2 0 1/2 -1/2 0 -1/2 1/2 0

Thus, P is symmetric and Q is skew-symmetric.

ReadYourFlow.COM
5.60 MATHEMATICS-XII

3 3 5/2 0 -1 5/2 3 2 3
Also, P + Q = 3 5 7/2 + 1 0 5/2 4 5 3 = A
5/2 7/2 5 -1/2 5/2 0 2 4 5
Thus, A is expressible as the sum of a symmetric matrix P and a skew-symmetric matrix Q.
LEVEL-2

EXAMPLE 9 Show that all positive integral powers of a symmetric matrix are symmetric.
SOLUTION Let A be a symmetric matrix and n eN. Then,
A" = AAA ... A upto H-times
=> (A'V = (AAA ... A upto/7-times)^
=> (A")T = (ATAT A7 ...A1 upto n-times) [By reversal law]
=> (AY = (ATf = A" [■■• AT =A]

low
Hence, A" is also a symmetric matrix.
EXAMPLE 10 Shoiu that positive odd integral powers of a skew-symmetric matrix are skew-symmetric
and positive even integral powers of a skew-symmetric matrix are symmetric.

ee
SOLUTION Let A be a skew-symmetric matrix. Then, AT = - A.
rF
Fr
We have, (A")T = (AT)n for all n e N. for [See Example 6]
(A")7 = (-A)" [v At=-A]
u
=> (A”)7 = (-1)” A”
ks
Yo

A" if 77 is even
oo

=> (A")t =
_ ^7i if 77 is odd
B
re

Hence, A" is symmetric if n is even and skew-symmetric if n is odd.


EXAMPLE ll A matrix which is both symmetric as well as skew-symmetric is a null matrix.
ou
ad

[NCERT EXEMPLAR]
Y

SOLUTION Let A = [Ojj] a matrix which is both symmetric and skew-symmetric.


nd
Re

Now, A = [Ojj] is a symmetric matrix


Fi

=> Ojj = Oji for all i',;'


Also, A = [ojj] is a skew-symmetric matrix.
Ojj = - ag for all i, j
=> ajj = - Ojj for all i, j ...(h)
From (i) and (ii), we obtain
Ojj = -Ojj for all/,;
=> 2Ojj = 0 for all i, j
=> Ojj = 0 for all i, j
=> A = [Ojj] is a null matrix.
EXERCISE 5.5
LEVEL-1

[2 31
1. If A = , prove that A - A T is a skew-symmetric matrix. [CBSE2001]
4 5

ReadYourFlow.COM
ALGEBRA OF MATRICES 5.61

[3-41 r
2. If A = ^ ^ , show that A - A is a skew-symmetric matrix.

5 2 x
3. If the matrix A = y z - 3 is a symmetric matrix, find x, y, z and t.
4 t -7
3 2 7
4. Let A = 14 3. Find matrices X and Y such that X + Y = A, where X is a symmetric
-2 5 8
and Y is a skew-symmetric matrix.
'4 2 -1'
5. Express the matrix A = 3 5 7 as the sum of a symmetric and a skew-symmetric
1 -2 1
matrix. [CBSE 2008]

w
2 4
6. Define a symmetric matrix. Prove that for A = , A + AT is a symmetric matrix where
5 6
A1 is the transpose of A.

Flo
ee
41
7. Express the matrix A = r ^3 ^ as the sum of a symmetric and a skew-symmetric

Fr
matrix. for
ur
3 -2 -4
8. Express the matrix 3 - 2 - 5 as the sum of a symmetric and skew-symmetric matrix
ks

-1 1 2
Yo

[CBSE 2010]
oo

and verify your result.


eB

ANSWERS

'3 3/2 5/2 0 1/2 9/2


r
ou

3. x = 4,1/ = 2, 2 e C, f = -3 4. X = 3/2 4 4 , Y = -1/2 0 -1


ad

-9/2 1 0
Y

5/2 4 8
'4 5/20 0 -1/2 -1
nd
Re

5. Symmetric matrix = 5/2 5 5/2 , Skew-symmetric matrix = 1/2 0 9/2


Fi

0 5/2 1 1 -9/2 0
„ 3 -5/2
7. Symmetric matrix = , Skew-symmetric matrix =
L - 3/2 0
3 1/2 -5/2 0 -5/2 -3/2
8. Symmetric matrix = 1/2 -2 2 , Skew-symmetric matrix = 5/2 0 -3
-5/2 -2 2 3/2 3 0

VERY SHORT ANSWER QUESTIONS (VSAQs)

Answer each of the following questions in one word or one sentence or as per exact requirement of the
question:
1. If A is an m x n matrix and Bisnx p matrix does AB exist? If yes, write its order.
'3 -1]
'2 1 4
2. If A = and B= 2 2 . Write the orders of AB and BA.
4 1 5
1 3

ReadYourFlow.COM
5.62 MATHEMATICS-XII

4 3 -4
3. If A = and B = ^ , write AB.
1 2
1
4. If A = 2 , write A47’. [CBSE 20091
3

5. Give an example of two non-zero 2x2 matrices A and B such that AB = O.

I
"2 3
6. IfA = , find A + AT.
5 7
i 0
7. If A = Q . , write A2.
i

cos x sin x 7Z T"


8. If A = find x satisfying 0 < x < — when A + A = 1
- sin x cos x '

w
cos x - sin x
9. If A = , find AA T
sin x cos x

10. If
[
1 0" + 2 "
y s i
0"

Flo
_ 2 = I, where / is 2 x 2 unit matrix. Find x and y.

ee
Fr
i -i
11. If A = , satisfies the matrix equation A = kA, write the value of k.
-i i for
ur
ri ii satisfies A4 = XA, then write the value of X.
12. If A =
1 1
ks

-1 0 O'
Yo
oo

13. If A = 0-1 0 , find A2.


0 0-1
eB

-1 0 O'
r

14. If A = 0-1 0 , find A3.


ou
ad

0 0-1
Y

-3 0] 4
15. If A = 0 -3 'hndA •
nd
Re
Fi

16- If[x 2] ^ = 2, find x

17,. If A = [Ojj] is a 2 x 2 matrix such that Ojj = i + 2j, write A. [CBSE 2008]
2 3' 3 -6'
18.. Write matrix A satisfying A +
-1 4 -3 8 ‘
19. If A =[ay] is a square matrix such that ^ =i:2 - j:2 , then
- write whether A is symmetric or
skew-symmetric.
20. For any square matrix write whether AA7 is symmetric or skew-symmetric.
21. If A = [fly] is a skew-symmetric matrix, then write the value of I 0^ .
22. If A = [fly ] is a skew-symmetric matrix, then write the value of I E ^ • .
' )
23. If A and B are symmetric matrices, then write the condition for which A B is also symmetric.
24. If B is a skew-symmetric matrix, write whether the matrix AB Ar is symmetric or skew-
symmetric.

ReadYourFlow.COM
ALGEBRA OF MATRICES 5.63

25. If B is a symmetric matrix, write whether the matrix AB A T is symmetric or skew-


symmetric.
26. If y4 is a skew-symmetric and n eN such that iAn)T =XAU, write the value of A.
27. If A is a symmetric matrix and n e N, write whether An is symmetric or skew-symmetric
or neither of these two.
28. If A is a skew-symmetric matrix and n is an even natural number, write whether A” is
symmetric or skew-symmetric or neither of these two.
29. If A is a skew-symmetric matrix and n is an odd natural number, write whether A" is
symmetric or skew-symmetric or neither of the two.
30. If A and B are symmetric matrices of the same order, write whether AB - BA is symmetric
or skew-symmetric or neither of the two.
31. Write a square matrix which is both symmetric as well as skew-symmetric.

low
1 3 O' 5 6
32. Find the values of x and y, if 2 + y [CBSE 20081
0 x 1 2 1 8 '
x+3 4 5 41 [CBSE 20081
33. If , find x and !/.
_3/ - 4 x + y_ 3 9

ee
rF
2x-y 5 _ 6 5

Fr
34. Find the value of x from the following: “ [CBSE 20091
.V = 3 -2 '
X -y 21 J2 2'
for [CBSE 20091
35. Find the value of i/, if ^
5 3 5'
u
1 2
ks

3x + y -y [CBSE 20091
36. Find the value of x, if
Yo

2y-x 3 -5 3 ’
oo

37. If matrix A =[1 2 3], write AA T . [CBSE 20091


B
re

T 2x + y 3y ' 6 0 [CBSE 2010]


3S- If 0 , then find x.
4 6 4
ou
ad

n 2i , find A + A T .
Y

39. IfA = [CBSE 2010]


3 4
nd
Re

a+b 2 [6 51
40. If “ , then find a.
2 2
Fi

3 b
41. If A is a matrix of order 3x4 and B is a matrix of order 4x3, find the order of the matrix of
AB. [CBSE 2010]
cos a - sm a [CBSE 2010]
42. IfA = is identity matrix, then write the value of a.
sin a cos a
1 2 3 1 '7 11' [CBSE 2010]
43. If , then write the value of k.
3 4 2 5 k 23
44. If l is the identity matrix and A is a square matrix such that A = A, then what is the value of
(/ + A)2 - 3A?
1 2
45. If A = is written as B + C, where B is a symmetric matrix and C is a skew-symmetric
0 3
matrix, then find B.
T T
46. If A is 2 x 3 matrix and B is a matrix such that A B and BA both are defined, then what is
the order of B?

ReadYourFlow.COM
5.64 MATHEMATICS-XII

47. What is the total number of 2 x 2 matrices with each entry 0 or 1?


x x-y 3 1
48. If = , then find the value of y. [CBSE2011]
2x + y 7 8 7

49. If a matrix has 5 elements, write all possible orders it can have. [CBSE2011]
50. For a 2 x 2 matrix A = [fl^] whose elements are given by Ojj - - , write the value of
1
[CBSE2011]
51. If.J23 + y -1i = 10
5 ' frHd the value of x. [CBSE2012]

9 -1 4 A 2 -1
52. If -A + , then find matrix A. [CBSE2013]
-2 1 3 0 4 9

low
a-b 2a+ c -1 5'
53. If , find the value ofb. [CBSE2013]
2a-b 3c+ d 0 13
0 1 -2
54. For what value of x, is the matrix A = -10 3 a skew-symmetric matrix? [CBSE 2013]

ee
x -3 0
rF
Fr
' 2 -2'
55. If matrix A = and A 2 =pA, then write the value of p. [CBSE 2013]
for
-2 2
u
56. If A is a square matrix such that 7l = A, then write the value of 7 A-(I + A) , where/ is the
ks
Yo

identity matrix. [CBSE 2014]


oo

"3 4] [1 y] [7 0'
B

57. If 2 + — , find x-y. [CBSE 2014]


5 x 0 1 10 5
re

1 0"
ou

58. If[x 1] = 0, find x. [CBSE 2014]


ad

-2 0
Y

a+ 4 3b '2a + 2 b + 2
59. If , write the value of a- 2b. [CBSE 2014]
nd
Re

8 -6 8 a-8b
Fi

60. Write a 2 x 2 matrix which is both symmetric and skew-symmetric. [CBSE 2014]
' xy 4 ~8 w~
61. If write the value of (x + y + z). [CBSE 2014]
z+ 6 x+y 0 6 '
-3^/1 , if z * j
62. Construct a 2 x 2 matrix A = [fly ] whose elements fly are given by Ojj = • 2
(I+7)2 ,iii=]
x+y “2 11T 1
63. If
[ x-y 4 3
2 , then write the value of (x, y).

0 2b -2
64. Matrix A = 3 13 is given to be symmetric, find the values of fl and Iz. [CBSE 2016]
3fl 3 -1
65. Write the number of all possible matrices of order 2x2 with each entry 1, 2 or 3.
[CBSE 2016]

ReadYourFlow.COM
ALGEBRA OF MATRICES 5.65

-1 0 -1 1
66. If [2 1 3] -1 1 0 0 = A, then write the order of matrix A. [CBSE2016]
0 1 1 -1
3 5
67. If A =
7 9 ]
is written as A = P + Q, where as A = P + Q, where P is symmetric and Q is

skew-symmetric matrix, then write the matrix P. [CBSE 2016]


68. Let A and B be matrices of orders 3x2 and 2x4 respectively. Write the order of matrix AB.
[CBSE 2017]
ANSWERS
1 2 3
-7
1. Yes, mx p 2. 2 x 2 and 3x3 3. 4. 2 4 6
2
3 6 9

w
'2 0" 0 0 4 8 -1 Cf
5. A = ,B= 6. 7.
3 0 2 -1 8 14 0 -1

8. -
3
TZ n oi
9. 0 1

Flo
m x - 0, y =-2 11. 2

ee
'81 0

Fr
12. 8 13. - A or, /3 14. A 15.
0 81
3 5 1-9 or
ur
16. -2 17. 18. -2 19. skew-symmetric
4 6 4
f
ks

20. symmetric 21. 0 2.2. 0 23. AB = BA


Yo

27. symmetric
oo

24. skew-symmetric 25. symmetric 26. (-If


B

28. symmetric 29. skew-symmetric 3-0. skew-symmetric 31. null matrix


re

32. x = 3, y = 3 33.x = 2,y=7 34. x=2 35. y =1


[2 5~
ou

36. x =1 37. 14 38. x = 3, y = 0 39.


ad

5 8
Y

40. 4 41. 3x3 42. a = 0 43. 17


n r
nd
Re

44. / 45. 46. 2x3 47. 16


1 3
Fi

48. 2 49. 1 x 5,5 x 1 50. 1/2 51. x = 3


8-3 5'
52. 53. b=2 54. x =2 55. 4
-2 -3 -6

56. -I 57. x - 2, y =-8 58. x = 2 59. 0

60.
ro oi
61. 0
r4 i/2"
62- 5/2 16 63. (-1,1)
0 0
2 h —3 3 6
64. a = -—,o= 65. 34 = 81 66. 1x1 67. 68. 3x4
3 2 6 9
MULTIPLE CHOICE QUESTIONS (MCQs)
1 0 0
1. If A = 0 1 0 , then A is equal to
a b -1
(a) a null matrix (b) a unit matrix (c) -A (d) A

ReadYourFlow.COM
5.66 MATHEMATICS-XII

2. lfA = q , n e N, then A4” equals

ro r ™r°
(b) 0 °i
"1 01
(a) f 0 0 (c)
0 1 (d) F D
3. If ^ and B are two matrices such that AB = A and BA = B, then is equal to
(a) B (b) A (c) 1 (d) 0
4, If AB = A and BA = B, where A and B are square matrices, then
(a) B2 =B and A2 =/l (b) B2 * Band A2 -A
(c) A2 * A, B2 =B (d) A2 * A, 32 ^ B
5. If A and B are two matrices such that AB = B and BA = A, then A2 + B2 is equal to
(a) 2 AB (b) 2 BA {c) A + B (d) AB

low
2n
cos--- - sin —
6. If 7 7 n oi , then the least positive integral value of k is
2 TC 2n 0 1
sm cos----
7 7

ee
(a) 3 (b) 4 (c) 6 (d) 7
rF
Fr
7. If the matrix 2lB is zero, then
(a) It is not necessary that either A = 0 or, B = 0 (b) = 0 or B = O
for
(c) A = O and B = O (d) all the above statements are wrong
ou
a 0 0
ks

8. Let A = 0 a 0 , then Ari is equal to


0 0 rt
oo
Y
B

a” 0 0 an 0 0 a'1 0 na 0 0
re

(a) 0 a” 0 (b) 0 r? 0 (c) 0 a" 0 (d) 0 na 0


0 0 0 0 rr 0 0 a" 0 0 na
ou
ad

9. If A, B are square matrices of order 3, A is non-singular and AB = 0, then B is a


Y

(a) null matrix (b) singular matrix (c) unit matrix (d) non-singular matrix
nd
Re

~n 0 0 a1 a2 a3
10. If 71= 0 n 0 and B = &2 ^3 / then AB is equal to
Fi

0 0 n ci c2 c3
(a) B (b) nB (c) B" (d) A + B

11. If A = fl 1 , then An (where neN) equals


0 1
"1 na (b) 1 n2 a '1 na n m
(a) 0 1 (c) 0 0
o 1
1 2 x 1 -2 y
12. If A = 0 1 0 and B = 0 1 0 and AB = l3 , then x + y equals
0 0 1 0 0 1
(a) 0 (b) -1 (c) 2 (d) none of these
13. If A = ^ 1 B- a j and (A + B)2 = A2 + B2, then values of (rand b are
-11 'B~ b
(a) a = 4,b =1 (b) a = 1,B = 4 (c) fl = 0,1) = 4 (d) a = 2,b =4

ReadYourFlow.COM
ALGEBRA OF MATRICES 5.67

a p is such that A2 =1, then


14. If A =
y -a
(a) 1 + a2 + Py = 0 (b)l-a2+pY=0 (c)l-a2-pY = 0 (d)l + a2-PY = 0
15. If S = [Sjj ] is a scalar matrix such that su = k and A is a square matrix of the same order, then
AS = SA = ?
(a) Ak (b) k + A (c)kA (d) kS
16. If A is a square matrix such that A 2 = A, then (/ + A)3 - 7A is equal to
(a) A (b) 1-A (c) f (d) 3A
17. If a matrix A is both symmetric and skew-symmetric, then
(a) A is a diagonal matrix (b) A is a zero matrix
(c) A is a scalar matrix (d) A is a square matrix

w
'0 5-7'
18. The matrix -5 0 11 is
7 -11 0
(a) a skew-symmetric matrix

Flo
(b) a symmetric matrix

ee
(c) a diagonal matrix (d) an upper triangular matrix

Fr
19. If A is a square matrix, then AA is a
(a) skew-symmetric matrix (b) symmetric matrixfor
ur
(c) diagonal matrix (d) none of these
20. If A and B are symmetric matrices, then ABA is
ks

(a) symmetric matrix (b) skew-symmetric matrix


Yo
oo

(c) diagonal matrix (d) scalar matrix


B

"5 xl
21. If A = q and A = A
re

Ly
(a) x = 0, y = 5 (b) x + y =5 (c) x = y (d) none of these
ou
ad

T rr
22. If A is 3 x 4 matrix and B is a matrix such that A B and BA are both defined. Then, B is of
Y

the type
nd
Re

(a) 3x4 (b) 3x3 (c) 4x4 (d) 4x3


•9 *9
Fi

23. If A = [Ojj] is a square matrix of even order such that Ojj -i - j , then
(a)A is a skew-symmetric matrix and | A | = 0
(b)A is symmetric matrix and | A | is a square
(c)A is symmetric matrix and | A | = 0
(d)none of these,
cos 0 - sin 0' , then A ^ + A = /2 , if
24. If A =
sin 0 cos 0

(a) 0 = uk ,n eZ (b) 0 = (2n + l)| rn eZ

(c) 0 = 2/7 rt + — , n e Z (d ) none of these


3
'2 0-3'
25. If A = 4 3 1 is expressed as the sum of a symmetric and skew-symmetric
-5 7 2
matrix, then the symmetric matrix is

ReadYourFlow.COM
5.68 MATHEMATICS-Xm

2 2-4 2 4-5 4 4-8 '1 0 O'


(a) 2 3 4 (b) 0 3 7 (c) 4 6 8 (d) 0 1 0
-4 4 2 -3 1 2 -8 8 4 0 0 1
26. Out of the following matrices, choose that matrix which is a scalar matrix:
'0 O' 'O'
ro oi -o o O'
(a) 0 0 (b) (c) 0 0 (d) 0
ooo
0 0 0

27. The number of all possible matrices of order 3x3 with each entry 0 or 1 is
(a) 27 (b) 18 (c) 81 (d) 512
28. Which of the given values of x and y make the following pairs of matrices equal?
"3.y + 7 5
and.
ro y - 2
y +1 2 - 3j 8 4

w
2 12
(a) Y = - — , y - 7 (b) y =7, y = - - (c) y =----, 4 = — (d) Not possible to find
3 3 5
29. H A = ^ ^

Flo
and kA = 0 3n , then the values of k, a, b, are respectively

ee
2a 24

Fr
(a) - 6, -12, -18 (b) - 6, 4, 9 (c) - 6, - 4, - 9 for (d) -6,12,18
ur
fl 0] 0 1 cos 0 sin 0
30. If I = and B = , then B equals
0 1 ,/ = -1 0 - sin 0 cos 0
ks
Yo

(a) / cos 0 + / sin 0 (b) I sin 0 + / cos 0


oo

(c) / cos 0 - / sin 0 (d) -1 cos 0 + / sin 0


B

1 -5 7
re

31. The trace of the matrix A = 0 7 9 is


ou

11 8 9
ad
Y

(a) 17 (b) 25 (c) 3 (d) 12


32. If .4 = [(7;y] is a scalar matrix of order nx rc such that % = k for all/, then trace ofA is equal to
nd
Re

(a) nk (b) n + k (d) none of these


Fi

0 0 4
33. The matrix A = 0 4 0 is a
4 0 0
(a) square matrix (b) diagonal matrix (c) unit matrix (d) none of these
34. The number of possible matrices of order 3x3 with each entry 2 or 0 is
(a) 9 (b) 27 (c) 81 (d) none of these
2y + y 4y] [7 7y-13’
35. If , then the value of .r - y is
5y-7 4y y+6

(a) y = 3, y -1 (b) y = 2, y = 3 (c) y = 2, y = 4 (d) y = 3, y = 3


36. If A is a square matrix such that A2 = 1, then (A -1)3 + (A + /)3 -7 A is equal to
(a) A (b) I-A (c) I + A (d) 3A

ReadYourFlow.COM
ALGEBRA OF MATRICES 5.69

37. IfA and B are two matrices of order 3 x m and 3 x n respectively and m = n, then the order of
5A -2B is
(a) mx 3 (b) 3x3 (c) m x n (d) 3xn
38. If A is a matrix of order m x n and B is a matrix such that AB1 and BT A are both defined, then
the order of matrix B is
(a) mxn (b) nxn (c) nxm (d) mxn
39. If A and B are matrices of the same order, then ABT -BrA is a
(a) skew-symmetric matrix (b) null matrix
(c) unit matrix (d) symmetric matrix
40. If matrix A = [«//]2x2 ' w^ere = | 0' if/t/ ' e£Iualt0
(a) / (b) A (c)C (d) -I

w
-cot 1 (kx) tan -If *
1 71
, then A- Bis equal to
41. If A = —
7T

Flo
sin-1 * -tan

ee
7C

Fr
(a) / (b) 0 (c) 21 (d) ^
for
42. If A and B are square matrices of the same order, then (A + B) (A - B) is equal to
ur
(a) A2 -B2 (b) A2 -BA-AB-B2
ks

(c) A2-B2 + BA-AB (d) A2-BA + B2+AB


Yo
oo

2 -1 3~
B

43. If A = and B =
-4 5 1
re

(a) only AB is defined (b) only BA is defined


ou
ad

(c) AB and BA both are defined (d) AB and BA both are not defined
Y

'0 -5 8~
nd
Re

44. The matrix A = 5 0 12 is a


-8 -12 0
Fi

(a) diagonal matrix (b) symmetric matrix


(c) skew-symmetric matrix (d) scalar matrix
'1 0 O'
45. The matrix A = 0 2 0 is
0 0 4
(a) identity matrix (b) symmetric matrix
(c) skew-symmetric matrix (d) diagonal matrix
____ANSWERS
1. (b) 2. (c) 3. (a) 4. (a) 5. (c) 6. (d) 7. (a) 8. (c) 9. (a)
10. (b) 11. (a) 12. (a) 13. (b) 14. (c) 15. (c) 16. (c) 17. (b) 18. (a)
19. (d) 20. (a) 21. (c) 22. (a) 23. <d) 24. (c) 25. (a) 26. (a) 27. (d)
28. (d) 29. (c) 30. (a) 31. (a) 32. (a) 33. (d) 34. (d) 35. (b) 36. (a)
37. (d) 38. (d) 39. (a) 40. (a) 41. <d) 42. (c) 43. (c) 44. (c) 45. (d)

ReadYourFlow.COM
5.70 MATHEMATICS-XII

SUMMARY
1. A set of mn numbers (real or imaginary) arranged in the form of a rectangular array of m
rows and n columns is called an m x n matrix.
2. A matrix having only one row is called a row matrix.
3. A matrix having only one column is called a column matrix.
4. A matrix in which the number of rows is equal to the number of columns, say n, is called a
square matrix of order n.
5. The elements of a square matrix A = [a^j] n xn for which i = j, i.e. the elements
all' a22' ' ann are called the diagonal elements and the line along which they lie is called
the principal diagonal or leading diagonal.
6. A square matrix A = [ajj]n * „ is called a diagonal matrix if all the elements, except those in
the leading diagonal, are zero i.e. Ojj = 0 for /' * /.
7. A square matrix A = [(ijj]nxlI is called a scalar matrix, if

w
(i) aij = 0 for all i * j and, (ii) fl,,- = c for all i, where c * 0.
8. A square matrix A = [Ojj]n xn is called an identity or a unit matrix, if

Flo
(i) Ojj = 0 for all i * j and, (ii) iijj = 1 for all i.

ee
9. A matrix whose all elements are zero is called a null matrix or a zero matrix.

Fr
10. A square matrix A = [djj] is called
(i) an upper triangular matrix, if cijj - 0 for all i > j for
ur
(ii) a lower triangular matrix, if Ojj = 0 for all i < j.
11. Two matrices A = [Ojj]mxn and B = [h,] tjim xn of the same order are equal, if
ks

Ojj - bjj for all z = 1, 2,..., w; j =1,2,... ,n


Yo
oo

12. Ii A = xn and 6 = [bjj]m XI1 are two matrices of the same order m x n, then their sum
A + B is an w x n matrix such that (A + B)(y = Ojj + bjj for / = 1, 2,..., m and j = 1,2, 3,..., n
B

Following are the properties of matrix addition:


re

(i) Commutativity: if A and B are two matrices of the same order, then A + B = B + A.
ou
ad

(ii) Associativity: If A, B, C are three matrices of the same order, then


Y

(A + B) + C = A + (B + C)
(iii) Existence of Identity: The null matrix is the identity element for matrix addition
nd
Re

i.e.. A +0 = A+0 x A
Fi

(iv) Existence ofInverse: For every matrix A = [Ojj] rn xn there exists a matrix - A = [- Ojj]m xn
such that A +(- A) = O =(- A) + A.
(v) Cancellation Laws: If A, B, C are three matrices of the same order, then
A + B = A + C => B =C and, B + A=C + A=> B=C.
13. Let A = [Ojj] be an mx n matrix and k be any number called a scalar. Then, the matrix
obtained by multiplying every element of A by k is called the scalar multiple of A by k and is
denoted by kA.
Thus, kA =[k aij]m xn.
Following are the properties of scalar multiplication:
If A, B are two matrices of the same order and k, l are scalars, then
(i) k (A + B) = kA + kB (ii) (k + I) A = kA + IA (iii) (kl) A = k (IA) = l {kA)
(iv) (-k) A=-(kA)=k(-A) (v) 1 A =A (vi) (-1) A = -A
14. If A and B are two matrices of the same order, then A - B = A + (- B).
15. Two matrices A and B are conformable for the product AB if the number of columns in A is
same as the number of rows in B.

ReadYourFlow.COM
ALGEBRA OF MATRICES 5.71

If A - [cijj]m xn and B — \bjj] n x p are two matrices, then AB is an m x p matrix such that
n
(AB)ij = Y, air brj ■
r=1
Matrix multiplication has the following properties:
(i) Matrix multiplication is not commutative.
(ii) Matrix multiplication is associative i.e. (AB) C = A (BC) wherever both sides of the
equality are defined.
(iii) Matrix multiplication is distributive over matrix addition
i.e. A(B + C)=AB + AC and (B+ C) A = BA+CA wherever both sides of the
equality are defined.
(iv) If A is an m x n matrix, then Im A = A = A In .

w
(v) If A is an m x n matrix and O is a null matrix, then A in x n O, i x p = 0m x p and
Op xmx A m xn = 0 p x n •

Flo
i.e., the product of a matrix with a null matrix is a null matrix.
16. If is a square matrix, then we define A] = A and A n+ 1 = An A

ee
Fr
17. If A is a square matrix and fl0/ fli/ / an are constants, then
Uq A” + An ^ + (I2 A n-2 + ... + aJt_lA+anI is called a matrix polynomial.
for
ur
T
18. Let A = [a.jj] be an m x n matrix. Then, the transpose of A, denoted by A , is an n x m matrix
ks

such that {AT)jj =ap for all i =1, 2, m; j =1, 2,..., n.


Yo
oo

Following are the properties of transpose of a matrix:


B

(i) (AT)T=A (ii) (A + B)T =AT +BT T


(iii) (kA) -k A
re

(iv) (AB)T =BT A7 (v) (ABC)7 =CT B7 A7


ou
ad

T*
19. A square matrix A = [(ijj] is called a symmetric matrix, if Ojj = iijj for all i, j i.e. A = A .
Y

20. A square matrix A = [cijj] is called a skew symmetric matrix, if = - cijj for all i, j
nd
Re

T
i.e. A = -A
Fi

21. All main diagonal elements of a skew-symmetric matrix are zero.


22. Every square matrix can be uniquely expressed as the sum of a symmetric and a skew-
symmetric matrix.
23. All positive integral powers of a symmetric matrix are symmetric matrices.
24. All odd positive integral powers of a skew-symmetric matrix are skew-symmetric matrices.

ReadYourFlow.COM
Re
Fiad
nd
Y
ou
Yo
re
B
oo
ur
ks
for
Flo
Fr w
ee

ReadYourFlow.COM
CHAPTER 6
DETERMINANTS

6.1 DETERMINANTS
DEFINITION Every square matrix can be associated to an expression or a number which is known as its
determinant. If A= [a-A is a square matrix of order n, then the determinant of A is denoted by det A or,
| A | or,

low
all a12 ••• ^ ••• aln
a2\ a22 ••• fl2; ••• a2n

“il 42 ••• °ij ••• “in

ee
rF
Fr
an2 ••• Onj ••• ann

6.1.1 DETERMINANT OF A SQUARE MATRIX OF ORDER 1


for
If A = is a square matrix of order 1, then the determinant of A is defined as
u
Ml = au or> II = fln
ks
Yo
oo

6.1.2 DETERMINANT OF A SQUARE MATRIX OF ORDER 2


B

If A = ^11 a\2 is a square matrix of order 2, then the expression an 022 ~al2 a21 is defined as
a2\ a22
re

the determinant of A.
ou
ad

a\\ a\2
i.e. Ml = - an a22 - «i2 a2l
Y

a2\ a22
Thus, the determinant of a square matrix of order 2 is equal to the product of the diagonal
nd
Re

elements minus the product of off-diagonal elements.


ILLUSTRATION Evaluate:
Fi

5 4 sin 0 cos 0
(i) -2 3 (ii) - cos 0 sin 0

x —1 1 x 2 + xy + y2 x+y
(hi) (iv)
x3 A-2 + X + 1 x 2 -xy + y2 x~y
1 log[) a
(v)
logM 1
SOLUTION By definition, we obtain
... 5 4
(0 -2 3 = 5 x 3-4x -2=15 + 8=23
sin 0 cos 0
(ii) = sin2 0 - (- cos2 0) = sin2 0 + cos2 0=1.
- cos 0 sin 0
x —1 1
(hi) = (x-l)(x2 + x + l)-x3 =(x3 —1) — x3 = —1.
x3 x2 + X + 1

ReadYourFlow.COM
6.2 MATHEMATICS-XII

^ 2 +xy + \f
2 x+y
-{x^ + xy + y2) (x-y)-(x2 - xy + y2) (x + y)
(iv)
x - xy + y2
2 x~y
= (x3 -y3) -(x3 + y3) = - 2y3
1 = 1 -log,, ax logflfc=l -1 =0
(v) ••• !ogb « = ■r~—r
log* b 1 log^

6.1.3 DETERMINANT OF A SQUARE MATRIX OF ORDER 3


fl12 rt13
If A = a2i a22 «23 is a square matrix of order 3, then the expression
. ^31 a32 a33 _

au a22 fl33 + al2 a23 fl31 + au a32 a2l - an a23 a32 - a22 fl13 a31 - al2 a2l a33
is defined as the determinant of A

w
al\ a\2 n\3
i.e. Ml = a2\ a22 a23
a3l a32 a33

Flo
ee
= an a22 a33 + au a^ a3l + a13 a32 a2l - au a23 a32 - a22 a31 a13 - a33 al2 a2l ...(ii)

Fr
flll fl12 a13
or. Ml = a21 a22 a23
for
ur
a3\ a32 a33
M I = flll (a22 a33 - a23 a32) ~ a\2 (fl33 fl21 “ a23 ff3l) + al3 (a32 a2\ ~ a22 fl3l)
ks
Yo

a22 a23 a2l a23 a2l a22


=>
oo

Ml = flu -fll2 + fll3 [Using notation given in 6.1.2]


a32 a33 a3l a33 ^31 a32
eB

a22 a23 fl2i a23


=> Ml = (-D 1+ 1 All a32 a33
+ (-l)
1+ 2
fll2 + (-1)1+3«,3 l21 T
a3\ a33 M31 M32
r
ou
ad

Thus the determinant of a square matrix of order 3 is the sum of the product of elements ay in first row
Y

with (-1)1 + i times the determinant of a 2x2 sub-matrix obtained by leaving the first row and colwnn
passing through the element.
nd
Re

The above expansion of \A\ is known as the expansion along first row. For example, if
Fi

"3-2 41
/! = 1 2 1 is a square matrix of order 3, then
0 1 -1
3-2 4
Ml = 1 2 1
0 1 -1
1
=> Ml = (-i)
1+ 1 x3?
1 1 +(-D
1+ 2
(-2) q +(-l) 1+ 3 4 10 ?1
=> Ml = 3 (-2-1) + 2(-l -0) + 4(l -0)=-9-2 + 4 = -7
There are three rows and three columns in a square matrix of order 3. The expression (ii) for the
determinant of a square matrix of order 3 can be arranged in various forms to obtain the
expansion of | A | along any of its rows or columns. Infact, to expand | A | about a row or a
column we multiply each element ay in Xth row with (-1)' + i times the determinant of the
sub-matrix obtained by leaving the row and column passing through the element.

ReadYourFlow.COM
DETERMINANTS 6.3

For example.
au fl12 fli3
| A | = n2\ ci12 ci23
a3l a32 a33

=> Ml = (-D
2+ 1
a2l
a\2 rt13
+ (-D 2+ 2 a22
aU a\3
+ (-D 2+3 a23 an ai2
a32 a33 a3\ a33 a31 a32
is the expansion of | A | about second row.
The expansion of | A | about 2nd column is given as
2+ 2 3+2 an fl13
MI = (-i) 1+ 2 aU a2l
a31
a23
a33
+ (-D a22
a\l
a3\
a\3
a33
+ (-D a32
a2\ a23
2 3-2
ILLUSTRATION! Evaluate A = 1 2 3 by expanding it along the second row.
-2 1 -3

low
SOLUTION By using the definition, of expansion along second row, we obtain
2 3-2
A = 1 2 3

ee
-2 1 -3
rF
Fr
2+ 1 {1) l3 -3 +(-1)2+2(2)_2 -3 +(-1) 2 + 3 (3) 2 3
A = (-1) -2 1
3-2 2 -2 3 3
for
=> A = - + 2 -3
-2 1
1 -3 -2 -3
ou
ks

=> A = -(- 9 + 2) + 2 (- 6 - 4) - 3 (2 + 6) = 7 -20-24 = 37.


oo

2 3-2
Y

ILLUSTRATION 2 Evaluate the determinant D = 1 2 3 by expanding it along first column.


B

2 1 -3
re

SOLUTION By using the definition, of expansion along first column, we obtain


ou
ad

2 3-2
Y

D = 1 2 3
-2 1 -3
nd
Re

=> D = (-1)1+1(2) i J +M)2+,(1) j3 I3 +(-l)3+1(-2) 3 -Z


3
Fi

3 -2 -2 3
=>
H? J 1 -3 2
D = 2 (-6-3)-(-9+ 2)-2 (9+ 4) = -18 + 7 -26 = -37.
-2
3

NOTE 1 Only square matrices have their determinants. The matrices which are not square do not have
determinants.
NOTE 2 The determinant of a square matrix of order 3 can be expanded along any row or column.
NOTE 3 If a row or a column of a determinant consists of all zeros, then the value of the determinant is
zero.
6.1.4 DETERMINANT OF A SQUARE MATRIX OF ORDER 3 BY USING SARRUS DIAGRAM
The determinant of a square matrix of order 3 can be evaluated by the following procedure:
an fl12 a13 all al2 a13
Consider the determinant a2i a22 a23 fhe square matrix A = a2l a22 a23
a3l a32 a33 a3l a32 a33

ReadYourFlow.COM
1

6.6 MATHEMATICS-XII

6.3 MINORS AND COFACTORS


MINOR Let A = [cijj] be a square matrix of order n. The minor M,y of a q in A is the determinant of the
square sub-matrix of order (n - 1) obtained by leaving ith row and fh column of A.
4 -7
For example, if A = 2 , then
-3
M-n = Minor of au = 2, M12 = Minor of fl12 = _ 3,
M21 = Minor of fl21 = -7, M22 = Minor of fl22 = 4
1 2 3'
If A = -3 2-1 , then
2-4 3
Mu ~ Minor a\\
=> Mu ~ Determinant of the 2x2 square sub-matrix obtained by leaving first row and
first column of A

w
2 -1
=> Mn = 3 =2'
-4
Similarly, we obtain
M12 = Minor of rti2 =
-3 -1

Flo
2 = -7, Mi3 = Minor of 013 =
-3 2 =8

ee
2 -4

Fr
2 3 1 3
M2i = Minor of a2i - M22 = Minor of «22 =
-4 3 = 18, for2 3
= - 3 etc.
ur
COFACTOR Let A = [ajj] be a square matrix of order n. The cofactor C,y of aq in A is equal to (-1)' + 1
times the determinant of the sub-matrix oforder (n -1) obtained by leaving ith row andjth column ofA.
ks

It follows from this definition that


Yo
oo

Cq = Cofactor of Ojj in A = (-1)' + i Mq, where is minor of a^ in A.


B

Thus, we have
re

Mjj if i + j is even
- My if i + j is odd
ou
ad

4 -7 1
Y

For example, if A = 2 /then


-3
nd

1+ l Mn=Mii=2, Ci2=(-1) 1+2


Re

Cu=(-1) m12 - Mi2=-(-3) = 3,


2+ 1 M2i = — M2i = — (—7) = 7, and C22=(—1) 2+2
Fi

C21=(-1) M22 = 4
1 2 3
If A = -3 2-1 , then
2-4 3
-1
Cn = (-D 1+ 1 M ll = Mn = _ 4 3 =2'
1+ 2 Mi2 = — Mi2 = -3 -1
Ci2= (-D = -(-9 + 2) =7
2 3
-3
C13 = (-1)
1+ 3
M13 = M13 = 2 -4 l = 8/
2+ 3 1 2
C23 = (-1) M23 - -M23 - 2 _4 = 8 etc.

REMARK Some authors define the minors and cofactors for the elements of a determinant which is not
correct. Infact, minors and cofactors are defined for the elements of a square matrix.

ReadYourFlow.COM
DETERMINANTS 6.7

ILLUSTRATIVE EXAMPLES

LEVEL-1

fl 31 , find the determinant of the matrix A 9 2A.


EXAMPLE 1 If A =
2 1
fl 31
SOLUTION We have, A =
2 1
'1 31 fl 3'
A1-2A =
2 1 2 1
2 21 31
1+6 3+3 [2 61 [7 61 \2 61 7-2 6-6' [5 01
=> A1-2A = 4-4 7-2
2+2 6+1 4 2 4 7 4 2 0 5

5 0
| A2 - 2A| = = 25-0 = 25.

w
0 5
1 2
EXAMPLE 2 If A = , then show that\ 2A\ = 41 A |.

Flo
4 2
SOLUTION We have. [NCERT]

ee
1 2 [2 4'
=> 2A =

Fr
A =
4 2 8 4
1 2 2 4
I A | = 4 2 = 2 - 8 = - 6 and | 2A | = 8 4 = 8 - 32 = -24 = 4 x (- 6)
or
ur
f
Clearly, 12A|=4|A|.
ks
Yo

x - 2 -3
= 3, find the values of x.
oo

EXAMPLE 3 If 3x 2*
B

SOLUTION We have,
re

x-2 -3
= 3
3x 2x
ou
ad

(x - 2) x 2x - (- 3) x 3x = 3
Y

=>
=> 2x (x - 2) + 9x = 3
nd
Re

=> 2x2 - 4x + 9x = 3
Fi

=> 2x2 +5x - 3 = 0 =e> (2x -1) (x + 3) = 0 => 2x -1 = 0 or, x + 3 = 0 => x = ^ -3.
2'

EXAMPLE 4 Let 3 y = 3 2 Find possible values ofx and y ifx, y are natural numbers.
x 1 4 1‘
SOLUTION We have,
3 y 3 2
x 1 4 1
=> 3 - xy = 3 - 8 => xy = 8 => x = 1, y = 8; x = 2, y = 4; x = 4, y = 2;x = 8, y =1
log 3 512 log4 3
EXAMPLE 5 Evaluate the determinant A =
log3 8 l°g 4 9 '
SOLUTION We have.
log 3 512 log4 3
A =
log 3 8 log4 9

ReadYourFlow.COM
6.8 MATHEMATICS-XII

: log329 1o822 3
=> A
log3 23 log22 32

91°g32 ^ log2 3
A = lo8ap ^ logo m
2
3 log3 2 1o82 3
2
1
9 log 3 2 — log2 3
=> A = 3 log 3 2 2
log2 3
A - (9 log3 2) x (log2 3) log2 3^(3 log3 2)

=> A = 9 (log3 2 x log2 3) -1 (log2 3 x log3 2)

w
=> A = 9 -— [v log;, a x logfl = 1]
2
15
=> A = —
2

Flo
ee
1 3 -2

Fr
EXAMPLE 6 Find the minors and cofactors of elements of the matrix A =[ajj] = 4 -5 6 .
for |_3 5 2
ur
SOLUTION Let M(y and C,y denote respectively the minor and cofactor of element Ojj in A. Then,

^11 = 5 2=-^® —30 = -40=> C|| = = -40


ks
Yo
oo

4 6
M12 = = 8 -18 = -10 => C12 - -M12 = 10
B

3 2
re

4 -5
Mi3 = = 20 + 15 = 35 C13 M13 =35
3 5
ou
ad

3 -2
Y

M 21 = 6 + 10 = 16 C21 — — A42| = —16


5 2
nd
Re

1 -2
M22 = 3 =2 + 6 = 8 (~22 ~ ^22 = 3
2
Fi

3
M23 = 3 =5-9 =-4 => C23 “ _M23 ~ 4
5

M31 = 3 -2
= 18-10 = 8 => C3i - M31 = 8
-5 6
1 -2
M 32 = 6 + 8 =14 => C32 ““ M32 = -14
4 6
1 3
M33 - = -5-12 = -17 => C33 = M33 = -17
4 -5

LEVEL-2

1 sin 0 1
EXAMPLE 7 Evaluate the determinant A = - sin 0 1 sin 0 . Also, prove that 2 < A < 4
-1 - sin 0 1

ReadYourFlow.COM
DETERMINANTS 6.9

SOLUTION We have.
1 sin 0 1
A = - sin 0 1 sin 0
-1 - sin 0 1
1 sin 0 - sin 0 sin 0 - sin 0 1
=> A = 1x - sin 0 +1 x
- sin 0 1 -1 1 -1 - sin 0
[Expanding along first row]
=> A = 1 x (1 + sin2 0) - sin 0 ( - sin 0 + sin 0) + 1 x (sin2 0 + 1)
=> A = (1 + sin2 0) -0 + (sin2 0 + 1)= 2 + 2sin2 0 = 2(1 + sin2 0)
We know that
-1 < sin 0 <1 for all 0
=> 0 < sin2 0<1 for all 0
1 + 0<1 + sin2 0<1 + 1 for all 0

low
=> 1 <1 + sin2 0 < 2 for all 0
=> 2 <2(1 + sin2 0) <4 for all 0
=> 2 < A < 4. for all 0

ee
EXAMPLE 8 If [•] denotes the greatest integer less than or equal to the real number under consideration,
rF
Fr
and -l <x <0,0 <y <l,l<z<2l then find the value of the following determinant:
M+i [y] [z]
for
A = [x] [y] + i [2]
ou
[x] [y] [2] +1
ks

SOLUTION We have, -l<x<0, 0<y<l and 1 < z < 2


oo

=> [x] = -1, [y] = 0 and [z] = 1.


Y
B

[x] +1 [y] [2]


re

A = [x] [y] + i [2]


[x] [y] [2] + i
ou
ad

0 0 1
Y

=> A = -1 1 1
-102
nd
Re

1 1 -1 1 -1 1
=> A = 0x -Ox +1x [Expanding along first row]
Fi

0 2 -1 2 -1 0
=> A = 0 (2 - 0) - 0 ( - 2 + 1) + 1 x (0 + 1) = 1.
x sin 0 cos 0
EXAMPLE 9 Prove that the determinant - sin 0 — x 1 is independent of 0.
cos 0 1 x
SOLUTION We have. [NCERT]
x sin 0 cos 0
A = - sin 0 - x 1
cos 0 1 X
-x 1 -sin 0 1 - sin 0 - x
=> A = x - sin 0 + cos 0
1 x cos 0 x cos 0 1
=> A = x (- x2 -1) - sin 0 ( - x sin 0 - cos 0) + cos 0 (- sin 0 + x cos 0)
=> A = -x3 - x + x sin 9 0 + sin 0 cos 0 - sin
• 0 cos 0 + x cos 9 0

=> A = -x3 - x + x (sin2 0 + cos2 0) = -x3 -x + x = -x3, which is independent of 0.

ReadYourFlow.COM
6.10 MATHEMATICS-XII

EXERCISE 6.1
LEVEL-1
1. Write the minors and cofactors of each element of the first column of the following matrices
and hence evaluate the determinant in each case:
5 20 -1 4
(i) /l = 0 -1 (ii) ^ =
2 3 [
1-3 2 1 a be
(iii) /\ = 4-12 (iv) A = 1 b ca
3 5 2 1 c ab
'0 2 6' a h S
(v) A = 1 5 0 (vi) A = h b f
3 7 1 8 f c

low
2-1 0 1 '
-3 0 1-2
(vii) A =
11-11
2-150

ee
2. Evaluate the following determinants:
rF
Fr
-7 cos 0 - sin 0
x 5x +1 (ii) sin 0 cos 0
cos 15° sin 15°
for
a + ib c + id
(iii) (iv) [CBSE 2008]
sin 75° cos 75° -c + id a-ib
u
ks

2 3 7 2
Yo
oo

3. Evaluate: 13 17 5
15 20 12
B

sin 10° -cos 10°


re

4. Show that
sin 80° cos 80° = 1
ou
ad

2 3-5
Y

5. Evaluate 7 1 - 2 by two methods.


-3 4 1
nd
Re

0 sin a - cos a
Fi

6. Evaluate: A = - sin a 0 sin p [NCERT]


cos a - sin p 0

cos a cos P cos a sin p - sin a


7. Evaluate: A = - sin p cos p 0 [NCERT]
sin a cos p sin a sin p cos a
2 5 4 -3
8. If 71 = and B = , verify that | AB | = | A 11 B\.
2 1 2 5
1 0 1
9. If A = 0 1 2 , then show that) 3 A | = 27 | A |.
0 0 4
10. Find the values of x, if
2 4 2x 4 2 3 x 3
(i) [NCERT] (ii) 4 5
5 1 6 x 2x 5

ReadYourFlow.COM
DETERMINANTS 6.11

3 x 3 2 3x 7
(iii) [NCERT] (iv) = 10
x 1 4 1 2 4
x+1 x—1 4 -1 2x 5 _ 6 5
(V)
1 3 [CBSE 2013] (vi) 8 x " 8 3 [NCERT EXEMPLAR]
x-3 x+2
x2 X 1
11. Find the integral value of x, if 0 2 1 = 28.
3 1 4

12. For what value of x the matrix A is singular?


x —1 1 1
T+x 7"
(i) A = 3 -x [CBSE 2012] (ii) A = 1 x —1 1
8 1 1 x-1
ANSWERS

low
1. Minors Cofactors
(i) M11=-1,M21=20 Cn = — 1, C21 = - 20
(ii) M11 = 3/M21=4 Cn = 3,C2i=-4

ee
rF C11=-12/C21=16/C31=-4

Fr
(iii) Mn = -12, M21 = -16, M31 = - 4,
(iv) Mn=a(b2-c2)/M2i=b(a2-c2)f Cn =a(b2-c2),C2i =~b(a2-c2),
or
M31 =c(fl2 “k2) C31 =c (a2 -b2)
sf
u
(v) M11=5/M21=-40,M31=-30 Cii=5,C21=40,C3a=—30
k
Yo
oo

(vi) Mn =bc - f 2, M21=hc -fg, Cn=bc-f2,c2i=fg-ch,c3i=hf-bg


eB

M31=hf-bg
(vii) =-9, M21 =9, M31 =-9, M41 =0 = - 9, C21 = - 9, C31 = - 9, C41 = 0
r
ou
ad

2. (i) 5x2 + 8x (ii) 1 (iii) 0 (iv) a2+b2 + c2 + d2 3. 0 5. -140


Y

6.0 7. 1 10. (i) ± yj~3 (ii) 2 (iii) ± 2v 2 (iv) 2 (v) 2 (vi) ± 3


nd
Re

13
11. 2 12. (i) ^ (ii) -1,2
15
Fi

HINTS TO NCERT & SELECTED PROBLEMS


0 sin a - cos a
6. We have, A = - sin a 0 sin p
cos a - sin p 0
On expanding along first row, we get
0 sin p - sin a sin p - sin a 0
A = 0 - sm a -cos a
- sin P 0 cos a 0 cos a - sin p

A = - sin a (0 - sin p cos a) - cos a (sin a sin P)


=> A = sin a cos a sin P - sin a cos a sin P = 0
cos a cos P cos a sin p - sin a
7. We have, A = - sin P cos P 0
sin a cos p sin a sin p cos a

ReadYourFlow.COM
6.12 MATHEMATICS-XII

On expanding along first row, we get


cos P 0 • r - sin P 0
A = cos a cos P - cos a sin p
sin a sin P cos a sin a cos p cos a
-sin P cos P
- sin a
sin a cos P sin a sin p
=> A = cos a cos p (cos a cos p - 0) - cos a sin p (- cos a sin p - 0)
9 9
- sin a (- sin a sin p - sin a cos P)
=> A = cos9 a cos 9 p + cos 9 a sin 9
p + sin 9 a
9 9 9 9 9 9
=> A = cos a (cos P + sin P) + sin a = cos a + sin a = 1
10. (i) We have.
2 4 2x 4
=> 2-20 = 2x2 -24 => 2x2 = 6 => x = ± Vs
5 1 6 x

low
(iii) We have,
3 x 3 2
=> 3 - x2 = 3 - 8 => X2 = 8 => x = ± 2V2
x 1 4 1

ee
rF
Fr
6.4 PROPERTIES OF DETERMINANTS
In section 6.1, we have defined the determinant of a square matrix of order 4 or less. Infact, these
definitions are consequences of the general definition of the determinant of a square matrix of
for
any order which needs so many advanced concepts. These concepts are beyond the scope of this
u
book. Using the said definition and some other advanced concepts we can prove the following
ks

properties. But, the concepts used in the definition itself are very advanced. Therefore we
Yo

mention these properties and verify them for a determinant of a square matrix of order 3.
oo

PROPERTY 1 Let A = [djj ] be a square matrix of order n, then the sum of the product of elements of any row
B

(column) with their cofactors is always equal to \A \ or, det (A).


re

i.e. I1 % Cij = i A I and' .f 1 “ij Cij = M I-


ou
ad

;=i
Y

^11 a\2 a\3


VERIFICATION Let A = fl2i fl22 rt23 be a square matrix of order 3. Then, by definition, we
nd
Re

. fl31 fl32 fl33 .


Fi

have
a\l a\2 a\3
\A\= a2i a22 «23
fl31 a32 a33
1+2
=> Ml = (-D 1+ 1 au a22 a23
a32 a33
+ (-D a12
a2\ a23
a3\ a33
+ (-l)
1+ 3
a13
a2\ a22
a3\ a32
[Expanding along first row]
=> Ml ~ aU CU + a12 C12 + a\3 C13 [By using the definition of cofactors]
Similarly, we have
Ml = fl21 C21 + a22 C22 + a23 Qz3' Ml = a3\ C31 + a32 C32 + fl33 C33'
Ml = all C11 + a2l C2l + a31 C31 etc-
PROPERTY 2 Let A= [fl,y] be a square matrix of order n, then the sum of the product of elements of any
row (column) with the cofactors of the corresponding elements of some other row (column) is zero.

ReadYourFlow.COM
DETERMINANTS 6.13

n
i.e. Z Ojj Ckj — 0 and, . ^ ^ Cik = °-

flll a12 aU
VERIFICATION Let A = a22 a23 *:)e a square matrix of order 3. Then, the sum of the
fl31 a 22 fl33
product of elements of first row with the cofactors of elements in second row is given by
flll ^21 + a12 ^22 + fl13 C-23
= an (-1) 2+1 al2 a13 + a12 (-1) 2+2 a\\ fl13 2+3 flll fl12
+ a13 (_1)
a32 a33 a31 a33 a3l a32
= - au (a12 a22 - «13 «32) + fl12 (an a33 ~ a13 a3l) ~ a13 (fln a32 ~ a\2 fl3l)
= 0

w
Similarly, we have
G11 C31 + g12 C32 + fl13 C33 =: g21 ^11 + a22 C12 + a23 C13 = 0 etc-
PROPERTY 3

Flo
Let A = [ajj] be a square matrix of order n, then \ A\=\ A7 \.

ee
By the abuse of language this property is also stated as follows:

Fr
The value of a determinant remains unchanged if its rows and columns are interchanged.
fl! *>1 c1 or fll ci2 a3
ur
VERIFICATION Let A = a2 b2 c2 beasquarematrixof orderS.Then, A7 = bl b2 b3 ■
sf
a3 b3 c3 C1 c2 g3
k
Yo

fll h
oo

Now, | A | = fl2 b2 c2
B

a3 b3 c3
re

1+ 1 * ^2 c2 1+ 2 a2 c2 1+3 «2 b2
Ml=(-D 1 b3 c3 + (-l)
+ (-l) 1 g3 h
ou

a3 c3
ad
Y

[Expanding along first row]


I A | = fli (b2 c3-b3 c2) -b1 (a2 c3-a3 c2) + Cj (a2 b3-a3 b2)
nd
Re

a1 a2 a3
Fi

and, \AT\=b^ b2 b3
ci c2 c3

G3 +(_1} 1+ 3 a2 a3
=> i^ti=(-d 1+ 1 ^ b3 +(_l)l+2b g2
1 c2 c3 V y 1 c2 c3 C1 b2 b3

[Expanding along first column]


| Ar | = fl! {b2 c3 - b3 c2) - b} ■ (a2 c3 - a3 c2) + c1 (a2 b3 - a3 b2) ...(h)
From (i) and (ii), we obtain | A | = | A71.

PROPERTY 4 Let A = [ Ojj ]bea square matrix of order n (> 2) and let B be a matrix obtained from A by
interchanging any two rows (columns) of A, then | B | = - | A |.
Conventionally this property is also stated as:
Ifany two rows (columns) ofa determinant are interchanged, then the value of the determinant changes by
minus sign only.

ReadYourFlow.COM
6.14 MATHEMATICS-XII

Cj
VERIFICATION Let A = ^2 c2 a sc[uare matrix of order 3 and let B be the matrix
a3 b3 c3
a3 b3 c3
obtained from A by interchanging first and third row i.e. B = (1^ b-, c2 ■
Ci

al bl ci
u2 b')
Then, \A\= a2 b2 c2 =(-l) 1+ 1 °ibb2 c2 +(-i),+2fc, a2 c2 +(-i) 1+ 3 Cl
a3 b3 c3 ^3 c3 a3 c3 a3 ^3

[Expanding along first row]


=> \ A\=a1(b2 c3 -b3 c2) -b1 (a2 c3 ~a3 c^ + Cj (a2 b3 -n3 b2)
a3 ^3 c3

w
and. \B\= ci2 b2 c2 =(-l) 3+ 1 b3 c3 +(-i)3+2i’i “3 cr3 +(-D 3+ 3 a3 b3
b2 c2 a2 c2 ci a2 b2
^1 bl Cj

Flo [Expanding along first row]

ee
=> I S| - -[«! (b2 c3 - b3 c2) -/?i (a2 c3 -a3 c2) + C] (a2 b3 -a3 b2)] ...(ii)

Fr
From (i) and (ii), we obtain | B | = -1 A |.
PROPERTY 5 If any tiuo rows (columns) of a square matrix /l = [ Oj^of order n (> 2) are identical, then
for
ur
its determinant is zero i.e. | A | = 0.
Conventionally this property is stated as:
ks
Yo

Ifariy two rows or columns of a determinant are identical, then its value is zero.
oo

a1 b-i ca
B

VERIFICATION LetA= a2 b2 c2 be a matrix having first and third rows identical and let B
re

a1 b1 Ci
ou
ad

be the matrix obtained from A by interchanging the first and third rows. Then, by property 4, we
Y

obtain
I B| = -Ml
nd
Re

al b1 Cl
Fi

But, B = a2 b2 c2 =A. Therefore, | B | = M | ...(ii)


al h C1
From (i) and (ii), we obtain
M| = -M| => 2M| = 0 => M| = 0.
PROPERTY 6 Let A = [ a1j ] be a square matrix of order n, and let B be the matrix obtained from 21 by
multiplying each element of a row (column) of A by a scalar k, then | B | =k | A |.
Conventionally this property is also stated as:
If each element of a row (column) of a determinant is multiplied by a constant k, then the value of the new
determinant is k times the value of the original determinant.
«1 bl ci
VERIFICATION Let A- a2 b2 c2 be a square matrix order 3, and let B be a matrix obtained
a3 b3 c3
from A by multiplying each element of second row by the same constant k, then

ReadYourFlow.COM
DETERMINANTS 6.15

«i b1 c-j
B = ka2 kb2 kc2 .
a3 b3 c3 ^

Now,
«1 bl
\A\ = a2 b2 c2 = flj {b2 c3-b3 c2) -b1 (a2 c3 -a3 c2) + q (a2 b3-a3 b2)
a3 b3 c3

«i h ci
and. \B\= ka2 kb2 kc2
a3 b3 c3

=> \B\=al (kb2 c3 -kb3 c2) -bj (ka2 c3 - ka3 c2) + c1 (ka2 b3 - kb2 a3)
[On expanding along first row]

w
=> \B\=k[al (b2 c3-b3 c2) (a2 c3 - a3 c2) + ca (a2 b3 - a3 b2)] ...(ii)
From (i) and (ii), we obtain\B\ = k\A\.
REMARK l

Flo
Itfollows from the above property that we can take out any common factor from any one row
or any one column of a given determinant.

ee
REMARK 2 Let A = [Ojj] be a square matrix of order n, then \kA\=kn\ A\, because k is common from

Fr
each row ofkA.
PROPERTY 7 Let A be a square matrix such that each element of a row (column) of A is expressed as the
for
ur
sum of two or more terms. Then, the determinant of A can be expressed as the sum of the determinants of
two or more matrices of the same order.
ks
Yo

Conventionally this property is also stated as:


oo

If each element of a row (column) of a determinant is expressed as a sum of two or more terms, then the
B

determinant can be expressed as the sum of two or more determinants.


re

flj + tt! b1 +0! C1 + Y!


VERIFICATION Let A = a2 b2 c2 be a square matrix such that each element in
ou
ad

a3 b3 c3
Y

first row of A is the sum of two elements. Then,


nd

«! + «! &1 + P1 q + Yi
Re

|A| = a2 b2 c2
Fi

a3 b3 c3
b2 c2 a2 b2 Expanding along
=> \A\=(al + a1) -(^l+Pl) a2 c2
+ (ci +Yi)
b3 c3 a3 c3 a3 b3 first row

-o a2 c2 + Cj a2 b2 b2 C2 a2 c2 a2 b2
|A| = + ai -Pi + Yi a3
1 a3 c3 a3 b3 b3 C3 a3 c3 b3

bl al Pi Yi
=> \A\=a2b2c2+a2b2c2
a3 b3 c3 a3 b3 c3

«1 bi cx <*1 Pi Yi
=>|A|=|B|+|C|, where B = a2 b2 c2 and C - a2 b2 c2 .
a3 b3 c3 a3 b3 c3 .

PROPERTY 8 Let A be asquare matrix and Bbea matrix obtainedfrom A by adding to a row (column) of
A a scalar multiple of another row (column) of A, then | B | = | A |.

ReadYourFlow.COM
6.16 MATHEMATICS-XII

This property is conventionally stated as:


If each element ofa row (column) ofa determinant is multiplied by the same constant ami then added to the
corresponding elements of some other row (column), then the value of the determinant remains same.
al Cl a^ + kb^ bl q
VERIFICATION Let ,4 = a2 b2 c2 be a matrix and let B = a2 + kb2 b2 c2 be the matrix
a3 b3 c3 a3 + kb3 b3 c3
obtained from A by multiplying the elements of second column by k and then adding them to
the corresponding elements of first column. Then,
+ kbi b\
\B\= a2 + kb2 b2 c2
a3 + kb3 b3 c3
al b-L c-i kbi bi Ci
| B | = a2 b2 c2 + kb2 b2 c2 [Using property 7]

low
a3 b3 c3 kb3 b3 c3
a\ bi q bi bi Ci
=> | B | = fl2 b2 c2 + k b2 b2 c2 [Using property 6]
a3 b3 c3 b3 b3 c3

ee
=> \B\=\A\ + kO
rF [Using property 5]

Fr
=> \B\=\A\
PROPERTY 9
for
Let Abe a square matrix order n (> 2) such that each element in a row (column) of A is
zero, then | A | = 0.
u
ks

Conventionally this property is also stated as:


Yo
oo

If each element of a row (column) of a determinant is zero, then its value is zero.
B

0 0 o ‘
VERIFICATION Let A = a2 b2 c2 be a square matrix. Then,
re

a3 b3 c3
ou
ad

0 0 0
Q b2 c2 _Q a2 c2 Expanding along
+ 0 a2 b2 = n
Y

\A\=a2 b2 c2 = «3 b3 a
b3 c3 a3 c3 first row
a3 b3 c3
nd
Re

PROPERTY 10 If A = [ajj] is a diagonal matrix of order n (> 2), then


Fi

\A\=“Uxa22xa33 x---x “an


PROPERTY 11 If A and B are square matrices of the same order, then | AB | =| A | | B |.
PROPERTY 12 Let A = [ajj ] be a square matrix of order n and let Cjj = cofactor of ay in A for
i, j = 1,2,..., n.lfC = [cy] is the matrix of cofactors of elements in A, then | C | = | A |^

aU a12 a\3 C11 c12 c13


VERIFICATION Let A = rt2l a22 a23 anc^ ^ = c2l c22 c23 ' where Cy = cofactor of ay in
a3i a32 a33 C31 c32 c33
A. Then,
C11 c12 C13 flll a21 a3l
CAT = C21 c22 c23 ai2 a22 a32
C31 c32 c33 a\3 a23 a33

ReadYourFlow.COM
DETERMINANTS 6.17

flll C11 + fl12 c12 + fl13 c13 fl21 cn+fl22 c12+fl23 c13 fl31 C11 + fl32 <:12 + rt13 c33
=> CAt = flll C21 + fl12 c22 + fl13 c23 fl21 C21 + fl22 c22 + fl23 c23 a31 C21 + fl32 c22 + fl33 c23
_a\\ C31 + fl12 c32 + fl13 c33 fl21 C31 + fl22 c32 + fl23 c33 fl31 C31 + ^32 c32 + c33_
| A| 0 0 '
=> CA T 0 |A| 0 [Using Properties 1 and 2]
0 0 |A|
|A| 0 0
|CA T 0 |A| 0 =| A|3
0 0 |A|
=> !c||Ar|=|A|3
^ !C||^|=|/i|3 [■■• |AT|=|A|]

=> |C|=H2

w
6.5 EVALUATION OF DETERMINANTS

Flo
If A is a square matrix of order 2, then its determinant can be easily found. But, to evaluate
determinants of square matrices of higher orders, we should always try to introduce zeros at

ee
maximum number of places in a particular row (column) by using the properties given in

Fr
section 6.4 and then we should expand the determinant along that row (column).
We shall be using the following notations to evaluate a determinant: for
ur
(i) Rj to denote row.
(ii) Rj <-» Rj to denote the interchange of /th and ;th rows.
ks

(iii) Rj -> Rj +X Rj to denote the addition of X times the elements of fh row to the
Yo
oo

corresponding elements of ;th row.


(iv) Rj (X) to denote the multiplication of all elements of z'th row by X.
B
re

Similar notations are used to denote column operations if R is replaced by C.


ou

ILLUSTRATIVE EXAMPLES
ad
Y

LEVEL-1
nd

Type I DETERMINANTS IN WHICH TWO ROWS (COLUMNS) BECOME IDENTICAL BY APPLYING THE
Re

PROPERTIES OF DETERMINANTS
Fi

41 1 5
EXAMPLE l Without expanding evaluate the determinant 79 7 9 .
29 5 3
41 1 5
SOLUTION Let A = 79 7 9 . Applying^-> Cj + (-8) C3, we get
29 5 3
1 1 5
A = 7 7 9
5 5 3
=> A =0 [v C} and C2 are identical]

1 w w2
2
EXAMPLE 2 If w is a complex cube root of unity. Show that w w 1 = 0.
w2 1 w

ReadYourFlow.COM
6.18 MATHEMATICS-XII

1 w w2
2
SOLUTION Let A = w w 1 . Applying Cj Cj + C2 + C3 , we get
w2 1 w

1 + W + w2 IV w2
A = W + IV 2 +1 IV 2 1
w2 +1 +w ^ IV

0 IV w2
=> A = 0 w2 1 [v 1 + w + w2 = 0]
0 1 Ztf

=> A=0 [•.• C1 consists of all zeros]

w
1 a b+c
EXAMPLE 3 Show that 1 b c + a = 0.
1 c a+b

1 a b+c

Flo
ee
SOLUTION Let A = 1 b c + a. ApplyingC2-» C2 + C3/we get

Fr
1 c a +b
1 a+b+c b+c
for
ur
A = 1 b+c+a c+a
1 c+a+b a+b
ks
Yo

1 1 fc + c
oo

=> A = (a + fc + c) 1 1 c + fl [Taking out a + b + c common from C2]


B

1 1 a +b
re

A = (a+ b + c)x0 = 0 [v Cj and C2 are identical]


ou
ad

b-c c-a a-b


Y

EXAMPLE 4 Show that c-a a-b b-c


a-b b-c c-a [NCERT, CBSE 2009]
nd
Re

b-c c-a a-b


Fi

SOLUTION Let A = c - a a-b b - c . Applying Cj -» C-j. + C2 + C3, we get


a-b b-c c-a
0 c-a a-b
A = 0 a-b b-c
0 b-c c-a
=> A =0 [v C1 consists of all zeros]
1 be a(b + c)
EXAMPLES Show that 1 ca b (c + a) = 0.
1 ab c(a + b) [NCERT]

1 be a (b + c)
SOLUTION Let A = 1 ca b (c + a) . Applying C3 -» C2 + C3, we get
1 ab c(a + b)

ReadYourFlow.COM
DETERMINANTS 6.19

1 be ab + be + ca
A = 1 ca ab + be + ca
1 ab ab + be + ca
1 be 1
=> A = (ab +bc + ca) l ca 1 [Taking out ab + bc + ca common from C3]
1 ab 1

=> A=(ab +bc + ca)x 0 = 0. [v Ci and C3 are identical]


x + y y +z z + x
EXAMPLE 6 Without expanding prove that: z x y = 0.
1 1 1 [NCERT]
x+y y+z z+ x
SOLUTION Let A = 2 X y . Applying -> R1 + R2' we get

low
1 1 1
x + y +z x + y +z x + y +z
A= 2 y
1 1 1

ee
rF
Fr
1 1 1
=> A = (x + y 4- 2) 2 x y [Taking out (x + y + 2) common from RJ
1 1 1
for
=> A=(x + y + 2)x0 = 0 [•.• Rj and R3 are identical]
u
2
ks

cosec 0 cot2 0 1
Yo
oo

EXAMPLE 7 Without expanding show that: A = cot2 0 cosec2 0 -1 = 0.


42 40 2
B
re

[NCERT EXEMPLAR]
SOLUTION Applying C-, -» C-! -C2, we obtain
ou
ad

cosec20 - cot2 0 cot2 0 ^


Y

A = cot 0 - cosec 0 cosec 0 -1


nd
Re

42-40 40 2
Fi

1 cot2 0 1
=> A = -1 cosec2 0 -1 = 0 [v Cj and C3 are identical]
2 40 2
2 3 4
EXAMPLES Find the value of the determinant A = 5 6 8
6x 9x 12x [CBSE 2009]
SOLUTION Taking 3x common from R 3 , we get
2 3 4
A =3x 5 6 8
2 3 4
=> A =3x x 0 =0 [v and R3 are identical]

ReadYourFlow.COM
6.22 MATHEMATICS-XII

0 2-1
=> A =2(39) 0 2 -1 [Taking 2 common from R-j and 39 common from R9j
4 17 11

=> A =78 x 0 [*.• R| and R2 are identical]


Type II EVALUATING DETERMINANTS BY USING THE PROPERTIES OF DETERMINANTS AND
PROVING IDENTITIES
1 1 1 1 1 1
9 9 9
EXAMPLE 14 IfA1=x y z and A2 = Vz z* *y , without expanding prove that A1 = A2.
x y z x y z
SOLUTION Applying Cj -> Cx (x), C2 C2 (y) and C3 -> C3 (z), we get

A 1 * y z
a2 - — xyz xyz xyz

w
xyz
x2 y2 z2

A2 “
xyz
xyz
1 1 1
Flo [Taking xyz common from R2]

ee
xyz 2 2
x y z2

Fr
1 1 1
for
=> A2 = - x y z
ur
[Applying R2 RJ
x2 2 z2
ks

1 1
1
Yo
oo

=> a2 = y2 z2 [Applying R2 R3]


eB

xyz
=> A2 -
r

Ax x2 1
ou
ad

ABC
EXAMPLE 15 Let A= By y2 1 and Aj =
Y

x y z , then show that Aj = A.


Cz z2 1 yz zx xy
nd
Re

SOLUTION We have,
Fi

ABC
Ai = x y z
yz zx xy
Ax By Cz
=> A] =~ x 2 yz2 z2 [Applying Cj -> C! x , C2 ^ C2 y, C3 C3 z]
xyz xyz xyz xyz
Ax By Cz
=> A1=^ * 2 y 2 z2 [Taking xyz common from R3]
*yz i i i
Ax By Cz
=> A1 = x2 y2 z2
1 1 1

ReadYourFlow.COM
DETERMINANTS 6.23

Ax x2 1
Ai= By y2 1 [Interchanging rows and columns]
Cz z2 1

=> A-j =A.


a b c q -b y
EXAMPLE 16 If A-j = x y z and A2 = - p a -x , without expanding or evaluating
p q r r -c z
A1 and A2, show that A^ + A2 = 0.
SOLUTION Taking -1 common from second row, we obtain
q -b y
A2=- p -a x
r -c z

w
q b y
A2- p a x
r c z

Flo [Taking (-1) common fromC2]

ee
q p r

Fr
A2= b a c [Interchanging row and columns]
y x z
for
ur
p q r
=> A2=- a b c [Applying C2 -^C!]
ks
Yo

x y z
oo

a b c
B

=> A2 - p q r [Applying R-y++R2]


re

x y z
ou
ad

a b c
x y z [Applying B2 <-> B3]
Y

A2 -
p q r
nd
Re

=> A2 - - Al
Fi

=> A| + A2 = 0
EXAMPLE 17 If A is a skew-symmetric matrix of odd order n, then | A | =0.
SOLUTION Since A is a skew-symmetric matrix. Therefore,
At = -A
=> |At| = I -A|
=> \AT\ = (-1)" | A | [■■ \kA\=k"\A\]
I A | = (-1)” | A | [••' |AT| = \A\]
I A | = -| A | [•.• n is odd]
=> 2| A | = 0
I A| =0
Hence, the determinant of a skew-symmetric matrix of odd order is zero.

ReadYourFlow.COM
6.24 MATHEMATICS-XII

0 a -b
EXAMPLE 18 Prove that: -a 0 - c = 0.
be 0
0 a b
SOLUTION Let A = -a 0 - c . Then,
b c 0
0 -a b
A = (-1)3 a 0 c [Taking (-1) common from each row]
-b -c 0
0 a -b
=> A = - -a 0 -c [Interchanging rows and columns]
b c 0

w
=> A = -A=>2A = 0=>A = 0.
0 a -b
ALITER Clearly, A = -a
b c 0
Flo
0 -c is a skew-symmetric matrix of odd order. Therefore,

ee
|A|=0.

Fr
0 b-a c-a
EXAMPLE 19 Without expanding or evaluating shoiv that a-b 0 c-b = 0.
for
ur
a-c b-c 0
ks

[NCERT EXEMPLAR]
Yo

0 b-a c-a
oo

SOLUTION Let A = a-b 0 c-b . Then,


B

a-c b-c 0
re

0 -{a-b) -{a-c)
ou
ad

A = {a-b) 0 ~{b-c)
Y

{a-c) {b-c) 0
nd
Re

0 ~{a-b) -{a-c)
Fi

A = {a-b) 0 ~{b-c)
{a-c) (b-c) 0
0 {a-b) {a-c)
A = (-l)3 -{a-b) 0 {b-c) [Taking -1 common from each row]
-{a-c) -{b-c) 0
0 -{a-b) -{a-c)
A = — (a-b) 0 -{b-c) [Interchanging rows and columns]
{a-c) {b-c) 0
=> A = -A => 2A = 0=> A = 0
AUTER Clearly, given determinant is the determinant of a skew-symmetric matrix of odd
order. So, its value is zero.

ReadYourFlow.COM
DETERMINANTS 6.25

a+ bx c + dx p + qx a c p
EXAMPLE 20 Without expanding, prove that ax + b cx + d px + q = (1 - x ) b d q .
U V IV U V IV

[NCERT]
SOLUTION Applying Rl-+R1-xR2 to A, we get
afl-AT2) c (1 - x2) pV-x2)
A = ax + b cx + d px + q
u v w

a c p
=> A = (1 - x2) ax + b cx + d px + q
u V iv

a c p

w
=> A = (1 -x2) b d q [Applying R2 -+R2- xRj]
u v iv
_ ab ac
Flo
ee
EXAMPLE 21 Prove that: ba -b1 be =4a2b2c2.

Fr
ac be- c2 for
ur
- a2 ac
SOLUTION Let A = ba -b2 &c . Then, [NCERT, CBSE 20111
ks

ac be _ c2
Yo
oo

-a b c
eB

A = abc a -b c [Taking a, b and c common from Rlf R2 and R3 respectively]


a b -c
r
ou
ad

-1 1 1
Taking a, b and c common from
Y

=> A = a2b2 c2 1 -1 1
ClfC2 andC3 respectively
1 1 -1
nd
Re

-10 0
Fi

=> A = a2b2 c2 1 0 2 [Applying C2 ->C2 +€2, C3 ->C3 +^1


1 2 0
0 2
=> A = a2 b2 c2 x (-1) x [Expanding along R^
2 0

=> A = a2 b2 c2 (-1) (0 - 4) = 4a2b2c


11 1 I
EXAMPLE 22 Prove that: 1 1 + x 1 = *y-
1 1 i+y

1 1 1
SOLUTION Let A = 1 1 + x 1 [NCERT]
1 1 \+y
Applying C2 -»C2 -Cj and C3 ->C3 -C-! , we get

ReadYourFlow.COM
6.26 MATHEMATICS-XII

1 0 0
A = 1 x 0
1 0 y
* 0 1 0 1 x
=> A = 1x -Ox +0X [On expanding along R^]
o y i y i o
A = xy
laa1
EXAMPLE 23 Evaluate: 1 b b2
[NCERT]
1 c c2

SOLUTION Let Abe the given determinant. Applying R2 #2 ~ and R3 -> R3 - Rp we get
1 a «2

w
A = 0 b-a b2 -a2
0 c —a c2-a2

1 a fl2

Flo
ee
=> A = (b - a) (c - a) 0 1 b + a [Taking out (b - a) common from R2&c(c - a) from R3]
0 1 c+a

Fr
1 a a2
for
ur
=> A = (b - a) (c - a) 0 1 b + a [Applying R3 -*R3 -R2]
0 0 c-b
ks
Yo

a2
oo

1 a
=> A = (b - a) (c - a) (c -b) 0 1 b + a [Taking out (c - b) common from R3]
B

0 0 1
re

1 b+a
ou

A = (b - a) (c - a) (c -b) x 1 x ^ [Expanding along Cj]


ad

1
Y

=> A = (b - a) (c - a) (c -b) x 1 = (a -b) (b - c) (c - a)


d

REMARK The reader is advised to remember the value of this determinant as a standard residt.
Re
n

x y z
Fi

EXAMPLE 24 Show that: x2 y2 z2 = xyz (x - y) (y -z) (z - x).


x3 y3 z3
[CBSE 2000,2010C, 2011]
xyz
2 2 2
SOLUTION Let A = x y z . Taking x, y and z common from , C2 and C3 respectively,
x3 y3 z3
we get
1 1 1
A = xyz xyz
2 2 z2
* y

ReadYourFlow.COM
DETERMINANTS 6.27

1 0 0
=> A= xyz X y-x z-x [Applying C2 -> C2 -Cj andC3 -> C3 -Cj]
2 2 2 2 2
x^ yz-xA Z -X

1 0 0
'Taking (y - x) and (z - x) common from
=> A = xyz (y - x) (z - x) x 1 1
C2 from C3 respectively.
x2 y+x z+x

A - xyz (y - x) (z - x) x 1 x ^ 1
=> [Expanding along R^]
y+x z+x
=> A = xyz (y - x) (z - x) (z + x - y - x)
=> A = xyz (x - y) (y - z) (z - x)
a Y

low
EXAMPLE 25 Prove that: a2 P2 y2 = (a - P) (P - y) (y ~ a) (a + P + Y)-
P + y Y + a ot + p
[NCFRT, CBSE 2007C, 2008, 2010 C]
a P Y

ee
SOLUTION Let A = a2 P2 y2
rF
. Applying R3 -> + R3, we get

Fr
P+Y Y+a a+p
a P Y
f or
Y2
ou
A= a2 P2
ks

a+p+Y a+P+Y a+P+Y


oo

a p y
Y
B

=> A= (a + p + y) a2 P2 Y2 [Taking out (a + p + Y) common from R3]


re

1 1 1
ou

P-a
a Y -a
ad

A = (a + P + y) a2 p2 - a Y 2 - a 2
2
Y

=> [Applying C| ->C2 -C^ and C3 ->C3 -Cj]


1 0 0
nd
Re

a 1 1
Fi

Taking (P - a) common from


A =(a + P + y) (P - ct) (y - a) a2 P+a Y + a
C2 and (y - a) from C 3
1 0 0

A = (a + p + y) (P - a) (y - a) x 1 x 1 1
=> [Expanding along R3]
P+a y +a
--=> A = (a + p + y) (P - a) (y ~ a) (y + a - p - a) = (a P)(P-Y)(Y-a)(a + p + Y).
1 1 1
EXAMPLE 26 In a A ABC, if 1 + sin A 1 + sin B 1 + sin C = 0, then prove that
9 9 9
sin A + sin A sin B +sin B sinC + sin C

A ABC is an isosceles triangle. [NCERT EXEMPLAR]


1 1 1
SOLUTION Let A = 1 + sin A 1 + sin B 1 + sinC . Then,
2 2 2
sin A + sin A sin B +sin B sinC + sin C

ReadYourFlow.COM
6.28 MATHEMATICS-XII

1 1 1
A= sin A sin B sinC [Applying R2
'7 9 9
sinA + sin A sinB + sin B sinC+sin C
1 1 1
=> A = sin A sin B sin C [Applying R3 -+R3-R2]
sin2A sin2B sin2C
1 0 0
A = sin A sin B-sin A sinC-sinA [Applying C2 -»C2 -C1/C3 -^■C3-C1]
sin2A sin2B-sin2A sin2C-sin2A
1 0 0
=> A = (sin B - sin A) (sin C - sin A) sin A 1 1

w
sin A sin B +sin A sinC + sin A
[Taking sin B-sin A common fromC2 and sinC - sin A fromC3]
=>
=>
Flo
A = (sin B-sin A) (sinC - sin A) {(sinC + sin A) -(sin B + sin A)} [Expanding along Rj]
A = (sin B - sin A) (sin C - sin A) (sin C - sin B)

ee
Now, A=0

Fr
=> (sin B - sin A) (sin C - sin A) (sin C - sin B) = 0
either sin B - sin A = 0 or, sin C - sin A = 0 or, sin C - sin B = 0
for
either sin A -sin B = 0 or, sin C = sin A = 0 or, sin C - sin B = 0
ur
=>
=> A = B or C - A or B=C
=> BC =CA or, AB = BC or CA = AB
ks
Yo

=> AABC is isosceles


oo

1 1 1
B

EXAMPLE 27 In a A ABC, if 1 + cos A 1 + cos B 1 + cosC = 0, show that AABC is


re

cos2 A + cos A cos2 B + cos B cos2 C + cosC


ou
ad

an isosceles. [NCERT EXEMPLAR, CBSE 2016]


Y

SOLUTION Proceed as in Example 26.


1 a a3
nd
Re

EXAMPLE28 Prove that: 1 b b3 =(a -b) (b - c) (c - a) (a + b + c)


Fi

1 c c3

[NCERT, CBSE 2011,2012,2013]


1 a a3
SOLUTION Let A = 1 b b3 Applying R2 -> R2 - R1 and R3 -» R3 - Rj, we obtain
1 c c3

1 a «3
A = 0 b-a b3-a3
0 c-a c3-a3

1 a a3
=> A = (b - a) (c - a) 0 1 b2 + a2 + ab [Taking out (&-a) from R2 and (c-o) from R3]
0 1 c2 + a2 + ac

ReadYourFlow.COM
DETERMINANTS 6.29

fl3
1 a
=> A =(b - a) (c - a) 0 0 (b2 - c2) + (ab - ac) [Applying R2 ->^2 -R3l
0 1 c 2 + a 2 + ac

1 a a3
=> A =(b - a) (c - a) 0 0 (b -c)(b + c + a)
2 a+
c+ 2 ac
0 1
1 a a3
=> A =(b - a) (c - a) (b - c) 0 0 a+b + c [Taking out - c) common from R2]
0 1 c2 + a2 + ac
0 a+b+c
=> A =(b - a) (c - a) (b -c)x l* [Expanding along Cj]
1 c2 + a2 + ac

w
■=>
A =(b - a) (c - a) {b - c) {0 -(a + b + c)} =(a -b) (b - c) (c - a) (a + b + c).
a b c 1 1 1
EXAMPLE 29 Show that a2 b2 c2 a2 b2

Floc2 =(a - b) (b -c)(c - a) (ab +bc + ca).

ee
be ca ab a3 b3 c3

Fr
[NCERT, CBSE 2007, 2011, 2013, 2014]
a b c for
ur
SOLUTION Let A = a2 b2 c2 . Multiplying Cj, C2 and C3 by a, b and c respectively, we get
be ca ab
ks
Yo
oo

a2 b2 c2
A = ^— «3 b3 c3
B

abc abc abc abc


re
ou
ad

a2 b2 c2
abc a3 b3 c3
Y

=> A = ---- [Taking abc common from R3]


abc 1 1 1
nd
Re

a2 b2 c2
Fi

A=- 1 1 1 [Applying R2++R3]


a3 b3 c3
1 1 1
=> A= «2 b2 c2 [Applying Rl <+> R2]
a3 b3 c3
1 0 0
=> A= fl2 b2_a2 c2_a2 [Applying C2 -+ C2 -C1 andC3 -> C3 -CJ
a3 b3 -a3 c3 -a3
1 0 0
=> A = a2 (b -a) (b + a) (c-a)(c + a)
a3 (b-a)(b2 +ba + a2) (c - a) (c2 + ca + a2)

ReadYourFlow.COM
6.30 MATHEMATICS-XII

1 0 0
A = (b-a)(c-a) a2 Taking (b - a) and (c - fl) common
=> b+a c+a
from C2 andCg respectively
a3 b2 + a2 + ab c2 + ac + a2
b+a c+a
=> A = (b - a) (c - a) x l x [Expanding along R1]
b2 + a2 + ab c + a 2 + ac
2

b -c c+a
=> A —(b fl) (c a) , 2 2 i [Applying C-l —» Cj -C2]
b -c + ab - ac c + a 2 + ac
2

b -c c+a
=> A=(b - a) {c - a)
(b2-c2) + a(b-c) c2 + a2 + ac
1 c+a
=> A= (b - a) (c - a) (b - c) [Taking {b - c) common from
b + c + a c2 + a2 + ac

w
=> A={b -a)(c -a) (b -c) (c2 + a2 + ac -be -c2 - ac - ab - ac - a2)
=>

Flo
A=(b - a) (c - a) (b - c) (- be - ab - ac) = (a - b) (b - c) (c - a) (ab + be + ca).
x x2 1+x3

ee
Fr
EXAMPLE 30 lfx*y*zand y y2 1+y3 = 0, then prove that xyz =-1.
z z2 1+z3
or
ur
[NCERT, CBSE 2011]
f
SOLUTION We have,
ks

x x2 1+x3
Yo
oo

A = y y2 1+y3
eB

z z2 1+z3

X x2 1 2 3
X x°
r
ou

Since each element of third


ad

=> A = y y2 1 + y y y 3
2
column is sum of two elements
Y

z z2 1 z z2 z3
nd
Re

X x2 1 1 x x2
Taking x, y and z common from C j,
Fi

A = y y2 1 + xyz 1 y y2
C2, andC3 in second determinant
z z2 1 1 z z2

X 1 x2
1 x X2
=> A = - y 1 y2 + xyz 1 y y2 [Interchanging C2 and C3 in first determinant]
z 1 z2 1 z z2

1 X x2 1 X x2
=> A = 1 y y2 + xyz 1 y y1 [Interchanging Cj and C2 in first determinant]
1 z z2 1 z z2
1
X x2
=> A = 1 y y2 (1 + xyz)
1 z z2

ReadYourFlow.COM
DETERMINANTS 6.31

1 x x2
=> A = 0 y-x y1 -x 2 (1 + ^/2) [Applying R2 ^2 “ ^1 an<^ ^3 ^3 ~
2 2
0 z-x Zz -X

1 X x2
Taking (y - x) and (z - x)
=> A = (y - x) (z - x) 0 1 y + x (1 + xyz)
common R2 and R 3 resp.
0 1 z+x

A = (y - x) (z - x) x 1 x1 y+x (1 + xyz) [Expanding along Cj]


1 z+x

=> A = (y -x) (z -x) (z + x -y -x) (1 + xyz)


=> A = (y - x) (z -x) (z - y) (1 + xyz) = (x - y) (y -z) (z - x) (1 + xyz)

w
A=0
(* - y) (y -2) (2 - *) (i + xyz) = 0
=>

Flo
1 + xyz = 0 [v x^y^z =>x-y*0, y-z*0 and z- x * 0]
xyz = -1.

ee
Fr
x x2 1 + px3
9 O
EXAMPLE 31 For any scalar p prove that A = y y 1 + py = (1 + pxyz) (x -y) (y -z) (z - x).
or
ur
z z2 1 + pz3
sf
[NCERT, CBSE 2010]
k
Yo

SOLUTION We have.
oo

2 3
XX 1 + px
B

A = y y2 1 + yy3
re

z z2 1 + pz3
ou
ad

2 3
X x2 1 X X px°
Y

2 3 ■/ Each element in III column is


A = y y2 1 + y y py sum of two elements
nd
Re

z z2 1 z z2 pz3
Fi

1 X2 X 1 x x2 Interchanging andC3 in first det.


A = - 1 y2 y + pxyz 1 y y2 Taking x, y, z common from R-i, R-2 ^3
1 z2 z 1 z z2 respectively and p from C3 in 2nd det.

1 X x2 1 X x2
=> A = 1 y y2 + yxyz \ y y [Interchanging C2 and C 3 in first determinant]
1 z z2 1 z z2

1 x x2
=> A = (1 + pxyz) 1 y y2
1 z z2

ReadYourFlow.COM
6.32 MATHEMATICS-XII

1 X x2
=> A = (1 + pxyz) 0 y-x y2-x2 [Applying R2 -> R2- and R3 -> R3 - Rj]
0 z -x z2-x2

1 X x2
Taking (y - x) and (z - x) common
=> A = (1 + pxyz) (y - x) (z - x) 0 1 y + x
from R2 and R3 respectively
0 1 z+x
1 y+x
A = (1 + pxyz) (y - x) (z - x) [Expanding along C-|]
1 Z+X
=> A = (1 + pxyz) (y - x) (z - x) (z + x - y - x) = (1 + pxyz) (x - y) (y -z) (z - x)
2
1 a a -be
EXAMPLE 32 Using properties ofdeterminants, shozv that 1 b b2-ca = 0.

w
2
1 c c -ab

Flo
1 a a2 -be
SOLUTION Let A = 1 b b2 - ca . Then, [CBSE 20021

ee
1 c c - ab

Fr
1 a a2 1 a -be
for
ur
A = 1 b b2 + 1 b -ca [Each element of third column is sum of two elements]
lee2 1 c -ab
ks
Yo
oo

1 a a2 1 a be
B

A = 1 b b2 1 b ca [Taking (- 1) common from C3 of second determinant]


re

1 c c2 1 c ab
ou

2
ad

1 a a2 a a abc
Multiplying Rj, R2 and R3 of second
Y

=> A = 1 b b2 '-b b2 abc


abc determinant by a, b and c respectively
1 c c2 c c2 abc
nd
Re
Fi

1 a a2 a a2 1
abc b b2 1
=> A = 1 b b2 [Taking abc common from C3 of second determinant]
abc
1 c c2 c c2 1

l a a2 a 1 a2
A - 1 b b2 + b 1 b2 [Applying C2 ++ C3 in second determinant]
1 c c2 c 1 c2

l a a2 l a a2
=> A = 1 b b2 1 b b2 [Applying Cj ++ C2 in second determinant]
1 c c2 1 c c2

=> A =0.

ReadYourFlow.COM
DETERMINANTS 6.33

a2 + 2a 2a +1 1
EXAMPLE 33 Prove that: 2a + l a+ 2 1 =(fl-l)3.
3 3 1

a2 + 2a 2a + l 1
SOLUTION Let A = 2a + l a+ 2 1 . Then, [NCERT EXEMPLAR]
3 3 1

a2 + 2a-3 2a-2 0
A= 2a-2 0-1 0 [Applying R-j -+ Rj -Rs, R2 -> R-2
3 3 1
(0+3) (0-1) 2(0-1) 0
A= 2(0-1) (0-1) 0

w
3 3 1
0+3 2 0
A=(0-1)2 2 1 0

Flo [Taking (0 -1) common from R2 and R3]

ee
3 3 1

Fr
9 0+3 2
A=(«-l)2 2 [Expanding along C 3]
1 for
ur
=> A =(0-l)2 (0+ 3-4) =(0-l)3
lab
ks
Yo

EXAMPLE 34 Let a, b and c denote the sides BC, CA and AB respectively of A ABC . If 1 c a =0,
oo

1 b c
B

9 9 9
then find the value of sin A + sin B + sin C.
re

SOLUTION We have,
ou

lab
ad

1 c a =0
Y

1 b c
nd
Re

1 a b
Fi

0 c-a a-b =0 [Applying R2 -> R2 -Ri, ^3-^1]


0 b-a c-b
c-a a-b
=> [Expanding along CJ
b-a c-b -r

=> (c-a) (c-b) -(a-b) (b-a) =0


=> a2 +b2 +c2 -ab-bc-ca = 0
=> 2a2 + 2b1 + 2c2 - 2ab - 2bc - 2c0 = 0 [Multiplying both sides by 2]
(a-b)2 + (b-a)2 + (c-a)2 =0
a-b = 0,b-c =0 and c - 0 = 0
a=b =c
AABC is equilateral
A = B =C= —
3

ReadYourFlow.COM
6.34 MATHEMATICS-XII

sin2A + sin2B + sin2C = 3sin2 — = 3x rvsf 9


3 2 4
a -1 0
EXAMPLE 35 Iff(x) = ax a -1 , using properties of determinants, find the value off(2x)-f(x).
ax2 ax a
[CBSE 2015]
SOLUTION We have.
a -1 0
/(*) = ax a -1
ax2 ax a
a -1 0
=> f(x) = 0 a + x -1 [Applying R3 -> R3 -xR2/ R2 -> R2 -^1]

w
0 0 a+x
a + x -1

Flo
=> f{x)=a 0 [Expanding along CJ
a+x

ee
f(x)=a(a + x)2

Fr
=> f(2x) =a(a + 2x)2 [Replacing x by 2x]
f(2x)-f{x) =a(a + 2x)2 -a {a + x)2 =a {(a + 2x + a + x)(a + 2x-a-x)} = ax(2a+ 3x)
for
ur
x p q
ks

EXAMPLE36 Show that: p x q =(x-p) (x2 +px-2q2).


Yo
oo

q q x
B

x p q
re

SOLUTION Let A = p x q . Then, [NCERT EXEMPLAR]


q q x
ou
ad

x-p p q
Y

A = p-x x q [Applying Cj ->C1-C2]


nd
Re

0 q x
Fi

1 p q
&=(x-p) -1 x q [Taking(x-p) common fromCj]
0 q x
1 p q
=> A = (x-p) 0 x + p 2q [Applying R2^>R2 + RJ
0 q x

x + p 2q
A=(x-p) [Expanding along C^]
X

=> A=(x-p) (x2 + px-2q2)


1 1 1
m+ 1 m+ 2
EXAMPLE37 IfmeNandm > 2, prove that: "'c 1 Ci Ci = 1.
mc2 m+ l
C2
m+ 2
C2

ReadYourFlow.COM
r-

DETERMINANTS 6.35

1 1 1
rn+ 1 m+ 2
SOLUTION Let A = Ci Cl . Then,
mr m+ 1 m+ 2
C2 C2 C2

1 1 1
m + 1c m+ 1 m+ n+ 1
A = mC 1 l Co + 1 •••Hcr + ',cr-i = Cr
tn m+ 1 m+ 1 m+ 1
C2 C2 Ci + C2

1 1 0
mr' m+ 1 m+ 1 [Applying C3 ->C3 -C2]
=> A = L1 Cl Co
mr' m+ 1 m + 1 Ci
l2 C2

1 1 0
mr m+ 1 n + 1Cr in C2]
=> A = Ll Cq + Li C0 [Applying : nCr + "Cr _ : =
mr , mr m+ 1

w
mr Li + C2 Cl
c2
1 0 0
=> A = mr
Cq
m+ 1
m+ 1
Co

Flo [Applying C2 C2 -CJ

ee
C2 Ll Cl

Fr
mr OT + 1 C0
C0 [Expanding along Rj ]
=> A =
m+ 1
Ll Cl or
ur
1 1
=> = (m + 1 - m) =1
sf
A =
m m+1
k

10! 11! 12!


Yo
oo

EXAMPLE 38 Evaluate: A = 11! 12! 13!


B

12! 13! 14!


re

SOLUTION We have,
10! 11! 12!
ou
ad

A = 11! 12! 13!


Y

12! 13! 14!


10! 11x10! 12x11x10!
nd
Re

=> A = 11! 12x11! 13x12x11!


Fi

12! 13x12! 14x13x12!


1 11 132
"Taking 10!, 11! and 12! common
A =10!x ll!x 12! 1 12 156
from R\, R2 and ^3 respectively
1 13 182
1 11 132
=> A = 10!x ll!x 12! 0 1 24 [Applying R2->R2- Rv R3 ^ R3 - RJ
0 2 50
1 24
=^> A = 10!x ll!xl2!x [Expanding along CJ
2 50

=> A = (10! x 11! x 12!) x 2

x+y X X
EXAMPLE 39 Prove that: 5x + 4y 4x 2x =x:>.
lOx + 8y 8x 3x
[CBSE 2002C, 2009,2014]

ReadYourFlow.COM
6.40 MATHEMATICS-XII

0 a2
A_(a+b + c)2 ab + ac - a
=> 0 bc + ba-b2 b2 [Applying Cj -> Cl (a), C2 -> C2 (&)]
ab
- lab - lab lab

ab + ac a2 a2
+ + c)2
=> b2 be + ba b2 [Applying C-j -> C1 + C3/ C2 -> C2 + C3]
ab
0 0 lab

b+c a a
A = (a + fc + c)2 Taking a, b and lab common
=> x ab x lab b c+a b
ab from R1 , R2 and R3 respectively
0 0 1

A = lab (a + b + c)2xlx ^ + c a
=> [Expanding along R3]
c+a

w
=> A = lab {a + b + c)2 {(b + c) (c + a) - ab} = labc (a + b + c)3

EXAMPLE46 Shoiv that:


(b + c)2
ab
ba
(c + a)2
ca
cb
Flo
= labc (a + b + C)2,

ee
ac bc (a + b)2

Fr
for [NCERT,CBSE 2006,101
ur
(b + c)2 ba ca
SOLUTION Let A = ab (c + a)2 cb . Multiplying R^, R2 and R3 by a, b and c
ks

ac be (a + b)2
Yo
oo

respectively, we get
B

a (b + c)2 ba2 ca2


re

1
A = ab2 (c + a)2 b cb2
ou

abc
ad

ac2 be2 (a + b)2 c


Y

(b + c)2 «2 «2
nd
Re

b2 (c + a)2 b2 Taking a, b and c common


=> A = abc
Fi

abc from C1 , C2 andC3 respectively


c2 c2 (a + b)2

=> A = labc (a + b + c)3 [Proceed as in Example 45]


EXAMPLE 47 Show that
l+a2-b2 lab -2b
lab 1 - a2 +b2 la = (1 + a2 +b2)3
2b -la 1 -a2 -b2 [NCERT, CBSE 2009, 2010 C]

1 + a2 -b2 lab -2b


SOLUTION Let A = lab l-a2+b2 la
lb -la l-a2-b2
We shall try to introduce zeros at as many places as possible keeping in mind that we have to
introduce the factor 1 + a2 +b2.

ReadYourFlow.COM
DETERMINANTS 6.41

Applying Cj ->C| -bC^ andC2 ->C2 + «C3 , we get

1 + a2 +b2 0 -2b
A= 0 l+a2+b2 2a
b(l + a2+b2) -a(l + a2+b2) 1 - a2 -b2

1 0 -2b
=> A = (l+a2+b2)2 0 1 2a [Taking (1 + a2+ b2) common from both Ci&c C2]
b -a 1-a2-b2

1 0 -2b
=> A = (l + a2 +b2)2 0 1 2a [Applying R3 -> R3 -bR^ + aR2]
0 0 l + a2+b2

w
1 2a
=> A = (1 + a2 +b2)2 x 1 x [Expanding along Cj]
0 \+a2+b2

Flo
ee
A = (l + a2 +b2)3

Fr
b2 + c2 ab ac
EXAMPLE 48 Shoiv that: ba c2 + a2 be = 4a2 b2 c2.
for
ur
ca cb a2 + b2
ks

b2 + c2 ab ac
Yo
oo

SOLUTION Let A = ba c2 + a2 be
B

ca ^ a2 + b2
re

Multiplying Ri, K2 and R3bya,b and c respectively, we get


ou
ad

a(b2 + c2) a2b a2 c


Y

1 b2a b (c2 + a2) b2e


A =----
abc
nd

2
Re

c a c2b c (a2 +b2)


Fi

b2+c2 a2 a2
abc Taking a, b and c common from
=> A =---- b2 c2 + a2 b2
abc C2 andC3 respectively
c2 c2 a2 + b2

2 (b2 + c2) 2 (a2 + c2) 2(a2+b2)


b2 2 2 b2 [Applying R^ ^ Ri + R2 + R3]
=> A = C + fl
c2 c2 a2+b2

b2 + c2 a2 + c2 a2 + b2
=> A =2 b2 c2 + a2 b2 [Taking 2 common from R-J
c2 c2 a2 + b2

ReadYourFlow.COM
6.42 MATHEMATICS-XII

b2 + c2 c1 + a2 a2+b2
A =2 -c2 0 -a2 [Applying R2 R2 - R\ and R3 -> R3 - RJ
-b2 -a2 0

0 c2 b2
=> A =2 -c2 0 -a2 [Applying R1 Rl + R2 + R3]
-b2 -a2 0

-c2 0
A =2 \-c2 + b2 [Expanding along R1 ]
-b2 0 -b2 «2

=> A = 2 (a2 b2 c2 + a2 b2 c2) = Aa2 b2 c2

w
a b ax + by
EXAMPLE 49 Prove that: b c bx + cy =(b2 - ac) {ax2 + 2bxy + cy2).
ax + by bx + cy 0

Flo
ee
a b ax + by

Fr
SOLUTION Let A = b c bx + cy. Applying C3 -> C3 - xCj - yC2 , we get
ax + by bx + cy 0 for
ur
a b 0
A = b c 0
ks

ax + by bx + cy -x {ax + by) -y {bx + cy)


Yo
oo

a b 0
B

=> A = b c 0
re

ax + by bx + cy - {ax2 + 2bxy + cy2)


ou
ad

a b
A = - {ax2 + 2bxy + cy2) [Expanding along C3]
Y

b c

=> A = - {ax2 + 2bxy + cy2) {ac -b2) = {b2 - ac) {ax2 + 2bxy + cy2).
nd
Re
Fi

EXAMPLE 50 Without expanding the determinant, show that (a+ b + c) is a factor of the determinant
a b c
b c a .
cab
a b c
SOLUTION Let A = b c a . Applying Cj -» C-! + C2 + C3 , we get
cab

a+b + c b c
A= b+c+a c a
c +a+b a b

1 b c
=> A = {a + b + c) lea [Taking {a + b + c) common from Cj]
1 a b

ReadYourFlow.COM
DETERMINANTS 6.43

1 b c
=> A = (a + b + c) 0 c-b a-c [Applying R2R-2 ~ R\ and ^3 ^3 _ ^1]
0 a-b b-c
c- -b a-c
=> A=(fl + & + c)xlx [Expanding along Cj]
a-b b-c
A =(a+ b + c) {-(b - c)2 - (a - c) {a - b)}
A=-(a + b + c)[(b - c)2 + (a - c) (a-b)] = -(a+ b + c) (a2 +b2 + c2 - ab - be - ca)
Clearly (a + b + c)isa factor of-(a + b + c) (a2 + b2 + c2 - ab - be - ca).
Hence, (a+b + c)isa factor of A.
a b c
EXAMPLE 51 If a are roots of the equation x3+px + q = 0,prove that b c a =0.

w
cab
o
SOLUTION It is given that a,b,c are roots of the equation x + px + q = 0.

Flo
a+ b + c = 0, ab+ bc + ca =p and abc = -q

ee
From example 50, we have

Fr
abc
b c a = -(a + b + c)(a2+b2 + c2-ab-bc-ca) for
ur
cab
abc
ks

b c a = 0 => (a2+b2+c2-ab-bc-ca) =0 [v a + b + c - 0]
Yo
oo

cab
B

abc
re

EXAMPLE 52 If a, b, c are positive and unequal, show that the value of the determinant b c a is
cab
ou
ad

always negative. [NCERT, CBSE 2010]


Y

abc
SOLUTION Let A = b c a . Applying Q Q + C2 + C3, we get
nd
Re

cab
Fi

a+b+c b c
A = b+c+a c a
c+a+b a b
1 b c
=> A = (a + b + c) 1 c a [Taking (a + b + c) common from CJ
1 a b
1 b c
=> A = (a + b + c) 0 c-b a-c [Applying R2 -> R2 - ^1 anc^ ^3 ^-3 ~
0 a-b b-c
/ 1 v c-b a-c [Expanding along C-J
=> A = (a + b + c) . .
J a-b b-c
A = (a + b + c){-(c-b)2 -(a-b) (a - c)} =(a + b + c) (-a2 -b2 - c2 + ab + be + ca)

=> A - - ^(a + b + c) (2a2 + 2b2 + 2c2 - 2ab - 2bc - 2ca)

ReadYourFlow.COM
6.44 MATHEMATICS-XII

=> A = - ^(a + b + c) {(a -b)2 + (b - c)2 + (c - a)2\

A < 0 ['•• a + b + c>0,(a-b)2 >0,(b-c)2 >0r(c-a)2 > 0]


n b c
EXAMPLE 53 Ifa+ b + c * 0 and b c a = 0, then prove that a =b =c.
cab
SOLUTION We have, [NCERT EXEMPLAR]
a b c
b c a = ~(a + b + c) {(a-b)2+(b-c)2+(c-a)2} [See example 52]
cab
a b c
b c a =0

w
cab
^(a + b + c) ^(a-b)2 +(b-c)2 +(c-a)2 ■ =0
=>

Flo
ee
=> {a-b)2 +(b-c)2 +(c-a)2 =0 [v a + b+ c * 0]

Fr
=> a-b = 0,b-c = 0 and c - a = 0
=> a = b =c. for
ur
EXAMPLE 54 If a, b, c are real numbers, prove that
a b c
ks
Yo

b c a = -(a + b + c) (a + biv + civ2) (a + bur + civ),


oo

cab
B

where w is a complex cube root of unity.


re

a b c
SOLUTION
ou

Let A = b c a . Applying^-»(^ + C2 + C3/we get


ad

cab
Y

a+b +c b c
nd
Re

A= b + c+ a c a
c+ a+b a b
Fi

1 b c
=> A = {a + b + c) 1 c a [Taking {a + b + c) common from CJ
1 a b
1 b c
=> A = {a + b+ c) 0 c-b a-c [Applying R2 R2 - R-[ and R3-> R3 - RJ
0 a-b b-c
. , , . c-b a-c
=> A = {a + b + c) , , [Expanding along Cj]
a-b b-c
A = {a + b+ c){-{b-c)2-{a-c){a-b)}
=> A = -{a + b + c){a2+b2 + c2-ab-bc-ca)
v a2+b2 + c2-ab-be-ca
=> A =-{a+ b + c) {a+ bw + cw2) {a + bw2 + cw)
= {a + bw + cw2) {a + bw2 + cw)

ReadYourFlow.COM
DETERMINANTS 6.45

a b-c c+b
EXAMPLE 55 Show that: a + c b c-a =(a+ b + c) {a2 + b2 + c2).
a-b b + a c

a b-c c+b
SOLUTION Let A = a + c b c-a . Multiplying first column by a, we get
a-b b + a c

*2 b-c c+b
A = — a2 + ac b c-a
0 a2-ab b + a c

a2+b2 + c2 b-c c+b


=> A = I a2+b2 + c2 b c-a [Applying C! Cj + bC2 + cC3]

w
U a2 +b2 + c2 b+a c

1 b-c
A = - (a2 +b2 + c2) 1 b
c+b
c-a

Flo [Taking a2 +b2 + c2 common from C!]

ee
a 1 b+a c

Fr
1 b-c c+b
=> A= - (a2 +b2 + c2) 0i c -a-b [Applying R2-+ R2- ^3 ^3 _
for
ur
a 0 a+c -b
+c -a-b
Ua2 + b2 + c2)xlx\aC
ks

=> A = [Expanding along C-J


Yo

-b
oo

A = - (a2 +b2 + c2) (- bc + a2 + ac + ba + be) = {a2 +b2 + c2) (a+ b + c)


B

a
re

3a -a + b -a + c
-b + a 3b -b + c = 3 (a + b + c) (ab + bc + ca).
ou

EXAMPLE 56 Show that:


ad

-c + a -c + b 3c
Y

[NCERT EXEMPLAR, CBSE 2006 C, 2013]


nd
Re

3a -a + b -a + c
-b + a 3b -& + c . ApplyingC],+C2+C3 , we get
Fi

SOLUTION Let A
— c + a -c + b 3c

a + b + c -a + b -a+ c
A = a+b+c 3b -b + c
a + b + c -c + b 3c
1 -a + b -a + c
=> A = (a + b + c) 1 3b -b + c [Applying Cj +C2 +C3]
1 -c + b 3c
1 -a + b -a + c
=> A=(a + b + c) 0 2b + a -b + a [Applying R2 —» R2 - R3 —> R3 - R\]
0 -c + a 2c + a
2b + a -b + a
=> A = (a + b + c) [Expanding along CJ
-c + a 2c + a
=> A = (a + b + c){(2h + a) (2c + a)-(-b + a) (-c + a)}

ReadYourFlow.COM
6.46 MATHEMATICS-XJl

=> A = (a + b + c) [(4bc + lab + 2ca + a2) - (be -ab - ac + a2)}


=> A = (a + b + c)(3bc + Sab + Sea)
=> A = S (a + b + c) (ab + be + ca).
Type III SOLUTION OF DETERMINANT EQUATIONS
a + x a-x a-x
EXAMPLE57 Solve: a - x a + x a-x =0. [CBSE 2004, 2005, 2011]
a-x a-x a + x
a + x a-x a-x
SOLUTION Let A = a-x a + x fl-.r . Applying C-l-XT-l+ C2+C3 , we get
a-x a-x a+x
3a-x a-x a-x
A = 3a-x a + x a-x

w
3a-x a-x a + x
1 a-x a-x
A = (3a-x) l a + x a-x
1 a-x a+x

Flo [Taking (3a - x) common from C-J

ee
1 a-x a-x

Fr
=> A =(3a-x) 0 2x 0 [Applying R2R2 - Ri and R3 -> R3 -R^
0 0 2x for
ur
lx 0
=> A = (3fl - x) x 1 x [Expanding along Cj]
0 lx
ks
Yo

=> A =(3a-x) 4x2


oo

A = 0 => (3a-x)4:X2 = 0 => x = 0, 3a.


B

x -2 2x - 3 3x - 4
re

EXAMPLE58 Solve: x-4 2x-9 3x -16 = 0.


ou

x - 8 2x - 27 3x - 64
ad
Y

x-2 2x-3 3x-4


SOLUTION Let A = x - 4 2x - 9 3x -16 . [CBSE 20111
nd
Re

x - 8 2x -27 3x - 64
Fi

Applying C2 ->C2 -2C-j andC3 ->C3 - SC^^e get


x-2 1 2
A = x-4 -1 -4
x - 8 -11 -40
x-2 1 2
=> A = -2 -2 -6 [Applying R2 -+ R2 - R1 and R3 -+ R3 - R-^]
-6 -12 -42
x-2 1 2
=> A = (- 2) (- 6) 1 1 3 [Taking (- 2) & (- 6) common from R2 & R3 respectively]
1 2 7
x-2 1 2
=> A = 12 1 1 3 [Applying R3 -+R3-R2]
0 1 4

ReadYourFlow.COM
DETERMINANTS 6.47

1 3 1 2
=> A = 12 (x-2) 1 1 4 [Expanding along C-J
1 4
A = 12 {(x - 2) (4 - 3) -(4 - 2)} =12 (x-4)
A = 0 => 12 (x - 4) = 0 => x = 4.
LEVEL-2

Type IV EVALUATING DETERMINANTS BY USING THE PROPERTIES OF DETERMINANTS AND


PROVING IDENTITIES
a a 3 a4-l
o
EXAMPLE 59 If a, b, c are all distinct and b b b4-l = 0.
c c3 c4-l

low
Shoiv that abc (ab + be + ca) = a+ b + c.
a a3 a4-1
SOLUTION Let A = b b3 b4 -1 . Then,
c c3 c4-l

ee
rF
Fr
a a 3 a4-l a a3 -1
A = b b3 b4-l + b b3 -1
for
c c3 c4 —1 c c3 -1
u
ks

1 a2 «3 a a3 1
Yo
oo

=> A = abc 1 b2 b3 -b b3 1
B

1 c2 c3 c c3 1
re

1 a2 a3 a a3 1
Applying R2 ^2 _
ou
ad

=> A =abc 0 b2-a2 b3 - a3 b-a b 3-a3 0


and ^ ^3 ~
Y

0 c2 - a2 c3 - a3 c-a c3-«3 0
nd
Re

1 a2 a3 a a3 1
Fi

=> A = abc (b -a)(c - a) 0 b + a b2 + a2 + ab -(b - a) {c - a) 1 b2+a2 + ab 0


0 c + a c2 + a2 +ac 1 c2 + a2 +ac 0

b + a b2 + a2 + ab 1 b2 + a2 + ab
=> A = abc (b - a) (c - a) -(b - a) (c - a)
c + a c2 + a2 + ac 1 c2 + a2 + ac
b + a b2 + a2 + ab 1 b2 + a2 + ab
=> A = abc (b - a) (c - a) ~(b - a) (c - a)
c-b c2-b2+a(c-b) 0 c2 -b2 + a (c -b)
[Applying R2 -> ^2 ~
b + a b2 + a2 + ab 1 b2+a2 + ab
=> A = abc (b -a)(c - a) (c - b) -(b-a) (c-a) (c-b) q
1 a+b+c 1 a+b +c

=> A = abc (b -a)(c - a) (c-b) [(b + a) (a + b + c) -(b2 + a2 + ab)}


- (b - a) (c - a) (c - b) (a + b + c - 0)
=> A = abc (a -b)(b - c) (c - a) (be + ca + ab) -(a-b)(b - c) (c -a)(a + b + c)

ReadYourFlow.COM
6.48 MATHEMATICS-XII

=> A = (a - b) (b - c) (c - a) [abc (ab + bc + ca) - (a + b + c)}


Now, A = 0
=> (a -b) (b - c) (c - a) {abc (ab + bc + ca) - (a + b + c)} =0
=> abc (ab +bc + ca) - (a + b + c) =0 [v a*b*c c-a*0]
=> abc (ab + be + ca) = a + b + c
EXAMPLE 60 Ifa,b,c are all positive and are pth, qth and rth terms of a G.P., then show that
log a p 1
A = log b q 1 = 0
log c r 1
SOLUTION Let A be the first term and R be the common ratio of the G.P. Then, we have
a = ARP ~ 1 => log a = log A + (p -1) log R
b = AR^-1 => logb = log A + (q -1) log R

low
c = ARr - 1 => log c = log A + (r -1) log R
log A +(p -1) log R p 1
A = log A +(q -l)log R q 1

ee
log A + (r -1) log R r 1
rF
Fr
log A + (p -1) log R p -1 1
A = log A + (<7-1) log R q-1 1 for [Applying C2 ->C2 -C3]
log A + (r -1) log R r -1 1
u
0 p-l 1
ks

=> A = 0 q-1 1 [Applying Cj - (log A)C3 - (log R)C2]


Yo
oo

0 r -1 1
B

=> A = 0.
re

xa yb zc abc
EXAMPLE 61 lfx + y+z = 0, prove that yc za xb = xyz cab
ou
ad

zb xc ya b c a
Y

SOLUTION We have. [NCERT EXEMPLAR]


nd
Re

xa yb zc
Fi

LHS = yc za xb
zb xc ya
za xb yc xb yc za
= xa -yb + zc
xc ya zb ya zb xc
= xa (yza -x be)-yb (y ac -zx b ) +zc (xyc -z ab)
= xyz(a3 +b3 + c3)-abc (x3 +y3 +z3)
= xyz (a0 + b3 + c3) - 3abc xyz [v x + i/ + z = 0 x3 +y3 +z3 =3xyz]
= xyz (a3 + b3 + c3 - 3abc)
= xyz(a + b + c) (a2 +b2 + c2-ab -bc-ca) ••■(i)

abc
RHS = xyz cab
b c a

ReadYourFlow.COM
DETERMINANTS 6.49

a+b+c a+b+c a+b+c


= xyz c a b [Applying R-^ -» Rj + R2 + ^3]
b c a
1 1 1
= xyz{a + b + c) cab [Taking a + b + c common from R^]
b c a
1 0 0
= xyz (a + b + c) c a-c b-c [Applying C2 ->C2-C1/C3 -+C3-C1\
b c-b a-b
a-c b-c
= xyz (a + b + c) [Expanding along R-J
c-b a-b

w
= (xyz) (a + b + c) {(a-c) (a-b)-(b-c) (c-b)J
= (xyz) (a + b + c) (a2 +b2 + c2 -ab -bc-ca) ...(h)
From (i) and (ii), we infer that

Flo
ee
xa yb zc a b c

Fr
yc za xb =xyz cab
zb xc ya b c a for
ur
, 2
ca-b2 ab-c2
be-a2 a b c
•j
EXAMPLE 62 Prove that: ca-b ab -c2 bc-a2 b c a .
ks
Yo

ab -c2 bc-a2 ca-b2 cab


oo
eB

[NCERT EXEMPLAR]
a b c
r

SOLUTION Let A= b c a and let a C,y = Cofactor of (A)zy in A. Then,


ou
ad

cab
Y

c a b a u2 r b c = ab-c2
cll - = bc-a2, C 12 = ac-o , Li3 =
nd

-~
Re

a b c b c a
Fi

b c a c a b
C21 -- = ac-b2, C22 = = ab-c2, C = bc-a2
a b c b 23="c «

b c a c a b
C31 “ = ab-c2, C 32 -~
= bc-a2, C 33 = = ac-b2
c a b a b c
Let C = [c(y] be the matrix of cofactors of elements of A. Then,
3-1 [By Property 12]
|C| = |A|
, 2
Cll ^12 c13 a b c
=> C2i C22 C23 b c a
C31 c32 c33 cab
, 2
bc-a2 ca-b2 ab-c2 a b c
=> ca-b2 ab-c2 bc-a2 b c a
ab-c bc-a ca-b cab

ReadYourFlow.COM
r
6.50 MATHEMATICS-XII

ALITER
be-a2 ca-b2 ab-c2
LHS = ca-b2 ab-c2 be-a2
ab-c2 be-a2 ca-b2
Applying Cj->C1+C2+C3/ we get
ab+bc + ca-(a2+b2+c2) ca-b2 ab-c2
LHS = ab+bc + ca-(a2+b2+ c2) ab-c2 bc-a2
ab+bc + ca-(a2+b2+ c2) bc-a2 ca-b2

1 ca-b2 ab-c2
2 Taking ab + be + ca-(a2+b2 + c2)
=> LHS = \ ab+ bc + ca-(a2+b2 + c2)\ 1 ab-c2 bc-a
common C i
1 J 1 bc-a2 ca-b2

low
1 ca-b2 ab -c2
=> LHS =-(a2+b2+c2-ab-bc-ca) 0 a(b -c) + (b2 -c2) b (c-a)+(c2-a2)
0 c(b-a)+(b2 -a2) a (c-b) + (c2-b2)

ee
rF
Fr
[Applying R2 R2 -Ri, R3 -> R3 -^i]

1 ca-b2
for ab-c2
u
=> LHS =-(a2+b2+c2-ab-bc-ca) 0 (b -c) (a + b + c) (c-a)(a + b + c)
ks

0 (b -a) (a + b + c) (c-b)(a + b + c)
Yo
oo

1 ca-b2 ab-c2 Taking (a + b + c)


B

=> LHS =-(a+ b + c)2(a2 +b2 + c2 -ab-bc-ca) 0 b-c c-a common from
re

0 b-a c-b R2and K3


ou
ad

=> LHS =-(a + b + c)2(a2+b2+e2^ab -bc-ca) b-c c-a [Expanding along C^]
Y

b-a c-b
n j
nd

=> LHS =-(a+ b + c)2(a2+b2+c2-ab-bc-ca) (Ibc-b2 -c -bc + ac + ab-a )


Re

=> LHS =-(a+ b + c)2 (a2+b2+c2-ab-bc-ca) (-a2-b2-c2 + ab+bc + ca)


Fi

=> LHS = (a + b + c)2 (a2+b2+c2-ab-bc-ca)2


=> LHS = {(a + b + c) (a2+b2+c2-ab-bc-ca)}2
Now,
a b c
RHS =b c a
cab
=> RHS = [-(a + b + c) (a2 + b2 + c2 - ab-be - ca)}2 [See Example 50]
=> RHS = (a + b + c)2 (a2+b2+c2-ab-bc-ca)2 ...(ii)
From (i) and (ii), we obtain LHS = RHS
9 9 9 2
bc-a ca-b ab-c a b c
i.e. ca-b2 ab-c2 bc-a2 b c a
ab-c2 bc-a2 ca-b2 cab

ReadYourFlow.COM
DETERMINANTS 6.51

bc-a2 ca-b2 ab-c2


example 63 Prove that: ca-b2 ab-c2 bc-a2 is divisible by a + b + c and find the
ab-c2 bc-a2 ca-b2
quotient. [NCERT EXEMPLAR, CBSE 2016]
SOLUTION From Example 62, we obtain
bc-a2 ca-b2 ab-c2 a b c2
ca-b ab-c bc-a = b c a
ab-c2 bc-a2 ca-b2 cab
From Example 50, we obtain
a b c
"y 'y 'y
b c a =-(a + b + c)(a +b +c -ab-bc-ca)

low
cab
bc-a2 ca-b2 ab-c2
ca-b2 ab-c2 bc-a2 =(a + b + c)2 (a2+b2 + c2-ab-be-ca)2

ee
ab-c2 bc-a2 ca-b2
rF
Fr
Clearly, RF1S is divisible by (a + b + c) and the quotient is (a + b+ c) (a2 +b2 + c2 -ab -bc-ca)2.
Hence, LHS is divisible by a + b+ c and (a + b + c) (a2 + b2 + c2 - ab -bc-ca)2 is the quotient.
for
EXAMPLE 64 Find a quadratic polynomial <t> (x) whose zeros are the maximum and minimum values of
u
ks

the function
Yo

1 + sin2x cos2 x sin 2x


oo

2
/(*) = sin2x 1 + cos x sin 2x
B

sin2 x cos2 x 1 + sin 2x


re

SOLUTION We have,
ou
ad

l + sin2x cos2x sin 2x


Y

f(x)= sin 2 x 1 + cos2 x sin2x


nd
Re

sin2x cos x 1 + sin 2x


Fi

2 cos2 x sin 2x
f(x) =2 1 + cos2 x sin 2x [Applying Cj ->C1+C2]
1 cos x 1 + sin 2x
2
2 cos x sin 2x
=> /(*) = 0 1 0 [Applying R2 -» R2 ^3 ^3
-1 0 1
f(x) = 2 + sin 2x [Expanding along Cj_]
-1 < sin 2x < 1 for all x e R
1 < 2 + sin 2x < 3 for all x e R
=> l</(x)<3 for all x e R
=> The maximum and minimum values of f(x) are 3 and 1 respectively.
Thus, a quadratic polynomial having 1 and 3 as its roots is <j)(x) =(x-l) (x-3) or,
(j)(x) =x2 -4x+ 3.

ReadYourFlow.COM
6.52 MATHEMATICS-XII

2
secx cos x sec * +cot.r cosec x n/2
2 2 8
EXAMPLE 65 Let f(x) = COS X COS X cosec 2 x . Prove that:
2 2 15
1 COS X COS X

SOLUTION Applying -> Rj -secx R3, we obtain


2
0 0 sec x + cot x cosec x - cos x
2 2
/(*) = COS X COS X cosec2 x
2
1 COS X cos2x
2
0 0 sec x + cot x cosec x - cos x
2
=> f(x) = 0 COS X cosec2 x Applying Ci ->C1-C2
sin2x cos2x 2
COS X

low
2 0 cos 2 x
=> /(x) = (sec x + cotx cosec x - cos x) 2
sin x cos 2 x

ee
1 cosx
/(x) = - sin x cos x
rF
-cosx

Fr
vcos2x sin2x
=> /(x) = - sin x-cos x + sin xcos x
for
/(x) =-sin2x -cos3x(l -sin2x)
u
=>
ks

=> /(x) = - sin x - cos x


Yo
oo

k/2 n/2 nil k/2


B

| f{x)dx= J (-sin2x-cos5x) rfx =


J sin2 dx - J cos5 x dx
re

0 0 0 0
n/2
ou

j 1
ad

J /(x) rfx = - J (1 -cos2x) dx - j cos0 x dx


Y

0 2 0 0
^/2
nd
Re

-|7l/2 1
j /(x)dx = ~ x- —sin2x J (1 - 2f2 + f4) dt, where t = cosx
Fi

2 JO
0 0
n/2
=> _oi-rt-2f3+it5i1 2 1 7t_ 8
3 5 Jo 4 3+5J 4 15

Type V ON ADDITION OF DETERMINANTS


Two or more determinants can be added by using the following property:
a1 x p bl x p * P ^1 + &! + Ci x p
a2 y q + h v q + c2 y q = a2+b2 + c2 y q
a3 z r b3 z r c3 z r a3 + b3 + c3 2 r
i.e. the sum of two or more determinants having all columns (or rows) identical except a specific
column (or row), say first, is a determinant whose first column (or row) is the sum of the
corresponding elements of first columns (or rows) of various determinants and the remaining
columns (or rows) remain same.

ReadYourFlow.COM
DETERMINANTS 6.53

^ n(n + l)
r
2
n
EXAMPLE 66 Let A,. = 2r -1 y «2 . Show that ^ Ar = 0.
r=1
n(3n -1)
3r - 2 z
2
n{n + l)
r
2
SOLUTION We have, \ = 2r -1 y n2
n(3n-l)
3r - 2 z
2
n x n(n + l)
Zr

w
r=1 2
n
Z^
Flo
Z(2r-1) y «2
r=1 r=1

ee
n (3n -1)
t(3r-2) z

Fr
r=1 2
n n (n +1)
for
ur
Now, ^ r = 1 + 2 +... + n =
r=1 2
ks
Yo

X (2r-1) = 1 + 3 + 5 +... + (2n-1) = -{1 + (2n-1)) =n2


oo

r=1 2
B

n(3n -1)
and. X (3r -2) = l + 4 + 7+... + (3n-2) = -{l+(3«-2)} =
re

r=1 2 2
ou
ad

n(n + l) ^ n(n + l)
x 2
Y

z^= n2 y n2 = 0 [■.• Ci and C3 are identical]


nd
Re

r=1 n(3n-l) __ n(3n-l)


Fi

2 2 2

2r ~ 1 2.3r ~ 1 45r "1 n


EXAMPLE 67 If \ = X V z . Show that ^ \ = Constant.
2n-1 3n -1 5" -1 r=1

SOLUTION Using the properties of determinants, we have


n
n
I 2r ~ 1
r -\ n
Z 2 X 3r
r 1 Z 4x5r -1
n r=1 r=1 r=1
Z 4 X y z
r-1 2n-1 3" -1 5” -1

(2n -1)
Now, 2 2r “ 1 = l + 2 + 22+... + 2"_1 = lx = 2"-1
r=1 (2-1)

ReadYourFlow.COM
r

6.54 MATHEMATICS-XII

2x 3r~1 =2(1 + 3+ 32 +... + 3”_1) = 3” -1


2x = 3"-l
r=1 3-1

2 4x 5r “ 1 = 4fl+5 + 52+... + 5""1 = 4 x 5" -ll


and. = 5” -1.
r=1 5-1

n ^2n -1 3” -1 5”-l
Z Ar = x y 2 = 0 [v R-i and are identical]
r=1 2” -1 3”-l 5” -1

2r -1 mCr 1
EXAMPLE 68 If m is a positive integer and Dr = m2 -1 m+1 . Prove that

w
sin2 On2) sin2 (m) sirt2 (w + 1)
m
E Dr = °-
r =0

Flo
ree
SOLUTION Using properties of determinants, we have

F
m m m
I (2r-l) E mCr E i or
ur
r=0 r=0 r=0
f
m
m2 -1 2)n
ks

Z Dr = m+1
Yo

r=0
oo

sin2 (m2) sin2 (m) sin2 (m + 1)


B
re

m
ou

X (2r -1) =-l + {1 + 3 + 5 +... + (2m -1)} = -1 + m2 -m 2-l


ad

r = 0~|
Y

m m
£ mCr =mC0 + OTC1+... + wCm=2m and, J 1 = (m +1)
nd
Re

r=0 r=0
Fi

m2 -1 2^
m m+1
E or = m2 -1 2m m+1 = 0 [•.• and R2 are identical]
r=0 sin2 (m2) sin2 (m) sin2 (m + 1)

Type V7 EVALUATION OF DETERMINANTS BY USING FACTOR THEOREM

Iff (x) is a polynomial such that/ (a) = 0, then (x - a) is a factor of f (x).

For example, x -6 x +llx-6 vanishes for x = 1. Therefore, (x -1) is its factor.


Thus, if a determinant is a polynomial in x such that its value is zero for x = a, then x -ais a
factor of A.
1 1 1
EXAMPLE 69 Without expanding evaluate the determinant A = a h c
a1 b2 c2

ReadYourFlow.COM
DETERMINANTS 6.55

SOLUTION If we put a in A, we find that its two columns C] and C2 become identical.
Therefore, A becomes zero and thus a-bis a factor of A. Similarly b - c and c - are factors of A.
The product of principal diagonal terms is be2 which is a third degree expression. Therefore, A is
of third degree. Since a -b, b - c and c - a are factors of A.Therefore, (a - b) (b -c) (c - a) is a third
degree factor of A. Thus, there cannot be any other factor of A in terms ofa,b and c. The only other
factor of A can be a constant, say X.
1 1 1
a b c = X (a - b) (b -c) (c - a)
a2 b2 c2
In order to find the value of X, let us give values to a, b and c such that calculations are easy and
the two sides do not vanish.
Putting a = 0, fr =1, c =-l, we have

w
1 1 1
0 1 -1 = -(-!)) (-1-0) => 2 = 2X=>X = 1.
0 1 1
1 1 1
Flo
ee
Hence, a b c = (a -b)(b - c) (c - a).

Fr
a2 b2 c2
for
ur
EXAMPLE 70 Without expanding, show that
(a — x)2 (fl-y)2 (a-zj2
ks

A = (b-x)2 (b-y)2 (b-z)2 = 2{a-b)(b-c)(c-a)(x-y)(y-z)(z-x).


Yo
oo

(c - x)2 (c-y)2 (c-z)2


B

SOLUTION If we put a =b, we observe that two rows R\ and R2 of A become identical, therefore
re

A = 0. Thus, fl - ^ is a factor of A. Similarly it can be easily shown thatb - c,c-a,x-y,y-z,z-x


ou

are factors of A. Therefore, (a - b)(b -c){c -a)(x - y) (y - z) (z - x) is a factor of A.


ad
Y

The product of diagonal elements of A is (a-xy (b -y) (c-z) which is a sixth degree
expression. Therefore, A can have six linear factors. Thus there cannot be any other factor of A
nd
Re

except a constant X (say).


Fi

(a-x)2 (fl-y)2 (a-z)2


{b-x)2 (b-y)2 (b-z)2 = X(a-b) (b - c) (c - a) {x-y) (y-z) {z-x)
(c-x)2 (c-y)2 (c-z)2

In order to find the value of X, we give some values to a, b, c, x, y, z such that two sides do not
vanish together.
Putting a = 0,b = -l,c = l, x = 1, y = 0,z = -1, we obtain
1 0 1
4 1 0 = M0 + 1)(-1-1)(1-0)(1-0)(0 + l)(-l-1) => 8 = 4A. => X = 2.
0 14
(a-x)2 (a-y)2 {a-z)2
{b-x)2 {b-y)2 {b-z)2 =2{a-b) {b - c) {c - a) {x-y) {y-z) {z - x)
{c-x)2 (c-y)2 (c-z)2

ReadYourFlow.COM
6.56 MATHEMATICS-XII

-2a a + b a + c
EXAMPLE 71 Prove that: b + a -lb b + c =4:(a + b){b + c)(c + a)
c +a c + b -2c
-2a a + b a + c
SOLUTION Let A = b + a -2b b + c . Putting b = - a, we obtain
c + a c + b -2c
-2a 0 a+c
A = 0 2a c - a
c + a c - a -2c
2a c-a . .0 2a
=> A = -2a „ +{a + c) [Expanding along RJ
c -a -2c ' c + a c -a
=> A = - 2a {-Aac - (c - a)2} - (a + c) {2a (c + a)}

w
=> A = 2a {(c - a)2 + Aac) -2a{c + a)2 =2a(c + a)2 -2a(c + a)2 = 0

Flo
Therefore, by factor theorem a + bis a factor of A. Similarly, we can show that (b + c) and (c + a)
are factors of A. We find that A is a third degree homogeneous polynomial in a, b and c and

ree
(b + c) (c + a) (a + b) is also a third degree homogeneous polynomial in a, b and c. Hence, we must

F
have
A = k (a + b) (b + c) (c + a), where A: is a constant. or
ur
-2a a + b a + c
f
or. b + a -2b b + c = k (a + b) (b + c) (c + a)
ks

c + a c + b -2c
Yo
oo

Putting a = 0,b =1 and c = 2 in (i), we get


B

0 12
re

1 -2 3 = k(l)(3)(2) => 24 = 6/c => A: = 4


2 3-4
ou
ad

Hence, A = A(a+ b) (b + c) {c + a).


Y

All ter Let a + b = 2C, b + c = 2A and c + a = 2B. Then,


a + b + b + c + c + a = 2C + 2A + 2B
nd
Re

=> a +b + c=A + B + C
Fi

=> a = (A + B + C) -(b + c) =(A + B + C) - 2A = B + C - A.


Similarly, we obtain b =C + A - B and c =^4 + B - C.
-2a a + b a + c
A = b + a -2b b + c
c + a c + b -2c
2A-2B- 2C 2C 2B
=> A = 2C 2B-2C- 2A 2A
2B 2A 2C-2A-2B
A-B-C C B
=> A = 8 C B-C-A A [Taking 2 common from Ci,C2 and C3]
B A C-A-B
A-B C+B B
=> A =8 B-A B-C A [ApplyingC] -+Ci+C2 and C2 -> C2 + C3]
B+A C-B C-A-B

ReadYourFlow.COM
DETERMINANTS 6.57

A-BC + B B
A = 8 0 2B A +B [Applying R2 R-2 + R-i and R3 -> B3 + B2]
2B 0 C-B
2B A + B C+B B
A = 8 (A - B) + 2B [Expanding along CJ
0 C-B 2B A + B

=> A = 8 [[(A - B) 2B (C-B) + 2B {(C + B)(A + B) - 2B2}]

=> A = 16B [(A - B) (C - B) + (C + B) (A + B)- 2B2)


A = 16B (2AC + 2B2 - 2B2) = 16B (2AC) = 32 ABC
'b + cV c + flV a + b'
=> A = 32 = 4 (a + &)(& + c) (c + a).
2 l 2 J 2
V yv ^ /V ^ /

EXERCISE 6.2

w
LEVEL-1

1. Evaluate the following determinant:


1 3 5 67 19 21
Flo a h g

ee
(i) 2 6 10 (ii) 39 13 14 (iii) h b f

Fr
31 11 38 81 24 26 8 f c
for
ur
1-3 2 1 4 9 6-3 2
(iv) 4-12 (v) 4 9 16 (vi) 2-12
ks

3 5 2 9 16 25 -10 5 2
Yo
oo

1 3 9 27
102 18 36
eB

. ... 3 9 27 1
(vu) 9 27 1 3 (viii) 1 3 4 [CBSE 2012]
17 3 6
27 1 3 9
r
ou
ad

2. Without expanding, show that the value of each of the following determinants is zero:
Y

8 2 7 6-3 2 2 3 7
2-12 (iii) 13 17 5
nd

(i) 12 3 5 (ii)
Re

16 4 3 -10 5 2 15 20 12
Fi

\/a a be a + b 2a+ b 3a + b 1 a a2 - be
(iv) 1/b b2 ac (v) 2a + b 3a+ b 4a + b (vi) 1 b b2 -ac
1/c c2 ab 4a + b 5a+ b 6a+ b 1 c c2-ab

49 1 6 0 * y 1 43 6
(vii) 39 7 4 (viii) -x 0 z (ix) 7 35 4
26 2 3 -y -z 0 3 17 2

l2 22 32 42
ba c (2X + 2~x)2 (2X -2~x)2 1
22 32 42 52
(x) (xi) a + 2x b + 2y c + 2z (xii) (3X + 3~X)2 (3X-3~X)2 1
32 42 52 62 z
y (4X+4~X)2 (4X -4~x) 1
42 52 62 72

ReadYourFlow.COM
6.58 MATHEMATICS-Xtl

sin a cos a cos (a + 5) sin2 23° sin2 67° cosl80°


(xiii) sin p cos p cos (P + 8) (xiv) -sin2 67° -sin2 23° cos2180°
sin y cos y cos (y + 8) cosl80° sin2 23° sin2 67°
cos (x + y) - sin (x + y) cos 2y V23 + V3 V5 V5
(XV) sinx cosx siny (xvi) VTs + Vie s VTo
-cosx sinx -cosy s + VTIs VTs s
sin2 A cot A 1
(xvii) sin2B cotB 1 , where A, B, C are the angles of AABC.
sin2C cotC 1

Evaluate the following (3 - 9):


2
a b+c a 1 a be

w
3. b c + a b2 [CBSE2006] 4. 1 b ca [NCERT,CBSE 2006]
c a + b c2 1 c ab

x+X x x

Flo a b c

ee
5. x x+X x [NCERT] 6. c a b [CBSE 2004]

Fr
x x x+ X b c a

X 1 1 0 xy2 xz2
or
ur
7. 1 X 1 8. x2y 0 yz2
sf
[NCERT EXEMPLAR]
1 1 X x2z zy2 0
k
Yo
oo

a+x y z
B

9. x fl + y z [NCERT EXEMPLAR]
re

x y «+z
ou
ad

1 x x2 1 1 1
r\
Y

10. IfA= 1 y y , t±.-y= yz zx xy , then prove that A + A-! = 0. [NCERT EXEMPLAR]


1 z z2 x y z
nd
Re

Prove thefollowing identities (11 -45):


Fi

a b c
11. a-b b-c c-a = rt3 +ft3 + c3 - 3abc [CBSE 2009]
b+c c+a a+b
b + c a -b a
12. c + a b-c b = 3abc - a3 -b''’ - c3
a+b c-a c
a +b b + c c + a a b c
13. b + c c + a a + b = 2 b c a [CBSE 2001,2004,2006 C, 2007]
c+a a+b b +c cab
a + b + 2c a b
14. c b + c + 2a b = 2(a + b + c)3 [NCERT, CBSE 2006C, 2008,2014]
c a c + a + 2b

ReadYourFlow.COM
DETERMINANTS 6.59

a - b -c 2a 2a
15. 2b b -c -a 2b = (a + b + c)3 (CBSE 2f)00C, 04,07, NCERT EXEMPLAR]
2c 2c c-a-b
1 b + c b2 + c2
16. 1 c + a c2+ a2 =(a~b)(b-c)(c-a) [CBSE 2002]
1 a + b a2+b2
a a + b a + 2b
17. a + 2b a a + b = 9 (a + b) b2 [CBSE 2002,2013,2017]
a + b a + 2b a
1 a be l a a2
18. 1 b ca 1 b b2
1 c ab 1 c c2

low
z x y x y z x2 y2 z2
19. z2 x2 y2 = x2 y2 z2 x4 y4 z4 = xyz(x-y)(y-z)(z-x) (x + y + z).
24 4 4 4 4 4 x y z
* y x y z

ee
(b + c)2 a2 hc
rF
Fr
20. (c + a)2 b2 ca =(a-b)(b-c)(c-a)(a + b + c)(a2+b2 + c2)
(a + b)2 c2 ab
for
(a + l)(a + 2) a+ 2 1
ou
21. (a+ 2) (a+3) a+3 1 =-2
ks

(a + 3) (a + 4) fl + 4 1
oo
Y

«2 a2-(b-c)2 bc
eB

22. b2 b2-(c-a)2 ca =(a-b) (b - c) (c - a) (a+ b + c) (a2+b2 + c2) [CBSE 2012]


r

c2 c2-(a-b)2 ab
ou
ad

1 a2 +bc a3
Y

23. 1 b2 + ca b3 =-(a-b) (b-c) (c-a) (a2+b2 + c2) [CBSE 2008]


d
Re

1 c2 + ab c3
n
Fi

*2
ac + c 2
bc
24. a2 + ab b2
ac = 4a2b2c2 [NCERT, CBSE 2014,2015]
ab b2 + bc c2
x+4 x x
25. x x + 4 x = 16 (3x + 4) [NCERT EXEMPLAR]
x x x+4
1 1+p l+p+q
26. 2 3 + 2p 4 + 3p + 2q = 1 [NCERT]
3 6+ 3p 10 + 6p+ 3q
a b-c c-b
27. a-c b c - a = (a + b - c) (b + c - a) (c + a - b)
a-b b-a c

ReadYourFlow.COM
6.60 MATHEMATICS-XII

a2 lab b2
28. b2 a2 lab = (a3+b3)2
lab b2 a2

a2+1 ac
29. ab fc2 +1 be = l + a2 +b2 + c2 [NCERT, CBSE 2014]
ca cb c2 +1
1 a a2
30. a2 1 a =(a3-l)2 [NCERT, CBSE 2013,2014,2015]
a a2 1
a+b + c -c -b
31. -c a+b +c -a = l(a + b)(b + c)(c + a)

w
-b -a a+b + c
b+c a a
32. b
c
c+a
c
b = 4flfrc
a+b
Flo [NCERT, CBSE 2006 C]

ee
Fr
b2 + c2 ab ac
33. ba c 2 + a2 be = 4«2fc2 c2 or
ur
ca cb a2 + b2
sf

0 b2a c2a
k
Yo
oo

34. a2b 0 c2b = la3 b3 c3 [CBSE 2003]


B

a2c b2c 0
re

a2+b2
c c
ou
ad

c
Y

b2 + c2
35. a a = Aabc
a
nd
Re

2 2
b b
Fi

-be b2 +bc c2 +bc


36. a2 + ac -ac c 2 + ac = (ab + bc + ca)3
a2 + ab b2 + ab -ab

x + k lx lx
37. lx x + X lx = (5x + X)(X-x)2 [CBSE 2014]
lx lx x + X
x + 4 lx lx
38. lx x + A lx = (5x + 4) (4 - x)2 [CBSE 2007,2011]
lx lx x + 4
y+z z y
39. z z+x x =Axyz [NCERT EXEMPLAR]
y x x+y

ReadYourFlow.COM
DETERMINANTS 6.61

-a(b2 + c2 - a2) 2b3 2c3


40. 2*3 -b(c2 + a2-b2) 2c3 = abc {a2 +b2 + c2)3
2a3 2b3 -c(a2 +b2 - c2)
1 +a 1 1
41. 1 1 + fl 1 =a3 + 3a2
1 1 \+a
2y y-z-x 2y
42. 2z 2z z-x-y =(x + y + 2)3 [CBSE2014]
x-y-z 2x 2x
y+z ^ y
43. z + x z x =(x + y+z) (x-z)z. [CBSE 2007]
x+y y z

w
a+x y 2
44. x a + y z =a (a +x + y+z) [CBSE 2014]
X y «+z
a3 2 a

Flo
ee
45. b3 2 b = 2{a-b) (b-c) (c-a) (a + b + c) [CBSE 2015]

Fr
c3 2 c
abc x y z y b q
for
ur
46. Without expanding, prove that x y z = p q r x a p
p q r abc z c r
ks
Yo

x+1 x+2 x+a


oo

47. Show that x + 2 x+ 3x + b = 0 wherefr, care in A.P. [CBSE 2005]


B

x+ 3 x+4 x+c
re

x- 3 x-4 x-a
48. Show that x - 2 x-3 x - (3 = 0, where a, P, y are in AP. [CBSE 2007]
ou
ad

x-1 x-2 x -y
Y

b + c c + a a +b
49. If fc, c are real numbers such that c + a a + b b + c = 0, then show that either
d
Re

a + b b + c c +a
n
Fi

a + b+ c = 0or, a=b=c. [NCERT]


P b c
50. If a q c = 0, find the value of — — ,p * a, q=b,r * c. [CBSE 2014]
a b r p-a q-b r-c

x -6 -1
51. Show that x = 2 is a root of the equation 2 -3x x-3 =0 and solve it completely.
-3 2x x + 2

52. Solve the following determinant equations:


x+a b c x+a x x
(i) a x+b c =0 (ii) X x+a x = 0, a * 0
a b x+c x x x+a

[CBSE 2003] [NCERT, CBSE 2011]

ReadYourFlow.COM
6.64 MATHEMATICS-XII

a+c c a+c
=> A = 2abc a + b b a
b+c b+c c
a + c -a 0
=> A = 2abc a + b -a -b [Applying Ci —> C-^ -C2 ,C3 ->C3 -C^]
b + c 0 -b
c -a 0
=> A = 2abc 0 -a -b [Applying C1^C1+C2+C3]
c 0 -b
1 -1 0
=> A =2abc x abc 0 -1 -1 [Taking c, a&cb common from Cj, C2 & C3 respectively]
1 0 -1

w
1 -1 0
=> A = 2a2 b2 c2 0 -1 -1 [Applying R3 -> R3 - ^il
0 1 -1
=> A = 4 a2 b2 c2
Flo [Expanding along Ci ]

ee
1 1+p l+p+q

Fr
26. Let A = 2 3 + 2p 4+3p + 2q . Applying C2C2 - pC^ and C3-+C3 - q C-^, we get
3 6 + 3p 10 + 6p + 3q
for
ur
11 1+p 1 1 1
A = 2 3 4+ 3p 2 3 4 [Applying C3 ->C3 -pC2]
ks
Yo

3 6 10 + 6p 3 6 10
oo

1 00
B

=> A = 2 12 [Applying C2 ~J>C2 -C1,C3 ->C3 -C^


re

3 37
ou
ad

1 2
=> A = 1 = 7-6 = 1 [On expanding along R]]
Y

3 7

a2+1 “b ac
nd
Re

29. Let A = ab b2 +1 be . Applying R^ -+ R^ (a), R2 -+ R2(b) &c R3-> R3 (c)r we get


Fi

ca cb c2 +1
a (a2 + 1) a2b a2 c
1 ab2 b(b2+l) b2c
A=
abc
c2 a c2 b c (c2 +1)

a2 +1 a2 a2
abc b2 b2+l b2
A=— [Taking a,b, c common from C2 & C3 respectively]
abc
c2 c2 c2+l

a2+b2+c2 + l a2+b2+c2+l a2+b2 + c2 + l


b2 b2+l b2 Applying
=> A=
1^1 —> R] + R2 + R3
c2 c2 c2+l

ReadYourFlow.COM
DETERMINANTS 6.65

1 1 1
=> A=(a2 +b2 + c2 +1) b2 b2 +1 b2 [Taking {a2 +b2 + c2 +1) common from R^]
c2 c2 c2+l

10 0
=> A = (a2 +b2 + c2 +1) b2 1 0 [ApplyingC2C2 -C1/C3 ->C3 -Cj]
c2 0 1

=> A-(a2 +b2 + c2 +1) [Expanding along


l a a2
30. Let A - a2 1 a . Applying Cj-» + C2 + C3 , we get
a a2 1

1 + a + a2 a a2

w
A = 1 + a + a2 1 a
1 + a + a2 a2 1

Flo 1 a a2

ee
=> A = (1 + a + a2) 1 1 a [Taking 1 + a + a common from C^]

Fr
1 a2 1
for
ur
1 a
A = (1 + a + a2) 0 1 - fl - a2 [Applying R2 —> R2 — Ri/ R3 —^ ^3 —
ks

0 a2 - a 1-a2
Yo
oo

1 «2
B

=> A = (1 + a + fl2) 0 1-a a(l-a)


re

0 - a (1 - a) (1 - rt) (1 + fl)
ou
ad

1 a a2
Taking (1 - a) common from
Y

=> A = (1 + a + «2) (1 - a)2 0 1 a


R2 andR3 respectively
0 -a 1+a
nd
Re

=> A = (l+a + a2)(l-a)2 (l + a + a2) [Expanding along Cj]


Fi

=> A = {(1+fl + fl2)|2 = (fl3-l)2

b+c a a
32. Let A = b c+a b . Applying Rj-» + R2 + R3 , we get
c c a+b
2(b + c) 2(a + c) 2 (a + b)
A = b c+a b
c c a+b
b+c c+a a+b
=> A = 2 b c+a b [Taking 2 common from R1]
c c a+b
b + c c + a a+b
^ A = 2 -c 0 -a [Applying R2 R2 - Rj, R3 -> R3 - RJ
b -a 0

ReadYourFlow.COM
6.66 MATHEMATICS-XII

0 c b
=> A = 2 -c 0 -a [Applying R1 + R2 + R3]
b -a 0

=> A = o 0 -a -c ~a i
-C 0
-a 0 C -b 0„ +b -b -a = 2(0 + abc + abc) = Aabc

b+c c+a a+b


49. We have, c + a a + b b + c = 0
a+b b + c c +a
2(a + b + c) c + a a + b
=> 2(a + b + c) a + b b + c = 0 [Applying Cj +C2 +C3]
2(a + b + c) b + c c + a
1 c + a a+b

w
=^> 2(a + b + c) 1 a + b b + c =0
1 b+c c+a

=>
1 c+a a+b
2(a + b + c) 0 b - c c - a = 0

Flo Applying R2 R2 - R3 -> R3 - R\

ee
0 b-a c-b

Fr
=> 2(a + b + c) {(b -c) (c -b) -(c -a) (b - a)) = 0 [Expanding along Cj]
2(a + b + c) \-(b2-2bc + c2)-(be-ca-ab + a2)} = 0 or
ur
=> 2 (a + b + c) (- a2 - b2 - c2 + be + ca + ab) = 0
f
ks

=> 2(a + b + c) (a2 + b2 + c2 - ab - be - ca) =0


Yo
oo

=> (a + b + c) (2a2 + 2b2 + 2c2 - 2ab -2bc-2 ca) = 0


B

=> (a + b + c) {(a-b)2 +(b - c)2 +(c- a)2} = 0


re

a + b + c = 0 or, (a-b)2 + (b-c)2 + (c - a)2 = 0


ou
ad

=> a + b + c = 0 or, a=b=c


50. We have.
Y

p b c
d
Re

a q c = 0
n

a b r
Fi

Applying Ri -+ R1-R3, R2 -> R2-R3,-we obtain


p-a 0 c-r
0 q-b c-r =0
a b r
q-b c-r 0 c-r 0 q-b
=> (P-a) \b -0 + (c~r) \ =0
r a r a b
=> (p-a) {r(q-b)-b(c-r)}-a(c-r)(q-b)=0
(p - a) (q-r) r + (p - q) (r - c) b + a (q-b) (r - c) =0
r b
=>
r-c q-b p-a
a-=o [Dividing by (p-a) (q-b) (r-c)]

/
r b7 ci ^
=> -----+ ------+ 1 + ------ + 1 = 1 + 1 [Adding 2 on both sides]
r-c [q-b J [p-a

ReadYourFlow.COM
DETERMINANTS 6.67

r
=> —=2
r-c q-b p-a
x+ a x x
52. (ii) We have, x x+a x = 0, a * 0
x x x + fl
3x + a x x
=> 3x + a x + a x = 0 [Applying C-! ^ Ca + C2 + C3]
3x + a x x+a
1 x X
=> (3x + <?) 1 x + a x = 0 [Taking (3x + a) common from
1 x x+a
1 x x
=> (3x + a) 0 a Q = 0 [Applying R2 R2 _ ' -^3 ^3 _

w
0 0a
=> (3x + a) a2 =0
=> 3x + fl = 0 => x = -a/3

Flo [•.• a^O]

ree
6.6 APPLICATIONS OF DETERMINANTS TO COORDINATE GEOMETRY

F
6.6.1 AREA OF A TRIANGLE
or
We know that the area of triangle whose vertices are (x^, yi),(x2 , y2)and(x3/ y3) is given by the
ur
expression:
sf
a = | j^i (y2 -y3) + ^2 (ys -yi)+ (yi -y2)J
k
Yo
oo

x\ y\ i Vl 1 yi 1 yi 1
B

Also, X2 y2 1 =xt — x2 + *3 [Expanding along CJ


ys 1 V3 1 y2 1
re

^3 ys 1
=(y2 - ys) - x2 (yi - ys)+ ^3 (yi - y2)
ou
ad

=xi (yi - ys) + ^2 (ys - yi) + ^3 (yi - y2) ...(h)


Y

From (i) and(ii), we obtain


a *1 yi 1
nd
Re

A = - ^2 Vl 1
Fi

x3 ys 1
Thus, the area of a triangle having vertices at (xlr y{), (x2, y2) and (x3, y3) is the absolute value
of A given by
! *1 yi 1
A = - *2 y2 1
x3 V3 1
NOTE Since area is always a positive quantity, therefore we always take the absolute value of the
determinant for the area.
6.6.2 CONDITION OF COLLINEARITY OF THREE POINTS
Let A (x-j, y-)), B (x2, y2) and C (x3, y3) be three points. Then,

1 xi yi 1 *1 yi 1
A, B, C are collinear <=> Area of triangle ABC = 0 <^> — x2 y2 1=0<=>x2 y2 1=0
2 ^3 ys 1 ^3 ys 1

ReadYourFlow.COM
6.68 MATHEMATICS-XII

6.6.3 EQUATION OF A LINE PASSING THROUGH TWO GIVEN POINTS


Let the two point be/l (xj, y{) and B (x2,1/2). LetP (x, y) be any point on the line joining A and B.
Then, points P, A and B are collinear.
* y 1
x-i yj 1=0
x2 yi 1
x y 1
Thus, the equation of the line joining points (x^, yj) and (x2/y2) is given by xq 1 =0
x2 V2 1

ILLUSTRATIVE EXAMPLES
LEVEL-1

EXAMPLE 1 Bind the area of the triangle with vertices A (5, 4), B (- 2, 4) and C (2, - 6).

w
SOLUTION The area A of triangle ABC is given by
„ 5 4 1
A = — -2
2
4 1
2 -6 1
Flo
ee
Fr
5 4 1
1
=> A = - -7 0 0 [Applying ^ -^2 “ anc* ^3 -> ^3 - ^i]
2 -3 -10 0
for
ur
1 -7 0
=> A = —x1x [Expanding along C3]
ks

2 -3 -10
Yo
oo

1
=> A = - (70 - 0) = 35 sq. units.
B

EXAMPLE 2 Show that the points (a, b + c), (b, c + a) and (c, a + b) are collinear.
re

SOLUTION We have,
ou
ad

a b+c 1 a a+b + c 1
A = b c+a 1 b b+c+a 1
Y

[Applying C2 ->C2 TCi]


c a+b 1 c c+a+b 1
nd
Re

a 1 1
=> A = (a + b + c) b 1 1 [Taking (a + b + c) common from C2]
Fi

c 1 1

=> A = (a + b + c) x 0 = 0. [v C2 and C3 are identical]


Hence, the given points are collinear.
EXAMPLE 3 If the points (a1, b^), (a2, ^2) and («i + «2' ^1 + ^2)are collinear, show that a^ b2 = a2 b-y.
SOLUTION If given points are collinear, then
h 1
a2 b2 1 = 0
ay + a2 + b'y 1

"i l
=> a2 — ay b2 — by 0 — 0 [Applying R2 —> R2 —Ry, R^ —> R3 — R^]
a2 b2 0
a2 - ay b2 -by = Q
[Expanding along C3]
a2 b2

ReadYourFlow.COM
DETERMINANTS 6.69

-<?i -b1
=> = 0 [Applying R-l^Rl- R2]
a2 b2
=> -fl1 b2 + ^2 = 0
=> flj = a2 ^1

EXAMPLE 4 If the points (2, - 3), (X, -1) and (0, 4) are collinear, find the value ofX
SOLUTION If given points are collinear, then
2 -3 1
X -1 1 = 0
0 4 1
2 -3 1
=> X-2 2 0 = 0 [Applying R2 R2 ~ and —» P3 - Rj]
-2 7 0

w
X-2 2
= 0 [Expanding along C3]
-2 7
=> 7X-14 + 4 = 0=> X = 10/7.

Flo
ree
EXAMPLE 5 Using determinants, find the area of the triangle whose vertices are (- 2, 4), (2, - 6) and
(5, 4). Are the given points collinear?

F
SOLUTION LetA be the area of the triangle. Then,
2 4 1 or
ur
1
A = - 2 -6 1
sf
2 5 4 1
k
Yo

-2 4 1
oo

A = - 4 -10 0 [Applying R2 R2 - R-^ and R3 ^ R3 - R-[]


B

2 7 0 0
re

1 4 -10
=> A = [By expanding along Cj]
2 7 0
ou
ad

A = ^(70) = 35 sq. units.


Y

=>
nd
Re

Clearly, A * 0, therefore given points are not collinear.


EXAMPLE 6 Find the equation of the line joining A (1, 3) and B (0, 0) using determinants and find k if
Fi

D (k, 0) is a point such that area of A ABD is 3 sq. units. [CBSE2013]


SOLUTION Let P {x, y) be any point on line AB. Then,
Area of A ABP = 0
13 1
-001 = 0
1 x y l
^ {1 (0-y) - 3(0-x) +1(0-0)} = 0

=> 3x - y = 0, which is the required equation of AB.


Now, Area of A ABD = 3 sq. units
1 1 3 1
=> 1 0 0 1 = ±3
2 0 1

ReadYourFlow.COM
6.70 MATHEMATICS-XII

1 3 1
=> 0 0 1 = ±6
k 0 1
=> 1 (0 - 0) - 3 (0 - fc) + 1 (0 - 0) = ± 6 => 3 A: = ± 6 => k = ±2

LEVEL-2
example 7 If A (xi, yf), B (x2, y2) and C (x3, yf) are vertices of an equilateral triangle whose each
*i yi 2 2
side is equal to a, then prove that x2 y2 2 = 3a4. [NCERT EXEMPLAR]
x3 V3 2
SOLUTION Let A be the area of triangle ABC. Then,
x *1 yi 1

w
A = 2 ^2 ^2 1
x3 V3 1

=>
*1 yi 1
2 A = x2 y2 1
Flo
ee
Fr
^3 ys 1
Xi yi 1 Xj yi 2
=> 4 A = 2 x2 y2 1 = x2 y2 2
for
ur
^3 ys 1 *3 ys 2
ks

9 yi 22
Yo

=> 16 A = x2 y2 2
oo

...(i)
x3 ys 2
B

V3 2
But, the area of an equilateral triangle with each side equal to a is a .
re

A = — a2 => 16 A2 = 3a4 ...(ii)


ou
ad

4
Y

*1 yi 2 2
From (i) and (ii), we obtain x2 y2 2 = 3a4.
nd
Re

x3 ys 2
Fi

EXAMPLE 8 A triangle has its three sides equal to a, b and c. If the coordinates of its vertices are
A (x1, y{), B (x2, y2) and C (x3, y3), show that
xi y\ 22
x2 y2 2 ={a + b + c) (b + c - a) {c + a -b) (a + b - c).
x3 V3 2
SOLUTION Let A be the area of triangle ABC. Then,
\ xt yi i
A = 2 *2 y2 1
x3 V3 1
*1 yi 1
2 A = x2 y2 1
x3 V3 1

ReadYourFlow.COM
DETERMINANTS 6.71

*1 yi 1
=> 4A = 2 *2 y2 1
*3 ys 1
*i yi 2
=> 4 A = x2 y2 2
x3 ys 2
02
9 *1 yi 2
16 Az = x2 t/2 2
^3 ys 2
We also know that the area of triangle ABC is given by
1
A = tJs (s - a) (s - b) (s - c), where s = -(a + b + c)
1 1
But, s = — (a + b + c) => s - a ~(a + b + c)- a = -(b + c - a).

low
2 2
1 ^{a + b-c).
Similarly, s -b = — (c + a -b) and s - c
1111
A2 = — (a + b + c)x-(b + c-a)x-(c + a-b)x-(a + b-c)

ee
rF
Fr
=> 16 £ = (a + b + c) (b + c - a) (c + a - b) (a + b + c) ...(h)
From (i) and (ii), we get
for
*1 yi 22
u
x2 Vl 2 =(ci + b + c) (b + c - a) (c + a - b) (a + b - c)
ks
Yo

x3 V3 2
oo
B

EXERCISE 6.3
re

LEVEL-1
ou
ad

1. Find the area of the triangle with vertices at the points:


Y

(i) (3, 8), (-4, 2) and (5,-1) (ii) (2, 7), (1,1) and (10, 8)
nd

(hi) (-1,-8), (-2,-3) and (3, 2) (iv) (0,0), (6,0) and (4, 3).
Re

2. Using determinants show that the following points are collinear:


Fi

(i) (5, 5), (-5,1) and (10, 7) (ii) (1,-1), (2,1) and (4,5)
(hi) (3,-2), (8, 8) and (5, 2) (iv) (2, 3), (-1,-2) and (5, 8)
3. If the points {a, 0), (0, b) and (1,1) are collinear, prove that a + b = ab.
4. Using determinants prove that the points (a, b), (a', b') and {a - a', b -b') are collinear if
ab' =a'b.
5- Find the value of A, so that the points (1, -5), (-4,5) and (A., 7) are collinear.
6. Find the value of x if the area of A is 35 square cms with vertices (x, 4), (2, - 6) and (5, 4).
7. Using determinants, find the area of the triangle whose vertices are (1,4), (2, 3) and
(-5, - 3). Are the given points collinear?
8- Using determinants, find the area of the triangle with vertices (- 3,5), (3,-6) and (7, 2).
9. Using determinants, find the value of k so that the points (k,2-2 k), (-k +1, 2k) and
(-4 -k, 6 - 2k) maybe collinear.
10. If the points (x, - 2), (5, 2) and (8, 8) are collinear, find x using determinants.

ReadYourFlow.COM
6.72 MATHEMATICS-XII

11. If the points (3, - 2), (x, 2) and (8,8) are collinear, find x using determinant.
12. Using determinants, find the equation of the line joining the points
(i) (1, 2) and (3, 6) (ii) (3,1) and (9, 3)
13. Find values ofif area of triangle is 4 square units whose vertices are
(i) (k, 0), (4, 0) and (0, 2) (ii) (-2,0), (0,4) and (0, k)
ANSWERS
75 47
1. (i) — sq. units (ii) — sq. units (iii) 15 sq. units (iv) 9 sq. units
13
5. k = -5 6. x = - 2,12 7. — sq. units. No 8. 46 sq. units
9. k =-l, 1/2 10. x = 3 11. x = 5
12. (i) y = 2x (ii) x = 3y 13. (i) k =0,8 (ii) 0,8

w
6.7 APPLICATIONS OF DETERMINANTS IN SOLVING A SYSTEM OF LINEAR
EQUATIONS

Flo
Consider a system of simultaneous linear equations given by

ee
fl-j x + y + c1 z =

Fr
a2 x + b2]/ + c2z = d2 ■ ...(i)
a3x + b3i/ + c3z = d3
or
ur
A set of values of the variables x, y, z which simultaneously satisfy these three equations is called
f
a solution set.
ks

For example, x = 3, y = 4 and z = 6 is the solution of the system of equations


Yo
oo

5x - 6y + 4z = 15
B

7x + 4 y - 3z = 19
re

2x + y + 6z = 46
ou
ad

A system of linear equations may have a unique solution, or many solutions, or no solution at
all. If it has a solution (whether unique or not) the system is said to be consistent. If it has no
Y

solution, it is called an inconsistent system.


d
Re

If d^ = d2 = d3 = 0 in (i), then the system of equations is said to be a homogeneous system.


n

Otherwise it is called a non-homogeneous system of equations.


Fi

6.7.1 SOLUTION OF A NON-HOMOGENEOUS SYSTEM OF LINEAR EQUATIONS


We now intend to solve a system of simultaneous linear equations by Cramer's rule named after
the Swiss mathematician Gabriel Cramer.
THEOREM 1 (Cramer's rule) The solution of the system of simultaneous linear equations
alx + b-ly = q -(i)
a2 x + b2 y = c2 •••(ii)
is given by x = ^, y = d2 , where D = al b\ f) _ cl b\ _ q q
and D-, - provided that
«2 V 1 c2 h a2 c2
D * 0.

PROOF We have, D = al h
a2 b2

ReadYourFlow.COM
DETERMINANTS 6.73

r bl fl-j x bi
xD =
a2 h a2 x b2

XD = alx + b'V [Applying C: ->CT +yC2]


=>
a2 x + b2 y b2

C| b-
=> xD = 1 =D1 [Using (i) and (ii)]
c2 b2
Similarly, we obtain
al C1
yo = a2 c2 = d2

x = and 1/ = , provided that D ^ 0. Q.E.D.

w
REMARK Here D = is the determinant of the coefficient matrix al b\
t2 a2 b2

Flo
The determinant D1 is obtained by replacing first column in D by the column on the right hand side of the

ee
given equations.

Fr
The determinant D2 is obtained by replacing the second column in D by the right most column in the
given system of equations. for
ur
THEOREM 2 (Cramer's Rule) The solution of the system of linear equations
ks

a-i x + b1y + CiZ = d1


Yo
oo

a2 x + b2y + c2z = d2 •••(ii)


a3 x + b3y + c3z = d3 ••.(iii)
eB

Dp D2 ^3
where
is given by x = y = — and z =
D'
r

D' D
ou
ad

ax bx cx d\ b\ q al dx cx a^ bi d1
Y

D = a2 b2 c2 , Dx — d2 b2 c2 , D2 = a2 d2 c2 and D = a2 b2 d2 ,
a3 b3 c3 d3 b3 c3 a3 d3 c3 a3 b3 ^3
nd
Re

provided that D * 0.
Fi

PROOF We have.
ax bx cx
D = a2 b2 c2 .
a3 b3 c3
ax bx cx ax x bx cx
xD = x a2 b2 c2 = a2x b2 c2
a3 b3 c3 a3x b3 c3
ax x + bx y + cxz bx cx
=> xD = a2 x + b2y + c2z b2 c2 [Applying Cj -> C! + y C2 + z C3]
a3 x + b3 y + c3z b3 c3
dx bx cx
xD = d2 b2 c2 = Dx [Using (i), (ii) and (iii)]
d3 b3 c3

ReadYourFlow.COM
6.74 MATHEMATICS-XII

Similarly, we obtain
t?! q a1 d]
i/D = n2 d2 c2 -D2 and zD = a2 b2 d2 = D3.
rt3 d3 c3 l73 d3
D-j D3 Do
x = —y = — and z = provided that D * 0. Q.E.D.
D J D
REMARK Here D is the determinant of the coefficient matrix. The determinant is obtained by
replacing the elements in first column ofD by d^, d2/ d3. D2 is obtained by replacing the elements in the
second column of D by rf-j, d2, d3 and to obtain D3, replace elements in the third column of D by
dl, d2, d3.
The above method of solving a system of three linear equations in three unknowns can be used
exactly the same way to solve a system of n equations in n unknowns as stated below.

w
THEOREM 3 (Cramer's Rule) Let there be a system ofn simultaneous linear equations n unknowns
given by
aU xl + a12 x2 + ■■■ +

Flo
(?2i x1 + a22 x2 +... + a2n x}1 =b2
Xn - b-i

ee
Fr
^hil xl + ^;2 x2 + ••• + ann Xn
aU a12 abi for
ur
Let D = a21 a22 ■■■ a2n and let Dj be the determinant obtainedfrom D after replacing the
ks

an 1 an2 ■■■ ann


Yo

bl
oo

Then, xi=^t D2
B

)th column by x2 = — , ■■•/ Xjl — provided that D * 0.


~ D
re

K
ou
ad

6.7.2 CONDITIONS FOR CONSISTENCY


Y

CASE 1 Tor a system of 2 simultaneous linear equations with 2 unknowns


(i) If D ^ 0, then the given system of equations is consistent and has a unique solution given
nd
Re

, D-j D2
^ D * D
Fi

(ii) If D = 0 and Dj = D2 = 0, then the system is consistent and has infinitely many solutions.
(iii) If D = 0 and one of and D2 is non-zero, then the system is inconsistent.
CASE II Tor a system of 3 simultaneous linear equations in three unknowns
(i) If D 0, then the given system of equations is consistent and has a unique solution given
u ^2 j D3
y D ^ D D
(ii) If D = 0 and Di=D2=D3 = 0, then the given system of equations may or may not be
consistent. However, if it is consistent, then it has infinitely many solutions.
(iii) If D = 0 and at least one of the determinants Dj, D2, D3 is non-zero, then the given system
of equations is inconsistent.
In order to solve a non-homogeneous system of simultaneous linear equations by Cramer's rule,
we may use the following algorithm.
ALGORITHM
STEP I Obtain D, Dj, D2andD3.

ReadYourFlow.COM
DETERMINANTS 6.75

step IIFind the value ofD.


IfD^O, then the system of equations is consistent and has a unique solution. To find the
solution, obtain the values ofDi, D2 and D3. The solution is given by
D2 , D3
x = Dl D ' lJ —D and z = —D
If D = 0, go to step III
STEP III Find the values of 0^,02, D^,-
If at least one of these determinants is non-zero, then the system is inconsistent.
If Dj = D2 = D3 = 0, then go to step IV.
STEP IV Take any two equations out of three given equations and shift one of the variables, say z, on the
right hand side to obtain two equations in x, y. Solve these tzvo equations by Cramer's ride to
obtain x, y in terms ofz. If these values of x and y satisfy the third equation, then the system is
consistent and the values ofx, y and z constitute a solution.

low
If the values ofx and y do not satisfy the third equation, then the system is inconsistent.
ILLUSTRATIVE EXAMPLES

LEVEL-1

ee
rF
EXAMPLE 1 Solve the following system of equations by Cramer's rule

Fr
2x-y = 17
3x +5y = 6
for
SOLUTION For the given system, we have
ou
D = l -15 = 2 x 5 - (-1) x 3 = 13*0
ks

3
oo

17 -1: =85 +6=91 and D2= ^ ^ =12-51 =-39.


Di =
Y
B

O D JO
re

So, by Cramer's rule, we obtain


= Di = 91 „ Do -39
ou

13 = 7 and *y = —^ = = -3.
ad

X D D 13
Y

Hence, x = 7 and y = - 3 is the required solution.


nd

EXAMPLE 2 Solve the following system of equations using Cramer's rule:


Re

5x-7y + z=ll, 6x - 8y - z =15 and 3x + 2y-6z=7.


Fi

SOLUTION The given system of equations is


5x -7y + z = 11
6x-8y -z = 15
3x + 2y - 6z =7
5 -7 1
D = 6 -8 -1 = 5 (48+ 2)+7 (-36+ 3)+ 1 (12+ 24)= 250-231 + 36=55*0
3 2 -6
11 -7 1
Di = 15 -8 -1 = 11 (48+ 2)+7 (-90+ 7)+ 1(30+ 56)= 550-581 + 86 = 55
7 2-6
5 11 1
D2 = 6 15 -1 = 5(-90+7)-ll(-36+3) + l(42-45) = -415+ 363-3 = -55
3 7-6

ReadYourFlow.COM
6.76 MATHEMATICS-XII

5 -7 11
and D3 = 6 -8 15 =5 (-56-30)+7(42-45)+ 11 (12+ 24) = -430-21 + 396 = -55
3 2 7
So, by Cramer's rule, we obtain
D1 55 , D2 =_55 , D3 55
x = — =— =1, V D -1 and 2 = —= - — = -l.
D 55 55 D 55
Hence, x = 1, y = -1 and 2 = -1 is the solution of the given system of equations.
EXAMPLE 3 Solve the system of equations x + 2y = 3 ami 4x + 8y = 12 by using determinants.
SOLUTION For the given system of equations, we have
1 2 3 2 1 3
D= = 0, D1 = = 0 and D2 = = 0.
4 8 12 8 4 12
Thus, D = Dj = D2 = 0

w
So, the given system has infinite number of solutions. Let y =k. Then,
x + 2y = 3 => x = 3 — 2k.

Flo
Hence, x = 3 - 2k, y = k is the solution of the given system of equations, where k is an arbitrary
real number.

ee
EXAMPLE 4 Shozv that the following system of equations is inconsistent:

Fr
2x + y = 3, 4x + 2y = 5.
SOLUTION For the given system of equations, we have for
ur
2 1 3 1
D= = 0 and D} = = 1*0.
4 2 5 2
ks

Thus, we have D = 0 and D1 * 0. So, the given system is inconsistent.


Yo
oo

EXAMPLE 5 By using determinants, solve the following system of equations:


x + y +z = 1
B

x + 2y + 32 = 4
re

x + 3y + 52 =7
ou
ad

SOLUTION For the given system of equations, we have


Y

1 1 1
D = 1 2 3 = 1 x (10 - 9) -1 x (5 - 3) + 1 x (3 - 2) = 0,
nd
Re

1 3 5
Fi

1 1 1
Dl 4 2 3 = 1 x (10 - 9) -1 x (20 - 21) + 1 x (12 -14) = 0,
7 3 5
1 1 1
D2 = 1 4 3 = 1 x (20 - 21) -1 x (5 - 3) + 1 x (7 - 4) = 0,
1 7 5
1 1 1
and. D3 = 1 2 4 = 1 x (14 -12) -1 x (7 - 4) + 1 x (3 - 2) = 0.
1 3 7
Thus, we have D = D-] = D2 = D3 =0.
So, either the system is consistent with infinitely many solutions or it is inconsistent.
Consider the first two equations, these equations can be written as
x + y = 1-2
x + 2y = 4-32

ReadYourFlow.COM
DETERMINANTS 6.77

In order to solve these equations let us use Cramer's rule.


1 1 1 -2 1
Here, D = = 2-1 =1, Di = = 2 - 22 - 4 + 32 = 2 - 2
1 2 4-32 2
1 1-2
and, D2 = = 4 - 32-1+2 = 3 - 22.
1 4-32
Di , D2
x = — and t/ = —^
D J D
=> x=2-2, y = 3-22.
Let 2 = fc, where /c is any real number. Then, we get
x = k - 2, y = 3 - 2k and z = k
These values satisfy the third equation.
Hence, x = k-2, y = 3-2k,z = A: is a solution of the given system of equation for every value

low
of k.
EXAMPLE 6 Using determinants, show that the folio-wing system of linear equation is inconsistent:
x-3y + 5z = 4
2x -6y + lOz = 11

ee
3x-9y + 152 = 12
rF
Fr
SOLUTION For the given system of equations, we have
1-3 5
for
D = 2 -6 10 = 0 [v C2 is proportional to C-J
ou
3 -9 15
ks

4-3 5 4 1 1
oo

D1 = 11 -6 10 =-15 11 2 2=0 [v C2 and C3 are identical]


Y
eB

12 -9 15 12 3 3
1 4 5 1 4 1
r

D2 = 2 11 10 =5 2 11 2=0 [v C-j and C3 are identical]


ou
ad

3 12 15 3 12 3
Y

1-3 4 1 1 4
and. D3 = 2 -6 11 =-3 2 2 11 =0 [y Ci and C2 are identical]
d
Re
n

3 -9 12 3 3 12
Fi

D = = D2 = D3 = 0.
So, the given system of equations may or may not be consistent.
If we now put 2 = kin any two of three equations, we find that the two equations obtained are
inconsistent as they represent a pair of parallel lines. Hence, the given system of equations is
inconsistent.
REMARK Ifwe examine the given system ofequations closely, wefind that the three equations represent
parallel planes. So, they have no point in common. Consequently the given system has no solution.
EXAMPLE 7 Using Cramer's rule, solve thefollowing system of linear equations:
(a + b) x - (a - b) y = Aab
{a -b) x + (a + b) y = 2(a2 -b2)
SOLUTION For the given system of equations, we have
a + b -(a-b) = (a + b)2 +{a-b)2 = 2(a2 +b2) * 0
D =
a-b a+b

ReadYourFlow.COM
6.78 MATHEMATICS-XII

So, the given system of equations has a unique solution.


Aab -(a-b)
Now, D} =
2 (a2 -b2) (a + b)
lab -(a-b)
=> = 2 (a + b) [Taking 2 common from C1 and (a + b) from R2]
a -b 1
=> D1 = 2(a + b){2ab + (a-b)2} = 2 (a + b) (a2 + b2)
a+b
and, D2 =
a-b 2(a2-b2)
a+b lab
=> D2 = 2(a-b) [Taking (a -b) common from R2 and 2 from C2]
1 (a + b)

=> D2 = 2(a-b)((a + b)2 - lab} = 2 (a -b) (a2 + b2)

low
By Cramer's rule, we obtain
Di _ 2 (a + b) (a2+b2) 2 (a-b) (a2 + b2)
x -a + b and, y = = a-b.
D 2(a2 +b2) 2 (a2 + b2)

ee
rF
Hence, x = a + b,y = a-b is the solution of the given system of equations.

Fr
EXAMPLE 8 Using determinants, shoiu that the follozuing system of equations is inconsistent:
for
2x - y + z= 4, x + 3i/ + 2z =12, 3x + 2y + 3z =10.
SOLUTION The given system of equations is
u
ks

2x - y + z = 4
Yo

x + 3y + 2z = 12
oo

3.r + 2y + 3z = 10.
B

2 -1 1
re

Here, D = 1 3 2 = 2 (9 - 4) + 1 (3 - 6) +1 (2 - 9) = 0
ou

3 2 3
ad
Y

4-11
and, 12 3 2 =4(9-4)+ 1(36-20)+ (24-30) = 30*0.
nd
Re

10 2 3
Fi

Hence, the given system of equations is inconsistent.


EXAMPLE 9 Solve the following system of equations by using determinants:
x+y+z = 1
ax + by + cz = k
a2 x + b2 y + c2 z = k2
SOLUTION For the given system of equations, we have
1 1 1
D = a b c
a2 b2 c2

1 0 0
D = a b -a c-a [ApplyingC^ ->C2 -Cj and C3 -C J
an2 bh2 -a 2 c 2 —a 2

ReadYourFlow.COM
DETERMINANTS 6.79

1 0 0
Taking (b - a) and (c - a) common
=> D =(b - a) (c - a) a 1 1
fromC2 andC3 respectively
b +a c+a
1 1
=> D =(b -a)(c -a)xlx [Expanding along R^]
b+a c+a
=> D =(b - a) (c - a) (c + a-b - a) ={b -c)(c - a) (a -b) -(i)
1 1 1
Dl = k b c =(b-c)(c-k)(k-b) [Replacing a by k in (i)]
k2 b2 c2

1 1 1
D2 = a k c = (k -c)(c - a) {a - k) [Replacing b by k in (i)]

low
a2 k2 c2
1 1 1
and. D3 = a b k =(a-b) (b -k){k - a) [Replacing cbyk in (i)]

ee
a2 b2 k2
rF
Fr
d2
x = —, i/ = and z =
D * D D for
_(b -c)(c -k) (k -b) _(k -c)(c - a) {a - k) (a-b) (b -k) (k - a)
=> ^ (b-c)(c-a)(a-b) andz =
u
(b-c)(c-a)(a-b)' (a-b) (b-c)(c-a)
ks

(c - k) (k -b) (k-c) (a-k) (b -k) (k - a)


Yo

Hence, x = and z =
oo

(c -a)(a -b)' (b-c) (a-b) (b -c)(c- a)


B

is the solution of given system of equations.


re

EXAMPLE 10 The sum of three numbers is 6. If we multiply the third number by 2 and add the first
number to the result, we get 7. By adding second and third numbers to three times thefirst number we get
ou
ad

12. Use determinants to find the numbers.


Y

SOLUTION Let the three numbers be x, y and z. Then, from the given conditions, we obtain
nd

x+y+z = 6 or. x + y +z - 6
Re

x + 2z = 7 x + Oy + 2z =7
Fi

3x + y + z = 12 3x + y + z = 12
1 1 1
Here, D= 1 0 2 = 1 (0-2)-1 (1 - 6) + 1 (1 - 0) = -2 + 5 + 1 = 4
3 1 1
6 1 1
Di = 7 0 2 = 6(0-2)-1 (7-24)+ 1 (7-0) = -12 + 17+7 = 12
12 1 1
1 6 1
D2 = 17 2 = 1 (7 -24)-6 (1 -6)+ 1(12-21) = -17 + 30-9 =4
3 12 1
1 1 6
and. Do = 1 0 7 = 1(0-7)-1(12-21)+ 6 (1-0) = -7 + 9 + 6 = 8
3 1 12

ReadYourFlow.COM
r
6.80 MATHEMATICS-XII

= — = — = 3, y = — = — 4 = i1 and
^ z = —^
D3 = —
8 = 2.
o
D 4 D 4 D 4
Thus, the three numbers are 3,1 and 2.
EXAMPLE 11 Solve the following- system of equations by Cramer's rule:
2 3 10 .465 ,6 9 20 _
- + - + — = 4,------ + - = 1 and - +---------= 2.
x y z x y z x y z
SOLUTION Let - = u, - 1 = v and
^ -1 w. Then, the above system of equations can be
written as x y z
2u + 3v + IQw = 4
4w -6v + 5w =1
6m + 9v - 20w = 2
2 3 10

w
Here, D=4 -6 5=2 (120 - 45) - 3 (- 80 - 30) + 10 (36 + 36) =150 + 330 + 720 =1200
6 9-20
4 3
D1 = 1 -6
10

Flo
5 = 4 (120 - 45) - 3 (- 20 -10) + 10 (9 + 12) = 300 + 90 + 210 = 600

ee
2 9 -20

Fr
2 4 10
D2 = 4 1 5 = 2(-20-10)-4(-80-30) + 10(8-6)=-60 + 440 + 20=400
for
ur
6 2 -20
2 3 4
ks

and, D3 = 4 -6 1 = 2(-12-9)-3(8-6)+ 4(36+ 36) = -42-6 + 288 = 240


Yo
oo

6 9 2
B

600 _ 1 11
= Pl =
re

u => => x = 2,
D 1200 ~ 2 x 2
ou

v = th _ 400 1 1
ad

1
= - => y = 3,
D 1200 3
Y

y 3
Do 240 1 1 1
and. w = —— = —-—- = - =>
nd

=>2=5
Re

D 1200 5 2 5
Fi

Hence, x - 2, y = 3 and 2=5.


LEVEL-2

EXAMPLE 12 If f (x) = ax2 + bx + c is a quadratic function such that f(l)=8, /(2)=11 and
/ (- 3) = 6, find f (x) by using determinants. Also, find f (0).
SOLUTION We have, / (x) = ax2 +bx + c
f (1) = 8=> a + b + c = 8
/ (2) = 11 => 4a + 2b + c =11
and, / (- 3) = 6 => 9a - 3b + c = 6
Thus, we obtain the following system of equations
a +b + c = 8
4a + 2b + c = 11
9a- 3b + c = 6

ReadYourFlow.COM
DETERMINANTS 6.81

For this system of equations, we have


1 1 1
D = 4 2 1 = 1(2+3)-1(4-9)+ 1(-12-18) = 5 + 5-30 = -20
9 -3 1
8 1 1
Di = 11 2 1 = 8(2+3)-l(ll-6) + l(-33-12) = 40-5-45 = -10
6 -3 1
1 8 1
D2 = 4 11 1 = 1(11 -6)-8 (4-9)+ 1(24-99) = 5 + 40-75 = -30
9 6 1
1 1 8
and, D3 = 4 2 11 = 1 (12 + 33) -1 (24 - 99) + 8 (-12 -18) = 45 + 75 - 240 = -120
9-3 6

low
D = -10
zii = = ! and C = ^3 = -120 = 6
a
D -20 2 D -20 2 D -20
Hence, / (x) = ^ 2 + ^ x + 6. Consequently, / (0) = 6.

ee
rF
Fr
EXAMPLE 13 Determine the values ofXfor which the following system of equations fail to have a unique
solution: for
Xx + 3y -z = 1
x + 2y + z = 2
u
-Xx + y + 2z = -1
ks
Yo

Does it have any solution for this value ofX?


oo

SOLUTION The given system of equations will fail to have unique solution, if
B

D = 0
re

X 3 -l
ou
ad

i.e. 1 2 1 =0
-X 1 2
Y

=> ^ (4 -1) - 3 (2 + A) - (1 + 2 A.) = 0


nd
Re

7
3X-6-3X-1-2?, = 0 => -2X-7 = 0=> X = -
Fi

2
7
For X — — , we obtain
2’
1 3 -1
D, = 2 2 1 16 * 0.
-1 1 2
7
Thus, for A, = —, we have D = 0 and D^ * 0.
7
Hence, the given system of equations has no solution for A, = - —.

EXAMPLE 14 For what values of a and b, the following system of equations is consistent?
x + y +z = 6
2x + 5y + az = b
x + 2y + 3z = 14

ReadYourFlow.COM
6.82 MATHEMATICS-XII

SOLUTION The given system of equations is consistent, if D * 0 or, if D = 0, then


D-y =D2 = D^ = 0.
We have.
1 1 1
D = 2 5a = 15-2a-6 + fl + 4-5 = 8-fl
1 2 3
6 1 1
Dy = b 5 a = 6(l5-2a)-(3b-Ua) + (2b-70) = 2a-b + 20
14 2 3
1 6 1
D2 = 2 b a = (3b-Ua)-6(6-a) + (28-b) = -8a + 2b-8
1 14 3
1 1 6

w
and. D3 = 2 5 = (70-2fe)-(28-fc) +6(4-5) = 36-/7
1 2 14
Now, D*0 => fl-8^0 a *8.

Flo
ee
Thus, the given system of equations will be consistent and will have unique solution for a * 8.

Fr
For a = 8, we have
D = 0 and Dy = 36-b,D2= 2b -72, D3 = 36-b for
ur
Clearly, Dy = D2 = D3 = 0 for/? = 36.
Thus, for a = 8 and b = 36, we have
ks
Yo

D = Dy = D2 = D3 = 0.
oo

Putting a = 8 and /? = 36 the given system of equations reduces to


B

x + y +z = 6
re

2x + 5y + 8z = 36
ou
ad

^ + 2y + 3z = 14
Y

Taking z = k, first and third equations become


nd
Re

x+y =6-k
Fi

x + 2y = 14 - 3k
Solving these equations by Cramer's rule, we get
6-k 1
14 - 3/f 2
x = = U-2k-U+ 3k = k-2
1 1
1 2
1 6-k
1 14 - 3k
y = = 14 - 3k - 6 + k = 8-2k
1 1
1 2
Thus, we have
x = k-2,y = 8-2k,z = k.
Clearly, these values satisfy the second equation.

ReadYourFlow.COM
DETERMINANTS 6.83

Thus, the given system of equations will be consistent and will have infinitely many solutions
for a = 8 and b = 36.
Hence, the given system of equation will be consistent if a ^ 8 and b e R or, if « = 8 and b = 36.
EXAMPLE 15 For what values of a and b, the system of equations
2x + ay + 6z = 8
x + 2y + bz = 5
x + y + 3z = 4
has: (i) a unique solution (ii) infinitely many solutions (iii) no solution.
SOLUTION For the given system of equations, we have
2 a 6
D = 1 2 b
1 1 3

low
=> D = 2 {6 - b) - a (3 -b) + 6 (1 - 2)
=> D = 12 - 2b - 3a + ab - 6 = 6 - 3a - 2b + ab = {b-3) (a-2)
8 a 6
D1 = 5 2 b

ee
4 1 3
rF
Fr
=> Dj = 8 (6 -b) - a (15 - 4b) + 6 (5 - 8)
=> D1 = 48-8b-15a + 4ab-18 = 30-15a - 8b + 4ab = (a-2) (4b-15)
for
2 8 6
u
D2 = 1 5 b
ks
Yo

1 4 3
oo

=> D2 = 2(15 -4b) -8(3 -b) + 6(4-5) = 30-8t-24 + 8&-6 = 0


B

2 a 8
re

and. D3 = 1 2 5 = 2(8-5)-fl(4-5) + 8(l-2) = 6 + a-8 = fl-2


1 1 4
ou
ad

(i) For unique solution, we must have


Y

D *0 => (a -2)(b - 3) *0 => Neither a *2, nor b * 3.


nd

(ii) For infinitely many solutions, we must have


Re

D — D1 = D2 = D 3 — 0
Fi

=> (a-2)(b-3) = 0, (a - 2) (4b -15) = 0 and a - 2 = 0


a = 2.
Putting a = 2 in the given system of equations, we obtain
2x + 2y + 6z = 8
x + 2y + bz =5
x + y + 3z = 4
This system is equivalent to the system
x + y + 3z = 4
x + 2y + bz =5
Putting z = k, we get
x+y = 4-k
x + 2y = 5-bk
Solving these two equations, we get
x = 3 -2k +bk, y = 1 -bk + k

ReadYourFlow.COM
6.84 MATHEMATICS-XII

Thus, the given system has infinitely many solutions given by


x = 3 -2k + bk,y = 1 - bk + k,z = k, where k eR.
Hence, the system has infinitely many solutions for a = 2.
(iii) For no solution, we must have
D = 0 and at least one of D| , D2 and D3 is non-zero.
Clearly, for fr = 3, we have
D = 0 and D3 * 0.
Hence, the system has no solution for b = 3.
EXERCISE 6.4
LEVEL-1

Solve thefollowing systems of linear equations by Cramer's rule:

w
1. *-2y = 4 2. 2x - y = 1 3. 2x-y=17
- 3x + 5y = - 7 7 x - 2y = - 7 3x + 5y = 6
4. 3x + y=19 5. 2x - y = - 2

Flo 6. 3x + fly = 4

ee
3x - y = 23 3x + 4y = 3 2x + fly = 2, fl 5* 0

Fr
7. 2x + 3y = 10 8. 5x + 7y = -2 9. 9x + 5y = 10
x + 6y = 4 4x + 6y = - 3 for 3y - 2x = 8
ur
10. x + 2y = 1
3x + y = 4
ks

Solve thefollowing system of the linear equations by Cramer's rule:


Yo
oo

11. 3x + y + z = 2 12. x - 4y -z = 11 13. 6x + y - 3z =5


B

2x - 4y + 3z = -1 2x - 5y + 2z = 39 x + 3y - 2z = 5
re

4x + y - 3z = -11 - 3x + 2y + z = 1 2x + y + 4z = 8
14. x + y =5 15. 2y - 3z = 0 16. 5x -7y +z =11
ou
ad

y +z = 3 x + 3y = - 4 6x - 8y -z =15
Y

x +z = 4 3x + 4y = 3 3x + 2y - 6z = 7
nd

17. 2x - 3y - 4z = 29
Re

18. x + y =1 19. x + y + z + 1 =0
-2x + 5y - z = -15 x +z=-6 ax + i7y + cz + d = 0
Fi

3x -y + 5z = -11 x-y-2z = 3 fl2x + b2y + c2z + d2 =0


20. x + y+z + zv =2 21. 2x - 3z + w = 1
x-2y + 2z + 2w=-6 x - y + 2w = 1
2x + y - 2z + 2w = -5 - 3y + z + w = 1
3x - y + 3z - 3w = - 3 x + y +z=1
Shozv that each of thefollozving systems of linear equations is inconsistent:
22. 2x-y =5 23. 3x + y = 5 24. 3x - y + 2z = 3
4x-2y =7 - 6x - 2y = 9 2x + y + 3z = 5
x-2y-z = 1
25. 3x - y + 2z = 6
2x -y + z = 2
3x + 6y + 5z = 20.

ReadYourFlow.COM
DETERMINANTS 6.85

Show that each of the following systems of linear equations has infinite number of solutions and
solve (26 - 30)
26. x-y + z = 3 27. x + 2y=5 28. x + y-z = 0
2x + y-z = 2 3x + 6y = 15 x-2y+ 2 = 0
-x-2y + 2z = l 3x + 6y - 52 = 0
29. 2x + y -2z = 4 30. x-y + 3z = 6
x-2y + z = -2 x + 3y - 32 = - 4
5x -5y + z = -2 5x + 3y + 32 = 10
31. A salesman has the following record of sales during three months for three items
A, B and C which have different rates of commission.
Month Sale of units Total commission drawn (in ?)

low
A B C
Jan 90 100 20 800
Feb 130 50 40 900
March 60 100 30 850

ee
Find out the rates of commission on items A, B and C by using determinant method.
rF
Fr
32. An automobile company uses three types of steel Si, $2 and S3 for producing three types of
cars C-j, C2 and C3. Steel requirements (in tons) for each type of cars are given below:
for
Cars
u
c]
ks

Steel C2 C3
Yo
oo

Si 2 3 4
B

S2 1 1 2
re

% 3 2 1
ou

Using Cramer's rule, find the number of cars of each type which can be produced using 29,
ad

13 and 16 tonnes of steel of three types respectively.


Y

ANSWERS
nd
Re

1. x = - 6, y = - 5 2. x = — 3, y = — 7 3. x =7, y = - 3 4. x = 7, y = - 2
Fi

12 16 2 7
6. x = 2, y = - — 7. x = 3'y 9 8. x = —2'y 2
5-x = ~ri'y = n a
10 92 7 1
9. * = - —,y = 10. x = 5'y 5 11. x = — 1,y = 2,2 = 3
37 37
12. x = -l, y = -5,2 = 8 13. x =1, y = 2,2 =1
14. x = 3,y = 2,2=1 15. x =5,y = -3,2 =-2 16. x =1, y = -l,2 = -l
17. x = 2,y = -3,2 = -4 18. x = - 2, y = 3,2 = - 4
(b-c)(c-d)(d-b) (a - tf) (rf - c) (c - fl) (a-b) (b-d) (d-a)
19. x = — (a -b) {b - c) (c - a)' y = —
(a - b) (b — c) (c — a)' (a - b) (b -c) (c - a)
3 1 2 1
20. x = - 2, y = 3,2 = - ,w =- 21. x =1, y = - — ,2 = ,10 = -
2 ^ 7 7 7
25. x = - 3, y = -1,2 =7 26. x = -, y = k--,z = k
3 ' 3
27. x = 5 -2k, y = k 28. x = k, y - 2k, z- 3k

ReadYourFlow.COM
6.86 MATHEMATICS-XII

6+3/c 8 +4k 7-3k 3k - 5 •


29. x = >y = --------,z = k 30. x = --------/ V =-------- / z = k
5 5 2*2
HINTS TO NCERT& SELECTED PROBLEMS
31. Let x, y andz be the rates of commission in ? of items A, B and C respectively. Then, we
have
90x + lOOy + 20z = 800,130x 4- 50y + 40z = 900 and, 60x 4- 100i/ 4- 30z = 850
6.7.3 SOLUTION OF A HOMOGENEOUS SYSTEM OF LINEAR EQUATIONS
In the previous sub-section, we have learnt about the solution of a non- homogeneous system of
linear equations and its consistency and inconsistency.
Let us now consider a homogeneous system of equations given by
fl} x 4- fq y 4- c-i z = 0

low
a2 x + b2y + c2z = 0
a3x + b3y + c3z = 0
For this system of equations, we have
0 h Ci fq 0 al b1 0

ee
ci
Dj =0 b2 c2 =0, T)2 —
rF
a2 0 c2 = 0 and, D3 = a2 b2 0 = 0.

Fr
0 b3 c3 a3 0 c3
a3 b 3 0
for
H h 'i D
If D = a2 b2 c2 ^ 0, then x = —1 = 0, y = ^ = Oandz = ^3=0.
ou
“3 i>3 C3 D D D
ks
oo

Thus, if D ^ 0, then the homogeneous system of equations has unique solution x = 0, y = 0, z = 0.


Y

This solution is called the trivial solution.


B
re

If D = 0, then a homogeneous system of equations has infinitely many solutions. Solutions other
than the trivial solution are called non-trivial or non-zero solutions.
ou
ad

In order to solve a homogeneous system of equations by Cramer's rule, we may use the
Y

following algorithm.
nd

ALGORITHM
Re

STEP I Obtain the system of equations and compute D i.e. the determinant of the coefficient matrix.
Fi

STEP II IfD * 0, then the system has only the trivial solution i.e. x=y=z = 0.So/x = 0= y=zis the
only solution of the given system.
STEP HI IfD =0, then take any two out of three equations and replace one of the variables z (say) by k.
Solve the system so obtained by Cramer's rule. The solution so obtained with z = k gives a
solution of the given system.
REMARK It is evident from the above discussion that a homogeneous system of equations will have
non-trivial solution iff \ D\=0.
ILLUSTRATIVE EXAMPLES

LEVEL-1
EXAMPLE l Solve the following system of homogeneous equations:
3x - 4y 4- 5z = 0
x + y-2z = 0
2x 4- 3y 4- z = 0

ReadYourFlow.COM
DETERMINANTS 6.87

SOLUTION For the given system of equations, we have


3-4 5
D =1 1 -2 = 46 * 0.
2 3 1
So, the given system of equations has only the trivial solution i.e. x =y =z = Q.
EXAMPLE 2 Solve the following system of homogeneous equations:
x + y -z = 0
x-2y + z = 0
3x + 6y -5z - 0
SOLUTION For the given system of equations, we have
1 1 -1
D = 1 -2 1 = lx(10-6)-lx(-5-3)-lx(6 + 6)= 4 + 8-12 = 0

w
3 6-5
So, the system has infinitely many solutions. Consider the first two equations. Putting z = k in
first two equations, we get
x+y = k
Flo
ee
x-2y = -k

Fr
Solving these equations by Cramer's rule, we obtain
k 1 1 k
for
ur
-k -2 -k k J D' 1 -k -2k _ 2k
X = = — and, y = —2 =
ks

D 1 1 -3 3 * D 1 1 -3 3
Yo

1 -2 1 -2
oo

k 2k
B

Thus, we have x = — , y = — ,z = k. Clearly, these values satisfy the third equation.


3 3
re

k 2k
Hence, x = — , y = — ,z = k gives the solution for each value of k.
ou

3 3
ad
Y

EXAMPLE 3 Find the value of Xfor which the homogeneous system of equations:
2x + 3y - 2z = 0
nd
Re

2x-y + 3z = 0
7x + Xy -z = 0
Fi

has non-trivial solutions. Find the solution.


SOLUTION The given system of equations will have non-trivial solution, if
D =0
2 3-2
i.e. 2-1 3 = 0
7 X -1
=> 2 (1 - 3 A) - 3 (- 2 - 21) - 2 (2 A. + 7) = 0
57
2-6 A+69-4 A-14 = 0 => 10 A + 57 = 0 => A = —
10
57
Hence, the given system of equations will have non-trivial solutions, if A = —
Let us now find solutions for this value of A. ^
Taking first two equations and replacing 2 by k, we get
2x + 3y = 2k
2x -y = -3k

ReadYourFlow.COM
6.88 MATHEMATICS-XII

Solving these two equations by Cramer's rule, we get


2k 3 2 2k
-3k -1 2k + 9k -7k 2 -3k -10 k 5k
x = y = 2
2 3 -2-6 8 ' 3 -8 4
2 -1 2 -1
Substituting these values of x and y in the third equation i.e. 7x + Xy - z = 0, we obtain
-7k , 5k . -49 57
LHS = 7 x + A.x----- k = k+5^5Ji-k = 0 = RHS v X=—
8 4 8 10 4 10
7k 5k
Hence, x =----- , y = — , z = k gives the solution of the given system of equations for each
8 4
value of k.
LEVEL-2

w
EXAMPLE 4 If the system of equations
x = cy + bz
y = az + cx
z = bx + ay
Flo
ee
has a non-trivial solution, shozv that a2 +b2 + c2 + 2abc -1

Fr
SOLUTION The given system of equations can be written as
x -cy -bz = 0
for
ur
cx-y + az = 0
bx + ay -z = 0
ks
Yo

If it has a non-trivial solution, then


oo

1 -c -b
eB

c -1 a = 0
b a -1
r
ou

=> 1 x (1 - a2) + c (- c - ab) -b (ca + b) = 0


ad
Y

=> 1-a2-c2-abc-abc-b2 = 0
a2 +b2 + c2 + 2abc = 1.
nd

=>
Re

EXAMPLE 5 If a, b, c are distinct real numbers and the system of equations


Fi

ax + a2 y + (a3 +l)z = 0
bx + b2 y + (b3 + 1) z = 0
cx + c2 y + (c3 + l)z = 0
has a non-trivial solution, show that abc = -1.
SOLUTION It is given that the given system of homogeneous linear equations has a non-trivial
solution.
a a2 a3 +1
b b2 b3+l = 0
c c2 c3+1

=> (a -b)(b - c) (c - a) (1 + abc) = 0 [See Example 30 on page 6.30]


=> 1 + abc = 0 [v a*b * c a- b*0,b-c*0,c-a*0]
=> abc = -1.

ReadYourFlow.COM
DETERMINANTS 6.89

EXAMPLE 6 Ifx, y, z are not all zero such that


ax + y + z = 0
x + by +z = 0
x + i/ + cz = 0,
1 1
then prove that — = 1.
\-a 1 -b 1 — c
SOLUTION It is given that x, y, z are not all zero. This means that there are non-trivial solutions
of the given system of equations.
all
1 b 1 = 0 => abc - a - c -b + 2 = 0=> abc = a + b + c - 2
1 1 c
1 1 (l-b)(l-c) + (l-c)(l-a) + (l-a)(l-b)
Now, + 1

w
1 -a 1 -b 1 -c (1-*)(!-&) (1-c)
3 - 2(a + b + c) + (ab + be + ca)

Flo
l - (a + b + c) + (ab +bc + ca) - abc
3 - 2(a + b + c) +(ab + bc + ca)

ee
[Using (i)]
1 - (a + b + c) + (ab + bc + ca) - (a + b + c) + 2

Fr
3 - 2(a+ b + c) + (ab + bc + ca)
= 1.
for
ur
3 -2(a + b + c) + (ab + be + ca)
ks

EXERCISE 6.5
Yo
oo

LEVEL-1
B

Solve each of the following system of homogeneous linear equations:


re

1. x + y - 2z = 0 2. 2x + 3i/ + 4z = 0 3. 3x + y + z = 0
ou

2x + y - 3z = 0 x+y+z = 0 x — 4y + 3z = 0
ad

5x + 4y-9z = 0 2x + 5y - 2z = 0 2x + 5y-2z - 0
Y

LEVEL-2
nd
Re

4. Find the real values of X for which the following system of linear equations has non-trivial
Fi

solutions. Also, find the non-trivial solutions


2 A. x - 2y + 3z = 0
x + Ay + 2z = 0
2x + A z = 0
5. If a,b, c are non-zero real numbers and if the system of equations
(fl-l)x = y + z
(b -1) y = z + x
(c-l)z = x + y
has a non-trivial solution, then prove that ab + be + ca = abc.
ANSWERS
1. x = kr y = k,z = /c, whereAeR 2. x = 0, y = 0, z = 0
3. x - -7k, y - 8k, z = 13k, where, k eR 4. A = 2, x =A, y = ^ , z = - A where k eR

ReadYourFlow.COM
6.90 MATHEMATICS-XII

VERY SHORT ANSWER QUESTIONS (VSAQs)

Answer each of the following questions in one word or one sentence or as per exact requirement of the
question:
If ^4 is a singular matrix, then write the value of | A|.
'5-x x + \~
For what value of x, the matrix ^ ^ is singular? [CBSE2011]

2 3 4
Write the value of the determinant 2x 3x 4.v
5 6 8
[2 31
State whether the matrix is singular or nonsingular.
6 4
4200 4201
Find the value of the determinant
4202 4203 ‘

w
101 102 103
Find the value of the determinant 104 105 106
107 108 109

Flo
ee
a l b+c

Fr
Write the value of the determinant b 1 c + a
c 1 a+b for
ur
[0 f| 0 1
If 71 = and B = , find the value of | 7l| +1 B|.
i 1 1 0
ks
Yo

21 and B = 1 0j , find | AB\.


oo

-1 -1 0
B

4785 4787
re

Evaluate:
4789 4791'
1 w w2
ou
ad

If w is an imaginary cube root of unity, find the value of w w2 1


Y

If A = J and B = J Z* 'findlABl- zo2 1 w


d
Re
n
Fi

If A = [ajj] is a 3 x 3 diagonal matrix such that a^ =1, fl22 = 2 and a^ = 3, then find | A |.
If A = [ajj] is a 3 x 3 scalar matrix such that an = 2, then write the value of | A|.
If I3 denotes identity matrix of order 3x3, write the value of its determinant.
A matrix A of order 3x3 has determinant 5. What is the value of 13A|? [CBSE2012]
On expanding by first row, the value of the determinant of 3 x 3 square matrix A = [ajj] is
flll ^11 + aU ^12 + a\3 ^13 ' where Qy is the cofactor of Ojj in A. Write the expression for its
value on expanding by second column.
Let A = [ajj] be a square matrix of order 3x3 and Cjj denote cofactor of Ojj in A. If | A| =5,
write the value of fl31 C31 + fl32 C32 + «33 C33.
19. In question 18, write the value of a^ C2| + «12 C22 + C23.
sin 20° - cos 20°
Write the value of
sin 70° cos 70° '
If A is a square matrix satisfying AT A- I, write the value of | A|.

ReadYourFlow.COM
DETERMINANTS 6.91

22. If A and B are square matrices of the same order such that| A| = 3 and AB = I, then write the
value of | B|.
23. ^ is a skew-symmetric of order 3, write the value of |v4|.
24. If A is a square matrix of order 3 with determinant 4, then write the value of |- A|.
T
25. If A is a square matrix such that | A | = 2, write the value of | AA |.
243 156 300
26. Find the value of the determinant 81 52 100 .
-3 0 4
2-3 5!
27. Write the value of the determinant 4 -6 10 .
6 -9 15
5x 2'
28. If the matrix is singular, find the value of x.

w
-10 1
29. If A is a square matrix of order nxn such that | A| = \ then write the value of | - A|.

Flo
2^ 23 24

ee
30. Find the value of the determinant 23 24 25

Fr
24 25 26 i
for
31. If A and B are non-singular matrices of the same order, write whether AB is singular or
ur
non-singular.
32. A matrix of order 3x3 has determinant 2. What is the value of | A (3/) |, where I is the
ks
Yo

identy matrix of order 3x3.


oo

33. If A and B are square matrices of order 3 such that |A| = -1,|B| = 3, then find the value of
B

| 3AB|.
re

a + ib c + id
34. Write the value of [CBSE 2008]
-c + id a -ib '
ou
ad

2-3 5
Y

35. Write the cofactor of a12 in the matrix 6 0 4 [CBSE 2008]


1 5 -7
nd
Re

2x + 5 3
36. If = 0, find x. [CBSE 2008]
Fi

5x + 2 9
X
37. Find the value of x from the following: 0
2 2x
4Uo [CBSE 2009]

2 3 4
38. Write the value of the determinant 5 6 8 IlCBSE 2009]
6x 9x 12x
39. If | A| = 2, where A is 2 x 2 matrix, find | adj A|. ICBSE2010]
0 2 0
40. What is the value of the determinant 2 3 4 [CBSE 2010]
4 5 6
6 -x 4
41. For what value of x is the matrix singular? [CBSE 2011]
3 -x 1
42. A matrix A of order 3 x 3 is such that | A| = 4. Find the value of 12 A|. [CBSE 2011]

ReadYourFlow.COM
6.92 MATHEMATICS-XII

cos 15° sin 15°


43. Evaluate: [CBSE2011]
sin 75° cos 75°
5 3 8"
44. If A — 2 0 1 . Write the cofactor of the element a 32- [CBSE2012]
1 2 3
X + 1 x-1 4 -1
45. If = , then write the value of x. [CBSE 20131
x-3 x+2 1 3
2x x+ 3 1 5
46. If = , then write the value of x. [CBSE 2013]
2 (x + 1) x +1 3 3
3x 7 8 7
47. If find the value of x. [CBSE 2014]
-2 4 " 6 4 '
2x 5 _ 6 -2
48. If , write the value of x. [CBSE 2014]

w
8 x “ 7 3
49. IfAisa3x 3 matrix,]/1| ^ 0 and 13/11 =/c | A | then write the value of [CBSE 2014]
50. Write the value of the determinant
P P+1

Flo [CBSE 2014]

ee
P-1 P

Fr
x+y y+z z+x
51. Write the value of the determinant z x y [CBSE 2015]
for
ur
-3 -3 -3
cos 8 sin 9
ks

52. If A = then for any natural number, find the value of Det (An). [CBSE 2015]
-sin 9 cos 9
Yo
oo

1 1 1
eB

53. Find the maximum value of 1 1 + sin 9 1 [CBSE 2016]


1 1 1 + cos 9
r
ou
ad

x+3 -2
54. If x e N and = 8, then find the value of x. [CBSE 2016]
Y

-3x 2x
nd

x sin 9 cos 9
Re

55. If -sin 8 -x 1 = 8, write the value of x. [CBSE 2016]


Fi

cos 9 1 x
56. If Aisa 3x 3matrix,thenwhatwillbethevalueof/cifDet(A 1)=(DetA)*r? [CBSE2017]
_____________________________________________________________________ANS WERS
1. 0 2. 3 3. 0 4. Non-singular 5.-2 6. 0 7. 0
8.0 9.0 10.-8 11.0 12.-70 13. 6 14. 8 15. 1
16. 135 17. fl-j2 Cj2 + fl22 ^'22 + ^32 ^-'32 1^' ^ 19. 0 20. 1 21. +1
22. 1/3 23. 0 24.-4 25. 4 26. 0 27.0 28.-4 29. (-1)” X
30. 0 31. Non-singular 32. 54 33. - 81 34. a2+b2 + c2 + d2 35. 46
36.-13 37. ±2 38.0 39.8 40.8 41. 2 42. 32 43. 0
44. 11 45. 2 46. 1 47. -2 48. ± 6 49. 27 50. 1 51. 0
52. 1 53. - 54. ± 2 55. - 2,- 2w, -2zu2 56.k=-l
2

ReadYourFlow.COM
DETERMINANTS 6.93

MULTIPLE CHOICE QUESTIONS (MCQs)


Mark the correct alternative in each of the following:
1. If A and B are square matrices of order 2, then det (A + B) - 0 is possible only when
(a) det (2l) = 0 or det(B)=0 (b) det (A) + det (B) =0
(c) det (,4) = 0 and det(B)=0 (d) A + B =0
2. Which of the following is not correct ?
(a) | A|=| A7 | , where A = [^]3 x 3

(b) \ kA \ =k5 \ A\, where A = [<i,y ]3 x 3


(c) If A is a skew-symmetric matrix of odd order, then | A | = 0
'a+b c+d a c b d
(d) +
e+f g+h e S / h
au an fii3

w
3. If A = a21 a22 «23 an<^ Q; is cofactor of Ojj in A, then value of | A | is given by
a31 a32 a33
(a) an C31 + fl12 C32 + a13 C33
(c) a21 Cn + a22 C12 + a23 C13

Flo
(b) an Cn + a12 C2i + a13 C31
(d) an Cn + a21 C21 + a31C31

ee
4. Which of the following is not correct in a given determinant of A, where A = [Ojj ] 3 x 3 •

Fr
(a) Order of minor is less than order of the det (A)
(b) Minor of an element can never be equal to cofactor of the same element
for
ur
(c) Value of a determinant is obtained by multiplying elements of a row or column
by corresponding cofactors
ks

(d) Order of minors and cofactors of elements of A is same


Yo
oo

* 2 x
5. Let x x 6 = ax +bx + cx + dx + e. Then, the value of 5a + 4b + 3c + 2d + e is equal
B

x x 6
re

to
ou
ad

(a) 0 (b) -16 (c) 16 (d) none of these


Y

*2 a 1
6. The value of the determinant cos nx cos (n + 1) x cos (n + 2) x is independent of
nd
Re

sin nx sin (» + 1) x sin (n + 2) x


Fi

(a) n (b) a (c) x (d) none of these


1 1 1 1 be a
7. If A3 = a b c , A2 = 1 ca b , then
a2 b2 c2 1 ab c

(a) Aj + A2 = 0 (b) A-] + 2 A2 — 0 (c) A-j = A2 (d) none of these


1 n n
n
8. If Dk = 2k n2 +n +2 n2 + n and X Dk = 48, then n equals
2k-l n2 n2 + n +2 k=l

(a) 4 (b) 6 (c) 8 (d) none of these


x2 + 3x x -1 x + 3
9. Let x +1 - 2x x- 4 = ax4 + bx3 + cx2 + dx + e
x-3 x+4 3x

be an identity in x, where a, b, c, d, e are independent of x. Then the value of e is

ReadYourFlow.COM
6.94 MATHEMATICS-XII

(a) 4 (b) 0 (c) 1 (d) none of these


10. Using the factor theorem it is found that a + b,b + c and c + a are three factors of the
-2a a + b a+c
determinant b + a -2b b + c . The other factor in the value of the determinant is
c+a c+b -2c
(a) 4 (b) 2 (c) a + b + c (d) none of these
0 x2-a x3-b
11. If a, b, c are distinct, then the value of x satisfying x +a 0 X2 + c = 0 is
x* +b x -c 0

(a) c (b) a (c) b (d) 0


a b 2a a + 3b
12. If the determinant b c 2b a + 3c = 0, then

w
2a a + 3b 2b a + 3c 0
(a) a, b, c areinH.P. (b) a is a root of 4ax + I2bx + 9c = 0 or/?, b, c are in G.P.
(c) a, b, c are in G.P. only

Flo
(d) a, b, c are in A.P.
C32"

ee
1 co"

Fr
13. If ©is a non-real cube root of unity and n is not a multiple of 3, then A = ©2” 1 ©" is
for ©" w2" 1
ur
equal to
(a) 0 (b) © (c) m2 (d) 1
ks
Yo

2r
oo

1 r n
14. If Ar = 2 n n2 , then the value of ^ Ar is
B

n n (« + 1) 2” + 1 r=1
re

2
ou
ad

(a) n (b) 2n (c) - 2n (d) n2


Y

a b ax + b
15. If a > 0 and discriminant of ax + 2bx + c is negative, then A = b c bx + c is
nd
Re

ax + b bx + c 0
Fi

(a) positive (b) (ac-b2) {ax2 + 2bx + c) (c) negative (d) 0


52 53 54
16. The value of 53 54 55 is
54 55 56

(a) 52 (b) 0 (C) 513 (d) 59


log3 512 log4 3 x log2 3 log8 3 =
17.
log38 log4 ^ logs 4 log3 4
(a) 7 (b) 10 (c) 13 (d) 17
x+ 2 x+ 3 x + 2a
18. If a, b, c are in A.P., then the determinant x + 3 x+4 x + 2b
^+4 a: + 5 x + 2c
(a) 0 (b) 1 (c) x (d) 2%

ReadYourFlow.COM
DETERMINANTS 6.95

sin (A+ B+C) sin(A+C) cosC


19. If A + B + C = tc, then the value of - sin S' 0 tan A is equal to
cos (A + B) tan (B + C) 0

(a) 0 (b) 1 (c) 2 sin B tan A cos C (d) none of these


cosec x sec x sec x
20. The number of distinct real roots of sec x cosec x sec x = 0 lies in the
sec x sec x cosec x
7U 71
interval — < x < - is
4 4
(a) 1 (b) 2 (c) 3 (d) 0
1 sin 0 1
21. Let A = -sin 0 1 sin 0 , where0 <6 < 2n.Then,

w
-1 - sin 0 1
(a) Det (A) = 0 (b) Det (A) e (2, a>) (c) Det (A) € (2, 4) (d) Det (A) e [2, 4]

Flo
2x 5 _ 6 -2
22. If , then x --
8 x " 7 3

ee
(a) 3 (b) ±3 (c) ±6 (d) 6

Fr
0 x-a x-b
23. If/(x) = x+fl 0 x-c , then
for
ur
x+b x+c 0
ks

(a) f(a)=0 (b) f(b)=0 (c)/(0)=0 (d) /(1)=0


Yo
oo

a-b b+c a
24. The value of the determinant b-a c+a b is
B
re

c-a a+b c
(a) a3 +b3 + c3 (b) 3 be (c) a:’ + b3 + c3 - 3abc (d) none of these
ou
ad

1+x 1 1
Y

25. If x, y, 2 are different from zero and 1 1 + y 1 =0, then the value of x 1 + y 1 + 2 1
nd
Re

1 1 1+2
Fi

is
(a) xyz <b) x-yh-1 (c) -x-y-z (d) -1
b -ab b-c bc-ac
26. The determinant ab-a1 a-b b -ab equals
bc-ca c-a ab-a

(a) abc (b-c) (c-a) (a-b) (b) (b-c) (c-a) (a-b)


(c) (a + b + c)(b-c)(c-a)(a-b) (d) none of these
cosx -sin x 1
27 If x, y eR, then the determinant A = sinx cosx 1 lies in the interval
cos (x + y) -sin(x + y) 0
(a) [-V2, V2] (b) [-1,1] (C) [-V2.1] (d) [-1, -V2]

ReadYourFlow.COM
6.96 MATHEMATICS-XII

1 1 1
28. The maximum value of A = 1 1 + sin 9 1 is(0 is real)
1 + cos 0 11
Vs Vs
(a4 x x+y
(c) V2
x + 2y
(d) 2

29. The value of the determinant x + 2y x x + y is


x+y x + 2y x
(a) 9x2(x + y) (b) 9y2(x + y) (c) Sy2(x + y) (d) 7x2(x + y)
COSX X 1
30. Let/(x) = 2 sin x x 2x , then lim /(*)
-y- is equal to
x-»0 x“
sin x x x

low
(a) 0 (b) -1 (d) 3(c) 2
1 -2 5
1 • There are two values of a which makes the determinant A = 2 a -1 equal to 86. The

ee
0 4 2a
rF
Fr
sum of these two values is
(a) 4 (b) 5 (c) -4 (d) 9 for
a p x p+q a+x a+p
ou
32. If cj y = 16, then the value of^ + y b + y b + q is
ks

c r z x+z c+z c+r


oo

(a) 4 (b) 8 (c) 16 (d) 32


Y
B
re

1 1 1
33. The value of ,1C1 n+ 2 n+ 4
Ci Cl is
ou
ad

nC2 n+2 •2+4


C2 C2
Y

(a) 2 (b) 4 (c) 8 (d) n2


nd
Re

ANSWERS
Fi

1. (d) (d) 3. (d) 4. (b) 5. (d) 6. (a) 7. (a) 8. (a) 9. (b)


10. (a) 11. (d) 12. (b) 13. (a) 14- (c) 15- (c) 16. (b) 17. (b) 18. (a)
19- (a) 20. (b) 21. (d) 22- (c) 23. (C) 24. (C) 25. (d) 26. (d) 27. (a)
28. (a) 29. (b) 30. (a) 31. (C) 32. (d) 33. (C)
SUMMARY
1 ■ Every square matrix can be associated to an expression or a number which is known as its
determinant.
an a12 .
(i) If A = is a square matrix of order 2x2, then its determinant is denoted by
a2l a22_
au a12
1^1 or and is defined as flji a22 - a^2 fl2i •
a2l a22

i.e. | | = a\\ a\2


- l7ll a22 ~ a\2 a2\
a2\ a22

ReadYourFlow.COM
DETERMINANTS 6.97

an au ai3
(ii) If A = rt2i «22 a23 *s a scluare matrix of order 3x3, then its determinant is
a3l a32 a33_
an au ai3
denoted by |A| or, tf2l a22 a23 anc^ ls eclual to flll a22 a33 + (712 a23 1131
a31 a32 a33
+ fl13 rt32 a2\ - Ci\i «23 a32 ~ a22 a\3 a3l ~ al2 a2\ a33
This expression can be arranged in the following form:
an fli2 a\3 1+1 1+2 1+ 3 (12i n22
a21 a23
a2\ a22 a23 =(_1) + aU a22 a23 + (-1) a12 + (-D rt13
a32 a33 1131 a33 a31 a32
a3\ a32 a33
This is known as the expansion of | A | along first row.

low
In fact, | A | can be expanded along any of its rows or columns. In order to expand | /4 |
along any row or column, we multiply each element Ojj of /th row (say) with (-1)' + 1 times

the determinant of the submatrix obtained by leaving the row and column passing through
the element and then they are added.

ee
rF
Similarly, we can find the value of the determinant of square matrices of order 4 or more.

Fr
2. A square matrix is a singular matrix if its determinant is zero. Otherwise, it is a
non-singular matrix.
for
3. (i) Let A = [qj \ be a square matrix of order n. Then the minor M,y or cijj in A is the determinant
u
of the sub-matrix of order (?? -1) obtained by leaving /th row and /th column of A.
ks
Yo

' 1 2 3"
oo

For example, if A = - 3 2-1 , then


B

2-4 3
re

2 -1 -3 -1
Mu = -4 3 = 2, M12 = = -7 and so on.
ou

2 3
ad
Y

(ii) The cofactor C,y of Ojj in A = [^] n xn is equal to (-1)' + - times M;y.
nd
Re

" 1 2 3"
For example, ifA= -3 2 -1 , then
Fi

2-4 3

Cn = (-D
1+ 1
Mu = Mil 2 and Ci2 = (-1) 1+ 2 M12 = A4P = 7 and so on.
4. Following are some important properties of determinants:
(i) Let A = [cijj ] be a square matrix of order n, then the sum of the product of elements of
any row (column) with their cofactors is always equal to | A | or, det (A).
n n
i.e. I ^ Ojj Cjj =| A I and _ 2^ a,j Cjj =| A |
/
(ii) Let A = [a(; ] be a square matrix of order n, then the sum of the product of elements of
any row (column) with the cofactors of the corresponding elements of some other row
(column) is zero.
n n
i.e. I Ojj Ckj = 0 and. I1 Cik = 0.
;=i /

ReadYourFlow.COM
6.98 MATHEMATICS-XII

(iii) Let A =[(ijj] be a square matrix of order then| A | =| .A7'|.


By the abuse of language this property is also stated as follows:
The value of a determinant remains unchanged if its rows and columns are
interchanged.
(iv) Let A = [fl/y j be a square matrix of order n (> 2) and let B be a matrix obtained from A by
interchanging any two rows (columns) of A, then \ B\ =- | A |.
Conventionally this property is also stated as:
If any two rows (columns) of a determinant are interchanged, then the value of the
determinant changes by minus sign only.
(v) If any two rows (columns) of a square matrix A = [Ojj ] of order n (> 2) are identical,
then its determinant is zero i.e. | A | = 0.
Conventionally this property is stated as:

low
If any two rows or columns of a determinant are identical, then its value is zero.
(vi) Let A = [cijj] be a square matrix of order n, and let B be the matrix obtained from A by
multiplying each element of a row (column) of A by a scalar k, then \ B\ =k | A |.
Conventionally this property is also stated as:

ee
rF
Fr
If each element of a row (column) of a determinant is multiplied by a constant k, then
the value of the new determinant is k times the value of the original determinant.
for
If A = [iijj] be a square matrix of order n, then \kA\=kn\ A\.
ou
(vii) Let A be a square matrix such that each element of a row (column) of A is expressed as
ks

the sum of two or more terms. Then the determinant of A can be expressed as the sum
oo

of the determinants of two or more matrices of the same order.


Y
B

Conventionally this property is also stated as:


re

If each element of a row (column) of a determinant is expressed as a sum of two or


more terms, then the determinant can be expressed as the sum of two or more
ou
ad

determinants.
Y

(viii) Let A be a square matrix and B be a matrix obtained from A by adding to a row
nd

(column) of A a scalar multiple of another row (column) of A, then | B | = | A |.


Re

This property is conventionally stated as:


Fi

If each element of a row (column) of a determinant is multiplied by the same constant


and then added to the corresponding elements of some other row (column), then the
value of the determinant remains same.
(ix) Let A be a square matrix of order n (> 2) such that each element in a row (column) of A
is zero, then | A | = 0.
Conventionally this property is also stated as:
If each element of a row (column) of a determinant is zero, then its value is zero.
(x) If A = [cijj] is a diagonal matrix of order n (> 2), then | A | = • «22 - a33 ■ ■ ■ amr
(xi) If A and B are square matrices of the same order, then | AB | = | A | | B |.
(xii) If A = [(7^ ] is a triangular matrix of order n, then | A| = . fl22 • ^33 antv
5. Area of a triangle with vertices (aq, iq), (aq, 1/2) and (ar3/ y3) is given by
y-i 1
A 1
A = r *2 V2 1
*3 ys 1

ReadYourFlow.COM
DETERMINANTS 6.99

6. (i) If ,4 is a skew-symmetric matrix of odd order, then | A | = O.


(ii) The determinant of a skew-symmetric matrix of even order is a perfect square.
7. Consider a system of simultaneous linear equations given by
a-^ x + b1y + c-^z =
a2 x + b2y + c2z = d2
«3 * + i,3 y + C3Z = d3
A set of values of the variables x, y, z which simultaneously satisfy these three equations is
called a solution.
A system of linear equations may have a unique solution, or many solutions, or no solution
at all. If it has a solution (whether unique or not) the system is said to be consistent. If it has
no solution, it is called an inconsistent system.

w
If di = d2 = d^ = 0 in (i), then the system of equations is said to be a homogeneous system.
Otherwise it is called a non-homogeneous system of equations.

Flo
(i) (Cramer's ride) The solution of the system of simultaneous linear equations
a-[x + b1y = Cj

ee
a2 x + b2y = c2

Fr
Dl
is given by x = , y = D ' where for
ur
D = ^1 and D2 = al C1 provided that D * 0.
a2 b2 c2 b2 a2 c2
ks
Yo

(ii) (Cramer's Rule) The solution of the system of linear equations


oo
B

f?! x + fo-j y + Cj z = d-j


re

^ + ^2 y + c2 2 =: d2
ou

a3x + b3y + c3z = d3


ad

Dl D2
Y

is given by x = —- and z = , where


D ' y D
nd
Re

a\ bi cj dl bl ‘I «1 d\ cx fl-j b-[ d]
D = a2 b2 c2 rDl= d2 b2 c2 ,D2= a2 d2 c2 and D = a2 b2 d2 ,
Fi

a3 b3 c3 d3 b3 c3 a3 d3 c3 a3 b3 d3
provided that D * 0.
8. (a) For a system of 2 simultaneous linear equations with 2 unknowns:
(i) If D * 0, then the given system of equations is consistent and has a unique solution
given by
Di D2
x = y = —
D ' J D
(ii) If D = 0 and Dj = D2 = 0, then the system is consistent and has infinitely many
i
solutions.
(iii) If D = 0 and one of D1 and D2 is non-zero, then the system is inconsistent.
(b) For a system of 3 simultaneous linear equations in three unknowns
(i) If D * 0, then the given system of equations is consistent and has a unique solution
given by

ReadYourFlow.COM
6.100 MATHEMATICS-XII

Dt , Do
D J D D
(ii) If D = 0 and Dj =D2 =D3 = 0, then the given system of equations is consistent with
infinitely many solutions.
(iii) If D = 0 and at least one of the determinants Dj, D2/ D3 is non-zero, then the given
system of equations is inconsistent.

w
Flo
ee
Fr
for
ur
ks
Yo
oo
B
re
ou
ad
Y
nd
Re
Fi

ReadYourFlow.COM
X
5

CHAPTER z
ADJOINT AND
INVERSE OF A MATRIX

7.1 ADJOINT OF A SQUARE MATRIX


ADJOINT let A - [cijj] be a square matrix oforder n and let C,j be cofactor ofaq in A. Then the transpose of
the matrix of cofactors of elements of A is called the adjoint of A and is denoted by adj A.

low
Thus, adj A = [Cy]7 => (adj A),y = Cy;- = Cofactor of iri A.

flll aU a13 c 11 C12 C13 Cn C21 C31


If A = ^21 ^22 ^23 ' ^ = ^-21 ^22 ^23 C12 C22 C32 /

ee
fl31 fl32 a33
rF
C 31 C 32 C 33 ^13 ^23 C33

Fr
where C,y denotes the cofactor of aq in A.
for
ILLUSTRATION! Find the adjoint of matrix A = [ajj] = ^ ■
u
ks

SOLUTION We have.
Yo

Cofactor of an= s, Cofactor of fl12 = -r. Cofactor of a2i=-qand. Cofactor of a22 = P-


oo

-r -iT s -q
B

adj A =
[-‘IP -r Pi
re

RULE Jt is evidentfrom this example that the adjoint ofa square matrix oforder 2 can be easily obtained by
ou
ad

interchanging the diagonal elements and changing signs of off-diagonal elements.


Y

-2 3 - F4 -3"
If A = -5 4 , then by the above rule, we obtain adj A =
nd

L- “2 ’
Re

'11 r
Fi

ILLUSTRATION 2 Find the adjoint of matrix A = [a,;] = 2 1 -3 .


-12 3
SOLUTION Let Cjj be cofactor of fl,y in A. Then, the cofactors of elements of A are given by
c _ 1 -3 2 -3 2 1
U11 - 2 3 =9' Ci2 - = - 3, C 13 “ =5
1 3 -1 2
1 1 1 1 1 1
C2i - = -l. C22 - = 4, C23 -- 3
2 3 -1 3 -1 2
1 1 r _ 1 1 1 1
C3l = = - 4, l-32 - - n2 = 5, C33 “ = -l
1 -3 -3 2 1
T
9-3 5 9 -1 -4
adj A = -1 4-3 -3 4 5
-4 5 -1 5 -3 -1
THEOREM 1 Let Abe a square matrix of order n. Then, A (adj A) = | A | = (adj A) A.
PROOF Let A = [ajj], and let Cy be cofactor of ay in A. Then,

ReadYourFlow.COM
7.24 MATHEMATICS-XII

-8 5
18. Show that A = satisfies the equation A2 + 4A - All = O. Hence, find A 1.
2 4
3 1
19. UA = ,showthaM2 -5A +71 =0. Hence, find A-^ [NCERT, CBSE 2007]
-1 2
[4 31
20. U A = , find x and y such that A2 -xA+yl =0. Hence, evaluate A-1.
2 5
21. If A = ^ 2"
^ ,find thevalueofXsothaM2 = XA -2I.Hence, find/l_ :1. [CBSE 2007]

22. Show that A = 2 satisfies the equation x2 - 3x - 7=0. Thus, find A~L.
-1

23. Show that A =


'6 51
satisfies the equation x2 -12 x + 1 = O.Thus, find A 1
7 6

w
111
24. For the matrix A = 1 2 - 3 . Show that A3 - 6A2 + 5A + 11 I3 =0. Hence, find A-1
2-1 3

Flo [NCERT]

ee
1 0 -2

Fr
25. Show that the matrix, A = -2 -1 2 satisfies the equation. A3 - A2 - 3A -13 = 0.
3 4 1
for
ur
Hence, find A ].
'2-11
ks

26. If A = -1 2 -1 . Verify that/]3 - 67l2 + 9A-4/=0 and hence find A-^ [NCERT]
Yo
oo

1-12
eB

-8 14]
27. If A=- 4 4 7 , prove that A-1 =AT.
r

9 1-8 4
ou
ad
Y

3-3 4
28. IfA=2 -3 4 , show that A"1 = A3.
nd
Re

0 -1 1
Fi

'-1 2 O'
29. If A = -1 1 1 , show that A2 = A-1.
0 1 0
[5 41 '1 21
30. Solve the matrix equation X= ^ , where X is a 2 x 2 matrix.
1 1 1
31. Find the matrix X satisfying the matrix equation: X ^ 3 '14 7'
-2 7 7 '
3 2 -1 1 '2 -1'
32. Find the matrix X for which: X
7 5 -2 1 0 4 ‘
'2 11 rs 3“ T O'
33. Find the matrix X satisfying the equation: X
5 3 3 2 0 1 '
1 2 2'
34. If A = 2 12 , find A-1 and prove that A2-4A-5/=0.
2 2 1

S'
ReadYourFlow.COM
ADJOINT AND INVERSE OF A MATRIX 7.25

35. If /I is a square matrix of order n, prove that | A adj A\ = | A |n.


3 -1 1 1 2 -2
36. If A -1 = -15 6 -5 and B = -13 0 ,find(AB)-'. [CBSE2012]
5 -2 2_ 0 -2 1
1 -2 3'
37. If A = 0-14 ,find(AT)_1. [CBSE 2015]
-2 2 1
-1 -2 -2
38. Find the adjoint of the matrix A = 2 1-2 and hence show that A (adj A) =| A113.
2 -2 1
[CBSE 2015]

low
0 1 1
39. If A = 1 0 1 , find A and show that A -1 l (A2-31). [NCERT EXEMPLAR]
2
1 1 0

ee
ANSWERS
rF
Fr
4 -5 d -b cos a - sin a 1 - tan a/2
l-(i) (ii) (hi) (iv)
- sm a cos a tan a/2 1
for
-2 -3 -c a
ou
-3 2 2 2 3 -13 -22 11 -11
ks

2-(i) 2-3 2 (ii) -3 6 9 (iii) 4-2 2


oo

2 2-3 5-3 -1 16-8 8


Y
eB

3 -1 1 25 0 0
(iv) -15 7-5 6. 0 25 0
r
ou
ad

4-2 2 0 0 25
Y

1 + be
ro ii -b
cos 0 - sin 0 a 1 1 -5'
d

7.(i)
Re

(ii) 1 0 (iii) (iv) —


sin 0 cos 0 17 3 2
n

-c a
Fi

-5 ! 7 4 17 3 3 1 -1
8.(i) i 7 _5 1
(ii) -1 -11 6 (iii) t 1 3 1
18 7 -5 1 27
5 1 -3 4 -1 1 3
3 -1 1 0 1 -1 -8 4 4
(iv) -15 6 -5 (v) 4 -3 4 (vi) t 11 -2 -3
5-2 2 3-3 4 4 -4 0 0

1 0 0 7 -3 -3 1 1 -1
9. (i) -1 1 0 (ii) \ -3 1 1
(vh) 0 cos a sm a
2 9
0 sin a - cos a -1 0 1 -5 -1
39 1+bc
-47 — -b -2 19 -27
11. 2 14. a 15. -2 18 -25
41 -17 -c a -3 29 -42

ReadYourFlow.COM
7.26 MATHEMATICS-XII

2 -3 1 f-4 5' 1 f2 -1"


17. 18. i9. -
-1 2 42 2 8 7 [1: 3:
20. * = 9, y=14, irj -3 i r-2 2"
21. X = l,A~1 = —
2-4 3
-3 4 5
22. ir * 6 -5 1
23. 24. — 9 -1 -4
7-1 -5 -7 6 11 5 -3 -1
-9 -8 -2 3 1-1
1 -3 -14' 3 1
25. 8 7 2 26. 1 3 1 30. 31- i -2
4 4 17
-5 -4 -1 ■113
-3 2 2 9-3 5
-16 3' 9 -14' 1
32. 33. 34. - 2-3 2 36. -2 1 0
24 -5 -16 25 5

low
2 2-3 1 0 2
-9 -8 -2
37. 8 7 2
-5 -4 -1

ee
rF
Fr
________________ _____________________ HINTS TO NCERT & SELECTED PROBLEMS
15. We have to find (AB)- 1 and we are given the values of A and B~ l. But, (AB)~ 1 = B- 1 A- 1.
for
So, we need to find A~ 1.
u
Now,
ks
Yo

5 0 4
oo

A = 2 3 2 =>|A| = 5 (3-4)-0(2-2)+ 4 (4-3) = -5 + 4 = -1*0


B

1 2 1
re

So, A 1 exists.
ou
ad

Let Cjj be cofactor of Ojj in A = [Ojj]. Then,


Y

c 11= 3-4 = —1, C12 = -(2-2) = 0, C13 = 4-3 = 1,C21 = -(0-8) = 8


C22 = 5-4 = 1, C23 = -(10-0) = -10,C31 = (0-12) = -12,
nd
Re

C32 = -(10-8) = - 2, C33 = 15


Fi

^11 C12 ^13 -1 0 1 -1 8 -12


adj A = C21 ^-22 ^-23 8 1 -10 0 1 -2
C31 C32 C33 -12 -2 15 1 -10 15

1 -8 12
So, A -1 1 adjA = 0 -1 2
Ml -1 10 -15
1 3 3 1-8 12 -2 19 -27
Hence, (AB)“ 1 = B"1 A" 1 = 1 4 3 0-1 2 -2 18 -25
1 3 4 -1 10 -15 -3 29 -42
[2 31
17. We have, A =
1 2

A2 = AA = F2 31 [2 31
'7 12'
1 2 1 2 4 7

ReadYourFlow.COM
ADJOINT AND INVERSE OF A MATRIX 7.27

A2 -4A + I =
'7 12'
-4
[2 3] ri oi
+ 0 1
ro o = O
4 7 1 2 0 0
=> A'1 (A2 -4A + I) = A'1 O [Multiplying both sides by A

A'1 A2 -4A-1 A + A'1 I = O


A-4I + A~l = O
'4 01 [2 3' 2-3'
A~ 1 = 41 -A =
0 4 1 2 1 2
3 1
19. We have, A =
-1 2
3 1 3 1 8 5
A2 = AA =
-1 2 -1 2 -5 3

low
So, A2 -5A+7I =
'8 5
-5
3 1
+7
ri oi 8-15+7 5-5 + 0 ro o" = 0
-5 3 -1 2 0 1 -5 + 5 + 0 3-10 + 7 0 0

Now, A2 -5A+7I = O

ee
=> A~ ^ (A2-5A+71) = A~^0
rF [Multiplying throughout by /l *]

Fr
=> A~l A2-5A~1 A+7A~l I = O for
=> A -51 + 7A~ 1 = O
u
ks

=> 77T1 = 51 -A
Yo
oo

'5 O' 3 1 '2 -1


=> 7 A -1
0 5 -1 2 1 3
B

1 [2 -1'
re

=> ,4 1 7 |_1 3.
ou
ad

'1 1 1
Y

24. We have, A = 1 2-3


2-13
nd
Re

111 111 4 2 1
Fi

A2 = AA = 1 2 - 3 1 2 - 3 = -3 8 -14
2-1 3 J [2 -1 3 J 7 -3 14
f 4 2 ll fl 1 1 8 7 1
and. A3 = A2 A = -3 8-14 1 2-3 - 23 27 - 69
7-3 14 2-1 3 32 -13 58
A3 -6A2 +5A + ll/3
"87 1 ' 4 2 1 1 1 1 1 0 0
= -23 27 -69 -6 -3 8 -14 + 5 1 2-3 + 11 0 1 0
32 -13 58 7 -3 14 2-13 0 0 1
"8-24 + 5 + 11 7-12 + 5 + 0 1-6+5+0 0 0 0
= -23 + 18+5 + 0 27 -48 + 10 + 11 -69 + 84-15 + 0 0 0 0 = O
32-42 + 10 + 0 -13 + 18-5 + 0 58-84 + 15 + 1 0 0 0
Now, A3-6A2+5A + 11/3 =0

ReadYourFlow.COM
7.28 MATHEMATICS-XII

A-1 (A3 - 6 A2 +5,4+11/3) = A~l0 [Multiplying both sides by A *]


=> A2 -6A+5I +UA~1 = O
=> UA~] = - A2 + 6A-5I
'4 2 1 ' 1 1 1 1 0 0
=> 11A -1 - -3 8 -14 +6 1 2-3 -5 0 1 0
7 -3 14 2-13 0 0 1
-4+6-5 -2+6+0 -1+6+0 -3 4 5
=> 1171 -1 3 + 6 + 0 -8 + 12-5 14-18 + 0 9 -1 -4
-7 + 12 + 0 3-6 + 0 -14 + 18-5 5 -3 -1
3 4 5
=> A~1 = 1 9 -1 -4
11 5 -3 -1

w
2 -1 1
26. We have, A = -1 2 -1
1 -1 2

Flo
ree
[2-11 2 -1 1 6-5 5
.-. A2 = AA = -1 2 -1 -1 2 -1 -5 6 -5

F
1 1 2 1 -1 2 5-5 6
6 -5 5 2 -1 1 or 22 - 21 21
ur
and. A3 = 7l2 71 = -5 6 -5 -1 2 -1 -21 22 -21
sf
5-5 6 1-12 21 -21 22
k
Yo
oo

A3 -6A2 + 9A -4/
B

22 - 21 21 - 36 30 - 30 18-9 9 -4 0 0
re

-21 22 - 21 + 30 - 36 30 + -9 18 -9 + 0-4 0
21 -21 22 -30 30 - 36 9 -9 18 0 0-4
ou
ad

22-36 + 18-4 -21 + 30-9 + 0 21 - 30 + 9 + 0 0 0 0


Y

= -21 + 30-9 + 0 22 - 36 + 18-4 -21 + 30-9 + 0 0 0 0 = O


21 -30 + 9 + 0 -21 + 30-9 + 0 22-36 + 18-4 0 0 0
nd
Re

Now, A 3 - 6A2 + 9A - 4/ = O
Fi

=> A~ 1 (A3 - 6A2 + 9A -4/) = A~ 1 O [Multiplying both sides by A l]


=> A2-6A + 9/-4A“ 1 =0
=> 4A_ 1 = A2 -6A + 9/
6 -5 5 ' -12 6 -6 9 0 0
=> 4A_1 = -5 6 -5 + 6 -12 6 + 0 9 0
5-5 6 -6 6 -12 0 0 9

'6-12 + 9 -5 + 6 + 0 5-6 + 0 3 1 -1
=> 4A-1 = -5 + 6 + 0 6-12 + 9 -5 + 6 + 0 1 3 1
5-6 + 0 -5 + 6 + 0 6-12 + 9 -1 1 3
3 1 -1
1
A -1 1 3 1
4 -1 1 3

ReadYourFlow.COM
ADJOINT AND INVERSE OF A MATRIX 7.29

7.3 ELEMENTARY TRANSFORMATIONS OR ELEMENTARY OPERATIONS OF


A MATRIX
The following three operations applied on the rows (columns) of a matrix are called elementary
row (column) transformations.
(i) Interchange of any two rows (columns)
If Ith row (column) of a matrix is interchanged with the yth row (column), it will be denoted by
Rj <—> Rj (Cj Cy).
2 1 3 2 1 3
For example, A = -1 2 1 , then by applying R2 R3we get: B = 3 2 4
3 2 4 -1 2 1
(ii) Multiplying all elements of a row (column) of a matrix by a non-zero scalar
If the elements of zth row (column) are multiplied by a non-zero scalar k, it will be denoted by
Rj -» Rj (k) [Cj -» Q (k)]or Rj —> kRj [Q -» kCj]

w
'32-1 3 2-1
If A = 0 1 2 , then by applying R2 —> 31^2/ we obtain: B = 0 3 6
-12-3

Flo -12 -3

ee
(Hi)Adding to the elements of a row (column), the corresponding elements of any other row (column)

Fr
multiplied by any scalar k.
If k times the elements of row (column) are added to the corresponding elements of the
for
row (column), it will be denoted by Rj -> Rj + k Rj (Cj -» Q + /c Cy).
ur
'2 13 1'
ks

IfA=-l -1 0 2, then the application of elementary operation R3 R^ + 2 R^ gives the


Yo

0131
oo

'2 131'
B

matrix B = -1 -1 0 2.
re

4 3 9 3
ou
ad

If a matrix B is obtained from a matrix A by one or more elementary transformations, then A and
Y

B are equivalent matrices and we write A ~ B.


'1 2 3 4'
nd
Re

Let A = 2 1 4 3 . Then,
Fi

3 12 4
12 3 4
A ~ 1-11-1 Applying R2 R2 + (-1) R\
3 12 4
12 3 1
=> A~ 1-11 -2 Apptying C4 ->C4 +(-l)C3
112 2
An elementary transformation is called a row transformation or a column transformation
according as it is applied to rows or columns.
ELEMENTARY MATRIX A matrix obtained from an identity matrix by a single elementary operation
(transformation) is called an elementary matrix.
1 3 0 0 0 1 1 0 0
For example, 0 1 0,01 0 0 0 1
0 0 1 1 0 0 0 1 0

ReadYourFlow.COM
7.30 MATHEMAT1CS-XII

are elementary matrices obtained from /3 by subjecting it to the elementary transformations


Rl -> + 3 R2,Ci <r^ C3 and R2 ^ R3 respectively.
'1-12 0'
Consider a matrix A = -3 5 12. Let B be a matrix obtained from A by applying
2 1 5 3j
elementary transformation R2 ^ R2 + 2 R1 and let £ be the elementary matrix obtained from
(as there are three rows in A) by subjecting it to the same transformation. Then,
1-12 0 '1 0 O'
B = -1 3 5 2 and £=210
2 15 3 0 0 1
"l 0 O' 1-12 0 1-12 0
Now, £A = 2 1 0 -3 5 12 -1 3 5 2 =B
0 0 1 2 15 3 2 15 3

w
Thus, we find that B can be obtained from A by pre-muItiplying with an elementary matrix
obtained from J3 by subjecting it to the same elementary row transformation.

Flo
Let C be a matrix obtained from A by the application of transformation C3 -> C3 + 2 C2, and let
£ be the elementary matrix obtained from I4 (as there are four columns in A) by subjecting it to

ee
the same column transformation. Then,

Fr
'1 0 0 0'
1-10 0
0 12 0
C = -3 5 11 2 and £ =
for
ur
0 0 10
2 17 3
0 0 0 1
ks

TOGO'
1-12 0 1-10 0
Yo

0 12 0
oo

Now, A£ -3 5 12 -3 5 11 2 = C
0 0 10
B

2 15 3 2 17 3
0 0 0 1
re

Thus, C can also be obtained from A by post-multiplying with an elementary matrix obtained
from /4 by subjecting it to the same elementary column transformations.
ou
ad

We now state the results obtained in the above discussion as two theorems, the proofs of which
Y

are beyond the scope of this book.


nd
Re

THEOREM 1 Every elementary row (column) transformation of an mxn matrix (not identity matrix)
can be obtained by pre-multiplication (post-multiplication) with the corresponding elementary matrix
Fi

obtained from the identity matrix lm (/„) by subjecting it to the same elementary row (column)
transformation.
THEOREM 2 LetC = AB be a product of two matrices. Any elementary row (column) transformation of
AB can be obtained by subjecting the pre-factor A (post-factor B) to the same elementary row (column)
transformation.
2 1 -3' 4 0
ILLUSTRATION 1 Verify Theorem 2, if A = 0 1 -1 , B=1 2 and the elementary
1 2 -1 2 3
row-operation is R2 —> R2 + (- 2) Rj.
SOLUTION We have,
'2 1 - 311"4 O' 3 -7
AB = 0 1-1 12 = -1 -1
1 2 -1JL2 3 4 1
Applying R2 R2 + (- 2) Rl on AB, we get

ReadYourFlow.COM
ADJOINT AND INVERSE OF A MATRIX 7.31

3 -7
AB-----7 13 = P (say) -(i)
4 1.
Applying R2 R-i + (-2) on A, we get
'2 1-3'
-4 -1 5 =Q(say) ...(h)
12-1
2 1-3 4 0 3 -7
Now, QB = -4 -1 5 1 2 -7 13 = R (say) ...(hi)
1 2 -1 2 3 4 1
From (i) and (hi), we get P = R.
Hence, the theorem is verified.
ILLUSTRATION 2 Use elementary column operation C2 -» C2 -2Cl in the matrix equation

w
4 2 '1 21 F2 01
ICBSE 2014]
3 3J L° 3J L1 1

Flo
SOLUTION If A, B, C are three matrices such that C = AB, then any elementary row (column)

ree
transformation of C can be obtained by subjecting the pre-factor (post-factor B) to the same
elementary row (column) transformation. Therefore, given matrix equation after applying

F
C2 -J>C2 ~2 Ci, becomes
'4 2-2x41 fl 21 [2 0-2x2' or
ur
3 3-2x 3J [0 3j [l 1-2x1
sf
'4 -6l_ri 21 \2 -4"
k
Yo

or. 3 -3j“[o 3J |_1 -1


oo
B

ILLUSTRATION 3 Apply elementaiy transformation R2 —> R2- SRj in the matrix equation
re

"11 -6l_fl 31 [2 0 1
6 -4j=|_0 2J [3 -2
ou
ad

SOLUTION Any elementary row transformation on the LHS of the given equation is obtained
Y

by subjecting the pre-factor on the RHS of the same transformation. Therefore, given matrix
nd

equation, by applying R2 ^ R2 - 31^, becomes


Re

' 11 -61 f 1 3l|"2 0"


Fi

-27 14 = -3 -7 3 -2

7.3.1 METHOD OF FINDING THE INVERSE OF A MATRIX BY ELEMENTARY TRANSFORMATIONS


Let A be a non-singular matrix of order n. Then A can be reduced to the identity matrix by a
finite sequence of elementary transformations only. As we have discussed every elementary
row transformation of a matrix is equivalent to pre-multiplication by the corresponding
elementary matrix. Therefore, there exist elementary matrices £}, £2 / • • • / El such that
(E/t Efc- 1 ••• £2 £l) A=In
=> {Ek Ek_i... £2 Ei) AA -1 = ^ A -1 [Post-multiplying by A ^

=> (EkEk-i~-E2Ei) /„ = A -1 [v /„ A'1 = A-1 and AA"1 = /„]

=> A -1 = (El Efc-1 ••• £2 Ei) ln.

Following algorithm maybe used for finding the inverse of a non-singular matrix by elementary
row transformations.

ReadYourFlow.COM
7.32 MATHEMATICS-XII

ALGORITHM
STEP I Obtain the square matrix, say A.
STEP II Write A = ln A
STEP in Perform a sequence of elementary roiu operations successively on A on the LHS and the
pre-factor In on the RHS till we obtain the result In = BA.
STEP IV Write A -1 = B.
ILLUSTRATIVE EXAMPLES

LEVEL-1

EXAMPLE 1 Find the inverse of the matrix A = ri si


2 7 , using elementary row transformations.

SOLUTION We know that


A = IA

w
'1 3] 1 01
or,
2 7 0 1
"1
0 1
31 ' 1 0~
-2 1
A

Flo [Applying R2^>R2+ (- 2) Ra]

ee
n oi 1 ! -? A

Fr
=> [Applying R1-+Rl+ (- 3) R2]
0 1 -2 1 for
'7-3'
ur
A"1
-2 1 ’
ks

EXAMPLE 2 By using elementary row transformations find the inverse of the matrix A = '1 2
Yo

3 7 '
oo

SOLUTION We know that


A = IA
eB

fl 2"
or.
3 7 0 1
°1 A
r
ou
ad

=>
'1 2' 1 ol
Y

0 1 -3 1 [Applying R2 ^2 + (" 3) Rl\


nd

1 01 _ r 7 -21
Re

=> 0 1 “ -3 1 A [Applying R1^R1+ (- 2) R2]


Fi

A'1 '7-2'
-3 1 •
We may use the following algorithm to find the inverse of a square matrix of order 3 by using
elementary row transformations.
ALGORITHM
STEP I Introduce unity at the intersection offirst row and first column either by interchanging two
rows or by adding a constant multiple of elements of some other row to thefirst row.
STEP II After introducing unity at (1, l)th place introduce zeros at all other places in first column.
STEP HI Introduce unity at the intersection of 2nd row and 2nd column with the help of 2nd and 3rd
rows.
STEP IV Introduce zeros at all other places in the second column except at the intersection of2nd row and
2nd column.
STEP V Introduce unity at the intersection of 3rd row and third column.

ReadYourFlow.COM
ADJOINT AND INVERSE OF A MATRIX 7.33

STEP VI Finally introduce zeros at all other places in the third column except at the intersection of third
row and third column.
3 -1 -2
EXAMPLE 3 Using elementary row transformation find the inverse of the matrix A = 2 0 -1
3-5 0
SOLUTION We know that
A =IA
3 -1 -2] 1 0 0
or. 2 0 -1 = 0 1 0 A
3-5 0 0 0 1
1 -1 -1 1 -1 0
=> 2 0-1 0 10 A [Applying Ri~^ R-[- R2]
3-5 0 0 0 1

low
1 -1 -1 1-10
=> 0 2 1 = —2 3 0 A [Applying R2 —> R2 (— 2) R3 R3 + (— 3) Rj]
0 -2 3 -3 3 1

ee
1 -1 -1 1 -1 0
rF
Fr
=> 0 1 1/2 -1 3/2 0 A [Applying R2 -> R2(l/2)]
0-2 3 -3 3 1
for
1 0 -1/2 0 1/2 0
ou
01 1/2 = -1 3/2 0 A [Applying R-l -» R} + R2 and R3 -> R3 + 2R2]
ks

0 0 4J -5 6 1
oo
Y

'1 0 -1/2 0 1/2 0


B

0 1 1/2 -1 3/2 0 A [Applying R3->1/4R3]


re

0 0 1 -5/4 3/2 1/4


ou
ad

1 0 0 -5/8 5/4 1/8


'Applying R2-» Rj + (1/2) R3
Y

0 1 0 -3/8 3/4 -1/8 A & R2 R2 + (1/2) R3]


0 0 1 -5/4 3/2 1/4
nd
Re

-5/8 5/4 1/8


Fi

Hence, A"1 -3/8 3/4 -1/8


-5/4 3/2 1/4
1 2 -2
EXAMPLE 4 Fine/ the inverse of the matrix A = -1 3 0 by using elementary row
transformations. 0 2 1 [CBSE 20101
SOLUTION We know that
A = IA
1 2 -2"| Fl 0 0"
or. -1 3 0 = 0 1 0 A
0 -2 ij [0 0 1
1 2 -2] [1 0 O'
=> 0 5 -2 = 1 1 0 A [Applying R2 -> R2 + R2]
0-2 1 001

ReadYourFlow.COM
7.34 MATHEMATICS-XII

1 2 -2 1 0 0
=> 0 1 0 1 1 2 A [Applying R2 R-2 + 2R3]
0 -2 1 0 0 0
1 0 -2 -1 -2 -4"
=> 0 1 0 1 1 2 A [Applying ^ R-! + (- 2) R2 , R3 R3 + 2R2]
0 0 1 2 2 5
1 0 0 3 2 6
0 1 0 1 1 2 A [Applying R1-> R1 + 2R3]
0 0 1 2 2 5
3 2 6
Hence, A -1 1 1 2
2 2 5
EXERCISE 7.2

w
LEVEL-1
Find the inverse of each of the following matrices by using elementary row transformations:
'7 1
2.
[5

Flo
21 2

ee
4 -3 3. [CBSE 20171
2 1 2 -1

Fr
0 1 2
[2 5] 3 101
4. [CBSE2010] 5. 6. 1 2 3
1 3 2 7
for
ur
3 1 1
2 0 -1 2 3 1 3 -3 4"
ks

7. 5 1 0 8. 2 4 1 9. 2 -3 4
Yo
oo

0 1 3 3 7 2 0 -1 1
eB

1 2 0 2 -1 3 1 1 2
10. 2 3 -1 11. 1 2 4 12. 3 1 1
1 -1 3 1 1
r

3 2 3 1
ou
ad

2 -1 4" 3 0 -1
Y

13. 4 0 2 [CBSE 2008] 14. 2 3 0 [CBSE 2009]


3-2 7 0 4 1
nd
Re

1 3 -2 -1 1 2
Fi

15. -3 0 1 [CBSE 2011] 16. 1 2 3 [CBSE 2012]


2 1 0 3 1 1
ANSWERS

- JLf3 1 2.
1-2" 1 ri 2"
25 I 4 -7 3. -
-2 5 5 2 -1

3 -5' 1/2 -1/2 1/2


7 -10"
4. 5. 6. -4 3 -1
-1 2 -2 3
5/2 -3/2 1/2
3 -1 1 1 1 -1 1-1 0"
7. -15 6 -5 8. -1 1 0 9. -2 3 -4
5-2 2 2-5 2 -2 3-3

ReadYourFlow.COM
ADJOINT AND INVERSE OF A MATRIX 7.35

-4/3 1 1/3 -2 4 -10 1 -2_ 5 -1


-1
1U. 7/6 -1/2 -1/6 11. 11 -7 -5 12. — -1-3 5
30 -5 -5 5 11 7 -1 -2
5/6 -1/2 1/6

-2 1/2 1 3-4 3 1 -2 -3 1 -1 1
13. 11 -1 -6 14. -2 3 -2 15. -2 4 7 16. -8 7 -5
4 -1/2 -2 8 -12 9 3 5 9 5-4 3

_____________________________________ VERY SHORT ANSWER QUESTIONS (VSAQs)


Answer each of the following questions in otic word or one sentence or as per exact requirement of the
question:
-3 4
1. Write the adjoint of the matrix A = “
-2 •
2. If A is a square matrix such that A (adj A) =51, where I denotes the identity matrix of the

w
same order. Then, find the value of | A|.
3. If A is a square matrix of order 3 such that | A| =5, write the value of |adjA|. [CBSE2009]

Flo
4. If A is a square matrix of order 3 such that | adj A| = 64, find | A|.

ee
5. If A is a non-singular square matrix such that| A| =10, find |A~ 1|.

Fr
6. If A, B, C are three non-null square matrices of the same order, write the condition on A
such that AB = AC => B=C.
for
ur
7. If A is a non-singular square matrix such that A- 1 = _ ^ 31 , then find (A ^ ^
-1
ks
Yo

'2 3 1 -2*
oo

8. If adj A = 4 1 and adj B = , find adj AB.


-1 -3 1
B

9. If A is a symmetric matrix, write whether A is symmetric or skew-symmetric.


re

10. If A isasquare matrix of order 3 such that | A | = 2, then write the value of adj (adj A).
ou
ad

11. If A isa square matrix of order 3 such that | A | = 3, then find the value of | adj (adj A) |.
Y

12. If A isa square matrix of order 3 such that adj (2A) = k adj (A), then write the value of k.
nd

13. If A isasquarematrix, then write the matrix adj (A ) - (adj A) .


Re
Fi

14. Let A be a 3 x 3 square matrix such that A (adj A) = 2 /, where I is the identity matrix. Write
the value of | adj A |.
15. If A is a non-singular symmetric matrix, write whether A~1 is symmetric or
skew-symmetric.
cos 0 sin 0" [k 0]
16. If A = and A (adj A) = , then find the value of k.
- sin 0 cos 0 0 k
17. If A is an invertible matrix such that | A- 1| = 2, find the value of | A|.
'5 0 O'
18. If A is a square matrix such that A (adj A) = 0 5 0 , then write the value of |adj A|.
0 0 5
3"
m _ ^ be such that A -l = k A, then find the value of k.

20. Let A be a square matrix such that A - A + / = O, then write A in terms of A.

ReadYourFlow.COM
7.36 MATHEMATICS-XII

2 -3 5
21. If Qy is the cofactor of the element Ojj of the matrix A = 6 0 4 , then write the value of
1 5 -7
a32 C32- [CBSE2013]

22. Find the inverse of the matrix


r ^3 -2
^ .

cos 0 sin 0’
23. Find the inverse of the matrix
- sin 0 cos 0
1 - 3"
24. IM = q , write adj A.
2
a b 1 0
25. lfA = /B= , find adj (AB).
c d 0 1

low
3 ^ , then find | adj A\.
26. IfA = [CBSE 20101
2 - 3j
"2 3 ~
27. IfA = 5 _2 'write A~ 1 in terms of A. [CBSE 2011]

ee
F2 5'
28. Write A 1 for A =
rF
Fr
1 3
29. Use elementary column operation C2 -> C2 + 2C! in the following matrix equation :
for
[2 ll [3 Ilf 1 01
2 OJ -[2 0j[-l 1 [CBSE 2016]
ou
ks

30. In the following matrix equation use elementary operation K2 -> R2 + Ri and the equation
thus obtained:
oo
Y

"2 31 n 0' '8 -3


eB

14 2-1 9 -4 [CBSE 2017]

ANSWERS
r
ou
ad

-2 -4 1
1. 2. 5 3. 25 4. ±8 5. —
-7-3
Y

10
5-21 -6 5
6. A must be invertible or | A | ^ 0
d

8. 9. symmetric
7* [3
Re

-1 -2 -10
n
Fi

10. 2A 11. 81 12. 4 13. Null matrix 14. 4


1 1
15. symmetric 16. 1 18. 25 19. —
2 19
[5 21 cos 0 - sin 0"
20. A-1=(I-A) 21. HO 22. 23.
7 3 sin 0 cos 0_
[ 0 31 d -b
24. [_2 ! 25.
-c a
26. -11 27. A-1 =—A
19
3 -5 2 5] [3 1 1 2 2 31 fl O' 8 -3'
28. A-1 = 29. 30.
-1 2 2 4 2 0 -1-1 3 7 2 -1 17 -7
HINTS TO NCERT& SELECTED PROBLEMS

11. adj (adj A) =| A | n-2 A ••• | adj (adj A) H A p'1)2


12. adj (kA) =kn~^ adj (A)
14. v A (adj (A)) = | A | / A (adj A) = 21 => | A | = 2
Now, | adj A | =| A [' -1 => | adj A | = 2 3-1 = 4

ReadYourFlow.COM
ADJOINT AND INVERSE OF A MATRIX 7.37

MULTIPLE CHOICE QUESTIONS (MCQs)

1- If ,4 is an invertible matrix, then which of the following is not true


(a) (Az)~1 = (A~1)2 (bJIA^hlAI -1
(c) (A7*)-1 = (A-1? (d)\A\ * 0
2. If /I is an invertible matrix of order 3, then which of the following is not true
(a) | adj .4 | =|/i|2 (b) (A"1)-1 = /l
(c) If BA = CA, then B ^ C, where B and C are square matrices of order 3
(d) (AB) -1 = B-1 A-1 , where B = [bjj] 3 x 3 and | B | ^ 0
3 4 -2 - 21
3. If A = , then (A + B) -1
2 4 -1
(b) A" 1 + B' 1

w
(a) is a skew-symmetric matrix
(c) does not exist (d) none of these

Flo
4. IfS = \ , then adj A is
c a

ee
-d -b d -b d b

Fr
(a) (b) (c)
-c a - c a c a
5- If A is a singular matrix, then adj A is or
ur
(a) non-singular (b) singular (c) symmetric (d) not defined
f
6- If A, B are two n x n non-singular matrices, then
ks
Yo

(a) AB is non-singular (b) AB is singular


oo

(c) (AB)-1 =A-1 B-1 (d) (AB) 1 does not exist


B

'a 0 O'
re

7. If A = 0 a 0 , then the value of | adj A | is


0 0 (7
ou
ad

(b) a9 (c) a6 (d) a2


Y

(a) a27
1 2 -1
nd
Re

8. If A = -1 1 2 , then det (adj (adj A)) is


2 -1 1
Fi

(a) 144 (b) 143 (c) 142 (d) 14


9- If B is a non-singular matrix and A is a square matrix, then det (B- 1 AB) is equal to
(a) Det (A-1) (b) Det (B- 1‘) (c) Det (A) (d) Det (B)
AO O'
10. For any 2x2 matrix, if A (adj A) = , then | A | is equal to
0 10
(a) 20 (b) 100 (c) 10 (d) 0
11- If A5 = O such that A” * 7 for 1 < n < 4, then (7 - A)- 1 equals
(a) A4 (b) A3 (c) 7 + A (d) none of these
-1
12. If A satisfies the equation x3 -5x2 + 4x + A, = 0, then A exists if
(a) A. * 1 (b) * 2 (c) A. * -1 (d) X * 0
13- If for the matrix A, A 3 = 7, then A- 1 =
(a) A2 (b) A3 (c) A (d) none of these

ReadYourFlow.COM
7.38 MATHEMATICS-XII

14. If A and B are square matrices such that B = -A 1 BA, then (A + B)2 =
(a) O (b) /i2 + B2 (c) A2 + 2AB + B2 (d) A + B
2 0 0
15. If /l = 0 2 0 , then A5 =
0 0 2
(a) 5A (b) WA (c) 16A (d) 32A
16. For non-singular square matrix A, B and C of the same order (AB 1 Q 1 =
(a) 2T1 BC -1 (b) C
-1 B~1A~l (c) CBA -1 (d) C"1 BA -1
5 10 3
17. The matrix - 2 - 4 6 is a singular matrix, if the value of b is
-1 -2 b

low
(a) -3 (b) 3 (c) 0 (d) non-existent
18. If d is the determinant of a square matrix A of order n, then the determinant of its adjoint is
(a) d" w d"-1 (c) d n+ 1 (d) d

ee
19. If A is a matrix of order 3 and | A | = 8, then | adj A | =
rF
Fr
(a) 1 (b) 2 (c) 23 (d) 26
20. If A2 - A + 7 = O, then the inverse of A is for
(a) A"2 (b) A + 7 (c) I-A (d) A-7
ou
21. If A and B are invertible matrices, which of the following statement is not correct,
ks

(a) adj A = | A | A~ 1 (b) det(A_1) = (det A)"1


oo
Y

(c) (A + B)* 1 = A" 1 + B“ 1 (d) (AB)* 1 = B~ 1 A -1


B
re

22. If A is a square matrix such that A2 = 7, then A - 1 is equal to


ou

(a) A + 7 (b) A
ad

(c) 0 (d) 2A
1 0
Y

23. Let A =
3 = 0 2 and X be a matrix such that A = BX, then X is equal to
nd
Re

1 2 4 1 -2 4 41
(a) x
2 3 -5 (b)2[ 3 5 M 3 -5 (d) none of these
Fi

'2 3
24. If A = j^5 2 be such that A -1 = kA, then k equals

(a) 19 (b) 1/19 (c) -19 (d) -1/19


1 1 2
1
25. If A = - 2 1 - 2 satisfies A A =1, then x + y =
3 x 2 y]
(a) 3 (b) 0 (c)-3 (d) 1
1 0 1
26. If A = 0 0 1 , then nl +bA + 2 A2 equals
a b 2
(a) A (b) - A (c) abA (d) none of these
1 - tan 0 1 tan 0l _ i l “Hthen
27. If
tan 0 1 -tan 0 1 b a

ReadYourFlow.COM
ADJOINT AND INVERSE OF A MATRIX 7.39

(a) a = l,b = l (b) fl = cos 2 0, & = sin 2 0


(c) = sin 2 0, fr = cos 2 0 (d) none of these
28. If a matrix A is such that 3A3 + 2A2 + 5A+1 = 0, then A 1 is equal to
(a) -(3A2 + 2A+5) (b) 3A2 +2A +5

(c) 3A2-2A-5 (d) none of these


-1 is equal to
29. If A is an invertible matrix, then det (A-i)
1
(a) det (A) (b) (c) 1 (d) none of these
det (A)
-1
30. If A = 3 , then A" =
-2

w
”1 0
(a)
0 1], if w is an even natural number

(b)
fl 0
0 1
, if n is an odd natural number

Flo
ee
f-i ol ,

Fr
(c) if n e N
0 1
(d) none of these
for
ur
x 0 0
31. If x, i/, z are non-zero real numbers, then the inverse of the matrix A = 0 y 0 , is
ks

0 0 2
Yo
oo

x_1 0 0 x"1 0 0
B

(a) 0 y~1 0 (b) xyz 0 y~1 0


re

0 0 2-l 0 0 z-l
ou
ad

*00 1 0 0
Y

1
(c) — 0 y o (d) 0 1 0
*yz 0 0 2 *yz o o i
nd
Re

ANSWERS
Fi

1. (a) 2. (c) 3. (d) 4. (b) 5. (b) 6. (a) 7. (c) 8. (a) 9. (c)


10. (c) 11. (d) 12. (d) 13. (a) 14. (b) 15. (c) 16. (d) 17. (d) 18. (b)
19. (d) 20. (c) 21. (c) 22. (b) 23. (a) 24. (b) 25. (c) 26. (d) 27. (b)
28. (d) 29. (b) 30. (a) 31. (a)
SUMMARY
1. If A = [cijj] is a square matrix of order n and Cjj denote the cofactor of cijj in A, then the
transpose of the matrix of cofactors of elements of A is called the adjoint of A and is denoted
by adj A i.e. adj A = [C^]7'.

flll aU a13 C11 C21 C31


If A = 7721 a22 a23 ' then adj A = C^2 ^-22 ^ 32
7731 77 32 7733_ fl3 C23 C33_
2. The adjoint of a square matrix of order can be obtained by interchanging the diagonal
elements and changing the signs of off-diagonal elements.

ReadYourFlow.COM
r

7.40 MATHEMATICS-XII

a b d -b
i.e. if A = then adj A =
c d ' -c a
3. IM is a square matrix of order n, then A (adj A) = \A\ In = (adj A) A.
4. Following are some properties of adjoint of a square matrix:
If A and B are square matrices of the same order n, then
(i) adj (AB) = (adj B) (adj A) (ii) adj ^4r = (adj A)T (iii) adj (adj A) = \ A \ n 2 A
(v) | adj (adj A)\ =\ A\(n~ 1)2
(iv) | adj A\=\A](1~l
5. A square matrix A of order n is invertible if there exists a square matrix B of the same order
such that AB = ln = BA.
In such a case, we say that the inverse of matrix A is B and we write A- 1 = B.
Following are some properties of inverse of a matrix:

w
(i) Every invertible matrix possesses a unique inverse.
(ii) If A is an invertible matrix, then (A- ^ -1 = A.

Flo
(iii) A square matrix is invertible iff it is non-singular.

ee
(iv) If A is a non-singular matrix, then A~1 = —(adj A).

Fr
(v) If A and B are two invertible matrices of the same order, then (AB) 1 = B 1 A l.
for
ur
(vi) If A is an invertible matrix, then (AT) -1 =(A-y.
(vii) The inverse of an invertible symmetric matrix is a symmetric matrix.
ks

1
Yo

(viii) If A is a non-singular matrix, then | A -1


oo

Ml
B

6. The following are three operations applied on the rows (columns) of a matrix:
re

(i) Interchange of any two rows (columns).


(ii) Multiplying all elements of a row (column) of a matrix by a non-zero scalar.
ou
ad

(iii) Adding to the elements of a row (column), the corresponding elements of any other
Y

row (column) multiplied by any scalar.


7. A matrix obtained from an identity matrix by a single elementary operation is called an
nd
Re

elementary matrix.
Fi

8. Every elementary row (column) operation on an m x n matrix (not identity matrix) can be
obtained by pre-multiplication (post-multiplication) with the corresponding elementary
matrix obtained from the identity matrix lm (In) by subjecting it to the same elementary row
(column) operation.
9. In order to find the inverse of a non-singular square matrix A by elementary operations, we
write A = 7A.
Now we perform a sequence of elementary row operations successively on A on the LHS
and the pre-factor I on RHS till we obtain I = BA.
The matrix B, so obtained, is the desired inverse of matrix A.

ReadYourFlow.COM
CHAPTER 8
SOLUTION OF SIMULTANEOUS
LINEAR EQUATIONS

8.1 INTRODUCTION
Consider the following system of m linear equations in n unknowns:
an xl + al2 x2 + • + alnxn=b\

low
#21 ^1 ■*" #22 x2 .+ a2nxn=b2
•••(i)

am\xl + am2x2+ ••• + =K

ee
rF
This system of equations can be written in matrix form as

Fr
#H #12 ... #]« x\
b2
fl21 a22 ••• a2n x2
for
: :
ou
fynl am2 ••• atnn
ks

h
oo

#11 #12 ... #1,,


b2
Y

fl21 fl22 ••• a2n x2


B

or AX = B, where A = ,X = and B = ■.
re

fl?«2 ••• amn m xn ^ j «X1 «x1


ou
ad

The mxn matrix A is called the coefficient matrix of the system of linear equations.
Y

ILLUSTRATION Express the following system of simultaneous linear equation as a matrix equation:
2x + 3y-z = 1
nd
Re

x + y + 2z = 2
Fi

2x-y + z = 3
SOLUTIONWe have,
2x + 3y -2 = 1
x + y + 2z = 2
2x~y+z = 3
This system of equations can be written in matrix form as
'2 3 -1] lYl [Y
1 1 2y = 2
2 -1 1 z 3
2 3-1 1
or. AX = B, where A = 1 1 2 , X = y and 8=2
2 -1 1 2 3
SOLUTION A set of values of the variables x^, x2,... xn ivhich simultaneously satisfy all the equations is
called a solution of the system of equations.

ReadYourFlow.COM
8.2 MATHEMATICS-XII

For example, x = 2, y = -3 is a solution of the system of linear equations


3x + i/ = 3
2* + y = 1
because 3 (2) + (-3) = 3 and, 2(2) + (-3)=l.
CONSISTENT SYSTEM If the system of equations has one or more solutions, then it is said to be a
consistent system of equations, othenvise it is an inconsistent system of equations.
For example, the system of linear equations
2x + 3y = 5
4x + 6y = 10
is consistent, because x = 1, y = 1 and x = 2, y = l/3 are solutions of it.
However, the system of linear equations
2x + 3y =5

w
4x + 6y = 9
is inconsistent, because there is no set of values of x, y which satisfy the two equations
simultaneously.

Flo
HOMOGENEOUS AND NON-HOMOGENEOUS SYSTEMS A system of equations AX=B is called a

ee
homogeneous system ifB = 0. Othenvise, it is called a non-homogeneous system of equations.

Fr
For example, the system of equations
2x + 3y = 0
for
ur
3x-y = 0
is a homogeneous system of linear equations whereas the system of equations given by
ks

2x + 3y =1
Yo
oo

3x-y =5
B

is a non-homogeneous system of linear equations.


re

8.2MATRIX METHOD FOR THE SOLUTION OF A NON-HOMOGENEOUS SYSTEM


In the previous section, we have seen that a system of simultaneous linear equations can be
ou
ad

expressed as a matrix equation. In this section, we shall discuss about a method for solving a
Y

system of non-homogenous simultaneous linear equations in which the number of unknowns is


same as the number of equations. In this method, we will use the inverse of the coefficient
nd
Re

matrix. So, it is also known as matrix method.


Fi

THEOREM 1 IfA is a non-singular matrix, then the system of equations given by AX = B has the unique
solution given byX = A~ lB.
PROOF We have, AX = B, where | A | * 0.
Now, | A | * 0. So, A- 1 exists.
Pre-multiplying both sides of AX = B by A~ ^ we get
A-1 (AX) = A_1B
=> (A_1A)X = A~lB
=> IX = A~lB
=> X = A~1B
Thus, the system of equations AX = B has a solution given by X = A” 1B.
Uniqueness: If possible, let X-^ and X2 be two solutions of AX = B. Then,
AX| = B and AX2 = B
=> AX! = AX2

ReadYourFlow.COM
SOLUTION OF SIMULTANEOUS LINEAR EQUATIONS 8.3

=> A~l(AXl) = A~l{AX2)


=> (A~ 1 A) X1 = (A~ 1 A) X2
=> IX^ = IX2
Xj = X2.
Hence, the given system of equations has the unique solution given by X = A~ lB.
Q.E.D.
In the above theorem, we have proved that a non-homogenous system AX = Bofn simultaneous
linear equations with n-unknowns has the unique solution given by X = A_1B, if A is a
non-singular matrix. Now, a natural question arises, what happens when v4 is a singular matrix?
In order to answer this, let us consider the following system of equations:
2x + y = 3
Ax + 2y = 6

w
This system of equations can be written in matrix form as
'2 1] [x 3
, or, AX = Br where A = * and B - ~3~
L6
4 2 [yj 6

Flo 4 2

ee
Clearly, | A | = 0. Also, the system of equations has infinitely many solutions as the two
equations represent coincident lines in xy-plane.

Fr
Now, consider the following system of equations:
2x + y - 3 or
ur
Ax + 2y =5
sf
For this system of equations also the determinant of the coefficient matrix A is zero i.e. A is a
k

singular matrix. But, the system has no solution i.e. it is an inconsistent system of equations, as
Yo
oo

the lines represented by the two equations are non-coincident parallel lines.
B

It follows from the above discussion that the system of equations AX = B may be inconsistent
re

or it may be consistent with infinitely many solutions when the coefficient matrix A is singular.
We now state and prove the following criterion for the consistency or inconsistency of a
ou
ad

non-homogenous system of linear equations.


Y

THEOREM 2 (Criterion of consistency) Let AX = B be a system ofn-linear equations in n unknowns.


nd

(i) 1/ | A | * 0, then the system is consistent and has the unique solution given byX = A~ 1B.
Re

(ii) I/| A | = 0 and(adj A) B = O, then the system is consistent and has infinitely 7nany solutions.
Fi

(iii) 1/ | A | = 0 and (adj A) B * O, then the system is inconsistent.


PROOF (i) See Theorem 1
(ii) We have.
AX = B, where | A | =0.
=> (adj A) (AX) = (adj A) B
=> ((adj A) A)X = (adj A) B
=> (I A | I„)X = (adjA)B [v (adj A) A = |A|y
=> | A | X = (adj A) B
If | A | =0 and (adj A) B =0, then | A | X = (adj A) B is true for every value of X.
So, the system of equations AX = B is consistent and it has infinitely many solutions.
(iii) If | A | = 0 and (adj A)B * O, then the equation | A | X = (adj A) B is not true because its
LHS is always a null matrix whereas the RHS is non-null matrix.
So, the system is inconsistent. Q.E.D.
The above discussion suggests the following algorithm to solve a system of simultaneous linear
equations.

ReadYourFlow.COM
8.6 MATHEMATICS-XII

1 2 1
Now, | ^4 | = 1 0 3 = 1 (0 + 9) - 2 (0 - 6) + 1 (- 3 - 0) = 9 + 12 - 3 = 18 * 0
2-3 0

So, the given system of equations has a unique solution given by X = A 1 B.


Let Cjj be the co-factors of elements Ojj in A = [cijj]. Then,
1+ 1 0 3 1 3
Cn = (-1) -3 0
= 9, C12 = (-D 1+2 2 0
= 6,

1+ 3 1 2+ 1 2 1
C13 = (-D 2
o
-3
= - 3, C21 = (-1) -3 0
= - 3,

1 1 1
= (-1) 2+ 2
= -2, ^23 = (-D 2+ 3 ? =7,
2 0 2 -3

2 1 1 1

w
3+ 1 3+2
C31 = (-1) = 6, C32 = (-1) = -2,
0 3 1 3
3+3 1 2

Flo
and. C33 = (-1) = -2
1 0

ee
T
9 6-3 9-3 6

Fr
adjA = -3-2 7 6 -2 -2
6 -2 -2 -3 7-2 for
ur
9-3 6
1
A -1 adj A = — 6 -2 -2
Ml
ks

18 -3 7-2
Yo
oo

Now, X = A-1 B
B

9-3 6 7 63 -33 +6 36 2
re

1 1 1
=> X= — 6 -2 -2 11 42 -22 -2 18 1
18 -3 7-2 1 18 -21 +77 -2 18 54 3
ou
ad

x
Y

2
=> y = 1 x — 2, y=l and z = 3
nd

z 3
Re
Fi

Hence, x = 2, y = 1 and z = 3 is the required solution.


Type II SOLVING THE GIVEN SYSTEM OF EQUATIONS WHEN THE COEFFICIENT MATRIX IS
SINGULAR
EXAMPLE 4 Use matrix method to examine the following system of equations for consistency or
inconsistency:
4x - 2y = 3, 6x - 3y = 5
SOLUTION The given system of equations can be written as
A X = B, where A = f -2 , X = * and B = 3
6 -3 Ly 5 '

Now, |A| = ^ -2
= -12 + 12 = 0
-3

So, the given system of equations is inconsistent or it has infinitely many solutions according as
(adj A) B * O or, (adj A) B = O respectively.
Let Cq be the co-factors of elements ajj in A = [Ojj ]. Then,

ReadYourFlow.COM
SOLUTION OF SIMULTANEOUS LINEAR EQUATIONS 8.7

Cii=(-D
1+ 1
(- 3) = - 3, C12 =(-l) 1+2 6 = - 6, C21 =(-l) 2+ 1 (-2) =2

2+2
and. C22=(-l) (4) =4
-3 -66]T _ f-3 2]
adj A =
2 4J ~ L_6 -94J +10 1
-3 2
So, (adj A) B =
-6 4 ][;]= -18 +20 2
Hence, the given system of equations is inconsistent.
EXAMPLE 5 Show that thefollowing system of equations is consistent.
2x -y + 3z = 5, 3x + 2y-z = 7, 4x + 5y -5z = 9
Also, find the solution.
SOLUTION The given system of equation can be written in matrix form as

w
'2-1 31f x 5
3 2-11/ 7
4 5-5 z 9
2-1 3

Flo
x 5

ee
or, AX = B, where A 3 2 -1 , X= y and B = 7

Fr
4 5-5 z 9
2-13 for
ur
Now, = 3 2 -1 = 2(-10 + 5) + l(-15 + 4)+3(15-8) = 0
4 5-5
ks

So, A is singular. Thus, the given system of equations is either inconsistent or it is consistent wit h
Yo
oo

infinitely many solutions according as (adj A) B*0 or, (adj A) B=0 respectively.
B

Let Cq be the co-factors of elements Ojj in A= [fl,-,]. Then,


3 -1
re

Cl! = (-1)1+1 5 Is = “5' c12=(- 1+2


4 -5 = 11,
ou
ad

1+ 3 3 2 2+ 1 -1 l = 10,
Ci3 = (-D 4 5
= 7, C21 = (-1) 5 -5
Y

2 3 2 -1
= (-1) 2+ 2 c23 = (-D 2+ 3
nd

5 = -14,
= -22,
Re

4 -5 4
Fi

-1 2 3 =11
c31 = (-D 3+ 1 2 -1
? = -5, C32 = (-1)
3+2
3 -1

and. 3+ 3 2 -1 =7
C33 = (-D 3 2'
-5 11 7 "|T -5 10 -5
(adj/l) = 10 -22 -14 11 -22 11
-5 11 7 7 -14 7
-5 10 -5 5 -25 +70 -45 0
=> (adj A) B = 11 -22 11 7 55 -154 +99 0 =0
7-14 7 J [_ 9 35 -98 + 63 0
Thus, AX = B has infinitely many solutions. To find these solutions, we put z = in the first two
equations and write them as follows:
2x-y = 5 - 3k and 3x + 2y = 7 + k
2 -1 5-3k
or,
3 2 y 7+k

ReadYourFlow.COM
8.8 MATHEMATICS-XII

2 -1 , X = * 5 - 3k
or. AX = B, where A = and B =
3 2 y 7+k
2 -1 2 1
Now, |A| = = 4+3=7* 0 and adj A =
3 2 -3 2
1 (adjA) = l 2 1
A -1
1^1 7 -3 2
Now, X = A-1 B
17-5 k
x 2 1 5 -3k
1 — —1 10 - 6 k + 7 + k 7
y 7 -3 2 7 +k 7 -15 + 9/c+2fc + 14 11 A:-1
7
17 -5 /c ll?c-l
x- >y = 7

low
7
These values of x, y and z = k also satisfy the third equation,
17-5 k llfc-l
Hence, x - /y = ^ and z = k, where k is any real number satisfy the given system of
7

ee
equations.
rF
Fr
Type III SOLVING A SYSTEM OF LINEAR EQUATIONS WHEN THE INVERSE OF THE COEFFICIENT
MATRIX IS OBTAINED FROM SOME GIVEN RELATION for
'1-1 11
u
EXAMPLE 6 If A -2 1-3 , find A~^ and hence solve the system of Imear equations
ks

111
Yo
oo

x + 2y + 2=4, -x + y + z = 0, x -3y + z = 2.
B

'1-1 1"
SOLUTION We have, A = 2 1-3
re

1 1 1
ou
ad

i -i i
Y

|A| = 2 1 -3 = 1(1 + 3)+ 1(2+3)+ 1(2-1) = 10*0


1 1 1
nd
Re

So, A is invertible.
Fi

Let Cy be the co-factors of elements ay in A [ay]. Then,


1+ 1 1-3 1+ 2 2 -3
Cn = (-1)
1 1
= 4, C12 = (-1) 1 1 = -5,
1+ 3 2 1 = 1, 2+ 1 1 1
Cis = (-1) C21 = (-1) 1 1 = 2,
1 1
1 1 1 -1
C22 = (-1) 2+ 2 C23 = (-1)
2+3 2
1 1 = 0, 1 1
3+ 1 -1 1
= 2, ^32 = (-D
3+2 1
l =5,
C31 = (-D 1 -3 2 -3

3+ 3 1 -1
and. C33 = (-D 1 =3
2

4 -5 1 4 2 2
adjA = 2 0-2 -5 0 5
2 5 3 1 -2 3

ReadYourFlow.COM
SOLUTION OF SIMULTANEOUS LINEAR EQUATIONS 8.9

4 2 2
1
=> A -1 = —adj A = -5 0 5 ~(i)
l^i 10 [ 1 -2 3
Now, the given system of equations is expressible as
i 2 11 r * 4
-Illy 0
1-3 1 z 2
x 4
or. AT X = B, where X = y and B = 0
2 2
Now, | AT = | .A | = 10 * 0. So, the given system of equations is consistent with a unique solution
given by

low
X = (A7)"1 B = (A-1)7 B [v (A7)"1 = (A"1)7]
x i r 4 2 2 iT * 4
=> y 0 [Using (0]
2 10 [_ 1 -2 3 J 2

ee
x [4-5 11[4
rF 16 + 0 + 2 9/5

Fr
1
=> y — 2 0-2 0 8 + 0-4 2/5
2 10 2 5 3 2 10 8 + 0 + 6 for7/5
=> x = 9/5, y = 2/5 and 2=7/5
u
ks

Hence, x = 9/5, y = 2/5,2=7/5 is the required solution.


Yo
oo

'-4 4 4 1 [l -1 1'
EXAMPLE? Determine the product -7 1 3 1-2-2 and use it to solve the system
B

5-3-1 2 13
re

of equations:
ou
ad

x-y + 2 = 4, x-2y-22 = 9, 2x + y+32 = 1.


Y

1 -1 1 ’_4 4 4'
SOLUTION Let A = 1 -2 -2 and C = -7 1 3 . Then the given product is
nd
Re

2 1 3 5 -3 -1
Fi

-444 1-11
CA = -7 13 1-2-2
5 -3 -1J [2 1 3
-4+4+8 4-8+4 -4-8+12 8 0 0 1 0 0
=> CA = -7+1+6 7-2+3 -7-2+9 0 8 0 = 8 0 1 0 = 87o
5-3-2 -5+6-1 5+6-3 0 0 8 0 0 1
1
=> =h
8

(ic> = '3

A -1 Ic [By definition of inverse]


8
-4 4 4
A"1 1
-7 1 3
8 5 -3 -1
The given system of equations can be written in matrix form as

ReadYourFlow.COM
r

8.10 MATHEMATICS-XII

1 -1 1 x 4
1 -2 -2 y 9
2 1 3 z i
1 -1 1 j 4
or. AX - B, where A = 1 -2 -2 , X = y and B = 9
2 1 3 z i
The solution of this system of equations is given by
X = 2T1 B
-4 4 4 4
1
=> X = - -7 13 9 [Using (i)]
8 5 -3 -1 1
x -16+36 + 4 24 3

w
1 1
y -28 + 9 + 3 16 -2
z 8 20-27-1 8 -8 -1
=> x = 3, y = - 2 and z = -1

Flo
ee
r1 2-3

Fr
EXAMPLES Find A \ inhere A = 2 3 2 . Hence solve the system of equations
3-3-4
or
ur
x + 2y - 3z = - 4, 2.v + 3y + 2z - 2, 3x - 3y - 4z = 11.
sf
SOLUTION We have.
1 2-3
k
Yo
oo

A = 2 3 2
3 -3 -4
B

1 2-3
re

|A| = 2 3 2 = -6 + 28 + 45 = 67 * 0
ou
ad

3 -3 -4
Y

So, A is invertible.
Let Cq be the co-factors of aq in A = [«;)]• Then,
nd
Re

C - f 1i1+1 3 2 1+2 2 2
C11 - (-1) -3 -4 = -6, ^12 = (-D = 14,
Fi

3 -4
1+ 3 2 3 2+ 1 2 -3
-3 = -15, C21 = (-1) -3 -4 = 17,
^13 = (-1) 3
2+2 1 -3 2+ 3 1 2
C22 = (-D 3 -4
= 5, ^23 = (-D 3 -3
= 9

3+ 1 2 -3 3+2 1 -3
C3I = (-D = 13, C32 = (-D = -8,
3 2 2 2
3+3 1 21
and. C33 = (-D = -1
2 3
T
-6 14 -15 -6 17 13
adj A = 17 5 9 14 5-8
13-8 -1 -15 9 -1
-6 17 13
A"1 1
So, — adjA = 14 5-8 -(i)
Ml 67 -15 9 -1

ReadYourFlow.COM
SOLUTION OF SIMULTANEOUS LINEAR EQUATIONS 8.11

The given system of equations is


x + 2y -?>z = - 4
2x + 3y + 2z = - 2
3x - 3y -4z = 11
1 1-3 x -4
or. A X = B, where A = 2 3 2 , X = y and B = 2
3 -3 -4 z 11
As discussed above A is non-singular and so invertible. The inverse of A is given by (i).
The solution of the given system of equations is given by
X = A~l B
x -6 17 13 -4 24 + 34 +143 3
1 =
=> y — 14 5 -8 2 -56 +10 -88 -2
z 67 [-15 9 -1 J[ 11 67 60 +18 -11 1

w
=> x = 3, y = - 2 and z = 1 is the required solution.
Tijpe IV ON APPLICATIONS OF SIMULTANEOUS LINEAR EQUATIONS

Flo
EXAMPLE 9 The sum of three numbers is 6. If we multiply the third number by 2 and add the first

ee
number to the result, we get 7. By adding second and third numbers to three times thefirst number, we get
12. Using matrices find the numbers.

Fr
SOLUTION Let the three numbers be x, y and z respectively. Then,
x+ y+z = 6 [Given]
for
ur
Also, .r + 2z =7
and, 3x + y + z = 12
ks

Thus, we obtain the following system of simultaneous linear equations:


Yo
oo

x+y+z = 6
x + Oy + 2z = 7
B

3x + y + z= 12
re

The above system of equations can be written in matrix form as


ou

'l 1 ll [Yl f 6"


ad

10 2 y = 7
Y

3 1 1 z 12
nd
Re

1 1 1 x 6
Fi

or. AX = B, where A = 1 0 2 ,X = y and B = 7


3 11 z 12
1 1 1
Now, |A| = 1 0 2 = l(0-2)-(l-6) + l(l-0)= -2 + 5 + 1 = 4*0
3 1 1
So, the above system of equations has a unique solution given by X = A" 1 B.
Let Cjj be the cofactor of Ojj in A = [^]. Then,
Cn = — 2, C|2 = 5,Ci3 = 1, C21 = 0,C22 — — 2, C23 = 2,
C31 = 2'C32 = -1 and C33 = -1
-2 5 llT -2 0 2
adjA = 0-2 2 5 -2 -1
2 -1 -1 1 2 -1
0 2
=> 1 ad)
A"1 = -—- rT A
A = —1 5S -2 -1
|7l| J 4 x
2 -1

ReadYourFlow.COM
8.12 MATHEMATICS-XII

Now, X = 1B
-2 0 2 6 i -2 + 0 +24 3
=> x=I4 5 -2 -1 7
4
30 -14 -12 1
1 2 -1 12 6 +14 -12 2
3
=> y 1
2 2
=> x = 3, y = 1 and z = 2.
Hence, the three numbers are 3,1 and 2 respectively.
EXAMPLE 10 An amount of^ 5000 is put into three investments at the rate of interest of 6%, 7% and 8%
per annum respectively. The total annual income is ? 358. If the combined income from the first two

w
investments is ? 70 more than the income from the third, find the amount of each investment by matrix
method.

Flo
SOLUTION Let x, y and z ? be the investments at the rates of interest of 6%, 7% and 8% per
annum respectively. Then,

ee
Total investment = ? 5000

Fr
=> x + y + z = 5000. for
Now, Income from first investment of ? x = ? -—
ur
100
Income from second investment of ? y = ? Zj/
ks

100
Yo
oo

8z
Income from third investment of ? z = ?
B

100
re

6x 7y 8z
Total annual income = ? ---- + +
100 100 100
ou
ad

=> -^ + -^- + — = 358 [v Total annual income = ? 358]


Y

100 100 100


==• 6x + 7y + 8z = 35800.
nd
Re

It is given that the combined income from the first two investments is ? 70 more than the
Fi

income from the third.


6x 7y 8z
70 + => 6x + 7y-8z =7000.
100 Too ~ 100
Thus, we obtain the following system of simultaneous linear equations:
x + y + z = 5000
6x + 7y + 8z = 35800
6x + 7y - 8z = 7000
This system of equations can be written in matrix form as follows:
1 1 1 x 5000'
6 7 8 V 35800
6 7-8 z 7000
1 1 1 x 5000
or, AX = B, where A - 6 7 8 , X = y and B = 35800
6 7-8 z 7000

ReadYourFlow.COM
SOLUTION OF SIMULTANEOUS LINEAR EQUATIONS 8.13

1 1 1
Now, |A| = 6 7 8 =1 (-56-56)-(-48-48)+(42-42) =-16 * 0.
6 7-8
So, A 1 exists and the solution of the given system of equations is given by X = A 1 B.
Let Cjj be the cofactor of a,y in A = [a;/ ]. Then,
Cn = -112, C12 = 96, C13 = 0,C2i = 15, C22 = -14,
C23 = = 1'C32 = -2 and C33 = 1-
T
-112 96 0 -112 15 r
adjA = 15 -14 -1 = 96 -14 -2
1 -2 1 0 -1 1
-112 15 l"
So, A-1 = — (adjA) = -- 96 -14 -2
|A| 16 0 -1 1

w
Hence, the solution is given by
-112 15 1 5000 -560000 +537000 +7000
1 1
X =A~ 1 B = -
16
96 -14 -2
0 -1 1
Flo
35800
7000 16
480000 -501200 -14000
0 - 35800 + 7000

ee
1000

Fr
=> y 2200
2 1800
for
ur
=> x = 1000, y = 2200 and z = 1800
Hence, three investments are of ? 1000, ? 2200 and ? 1800 respectively.
ks
Yo

EXAMPLE 11 A mixture is to be made of three foods A, B, C. The three foods A, B, C contain nutrients
oo

P,Q, R as shown below:


Ounces per pound of Nutrient
B

Food
re

P Q R
A 1 2 5
ou
ad

B 3 1 1
Y

C 4 2 1
Hoio to form a mixture which will have 8 ounces of P, 5 ounces ofQ and 7 ounces ofR?
nd
Re

SOLUTION Let x pounds of food A, y pounds of food B and z pounds of food C be needed to
Fi

form the mixture.


Since one pound of food A contains 1 ounce of nutrient P. So, x pounds of food A will contain x
ounces of nutrient P. Similarly, the amount of nutrient P in y pounds of food B and z pounds of
food C are 3y and 4z ounces respectively. Therefore,
Total quantity of nutrient P in .r pounds of food A, y pounds of food B and z pounds of food C is
x + 3y + 4z ounces.
a: + 3y + 4z = 8
Similarly, 2.r + y + 2z =5 [For nutrient Q]
and 5x + y + z = 7 [For nutrient R]
The above system of simultaneous linear equations can be written in matrix form as
"l 3 4] [V 8
2 12 y 5
5 11 z 7
1 3 4 x 8
or, AX = B, where A = 2 1 2 / x = y and B = 5
5 1 1 z 7

ReadYourFlow.COM
8.14 MATHEMATICS-XII

1 3 4
Now, |A| = 2 1 2 = l(:L-2)-3(2-10) + 4(2-5)= -1 + 24-12 = 11 * 0
5 1 1
So, A 1 exists.
Let Cjj be the cofactor of Ojj in A = [ajj]. Then,
Cn = -1, C12 = 8, C13 = - 3, C21 = 1, C22 = -19,
C23 = 14, C31 = 22, C32 = 6 and C33 = — 5
-1 8 -3f -1 1 2
adj A = 1 -19 14 8 -19 6
2 6 -5 -3 14 -5
1 I"112"
=> A" 1 = -— adj A = — 8-19 6
1^1 11 -3 14 -5

w
Thus, the solution of the system of equations is given by
-1 12] [81 -8 +5 +14 11 1
X = A~ 1 B = — 8 -19 6
11 -3 14-5 7
Flo
5 =— 64 -95 +42
11 -24 +70 -35
1
11
11 1

ee
11 1

Fr
x 1
=> y 1
1
for
ur
2

=> x = 1, y = 1 and z = 1.
ks

Hence, the mixture is formed by mixing one pound of each of the foods A, B and C.
Yo
oo

EXERCISE 8.1
B

LEVEL-1
re

1. Solve the following system of equations by matrix method:


ou

(i) 5x + 7y + 2 = 0 (ii) 5x + 2y = 3
ad

4x + 6y + 3 = 0 3x + 2y = 5
Y

(iii) 3x + 4y -5 = 0 (iv) 3x + y = 19
nd
Re

x-y + 3 = 0 3x - y = 23
Fi

(v) 3x + 7y = 4 (vi) 3x + y =7
x + 2y = -1 5x + 3y = 12
2. Solve the following system of equations by matrix method:
(i) x + y-z = 3 (ii) x + y + z = 3
2x + 3y + z = 10 2x -y +z = -1
3x -y - 7z = 1 2x + y - 3z = - 9 [CBSE 2004,2005]
(iii) 6x - 12y + 25z = 4 (iv) 3x + 4y + 7z =14
4x + 15y - 20z = 3 2x - y + 3z = 4
2x +18y +15z = 10 x + 2y-3z = 0
(V)--- + - = 10 (vi) 5x + 3y + z = 16
x yz
1
- + - +1- 1 = 10
10
2x + y + 3z =19
x yz
2-i+2 =13 [CBSE 2005, 07] x + 2y + 4z = 25
x yz

ReadYourFlow.COM
SOLUTION OF SIMULTANEOUS LINEAR EQUATIONS 8.15

(vii) 3* + 4y + 2z = 8 (viii) 2x + y + 2 = 2
2y - 3z = 3 x + 3y -2 =5
x-2y + 6z = -2 3x + y - 22 = 6 [CBSE 2008]
(ix) 2x + 6y = 2 (x) x-y + 2 = 2
3x -2 = - 8 2x -y = 0 [CBSE2003]
2x-y+2 = - 3
2y-z =1 [CBSE 2003]
(xi) 8x + 4y + 32 = 18 (xii) x + y + 2 = 6
2x + y + 2 =5 x + 22 = 7
x + 2y +z =5 [CBSE 2008] 3x + y + 2 =12 [CBSE 2009]
3 10
(xiii) - + — + — =4, (xiv) x-y + 22 =7
x y z
4 _6+ r5=1,

w
3x + 4y -52 = -5
x y z
6 9 20

Flo
- +-------- = 2;x,y,z*Q [CBSE 2011] 2x-y + 32 =12 [CBSE 2012]
x y z

ee
3. Show that each of the following systems of linear equations is consistent and also find their
solutions:

Fr
(i) 6x + 4y = 2 (ii) 2x + 3y = 5
9x + 6y = 3 6x + 9y = 15
for
ur
(hi) 5x + 3y + 72 = 4 (iv) x-y + 2 = 3
3x + 26y + 22 = 9 2x + y -2 = 2
ks
Yo

7x + 2y + 10 2 = 5 - x - 2y + 2z = 1
oo

(v) x + y + 2 = 6 (vi) 2x + 2y - 22 = 1
B

x + 2y + 32 = 14 4x + 4y - 2 = 2
re

x + 4y + 72 = 30 6x + 6y + 22 = 3
4. Show that each one of the following systems of linear equations is inconsistent:
ou
ad

(i) 2x+5y = 7 (ii) 2x + 3y = 5


Y

6x + 15y = 13 6x + 9y = 10
(hi) 4x - 2y = 3 (iv) 4x - 5y - 22 = 2
nd
Re

6x - 3y = 5 5x - 4y + 22 = - 2
Fi

2x + 2y + 82 = -1
(v) 3x - y - 22 = 2 (vi) x + y - 22 =5
2y -2 = -1 x -2y + 2 = -2
3x-5y = 3 - 2x + y + 2 = 4
'l -1 0" 2 2-4
5. If A = 2 3 4 and B = -4 2-4 are two square matrices, find AB and hence
0 1 2 2-1 5
solve the system of linear equations:
x-y = 3, 2x + 3y + 42 =17, y + 22 =7 [CBSE 2010, 2012]
2-3 5 "I
6. lfA = 3 2 - 4 , find and hence solve the system of linear equations:
1 1 -2j
2x- 3y + 52 = 11, 3x+2y-4z = -5, x + y-2z = - 3 [CBSE 2007, 2009, 2012]

ReadYourFlow.COM
8.16 MATHEMATICS-XII

1 2 5
7. Find A-1, if A = 1 -1 -1 . Hence, solve the following system of linear equations:
2 3 -1 J
x+ 2y+5z =10, x-y-z = -2, 2x+ 3y-z = -11 [CBSE 2010, 2012]
' 1 -2 0 1
8. (i) If A = 2 13 , find A-1. Using A~\ solve the system of linear equations:
0-2 1
x-2y =10, 2x + y + 3z = 8, - 2y + z =7
'3 -4 2]
(ii) A = 2 3 5, find A- 1 and hence solve the following system of equations:
1 0 ij
3x - 4y + 2z = -1 , 2.r + 3y + 5z = 7 , x + z = 2 [CBSE 2011]
1 -2 0 '7 2-6'

w
(hi) A = 2 13 and B = - 2 1 - 3 , find AB. Hence, solve the system of equations:
0 -2 1 -4 2 5

Flo
x-2y = 10, 2x + y+3z = 8 and -2y + z=7
‘ 1 2 O]
[CBSE 2011]

ee
(iv) If A = -2 -1 -2 , find A-1. Using A-1, solve the system of linear equations

Fr
0 -1 ij
,t-2y = 10, 2x-y-z = 8, -2y+ z =7 for [NCERT EXEMPLAR]
ur
'2 2-4" 1 -1 0
(v) Given A = -4 2 -4 , B = 2 3 4 , find BA and use this to solve the system of
ks

2-15 0 12
Yo
oo

equationsy + 2z =7, x-y = 3, 2x+3y + 4z=17 [NCERT EXEMPLAR]


eB

[231'
(vi) If A = 1 2 2 , find A 1 and hence solve the system of equations
r

-3 1 -1
ou
ad

2x + y-3z=13, 3x + 2y+z = 4, x + 2y-z = 8. [CBSE 2017]


Y

'1 -1 2] [-2 0 l'


(vii) Use the product 0 2-3 9 2-3 to solve the system of equations
nd
Re

3-2 4 6 1 -2
Fi

x + 3z = -9, -x + 2y - 2z = 4,2x - 3y + 4z = - 3. [CBSE 2017]


9. The sum of three numbers is 2. If twice the second number is added to the sum of first and
third, the sum is 1. By adding second and third number to five times the first number, we
get 6. Find the three numbers by using matrices.
10. An amount of ? 10,000 is put into three investments at the rate of 10,12 and 15% per annum.
The combined income is ? 1310 and the combined income of first and second investment is ?
190 short of the income from the third. Find the investment in each using matrix method.
11. A company produces three products every day. Their production on a certain day is 45
tons. It is found that the production of third product exceeds the production of first product
by 8 tons while the total production of first and third product is twice the production of
second product. Determine the production level of each product using matrix method.
12. The prices of three commodities P,Q and R are ? x, y andz per unit respectively. A
purchases 4 units of R and sells 3 units of P and 5 units of Q. B purchases 3 units of Q and
sells 2 units of P and 1 unit of R.C purchases 1 unit of P and sells 4 units of Q and 6 units of
R. In the process A, B and C earn ^ 6000, ? 5000 and ^ 13000 respectively. If selling the units
is positive earning and buying the units is negative earnings, find the price per unit of three
commodities by using matrix method.

ReadYourFlow.COM
SOLUTION OF SIMULTANEOUS LINEAR EQUATIONS 8.17

13. The management committee of a residential colony decided to award some of its members
(say x) for honesty, some (say y) for helping others (say 2) for supervising the workers to
keep the colony neat and clean. The sum of all the awardees is 12. Three times the sum of
awardees for cooperation and supervision added to two times the number of awardees for
honesty is 33. If the sum of the number of awardees for honesty and supervision is twice the
number of awardees for helping others, using matrix method, find the number of awardees
of each category. Apart from these values, namely, honesty, cooperation and supervision,
suggest one more value which the management must include for awards. [CBSE 2013]
14. A school wants to award its students for the values of Honesty, Regularity and Hardwork
with a total cash award of ? 6000. Three times the award money for Hardwork added to that
given for honesty amounts to ? 11000. The award money given for Honesty and Hardwork
together is double the one given for Regularity. Represent the above situation algebraically
and find the award for each value, using matrix method. Apart from these values, namely.
Honesty, Regularity and Hardwork, suggest one more value which the school must
include for awards. [CBSE 2013]
15. Two institutions decided to award their employees for the three values of resourcefulness,

w
competence and determination in the form of prizes at the rate of ? x, ? y and ? z respectively
per person. The first institution decided to award respectively 4,3 and 2 employees with a
total prize money of ? 37000 and the second institution decided to award respectively 5, 3

Flo
and 4 employees with a total prize money of ? 47000. If all the three prizes per person
together amount to ? 12000, then using matrix method find the value of x, y and 2. What

ee
values are described in this equations? [CBSE 2013]

Fr
16. Two factories decided to award their employees for three values of (a) adaptable to new
techniques, (b) careful and alert in difficult situations and (c) keeping calm in tense
situations, at the rate of tx,ty and ? 2 per person respectively. The first factory decided to
for
ur
honour respectively 2, 4 and 3 employees with a total prize money of ? 29000. The second
factory decided to honour respectively 5, 2 and 3 employees with the prize money of
? 30500. If the three prizes per person together cost ? 9500, then
ks
Yo

(i) represent the above situation by a matrix equation and form linear equations using
oo

matrix multiplication.
B

(ii) Solve these equations using matrices.


(iii) Which values are reflected in the questions? [CBSE 2013]
re

17. Two schools A and B want to award their selected students on the values of sincerity,
ou

truthfulness and helpfulness. The school A wants to award ? x each ? y each and f 2 each for
ad

the three respective values to 3, 2 and 1 students respectively with a total award money of
Y

^ 16,00. School B wants to spend ? 2,300 to award its 4,1 and 3 students on the respective
values (by giving the same award money to the three values as before). If the total amount
nd
Re

of award for one prize on each value is ^ 900, using matrices, find the award money for each
value. Apart from these three values, suggest one more value which should be considered
Fi

for award. [CBSE 2014]


18. Two schools P and Q want to award their selected students on the values of Discipline,
Politeness and Punctuality. The school P wants to award ? x each, ? y each and ? 2 each for
the three respectively values to its 3, 2 and 1 students with a total award money of ? 1,000.
School Q wants to spend ? 1,500 to award its 4,1 and 3 students on the respective values (by
giving the same award money for three values as before). If the total amount of awards for
one prize on each value is ^ 600, using matrices, find the award money for each value.
Apart from the above three values, suggest one more value for awards. [CBSE 2014]
19. Two schools P and Q want to award their selected students on the values of Tolerance,
Kindness and Leadership. The school P wants to award ^ x each, ? y each and ? 2 each for
the three respective values to 3,2 and 1 students respectively with a total award money of ?
2,200. School Q wants to spend ? 3,100 to award its 4, 1 and 3 students on the respective
values (by giving the same award money to the three values as school P). If the total amount
of award for one prize on each values is ? 1,200, using matrices, find the award money for
each value.
Apart from these three values, suggest one more value which should be considered for
award. [CBSE 2014]

ReadYourFlow.COM
8.18 MATHEMATICS-XII

20. A total amount of ? 7000 is deposited in three different saving bank accounts with annual
interest rates 5%, 8% and 8—% respectively. The total annual interest from these three
accounts is ? 550. Equal amounts have been deposited in the 5% and 8% savings accounts.
Find the amount deposited in each of the three accounts, with the help of matrices.
[CBSE 20141
21. A shopkeeper has 3 varities of pens 'A', ‘B‘ and 'C‘. Meenu purchased 1 pen of each variety
for a total of ? 21. Jeen purchased 4 pens of 'A' variety, 3 pens of 'B' variety and 2 pens of 'C'
variety for ? 60. While Shikha purchased 6 pens of 'A' variety, 2 pens of 'B' variety and 3
pens of 'C variety for ? 70. Using matrix method find the cost of each pen. [CBSE 2016]
ANSWERS
7
1- (i) * = f, y = -2 (ii) x=-l, y =4 (iii) x = -l, y = 2
<vi> *=! 1
(iv) x=7, y = -2 (v) x = -15, y =7
4 'y = 4

w
.... 8 10 19 1 1
2. (i) x = 3, y =1, 2 =1 (u) x = --,y = j,z = — (iii) x = -, y = —, 2 =—
7 2*3 5

(iv) x =1, y =1, 2 = 1


, . 1

Flo
1
(v) X= -,y=-,2=r
1
(vi) x = 1, y = 2, 2=5

ee
5

Fr
(vii) x = -2, y = 3, 2 = 1 (viii) x=l, y =1, 2=-l (ix) x = -2,y =1,2 = 2
(x) x=l,y = 2,2 = 3 (xi) x = 1, y =1, 2 = 2 for (xii) x = 3,y = 2,2=1
ur
(xiii)x = 2, y = 3,2 = 5 (xiv) x = 2, y = 1,2 = 3
\-2k 5-Bk
3. (i) x = — y=* (ii) x = --------- / y = (xii) x = 3, y = 1, 2 = 2
ks

2 *
Yo

7 -16fc k+3
oo

(iii) x = / y= , z=k — y = —~ + k, z = k
n 11 3
eB

(v) x-k -2, y = 8 -2 k, z = k (vi) x = ^-fc, y=k, 2 = 0


r
ou

5. x =2, y = -1, 2 = 4 6. x = 1, y = 2, 2 = 3 7. x = -1, y = -2, 2 = 3


ad

8. (i) x = 4, y = -3, 2=1 (ii) x = 3, y = 2,2 =-1 (iii) x = 4, y = -3, z = 1


Y

(iv) x = 0, y = -5,2 = -3 (v) x = 2, y = -1,2 = 4 9. 1,-1, 2


nd

10. Rs 2000, Rs 3000, Rs 5000 11. 11,15,19 12. x = 3000, y = 1000,2 = 2000
Re

13. x = 3, y = 4,2 =5 14. x = 500, y = 2000,2 = 3500 15. x = 4000, y = 5000,2 = 3000
Fi

16. x = 2500, y = 3000, 2 = 4000 17. x = 200, y = 300,2 = 400 18. x = 100, y = 200,2 = 300
19. x = 300, y = 400,2 = 300 20. ? 1125,? 1125, ? 4750
21. Variety A : ? 5, Variety B: ? 8, Variety C : ^ 8
HINTS TO NCERT& SELECTED PROBLEMS
13. The given data suggests the following equations:
x + y + 2=12, x-2y+2 = 0, 2x+ 3y+ 3z = 33
14. Let the award money for Honesty, Regularity and Hardwork be ? x, y and z respectively.
Then, x + y + z = 6000, x + 3z =11000 and x - 2y + z = 0.
15. 4x + 3y + 2z = 37000, 5x + 3y + 4z = 47000, x + y+2=12000
16. 2x + 4y+3z = 29000, 5x + 2y + 3z = 30500, x + y+2 = 9500
8.3 SOLUTION OF HOMOGENEOUS SYSTEM OF LINEAR EQUATIONS
In chapter 6, we have learnt about determinant method to solve a homogeneous system of linear
equations. In this section, we shall discuss matrix method to solve the same.
Let AX = O be a homogeneous system of n linear equations with n imknowns.
Let us now discuss two cases:

ReadYourFlow.COM
SOLUTION OF SIMULTANEOUS LINEAR EQUATIONS 8.19

CASE I When | A | * 0 i.e. matrix A is non-singular.


If | A | * 0, then A- 1 exists.
AX = O
=> A-1 (AX) = A_10
=> (A- aA) X = O => IfjX = O => X = O
=> x-[=x2=...=xn = 0
Thus, if the coefficient matrix A is a non-singular, then the homogeneous system of equations
has the unique solution X = 0 i.e. Xj = x2 = ... = x7l =0.
This solution is known as the trivial solution.
CASE II When | A | = 0 i.e. matrix A singular.
If | A | = 0, then (adj A) B = (adj A) 0=0
i.e. the condition of consistency is always satisfied. So, the given system of equations is

low
consistent and it has infinitely many solutions which can be obtained by giving any real value to
one of the variables and then solving the remaining equations by matrix method.
In order to solve a homogeneous system of the three linear equations with 3 unknowns x, y, z,
we may use the following algorithm.

ee
ALGORITHM
rF
Fr
STEP I Obtain the system of equations and express it in the matrix equation of the form AX = O.
STEP II Find | A |.
for
STEP III//1 A | * 0, then x=y=z = 0 is the only solution of the homogeneous system. So, write x = 0,
ou
y = 0,z = 0as the solution.
ks

STEP IV 1/ | A | = 0, then the system has infinitely many solutions. In order to find these solutions put
z=k (any real number) and solve any two equations for x and y by the matrix method. The
oo

values ofx and y so obtained with z = k give a solution of the system.


Y
B

Following examples will illustrate the above procedure.


re

ILLUSTRATIVE EXAMPLES
ou
ad

LEVEL-1
Y

Type I WHEN THE DETERMINANT OF THE COEFFICIENT MATRIX IS NON-SINGULAR


nd
Re

EXAMPLE l Solve the following system of homogeneous equations:


Fi

2x + 3y -z = 0
x-y -2z = 0
3x + y + 3z = 0
SOLUTION The given system of homogeneous equations can be written as
'2 3-1 x 0
1 -1 -2 y 0
3 1 3 2 o
2 3-1 x 0
or. AX = O, where A = 1 -1 -2 , X = y and O = 0
3 1 3 2 0
2 3-1
Now, | A | = 1 -1 -2 = -2-27 -4 = - 33 * 0.
3 1 3
Thus | A | * 0. So, the given system has only the trivial solution given by x = y =2 = 0.

ReadYourFlow.COM
8.20 MATHEMATICS-XII

Type II WHEN THE DETERMINANT OF THE COEFFICIENT MATRIX IS SINGULAR


EXAMPLE 2. Show that the homogeneous system of equations
x-ly + z = 0, x + y-z = 0, 3x + 6y-5z = 0
has a non-trivial solution. Also, find the solution.
SOLUTION The given system of homogeneous equations can be written in matrix form as
"i -2 iir* 0
1 1-1 y 0
3 6 -5 z 0
1 -2 1 0
or. AX = O, where A = 1 1 -1 , X = y and O = 0
3 6-5 z 0
1 -2 1

low
Now, \A\=1 1 -1 = l(-5 + 6) + 2(-5+3) + l (6-3) = 0.
3 6-5
So, the given system of equations has a non-trivial solution. To find these solutions, we put z-k
in the first two equations and write them as follows:

ee
x-2y = -k and x + y = k.
rF
Fr
1 -21f x~ -k
or.
1 1 V k for
1 -2 -k
or. AX = B, where A = ^ , X = and B =
1 k '
ou
y
ks

1 -2 = 3 * 0. So, 2l 1 exists.
Now, |A| =
1 1
oo
Y

1 2
B

Clearly, adj A =
-1 1 •
re

1 2
A~l = 1 ad^=-1
ou
ad

IA| -1 1
Y

Now, X = A -l B
1 1 2 -k k/3
nd
Re

V 3 -1 1 k 2k/3
Fi

x = k/3, y = 2k/3.
These values of x, y and z also satisfy the third equation.
Hence x = k/3,y=2k/3 and z = k, where k is any real number satisfy the given system of
equations.
EXERCISE 8.2
LEVEL-1
Solve thefollowing systems of homogeneous linear equations by matrix method:
1. 2x -y + z = 0 2. 2x - y + 2z = 0 3. 3x - y + 2z = 0
3x + 2y -z = 0 5x + 3y - z = 0 4x + 3y + 3z = 0
x + 4y + 3z = 0 x + 5y - 5z = 0 5x + 7y + 4z = 0
4. x + y - 6z = 0 5. x+y +z = 0 6. x + y-z = 0
x -y + 2z = 0 x - y - 5z = 0 x-2y+z = 0
-3x + y + 2z = 0 x + 2y + 4z = 0 3x + 6y -5z = 0

ReadYourFlow.COM
SOLUTION OF SIMULTANEOUS LINEAR EQUATIONS 8.21

7. ?>x + y - Tz = 0 8. 2x + 3y -2 = 0
x + y + z =0 x-y - 2z. = 0
x-2y +z = 0 3x + y + ?>z = 0
^ANSWERS
-5k 12 k -9k k ,
1. x = y =z = 0 2. x = z =k 3. x =
11 '
y = 11 ' 13 ' y = -^r'z=k
13
4. x = 2k, y = 4k, z=k 5. x = 2k, y =-3k, z=k 6. x-k, y = 2k, z = 3 k
7. x = y =z = 0 8. x=y=z = 0
_____________________________________VERY SHORT ANSWER QUESTIONS (VSAQs)
Answer each of the following questions in one word or one sentence or as per exact requirement of the
question:

w
1 0 0 x 1
1. If 0 1 0 y = -1 , find x, y and z.

Flo
0 0 1 2 0

ee
1 0 0 X 1
2. If 0-1 0 y = 0 , find x, y and z.

Fr
0 0-1 2 1
for
ur
1 0 0 x 1
3. If 0 y 0 1=0 , find x, y and z.
ks

0 0 1 2 1
Yo
oo

4. Solve r 3^
"10"
^ for x and y.
eB

1 0 0 x 2
r
ou
ad

5. If 0 0 1 y\= -i , find x, y, z.
Y

0 1 0 2 3

r si and AX = B, then find n.


nd

'2 4“
Re

6. If A = /B=
4 3 1 11
Fi

ANSWERS

1. X =1,1/=-1,2 = 0 2. x =1, y = 0,z =-l 3. x = l,y = 0,2 = 1


2
5. x = 2, y = 3,2 = -l 6. 2
4-*=I'y = -2
_________________________________________ MULTIPLE CHOICE QUESTIONS (MCQs)
1. The system of equation x + y + z = 2, 3x-y + 2z = 6and3x + y+z = -18 has
(a) a unique solution (b) no solution
(c) an infinite number of solutions (d) zero solution as the only solution
2x + y -z =7
2. The number of solutions of the system of equations: x - 3y + 2z = 1, is
x + 4y - 3z = 5
(a) 3 (b) 2 (c) 1 (d) 0

ReadYourFlow.COM
8.22 MATHEMATICS-XII

*1 1 -1 2 3
3. LetX = ,A = 2 0 1 and B = 1 . If AX = B, then X is equal to
*3 3 2 1 4

1 -1 -1 -1 0
(a) 2 (b) -2 (c) -2 (d) 2 (e) 2
3 -3 -3 3 1
4. The number of solutions of the system of equations:
2x + y-z =7
x -2>y + 2z = 1, is
x + Ay -3z = 5
(a) 3 (b) 2 (c) 1 (d) 0
5. The system of linear equations:

w
x + y+z =2
2x+y-z=3
?>x + 2y + kz = A has a unique solution if
(a) k (b) -1 <k <1

Flo (c) -2 <k <2 (d) k = 0

ee
6. Consider the system of equations:

Fr
a1 x + b1y + c1z = 0
or
ur
a2 x + b2y+ c2z = 0
sf
a3 x + b^ y + c3z = 0.
k
Yo

bi q
oo

If a2 b2 c2 = 0, then the system has


B

a3 b3 c3
re

(a) more than two solutions (b) one trivial and one non-trivial solutions
ou

(c) no solution (d) only trivial solution (0, 0, 0)


ad
Y

7. Let fl, b, c be positive real numbers. The following system of equations mxry and z
x1 y1 z2 *2 y2 z2 x2 y2 z2
nd

+
Re

+ —----- =1, — = 1,------ + ~ — 1 has


a2 b2 c2 a2 b2 c2 a2 b1 c1
Fi

(a) no solution (b) unique solution


(c) infinitely many solutions (d) finitely many solutions
8. For the system of equations:
x + 2y + 3z = 1
2x + y + 3z = 2
5x + 5y + 9z = 4
(a) there is only one solution (b) there exists infinitely many solution
(c) there is no solution (d) none of these
9. The existence of the unique solution of the system of equations:
x + y + z= X
5x - y + p z = 10
2x+ 3y - z = 6 depends on
(a) p only (b) X only (c) X and p both (d) neither X nor p

ReadYourFlow.COM
SOLUTION OF SIMULTANEOUS LINEAR EQUATIONS 8.23

10. The system of equations:


x + y + z=5
x + 2y + 3z = 9
x+ 3y + X.z =|i
has a unique solution, if
(a) X = 5, q = 13 (b) ^ 5 (c) A. = 5, q * 13 (d) q * 13
_____________________________________________________________________ ANSWERS
1. (a) 2. (d) 3. (d) 4. (d) 5. (a) 6. (a) 7. (b) 8. (a) 9. (a) 10. (b)

SUMMARY
1. A system of n simultaneous linear equations inn unknowns x-^, x2, x3/is
a\\ x\ + a\2 x2 + • + aViXn=b-l
fl21 ^i + «22x2+ ••• + a2ii Xn =b2

w
^1 X1 + ^12 x2 + ••• +

Flo
This system of equations can be written, in matrix form, as
Xn =bn

ee
au au ... ayj h

Fr
x\
a2l a22 ••• a2n x2 b2 for
ur
an\ an2 ■■■ ann X,1 bn
an au ••• “v,
ks

X1
Yo

a21 a22 ••• a2n ,X= x2 and B =


oo

or, AX = B, where A =
B

_anl an2 ■■■ ann x„ bn


re

2. A set of values of the variable Xj, x2,..., x„ satisfying all the equations simultaneously is
called a solution of the system.
ou
ad

3. If a system of equations has one or more solutions, then it is said to be a consistent system of
Y

equations, otherwise it is an inconsistent system of equations.


4. A system of equations AX = B is called a homogeneous system, if B = O. Otherwise, it is
nd
Re

called a non-homogeneous system of equations.


5. A system AX = B of n linear equations in n equations has a unique solution given by
Fi

X = A- 1 B, if| A | * 0.
If | A | = 0 and (adj A) 8=0, then the system is consistent and has infinitely many solutions.
If | A | = 0 and (adj A) 8*0, then the system is inconsistent.
6. A homogeneous system of n linear equations in n unknowns is expressible in the form
AX =0.
If | A | * 0, then AX = O has unique solution X = 0 i.e. x1 = x2 =... = x„ = 0. This solution is
called the trivial solution.
If | A | = 0, then AX = O has infinitely many solutions.

ReadYourFlow.COM
Re
Fi ad
nd
Y
ou
r
Yo
eB
oo
ur
ks
for
Flo

Fr w
ee

ReadYourFlow.COM
CHAPTER 9
CONTINUITY

9.1 INTUTIVE NOTION OF CONTINUITY


Intutively a function is continuous in its domain if its graph is a curve without breaks or jumps
throughout its domain and a function is continuous at a point in its domain if its graph does not
have breaks or jumps in the immediate neighbourhood of the point. Consider the graph of a

w
function / (x) shown in Fig. 9.1. It is evident from the graph that / (x) is not defined at x = a.
Consequently, there is hole in the curve y = / (x) and so f (x) is not continuous at x = a. We also
observe that L = R i.e. lim
x —>a x -> a+

Flo
/ (x) = lim / (x) and so lim / (x) exists. Thus, the continuity
x —> n

ree
of / (x) at x = fl is destroyed, if lim / (x) exists but / (x) is not defined at x = a.

F
X —> fl

or
ur
sf
k
Yo
oo
B
re
ou
ad
Y

HOLE :f(a) is not defined, lim/(.r) exists


x—> a
nd
Re

Fig. 9.1
Fi

Let us now consider the function / whose graph is shown in Fig. 9.2. Clearly, L = R i.e.
lim / (x) = lim / (x). Consequently lim /(x) exists. But, there is hole in the curve because
x ->a x -> a+ x a

y i" T
/(«)

;
L ; R

•ot a
rr
L
HOLE :/(fl) is defined, lim/(x) exists and not equal to f(a)
x—» a
Fig. 9.2

ReadYourFlow.COM
9.2 MATHEMATICS-XII

lim f(x) is not equal to/ (a).So,f (a-) becomes discontinuous atx =a if, lim f(x) exists but it is
x-+ a x ->a
not equal to the value of / at x = a.

In Fig. 9.3, we observe that L* R i.e. lim /(x) * lim f{x). So, lim /(x) does not exist.
x —> a x—>a+ x a

Also, / (x) is not continuous at x = a. Thus, the continuity of / at x = a is also destroyed if


lim / (x) does not exist. This happens due to the jump in the values off (x) as x crosses' a'.
x a

w
Flo
ee
JUMP: lim f(x) is not the same as lim f(x)

Fr
x-> a~ .t-> a+
Fig. 9.3 for
ur
The continuity of a function / is also destroyed if either of the two limits lim /(x) and
x—>a~
ks

lim / (x) or both tend to + oo or - oo and / {a) is finite as is evident from in Fig. 9.4
Yo
oo
B
re
ou
ad
Y
nd
Re
Fi

f[x) is defined atx = a, lim f{x) = + oo and


f{x) is defined atx = a, lim _ fix) = + oo
x-> a lim f(x) = + co x-> a~
x—> fl+
Fig. 9.4

It follows from the above discussion that a function/(x) can be continuous at a point x = a iff

(i) f{a) is defined. (ii) lim /(x) exists and, (iii) lim /(x) =f(a).
x —> fl x ->a

This is also evident from Fig. 9.5


Thus, we define continuity of a function at a point as follows.

ReadYourFlow.COM
CONTINUITY 9.3

lim f(a) exists and is equal to f(a)


Function/(x) is continuous at a point .r = n
Fig. 9.5

w
9.2 CONTINUITY AT A POINT
DEFINITION Afunction f(x) is said to be continuous at a point x = a of its domain, iff lim f(x) =f(a).

Flo
Thus,
(/(x) is continuous at x = a) <=> lim f (x) = f (a) lim / (x) = lim f{x)=f(a)

ee
x x-^a x ->a+

Fr
lff(x) is not continuous at a point x = a, then it is said to be discontinuous at x =
If lim
/ (x) = lim /(x) ^ f(a) , then the discontinuity is known as the removable
for
ur
x a x -+a+
discontinuity, because/(x) can be made continuous by redefining it at point x = a in such a way
ks
Yo

that f(a) = lim /(x) .


oo

x a
eB

If lim /(x)^ lim / (x), then/(x) is said to have a discontinuity offirst kind.
x -> a
r

A function/(x) is said to have a discontinuity of the second kind at x = a iff


ou
ad

lim / (x) or, lim /(x) or, both do not exist.


Y

x— x-+a+
nd
Re

A function / (x) is said to be left continuous or continuous from the left atx = a, iff
Fi

(i) lim / (x) exists and, (ii) lim /(x) =f(a)


x ->a
A function / (x) is said to be right continuous or continuous from the right at x = a, iff
(i) lim / (x) exists and, (ii) lim / (x) = / (a)
x—»«+ x -»a+
It follows from the above definitions that
/ (x) is continuous at x = « iff it is both left as well as right continuous at x = a.
REMARK A function f (x) fails to be continuous at x = a for any of thefollowing reasons.
(i) lim / (x) exists but it is not equal to f (a). (ii) lim f (x) does not exist.
x —>« x —> a

This happens ifeither lim /(x) does not exist or, lim f (x) does not exist or both lim / (x) and
x -» fl <j+ x —> a~
lim / (x) exist but are not equal.
a+
(iii) fis not defined at x = a i.e. f {a) does not exist.

ReadYourFlow.COM
9.4 MATHEMATICS-XII

9.3 ALGEBRA OF CONTINUOUS FUNCTIONS


Regarding the continuity of the sum, difference, product and quotient of functions, we have the
following theorems.
THEOREM 1 Let f and g be two real functions, continuous atx=a. Let a be a real number. Then,
(i) f + g is continuous at x =a. (ii) f -gis continuous at x= a.
(hi) a / is continuous atx-a. (iv) fg is continuous at x = a.
(v) \ is continuous at x = a, provided that f (a) * 0.
/
/
(vi) — is continuous at x=a, provided that g (a) * 0.
£
PROOF1 Since/ and g are continuous atx = a. Therefore, lim f (x) =f (a) and lim g(x) =g (a).
x->a x->a
(i) We know that

w
lim (f + g)(x) = lim [f(x)+g(x)]
x ->a x —>a
=> lim (f + g)(x) = lim f (x) + lim g(x)

Flo
x ->a x —>a x->a

ree
=> lim (/ + g) (x) = f (a)+g (a) lim / (x) = / (a) and lim g(x) = g (a)
x —> a x a x a

F
=> lim (f + g) (x) = (J + g) (a)
x a or
ur
f + gis continuous atx = a.
f
(ii) We know that
ks
Yo

lim {f-g){x) = lim [/r(x)-g(x)J


oo

x —* a x-> a
B

=> lim {f-g)(x) = lim /(x)-lim g(x)


re

x —> a x -> a x ^*a


ou
ad

=> lim (f-g)(x) = f (a)-g{a) ■: lim / (x) = / (a) and lim g(x) = g (a)
Y

x -> a x -ta x a
=> lim (f~g)(x) = (f-g)(a)
nd
Re

x-^a
Fi

f - g is continuous at x = a.
(iii) We know that
lim (a /) (x) = lim a f (x)
x —> fl x a

=> lim (a /) (x) = a lim /(x) = a / (a) ••• lim / (x) = / (a)
X —> 0 x-+a x->a
a/ is continuous at x = a.
(iv) We know that
lim (fg) (x) = Urn (f(x)g(x)}
x —> a x a
=> lim (fg) (x) = lim / (x) lim g (x) = f (a) g (a) = (fg) (a)
x x ->a x -> a

So, fg is continuous at x = a.

ReadYourFlow.COM
CONTINUITY 9.5

(v) We know that


1
lim - (x) = lim
x->a / x -> a /Wj
r* ^
1 1 1 1
=> lim (x) = = ~z («)
x->a f lim f(x)
x—>a
f{a) fj

So, — is continuous at x = <7


/
(vi) We have.
lim / (x)
lim
r
— (x) = lim <
fix) x-*a
x->a x-+a g{x) lim ^(x)
x-> a

w
lim r (X) = - («)
g(a) g

So,
x^a Vg;
f— is continuous at x = a.
Flo
ee
g

Fr
THEOREM 2 Let f and g be real functions such that fog is defined. If g is continuous at x = aandf is
continuous at g (a), show that fog is continuous at x = a. or
ur
PROOF Since fog is defined. Therefore,
sf
Range (g) c Domain (f) => g (*) e Domain (/) for all x e Domain {g)
k

Now,
Yo
oo

g (x) is continuous at x = fl
B

=> lim g(x) = g (a) ...(i)


x ->a
re

/ is continuous at g (a)
ou
ad

=> lim f(g(x)) = fig (a))


gix)-*g(a)
Y

=> lim f{g(x)) = fig (a)) [From (i), xa => gix)-^g (fl)]
nd

x —>a
Re

=> lim {fog) (x) = ifog) ia)


Fi

x -> fl

ILLUSTRATIVE EXAMPLES
LEVEL-1
Type I ON TESTING CONTINUITY OF A FUNCTION AT A POINT WHEN THE FUNCTION HAS SAME
DEFINITION ON BOTH SIDES OF THE GIVEN POINT

; x*0
EXAMPLE l Test the continuity of the following function at the origin: f ix) = < x
; x=0
1
SOLUTION We observe that:
(LHL at x = 0) = lim / (x) = lim f iO-h) = lim f i-h)
x-»0 h-+0 /l 0

-h\
lim lim = lim -1 = -1
h-*0 -h h-+0 -h h->0

ReadYourFlow.COM
9.6 MATHEMATICS-XII

and. (RHL at x = 0) = lim f (x) = lim f (0 + h) = lim f (h)


x -> 0+ h->0 >0

= lim = lim - = lim 1 = 1


h->0 h h->0 h h->o

Thus, we have lim / (x) * lim / (*).


x->0~ x^0+
Hence,/(x) is not continuous at the origin.
ALITER We have.
;x>0
x
; x* 0 -x x, x>0
/(*) = *
1
; x=0
or, /(x) =
x
= -1 ; x < 0 *1 = - x, x<0 ]

low
1 ; x=0

(LHL at x = 0) = lim /(x) = lim -1 = -1 v /(x) = -1 for x < 0 and x -» 0


x -> 0“ ^->0 means that x < 0 such that x -» 0

ee
rF
Fr
(RHL at x = 0) = lim f(x) = lim 1=1 v /(x) =1 for x > 0 andx -> 0+
x->0+ x -> 0
for means that x > 0 such that x -> 0
lim /(x) ^ lim f(x).
x 0 x -> 0+
u
Hence,/(x) is not continuous at the origin.
ks
Yo

EXAMPLE 2 Show that the function f(x) given by


oo
B

x sin — , x*0
f(x) = x is continuous at x = 0. [NCERT EXEMPLAR]
re

0 , x=0
SOLUTION We observe that:
ou
ad

1
Y

(LHL at x = 0) = lim /(x) = lim f(0-h) = lim f(-h) = lim - h sin


x -> 0 h 0 h->0 l!->0 {-h)
nd
Re

= lim /? sin ( i ] = 0 x (an oscillating number between - 1 and 1) = 0


Fi

/* -»0 yhj
(RHL at x = 0) = lim /(x) = lim f{0 + h) = lim f(h) = lim /j sin f—1
x->0+ /? -> 0 h —> 0 h^O \hj
= 0 x (an oscillating number between -1 and 1) = 0
and, /(0) = 0.
Thus, we obtain lim /(x) = lim /(x) =/(0).
x -> 0~ x -> 0+
Hence, /(x) is continuous at x = 0.
sin x
EXAMPLE 3 Show that thefunction /(x) given by f(x) = • ^ + cos x , x ^ 0
2 , x =0
is continuous at x = 0.
SOLUTION We observe that

(LHL at x = 0) = lim /(x) = lim f(0-h) = lim f(-h)


x->0_ Jj -> 0 /j 0

ReadYourFlow.COM
CONTINUITY 9.7

sin(-fi) ... .. sink


= lim ----- + cos (- h) = lim —-— + lim cos h =1 + 1 = 2
h->0 h h^>0 h h~+0

(RHL at x = 0) = lim f(x) = lim f{0 + h)= lim f(h)


x->0 + /i -> 0 h —> 0
sin h sin h
lim + cos h = lim + lim cos h =1+1 =2
h-^Q h /i->0 h h->0
and. m =2
lim f(x) = lim f(x) = /(0).
x —> 0 0+

Hence, / (x) is continuous at .r = 0.


cos t

w
; t * tt/2
EXAMPLE 4 Examine thefunction f(t) given by f(t) = n/2 -1 for continuity at t = %/2.
1 i t = n/2

SOLUTION We observe that:

Flo
ee
(LHLatf =ti/2) = lim f(t)
t -> ji/2-

Fr
sin h
= lim fW2-h) = lim = lim = 1
h->0 J /t->0 rc/2 -(7c/2 -h) h—>0 h
for
ur
and, (RHL at f = ti/2) = lim f(t)
t n/2+
ks

.. ,, /n ,, COS 71/2 +ft) - sin h


Yo

= hm /(ti/2 + ft) = lim ——————— = lim


oo

h-+0 h-+0 ti/2-(rc/2 + ft) /i->0 -ft


B

sin ft
= lim
re

h->0 ft
and. /(7r/2)=l.
ou
ad

lim /(f) = lim /(f) = /(it/2).


Y

f -> rc/2 f -» rc/2 +


d

So, /(f) is continuous at f = 7t/ 2.


Re
n

, wftcn x ^ 0
Fi

EXAMPLE 5 Show that thefunction f(x) given by f(x) = • gl / * + ^ is discontinuous


0 , when x = 0

af x = 0. [NCERT EXEMPLAR]
SOLUTION We observe that:
(LHLat x = 0) = lim /(x) = lim /(0 - ft) = lim /(-ft)
x -> 0 h —> 0 h —> 0

o-i i
= lim —3-77— = lim = -l v lim - T7ft=°
h->0 e~1/h+ 1 h ->0 1+1 0+1 /i -»0 g
el/h
and. (RHL at x = 0) = lim /(x) = lim /(0 + ft) = lim /(ft)
x -»0+ ft->0 ft ->0
,1/ft _1 1 -1/e 1/ft 1-0
= lim = lim = 1
ft -> 0 e 1/ft + 1 ft -+0 1 + 1/e 1/ft 1+0

ReadYourFlow.COM
9.8 MATHEMATICS-XII

(LHL at x = 0) * (RHL at .r = 0)
So, / (x) is not continuous at x = 0 and has a discontinuity of first kind at x = 0.
Type 11 ON TESTING CONTINUITY OF A FUNCTION AT A POINT WHEN THE FUNCTION HAS DIFFERENT
DEFINITIONS ON BOTH SIDES OF THE GIVEN POINT
Let a function /(x) be defined as
()> (x) ; if x < <M*) ; if x < fl
/w={ \|/ (x) ; if x >
or, /(x) =
<|)(x) ; if x<fl
vj/ (x) ; if x > a
or, /(x) = k ; if x = a
v(/ (x) ; if x > fl
To test the continuity of such functions at x = a, we have to find left hand and right hand limits of
f(x) at x = a. For finding these two limits one can use the method which we have used in
previous examples or we can use the following method:
v x -> a <=> x < and x -> a

w
(LHL at x = a) = lim /(x) = lim (}>(x) lim /(x) = lim <j)(x) [v /(x) = <j)(x) for x<a]
x —> a X->fl
x-> a x —> a

Flo
Now, lim <))(x) can be calculated by various methods of evaluating limits as discussed in the
x —> a

ee
chapter on limits.

Fr
Similarly, we have
v x —> q+ <x> x> a&c x a
for
ur
(RHL at x - a) - lim /(x) - lim \\i(x) y (^ = ^ [•.• y(x) = y for x>d\
x->a x^a x->a+ x~>a
ks
Yo

Now, lim vj/(x) can be calculated by various methods of evaluating limits.


oo

X —M7
B

EXAMPLE 6 Discuss the continuity of the functionf(x) at x =1/2, ivhere


re

1/2-x ; 0 < x < 1/2


ou

/(*) = 1 ; x =1/2 [CBSE 2011]


ad

3/2-x ; 1/2 <x <1


Y

SOLUTION We observe that:


nd
Re

(LHL at x =1/2) = lim f(x)= lim (1/2-x) v /(x) = ^ - x for 0 < x < -^j
Fi

x —>l/2~ x -»1/2
= 1/2 -1/2 = 0 [Using direct substitution method]
and. (RHL at x = 1/2) = lim /(x) =
x->l/2+
lim (3/2-x)
x ->■ 1/2
••• /w=| - x for —2 <x<l
= 3/2-1/2 = 1 [Using direct substitution method]
Clearly, lim /(x) ^ lim /(x)
x -»1/2- x —> l/2+

Hence, /(x) is not continuous at x = 1/2. Clearly, /(x) has discontinuity of first kind at x = 1/2.
2 - x, x < 2
EXAMPLE 7 Discuss the continuity of thefunction f(x) given by f(x) = at x = 2.
2 + x, x > 2
SOLUTION We observe that:
(LHL at x = 2) = lim /(x) = lim (2-x) [’•• /(*) = 2 - x for x < 2]
x—>2

= 2-2 = 0

ReadYourFlow.COM
CONTINUITY 9.9

and. (RHL at x = 2) = lim /(x) = lim (2 + x) [y f{x) = 2 + x = for x > 2]


x -> 2 + x -> 2

=2+2=4
lim f(x) + lim fix).
x -> 2“ x -> 2+
Hence,/(x) is not continuous at x = 2.
5x -4 , when 0 < x < 1
EXAMPLE 8 Show that fix) = is continuous at x =1.
4x3 - 3x / w/ien 1 <x <2
SOLUTION We have,
(LHL at x = 1) = lim fix) = lim 5x - 4 [y fix) =5x - 4, when x < 1]
x->l"
= 5x1-4 =1
(RHL at x =1) = lim fix) = lim 4x3 - 3x [y fix) =4x3 - 3x, x >1]

w
x->l+
= 4(1) ^ - 3(1) = 1
and, /(l) = 5 x 1 - 4 = 1
lim fix) = /(l) = lim fix) .
Flo [•.■ fix) =5x - 4, where x < 1]

ee
x -»1" x ->1+

Fr
So, fix) is continuous at x = 1.
EXAMPLE 9 Shoiv that the function fix) = 2x -1 x | is continuous at x = 0.
for [CBSE 2002]
ur
SOLUTION We have.
2x-x , if x>0
fix) = 2x-|x| = 2x -(-x) , if x<0
ks
Yo

x , if x>0
oo

=> /(*) = 3x , if x < 0


B
re

Now,
(LHL at x = 0) = lim /(x) = lim 3x = 3 x 0 = 0
ou

x—»0
ad

x -> 0
Y

(RHL at x = 0) = lim fix) = lim x = 0


x->0+ x -> 0+
nd
Re

and. /(0) = 0
lim fix) = lim /(x) = / (0).
Fi

x —> 0 0+

So, / (x) is continuous at x = 0.


EXAMPLE 10 Discuss the continuity of the function of given by / (x) = | x -11 +1 x - 21 at x =1 and
x = 2.
SOLUTION We have,
fix) = |x-l|+|x-2|
-(x-1) -(x-2) , if x<l
/(*) = (x -1) - (x - 2) , if 1 <x<2
(x -1) + (x - 2) , if x > 2
-2x+3, ifx<l
=> /w = 1 , if 1 <x<2
2x - 3 , x > 2
Continuity at x = 1:
lim /(x) = lim (-2x + 3) = - 2 x 1 + 3 =1, lim /(x) = lim 1 =1 and, / (1) =1.
x-+l“ x->l_ x->l+ 1+

ReadYourFlow.COM
9.10 MATHEMATICS-XII

lim f (x) = f (1) = lim / (x)


x->r X -» 1+
So, / {x) is continuous at x = 1.
Continuity at x = 2:
lim f (x) = lim 1=1, lim f (x) = lim (2x - 3) =2x2-3 =1
x^>2~ x->2~ x —> 2 + AT —> 2

and. / (2) =2x2-3 =1.


lim / (x) = lim / (x) = / (2)
x -> 2- x-»2+
So, / (x) is continuous at x = 2.
Type III ON FINDING THE VALUE(S) OF A CONSTANT GIVEN IN THE DEFINITION OF A FUNCTION WHEN

w
IT IS CONTINUOUS AT AN INDICATED POINT
A function /(x) is continuous at a point x = iff lim /(x) = lim /(x) = f(a).
x ->a~ x -> a+
But, lim /(x) = lim /(x) => lim /(x) exists.
X « x -* fl+ X

Flo
ee
Fr
Thus, /(x) is continuous at x = a iff lim /(x) =/(«) .
x -* a
for
ur
We will use this result in finding unknown quantity in the definition of a function when it is
given to be continuous at a given point.
ks

EXAMPLE 11 Determine the value ofkfor which thefolloivingfunction is continuous at x = 3.


Yo
oo

x2 -9 , x* 3
eB

/(*) = x- 3
k ' *=3
r
ou
ad

SOLUTION Since /(x) is continuous at x = 3.


Y

lim /(x) = /(3)


x —> 3
nd
Re

=> lim /(x) = k [••7(3) =fc]


x —> 3
Fi

2 -9 (x - 3) (x + 3)
=> lim - = k => lim = /c => lim (x+3)=k=>6=k
x-> 3 x - 3 x —>• 3 x-3
Thus, /(x) is continuous at x = 3, if /c = 6.
EXAMPLE 12 Find the value ofthe constant Xso that thefunction given below is continuous at x = -l.
x2 - 2x - 3 , x * -1
/(*) = x +1
X x =—1

SOLUTION Since /(x) is continuous at x = - 1.


lim /(x) =/(-!)
x —> — 1
x2 - 2x - 3
=> lim = X [v /(-1)=^]
x —> — 1 x +1

ReadYourFlow.COM
CONTINUITY 9.11

(*-3) (x + 1)
lim = A, => lim (x - 3) = X => - 4 = X
x —> —1 x+1 x —> — 1
So, /(x) is continuous at x = -1, if A, = - 4.
EXAMPLE 13 Find the value of the constant k so that the function given belozv is continuous at x = 0.
1 - cos 2x , x * 0
/(*) = 2x2
k / x=0
SOLUTION It is given that the function /(x) is continuous at x = 0.
lim /(x) = /(0)
x->0
1 - cos 2x
=> lim i = k [••7(0) =fc]
x-^0 2x2

low
2 sin2 x
=> lim = k
x —> 0 2x2

sin x7
=> lim = k=> l2 = k=>k=l

ee
x -> 0 X
rF
Fr
Thus, /(x) is continuous at x = 0, if /c = 1.
2x-l , x<2
EXAMPLE 14 Find the value of 'a' z/f/ze function fix) defined by /(x) = a , x = 2 is continuous
for
at x = 2. x+1 , x>2
u
SOLUTION We observe that:
ks
Yo

(LHL at x = 2) = lim /(x) = lim (2x-l) =2x2-1 = 3,


oo

x —>2_ x~*2
B

(RHL at x = 2) = lim /(x) = lim (x + 1) = 2 + 1 = 3,


re

x —»2+ ^->2
and. /(2) = «
ou
ad

Since /(x) is continuous at x = 2. Therefore,


Y

lim /(x) = lim /(x) = /(2) =^3 = 3 = a=>a = 3


nd

2+
Re

Thus, /(x)is continuous at x = 2, if a - 3.


Fi

log (1 + ax) - log (1 -bx) , if x * 0


EXAMPLE 15 If the function f(x) defined by /(x) = • x
zs continuous at x = 0,find k. k , if x = 0
SOLUTION Since /(x) is continuous at x = 0.
lim /(x) = /(0)
x -» 0
log (1 + ax) - log (1 ~bx) _k
=> lim [■■■f(0)=k]
x -> 0 x
| log (1 + ax) log (1 -bx) \ =k
=> lim
x -> 0
log (1 + ax)
=> lim - lim log (1 k
x-»0 x x —> 0 x
log (1 + ax) log (1 -bx) = k
=> a lim -(-b) Urn
x —> 0 ax x -> 0 {-b)x

ReadYourFlow.COM
9.12 MATHEMATICS-XII

log(l + x) =1
=> a(l)-(-b)(l) = k Using: lim
*->■0 x
a+b = k
Thus, f{x) is continuous at x = 0, if /c = « + fo.
example 16 Find the values of 'a' so that the function f(x) defined by
sin2 ax , x * 0
/(*) = x2 may be continuous at x = 0.
1 , x = 0
SOLUTION The function /(x) will be continuous at x = 0, iff
lim /(x) = /(0)
x —> 0
•2
sm ax

w
=> lim = 1 [• /(0) = 1]
x-»0 x2

2 sm ax \2

Flo
}
=> a lim = 1
ax

ee
a2 (l)2 = 1 => fl = ± 1

Fr
Thus, /(x) will be continuous at x = 0, if fl = ± 1.
for
3ax + b , if x>l
ur
example 17 If thefunction f (x) given by f (x) = 11 , if x =1 is continuous at x = l,find
5ax -2b , if x < 1
ks
Yo

the values ofa and b. fCBSE 2002,2010,2011,20121


oo

SOLUTION We observe that:


eB

(LHL at x = 1) = lim / (x) = lim (Sax -2b) = 5a -2b


x^r x —> 1_
r
ou
ad

(RHL at x = 1) = lim. / (x) = lim (3ax + b) =3a + b


x -> 1+ x->l+
Y

and. /(l) = 11-


nd
Re

Since / (x) is continuous at x = 1.


lim / (x) = lim / (x) = / (1)
Fi

x->r 1+
=> 5a-2b = 3a+ b = 11 => 5a-2b =11 and 3a + b = 11 => a = 3 and b = 2
1 - cos 4x
, if x < 0
x2
EXAMPLE 18 Letf(x) = ■ a , if x = 0
4~x , if x > 0
^16 + Vx - 4
Determine the value of a so that f (x) is continuous at x = 0.
[CBSE 2010, 2012, 2013 NCERT EXEMPLAR]
SOLUTION For/(x) to be continuous at x = 0, we must have
Urn / (x) = lim / (x) = / (0)
x -> 0 X (T
=> lim /(x) = lim / (x) = a -(i)
x -> 0 x —> 0 +

ReadYourFlow.COM
CONTINUITY 9.13

1 - cos 4x 1 - cos 4x
Now, lim /(x) = lim V /(X) = for x < 0
x-»0 x -» 0 X2 x2

2 sin2 2x
=> lim /(x) = lim
x->0 x ->Q x2

sin 2x'i2 sin 2x^2


=> lim / (x) = 2 lim = 2 x 4 x lim = 8 (l)2 = 8 ...(ii)
x-»0 x-»0 x x-»0 2x

Vx for x > 0
and. lim /(x) lim ••• / (x) =
0+ x -> 0 -^16 + Vx — 4 ■Jl6 + Vx -4

=> lim /(x) = lim •^16 + Vx + 4


x —> 0 + x -> 0 16 + Vx -16

low
=> lim / (x) = lim ^16 + Vx + 4 = 4+4 = 8 ...(iii)
0+' x -> 0
From (i), (ii) and (iii), we get a = 8.

ee
(4* -l)3
rF
Fr
example 19 Determine f (0) so that the function f (x) defined by f (x) =
. X
x2'
sm log 1 +
4 6 3
for
u
becomes continuous at x = 0.
ks

SOLUTION For/(x) to be continuous at x = 0, we must have


Yo
oo

lim /(x) = /(0)


x->0 \3
B

4* -1
re

(4X -l)3 x
=> / (0) = lim / (x) = lim = lim \\
2
ou

x->0 x->0 X2
ad

X . , , X
sm - log 1 + — x
4 3 sm - 0log 1 + —
3
Y

4
^4 x -
* x2
nd

—x 3
Re

4 3
Fi

(loge 4)3
/(0) = = 12 (log, 4)3.
1 1
—x—
4 3
V2 cos x—1 , x + .Find the value off ^ jso that f(x) becomes continuous
EXAMPLE 20 If f (x) =
cot x —1
atx = n/4. [NCERT EXEMPLAR]
71
SOLUTION For / (x) to be continuous at x = -, we must have

lim /M=/ff|
x-^n/4: v4y
V2cosx-l
=> lim
COtX-1

ReadYourFlow.COM
MATHEMATICS-XII

V2( cos x-cos-K 2sin[ — sin


x tt;

U 8 2+8 . TC
=> lim — = -V2 lim x sinxsin —
x n/4 cot x-cot— X —> 71 / 4 . TC TC 4
cos x sin — sin x cos-
4 4 4
sin ( — I sin f —+ -
=> /f -jl = -2-v/2 lim 2 8 2 8
x sm x sin —
. TC

v4^ x —► it / 4 -sin [ x — — 4
4
X TC ) . I X TC
sm ------ I sm I - + -
f[~'\ = -2V2 lim 2 8 2 8 x sm x sm —
. TC

u; x-»7t/4 _2sin f ^^ COS


X TC 4
U 8 2 8
. X TC
sin - + -

w
{2 8
=> /ff|
V4y
= V2 x —>lim7c / 4 X TC
x sm x sm -
4
. TC

COS

Flo
2 8
. f TC TC
sm - + -

ee
2
=>
18 8 x sin — sin — =f sin — 1

Fr
TC TC 4 4 4 2
COS
8 8 for
ur
LEVEL-2

EXAMPLE 21 Prove that the greatest integer function [x] is continuous at all points except at integer
ks

points. [NCERT]
Yo
oo

SOLUTION Let/(x) = [x] be the greatest integer function and let k be any integer. Then,
eB

k-l , ii k-1 <x <k


/(*) = M = k , if k<x<k + l
[By definition of [x]]
r

Now,
ou
ad

(LHLatx=fc) = lim /(x) = lim f(k-h) = lim [k-h]


Y

x —> k /1 -> 0 h -> 0


nd

= Urn (k-l) = k-l [: k-l<k-h<k [k-h] = k-l]


Re

/i->0
Fi

and.
(RHLatx = /c)= lim f(x) = lim f(k + h) = lim [k + h]
X -4 k+ h —> 0 /!->0

lim k = k [v k <k + h <k + 1 [k + h] = k]


h-+0
lim /(x) ^ lim /(x) .
x->k~ x ->
So, /(x) is not continuous at x = /c.
Since k is an arbitrary integer. Therefore,/(x) is not continuous at integer points.
Let a be any real number other than an integer. Then, there exists an integer k such that
k-l <a <k.
Now,
(LHL at x = <?) = lim f(x) = lim f(a - h) = lim [a - h]
X —HI h-*0 h —> 0

lim k-l = k -1 [•.• k -1 <a-h <k [a -h]-k -1]


/!-»0

ReadYourFlow.COM
CONTINUITY 9.15

(RHLatA: = a) = lim f(x) - lim f(a + h)


a+ h^O
•/ k -1 <a + h <k
= lim [a + h] = lim (k -1) =k -1
/i->0 h-+0 [a + h] = k -1
and, f(a) =[a] =k-l [•: k - l <a <k [a] = -1]
Thus, lim f(x) = lim f(x) = f(a)
x-a x->a+
So, f(x) is continuous at x = a. Since a is an arbitrary real number, other than an integer.
Therefore, f(x) is continuous at all real points except integer points.
EXAMPLE 22 Letf (x + ij) =f (x) + f (y)for all x, y <= R. Iff (x) is continuous at x = 0, show thatf(x)
is continuous at all x.
SOLUTION Since/(discontinuous atx = 0. Therefore,
lim / (x) = lim / (x) = f (0)

w
.* -» 0 x 0+
lim / (0 - /j) - lim f (0 + h) = f (0)

=>
h->0 h-+0
lim / (0 + (- h)) = lim f (0 + h) = f (0)
Flo
ee
/i -> 0 /? -> 0

Fr
=> lim [f (0) + f {-h)] = Urn \f (0) + f {h)] = f (0) [Using :/(x + y) =/(x)+/(y)]
h->0 h 0 or
ur
=> /(0)+ lim /(-&) =/(0)+ lim /(ft) =%(0)
h —^ 0 /i -> 0
sf

=> lim / (- h) = lim f (h) = 0


k
Yo

/i -> 0 h->0
oo

Let a be any real number. Then,


B

lim / (x) = lim f(a-h) = lim f(a + (-hft.


re

x->a h^0 /i 0
ou
ad

=> lim /(x) = lim lf(a)+f(-h)] [■•■ / (x + y) =/(d+/(y)]


x -> a li->0
Y

=> lim / (x) = / (a) + lim / (- h)


h->0
nd

x->a
Re

=> lim / (x) = / (a) + 0 [Using (i)]


Fi

=> lim / (x) = / (a).


x-> a
and. lim / (x) = lim f (a + h)
a+ /z -> 0
=> lim / (x) = lim \f (a) + f (h)] [••• f (x + y) =f (x) + f (y)]
a+ h ->0

=> lim / (x) = f (a) + lim / (h)


X -> /!->()
=> lim / (x) = / (fl) + 0 = / (fl) [Using (i)]

Thus, we have
lim /(x) = lim /(x) =f(a)
x-*a a+

f (x) is continuous at x = a.
Since is an arbitrary real number. So, / (x) is continuous at all x e K.

ReadYourFlow.COM
r

9.16 MATHEMATICS-XII

Type IV ON CONTINUITY OF COMPOSITE FUNCTION


EXAMPLE 23 Show that thefunction f (x) = | sin x + cos x | is continuous at x = n.
SOLUTION Let g (x) = sin x + cos x and h(x) = | x |. Then,
(hog) (x) = h (g(x)) = h (sin x + cosx) = | sin x + cosx| = /(x)
In order to prove that/ (x) is continuous at x = n. It is sufficient to prove that g (x) is continuous
atx = re and (x) is continuous at g (tt) = sin tt + cos n =-l.
Now,

lim g(x) = lim (sin x + cos x) = sin cos 7r = -l and, g(7r) =-l
x —> n X->K

lim g(x) = g(n)


X —» 7t

So, g (x) is continuous at x = n.

w
Let y = g(7t) = -1.
Now, lim h (y) = lim |y| = lim -y--(-l) = 1
y-»-l y->-i ' y-»-1

Flo
ee
and. h(g(K)) = h(-l) = |-1| = 1.

Fr
lim h (y) = h(g(n))
y->-i
for
ur
=> lim h(g(x)) = h (g(n))
g(x)-+~ 1
ks
Yo
oo

=> lim MgM) = /J(g(^))


B

g(x)->g(n)
re

■=>
h(x) is continuous atg (tt)
ou

Hence,/(x) = /tog (x) is continuous at x = n.


ad
Y

EXERCISE 9.1
nd

LEVEL-1
Re
Fi

1. Test the continuity of the following function at the origin:

u \ = Irr
f(x) j |x| ' x*°
1 , x=0
x2 - x - 6
2. A function /(x) is defined as /(x) = • ; if x 9* 3
x-3
5 ; if x=3
Show that/(x) is continuous at x = 3.
x2 -9
; if x* 3
3. A function /(x) is defined as /(x) = x-3
6 ; if x = 3
Show that/(x) is continuous at x = 3.

ReadYourFlow.COM
CONTINUITY 9.17

X2-l . .
; for x . Find whether /(x) is continuous atx = 1.
4. If/(^) = x-1 '
2 ; for x = 1
sin 3x
<v5)lf f(x) = , when x* 0 . Find whether f(x) is continuous at x = 0.
x
1 , when x = 0

e1/x , if x * 0 Find whether/ is continuous at x = 0.


6. If /(x) =
1 , if x = 0 •
1 - cos X
, when x ^ 0
7. Let /(x) = • x2 . Show that/(x) is discontinuous at x = 0.
1 , when x = 0

w
x -| x|
, when x * 0 is discontinuous at x = 0.
8. Show that /(x) = 2
2 , when x = 0

Flo
ee
x-a\
, when x±a

Fr
9. Show that /(x) = x-a is discontinuous at x = a.
1 , when x = a
or
ur
10. Discuss the continuity of the following functions at the indicated point(s):
sf

*1 cos -] , x*0 [NCERT EXEMPLAR]


atx = 0
k

x
Yo

(i) /(*) =
oo

0 , x=0
B

x2 sinf-
re

, x^0
(ii) /(*) = x at x = 0
0 , x =0
ou
ad
Y

1
(x - a) sin , x* a
(hi) /(x) = x — Cl at x =
nd
Re

0 , x=a
Fi

ex -1
, ifx^O
(iv) /(x) = i0g (i + 2x) at x = 0
7 , if x = 0

1-x"
, X^1
(v) f(x) = 1 -x neN at x =1
n-1 , x =1

i , for x ^ 1
(Vi) /(x) = x-1 at x = 1
2 , for x = 1

21 x| + x2
(vh) f(x) = ,x* 0 at x = 0
x
0 ,x=0

ReadYourFlow.COM
9.18 MATHEMATICS-XII

I A' - a I sin I , for x * a


(Viii) /(A) \x-aj at x =n [NCERT EXEMPLAR]
0 , for x = a

l+x2 / if 0 <x <1


11. Show that/(a) = is discontinuous at a = 1.
2- x , if a > 1

sin 3a
, if a < 0
tan 2a
3
12. Show that /(a) = , if A = 0 is continuous at A = 0
2
log (1 + 3a)
, if a > 0
e2x-l

w
13. Find the value of 'a' for which the function / defined by

Flo
rt sin ^ (a + 1), a < 0
/(*) = is continuous at a = 0. [CBSE2011]

ee
tan a -- sin a
,x>0
A3

Fr
3x - 2 , a < 0
14. Examine the continuity of the function /(a) = for at a = 0.
ur
A+1 , A>0
Also sketch the graph of this function.
ks

a , a>0
Yo
oo

15. Discuss the continuity of the function/(a) = • 1 , a = 0 at the point a = 0.


-A , a<0
B
re

a , 0 < a <1/2
16. Discuss the continuity of the function/(a) = 1/2 , a =1/2 at the point a =1/2.
ou
ad

1 —A , 1/2 < A <1


Y

2a-1 , a<0
17. Discuss the continuity of /(a) = at a = 0. [CBSE2002]
nd

2a + 1 , a>0
Re
Fi

a2
-1
, x*1
18. For what value of k is the function f(x) = x-\ continuous at a = 1 ?
k , a =1

a2 - 3x + 2
, if x*l
19. Determine the value of the constant k so that the function /(a) = < A —1
k , if x=l
is continuous at a = 1
sin 5 a
, if a ^ 0
20. For what value of k is the function /(a) = 3a continuous at a = 0 ?
k , if x = 0
21. Determine the value of the constant k so that the function
kx2 > if a <2
^=3 is continuous at a = 2. [NCERT]
, if a > 2

ReadYourFlow.COM
CONTINUITY 9.19

22. Determine the value of the constant k so that the function


sin 2.r
, if x * 0
fix) = 5x is continuous at x = 0. ICBSE2007]
k , if x = 0
ax + 5 , if x < 2
23. Find the values of a so tliat the function/(x) = is continuous at x = 2.
x -1 , if x > 2
[CBSE2002]
x
, x*0
24. Prove that the function /(x) = • | x | + 2x2 remains discontinuous at x = 0,
k , x =0

regardless the choice of k. [NCERT EXEMPLAR]

w
k cos x , x * n/2
25. Find the value of k if/(x) is continuous at x = n/2, where/(x) = n — 2x
, x = n/2

Flo 3

ee
[NCERT]

Fr
26. Determine the values of a, b, c for which the function
sin (fl + 1) x + sin x , for x < 0 for
ur
x
fix) = c , for x = 0 is continuous at x = 0.
ks

yjx + bx2 -Vx


Yo

, for x > 0
oo

bx3/2
eB

1 - cos kx
, x*0
27. If/(x) = ■ x sin x is continuous at x = 0, find k.
r

1
ou
ad

, x=0 [NCERT EXEMPLAR]


2
Y

x-4
+a , if x <4
nd
Re

x -4|
Fi

28. If/(x) = • a+b , if x = 4 is continuous at x = 4, find a, b.


x-4 [NCERT EXEMPLAR]
+b
x - 41 , if x>4
sin 2x , x * 0
29. For what value of k is the function /(x) = • x continuous at x = 0 ?
k , x =0
log(l + -;)-log(l-^
30. Let /(x) , x * 0. Find the value of / at x = 0 so that / becomes
x
continuous at x = 0.
2* + 2 -16
, if x * 2
31. If/(x) = 4 'v -16 is continuous at x = 2, find/c.
k , if x = 2 [NCERT EXEMPLAR]

ReadYourFlow.COM
9.20 MATHEMATICS-XII

cos 2 x - sin 2 x -1
r x*0
32. lff(x) = ^x2 +1 -1 is continuous at x = 0, find k.
k , x=0
1 - cos 7 (x - n)
33. Extend the definition of the following by continuity / (x) = at the point
5 (x - it)2
X = K.
2x + 3 sin x
34. If / (x) = x 0 is continuous at x = 0, then find / (0).
3x + 2 sin x '
1 - cos 4x
, when x * 0
35. Find the value of k for which / (x) = • 8x2 is continuous at x = 0.
k , when x = 0

w
[CBSE 2000 C, 2017, NCERT EXEMPLAR]
36. In each of the following, find the value of the constant k so that the given function is
continuous at the indicated point:
1 - cos 2kx if x * 0

Flo
ee
(i) /(*) = x2 at x = 0

Fr
8, if x = 0

if x * 1
for
ur
at x=l
k , if x = 1
ks
Yo

_ J (x2 - 2x) , if x < 0


oo

(hi) /(x) at x = 0 [NCERT]


cos x , if x > 0
B

kx + 1 , if x <
re

tc
(iv) /(x) = at x = n [NCERT]
COS X , if X > 7T
ou
ad

fcx +1 , if x < 5
Y

(v) f(x) = at x = 5
3x - 5 , if x > 5
nd
Re

x2 -25
r x*5
Fi

(vi) /(*)=< x -5 at x = 5 [CBSE 2007]


k , x =5

(vu) at x =1 [CBSE 2007]

(x2 + 2) , if x < 0
(viii) f(x) = at x = 0. [CBSE 2010]
3x +1 , if x > 0
x3 +x2 -16x + 20
,x^2
(ix) /(x)=- (x-2)2 at x-0. [NCERT EXEMPLAR]
k /X = 2
37. Find the values of a and b so that the function / given by
1 , if x < 3
f(x) = lax+ b , if3<x<5 is continuous at x = 3andx =5. [CBSE 2013]
7 , if x >5

ReadYourFlow.COM
CONTINUITY 9.21

x2
if 0 < x < 1
38. If/(x) = • 2 ' . Show that/ is continuous at x = 1.
2x2 - 3x + — , if 1 < x < 2
2 '
39. Discuss the continuity of the / (x) at the indicated points:
(i) /(X) = |x| + |x-l| at x = 0,1. [NCERT EXEMPLAR]
(ii) / (x) = | x -11 +1 x + 11 at x = -1,1.
x -| x| , x * 0
40. Prove that/ (x) = x is discontinuous at x = 0.
2 , x=0

2x2 + /c , if x > 0
41. If/(*)=- , then what should be the value of k so that / (x) is
- 2x2 + , if x < 0

w
continuous at x = 0.
_ < A, (x2 - 2x) , if x < 0
42. For what value of A. is the function / (x)

Flo 4x +1 , if x > 0
continuous at x = 0?

ee
What about continuity at x = ± 1 ? [NCERT]

Fr
43. For what value of k is the following function continuous at x = 2?
f 2x + 1 ; if x < 2 or
ur
/(*) = k ; x =2 [CBSE 2008]
sf
3x -1 ; x > 2
k
Yo

1 - sin 3 x
oo

., TC
if X < —
3 cos2 x 2
B

44. Let/(x) = a , if x = ^ . If / (x) is continuous at x = —,


^ find a and b.
re

(1 - sin x) K
ou

(n-2xf ' if X > —


ad

2
Y

[CBSE 2008, 2016]


45. If the functions / (x), defined below is continuous at x = 0, find the value of k:
nd
Re

1 - cos 2x
Fi

, x<0
2x2
f(x) = k , x=0 [CBSE 2010]
x
, x>0

46. Find the relationship between 'a1 and 'b' so that the function '/' defined by
ax +1, if x < 3
f(x) = is continuous at x = 3. [CBSE 2011]
bx + 3 , if x > 3
_________ ANSWERS
1. Discontinuous 4. Continuous 5. Discontinuous 6. Discontinuous
10. (i) Continuous (ii) Continuous (iii) Continuous (iv) Discontinuous
(v) Discontinuous (vi) Discontinuous (vii) Discontinuous (viii) Continuous
1
13. a=- 14. Discontinuous 15. Discontinuous 16. Continuous
2
17. Discontinuous 18. 2 19.-1 20. 5/3

ReadYourFlow.COM
9.22 MATHEMATICS-XII

21. 3/4 22. 2/5 23. -2 25. 6


26. fl = --/be2?-{0}/c = - 27. ±1 28. a = l,b = -l
2 2
a +
29. 2 30. 31. 1/2 32. -4
flb
33. 49/10 34. 1 35. 1
36. (i) A: = ± 2 (ii) k = — (iii) No value of £(iv) k =— (v) /e=- (vi) A: =10
n , n 5
(vii) A: = 4 (viii) k=l/2 (ix) k =7 37. a = 3,b=-8
39. (i) Continuous, (ii) Continuous 41. k is any real number.
42. There is no value of X. for which it is continuous at x = 0.
At X = ± 1, f (x) is continuous. 1
43. k=5 44. a = 2-b-i
45. k =1 46. 3a-3b =2

w
HINTS TO NCERT& SELECTED PROBLEMS
kx2 , x< 2
21. lff(x) =
3 , x>2
is continuous at x = 2, then

Flo
ee
lim / (x) = lim f(x) = f (2)

Fr
x -»2~ x-+2+
=> lim kx2 = lim 3 = k (2)2 for
ur
■Y -> 2~ a:^2+

=> 4A: = 3 =>k= —


ks

4
Yo

A: cos x
oo

71

25. It is given that / (x) = ■ n-2x ^ is continuous at x = -.


B

K 2
re

3 , x =—
2
ou
ad

lim /(x)=/ff)
y -> n/2 V2/
Y

k cos x sin (7t/2 - x) = kxl


=> lim - 3 => k lim
nd

3 => k = 6
Re

x —> jt/2 7t-2x x —> ti2 2(k/2-x) 2


Fi

k(x2 -2x), x < 0


36. (iii) It is given that / (x) =
cos x , x > 0
Now,
lim / (x) = lim A- (x2 - 2x) = 0 for all k
Y —> 0 Y —> 0_

lim / (x) = lim cos x =1 and,/(0) =cos 0 =1


Y —> 0
+ Y —> 0
+

Clearly, there is no value of k for which lim / (x) = lim / (x) =/ (0) may hold good.
Y —> 0 ~ Y->0 +

Hence, there is no value of k for which / (x) is continuous at x = 0.


Ax + 1 , X < 71
(iv) It is given that / (x) = is continuous at x = tc
COS X , X > 71

lim /(x) = lim f(x)=f(n)


Y —> 7t Y —> TC

ReadYourFlow.COM
CONTINUITY 9.23

=> lim kx + 1 = lim cos x = kn + 1


X -> K
+
2
=> k k + 1 = cos k => kn + 1 = -1 => k=-~
n
9.4 CONTINUITY ON AN INTERVAL
CONTINUITY ON AN OPEN INTERVAL A function f(x) is said to be continuous on an open interval (a, b)
iff it is continuous at every point on the interval (a, b).
CONTINUITY ON A CLOSED INTERVAL A function f(x) is said to be continuous on a closed interval [a, b]
iff
(i) f is continuous on the open interval (a, b) (ii) lim f(x)= f(a) and, (Hi) lim f(x)=f(b).
x —> x —> b~
In other words, f(x) is continuous on [a, b] iff it is continuous on (a, b) and it is continuous at a from the
right and at bfrom the left.

low
CONTINUOUS FUNCTION A function f(x) is said to be continuous, if it is continuous at each point of its
domain.
EVERYWHERE CONTINUOUS FUNCTION A function f(x) is said to be everywhere continuous if it is
continuous on the entire real line (- co, oo).

ee
9.5 PROPERTIES OF CONTINUOUS FUNCTIONS
rF
Fr
In this section, we shall learn some properties of continuous functions and prove the continuity
of some standard real functions in their domains. for
THEOREM 1 Iff and g are two continuous functions on their common domain D, then
u
(i) f + g is continuous on D
ks

(ii) f - g is continuous on D
Yo
oo

(iii) fg is continuous on D
(iv) a f is continuous on D, where a is any real number.
B
re

(v) — is continuous on D - [x: g (x) * 0}


g
ou
ad

1
(vi) — is continuous on D ~{x:f (x) * 0}
Y

f
PROOF Let fl be an arbitrary point in common domain D.
nd
Re

Since / and g are continuous on D. So, they are also continuous at V.


Fi

lim f(x) = f (a) and lim g(x) = g(a) -(i)


x-> a x a
(i) We have.
lim (f + g) (x) = lim (/ (x) + g (x))
a: -» fl x —> a
=> lim (/ + g) (x) = lim / (x) + lim g (x)
x —> <J X -> (7 x —> a

=> lim (J + g)(x) = f (a) + g (a) = (f + g) (a) [Using (i)]


x->a
f + gis continuous atx = a.
Since a is an arbitrary point in D. Hence, / + g is continuous on D.
(ii) We have.
lim (f-g) (x) = lim (f(x)-g(x))
x —> rt X ->rt

=> lim (f-g) (x) = lim f(x)~ lim g(x)


X ->(7 x -+a x -> a

ReadYourFlow.COM
9.24 MATHEMATICS-XII

=> lim (f-g) (x) = f (a)-g(a) = {f-g)(a) [Using (i)]


x a

f-g is continuous at x = a.
Since a is an arbitrary point in D. Hence, / - g is continuous in D.
(iii) We have.
lim (/£)(*) = lim (J(x) g(x))
x—>a x->a

=> Ihn (Jg) (x) = lim f (x) x lim g (x) [Using (i)]
x—>a x ->a x-+a

=> lim ifg) (x) = f(a) g(a) = (Jg) {a)


x —> u
fg is continuous at x = a.
Since a is an arbitrary point in D. Hence, fg is continuous in D.

w
(iv) We have.
lim (of) (x) = lim (a/(x))
x a x —>n
=> lim (of) (x) = a lim / (x)

Flo
ee
x->a x-+a

Fr
=> lim (of) (x) = a/(a) [Using (i)]
x -* a
=> lim (of) (x) = (a f) (a)
for
ur
x->a
a / is continuous at x = a.
ks

Since a is an arbitrary point in D. Hence, of is continuous in D.


Yo
oo

(v) Let a eD such thatg (a) * 0. Then,


B

lim - (x) = lim


fix)
re

X -»fl g(x)
ou

lim f (x)
ad

=> lim 7y- x—


Y

(X) =
UJ lim g(x) g(a) 8)
x —> a
nd
Re

f— is. continuous at x = a.
Fi

8
f continuous onD - {x :g (x) ^ 0).
Since a is an arbitrary point in D such thatg (a) * 0. Hence,—is
8
(vi) Let a eD such that / (a) * 0. Then,
lim \\ (x) = lim * 1 1
[Using (i)]
x —y a / x —> fl fix) lim f(x) f(a)
x->a

=> lim
X —> <3
j
4lw = l/J4 w
— is continuous at x = a.
f
Since a is arbitrary point in D such that f(a) * 0. Hence, 1// is continuous on D - (x: /(x) * 0}.
Q.E.D.

ReadYourFlow.COM
CONTINUITY 9.25

THEOREM 2 The composition of two continuous functions is a continuous function.


PROOF Let / and g be two real functions such that go/exists. Then, Range (/) c Domain (g).
Let a be an arbitrary point in the domain of /.
Then, o e Domain (/)=>/ (a) e Range (/)=>/ (a) e Domain(g) [v Range (/) c Domain (g)]
Since / and g are continuous on their domains. Therefore,
a e Domain (/) and / (a) e Domain (g)
=> /is continuous atx = a andgis continuous at / (a)
lim f(x) = f (a) and lim g (y) = g (f(a))
X —> (7 y->/(«)
=> lim f(x) = f (a) and lim g (/(*)) = gif(a)), where y = f(x)
X —> fl /(x)->/(a)

=> lim g(/(.t)) = g(/(fl)) [ ■• ^ A => /(*)->/ («)]

w
x -> fl

=> lim go/ (x) = go/ (o)


x —> fl

=> go/ is continuous at x = o.

Flo
ee
Since o is an arbitrary point in its domain. Hence, go/ is continuous. Q.E.D.

Fr
THEOREM 3 Iff is continuous on its dot?iain D, then \f\is also continuous on D.
PROOF Recall that | /1 (known as absolute function) is defined as | /1 (x) = | / (x) |.
or
ur
Let a be an arbitrary real number in D. Then, / is continuous at a.
f
ks

lim fix) = / («)


Yo

x —>a
Now,
oo

[By definition of | /|]


eB

lim | /1 (x) = lim | fix) \


x —> ci x —> a
r

=> lim | /1 (x) = lim fix) = | /(fl) | = | / | (fl)


ou
ad

x' —> n x ->n


Y

| / | is continuous at x = a.
nd

Since a is an arbitrary point in D. Therefore, | /1 is continuous in D. Q.E.D.


Re

REMARK The converse of the above theorem may not be true. For example, consider the function
Fi

1 , if x eZ
fix) = -1, if xeR-Z

Let a be an arbitrary integer. Then,


lim fix) = lim f(a-h) = lim -1=-1 [v h >Q, a -h <£Z as /z is very small]
x —> Cl h —> 0 h-^0

lim / (x) = lim f ia + h) - lim -1 = -1


a+ h —> 0 h —> 0

and. fia) = 1.
lim / (x) = lim / (x) * / ia)
x —> a x -> a+

So, / is discontinuous at x = a.
Now, | / | (x) = | / (x) | = 1 for all x e R. So, |/| is a constant function and hence , it is
everywhere continuous.

ReadYourFlow.COM
9.26 MATHEMATICS-XII

THEOREM 4 A constant function is everywhere continuous.


PROOF Letf(x) = c, where c is a constant. Clearly, the domain of a constant function is R.
Let a be any real number. Then,
lim f (x) = lim c = c and,/(a) = c.
x a x —m

lim f(x) = f (a)


x-*a
=> f (x) is continuous at x = a.
But, a is an arbitrary real number. Hence, f (x) is continuous on R. Q.E.D.
REMARK 1 It is evident from the graph of a constant function that is everywhere continuous.
THEOREM 5 The identity function is everywhere continuous.
PROOF Let / (x) = x for all x e Rbe the identity function. Let a be any real number. Then,
lim / (x) = lim x = a and, / (a) = a.

w
x -> a x —> a
lim / (x) = / (fl)
x —> a
So, / (x) is continuous at x = a.

Flo
ee
Since a is an arbitrary real number. Hence, / (x) is continuous on R i.e. it is everywhere
continuous.

Fr
REMARK 2 The abovefact can be easily observedfrom the graph of the identityfunction. Q.E.D.
for
ur
THEOREM 6 A polynomial function is everywhere continuous.
PROOF Let / (x) =a0 + a^ x + a2x2 + ... + anyf ,neZ,n> 0, x e R be a polynomial function.
ks

We shall prove the theorem by induction on n.


Yo
oo

STEP 1 Whenn = 0, we have


eB

/(x) = fl0
Clearly, / (x) is a constant function which is everywhere continuous
r

When n = 1, we have
ou
ad

/ (*) = *•
Y

Clearly, / (x) is the sum of a constant function and a multiple of the identity function. So, being
the sum of two everywhere continuous functions, / (x) is everywhere continuous.
nd
Re

STEP II Let every polynomial function of degree at most n be everywhere continuous.


Fi

Consider a general polynomial function of degree (n + 1).


g(x) = a0 + a-L x + a2 x2 +... + an x" + a,, + ! x" + 1 , where + 1 ^ 0.

=> g{x) = + x+ a2x + ... + an xn 1 + fl,I+ x x")


Clearly, it is the sum of a constant function (which is everywhere continuous) and the product
of the identity function x (which is everywhere continuous) and the polynomial function
a-! + fl2 * + ••• + + i degree at most n (which is everywhere continuous by induction
assumption). Therefore, g (x) is everywhere continuous.
Hence, by the principle of mathematical induction, a polynomial function is everywhere
continuous.
A simple consequence of the above theorem is the following: Q.E.D.
COROLLARY Every rational function is continuous at every point in its domain.
PROOF Let/ (x) = g(x) h(x) * 0 be a rational function. Then, g (x) and h (x) are polynomial
h(x)'
functions.

ReadYourFlow.COM
CONTINUITY 9.27

The domain of / (x) is the set D = .R - {x: g (x) = 0}.


Since polynomial functions are everywhere continuous. Therefore, g (x) and h (x) are
continuous on D.

Hence, by theorem 1,/ (x) = g(x) is continuous on D.


h(x)
THEOREM 7 The modulus function is everywhere continuous.
PROOF We know that the identify function is everywhere continuous.
Also, if / is continuous, then | /1 is also continuous. Therefore, | x | is everywhere continuous.
Q.E.D.
THEOREM 8 The exponential function ax, a>0 is everywhere continuous.
PROOF Letf(x) = flv.Then,
ax-l
lim ax = lim < x +1 ■ = lim x lim x + lim 1 = (loge a x 0) + 1 = 1.

w
x —> 0 x —> 0 x x-» 0 x x-> 0 x -> 0
Let c be an arbitrary real number. Then,

h^O h->0
Flo
lim / (x) = lim f (c -h) = lim ac h - ac lim a h = ac lim ^T = ac x ^ = «c=/(c)
h->0 h->0 ah

ee
x —> c
+ h = ac

Fr
and. lim / (x) = lim / (c + h) = lim aL lim a1' = x 1 - ac = f (c)
c+ h->0 h -*Q /!-»0
for
ur
lim / (x) = lim / (x) = / (c)
X -> c X -» c+
ks

So, / (x) is continuous at x = c.


Yo
oo

Since c is an arbitrary real number. Hence, / (x) = ax is everywhere continuous. Q.E.D.


B

COROLLARY cx is everywhere continuous.


re

THEOREM 9 The logarithmic function is continuous in its domain.


ou
ad

PROOF Let/ (x) = logc x, where c > Obe the logarithmic function. Clearly, domain (/) = (0, co).
Y

Let fl be an arbitrary point in (0, co). Then,


lim / (x) = lim f (a + h)
nd
Re
Fi

=> lim / (x) = lim logc (a + h)


x ~>a+ /? —>0

=> lim / (x) = lim logc all + -


■ a+ h->0 V aj

=>
f
lim f (x) - lim •! logc a +logc 1 + —
f h
x->a+ l'~>0 { V a

=> lim / (x) = logc a + lim logc [1 + — )


x^a+ V a)

log,f1 + -
a h
=> lim / (x) = log, a + lim • r x —
x->a+ h-*0 h a
a

ReadYourFlow.COM
9.28 MATHEMATICS-XII

logc I 1 + -
a h
=> lim f (x) = logc a + lim x lim
x->a+ h —^ 0 h /! —> 0 a
a
=> lim / (x) = logc a + loge c x 0 = logc a = f (a)
x —> a+
Similarly, we have
lim / (x) = / (a)
* -»a

lim f(x)=f(a)= lim f (x)


x —> a h -> a+

So, f (x) is continuous at x = a.


Since a is an arbitrary point in (0, co). Hence, / (x) is continuous on (0 , oo).

w
Q.E.D.
THEOREM 10 The sine function is everywhere continuous.

Flo
PROOF Let / (x) = sin x and let a be an arbitrary real number. Then,
lim / (x) = lim f (a + h)

ee
x->a+ h->0

Fr
=> lim / (x) = lim sin (a + h)
x -» a+ for
ur
=> lim / (x) = lim {sin a cos h + cos a sin h]
&->0
ks
Yo

=> lim / (x) = sin a lim cos h + cos a lim sin h


oo

x -> rt+ Kh->0 /; —> 0


B

=> lim /(x) = sin x 1 + cos ax 0 lim sin /? = 0 and lim cos h = 1
re

a+ /i->0 /i -» 0
ou
ad

=> lim / (x) = sin a = f (a)


a+
Y

Similarly, we have
nd
Re

lim / (x) = / (a)


x a
Fi

lim /(x) = f (a) = lim /(x)


x-+a fl+

=> / (x) is continuous at x = a.


Since a is an arbitrary real number. Hence,/ (x) = sin x is everywhere continuous. Q.E.D.
THEOREM 11 The cosine function is everywhere continuous.
PROOF Let / (x) = cos x and let a be any real number. Then,
lim / (x) = lim f (a + h)
x->a+ h^O

=> lim / (x) = lim cos (a + h)


/i -»0

=> lim / (x) = lim {cos a cos h - sin a sin h\


x -* a+ h->0

ReadYourFlow.COM
1
CONTINUITY 9.29

=> lim / (x) = cos a lim cos h - sin a lim sin h


x->a+ h^>0 [h^O

lim / (x) = (cos a) x 1 - sin fl x 0 lim cos h =1 and lim sin h = 0


/i->0 /i->0

=> lim / ( j) = cos a = / {a)


x -»1?+

Similarly, we have
lim f{x) = f{a)
X ->(1

lim / (x) = lim / (x) = / {a)


x —> ti a+

So, / (x) is continuous at x =

w
Since a is an arbitrary real number. Hence, / (x) is everywhere continuous. Q.E.D.
THEOREM 12 The tangent function is continuous in its domain.

Flo
PROOF Let / (x) = tan x. Clearly, domain (/) -R - j (2n + 1) ^: n e Z j

ee
siir x

Fr
We have, f(x) = tan x =
cos x
for
Since sin x and cos x are everywhere continuous. Therefore, / (x) = tan x is continuous for all
ur
x e R except when cos x * 0. But, cos x = 0 at x = (2n + 1) n/2, n e Z.
Hence, / (x) = tan x is continuous for all x eR - {(2« + 1) n/2:n e Z}. Q.E.D.
ks
Yo

THEOREM 13 (i) The cosecant function is continuous in its domain.


oo

(ii) The secant function is continuous in its domain.


eB

(iii) The cotangent function is continuous in its domain.


PROOF It is the direct consequence of the above Theorems and Theorem 1.
r

THEOREM 14 / (x) = sin- 1 x is continuous on [-1,1].


ou
ad

PROOF Let a be an arbitrary point in [-1, -1].


Y

Let y - sin x. Then, x = sin y


nd
Re

x —» => sin y —m? => y —» sin 1 a.


Fi

Thus, lim /(x) = lim sin_1x lim y = sin 1 a = f (a)


x —> a x -> a y -» sin- ^ a
So, f (x) is continuous at x = a.
Since ^is an arbitrary point of [-1,1]. Hence,/(x) = sin-1 xis continuous on[-1,1]. Q.E.D.
REMARK Proceeding as above, it can be shown that all inverse trigonometric functions are continuous
in their respective domains.
ILLUSTRATIVE EXAMPLES

LEVEL-1

Ti/pe I ON TESTING THE CONTINUITY OF A FUNCTION IN ITS DOMAIN


’ l x-41 x*4
EXAMPLE l If a function f is defined as f(x) = • x-4 '
0 , x =4
Show that f is everywhere continuous except at x = 4.

ReadYourFlow.COM
9.30 MATHEMATICS-XII

SOLUTION We have.
x -4\ x*4
fix) = x-4 '
0 , x=4
-(x-4)
= -l ; x <4
x-4
x-4 -(x-4), x <4
=> /(*) = =1 ; x>4 V | x-4| =
x-4 x-4, x>4
0 ; x=4

When j < 4, we have f(x) = -1, which, being a constant function, is continuous at each point
x <4.

low
Also, when x > 4, we have/(x) =1, which, being a constant function, is continuous at each point
x> 4.
Let us consider the point x = 4.

ee
We have.
rF
Fr
lim f(x) = lim -1 = -1, lim f(x) = lim 1=1 and, /(4) = 0.
x —>4 x -> 4 x 4+ x ->4 for
lim f(x) * lim f(x)
x —> 4 +
ou
x —>4
ks

So, f(x) is not continuous at x = 4.


oo

Hence, f(x) is everywhere continuous, except at x = 4.


Y
B

sin 2x , if x < 0
re

EXAMPLE 2 Discuss the continuity of the function f(x) = x


x+2 , if x > 0
ou
ad

SOLUTION When x < 0, we have


Y

sin 2x
f(x) =
nd
Re

x
Fi

We know that sin 2x as well as the identity function x both are everywhere continuous. So, the
quotient function-------- = f(x) is continuous at each x < 0.
x
When, x > 0, we have
/(x) = x + 2,which being a polynomial function, is continuous at each x > 0.
Let us now consider the point x = 0.
We have.
sin 2x sin 2x
lim /(x) = lim = 2 lim = 2 (1) = 2, lim /(x) = lim x + 2 = 2
x -> 0“ x -» 0 X x —> 0 2x 0+ x 0

and. /(0) =0 + 2 = 2
lim /(x) = lim /(x) = /(0).
x —> 0 x -> 0 +

So, /(x) is also continuous at x = 0. Hence,/(x) is everywhere continuous.

ReadYourFlow.COM
CONTINUITY 9.31

, if x*0
EXAMPLE 3 Discuss the continuity of thefunction f{x) = x
0 , if ^ = 0

SOLUTION We have.
-x
, if x < 0
^1 if x > 0
f(x)=\ 1—1
x
, if x * 0
=> f(x) = • -=1
X
, if x > 0
- x, if x < 0
0 , ifx = 0
0 , if X = 0

We observe that/(x) is a constant function for all x < 0 as well as for x > 0. So, it is continuous for
all x > 0 and for all x < 0.

w
Consider the point x = 0. At x = 0, we have

(LHL at x = 0) =

Flo
lim /(x) = lim -1 = -1 and, (RHL at x = 0) = lim /(x) = lim 1=1
x->0+ x->°

ee
(LHL at x = 0) * (RHL at x = 0)

Fr
So, /(x) is not continuous at x = 0. Hence, /(x) is continuous at each point, except at x = 0.
for
2 x -1 , if x <0
ur
EXAMPLE 4 Discuss the continuity of the function f(x) given by /(x) =
2x +1 , if x > 0
ks

[CBSE 2002]
Yo
oo

SOLUTION When x < 0, we have /(x) = 2x -1.


eB

Clearly, /(x) is a polynomial function for x < 0. So, /(x) is continuous for all x < 0.
When x > 0, we have /(x) = 2x + 1.
r
ou
ad

Clearly, /(x) is a polynomial function for x > 0. So, it is continuous for all x > 0.
Y

Let us now consider the point x = 0. At x = 0, we have


lim /(x) = lim (2x-l) = -1 and, lim /(x) = lim (2x +1) = 1.
nd
Re

x->0 x -» 0 x -» 0+ x 0
Fi

lim /(x) * lim /(x)


x ->■ 0~
r,~ x-»0+

So,/(x) is not continuous at x = 0. Hence,/(x) is everywhere continuous except at x = 0.


EXAMPLE 5 Show that the function f defined by /(x) = 11 - x +1 x | | is everywhere continuous.
[NCERT]
SOLUTION Let g(x) = 1 - x + | x | and h(x) = | x | be two functions defined on R. Then,
(hog)(x) = h(g{x)) = h(l-x + \x\) = |1 -x + | x 11 = /(x) for aUxeR.
Since (1 - x), being a polynomial function and | x | being a modulus function are continuous on R.
Therefore, g{x) = 1 - x +1 x | is everywhere continuous.
Also, h(x) = | x | is everywhere continuous. Hence, / = hog is everywhere continuous.
ALITER We have.
11 — x — x|, if x<0
/« = |i-*+M|={ j 1 —x + x| , if x>0

ReadYourFlow.COM
9.36 MATHEMATICS-XII

2 , ifx<3
(iv) f(x) = - ax+ b , if 3 < x < 5
9 , if x > 5
4 , if x < -1
(v) /(x) = < ax2 +b , if-l<x<0
COS X , if x > 0

v'l + px - yi -px
, if -1 < x < 0
X
(vi) f(x)=\ [CBSE 2013, NCERT EXEMPLAR]
2x +1
, if 0 < x < 1
x-2
5 , if x < 2
(vii) /(x) = \ax + b , if 2 < x <10 [NCERT]
21 , if x > 10

w
k cos x 71
X <—
k-2x 2
(viii) /(*) = 3

Flo
x =—
2
Tt
[CBSE 2010]

ee
3 tan 2x
rr
x >—

Fr
2x - n 2
x2 , if 0 < x < 1
for
ur
a
5. The function /(x) = • a , if 1 <x < V2
ks

2b1 - 4b
Yo

, if V2 < x < oo
oo

x2
B

is continuous on [0, oo). Find the most suitable values of a and b.


re

6. Find the values of a and b so that the function / (x) defined by


ou

x + flV2 sin x , if 0 < x < tx/4


ad

/ (x) = • 2x cot x + fr , if n/4<x < k/2


Y

a cos 2x —b sin x , if it/2<x<k


nd
Re

becomes continuous on [0, n].


Fi

x2 + ax +b , 0 < x < 2
7. The function / (x) is defined by / (x) = 3x + 2 , 2 <x <4
2ax + 5b , 4 <x <8
If / is continuous on [0, 8], find the values of a and b.
tan f— -x
8. If / (x) = 4
for x * find the value which can be assigned to / (x) at x = 7r/4 so
cot 2x
that the function/ (x) becomes continuous every where in [0, tt/2].
2x -1 , if x < 2
9. Discuss the continuity of the function /(x) = 3x
, if x > 2 '
2
10. Discuss the continuity of f(x) = sin | x |. [NCERT]

ReadYourFlow.COM
CONTINUITY 9.37

sin x
, x<0
11. Prove that f(x) = x is everywhere continuous.
x+l , x>0
12. Show that the function g (x) = x - [x] is discontinuous at all integral points. Here [x] denotes
the greatest integer function. [NCERT]
13. Discuss the continuity of the following functions:
(i) / (x) = sin x + cos x (ii) / (x) = sin x - cos x (iii) / (x) = sin x cos x [NCERT]
14. Show that / (x) = cos x2 is a continuous function. [NCERT]
15. Show that/ (x) = | cos x | is a continuous function. [NCERT]
16. Find all the points of discontinuity of / defined by / (x) = | x | -1 x + 11. [NCERT]
1
x"2 sin — , x * 0 [NCERT]
17. Is / (x) = • x a continuous function?
/ x=0

w
0
1
18. Given the function/(x) = . Find the points of discontinuity of the function/(/(x)).
x+2

Flo [NCERT EXEMPLAR]

ree
19. Find all point of discontinuity of the function /(f) = 2 ---- , where f = —
x —1

F
[NCERT EXEMPLAR!
or
ur
________ ANSWERS
f
2. Discontinuous at x = 0
ks

3. (i) x = 1 (ii) x = 2 (iii) x=0 (iv) x = 0 (v) x = 0


Yo
oo

(vi) x = 0 (vii) x = 0 (viii) Nowhere discontinuous


(x) Discontinuous at x = 1
B

(ix) Discontinuous at x = 3
(xi) Discontinuous at x = 1 (xii) Everywhere continuous
re

(xiii) Everywhere continuous (ii) k = -2


ou

4- 0)
ad

15
Y

(iii) No value of k can make/ (iv) a =7/2,b = -17/2


nd

(v) a = 3,b=l (vi) p = -1/2


Re

(vii) a = 2, b = 1 5. a =-l,b -1 or a - l,b =1 ± 42


Fi

6. a = k/6, b - - k/12 7. a = 3,b = —2


8- f(i\ - \
9. Everywhere continuous. 10. Everywhere continuous
13. (i) Everywhere continuous (ii) Everywhere continuous
(iii) Everywhere continuous 16. No. point of discontinuity
17. Continuous 18. Discontinuous at x = -2 and x = -5/2
19. Discontinuous x = 1 / 2,1, 2.
HINTS TO NCERT & SELECTED PROBLEM
3. (x) We have.
X10 -1 , X 5a 1
/(*) =
x2 , x >1
Clearly, / (x) is a polynomial function for all x < 1 as well as for all x > 1. So, / (x) is
everywhere continuous except possibly at x = 1.

ReadYourFlow.COM
9.38 MATHEMATICS-XII

Now, lim f (x) = lim ^r10-1=1-1=0 and, lim f (x) = lim x2 = l2 = 1


x->l_ x->r X->1 4-
X->] +

Clearly, lim / (x) ^ lim / (x). So, / (x) is not continuous at x = 1.


x -> l ~ x -> 1+
Hence, / (x) is everywhere continuous except at x = 1.

(xi) We have.
2x , x < 0
f(x) = 0 , 0<x<l
4x , x >1
At x = 0, we have

w
lim / (x) = lim 2x = 2 x 0 = 0, lim / (x) = lim 0=0 and, / (0) = 0
x -»0- x —> 0 x —> 0+ 0+
Thus, lim /(x) = lim /(x)=/(0).
x -> cr x-»0+

Flo
ee
So, / (x) is continuous at x = 0.

Fr
At x = 1, we have
lim / (x) = lim 0 = 0, lint / (x) = lim 4x = 4
x->r x->r i+ x-»l+
for
ur
lim / (x) * lim / (x)
x-^r x—>.1+
ks
Yo
oo

So, / (x) is not continuous at x = 1.


B

For x < 0, / (x) is a polynomial function which is everywhere continuous. For x e [0,1], / (x)
is a constant function which is also continuous. For x >l,/(x) is a polynomial function
re

which is everywhere continuous.


ou
ad

Hence, / (x) is everywhere continuous except at x = 1.


Y

(xii) We have.
sin x - cos x , x * 0
nd

/(*) =
Re

-1 , x = 0
Fi

Clearly, / (x) is continuous for all x ^ 0.


Now, lim / (x) = lim (sin x-cos x) = sin 0-cos 0 = -1 = f (0)
x -> 0 x -* 0

So, / (x) is continuous at x = 0. Hence, / (x) is everywhere continuous,


(xiii) We have.
-2,x<—1
/ (x) = < 2x , -1 < x < 1
2 , x>l
As / (x) is a constant function for all x < -1 and all x > 1. So, / (x) is continuos for all x < -1
and all x >1. For x e(-l, 1),/ (x) is a polynomial function which is always continuous.
Thus, / (x) is continuous for all x except possible at x = -1,1.
Continuity at x = -V. We observe that
lim /(x) = lim -2 = - 2, lim / (x) = 2x = -2 and, /(-l)=-2
x->-r x->-r x-».-l+

ReadYourFlow.COM
CONTINUITY 9.39

lim f(x) = lim /(*) =/(-!)


x^-r 1+

So,/ (x) is continuous at x = -l.


Continuity at x =1: Clearly,
lim/ (x) = lim 2 = 2, lim / (x) = lim 2x = 2 x 1 = 2 and, / (1) =2
1+ x —> 1+ .r -> 1“

lim f(x) = lim / (x) = f (1)


x->r x^>i +
So, / (x) is continuous at x =1. Hence, / (x) is everywhere continuous.
5 , x<2
4. (vii) It is given that / (x) =< ax + b , 2 < x < 10 is everywhere continuous.
21 , x > 10

w
So, it is continuous at x = 2 and x = 10.
lim / (x) = lim /(x)=/(2) and. lim / (x) = lim / (x) = / (10)
x -*2~ x ->2+

Flo x -»10 x -> 10+

ee
=4> lim 5 = lim ax + b - 5 and. lim ax + b = lim 21 = 21

Fr
x -» 2+ x 10~ x 10 +

=> 5 = 2a + b and 10fl + b = 21 for


ur
=> a = 2, b = 1
10. Let a be any real number. Then,
ks

lim / (x) = lim sin | x | = lim sin | fl - /i | = sin | a \


Yo
oo

x -> x —> a~
B

lim / (x) = lim sin | x | = lim sin | a + ft | = sin | a | and, / (a) = sin | a |
re

«+ a+ /i -> 0
ou

lim / (x) =lim / (x) = / (a)


ad

x -» a x -> a+
Y

So, / (x) is continuous at x = fl.


nd

Since a is an arbitrary real number. Hence, / (x) is everywhere continuous.


Re

12. Let a be any integer. Then,


Fi

lim y(x) = lim =g(a-h)= lim (a-h)-[a-h]= lim (a-h)-(a-1) = a-{a-l) =1


x -» fl+ h ->0 /i -> 0 h^0

lim y (x) = lim ?(a + h) = lim (a + h) - [a + h]= lim (a + h)-a=a-a = 0


x->a+ h-+0 /j ->0 h->0

lim y(x) * lim y(x).


x -> a x —» a+
So, y (x) is discontinuous at x = a.
Since a is an arbitrary integer. Hence, y (x) is discontinuous at all integral points.
13. We know that sin x and cos x are everywhere continuous. Therefore, sin x + cos x,
sin x - cos x and sin x cos x are everywhere continuous.
14. Let / (x) = cos x2 and abe any real number. Then,
7 2
lim f (x) = lim f (a-h) = lim cos (a - h) = cos a
x -> a h-+0 h —> 0

ReadYourFlow.COM
9.40 MATHEMATICS-XII

lim / (x) = lim f (a + h) = lim cos (a + h)2 = cos a2 and, f (a) = cos a 2
x^a+ h->0 h-+0

lim f (x) = lim f(x)=f(a)


x->a+

So, / (x) is continuous at x = a.


Since 'a' is an arbitrary real number. Hence, / (x) is everywhere continuous.
15. Let / (x) =| cos x j and a be any real number. Then,
lim / (x) = lim / (fl - h) = lim | cos (a -h)\ = cos a
-T —> (J /»-> 0 /z -> 0

lim / (x) = lim f (a+ h) = lim cos (a + h)\ = | cos fl | and, / (a) = cos a
x —> a+ h —*0 h ->0

low
lim / (x) = lim / (x) = f (a)
x -> n x ->a+
So, / (x) is continuous at x = a.

ee
Since 'a' is an arbitrary real number. Therefore, / (x) is everywhere continuous.
rF
Fr
16. We have.
- x + x + 1 , x < -1 for
/(x) = | X| — | X + 11 = ■ -x-(x + l) , -1 <x <0
u
x - (x + 1) , x > 0
ks

1 X <—1
Yo
oo

=> / (x) = -| - 2x -1 , -1 < x < 0


-1 x>0
B
re

Clearly, / (x) is continuous for all x satisfying x < -1, -1 <x<0 and x > 0. So, possibly
points of discontinuity are x = -1 and x = 0.
ou
ad

Continuity at x=-1: Clearly,


Y

lim /(x) = lim 1=1, lim f(x)= lim (-2x-1) =-2 x-1-1 =1
nd

*->- r +
Re

y->- r -Y —> — 1 .Y —» -1+


Fi

and, /(-l) = -2x -1 -1 =1

lim /(x) = lim /(x) = /(-l).


,Y->-r 1+

So, / (x) is continuous at x = 1.


Continuity at x = 0: Clearly,
lim /(x) = lim (-2x -1) = -2 x 0-1 = -1,
x —> 0 x -> 0

lim /(x) = lim -1 = -1 and, /(0) =-2x0-1 = -1


X 0+ x->0 +

lim / (x) = lim / (x) = / (0)


.Y —> 0 x —> 0 +

So, / (x) is continuous at x = 0. Hence, / (x) is everywhere continuous.

ReadYourFlow.COM
CONTINUITY 9.41

17. We have.
x2 sin - , x*0
/(*) = x
0 , x=0
Clearly, / (x) is continuous for all x * 0.
Now,
- 1
lim / (x) = lim x sin — = 0 x (An oscillating number between -1 and 1) = 0 =/ (0)
x ->■ 0 * -> 0 x
So, / (x) is continuous at x = 0. Hence, / (x) is everywhere continuous.
1
18. Clearly, /(x) = is discontinuous at x = -2. Also, it is not defined at x = -2.
x+ 2
For x * - 2, we have
1 x+2

w
/(*)=/ 1
-—— + 2 2x +5
x+2

Flo
We observe that/(/(x)) is discontinuous and undefined at x = -5/2.
Hence, /(/(x)} is discontinuous at x = - 2 and x = -5/2.

ee
19. We have.

Fr
/(x) = “2 ----- and f = .
t2 +t-2 x-1 for
ur
Clearly, t = - - is discontinuous and undefined at x = 1.
x-1
ks
Yo

For x * 1, we have
oo

1
f(t) t1 + t-2 (f+2)(i-l)
B
re

This is discontinuous at f = 2 and f = 1.


1
ou

For t = -2, t= — => x = —


ad

x-1 2
Y

1
For f =1, f = =s> x = 2
x-1
d
Re
n

Hence, / is discontinuous atx=l/2,x=l and x = 2.


Fi

VERY SHORT ANSWER QUESTIONS (VSAQs)


Answer each of thefollowing questions in one word or one sentence or as per exact requirement of the
question:
1. Define continuity of a function at a point.
2. What happens to a function / (x) at x = a, if lim / (x) =f (a) ?
x -> a
x
3. Find / (0), so that / (x) = becomes continuous at x = 0.
i-

— ,x*0
4. If / (x) sin 3x is continuous at x = 0, then write the value of k.
k ,x=0
sin 10 x
5. If the function / (x) = , x * 0 is continuous at x = 0, find / (0) .
x

ReadYourFlow.COM
9.42 MATHEMATICS-XII

x1 -16
, if x ^ 4
6. If/(x) = x -4 is continuous at x = 4, find k.
k , if x = 4
sin x2
7. Determine whether/(x) =* “ , x ^ 0 is continuous at x = 0 or not.
0 , x=0
1 - cos x
, x*0
8. If/(x) = x2 is continuous at x = 0, find k.
k , x=0
sin ^ x
9. If/(x) = , x * 0 is continuous at x = 0, write the value of k.
x
k , x=0
5x -4 0<x<l

low
10. Write the value of b for which / (x) = is continuous at x = 1.
4x2 + 3bx 1 < x < 2
kx
■—-, x < 0
11. Determine the value of constant 'k' so that the function / (x) = -M x is continuous at

ee
3 ,x>0
rF
Fr
x = 0. [CBSE 2017]
x2 + 3x-10
,x*2
12. Find the value of k for which the function / (x) = •
for x-2 is continuous at x = 2.
k , x2
u
ks

[CBSE 2017]
Yo
oo

_ ANSWERS
B

2. /(x) becomes continuous atx=a 3. 2 4. I 5. 10 6. 8


3
re

1
7. continuous 8. 9. 1 10. -1 11. -3 12. 7
ou

2
ad
Y

MULTIPLE CHOICE QUESTIONS (MCQs)


4 -x2
1. The function/(x) =
nd
Re

4x -x3
Fi

(a) discontinuous at only one point (b) discontinuous exactly at two points
(c) discontinuous exactly at three points (d) none of these
2. If / (x) =| x - a | (|) (x), where <j) (x) is continuous function, then
(a)/'(fl+) = 4>(fl) (b) f(a~) =-tf>(a) (c) /' (a+) =/' (O (d) none of these
3. If/(x) = |logio x|, then at x =1
(a) / (x) is continuous and /' (1 +) = log10 e
(b) / (x) is continuous and /' (1 +) = log10 e
(c) / (x) is continuous and /' (1~) = log^Q ^
(d) / (x) is continuous and /' (1 ) = - log10 e
36* - 9* - 4* + 1
, x* 0
4. If/(x) = V2-VT + cos x is continuous at x = 0, then k equals
k , x=0
(a) 16V2 log 2 log 3 (b) 16Jl In 6 (c) 16^2 In 2 In 3 (d) none of these

ReadYourFlow.COM
CONTINUITY 9.43

, x *0,1

5. Iff (x) defined by / (x) = ,x=0 then / (.r) is continuous for all
,x=l

(a) x (b) x except at a: = 0


(c) x except at x = 1 (d) x except at * = 0 and a: = 1.
1 - sin x log sin x 71
, X ^ -
(TC - 2x)2 (log (1 + K2 - 47TX + 4x2) 2 is continuous at x = —, then k =
6. If/(x) = •
n 2'
k / x=~
2

low
1 1 1
(a) -T7 (b) - — (0-- (d)
16 32 64 28
7. If/(x)=(x + l) cot X be continuous at x = 0, then / (0) is equal to
(a) 0 (b) l/e (c) e (d) none of these

ee
log (1 + ax) - log (1 - bx)
rF
Fr
, x^O
8. If/(x) = x and / (x) is continuous at x = 0, then the value
k , x=0 for
of A: is
u
(a) a- b (b) a+ b (c) log a + log b (d) none of these
ks
Yo

~ e1/x-l
oo

, X 0
9. The function / (x) = • gl/ x +
B

0 , x=0
re

(a) is continuous at x = 0 (b) is not continuous at x = 0


ou
ad

(c) is not continuous at x = 0, but can be made continuous at x = 0


Y

(d) none of these


x -4
nd

x <4
Re

+ a,
Fi

10. Let / (x) = • x = 4 . Then, / (x) is continuous at x = 4 when


x -4
+ b,
I x -41 x>4

(a) a=0,b = 0 (b) a =l,b =1 (c) a=-l,b=l (d) a=l,b=--[


(cos x) 1/x , x* 0
11. If the function / (x) = ■{ v is continuous at x = 0, then the value of k is
k , x=0

(a) 0 (b) 1 (c)-l (d) e


12. Let/(x) =| x| +1 x-11, then
(a) / (x) is continuous at x = 0, as well as at x = 1
(b) / (x) is continuous at x = 0, but not at x = 1
(c) / (x) is continuous at x = 1, but not at x = 0
(d) none of these

ReadYourFlow.COM
9.44 MATHEMATICS-XII

xA - 5 x2 + 4
| (x -1) (x - 2) | ' x *1,2
13. Let / (x) = • 6 , x = 1 . Then,/(x) is continuous on the set
12 x =2

(a) R (b) R -{1} (c) R -{21 (d) *-{1,2}

sin (a + 1) x + sin x
, x <0
x
14. If/(x) = c , x=0 is continuous at x = 0, then
^x + bx2 - Vx
, x>0
bx Vx

low
3 / n 1 3 , , 1
(a) a = - (b) a = - — ,b =1, c = —
2 2 2 2
(c) fl =- —, b g R — {0}, c = — (d) none of these
2 2

ee
K
mx + 1 / x<—
rF
Fr
9 71
15. If / (x) = • is continuous at x = —, then
K 2'
sin x + n , x>— for
2
u
mit
ks

(a) w =l, n = 0 (b) m = — + l (c)n = — (d) m - n = —


Yo

2 2
oo
B

2 -^a2 + ax + x2
- flX + X
re

16. The value of /(0), so that the function f (x) = becomes


-Ja + x - / a - x
ou
ad

continuous for all x, given by


(a) a 3/2 1/2
Y

(b) a (c) -fl1/2 (d) -fl3/2

1 , | x| > 1
nd
Re

1 1 i i 1
17. The function/(x) = • 2 ' -<M <— ,n=2,3,...
Fi

n n n-l
0 / x=0
(a) is discontinuous at finitely many points
(b) is continuous everywhere ^
(c) is discontinuous only at x = ± -, n e Z - {0} and x = 0
n
(d) none of these
(27 — 2 x) 1/3 . 3
18. The value of / (0), so that the function / (x) = jj^r (x * 0) is continuous, is
9- 3 (243 + 5 x)
given bv
(a)!
(b) 6 (c) 2 (d) 4

2-(256-7 x) 1/8
19. The value of /(0) so that the function / (x) = , x ^ 0 is continuous
(5 x + 32)1/5 -2
everywhere, is given by

ReadYourFlow.COM
CONTINUITY 9.45

(a) -1 (b) 1 (c) 26 (d) none of these


vrr+ px - yri - px , -1<x < 0
20. /(*)=• is continuous in the interval [- 1, 1], then p is
2 .t +1
0 < .r < 1
x-2
equal to
(a) -1 (b) -1/2 (c) 1/2 (d) 1

x2 a , 0 <x <1
21. The function / (x) = a , i <x <s[2 is continuous for 0 < x <oo, then the most
2b2 -4b
/ <x <cc
x2

w
suitable values of a and b are
(a) a = l,b =-1 (b) « = -!,&=! + V2
(c) a = -l,b=l (d) none of these

Flo
22. If / (x) = 1 sin x^ ^ vvhen x ^ — and/ f - ] = A., then / (x) will be continuous function at

ree
2 2 ' V2/
(n-2 x)

F
x = 71/2, where A. =
(a) 1/8 (b) 1/4 (c) 1/2 (d) none of these
or
ur
(4*-l)3
f
/ x*0
may be
ks

23. The value of a for which the function /(x) =• sin (xa) log {(1 + x23)}
Yo

12 (log 4)3 , x = 0
oo
B

continuous at x = 0 is
(d) none of these
re

(a) 1 (b) 2 (c) 3


24. The function / (x) = tan x is discontinuous on the set
ou
ad

(a) {n K-.neZ] (b) {2n n:n eZ}


(c) |(2n + 1) ^ :n e Z j (d) |?y :nezj
Y

sin 3 x
nd

, x*0
Re

25. The function / (x) = x is continuous at x = 0, then k =


Fi

k
, x =0
2
(a) 3 (b) 6 (c) 9 (d) 12
2x - sin 1* x
26. If the function / (x) = -— is continuous at each point of its domain, then the
2x + tan 1 x
value of / (0) is
1 <d>!
(a) 2 (b)-
5x - 4 , 0 <X <1
27. The value of b for which the function / (x) = is continuous at
4x2 + 3bx , 1 < x < 2
every point of its domain, is
(a) -1 (b) 0 (c) 13/3 (d) 1
28. If/(x) = -— , then the set of points discontinuity of the function / (/ (/ (x))) is
1 -x
(a) dl (b) (0,1} (c) (-1,1} (d) none of these

ReadYourFlow.COM
9.46 MATHEMATICS-XII

rt
tan —x
4
29. Let / (.t) = , x * The value which should be assigned to/ (x) at x = ^, so that
cot 2x
it is continuous everywhere is
(a) 1 (b) 1/2 (c) 2 (d) none of these
x3 + a-2 -16x + 20
30. The function / (x) = is not defined for .r = 2. In order to make / (x)
t-2
continuous at x = 2, / (2) should be defined as
(a) 0 (b) 1 (c) 2 (d) 3
a sin ^ (x + 1), x <0
31. If f(x) = - is continuous at x = 0, then a equals
tan x - sin x
x >0
x3

w
1 1 1 1
(a) 2
(b)3 (C)I (d) 6

Flo
ax2 + b , 0 < x < 1
32. If fix) = 4 , x =1 , then the value of {a,b) for which /(x) cannot be

ee
x+ 3 , 1 <x <2

Fr
continuous at x = 1, is
(a) (2, 2) (b) (3,1) (c) (4, 0) for (d) (5, 2)
ur
log (1 + 3x) - log (1 - 2x)
, x^0
33. If the function / (x) defined by / (x) = • x is continuous at
ks

k , x=0
Yo
oo

x = 0, then k =
(a) 1 (b) 5 (c) -1 (d) none of these
B
re

1 - cos 10 x
, x<0
x2
ou
ad

34. If/(x) = • a , x = 0 , then the value of a so that / (x) may be continuous at


Y

, x>0
^625 + Vx - 25
nd
Re

x = 0, is
Fi

(a) 25 (b) 50 (c) -25 (d) none of these


1
35. If / (x) = x sin , x * 0, then the value of the function at x = 0, so that the function is
x
continuous at x = 0, is
(a) 0 (b) -1 (c) 1 (d) indeterminate
sin — , x * 0
36. The value of k which makes / (x) = • x continuous at x = 0, is
k , x =0
(a) 8 (b) 1 (c) -1 (d) none of these
37. The values of the constants a, b and c for which the function
(1 + ax) l/.r , x<0
b
/(•?) =
, x = 0 may be continuous at x = 0, are
(x + c) 1/3 -1
, x>0
1/2 -1
(x + 1)

ReadYourFlow.COM
CONTINUITY 9.47

(a) a

(c) a
=m) 'b=
= los'(!)' b = -,c = l
2
3'C = 1 (b) a =log' (!)

(d) none of these


3

2-Jx , 0<x<l
38. The points of discontinuity of the function f (x) = ■ 4-2x , 1 < x < ^ is (are)

2x -7 , — < x < 4
2
(a) x = 1, x = | (b) x = | (c) x = 1/ T/ 4 (d) x = 0, 4
2'
1 - sin2 x 7C

w
' ^2
3 cos2 x
K ft
39. If/(x) = • a , x = — . Then, / (x) is continuous at x = —, if
& (1 - sin x) 71

Flo
ee
/ X > —
(n-2xf 2

Fr
(a) a = —,b = 2 (b) a = — (c) a = —, b = — (d) none of these
for
ur
3 3' 3 3 3
^ (2x2 + 3), x<l
ks
Yo

40. The points of discontinuity of the function / (x) = j 6 - 5x , 1 < x < 3 is (are)
oo

x-3
, x>3
eB

(a) x = 1 (b) x = 3 (C) x = 1, 3 (d) none of these


r

5x - 4 , if 0 < x < 1
ou
ad

41. The value of a for which the function / (x) = is continuous at


4x2 + 3ax , if 1 <x <2
Y

every point of its domain, is


nd
Re

(a) 13/3 (b) 1 (c) 0 (d) -1


Fi

sin (cos x) - cos x K


, x*-
(n-2x)2 2 .
42. If/(x) = is continuous at x = ^, then k is equal to
K
k / x=—
2
1
(a) 0 (b)- (c)l (d) -1

______ ANSWERS
1. (c) 2. (a), (b) 3. (a), (d) 4. (c) 5. (d) 6. (c) 7. (c) 8. (b) 9. (b)
10. (d) 11. (b) 12. (a) 13. (d) 14. (c) 15. (c) 16. (c) 17. (c) 18. (c)
19. (d) 20. (b) 21. (c) 22. (a) 23. (d) 24. (c) 25. (b) 26. (b) 27. (a)
28. (b) 29. (b) 30. (a) 31. (a) 32. (d) 33. (b) 34. (b) 35. (a) 36. (d)
37. (c) 38. (b) 39. (b) 40. (b) 41. (d) 42. (a)

ReadYourFlow.COM
9.48 MATHEMATICS-XII

SUMMARY
1. A real valued function function f (x) is continuous at a point 'a' in its domain iff
lim f(x) =f (a).
x —> a
i.e. the limit of the function at x = ais equal to the value of the function atx-a.
2. A function / (x) is said to be continuous if it is continuous at every point of its domain.
3. Sum, difference, product and quotient of continuous functions are continuous i.e, if / (x)
and g (x) are continuous functions on their common domain, then / ± g, fg, -,kf (k is a
8
constant) are continuous.
4. Let / and g be real functions such that fog is defined. If g is continuous at x=a and / is

low
continuous at g {a), then fog is continuous atx = a.
5. Following functions are everywhere continuous:
(i) A constant function (ii) The identity function
(iii) A polynomial function (iv) Modulus function

ee
(v) Exponential function (vi) Sine and Cosine functions
rF
Fr
6. Following functions are continuous in their domains:
(i) A logarithmic function for
(ii) A rational function
(iii) Tangent, cotangent, secant and cosecant functions.
u
ks

7. If / is continuous function, then | /1 and y are continuous in their domains.


Yo
oo

8. sin~ 1 x, cos' 1 x, tan- 1 x, cot- 1 x, cosec- 1 x and sec- 1 x are continuous functions on
B
re

their respective domains.


ou
ad
Y
nd
Re
Fi

ReadYourFlow.COM
CHAPTER 10
DIFFERENTIABILITY

10.1 DIFFERENTIABILITY AT A POINT


DEFINITION Let f(x) be a real valuedfunction defined on an open interval (a, b) and let c s {a, b). Then,
/(*) -f(c) exists finitely.
f(x) is said to be differentiable or derivable at x = c, iff lim
x —> c x-c

low
This limit is called the derivative or differential coefficient of the function f(x) at x = c, and is
df(x)^
denoted by /' (c) or, Df (c) or,
v dx S x = c

ee
f(x)~f(c)
rF
Fr
Thus, /'(c) = lim
x->c x -c
Now,
for
f(x) is differentiable at x = c
u
m -m exists finitely
ks

<=> lim
Yo

x-»c x-c
oo

f(x) -f(c) =
B

fix) -f{c)
<=> lim lim
re

x —> c x-c c+ x-c


fjc-h)-f(c) fjc + h) -/(c)
ou
ad

<=> lim = lim


h^O -h h->0 h
Y

f(x)-f(c) f(c-h) -/(c)


nd

lim or, lim is called the left hand derivative of / (x)


Re

x —> c x-c /i->0 -h


Fi

at x = c and is denoted by f'{c~) or, Lf '(c).

fix)-f(c) fjc + h) -/(c)


lim or, lim is called the right hand derivative of/fx)
c+ x-c h-+0 h
at x = c and is denoted by /'(c+) or, Rf '(c).
Thus,
/(is differentiable at x = c o L/ '(c) = R/' (c).
If Lf'jc) * Rf '(c), we say that/(x) is not differentiable at x = c.
MEANING OF DIFFERENTIABILITY AT A POINT As we have seen in the chapter on continuity of a
function that if a function / (x) is continuous at a point x=a (say), then its graph is an unbroken
curve at ja, fja)) and there are no holes and jumps in the graph of the function in the
neighbourhood of point x= a. Now, a natural question arises: What do we mean when we say
that a function /(x) is differentiable at a point x = c? In the following discussion we shall try to
answer this question.

ReadYourFlow.COM
10.2 MATHEMATICS-XII

Consider the function f(x) defined on an open interval (a, b). Let P (c,/(c)) be a point on the
curve y =f (x), and let Q(c -h, f(c - h)), and R(c + h, f(c + h)) be two neighbouring points on
the left and right hand side respectively of point P as shown in Fig. 10.1. Then,

y=/M
R(c + h,f(c+h))

X' Q(c-li/f(c-h)) X
o
7

w
r

Flo Fig. 10.1

ee
f(c-h)-f(c) f(c + h) -f(c)

Fr
Slope of chord PQ = and. Slope of chord PR =
-h h
We know that tangent to a curve at a point P (say) is the limiting position of chord PQ when Q
for
ur
tends to P. Therefore, ash ->0 points Q and R both tend to P from left hand and right hand sides
respectively. Consequently, chords PQ and PR become tangent(s) at point P.
ks
Yo

f(c-h)-f(c)
oo

Thus, lim = lim (Slope of chord PQ)


h-+0 —h h->0
B
re

= lim (Slope of chord PQ)


Q -*P
ou
ad

= Slope of the tangent at point P, which is the limiting position


Y

of the chords drawn on the left hand side of point P. ...(i)


f(c + h) -f(c)
nd
Re

and. lim = lim (Slope of chord PR)


h^O h h —>0
Fi

= lim (Slope of chord PR)


R —> P
= Slope of the tangent at point P, which is the limiting position
of the chords drawn on the right hand side of point P -(ii)
Now,
f(x) is differentiable atx = c.
f(c-h) — f{c) = f(c + h)-f(c)
<=> lim lim
h->0 -h /i->0 h
<=> Slope of the tangent at point P, which is limiting position of the chords drawn on the
left hand side of P is same as the slope of the tangent at point P, which is the limiting
position of the chords drawn on the right hand side of P
<=> There is a unique tangent at point P.
Thus,f(x) is differentiable at point P, iff there exists a unique tangent at point P. In other words, f(x) is
differentiable at a point P iff the curve does not have P as a corner point.

ReadYourFlow.COM
DIFFERENTIABILITY 10.3

Consider the function f(x) = | x |. This function is not differentiable at x = 0, because if we draw
tangent at the origin as the limiting position of the chords on the left hand side of the origin, it is
the line y = -x whereas the tangent at the origin as the limiting position of the chords on the
right hand side of the origin is the line y =x. Mathematically, left hand derivative at the origin is
-1 (slope of the line y = -x) and the right hand derivative at the origin is 1 (slope of the line y = x).
Y

X' O X

w
Flo
r
Fig. 10.2

ee
Fr
Let/(x) be a differentiable function at a point P. Then the curve y =/(x) has a unique tangent at
P. Since tangent at P is the limiting position of the chord PQ when Q —> P. So, if f{x) is
or
ur
differentiable at a point P, then chords exist on both sides of point P. This means that the curve
sf
exists on both sides of P. Consequently f{x) is continuous at P.
k

It follows from the above discussion that, if a function is not differentiable at x = c, then either it
Yo
oo

has (c, /(c)) as a comer point or it is discontinuous at x = c.


B

Also, every differentiable function is continuous as proved below.


re

THEOREM If a function is differentiable at a point, it is necessarily continuous at that point. But, the
converse is not necessarily true.
ou
ad

OR
Y

/(x) is differentiable atx = c=> f(x) is continuous at x = c


nd
Re

/(*) ~/(c)
PROOF Let a function /(x) be differentiable at x = c . Then, lim exists finitely.
x->c x-c
Fi

/(*) -f{c) =
Let lim f'{c)
x -»c x -c
In order to prove that/(x) is continuous at x = c, it is sufficient to show that lim /(x) = /(c).
x-tc
Now,
Y /(x)-/(c)
lim /(x) = lim (x - c) + /(c)
X -> c X —> c x-c

=> lim /(x) = lim /(*) -/(0 (x-cU +/(c)


X —> c x —> c x -c

=> lim /(x) = lim MzM)x lim (x - c) + /(c) =/ '(c) x 0 + /(c) [Using (i)]
.r -> c x ->c x -c x->c
=> lim /(x) = /(c)
X -» c

Hence, /(x) is continuous at x = c. Q.E.D.

ReadYourFlow.COM
10.4 MATHEMATICS-XII

REMARK The converse of the above theorem is not necessarily true i.e., a function may be con tin nous at
a point but may not be differentiable at that point. For example, the function f(x) =| x \ is continuous at
x = 0 but it is not differentiable at x = 0 (See Example 1 below.)
ILLUSTRATIVE EXAMPLES

LEVEL-1

EXAMPLE i Show that f(x) = | x | zs not differentiable at x = 0.


SOLUTION We observe that:

(LHDatx = 0) = lim /(*) -/(Q)


x^0~ x-0

(LHD at x = 0) = lim
f(o-h)-m
/z->0 0-h-0

w
=> (LHD at x = 0) = lim
f(-h)-f(0)
h-+0 -h

=> (LHD at x = 0) = lim


/z -> 0
~h\-\ 0|
-h
= lim

Flo
/z -> 0 - h
lim A =
/z —> 0 — h
lim -1 = -1
/z —> 0

ee
f(x) -/(0)

Fr
and. (RHD at x = 0) = lim
x -> 0+ x-0 for
ur
/(0 + h) ~/(0)
=> (RHD at x = 0) = lim
h->0 h
ks

f(h) -/(0)
Yo

=> (RHDat x = 0) = lim = lim = lim 1=1


oo

/z —> 0 h h^O h h —>0 h /i->0


eB

(LHD at x = 0) * (RHD at x = 0).


So, /(x) is not differentiable at x = 0.
r

x -1, if x <2
ou

EXAMPLE 2 Show that the function f(x) =


ad

is not differentiable at x = 2.
2x - 3, if x>2
Y

SOLUTION We observe that:


f(x) -m
nd
Re

(LHD at x = 2) = lim
x —»2- x-2
Fi

(* “!) ~(4 - 3)
=> (LHD at x = 2) = lim [v /(x) =x-1 for x < 2]
x -> 2 x-2
x -2
(LHD at x = 2) = lim = lim 1=1
x —» 2 X - 2 x -» 2

and. (RHD at x = 2) = lim


fjx) -f(2)
2+ x-2
(2x - 3) -(4 - 3)
=> (RHD at x = 2) = lim [••' fix) = 2x - 3 for x > 2]
x->2 x-2
2x - 4
=> (RHD at x = 2) = lim = lim 2 = 2
x 2 X -2 x->2
(LHD at x = 2) * (RHD at x = 2).
So, f(x) is not differentiable at x = 2.

ReadYourFlow.COM
DIFFERENTIABILITY 10.5

x2 sin f — , ifx^O
EXAMPLE 3 Shozv that the function f(x) = • x is differentiable at x = 0 and
, if ^ = 0
0
/'(0)=0. [NCERT EXEMPLAR!
SOLUTION We observe that:
/(*) -/(Q)
(LHD at x = 0) = lim
x->0 x-0
/(0 - h) -/(0)
=> (LHD at x = 0) = lim
/7->0 0-h-0

=> (LHD at x = 0) = lim


n-h)-m
h-*0 -h

(-h)2 sin -1- -0

low
{-h)
=> (LHD at x = 0) = lim
h~>0 —h

=> (LHD at x = 0) = lim h sin (|

ee
/i->0 KhJ
rF
Fr
=> (LHD at x = 0) = Ox (an oscillating number between - 1 and 1) = 0
/(*) -/(Q)
and, (RHD at x = 0) = lim
for
x -> 0 + x-0
u
/(0 + h) ~/(0)
ks

=> (RHD at x = 0) = lim


/i -> 0 0 + ^-0
Yo
oo

-0
m -m =
B

(RHD at x = 0) = lim lim


re

/j-»0 h h->0 h

(RHD at x = 0) = lim sin ( — I


ou
ad

=>
i —► 0 UJ
Y

=> (RHD at x = 0) = 0 x (an oscillating number between -1 and 1) = 0


nd
Re

(LHD at x = 0) = (RHD at x = 0) = 0.
Fi

So,/(x) is differentiable at x = 0 and / '(0) = 0.


EXAMPLE 4 Show that f{x) = x is differentiable at x=l and find f (1).
SOLUTION We observe that:
(LHD at x = 1) = lim
f(x)-f(D
x->l~ x —1
f(l-h)-f(l)
=> (LHD at x =1) = lim
/i ->0 1-h-l
(\-h)2-l2 -2h + h2
=> (LHD at x = 1) = lim lim = lim (2-h) = 2.
/i->0 -h h->0 -h /; —> 0

(RHD at x = 1) lim
fix) -/(l)
and.
1+ x —1
f(l + h)-f(\)
=> (RHD at x = 1) = lim
/i->0 1 + ^-1

ReadYourFlow.COM
10.6 MATHEMATICS-XII

(l+h)2 -1 2h + h2
=> (RHD at x = 1) = lim lim = lim (2 + h) =2
/i->0 h /i -»0 h h ->0
(LHD at x = 1) = (RHD at x = 1) = 2.
So, f(x) is differentiable at x = 1 and /'(!)= 2.
EXAMPLE 5 Shozv that thefunction f(x) =\ x + 1\+ \ x-l\for all x <= R, is not differentiable atx = -l
andx=l. [CBSE 20151
SOLUTION We have.
- (x +1) - (x -1) = - 2x, if x < -1
/(x) =|x-l|+|x + l| = - X + 1 -(x-1) =2 , if -1 < x < 1
X + 1 + X -1 = 2x ,ifx>l
Differentiability at x = -1: We observe that
(LHD at x = -1) = lim /(*)-/(-!)
x-(-l)

w
-2x -2 -2 (x + 1)
= lim — = lim lim (-2)=-2
x->-r x+1 x->-r x+1 x->-r

(RHD at x = -1) = lim


x-(-l)
Flo
/(x)-f(-i)

ee
-l+

Fr
2-2
= lim = lim — = lim 0=0
_1+ x + 1 X - 1+ x + 1 X 1+ for
ur
(LHD at x = -l) * (RHD at x = -l)
So, /(x) is not differentiable at x = -1.
ks

Differentiability at x = 1: We observe that


Yo

2-2
(LHD at x = 1) = lim /(*)-/(!) .
oo

lim = lim — = lim 0 = 0


x-> r x —1 r x->r X —1 x->r
B

/(a-)-/(D
re

(RHD at x =1) = lim


x -> l+ x —1
ou
ad

2x-2 x—1
= lim = lim 2 lim 2 = 2
Y

1+ x-1 1+ x —1 X -»1+
d

(LHD at x =1) ^ (RHD at x =1)


Re
n

So, /.(x) is not differentiable at x = 1.


Fi

Hence, /(x) is not differentiable at x = -1 and x = 1.


EXAMPLE 6 Discuss the differentiability off (x) = x | x | at x = 0.
SOLUTION We have. [NCERT EXEMPLAR]
x2 , x > 0
/(*) = *l*| = - x2 , x < 0
/(*)-/(Q)
Now, (LHD at x = 0) = lim
x -> 0~ x-0

-x2 -0
(LHD at x = 0) = lim [Using definition of / (x)]
x-0
=> (LHD at x = 0) = lim -x = 0
x -* 0

/(*)-/(Q)
and. (RHD at x = 0) = lim
x -> 0+ x-0

ReadYourFlow.COM
DIFFERENTIABILITY 10.7

x2 -0
=> (RHDatx = 0) = lim [Using definition of / (x)]
* 0 x -0
=> (RHDatx = 0) = lim x = 0.
x -> 0
(LHDatx = 0) = (RHDatx = 0)
So, / (x) is differentiable at x = 0 . . 1
x sm — , when x * 0
EXAMPLE? Show that the function f(x) = • x is continuous but not
, when x = 0
differentiable at x = 0. 0
SOLUTION For the continuity of the function refer Example 2 on page 9.6 of Chapter 9.
Now,
/(*) -/(0)
(LHD at x = 0) = lim
x -» 0 x-0

w
/ (0-h) -/ (0)
lim
0-/7-0

Flo
1
-/isin
-h
= lim = lim

ee
/i->0 -h h->0 —h

Fr
1
= - lim sin
h^0 h for
ur
= A number which oscillates between - 1 and 1
(LHD at x = 0) does not exist.
ks

Similarly, it can be shown that RHD at x = 0 does not exist.


Yo
oo

Hence, /(x) is not differentiable at x = 0.


B

LEVEL-2
re

JL 1
ou

jV*,
ad

EXAMPLE 8 Discuss the differentiability of f (x) = • x e , x * 0


at x = 0.
Y

0 , x =0
SOLUTION We observe that:
nd
Re

1+ 1
= x e - 2/x , x > 0
Fi

X X
xe
0 , x =0
/(*) =
-1 1
—+ —
xe x x =x , x <0

/(T)-/(0)
Now, (LHD at x = 0) = lim
x —> (T x —0
x-0
=> (LHD at x = 0) = lim = 1 [•/ / (x) = x for x < 0 and/ (0) = 0]
x —»0 x - 0
f(x)-f(0)
and. (RHD at x = 0) = lim
x->0 + x-0

x e ~2/x -0 - 2x for x > 0 and / (0) = 0]


=> (RHDatx = 0) = lim [•■ f(x)=xe
x-> 0 X

ReadYourFlow.COM
10.8 MATHEMATICS-XII

=> (RHD at x = 0) = lim e 2/x = 0.


x->0
(LHDatx = 0) ^ (RHD at x = 0)
So, f(x) is not differentiable at x = 0.
x2 f (a) - a2 f (x)
EXAMPLE 9 Iff (x) is differentiable at x = a, find lim
x a x-a
SOLUTION It is given that/ {x) is differentiable at x = a. Therefore,
/(*)-/(«)
lim --------------- exists mutely.
x —>a x-a
f(x) -f (a)
Let lim = /'(«) ...(i)
j —» a x-a
x2 / (a) -a2 f (x)
Now, lim

low
x->a x-a
x2 f(a)-a2 f{a) + a2 f{a)-g2 f (x)
lim
x —> a x-a

ee
(x2 -a2)f(a)-a2 {f {x) -/(«)}
= lim
rF
Fr
x -> a x-a
(x2 - a2) f (a) a2ff(x)-f(a)
lim
for
x —> a x-a x-a
u
ks

= lim (x + a) f (a) - a2 lim /(*)-/(«) = 2a f(a)-a2f'(a) [Using (i)]


Yo

x ->a x a x-a
oo

x2 , X< c
B

EXAMPLE 10 For what choice of a and b is the function f (x) =■ is differentiable


at x = c. ax + b , x >c
re

SOLUTION It is given that / (x) is differentiable at x = c and every differentiable function is


ou
ad

continuous. So, / (x) is continuous at x = c.


Y

lim /(x) = lim f(x) = /(c)


X-+C x —> c+
nd
Re

lim x2 = lim {ax + b) = c2 [Using definition off (x)]


X —> c x —> c
Fi

=> c2 = ac + b
Now, / (x) is differentiable at x = c
=> (LHD at x = c) = (RHD at x = c)
f(x)-fic) f(x)-fic)
=> lim lim
x->c x -c x —> c+ x-c
X*2 - C 2 (ax + b) -c2
=> lim = lim [Using definition of/(x)]
x ->c x—c X —> c x-c
X*12 -C 2 ax + b - (ac + b)
=> lim = lim [Using (i) ]
x -> c x -c X -> c x -c
a (x - c)
lim (x + c) = lim
X —> c X —> C X -c

lim (x + c) = lim a
X —> c x —»c

ReadYourFlow.COM
DIFFERENTIABILITY 10.9

=> 2c = a ...(ii)
From (i) and (ii), we get
c2 = 2c2 +b => b = -c2
2
Hence, a = 2c and b = -c .
xf(2)-2f(x)
EXAMPLE 11 Iff (2) = A and f' (2) =1, then find lim
x->2 x-2
SOLUTION Using definition of derivative, we have
/(*)-/(2)
lim = /'(2)
x->2 x-2

f(x)-f(2)_1
=> lim [••• /' (2) = 1]
x —>2 x-2

w
Now,
xf(2)-2f(x)
lim
x->2 x-2

Flo
ree
xf (2) - 2 / (2) + 2/ (2) - 2 / (x)
= lim

F
x —>2 x-2
(x-2)f(2)-2(f(x)-f(2)) or
ur
= lim
x^>2 x-2
sf
(x-2)f(2) /(*)-/(2)
= lim -2 lim
k
Yo

x->2 x-2 x-*2 x-2


oo

= / (2) - 2 /' (2) = 4-2x1 = 2 [Using (i) and /(2) = 4]


B
re

EXAMPLE 12 A function f:R^R satisfies that equation f(x + y)=f(x) f{y) for all x,yeR,
f(x) * 0. Suppose that thefunction f(x) is differentiable at x = 0andf (0) = 2. Prove thatf (x) = 2 f(x).
ou
ad

[NCERT EXEMPLAR]
Y

SOLUTION We have, f(x + y) = f(x) f(y) for all x,y <=R


nd
Re

/(0 + 0) =/(0)/(0) [Putting x = 0, y = 0]


Fi

/(0)=/(0)/(0)
/(0) (1-/(0)) =0 [•.• /(x) 0 for any x .'./(0) ^ 0]
=> 1-/(0) =0
=> /(0)=1
It is given that f(x) is differentiable at x = 0 and /' (0) = 2.
/(0 + h) -/(0) /(c + /i) -/(c)
/'(0) = lim Putting c = 0 in /' (c) = lim
h —> 0 h Ii -» 0 h
/(^)-/(0)
2 = lim
h-+0 h

=> 2 = lim
/i-»0 h
Now, f(x + h) -f(x)
/'(x) = lim
/i->0 h

ReadYourFlow.COM
10.10 MATHEMATICS-XII

=> f'(x) = lim


mm-m [■•• /(* + 3/) =f(x) f(y) for all x, y e /. f(x + h) = f(x) f(h)]
h->0 h

=> f'(x)=f(x) lim


m-i
h-+0 h
=> f'(x)=2f(x) [Using (i)]
Hence, /' (x) =2 f (x).

EXERCISE 10.1
LEVEL-1

1. Show that f{x) = | x - 31 is continuous but not differentiable at x = 3. [CBSE2012,2013J


2. Show that /(x) = x1^ 3 is not differentiable at x = 0.

w
12 x -13, if * < 3
3. Show that/(x) = is differentiable at x = 3. Also, find / '(3).
2x2 + 5, H x > 3
4. Show that the function / defined as follows
f 3x - 2 , 0 < x <1
Flo
ee
Fr
f(x) = 2x2 -X, 1 < x < 2
5x - 4 , x > 2 for
ur
is continuous at x = 2, but not differentiable thereat. [CBSE 20101
5. Discuss the continuity and differentiability of the function/(x) = | x | +1 x -1| in the interval
ks

(-h 2). [CBSE 2015]


Yo
oo

6. Find whether the following function is differentiable at x = 1 and x = 2 or not:


B

x , x <1
2 -x , l<x<2
re

/(*) = [CBSE 2015]


-2+3x-x2/ x>2
ou
ad

LEVEL-2
Y

1
d

x7" sin - , x*0


Re

7. Show that the function /(x) = - x is


n
Fi

0 , x=0
<
(i) differentiable at x = 0, if m > 1
(ii) continuous but not differentiable at x = 0, if 0 < w < 1
(iii) neither continuous nor differentiable, if m < 0
i
x2 + 3x + a / if x <1
8. Find the values of a and b so that the function /(x) =
bx + 2 , if x > 1
is differentiable at each x eR. [NCERT EXEMPLAR]
| 2x - 31 [x]
x>l
9. Show that the function f(x) =< 71 X is continuous but not differentiable
sin / x <1
2
at x = 1.

ReadYourFlow.COM
DIFFERENTIABILITY 10.11

ax2 -b
, if | *| <1
10. If f{x) = 1 is differentiable at .v = 1, find a, b.
, if |x| >1
x
x2+3x + a / x<l .
11. Find the values of a and b, if the function/(x) defined by f(x) =
bx + 2 , .t >1i is
differentiable at x = 1. [CBSE2016]

ANSWERS
3. 12 5. Continuous on (-1, 2) but not differentiable at x = 0,1.
6. Not differentiable at x = 1, but differentiable at x = 2. 8. a - 3rb =5
10. fl = -1/2,1? =-3/2 ll.a = 3,b=5
HINTS TO NCEART& SELECTED PROBLEMS
8. Use (LHD at x = 1) = (RHD at x = 1) and, lim /(x) = lim /(x) = /(l)

w
x-^1" X -> 1+

3
(2x-3) [x] '*"2

Flo
9. / (x) can be re-written as follows: / (x) = - (2x - 3) , l<x<-

ee
2

Fr
TCX
sin , x <1
2 for
ur
Now, check continuity and differentiablility off (x).
1
, if x<-l
x
ks
Yo

10. / (x) can be re-written as follows: / (x) = (ax2-b) , if -1 <x < 1


oo

1
, if x>l
B

x
re

Now, check continuity and differentiability of / (x).


ou
ad

10.2 DIFFERENTIABILITY IN A SET


Y

A function f(x) defined on an open interval (a, b) is said to be differentiable or derivable in open interval
(a, b) if it is differentiable at each point of (a, b).
nd
Re

A function/(x) defined on [a, b] is said to be differentiable or derivable at the end points a and
Fi

b if it is differentiable from the right at a and from the left at b.

In other words, lim /(*) -/(«) and lim /(*) -m both exist.
x —> a+ x —a x->b~ x-b
Iff is derivable in the open interval (a, b) and also at the end points a and b, then f is said to be derivable
in the closed interval [a, b].
A function/is said to be a differentiable function if it is differentiable at every point of its
domain.
If a function is differentiable at each x e R, then it is said to be every where differentiable.
For example, a constant function, a polynomial function, sin x, cos x etc. are everywhere
differentiable.
SOME USEFUL RESULTS ON DIFFERENTIABILITY
(i) Every polynomial function is differentiable at each x eR.
(ii) The exponential fimction ax, > 0 is differentiable at each x eR.
(iii) Every constant function is differentiable at each x eR.

ReadYourFlow.COM
10.12 MATHEMATICS-XII

(iv) The logarithmic function is differentiable at each point in its domian.


(v) Trigonometric and inverse-trigonometric functions are differentiable in their respective
domains.
(vi) The sum, difference, product and quotient of two differentiable functions is
differentiable.
(vii) The composition of differentiable function is a differentiable function.
(viii) If a function / (x) is differentiable at every point in its domain, then
f(x + h)-f(x) f(x-h)-f(x)
lim or, lim
/i->0 h /! ->0 -h
is called the derivative or differentiation of / at x and is denoted by / '(x) or, — (/ (x)).
dx
ILLUSTRATIVE EXAMPLES
LEVEL-1

w
EXAMPLE 1 Iff (x) = x2 + 2x + 7, find /' (3).
SOLUTION We know that a polynomial function is everywhere differentiable. Therefore, / (x)
is differentiable at x = 3.
f (3 + h) - f (3)
Flo
ee
/' (3) = lim
h

Fr
/i->0

{(3 + h)2 + 2(3 + h) + 7} -[9 + 6 + 7}


/' (3) = lim
for
ur
h->0 h
Sh + h2
ks

=> /' (3) = lim = lim (8 + h) = 8.


h
Yo

h-+0 /; -> 0
oo

EXAMPLE 2 Find f' (2) and f' (5) zuhen f (x) = x2 + 7x + 4.


B

SOLUTION We know that a polynomial function is everywhere differentiable. Therefore, f(x)


re

is everywhere differentiable. The derivative off at x is given by


f(x + h)-f(x)
ou
ad

f'(x) - lim
h —► 0 h
Y

{(x + h)2 + 7(x + h) + 4} - {x2 + 7x + 4}


=> /'(x) = lim
nd
Re

/i->0 h
Fi

2hx + 7h + h2
=> / '(x) - lim = lim (2x + 7 + h) = 2x + 7
// ->0 h /(->0
Putting x = 2 and x =5 respectively in/'(x) = 2x + 7, we get
/ '(2) =2x2 + 7 = 11 and /'(5) =2x5 + 7 =17.
EXAMPLE 3 For the function f given by f(x) = x2 - 6x + 8, prove that f '(5) - 3 /' (2) = / '(8).
SOLUTION Clearly, f(x) being a polynomial function, is everywhere differentiable. The
derivative of / at x is given by
/ (x + h) — f (x)
/'(x) = lim
h->0 h
{(x + h)2 - 6(x + h) + 8} - {x2 - 6x + 8}
f'(x) = lim
h -> 0 h
2hx - 6h + h2
=> f'(x) = lim = lim (2x - 6 + /z) = 2x - 6
/i->0 h h-+0

ReadYourFlow.COM
DIFFERENTIABILITY 10.13

/ ' (5) - 3/' (2) = (2 x 5 -6) - 3 (2 x 2-6) = 4 + 6 = 10 and, /'(8) =2x 8-6 =10.
Hence, / '(5) - 3/ '(2) = /'(8).
1 -x , x <1
EXAMPLE 4 Discuss the continuity and differentiability of f (x) = -j (1 - x) (2 - x), 1 < x < 2
3 -x , x>2
SOLUTION When x < 1, we have / (x) = 1 - x.
We know that a polynomial function is everywhere continuous and differentiable. So, / (x) is
continuous and differentiable for all x < 1.
Similarly, / (x) is continuous and differentiable for all x e (1, 2) and x > 2.
Thus, the possible points where we have to check the continuity and differentiability of / (x) are
x = 1 and x = 2.
Continuity at x = l: We observe that:

w
lim / (x) = lim (1 - x) [v / (x) = 1 - x for x < 1]
x->r
= 1-1 = 0
lim / (x) = lim (1 - x) (2 - x)
Flo [v / (x) = (1 - x) (2 - x), for 1 < x < 2]

ee
x -> 1+ x —> 1

Fr
= 0
and, /(l) = (1-1) (2-1) = 0. for
ur
lim /(x) = lim /(x)=/(l)
x->r x —> 1+
ks
Yo

So, / (x) is continuous at x = 1.


oo

Continuity at x = 2: We observe that:


B
re

lim / (x) = lim (1 - x) (2 - x) [v / (x) = (1 - x) (2 - x) for 1 < x < 2]


x ->2~ x^2
ou
ad

= (2-1) (2-2) = 0
Y

and. lim /(x) = lim (3-x) = 3-2=1 [••• / (*) = 3 - x for x > 2]
x -> 2
nd

2+
Re

lim /(x) * lim /(x)


Fi

x -> 2
So, / (x) is not continuous at x = 2.
Differentiability at x = 1: We observe that:
(LHD at x = 1) = lim /(*)-/(D
x->r x —1
(l-x)-O
=> (LHD at x = 1) = lim [Using definition of / (x)]
x —> 1 x —1
X—1
=> (LHD at x = 1) = - lim = -1
x —> 1 x -1
/(*)-/(!)
and. (RHD at x = 1) = lim
x -» 1+ x —1
(l-x)(2-x)-0
(RHD at x = 1) = lim [Using definition of / (x)]
x —> 1 x—1

ReadYourFlow.COM
10.14 MATHEMATICS-XII

(*-l) (x-2)
=> (RHD at x = 1) = lim
x —> 1 X —1
=> (RHD at x = 1) = lim x-2 = 1-2 = -1
a- -> 1

Clearly, (LHD at x = 1) = (RHD at x = 1). So,/(x) is differentiable at x =1.


Differentiability at x= 2:
Since / (x) is not continuous at x = 2. So, it is not differentiable at x = 2.
EXAMPLE 5 Discuss the differentiability o// (x) = | x -11 +1 x - 21.
SOLUTION We have,
f(x) = |x-l|+|x-2|
-(x-l)-(x-2) for x < 1
=> f(x)=\ x -1 - (x - 2) for 1 < x < 2
(x -1) + (x - 2) for x > 2

w
-2x + 3 , x <1
=> f(x)=l 1 , 1 <x <2
2x - 3 , x>2

Flo
ee
When x < 1, we have / (x) = - 2x + 3 which, being a polynomial function is continuous and

Fr
differentiable.
When 1 < x < 2, we have / (x) = 1 which, being a constant function, is differentiable on (1, 2).
or
ur
When x > 2, we have / (x) = 2x - 3 which, being a polynomial function, is differentiable for all
x > 2.
f
ks

Thus, the possible points of non-differentiability of / (x) are x = 1 and x = 2. So, let us check the
Yo

differentiability of/(x) at these points.


oo

Differentiability at x = 1:
eB

(LHD at x = 1) = lim /(*)-/(!)


*->r x —1
r
ou
ad

(-2x+ 3) -1
=> (LHD at x = 1) = lim [v / (x) = - 2x + 3 for x < 1]
Y

* -> l x —1
-2 (x -1)
nd

=> (LHD at x = 1) = lim = lim =-2 = -2


Re

1 x —1 x^l
Fi

and. (RHD at x = 1) = lim /(*)-/(P


X -> l+ x—1
1 -1
=> (RHD at x = 1) = lim = 0. [y / (x) = 1 for 1 < x < 2]
A' —> 1 X — 1
(LHD at x = 1) * (RHD at x = 1)
So, / (x) is not differentiable at x = 1.
Differentiability at x = 2:
/(*)-/(2)
(LHD at x = 2) = lim
x-2
1-(2x2-3)
=> (LHD at x = 2) = lim [v / (x) = 1 for 1 < x < 2 &/ (2) = 2 x 2 - 3]
x —> 2 x-2
1 -1
=> (LHD at x = 2) = lim = 0.
x->2 x-2

ReadYourFlow.COM
DIFFERENTIABILITY 10.15

/(*)-/(2)
and/ (RHDatx = 2) = lim
x->2 + x-2
(2x - 3) -(2x2-3)
=> (RHDatx = 2) = lim [•.• / (x) = 2x - 3 for x > 2]
x —» 2 x-2
2x - 4 2 (x - 2)
=> (RHDatx = 2) = lim = lim = 2
x 2 x-2 x -> 2 x - 2
(LHDatx = 2) ± (RHD at = 2)
So, / (x) is not differentiable at x = 2.
REMARK The function f(x) given by f (x) - \ x - + \ x - a2\+ \ x - a3\ + ... +1 x - is not
differentiable at x = alf a2, a3,..., an. However, it is continuous at these points.

LEVEL-2

low
x2 + 3x + a , for x < 1
EXAMPLES If f(x) = is everywhere differentiable, find the values of
bx + 2 , for x>l
a and b.
SOLUTION For x < 1, we have / (x) = x + 3x + fl which is a polynomial.

ee
rF
Fr
For x > 1, we have / (x) =bx + 2 which is also a polynomial.
Since a polynomial function is everywhere differentiable. Therefore, / (x) is differentiable for all
for
x > 1 and also for all x < 1. Thus, we have to use the differentiability of / (x) at x = 1 to find the
values of a and b.
u
ks

Now,
Yo

/ (x) is differentiable at x = 1
oo

=> / (x) is continuous at x = 1


B

=> lim /(x) = lim /(x) = /(l)


re

1+
ou

=> lim x2 + 3x + a = lim bx + 2 = 1 + 3 + fl


ad

X —> 1 x —> 1
Y

=> 1 + 3 + fl = b + 2
=> a-b+2 = 0
nd
Re

Again, / (x) is differentiable at x = 1.


Fi

=> (LHDatx = 1) = (RHDatx = 1)

=> lim
/(*)-/(!)
lim /W-Zd)
x->l_ x —1 x —> 1"*" x —1

x2 + 3x + fl - (4 + fl) (bx + 2) - (4 + fl)


=> lim = lim
x —1 x 1 x —1
x2 + 3x - 4 bx -(2 + a)
=> Urn lim
x —> 1 x —1 x —> 1 x —1
(x + 4) (x -1) bx-b
=> lim = lim [From (i), 2 + a = b]
x —> 1 x —1 x —> 1 x — 1
=> lim (x + 4) = lim b
X —> 1 x->l

=> 5 = b.
Putting b = 5in (i), we get a = 3. Hence, a = 3 and b = 5.

ReadYourFlow.COM
10.16 MATHEMATICS-XII

EXAMPLE 7 Discuss the differentiability off (x) =| log{. x| for x >0.


SOLUTION We have.
f(x) — | loge x] -1 - log(, x , for 0 < x < 1
loge x , for x > 1
Clearly, /(x) is differentiable for all x>l as well as for all x<l. So, we have to check its
differentiability at x = 1.
We have,
/(*)-/(!)
(LHDatx = 1) = lim
x->r x —1
- log x - log 1
=> (LHD at x =1) = lim [•.• /(x) = - loge x for 0 < x < 1]
x->l~ x—1
logx = log(l -/2) log(l -h)
=> (LHD at x = 1) = - lim - lim = - lim = -1

w
x->r x — 1 /j ->■ 0 1 - /? -1 /i -»o -h
/(-y)-/(D
and. (RHD at x = 1) = lim

Flo
X -> l+ x —1
log x - log 1 log (1 + h)
= lim log (1 + h) = 1

ee
=> (RHD at x = 1) = lim = lim
x->l+ x — 1 It-+0 1 + h -1 /j->0 h

Fr
Clearly, (LHDatx = 1) ^ (RHD at x = 1). So, / (x) is not differentiable at x = 1.
or
ur
EXERCISE 10.2
f
LEVEL-1
ks
Yo

1. If / is defined by / (x) = x2, find / '(2).


oo

2. If/ is defined by / (x) = x2 - 4x + 7, show that /'(5) = 2/'^J


eB

3. Show that the derivative of the function / given by / (x) = 2x3 - 9x2 + 12x + 9, at x = 1
r
ou
ad

and x = 2 are equal.


4. If for the function <t> (x) = ?ix2 + 7x - 4, <D'(5) = 97, find X,
Y

5. If/(x) = x3+7x2 + 8x-9,find/'(4).


nd
Re

6. Find the derivative of the function / defined by / (x) = mx + c at x = 0.


Fi

7. Examine the differentiability of the function / defined by


2x + 3 , if - 3 < x < - 2
f(x)=\x + l , if -2 <x <0 [NCERT EXEMPLAR]
x+2 , if 0 < x < 1
8. Write an example of a function which is everywhere continuous but fails to be
differentiable exactly at five points.
LEVEL-2
9. Discuss the continuity and differentiability of / (x) = | log |x| |.
10. Discuss the continuity and differentiability of f (x) =
1
(x - c) cos , X it c
11. Discuss the continuity and differentiability of / (x) = ■ x -c
0 X = c

12. Is | sin x | differentiable? What about cos | x | ?

ReadYourFlow.COM
DIFFERENTIABILITY 10.17

ANSWERS
1. 4 4. 9 5. 112 6. m
7. Not differentiable atx = 0 and x = -2. 8. f(x) = | x| + | x -11 +1 x - 2 j +1 x - 31 +1 x - 4
9. Not differentiable at x = ± 1 10. Not differentiable at x = 0
11. Not differentiable at x = c
12. | sin x | is not differentiable at x = rnt, n <= Z, cos | x | is everywhere differentiable .
_________________________________ VERY SHORT ANSWER QUESTIONS (VSAQs)
Answer each of the following questions in one word or one sentence or as per exact requirement of the
question:
1. Define differentiability of a function at a point.
2. Is every differentiable function continuous?
3. Is every continuous function differentiable?
4. Give an example of a function which is continuous but not differentiable at a point.

w
5. If / (x) is differentiable at x = c, then write the value of lim / (x).
x -> c
6. If / (x) = | x - 21 write whether /' (2) exists or not.

Flo
7. Write the points where / (x) = | log,, x | is not differentiable.
8. Write the points of non-differentiability off (x) = | log | x| .

ee
Fr
9. Write the derivative off (x) =| x |3 at x = 0.
10. Write the number of points where/ (x) = | x | +1 x -11 is continuous but not differentiable,
for
11. If lim f(x)-f(c) exists finitely, write the value of lim / (x).
ur
x-*c x-c -V -> c
ks

12. Write the value of the derivative of / (x) = j x -11 +1 x - 31 at x = 2.


Yo
oo

13. If f (x) = Jx2 + 9, write the value of lim /(*)-/(4)


x -> 4 x-4
B
re

ANSWERS
2. Yes 3. No 4. / (x) = | x | at x = 0 5. / (c) 6. Does not exist 7. 1
ou
ad

8. ± 1 9. 0 10. x = 0,1 11./(c) 12.0 13. -


Y

5
MULTIPLE CHOICE QUESTIONS (MCQs)
nd
Re

1. Let/(x) = |x| and g (x) = |x3|, then


Fi

(a) / (x) and g (x) both are continuous at x = 0


(b) / (x) and g (x) both are differentiable at x = 0
(c) / (x) is differentiable but g (x) is not differentiable at x = 0
(d) / (x) and g (x) both are not differentiable at x = 0
2. The function / (x) = sin~ 1 (cos x) is
(a) discontinuous at x = 0 (b) continuous at x = 0
(c) differentiable at x = 0 (d) none of these
3. The set of points where the function/(x) =x|x| is differentiable is
(a) (-co,co) (b) (-co, 0) u(0, oc) (c) (0, co) (d) [0, co]
I x + 2|
,x*-2
4. If/ (x)= tan-1 (x-t-2) , then / (x) is
2 , x =-2
(a) continuous at x = - 2 (b) not continuous at x = - 2
(c) differentiable at x = - 2 (d) continuous but not derivable at x = - 2

ReadYourFlow.COM
10.18 MATHEMATICS-XII

5. Let f(x) = (x +1 x|) | x |. Then, for all x


(a) / is continuous (b) / is differentiable for some x
(c) /' is continuous (d) f" is continuous
6. The function/(x) = tT ^ is
(a) continuous everywhere but not differentiable at x = 0
(b) continuous and differentiable everywhere
(c) not continuous at x = 0
(d) none of these
7. The function / (x) = | cos x) is
(a) everywhere continuous and differentiable
(b) everywhere continuous but not differentiable at (2n + 1) k/2 , n e Z
(c) neither continuous nor differentiable at (2n + 1) k/2 , « e Z
(d) none of these

low
8. Iff (x) = - y/l -x2, then / (x) is

(a) continuous on [-1,1] and differentiable on (-1,1)


(b) continuous on [-1,1] and differentiable on (-1, 0) u <{) (0,1)

ee
(c) continuous and differentiable on [-1,1]
rF
Fr
(d) none of these
9. If / (x) = | sin x) + b + c | x|3 and if / (x) is differentiable at x = 0, then
for
(a) a = b - c = 0 (b) a = 0,b = 0; c e R
u
(c) b = c = 0, a eR (d) c = 0, a = 0,b e R
ks

x2 x2 x2
Yo

10. If/(x) = x2 + +... + + then at x = 0, / (x)


oo

l + x2 (l + x2)2 (1 + x2)”
B

(a) has no limit (b) is discontinuous


re

(c) is continuous but not differentiable (d) is differentiable


11. If / (x) = | loge x |, then
ou
ad

(a) /' (1+) = 1 (b)/'(l-)=-l (c)/'(l) = 1 (d) /'(l) = -1


Y

12. Iff (x) =| log,, |x|, then


nd
Re

(a) / (x) is continuous and differentiable for all x in its domain


(b) / (x) is continuous for all for all x in its domain but not differentiable at x = ± 1
Fi

(c) / (x) is neither continuous nor differentiable at x = ± 1


(d) none of these
1
for | x | > 1
13. Let/(x) = • x . If / (x) is continuous and differentiable at any point, then
ax 2 + b for | x | < 1
3 1 , 3
(b)
= - -,b = —
a
2 2 2
(c) fl = l,b = -1 (d) none of these
14. The function / (x) = x - [x], where [•] denotes the greatest integer function is
(a) continuous everywhere (b) continuous at integer points only
(c) continuous at non-integer points only (d) differentiable everywhere
ax2 + 1 , x > 1
15. Let / (x) = Then, / (x) is derivable at x = 1, if
x + 1/2, x<l.

ReadYourFlow.COM
DIFFERENTIABILITY 10.19

(a) a = 2 (b) a=l (c) fl = 0 (d) a =1/2


16. Let f (x) =| sin .’v: |. Then,
(a) / (x) is everywhere differentiable.
(b) f (.v) is everywhere continuous but not differentiable at x =nn ,n eZ
71
(c) / (a) is everywhere continuous but not differentiable at x = {In + 1) — , n <=Z.

(d) none of these


17. Let f (x) - \ cos x \. Then,
(a) f (x) is everywhere differentiable
(b) f (x) is everywhere continuous but not differentiable at x =n n,n eZ
(c) f (x) is everywhere continuous but not differentiable at x = {In + 1) - , n eZ
(d) none of these
18. The function f {x) =1 +\ cos x \ is
(a) continuous no where (b) continuous everywhere

w
(c) not differentiable at x = 0 (d) not differentiable at x = n rc, neZ
19. The function / (x) = | cos x | is
(a) differentiable atx ={2n + 1) n/2 , n eZ

Flo
(b) continuous but not differentiable at x =(2n + 1) tc/2, neZ

ee
(c) neither differentiable nor continuous at x = n n, n e Z

Fr
(d) none of these
sin (k [x - jt])
20. The function / (x) = -, where [•] denotes the greatest integer function, is
for
ur
4 + [x]2
(a) continuous as well as differentiable for all x e R
ks

(b) continuous for all x but not differentiable at some x


Yo
oo

(c) differentiable for all x but not continuous at some x.


(d) none of these
B

21. Let /(x) =fl + /?| x| + c| x|4, where a,fr, andc are real constants. Then, /(x) is
re

differentiable at x = 0, if
ou
ad

(a) fl = 0 (h) b = 0 (c) c = 0 (d) none of these


Y

22. If / (x) = | 3 - x | + (3 + x), where (x) denotes the least integer greater than or equal to x, then
/ (*) is
nd
Re

(a) continuous and differentiable at x = 3


(b) continuous but not differentiable at x = 3
Fi

(c) differentiable but not continuous at x = 3


(d) neither differentiable nor continuous at x = 3
1
, x*0
23. If/(x) =\\+ellx , then / (x) is
0 , x =0
(a) continuous as well as differentiable at x = 0
(b) continuous but not differentiable at x = 0
(c) differentiable but not continuous at x = 0
(d) none of these
1 - cos x
, x*0
24. If/(x) = ix sm x then at x = 0, / (x) is
, x=0
2
(a) continuous and differentiable (b) differentiable but not continuous
(c) continuous but not differentiable (d) neither continuous nor differentiable

ReadYourFlow.COM
10.20 MATHEMATICS-XII

25. The set of points where the function / (.t) given by / (x) = | x - 31 cos x is differentiable, is
(a) R (b) R-{3] (c) (0, qo) (d) none of these
x<-l
26. Let /(x) = <!|xi , -1 < x < 1 . Then, / is
0 , x>l
(a) continuous at x = -1 (b) differentiable at x = -1
(c) everywhere continuous (d) everywhere differentiable

ANSWERS
1. (a) 2. (b) 3. (a) 4. (b) 5. (a), (c) 6. (a) 7. (b) 8. (b)
9. (b) 10. (b) 11. (a),(b)12. (b) 13. (b) 14. (c) 15. (d) 16. (b)
17. (c) 18. (b) 19. (b) 20. (a) 21. (b) 22. (d) 23. (d) 24. (a)

w
25. (b) 26. (a)
SUMMARY

Flo
1. A real valued function / (x) defined on (a, b) is said to be differentiable at x = c e (a, b), iff
lim exists finitely

ee
X -> c x -c

Fr
/(*) -f(c) f(x) -f{c)
<=> lim lim
x -> c x-c h^c+ x-c for
ur
f(c-h) —f (c) / (c + h) -f (c)
<=> lim = lim <=> (LHD at x = c) = (RHD at x = c)
/; —> 0 -h h
ks

h-^0
Yo

2. A function is said to be differentiable, if it is differentiable at every point in its domain.


oo

3. Every differentiable function is continuous but, the converse is not necessarily true.
B

4. Following are some results on differentiability:


re

(i) Every polynomial function is differentiable at each x e R.


ou
ad

(ii) The exponential function ax, a > 0, a * l is differentiable at each x e R.


Y

(iii) Every constant function is differentiable at each x e R.


(iv) The logarithmic function is differentiable at each point in its domian.
d
Re

(v) Trigonometric and inverse-trigonometric functions are differentiable in their


n
Fi

respective domains.
(vi) The sum, difference, product and quotient of two differentiable functions is
differentiable.
(vii) The composition of differentiable function is a differentiable function.
(viii) If a function / (x) is differentiable at every point in its domain, then
f(x + h)-f(x) f(x-h) -/(x)
lim . 'J1/ lim
h —> 0 h -h

is called the derivative or differentiation off at x and is denoted by/' (x) or — (/(x)).
dx

ReadYourFlow.COM
CHAPTER 11
DIFFERENTIATION

11.1 INTRODUCTION
In the previous chapter, we have learnt about differentiability of a function at a point. The same
was extended to the domain of a function. In case, a function is differentiable at every point of its

w
domain, then each point in its domain can be associated to the derivative of the function at that
point. Such a correspondence between points in the domain and the set of values of derivatives

Flo
at those points defines a new function which is known as the derivative or differentiation of the
given function. In the previous class, we have studied that the derivative of a function f(x) is

ee
given by

Fr
f(x + h)-f(x) d f(x-h)-f(x)
= lim or. = lim
h->0 h dx /!->0
for -h
ur
This is also called the derivative or differentiation with respect to x and is also denoted by /' (x)
or, D / (x). Sometime the derivative or differentiation of a function / (x) is also called the
ks

differential coefficient of / (x). The process of finding the derivative of a function by using the
Yo
oo

above definition is called the differentiation from first principles or by ab-initio method or by
delta method.
B

Following are derivatives of some standard functions which have been derived in Class XI from
re

first principles.
ou
ad

/•\ d . yi. n-\


(i) —{x)=nx (ii) ~(ex) = ex
Y

dx dx
1
d

(hi) — (ax) = ax loge a , a>0 (iv) ~ (logt. x) =


Re

dx dx x
n
Fi

1
(v) (logfl x) = a > 0, fl * 1 (vi) — (sin x) = cos x
dx x logt, a dx

(vii) — (cos x) = - sin x (viii) — (tan a-) = sec2 x


dx dx
2
(ix) (cot x) = - cosec x (x) — (sec x) - sec x tan x
dx dx

(xi) — (cosec x) = - cosec x cot x


dx
Let us now have a brief recall of what else we have studied in Class XI.

11.2 RECAPITULATION

In the previous class, we have learnt about the following fundamental rules for differentiation.

(i) Differentiation of a constant functions zero i.e., — (c) = 0


dx

ReadYourFlow.COM
11.2 MATHEMATICS-XH

(ii) Let f(x) be a differentiable function and let c be a constant. Then, c f (x) is also
differentiable such that

i.e. the derivative of a constant times a function is the constant times the derivative of the
function.
(iii) Product rule: If f (x) and g(x) are differentiable functions, then / (x) g (x) is also
differentiable function such that

I <*>*(*»=!(/(*>>(*)+/(*)• |(* w)
If / (x), g (x) and h (x) are differentiable functions, then
d [f (x) 8(x) h = ^ (/(^)],? (x) h (x) + / (x) ^ (x) J h (x) + / (x) g(x) ~ [h(x)j
dx

low
(iv) Quotient rule: Iff (x) and g (x) are two differentiable functions and g (x) * 0, then

d_ \ f(x)
dx g(x) IgWl2

ee
rF
ILLUSTRATION 1 Differentiate the following functions with respect to x:

Fr
2X cot x
(i) (ii) ex log yjx tan x
Vx
for
SOLUTION (i) We have,
ou
ks

d 2X cot x
dx ffx
oo
Y
B

= *-l2zcotx x-1/2
re

dx
= \ 4~ (2X) \ (cot x) x" 1/2 + 2X (cot x)| x~ 1/2 + 2X cot x |£ (x" 1/2)
ou
ad

dx
Y

= 2X log^ 2 cot x x -1/2 + 2X (- cosec2 x) x_ 1/2 + 2X cot x x - — x~ 3/^2


nd

2
Re

2X loge 2 cot x 2X cosec2 x 2 x — 1 cot X


Fi

Vx rx x Jx

(ii) {ex log -Jx tan x)


dx
d x 1
e x — log x x tan x
dx
1 d j ex log x tan x|
2 dx

\ [ix{eX)} X d
tan x + e log x <[ — (tan x)
dx

lx, , ex tan x , x , 2
- ie log x tan x +------ + e log x sec x •
2 x
~ ex | log x tan x + tan x + log x sec2 x
x

ReadYourFlow.COM
DIFFERENTIATION 11.3

ILLUSTRATION 2 Differentiate the following functions with respect to x:


ex + sin x sin x - x cos x
(i) (ii)
l + log x x sin x + cos x
SOLUTION (i) We have.
d e + sin x
dx 1 + log x

(1 + log x) — (ex + sin x) - (ex + sin x) (1 + log x)


dx ax
(1 + log x)2

(1 + log x) (ex + cos x) — (ex + sin x) I 0 + — e x + sin x


(1 + log x) (ex + cos x) -

w
x
(1 + log x)2 (1 + log X)2

(ii)
d
dx
sin x - x cos x
x sin x + cos x
Flo
ee
Fr
d
(x sin x + cos x) (sin x - x cos x) - (sin x - x cos x) (x sin x + cos x)
dx dx
2 for
(x sin x + cos x)
ur
(x sin x + cos x) (cos x - cos x + x sin x) - (sin x - x cos x) (sin x + x cos x - sin x)
2
ks

(x sin x + cos x)
Yo
oo

(x sin x + cos x) (x sin x) - (sin x - x cos x) (x cos x)


(x sin x + cos x)2
B
re

2 2 2 2
(x sin x + x sin x cos x) - (x sin x cos x - x cos x)
= 2
ou

(x sin x + cos x)
ad

x 2 (sin 2 x + cos2 x)
2
Y

x
2 (x sin x + cos x)2
(x sin x + cos x)
nd
Re

ILLUSTRATION 3 Ify =(1 + x) (1 + x2) (1 + x4) (1 + x8)... (1 + x2 ),find


Fi

SOLUTION We have.
y =(1 + x) (1 + x2) (1 + x4) (1 + x8)... (1 + x2")

(1 - x) (1 + x) (1 + x2) (1 + x4) (1 + x8)... (1 + x2”)


=> y~ 1 —x

(1 -x2) (1 + x2) (1 + X4) (1 + X8) ... (1 + x2”)


y_ 1 —X
2«+ 1
1 -X
=> y=
1 -X
d 2>i+ 1 2n+\ d
)-(l-x (1-x)
rfy _ (1-x) dx (1-x )
dx
=>
dx (1-x)2

ReadYourFlow.COM
11.4 MATHEMATICS-XII

2«+ 1 2» + l
=>
dy_{\- x) (- 2” +1 x Va-x )
dx a-xf
2« + l- 1 n + l 2'7 + 1 2" + 1
dy=z2 jj + 1 x +2 X + 1 -x
=>
dx (1 -x)2
+ 1 -1 2" + 1 (2" + 1-l)
dy_- 2n+1x2n +1 + x
=>
dx (1-x)2
ILLUSTRATION 4 Iff(x) = \ cos x |, find f
SOLUTION We have.
!)-■<?)
[NCERT EXEMPLAR]
cosx, if 0 < x < —
2

w
/(x) =| cosx
- cos x, if — < X < 7t
2

- sin x, if 0 < x < -

Flo
ee
=> 2
/'(*) =

Fr
sin x, if — < x < 71
2
for
ur
Note that/(x) is not differentiable at x =
fl(n] ■ k 1 , fl( Srt) . 3;:
ks

/ — =-sin-=—p= and, / — = sin — =


UJ 4 V2 j {4 )
Yo
oo

ILLUSTRATION 5 Iff(x) = | cos x - sin x |, find f -


B

SOLUTION We have, 6 [NCERT EXEMPLAR]


re

cos x - sin x, if 0 < x < ^


/(x) =| cosx -sin x | =
ou
ad

- (cos x - sin x), if — < x < —


Y

4 2
nd

cosx - sin x, if 0 < x < —


Re

4
=> /(*H
Fi

sin x - cosx, if - < x < -


4 2
-sin x - cosx, if 0 < x < -^
=> f(x)=\ . , * TZ 7U
cosx + sinx, if - < x < -
4 2
71 71 ^3+1
1+1 71 n a/3+1
=> = - sin—cos— = ---- and4 /r>\ —
K
= cos—+ sin — =
6 6 2 3 3 3 2
ILLUSTRATION 6 Iff(x) =| log x |, x > 0, find f (\/e) and f {e).
SOLUTION We have, [NCERT EXEMPLAR]
-log x, if 0 <x <1
/(x)=|logx| =
log x, if x > 1
—1/x, if 0 < x < 1
=> /'(•t) =
1/x, if x > 1
/' (e) =1/e and /' (1/e) = - e

ReadYourFlow.COM
DIFFERENTIATION 11.5

11.3 DIFFERENTIATION OF INVERSE TRIGONOMETRIC FUNCTIONS FROM FIRST


PRINCIPLES
In the previous class, we have learnt that the derivative of a function / (x) is given by
f(x + h)-f(x) or f(x-h)-f(x)
i {f = lim
h-+0 h
lim
h^O -h
The process of finding the derivative of a function by using the above definition is called the
differentiation from first principles or, by flfr-initio method or, by delta method.
In this section, we will find the derivatives or differentiations or differential coefficients of
sin- 1 x, cos 1 x, tan -1 x, sec - 1 x, cosec -1 x and cot 1 x from first principles.
Following results will be very useful to find the same:
(i) sin- 1 x ± sin- 1 y - sm 1 j* ± !/ ^ - *2 j

w
(ii) cos 1 x ± cos 1 y - cos 1 xy +

(iii) tan 1 x ± tan -l y =tan i x±y


1 + xy
Flo
ee
Fr
n-an =nan~l
(iv) lim —
x —» a x — a for
ur
sin x sin (x - a) _ ^
(v) lim = 1 , lim
x -> 0 x x -» fl x -a
ks

tan x tan (x - a)
(vi) lim = 1 , lim
Yo

x-a
oo

x->0 x x —> (?
tan -1 i=i
B

sm
(vii) lim — =1, lim
re

x -> 0 x x -> 0 x
x -1
ou
ad

(viii) lim - = log,, a , a>0, a


x->0 X
Y

log, (1 + x) 1
(ix) lim
x -> 0 x
d
Re
n

logfl (1 + x)
(x) lim
Fi

= iogrt ^ •
x —> 0 x
THEOREM 1 If x e (-1,1), then the differentiation of sm 1 x with respect to x is - ... ...

i.e.. — (sin- 1 x) = for xe (-1,1)


dx

PROOF Let /(x) = sin 1 x. Then, f (x + h) = sin 1 (x +/;)


/ (x + h) - / (x)
lim
dx h^O h
sin 1 (x + h) - sin -1 x
=> -j- (/ (*)) = lim
dx /j -> o h
in 1 • (x + /z) ^1 -x2 - x -yjl -(x + h)2
=> 4- if (*)) = lim
dx /(-»0 h

ReadYourFlow.COM
11.6 MATHEMATICS-XII

-1 + ^-x2 -Xyjl-(x + h)2^


sin
=> 7“ (/ (*)) = lim
dx h —> 0
.r-Jl -(x + /i)2 j
| (x + h) *Jl -x2 Xyjl — (X + /?)2 j
x -----------------------
h

=> (/(x)) = lim


dx /z->0 /z
(x + /z)2 (l-x2)-x2 (l-(x + /i)2}
=> -j- (/ (*)) = lim

w
dx h ->0 h
1
X -7_______ __ ____ y

Flo
|(x + /z) ~x2+xyi-(x + /z)2 1

ee
4- (/ (*)) = lim (x + h)2-x2 1

Fr
dx h->0 h
x y-------) ---- y
|(x + /z) 4-x2 +xjl-(x + h)2 1
or
ur
1
f
=> — (/(x)) = lim (2x + h)x-
dx h —> 0 (x + /1) -x2 + x-Jl -(x + h)21
ks
Yo
oo

2x 1
=>
B

dx 2x^/l - x2
re

1
Hence, — (sin 1 x) =
ou

, where -1 <x <1. Q.E.D.


ad

dx t/i-*2
Y

//x e (-1,1), 1/zczz the differentiation of cos 1 x with respect to x is - 1


THEOREM 2
nd
Re

— (cos 1 x) = 1
i.e..
Fi

dx

PROOF Let/(x) = cos 1x. Then,/(x + Iz) = cos 1 (x + h)


4 (/ (*)) = iim f(x + h)-f(x)
dx h->0 h
cos 1 (x + /z) - cos 1 x
=> 7- (/ (x)) = lim
dx /j->0 h
- - sin- 1 (x + h) - JT - sm
. -1
x
d 2 2 71 . -1
4 (/ (x)) = lim — -1
=> v cos x =----sin x
h 2
sin 1 (x + h) - sin 1 x
=7 = - Urn
dx /z —> 0 h
1
=> /(/M) = [See Theorem 1]
dx

ReadYourFlow.COM
DIFFERENTIATION 11.7

Hence, — (cos 1 x) = 1 Q.E.D.


dx
THEOREM 3 The differentiation o/tan 1 x with respect to x is ^
1 l + x2
i.e., — (tan_1x) =
dx l + x2
PROOF Let / (x) = tan 1 x. Then, f (x + h) - tan 1 (x + h)
d f(x + h)-f(x)
Now, (f(x)) = lim
dx /7->0 h
i -1 x + h -x
tan
d tan 1 (x + h) - tan -1 x 1 + x (x + h)
=> (/(x)) = lim lim
dx /!->0 h /i->0 h

w
tan 1 h
l + x2 + hx
d
=> —(f(x)) = .lim

Flo 1 1
x------- 2------- = 1 X l + x2
1

ee
dx /; -> 0
h (1 + x + hx) l + x2
2

Fr
l + x + hx
for
ur
d (tan 1 x) = 1
Hence, for all x e K. Q.E.D.
dx l + x2
ks
Yo

THEOREM 4 The differentiation o/cot~ 1 x with respect to x is - 1


oo

1 + x2 ’
B

1
i.e. ^ (cot 1 x) =
re

' dx l+x2
ou
ad

PROOF Let / (x) = cot- 1 x .Then, / (x) = - - tan- 1x and sof{x + h)=—~ tan - 1 (x + h)
Y

2 2
d f {x + h) — f (x)
(fix))
d

lim
Re

dx h —> 0 h
n
Fi

- - tan- 1 (x + h)[ - it -tan


. -1
x
d 2 2
lim
dx /j->0 h
d tan -1 x - tan 1 (x + h)
=> —(/(*)) lim
dx h —> 0 h

tan 1 x -(x + h)
1 + x(x + h)
d
=> -f
dx
(/W) lim
h->0 h

-1 -h
tan
1 + x2 + x/i
=> -f
dx
(/(*)) lim
h-*0 h

ReadYourFlow.COM
11.8 MATHEMATICS-XII

tan 1 -h
d 1 + x2 +hx 1 -1 -1
^ T-(/(x)) = lim < x------------ 0------------- 1x
dx /i ->o -h 1 + x +hx l + x2 l + x2
1+ x2 +hx
d 1
Hence, — (cot -l x) = - Q.E.D.
dx l + x2
THEOREM 5 IfxeR -[-1,1], then the differentiation of sec~ 1 x zuith respect to x is 1

1 *
i.e., —(sec- 1 x) =
dx -1

w
tan 1 ^x2 -1 , if x>l
PROOF Let / (x) = sec 1 x. Then, / (x) =
CASE I When x > 1.

Flo n - tan 1 -Jx2 -1 , if x < -1

ee
We have, /(x) = tan-1 ^x2-1 and/(x + /z) = tan

Fr
d f{x + h)-f(x)
••• -r(f(x)) = lim
dx h
/i->0
for
ur
d tan ^(x + h)2 -1 tan 1 tJx2 -1
=>
ks

(f(x)) = .lim
dx h —> 0 h
Yo
oo

^(x + /j)2-1 -1
B

d
=> —(f (x)) = lim -tan
re

dx h-+0 h 1 + ■J(x + h)2 - lx -Jx2 -1


ou
ad
Y

, -1
tan
1 + -J(x + /i)2 -1 x ^x2 -1
d
Re
n

— (/ (^)) = lim
dx h->0 t](x + h)2 -1 - jx2-l /i |l + J(x + h)2
Fi

-1 X

1 + *J(x + h)2-1 x Jx2 -1

1 (x + /i)2-l-(x2-l) 1
(/ (x)) = lim • x
dx h —> 0 1+ xj(x +h)2 -1 x yx2 -1 ■J(x + h)2 -1 +^? -1

d 1 2hx + h2 1
=> —(f(x)) = lim • x
dx h^O h 1 + J(x + h)2 -1 xjx2 -1 xjix + h)2 -1 +Jx2 -1

d 2x + h 1
=> — (/ (x)) = lim x
dx fr->0 1 + J(x + h)2 -1 x yx2-! ij(x + h)2 -1 +-J? -1

ReadYourFlow.COM
DIFFERENTIATION 11.9

=> -f (/«) = 21
dx 1 + x2 - 1 -j?-! +-/? -1
1

1
=> =
dx xjx2 -1
CASE II When x <-l.
Proceeding as in Case I, we obtain
d , -i x 1
— (sec x) = -
dx xiJ7
-1
Thus, we obtain
1

w
for x > 1
d . -i . xyjx2 -1
— (sec x) =
dx 1

Flo
for x < — 1
x^

ee
d . -i . 1

Fr
Hence, — (sec x) = for all x * ± 1.
dx -1 for
ur
Q.E.D.
ks

THEOREM 6 Ifx e R-[-1,1], then the differentiation of cosec lx with respect to xis -1
Yo

|x|-J? -1
oo

-1
B

i.e.. — (cosec- 1 x) = for all x * ± 1.


dx
re

x
-1 x. Then, f (x + h) = cosec 1 (x + h)
ou
ad

PROOF Let / (x) = cosec


Y

f(x + h)-f(x)
-f (/(X)) = lim
dx h^O h
d
Re

1 “1
n

4- if (x)) = lim cosec (x + h) - cosec x


Fi

=>
dx h-+0 h
n TZ
----sec Ux + h) ----sec -l x
2 2
=> (/ (x)) = lim
dx h->0 h
-l (x + h) - sec 1 x
sec
=> -J- if (x)) = - lim
dx l/->0 h
1
=> 4-(fix))
dx
=- [See Theorem 5]
x

Hence, — (cosec 1 x) = - 1
for all x ^ ± 1.
dx -1
Q.E.D.

ReadYourFlow.COM
11.10 MATHEMATICS-XII

The above results and derivatives of other standard functions are listed below for ready
reference.
d , x. x
(i) -forVn*"-1
dx (ii) Tx(e)=e

(hi)
dx
= a'x loge a (iv)
1
i{los'x)=
(v) ~~~ (log(j = ------- (Vi) — (sin x) = cos x
dx x log^ a dx
(vii) ~ (cos ,t) = - sin x (vhi) -7- (tan x) = sec2 x
dx dx
d
(ix) — (cot x) -- cosec2 x (x)
-f (secx)=secxtan^
dx dx
— (sin-1 x) = 1
(xi) —(cosec x) = - cosec x cot x (xii)
dx dx

w
(xiii) —(cos-1 x) = - 1 -p(tan-3 x) = 1
(xiv)
dx yJl-X2 dx 1+x2
(xv) — (cot- 1 x) = - 1

Flo (xvi)
d , -1 .
— (sec x) =
1

ee
dx 1 +x2 dx Ixl^2-1

Fr
(xvii) — (cosec- 1 x) = -1
dx for
ur
-1
Following examples will illustrate some more applications of differentiation by first principles.
ks
Yo

ILLUSTRATIVE EXAMPLES
oo

LEVEL-1
B

EXAMPLE 1 Differentiate the following functions with respect to x fro?n first-principles:


re

2 fx
(i) e* (ii) e2v [CBSE 2003] (iv) esin'Y
ou

(iii) *
ad

2 e(x+h)2
Y

SOLUTION (i) Let /(x) = ex . Then, / (x + h) =


f(x + h) -f(x)
d

= lim
Re

dx h
n

h->0
Fi

e(x+h)2 _ex2
=> = lim
dx h —> 0 h
x2 2hx+ h2 x2
f-(f(x)) = lim — -e
=>
dx h ->0 h
2h x+ h2 ' 2hx + h2'
-1
=> -j- (fix)) = lim ex ■ - x
dx h ->Q 2h x + h2 h

2hx+h2
~~ (f(x)) =ex lim J - -1
=> x lim (2x + h)
dx /j-»0 2hx + h2 /j->0

0
e -1 x lim (2x + h), where 0 = 2h x + h2
=> -J- (fix)) = ex lim
dx 0 -> 0 0 h->0

ReadYourFlow.COM
DIFFERENTIATION 11.11

d X2 X2
— (f(x)) = e X 1 X 2x = 2x e
dx
— {e*2) = Ixe*2
dx
e2(x+h)
(ii) Letf(x) = e2x. Then, / (x + h) =
d f(x + h) -f(x)
— (/(*)) = lim
dx h —> 0 h
e2(x+h)_e2x 2x 2/i
. e -e
2x
d = lim -
— (/(x)) = lim
dx /i -> o h /i-»0 h
2h e*-!
d 2x e -1 = 2e2x lim
=> ^-(/(*)) = lim , where y = 2h
dx h-^0 2/z y->0 y

low
d (f(x)) = 2e2x x 1 = 2e 2x y-i
=> — v lim - =1
dx y —> 0 y

ee
d (e2x) = 2e 2x
rF
Fr
dx

(hi) Let/(x) = e^x. Then, / (x + h) = for


d f{x + h) — /(x)
ou
— (/(x)) = lim
dx h->0 h
ks

eylxTh _e^ Jx+ h-yfx


oo

d -Jx -1
=> — (/(x)) = lim lim
Y
B

dx fi->0 h h->0 h
re

JxTTl-yfx Jx + h --Jx
d ev -1
ou
ad

=> lim x ----------------


dx h->0 ^jx + h - -Jx h
Y

Jx+Tl-yfx
nd

Qx + h - -Jx) Qx + h + -Jx)
Re

d -Jx -i
=> J-if(x)) = e lim
Fi

dx /i->0 yjx + h-Jx h (yjx + h + -Jx )

d Jx ey _i x + h- x
=> J-(f(x)) = e lim -------- x lim
h-+0 h ^x + h + j
, where y = Jx + h - Jx
dx y —> 0 y

[v when h-^0,y ->0]

d %/x _______
1
=> ^(/W) - e X 1 X vita
dx Jx + Jx 2-Jx y->0 y
/
sin x . Then, / (x + h) = esin("+,j)
(iv) Let/(x) =<?
d f(x + h)-f(x)
— (/(x)) = lim
dx /i ->0 /z
^sin(x+ h) _tJsinx
d
=> — (/(X)) = lim
dx /i 0 h

ReadYourFlow.COM
11.12 MATHEMATICS-XII

^sin(r+ h)-smx
=> 4-am
dx
= e sin - h->0
lim
a
h
t?sin(x+ h) - sin a _ ^
=> 4-am
dx
= e sin a
lim
h^O sin (x + h) — sin x
^ J sin (x + h) - sin x
h

t>sin(A+ h)-sinx
-1
=> 4-am
dx
= esinx h->0
lim <
sin (x + h) - sin x
■ x lim
h->0
sin (x + h) - sin x
h

where
ey-l 2 sin (/z/2) cos (x + h/2)
4~
dx
(f (x)) = e sin a
lim
y -> 0 y
x lim
/i->0 2 (h/2)
, y = sin (x + /z) - sin x
[•■• when /j -» 0, y 0]

low
^-1
=> 4am
dx
= sin a
lim
y^-0 y
x lim
/i -> 0
sin (h/2)
h/2
x lim cos(x + /z/2)
/;->0

=> 4am = ^ sin a


(1) x (1) x (cos x) = e sin a X cos X

ee
dx
rF
Fr
EXAMPLE 2 Differentiate xex from first principles.
SOLUTION Let/ (x) = xe* Then, f (x + h) = (x + h) e(x + h)
for
f (x + h) — f (x)
4am =
ou
lim
ks

dx h 0 h
A+ h
oo

X
(x + h)e -xe
=> = iim
Y
B

dx h->0 h
re

x+h x+h
(xe - xex) + he
=> = lim
dx h-+0 h
ou
ad
Y

x eh-l'
a + h
=> ~ (f (*)) = lim < xe +e
dx h-+0 h
nd
Re
Fi

(eh-l
=> 4-(f W) = xex lim + lim e
x+h
= xex + ex = (x + 1) ex.
dx h->0 h h -+0

LEVEL-2
EXAMPLE 3 Differentiate log sin x by first principles.
SOLUTION Let / (x) = log sin x. Then, / (x + h) = log sin (x + h).
4-(f(x)) = lim f(x + h)-f(x)
dx h ->0 h
log sin (x + h) - log sin x
=> 4-(fix))=
dx
iim
>!->0 h
sin (x + h)
log
sin x
=> 4~(f(x))=
dx /?iim
-> o h

ReadYourFlow.COM
DIFFERENTIATION 11.13

sin (x + h) ^
log j 1 +
sin x
=> -j- if (x)) = lim
dx h —> 0 h
sin (x + h) - sin x
log i 1 +
sin x
=> ~7~ if (x)) = lim
dx h->0 h
sin (x + h) - sin x
log 1 +
sin x sin (x + h) - sin x
=> y-(/(x)) = lim
dx 0 ^ sin (x + h) - sin x sin x
sin x

w
sin (x + h) - sin x
log 1 +

Flo
sin x
sin (x + h) - sin x 1
=> 4-(fix)) = lim x

ee
dx h->0 sin (x + h) - sin x h sin x

Fr
sin x

sin (x + h) - sin x
for h h"
ur
log U + 2 sin cos x + —
sinx 2 2 1
=> -T (/(*)) = lim x lim —x
dx sin (x + h) - sin x fc->0 h sin x
ks

/i 0

1
Yo

sinx
oo

sin (x + h) - sin xl
B

h
log i 1 + sm
■ h
COS X 4- -
re

sinx 2 2 1
=> = Um x lim
dx h^O sin (x + h) - sin x /i-»0 (h'\ sin x
ou
ad

sinx 1 2/
Y

=> if (x)) = 1 X cos x X -4— = cot X.


nd

dx sm x
Re
Fi

EXAMPLE 4 Differentiate log sec xfrom first principles.


SOLUTION Let / (x) = log sec x. Then, f (x + h) = log sec (x + h)
f(x + h)-f(x)
f(/M)
dx
lim
h->0 h
log sec (x + h) - log sec x
=> lim
dx h-+0 h

sec (x + h)
log.
sec x
=> lim
dx /i-*0 h

cos X
log 1 + -1
cos (x + h)
/(/W) \
=> lim
dx h-^0 h

ReadYourFlow.COM
11.14 MATHEMATICS-XII

cos x - cos (x + h)
log 1 +
cos (x + h) ^ cos x - cos (x + h)
=> ~(/W) = lira
dx h->0 ^ cos x - cos (x + h) cos (x + h)
cos (x + h)

cos x - cos (x + h) . h
log 1 + 2 sin ( x + — sin
cos (x + h) 2 2
=> — (/(*))= lim x lim
dx /; —> 0 cos x - cos (x + li) // —> 0 h cos (x + h)
cos (x + h)

h
sin x + h)
2 sin
sin x

w
=> — (/ (x)) = 1 x lim x 2 = lx x 1 = tan x.
dx ' h ->o cos (x + h) h cos X
2
EXAMPLE 5

Flo
Iff (x) = x tan" 1 x, find f (f~2>) by first principles.

ee
SOLUTION We have,

Fr
/ (x + h) -f (x)
f' (x) = lim
h ->0 h for
ur
f'(yj3) = lim
h ->0 h
ks

(y[3 + h) tan 1 (-/3 +h) --fS tan 1


Yo

=> /'(V3) = Hm
oo

/i->0 h
B

/3 | tan 1 + h) - tan 1 ~J3 j + h tan 1 + h)


re

=> /'(V3) = lim


h -> 0 h
ou
ad

V3 t -if j3+h-j3
Y

=> f'{j3) = lim ---- tan --------—=--------- + lim tan 1 (f3 + h)


/; ->0 h 1 +y[3U3 + h) h —> 0
d
Re
n

tan 1 h
Fi

4 + ^3/^ 1
=> f' (V3) = ^3 lim ■ + lim tan 1 (^3 + h)
>X4 + V3/i h->0
/i-»0 h
4 + j3h

=^> /' (/3) = x x ^ + 1:311 -1 V3^ + tan -1 V3.


4
EXAMPLE 6 Differentiate cos- 1 (2x + 3) from first principles.

SOLUTION Let / (x) = cos ^ (2x + 3). Then,/ (x + //) = cos- 1 (2x + 3 + 2h).

f(x + h)-f(x)
-j- if (*)) = lim
dx /; -> o /i

cos 1 (2x + 3 + 2/i) - cos 1 (2x + 3)


=> if ix)) = lim
dx /i -> 0 h

ReadYourFlow.COM
DIFFERENTIATION 11.15

71
71 - sin-1 (2x + 3 + 2/i) ---- sin 1 (2x + 3)
><*» = lim 2 2
=>
h->0 h
sin 1 (2x + 3) - sin 1 (2x + 3 + 2/j)
=> -fi/M)
dx
= lim
h->0 h
sin- 1 l(2x + 3) ~(2x+3 + 2h)2 -(2x + 3 + 2h)^\ -(2x+3)2J

±(f(x))=hm
dx h 0 h
sin 1Z Z
----------- x —
dx /i -»0 Z h '
where Z =(2x + 3) Jl ~(2x + 3 + 2h)2 -(2x + 3 + 2h) ~(2x + 3)2

low
. -1 -7
sm Z
=> ^(X))
dx = ^0 I lim
h ->0 Z
= 1

ee
(2x + 3) ~(2x + 3 + 2/j)2 ~(2x + 3 + 2h) -(2x+3)2
rF
Fr
=> ~(f(x)) = lim
dx /i-»0 h
(2a- + 3)2 (1 ~(2.r + 3 + 2^)2} ~(2x + 3 + 2h)2 (1 -(2a + 3)2}
for
=> 7" (/(a)) = lim
dx h —> 0 /j |(2a + 3) -(2a + 3 + 2h)2 + (2a + 3 + 2ft) -(2a+ 3)2 j
ou
ks

_________________ (2a + 3)2 ~(2a + 3 + 2h)2_________________


oo

=> y-(/(A)) = lim


Y

rfA ft 0 h |(2a + 3) -(2a + 3 + 2h)2 + (2a +3 + 2h) -(2a+ 3)2 j


B
re

________________ - 4h (2a + 3) - 4h2


ou

=> lim
ad

rfA /i->0 h |(2a + 3) -(2a + 3 + 2h)2 + (2a + 3 + 2ft)^l-(2x+3)2|


Y

____________________- 4 (2a + 3)-Ah


nd
Re

=> -7- (/ (*)) = lim


dx h->0 (2a + 3) -(2a + 3 + 2h)2 + (2a + 3 + 2h) -Jl -(2a + 3)2
Fi

-2
=> -?-(/«)
dx
= - 2 (2 a
4 (2a + 3)
+ 3) -(2a + 3)2 ,]l-(2x+3)2
^tan x
EXAMPLE 7 Differentiate e from first principle.
e^.Then,f(x + h)=e^n{X+h)
SOLUTION Let / (a) =
f(x + h) — f (a)
=> fvM)
dx
= lim
/i->0 h
f tan (x + h) ftan x
=> -j- (/ (x)) = lim
dx h->0 ft
^tan(A+ fr) - Jtanx ^
I tan a
=> T" (/ (*)) = lim ev
dx ft—>0 ft

ReadYourFlow.COM
11.16 MATHEMATICS-XII

^tan(x + It) - ^/tan x


-1 Jtan (x + h) - /tan x
=> lim x ----------------------- ----------
/i -> 0 ^/tan (x + /?) - ^/tan x h

^%/tan(x+ /i) - v/tan .r


1 /tan {x + h) - /tan x
4-(f(x))=e^x
dx
x lim x lim ----------------- --------
to -> 0 v'tan (x + h) - an x h->0 h

C? . r . ,, Jtan x tan (x + to) - tan x 1


=> — (/ (x)) = x1x lim X . ^------ ---------
dx to -» 0 to ^/tan (x + to) + ^/tan x
sin to 1
=> = c lim -------------------------------- x ■ - ^—=------------------
dx to -> 0 to cos (x + to) cos x .Jtan (x + to) + .Jtan x
,/tan x
d Jtan .r 1 1 ev 2
=>

w
^(/W)=J x 2 x sec x.
UA cos x 2 .Jtan x 2 Jtan x
EXAMPLE 8 Differentiate x tan -1 x from first principles.
SOLUTION

Flo
Let / (x) = x tan -1 x. Then, / (x + to) = (x + to) tan -1 (x + to)

ee
/(x + to)-/(x)

Fr
-7-(/(*)) = lim
dx to -»• 0 to
(x + to) tan 1 (x + to) - x tan 1 x
for
ur
-7-(/(*)) = lim
Jx to -»0 /i
ks

d ^ tan 1 (x + to) - tan 1 x to tan 1 (x + to)


Yo

=> — (/ (x)) = lim +


oo

dx to->0 to to
B
re

x + to -x
x tan -1
1 + x(x + to)^
ou

=> tan 1 (x + to)


ad

lim • ■ + lim
dx to -»0 to to —> 0
Y
nd
Re

to
tan -1
Fi

=> -^-(f(x))=x lim 1 + x (x + to) 1 -1 x . -1


x > + tan x = ------ r- + tan x
dx to -> 0 to {1 + x (x + to)} 1 +x2
1 + x (x + to)

EXAMPLE 9 Differentiate sin- 1 Jx (0 < x < 1) from first principles.


SOLUTION Let / (x) = sin 1 Jx. Then, / (x + to) = sin- 1 Jx + to
f (x + h)-f{x)
-7-(/(x)) = Urn
dx to 0 to
1
in 1 Jx + to - sin 1 Jx
=> ^-(/(x)) = lim
dx to 0 to
sin-1 j Jx + to yJl^-X - Jx JT -x-to
=> (/(x)) = lim
dx to —> 0 to

ReadYourFlow.COM
DIFFERENTIATION 11.17

• “I Z
sin -7 2 ____ ____ _______
=> “7" (/ (■'■)) = lim
x —, where Z = +h - x - ~Jx -x - h
dx Z

in Z sin 1 Z
=> 7-(/(*)) = Hm 7 v lim = lim = 1
rfx /i -» 0 h /! ->0 Z z ->0 Z

(x + /i) (1 -x)-x(l -x-/i) ^ 1


=> lim
h->0 h yjx+hjr^ x + Vx - x - /i

=> Z(/W) = lim h(l-x + x) ^ 1


dx /i->0 h Jx + hfi -X + Vx yjl-x-h
Z(/W)= Um 1 1
dx h->0 ^jx + h^jl-x + ^jx^l-x-h 2 Jx Jl-x

w
REMARK It should be noted that — is an operator such that wheii it is applied on y = fix) gives us
dx

dx dx

Flo
— (/ (x)) = —. Also, — is not simply a fraction obtained by dividing dy by dx. For example, if — is
dx dx

ee
applied on sin x it gives us cos x i.e., (sin x) = cos x. The operator — is called the differential operator.

Fr
Input
or
ur
fix)
sf
\ /
Operator
k
Yo

d
oo

dx
B


re

Output
ou
ad
Y

Fig. 11.1
nd
Re

EXERCISE 11.1
Fi

LEVEL-1

Differentiate the following functions from first principles:

2. e3x ax+ b cosx -fix


1. 3. e 4. e 5. e

LEVEL-2
Differentiate each of the following functions from first principal:
fcotx
6. log cos x 7. e 8. x2 / 9. log cosec x 10. sin 1 (2x + 3)
ANSWERS
e^x
1. -e~x 2. 2>e^x ax+ b COS X
5. ------ 6. - tan x
3. ae 4. — e sm x

/cot X
e^ 2
7. - cosec2x 8. (x2 + 2x) ex 9. - cot x 10.
2 ^cot x f-(2x + 3)2

ReadYourFlow.COM
11.18 MATHEMATICS-XII

11.4 DIFFERENTIATION OF A FUNCTION OF A FUNCTION


In this section, we will study about the differentiation of composition of two or more functions.
THEOREM (Chain Rule) Iff(x) and g(x) are differentiablefunctions, thenfog is also differentiable and
(fog)'{x) = / '^(x) J £'(x)

or. = w)

PROOF Since / (x) and g (x) are differentiable functions. Therefore,


f(x + h)-f{x)
-7-(/(*)) = lim
dx /z->0 h
...(i)
g(x + h)-g(x)
-j- (g (*)) = lim
dx /; —> 0 h

w
fog (x + h) - fog (x)
Now, /{ww} = lim h->0 h

=> /{ww} = lim


Flo
f{g(x + h)}-f{g (x)}

ee
/1 -> 0 h

Fr
/ {g (^ + ^)> -fig (*)} y8(x + h)-g(x)
=> = lim
h->0 g(x + h)-g(x) h for
ur
f{g(x + h))-f{g(x)} g(x + h)-g(x)
=> /{ww} = lim
/i-»0 g(x + h) —g (x)
x lim
/i->0 h
ks

g(x + /z)-g(x)
Yo

=> lim x lim


oo

g(x+/z)-.^(x) g(x + h)-g(x) /j-»0 h


B

■: g (x) is differentiable, g (x) is continousand hence lim ^(x + h) =g(x)


re

/z -> 0
ou
ad

d
=>
^Hw}xl{x(4
Y

dx
REMARK 1 The above rule can also be restated as follows:
nd
Re

V z=/(y) and y =g(x), then =


Fi

dx dy dx
OR
Derivative ofz with respect tox = (Derivative ofz with respect to y) x (Derivative ofy with respect to x)
REMARK 2 This chain ride can be extended further.
Derivative ofz with respect to x = (Derivative ofz with respect to u) x ( Derivative ofu with
respect to v) x (Derivative of v with respect to x)
ILLUSTRATIVE EXAMPLES

LEVEL-1
EXAMPLE 1 Differentiate thefollowing functions with respect to x:
(i) sin (x2 + 1) sin x
(ii) * (iii) log sin x
SOLUTION (i) Let y = sin (x2 + 1). Putting u = x2 + 1, we get 3
9
y = sin u and u = x +1

ReadYourFlow.COM
DIFFERENTIATION 11.19

dy du
— = cos u and — = 2x
du dx
dV dy du
Now, = —x —
dx du dx
dy n
=> ---- = (cos u) 2x = 2x cos (x + 1) [v m = x2 +1]
dx
Hence, — (sin (x2 + 1)} = 2xcos(x2+1)
dx
ALITER We have.
d {sin (x2 + 1)} x (x2 + 1)
— (sin (x2 + 1)} =
dx d(x2 +1) dx

d {sin (x2 + 1)) = cos (x2 + 1)


=> — {sin (x2 + 1)} = {cos (x2 + 1)} x 2x

w
dx d(x2+l)

=> — {sin (x2 +1)} = 2x cos (x2 + 1).


dx
sin a-

Flo
ree
(ii) Let y = e . Putting u = sin x, we get
y = eu and u = sin x

F
dy u , du
— = e and — cos x.
for
ur
du dx
Now, rfy = rfy x —
ks

dx du dx
Yo
oo

dy u sin x [-.• u = sin x]


J - e cos x = cos x e
dx
B

sin x sin x
Hence, d .
re

e cos x.
dx
ou

d
ad

ALITER — d , sin x } x — {sin x} = e


siii x
x cos x.
dx
le d (sin x) dx
Y

(iii) Let y = log sin x. Putting!/ = sin x, we get


nd
Re

y = log w and w = sin x


Fi

dy i , du
---- = — and — cos x.
du u dx
dy dy du
Now, = x—
dx du dx
dy = -i x cos x i
=> X cos X = cot X.
dx u sin x

Hence, -J- !log sin x} = cot X.


dx
d 1
ALITER — {log sin x} = {log sin x} X — (sin x) = — x cos x - cot x
dx d (sin x) dx sin x

EXAMPLE 2 Differentiate the following functions with respect to x:


sin x2 2
(i) log sin x2 (ii) e (iii) sin (ex ).

ReadYourFlow.COM
11.20 MATHEMATICS-XII

SOLUTION (i) Let \j = log sin x2. Putting v = x2 and u = sin x 2 = sin v, we get
y - log u, u - sin v and v = x2
dy 1 du dv
— = cos v and — = 2x.
du u ' dv dx
dy dy du dv
Now, = X ----- X ---- -
dx du dv dx
dV 1 ,1
=> — = - x cos vx 2x =------- cos v x 2x [•.• u = sin v]
dx u sin v
dV i
=> —— = (cot v) 2x = 2x cot x [v v=x2]
dx
d 2 2
Hence,— (log sin x ) = 2x cot x
dx

w
d d j-(sinx2)x A(x2)
ALITER — {log sin x2} = {log sin x2} x
dx d (sin x2) d(x Z'\) dx

Flo
1 2 2
x cos x x 2x = 2x cot x .

ree
sin x2
sin x2 2 2

F
(ii) Let y = e . Putting x =v and u = sin x - sin v, we get
y = elt, u = sin v and v = x2
for
ur
dy u du , dv
= 2x.
ks

du dv dx
Yo

dy _ dy du dv
oo

Now,
dx du dv dx
B
re

=> — - e11 x cos vx 2x = esm v x cos x 2x [v u = sin z;j


dx
ou
ad

dy sin x2 2
= e x cos x x 2x [■■■V = x1]
Y

dx
f 9^ . 2
d .sinxZ d sinx 2] d( ■ 2
x — sinx 2\ x —(x
d 2. sin x
nd

ALITER e ) x cosx x 2x
Re

dx ----------T e dx dx
d (sin x ) ^
Fi

x2 2 x2
(iii) Lety = sin(t? ). Putting x =v and u = e e , we get
2
y = sin u, where u = and v = x .
du
dy = cos u, — v dv „
= e and — = 2x.
du dv dx
dy _ dy ^ du ^ dv
Now,
dx du dv dx
=> — = cos ux eV x r,2x = cos
dl
J
{ev) x ev x 2x r
[■.■ u = e ]
dx

=> — = cos(ex )x ex~ x 2x [:v = x2]


dx
d 2 d 2
ALITER — {sin ex ) = 2 (sin eX ) x ^ ^x ~T ^ = cos ^ ^ x eX x 2x
dx xZx z\
d(x-) dx
d(e* )

ReadYourFlow.COM
DIFFERENTIATION 11.21

EXAMPLE 3 Differentiate the following functions zvith respect to x:


(ii) y]x2 + x + 1 1
(i) (x2+x + l)4 (hi) sin 3 x (iv)
SOLUTION (i) Let y = (x2 + x + l)4. Putting x2 + x + 1 = w, we get ^7
y = u4 and u = x2 + x +1

^ = 4u3 and ^ = 2x + 1
du dx
dy _ dy; du
Now,
dx du dx

=> ^ = 41/3 (2x + 1) = 4 (x2 + x + 1)3 (2x +1).


dx
ALITER We have,
d

w
4-((*2+*+i)4) = {(x2 +x + l)4}x — (x2 +x + l) = 4 (x2 + x + 1)3 (2x + 1)
dx dx
d(x2 + x +1)

(ii) Let y = -Jx2 + x + 1. Putting x2 + x + 1 = u, we get

Flo
ree
y = -fii and ii = x2 + x +1

F
1 du
and — = 2x + 1 or
ur
du 2 2y[ll dx
sf
dy _ dy ^ du
Now,
dx du dx
k
Yo
oo

^ = _^=x(2x + l) = 1 [•.• W = X2 + X + 1]
=> x (2x +1)
dx 2^fii
B

2 Jx2 + x + 1
re

ALITER We have.
ou
ad

2 * d |(x2 + x + 1)1/2| X (x2 + x + 1)


X + X+1 > =
Y

d (x2 + x + 1)

= - (x2 + x + 1) -1/2 (2x +1)


nd
Re

2
Fi

3
(iii) Let y = sin x. Putting u = sin x, we get
y = u3 and u = sin x

^ = 3»2 and ^ = COS X.


du dx

Now, dy = dy^^
dx du dx

=> = 3u2 x cos x = 3 (sin x)2 x cos x [v u = sin x]


dx
dy r,-2
=> — = 3 sin x cos x.
dx
— (sin 3 x) = — {(sin x)3} = d ((sin x)3) x — (sin x)
ALITER
dx dx d (sin x) dx
3-1 2
= 3 (sin x) x cos x = 3 sin x cos x.

ReadYourFlow.COM
11.22 MATHEMATICS-XII

1 2 2
(iv) Let y = . Putting u = a - x , we get
2
-X
1 1/2
V ~ ~r
a
and u=a 2 - x 2
-Ju ~ u
du
fy = _Im-3/2 and — = - 2x.
du 2 dx
dy _dy ^ du
Now,
dx du dx
dy=_lu-3/2 1 x [v u = a2 -x2]
x (- 2x) =- 3/2 x (~ 2x) = (*2-*2)3/2
dx 2 2

ALITER
dx
{(«2-x2r1/2} = d
d(a2-x2)
[,2 2>-l/2l ^ f 2 2
\xTAa~x)

w
1 2 2V3/2 x
a -x (0-2*) =
2 («2-,2)3/2

Flo
EXAMPLE 4 Differentiate thefolloivingfunctions with respect to x:

ree
x sin x
(i) log (sec * + tan x) (ii) e

F
a + b cos x
(iii) sin-1 (x3) (iv) sin -1 or ,b > a
ur
b + a cos x
f
SOLUTION (i) Let y = log (sec x + tan x). Putting u = sec x + tan x, we get
ks

y = log u and u = sec x + tan x


Yo
oo

— = - and — = sec x tan x + sec2 x.


B

du u dx
re

dy _ dy du
Now,
dx du dx
ou
ad

— = - x (sec x tan x + sec2 x) = 1


=> sec x (tan x + sec x) = sec x.
Y

dx u sec x + tan x
x sin x
nd

(ii) Lety = e . Putting u = x sin x, we get


Re
Fi

y = e11 and u = x sin x


dy = eu and, -du
— = x cos x + sin x.
du dx
dy _ dy du
Now,
dx du dx
^ - eu (x cos x + sin x) = e x sm x (x cos x + sin x)
dx
d x sin A) x — (x sin x) = £ x sin x x (x cos x + sin x).
ALITER — (e X Sm * ) =
dx d (x sin x) dx
i (iii) Let y = sin x . Putting u = x , we get
. -1
y = sin u and » = x3
^ = -pi— and ^ = 3a:2
dx

ReadYourFlow.COM
DIFFERENTIATION 11.23

dy _ dy ^ du
Now,
dx
=> dy _ x 3x2 -
dx

d (sin
.. -i x 3,) = ----d
ALITER —
dx d (x

a+ b cos x a + b cos x
(iv) Let y = sin -1 . Putting u =
b + a cos x b + a cos x
a + b cos x
y = sin -1 u and u =
b + a cos x
1 du (b + a cos x) (0 -b sin x) - (a + b cos x) (0 - a sin x)
and, — = 2

w
du dx (b + a cos x)

du (a2 -b2) sin x

Flo
dy_ - and, —
du yjl -u 2 dx 2
(b + a cos x)

ee
dy = dyxdu

Fr
Now,
dx du dx
(a2 - b2) sin x 2 2
dy = , 1 .... x-----------------— 1 or (a -b ) sin x
ur
=>
dx ijl-u2 (b + acosx) 2
sf
a + b cos x (b + a cos x)
1-
1 b + a cos x
k
Yo
oo

dy _ (b + a cos x) (a2 -b2) sin x


B

=> x 2~
dx (b + a cos x)
re

J(b + a cos x)2 -{a + b cos x)2


ou
ad

(a2 -b2) sin x 1


d± = x
Y

dx {b + a cos x)
Jb^l^-cos^x^-a^il-cos^x)
nd
Re

dy {a2 -b2) sin x (b2 - a2) sin x ^


Fi

1 1
=>
dx b + a cos x b + a cos x b + a cos x
yj(b2 - a2) sin2 x sin x

EXAMPLE 5 Differentiate the following functions with respect to x:


ex (ii) log7 (log7 x)
(i) e (iii) logx 2
*
SOLUTION (i) Lety = e . Puttinge =»,weget
y = etl and u = ex
dy - eu and, —
du = e x

du dx
dy _ dy ^ du
Now,
dx du dx
dy ex
J — eu x e = e x e
X X
=> [■cu = ex]
dx

ReadYourFlow.COM
11.24 MATHEMATICS-XII

d ex drex) d ,x. ex x
ALITER e e x —(«? ) xe
dx J d{ex){ dx
(ii) Let y = log7 (log7 x). Putting u = log7 x, we get
y - log7 u and u = log7 x
dy_ = 1 du 1
and
du u loge 7 dx x loge 7
dy _ dy ^ du
Now,
dx du dx
dy = 1 1
=>
dx u log,, 7 x log,, 7
= 1 1
=> [••' u = log7 x]
dx log7 X X log,, 7 XX loge 7 x(log7 x)(loge 7)2

w
d (log7 (log7 X') )X~^(l°87 X) = 1 1
ALITER T“ flog7 (log7 = x
dx d (log7 x) (log7 x) loge 7 a: log? 7
1

Flo
ee
x (log7 x) (log, 7)2

Fr
1
(iii) Let y = log* 2. Then, y =
log2x or log,, b
ur
Putting u = log2 x, we get
f
ks

y = - and u = log2 x
Yo
oo

dl - 1 du 1
--2 and —
eB

du u dx x loge 2
dy _ dy du _ 1 1 1 1
Now,
r

[••• W = log 2 x]
dx du dx
ou

u2 X loge 2 x \oge 2
ad

(!og2 x)2
Y

ALITER Using chain rule, we obtain


nd

1 d 1
Re

^-(logl2) = 4- 1 1
x T“ (log2 x) = -----------_ x-----------
dx dx log2 x J d (log2 x) ^ log2 x dx (log2 x)z X log,, 2
Fi

EXAMPLE 6 Differentiate the following functions with respect to x:


/ \ 2
(i) sec (log xn) (ii) log tan ~ ^ [CBSE 2002] (iii) log - sin —---- 1
\ 3

SOLUTION (i) Let y = sec (log xn). Putting xn = v, u = log xn = log v, we get
y -sec u, u =log v and v = xn
dy , du 1 , dv n-l
— =sec u tan u, — = - and — = nx
du dv v dx
dy dy du dv
Now,
dx du dv dx
1
dy = secu tan ux -xnxn-l
=>
dx V

ReadYourFlow.COM
DIFFERENTIATION 11.25

n-1
=> — = sec (log x") tan (log x") x x nx - x sec (log x") tan (log xn)
dx x

(ii) Let y = log tan — + - . Putting - + — = u, tan - + —


- = tan n = u, we get
14 2^ 4 2 1^4
v4 2
y = log u, u = tan v and v ~ ^ + ^

dy _ 1 du
2 , du 1
= sec v and — =
du u dv dx 2
dy dy du dv
Now,
dx du dv dx
dy _ 1 2 1 1 2 1
-x sec vx — = sec u x — [y = tan v]
dx u 2 tan v 2

w
rfy = 1 1 1 1 n x
=> = sec x. v = —I—
dx 2 sin u cos v sin 2 u cos X 4 2
sin - + x

Flo
2

ee
2
(iii) Let y = log 1 sin------ 1

Fr
3
1 or
ur
2 2 ^ 2
X X
sf
Putting —r - -1 = u, sin------ 1 = sin v = u and log -! sin 1------ 1 v = log u = z, we get
3 3 3
k
Yo

2
oo

y = ^Jz, z = log u, u = sin v and v - ----- 1


B

3
re

= 1 dz 1 „„
du , dv 2x
— = cos v and — = —
2 -Jz du u ' dv dx 3
ou
ad

dy dy dz du dv
Y

Now, — = x—x—x —
dx dz du dv dx
nd
Re

dy = 1 1 2x X cos V
=> (cos v) - —X [v 2 = log u]
Fi

dx 2 yfz u 3J 3 u v- log u

2 .(x2 .
cos --—i x cot -1
3 3
dlJ x
=> — — X
dx 3 r 2
sin * -1 log sin------ 1
x2
3 log 1 sin
x2 -1 1
3 3 \ 3

EXAMPLE 7 Differentiate the following functions with respect to x:

(i) log ( x + Ja2 + x2 [CBSE 2003, NCERT EXEMPLAR] (ii) log a + b sin x
a - b sin x
-x
ex + e 1 + sin x
(iii) (iv) log
-V -x 1 - sin x
e -e

ReadYourFlow.COM
11.26 MATHEMATICS-XII

SOLUTION (i) Let y = log x + yja2 + x21 Then,

dy _ d
log (x +
dx dx
dy _ 1 d
X ----- x+
dx X + a2 + x2 d*
dy _ 1
=>
dx 2
r{i+!<*2+*2)2s-1/2
) x—d 2 , 2
dx
(a + x ) n

x+ a +x
I 2 2
dy_ 1 1 1 a +x +x 1
=> 1+ x 2x
dx X + a2+x2
2^/777 X+ ',2 + x2 a +x
2 2 2
a +x
2

low
a + b sin x
(ii) Let y = log • .Then,
a - b sin x
y = log (a + fr sin x) - log (a -b sin x)

ee
rF
fy. = jL {i0g (a + b sin x)} - — (log (a -b sin x))

Fr
=>
dx dx dx
it = 1 d 1 d
=> x — (a + b sin x) - x —— (a —b sin x)
for
dx a + b sin x dx a -b sin x dx
ou
^ = ---- ttt----^(a d + bsin x)- 1 d
=>
ks

x —— (a b sin x)
ax a + b sin x ax a - b sin x dx
oo

it = 1 1
(0 + b cos x) - (0 - b cos x)
Y
B

dx a + b sin x a-b sin x


re

dy = b cos x b cos x 1 1
=> + = b cos x •
ou

dx a + b sin x a-b sin x a + b sin x a - b sin x


ad
Y

=> = b cos x • a -b sin x + a + b sin x lab cos x


dx (a + b sin x) (a -b sin x) 2,2.2
d

a -b sin x
Re
n

x -X
e +e
Fi

(iii) Let y = - — .Then,


e -e

iL = A ex + e x
dx dx ex -e - x

(ex - e~ x) ^ (ex + e-x)-(ex + e-x)^-(ex-e~x)


dy _ dx dx
dx (ex-e-x)2
dy _ (ex - e~ x) (ex -e~x)~ (ex + g~ x) (ex + e~ x) d d
=> vA(e-x)=e-xA(-x) = -e -x
dx (ex-e-x)2 dx dx
dy _ (ex-e~x)2 ~(ex + e~x)2 4
=>
dx (ex-e-x)2 z x
(e -e
-Xs2
)

ReadYourFlow.COM
DIFFERENTIATION 11.27

1 + sin x
(iv) Let y = log . Then,
1 - sin x
1/2
1 + sin x 1 + sin x
y = log ^
- sin x
l2 log < 1 - sin x
> = - {log (1 + sin x) - log (1 - sin x)}
2

- -\ — d
{log (1 + sin x)} - — {log (1 - sin x)}
dx 2 ) dx dx

rfy _ 1 1 d 1 d
=> x — (1 + sin x) - x — (1 - sin x) •
dx 2 1 + sin x dx 1 - sin x dx

=> rfy = I. 1
(0 + cos x)------- - (0 - cos x) •
dx 2 1 + sin x 1 - sin x

w
dy _ 1 cos x cos x
=> +
dx 2 1 + sin x 1 - sin x

dy = 1 1 - sin x + 1 + sin x

Flo
1 2 cos x

ee
— COS X < = - COS X = sec x.
2 1 - sin2 x 2
dx 2 1 - sin 2 x COS X

Fr
dy
EXAMPLE 8 find -2. / when
dx for
ur
ax ex + log x
(i) y = e cos (bx + c) (ii) y =
sin 3x
ks

2
Yo

sm x + x
oo

(iii) y = ex log (1 + x2) (iv) y =


cot 2x
B

SOLUTION (i) Using product rule, we get


re

dy = eax d d . ax\
x — {cos (bx + c)} + cos (bx + c) x (e )
dx
ou

dx dx
ad

= d d
Y

x - sin (bx + c)x (bx + c) + cos (bx + c) x eax x — (ax)}


dx dx dx
nd
Re

=> = eax x {- sin (bx + c)} xb + cos (bx + c) x eax x a


dx
Fi

= eax {-b sin (bx + c) + a cos (bx + c)}.


dx
(ii) Using quotient rule, we get
d Y \ d
sin 3x x — (e + log x) - (e' + log x) x (sin 3x)
dy = dx dx
dx (sin 3x)2
■ x
sin 3x e* + —
1 - (ex + log x) (cos 3x) (3x)
dy = x dx
=> 2
dx sin 3x
1
sin 3x ex + - (ex + log x) (cos 3x) x 3
dy _ x
2
dx sin 3x
dy _ (ex + 1/x) sin 3x - 3 (ex + log x) cos 3x
=>
dx sin2 3x

ReadYourFlow.COM
11.28 MATHEMATICS-XII

(iii) Using product rule, we get


{log (1 + X2)} + log (1 +X2)x-^- (ex)
dx ax dx
1
=> -------T x — + x2) + log (1 + x2) x ex
dx 1 + X2 dx
x
=> ~ j x 2a: + x log (1 + x2) = ex < 2x 2 + lo8 (! + ^2)
dx i + *2 1+x
(iv) We have,
sin x + x2 2
y= = (sin x + x ) tan 2x
cot 2x

w
Using product rule, we get
= (sin x + x2)-^- (tan 2x) + tan 2x — (sin x + x2)
ax dx dx
=> ^ = (sin x + x2) (sec2 2*)

Flo
(2x) + (tan 2x) (cos a: + 2x)

ree
dx dx

F
=> ^ = (sin x + x2) (sec2 2x) x 2 + (tan 2x) (cos x + 2x)
or
ur
=> dy = 2 (sin x + x 2 ) sec2 2x + (cos x + 2x) tan 2x.
f
dx
ks
Yo

Jx2 + a21 , then move that — =


oo

EXAMPLE 9 Ify = \x +
v J dx
B
re

SOLUTION We have, y [CBSE 2005]


ou
ad
Y
nd
Re

dy =njx+-Jx2 + fl21 x£[x+A/777j


Fi

=>
dx
n- 1
dy d d
=> = n{x + x2+a2 x —(x) + ^-
dx dx dx

dy = n\x + Jx2 + a2
=>
dx
X{l + l(S + a>)-U>xA.(x>+A

=> fty _ n jx + -Jx2 + a2 | x


1+
1
x 2x
dx
2^/777
M-l
=> dy - n | x + ^x2 + a2 | x
1+
dx

ReadYourFlow.COM
DIFFERENTIATION 11.29

H | X + -Jx2 + fl2 |
n-\ yjx2 + a2 + x
dy
=> -J- = n x+
P2 + a 2
dx x2 + a2 Jx2 + a2 ^x
. -1 -1
EXAMPLE 10 Ify =
x sm Y + log - x2, then prove that rfy = sin x
2,3/2'
dx (1-x")
V1
SOLUTION We have,
2,-l/2 + ^log (1 -x2)
y =x sin-1 x(1 -x")

Differentiating with respect to x, we get


dy = . -1 2,- 1/2 1 d
x sm x(l - X ) + 2 dx
| log (1-x2)}

w
dx dx 1
=> ^ = sin-1 x (1 - x2)_1/2 — (x) + x — (sin-1 x) (1 - x2)-1/2
rfx dx dx

Flo
+ x sin-1 x(1 -x2)-1/2 +4X 1
2
d
dx
2,
— (1 “ X )

ee
x

Fr
1 1 3/2 d (1
=> x1+xx X 2)
Jl-X2 dx
or
ur
f
ks
Yo

-1 x (sin 1 x)
oo

dy _ sin x x
=>
B

dx i-*2 ^ — x2)3/2
v1 -j2
re

-1
x 2 sm
. -1
dy _ sin x x
=>
ou
ad

dx /l- *2 2,3/2
d-x")
Y

EXAMPLE 11 Differentiate the following functions with respect to x:


nd
Re

, . -1 a:),2 cos -1
-1 (sin
Fi

(i) sin (m sin x) (ii) a (hi) e


SOLUTION (i) We have,
i/ = sin (w sin-1 x)
dy = | sin (m sin 1 x)|
dx
=> — = cos (m sin 1 x) x — (m sin 1 x)
dx dx
=> dy = cos (m sin -i x) x m — (sin 1 x)
dx dx
dy = cos (m sin 1 x) x m x w -i
=> cos (m sin x)
t/x

(ii) We have.
,(sm
. -1 x),2
y=a

ReadYourFlow.COM
11.30 MATHEMATICS-XII

= ±\ a(sin 1 x)2
dx dx
dy (sin-^ x)2 . d . . k2
~ = av log a x — ^ (sin x)
dx dx
dy = ^(sin-1 x)2 log <? x 2 (sin 1 x)2 1 d , . -i .
=> x — (sin x)
dx dx
dy = fl(sin_1x)2 1 2 log a ■ sin-1 x (sin 1 x)2
=> log « x 2 sin -1 x: x xa
dx

(iii) We have,

y=e

w
dy_ _ d_

Flo
F ree
or
ur
f
ks
Yo
oo
B
re
ou
ad
Y

EXAMPLE 12 Differentiate the following functions with respect to x:


nd
Re

3 - x 2 + (3 -x2)5
(i) log10 a: + logx 10 + log* X + log10 10 (ii) 5
Fi

SOLUTION (i) Let y = log10 x + log* 10 + log* x + log10 10. Then,


1 + 1 + 1 = log10 x + (log10 x) 1 + 2.
y = log10 x +
logl0 ^
Differentiating with respect to x, we get
dy = 1 + (-1) (log10 x) 2 x~ (log10 x) + 0
dx x loge 10 dx
dy = 1 "(ioglO *) 2 x 1
=>
dx x log^ 10 x loge 10
1
dy = ___________________
1 1
=>
dx ^logglO x (log10 x)2 loge10

*1 = 1 1
=> logg 10
dx x logg 10 X (logic x iogg llJ)2

ReadYourFlow.COM
DIFFERENTIATION 11.31

=>
fty = 1 1
2 loge 10-
dx x logg 10 x (log,, x)

3 - a: 2 ,0 2s5
(ii) Let y =5 + (3-a: ) .

Differentiating with respect to x, we get


^ = A(5 3 - x2,) + ^ d
(3-^)
2v5
dx dx dx .
2 2^5-1 x
— = 5
3-x loge5x—(3-x 2 )+5(3-x ) (3-x2)
dx dx

=> 4/ =5 3o - x2 loge 5 x (0 - 2x) + 5 (3 - x2)4 x (0 - 2x)


dx
dy_ = 2
=> -2x^5 3 - x logt, 5 + 5(3 -x2)4 • .

w
dx

«2+x2 + a2-x2 , rfy 2a2 2

Flo
a
EXAMPLE 13 I/y = , show that — - --------- 3“ 1 +
2 2 a2-*2 X 4 4
a +x a -x

ee
Fr
SOLUTION We have.
,«2 + x2+J«2-x2 a2+x2+Ja2-x2 '.2+*2 + a2 - x2
for
ur
y = l 2 2 i 2 2 I 2 2 a2-x2 a2 + x2 + Ja2 - x2
a +x a -x a +x
ks

2
Yo

' 2 2
oo

+ fl -X 2 2 2 - x2 + 2 ija2 + x2 Jo2 2
a +x +a -x
B

=> y =
(a2 +x2)-{a2 -x2) 2x2
re

2«2 + 2-^fl4 -X 4 2 1 4 4
a fl -X 2 -2 -2
ou
ad

=> y = fl X + x
2 2
2x x
Y

rfy 2d. -2, ^


-Z- = a — (x ) + —
nd
Re

dx dx dx
Fi

=> iL
j = - 2a2 x 3 + (- 2) x -3 , -2X 1 , 4 4x-l/2 ^ , 4
+ (x )-(fl -x )
4,
x—(« -X )
dx 2 rfx
dy la2 2 3
=>
3 3
= (_4x )
rfx x x 2 4 4
2x a -x
2
dy _ 2a 2 2x
=>
dx 3 3 4 4
x x a -x

i 4 4
dy la2 2 a -x x
=>
3 3
+
X X 4 4
fl -X

dy _ la2 4 4 4
-X + X
=>
dx 4-2-
x3 1 4
x 3 a -x
4

ReadYourFlow.COM
11.32 MATHEMATICS-XII

dy la2 t 4 2a1 2
2a a
=>
dx 3 3 4 4
X X a -x X yja4 -X4

1 -x prove that (1 - x2) ^-+ y = 0.


EXAMPLE 14 Ify = [CBSE 2004]
1+x' dx
1 -x
SOLUTION We have, y =
1+x
Differentiating with respect to x, we get
U/2-l
dy _1 1 - x d \ 1 -x
X
dx 2 1 + x dx 1+x

w
(l + x)-^(l-x)-(l-x)4-tt+x)
rfy =I 1+xx dx dx
dx 2 y 1 -x (1 + x)2

Flo
ee
= 1 ll + x x (l + x)(-l) -(1 -x)(l)
=>
dx 2 \ 1 -x

Fr
(1 + x)2

dy 1 11 + x -1 - x -1 + x
=>
for
ur
dx 2 VI-x x------------2----
(1 + x)2
ks

= 1+x 1
=>
Yo

*----------T
oo

dx 1 -x d + .t)2
B

2, dy 1+x 1 2(1-^)
=> (1 -x )-2- =
re

dx 1 -* (1+x)
ou
ad

1 -x => (1-x2)-^ = -y => (1 - x2) — + y = 0


=>
Y

dx 1+x dx dx
nd
Re

X
1 +e dy e
EXAMPLE 15 If y = x' show that ---- =
Fi

1-e dx (l-ex) f-e2x


l + ex
SOLUTION We have, y =
1 -ex
Differentiating both sides with respect to x, we obtain.
xY"2-1
dy _ 1 1 + e d l + ex
Tx~ 2 ^Tex dx i-ex

x (i-ex)^-(l + ex)-(l + ex)--(l-ex)


d1 = \ l~e_x dx dx
=>
dx 2]i + ex Xx2
(1-e )

dy _ 1 1-e ^ (1 - ex) ex + (1 + ex) ex


(l-ex)2
L2 \llM!
+ ex x
2ex
(l-eA)
x,2
dx 2 V1 + e*

ReadYourFlow.COM
DIFFERENTIATION 11.33

x
1 e
=>
-ox,2 /!T~ xX\3/2
yjl + e (1 -e )
X

=>
dx f77^?(l-ex) (l-ex)^l-e2x

EXAMPLE 16 //(l + *)” = C0 + C1 x + C2 x2 +... + C„ x", wsmg derivatives prove that


(i) Cj + 2C2 + ... + nCn = n.2" -1 (ii) C] — 2 C2 + 3 C3 + ... + 1 « C„ = 0
SOLUTION We have,
(1 + x) = Cq + Ci x + C2 x + ... + CH x
Differentiating both sides with respect to x, we get

w
n (1 + x) = C-j + 2 C2 x + 3 C3 x +... + n Cn x
Putting x = 1 and -1 successively, we get

Flo
C1 + 2C2 + 3C3 +... + hCn = ti.2"-1 and, C! -2C2 + 3C3 +... + (-1)”'17iC„ = 0

ee
EXAMPLE17 Using thefact: sin (A + B)=sm A cos B + cos A sin Band the technique ofdifferentia-

Fr
tion, obtain the sum formula for cosines.
SOLUTION We have, for
ur
sin (A + B) = sin ,4 cos B + cos A sin B
Taking B as a constant, A as a variable and differentiating both sides with respect to A, we get
ks

d
Yo

(sin (A + B)) = cos B — (sin A) + sin B (cos A)


oo

dA dA dA
B

=> cos (A + B) = cos B cos A - sin B sin A or, cos (A + B) = cos A cosB - sin A sin B.
re

x +1
EXAMPLE 18 Iffix) = \lx2+l, g{x) = andh(x) = 2x - 3, thenfind f (h'(g'{x)).
x2+l
ou
ad

SOLUTION We have. [CBSE 2015]


Y

f(x) = ylx2+l,g(x) = X +1 and /i(x) = 2x-3


x2+l
d
Re
n

1 - 2x - x2
Fi

f'(x)=-]=L=,g'(x) = — and h'(x) = 2 for all x e R


Vx2+1 (x2+ir
Now,
h'(x) = 2 for all x e R
=> h' (*)) = 2 for all x e R

f'(2) for all x e R

=> 4= for all x e R /'W--T#— •■•/’(2)=4


2 + l=V5 v5
Vx2+1

11.4.1 DIFFERENTIATION OF INVERSE TRIGONOMETRIC FUNCTIONS BY CHAIN RULE


In section 11.3, we have obtained the derivative of inverse trigonometric functions from first
principles. In this section, we will obtain the same by using chain rule.

ReadYourFlow.COM
11.34 MATHEMATICS-XII

THEOREM 1 Ifx e (-1,1), then the differentiation of sin -1 x with respect to x is 1

i.e.. — (sin- 1 x) =
dx -/l
1
= , for ^€(-1,1). -J1 -*2
PROOF Let y = sin 1 x. Then,
sin (sin 1 x) =x => sm y = x
Differentiating both sides with respect to x, we get
-7- (siny)
dx
— (sin y)x — [By chain rule]
dy dx
cosy
dy

w
dx
dy _ 1
dx cos y
dy _ 1

Flo
ee
Fr
for
ur
ks
Yo
oo

-1
THEOREM 2 Ifx e (-1,1), then the differentiation of cos -1 x with respect to x is
B

i.e.. — (cos 1 x) = -1 for x e(-l, 1).


re

dx
-Jl -*2
ou
ad

PROOF Let y = cos 1 x. Then,


Y

cos (cos- 1 x) = x
nd
Re

=> cos y-x

i
Fi

=> [Differentiating both sides with respect to x]

=> -y- (cos y) x ^ = 1


[By chain rule]
dy dx
-sm y — =1
y dx
=> dy _ -1
dx sin y
dy_ -1
=>
dx yl - cos2 y

dy = -1
=> -1 <x<l [•.■ a: = cos y]
dx

or. —d /(cos -I x)\ = —7=1 -1 < X < 1


dx

ReadYourFlow.COM
DIFFERENTIATION 11.35

THEOREM 3 The differentiation o/tan~1 x with respect to x is 1


i^2'
i.e..
j- (tan-1 x)=-^—2
dx i+
PROOF Let y = tan - 1 x. Then,
tan (tan- 1 x) = x
tan y = x
=> y- (tan y)=-^-(x) [Differentiating both sides with respect to x]
dx dx
=> — (tany)x^ = 1 [By chain rule]
dyV * dx
2 dy
=> sec y -f-=l
dx
dy ___ 1

w
dx sec-2 y
1
dy=______

Flo
=>
dx 1 + tan2 y

ee
or. -^-(tan- 1 x) =— [•.• y = tan 1 x and tan y = x]

Fr
dx l+x2-
l -1
THEOREM 4 The differentiation of cot x with respect to x is for
ur
1 + x2
i.e.. — (cot 1 x) = -1
dx l + x2
ks
Yo

PROOF Let y = cot- 1 x. Then,


oo

cot (cot 1 x) = X
B

cot y = x
re

=> y- (c°t y)= y- (x) [Differentiating both sides with respect to x]


ou
ad

dx dx
— (coty)x^=l
Y

[Using chain rule]


dy dx
d
Re

- cosec 2 y x -y
dy = 1
n

dx
Fi

=> dy = -1
dx cosec2 y
dy = -1
=>
dx i + cot2 y

or. — (cot-1 x) = -1 [v y = cot 1 x and x = cot y]


dx l + x2
1
THEOREM 5 If x e R - [-1,1], then the differentiation of sec~1 x with respect to x is
d , -i . 1
i.e.. — (sec x) = , x eR -[-1,1].
dx

PROOF Let y = sec 1 x. Then,


sec (sec- 1 x) = x
=> sec y = x

ReadYourFlow.COM
11.36 MATHEMATICS-XII

=> — (sec y) = — (x) [Differentiating both sides with respect to x]


dx * dx
=> — (sec y) x — = 1 [Using chain rule]
dy dx
dy
=> sec y tan y — = 1
dy = 1
=>
dx sec y tan y
If a: > 1, then y e (0, tc/2)
/. sec y > 0, tan y > 0 =^> | secy | | tan y | = secy tan y
dy = 1
=> if a: < -1, then y (71/2, k)
dx | sec y | | tan y |
sec y < 0, tan y < 0
=> | sec y | | tan y | = (- sec y) (- tan y) = sec y tan y

w
dV 1
=>
dx I secy| ^/tan2 y

=>
dy 1

Flo
ee
dx | sec y | sec2 y -1

Fr
— (sec- 1 a:) = 1
or.
dx
1
for
ur
THEOREM 6 //.t eft -[-1,1], f/7cn the differentiation of cosec 1 a: with respect to x is -1
ks

, -1 \ 1
i.e.. — (cosec x) =
Yo

dx
oo
B

PROOF Let y = cosec" 1 x. Then,


re

cosec (cosec- 1 a:) = x


ou
ad

cosec y = a:
Y

=> -j- (cosec y) =~ (a^) [Differentiating both sides with respect to x]


ax dx
-7- (cosec y) x ^ = 1
nd
Re

=> [Using chain rule]


rfy rfx
Fi

=> - cosec y cot y x =1


dx
dy -1
rfx cosec y cot y
If x > 1, then y (0, k/2)
=> cosec y > 0, cot y > 0 => | cosec y 11 cot y | = cosec y cot y
rfy -1
If x < -1, then y e (- 7t/2,0)
dx | cosec y 11 cot y |
cosec y < 0 and cot y < 0
=> | cosec y 11 cot y | = (- cosec y) (- cot y)
dy -1
=>
| cosec y | ^cosec2 y -1

— (cosec- 1 x) = 1
or.
dx

ReadYourFlow.COM
DIFFERENTIATION 11.37

EXERCISE 11-2
LEVEL-1
Differentiate thefollowing functions with respect to x (1-57):
1. sin (3x + 5) 2. tan2 x ^ tan {x° + 45°)
sin -/x x
4. sin (log x) 5. e
7. sin2 (2x +1) 8. log7 (2x - 3) ^ tan 5x°
3
10. 2X 11. 3
2 2
x2 + 2x a -x
13. 3 14. 2 2
a +x

1 + sin x 18/flog sin x)2

w
16.
1 - sin x

Flo
1+x /g3x cos 2x
19.
1 -x

ee
Fr
tan 3 x
22. sin (log sin x) 23. e
sin x 1 - cos x
for
ur
25. log 26. log [CBSE 20031
1 + COS X 1 + COS X
ks

ex log x
Yo

27. tan (esin x) 28. 29. 2


oo

X
B

^ 2 ^
e 2x +. e - 2x X +X + 1
36*4 log (cosec x - cot x) 31. 32. log
re

e 2x -e - 2x X —X+1
ou

sin -1 2x
ad

33. tan 1 (e*) -1


34. e 35. sin (2 sin x)
Y

-1
rx 37. Jtan
-1 x 38. log (tan 1 x)
d
Re
n
Fi

2X cos x ^^O^x sin 2x + 5X + kk + (tan2 x)3


—2-------2
(x2 + 3)2
3x2 sin x
41. log (3 x + 2) -x2 log (2x -1) 42.
^7-jc2

/2 +1+ f2-1
if. sin2 [log (2x + 3)} 44. ex log sin 2x 45.
-Jx2 +1 ~yjx2 -1

■^x2 + 4x +1 . -1 X
47. (sin 1 x4)4 48. sin

x2 +2
50. 3 e 3x log (1 + x) 51.
(x2 + 2)3 ^cos X

ReadYourFlow.COM
11.38
MATHEMATICS-XII

i nl x
53. log J cot - +- 54. eax sec x tan 2x
cos 2x

55. log (cos x2) 56. cos (log A')2 x-1


57. log
x+1
-1
log jv*-! yjx + 11 , show that =
2^
1

59. If y = yjx + 1 + tJx -l, prove that ^jx2 -1 = ~ 1/

60. If y = , prove that x = (1 - y) y


^ 2 dx

^6?. If y = log V^ + 4 , prove that — = X ^

low
dx 2x (a + 1)
1 + tan x dy
62. If y = log prove that— = sec 2a. [CBSE2011]
1 - tan a dx

ee
1
63. if y = V-k + -?= , prove that 2 a — = V* - 4=
rF
Fr
4~x dx
-1
a sin x 2 y
for
Ify = -, prove that (1 - a )----- = A + —.
2 dx A
u
ks

A
-X
65.ify=^ - e — , prove that — = 1 - y2
Yo
oo

e +e x dx
B

66. If y = (a -1) log (a -1) - (a + 1) log (a + 1), prove that ^ = log x-1
re

dx 1+A
ou
ad

n
67. If y = eX cos a, prove that — = ^ 2 cos a + -
Y

dx 4
nd

1 1 - cos 2a
Re

dy
68. if y = - log prove that = 2 cosec 2a
1 + cos 2a
Fi

69. If y = a sin 1 a + Jl - a2 , prove that — = sin -1 a


v rfA
If y = -Ja2 + a2, prove that y ^ — A = 0

71. If y = +e~x, prove that ^ = ^y2 -4

72. Ify = 4? a2 ,
prove that y — + A = 0
dx
/ . \
73. If -vy = 4, prove that a — + y2 = 3 y
dx
2
d x I 2 2 a . -1 A l 2 2
74. Prove that — < — J a - x + — sin — a -x [CBSE2004]
dx 2 * 2

ReadYourFlow.COM
DIFFERENTIATION 11.39

ANSWERS

1. 3cos(3x+5) 2. 2 tan x sec2 x 3. — sec2 (x0 + 45°)


180
^esin^
cos tan x 2
4. — cos (log x) 5. 6. e sec x
X 2s[x
2 9. sec2 (5x°)
7. 2 sin (4x + 2) 8.
(2x - 3) log^ 7 180

11. 3e* log 3 • ex 1


10. 3x2-2x log 2 12. -
x l°ge 3 (log 3 x)2

x2 + 2x - 2fl2 x
13. (3 log 3) (2x + 2) 14. 15. 3xlogx (log 3) (1 + log x)
^~7(a2+x2)3/2

w
- 2x
16. sec x (tan x + sec x) 17. 18. 2 (log sin x) cot x
yji^a+x2)3'2

Flo l+x2

ee
1 4x 21. e3x (3 cos 2x - 2 sin 2x)
19. 20. cos
3/2 2,2

Fr
y/l+Xil -X) 1-x2
(1 - x )
for Jcotx
1 ^ 2
ur
22. cos (log sin x) • cot x 23. 3 ctan 3;c ■ sec2 3x 24. - x cosec x
2 ^cot x
ks

25. cosec x 26. cosecx 27. sec2 {e sin x ) e sin x cos x


Yo
oo

28.
1 29. x 2 [ log x + — ^ logxj
B

x x
re

-8 2 (x2 -1)
ou

32. -
ad

30. cosec x 31. -2xa


2x x4 + x2 + 1
{e - e )
Y

sin -l 2x
X
e 2 -1 2
33. 34. e 35. cos (2 sin x) ■
d
Re

\+e2x
n
Fi

tan- 4
e 1 1
36. 37. 38.
2-Jx (l + x) (1 + x2) tan 1 x
(4 + x2) tan -1 x
2

2X 4x cos x
39. cos x • loge 2 - sin x -
(x2 + 3)2 x2 + 3

/ x 5 2
40. sin 2x + 2x cos 2x + 5 log 5 + 6 tan x sec x
2
3 2x2 6x sin x + 3x cos x 3x3 sin x
41. - 2x log (2x -1) 42.
3x + 2 2x -1 ^7 2,3/2
(7 - x )

2
sin [2 log (2x + 3)} • 44. 2 ex cot 2x + ex log sin 2x
2x+ 3

ReadYourFlow.COM
11.40 MATHEMATICS-XII

2* 3 1
45. 2x + 46.
^ "I ^x2 + 4x + 1

^ -i16x
s 3 , ■ -1 4n 3
(sin x )
48.
2 2
-Jl -^8 a +x

ex sin ^ + i,'x cos x 6x ex sin a: -3 a-


49. 50. 3 c - 3 log (a: +1) •
(x2 + 2)3 (x2 + 2)4 x +1

1 x2 + 2
51. 2x + tan x 52. 2x (1 - x2)2 sec 2x {1 - 4x2 + x (1 - x2) tan 2x}
^cos x 2

53. - sec x 54. eax sec x [a tan 2x + tan x tan 2x + 2 sec2 2x)

w
56 - 2 log x sin (log x)2 1
55. - 2x tan x2 57.

Flo x x2-l

ee
11.5 DIFFERENTIATION BY USING TRIGONOMETRICAL SUBSTITUTIONS

Fr
Sometimes, it becomes very easy to differentiate a function by using trigonometrical
transformations. Usually this is done in case of inverse trigonometrical functions. Some
important results on trigonometrical and inverse trigonometrical functions are given below for
for
ur
ready reference.
(i) sin 2x = 2 sin x cos x (ii) 1 + cos 2x = 2 cos2 x or, cos 2x = 2 cos2 x -1
ks
Yo

2 2
oo

(iii) 1 - cos 2x = 2 sin x or, cos 2x = 1 - 2 sin x


B

2 tan x 1 - tan 2 x
(iv) sin 2x = (v) cos 2x =
re

2
1 + tan2 x 1 + tan x
ou

2 tan x
ad

3
(vi) tan 2x = (vii) sin 3x = 3 sin x - 4 sin x
1 - tan 2 x
Y

3
3 tan x - tan x
nd
Re

(viii) cos 3x = 4 cos3 x - 3 cos x (ix) tan 3x =


1-3 tan 2 x
Fi

(x) sin -1

sm x + sin- 1 y , if -1 < x, y < 1 and x2 + y2 < 1 or, if xy < 0 and x2 + y2 > 1


tc - (sin- 1 x + sin- 1 y) , if 0 < x, y < 1 and x2 + y2 > 1

- re - (sin -1 x + sin- 1 y) , if -1 < x, y < 0 and x2 + y2 > 1

sin

sin 1 x - sin 1 y , if -1 < x, y < 1 and x2 + y2 < 1 or, if xy > 0 & x2 + y2 > 1
= re - (sin - 1 x - sin- 1 y) ,if0<x<l,-l<y<0 and x2 + y2 > 1
— it — (sin - 1 x - sin -1 y) , if -1 < x<0, 0<y <1 and x2 + y2 >1

ReadYourFlow.COM
DIFFERENTIATION 11.41

(xi) cos 1 *y-


-1 -1
cos x + cos y , if -1 < x, y < 1 and x + y > 0
2tc - (cos 11 x + cos 1 y) , if -1 < x, y < 1 and x + y < 0

-1
cos xy +

-1 -1
cos x - cos y , if -1 < x, y < 1 and x < y
- (cos J x - cos 1 y) , if -1 < y < 0, 0 < x < 1 and x > y

-1 -1
tan x + tan y / if ^y < i
l x+y

w
-i x + tan 1 y) , if x > 0 , y > 0 and xy > 1
(xii) tan re - (tan
v1 - xy j if x < 0, y < 0 and xy > 1
re + (tan - 1 x + tan- 1 y) ,

tan

Flo
-1 x - tan 1 y , if xy > -1

ree
(xiii) tan 1 x~y = < 71 - (tan- 1 x - tan" 1 y) , if x > 0 , y < 0 and xy < -1

F
f+xyj if x < 0, y > 0 and xy < -1
— tc — (tan 1x-tan 1 y) , for
ur
-1 -1
(xiv) sin ~ x + cos x = —, if -1 <x<l
ks

2
Yo

-l -l
oo

(xv) tan " x + cot x = - for all x e R


2
B

-1 -1 x = — , if x e(-oo , -1] u [1 , co)


(xvi) sec ~ x + cosec
re

2
-l -1
ou

(xvii) cos 1 {- x) = 7T - cos " x for x e [-1,1]


ad
Y

(xviii) tan 1 (- x) = - tan 1 x for x e R

(xix) sin- (- x) = - sin x for x e [ -1,1]


nd
Re

-1 -ifl
Fi

(xx) sin x = cosec , if x e ( - oo , -1] u [1, co)


x
(xxi) cos 1 x = sec 1 — , if x e (- oo , -1] u [1 , oo)
x

cot -1 f , if x > 0
-l x)
(xxii) tan x =<
- tc + cot
-in , if x < 0
X

(xxiii) sin 1 (sin 0) = 0 , if - -<e<- -1


— ; cos (cos 0) = 0,ifO<0<7c
2 2 '
1 TC n TC 7C A 7C
tan - < 0 <- ; cosec 1(cosec 0) = 0 , if - - <0 <-
(tan 0) = 0 / if - ^ -,0^0
2 2 2
-1
sec (sec 0) = 0,ifO<0<Tc/0^^ ; cot- 1 (cot 0) = 0/ifO<0<rc

ReadYourFlow.COM
11.42 MATHEMATICS-XII

Following are some substitutions useful in finding derivatives:


Expression Substitution
a2 + x 2 x = a tan 0 or, a cot 0
2 2
a -x x = a sin 0 or, a cos 0
2 2
x -a x = a sec 0 or, a cosec 0
a-x a+x
or. x=a cos 20
a+x a-x

a 2 -x 2 a 2 +x 2
x1 - a2 cos 20
a 2 + x 2 °r' 2 2
a -x

w
ILLUSTRATIVE EXAMPLES

LEVEL-1

EXAMPLE 1 Differentiate the following functions with respect to x:


Flo
ree
(i) sin - 1 (sin x), x e [0,2 n] (ii) cos 1 (cos x), x e [0, 2 n]

F
(iii) tan- 1 (tan x), x e [0, tc] -1 —
or
ur
2
sf
SOLUTION (i) Leti/ = sin 1 (sin x). Then,
k
Yo
oo

x , if x e 0, —
2
B
re

3 K
y = sin 1 (sin x) = k-x , if x e -
2' 2
ou
ad
Y

3n
-2n + x , ifxe ----, 2 n
2
nd
Re

We observe that
Fi

LHDatx = - = 1 and, RHD at x = — = -1


2J 2

3 71 3n
LHD at x = — I = -1 and, RHD at x = — = 1
2 2
3K
So, y = sin 1 (sin x) is not differentiable at x = —
2' 2

1 , if x e 0, —
2

= 1 -r 7t 3 71^1
-1 , if x e — ,----
dx 2 2

3 71
1 , ifxe —,27i
2

ReadYourFlow.COM
1
DIFFERENTIATION 11.43

(ii) Let y - cos 1 (cos x). Then,


, if x e[-n, 0]
x , if xe [0,71]
y = cos 1 (cos x) = •
2 7t - x , if x e [ti, 2 tt]
— 2 7i + x , ifxe[2 7i/3 7r] and so on.

Clearly,
(LHD at x = 0) = -1 and (RHD at x = 0) = 1; (LHD at x = ti) = 1 and (RHD at x = tc) = -1
(LHD at x = 2 7t) = -1 and (RHD at x = 2 tt) = 1
-1
So, y = cos (cos x) is not differentiable at x = 0, ti, 2 n.
1 , if x e(0,7t)
Hence =
dx -1 , if x s(tc , 2 ti)

w
(iii) Let y = tan - 1 (tan x). Then,

Flo
x , if x e 0, —
2j

ee
Fr
3 Tt
y = tan 1 (tan x) = • X - TC , if x e — and so on
2 ' 2 for
ur
3 71 5 71
x-2 tc , if x e ----
2 ' 2
ks
Yo
oo

^ = l.ifxE 0,|j U TC
— , TC
B

dx 2
re

Differentiate sin 1 j^2x -Jl -x2 j with respect to x, if


ou

EXAMPLE 2
ad
Y

1 1 1
®-j2<X<j2 (ii) 4 < X <1 (iii) -1 <x<-
V5 V2
nd
Re

Let y = sin - 1 2x
Fi

SOLUTION . Putting x = sin 0 , we get

i/ = sin- 1 (2 sin 0 cos 0) = sin 1 (sin 2 0)

(i) If - -4= < x < -4=, then


V2 V2
x = sin 0 => —]= < sin 0 < -4= => <0< — => <20< —
V2 V2 4 4 2 2
y = sin 1 (sin 2 0)

=> y = 20 v -— < 2 0 < —


2 2

=> y = 2 sin- 1 x [•.• sin 0 = x => 0 = sin 1x]


dy =
dx

ReadYourFlow.COM
11.44 MATHEMATICS-XII

1
(ii) If —f= < x <1, then
V2
x = sin 0 -4= <sin0 <1 => - <0<- => —<20<ti:
V2 4 2 2
y = sin 1 (sin 2 0)
y = sin-1 (sin(7r-2 0))

=> y = 71 - 2 0 .• —<20<7r=>O<7t-20 < —


2
y = Ti - 2 sin-1 x
dy = 0 - 2 2
=>
dx
1

w
(iii) If -1 < x < —t= , then
V2
x = sin 0 => -1 < sin 0 < - -4= => -- < 0 <-— => -7t<20<--
V2

Flo
2 4 2

y = sin-1 (sin 2 0)

ee
Fr
=> y = sin-1 (-sin (tih-2 0))
y = sin-1 (sin (-71-2 0)) for
ur
=> y = - 7i - 2 0 —7t<20<—— => -— <7T+20<O
2 2
ks

— 7t — 2 sin-1 x
Yo

=> y =
oo

rfy = -0- 2 2
=>
eB

dx Jl-x2 ■Ji -^2


—1 3
r

EXAMPLE 3 Differentiate sin (3x - 4x ) with respect to x, z/


ou
ad

... 1 1
(i) — — < x < (ii) — < x < 1 (iii) -1 < x < — —
Y

2 2 2 2
SOLUTION Let y = sin 1 (3x - 4x3). Putting x = sin 0, we get
nd
Re

y = sin- 1 (3 sin 0 - 4 sin 3 0) = sin- 1 (sin 3 0)


Fi

(i) If - — < x < -, then


22
x = sin 0 => - — < sin 0 < - => -—<0 < — - - < 3 0 <-
2 2 6 6 2 2
y = sin- 1 (sin 3 0)

=> y = 30 •<30<-
2 2
y = 3 sin- 1 x
dy _ 3
dx
f-x2
(ii) If — <x <1, then
2
x = sin 0 => - <sin0<l => - <0<- => *<30<-^
2 6 2 2 2

ReadYourFlow.COM
DIFFERENTIATION 11.45

y = sin 1 (sin 3 0)

=> y = sin- 1 {sin (k - 3 0)}


3n Tt
=> y = Tt - 3 0 v-<30< - 3 0 <—
=>----- < Tt
2 2 2 2
=> y = Tt - 3 sin
-1
x [•.• x = sin 0 => 0 = sin 1x]
dy = 3 3
=> 0-
dx

(iii) If-1 <x<-- , then


2
Tt „ Tt 3 Tt
x = sin 0 => -1 < sin 0 < - -=>--<
- 0 <--=> < 3 0 <- —
2 2 6 2 2
y = sin 1 (sin 3 0)

w
3 it o it K
=> y = sin 1 {sin (- Tt - 3 0)} •-------<30 < — => — <- Tt — 3 0 < —

Flo
2 2 2 2
=> y = - Tt - 3 0

ee
=> y = - Tt - 3o sin
• x

Fr
t/y 3 3
=> = -0- for
ur
dx

-1 o
(2x -1) with respect to x, if
ks

EXAMPLE 4 Differentiate cos


Yo

(i) 0 <x <1 (ii) -1 < x < 0


oo

i 2
SOLUTION Let y = cos (2x -1). Putting x = cos 0, we get
B
re

y = cos 1 (2 cos2 0-1) = cos 1 (cos 2 0)


(i) If 0 <x <1, then
ou
ad

x = cos 0 => 0 < cos 0<1=> O<0<— => O<20<Tt


Y

2
y = cos 1 (cos 2 0)
d
Re
n

=> y - 20 [v 0 < 2 0 < Tt


Fi

=> y = 2 cos 1 x [v x = cos 0 => 0 = cos 1x]


dy 2
=>
dx ^ -x2

(ii) If-1 < x < 0, then


x = cos 0 => - 1 < COS 0<O=>— <0<Tt=> Tt<20<2Tt
2
y = cos 1 (cos 2 0)

=> y = cos 1 {cos (2 Tt - 2 0)}


=> y = 2 Tt - 2 0 [v it<20<2Tt=> O<2Tt-20<Tt]
=> y = 2 Tt - 2 cos 1 x
dy 2 2
= 0+
dx

ReadYourFlow.COM
11.46 MATHEMATICS-XM

EXAMPLE 5 Differentiate cos 1 (1 - 2x2) with respect to x, if


(i) 0 < x < 1 (ii) -1 < x < 0
-1 (1 - 2x2). Putting x = sin 0 , we get
SOLUTION Let y = cos
y = cos 1 (1 -2 sin2 0) = cos 1 (cos 2 0)
(i) If 0 < x < 1, then
x = sin 0 => 0 < sin 0<1=>O<0<— => O<20<tc
2
-1
y = cos (cos 2 0)
=> y = 20
-1 [v x = sin 0 => 0 = sin 1 x]
=> y = 2 sin x

=>

w
(ii) If -1 < x < 0, then
x = sin0=>-l < sin0<O=> -~<0<O =^-7t<20<O
2

Flo
ree
y = cos 1 (cos 2 0)

F
-1
=> y = cos- 1 (cos (- 2 0))
y = -20 or
[•.• - 7t<20<O =>O<-20<7r]
ur
-1
y = - 2 sin
f
X
ks

=>
Yo
oo
B

EXAMPLE 6 Differentiate cos 1 (4x3 - 3x) with respect to x, if


re

(i) xe -
1 f (ii) x e
1
,1 (hi) xe
ou
ad

2 ' 2 2 2
Y

SOLUTION Let y = cos 1 (4x3 - 3x). Putting x = cos 0, we get

y = cos 1 (4 cos3 0-3 cos 0) = cos 1 (cos 3 0)


nd
Re
Fi

(i) If x € ' t^en

1 1
X = cos 0 => — < cos 0 < - => — < 0 < — => 71 < 3 0 < 2 7C
2 2 3 3
y -- cos 1 (cos 3 0)

=> y = cos 1 {cos (2 7t - 3 0))


=> y = 2;t-30 [v 7i<30<2ti=> O<27t-30<7i]
=> y' =
= 2 7i - 3 cos 1 x [•.* X = cos 0 => 0 = cos 1 x]
dy
— = o-3x-
=>
dx

(ii) Ifxef^l , then


2

x = cos 0 => — < cos 0<1 => O<0< — => 0 < 3 0 < 7T
2 3

ReadYourFlow.COM
DIFFERENTIATION 11.47

y = cos 1 (cos 3 0)
=> y = 30 [v 0 < 3 0 < re]
=> y = 3 cos x [v X = cos 0 => 0 = cos 1 x]

3
=>
rfx t/i-*2

(iii) If-1 < x < — —, then


2
2 71
x = cos 0 => — 1 < cos 0 < - - => — <0<tt;=> 27r<30<37i
2 3
y = cos 1 (cos 3 0)

=> y = cos 1 {cos (2 n - 3 0)}

w
=> y = cos 1 {cos (3 0 - 2 rc)} [v 2tc<30<37t=> O<30-27r<7r]

Flo
=> y = 30-2K
=> y = 3 cos 1 x - 2 7i

ee
dy -3 -3

Fr
-0 =
dx
for
ur
EXAMPLE 7 Differentiate tan 1 2x
- zuith respect to x, if
ks

1 -x
Yo
oo

(i) -^(-1,1) (ii) x e (- oo, -1) (iii) x e(l, oo).


B

SOLUTION Let y = tan - 1 2x


Putting x = tan 0 , we get
re

1 -x2
ou
ad

y = tan 1 (tan 2 0)
Y

(i) If-1 <x <1, then


x = tan 0 => -1 < tan 0 <1=> - — < 0 < — => <20< —
nd
Re

4 4 2 2
Fi

y = tan 1 (tan 2 0) =20 , -— <2 0 <—


2 2_

=> y = 2 tan 1 x [v x = tan 0 => 0 = tan_ 1 x]


dy = 2
=>
^ l + x2
(ii) If — co<x< — 1, then
x = tan 0 - oo < tan 0 < -1 => - — < 0 < - — => — tt<20< — —
2 4 2
y = tan 1 (tan 2 0)

=> y = tan- 1 {tan (ti + 2 0)}

-1 Y -7l<20<- — => O<7t+20<-


=> y = tan {tan (tc + 2 0)}
2 2
=> y = 7i + 2 0
-1
y = 7i + 2 tan x

ReadYourFlow.COM
11.48 MATHEMATICS-XII

^ = 0+ 2 2
==>
dx 1+x2 1+x2
(iii) If x e (1, oo), then
7t _ 7t
x = tan 0 =r> 1 < tan 0 < oo — < 0 < — => — < 2 0 < 7C
4 2 2
y = tan 1 (tan 2 0)
=> y = tan 1 {-tan (ti - 2 0)}
=> y = tan 1 {tan (20-7r)}

y = 2 0 - ti — < 2 0 < k => — <20-ti<O


2 2
=> y = 2 tan -1 x - ti

low
dy = 2 2
-0 =
dx i+x2 i+^2

3x -x3
EXAMPLES Differentiate tan 1 Df

ee
1 - 3x2
rF
Fr
1 1 1
(i) ~i<x<73 [NCERT] (ii) ^>-7^ (in) x<--/=
V3 for
3x - x3
ou
SOLUTION Let y = tan 1 . Putting x = tan 0 , we get
ks

1 -3x2
oo

3 ^
3 tan 0 - tan 0
Y

-1
B

y = tan 1 = tan (tan 3 0)


2
1-3 tan 0
re

1 1
ou
ad

(i) H“73 <A;<73'then


Y

x = tan 9 => - -L < tan 9 < -L <0<— => ^ < 3 0 <—


V3 V3 6 6 2 2
nd
Re

y = tan 1 (tan 3 0)
Fi

y = 30 -— < 3 0 < —
2 2

=> y = 3 tan 1 x [v x = tan 0 => 0 = tan 1 x]


dy = 3
=>
dx i+x2
i
(ii) If x > , then
V3
x = tan 0 => tan 0 > ~ =>—<0<— => ^<30<
-J3 6 2 2 2
y = tan 1 (tan 3 0)
=> y = tan 1 {- tan (t: - 3 0)}
=> y = tan 1 {tan (3 0 - ti)}

ReadYourFlow.COM
DIFFERENTIATION 11.49

v ^<30<^ <30-7I< —
=> y = 3 0 - ji
2 2 2 2
[••• x = tan 0 => 0 = tan 1 x]
y = 3 tan 1 x - k
dy = 3 3
=> -o =
dx i+x2 1 + x2

(iii) If x < - then

TC „ 71 3 tc 0 - n
x = tan 0 => - oo < tan 0 < —7= => ---- < 0 <----- ------< 3 e < —
V3 2 6 2 2
y = tan 1 (tan 3 0)

=> y = tan 1 {tan (71+3 0)}


3 7t 7t

w
y = 7i + 3 0 -------< 3 0 < — => --<71+ 30<-
2 2 2 2

y == 7r+3 tan 1 x

Flo
=>
rfy _ 3 3
= 0+

ee
=>
dx i+,2 1 + x2

Fr
EXAMPLE 9 Differentiate sin 1
2x
- with respect to x, when or
ur
1+x
f
(i) x € (—1/1) (ii) x e(l, oo) (iii) xe (-co,-!)
ks
Yo

2x
oo

SOLUTION Let y = sin _ 1 Putting x = tan 0, we get


1 +x2
B
re

2 tan 0
y =
= sin- 1 = sin 1 (sin 20)
ou

1 + tan2 0
ad
Y

(i) If x e(-l, 1), then


TC - ^ K
x = tan 0 => -1 < tan 0<1 => - — < 0 < — => --<2 0<-
nd
Re

4 4 2 2
Fi

y = sin 1 (sin 2 0)

y = 20 -— <2 0 < —
2 2

=> i/ = 2 tan 1 x
rfy = 2
dx l + x2
(ii) If x e (1, co), then
x = tan 0 => 1 < tan 0<cor=>—<0< — —<20<7t
4 2 2
1
y = sin (sin 2 0)
-1 [v sin (ti - 2 0) = sin 2 0]
y = sin {sin (7t-2 0)}

=> y = ti-20 v - <20<ti=> O<7r-20< —


2 2

ReadYourFlow.COM
11.50 MATHEMATICS-XH

=> y = ti-2 tan 1 x [v a: = tan 0 => 0 = tan 1 x]


dy 2 2
— = 0-
dx l+x2 l + x2
(iii) If a: e (- co, -1), then
71 _ 71
x = tan 0 => - oo < tan 0 < -1 => - - < 0 <---- => — 7T<20< — —
2 4 2
y = sin- 1 (sin 2 0)
y = sin - 1 (- sin (n + 2 0))
y = sin-1 {sin (-7u-2 0))
7C
y = -ti-2 0 -ti<20<- — => — <-7t-20<O
2 2

w
-1
y = - 7i - 2 tan x a: = tan 0 => 0 = tan 1 x]
dy = 0- 2 2
dx 1 + x2 1+x2

Flo
ee
l-x2

Fr
EXAMPLE 10 Differentiate cos 1 with respect to x, when
1 + x2
or
ur
(i) x e(0, oo) (ii) x e (- oo, 0)
f
-1 f l-x22
ks

SOLUTION Let y = cos . Putting x = tan 0, we get


Yo

1 + x2
oo

\
1 1 - tan2 0
B

y = cos cos 1 (cos 2 0)


re

1 + tan2 0
ou

(i) When x e(0, oo)


ad

x = tan 0 => 0 < tan 0<co=> O<0<-^=>O<20<ti:


Y

2
y = cos- 1 (cos 2 0)
nd
Re

y = 20 [v 0 < 2 0 < tt]


Fi

=> y = 2 tan 1 x [v x = tan 0 => 0 = tan 1 x]


dy = 2
=>
dx 1+x2
(ii) When x e (- co, 0)
x = tan 0 => - oo < tan 0<O=> -—<0<O=>-tc < 2 0 < 0
2
y = cos 1 (cos 2 0)

=> y = cos 1 {cos (-2 0)) [v -7t<20<O=> O<-20<tc]


y = -20
=> V = - 2 tan 1 x
dy = 2
=>
dx i + x2

ReadYourFlow.COM
DIFFERENTIATION 11.51

EXAMPLE 11 Differentiate each of thefollowing functions with respect to x


1-x2 2 'l
1 -x
(i) cos- 1 ,0 <x <1 [NCERT] (ii) sin -1 , 0 <x <1 [NCERT]
1+x2 1+x2

(iii) cos 1 2x , —1 <x <1 [NCERT] (iv) sec-1 1 , 0 < x < i [NCERT]
1 + x2 2x2 -1 V2
, 2
-1 1-x
SOLUTION (i) Let y = cos , where 0 < x < 1.
i+^2
Putting x = tan 0, we have
-1 1 - ttan 2 ^
i
y = cos

w
1 + tan2 0

=> y = cos 1 (cos 2 0)


y = 20

Flo
v O<x<l=>O<tan0<l=>O<0< — =>O<20<-

ee
4 2
y = 2 tan -1 x

Fr
[v x = tan 0 => 0 = tan 1 x]
dy = 2
=> for
ur
dx i + x2

-ifizZ
ks

(ii) Let y = sin , where 0 < x < 1. Putting x = tan 0, we get


Yo

l + x2
oo

\
f 2
B

_1 1 - tan 0
y = sin
re

1 + tan2 0
ou
ad

=> y = sin (cos 2 0)


Y

-1
=> y = sin sin --20
2
nd
Re

y = --20 v 0 < x < 1 => 0<tan 0<1=> O<0<-=> O<20<-=> O<--20< —


Fi

2 4 2 2 2

y = - - 2 tan 1 x [y x = tan 0 => 0 = tan 1 x]


2
= 0- 2 2
=>
dx l + x2 1 +x2

2x
(iii) Let y = cos 1 , where -1 < x < 1. Putting x = tan 0 , we get
! + x2

-1 2 tan 0
y = cos
1 + tan2 0
=> y = cos 1 (sin 2 0)
-1
=> y = cos cos --20
2

ReadYourFlow.COM
11.52 MATHEMATICS-XII

=> --20 v -1 <x <1 => -1 < tan 0<1=> --<0<-=> - -<20<-=> O<--20<tc
2 4 4 2 2 2

=> y = — - 2 tan 1 x
2
-2
=_____
=>
dx i+

-1 i i
(iv) Let y = sec , where 0 < x < —?=
2x2 -1 V2
Putting x = cos 0, we have
-1 1
y = sec
2 cos2 0-1

w
y = cos- 1 (2 cos2 0-1) -1 1 -1
v sec - COS X
X

=> y = cos 1 (cos 2 0)

Flo
ree
=> y = 20 v 0 < x < -1= => 0 < cos 0 < -U =>O<0<—=>O<20<-
V2 4 2

F
=> y = 2 cos 1 x or [•.• x = cos 0 => 0 = cos 1 x]
ur
dy _ _ 2
=>
f
rfx
ks
Yo

EXAMPLE 12 Differentiate each of the following functions with respect to x:


oo

f
-1 2x{l -x 2 1 1 1 1
B

(i) sin /----- 7= < * <-?= (ii) cos- 1 2x ----- j= < X < —j=
J V2 V2 y/2
re

-1 1 1
SOLUTION (i) Lety = sin 2x , where - —j= <x < -j=. Putting x = sin 0, we get
ou
ad

V2 -V2
Y

y = sin 1 (2 sin 0 cos 0)


nd

y = sin- 1 (sin 20)


Re
Fi

=> ■ --L= < x <-^==> —< sin 0 <^==> -—<©<-=> - —<20< —


y
V2 V2 -y/2 4 4 2 2
=> [v x = sin 0 => 0 = sin- 1 x]

i i
(ii) Let y = cos , where - < x < -^=. Putting x = sin 0, we get
72
-1 71
y = cos 1 (2 sin 0 cos 0) = cos 1 (sin 2 0) = cos cos---- 2 0
2

1 1 1 1 n n
----- r= < X < -7= =>----- F=r < sin 0 < -F= => — <0 <-
=>
72 72 72 72 4 4
y = --20
2
=> - -<20<- => O<--20<k
2 2 2

ReadYourFlow.COM
DIFFERENTIATION 11.53

=> [v x = sin 0 => 0 = sin 1 x]

=>

=>

EXAMPLE 13 Differentiate the following functions with respect to x:

1 1 - cos x
(i) tan < , —K<X <K
sin x

w
-1 1 - cos x [NCERT EXEMPLAR]
(ii) tan < , -ti<x <n
1 + COS X

(iii) tan
-1
<
1 + COS X
• , 0 < X < rc

Flo
ee
1 - cos x

Fr
cos x
(iv) tan 1 • • , 0 <X <71
or
ur
1 + sin x
f
ks

-1 1 + sin x 71 71
(v) tan < > . — <x < —
Yo

1 - sin x 2 2
oo
B

(vi) tan 1 (sec x + tan x), - - < x < — [NCERT EXEMPLAR]


re

2 2
ou

-1 1 - COS X
ad

SOLUTION (i) Let y = tan .Then,


sin x
Y

2 X
nd

2 sin
Re

-1 2 -1 x X 7t X 71
y = tan • = tan tan — Y-ft<X<ft=>----- <—<-
x X 2 2 2 2 2
Fi

2 sin — cos —
2 2
=I
dx 2

-1 1 - COS X
(ii) Lety = tan • .Then,
1 + cos x

-1 x x
tan tan — if tan >0
-1 2 sin2 x/2 -1 x 2j ' 2
y = tan = tan tan —
2 cos2 x/2 2 , -1 x x
tan - tan — if tan <0
2j ' 2

x 1
if 0 <x < ft if 0 < x < ft
=> y =
2 ' 4/ I2 '
X i
if - ft < x < 0 if - ft < x < 0
2 ' 2 '

ReadYourFlow.COM
11.54 MATHEMATICS-XII

-1 1 + COS X
(iii) Lety = tan • .Then,
1 - cos x
2
-1 2 cos x/2 -1 x 1 cot^ —
*
y = tan 2
= tan cot — = tan
2 sin x/2 2 2

K X K X
y = tan -1 tan
2 2 2 2
1
^ = 0-^ 1
2 2
cos X
(iv) Lety = tan -1 . Then,
1 + sin x

w
n n x TC X
sin -+x 2 sin + cos ----- 1-----
2 4 2 4 2

Flo
-1 -1
y = tan = tan
2 sin 2
;T 71 X
-+x +-

ee
1 - COS
2 4 2

Fr
-1 . n x -1 71 71 X n x
=> y = tan cot — + - = tan tan for ----1----
ur
4 2 2 4 2 4 2
1
^ = 0-^ 1
ks

rfx 2 2
Yo
oo

1 + sin x
(v) Lety = tan -1 • .Then,
B

1 - sin x
re

2
1 - cos (n/2 + x) 2 sin (ti/4 + x/2)
tan -1
ou

y = tan 1 -
ad

=> 2
1 + cos (n/2 + x) 2 cos (7t/4 + x/2)
Y

n x
-1
tan H TC 7C _ X X 7C
d

=>
Re

y = tan —I— — <x< — => 0< —i— < —


n

4 2 2 2 4 2 2
Fi

rfy _ 1
dx 2
(vi) Lety = tan 1 (sec x + tan x).Then,
ji
1 - cos -+x
-1 1 sin x 2
y = tan + • = tan -1
cos x cos x TC
sin -+x
v 2

2 sin 2
-1 4 2
• = tan -1
TC X
=> y = tan tan — + - ----- 1-----
» . 7C X | 7C X 4 2J 4 2
2 sin + - I cos — + -
v4 2 v4 2
dy = 1
dx 2

ReadYourFlow.COM
DIFFERENTIATION 11.55

EXAMPLE 14 Differentiate the following functions with respect to x:


(i) tan 1 j-^1 + x2 + a:1 , x eR

(ii) tan -1 -x , x eR

(iii) tan -1 <


f + x2 -1 ■ ,x*0 [CBSE 2004,2012]
x

^Jl + x2 +1
-1
(iv) tan ■ ,x*0
x

w
(v) cot -1 +X

Flo
-l -f sin x + - sin x
(vi) tan • , 0 < x < ft. [CBSE 2004]
+ sin x - - sin x

ee
Fr
SOLUTION (i) Let y = tan 1 (-Jl + x2 + x). Putting x = cot 0, we get

-1 -1 1 cos 0 1 + cos 0
or = tan -1
ur
y = tan (cosec 0 + cot 0) = tan
sin 0 sin 0 sin 0
sf

2 cos 2 0
k
Yo

, -1
oo

=> 2
y = tan 0 . 0 0
2 sin — cos -
B

2 2
re

0 ft 0 ft 1
=> y = tan -1 cot - = tan • tan — --0
ou
ad

2 2 2 2 2
Y

ft 1 ,-l [y X = cot 0 0 = cot 1 x]


=> y =------cot x
y 2 2
nd
Re

1
^ = 0-^ 1 1
Fi

dx 2 1 + x2 2 (1 + x2)
(ii) Let y = tan 1 (Jl + x2 - x). Putting x = cot 0, we get

2 0
1 1 - cos 0 2 sin
-1
y = tan (cosec 0-cot 0) =tan = tan -1 2
sin 0 0 . 0 0
2 sin — cos —
2 2
-1 0 1 1-1
=> y = tan tan — = - 0 = - cot 1 x
2 2 2
rfy _ 1 1 1
dx 2 1 + x2 2(1+/)

f+X2 -1
(iii) Let y = tan -1 . Putting x = tan 0, we get
x

ReadYourFlow.COM
11.56 MATHEMATICS-XII

2 0
2 sin
-1 sec 0 -1 -1 1 - cos 0 -1 2
y = tan = tan tan
tan 0 sin 0 0 0
2 sin cos
2 2
-1 Q -0 = — tan-1 x
y = tan tan —
2 2 2

dy = l 1
dx 2 1 + x2

-1 •Jl + x2 + 1
(iv) Let y = tan . Putting x = tan 0, we get
x

2 0

low
-1 sec 0 + 1 1 + cos 0 2 cos
-1 -1 2
y = tan = tan = tan
tan 0 sin 0 0 0
2 sin cos
2 2)
n 0

ee
-1 0 n 1 -1
=> y = tan cot - = tan
-1
tan *-l0 = tan x
2
rF
2 ~2 2 2 2 ~2

Fr
1
^ = 0 - —x 1 1 for
dx 2 l + x2 2(l + x2)
ou
(v) Let y = cot-1 (-Jl + x2 + x). Putting x = cot 0, we get
ks

2 0
oo

1 + cos 0 2 cos
-1 -1 2
Y

i/ = cot 1 (cosec 0 + cot 0) = cot = cot


B

sin 0 0 0
2 sin - cos
re

2 2
0 1 1
ou

-1 -l
ad

=> y = cot cot - = — 0 = — cot x


2 2 2
Y

dy _ 1
nd

dx 2(1 +i2)
Re
Fi

(vi) Let y = tan


-1 f1 + sin x + - sin x
+ sin x - - sin x

We know that:
2
. 2 X 2 X X x X . X x x
sin — + cos — + 2 sm - cos — cos - + sm — = cos — + sm —
2 2 2 2 2 2 2 2
i--------------- X X
'l + sin x = cos — + sin — , for 0 < x < jt.
2 2
\2
2 X . 2 X
• —
* cos — x x X X X
and. - sm x = cos — + sm — - t2 sm cos — sm — cos — sm —
2 2 2 2 2 2 2 2
x x
cos — sin if 0 < x < -
2 2 ' 2
=> - sm x
x X Tt
cos---- sm — if — <X < JT
2 2) 2

ReadYourFlow.COM
DIFFERENTIATION 11.57

Thus, we have following cases:


CASE I When 0 < .r < -.
2
In this case, we have
x X x x
cos + sin + cos - sin
2 2 2 2; -1 n x
y - tan 1 - ■ = tan cot — = tan •; tan
x
. x x . x 2 2 2
cos + sin - cos - sin -
2 2 2 2

71 _ X 71 71 X 71
=> y v 0 <x < — => - <------- <-
- =^
2~2 2 4 2 2 2
1
^ = o-1
dx 2 2

w
CASE II When - < x < ti.
2

Flo
In this case, we have
x

ee
X . X . X
cos + sin - + Sin - cos
2 2 2 2 -1 x x

Fr
y = tan 1 tan tan —
x X - sin * x 2 2
cos + sin -cos
2 2 2 2 or
ur
dy _ l
sf
dx 2
k
Yo

EXAMPLE 15 Differentiate the following functions with respect to x:


oo

-1 a+x
B

(i) tan
1 — ax
re

a cos x-b sin x


ou

-1 71
ad

n
(ii) tan — <x < — andJ -
a L.
tan x > -11 [NCERT EXEMPLAR]
b cos x + a sin x / 2 2
Y

-l 3a2 x - x 3 'l 1 x _1_


nd
Re

(iii) tan [NCERT EXEMPLAR]


7?3 - 3flx2 ' f3< aK J3
Fi

-1 a-x
(iv) tan ,-a <x <a
a+x

d -1 a+x
SOLUTION (i) — tan
dx 1 - ax

= — d (tan 1 x) = 0 +
d (tan 1 77) + —
d {tan -l1 77 + tan -11 x) = — 1 1
dx dx dx 1 + x2 l + x2

-1 a cos x -b sin x
(ii) Let y = tan . Then,
b cos x + 77 sin x

c a ,
- tan x
-1 b [Dividing numerator and denominator by b cosx]
y = tan
1 a x
1 + tan x
b

ReadYourFlow.COM
11.58 MATHEMAT1CS-XII

-1 a - tan 1 (tan x) = tan -1 K K


=> y = tan -x ----- < X < —
b 2 2

=> ^ = 0-1 =-l


dx
f 3a
x 2 3^1
x-x
(iii) Let y = tan 1 . Then,
a3 - 3ax2

\3
3x x
-1 a a)
y = tan [Dividing numerator and denominator by a3]
2
1-3 *
a

w
x
Putting - = tan 0, we get
a

Flo
-i 3 tan 0 - tan 3 0
y = tan = tan 1 (tan 3 0)
1-3 tan 2 0

ree
1 x 1

F
Tl TC
=> y = 30 ------- 7= <- <-7= => ---- <0 <- => -— < 3 0 < —
y[3 a yf?, or 6 6 2 2
ur
-1 *
=> y = 3 tan
f
a
ks

3a2 1
Yo

dy 3 d (x 3a
=> — = -------- — x — —
oo

dx x2 dx\a 2----- 2 x ~ 2 2
a z +■ x z a a +x
B

1+-2
a
re

a-x
(iv) Let y = tan -1
ou

•, where - a <x <a. Substituting x = a cos 0, we get


ad

a+x
Y

a- a cos Q 1 - cos 0
nd

y = tan 1 = tan’1
Re

a + a cos 0 1 + cos 0
Fi

2 0
2 sin
2 -1 0
=> y = tan 1 • 2 0
tan tan —
2
I 2 cos
2

Now,

-a<x<a and x = flcos0=> -a<a cos Q<a=$ -1<cos0<1 => 0e(0,7i)=>

1 -11 x
y = tan -1 tan — = tan- 11 tan -1 = 10 = — cos
2 2J 2 2 a
dy
=>
1
------ X
1
=x — —
d fx 1
dx 2 a.2 dx a 2^/7 -X 2
2
a

ReadYourFlow.COM
DIFFERENTIATION 11.59

dy
. -1
EXAMPLE 16 //y = sin jx^/1 -x-Jx and 0 < x < 1, then find —.
dx
SOLUTION We have. [CBSE 2010]
. -1
y = sm j^V1 - x - yfx tJ1 - x2} , where 0 < x < 1
. -1
=> y = sm

-1 . -1
y = sin
-1 x - sin 1 -Jx Using : sin -1 x - sin ~ y = sin

Differentiating with respect to x, we get


dy = ___1 1 1 1 1
dx f-x2 f-(^) 2 dx

w
-1 dy
EXAMPLE 17 If y = cos -X + and 0 < x < 1, find

Flo
dx
SOLUTION We have.

ee
-1 | xfl -x + yfx^l-X2 |

Fr
y = cos

-1 |x^l-(Vx)2 + yfx \ZU-X2 I or


ur
=> y = cos
sf
K . -1 jx-Jl -(Vx)2 +y[x yjl -x2 |
Jy = —
2 sin
k
Yo
oo

y = ^-1sin ]x + sin 1Vr|


B
re

=> — -sin 1 x -sin


y= 2
ou
ad

-1 x -sin 1Vx
Y

=> y = cos
Differentiating with respect to x, we get
nd
Re

dy _ 1 1 1 1
i(4~x) =
Fi

dx 2 dx Vl-x2 2Vx-/l-x2
yi-(V^)
yjl+X2 +Jl -X2
-i dy
EXAMPLE 18 If y = tan < , -1 < x < 1, x # 0 /mrf —.
Jl+X2 -y/l-X2 dx
[CBSE 2015]
SOLUTION Putting x2 = cos 20, we get

1 + cos 20 + ^ - cos 20 ^2 cos2 0 + J2 sin2 0


-1
y = tan = tan
+ cos 20 - - cos 20 J2 cos2 0 - -y 2 sin 2 0

\
-1 cos 0 + sin 0 1 1 + tan 0 -1
=> y = tan = tan = tan tan ^ + 0
cos 0 - sin 0 1 - tan 0 4
//

=> ^+0 v 0 < x2 < 1 => 0 < cos 20<1=> O<20< — => O<0< — => — < — + 0< —
y=i 2 4 4 4 2

ReadYourFlow.COM
11.60 MATHEMATICS-XII

7t 1 -12
1/ = — + — COS X ■: cos 2Q = x2 => 0 = — cos 1 x2
4 2 2
dy _ d ( k 1 d
+ (cos 1 x2)
dx dx 1 4 2 dx
dl = 0 + -1 (-D d (x2)2 =-4x_=
1 2x
=
-x
dx 4 dx
2 Ji-*4 V1--4
EXAMPLE 19 Differentiate sin -1
2j-
+ cos
-ifi-,2 . with respect to x, if
1 +x2 l + .v2
(i) xe(0,l) (ii) x e(-l, 0) (iii) x e(l, oo) (iv) X €(-00,-1)
2^
2x _1 1 -x
SOLUTION Lety = sin -1 + cos . Putting x = tan 0, we get

w
1 + *2 l + x2

f 2 ^
2 tan 0 - 1 1 - tan “ 0

Flo
. -1
1/ = sin + cos sin (sin 2 0) + cos 1 (cos 2 0)
1 + tan2 0 1 + tan2 0

ee
(i) When 0 < x < 1.

Fr
We have.
x = tan 0 and O<x<l=>O<tan0<l => 0<0< — => O<20<- for
ur
4 2
V = sin 1 (sin 2 0) + cos~ 1 (cos 2 0)
ks
Yo

=> -1
y = 20 + 20 = 40 = 4 tan x [v x = tan 0 => 0 = tan 1 x]
oo

dy_ = 4
B

=>
dx i + x2
re

(ii) When -1 < x < 0.


ou
ad

We have.
Y

x = tan 0 and - l<x<0=> -l<tan0<O=> --<0<O=> --<20<O


4 2
nd
Re

=> sin 1 (sin 2 0) = 2 0 and cos- 1 (cos 2 0)= cos- 1 {cos (- 2 0)} = - 2 0.
Fi

y = sin- 1 (sin 2 0) + cos- 1 (cos 2 0)

=> y = 2 0 + (- 2 0)
=> y = 0

=> ^ =0.
dx
(iii) When x e (1, oo).
We have.
x = tan 0 and 1 < x < oo => 1 < tan 0 < go => —<0<—=> - <20<7i
4 2 2
-1
=> cos ~ (cos 2 0) = 2 0 and sin - 1 (sin 2 0) = sin - 1 {sin (rc - 2 0)} = tt - 2 0
y = sin- 1 (sin 2 0) + cos- 1 (cos 2 0) = 71-20 + 20 = n

=> ^ = 0.
dx

ReadYourFlow.COM
. DIFFERENTIATION 11.61

(iv) When * e (- co, -1).


We have.
x = tan 0 and -oo<x<-l => -ce>< tan 0<-l=> <0<- — => - tt:<20<--^
2 4 2
=> sin 1 (sin 2 0) = sin 1 {- sin (tt + 2 0)} - sin -1 {sin (-it — 2 0)1 = - 7T - 2 0
-1
and. cos 1 (cos 2 0) = cos {cos (-2 0)) = - 2 0

y = sin 1 (sin 2 0) + cos 1 (cos 2 0)


y = -7t-20-20
y = - ix - 4 tan- 1 x [•.• x = tan 0 0 = tan ~lx]
^ = 0 42 = 4
=>
dx 1 +xl i+I2

w
EXAMPLE 20 Differentiate tan 1 2x
+ cos
-l[ 1 -x2 with respect to x, when
i 2 1 + x2
1 -x \ /
(i) x e(0,1) (ii) x e (1, oo)

Flo (hi) x e(-l, 0) (iv) X €(-00,-1)

ee
-1 2x

Fr
SOLUTION Let y = tan + cos . Putting x = tan 0 , we get
1 + x2
or
ur
-1 2 tan 0 _ 1 1 - tan 2 0 -1 (tan 2 0) + cos 1 (cos 2 0)
y = tan + cos = tan
f
1 - tan 2 0 1 + tan“ 0
ks
Yo

(i) When x e (0,1).


oo

We have.
B

0 < x < 1 and x = tan 0 => 0 < tan 0<1=?> O<0<—=> O<20<^
re

4 2
-1 (tan 2 0) = 20 and cos- 1 (cos 20) =20
=> tan
ou
ad

y = tan- 1 (tan 2 0) + cos 1 (cos 2 0)


Y

-1 -1
=> y = 20 + 20 = 40 = 4 tan x [y x = tan 0 => 0 = tan x]
nd
Re

dy = 4
=>
Fi

dx i + x2
(ii) When x e (1, oo).
We have.
x > 1 and x = tan 0
tan 0>1 => - <0< — => —<20<7r=> —<20<7t and - — <20-7t<O
4 2 2 2 2
-1
cos (cos 2 0) =20

and. tan- *(tan 2 0) = tan 1 tan (ti: - 20) j = - tr "i -1 {tan (tc - 20)} = -(tc - 20) = 20 - ti .

y = tan 1 (tan 2 0) + cos 1 (cos 2 0)


-1
=> y = 20-?t + 20 = 40-71 = 4 tan X - 7t
dy = 4 4
=> -0 =
dx i + x2 1 +x2

ReadYourFlow.COM
11.62 MATHEMATICS-XII

(iii) When x e(-l, 0).


We have,
-1 < x < 0 and x = tan 0 => -l<tan0<O=> <0<O=> --<20<O
4 2
tan 1 (tan 2 0) = 2 0 and cos 1 (cos 2 0) = cos- 1 jcos (-2 0)} = -20
- 1 (tan 2 0) + cos- 1 (cos 2 0) = 2 0 + (- 2 0) = 0
y = tan
=e> ^ = 0
dx
(iv) When x e (- oo, -1).
We have,
- co < x < -1 and x = tan 0
=> - oo < tan 0 < -1 => - -<©<-—=> - t:<20<- — => — < - 2 0 < tt and O<tx + 20< —
2 4 2 2 2
cos 1 (cos 2 0)= cos- 1 jcos (-2 0)} =-20

low
and. tan 1 (tan 2 0) = tan- 1 (tan (tt + 2 0)} = tt+20
y = tan 1 (tan 2 0) + cos- 1 (cos 2 0)
=> y = ti:+2O-20 = tt

ee
=> dX = o.
dx
rF
Fr
EXAMPLE 21 If y = sin 1 x + sin 1 yjl -x2, find in each of the following cases:
for
(i) x e (0,1) (ii) x e(-1, 0) [NCERT1
u
SOLUTION Putting x = sin 0, we get
ks

-1 (sin 0) + sin - 1 (cos 0)


y = Sin
Yo
oo

(i) We have,
B

x e (0,1) and x = sin0=> O<sin0<l=> O<0<-=> 0<—-0<-


2 2 2
re

-1 (sin 0) + sin 1 (cos 0)


i/ = sin
ou
ad

=> y = sin 1 (sin 0) + sin - sin — - 0 = 0 + ^-0 = ^


Y

v2 2 2
nd
Re

= 0
dx
Fi

(ii) We have,
x e (-1, 0) and x = sin0=> -l<sin0<O=> -—<0<O
2
-1 (sin 0) = 0 and sin 1 (cos 0) = sin -1 n _
=> sin sin — + 0 =- + 0
2 2
i/ = sin 1 (sin 0) + sin 1 (cos 0) = 0 + -^ + 0= ^ + 2sin 1 x
2 2
dy 2 2
=> = 0+
dx

EXERCISE 11.3
LEVEL-1
Differentiate the following functions with respect to x:
-i 2x,/l -x2 4=<x<i -1
1. cos 2. cos < • , -1 < x < 1
'V2

ReadYourFlow.COM
DIFFERENTIATION 11.63

• ,0 <x <1 4. sin ,0 <x <1

-1 x
- ,-a<x<a 6. sin <
[V -X
2
Jx2 +a 2
7. sin-1 (2x2 -1) /0<x<l -1 (1 -2x2)/0<x<1
8. sin

-1 x -1 sin x + cos x 3 K n
9. cos < 10. sin --------<x <-
J2
I2 x +a
2 4 4

-1 cos x + sin x n n
11. cos , — <x <- [NCERT EXEMPLAR]
J2 4 4

w
-1 X
12. tan = •,-1<x <1

Flo
1 + V1- ^

ee
2
X -1 -x
-1

Fr
13. tan ■ ,- a <x <a 14. sin ,-1 < X < 1
-X 2 42
or
ur
X+ Jl-x2 4x 1 1
f
-1 -l
15. cos < •,-1<x <1 16. tan , — <x <-
V2 2 2
ks

1 - 4x2
Yo
oo

/ 1 \
2 -l 2flx
-l
B

17. tan , -CO <X <0 18. tan ,fl>l/-co<X<0


2x
1 -4X 1 -a
re
ou

l V1 + a2 x2-l
ad

+X + -1
-1 20. tan ,x*0
19. sin < , 0 <x <1
Y

2
nd
Re

-1 sin x -1
Fi

21. tan -n<x<7i: [NCERT] 22. sin


1 + COS X

2/1
-1 1 -X
23. cos , 0 <x <co 24. sin
2n
1 +X

-1 a+x -l -Jx + yja


25. tan 26. tan
1 -flX
f

-1 a + b tan x -1 a + bx
27. tan 28. tan
b - a tan x b - ax

_l x — a -1 x
29. tan 30. tan
x+a 2
1+6x

ReadYourFlow.COM
11.64 MATHEMATICS-XII

31. tan 1 5x 1 1 -1 cos x + sin x 71 K


32. tan , — <x <—
1 - 6 x2 ’ V6 V6 cos x - sin x 4 4

x+1
-1 . -1 2
33. tan < 34. sin [CBSE2013]
1/3
1 -(a x) 1 + 4'r

-1 2x -1 1+x2 dy _ 4
35. If y = sin + sec 7T , 0 < x < 1, prove that —
1 + x2 1 -x z dx "l+x2'

-1 X 1
36. Ify = sin + cos -1 dy =
= , 0 < x < oo, prove that — 2

i1 +X
2

37. Differentiate the following with respect to x:


+X 2 dx " l + x2'

low
(i) cos-1 (sin x) l 1 —x
(ii) cot [NCERT, CBSE 2004]
l+x

-1 + sin x + - sin x
38. If}/= cot < , show that — is independent of x.

ee
INCERTJ
+ sin x - - sin x dx
rF
Fr
2^
-1 2x -l l + x dy _ 4
39. If y = tan + sec y , x > 0, prove that —
1-x2 dx l+x2'
for
1 — Xz
u
-1 x + 1 . -1 x -1
ks

40. If y = sec + sin , x > 0. Find


Yo

x —1 x+1 dx
oo
B

1 -x
41. If y = sin 2 tan 1 • , find
re

l+x dx
ou
ad

1
42. If y = cos 1 (2x) + 2 cos 1 ^1 - 4x2 , 0 < x < —, find —.
2'
Y

dx
43. If the derivative of tan (a + fcx) takes the value 1 at x = 0, prove that l + a =b.
nd
Re

44. If y = cos 1 (2x) + 2 cos- 1 -Jl _4 x2 -A


- < x < 0, find
Fi

7 2 dx

45. If y = tan 1 dy
, find —. [CBSE 2003,2008]
+x+ dx

! 2x-3^7
46. Ify = cos , find —. [CBSE 2010]
Vl3 dx

x+1
3X . .
47. Differentiate sin-1/ — — with respect to x. [CBSE 2013]
1 +(36)
. -1 6xVl-9x2 \---- -r-
48. Ify = sin <x < , then find — . [CBSE 2017]
3V2 dx
ANSWERS
2 1 1 1
1. 2. - 3. - 4. -
2^? 2^?

ReadYourFlow.COM
DIFFERENTIATION 11.65

1 A a
5.

f2 -X

a
2 2
a +x
2

9. - 10. 1 11. -1
2 2 2
a +x

1
13.
2>2 -X 2 1 + 4x2

X 4- 1
iog,- 2 18. 2<lQgi 20. I (7
17. -
1 +4r 1 + a2x 2 yjl-X2 2 l+«2 x2

ii-l
1 2 nx
21.1 22. - 23. 24. 0
2 »i

w
2 1 +x2 1+x
1 1 1
25. 26. 27. 1 28.
2 Jx (l + x)

Flo
l+x2 1 + x2
3 2 3 2

ee
29.^4 30. 31. 32. 1
+x 1 + 9 x2 1 + 4x2 1 + 9 x2 1 + 4 x2

Fr
1 x-2/3 1
33. - 37. (i) -1 (ii)
2/3 or l+x2
ur
3 1 +x
f
2 6
40. 0 42. 44. -
ks

Jl-4x2
Yo
oo

1 47. 2(log6)6x 6
B

45. 48.
2^~x2 1 + 36x Vl~9x2
re

H/WTS TO NCERT& SELECTED PROBLEMS


ou
ad
Y

26. Given function = tan 1 yfx + tan 1


nd

(fl/b) + tan x
Re

27. Given function = tan


-1 = tan 1 (a/b) + tan -11 (tan x)
1 - (a/b) tan x
Fi

-1 (a/b) + x -1
28. Given function I = tan > = tan (a/b) + tan x
l-(a/b)x

-l 1 ~(a/x) = tan 1 (1) - tan -11 (a/x)


29. Given function = tan
1 + (a/x)

-1 3x - 2x -1 -l
30. Given function = tan = tan 1 3x - tan 2x
1 + (3x) (2x)
-1 -1
31. Given function = tan J 3x + tan ‘ 2x

-1 1 + tan x = tan 1 (1) + tan 1 (tan x) = — + x


32. Given function = tan
1 - tan x 4

33. Given function = tan 1 x1^3 + tan 1 fl1//3

35. Puttingx = tan 0, we gety = 2 tan-1 x + 2 tan -1ix=4tan -1ix

ReadYourFlow.COM
11.66 MATHEMATICS-XII

38. We have.
! ^1 + sin a: + - sin x K
y = cot
+ sin x - - sin x 2

2 \2
X X X X
cos - + sin — + cos — + sin
2 2 2 2J
=> y = cot 1
2 2
X X X x
cos + sin — cos — sin
2 2 2 2

x X X X
cos — + sin + cos---- sin
1 2 2 2 2
=> y = cot
x X X X

w
cos + sin cos — sin
2 2 2 2

Flo
x X X X
cos — + sin - + cos — sin -
. -1 2 2y 2 2J

ee
71
cot ,0 <x < —
x 4

Fr
X X X
cos - + sin - cos---- sin
2 2 2 2
^ y =
for
ur
x X X X
cos + sin — cos — sin
2 2 2 2
ks

1 71 7T
cot - <x < —
Yo

x 2
oo

X X X ' 4
cos - + sin - + cos — sin
2 2 2 2
B
re

X
cot 1 cot — 71
, 0 <x x <—
ou
ad

2y 4
=> y =
Y

1 X 7t 71
cot tan — — <x <—
2 2
nd

'4
Re
Fi

X
cot 1 cot - n
, 0 <x <—
2 4
^ y =
cot 1 i cot — 71 n
-------* , — < x < —
2 2J 4 2

x n
0 <x <-
2' 4
=> y = 71 X 71 71
- <x <-
2 2'4 2
1 n
, 0< x < -
^y _ 2 4
dx 1 n n
< x <
2 ' 4 2
dV
Hence, — is independent of x.
dx

ReadYourFlow.COM
DIFFERENTIATION 11.67

-i x + 1 -1 x -1 -1 -1
40. Use: sec = cos and cos 0 + sin e=-
x —1 x+1 2
42. Putting 2x = cos 0, we get
i/ = 0 + 2 cos-1 (sin 0) = 0 + 2 cos-1 {cos (7r/2 - 0)} =0 + 2 (n/2 - 0) = rc - 0 = tt - cos 1 (2x)

‘UJ dx
11.6 RELATION BETWEEN AND
dx dV
Let x and i/ be two variables connected by a relation of the form /(x, i/) = 0. Let Ax be a small
change in x and let Ai/ be the corresponding change in y. Then,
dy _ dx Ax
lim — and, — = lim
dx A.t—>0 Ax dy A a- -> 0 Ay
Ay Ax .
Now, — x — = 1

w
Ax Ay
/ \
Ay Ax
=> lim = 1

Flo
A x -+ 0 ^ Ax Ay j
Ay x Ax

ee
=> lim lim = 1 [•.• Ax ^ 0 « Ay ^ 0]
Ax 0 Ax A i/ —»0 Ay

Fr
dy dx .
=^>
dx dy or
ur
*1 = 1
f
=>
dx dx/dy
ks
Yo

11.7 DIFFERENTIATION OF IMPLICIT FUNCTIONS


oo

Up till now we have discussed derivatives of functions of the form y = / (x). If the variables x and
B

y are connected by a relation of the form/ (x, y) = 0 and it is not possible or convenient to express
re

y as a function x in the form y = <j) (x), then y is said to be an implicit function of x. To find •— in
ou
ad

such a case, we differentiate both sides of the given relation with respect to x, keeping in mind
Y

that the derivative of <|) (y) with respect to x is — •


dy dx
nd
Re

dy d . 2, 0 dy
For example, — (sin y) =
COS!/ /'*(y) = 2ys-
Fi

dx
It should be noted that — (sin y) = cos y but — (sin y) = cos y • —.
dy dx dx
Similarly, — (y3) = 3y2 whereas ~ (y3) = 3 y2
dy dx dx
ILLUSTRATIVE EXAMPLES

LEVEL-1

EXAMPLE i Ifax2 + 2 hxy + by2 + 2gx+2fy + c = 0, find —and—.Also, show that — •— = !.


dx dy dx dy
SOLUTION We have,
ax2 + 2 hxy + by2 + 2gx + 2fy + c = 0
Differentiating both sides of this with respect to x, we get

4- (“X2) + 4-V h*y) + 4- (ty2) +


dx dx
(2 ^x) + ^- (2 fy) + 4
dx dx
(c) = -f-(O)
dx
dx dx

ReadYourFlow.COM
11.68 MATHEMATICS-XII

=>
aTy)+2hTX™+biy)+2STXM+2fiM+°=°
=> 2ax + 2h x~- + y +b2y —+ 2gxl+2fx^- = 0
dx dx dx
dy_
=> 1 (2hx + 2by + 2/) + lax + 2 hy + 2g = 0
/
dy _ 2 {ax + hy + g) _ ax + hy + g
=> •••(ii)
dx 2 (hx + by + /) hx + by + f
Differentiating both sides of (i) with respect to y, we obtain
-f(ax2) + -f (2ft xy) + -f (by2) + ^-(2gx) + -l (Ify) + -f M =-f (0)
ay 01/ rfy dy dy dy dy

w
a + 2h T^xy) + b 1~^2) + 2gT'+ 2/ T"(y)+ T-(c) = 0
rfy dy dy dy dy dy

Flo
dx
=> a 2x— +2h y — + x +b (2y) + 2g — + 2fxl +0 = 0
dy) { dy ) ' dy

ee
Fr
dx _ 2 (hx + by +/) _ hx + by + f
...(hi)
dy 2 {ax + hy + g) ax + hy + g
for
ur
From (ii) and (iii), we obtain
\ / \
dy dx _ ax + hy + g _ hx + by+f n
ks

dx dy hx + by + f J ax + hy + g ^
Yo
oo

EXAMPLE 2 If X2 + 2xy + y 3 = 42, find ^


B
re

SOLUTION We have,
x2 + 2xy + y3 =42.
ou
ad

Differentiating both sides of this with respect to x, we get


Y

£(*2)+2l(*>+^3)=l(42)
nd
Re

2.y + 21 x — + y 1 + 3 y2 — = 0
=>
Fi

J
dx y dx

=> 2x + 2y + ^- (2x + 3 i/2) = 0


dx
~ (2x + 3 y2) = - 2 (x + y)
dx
rfy = 2 (.y + y)
=>
dx (2y + 3y2)

EXAMPLE3 f/x3 + i/3 = 3(m/, find


dx
SOLUTION Differentiating both sides of the given relation with respect to x, we get
T-(x3) + 4-(y3) = 3«-T(xy)
dx dx dx
=> 3x2 + 3y2 ~ = 3a i x —
dx dx
+y

ReadYourFlow.COM
DIFFERENTIATION 11.69

=> (3y2-3flx)— = 3fly - 3x2


dx
=> 3(y2-flx)^ = 3(ay -x2)
dx
dy_ = ay-x1
=>
dx y-2 - ax

EXAMPLE 4If log (x2 + y2) = 2 tan -ify , show that dy_ = x + y
x) dx x -y
SOLUTION Differentiating both sides of the given relation with respect to x, we get

^ jlog (x2 +y2) 2 — < tan -i y


dx U

w
J S' \
1 1 d y
=> — X----- -?-(i2 + y2) = 2 x X -—
x2 + y 2 dx l+(,y/x)2 dx{x)

Flo
2 x^ -y x 1

ree
1 d , 2S d . 2, dx
=> 2
— {x) + — {y) 2
x + y 2 Ux dx x +y x

F
2
for
ur
=> rb|2”2'*j ■ x2 + y
2
ks

|”»S}-2i-S-')
Yo
oo
B

dy dy
x + y -f- = x-f-y
re

dx dx
=> fj- (y~x) = ~(x + y)
ou
ad

dx
dy _ x + y
Y

=>
dx x-y
nd
Re

1
EXAMPLE 5 Ifx +y+y + x = 0 and x ^ y, yroue that =
Fi

(* + l)2'
[CBSE 2012, NCERT]
SOLUTION We have,
x^y + y^x = o
=> x^y y/ + X
=> X2 (1 + y) = y2 (1 + X) [On squaring both sides]
2 2 2 2
=> * -y = y *-x y
=> (x + y) (x - y) = -xy(x-y)
=> * + y = -*y [■•• x*y\
=> * = -y-xy
y (1 + x) = - x
x
=> y = -1+x

ReadYourFlow.COM
11.70 MATHEMATICS-XII

dy _ (1 + A') X 1 - X (0 + 1)
dx (1 + x)2
dy = 1
=>
dx (i+^)2

(2 2n
-1 ^ -y - tan 1 a, prove that — = —
EXAMPLE 6 If COS
2 2 dx x
X + 1/
SOLUTION We have.
r 2 2
-i * -V -i
cos 2 = tan a
2
* +y
2 2

w
* -y -i
=>
2
2 = cos (tan <?) = X, say
x +y

Flo
2x2 X+l
[Applying Componendo and dividendo]
2y2 X-l

ree
2
x 1 +?.

F
=>
y2 i -^
or
ur
Differentiating both sides with respect to x, we get
f
2 d , 2s 2 d 2.
ks

y (x ) - x — (y )
dx dx
Yo

=> = 0
oo

2x2
(y )
B

y2x2x-x2x2yd;/
re

dx = Q
4
ou

y
ad

2x/-2x2y^ = 0 => 2x2y^ = 2xy2 => ^ = ^


=> dy = 1
Y

=>
y dx - dx y dx 2x2y dx x
nd
Re

AL1TER We have,
r 2 2
Fi

-1 x -y -l
cos = tan <7
2 2
* +y ;

-1 i-(y/42 -1
cos = tan a
l + (y/x)2

-1
2 tan tan a

=> tan
-1 f —
y 1 = —1 tan -1 a
x 2

y = tan 1 tan 1 a
=>
x 2

-yx 1
A [l =0
2
= 0=>x^-y = 0=» ^ = y
dx U'. x dx
J: ' dx x

ReadYourFlow.COM
DIFFERENTIATION 11.71

2
dy _ sin {a + y)
EXAMPLE 7 //sin y = x sin (a + y), prove that
dx sin a [CBSE 2009,2011,2012]
SOLUTION Differentiating both sides of the given relation with respect to x, we get
J-(siny) = ^ jx sin (a + i/)J

cos y — = lx sin (a + y) + x cos (a + y) ~(a + y)


dx dx
dy dy
=> cosy — = sin (a + y) + x cos (a + y) —
dx dx
=> cos y — - x cos (a + y) -- = sin (a + y)
dx dx
dy
| cos y - x cos (a + y) j —
=> = sin (a + y)
dx

w
•: sin y = x sin (a + y)
sin y dy sin y
=> • cos y - cos (« + y) — = sin (a + y)
sin (a + y) dx x=
sin (a + y)

Flo
sin {a + y) cos y - sin y cos {a + y) dy

ee
=> ~r = sm (a + y)
sin (a + y) dx

Fr
sm(a + y-y)xjy = sin(fl + y)
=>
sin (a + y) dx for
ur
dy _ sin 2 (<7 + y)
ks

dx sin a
Yo
oo

ALITER1 We have,
sin y = x sin (a + y)
B

sin y
=>
re

x
sin (a + y)
ou
ad

Differentiating both sides with respect to y, we get


dx sin (a + y) cos y - sin y cos {a + y) _ sin (a + y - y)
Y

dy sin 2 {a + y) — sin 2 {a + y)
d
Re

1
n

-
Fi

dx dx/dy
dy _ sin1 (a+ y)
=>
dx sin «
LEVEL-2
6
EXAMPLE 8 If-Jl -x6 +yjl-y6 = a(x3 -y3), prow that ^ \ -r , where -1 x < 1
O
and -l<y<l. y yl - x
SOLUTION Putting x3 = sin A and y3 = sin B in the given relation, we get
- sin2 A + ijl - sin2 B = a (sin A - sin B)

cos A + cos B = a (sin A - sin B)


A+B A-B) A-B) A + B}
=> 2 cos cos = 2a sin cos
2 2 2 2

=> cot a
2

ReadYourFlow.COM
11.72 MATHEMATICS-XII

=> A~B t-l ^


-y- = cot (a)

A - B = 2 cot-1 (a)
=> sin-1 x3 - sin-1 y3 = 2 cot-1 (a).
Differentiating both sides with respect to x, we get
1 d , 3, 1 *^(y3) = o
x^-(^ ) -
dx
Vi-y6 dx
1 x 3x2 - 1 x3y2t = 0
=>
dx
^/l^7
i-y6
=> - ?L
dx y2 ii-*6

w
ALITER 1 We have, -Jl-x6 +-Jl-y6 =a(x3-i/3)

Flo
Differentiating both sides of the given relation with respect to x, we get

ee
1 ^(l-x6) + 1 d ,i 6X d . 3 3.
-(1-y ) =a — (x -y )

Fr
2V1-*6 dx 6 dx dx
2 V1 -y
1 1 - 6y3 — = fl f 3X2 - 3y2 — or
ur
x - 6x5 +
2yll-x6 dx dx
2^7
f
ks

-3x5 3y5 dy
Yo

=> = 3ax2 - 3fly2 —


oo

6 dx dx
y
eB

5
av2 - 2 -Y
=>
r

> = fl.Y +
dx
ou

Vl-AT6
ad
Y

jfl7V-y3 -X6 +*3|


nd

y2^
Re

=X
dx
7^7 Vi -x6
Fi

dy _ x2 ^/l-y6 <7 Vl -A'6 + .Y3


=>
^=y2V^7U^7-y3
i-/
3
+X
dy _ x2 -yjl -y6 3 3
* -y [Using (i)]
dx 2 /I 6
y Vi jv'1-*6 +v/;1-y6j J i-y6
3
3 3 -y
^ -y
dy _ x2 yl -y6 1 - X6 + yjl -x6 - y6 + x6 3..3
=> -•v y
dx y2 VI-a: 6 L 6 6 T 6 3 3 6
Vl-x yjl -y +1 -y -x y +y

ReadYourFlow.COM
1

DIFFERENTIATION 11.73

dy _ x1 yi-y6 l-A-3y3+Vl-x6 yjl -y 6


=^-
dx y2 -Jl-x6 1 -x3y 3 + \l -x6^ 6
"!/
dy = x^ jl-y6
=>
dx ~ y~ y 1 — x6
EXAMPLE 9 If x2 + i/2 = f - - and x4 + y4 = f2 + -i-, then prove that — = —^
/ f2 dx x3 y
SOLUTION We have,
x 2 + y 2 =^--
t

=> (x2 + y2) t-


if

w
t
=> x4 + y4 + 2x2y2 =f2 + 4 2

Flo
f2
4 4 ? 4 4
vx + y 4=f2+i

ee
=> x + y + 2x y = x + y4 - 2
f2

Fr
=> 2x2y2 = - 2
or
ur
2 2 1 2 -2
x y = -1=> }f 2 ^ ^ -x
f
X
ks

Differentiating with respect to x, we get


Yo

dy dy
oo

-3 1 ^ dy _
2y = -(-2)x => y 3 x'3 y
B

(fx rfx x d*
re

-1 x rfy
EXAMPLE 10 If y= b tan - + tan -i y , find
a dx
ou

x
ad

SOLUTION We have,
Y

-l x -iy
y = b tan — + tan
nd
Re

x
Fi

=> 1= tan
-1 x
— + tan -i y
b a x
V x ,
=> tan — = — + tan -iy
b a x
Differentiating both sides with respect to x, we get
dy
x — -y
1 2{l\dy = 1 + 1 dx
- sec 2X
b UJ^X a x2
i+ y
v x~ /
dy
x
=> 1 sec
- = I, dx -y
b b dx a x2 + y2

dy 1 sec 2 J/|- x 1
- 2
dx b b J x~ + y 2 a

ReadYourFlow.COM
11.74 MATHEMATICS-XII

l__ JL_
x2 + 1/
=> *1 = - / \
dx 1 *
sec 2fy _
b 2 2
x +y
-l cos 3.t 3
EXAMPLE 12 Ify = COS then show that — =
cos'3 x dx cos x cos 3x

SOLUTION We have.
-1
y = cos

cos 3x
cos y =
3
\ COS X

low
3
4 cos x - 3 cos x
=> cos y =
3
cos x

ee
=> cos y = -^4-3 sec9 x
rF
Fr
=> cos2 y = 4 - 3 (1 + tan 2 x)
2 2
for
=> 1 - cos y = 3 tan x
u
2 2
=> sin y = 3 tan x
ks
Yo

=> sin y = ^3 tan x


oo

Differentiating both sides with respect to x, we get


B

2
cos y— = j3 sec x
re

dx

fy = __ V3
ou
ad

=> 2
[Using (i)]
dx cos y cos x
Y

=> fy.- 3
nd
Re

dx cos x cos 3x
Fi

EXERCISE 11.4
LEVEL-1
dy
Find — in each of the following (1-11):
dx
9 o
2. y 3 - ^3xy2 = x 3 +, 03x2 y 2/3 2/3 =(?2/3
1. xy = c 3. x +y
2 2
, 5 5 r
4. 4x + 3y = log (4x - 3y)
a2 b2
5. VVl 6. x + y =5 xy

7. (x + y)2 =2axy 8. (x2 + y2)2 =xy [CBSE 2009] q . -1/2, 2,


9. tan (x + y ) = a
10. = logf - 11. sin xy + cos (x + y) = 1
v y

12. If-Jl-x2 +-Jl-y2 = (x-y), prove that ^ [NCERT EXEMPLAR]

ReadYourFlow.COM /
DIFFERENTIATION 11.75

13. If y -^1 - a:2 + x -Jl -i/2 =1, prove that ^ =

14. If xu = l, prove that ^ + i/2 = 0.


dx
15. If xy2 =1, prove that 2^
+ .V 3=0.
dx
16. If x +y +y + x = 0, prove that (1 + x) 2^ + l=0. [CBSE 20111
dx
17. If log -Jx2 + y2 = tan -i y , prove that ^ = X + y
x dx x~y
x + y = a, prove that ^ ^
18. If sec

w
*-y dx x
r 2 2^ rfy _ x (1 - tan fl)

Flo
19. If tan-1 V* 2 prove that
x+y dx y (1 + tan (?)

ee
20. If xy log (x + y) = 1, prove that ^ = - ^ +x+

Fr
dx x (xy + x + y)
or
ur
sin2 (fl + y)
21. If y = x sin (a + y), prove that — =
f
dx sin ((7 + y)-y cos (fl + y)
ks

2
Yo

__ ti- • / . . , . dy sin (fl + y)


22. If x sin (fl + y) + sin a cos (a + y) = 0, prove that— =
oo

dx sin a [ NCERT EXEMPLAR]


B

y
23. If y = x sin y, prove that — =
re

dx (1 - x cos y)
ou

If y Jx2 + 1 = log ^x2 + 1 - x , show that (x2 + 1) ^ + xy + 1 = 0.


ad

24.
Y

25. If sin (xy) + ^ = x2 - y2, find ^ .


nd

x
Re

dx
26. If tan (x + y) + tan (x - y) = 1, find ^ .
Fi

dx i

eA (elJ -1) dy
27. If cA + =cA + prove that — = ------------- or,-^- + e }/~x = 0 [CBSE 2014]
dx ^(cT-l) ' dx
--
2
28. If cos y = x cos (a + y), with cos a^±l, prove that — = C°S ^ + ^ [NCERT]
dx sin a
29. Ifsin2y + cosxy = /c, find — atx =1, y =-. [CBSE 2017]
dx ^ 4
LEVEL-2

30. If y = {log COSX sin x} {logsin y cos x} -1 + sin

31. If yjy + x + yjy -x = c, show that

ReadYourFlow.COM
11.76 MATHEMATICS-XII

ANSWERS

(* + y)2 xl/ 3
1. -I 2. ^ =
x dx y2 - 2xy - x2
4
4 4 (1 - 4 x + 3y) . y~x
3 (4 x - 3 i/ + 1) 2 6- ~i—
a y y -*
4x (x2 + y2) - y
7. gy-x-y 8. 9. — —
x + y - ax x - 4 y (x2 + i/2) >/
sin (x + y) - y cos (xy) 2x3 + y - x2y cos (xy)
10. 11. 25. 2
x~y -1) x cos (xy) - sin (x + y) x {x cos xy + 1 + 2xy}
x (y^

w
2 2
sec (x - y) + sec (x + y) 4 1
26. 29. 2(V2 + 1) 30. 8
sec 2 (x - y) - sec2 (x + y) 7T2 + 16 2

Flo HINTS TO NCERT & SELECTED PROBLEMS

ee
Fr
12. Put x = sin A, y = sin B and proceed as in Ex. 8.
13. Put x = sin A and y = sin B
for
ur
14. We have.
dy , 2
xy=l=>y = — => — = -^=- Therefore , — + i/ = - 4 + J2=°
ks

7 y x dx x2 dx X X
Yo
oo

15. We have,
B

2 1 dx 2 ^ ^ 2~ + y3 =0
xy = 1 => x = 2 ^ j,, dx
re

y* dx 2
y
28. We have.
ou
ad

cos y = x cos (a + y)
Y

cos y
=> x =
cos (a + y)
d
Re
n

dx - cos (a + y) sin y - cos y x - sin {a + y)


Fi

=>
dy {cos {a 4- y)}2
dx sin (a + y) cos y - cos (fl + y) sin y _ sin (fl + y - y) sin a
=> — 2
2
cos2 (fl + y) cos (rt + y) cos {a + y)
2
<2y _ cos (g + y)
dx sin a

11.8 LOGARITHMIC DIFFERENTIATION


We have learnt about the derivatives of the functions of the form [f (x)]”, n^x'> and nn, where
/ (x) is a function of x and n is a constant. In this section, we will be mainly discussing derivatives
of the functions of the form [/(x)] g(*) where /(x) and g(x) are frmctions of x. To find the
derivative of this type of functions we proceed as follows:
Let y =|/(x)]‘"(A). Taking logarithm of both the sides, we get

log y = g(*) • log {f(x)}

ReadYourFlow.COM
DIFFERENTIATION 11.77

Differentiating with respect to x, we get


1 i \ 1

y dx
dy d
-r [/wj + log {/■(%)}• •
dx = y f{x) dx
Alternatively, we may write
y=[/«]* w=r?w '°8i?:W)
Differentiating with respect to x, we get
1
ffy = eg(^)log l/‘(ar)} <
dx
g(x) ■
/W£(/w)+lo8^)}i^w)

w
dy = g(x) g (X) d [/w) + log (fCx)} • ~ (V(x) j •
=> \f(x)]
dx f{x) dx

Flo
ILLUSTRATIVE EXAMPLES

ee
LEVEL-1

Fr
EXAMPLE 1 Differentiate the following functions with respect to x:
or
ur
(i) xx (ii) xsina: [NCERT] (hi) (sin x) l°g*
f
SOLUTION (i) Let y = x* Then,
ks
Yo

ex-loSx glogab = ^I'logrtj


oo

y = [v
B

On differentiating both sides with respect to x, we get


re

dy. = ex log * A (x log x)


dx dx
ou
ad

=> = xA i log xx — (x) + xx — (log x) [., ^log* = ^


Y

dx dx dx
dy = x x ,log x + x x —
1
nd
Re

=> —
dx x;
Fi

dy
=> dx = ^ (1 + 108 X)
sin x
(ii) Lety = x . Then,

y = e
sin x log .r [v ab = e]°ZI,h eb log a]
On differentiating both sides with respect to x, we get
dy sin x. log x d . . ,
-f- = e — (sm x. log x)
dx dx
dy .t jiog X ~ (sin x) + sin x (log x)J . ^sin .r log .v _ ^.sin
=^> = X
dx
dy sin x sm x
=> — = X cos x. log x +
dx x
(iii) Let y = (sin x) log x . Then,
(jlog x. log sin x b
y = [••• A e b log a ]

ReadYourFlow.COM
11.78 MATHEMATICS-XII

On differentiating both sides with respect to x, we get


= A (iogx.log sin 4
dx dx
dy = logx (log sin x)j
(sin x) log sin .r — (log x) + log x —
dx dx dx

dy = (sin x) log X log sin x + log X X 1


x cos x
dx x sin x

dy = (sin x) logx log sin x


=> — + cot x. log x
dx x
EXAMPLE 2 Differentiate the following functions with respect to x:
(i) (cos x)A' (ii) x
Vx (hi) (logx)
sin x
(iv) (sin x)
cosx

w
SOLUTION Let t/ = (cos x)A. Then,
^x log cos x
y =

Flo
On differentiating both sides with respect to x, we get

ee
dy = /leg cos x A (jr log cos x)

Fr
dx dx
— = (cos x)'v log cos x — (x) + x — (log cos x)
for
ur
dx dx dx
1
ks

=> — = (cos x)A < log cos x + x (- sin x)


Yo

dx COS X
oo
B

=> = (cos x) A (log cos x - x tan x)


dx
re

(ii) Lett/ = xv'Y. Then,


ou
ad

eJx log X
V =
Y

On differentiating both sides with respect to x, we get


nd

A/ = .Alogx A(^logx)
Re

dx dx
Fi

dy _
dx ^{(Iog5£)l(v")+vif(bgJ:)}
d_i=xrx
dx {{losx)^+rxxl}
dy _ Yfx f log T 1 "
=>
"
dx 2^ r*;
(iii) Lett/ = (log x)‘smA. Then,
^sin x log (log x)
y =
On differentiating both sides with respect to x, we get
dy = ^sin x. log (log x) {sin ^ log (log j
dx dx
= (log X) sin x | log (log x) (sin x) + sin x (log (log x))|
=>
dx

ReadYourFlow.COM
DIFFERENTIATION 11.79

^ = sin x ■ log (log x). cos x + sin x x —1 1


=> dx (log.) ------ X —
log X X

dy = sin x sm x
=> (log x) • log (log x). COS X +
dx X log X

(iv) Let y = (sin x) COS X . Then,


gcos x. log sin x
y =
On differentiating both sides with respect to x, we get
dy cosx.log sin x d . , .
— = e ° — (cos x. log sm x)
dx dx
dy = ,log sm
• x—d (cos x) + cos x — (log sin x) 1
(sin x) cosx

w
=>
dx dx dx j

Flo
dy - sin x log sin x + cos x x —^— x cos x
=> = (sin x) cosx
dx sin x

ee
Fr
2
dy = cosx COS X
=> (sin x) • - sin x log sin x + —
dx sm x for
ur
EXAMPLE 3 Differentiate thefollowingfunctions with respect to x :
ks

-1 -1
COS X COS X
Yo

(i) X (ii) (sin x)


oo

-1
B

SOLUTION Lety = x COS A. Then,


re

1 X . log X
y = e cos
ou
ad

On differentiating both sides with respect to x, we get


Y

dx = e
dx
cos 1 x. log x
dx
(cos 1 X . log x)
d
Re
n

dy -l
— = x cos x ] log x — (cos 1 x) + cos 1 X — (log x) 1
Fi

=>
dx l dx dx j

dy 1X cos -1 x
=> — = x cos +
dx x

-1
(ii) Let y = (sin x) cos *. Then,
-1 x. log sin x
y = e cos
On differentiating both sides with respect to x, we get
cos 1 x. log sin x -j-
d (cos 1 x. log sin x)
dx dx
dy -l
=> — = (sin x) COS X cos -1 x — (log sin x) + log sin x — (cos 1 x)
dx dx dx

ReadYourFlow.COM
11.80 MATHEMATICS-XII

dy _ COS
-1 X -1 1
(sin x) cos xx x cos x + (log sin x) x
dx sin x

-1 l log sin x
=> = (sin x) COS X
cos x cot x-----,
dx

EXAMPLE 4 Differentiate the following functions with respect to x:


xx
(i) x (ii) (* Y
SOLUTION (i) Let y = x . Then,
. log x
y =

low
On differentiating both sides with respect to x, we get
dy = e^.logx T^ x)
dx dx
iL = xxX ~ (exl°Zx\oSx)
=>

ee
dx dx
rF
Fr
dy xx x log x
=> -- = x +e
dx dx x Tx (1°8 for
X log X d x log x x 1
log x. e (x log x) + e
ou
=>
dx dx x
ks

dy xx 1 , x 1
oo

=> — = x (log x) xA xx — + log X + X x


dx X
Y

X
B

X
re

dy xx
=> _y_ = x < xA (1 + log x) log x +
dx
ou
ad
Y

dy 1 xA |(1 + log x) log x + -


= x
dx x
nd
Re

(ii) Let y = {xx)x. Then,


Fi

2 2
X. X X X . log X
y = x = X =>}/ = £

On differentiating both sides with respect to x, we get


dy =/.log* d ^2-, )
dx dx
dy = ^x2. log x d . 2. 2 d n ,
=> logx - (x ) + x — (log x)
dx dx dx
? 2
= xx j (log x) 2x + x"^ x 1 X log X
= X
x i
]
=>
dx [ X

^ = xA (2x log x + x)
=>
dx
2
=> ^ = x xx (2 log x + 1).
dx
tan x sec x dy [CBSE 2007]
EXAMPLE 5 If y = (sin x) + (cos x) , find -f-.
dx

ReadYourFlow.COM
1

DIFFERENTIATION 11.81

SOLUTION We have,
y = (sin x)tan A + (cos x) sec x _ ^ tan x. log sin x + e sec x. log cos x
On differentiating both sides with respect to x, we get
dy _ d . tan x log sin + , sec.v log cos x,
dx dx dx
dy tan* log sin x d . . . . secy log cosy d . ^
=> — = e 6 —(tan x log sin x) + e 6 —(sec x log cos x)
dx dx dx
= tan x
=> (sin x) — (tan x) x log sin x + tan x x — (log sin x)
dx dx dx
sec x
+ (cos x) — (sec x) x log cos x + sec x x — (log cos x)
dx dx

dy = (sin x) tan x 2 1

w
=> sec x log sin x + tan x x x cos x
dx sin x

Flo
secx 1
+ (cos x) sec x tan x log cos x + sec x (-sin x)
COS X

ee
dy

Fr
tan x jsec2x log sin x + 11 + (cos x) sec x
=> ~ = (sin x) {sec x tan x. log cos x - sec x tan x}
dx
EXAMPLE 6 Differentiate: (log x)'v + xlog x with respect to x.
for
ur
SOLUTION Let y = (log xf + xlog v. Then,
ks
Yo

log x
glog (log X)A log(x ) = ex log (log x) + e log X. log X
oo

y = +e
B

On differentiating both sides with respect to x, we get


re

dy = ex log ^log ^ x — {x log (log x)} + c(log x — (log x)2


dx dx dx
ou
ad

dy = (log x)"' | log (log x) X ^


d . . d {log (log x)) J + xlog A 12 (log x) ^ (log x)) J
Y

=> — (x) + X X---


dx dx
nd
Re

* = <108 X)X
1 1
log (log x) + x x --------X — > + X
logx
{2(,ogx)l}
Fi

log X X

dy v 1 logx 2 log x
=> = (log x)’ < log (log x) + — +X
dx logx X

EXAMPLE 7 Differentiate the following functions with respect to x :


cot x 2x2 -3
(i) -v + ICBSE2012] (ii) cos (xx)
x2 + x + 2
x 2 (iv) xx e2i'x+ 3)
(iii) log (x + cosec x)

cot X 2x2 -
SOLUTION (i) Let t/ = x + —. Then,
x2 + x + 2

j,cot x logx 2x2 - 3 cot X glogxCOtAr _ gCOt x log Xj


y = + [••• *
x2 + x + 2

On differentiating both sides with respect to x, we get

ReadYourFlow.COM
11.82 MATHEMATICS-XII

dy_ _ d_ , cot x log .V\ + d_ 2x2 - 3


=>
dx dx dx x2 + x + 2
(x2+x + 2)4- (2-x2 -3)-(2x2-3)~ (x2 + x + 2)
dy = ^cot* logx_l(cotx.logx) + dx dx
=>
dx BX (x2 +x + 2)2

cot .V | (log x) (cot x) + (cot x) J- (log x) j (x2 + x + 2) (4x) -(2x2 - 3) (2x + l)


= X +
dx (x2 +x + 2)2

dy _ ^cot x -cosec 2 x. log x + cot X + 2x2 + 14x + 3


dx x (x2 + x + 2)2

(ii) Let y = cos (x'r). Then,

w
(.xlogX) [v ^ = ex log x]
y = cos
On differentiating both sides with respect to x, we get
^ = A Los^1081)
dx dx
Flo
ee
Fr
=> dy - - state1108
dx dx
or
ur
=> ^ = - sin (.^Je^1081 ~ (xl°gx')
dx dx
sf
^ = -sm(x*)xxx 4~ (x) • log x + x 4~ (log x) j
=>
k
Yo

dx dx dx
oo
B

dy _ xx sin (xx) | log x + x. -


=>
re

dx
ou

dy _ - xs sin (x*) (log x + 1)


ad

=>
dx
Y

(iii) Let y = log (xA + cosec2 x). Then,


nd
Re

dy = 1 dr 2
x —^ (x + cosec x)
Fi

dx xx + cosec2 x dx
dy _ 1 — (xA) + — (cosec2 x)
=> x 2
dx x + cosec x dx dx

dy = 1 A (e*108V —(cosec2*)
dx x' + cosec2 x
x dx dx

dy _ 1 / log ^ A (x log x) + 2 cosec x — (cosec x) l


=>
dx x + cosec2 x
x dx dx J
= ------- -—-— jxA (1 + log x) - 2 cosec2 x cot xl
=>
dx xx + cosec x t J

(iv) Lety = x^2(A+3) . Then,


y = ^Xiogx e2(x+ 3) [V xx = elog xX = ex log 1 ]

y = £>xlogx+2(.r+ 3)
=>

ReadYourFlow.COM
DIFFERENTIATION 11.83

On differentiating with respect to x, we get


dy = e*'°g*+2(*+3)Alxiosx + 2(x+3)i
ax dx
dy = gxlogx e2(x+ 3)
— (x log x) + 2 — (x + 3)
dx dx dx

=> = Xx e2(x+3) (l + logx + 2) = xx e2x+ 3 (3 + log x)


dx
dy_ _ log*
EXAMPLE 8 IfxV = e'x ^, prove that
dx (1 + log x)2
[CBSE 2000 C, 2010 C, 2011, 2013, NCERT EXEMPLAR]
SOLUTION We have,
xJi/ = e x - Jy

low
^ y log .r = ex-\j [v ^ = el°S*y = e^°Zx]
=>
X
=> y log x = x — y => y log x + y = x => y (1 + log x) = x => y =
1 + log X
On differentiating both sides with respect to x, we get

ee
/
rF
Fr
(1 + log x) x 1 - x 0 +
dy _ X
tog*
dx (1 + log x)2 (1 + log x)2
for
ou
EXAMPLE 9 lfxy + yx = 2, find [NCERT]
ks

dx
SOLUTION We have,
oo
Y

xy + yx = 2
B
re

=> ey los * + e* log y = 2


On differentiating both sides with respect to x, we get
ou
ad
Y

dx
d

e}/ log x d ^ Jog ^ + gx log y A (x log y) = 0


Re

=>
n

dx dx
Fi

=> xy ^x tog * + y x 1 + yx J1 x log y + x x - ^


= 0
dx x y dx

=> xy log x + yx — l — + xy x — + yX x log y


= 0
y dx x

=> xy log X + * y x-l dy + y*y-i + /togy = 0


dx

dy = y*y -1 + yx log y
=>
dx xy log x + x y x-l

EXAMPLE 10 IfxlJ = yx, /mrf [NCERT]


dx
SOLUTION We have,
V x
*” =y

ReadYourFlow.COM
11.84 MATHEMATICS-XII

Taking log on both sides, we get


•/ log x = xlogy
Differentiating both sides with respect to x, we get
y -y- (log x) + log a: (y) = x~ (log y) + log y (x)
dx dx dx dx
1* + ,log x x rfy = a- x I ^ + (Jog y) l
yx —
x dx y ^x
=> log, - logy
dx y dx x
dXJ log x—— = logy- —
rfx X

dy y log x -x j = xlogy-y
=>

low
dx y X

dy = y y logy-y
dx x l^y log x -x

ee
EXAMPLE 11 If (cos x)'J = (sin y)x, find ^j-. [NCERT, CBSE 2009]
rF
Fr
SOLUTION We have, for
(cos x)y =(sin y)A
u
Taking log on both sides, we get
ks

y log cos x = x log sin y


Yo
oo

Differentiating both sides with respect to x, we get


B

y (log cos x) + ^ (log cos x) = x — (log sin y) + (log sin y) 1


dx dx dx
re

y sin x + dy . x dy + log sin y


=> cos y —
ou
ad

cos x dx sin y dx
Y

dy
=> — (log cos x - x cot y) = log sin y + y tan x
dx
nd
Re

dy _ log sin y + y tan x


=>
Fi

dx log cos x - x cot y


EXAMPLE 12 Ify = ax + ex + xx + xa,find ^ at x = a. [NCERT]
dx
SOLUTION We have,
y = ax +ex + xx + xa
=> y = a*+e*+e*'°s*+x“
=> + + f (**"**) +
dx
dx dx dx dx
=> ^ = ax log a + ex + exlosx — (xlog x) + axa-l
dx dx
a -1
=> — = ax log a + ex + xx (1 + log x) + « x
dx

=> dy a1 log a + ea + aa (1 + log a) + a a = e1' + 2an (1 + log a)


dx / x = a

ReadYourFlow.COM
DIFFERENTIATION 11.85

REMARK In order to find the derivative of a product of a number offunctions or a quotient of a number
offunctions, zvefirst take logarithm of both sides and then differentiate. The procedure is illustrated in the
following examples.

f-x2 (2x - 3) 1/2


EXAMPLE 13 If \J , find
(x2 + 2)2/3 dx
SOLUTION Taking log of both sides, we get
logy = \ log(l - *2) + | log(2* - 3) -1 log(x2 + 2).
On differentiating both sides with respect to x, we get
1 dy _ 1 1 o 2 1
Y 2x) + ------------x 2-----x x 2x
y dx 2 (1 - x ) 2 (2x - 3) 3 x2 + 2

low
^dx■ = yJ_—
l-x2
1
2x - 3
4x
3 (x2 + 2)

dy jl-x2 (2x - 3) 1/2 x 1 4x

ee
=>
dx (x2 + 2)2/ 3
rF
l-x2 2x - 3 3 (x2 + 2)

Fr
xfx (x +4) 3/2 for
EXAMPLE 14 Find the derivative of 4/3 with respect to x.
(4x - 3)
ou
yfx (X + 4) 3/2
ks

SOLUTION Let y =
4/3 •
oo

(4x - 3)
Y
B

Taking log of both sides, we get


re

13 4
log y = - log x + - log (x + 4) - - log(4x - 3)
ou
ad

On differentiating both sides with respect to x, we get


Y

1 dy _ J_ | 3 1 id , A\ 4 1 ^(4at-3)
— (x + 4)----x-----------
y dx 2x 2 x + 4 dx 3 4x — 3 dx
nd
Re

4
Fi

dy _ __+_________
1 3
=> dx ~ V 12x + 2 (x + 4) 3 (4x - 3) X 4

dy _ yfx (x + 4) 3/2 1 3 16
=> 2x + 2 (x + 4) 3 (4x - 3)
dx (4x - 3) 4/3

EXAMPLE 15 Ifxmyn = (x + y) m + n ,prove that —


dx x
[CBSE 2000, 2014, 2017, NCERT EXAMPLAR]
SOLUTION We have,
xm ■ y" = (x + y) m + n
Taking log on both sides, we get
m log x + n log y = (m + n) log(x + y)
Differentiating both sides with respect to x, we get
1 1 dy m+n d
my. —h 77 x-----=----------------- {X + y)
x y dx x + y dx

ReadYourFlow.COM
11.86 MATHEMATICS-XII

m n dy m+n
=> i3
a: y rfo: ^ + y l, dx;
n m+n . it = m +n m
=>
y x+y dx x+y x
nx + ny - my -ny dy _ \ mx + nx - mx - my
y{x + y) dx {x + y) x
nx -^y dy = nx-my ^ dy = y
=>
y(x + y) dx (x + y) x dx x
LEVEL-2
2 c
ax bx
EXAMPLE 16 Ify = + — + 1, prove that

w
(x -a)(x- b) (x - c) (x - b) (x -c) x-c
dy = y a b c
+
dx x a-x b -x

Flo c -x

ee
SOLUTION We have.
2

Fr
ax bx ^+i
{x-a) (x -b)(x- c) (x - b) (x - c) x -c for
ur
ax2 + bx + c + x -c
=> ^ (x - a) (x - b) (x -c) (x - b) (x - c) x -c
ks
Yo

- axl + bx + x
oo

=> y (x-a)(x- b) (x - c) (x - b) (x -c) x-c


B
re

ax 2 bx + x(x-b)
^ (x - a) (x - b) (x - c) (x - b) (x - c)
ou
ad

2 2
Y

ax X
=> y (x — a) (x — b) (x — c) (x —b)(x — c)
nd
Re

ax 2 + x ^ (x-a)
Fi

=> V (x - a) (x-b) (x - c)
3
x
=> y (x - a) (x-b) (x - c)
3
x
=> log y = log •
(x-a)(x-b) (x-c)

=> logy = 3 log x - log (x -a) + log (x -b) + log (x-c)|


On differentiating with respect to x, we get
Idy = 3 f 1 1 1
+ +
y dx x x-a x-b x-c

dy 1 1 1 1 1 1
=> -r + +
dx = y < x x-a x x-b X x-c

ReadYourFlow.COM
DIFFERENTIATION 11.87

dy a b c
=> , = y <
dx x(x-a) x(x-b) x(x-c)

dy = l a b c
=>
dx x a -x b -x x-c

EXAMPLE 17 Prove that the derivative ofan even function is an oddfunction and that ofan oddfunction
is an even function.
SOLUTION Let / (x) be an even function. Then,
/(-*) = f(x)
=> 4-
dx
in-x)} = 4-vw)
dx
d
=> f'(-x). — (-x) =f'(x)

w
dx
=> -f'(-x) = f'{x)

Flo
=> f'(-x) = -f'(x)
=> / ' (x) is an odd function.

ee
Let / (x) be an odd function. Then,

Fr
/(-x) = -/(x)

4-{n-x)} =-
=>
for
ur
dx dx {f«>
=> f(-x)4-(-x) = -f'[x)
ks

dx
Yo
oo

=> -f'(-x) = -/'(x)


B

=> f'(-x) = f'(x)


re

=> / '(x) is an even function.


ou
ad

2x -1 ' (x) = sin x2rfind —.


EXAMPLE 18 Ify=f
Y

x2 +1 dx
2x -1
nd
Re

SOLUTION Let z = . Then,


x2 +1
Fi

y =fW
=>
dx dx dz dx
dy d 2x -1
j =/'(,*) -T-
dx dx x2 + 1

dy 2 (x2 + 1) - (2x -1) 2x


=> = fix)
dx (x2 +1)2

dy_ = 2. 2 (x2 + 1) - (4x2 - 2x)


=> (sin z ) [•.• /' (x) = sin x2 :.f (z) = sin z2]
dx (x2 +1)2
2
dy 2x -1 1 +X-X2
=> = 2 sin
dx x2 + l (x2 +1)2

ReadYourFlow.COM
11.88 MATHEMATICS-XII

X x x sin x
example 19 Given that cos - . cos —. cos -.. , prove that
2 4 8 -Y
1 2 Y 1 2 Y 2 1
— sec — + — sec — + ...= cosec x -
2
22 2 24 4 Y
SOLUTION We have.
x X X sin x
cos — . cos —. cos — ...
2 4 8 Y
Taking log on both sides, we get
XXX
log cos — + log cos — + log cos - ... = log sin y - log x
2 4 8
Differentiating both sides with respect to y, we get
- Y Y x
1 sin - t sin sin cos y 1
_I__ 2 _i 4 _! 8

w
2cos
___ * 4 cos * 8 Y Sin Y Y
cos -
2 4 8
1 x 1 * 1 , * 1
=>
2 4 4 8 8

Flo
- tan------- tan-------- tan -... = cot Y-----
2 Y

ee
Differentiating both sides with respect to y, we get

Fr
1 2 y 1 2 y 1 2 y 2 1
sec ------- ^ sec------- y sec - - cosec y + —^
2
22 2 42 4 g2 8 Y
I
1 1 I
for
ur
2 Y 2 Y 2 Y 2
=> — sec - + sec — + —^ sec ~ • = cosec -Y---- 2~
Y
ks

EXERCISE 11.5
Yo

3
oo

LEVEL-1
B

Differentiate the following functions with respect to x: (1-18)


re

1/x sin x
. Y 2. Y
ou

-1
ad

COS X
+ COS y) A 4. Y
Y

COS X
5. (log y)j (log x)
nd
Re

7. (sin y)
COS X S.ex^x
Fi

logx 10. 10logsinr


^^fsin y)

11. (log X)
log X
[NCERT] io(10'v)
13. sin (ya ) 14. (sin 1 y)a

sin -1 x
16. (tan y) 1/x
2-i
tan -1- x 18. (i) (yv) yfx (ii) y
(sin x - cos x)
+ X
17. Y
Y2 + 1

Y2 +1
(hi) -v
X cos X
+ [NCERT, CBSE 2011] (iv) (y cos y) v + (y sin y) l/.V [NCERT]
2..
Y -1

(V) ( Y + i
1 sin x
+Y xJ [NCERT] (vi) e + (tan y)a [CBSE 2003]
Y

1/x x2 - 3 x2
(vii) (cos y)a + (sin y) [CBSE 2010] (viii) ya 3 + (y - 3)x [NCERT]

ReadYourFlow.COM
DIFFERENTIATION 11.89

dy
Find , {19-32) when
x
= ex + 10v + x'r
20. y = xn + nx + xx + n"
^v_(x2-1)3(2x-1)
’ V p-3) (Ax -1)

enx sec x log x


22. y =

23. = e 3x sin 4x ■ 2X
24/ i/ = sin x sin 2x sin 3x sin 4x
25. y = xsm x + (sin x)x [NCERT]

low
COS X sin x
26. y = (sin x) +(cos x)
v= (tan x)
cot x
+ (cot x) tan x
28. y =(sin x)A + sin -1 Vx [NCERT, CBSE 2009,2013,2017]

ee
cos x tan x
29. (i) y=x + (siri x) [CBSE 2009]
rF(ii) y = xx + (sin x)A [CBSE2008]

Fr
jC 1/ =(tan x)log A + COS
2 TC
4
for
IA­
ou
SI. y = xx + x
ks

32. y = xlog x + (log x)A [NCERT, CBSE 2013]


oo

33. If x13 y7 = (x + t/)20, prove that ^ ^


Y
B

dx x
re

34. If x16 y9 =(x2 + y)17, prove thatx^ = 2y


ou
ad

35. If y = sin (xA), prove that ^ = cos (xA) • xA (1 + log x)


Y

dx
xx (1 + log x) + yA • log y
nd
Re

36. If xA + yA = 1, prove that — =


dx x-y(-v-1)
Fi

37. Ifx^yA-=l,provethat^=-y(^yl0g^
dx x (y log x + x)
dy
38. If x'/ + yA = (x + y)A + l// find
dx
39. If xm v” = 1, prove that — = - —
dx nx
dy (1 + log y)2
40. If yA =c1/ A, prove that [NCERT EXEMPLAR]
dx logy
41. If (sin x)> =(cos y)^ prove that ^ = ^ cos V ^ cot ^
dx log sin x + x tan y
42. If (cos x)» = (tan y)^, prove that ^ = l°g ^ y + y tan x
dx log cos x - x sec y cosec y
dy [CBSE 2014]
43. If e'T + e-1 = cA + l/, prove that + e^~x = 0
dx

ReadYourFlow.COM
11.90 MATHEMATICS-XII

ty _ Hog y)2
44. If ex/ = yx, prove that
dx log y -1
45. If ex + XJ - x = 0, prove that — = -—^
dx x
sin2 (fl + y)
46. If y = x sin (« + y), prove that — =
dx sin (a + y) - y cos (a + y)
2
47. Ifx sin (a + y) + sin a cos (a + y) = 0,prove that— =^£l—^ [CBSE2013]
dx sin a
48. If (sin x)^ = x + y, prove that — =
dx (x + y) log sin x -1
y (x2y + x + y)
49. If xy log (x + y) = 1, prove that — =
dx

low
x (xy2 + x + y)
50. If y = x sin y, prove that ^ = —:---- y
dx x (1 - x cos y)
51. Find the derivative of the function / (x) given by / (x) = (1 + x) (1 + x2) (1 + x4) (1 + x8)

ee
rF
Fr
and hence find /' (1)
x2 + x + 1 2 -1 V3 x dy
52. If y = log ^ + —7= tan
for
x -x +1 1 -X 2 JindfX
ou
sin x - cos x 7T 371 dy
ks

53. If y = (sin x - cos x) — <x < — , find —. ICBSE2010]


'4 4 dx
oo

dy
54. Ifxy = e x-y , find —.
Y

[NCERT]
B

dx
re

55. If y* + x^ + xx =ab, find —. [NCERT]


dx
ou
ad

56. If(cosx)^ =(cosy)A find—. [CBSE2012]


Y

dx
2
nd

57. If cosy = xcos (a+ y), where cos a * ± 1, prove that^ _ cos (a + y)
Re

[CBSE2014]
dx sin a
Fi

X
dy
58. If(x-y) e x-y = a, prove thaty — + x = 2y. [CBSE2014]
dx
59. If x = c-t/ v, prove that [NCERT EXEMPLAR]
dx xlogx

tanx
60. Ify = x + find — [NCERT EXEMPLAR]
dx
y/x2
61. if y=1 + 7/”— \+ +7 , find —.
1 1 1 1 1 dx
—a
x
—a
X X
-P -a
X
-P --Y
V*
ANSWERS
l/x( 1 -logxN sin x
1. X
2
2. x sin x + (cos x) log X
X X

ReadYourFlow.COM
DIFFERENTIATION 11.91

-1 -1
x sin x cos x
3. (1 + cos x)x log (1 + cos x) - 4. x cos x
1 + COS X X

1 COS X
5. (log x)* log (log x) + 6. (logx) cos x ■ - sin x • log (log x) +
logx x log x

7. (sin x) cos* {— sin x log sin x + cos x cot x} 8. xx (1 + log x)


logx 1 10. iolog sin x ■ log 10 ■ cot x
9. (sin x) - log sin x + log x • cot x
x
log x J 1 + log (log x) 12. 1010*10A'(loge 10)2
11. (logx)
X

14. (sin 1 x)x • log sin 1 x + —^ 1


13. x* (1 + log x) cos (x*)

low
sin -l

-1 2
-1 x sin x 1/x ^2 log tan x + 1 sec x
15. x sin 16. (tan x)

ee
x X x tan x
rF
Fr
-1 x tan 1 x log x 18. (i) xx+1/2- 2x +1
17. x tan for + log X
x l 2x
ou
(sinx-cosx) sinx-cosx + (cosx + sin x) log x ^ + 4x
ks

(ii) x
X (x2+l)2
oo

4x
Y

xcosx
B

(iii) x {(1 + log x) cos x - x log x sin x)- / 2 1X2


(X -1)
re

lx {1 + x cot x - log (x sin x)}


(iv) (x cos x)* {1 - x tan x + log (x cos x)) + (x sin x)
ou

2
ad

X
Y

1
1 x 2 -11 . i
1 1+x J * + 1 log *
(v) U + - <^-2-----+ log X + - +X
2 2
nd

x2 +1 X
Re

X X X
Fi

sin x
(vi) e cos x + (tan x)x (log tan x + x sec x cosec x}
1/x 1 . . cotx
(vii) (cos x)* (log cos x - x tan x) + (sin x) — log sin x + —
Y X

2-3 2-3 2 -2
(viii) xx • ------- + 2x log x • + (x - 3)a • + 2x log (x - 3) •
x

19. ex + 10A log 10 + xx log (ex) 20. nxn -1 + nx log n + xx log (ex)

(x2 -l)3 (2x -1) 6x 2 1 2


21. +
p-3) (4x -1) |^2_i 2x -1 2 (x - 3) 4x -1

12 eax sec x log x 1 1


a + tan x + +
X log X 1 -2x

ReadYourFlow.COM
11.92 MATHEMATICS-XII

23. e3r sin 4x 2A (3 + 4 cot 4x + log 2)


24. sin x sin 2x sin 3x sin 4x (cot x + 2 cot 2x + 3 cot 3x + 4 cot 4x)
sin x sin x
25. x cos x log x + + (sin x)A {log sin x + x cot x}
x
cos x
26. (sin x) {- sin x log sin x + cos x cot x} + (cos x) sin x {cos x log cos x - sin x tan x}
cot x cosec2 x (1 - log tan x) + (cot x) tan x 2
27. (tan x) sec x{logcotx-l}
1
28. (sin x)x (x cot x + log sin x) +
2 }Jx -x2

cos x COS X 2
29. (i) x - sin x log x ^ + (sin x) tan x • 1 + sec x log sin x •
x

w
(ii) xA (1 + log x) + (sin x)A (x cot x + log sin x)

sec 2 x log tan x 31. x* (1 + log x) + x1/x 1 l°2gX


30. (tan x) log* • log x
tan x x

Flo x

ee
log x I 2 log x 1

Fr
32. x + (log x)A log (log x) +
X logx
(*+ y) {1 + log (x + y)}-yxy-i -/ logy
for
ur
38. ^
x-v log x + xy X —1 -(x + y) x + y {1 + log (x + y)}
ks

51. 1 + 2 x + 3x2 +... + 15x14,/' (1) =120 4


Yo

52.
oo

x4 + x2 + 1
eB

53. (sin x - cos x) sin x - cos x ((sin x + cos x) log (sin x - cos x) + (cos x + sin x)}
y(x-l) yA jpg y+yx,J~1 + xA (i + log x) log cosy+ y tan x
r

54. 55. - 56.


ou
ad

x (y + 1) x-1 log cosx + xtany


XV + xJ/logx
Y

tan x 2 tanx x a
61. y, Y
nd

60. x sec xlogx +


Re

x ■Jlx2 + 2 X 1
—a -P --Y
Fi

x X X

HINTS TO NCERT& SELECTED PROBLEMS


log X
11. Let y = (log x) . Then,
log |(logx) logx
y = e l
e(iog x) log (log x)
=> y =
^y = e(log x) log (log x) | (logx) log (logx) |
dx dx

= (log *)
logx ( tlx (l08 ^ 108 ^08 ^ + 108 ^ ^ 108 (l08
= (iog:c)iog,iiogaogx) + iogxx 1 1
--------- X —
X log X X

ReadYourFlow.COM
DIFFERENTIATION 11.93

log x 1 log (log *) | 1 log -V 1 + log (log .V)


= (log x) = (log x)
X X X

•V COS -T
x2 +1
18. (iii) Lety = x + . Then,

a: cos .y log Y ,
*2+i
y = e ° +
x2-l
d(x2 + l
dij _ d (cos * log x
dx dx 1 dx x2 _i

excosX,°ZX ~(x cos x log x) + — 4^


dx dx xl _i

low
X COS Y
(x2 -1) 2x - (x2 +1) 2x
x {cos x log x - x sin x log x + cos x) +
(X2 -l)2

Y COS Y 4x
- x (cos x log x - x sin x log x + cos x( -

ee
^ 2 1X2
(X -1)
rF
Fr
1/x
18. (iv) Lety = (x cos x)A + (x sin x) . Then,
%
log (y sin y) l/.v
e log (y cosy)
+e
for
y =
ou
^Y log (y cosy) 1/y log (y sin y)
=> y = +e
ks

di = d_ gX log (y cosy)
oo

+A el/x log(X sin y)


=>
dx dx dx
Y
B

dl =ex log <* cos x) ± |x log (I cos ^ + eVx log(isin») ± - log (x sin x)
re

=>
dx dx dx x
ou
ad

— = (x cos x)A — {x (log x + log cos x)) + (x sin x)1^ J — (log x + log sin x) |
Y

dx dx dx [x J
nd
Re

=> — = (x cos x)'Y (log x + log cos x) + x - tan x


dx
Fi

1/x f 1 (1 j- (log x + log sin x) j


+ (x sin x) + cot x
X X

1/a- 1 + cot x log x log sin x


= (x cos x)x {log (x cos x) + (1 - x tan x)} + (x sin x) 2 2 2
X X X X

l/x j 1 + x cot x - log (x sin x)


= (x cos x)A {log (x cos x) + 1 - x tan x} + (x sin x) 2
X

18. (v) Lety = x+-


if + X 1+ 1/y . Then,
X
X log ( Y + 1/y) (1+ 1/y) log Y
y = e +e
dy = exlog(.x+ l/x) d_lx\0g(x + - (1+ 1/y) log y d 1^
=> +e 1 + — log x •
dx dx x dx x

ReadYourFlow.COM
11.94 MATHEMATICS-XII

=> _ x+-
if i
log x + - +
X d
X+—
i 1+ Vx i
-^logx+li + Iji
+ X
dx X X X + 1 dx X
X
2
=> JLj X + -if
d 1
log X + - +
X
if +X
1 + l/x 1 + * - log x
dx x X x2 + 1 2
X X

=> if 1 1
x2 -1 1/x-l
X+— fog X+- + > +X {1 + X - log x}
dx x x2 +1
2 2
18. (viii) Let y = x^ " 3 + (x - 3)x . Then,
(x2 - 3) 2
log(x- 3)*
y = +e

w
^(x2 - 3) log x x2 log (x — 3)
=> V = +e

Flo
*1 = A L (x2 - 3) log x [ + J ^x2 log (x - 3)
=>
dx dx dx

ee
Fr
= £>(x2-3)logx d_
=> (x2 - 3 ) log x • + ex 2 log(x- 3) d 2 , ,
dx dx fa \ x los(*-3) •
for
ur
=> ^ = xA _ 3 • 2x log x + —----- - . + (x - 3)x • 2x log (x - 3) + —— •
dx x x-3
ks
Yo
oo

25. We have,
y = x sin x
B

+ (sin xf
sin x
re

=> el08* log (sin xf


y = + <?
^sin x log x
ou

+ c x log sin x
ad

=> y =
Y

=> *1 = f_ /^.sin x log x) + — (ex 108 sin;<r)


dx dx dx
d
Re

^ = esin ^ Io§ ^ T fsjn ^ iog x) + ex ^ sin x ~ (x !og sin x)


n
Fi

dx dx dx
sm x
=> — = xsin v -j cos x log x + + (sin x)x {log sin x + x cot x}
dx [ x
28. Lety = (sin x)x + sin- 1 fx. Then,
gX log sin x
y = + sm

=> ^ = A (eIlossin^ + T(sin-ivj)


dx dx dx
=> ^- = ex los sm A — (x log sin x) + — (sin- 1 )
dx dx dx
1 1
=> ~ = (sin x)x {log sin x + x cot x) +
dx -x 2^
1
=> , = (sin xf {log sin x + x cot x) +
dx 2yjx-X2

ReadYourFlow.COM
DIFFERENTIATION 11.95

32. We have.
y = x^x + (log x)x
log x
=> y = elo8<* ) + e log (log x)x

t,log .V log X + e X log (log x)


=> y =
e0°s*>2 X log (log x)
=> y = +e
iL = /og j A (log X)2 + e* l0«<log r) 4-1* log (log X))
dx dx dx
^ = Ylogx 1^ x ^ 1
=> 2 log xx— + (log x) log (log x) + -—— x —
dx log X X

low
, r* lOg X -l
=> — = —----- log X + (log x)'^ log (log x) + -—
dx x logx
54. We have.
xy = ex -v => log(xy) = log(ex y) => log x +logy = x-y

ee
rF
Fr
Differentiating with respect to x, we get
rfy = y(x-l)
I + I *1 = — =>
for
x y dx rfx dx v-7y / x dx x (y + 1)
ou
55. We have.
ks

yx + J + xx = ah
oo
Y
B
re

=> ex log y + e J/logx + e ^logx = ab


ou
ad

Differentiating both sides with respect to x, we get


Y

A. log y) +A (ey i°g I) +A^iog^j = ±(ah)


dx dx dx dx
nd
Re

=> log y A log y) + ey log1 A (y log X) + e1 log1 (X log x) = 0


Fi


dx dx dx
x dy
=> yx iogy + - + X y —log x + - + x'v (1 + log x) = 0
y dx \ dx X

dy . x-1 xy log x) = - [yx log y + y xy 1 + xx (1 + log x)}


=> ~r(xy +
dx
dl = [yx \ogy + yxy 1 + xY (1 + log x)}
=>
dx xyx 1 + xy log x

11.9 DIFFERENTIATION OF INFINITE SERIES


dy
Sometimes the value of y is given as an infinite series and we are asked to find In such cases

we use the fact that if a term is deleted from an infinite series, it remains unaffected. The method
of finding — is explained in the following examples.
dx

ReadYourFlow.COM
11.96 MATHEMATICS-XII

ILLUSTRATIVE EXAMPLES

LEVEL-1
x-00
EXAMPLE 1 Ify=XX , find dy
dx
SOLUTION Since by deleting a single term from an infinite series, it remains same. Therefore,
the given function may be written as
y = xy
=> log y = y log x [On taking log of both sides]
\ dy dy . d .. ,
=>
y±=fX^X + yT^X) [Differentiating both sides with respect to .r]

=> if
y dx
= f'logoi'
dx x

low
=>
rfyj1- - log
. X =l
dx\y X

dy (1 -y log x) _ y_
=>
dx X

ee
V
2
rF
Fr
=> d}L = y
dx x (i -yiogx) for
EXAMPLE 2 Ify = Jsin x + ^jsinlc^{~js^x=+:. .. to co , prove that — = cos x
u
dx 2 y -1
ks

SOLUTION The given series may be written as


Yo
oo

y = Jsin x + y
2 .
=> y = sm x + y [Squaring both sides]
B

o dy dy
re

=> 2y — = cos x + [Differentiating both sides with respect to x]


dx dx
ou

dy
ad

-f-(2y-l) = cosx
dx
Y

dy _ cos x
=>
nd

dx 2 y -1
Re
Fi

x"00
dy _ y2 log y
EXAMPLE 3 Ify -a , prove that
dx x (1 - y log x ■ log y)
SOLUTION The given series may be written as
(*y)
y = « ’
=> log y = xy log a [Taking log of both sides]
=> log (log y) = y log x + log (log a) [Taking log of both sides]
1 7- (log y) = ^ log x + y (log x) + 0
=> [Differentiating both sides w.r.t. x]
log y dx dx dx
1 1 dy dy 1
=> ----------- log x + 1/ x —
log y y dx dx x

dy 1
=> - log X • = y
dx y log y x

ReadYourFlow.COM
DIFFERENTIATION 11.97

dy 1 -y log 1/ log x = y
=>
dx y log y x

dy = y2 log y
=>
dx x {1-y log i/logx}
+ ... to CO
x+ , shozu that — = - ■
EXAMPLE 4 Ify = e
dx 1 - y
SOLUTION The given function may be written as
y = ex+v
=> log y = (x + y) log e [Taking log of both sides]
=> logy = x + y [••• log c = 1]
1 dy dy

w
=> = 1 + [Differentiating with respect to x]
y dx dx
dy 1
=> --1
dx y
= 1 => dy = _y_
dx l-y

Flo
ee
(V^r00 2
dy

Fr
y
EXAMPLE 5 lfy=(^){'f*) , show that — =
dx x (2 - y log x)
for
ur
SOLUTION The given function can be written as
y =
ks

y = xy/2
Yo
oo

i°gy = ^ log ^ [On taking log of both sides]


B

1 dy y 1 1 . dy
re

----- = — x —i— log x — [Differentiating both sides with respect to x]


y dx 2x2 dx
ou
ad

^ U-Ilogx __y
Y

dx \y 2 * 2x
nd
Re

dy] 2-y log x = y_


=>
Fi

dx 2-y 2x
2
dy =____ y
=>
dx x (2-y log x)
1 , prove that ^ = —-—
EXAMPLE 6 If y = X +
1 dx 2y - x
X+
1
X+
X + ...
SOLUTION We have.
1
y = x+
1
x+
1
X+
X + ...
1
=> y = x+-
y

ReadYourFlow.COM
11.98 MATHEMATICS-XII

=> y2 = xy + \
dy n
=> — = y+ x
2^x +0 [Differentiating both sides with respect to a']
dx
=> di«y-*)=y
=> dy = y
dx 2y -x
sin x , prove that ^ = d + V) c°s ^ y sin x
EXAMPLE 7 If y = -----
cos x dx 1 + 2y + cos x - sin x
1+
sm x
1+
cos x
1 +
1 + ... to CO

low
SOLUTION We have,
sin x
y = COS X
1 +
1+y

ee
(1 + y) sin x
rF
Fr
1 + y + cos x
2
y + y + y cos x = (1 + y) sin x
for
ou
Differentiating both sides with respect to x, we get
ks

dy „ dy dy
~ + 2y -f- + cos x - y sin x = — sin x + (1 + y) cos x
oo

dx dx dx dx
Y
B

=> ^ {1 + 2y + cos x - sin x} = (1 + y) cos x + y sin x


re

dy _ (1 + y) cos x + y sin x
ou
ad

dx 1 + 2y + cos x - sin x
Y

EXERCISE 11.6
nd
Re

LEVEL-1
Fi

dy i
1. Ify = Jx + x +... to co , prove that — =
dx 2 y -1
2. If y = ^ cos x + ^cos x + ^cos x +... to co , prove that dy _ sin x
dx~ 1- 2 y'
3. Ify = ^|log x + ^log x + ^log x +... to co", prove that (2 y -1)

, 2
. , dy sec x
4. If y = ^tan x + ^tan x + Jtim x +... to co , prove that — =-------- .
dx 2 y - 1
2
(sin x) (sin x)"' 00 y cot x
5. If y = (sin x) , prove that — =
dx (1 - y log sin x)
(tan .t)" ' ^
6. If y = (tan x)
(tan x) , prove that ^ = 2 at x = - .
dx 4

ReadYourFlow.COM
DIFFERENTIATION 11.99

7. If 1/ = ex + xe + ex , prove that
eX
dy xe ex ex x , e xe
ex 1 + ex ■ log x !■ + ex
— x {1+e log x}
— =e ■ x <— + e ■ log xl + x ■ e ■ x
dx X x

(cosx)... CO 2
(cos x) y tan x
8. If y = (cos x) , prove that — = [NCERT EXEMPLAR]
dx (1 - y log cos x)

11.10 DIFFERENTIATION OF PARAMETRIC FUNCTIONS


Sometimes * and y are given as functions of a single variable e.g. x = y = v|/ (f) are two
functions of a single variable. In such a case x and y are called parametric functions or parametric
equations and t is called the parameter. To find ^ in case of parametric functions, we first obtain

w
the relationship between x and y by eliminating the parameter t and then we differentiate it with
respect to x. But, it is not always convenient to eliminate the parameter. Therefore, ^ can also be

obtained by the following formula

Flo
ee
dy _ dy/dt

Fr
dx dx/dt
To prove it, let A x and A y be the changes in x and y respectively corresponding to a small
for
ur
change At int. Then,
lim Ay dy
ks

Ay = Ay/At ^ dy = Ay Af —>0 At dt
Yo

lim
oo

Ax Ax/At dx A x-*0 Ax Ax dx
lim
B

At ~>0 At dt
re

ILLUSTRATIVE EXAMPLES
ou
ad

LEVEL-1
Y

EXAMPLE 1 Find — in each of the following:


dx
nd
Re

1 2 t
(i) x=a cos t + - log tan - and y - asm t. 1CBSE 2011, NCERT]
Fi

2 6 2
(ii) x = fl (0 - sin 0) and y = a(l - cos 0). [NCERT]
1 2 t
SOLUTION (i) x = a j cos f + — log tan “ - j> and y = a sin t

=> x = a\ cos f + -^ x 2 log tan and y - a sin t

x = a | cos t + log tan


=> and y - a sin t.

Differentiating with respect to t, we get


dx 1 2 t 1
— = a < - sin t + sec x — and — = cos t
dt tan t/2 2 2 dt

dx 1
=> a - sin f + > and ~ = a cos t
dt 2 sin (f/ 2) cos (f/2) dt

ReadYourFlow.COM
11.100 MATHEMATICS-XII

=>
dx
a - sin t +
1
and ty.. -= a cos t
dt sin t dt

dx - sin 2 t + 1 dy
=> a ■ and = a cos t
dt sin f dt
2
dx a cos t. and ^- = a cos t
=>
dt sin t dt
dy _ dy/dt a cos t
7
= tan t
dx dx/dt n cos" t
sin t
(ii) We have,

w
x = a (Q - sin 0) and y = a (l - cos 0)
Differentiating with respect to 0, we get
dx

Flo
— - a (1 - cos 0) and — = o sin 0
dQ dQ

ee
dy _ dy/dQ a sin 0 2 sin (0/2) cos (0/2) 0

Fr
- = cot —.
dx dx/dQ a(l - cos 0) 2 sin2 (0/2) 2
for
ur
EXAMPLE 2 Ifx = a sec 3 0 and y = n tan 3 0, find — at 0 = —. [NCERT EXEMPLAR]
dx 3
ks

SOLUTION We have,
Yo

3 3
oo

x - nsec' 0 and y = a tan 0


B

Differentiating with respect to 0, we get


re

— = 3a see2 0 — (sec 0) and ~ = 3a tan 2 0 — (tan 0)


dQ dQ dQ dQ
ou
ad

=> — = 3a sec2' 0 tan 0 and — = 3a tan9 0sec2 0


Y

dQ dQ
nd

dy _ dy/dQ _ 3a tan2 0 sec2 0 tan 0


Re

sin 0
dx dx/ dQ 3a sec3 0 tan 0 sec 0
Fi

dy n V3
=> = sin — =
dx 3 2
0 = 71/3

EXAMPLE 3 Find — , when x = a cos 3 t and y = a sin 3 t.


dx
SOLUTION We have, x = a cos 3 t and y = a sin 3 f
dx 2 , d
— = 3a cos t — cos (t) = - 3a cos2 t sin t and, — = 3a sin2 f — (sin t) = 3a sin2 t cos t
dt dt dt dt
2
dy _ dy/dt 3a sin" t cos t
2 = - tan t
dx dx/dt - 3a cos" t sin t
-1 t cos -1 t , a > 0 and -1 < f < 1, show that — = - ^
EXAMPLE 4 Ifx=yjasin >y = \a
dx x
[CBSE 2012, NCERT]

ReadYourFlow.COM
DIFFERENTIATION 11.101

SOLUTION We have.

X and y
1/2 1/2
dx _ 1 ( sin -i11 > d sin ^ t —1 d t
a and ^ = i a cos t
a
cos
=> a
It ~ 2 dt dt 2 dt
-1/2
=>
dx
dt
if/""1''
2
a
sin 11 logt, a d
— (sin
dt
1 t)

and,
x-1/2/
dy _ 1 cos-1 t cos-1 t loge a — (cos 11)
a a
~dt ~ 2 dt

low
dx _ 1 -1
sin 11
V/2 X loge (1
=> a (l°ge rt) x
~di ~ 2

_ 1 > 1/2

ee
dy _ 1 cos t = ~yloSe a
and. a (log,, a) x
rF
Fr
~dt ~ 2 2^1-t2

dv -i2
for
*1 = 11 = -y lQge „2 v1
=
u
dx dx 2^/lL? X loge x
ks

dt
Yo
oo

.x . 2 2 „ sin 1 i + cos ^ f - *2 y2 = «%n


ALITER Clearly, x y =a sin

t + cos
,
t =—
-1* 71
B

2
re

Differentiating with respect to x, we get


2xy2 + 2x2 y — = 0 => ^
ou
ad

dx dx x
Y

2t -1 2t
EXAMPLE 5 7/x = sin -1 - awd y = tan , f > 1. Prow that — = -1.
nd
Re

1 +t 1-f2 dx
Fi

SOLUTION Let t = tan 0. Then,


t > 1 => tan 0>1 — < 0 <— => - <20<ti
4 2 2

. -1 2t 2 tan 0
x = sin = sin - 1
l+t2 1 + tan 2 0

=> x = sin- 1 (sin 2 0)= sin -1 (sin (n-2 0)} -- 7r-2 0 = 71-2 tan -1 t
dx 2 -2
— = 0-
dt 1+f2 1+t2

2t
and. y = tan 1
i-t2

2 tan 0 = tan 1 (tan 2 0) = tan -1 {-tan (ti-20)}


y = tan 1 •
1 - tan2 0

ReadYourFlow.COM
11.102 MATHEMATICS-XII

y - ~ tan 1 {tan (ti-2 0)) = -(tc-2 0) = -71 +2 tan lt


^=0+ 2 2
=>
dt 1+f2 1+f2
2
dy
l+t2
dy = dt_ =
= -1.
dx dx 2
dt 1+f2

LEVEL-2

EXAMPLE 6 Ifu = sin (m cos 1x),v = cos (m sin 1 x), prove that — =
dv
SOLUTION We have,

w
u = sin (m cos-1 x) and v = cos (w sin-1 x)

=> sin lu=m cos-1* and cos-1y =m sin-1*

Flo
=> sin -1 u + cos-1 y = w (cos-1 * + sin -1 *)
. -1 -1 mn

ee
=> sin u + cos v = . -1 -1 7C
•.* sin * + cos * = —
2 2

Fr
Differentiating both sides with respect to v, we obtain
1-u2
1 du 1 dll
for
ur
= 0 => — =
dv
Vi-*2* Vi-2
ks

EXAMPLE 7 /f* = sec 0 - cos 0 and y = sec” 0 - cos” 0, prove that


Yo
oo

(*2+4)(i?) ="2(y2+4>
B
re

SOLUTION We have,
ou

* = sec 0 - cos 0 and y = sec” 0 - cos” 0


ad
Y

— = sec0 tan 0 + sin 0 and, — =n sec” 0 tan Q + n cos”-10 sin 0


dQ dQ
nd

^ = tan 0 (sec 0 + cos 0) and, ^ ® (sec” 0 + cos” 0)


Re

------*v
Fi

dy _ dy/dQ
dx dx/dQ
dy _ntan0(sec”0 + cos”0) _ sec”0 + cos”0
=>
dx tan 0 (sec 0 + cos 0) sec 0 + cos 0
dy _^2 (sec”0 + cos”0)2
dx (sec 0 +cos 0)2
^ dy^\ _ 2 (sec” 0 - cos” 0)2 + 4 sec” 0 cos” 0
=> [v (a + b)2 =(a-b)2+ 4:ab]
V
dx y (sec 0-cos 0) +4sec0cos0

=> dy n2
\ dx

=> (x2 + 4) ^ dy] = «2(y2 + 4)


V. dx

ReadYourFlow.COM
DIFFERENTIATION 11.103

EXERCISE 11.7
LEVEL-1
dy
Find —, when
dx
, Tr~x = at2 and y = 2 at [NCERT]
2. x = a (0 + sin 0) and y = «(1 - cos 0)
>^7 x = a cos 0 and y = b sin 0 [NCERT]
A Q
4. x = ae (sin 0 - cos 0), y =ae (sin 0 + cos 0)
[CBSE2014]
0x =b sin2 0 and y =a cos2 0
6. x = a(l - cos 0) and y = fl (0 + sin 0) at 0 = -^ [NCERT]

el + e~l t -t
e i
7. x = and y =------

w
2
3 at 3at2
8. x = and y =

Flo
l+f2 1+t2
9. x=a (cos 0 + 0 sin 0) and y = a (sin 0 - 0 cos 0) [NCERT]

ee
0 -0 0-U

Fr
10. x = e 0 + — and y = e [NCERT EXEMPLAR]
0
21 1 -t2 or
ur
11. x = and y =
l+t2 i+(2'
f
ks

-1 1
12. x = cos = and y = sin ,teR
Yo

yi+f 2 *
oo
B

1-f2 , 2t
13. x = and y =
re

l+i2 l+f2
ou

30^1
ad

dy
14. If x = 2 cos 0 - cos 2 0 and y = 2 sin 0 - sin 2 0, prove that ~ = tan [CBSE2013]
l 2
Y

cos 2f sinlt ,provethat^=-i^


15. If x = e and y = e [NCERT EXEMPLAR]
d
Re

dx x log y
n

2 7T
Fi

16. If x = cos t and y = sin f, prove that at f = —


dx V3 3
1
17. If x = a f + - and y =a \ t — prove that — = —
b dx y

-1 2t -1 2t
18. If x = sin and y = tan , - l <t <1, prove that — = 1
1 +f2 1-t2 dx

sin 3 f ^ cos 3 *r dy
[NCERT]
19. If x = ^cos 21 ' ^ ^cos 21 ,find —
dx

20. Ifx= U + - ,y=a f , find d}/


t dx

1+t2 2t dy
21. If x = fl 2 andy = , find — [CBSE2005]
1 -t 1 -t2 dx

ReadYourFlow.COM
11.104 MATHEMATICS-XII

dy
22. If x =10 (f - sin t), i/ = 12 (1 - cos t), find^. [NCERT]
dx
23. If x = (0 - sin 0) and, y = rt (1 + cos 0), find ^ at 0 = —.
[CBSE 2011]
dx 3
24. If x = sin 2f (1 + cos 2f) and i/ =b cos 2f (1 - cos 2f), show that atf =-, ^ = -.
4 dx a
[CBSE 2014,2016, NCERT EXEMPLAR]
25. If x = cost (3 - 2cos21) and y = sin f (3 - 2 sin 2 f) find the value of — at f = —.
dx 4
[CBSE 2014, NCERT EXEMPLAR]
1 + logf ^ _ 3 + 2 logi
26. If x = , find —. [NCERT EXEMPLAR]
f2 t dx

w
27. Ifx = 3 sin f-sin 3f,y = 3cosf - cos 3f, find —atf = —. [NCERT EXEMPLAR]
dx 3
2t It
28. If sin x =
1+f2'
tany =

Flo
1-f2
, find —.
dx
[NCERT EXEMPLAR]

ee
ANSWERS

Fr
1 0 b
1. 2. tan — 3. — cot 0 4. cot 0
t 2 n
for
ur
a 2t
5. — 6. 1 7. — 8.
b y i-f2
ks
Yo

-2o(92-Q3 + e + i) x
oo

9. tan 0 10. c 11. - 12. 1


(03 + 02 + 0 -1) y
B

i
re

f+
f2-i a t i+f2
13. 19. - cot 3 f 20. 21.
ou
ad

2f .a - 1 2at
a(t+t
Y

t
6 f
nd
Re

22. — cot 23.-73 25. 1 26. f


5 2
Fi

1
27. —t= 28. 1
73
HINTS TO NCERT & SELECTED PROBLEMS
1. We have.
x = at2 and y = 2at => — = 2at and — = 2fl
* dt dt
d\£ _ dy/dt _ 2a_ _ 1
dx dx/dt 2at t
3. We have.
x = a cos 0 and y = sin 0
dx . dy
=> — - -flsin0, — = b cos 0
rf0 de
dy _ dy/dQ b cos 0 b
— cot 0
dx dx/dQ - sin 0 n

ReadYourFlow.COM
DIFFERENTIATION 11.105

6. We have.
x = fi (1 - cos 0), y = a (0 + sin 0)
dx
=> —= a sin 0, = a (1 + cos 0)
dQ dO
2 0
2 cos
dy _ dy/dQ _ n(l + cos 0) 0
-— = cot
dx dx/dQ a sin 0 0 0 2
2 sin cos
2 2
'dy
=> = cot — =1.
{dxM = ” 4
2
9. We have.
x = a (cos 0 + 0 sin 0), y = a (sin 0-0 cos 0)
dx dy
=> = 7? (— sin 0 + sin 0 + 0 cos 0), = a (cos 0 - cos 0 + 0 sin 0)

w
dQ
dx dy =
=> --- = (7 0 cos 0, 77 0 sin 0
dQ dQ
dy _ dy/dQ
dx dx/dQ
77 0 sin 0
77 0 COS 0
= tan 0

Flo
ee
COS 27 sin 27

Fr
15. We have, x = e ,y = ^
dx cos 27 (-2 sin 2f)/ — = esin2t (2 cosit)
=> = 7? for
ur
dt
dr
=> — = - 2x sin 27, — = 2y cos 27
dt dt
ks
Yo

cos 27 sin 27
dx dy V V = <? ,y = e
oo

=> = - 2x log y, — = 2y log x


dt dt => log v = cos27, logy = sin 27
B
re

rfy _ rfy/7f7 _ 2y logx Vog*


tfx dx/dt -2x log y xlogy
ou
ad

ALITER We have,
Y

cos 27 sin 27
x=e and, y = e
nd
Re

=> logx = cos27 and, logy = sin 27


=> (logx)2 + (logy)2 = cos2 27 + sin2 27
Fi

=> (logx)2 + (logy)2 =1


Differentiating both sides with respect to x, we get
2 (logx) - + 2 (logy) --f = 0 ^ dy= ylog^
x y dx dx xlogy
19. We have.
• 3 7f
sin cos 3 7x
x >y =
^cos 27 ^/cos 27
1/2
=> x = sin 7 (cos 27)' / y = cos37(cos27) 1//2
2
dx 3 sin 7 cos 7 3 1 - 3/2 7/
=> + sin' ^ x _ 2 ^cos ^0 — (cos 27)
dt y^cos 27 dt
9
dy _ - 3 cos- 7 sin 7 1 -3/2 d
+ cos 3 x7 x - — (cos 27) — (cos 27)
dt ^/cos 27 dt

ReadYourFlow.COM
11.106 MATHEMATICS-XII

2
dx 3 sin t cos t sin 3 t sin 2t dy _ - 3 cos2 t sin t cos3 t sin 2t
=> + +
dt ^cos 2t (cos 20 37 2 ' dt ^cos 2t (cos 2t) 3/2
dx _ 3 sm2 t cost cos 2t + sin3 f sin 2f dy _ - 3 cos2 f sin i cos 2t + cos3i sin 2t
=>
dt (cos 203/2 ' dt 3/2
(cos 20
dx 3 sin2 t cos t(l -2 sin2 0 + 2 sin4 t cos t
=>
dt 3/2
(cos 20
dy _ ~ 3 cos2 t sin t (2 cos2 f -1) + 2 cos4 £ sin f
dt 3/2
(cos 20
rf.t _ 3 sin2 t cos f - 4 sin4 f cos t dy _ -4 cos41 sin t + 3 cos21 sin t
(cos 20 372 ' dt 3/2
(cos 20

low
3 3
dx _ sin t cos t (3 sin f - 4 sin t) dy sin t cos t (4 cos t - 3 cos t)
dt (cos 20 372 3/2
(cos 2t)
dx _ sin 2t sin 3t dy _ - sin 2t cos 3f

ee
dt 2 (cos 20 37 2 ’ dt (cos 20 3/2
rF
Fr
dy _ dy/dt _ - sin 2t cos 3f
= - cot 3t
dx dx/dt sin 2t sin 3f for
22. We have, x = 10 (f - sin t)r y = 12 (1 - cos t)
u
=> ^ = 10 (1 - cos 0 / — = 12 sin f
ks

dt dt
Yo
oo

dy _ dy/dt _ 12 sin f 24 sin t/2 cos t/2 6 t


= — cot —
B

dx dx/dt 10 (1 - cos 0 20 sin2 t/2 5 2


re

11.11 DIFFERENTIATION OF A FUNCTION WITH RESPECT TO ANOTHER FUNCTION


ou
ad

So far we have discussed derivative of one variable, say, y with respect to other variable, say, x.
Y

In this section, we will discuss derivative of a function with respect to another function.
Let u=f (.r) and v = g (x) be two functions of x. Then, to find the derivative off (x) with respect to
nd
Re

g (x) i.e., to find — we use the following formula


Fi

dv
du _ du/dx
dv dv/dx
Thus, to find the derivative of f(x) with respect to g(x), we first differentiate both with respect
to x and then divide the derivative of /(x) with respect to x by the derivative of ^ (x) with
respect to x.
Following examples will illustrate the procedure.
ILLUSTRATIVE EXAMPLES

LEVEL-1

EXAMPLE 1 Differentiate log sin x with respect to fcos x.


SOLUTION Let u = log sin x and v = ^jcos x. Then,
du L , dv sm x
— = cot x and — =
dx dx 2 ^jcos x

ReadYourFlow.COM
DIFFERENTIATION 11.107

du _ du/dx cot a:
--2 yjcos x cot .r cosec x
dv dv/dx sinx
2 ^cos x

-1 fl + 2x^l with respect to -Jl + 4x2.


EXAMPLE 2 Differentiate tan
1 -2x

-1 1 + 2x and v = ■^1 + 4x2. Then,


SOLUTION Let u = tan
1 -2x

u = tan -1 1 + tan 1 2x and v = -Jl + 4x2


du 2 dv 1 4x
=> and x 8x =
dx 1 + 4X2 dx 2^1 + 4x2 f+4*2

w
2
du
du = dx = 1 + 4x2 1
dv

Flo
4x 2x Jl + 4x2

ee
dx ■Ji + fe2

Fr
EXAMPLE 3 Differentiate tan
-1 f+x2 -1 with respect to tan 1 x, x * 0.
for
ur
x
ks

f+x2-!
Yo

-1 -1
oo

SOLUTION Let « = tan and y = tan x.


x
B
re

Putting x = tan 0, we get


ou

f + X2~l
ad

sec 0 - 1 _1 1 - cos 0
u = tan
-1 = tan -1 = tan
Y

x tan 0 sin 0
nd
Re

2 sin2 (0/2) 0 1 -1
u = tan -1 > = tan -1 tan - 10 =
Fi

=> - tan ~ x
2 sin (0/2) cos (0/2) 2J 2 2

Thus, we have
u = — tan 1 x and z; = tan 1 x
2
du = 1 1 dv 1
=> 2and &
dx 2 i+x 1 +x2
du du/dx y- x C1 + x2) = y-
dv dv/dx 2 (1 + x2) 2

2x
EXAMPLE 4 Differentiate sin
-1 - with respect to tan -1 x , -1 < x < 1.
1+x

. -1 2x -1
SOLUTION Letu = sin and v = tan x.
l+x2

ReadYourFlow.COM
11.108 MATHEMATICS-XII

Putting x = tan 0, we get


2 tan 0
u = sin -1 = sin
-1
(sin 20)
1 + tan2 0

u = 20 = 2 tan 1 x n n K 71 0 _ 7t
=5> V - 1 < X < 1 => -- <0 <-=> <2 0 <-
4 4 2 2
Thus, we have
w = 2 tan “ 1 x and v = tan “ 1 x
du 2 dv 1
and — =
dx 1 + x2 dx 1 + x2
du _ du/dx 2/1 + x2
= 2

w
dv dv/dx 1/1 + x2

EXAMPLE 5 Differentiate xx zuith respect to x log x.

SOLUTION Let u = x* and v = x log x. Then,

Flo
ee
u = x * = elogxx = exlogx and v = x log x

Fr
=> du - e x logb x x —
d (x
t log
\ x)\ andj —
dv = xx
— +1 x log x for
ur
dx dx dx x
=> = xx (1 + log x) and — = 1 + log x
ks

dx dx
Yo
oo

du _ du/dx = xx (i + log -y) = Yx


B

dv dv/dx (1 + log x)
re

ALITER We have,
u = xx => log u = x log x = v =>u = ev
ou
ad

du d . v v du x
Y

— = —(e ) = e = u => = X .
dv dv dv
nd
Re

EXAMPLE 6 Differentiate tan 1 •


f + x2-f-x2= • zuith respect to cos -1 x 2 .
Fi

-Jl + x2 +-Jl -X
-Jl +X2 -yjl -X2
SOLUTION Let tan -1 < and v = cos—1 x 2 .
Jl+x2 +J1-X2
2
Putting x = cos 0, we get
+ cos 0 - ^/l - cos 0
u = tan -1
+ cos 0 + ^1 - cos 0
-J2 cos2 0/2 - y/2 sin2 0/2
=> u = tan -1
■J2 cos2 0/2 + ^2 sin2 0/2

-1 cos 0/2 - sin 0/2


=> u = tan
cos 0/2 + sin 0/2

ReadYourFlow.COM
DIFFERENTIATION 11.109

1 1 - tan 0/2 0
=> u = tan Dividing numerator and denominator by cos —
1 + tan 0/2 2

-1
=> u = tan

K 1 2
[v X = COS 0 0 = cos

-1
Now, V = cos

du du/dx

w
dv dv/dx
-1

Flo
EXAMPLE 7 Differentiate x with respect to sin x.
-1 -1

ee
SOLUTION Let m = x sin * and v = sin x. Then,

Fr
sin -1 a:
U — X

sin - 1‘ -r. log x


for
ur
=> u — e
du ^sin-1 x.logx _d {sin 1 x. log x}
=>
ks

dx dx
Yo
oo
B
re
ou
ad
Y
nd
Re
Fi

sin ^ x
=X

dx

1 with respect to cos 1 ^2x -Jl - x2 .


EXAMPLE 8 If x e -j= , 1 , differentiate tan
V2
[CBSE 2014, NCERT EXEMPLAR]

SOLUTION Let u = tan

Let x = sin 0. Then,


1 -4= <sin 0<1 => —<0< —
x e —f= , 1 =>
V'2 V2 4 2

ReadYourFlow.COM
11.110 MATHEMATICS-XII

Now,

1 yjl-sm2Q
=> u = tan = tan 1 (cot 0) = tan -1 tan ^-0
sin 0 2

*-0 Tl „ Tl _ 71 „ 7T
=> U = — < 0 < — => 0 <---- 0 < —
2 4 2 2 4
n
=> u =---- sin
2
du
=> ---- = 0-

w
Flo
ee
=>

Fr
K
v =---- sin 1 (sin 2 0) for [v x = sin 0]
ur
2
n 1
=> v ---- sin |sin (ti - 2 0)}
2
ks
Yo

^ -(jr-2 0)
oo

V •.•-<0<-=> O<7i-20< —
2 4 2 2
B

=> v -— + 20 = - — + 2 sin 1 x
re
ou

=>
ad
Y
nd
Re
Fi

2x
EXAMPLE 9 Differentiate tan 1 with respect to sin -1 'if
1+x2

(i) x e (-1,1) (ii) x e(l, co) (iii) x e (— oo , — 1)

SOLUTION Let u = tan 1 2x 2x


- and v = sin 1
1 -x 1 + x2 ‘

Putting x = tan 0, we get


2 tan 0 2 tan 0
u = tan- 1 and v — sin 1
1 - tan2 0 1 + tan2 0

=> u = tan 1 (tan 2 0) and v = sin 1 (sin 2 0)

ReadYourFlow.COM
DIFFERENTIATION 11.111

(i) WTie?? * e (-1 ,1).


a: £(-1,1) and x = tan 0
-1 < tan 0 < 1 — — < 0 < — => - — < 2 0 < —
4 4 2 2
tan 1 (tan 2 0) = 2 0 and sin 1 (sin 2 0) =20
=> u = 2 Q and = 2 0
u = 2 tan- 1 x and t» = 2 tan- 1 x [•.• x = tan 0 => 0 = tan 1 x]
du 2 , dv 2
=>
dx l+x ?ands = l + x2
2
du
du _ dx _ l + x2
= 1
dv dv 2

w
dx i + x2
(ii) When x e(l, co).
xe(l,oo) and x = tan 0
l<tan0<oo=> - <0<-=> —<20<ti

Flo
ee
4 2 2
u = tan 1 (tan 2 0)= tan 1 {-tan (tt - 2 0)} = tan 1 {tan (2 0 - tt)} = 2 0-rc

Fr
=> u = 2 tan -1 X - 7T for [•.• 0 = tan- 1 x]
ur
du 2 2
-0 =
dx l+x2 l + x2
ks

and. v = sin 1 (sin 2 0)= sin 1 {sin (jt-2 0)) = k-2 0 = 7r-2tan ^x
Yo
oo

^ = 0- 2 -2
=> =
B

dx l + x2
l+x2
re

2
du
du = dx = l + x2 = -1
ou
ad

dv dv -2
Y

dx 1 +x2
nd
Re

(iii) When x e(- oo, -1).


We have.
Fi

x = tan 0 and x e (- co, -1) => - co < tan 0 < -1 => <0<- — => - n <2 6 < - —
2 4 2
u = tan- 1 (tan 2 0)= tan- 1 {tan (ti + 2 0)} = 7t + 2 0 = re + 2 tan- 1 x
du 2 2
— = 0+
dx l+x 2 1+*2
and. v = sin- 1 (sin 2 0)= sin 1 {- sin (tt + 2 0)}
v = sin-1 (sin (-7t - 2 0)) = - rt - 2 0 = - r: - 2 tan 1 x
dv 2
=>
dx 1 + x2
2
du
du _ dx _ l + x2
= -1
dv dv 2
dx 1 + x2

ReadYourFlow.COM
11.112 MATHEMATICS-XII

,r 1 -1 3x - x3 2x
EXAMPLE 10 If- -j^<x< -j=, differentiate tan ivith respect to tan -1
1 - 3x2 1-x2

3x-x3 2x
SOLUTION Let u = tan 1 and v - tan 1
1 - 3x2 1-x2
Putting x = tan 0, we have
u = tan ~ 1 (tan 3 0) and v = tan ~ 1 (tan 2 0)
1 1
----- 7= < X <-==> -4= < tan 0 < -4= => ----71< 0A< -71
u = 3 Q and v = 2 Q V3 y[3 V3 73 6 6
- — < 3 0 < - and --<20<-
2 2 3 3

u = 3 tan -1 x and v = 2 tan -1 x

w
=>
du 3 dv 2
=> and

Flo
d* 1 + x2 dx i + x2
3

ee
du
du _ dx_ _ 1 +x2 3

Fr
=>
dv dv 2 2
dx i+x2
for
ur
EXERCISE 11.8
ks

LEVEL-1
Yo

1. Differentiate x 2 with respect to x 3 .


oo

2. Differentiate log (1 + x2 ) with respect to tan —1 x.


B
re

Differentiate (log x)'v with respect to log x.


ou

“1 / 2 _i
ad

4. Differentiate sin Jl-x with respect to cos x, if


Y

(i) x e (0,1) (ii) X £ ( — 1, 0)


-1 (4x - 4x2) with respect to - 4x2, if
nd

Differentiate sin
Re
Fi

... 1 1 .... ( 1 1 1 1
(i) xe - — (n) x e —j= , - (iii) xe -
2^2' 2 f2 2f2 2 2' 2 f2

-1 f + x2-l 2x
6. Differentiate tan with respect to sin -1 , if-1 <x <1, x * 0.
X 1 +x2
ICBSE2014,2016J
1
7. Differentiate sin -1 (2x -Jl - x2) with respect to sec -1 .if

(i) xe(0,l/72) (ii) x e (1/f2,1)


sin x cos x
8. Differentiate (cos x) with respect to (sin x)

-1 2x 1-x2
9. Differentiate sin with respect to cos -1 , if 0 < x < 1.
l + x2 l + x2

ReadYourFlow.COM
DIFFERENTIATION 11.113

-1 1 + ax 2 2
10. Differentiate tan wiith respect to -jl + a x .
1 - ax

-1 ^ 2x -Jl - x2 with respect to tan


11. Differentiate sin

-1 2x
12. Differentiate tan - with respect to cos , if 0 <x <1.
1 -x

-1 x—1 —1 3 1 i
13. Differentiate tan with respect to sin (3x - 4x ), if — < x < — .
x+1 2 2

-1 COS X -1
14. Differentiate tan with respect to sec x.

low
1 + sin x

-1 2x -1 2x
15. Differentiate sin with respect to tan y , if -1 < x <1.
i + *2

ee
rF
Fr
_i 3
16. Differentiate cos (4x - 3x) with respect to tan
for
u
17. Differentiate tan 1 X 1
== with respect to sin < X < -7= .
ks

V2
Yo

1-X
oo

\ [CBSE2014]
B

-1 - x2 with respect to cot -1 x


18. Differentiate sin ,ifO <x <1.
re

U^2
v ' /
ou
ad

-1 2 ax -Jl - fl2 x2 with respect to 2 2 .r 1 1


19. Differentiate sin -a x , if —t= <ax < —f=.
Y

J2
1 1 -x
nd
Re

20. Differentiate tan w ith respect to ^'1 - x2 , if -1 < x < 1.


1+x
Fi

ANSWERS
2 x—1
1. — 2. 2x 3- ^(logx) {1 + log X • log (log x)}
3x
1 1
4. (i) 1 (h) -1 5. (i) -- (ii) - (iii) -
x X X
6.1 7- (i) 2 (ii) -2
4
sin x
(cos x) {cos x • log cos x - sin x • tan x} 1
8. 9. 1 10. 11. 2
COS X / 2 2
(sin x) {- sin x log sin x + cos x • cot x} ax J1 + a x

- x a/*2"1 1
12. 1 13. 14. 15. 1 • ■ 3 17. -
3 (1 + x2) 2 2

2
18. 1 19. - 20.
ax x (1 + x2)

ReadYourFlow.COM
11.114 MATHEMATICS-XII

11.12 DIFFERENTIATION OF DETERMINANTS


In the previous sections, we have studied differentiation in detail. In this section, we shall
discuss the differentiation of determinants.
To differentiate a determinant, we differentiate one row (or column) at a time, keeping others
unchanged.
For example, if
f(x) g(x)
A(x) = , then
u(x) v (x)

f-{AWi = /'(*) g'(x) f (x) g (x)


+
dx u (x) V (x) u' (x) v' (x)
Also,

w
f'(x) g (x) + /(x) g'(x)
4- {A(*)( =
dx t/' (x) v (x) u (x) v' (x)

Flo
Similar results hold for the differentiation of determinants of higher order. Following examples
will illustrate the same.

ee
Fr
ILLUSTRATIVE EXAMPLES

LEVEL-2 or
ur
f
2 ab
x+a ac
ks

2
EXAIN
Yo

V fix) = ab x + b^ be ,find f' (x).


oo

ab be 2
x+c
B
re

SOLUTION We have.
x + a2 ab
ou

ac
ad

2
Y

fix) = ab x+b be
ac be 2
x+c
nd
Re

2
Fi

1 0 0 x+a ab ac x + a2 ab ac
/' (x) = ab x + b2 be + 0 1 0 + ab x + b2 be
2 2
ac be x+c ac be x + c 0 0 1

x + b2 be 2 ac 2 ab
x+ a x+a
=> /'(*) = 2 + 2 + 2
be x+c ac x+ c ab x+b

=> j(x + b2) (x + c2)-b2 c2j + |(x + ^ (x + c2) -a 2 c 2


f'(x) =

+ j(x + a)2 (x + b2) -a 2,2


b

=> /' (x) = x2 + x (b2 + c2) + x2 +x {a2 + c2) + x2 + x {a2 + b2)

=> f'{x) = 3x2 + 2x (a2 +b2 + c2).

ReadYourFlow.COM
DIFFERENTIATION 11.115

EXAMPLE 2 Iffr (x), gr (x) and hr (x) ;r =1,2, 3 are polynomials in x such that fr (a) = gr(a) = hr (a);
r =1,2, Sand
fl(x) f2(x) f3 (x)
F(x) = 8l(x) g2(x) g3(x) , find F' (x) at x = a.
h\ (x) h2 (x) h3(x)
SOLUTION We have.
Mx) f2(x) f3 (x)
F(x) = gl(x) g2(x) g3(x)
h (x) h2 (x) h3(x)

fl'(x) f2 (x) f3'(x) fl (x) f2 (x) f3 (x) A (x) f2 (x) f3 (x)


F '(x) = gl (x) g2 (x) g3(x) + gl'(x) g2'(x) g3'(x) + gl (x) g2{x) g3(x)

low
hi (x) h2 (x) h3{x) h (x) h2 (x) h3(x) h' (x) h2 (x) h3'(x)

A'(«) A'(«) A' (fl) A (fl) A (fl) /s (A A (fl) A (fl) A (fl)


gl(«) 82(0) g3 (a) + gl'(a) gi'W 83 +

ee
F'(fl) =
rF
Fr
hi (a) h2 (a) h3(a) h (a) h2 (a) h3(a) A'(«) A'(«) /j3'(fl)

A(«) A(«) A(«) A (fl) A («) A (A


/'i(«)/'2(«) /'3(«)
for
=> F'(fl) = gl («) g2 («) g3 (fl) + g7! («) g'2 (fl) g' 3 (fl) + A (fl) A(fl) A (fl)
u
A(fl) A (a) A(«)
ks

gl g2 (a) g3(a) gl' («) g2(a) 83 (a)


Yo
oo

[Using: fr (a) =gr (a) = hr (a) ;r =1,2, 3]


B

=> F'(a) = 0 + 0 + 0 = 0 [•.• Two rows are identical in each of the determinants]
re

EXAMPLE 3 Iff (x), g (x) and h (x) are three polynomials of degree 2, then prove that
ou
ad

/ (A g (A h (x)
Y

<t»(A = /' (A g' (A h' (A fs a constant polynomial.


/"(A g" (A h" (x)
nd
Re

2 2 2
SOLUTION Let / (x) = ^ x + fl2 * + ^3, g (x) = A x +b2 x + b3 and h (x) =clx + c2 x + c3.
Fi

Then,
/' (A = 2 ^ x + a2, ^ (x) = 2b1x + b2 and h ' (x) = Iq x + c2
/"(A = 2q, g" = 2}}1,h"(x)=2c1
and. /'"(A = g'"(A = h'" (x) = 0
In order to prove that (j) (x) is a constant polynomial, it is sufficient to show that <j>" (x) = 0
for all x.
Now,
/ (A g (A h (A
<KA = /'(A g'(A h'(x)
/"(A g" (A h" (x)

/' (A g' (A h' (A / (A g (A h (A / (A g (A h (A


f(A = /'(A g'(A h' (x) + /"(A g"(A h"(x) + /'(x) g'(x) h' (x)
/"(A g" (A h" (x) /"(A g"(A /i"(A /'"(A g"'(A /r"'(A

ReadYourFlow.COM
11.116 MATHEMATICS-XII

/ (x) g (x) h (x)


=> (|)' (x) = 0 + 0 + f' (x) g'(x) h'(x) [Using (i)]
0 0 0

=> <K(x) = 0 + 0 + 0 = 0 for all x.


=> <j>(.r) = Constant for all x.
Hence, <J>(x) is a constant polynomial.

/ g h
EXAMPLE 4 f> grh we differentiable functions of x and A = (x/)' (x-£)' (x/?)'
(x2/)" (x2 g)" (x2 h)"

f g h

low
prove that A' = / g: h'
(x3 f")' (x3 g"y (x3h"y

SOLUTION We have,

ee
(xf)' = xf'+f, (xg)' = xg + g, (xh)' = xh' + h
rF
Fr
(x2 /)' = x2 /' + 2x/ , (x2 g)' = x2 g' + 2xgr (x2 h)' = x2h' + 2xh

(x2/)" = x2 / " + 4x /' + 2/, (x2 g)'' = x2 g" + 4xg' + 2g


for
ou
and. (x2 h)" = x2 h" + 4x/j' + 2h
ks
oo

/ g h
Y
B

A = xf'+f xg'+g xh' + h


re

x2 /" + 4x/' + 2/ x2 g" + Axg' + 2g x2 h" + 4xh' + 2h


ou
ad

/ g h
Y

=> A = xh' Applying R2 -> R2 ~


xf
and -+ R3 - 2Ri
nd
Re

x2 /” + 4x/' x2g" + 4xg' x2h" + 4xh'


Fi

f g h
=> A = xf' xg' xh' Applying R3 ->R3 -4K2
x2f" x2g" x2h'

/ g h
=> A = x f g' h' [v Taking x common from R2]
x2f" x2g" x2h"

/ g h
A = f g1 h' [Multiplying R3 by x]
x3/" *3r x3h"

ReadYourFlow.COM
DIFFERENTIATION 11.117

/' S' h' f S h f S h


A' = /' S' h' + /" s" h" + f S' /z'

x f x g x h x3f" x3 g" x3 h" (x3/'')' (*3 S")' U-3^")'

f S h f S h
=> A' = 0 + 0 + /' S' h' f S' h'
{x3 f")' (xV)' (*3'>")1 (x3f)' (x3 g’)' (x3 h"Y
/(a:) gW ftW f'(x) g'(x) h'(x)
EXAMPLE 5 If y = l m n , prove that ~ = l m n [NCERT]
dx
a b c a b c

f{x) g(x) h(x)

w
SOLUTION We have, y = 1 m n
a b c

Flo
-r; (/(*)) 7- (s (x)) -j- (h (x»

ee
dx dx dx /(x) g(x) h (x) f(x) g (x) h (x)
fl _

Fr
1 m n 0 0 0 + 1 m n
dx
a b c a b
for c 0 0 0
ur
/'(x) g'(x) h'(x)
ks

=> _ / m n
Yo

dx
oo

a b c
B
re

_____________________________________VERY SHORT ANSWER QUESTIONS (VSAQs)


Answer each of the following questions in one ivord or one sentence or as per exact requirement of the
ou
ad

question:
Y

1. If/ (x) = logt, (logp x), then write the value off' (e).
nd
Re

2. If / (x) = x + 1, then write the value of— (fqf) (x).


dx
Fi

dy
3. If /' (1) = 2 and y =f (log,, x), find — at x = e.
dx
4. If / (1) = 4, /' (1) = 2, find the value of the derivative of log (/ (ex)) with respect to x at the
point x = 0.
5. If /' (x) = J2x2 -1 and y = f (x2), then find — atx =1.
v dx
6. Let g (x) be the inverse of an invertible function / (x) which is derivable at x = 3. If / (3) = 9
and/' (3) = 9, write the value of g' (9).
7. If y = sin - 1 (sin x), - - < x < -. Then, write the value of — for x e f - - , - .
2 2 dx {2 2)
8. If — < x < — and y = sin ~ 1 (sin x), find —.
2 2 J dx
9. If ji < x < 2ti and y = cos- 1 (cos x), find —.
dx

ReadYourFlow.COM
11.118 MATHEMATICS-XII

2x
10. If y = sin -1 dV for x > 1.
, write the value of —
1 +x2 dx

11. Iff (0) =/ (1) = 0,/' (1) = 2 and y =/ (eA) e write the value of — at x = 0.
dx
12. If y = x | x I, find — for x < 0.
dx
13. If y = sin- 1 x + cos 1 x, find —.
dx
14. If x = fl (0 + sin 0), y = fl (1 + cos 0), find — .
dx
1 - cos 2x rfy
15. If - ^ < x < 0 and y = tan 1 , find —.
1 + cos 2x dx

low
16. If y = xx, find — at x = e.
dx
1 -x
17. If y = tan -i dy
, find —.
1+x dx

ee
dy
Ify = log„ x, find -f-.
rF
Fr
18.
dx
19. If y = log ^tan x, write
for
ou
1-x2 , 2
If y = sin 1 -1 1-X dy
ks

20. + cos , find —.


l+x2 1 + x2 dx
oo
Y

x + 1 + sin 1 x —1 dy
B

21. If y = sec -i , then write the value of —.


x —1 x + ly dx
re

If | x | < 1 and y = 1 + x + x2 +... to oo, then find the value of


ou

22.
ad

dx
Y

If w = sin 1 2x -1 2x
23. and v = tan , where -1 < x < 1, then write the value of
nd

l + x2 1-x2
Re
Fi

du
dv
u(x)
24. If / (x) = log • u (1) =v{\) and u' (1) = v' (1) = 2, then find the value off' (1).
v(x) '

25. If y = log | 3x | , x * 0, find dy

26. Iff (x) is an even function, then write whether/' (x) is even or odd.
27. If / (x) is an odd function, then write whether/' (x) is even or odd.
28. Write the derivative of sin x with respect to cos x. [CBSE2014]
^ANSWERS

1. 1 2. 1 3. ^ 4. 2 5. 2
e 2
1 -2
6. 7. 1 8. -1 9. -1 10.
9 l + x2

ReadYourFlow.COM
DIFFERENTIATION 11.119

14. - tan
e
2 •2. -2x 13. 0 15. -1
2
1 1
16. 2 ee 17. 18. 19. cosec lx 20. 0
1 + x2 x loge a
1 1
0 22. 23. 1 0 25. -
(I-*)2 .r
odd 27. even 28. - cot*
MULTIPLE CHOICE QUESTIONS (MCCte)
If / (x) = log 2 (log .t), then f' (x) at x = e is
(a) 0 * (b) 1 ' (c) 1/e (d) 1/2e
The differential coefficient of / (log x) with respect to x, where / (x) = log x is
logx -1
(a) 7” (b) (c) (x log x) (d) none of these
logx X

w
The derivative of the function cot 1 (cos 2 x) 1/2 • at x = tc/6 is

(a) (2/3)
1/2
(b) (1/3)
1/2

Flo
(c) 3
1/2
(d) 6
1/2

ree
-1
Differential coefficient of sec (tan x) is

F
x 1 x
(a) (b) -t (c) (d)
v*+*2 or i T
ur
1 + x2 V1 + *
f
-1 1 + sin x
ks

If / (x) = tan 0 < x < n/2, then/' (n/6) is


1 - sin x
Yo
oo

(a) -1/4 (b) -1/2 (c) 1/4 (d) 1/2


B

Yr
1
re

6. If y = 1 + - , then — =
x dx
ou
ad

\X
1 1 1 (b) fl + -1 logfl +-
Y

(a) 1+ log 1 + -
X X x +1 X X
nd
Re

(c) fx + —1 • log (x + 1)
X
(d) [ x + — log 1 + - + 1
Fi

X x +1 X X X+1

If x-*7 = e* ~ y, then —^ is
dx
1+x 1 - log X logx
(a) (b) (c) not defined (d)
1 + log x 1 + log x (1 + log I)2
8
Given / (x) = 4x , then
1 1

wf[\
If x = cos 3 0, y = 0 sin 3 0, then J1 + — =

ReadYourFlow.COM
11.120 MATHEMATICS-XII

(a) tan2 0 (b) sec- 0 (c) sec 0 (d) | sec 01

10. Ify = sin , then d}L =


dx
(a)----- ^ 2 1 2
(b) (c) (d)
1 + xl 1 4- A'2 2 -x2 2 - A'2

1
11. The derivative of sec -1 with respect to + 3 x at x = -1 / 3
2 x2 +1
(a) does not exist (b) 0 (c) 1/2 (d) 1/3
12. For the curve -Jx + Jy = 1, — at (1/4,1 /A) is
v dx
(a) 1/2 (b) 1 (c) -1 (d) 2

w
13. If sin (x + y) = log (x + y), then — =
dx

Flo
(a) 2 (b) -2 (c) 1 (d) -1]
2x 2x dU

ee
14. Let U - sin -l and V = tan -l , then — =
1 + x2 1-x2 dV

Fr
1-x2 for
(a) 1/2 (b) x
ur
(c) (d) 1
1 + x2
ks

d COS X
15. tan -1 equals
Yo
oo

dx 1 + sin x
B

(a) 1/2 (b) -1/2 (c) 1 (d) -1


re

3/4
x x-2
16. equals
ou
ad

dx x+2
Y

2-l x2 + 1 2 -1
d

(a) —2 (b) 1 (d)


Re

(c) -5------
n

x -4 A- -4 x -4
Fi

dy =
17. If y = y sin x + y, then
dx
sin x sin x cos x cos x
(a) (b) (c) (d)
2^1 i -2y l-2y 2 y -1
dy _
18. If 3 sin (xy) 4- 4 cos (xy) =5, then —
dx
3 sin (xy) 4- 4 cos (xy)
(a) (b)
x 3 cos (xy) - 4 sin (xy)
3 cos (xy) 4- 4 sin (xy)
(c) (d) none of these
4 cos (xy) - 3 sin (xy)
dy .
19. If sin y = x sin (a + y), then — is
dx
sin a sin 2 (a + y)
(a) 2 (b)
sin a sin - (a 4- y) sin n

ReadYourFlow.COM
DIFFERENTIATION 11.121

2 sin2 ja-y)
(c) sin a sin (a + y) (d)
sin a
_1 2 —1
20. The derivative of cos (2x -1) with respect to cos xis
1 (d) \-x2
(a) 2 (b) (c) 2/x
2^?
21. If f (x) = ijx2 + 6x + 9, then /' (x) is equal to

(a) 1 for a: < - 3 (b) -1 for x < - 3 (c) 1 for all x e R (d) none of these
22. If / (.v) =| .t2 - 9x + 201 then /' (x) is equal to
(a) - 2 x + 9 for all x eR (b) 2x - 9 if 4 < x < 5
(c) -2.r + 9 if 4 <x <5 (d) none of these
23. If f (x) = yjx2 - lOx + 25, then the derivative off (x) in the interval [0, 7] is

low
(a) 1 (b) -1 (c) 0 (d) none of these
24. If f (x) = | x - 31 and g (x) = fof (x), then for x > 10, g' (x) is equal to
(a) 1 (b) -1 (c) 0 (d) none of these

ee
I+m \in + ii/ N« + /
I m n
rF
Fr
X X
25. If/M = 4, / , then /' (x) is equal to
x X X for
l+m+ n
(a) 1 (b) 0 (c) x (d) none of these
ou
1 1 1
ks

26. If,i/ = + i , b-a c-


- , then — is equal to
1 +Xa
-b
+X
c-b
1 +X
b- c
+X
a- c
1+x +X a dx H
oo

+ + c- 1
Y
B

(a) 1 (b) (a + b + c)
re

(c) 0 (d) none of these


27. If -x6 + yjl -y6 = fl3 (x3 - y3), then ^ is equal to
ou
ad
Y

/(c)^
\ x 2 (T7 (d) none of these
d
Re

6
1-y
n

y
Fi

dy
28. If y = log ^tan x, then the value of ^ at x = ^ is given by
1
(a) » (b) 1 (c) 0 (d) 2
(2 2^
— 1 —r—=
x —V dy
29. If sin log a then — is equal to
x2 + y2 dx
2 2
(a) (b) 1 (c) - (d) none of these
x +y X v
dy
30. If sin y = x cos (a + y), then is equal to
dx
2 ■ 2
cos (<? + y) cos a sin y
(a) (b) 2 (c) (d) none of these
cos a cos {a + y) cos a

ReadYourFlow.COM
11.122 MATHEMATCS-XII

32. If y = Jsin x + y, then — equals


v dx
cos .r cos x
(a) (b)
2./-1 l-2y l-2.y 2y-1
_ i sin x + cos x dy is equal to
33. If y = tan , then —
cos x - sin x dx
1

low
(a) 2 (b) 0 (c)l (d) none of these

ANSWERS
1. (d) 2. (c) 3. (a) 4. (d) 5. (d) 6. (a) 7. (d) 8. (c) 9. (d)

ee
10. (a) 11. (a) 12. (c) 13.
rF
(d) 14. (d) 15. (b) 16. (a) 17. (d) 18. (a)

Fr
19. (b) 20. (a) 21. (b) 22. (c) 23. (d) 24. (a) 25. (b) 26. (c) 27. (a)
28. (b) 29. (b) 30. (a) 31. (b) 32. (a) 33. (c)
for
ou
SUMMARY
ks

1. Let/(x) be a differentiable or derivable function on {a, b]. Then,


oo

f(x + h)-f(x) ^ f(x-h) -/(x)


Y

lim lim
B

•~(i)
/i -»0 h k->0 -h
re

is called the derivative or differentiation of / (x) with respect to x and is denoted by


ou
ad

/'(x) or — (/(x)) or, D/(x), where D= —


dx dx
Y

dV
2. If y =/(x), then — gives the slope of the tangent to the curve y = / (x) at point P.
d
Re

Kdxh
n
Fi

3. Following are derivatives of some standard functions:


(i) —(x ) = nx -1
dx dx
(iii) ^-{ax) = ax log^ a 1
(iv) T (log,, x) =
dx dx x
(v) A (i0grt *) =-J--- (vi) — (sin x) = cos x
dx x log,, a dx

(vii) — (cos x) = - sm x (viii) — (tan x) =sec2 x


dx
dx
(ix) -j— (cot x) = - cosec 2 x
dx (x) — (sec x) = sec x tan x
dx
(xi) — (cosec x) = - cosec x cot x 1
dx (xii) — (sin - 1 x) = -1<X<1
dx

ReadYourFlow.COM
DIFFERENTIATION 11.123

(xiii) — (cos 1 x) =
dx

(xiv) — (tan 1 x) =
dx
(xv) -^-(cot_1x)=-
dx

(xvi) — (sec- 1 x) = ■
dx

d -1
(xvii) — (cosec x) =
dx

w
. x>l

Flo
(xviii) — • sin 1 2x
dx l + x2

ee
Fr
, x <-l
for
ur
,x>0
d 1 (l-x2
ks

(xix) — < cos


dx l + x2
Yo

,x<0
oo

l + x2
B

2
re

, x < — 1 or x>l
d l + x2
(xx)
ou
ad

dx 2
,-l< x < 1
Y

l + x2
d
Re

1 1
; - < x < 1 or, - l<x< —
n

2
Fi

(xxi) jsin 1(3x-4x3)| = <

d | cos 1(4x3-3x)|
(xxii) dx 1 1
,if — <x< — or, -1 <x< —
2 2 2

1 1
x< or x >
3^ V3
d -1 3x -x
(xxiii) — tan
dx 1 -3x2

ReadYourFlow.COM
11.124 MATHEMATICS-XII

(xxiv) £ {sin (sin


= 1, if -1 < x < 1

(xxv) £ { cos(cos 1 x) = 1, if -1 <x<l

(xxvi) | (tan (tan ” 1 x) j = 1 for all x e R

(xxvii) — {cosec(cosec _ ^r)} = 1 for all A-eR-(-l/l)


dx { J

(xxviii) £ { sec (sec ^x) = 1 for allx e R -( -1,1)

(xxix) — {cot (cot ~ 1 x) 1 = 1 for all x e R


dx l J

w
-1, - 3jt/2 < x < - ji/2
1 , - k/2 <x < 7r/2
and so on

Flo
-1 , k/2 <x < 3nl2
1 , 37t/2 < x <5tt/2

ee
, 0 < X < 71

Fr
(xxxi) — {cos 1 (cos x) 1 = •
and so on
dx l J -1 , TC<X <2/71
for
ur
(xxxii) — < tan
dx {
-1
(tan x)J = j ,1 rnn —rt2 <x < —2 + nn ,neZ_
tc
ks

1 (cosec x)| = j 1 , - 7i/2 < x < 0 or, 0 x < tt/2


Yo

(xxxii:) f 1 cosec
oo

and so on
-1,7r/2 <x < tc or n<x < 3n/2
B

(xxxiv) A { sec -1 (sec x) j = | 1,0 < x < tt/2 or , k/2 < x < k
re

-1 , k < x < 3k/2 or, 3k/2 < x < 2k


ou
ad

(XXXV) A J cot -1 (cot x) j = 1 , (;t -1) k < x < «k ,n eZ


Y

4. Following are the fundamental rules for differentiation:


nd
Re

(i) — (Constant) = 0
Fi

dx

(ii) -f (c/M}=c -fvwi


dx dx
(iii) -f (/-(*) ±gM) = -f (fWl ± ~ te(.v)l
dx dx dx
r {fix) g (*)} =f(x) ~ {§<x)} + g(x) ft- {fix)}
(iv)
dx dx dx
g(x) ~ {fix)}-fix) 4-{gix)}
d fix)
(v) -f- = dx dx
dx) g(x)
te(x)l2
(Vi) t = 1
dx dx
dy

ReadYourFlow.COM
DIFFERENTIATION 11.125

(vix) ~ {fW}fe(A:)> = if Ml* w “ -f If M) + ‘“S / (*) ■ -f l? (*))


dx / (x) dx dx

(viii) Ifx = (j)(0 and y = v (0, then ^


ax dx/dt
du du/dx
(ix) If u and v are functions of x, then — = , , ,
dv dv/dx
5. Iff (x), g (x), u (x) and u (x) are function of x and A is a determinant given by
f(x) g(x)
A(x) = . Then,
u (x) v (x)

^{A(X)} = f'(x) g'(x) +


f(x) g(x)
dx u (x) v (x) u' (x) v' (x)

w
f'(x) g(x) fix) g' (x)
Also, — {A (x)} = +
dx u! (x) v (x) u (x) y' (x)

Flo
Similar results hold for the differentiation of determinants of higher order.

ee
Fr
for
ur
ks
Yo
oo
r eB
ou
ad
Y
nd
Re
Fi

ReadYourFlow.COM
Re
Fi ad
nd
Y

.
ou
Yo
re
B

_
oo
ur
ks
for
Flo
w Fr
ee

_
ReadYourFlow.COM
CHAPTER 12
HIGHER ORDER DERIVATIVES

12.1 DEFINITION AND NOTATIONS


dy
If y = f(x), then the derivative of y with respect to x, is itself, in general, a function of x and

low
can be differentiated again. To fix up the idea, we shall call — as the first order derivative of y

with respect to x and the derivative of with respect to x as the second order derivative of y
d2y d2y
with respect to x and will be denoted by —Similarly the derivative of —y with respect to x

ee
dx1 dx
rF
Fr
will be termed as the third order derivative of y with respect to x and will be denoted by -—y and
dx6
for
dny
so on. The order derivative of y with respect to x will be denoted by
u
dxn
ks
Yo

If y =/(x), then the other alternative notations for


oo

dy d2y d^y
- — — ... are
B

dx' dx2' dx3' ' dx"


re

1/2' vs* y»
yi'
ou

, y(n)
ad

y'' y"' y'"'


Dy, D2 y, D3 y, •••, D" y
Y

fix), f"{x), f"'(x), •••' fn(x)


nd
Re

(d"v)
Fi

The values of these derivatives at x = are denoted by y„ (a), y11 {a), Dny (a), fn{a) or.
{dxn 'x = a
ILLUSTRATIVE EXAMPLES
LEVEL-1
Type l ON PROVING RELATIONS INVOLVING VARIOUS ORDER DERIVATIVES OF CARTESIAN
FUNCTIONS

EXAMPLE 1 If y = sin 1 x, show that dhdx2= (1-xOx2,3/2 '


SOLUTION We have, y = sin 1 x.
On differentiating with respect to x, we get
dy = 1
dx

ReadYourFlow.COM
12.2 MATHEMATICS-XII

On differentiating again with respect to x, we get


d2y _ d 1 |(l-x2)-1/2} =-1(1
dx2 dx dx 2 dx

d2y -____________
1 x
=> (-2x) =
dx2 2(1-x2)3/2 (l-x2)3/2'
d2y
EXAMPLE 2 Ify = A cos nx + B sin nx, show that + n2y = 0.
dx2
SOLUTION We have, [CBSE 2001C]
y = A cos nx + B sin nx
On differentiating with respect to x, we get
— = - An sin nx + Bn cos nx

w
dx
On differentiating again with respect to x, we get
^y = - An2 cos nx - Bn2 sin nx = -n2 (A cos nx + B sin nx) =
dx2

Flo - n 2y

ree
d^L + n2y = 0.

F
dx2
d2y dy + mny = 0.
or
ur
EXAMPLE 3 Ify = Aemx + Benx, show that - (m + n) —
dx2 dx
sf
[NCERT, CBSE 2007, 2014]
k
Yo

SOLUTION We have, y = Aemx + Benx


oo

^ = Amemx + Bne nx
B

dx
re

d2y
=> = Am2 emx + Bn2 e"x
ou

dx2
ad

d2y
Y

dy + mny
- (w + n) —
dx2 dx
nd
Re

= (Am2 emx + Bn2 enx) - (m + n) (Amemx + Bne"x) + mn (Aemx + Benx) = 0.


Fi

2 d y dy
EXAMPLE 4 Ify = A cos (log x) + B sin (log x), prove that x --- 7T + X -2- + U =0.
dx2 dx
SOLUTION We have, [CBSE 2007,2009]
y = A cos (log x) + B sin (log x).
On differentiating with respect to x, we get
1
dy_ - B
sin (log x) + — cos (log x)
dx x x
=> x — = - A sin (log x) + B cos (log x)
dx
On differentiating again with respect to x, we get
d2y dy cos (log x) sin (log x)
xJ+ = dx
= -A

dy _
x
—B
x

x2 + x — = - {A cos (log x) + B sin (log x)}


dx2 dx

ReadYourFlow.COM
HIGHER ORDER DERIVATIVE 12.3

=> *2 +x dy _
= - y
dx2 dx
2 dy dy „
=> x —tt + x — + y = 0
dx1 dx
d*y = cos x [NCERT EXEMPLAR]
EXAMPLES If y = tanx + secx, prove that
dx2 (1 - sin x)2
SOLUTION We have, y = tan x + sec x
— = sec2 x + sec x tan x = —^ + sin x 1 + sin x
2 2
dx cos
COS X COS X COS X

dy _ 1 + sin x 1
=>
dx 1 - sin 2 x 1 - sin x
d2y d
d 1 = ^{(l-sinx)-1}

w
=>
dx2 dx 1 — sin x dx
d2y -1 COS X
=>
dx2 dx

Flo
= (-l)(l-sinx) -2 — (1 - sin x) =
(1 - sin x)
2 (~ cos *) =
(1 - sin x)2

ee
EXAMPLE 6 /f y = tan x, yroi't? that y^ = 2i/yi.

Fr
SOLUTION We have, y = tan x
dy = sec2 x
for
ur
-Z-
dx
ks

2
or. yj = sec x ••• yi = dx
Yo
oo

=> y- (yi) = y- (sec2 x)


B

dx dx
re

d 2
=> y2 = 2 sec x —(sec x) = 2 sec x sec x tan x = 2tan x sec x
ou
ad

=> III = 2yyi [•.• y = tan x and y^ = sec x]


Y

2
EXAMPLE 7 If y = tan-1 x,/mrf —y in terms ofy alone.
nd
Re

dx1 [NCERT EXEMPLAR]


Fi

SOLUTION We have,
y = tan_1x
=> x = tan y
Differentiating with respect to y, we obtain
dx 2
= sec y

dy =_j_ V ^= 1
sec2 y dx dx/dy
Differentiating both sides with respect to x, we obtain
d2y d , 2 N
-^-=—(coszy)
dx2
^y = dy
=> -2 cosy siny —
dx2 dx

ReadYourFlow.COM
12.4 MATHEMATICS-XII

=> ^1 = - 2 cos i/ sin y x cos y .. dv = cos2 y


rfx2
rf2y . 3
=> —y = - 2 sin y cos y
dx2

EXAMPLE 8 If yf'Y = (x + J/) m + n , prove that = 0. [NCERT EXEMPLAR]


dx2
SOLUTION We have,
xV=(^y) m + n
=> dy = y
dx x [See Example 15 on page 11.85]
Differentiating both sides with respect to x, we obtain
A_(dy]=A_(y]
dx{dx) dxyx)

w
x^ -yx 1
d2y _ dx
=>
dx2

Flo
ee
d*y

Fr
=> Using: ^
dx2 dx x
d2y
for
ur
=>
dx2
ks

d2y _ 24y
EXAMPLE 9 Ify 3 - y = 2x, prove that
Yo

dx2 (3y2 -l)3


oo

SOLUTION We have,
B

y3-y = 2x
re

Differentiating both sides with respect to y, we obtain


ou
ad

(3y2-D = 2^
Y

dy
=> dy _ 2
nd
Re

dx (3y2 -1)
Fi

Differentiating both sides with respect to x, we obtain


d^y_ 2 2 dy
r(3y2-i)=- x 6y-f
dx2 (3y2 -l)2 dx (3y2 -i)2 dx

_12.y 2
x_____ dy _ 2
=>
dx2 (3y2-!)2 (3y2 -1) dx 3y2 -1
24y
=>
dx2 (3y2 -i)3
d^y -(dy)2
EXAMPLE 10 If e^ (x + 1) =1, s/ioze that [NCERT]
dx2 dx
SOLUTION We have,
(x + 1) = 1
=> V = ------
1
x +1

ReadYourFlow.COM
HIGHER ORDER DERIVATIVE 12.5

1
=> log e V ■= log
x+l/
=> y = - log (x + l)
dy_ _ 1 = l
=> and
dx X+l dx2 (x +1)2
d^y = dy]2
=>
dx1 dx
2
EXAMPLE 11 Ify = xX,find ^ .
dx1
SOLUTION We have, y = xx
logy = x log x

w
Differentiating with respect to x, we get
\ dy . , 1

Flo
y dx x
^ = y(l + log x)

ee
=> •••(i)
dx

Fr
Differentiating both sides of (i) with respect to x, we get
2
^ (1 + log x) + y d (1 + log x) for
ur
dx2 dx dx
^ ^ (1 + log x) + y X - = y (1 + log x)2 + —
ks

[Using (i)]
Yo

dx2 dx x x
oo

'Ll = Xx |(l + logx)2 + ^


=>
B

dx2
re

3 d2y j
dy \2

EXAMPLE 12 if[ax + b) e^^x = x or,y = x log —- prove that x = x — -y .


ou

a + bx j dx2 dx
ad

[CBSE 2005, 2013, 20151


Y

SOLUTION We have,
d
Re

[ax + b) ey/x = x
n
Fi

=> gy/x =
ax + b
=>

=> y = x log
^ + bxj
=> y = x (log x - log (a + fox)}
y
=> — = log x - log (a + fox)
x
On differentiating with respect to x, we get

1 1 d ... 1 b
---------------- (a + fox) =---------
x a + bx dx x a + bx
.........
dy 2 1 b
=> x-r-y = * -X
dx + fox

ReadYourFlow.COM
12.6 MATHEMATICS-XII

dy ax
=> x ,-y
dx a + bx
Differentiating both sides of (i) with respect to x, we get
^ d2y + dy _ dy _ (a + bx) a-ax(0 + b)
dx2 dx dx (a + bx)2
d2y _ a2
=> x
dx2 (a + bx)2
3 d2y = a 2 x 2
=> x [Multiplying both sides by r2]
dx2 (a + bx)2
\2
ax
=> x 3 •••(ii)
dx2 a + bx /

w
From (i) and (ii), we obtain
_v3 d2y = (xdy_f'
dx2 dx

Flo
ee
2
EXAMPLE 13 Ify = log | x + yjx2 + a2 }, prove that: (x2 + a2) + x^ = 0.

Fr
dx2 dx
SOLUTION We have. for [CBSE 2013]
ur
y = log jx +-^x2 + a2 j
ks

On differentiating with respect to x, we get


Yo
oo

dy _ 1 1 2x
1+
B

dx x + a x2 + a x+ tJx2 + a2 2 -Jx2 + a2
re

dy _ 1 x2 + a2 + x
ou
ad

=> x
dx x + -Jx2 + a2 x2 + a 2
Y

1 dy
= , where yi =
nd

=> Vl =
Re

yjx2 + a 2 dx
Fi

=> i/!2 (x2 + a2) = 1


Differentiating with respect to x, we get
*12 ~ (x2 + a2) + (x2 + a2) — (y^2) = 0
dx dx
=> yi2 (2-r) + (x2 + a2) x 2 y: y2 = 0 d
3-(yi2)=2(yi)
dx
2yi |y2 (*2 +ij2) + xyi
= 0

=> y2 (X2 + a2) + xyx =0 [•■• yi ^ o]


2
EXAMPLE 14 Ify = sin- 1 x, then show that (1 - x2) —~- = 0.
dx2 dx
SOLUTION We have, [CBSE 2012, NCERT]
y = sin - 1 x

ReadYourFlow.COM
HIGHER ORDER DERIVATIVE 12.7

= 1
=> [Differentiating with respect to x]
* ^l-x1

=>
ax
Differentiating both sides with respect to x, we get
fTTJ A * iy _ o

d-^) [Multiplying both sides by yl -x2]


dx2 dx
A LITER We have,
y = sin- 1 x

w
dy _ 1
*

=> = 1

Flo
ee
dx

Fr
Differentiating both sides with respect to x, we get
j \2
dy
‘'-‘’H'ilihi = 0 for
ur
dx y
2^ dy d2y
=> - 2x dy = 0
ks

2(1-*")
dx dx2 dx J
Yo
oo

2X d2y xdy = Q
=> d-*")
B

dx2 dx
re

sin 1 x , prove that (1 -x2) -x — -m2y = 0.


EXAMPLE 15 If y =
rfx2 rf*
ou
ad

SOLUTION We have. [CBSE 2015]


Y

y=e
nd
Re

Differentiating with respect to x, we obtain


Fi

dy _„m
— C
sin-1 x --------------
rn
dx
dy _ my w sin 1x
=> =y]
dx 7^17
dy]2 _ ™2y2
c/x l-x2

=> = m 2..2
y

=>
(1 -x2) y2 =m2y2, where >/i = ^

Differentiating with respect to x, we obtain


(I-X2)f (yi2) + (y!2) f(l-x2)=m2f (y2)
dx dx dx
=> (1 -x2) 2y1y2 +y12(-2x) =m2 (2yy1) ••• -7-(yi2) = 2y1y2 and 4- (y2)=2yyi
dx dx

ReadYourFlow.COM
12.8 MATHEMATICS-XII

2yi |(i-x2) 1/2 ~xyi -m2yj =0

=> (i-*2)i/2-*yi-m2y = 0 [••• yi * 0]


EXAMPLE 16 y = | 'r + +1 J / show that (x2 +l)\j2+ xyx -m1 y = 0.

SOLUTION We have. ICBSE2013, 2U15]

Differentiating with respect to x, we get


dy_ = m-1 d
| x + tJx2 +1 | x—^ x+
dx dx
m -1

w
rfy _ m | [x + -Jx2 + 1 | 2x
x• 1 +
dx 2 -Jx2 +1

Flo
m
, m X2 + 1 + X
^ = _

ee
=>
rfx

Fr
=> yi = for
ur
yijx2+l = my
ks
Yo
oo

=> y!2(x2 +1) = m 2..2


y [Squaring both sides]
B

Differentiating with respect to x, we get


re

2yi yi C1 + *2) + y/ (2x) = 2m2 yyi


ou
ad

y2(l + x2) + xy1-m2y = 0


Y

EXAMPLE Y7 if y = Sjn - - , show that (1 - x2) dy [CBSE 2013]


- 3x — - y = 0.
d
Re

rfx2 dx
n
Fi

sin- 1 x
SOLUTION We have, y =
1-x2

=> y Jl - x2 = sin 1 x

Differentiating both sides with respect to x, we get


x 1
iy
rfx
V1-*2 = i—i
Jl-x2 Jl-x2
=> ^ (1 - x2) - xy = 1
dx
Differentiating both sides with respect to x, we get
i|(1_x2)_2^ dy - y = 0
x—
dx dx

=> = 0
dx2 dx

ReadYourFlow.COM
HIGHER ORDER DERIVATIVE 12.9

f- log i/l, show that (1 + x2) + {2x - a) ^-=0.


EXAMPLE 18 If x = tan
U J dx2 dx
SOLUTION We have. [CBSE 2011, 2013]

x = tan
Giosy)
=> tan 1 x=-logy
a
-1
=> a tan ‘ x = log y
Differentiating with respect to x, we get
a ldy_
l + x2 V dx
(1 + x2) ^ = ay

w
■=>

dx
Differentiating with respect to x

(1 + T2) + 2x — = a —

Flo
ee
dx dx

Fr
(l + x2)^+(2x-a)^- = 0
=>
dx2 dx for
ur
d2 V _ 1 T dy^
EXAMPLE 19 y = X*, prove that -*=0. [CBSE 2014,2016]
dx2 y v dx ) x
ks
Yo

SOLUTION We have, y = xx
oo

e log = ex log x
B

or. y =
re

Differentiating with respect to x, we get


ou

/y = exl°Zx — (xlogx)
ad

dx dx
Y

=> (1 + log x)
nd

dx
Re

dy = y (1 + log x)
Fi

=>
dx
Differentiating with respect to x, we get
d2y
= y* — (1 + log x) + x (1 + log x)
dx2 dx dx
d2y
= y x — + — x (1 + log x)
dx2 x dx
d2y = y + dy(l^i 1 dy
=> From (i), 1 + log x = —
dx2 X dx [y dx y dx _
d2y
=>
dx2 x y {dx]

d2y 1 r dyf -1 = 0.
dx2 y^dx) X

ReadYourFlow.COM
12.10 MATHEMATICS-XII

Type II ON FINDING SECOND ORDER DERIVATIVES OF PARAMETRIC FUNCTIONS


d2y
EXAMPLE 20 Find > ifx = at2, y -2at.
dx2
SOLUTION We have,
x = at2 and y = 2at
dx
=> — - 2at and d
-Z = ...(i)
dt dt
dy _ dy/dt _ 2a _ 1
dx dx/dt 2at t
Differentiating both sides with respect to x, we get
d2y _ d fl
dx2 dx 11

low
=>
=_rff__ J_ J_ dx dt 1
From(i), — = 2at
dx2 t2 dx t2 2at dt dx 2at
d2y = 1
=>
dx2 2at3

ee
2
rF
Fr
EXAMPLE21 Ifx = a cos3 0, i/ = rt sin 3 0, find —Also, find its value at Q-—.
dx2 6
SOLUTION We have, x - a cos 3 0 and y - a sin 3 0 [CBSE 2013]
for
— = - 3a cos2 0 sin 0 and — = 3a sin2 0 cos 0
ou
dB dQ
ks

dy _ dy/dQ 3a sin2 0 cos 0


oo

So, 2 = - tan 0
dx dx/dd - 3a cos 0 sin 0
Y
B

Differentiating both sides with respect to x, we obtain,


re

1 1
= — (- tan 0) = - sec2 0 — = - sec2 0 x sec4 0 cosec 0
ou

- 3a cos2 0 sin 0 3a
ad

dx2 dx dx
Y

1 4 71 n 1 2 32
- — sec — cosec — = — x i —= x2—
3a 6 6 3a l VI 27 a
nd
Re
Fi

EXAMPLE 22 If x = a sin t and y = a ^cos t + log tan j ,find d2y


dx2
SOLUTION We have. [CBSE 2013]
x = a sin t and y -a ^cos t + log tan

dx . i dy . , 1
=> — = acos t and — = a -sinf + — x sec
dt dt t 2 2
tan
2
— =acosfand — = a f - sin f + —1
=>
dt dt V sinty
dx , , dy a (1 - sin 2 f)
=> — = a cos t and — = ----------------
dt dt sint
dx , dy acos2f
=> — = a cos t and — =
dt dt sinf

ReadYourFlow.COM
HIGHER ORDER DERIVATIVE 12.11

dy acos2 1
fy=ll = sin t
cos t
= cot t
dx dx acos t
sin t
dt
= —f^i = —(cot f) =-cosec2 f —
=>
dx2 dx dx) dx dx
d2y 2x1 1
—~r = -cosec r x-------
dx2 a cos t a sin2 t cos t
Type III ON PROVING RELATIONS INVOLVING VARIOUS ORDER DERIVATIVES OF PARAMETRIC
FUNCTIONS
2 d y dy „
EXAMPLE 23 If x = a cos Q + b sin 0 and y = a sm Q - b cos 0, prove that y —^ -x -2- + y = 0.
dx2 dx

w
[CBSE2013, 2014, 2015]
SOLUTION We have, x =a cosG + frsinG and y = fl sin 0-fc cosG

Flo
x2 + y2 = {a cos G + b sin G)2 + (a sin 0 - fc cos 0)2
x2 + y2 = a2 (cos2 0 + sin2 G) + b2 (sin2 0 + cos2 0)

ee
=>

Fr
=> x2 + y2 = a2 +b2
Differentiating with respect to x, we get for
ur
2x + 2y ^ = 0
ks

dy _ x
Yo

=>
oo

dx y
eB

Differentiating with respect to x, we get


x
yx 1 - y~x
r

dx y)
ou

[Using (i)]
ad

dx2 y2 y2
Y
nd

d2y = (-v2 + y2)


Re

=> ...(h)
dx2
Fi

+y=0 [Using (i) and (ii)]

2
EXAMPLE 24 If x = sin t and y = sin pt, prove that (1 - x2) - x ^ + p2y = 0
dx2 dx
[NCERT EXEMPLAR, CBSE 2016]
SOLUTION We have,
x = smt, y = sin pt => ^ = cost and, — = pcospt
dt
dy _ dy/dt _ pcospt _ P yl-sin2pt
dx dx/dt cost •\/l-sin2f
dy _pfl-y2
=>
dx

ReadYourFlow.COM
12.12 MATHEMATICS-XII

=> (gVxWa-y2) [Squaring both sides]

Differentiating with respect to x, we obtain

=> (-2x)=-2p2y^-
dx

=> =0

=> (l-x2)^-x^ + p2y=0


v 2 — *0
dx2 dx dx

low
EXAMPLE 25 If x = sin Q , y = cos p 0, prove that
(l-J)y2-xy1 + P2y = 0, where y2=—j and yx = ^
dxz dx

ee
SOLUTION We have,
rF
Fr
x = sin 0 and y = cos p 0.
*1 -p sin pQ _ -p -Jl - cos2 p 0 for
= dQ =
dx dx cos 0 -Jl - sin2 0
ou
dQ
ks

dy = -p^-y2
oo

dx
Y
B
re

dy)2 _ (i -y2)
=> [Squaring both sides]
dx (1-x2)
ou
ad
Y

= p2 (i-y2)
nd
Re

Differentiating both sides with respect to x, we get


Fi

j \2

dx dxz dx j
2v^y _ dy 2
d-x") x — + py = 0
dx2 dx
LEVEL-2
Type ZV ON PROVING RELATIONS INVOLVING VARIOUS ORDER DERIVATIVES
3/2
i+ rfyf
dx
example 26 If (x - a)2 + (y - b)2 = c2, prove that is a constant independent of
d2y
dx2
a and b. [NCERT]
SOLUTION We have,
(x - a)2 + (y -b)2 =c2

ReadYourFlow.COM
HIGHER ORDER DERIVATIVE 12.13

Differentiating with respect to x, we get


2(x-a) + 2(y-b)^ = 0
dx
(x - a) + (y -b) = 0 ...(h)
dx
Differentiating with respect to x, we get
d2y
+ = 0
dx2 dx

=> (y-b) 1+
dyf
dx2 dx J

i+ dyf

w
dx
d^ =
=> ...(hi)
dx2 y-b
From (ii), we obtain

Flo
ee
x -a
= -

Fr
dx y-b
dy]2 for
ur
i+ = i+
dx
dy]2 (x-a)2 + (y-b)2
ks

=> 1 +
Yo

dx (y-fc)2
oo

c2
B

=> 1 + [Using (i)] ...(iv)


dx (y-b)2
re

3/2 3/2
(dy]2 c2 c3
ou
ad

=> 1 + = *
v dx (y-b)2 (y-b)3
Y

From (hi) and (iv), we obtain


nd
Re

1+
(I)'
Fi

d^y __ c2/(y-b)2 -c2


...(Vi)
dx2 y-b (y-b) (y-b)3
From (v) and (vi), we obtain
dy]2
i+
dx
= -c, which is independent of a and b.

(y-b)3
d^y a2b2
EXAMPLE 27
Vy1 = a 2 cos2 x + fr2 sin2 x, then prove that dx2 + y = y3'

SOLUTION We have,
y = a cos x + & sin x
=> 2y2 = a2 (2 cos2 x) + b2 (2 sin2 x)

ReadYourFlow.COM
12.14 MATHEMATICS-XII

=> ly1 = a2 {1 + cos 2x) + b2 (l - cos 2x)


=> 2y2 = {a2 + b2) + {a2 - b2) cos 2x ...(i)
Differentiating with respect to x, we get
41/ — = - 2 (rt2 -b2) sin 2x
dx
=> 2y — = -(a2 -b2) sin 2x ...(ii)
dx
From (i), we obtain
2y2 - (a2 + b2) = (a2 -b2) cos 2x ...(hi)
Squaring (i) and (ii) and adding, we get

4j/ 2^)2
dx
+ {2yM«2+>2)}2 = (a2 - b2)2 (sin22x + cos22x}

low
2 f dy)2
=> 4y + 4y4 -4y2(a2 +b2) + (a2 +b2)2 =(a2 -b2)2
dx

dyf + y2 - (a2 + b2)

ee
=> 4]/2 = (a2 -b2)2 - (a2 +b2)2
dx y
rF
Fr
=>
(l)2+y2-(a2+fc2) = -4 a2 b2 for
ou
a2 b2
ks

=> + y2 - (a2 +b2) =


, dx y2
oo

Differentiating both sides with respect to x, we get


Y
B

2(dy)d*y + 2l/dy = 2a2 b2 dy


re

[dxJdx2 dx y3 dx
ou
ad

d*y_ a2b2 dy
=> Dividing both sides by 2 —
Y

+y =
dx2 y3 dx
nd

EXAMPLE 28 Iff (x) = | x |3, show that / " (x) exists for all real x and find it.
Re

SOLUTION We have,
Fi

[NCERT]
x3 , if x > 0
/(*) = l^|3 = (-x)3 =-x3 , if x <0
Now,
/(■v)-/(0) - x 3-0
(LHD at x = 0) = lim = lim = lim_ -x2 =0
x ^ 0~ x -0 x-»0- x x -> 0
fM-f(0) x3 -0
(RHDat x = 0) = lim lim lim x2 = 0
x -»0 + x-0 x -> 0+ x
(LHD of /(x) at x = 0) = (RHD of /(x) at x = 0)
So, / (x) is differentiable at x = 0 and the derivative of/(x) is given by
3x2 , if x > 0
/'(*) =
- 3x2 , if x < 0
Now,

ReadYourFlow.COM
HIGHER ORDER DERIVATIVE 12.15

- 3x2 0 i-
(LHD of /' (x) at x = 0) = lim lim — = km - 3x = 0
x-»0- x-0 x -> 0- x x->0-
3x2 -0
(RHD of /' (x) at x = 0) = lim = lim = lim 3x = 0
x->0+ x -0 0+ x-0 x->0+
(LHDof /' (x) at x = 0) = (RHDof /' (x) at x = 0)
So, /' (x) is differentiable at x = 0.
6x , if x > 0
Hence, f " (x) =
- 6x , if x < O'
Type V MISCELLANEOUS PROBLEMS
EXAMPLE 29 In x is independent variable and y is the dependent variable. If independent and
dx
dV becomes dx and these two are connected by the relation

w
dependent variables are interchanged —
dx dV
2
d2x

Flo
— • — = 1. Find a relation between and
dx dy dx1 dyr

ee
SOLUTION We know that

Fr
d2y _ d f dy
dx2 dx V dx for
ur
=> d2y = d ( 1 1 - ^ f—
dx2 dx{dx/dy) dx { dy y
ks
Yo
oo

-2
d2y _ d f dx'" dx d_(dx 1
=> X -------------
eB

dx2 dy\[dyj dx dyydy dx! dy


dy
-2 d2X
r

d2y _ dx d2x dx'' dx'


ou

=>
ad

dx2 ■ dy) dy2 [dy <dy) dy2


Y

-3 d2x dy']3 d2y _ d2x


dx
dy2 ^ [di) ~d?~
nd

Hence,
Re

dx2 dy dy2
Fi

^yJ^y)2
EXAMPLE 30 Find the equation to which the equation x y — = 0 is transformed by
dx2 dx dx
interchanging the independent and dependent variables.
SOLUTION We know that
dy = 1 and d2y _ 1 d2x
[See Example 29]
dx dx/dy dx2 (dx/dy)3 dy2
d2y (dy]2 f dy]
Substituting these values in the equation x
dx2 -+[i) ii}-°'weset
\2
x d2x 1 ^=0
dx]3 dy2 dx dx'1
\dy) dy) dy)

ReadYourFlow.COM
12.16 MATHEMATICS-XII

d2x dx f dx)2
=>
'V =0 Multiplying both sides by

d2x dx)2
=> -^ = 0
*~9 +y
dy dy) dy
EXERCISE 12.1
LEVEL-1
1. Find the second order derivatives of each of the following functions:
(i) x3 + tan x (ii) sin (log x) [NCERT] (iii) log (sin x) [NCERT]
(iv) ex sin 5x [NCERT] (v) e6x cos 3x [NCERT] (vi) x3 log x INCERTI
(vii) tan-1 x [NCERT] (viii) [NCERT] (ix) log (log x) [NCERT]

w
X COS X

d2y
2. tty =e x cos x, show that = 2c~ * sin x.

Flo
dx2
3. If y = x + tan x, show that cos2 x - 2y + 2x = 0. [CBSE2U07]

ee
dx2

Fr
4. If y = x3 log x, prove that .
dx4 ^ for
ur
5. If y = log (sin x), prove that —y = 2 cos x cosec3 x.
dx3
ks

d2y
Yo

6. If y = 2 sin x + 3 cos x, show that —j + y = 0.


oo

dx1
eB

d2y _ 2 log x-3


7. If y = log* , show that
x dx2 x3
r
ou

8. If x = 0 sec Q, y =b tan 0, prove that = —r^—


ad

dx1 a~2 y..S'


Y

9- If x = a (cos 0 + 0 sin tt),y = a (sin 0-0 cos 0), prove that


nd
Re

d2x
:i-^r (cos©-©sin©), —r (sin0 + 0cos0) and —^ ^
Fi

dQ2 dQ2 dx1 a0


[NCERT, CBSE 2012, 2017]

10. If y = cr cos x, prove that = 2ex cos j^x + -1. [CBSE 2012]
rfx2 2
h4
11- If x = a cos Q,y =b sin 0, show that -
dx1 2 ..3 -
a y
2
12. If x = a (1 - cos 3 0), y = sin 3 0, prove that = at 0 = —.
dx1 27 a 6
2
13. If x = a (0 + sin 0), y = (1 + cos 0), prove that -^-y = - —
dx2 y2

14. If x = a (0 - sin 0), y = a (1 + cos 0) find [CBSE 2011]


rfx2’

15. If x = a(l - cos 0), y = c (0 + sin 0), prove that d2y- iate=^.
dx2 2

ReadYourFlow.COM
HIGHER ORDER DERIVATIVE 12.17

^=^-ate = ^.
16. If x = (1 + cos 0), y = (0 + sin 0), prove that
dx2 a 2
d2y j \2
17. If .r = cos 0,y = sin 3 0, prove that y + dy = 3 sin 2 0 (5 cos2 0 -1). [CBSE 2013]
dx2 dx y
18. If y = sin (sin x), prove that + tan x • -^ + y cos2 x = 0.
dx1 dx
r\

7 d^v dy 2
19. If x = sin t, y = sin pt, prove that (1 - x ) —j -x-f- + p y = 0.
dx~ dx
20. If y = (sin-1 x)2, prove that (1 - x2) y2 - xi/i -2 = 0.

21. Ify =e tan 1 x , prove that (1 + x2) y2 + (2x -1) yi = 0.


22. If y = 3 cos (log x) + 4 sin (log x), prove thatx2y2 + x y^ + y = 0.

w
INCERT, CBSE 2009,2012,2016]
23. If y = e2x (ax + b), show that y2 - 4 yj + 4 y = 0.
24. If x = sin log y j , show that (1 - x2) y2~x Vi

Flo
-a2 y = 0. [CBSE 2010]

ee
25. If log y = tan-1 x, show that (1 + x2) y2 + (2x -1) y-! = 0.

Fr
26. If y = tan-1 x, show that (1 + x2) + 2x ^- = 0.
dx1 dx
or
ur
2\ d2y
27. if y = | log (x + ^/x2 +1) | , show that (1 + x ) — = 2. [CBSE 2008]
f
dx22 +X~T
dx
ks

28. If y =(tan- 1 x)2, then prove that (1 + x2)2 y2 + 2x (1 + x2) y-^ =2. [CBSE 2012, [NCERT]
Yo
oo

d2y . dy „
29. If y = cot x show that —+ 2y = 0.
B

ix2 dx
2 (2^
re

30. Find , where y = log ~ [CBSE 2000]


dx .
ou
ad

d2y dy
Y

31. If y = ae2x +be x, show that. -2y = 0. [CBSE 2000C]


dx2 dx
nd
Re

32. If y = ex (sin x + cos x) prove that -2 — + 2y = 0. [CBSE 2002,2009]


dx2 dx y
Fi

2
33. If y = cos- 1 x, find —y in terms of y alone. [NCERT]
dx2
-1x , prove that (1 - x2)
-x^--a2 y = 0.
34. Uy = ea cos [NCERT, CBSE 2012]
dx2 dx
d2
35. Ify =500 e7* + 600 c-7;t,showthat—| =49 y. [NCERT]
dx2
d2y
36. If x = 2 cos f - cos 2t,y = 2 sin t - sin 2t, find -—y atf =-.
^ dx2 2
37. Ifx=4z2+5, y = 6z2+7z+3,find^-^.
dx2

38. If y = log (1 + cos x), prove that


d^L cfy dy = Q [CBSE 2005]
dx3 dx2 dx
2 d2y dy
39. Ify = sin (log x),prove that x + x— + y = 0. [CBSE 2007]
dx2 dx

ReadYourFlow.COM
12.18 MATHEMATICS-XII

40. Ify = 3 e2x + 2 e3x, prove that -5 — + 6y = 0. IN CERT, CBSE 2007,2009]


dx2 dx
41. If y=(cot- 1 x)2, prove that y2 (x2 + l)2 + 2x(x2 + 1) y-j =2.
d2y
42. If y = cosec- 1 x, x > 1, then show that x (x2 -1) j + (2x2 -1) — = 0. [CBSE 2010]
dx1 ^ dx
t 71
43. If x = cost + log tan -, y = sin f, then find the value of and t [CBSE 2012]
2 dt2 4
44. If x = flsinf andy = ^cosf + log tanfind d2y
[CBSE 2013]
dx2'
2
45. Ifx = (cost + f sin f) and y = a (sin f-f cost), then find the value of—^ at t = -. [CBSE 2014]
dx2 4
46. Ifx =fl^cosf+ logtan^j. d2y t, n

w
y = a sin f, evaluate [CBSE 2014]
dx2 3
d2y
47. Ifx =a (cos2f + 2f sin 2f)andy =fl (sin 2f -2f cos 2f), then find [CBSE 2015]

Flo d2y
rfx2'

ee
48. Ifx = 3cot-2cos3f, y = 3sinf-2sin3f,find [CBSE 2017]
dx2'

Fr
LEVEL-2
for
ur
49. If x = o sin f -b cos t,y = a cos t + b sin t, prove that
d^L _
dx2
ks

50. Find A and B so that y - A sin 3x + B cos 3x satisfies the equation


Yo
oo

+ 4 ^ + 3y = 10 cos 3x.
dx2 dx
eB

51. \iy = A e~ kt cos (pt + c), prove that + 2k — + n2 y = 0, where n2 =p2 + k2.
dt2 dt
r

2
ou
ad

52. If y = x" [a cos (log x) + b sin (log x)}, prove that x2 + (1 - 2n) ^ + (1 + n2) y = 0.
Y

dx dx
53. If y = a |x +Vx2 +l| +/? jx-Vx2 + l| , prove that(x2-l) rf2y dy 2y = r>0.
nd

+ x —-?7
Re

dx2 dx
Fi

ANSWERS
- [sin (log x) + cos (log x)] n
1- (i) 6x + 2 sec x tan x (ii) (iii) -cosec x
x2
(iv) 2ex (5 cos 5x -12 sin 5x) (v) 9c6* (3 cos 3x - 4 sin 3x) (vi) x (5 + 6 log x)
- 2x
(vii) (viii) - x cos x - 2 sin x (ix) - (1 + log x)
(1 + x2)2 (x log x)2
2
14. (ii) — cosec 4 e 30. 33. - cot y cosec2 y
4a 2 x2
3 ( jl \
7
36. 37. 43. dy 1
= 242
2 64z3 dt2 42 ' [dx2 J,=n
Jt=«
4 4
44. 1 45. M 8^3
. o
46. 47. — sec32f
asm tcost TUI a 2a
48. 1 1
50. A = —
—, B = —
3 sin 3 f cos 2t 3 3

ReadYourFlow.COM
HIGHER ORDER DERIVATIVE 12.19

HINTS TO NCERT & SELECTED PROBLEMS


1. (ii) Lety = sin (log x). Then,
dy _ cos (log x)
dx x

=> — = cos (log + {cos (log x)}


dx2 dx \xj x dx
d2y cos (log x) sin (log x) _ —2" |cos (log x) + sin (log x)|
=> —=
dx2 x2 x2
(iii) Let y = log (sin x).Then,
dy _ - - cosec 2 x
= cot X =>
dx dx2
(iv) Let y = ex sin 5x. Then,

low
^ sin 5x + e'r (5 cos 5x) = (sin 5x + 5 cos 5x)
dx
_> djj' = ^gin c;x + 5 cos 5^ + e* A- (5^ 5x + 5 cos 5x)

ee
dx2 dx dx
rF
Fr
=> ddx2
h_ = ex (sin 5x + 5 cos 5x) + ex (5 cos 5x - 25 sin 5x) = ex (- 24 sin 5x + 10 cos 5x)
for
(v) Let y = e6x cos 3x. Then,
ou
ks

= — (e6x) cos 3x + e6x — (cos 3x)


dx dx dx
oo

dy 6x
Y

=> = 6 e bx cos 3x -3c sin 3x


B

dx
re

dy = 3 ebx
(2 cos 3x - sin 3x)
dx
ou
ad

d2y _= 3 — {ebx) (2 cos 3x - sin 3x) + 3 ebx — (2 cos 3x - sin 3x)


Y

=>
dx2 dx dx
nd
Re

d2y
=> = 18 ebx (2 cos 3x - sin 3x) + 3e6x (- 6 sin 3x - 3 cos 3x)
Fi

dx2
d2y
= 9 c6* {4 cos 3x - 2 sin 3x - 2 sin 3x - cos 3x} = 9 (3 cos 3x - 4 sin 3x)
dx2
(vi) Let y = x3 log x. Then,

= logx f (x3)+^34(logx)
dx dx dx
1
dy = 3x2 log x + x3 x -
=>
dx x
=>
dy = x2 (3 log x +1)
dx
d^L
=> = (3 logx + 1) ~T~ (x2) + X2 -~-(3 log X + 1)
rfx2 dx dx
d2y 3
=> = 2x (3 log x + 1) + x2 x —
- = x(6 log x + 5)
dx2 X

ReadYourFlow.COM
12.20 MATHEMATICS-XII

(vii) Let y = tan 1 x. Then,


rfy _ = (1 + AT2)- 1 => = — (1 + .t2)- 2 ~ (1 + x^) = 2x
dx l + x2 dx2 dx (1 + x2)2
(viii) Let y = .t cos x. Then,
dy _
cos x - x sin x
dx

=> —y = - sin x - (sin x + x cos x) = - 2 sin .r - x cos x


dxz
(ix) Let y = log (log x). Then,
= Id... 1
= (x log x) 1
& lo^*(l0g3;) =

low
=> (Ulogx)-1}
dxz dx
-(1 + logx)
=> yf = "(^ log xV 2 4- (* log x) = ~ 2 (1 + log x) =
dx* dx (x log x) (x log x)2

ee
rF
Fr
22. We have,
y = 3 cos (log x) + 4 sin (log x) for
= — 3 4
=> sin (log x) + - cos (log x)
dx x
ou
X
ks

dy
=> x— = - 3 sin (log x) + 4 cos (log x)
dx
oo
Y

Differentiating both sides with respect to x, we get


B
re

d_y dy -3 . 4 . .. ,
x —= — cos (log x) — sin (log x)
dxz dx x x
ou
ad

x2^y
+ x dy = - {3 cos (log x) + 4 sin (log x)}
Y

=^>
dx2 dx
nd
Re

=>
x2d^ + xdy =
dx2 dx
- y => x 2fy^+y =°
Fi

dx1 dx
28. We have.
y = (tan 1 x)2
dy = 2 (tan 1x) 2-1 — (tan 1 x)
=>
dx dx
^ ?y = 2 tan 1 x
dx i + x2

=> (1 + x2) ^ = 2 tan" 1 x


dx
r a \2
2\2 (dy i
(1 + x*) = 4 (tan" 1 x)2 [Squaring both sides]
dx
2n2 (dy^l2
d + x") = 4y
dx

ReadYourFlow.COM
HIGHER ORDER DERIVATIVE 12.21

Differentiating both sides with respect to x, we get


2(1 + x2)x 2x dyf + 2 (1 + x^) = 4^y
dx dx dx2 dx
2^2 d2y
2x (1 + x2) ^ + (1 + x^)
=> = 2 => (1 + x2)2 y2 + 2x (1 + x2) y1 = 2
dx rfx2
33, We have,
y = cos- 1 x
dy 1
dx
= -(i-.2r1/2
d2y _ 1
=>
dx2 2 dx
d2y = -x

w
=>
dx2 (l-x2)3/2
d*y = - cos y -1 x => x = cos y]

Flo
=> [••• 3/ = cos
dx2 (1 - cos2 y)3/2

ee
2
=> - cot y cosec y

Fr
dx2
34. We have, for
ur
y = ea cos ~lx
dy lx d , -1 .
— = e (7 cos
ks

=> — {a cos x)
dx dx
Yo
oo

dy = eacos ^ x x a
=>
B

dx
re

=> dy _
ou
ad
Y

=> -«y
d
Re
n

2 ..2 [On squaring both sides]


=> = « y
Fi

Differentiating with respect to x, we get


dy d2y =
- 2x i-)2
Ux
+ (l -X2) 2
9
dx dx2
2fl2 y—

9
X—dy + (l - XZ)
?\ ---f
« V = 772 y => ,1 2\ —^
(l - xz) d y x — - a2 y = 0
dx dx2 dxz dx
35. We have.
y = 500 e7x + 600 e -7x

=> = 3500 e7x - 4200 e -7x .


dx
d*y _
=> = 3500 x7 e7x + 4200 x 7 e -7x
dx2
^2y = ^ =
=> 49 (500 e7* + 600 c -7x ) => 49y
dx2 rfx2

ReadYourFlow.COM
12.22 MATHEMAT1CS-XII

40. We have,
y - 3 <? 2a + 2 e 3'v
^y
6 elx + 6 e3x and = lie2* +18 e3*
dx dx2
2
H-5^L + 6y = (12 e2x + 18 e^) -5 (6 e2x + 6 e3x) + 6(3 e2x + 2 e3*) = 0
‘ d*2 dx

_____________________________________VE/7V SHORT ANSWER QUESTIONS (VSAQs)


Answer each of the following questions in one word or one sentence or as per exact requirement of the
question:
2
1. If y = x" + 1 + bx~ n and x2 —^ = Xy, then write the value of A.
dx
d2x
2. If x=a cos nt -b sin nt and = Xx, then find the value of X.

w
dt2

3. If x =t2 and y =f3, find—y.

Flo
dx2
1

ee
4. If x = 2at, y = at2, where is a constant, then find —~ ~atx = —.
dx 2 2

Fr
5. If x = / (f) and y = g (t), then write the value of A
dx2
for
ur
2
x2 X3 X4
6. If y = 1 — x + —— ——ir — to oo, then write in terms of y.
ks

2! 3! 4!
Yo

d2x
oo

7. If y = x + ex, find
dy2'
eB

8. If y = | x - x2 |, then find —y
r

dx2
ou
ad
Y

9. If y = | loge x |, find .
dx"
ANSWERS
nd
Re

3. i 1 5
Fi

1. «(n +1) 2. n2 4.
4f 2« /'3
1
, 0 <x <1
- er d2y _ f-2,0 <x <1
6. y 7. 9.
(1 + cr)3 d?_i 2 , x > 1, x <0 dx2 1
--y , X >1
xz
MULTIPLE CHOICE QUESTIONS (MCQs)
Write the correct alternative in each of the following:
d2x.
1. If x = a cos nt -b sin nt, then —y18
dt2
(a) n2 x (b) -n2 x (c) -rcx (d) nx
d2}/
2. If x = 2, y = 2 , then
dx2

ReadYourFlow.COM
HIGHER ORDER DERIVATIVE 12.23

1 1
(a)------ (b) (c) -
t2 2 at3 t
2 d2y _
3. If y - axn *1 + bx ", then x
dx2
(a) n(n-l)y (b) n (n + l)i/ (c) ny (d) n2y
d20
4. (2 cos x cos 3 x) =
dx20
(a) 220 (cos 2 x - 220 cos 4 x) (b) 220 (cos 2 x + 220 cos 4 x)
(c) 220 (sin 2 x + 220 sin 4 x) (d) 220 (sin 2 x - 220 sin 4 x)

5. If x =f2, i/=f3, then ^1-


dx2

w
(a) 3/2 (b) 3/4t (c) 3/2t (d) 3t/2
6. If y = a + bx2, a, b arbitrary constants, then
fy cfy

Flo
rf2y - — + y = 0 (d) x ^ = 2 xy
(a) = 2xy (b) X = !/l (c) x
dx2 dx2 dx dxz

ee
dx2

Fr
7. If / (x) = (cos x + i sin x) (cos 2x + i sin 2x) (cos 3x + i sin 3x).... (cos nx + i sin nx) and
/ (1) = 1, then / " (1) is equal to
n(n + l) n (n + l)\2 n (n + 1) | 2 for
ur
(a) (b) (c) - (d) none of these
2 2 2
ks

d2y
Yo

8. If y = a sin mx + fr cos mx, then 2- is equal to


oo

dx
(b) m2y (d) my
B

(a) -m2y (c) -my


re

siii 1 x
9- If f (x) = = , then (1 - x2) / " (x) - x / (x) =
ou
ad

(b) -1 (c) 0 (d) none of these


Y

(a) 1

10. If y = tan 1
logt* (g/-y2)
+ tan
3 + 2 logt, x
then A=
nd
Re

logt, {ex2) 11 - 6 loge J' dx2


Fi

(a) 2 (b) 1 (c) 0 (d) -1


11. Let f (x) be a polynomial. Then, the second order derivative of / (ex) is
(a) f"(ex) e2x + f' (ex) ex (b) f"(ex)ex +f'(ex)
(c) f"(ex)e2x+f"(ex) ex (d) f"{ex)
12. If y = rt cos (logt, x) + b sin (log,, x), then x2 y2 + xy1 =
(a) 0 (b) y (c) -y (d) none of these
1 .
13. If x = 2 at ,y = at2, where <i is a constant, then —^ at x = — is
dx2 2
(a) l/2a (b) 1 (c) 2a (d) none of these
d2y
14. If x = / (f) and y=g (t), then j is equal to
dx

(a) (b) (d)


f" g'-g"f
(/o3 in2 /

ReadYourFlow.COM
12.24 MATHEMATICS-Xli

15. If 1/ = sin (m sin x), then (1 - x ) y2 - x]h is equal to


(a) m2y (b) my (c) -m2y (d) none of these
16. If y = (sin x) , then (1 - x ) y2 is equal to
(a) xy1 + 2 (b) xyl-2 (c) - xyl + 2 (d) none of these
o
tan x
17. lfy=e , then (cos x) y2 =
(a) (1 - sin 2x) yl (b) - (1 + sin 2x) y-^
(c) (1 + sin 2x) yl (d) none of these
\
2 X
w. Ify= tan tan — ,a>b>0. then
-Ja2~b2 2
-1 b sin x
(a) yi = (b) y2 =
a + b cos x (a + b cos x)

w
1 -b sin x
(c) yi = (d) y2 =
a - b cos x (a -b cos x)2

Flo
ax + b
i9. Ify=72 /then(2xy1+y)y3 =
x +c

ee
Fr
(a) 3 (xy2 + y{) y2 (b) 3 {xyl + y2) y2 (c) 3 (xy2 + yx) (d) none of these

X
then x 3 y2 =
for
ur
2o. Ify=1°gc
a + bx j
2
ks

(a) (*yi -y)2 (b) (1 + y)2 (c) (d) none of these


Yo
oo
B

22 Ifx=/(f) cost-/'(f) sin f andy=/(t) sin f+/'(f) cost, then=


re

(a)/(f)-/"(f) (b) {f (t) - f " (t)}2 (c) {f (t) + /" (f)}2 (d) none of these
ou
ad

22 If y1//n + y~ :1'/" = 2x, then (x2 -1) y2 + xy1 =


Y

(a) -n2 y (b) n2y (c) 0 (d) none of these


nd
Re

23- If {x" - x” _ 1 + a2 x” _ 2 +... + (-1)" an} ex = xn ex, then the value of iy , 0 < r < n, is
Fi

equal to
(«-/-)! nl
(a) (b) (c) (d) none of these
r\ r\ (n-r)\
22 If y = x” 1 log x then x2 y2 + (3 - 2ri) xyj is equal to
(a) -(n-1)2 y (b) (n-l)2y (c) -n2y (d) n2y
2
2^ If xy - loge y = 1 satisfies the equation x (yy2 + y1 ) - y2 + X yyT = 0, then X =
(a) -3 (b) 1 (c) 3 (d) none of these
2
If y2 = ax2 +bx + c, then y 3 —^ is
26.
dx2
(a) a constant (b) a function of x only
(c) a function of y only (d) a function of x and y

L
ReadYourFlow.COM
HIGHER ORDER DERIVATIVE 12.25

____ ANSWERS

1. (b) 2. (d) 3. (b) 4- (b) 5. (b) 6. (b) 7. (c) 8. (a) 9. (a)


10. (c) 11. (a) 12. (c) 13- (a) 14. (a) 15. (c) 16. (a) 17. (c) 18. (b)
19. (a) 20. (a) 21. (c) 22. (b) 23. (c) 24. (a) 25. (c) 26. (a)

SUMMARY
1. If y =/ (x), then — ^ is called second order derivative of y with respect to x and is
dx VdxJ
rf2y
denoted by —w or, y2 or, y Similarly, third and higher order derivatives are defined.
dx2
2. If x=f (t) and y = g(t),then

= A.

w
dx2 dx

or =d_ lV_(0 dt
' dx2 dt \f'(t) dx dx2

Flo {f'm3

ee
Fr
for
ur
ks
Yo
oo
B
re
ou
ad
Y
nd
Re
Fi

ReadYourFlow.COM
Re
Fi ad
nd
Y
ou
Yo
re
B
oo
ur
ks
for
Flo
w Fr
ee
*

ReadYourFlow.COM
CHAPTER 13
DERIVATIVE AS A RATE MEASURER

13.1 DERIVATIVE AS A RATE MEASURER


Let y =j(x) be a function of x. Let Ay be the change in y corresponding to a small change Ax in x.
Then, — represents the change in y due to a unit change in x. In other words, — represents the
Ax Ax-
average rate of change of y with respect to x as x changes from x to x + Ax.

low
As A x -» 0, the limiting value of this average rate of change of y with respect to x in the interval
[x, x + Ax] becomes the instantaneous rate of change of y with respect to x.
Thus,
Ay = Instantaneous rate of change of y with respect to x
lim

ee
Ax -> 0 Ax
rF
Fr
dy ^y =^y
=> — = Rate of change of y with respect to x lim
dx for Ax -> 0 Ax dx
The word "instantaneous" is often dropped.
ou
dy
ks

Hence, represents the rate of change of y with respect to x for a definite value of x.
dx
oo

dy / dy
Y
B

REMARK 1 The value of at x = x0 i.e. represents the rate ofchange ofy with respect to x
dx dx
re

'X = X0

at x - x0.
ou
ad

dy
dy ff; dx
Y

REMARK 2 7/x = <]) (f) and y = \\i (t), then---- = , provided that---- ^ 0.
dx dx dt
nd
Re

dt
Thus, the rate of change ofy with respect to x can be calculated by using the rate of change ofy and that ofx
Fi

each with respect to t.


REMARK 3 Throughout this chapter, the term "rate of change" will mean the instantaneous rate of
change unless stated otherwise.

ILLUSTRATIVE EXAMPLES

LEVEL-1
EXAMPLE 1 A balloon, which always remains spherical, has a variable radius. Find the rate at which its
volume is increasing with respect to its radius when the radius is 7 cm.
SOLUTION Let x be the radius and y be the volume of the balloon. Then,
4 = An(7)2 =19671 cm2
y = — kx
3 dx l, dx ) x = 7
Hence, the volume is increasing with respect to its radius at the rate of 196 n cm , when the
radius is 7 cm.

ReadYourFlow.COM
13.2 MATHEMATICS-XM

EXAMPLE 2 Find the rate of change of the area of a circle with respect to its radius. How fast is the area
changing with respect to the radius when the radius is 3 cm ? [NCERT1
SOLUTION Let A be the area of the circle. Then,
a 2 dA
A = nr => — = 2rc r
dr
Thus, the rate of change of the area of the circle with respect to its radius r is 2ti r.
When r - 3 cm, we obtain
— = (2nx 3) cm = 671cm.
dr 3
EXAMPLE 3 A balloon, which always remains spherical, has a variable diameter —(2x + 3). Determine
the rate of change of volume with respect to x. 2
SOLUTION Let V be the volume of the balloon. Then,
^{|(2x+3)}3 (2*+3)3
V

w
=> dV = ^ x 3 (2x + 3)2 — (2x + 3) = 27 n {2x + 3)2
dx 16 dx 8

Flo
EXAMPLE 4 The total cost C (x) associated with the production ofx units of an item is given by
C (x) = 0.005.V3 - 0.02*2 + 30* + 5000

ee
Find the marginal cost zvhen 3 units are produced, where by marginal cost we mean the instantaneous rate

Fr
of change of total cost at any level of output. [NCERT]
SOLUTION Since the marginal cost is the rate of change of total cost with respect to the output.
for
ur
Marginal cost (MC) = — (C(x)) = —(O.OOS.r0-0.02*2 + 30x + 5000)
dx dx
ks

= 0.005 (3x2)-0.02 (2x)+ 30


Yo
oo

When x = 3, we get
Marginal cost (MC) = 0.005 x 3 x 32 - 0.02 x 2 x 3 + 30 = 0.135 - 0.12 + 30 = 30.015
B
re

Hence, the required marginal cost is ? 30.02 (nearly).


EXAMPLES The total revenue received from the sale of x units of a product is given by
ou
ad

R (x) = 3x2 + 36x + 5. Find the marginal revenue zvhen x = 5, where by marginal revenue zve mean the
Y

rate ofchange of total revenue with respect to the number of items sold at an instant. [NCERT]
nd

SOLUTION Since the marginal revenue is the rate of change of total revenue with respect to the
Re

number of units sold.


Fi

Marginal revenue (MR) = — = — (3x2 + 36x + 5) = 6x + 36


dx dx
When x = 5, we get
Marginal revenue =6x5+ 36 = 66
Hence, the required marginal revenue is ? 66.
'C EXAMPLE 6 A car starts from a point P at time t = 0 second and stops at point Q. The distance x, in
metres, covered by it, in t seconds is given by x = t2 ^2 -Find the time taken by it to reach at Q and

alsofind distance between P and Q. [NCERT]


SOLUTION We have,
* = f2 f2- t => x = 212 - t3 * =4(-(2
3 3 dt
This gives velocity of the car at any time t.
Suppose the car stops at Q after ^ second. Then, at f = fj

ReadYourFlow.COM
DERIVATIVE AS A RATE MEASURER 13.3

dx
— = 0
dt P R Q
dx
or. = 0 Fig. 13.1
dt f=fl

=> *l-h2 = 0
=> h(*-h) = 0
=> ^ = 4 [•.• = 0 is for point P]
Thus, the car takes 4 seconds to reach at Q.
The distance between P and Q is the value of x at f = f j i.e. at f = 4.
? 43 64 32
PQ = (Valueof X att = 4) = 2x 42 -— = 32------= — m
3 3 3
EXAMPLE 7 Find the rate of change of volume ofa sphere with respect to its surface area when the radius
is 2 cm.

w
SOLUTION Let r be the radius, V the volume and S be the surface area of the sphere. Then,
V = — 7rr 3 and S = 47tr2

Flo
3
dV
We have, to find — when r = 2.

ee
dS

Fr
Now, V - — nr 3 and S = 4ra'2
3
for
ur
=> — = 4rcr2 and — = 8nr
dr dr
dV
ks

dV=Jr_
Yo
oo

dS dS
B

dr
dV _ 4nr2 _ r
re

=>
ou
ad

dV = 2=1.
Hence,
Y

dSJr = 2 2
d

EXAMPLE 8 If x and y are the sides of two squares such that y = x — a2. Find the change of the area of
Re
n

second square with respect to the area of thefirst square. [NCERT EXEMPLAR]
Fi

SOLUTION Let and A2 denote the areas of squares of sides a and y respectively. Then,
= a2 and A2 = y2
=> A-y = a2 and A2 = (a - a2)2 [•• y = a-a2 (given)]
dAy
=> = 2a and ——= 2 (a—a2) (1 - 2a)
dx dx
dA2 _ dA2/dx
Now,
dAy dAy/dx
dA2 _ 2 (a - a2) (1 -2a)
=> = (1 - a) (1 -2a) = 1 - 3a + 2a2.
dAy 2a
EXAMPLE 9 A swimming pool is to be drainedfor cleaning. IfL represents the number of litres of water
in the pool t seconds after the pool has been plugged off to drain and L = 200 (10-t)2. Hoiu fast is the
water running out at the end of 5 seconds? What is the average rate at which the water flows out during
thefirst 5 seconds? [NCERT EXEMPALR]

ReadYourFlow.COM
13.4 MATHEMATICS-XII

dL
SOLUTION We have to find — at I =5.
dt
Now, L = 200 (10-f)2

=> dL =-400(10-f)
dt
dL
= -400 (10-5) =-2000
dt A =5
Thus, the water is running out at the rate of 2000 litres per second at the end of 5 seconds.
The average rate at which the water flows out during the first 5 seconds is given by
L(0) - L(5) _ 200 (10 - 0)2 - 200 (10 -5)2 20000 -5000
= 3000 litres/sec.
5 5 5

EXERCISE 13.1

low
LEVEL-1
1* Find the rate of change of the total surface area of a cylinder of radius r and height h, when
the radius varies.
2. Find the rate of change of the volume of a sphere with respect to its diameter.

ee
3. Find the rate of change of the volume of a sphere with respect to its surface area when the
rF
Fr
radius is 2 cm.
4. Find the rate of change of the area of a circular disc with respect to its circumference when
for
the radius is 3 cm.
5- Find the rate of change of the volume of a cone with respect to the radius of its base.
ou
6. Find the rate of change of the area of a circle with respect to its radius r when r = 5 cm.
ks

7. Find the rate of change of the volume of a ball with respect to its radius r. How fast is the
oo

volume changing with respect to the radius when the radius is 2 cm?
Y
B

8. The total cost C (x) associated with the production of x units of an item is given by
C (x) = 0.007x3 - 0.003x2 + 15x + 4000. Find the marginal cost when 17 units are produced.
re

[NCERT]
ou
ad

9. The total revenue received from the sale of x units of a product is given by
Y

R (x) =13x2 + 26x + 15. Find the marginal revenue when x =7. [NCERT]
10. The money to be spent for the welfare of the employees of a firm is proportional to the rate
nd
Re

of change of its total revenue (Marginal revenue). If the total revenue (in rupees) received
Fi

from the sale of x units of a product is given by R(x) - 3x2 + 36x + 5, find the marginal
revenue, when x = 5, and write which value does the question indicate. [CBSE2013]
ANSWERS
2.^
1. 4:nr + 2nh —, r is the diameter 3. 1 cm
2
4. 3 cm 5. — nr h 6. 10 7i cm2/cm
3
7. 47tr2 , 16tt m3/m 8. ? 20.967 9. ? 208
10. MR = ? 66. It indicates the extra money spent when number of employees increase from
5 to 6.
HINTS TO NCERT & SELECTED PROBLEMS
3. We have,
y = — 7ir3and, S = 4 7rr2=> — = 4 7r:r2and, — = 8 nr
3 dr dr

ReadYourFlow.COM
DERIVATIVE AS A RATE MEASURER 13.5

dV dV/dr _ 4tt: r2 _ r dV 2 .
=i- - =1 cm
US ~ dS/dr ~ 8nr ~2 dS )r = 2 2
4. We have.
A = nr2 and, C=2nr =e> — = 2nr and, ^- = 2n
dr dr
dA _ dA/dr dA
= r => = 3 cm
dC ~ dC/dr dC)r = 3
8. We have.
C (x) = 0.007x3 - 0.003 x2 + 15x + 4000
=> — (C (x)) = 0.021 x2 - 0.006x + 15
dx

(JH = 0.021 x!72 -0.006x 17 +15

w
x = 17
Hence, marginal cost = ? 20.967
9. We have, R (x) = 13x2 + 26x + 15

Flo
ee
— (K (x)) = 26x + 26 => = 26 x 7 + 26 = 208

Fr
dx x-7
13.2 RELATED RATES for
ur
Generally we come across with the problems in which the rate of change of one of the quantities
involved is required corresponding to the given rate of change of another quantity. For example,
ks

suppose the rate of change of volume of a spherical balloon is required when the rate of change
Yo

of its radius is given. In such type of problems, we must find a relation connecting such
oo

quantities and differentiate this relation w.r. to time. The procedure is illustrated in the
B

following examples.
re

ILLUSTRATIVE EXAMPLES
ou
ad

LEVEL-1
Y

EXAMPLE 1 An edge of a variable cube is increasing at the rate of 10 cm/sec. How fast the volume of the
nd
Re

cube is increasing when the edge is 5 cm long?


Fi

SOLUTION Let x be the length of the edge of the cube and V be its volume at any time t. Then,
V = x3 and — = lOcm/sec [Given]
dt
Now, V = x3
dV 2 dx
=> — = 3x
dt dt
dV 9 dx
=> ~ = (3*2)(10) — =10
dt dt

=> ™ = 30 x2
dt
=> = 30 (5)2 = 750 cm3sec.
v dt )x = 5

Thus, the volume of the cube is increasing at th e rate of 750 cm3/sec when the edge is 5 cm long.

ReadYourFlow.COM
13.6 MATHEMATICS-XII

EXAMPLE 2 The radius of a circle is increasing uniformly at the rate of 4 cm/sec. Find the rate at which
the area of the circle is increasing when the radius is 8 cm.
SOLUTION Let r be the radius and A be the area of a circle at any time t. Then,
A = 7tr2 and — - 4cm/sec [Given]
dt
Now, A = nr2
dA dr
=> — = 2nr —
dt dt
dA 9 9
=> = 2 rex 8 x 4 cm“/sec - 64 tt cm /sec.
dt Jr = 8

EXAMPLE 3 If the area of circle increases at a uniform rate, then prove that the perimeter varies inversely
as the radius. [NCERT EXEMPLAR]
SOLUTION Let r be the radius P be the perimeter and A be the area of the circle at any time t.

w
Then, 2l = nr2 and P = 2nr.
dA
It is given that — = constant (k), wherek >0
dt

Flo
ee
Now,

Fr
A = nr2 and P = 2iir

p
=> A =K
or v P = 2nr => r = —
ur
2n
sf
=> A= Lp2
4k
k
Yo
oo

dA I dP
=> = —x2P
B

dt 4k dt
1
re

=> k= —
2k dt
ou
ad

I dP
=> k = ——(2Kr) [v P=2Kr]
Y

2k dt
dP__k_
=>
nd
Re

dt ~ r
Fi

=> P varies inversely as the radius r.


ALITER We have,
A -Kr2 and P = 2nr
dA dr dP dr
=> — ~ 2nr— and = 2n—
dt dt dt dt
dr dP dr dA
=> k = 2nr —- and = 2n— — =k
dt dt dt dt
dP k dr
=> — = 2k On eliminating —
dt 2nr dt
dP_ = k
=>
dt ~ r
=> P varies inversely as the radius r.
EXAMPLE 4 The side of an equilateral triangle is increasing at the rate of 2 cm/sec. At what rate is its
area increasing when the side of the triangle is 20 cm? [CBSE2015]

ReadYourFlow.COM
DERIVATIVE AS A RATE MEASURER 13.7

SOLUTION At any time t, let x cm be the length of a side of an equilateral triangle and A be
its area. Then,
A=^x2
4
dA J3 dx 43 dx
=> — = — x 2x— = x—
dt 4 dt 2 dt
dA dx
= — x 20 x 2 = 20^3 cm2/sec : — -2 cm/sec (given)
dt Jx = 20 2 dt

Hence, the area is increasing at the rate of 2043 cm2/sec.

EXAMPLE 5 The radius of a balloon is increasing at the rate of 10 cm/sec. At what rate is the surface area
of the balloon increasing when the radius is 15 cm?
SOLUTION Let r be the radius and S be the surface area of the balloon at any time t. Then,
9 dr

w
S = 4%r and — =10 cm/sec
dt
Now, S = 4nr2

=>
dS dr
— = Stx r —

Flo
ee
dt dt
dS dr

Fr
=> — = 8071 r — =10 cm/sec.
dt dt
dS
for
ur
=> = SOtx (15) = 120071 cm2/sec.
dt Jr = 15
ks

EXAMPLE 6 A spherical ball of salt is dissolving in water in such a manner that the rate of decrease of
Yo
oo

volume at any instant is proportional to the surface. Prove that the radius is decreasing at a constant rate.
[NCERT EXEMPLAR]
B

SOLUTION Let V, S and r denote respectively the volume, surface area and radius of the salt
re

ball at any instant f. Then,


ou
ad

V = — 7cr 3 and S = 4tu'2


3
Y

It is given that the rate of decrease of the volume V is proportional to the surface area S.
dV
nd
Re

i.e. —- cc S
dt
Fi

dV
=> —= -kS, where /c > 0 is the constant of proportionality

It is given that V is decreasing with time, so that is why negative sign is taken.
Now,
dV
—— =-kS
dt
=>

2 dr
=> 4nr = -4nkr2
dt
dr
=> = -k
dt
=> r decrease with a constant rate
Hence, the radius is decreasing at a constant rate.

ReadYourFlow.COM
13.8 MATHEMATICS-XII

EXAMPLE 7 Find an angle 0, 0 < 0 < ^, which increases twice as fast as it sine.

[NCERT EXEMPLAR]
SOLUTION It is given that
dQ
= 2 — (sin 0)
dt dt
dQ dQ
=> — = 2 cos 0 —-
dt dt
dQ
=> 2 cos 0=1 *0
dt
1
=> cos 0 = —
2
0 = 60°
Hence, the measure of angle is 60°.

w
EXAMPLE 8 A stone is dropped into a quiet lake and zuaves move in a circle at a speed of 3.5 cm/sec. At
the instant when the radius of the circular ivave is 7.5 cm, how fast is the enclosed area increasing?

Flo
[NCERT]
SOLUTION Let r be the radius and A be the area of the circular wave at any time t. Then,

ee
A^ = nr 2 andi —dr = 3-5 cm/sec.
[Given]

Fr
dt
Now, A = nr2 for
ur
dA dr dr
=> — = n 2r — = 2nr —
dt dt dt
ks
Yo

dA dr
=> — = 2nr(3-5) = 7nr
oo

— = 35 cm/sec
dt dt
B

dA
=> -7k(7-5) = 52-5ti cm2/sec.
re

d* Jr =7-5
ou
ad

EXAMPLE 9 A particle moves along the curve 6y = x3 + 2. Find the points on the curve at which the
Y

y-coordinate is changing 8 times as fast as the x-coordinate. [NCERT]


SOLUTION Let the required point be P(x, y). It is given that
nd
Re

Rate of change of y coordinate = 8 (Rate of change of x-coordinate)


Fi

dy_ = 8dx
i.e. -(i)
dt dt
Now, 6y = x3 + 2
2 dx
=> 6™ = 3x [Differentiating both sides with respect to t]
dt dt
dx 2 dx
=> 6 8— = 3x [Using (i)]
dt dt
=> 3x2 = 48 => x2 = 16 => x = ± 4

Now, x = 4 => 6y =43 + 2 = 66 => y = 11

and, x = -4 =>6y = (-4)3 + 2 = -62 y =

So, the required points are (- 4, - 31/3) and (4,11).


EXAMPLE 10 The volume of a cube is increasing at a rate of 7 cm3/ sec . How fast is the surface area
increasing when the length of an edge is 12 cm?

ReadYourFlow.COM
DERIVATIVE AS A RATE MEASURER 13.9

SOLUTION Let x be the length of an edge of the cube, V be the volume and S be the surface area
at any time t. Then, V = x3 and S = 6x2. It is given that

— = 7cm'V sec
dt
=> — (x3) = 7 =>3x2 — = 7 => dx 7
dt dt dt 3x2
Now, S = 6x2
dS dx
=i> — = 12x —
dt dt
dS „ 7 dx 7
— = 12xx —
dt 3x2 dt 3x2
dS 28
=>
dt x

low
=> = — cm 2/sec = — cm2/ sec
= 12 12 3

EXAMPLE 11 The volume of a cube is increasing at a constant rate. Prove that the increase in surface

ee
area varies inversely as the length of the edge of the cube. [NCERT EXEMPLAR]
rF
Fr
SOLUTION Let x be the length of each edge of the cube, S be its surface area and V be its volume
at any time t. Then, S = 6x2 and V = x3. It is given that — = k (constant).
for
dt
Now, V = x3
ou
— = 3*2* ^ k = „3x 2 — dx dx k
ks

=> => — = 7T
dt dt dt dt 3x2
oo

and. S = 6x2
Y
B

dS dx
re

— = 12 x —
dt dt
ou

dS k
ad

=> — = 12x [Using (i)]


dt 3x2
Y

dS 4fc => — dS
cc
1
nd
Re

dt X dt x
Fi

Hence, the rate of increase in surface area varies inversely as the length of the edge of the cube.
EXAMPLE 12 Two men M-y and M2 start with velocities v at the same timefrom the junction of two roads
inclined at 45° to each other. If they travel by different roads, find the rate at which they are separated.
[NCERT EXEMPLAR]
SOLUTION Let O be the junction and OA and OB be two roads inclined at an angle of 45°. Let
men My and M2 travel by roads OA and OB respectively and let at any time P and Q be their
positions such that OP = OQ = x (both men travel with same speed v). Then,
B
dx
— =v
dt
dy Q
Let PQ = y. We have to find -A

Using cosine formula in AOPQ, we obtain


PQ2 =0P2+0Q2-2 0P 0Q cos 45° //
45°
=> y2 = X 2 +X 2^2
- 2x X
1
o x —P A
V2 Fig. 13.2

ReadYourFlow.COM
13.10 MATHEMATICS-XII

y = V2-V2 x

dt dt
=> ^=Vwft,
dt
Hence, two men and M2 are separated at the rate ^2 - V2 j i?.

EXAMPLE 13 For the curve y =5x- 2x , z/x increases at the rate of2 units/sec, then howfast is the slope
of the curve changing when x = 3? [NCERT EXEMPLAR]
SOLUTION Let m be the slope of the curve at an arbitrary point (x, y) on it. Then,
m--2-dV
dx

low
=> m = 5 - 6x2 v y =5x-2x3 = 5 - 6x2
dx
dx dm
It is given that — = 2 units/sec and we have to find — when x = 3.
dt dt

ee
Now,
rF
Fr
m = 5 -6x2
dm 1 _ dx
=> — = -12 x — for
dt dt
dm^
ou
=> = -12x 3x2 - -72 units/sec v x = 3 and — = 2
ks

dt )x=3 dt
oo

Thus, the slope of the curve is decreasing at the rate of 72 units/sec when x is increasing at the
Y

rate of 2 units/sec.
B
re

EXAMPLE 14 The length x of a rectangle is decreasing at the rate of 2 cm/sec and the width y is
increasing at the rate of 2 cm/sec. When x = 12 cm and y = 5 cm, find the rate of change of
ou
ad

(i) the perimeter and (ii) the area of the rectangle. IN CERT]
Y

SOLUTION Let P be the perimeter and A be the area of the rectangle at any time t. Then,
P = 2 (x + y) and A = xy
nd
Re

It is given that — = - 2 cm/sec and ^- = 2 cm/sec.


Fi

dt dt
(i) We have,
P = 2 (x + y)
dP ~(dx dy''
=> = 2 (- 2 + 2) = 0 cm/sec i.e. the perimeter remains constant.
dt dt dt
(ii) We have,
A = xy
dA dx
=>
dt ydt y + x dt
dA
=> — = -2x5 + 12x2 [v x = 12 cm and y = 5 cm (given)]
dt
dA _. 2/
— = 14 cm /sec.
dt
EXAMPLE 15 A man 2 metres high, walks at a uniform speed of 6 metres per minute away from a lamp
post, 5 metres high. Find the rate at which the length of his shadow increases. INCERT]

ReadYourFlow.COM
DERIVATIVE AS A RATE MEASURER 13.11

SOLUTION Let AB be the lamp-post. Let at any time t, the man CD be at a distance x metres
from the lamp-post and y metres be the length of his shadow C£. Then,
dx
— = 6 metres/minute [Given]
dt
' learly, triangles ABE and CDE are similar.
AB AE B
CD ~ CE
5 x+y
=>
2 y
3«/ = 2*

=> 3— 2—
dt dt

w
=> 3% =2(6) [Using (i)]
dt /l C £
Fig. 13.3

Flo
=> = 4
dt

ee
Thus, the shadow increases at the rate of 4 metres/minute.

Fr
EXAMPLE 16 A man is walking at the rate of 6.5 km/hr towards thefoot of a tower 120 m high. At what
rate is he approaching the top of the tower when he is 50 m away from the tower?
for
SOLUTION Let at any time t, the man be at distances of x and y metres from the foot and top of
ur
the tower respectively. Then,
y2 = x2+ (120)2
ks
Yo

dy dx
oo

=> 2i/— = 2x —
dt dt
B

B
dy_ = xdx
re

=>
dt y dt
ou
ad

dx
We are given that — = - 6 - 5 km/hr (negative sign due to
Y

dt ■/, 120 m
decreasing ,r). Therefore,
nd
Re

dy _ 6-5x
...(h)
dt
Fi

y
Putting x =50 in (i), we get: y = -\/502 + 1209 = 130 x meter
A (Man) O
Putting x = 50, y = 130 in (ii), we get Fig. 13.4
dy 6-5x50
= -2-5
dt 130
Thus, the man is approaching the top of the tower at the rate of 2 - 5 km/hr.
2 1
EXAMPLE 17 A man 2 m tall, walks at the rate of l — m/sec towards a street light which is 5— m above
3 3
the ground. At what rate is tip of his shadow moving? At what rate is the length of the shadow changing
when he is 3—mfrom the base of the light? [NCERT EXEMPLAR]
3
1
SOLUTION Let OA be the street light of height 5 m. At any time f, let PQ be the position of the
man and let PR be the length of his shadow such that PR = x and OP = y.

ReadYourFlow.COM
13.12 MATHEMATICS-XII

It is given that the man is walking at the rate of — m/sec towards the street light.
3
— —m/sec
dt 3
We have to find the rate at which the tip of the shadow is moving i.e. we have to find — {x + i/).
dt '
For this we require the value of So, let us first find

A's AOR and QPR are similar triangles.


AO OR A
PQ ~ PR
16/3 _x + y
=>
2 ~ x 5|m

low
==> 8x = 3x+3y
5x = 3y
Differentiating with respect to t, we obtain
R x •*— P—— y —— o

ee
5— = 3 —
dt dt Fig. 13.5
rF
Fr
=> 5— = 3x - -
dt 3 for
dx
=> = -1 m/sec ...(h)
u
dt
ks

d /^r,\ d , . dx dy . 5 8
— {OR) = — {x+y)= — + = — [Using (i) and (ii)]
Yo

dt dt ~ dt dt 3 3
oo

Thus, the tip R of the shadow PR is moving towards the base of the street light at the rate of
B

8/3 m/sec.
re

From (ii), we have


dx
ou
ad

— = -1 for all x, y
dt
Y

1
Thus, the length of the shadow is reducing at the rate of 1 m/sec when the man is 3 — m from the
nd

3
Re

base of light.
Fi

EXAMPLE 18 A ladder 5 m long is leaning against a wall. The bottom of the ladder is pulled along the
ground awayfrom the wall at the rate of 2 m/sec. Howfast its height on the wall decreasing when thefoot
of the ladder is 4 m awayfrom the wall? [CBSE 2012, NCERT]
SOLUTION Let AB be the position of the ladder at any time t such that OA = x and OB = y.
Then,
OA2+OB2 = AB2 => x2 + y2 = 52 -(i) y
It is given that the bottom of the ladder is pulled along the ground
away from the wall at the rate of 2 m/sec. B
dx
— = 2 m/sec.
dt
Now, x2 +y2 = 52

=> 2x — + 2y— = 0
dt dt
..dx n o A X
=> 2x (2) +2y— = 0 . —=2
dt dt
Fig. 13.6

ReadYourFlow.COM
DERIVATIVE AS A RATE MEASURER 13.13

dy _ _lx
=> .••(ii)
dt y
Putting .r = 4 in (i), we get: y = y25 -16 = 3.
dy _ _ 8
Putting x = 4 and y = 3 in (ii), we get: — — m/sec.
dt 3
8
Hence, the rate of decrease in the height of the ladder on the wall is — m/sec.
3
EXAMPLE 19 The two equal sides of an isosceles triangle with fixed base b are decreasing at the rate of 3
cm/sec. Howfast is the area decreasing when the two equal sides are equal to the base? [NCERT]
SOLUTION Let at any time t, the length of each equal side be x cm and area of the triangle be A.
Then,
1
A = ^(BCxAD)

w
1 X ,0 X
=> A = —
2

Flo
b
=> A = -
4

ee
dA b 1
i«*2-b2)

Fr
=> = — X
dt 4 2 yAx2 -b2 dt
for
ur
dA b dx
=> x 8x —
dt 8^4x2-b2 dt
B D c
ks
Yo

dA bx dx Fig. 13.7
oo

=>
dt
B
re

dA 3bx dx
=> — = 3 cm/sec (given)
dt dt
ou
ad
Y

dA 3b2
=> = f3b cm2/sec.
dt ?x = b yfAb2 -b2
nd
Re

LEVEL-2
Fi

EXAMPLE 20 An airforce plane is ascending vertically at the rate of 100 km/h. If the radius of the earth is
r km, how fast is the area of the earth, visible from the plane, increasing at 3 minutes after it started
ascending? Given that the visible area A at height h is given by A = In r2
r+h
SOLUTION It is given that the plane is ascending vertically at the constant rate of 100 km/h.
— = 100 km/h
dt
3
=> Height of the plane after 3 minutes = 100 x ^ = ^ ■'<m- [Using/? =vt]
r> 2 h
Now, A = 2nrz -----
r+h
dh , dh
dA n 2 d h
(r + h)^-hf(r + h) (r + h) —- -h —
— = 2nr — ------ = 2nr2 • dt dt ■ = 2k r2 ■ dt dt
dt dt \\ r + h (r + h)2 (r + h)2

ReadYourFlow.COM
13.14 MATHEMATICS-XII

dA _ 2kv3 dh
=>
dt ~ (r + /z)2 dt

dA = 2nr3 200nr3 dh
=> x 100 = — =100 km/h
dt (r + h)2 (r + fi)2 dt
dA
We have to find when t = 3 minutes and at t = 3, we have h = 5 km.
dt
dA 200ti r 3
dt)t = 3 (r+5)2'

EXAMPLE 21 Water is dripping out from a conical funnel of semi-vertical angle ^ at the uniform rate

of 2 cm2/sec in its surface area through a tiny hole at the vertex in the bottom. When the slant height of the

w
water is 4 cm,find the rate of decrease of the slant height of the water. [NCERT EXEMPLAR!
SOLUTION Let VAB be a conical funnel of semi-vertical angle At any time t the water in the

Flo
cone also forms a cone. Let r be its radius, / be the slant height and S be the surface area. Then,

ee
WL = /, O' A' =r and ZA' VO' = —.
4

Fr
In A VO' A', we have
n VO' VO' O'A' O'A'
for
ur
cos — = and, sin — =
4 VA' l 4 VA l
ks
Yo

=> VO' = / cos - and, O'A = Ism—.


i ■ n
oo

4 4
B

The surface area S of the conical funnel is given by Fig. 13.8


re

S = k(0'A')(VA') [Using: S = nr l]
ou

Kl2
ad

=> S = k\ l sin — I/ = k I,2 sin


■ n
-=
Y

4 4 V2
dS 2k I dl
=>
d
Re

dt ~ \[2 dt
n
Fi

2k l df dS 2/
-2 = • — = -2 cm /sec
42 dt dt
dl 42
=>
dt TC /

dl} 42
------ cm/sec.
dtJi = 4 4k

Thus, the rate of decrease of the slant height is — cm/sec.


42
4rc
EXAMPLE 22 Sand is pouringfrom a pipe at the rate of 12 cm3/sec. The falling sand forms a cone on the
ground in such a way that the height of the cone is always one-sixth of the radius of the base. Howfast is the
height of the sand-cone increasing when the height is 4 cm ? [NCERT, CBSE 2011)
SOLUTION Let r be the radius, h be the height and Vbe the volume of the sand-cone at any time
t. Then,
V = — 7r r2 h
3

ReadYourFlow.COM
DERIVATIVE AS A RATE MEASURER 13.15

=> V = ^n(36h2)h = 12k h3 [v r = 6 h]

dV 2 dh
=> — - 36kh
dt dt h
dh 1 dV
— =12 (Given)
dt 3k h2 dt r
A' B
O
dh 1 1
=> =
3k (A)2 48 n Fig. 13.9
dt Jh = 4
1
Thus, the height of the sand-cone is increasing at the rate of cm/sec.
48:i
EXAMPLE 23 An inverted cone has a depth of 10 cm and a base of radius 5 cm. Water is poured into it at

w
the rate of 3/2 c.c. per minute. Find the rate at which the level of water in the cone is rising when the depth
is 4 cm.

Flo
SOLUTION Let a be the semi-vertical angle of the cone VAB a O B
5 cm
whose height VO is 10 cm and radius OB = 5 cm. Then,

ee
5 1 O'

Fr
tan a = —
10 2 A
for 10 cm
Let V be the volume of the water in the cone i.e. the volume of h
ur
the cone VA1 B' after time t minutes and h be the height of water.
V /
Then,
ks
Yo
oo

V = -k(OB')2 (VO') Fig. 13.10


3
B

O' B' O' B' .


re

=> V = — Kh3 tan2 a Y tan a = ------ =------- => O B = h tan a


3 VO' h
ou
ad

1
=> V = ^ y tan a
Y

2
dV=JL3h2dh k , 2 dh
nd

=> -h —
Re

dt 12 dt 4 dt
Fi

3 _ Kh2/ dh dV 3 o
=> — cm /minute (given)
2 ~ 4: dt dt
dh 6
=>
dt Kh2
dh 6 3
=> = — cm/min.
dt Jh = 4 rc(4)2 8tc

EXAMPLE 24 Water is dripping out from a conical funnel at a uniform rate of 4 cm3/sec through a tiny
hole at the vertex in the bottom. When the slant height of the water is 3 cm, find the rate of decrease of the
slant height of the water-cone. Given that the vertical angle of thefunnel is 120°.
(NCERT EXEMPLAR]
SOLUTION Let at any time t, V be the volume of the water in the cone i.e., the volume of the
water-cone VA' B', and let / be the slant height. Then,
V3 / /
O'A' = / sin 60° = and VO' = / cos 60°= -.
2 2

ReadYourFlow.COM
13.16 MATHEMATICS-XII

1
V = -K
V3/f I Kl3

3 2 2 8 O
/I B
dV 3nP_ cU
=> -(i) O'
dt 8 dt
dV q B'
We are given that — = - 4 cm /sec (negative sign due to
dt
decreasing V). .60° 60°.

_ 4 _ ^ j2 dV
Putting----= - 4 in (i)
8 dt dt V
dl 32 Fig. 13.11
=>
dt 3 nl2

w
When / = 3, we get
dl 32 32
cm/sec
dt 37r(3)2 27 n

Flo
Thus, the slant height of the water-cone is decreasing at the rate of
32
cm/sec.

ee
27

Fr
EXAMPLE 25 Wnter is running into a conical vessel, 15 an deep and 5 cm in radius, at the rate of 0.1
cm3/sec. When the water is 6 cm deep, find at what rate is for
ur
(i) the water level rising? (ii) the water-surface area increasing?
(iii) the wetted surface of the vessel increasing?
ks

SOLUTION Let Vbe the volume of the water in the cone i.e. the volume of the water-cone VA' B'
Yo
oo

at any time t. Let VO' = h, O' A' = r and VA' = /. Let a be the semi-vertical angle of the cone. Then,
OA 5 1
eB

tan a =
VO 15 3 A O 5 cm B
O'A' r
r

Also, tan a =
ou

VO' h
ad

l r
=> r,3r = h,
Y

- = - O'
3 h A'
nd

(i) We have. yi
Re

/
1 /if h
Fi

i ft i 5
V = - nr2 h = — n h = — lr [y 3r = h\
3 3 3 27 t
7
dV_ _ 3n
dt ~ 27 dt Fig. 13.12
L.

371 ^2 dh dV q
=> 0.1 = — — = 0.1 cm /sec (Given)
27 dt dt
dh 2.7
=>
dt 3 nh2
(dh 2.7 1
=>
\dt 'h = 6 37r(36) 4071

Thus, the water level is rising at the rate of cm/sec.


407t
(ii) Let A be the water surface area at any time t. Then,
A = nr2

ReadYourFlow.COM
DERIVATIVE AS A RATE MEASURER 13.17

h2
=> A 71 [v 3r = h]
9
dA _ 2k h dh
=>
It ~ ~9~ It
When h = 6, — = , we get
dt 40ti
dA 2k x 6 1
= — cm2/sec
dt 9 X 407t 30
1 2
Thus, the water-surface area is increasing at the rate of — cm /sec

(iii) Let S be the wetted surface area of the vessel at any time t. Then, S = nrl.
From Fig. 13.12 , we have
l2 = VA'2 = VO'2 + O'A'2

w
=> l2 = h2 + r2
, 2 h2
l2 = hA + —

Flo
=> [v 3r=h]
9
VlO h

ree
/ =
3

F
S = KrI
fh^fy/iO h or
ur
=> S = K
U 3
f
ks

=> S = ^Vl0/72
Yo

9
oo

dS 2 k VlO h dh
=>
B

dt 9 dt
re

dh 1
Since h = 6 and, — = . Therefore,
dt 40 7i
ou
ad

dS 2k V10 , 1 Vio 2,
x 6 x----- =------ cm / sec.
Y

dt 9 407i 30
Vio
nd
Re

Thus, the wetted surface area of the vessel is increasing at the rate of cm /sec.
30
Fi

EXAMPLE 26 A water tank has the slope of an inverted right circular cone with its axis vertical and
vertex lower most. Its semi-vertical angle is tan - 1 (05). Water is poured into it at a constant rate of 5
cubic metre per hour. Find the rate at which the level of the water is rising at the instant when the depth of
water in the tank is 4 m. [NCERT]
SOLUTION Let a be the semi-vertical angle of the water tank in the form of cone. Then,
1 r 1 h
tan a = 0.5 = => r =
2 h 2 2 A .0. B

Let V A' B' be the water cone of volume V. Then,


dV A' &L'.7.\V::/b-
= 5 m3/hr [Given]
dt h
dh
We have to find when h = 4 m.
dt
Now,
V
V = - r2 /7
3 Fig. 13.13

ReadYourFlow.COM
r

13.18 MATHEMATICS-XII

1 h)2
=> V = -n h1 = —
71 r 3

3 2 12
dV _ K ^2 ^
=>
dt 4 dt
=> r K a
5 = —x 4 x —2 dh
4 dt
dh ^ 5_ = — 5 7 m/h /u = — 35 m/h /u
=> x—
dt 4k 4 22 88
35
Thus, the rate of change of water level is — m/h.
88
EXAMPLE 27 A man is moving away from a tower 41.6 m high at the rate of 2 m/sec. Find the rate at
which the angle of elevation of the top of tower is changing, when he is at a distance of 30 m from the foot
of the tower. Assume that the eye level of the man is 1.6 m from the ground.

w
SOLUTION Let AB be the tower. Let at any time t, the man be at a distance of x metres from the
tower AB and let 0 be the angle of elevation at that time.Then,
tan 0 = —
PC
Flo B

ee
40
tan 0 = —

Fr
x
=> x = 40 cot 0 -(i) for
ur
d* a r\ 2a dQ
=> — = -40 cosec 0 — 41.6 m
dt dt
ks

dx
We are given that ^ = 7 rn/sec.
Yo
oo

2 = - 40 cosec2 0 —
B

dt C
re

dQ 71
1
=> ...(ii) Fig. 13.14
dt 20 cosec2 0
ou
ad

When x = 30, we get


Y

30 3
cot 0 = — [Putting x = 30 in (i)]
nd

40 4
Re

o o 9 25
cosec- 0=1+ cot2 0 = 1 + — = —
Fi

16 16
2 25
Substituting cosec 0 = —^ in (ii), we get
16
dO 1
—— radians/sec
dt 20 x — 125
16
Thus, the angle of elevation of the top of tower is decreasing at the rate of 4/125 radians/sec.
EXAMPLE 28 A kite is moving horizontally at the height of 151.5 meters. If the speed of kite is 10 m/sec,
how fast is the string being let out; when the kite is 250 m away from the boy who is flying the kite? The
height of the boy is 1.5 m. [NCERT EXEMPLAR]
SOLUTION Let OA be the boy of height 1.5 m and kite be flying at a height OB = 1515 m from
the horizon OX. Therefore, AB = OB-OA =(1515 -15) m = 150m.
Let at any time t, kite be at P such that BP =x and AP = y.
It is given that the kite is moving horizontally at the speed of 10 m/sec.
— =10 m/sec
dt

ReadYourFlow.COM
DERIVATIVE AS A RATE MEASURER 13.19

dy
We have to find the rate at which the string is being let out i.e. — when y = 250 m.
dt
Applying Pythagoras theorem in AABP, we obtain
AP2 =AB2 +BP2
y2 = 1502 + x2
Differentiating with respect to t, we obtain
dx 151.5 m
2y~ = 2x —
' dt dt
=> — = —x 10
dt y
dy _ lOy ...(h) o
dt y Fig. 13.15

low
Whenj/=250
y2 = 1502 + A'2 => 2502 = 1502 + a2 => a2 = 40000 => a = 200
Putting a = 200 and i/ = 250 in (ii), we obtain
^=10x 200 = 8

ee
dt 250
rF
Fr
Hence, the string is being let out at the rate of 8 m/sec.
for
EXERCISE 13.2
u
LEVEL-1
ks
Yo

1. The side of a square sheet is increasing at the rate of 4 cm per minute. At what rate is the area
oo

increasing when the side is 8 cm long?


B

2. An edge of a variable cube is increasing at the rate of 3 cm per second. How fast is the
re

volume of the cube increasing when the edge is 10 cm long?


3. The side of a square is increasing at the rate of 0.2 cm/sec. Find the rate of increase of the
ou
ad

perimeter of the square.


4. The radius of a circle is increasing at the rate of 0.7 cm/sec. What is the rate of increase of its
Y

circumference? [NCERT]
nd

5. The radius of a spherical soap bubble is increasing at the rate of 0.2 cm/sec. Find the rate of
Re

increase of its surface area, when the radius is 7 cm.


Fi

6. A balloon which always remains spherical, is being inflated by pumping in 900 cubic
centimetres of gas per second. Find the rate at which the radius of the balloon is increasing
when the radius is 15 cm. [NCERTl
7. The radius of an air bubble is increasing at the rate of 0.5 cm/sec. At what rate is the volume
of the bubble increasing when the radius is 1 cm?
8. A man 2 metres high walks at a uniform speed of 5 km/hr away from a lamp-post 6 metres
high. Find the rate at which the length of his shadow increases.
9. A stone is dropped into a quiet lake and waves move in circles at a speed of 4 cm/sec. At the
instant when the radius of the circular wave is 10 cm, how fast is the enclosed area
increasing? [NCERT]
10. A man 160 cm tall, walks away from a source of light situated at the top of a pole 6 m high,
at the rate of 1.1 m/sec. How fast is the length of his shadow increasing when he is 1 m
away from the pole?
11. A man 180 cm tall walks at a rate of 2 m/sec. away, from a source of light that is 9 m above
the ground. How fast is the length of his shadow increasing when he is 3 m away from the
base of light?

ReadYourFlow.COM
r

13.20 MATHEMATICS-XII

12. A ladder 13 m long leans against a wall. The foot of the ladder is pulled along the ground
away from the wall, at the rate of 1.5 m/sec. How fast is the angle 0between the ladder and
the ground is changing when the foot of the ladder is 12 in away from the wall.
13. A particle moves along the curve y = x2 + 2x. At what point(s) on the curve are the x and y
coordinates of the particle changing at the same rate?
14. If y = 7x - x3 and x increases at the rate of 4 units per second, how fast is the slope of the
curve changing when x = 2?
15. A particle moves along the curve y = x3. Find the points on the curve at which the
y-coordinate changes three times more rapidly than the x-coordinate.
16. Find an angle 0
(i) which increases twice as fast as its cosine.
(ii) whose rate of increase twice is twice the rate of decrease of its consine.
17. The top of a ladder 6 metres long is resting against a vertical wall on a level pavement,
when the lad der begins to slide outwards. At the moment when the foot of the ladder is

w
4 metres from the wall, it is sliding away from the wall at the rate of 0.5 m/sec. How fast is
the top-sliding downwards at this instance?

Flo
How far is the foot from the wall when it and the top are moving at the same rate?
18. A balloon in the form of a right circular cone surmounted by a hemisphere, having a

ree
diametre equal to the height of the cone, is being inflated. How fast is its volume changing
with respect to its total height h, when h = 9 cm.

F
19. Water is running into an inverted cone at the rate of tt cubic metres per minute. The height
or
ur
of the cone is 10 metres, and the radius of its base is 5 m. How fast the water level is rising
f
when the water stands 7.5 m below the base.
ks

20. A man 2 metres high walks at a uniform speed of 6 km/h away from a lamp-post 6 metres
Yo

high. Find the rate at which the length of his shadow increases.
oo

21. The surface area of a spherical bubble is increasing at the rate of 2 cm2/s. When the radius
B

of the bubble is 6 cm, at what rate is the volume of the bubble increasing? [CBSE2005]
re

22. The radius of a cylinder is increasing at the rate 2 cm/sec. and its altitude is decreasing at
the rate of 3 cm/sec. Find the rate of change of volume when radius is 3 cm and altitude
ou
ad

5 cm. [CBSE 2017]


Y

23. The volume of metal in a hollow sphere is constant. If the inner radius is increasing at the
rate of 1 cm/sec, find the rate of increase of the outer radius when the radii are 4 cm and
nd
Re

8 cm respectively.
24. Sand is being poured onto a conical pile at the constant rate of 50 cm3/minute such that the
Fi

height of the cone is always one half of the radius of its base. How fast is the height of the
pile increasing when the sand is 5 cm deep.
25. A kite is 120 m high and 130 m of string is out. If the kite is moving away horizontally at the
rate of 52 m/sec, find the rate at which the string is being paid out.
o
26. A particle moves along the curve y = (2/ 3)x + 1. Find the points on the curve at which the
y-coordinate is changing twice as fast as the x-coordinate.
27. Find the point on the curve y2 = 8x for which the abscissa and ordinate change at the same
rate. [CBSE 2002C]
Q
28. The volume of a cube is increasing at the rate of 9 cm /sec. How fast is the surface area
increasing when the length of an edge is 10 cm?
29. The volume of a spherical balloon is increasing at the rate of 25 cm /sec. Find the rate of
change of its surface area at the instant when radius is 5 cm. [CBSE 2004,2017]
30. The length x of a rectangle is decreasing at the rate of 5 cm/minute and the width y is
increasing at the rate of 4 cm/minute. When x = 8 cm and y = 6 cm, find the rates of change
of (i) the perimeter (ii) the area of the rectangle. [CBSE 2009]

ReadYourFlow.COM
DERIVATIVE AS A RATE MEASURER 13.21

31. A circular disc of radius 3 cm is being heated. Due to expansion, its radius increases at
the rate of 0.05 cm/sec. Find the rate at which its area is increasing when radius is 3.2 cm.
[NCERT]

________ANSWERS
1. 64 cm2/minute 2. 900 cm3/sec 3. 0.8 cm/sec 4. 1.4 Tt cm/sec
O
5. 11.27Tcm2/sec 6. 1/7i cm/sec 7. 2tt: cm“ /sec 8. 5/2 km/h
9. 807icm2/sec 10. 0.4 m/sec 11. 0.5 m/sec 12. 0.3 radian/sec
13. (-1/2,-3/4) 14. 48 15. (1,1), (-1,-1) 16. (i) 7rc/6 (ii) n/6
17. —?= m/sec, 3V2m 18. 127ccm3/sec 19. 0.%4 metre/minute 20. 3 km/hr
,f5
21. 6 cm3/sec 22. 337Tcm3/sec 23. 1/4 cm/sec 24. 1/2ti: cm/minute
25. 20 m/sec. 26. (1,5/3) and(-l, 1/3) 27. (2,4)

w
28. 3.6 cm2/sec 29. 10 cm2/sec 30. (i) -2 cm/minute (ii) 2 cm/minute
31. 0.320 k cm2/sec

Flo HINTS TO NCERT & SELECTED PROBLEMS

ee
2dx
1. We have, A = x and — =4 cm.

Fr
dt
. 2 dA - dx
Now, A = x => — = 2x — for
ur
dt

2. We have, V =x^ and — = 3.


ks

dt
Yo
oo

., 3 dV „ ? dx
Now, V = x* => — = 3x — 900 cm3/sec
B

dt dt
re

dx
3. We have, P = 4x and — = 0.2
dt
ou
ad

Now, P = 4.x => — = 4 — => — = 4 x 0.2 = 0.8 cm/sec


Y

dt dt dt
4. Let r be the radius and C the circumference of the circle. Then, C = 2nr
nd
Re

It is given that — - 0.7 cm/ sec.


dt
Fi

dC dr dC
Now, C = 2nr => — = 2ti => — =2n x 0.7 cm/sec = 1.4tt cm/sec
dt dt dt
5. We have, S = 47tr2 and — = 0.2
dt
Now,
dS
S = 4nr2 => — = 87ir — => — = Snr (0.2) = 1.671 r => r=7 = 1.6 rex 7 =11.2tc
dt dt dt dt
4 3
6. Let r be the radius and V be the volume of the balloon. Then, V = — nr .
3
dV a
It is given that - 900 cm /sec.

Now, V = — tt r3
3
dv * 2 dr
=> —=4rcr
dt dt

ReadYourFlow.COM
r

13.22 MATHEMATICS-XII

2 dr dv
900 = 4k x 15 = 90 and r = 15
dt dt
dr =1
=>
df k

7. We have, V=~nr3 and — = 05


3 dt
dV 2 dr dV ?
V = — Ter3 => —=4Kr => =47r(l)2(05) =2k
3 dt dt dt
9. Let r be the radius of the circular region and A be its area. Then,
. 2 dA dr o dA _ ^ „ dr
A = nr => — = 2nr — => ----= 2nrx4: = 8nr — =4 cm/sec
dt dt dt dt
dA
= 8ttx 10 = 80k cm2 /sec

low
dt Jr=10
12. Let the bottom of the ladder be at a distance x m from the wall and the top be at a height y
from the ground. Then,
x2 + y2 = 132 and tan 0 - —

ee
x
rF
Fr
x dy dx
dx d0 -y
2x — + 2y — = 0 and sec2 0 dt dt
for
dt dt dt x2
u
dy 3y
ks

3r+ 2y = 0 and sec2 0 — = —^ 2 dx


Yo

— = 15
oo

dt dt x2 dt
B

3x _3y
xx --
re

d-Z=-^ and sec2 0 — = 2y 2


dt 2y dt x2
ou
ad

d0 = _ 3 (a-2 + y2) 3 (*2 + i/2) =_3 (x2 +y2) 3


Y

dt 2 x2y sec2 0 2 x2i/(l + tan20) 2 ? y-2 N 2y


xy 1 +
nd
Re
Fi

When .y=12, A'2+y2=132=> y=5


d0=_^
" dt 10
13. We have.
dx 1
y = x2 + 2x => dy = (2x + 2) — => 1 = 2x + 2 => x = -
dt dt 2
14. We have,
m = Slope of the curve = — =7 - 3x2.
dx
Now, m =7 - 3x2
dm dx
= - 6x
dt dt
dm dm dx
= - 6x (4) => - 24x = -48 — = 4 (given)
dt dt Jx = 2 dt

ReadYourFlow.COM
DERIVATIVE AS A RATE MEASURER 13.23

16. (i) We have.


dQ d dQ
— = 2 — (cos 0) => — = -2sin0 — => sin 0 = -
dt dt dt dt
(ii) It is given that
dQ d dQ
2 (cos 0) => = 2sin0 — => sin0 = — => 0=-
dt dQ
21. We have,
S = 4ti r2 —=8nr

V = — Ter

low
=> dv = 47t r 2 x----
— 2 [Using (i)]

ee
rF
Fr
Hence,— = 6 when r = 6.
dt for
22. We have, V - nr2 h, — = 2 and ^ = - 3
ou
dt dt
ks

T/ 2. dV \ , dr
V = nr h => — = n <2rh —
oo
Y
B

When r = 3, h =5, we obtain


re

dV
— = n (60 - 27) = 337t
dt
ou
ad

25. We have.
Y

y2 =*2 +(120)2
nd

~ dy . dx
Re

=> 2y = 2x —
rft dt
Fi

120 m
dy _ x dx
dt y dt
dx
=> — =52 — -7- =52
dt y df J
Fig. 13.16
Putting 1/ = 130 in y2 = x2 + (120)2, we get x =50.
dy _ 52 x 50 _ 2()
df 130
31. Let r be the radius and A be the area of the disc at any time t. Then, A =nr2.
It is given that ^ = 0.05 cm/sec.

Now, A = nr2
dA dr = 2 7i x 3.2 x 0.05 = 0.320 n cm2/sec.
— = 2nr—=>
dt dt

ReadYourFlow.COM
13.24 MATHEMATICS-XII

_____________________________________VERY SHORT ANSWER QUESTIONS (VSAQs)


Answer each of the following questions in one word or one sentence or as per exact requirement of the
question:
1. If a particle moves in a straight line such that the distance travelled in time t is given by
s = f3 — 6f2 + 9f + 8. Find the initial velocity of the particle.
2. The volume of a sphere is increasing at 3 cubic centimeter per second. Find the rate of
increase of the radius, when the radius is 2 cms.
3. The sides of an equilateral triangle are increasing at the rate of 2 cm/sec. How far is the area
increasing when the side is 10 cms? [NCERT EXEMPLAR]
4. The side of a square is increasing at the rate of 0.1 cm/sec. Find the rate of increase of its
perimeter.
5. The radius of a circle is increasing at the rate of 0.5 cm/sec. Find the rate of increase of its
circumference.

low
6. The side of an equilateral triangle is increasing at the rate of — cm/sec. Find the rate of
3
increase of its perimeter.
7. Find the surface area of a sphere when its volume is changing at the same rate as its radius.

ee
8. If the rate of change of volume of a sphere is equal to the rate of change of its radius, find the
radius of the sphere.
rF
Fr
9. The amount of pollution content added in air in a city due to x diesel vehicles is given
by P (x) = 0.005.t3 + 0.02x2 + 30*. Find the marginal increase in pollution content when 3
for
diesel vehicles are added and write which value is indicated in the above questions.
ou
[CBSE 2013]
ks

10. A ladder, 5 meter long, standing on a horizontal floor, leans against a vertical wall. If the
oo

top of the ladder slides down wards at the rate of 10 cm/sec, then find the rate at which the
Y
B

angle between the floor and ladder is decreasing when lower end of ladder is 2 metres from
the wall.
re

[NCERT EXEMPLAR]
ANSWERS
ou
ad

1. 9 units/unit time 2. 3/16jr cm/sec 3. 10 cm2/sec 4. 0.4 cm/sec


Y

5. rc cm/sec 6. 1 cm/sec 7. 1 square unit 8. 1/2Vti: units


9. 30.255 imits, Pollution level due to x diesel vehicles. 10. 1/20 radian/second
nd
Re
Fi

MULTIPLE CHOICE QUESTIONS (MCQs)


Mark the correct alternative in each of the following:
4 ^ cly
1. If V = — 7tr , at what rate in cubic units is V increasing when r = 10 and — = 0.01 ?
3 dt
(a) k (b) An (c) 407T (d) An/ 3
2. Side of an equilateral triangle expands at the rate of 2 cm/sec. The rate of increase of its area
when each side is 10 cm is
(a) 10V2 cm2/sec (b) 10^3 cm2/sec (c) 10 cm2/sec (d) 5 cm2/sec
3. The radius of a sphere is changing at the rate of 0.1 cm/sec. The rate of change of its surface
area when the radius is 200 cm is
(a) 87icm2/sec (b) 127icm2/sec (c) IOOtt cm2/sec (d) 200 cm2/sec
4. A cone whose height is always equal to its diameter is increasing in volume at the rate of
40 cm /sec. At what rate is the radius increasing when its circular base area is 1 m ?
(a) 1 mm/sec (b) 0.001 cm/sec (c) 2 mm/sec (d) 0.002 cm/sec
5. A cylindrical vessel of radius 05 m is filled with oil at the rate of 0.25 tt m 3 /minute The rate
at which the surface of the oil is rising, is

ReadYourFlow.COM
DERIVATIVE AS A RATE MEASURER 13.25

(a) 1 m/minute (b) 2 m/minute (c) 5 m/minute (d) 1.25 m/minute


o 9
6. The distance moved by the particle in time t is given by x =t -12t + 6t + 8. At the instant
when its acceleration is zero, the velocity is
(a) 42 (b) -42 (c) 48 (d) -48
7. The altitude of a cone is 20 cm and its semi-vertical angle is 30°. If the semi-vertical angle is
increasing at the rate of 2°per second, then the radius of the base is increasing at the rate of
160
(a) 30 cm/sec cm/sec (c) 10 cm/sec (d) 160 cm/sec
9 9
8. For what values of x is the rate of increase of x -5x +5x + 8 is twice the rate of increase of
x?
(a) -3,-1/3 (b) -3,1/3 (c) 3,-1/3 (d) 3,1/3
9. The coordinates of the point on the ellipse 16x2 + 9y2 = 400 where the ordinate decreases at
the same rate at which the abscissa increases, are
(a) (3,16/3 ) (b) (-3,16/3) (c) (3,-16/3 ) (d) (3, - 3)

low
10. The radius of the base of a cone is increasing at the rate of 3 cm/minute and the altitude is
decreasing at the rate of 4 cm/minute. The rate of change of lateral surface when the
radius = 7 cm and altitude 24 cm is
(a) 547tcm2/min (b) 77rcm2/min (c) 27 cm2/min (d) none of these

ee
11. Tire radius of a sphere is increasing at the rate of 0.2 cm/sec. The rate at which the
rF
volume of the sphere increases when radius is 15 cm, is

Fr
(a) 1271 cm3/sec (b) 1807tcm3/sec (c) 2257rcm3/sec (d) 37i:cm3/sec
O
12. The volume of a sphere is increasing at 3 cm /sec. The rate at which the radius increases
for
when radius is 2 cm, is
u
3 3 3 1
ks

(a) -----cm/sec (b) ----- cm/sec (c) ----- cm/sec (d) cm/sec
32tt 167t: 48ti 24 h
Yo
oo

13. The distance moved by a particle travelling in a straight line in t seconds is given by
B

s = 45f + Ilf2 -13. The time taken by the particle to come to rest is
re

(a) 9 sec (b) 5/3 sec (c) 3/5 sec (d) 2 sec
14. The volume of a sphere is increasing at the rate of 47t cm /sec. The rate of increase of the
ou
ad

o
radius when the volume is 288 re cm', is
Y

(a) 1/4 (b) 1/12 (c) 1/36 (d) 1/9


15. If the rate of change of volume of a sphere is equal to the rate of change of its radius, then its
nd
Re

radius is equal to
Fi

(a) 1 unit (b) y/2n units (c) l/Jln unit (d) 1/2Vti unit
16. If the rate of change of area of a circle isequal to the rate of change of its diameter, then its
radius is equal to
(a) 2/tt unit (b) l/7rimit (c) k/2 units (d) tt units
17. Each side of an equilateral triangle is increasing at the rate of 8 cm/hr. The rate of increase
of its area when side is 2 cm, is
(a) 8cm2/hr (b) 4V^ cm2/hr (c) vr3/8cm2/hr (d) none of these
18. If s = f3 - 4f2 + 5 describes the motion of a particle, then its velocity when the acceleration
vanishes, is
(a) 16/9unit/sec (b) - 32/3 unit/sec (c) 4/3 imit/sec (d) -16/3 unit/sec
9
19. The equation of motion of a particle is s = 2f + sin It, where s is in metres and f is in
9
seconds. The velocity of the particle when its acceleration is 2 m/sec , is
(a) 7i + \/"3 m/sec (b) — + V3 m/sec (c) — + V3 m/sec (d) — + i= m/sec
3 3 3 V3

ReadYourFlow.COM
13.26 MATHEMATICS-XII

20. The radius of a circular plate is increasing at the rate of 0.01 cm/sec. The rate of increase of
its area when the radius is 12 cm, is
(a) 144 7icm2/sec (b) 2.4 tc cm2/sec (c) 0.24 rc cm2/sec (d) 0.024 71 cm2/sec
21. The diameter of a circle is increasing at the rate of 1 cm/sec. When its radius is n, the rate of
increase of its area is
(a) 7i:cm2/sec (b) 27rcm2/sec (c) tt2 cm2/sec (d) 2tc2 cm2/sec2
22. A man 2 metres tall walks away from a lamp post 5 metres height at the rate of 4.8 km/hr.
The rate of increase of the length of his shadow is
(a) 1.6 km/hr (b) 6.3 km/hr (c) 5 km/hr (d) 3.2 km/hr
23. A man of height 6 ft walks at a uniform speed of 9 ft/sec from a lamp fixed at 15 ft height.
The length of his shadow is increasing at the rate of
(a) 15 ft/sec (b) 9 ft/sec (c) 6 ft/sec (d) none of these
24. In a sphere the rate of change of volume is
(a) tt times the rate of change of radius

low
(b) surface area times the rate of change of diameter
(c) surface area times the rate of change of radius
(d) none of these
25. In a sphere the rate of change of surface area is

ee
(a) 8k times the rate of change of diameter
rF
Fr
(b) 2k times the rate of change of diameter
(c) 2k times the rate of change of radius for
(d) 8k times the rate of change of radius
26. A cylindrical tank of radius 10 m is being filled with wheat at the rate of 314 cubic metre per
u
hour. Then the depth of the wheat is increasing at the rate of
ks

(a) 1 m/hr (b) 0.1 m/hr (c) 1.1 m/h (d) 0.5 m/hr
Yo
oo

____ ANSWERS
B

1. (b) 2. (b) 3. (c) 4- (d) 5- (a) 6. (b) 7. (b) 8. (d) 9. (a)


re

10. (a) 11. (b) 12. (b) 13. (a) 14. (c) 15. (d) 16. (b) 17. (a) 18. (d)
ou
ad

19. (b) 20. (c) 21. (c) 22. (d) 23. (c) 24. (c) 25. (d) 26. (a)
Y

SUMMARY
dv
nd
Re

1. If y = / (x), then — measures the rate of change of y with respect to .r.


dx
Fi

2. represents the rate of change of y with respect to x at r = Xq.


= *0
3. If the displacement of a particle moving in a straight line at time t is given by s = / (f), then
ds dv d2s dv
(i) v = Velocity at timet = — ,a = Acceleration at timet = —
dt dt ~~d?~V~' ds

(ii) If a particle moving in a straight line comes to rest, then — = 0 and —= 0.


dt dt2
ds d2s
(iii) If a particle moving in a straight line is instantaneously at rest, then — = 0 but —- ^ 0.
dt dt2

ReadYourFlow.COM
CHAPTER
DIFFERENTIALS, ERRORS
AND APPROXIMATIONS

14.1 DIFFERENTIALS

In the chapter on differentiation we defined derivative of 1/ with respect to x i.e. — as the limit of
dx
dy
the ratio — as Ax 0 and considered — as a symbol not as a quotient of two separate quantities
Ax dx

w
dy and dx. In this chapter, we shall give a meaning to the symbols dx and dy in such a way that
the original meaning of the symbol — coincides with the quotient when dy is divided by dx.

Flo
dx
Let y - f(x) be a function of x, and let Ax be a small change in x. Let Ay be the corresponding

ee
change in y. Then,

Fr
lim QL = = /'(*)
Ax -> 0 Ax dx
or
ur
=> = / '(x) + e, where e -> 0 as Ax -> 0
sf
Ax
=> Ay = / '(x) Ax + e A x
k
Yo
oo

=> Ay = /'(x) Ax, approximately


= ^ Ax , approximately
B

=> ■:f'{x)=-^
dx dx
re

NOTE This formula is very useful in the calculation of small changes (or errors) in dependent variable
ou
ad

corresponding to small changes (or errors) in the independent variable and is of great importance in the
theory of errors in Engineering, Physics, Statistics and several other branches of the science.
Y

SOME IMPORTANT TERMS


nd
Re

ABSOLUTE ERROR The error Ax in x is called the absolute error in x.


Fi

RELATIVE ERROR If Ax is an error in x, then — is called the relative error in x


x
Ax-
percentage ERROR If Ax is an error in x, then — x 100 is called percentage error in x.
x
REMARK 1 We have, Ay = /' (x) • Ax + e • Ax.
Since e • Ax is very small, therefore principal value of Ay is f' (x) Ax ivhich is called differential ofy and is
denoted by dy.
i.e. dy = /'(*) Ax or, dy ■ Ax
dx
So, the differential ofx is given by
dx = — • Ax = 1 ■ Ax = Ax
dx
tfy = — A x => dy = — dx
dx dx

ReadYourFlow.COM
14.2 MATHEMATICS-XII

GEOMETRICAL MEANING OF DIFFERENTIALS


In order to understand the geometrical meaning of differentials, let us take a point P (x , y) on
the curve y =f (x), where / (x) is a differentiable real function. Let Q(x + Ax, y + A y) be a
neighboring point on the curve, where A x denotes a small change in x and A y is the
corresponding change in y. It is evident from the Fig. 14.1, that is the slope of secant PQ. But,
Ax
dy
as Ax 0, Ay approaches the limiting value (slope of the tangent at P). Therefore, when
A x —» 0, A y (=QS) is approximately equal to dy (= RS) as shown in Fig. 14.1.
Y

Q(x +Ax,y +Ay)


1

w
R i Ay
dy
P(x, y). r t
s
y=fM

Flo
ee
Fr
o / X
or
ur
/I
f
Fig. 14.1
ks
Yo

Geometrically the values of dx and dy are as shown in Fig. 14.1.


oo

REMARK 2 Let y =/(x) be a function of x, and let Ax be a small change in x. Let the corresponding
B

change in y be Ay. Then ,


re

y + Ay = / (x + Ax)
ou

But, Ay = — • Ax =/ '(x) Ax, approximately


ad

dx
Y

f(x + Ax) = y + Ay
=> / (x + A x) = y + f '(x) ■ Ax, approximately
nd
Re

/ (x + A x) ~ 1/ + fp' A*/ approximately


Fi

Let x be the independent variable and y be the dependent variable connected by the relation
y =/(x). We use the following algorithm to find an approximate change Ay in y due to a small
change Ax in x.
ALGORITHM
STEP I Choose the initial value of the independent variable as x and the changed value as x + Ax.
STEP II Find Ax and assume that dx = Ax.
STEP III Find — from the given relation y =/ (x).
dx
dy
STEP IV Find the value of— at (x, y).
dx
STEP V Find dy by using the relation dy = — dx.
dx
STEP VI Put Ay = dy to obtain an approximate change in y.

ReadYourFlow.COM
DIFFERENTIALS, ERRORS AND APPROXIMATIONS 14.3

ILLUSTRATIVE EXAMPLES

LEVEL-1

EXAMPLE 1 If ij =■ x4 -10 and if x changes from 2 to 1.99, what is the approximate change in y?
Also, find the changed value ofy.
SOLUTION Let x = 2,x + Ax = 1.99. Then, Ax = 1.99 - 2 = - 0.01.
Let dx = Ax = - 0.01
We have.
y = x4 -10

=> —
dx
= 4*3 => Vf —1
d':)
= 4(2)3 = 32
x= 2
dy
dy = — dx
dx

w
dy = 32 (-0.01) = -0.32
=> Ay = - 0.32 approxiamtely [••• A y = dy]
So, approximate change in y = - 0.32.

Flo
ee
When x = 2, we have

Fr
y = 24 -10 = 6
So, changed value of y = y + Ay = 6 + (- 0.32) = 5.68. or
ur
EXAMPLE 2 A circular metal plate expands under heating so that its radius increases by 2%. Find the
sf
approximate increase in the area of the plate if the radius of the plate before heating is 10 cm.
SOLUTION Let at any time, x be the radius and y be the area of the plate. Then, y = re x .
k
Yo
oo

Let Ax be the change in the radius and let Ay be the corresponding change in the area of the plate.
B

Then,
Ax
re

— x 100 = 2 (given)
x
ou
ad

When x = 10,
Ax
Y

— X 100 = 2 — x 100 = 2 Ax = — => dx = — [v dx = A x] •••(i)


x 10 10 10
nd
Re

Now, y = tix2=> — = 27ix=> dy)


= 2071
dx dx Jx = io
Fi

dy - — dx => dy - 20ti x — = Arc => Ay = 4:11 [" dy = Ay]


dx 10
Hence, the approximate change in the area of the plate is 4k.
examples Find the percentage error in calculating the volume of a cubical box ifan error of 1% is made
in measuring the length of edges of the cube.
o
SOLUTION Let x be the length of an edge of the cube and y be its volume. Then, y = x . Let Ax be
the error in x and Ay be the corresponding error in y. Then,
— x 100 =1 (given)
x
dx
=> — x 100 = 1 [•.• dx = A x] ...(i)
x
We have to find — x 100.
y

ReadYourFlow.COM
14.4 MATHEMATICS-XII

Now, *3 => ^ = 3^2


y = dx
dy = dlJ dx
dx
dy _ 3A'2
=> dy = 3x2 dx ^ dx => -o- dx fv y = a*3]
y y y x*
rfy =
=>
y X

— x 100 = 3 f — x 100 = 3 [Using (i)]


y x
=> — x 100 = 3 [••• dy = A y]
y
So, there is 3% error in calculating the volume of the cube.

w
EXAMPLE 4 The time T ofa complete oscillation ofa simple pendulum of length I is given by the equation
l
T = 2n

Flo
g
where g is constant. What is the percentage error in T when I is increased by 1%?

ee
SOLUTION Let A/ be the change in / and AT be the corresponding error in T. Then,

Fr
— x 100 = 1 (given)
/
or
ur
dl
=> — xl00 = 1 [v dl = A/]
f
/
ks

I
Now, T = 2k
Yo
oo

log T = log 2k + (1/2) log / - (1/2) log *


B

=>
fdT 11
re

=>
T dl ~ 2 !
ou
ad

dT = Jf
Y

dl ~ 21
dT = — dl
nd
Re

dl
T
Fi

=> dT = — dl
21
dT = fed
T ~ 2 l

=> — x 100 = -f -x 100


T 2 /
dT 1
=> — xlOO = [Using (i) ]
T 2
=> — x 100 = - [v dT = AT]
T 2
So, there is (1/2)% error in calculating the time period T.
EXAMPLE 5 Find the approximate change in the volume V ofa cube of side x meters caused by increasing
the side by 2%.
SOLUTION Let Ax be the change in x and A V be the corresponding change in V.

ReadYourFlow.COM
DIFFERENTIALS, ERRORS AND APPROXIMATIONS 14.5

Ax
It is given that — x 100 = 2.
x
We have.

=>
dx
dV
A V =----Ax
dx
AV = 3x2 Ax
AV = 3x2 x 2x Ax 2x
=> v — x 100 = 2 => Ax =
100 x 100
AV = 0.06x3

w
Thus, the approximate change in volume is 0.06x m .
EXAMPLE 6 If the radius of a sphere is measured as 9 cm with an error of 0.03 cm, then find the

Flo
approximating error in calculating its volume. [CBSE 2011]
SOLUTION Let r be the radius of the sphere and Ar be the error in measuring the radius. Then,

ee
r = 9 cm and Ar = 0.03 cm.

Fr
Let V be the volume of the sphere. Then,
..4 3 dV 2 dV for
= 4tt x 92 = 324 n
ur
V =— nr0 => — = 4rrr =>
3 dr dr Jr = 9
ks

Let AV be the error in V due to error Ar in r. Then,


Yo
oo

AV = — Ar => AV = 324tt x 0.03 = 9.72 ti cm3.


dr
B

Thus, the approximate error in calculating the volume is 9.72 tc cm .


re

EXAMPLE 7 Find the approximate value off( 3.02), where f(x) = 3x2 + 5x + 3.
ou
ad

[NCERT, CBSE 2014]


SOLUTION Let y =f (x), x = 3 and x + Ax = 3.02. Then, Ax = 0.02.
Y

For x = 3, we get
nd
Re

y =/(3) = 3x 32 +5x 3 + 3 = 45
Fi

Now,
dy
y =f (x) => y = 3x2 +5x+3 => — = 6x + 5 => = 6x3 + 5= 23
dx dxJx=3
Let Ay be the change in y due to change Ax in x. Then,
Ay = — Ax => Ay = 23 x 0.02 = 0.46
* dx y
/ (3.02) = y + Ay = 45 + 0.46 = 45.46.
EXAMPLE 8 Find the approximate volume of metal in a hollow spherical shell whose internal and external
radii are 3 cm and 3.0005 cm, respectively. [NCERT EXEMPLAR]
SOLUTION Let x be the radius and y be the volume of a solid sphere. Then,
4 3

=> d-l=inx2
dx

ReadYourFlow.COM
14.6 MATHEMATICS-XII

We have, x - 3 cm, x + Ax = 3.0005 cm. Therefore, A.t = 0.0005 cm.


Let dx = Ax = 0.0005. Then,
dy
dx
n
=> dy = 4nx dx
=> dy = 47i (3)2 x 0.0005 = O.OlSn cm 3
=> Ay = O.OISti
Hence, the approximate volume of the metal is 0.018rr cm3.
Type II ON FINDING THE APPROXIMATE VALUE USING DIFFERENTIALS
In order to find the approximate values by using differentials, we may use the following
algorithm:
ALGORITHM

w
STEP! Define afunctional relationship between the independent variable x and dependent variable y
by observing the given expression. For example, ifwe have to find the approximate value of the

Flo
square root or cube root of a number, then we define y = 2 or x1^ 3.Ifwe have to find the
approximate value of logarithmic of a given number, then we consider y = log x.

ee
STEP II Choose a value ofx nearest to the valuefor which we have to find y in such a way that either y is

Fr
given for the chosen x or y can be easily computedfor chosen x. For example, if we have to find an
approximate value of(65r^ we take x as 64, because cube root of 64 can be easily calculated.
for
ur
STEP HI Denote the value ofx at which we have to find i/by x + Ax.
STEP IV Find Ax and assume that dx = Ax.
ks
Yo

STEP V Find ^ from the relation obtained in step I.


oo

dx
B

dy
STEP VI Find the value °f~j~ ty putting the value ofx chosen in step II.
re
ou
ad

STEP VII Find dy by using the relation dy = — dx


dx
Y

STEP VIII Assume that Ay = dy.


d

STEP IX Find the value ofy by putting the value ofx chosen in step II in the relation obtained in step I.
Re
n

STEP X The approximate value ofy is y + Ay.


Fi

EXAMPLE 9 Use differentials to approximate V25.2.


SOLUTION Consider the function y =/(x) = Vx. Let x = 25 and x + Ax = 25.2. Theri,
Ax = 25.2 - 25 = 0.2
For x -- 25, we obtain
y = V25 =5 [Putting x =25 in y = Vx]
Let dx - Ax = 0.2.
dy = 1 dy 1 1
Now, y = Vx =>
dx 2V? dxJ x = 25 2 (5) 10
dy 1
dy = — dx => du = — (0.2) = 0.02 => Ay = 0.02 Ay = dy]
J dx * 10
Hence, V25.2 = y + Ay = 5 + 0.02 = 5.02.

ReadYourFlow.COM
DIFFERENTIALS, ERRORS AND APPROXIMATIONS 14.7

EXAMPLE 10 Use differentials to approximate the cube root of 127.


SOLUTION Since we have to find the approximate value of the cube root of 127. So, we consider
the functiony=/(x)=x1/3.
Let .y = 125 and y + Ay = 127. Then, Ay = 127 -125 = 2.
For y = 125, we obtain
y -(125) 1/3 = 5. [Putting y = 125 in y = xl/ 3]
Let dx = Ay = 2.
Now,
i/3 ^ <?y = i dy i i 1
y=Y
dx 3y 2/3 d-v J x= 125 3 (125)2/3 3(5j3^2/3
) 75

dy
dy = — dx
dx

w
1 2
=> ^ = 75(2) = 75

Flo
Ay = ^ [••• Ay = dy]
75

ee
Hence, (127) 1/3 = y + Ay = 5 + — = 5.026.

Fr
75
EXAMPLE 11 Use differentials to find the approximate value of f0.037.
for
ur
SOLUTION Let i/ =/(y) = Vy, y = 0.040 and x + Ax = 0.037. Then, Ay = 0.037 - 0.040 = - 0.003.
For y = 0.040, we obtain
ks

y = VO.040 = 0.2 [Putting y = 0 .040 in y = Vy]


Yo

Let dx = Ax = - 0.003.
oo

dy i i
Now, y = Vy => dy ___
B

=>
dx 2 Vy “x) x= 0.040 2 V0.040 0.4
re

dy dx => dy = —1 3 A
dy = (- 0.003) = - — Ay = - —3 [••■ Ay = dy]
ou
ad

dx J 0.4 400 400


Y

Hence, V0.037 = y + Ay = 0.2 - — = 0.2 - 0.0075 = 0.1925.


J J 400
d
Re

EXAMPLE 12 Use differentials to find the approximate value of log^ (4.01), having given that
n

log,, 4=1.3863.
Fi

SOLUTION Let y =/(y) = log,, y, y = 4 and x + Ax - 4.01. Then, Ay = 0.01.


For y = 4, we obtain
y=/(4)=loge 4=1.3863 [Given]
Let dx = Ax - 0.01
Now,
dy _ i => dy i
y = log,, y =>
dx Y dx x = 4 4
dy dx => dy = - x 0.01 = 0.0025 => Ay = 0.0025
dy = -2- [v A y = dy]
Hence, log,, (4.01) = y + Ay =1.3863 + 0.0025 = 1.3888.
EXAMPLE 13 Using differentials find the approximate value of tan 46°, if it is being given that 1° =
0.01745 radians.
SOLUTION Let y =/(y) = tan x, x = 45°= (re / 4)L' and x + Ax = 46°. Then, Ay = 1° = 0.01745
radians.

ReadYourFlow.COM
14.8 MATHEMATICS-XII

For x = tc/4, we obtain

y = f(n/4) = tan 71/4 = 1


Let dx = Ax = 0.01745.
dy _ dy_ 2 K
Now, y = tan x => -iL sec 2 x => sec — = 2
dx x = k/4
4

dy
dy = ^-dx => dy = 2(0.01745) = 0.03490 => Ay = 0.03490 [••• Aysrfy]
dx
Hence, tan 46° = y + Ay = 1 + 0.03490 = 1.03490.

LEVEL-2

EXAMPLE 14 If in a triangle ABC, the side c and the angle C remain constant, while the remaining
db
elements are changed slightly, using differentials show that = 0.

w
cos A cos B
SOLUTION We are given that the side c and angle C remain constant.

Flo
Q
= k (constant)
sin C

ee
a b a b c

Fr
=> = k
sin 71 sin B sin A sin B sin C
=> a = k sin A and b = k sin B
for
ur
da db
=> — = k cos A and - k cos B
dA dB
ks

da da
Yo

Now, da = — dA => da = k cos A dA => = kdA


oo

dA cos A
B

db db
and. db = dB => db - k cos B dB => = k dB
re

dB cos B
da db
ou
ad

+ = kdA + kdB = kd(A + B) = kd(n-C)


cos A cos B
Y

da db
=> = k(0) = 0 [v k-C = Constant .\ d (rc -C) = 0]
d
Re

cos A cos B
n

da db
Fi

Hence, + = 0.
cos A cos B
EXAMPLE 15 If a triangle ABC, inscribed in a fixed circle, be slightly varied in such away as to have its
vertices always on the circle, then show that + db + dc = 0.
cos A cos B cos C
SOLUTION We know that
a - 2R sin A,b = 2R sin B and c = 2R sin C
da
=> — = 2R cos A ^ = 2R cos B and = 2R cos C [v R = constant]
dA ' dB dC
da db dc
But, da = — dA, db = — dB and dc = — dC
dA dB dC
da = 2R cos A dA, db = 2R cos B dB and dc = 2R cos C dC
da db dc
=> + = 2R {dA + dB + dC)
cos A cos B cos C

ReadYourFlow.COM
DIFFERENTIALS, ERRORS AND APPROXIMATIONS 14.9

da db dc
=> + + = 2Rd (A + B + C) =2R d (n) [v A + B + C = n]
cos A cos B cos C
da db dc
=> + = 2R (0) = 0
cos A cos B cos C
EXERCISE 14.1
LEVEL-1
1. If 1/ = sin .r and .r changes from k/2 to 22/14, what is the approximate change in y?
2. The radius of a sphere shrinks from 10 to 9.8 cm. Find approximately the decrease in its
volume.
3. A circular metal plate expends under heating so that its radius increases by k%. Find the
approximate increase in the area of the plate, if the radius of the plate before heating is

w
10 cm.
4. Find the percentage error in calculating the surface area of a cubical box if an error of 1% is
made in measuring the lengths of edges of the cube.

Flo
5. If there is an error of 0.1% in the measurement of the radius of a sphere, find approximately
the percentage error in the calculation of the volume of the sphere.

ee
6. The pressure p and the volume i> of a gas are connected by the relation pvl A = const. Find the

Fr
percentage error in p corresponding to a decrease of 1/2% in v.
7. The height of a cone increases by k%, its semi-vertical angle remaining the same. What is
or
ur
the approximate percentage increase (i) in total surface area, and (ii) in the volume,
f
assuming that k is small ?
ks

8. Show that the relative error in computing the volume of a sphere, due to an error in
Yo

measuring the radius, is approximately equal to three times the relative error in the radius.
oo

9. Using differentials, find the approximate values of the following:


B

(i) V25.02 (ii) (0.009) 1/3


re

(hi) (0.007) 1/3 (iv) Viol


ou
ad

1
(v) (15),/4 (vi) (255)1/4 (vii)
Y

(2.002)2
d

(viii) log(, 4.04, it being given that logig 4 = 0.6021 and log10 e = 0.4343.
Re
n

(ix) logt, 10.02, it being given that log£, 10 = 2.3026.


Fi

(x) log10 10.1, it being given that log10 e = 0.4343.


(xi) cos 61 °, it being given that sin 60°= 0.86603 and 1° = 0.01745 radian.
1 (xiii) sin f —
(xii)
V25l 14
11 7C
(xiv) cos (XV) (80) 1/4
36
(xvi) (29) 1/3 (xvii) (66) 1/3

(xviii) V26 [CBSE2000] (xix) V37 [CBSE 2000]


1/4
(xx) V048 [CBSE 2002C] (xxi) (82) [CBSE 2005]
1/4
(xxii) |) (xxiii) (33) 1/5

(xxiv) V36.6 (xxv) 25 1/3


(xxvi) V495 [CBSE 2012] (xxvii) (3.968) 3/2 [CBSE 2014]

ReadYourFlow.COM
14.14 MATHEMATICS-XII

SUMMARY
1. Let 1/ = f(x) be a function of x, and let Ax be a small change in x and Ay be the corresponding
change in y. Then,
Ay = — Ax approximately.
dx
dy
— Ax is called differential of y and is denoted by dy.
dx
2. Following are some useful results on differentials:
(i) If / (x) is a constant function, then its differential is zero.
(ii) If i/ = cu, then dy = c dn, c is a constant.
(iii) If y = u ± v,. then dy = du±dv
(iv) If y = uv, then dy = u dv + v du
v du -u dv
(v) If y = - , t hen dy =

w
V v2
(vi) If y = / (x),r then dy = f (x) dx.

Flo
3. (i) Lety = /(x)beagivenfunctionofx. If Ax is an error in x, then the corresponding error
Ay in y is given by Ay = — Ax.

ee
dx

Fr
The error A.x in x and Ay in y are known as absolute errors.
Ax
(ii) If Ax is an e rror in x, then — is called relative error in x.
or
ur
x
f
Ax
(iii) If Ax is an error in x, then x 100 is called the percentage error in x.
ks

x
Yo
oo
B
re
ou
ad
Y
nd
Re
Fi

ReadYourFlow.COM
CHAPTER 15
MEAN VALUE TH EOREMS

15.1 ROLLE’S THEOREM


STATEMENT Let f be a real valued function defined on the closed interval [a,, b] such that
(i) it is continuous on the closed interval [a, b],
(ii) it is differentiable on the open interval (a, b),
and, (Hi) f(a) - f(b).

w
Then, there exists a real number c e (a, b) such that f'(c)= 0.

Flo
GEOMETRICAL INTERPRETATION OF ROLLE’S THEOREM Let f(x) be a real v.mluedfunction defined on
la, b] such that the curve y =f(x) is a continuous curve between points A(a, /(a)) and B(b, f(b)) and it is

ee
possible to draw a unique tangent at every point on the curve y =/(x) between points A and B. Also, the

Fr
ordinates at the end points of the interval [a, b] are equal. Then, there exists at least one point (c, f(c))
lying between A and B on the curve y =f(x) where tangent is parallel to x-axJs.
(c,m yv
for
ur
y,
A(a,f(a)) B(b,f(b))
ks

B(b,fm y =/(*)
Yo

A(a,f(a))
oo

(5 /(<-'))
eB

/ X
y = /(*) o (o,0) (b, 0)
r
ou
ad

o (a,0) (c, 0) (b,0) X


Y

'•.........I* -

(c,/(c)) (c,f(c))
Fig. 15.1 Fig. 15.2
nd
Re

ALGEBRAIC INTERPRETATION OF ROLLE’S THEOREM Let f(x) be a polyi aomial with a and b as its
Fi

roots. Since a polynomial function is everywhere continuous and diffe rentiable and a and b are
roots of f(x), therefore f(a)=f(b)=0. So, f(x) satisfies conditions of Rolle's theorem.
Consequently, there exists c e (a, b) such that/'(c) = Oi.e.f'(x) = Oatx = = c. In other words x = cis
a root of / '(x). Thus, algebraically Rolle's theorem can be interpreted as follows:
Between any two roots of a polynomial f(x), there is always a root of its den i mtive f '(x).
REMARK On Rolle's theorem generally two types of problems are formula] ed.
(a) To check the applicability of Rolle's theorem to a given function on a /given interval,
(b) To verify Rolle's theoremfor a givenfunction on a given interval. In bo ih types ofproblems wefirst
check whether f(x) satisfies conditions of Rolle's theorem or not. Th e following results are very
helpfid in doing so.
(i) A polynomial function is everywhere continuous and differentiable.
(ii) The exponential function, sine and cosine functions are emery where continuous and
differentiable.
(iii) Logarithmic function is continuous and differentiable in its domain.
(iv) tan x is not continuous at x = ± n/2, ± 3k/2, ± 5n/2...

ReadYourFlow.COM
15.2 MATHEMATICS-XII

(v) | .v | is not differentiable at x = 0.


(vi) Iff '(x) tends to ±ccas x -> k, then f(x) is not differentiable at x = k.
1
For example, if f(x) =(2x -1) 1/2 , then f '(x) = is such that lim f '(x) = co.
a: —>( 1/ 2) + '
So, f(x) is not differentiable at x = 1/2.
(vii) The sum, difference, product and quotient of continuous (differentiable) functions is
continuous (di fferentiable).

ILLUSTRATIVE EXAMPLES

LEVEL-1

Type I TO CHECK THE APPLICABILITY OF ROLLE’S THEOREM


EXAMPLE 1 Discuss the applicability of Rolle's theorem for the following functions on the indicated

w
intervals:
(i) f(x) =|*l on [-I,!]) (ii) f(x) = 3 + (x - 2// 3 on [1, 3]

Flo
(iii) f(x) = tan x on [0, 71:]
SOLUTION (i) We have,

ee
-:c, when -1 < x < 0

Fr
f(x)=\x\ =
when 0<x<l
for
Since a polynomial function is everywhere continuous and differentiable. Therefore, f(x) is
ur
continuous and differenitiable for all x < 0 as well as for all x > 0 except possibly at x = 0.
So, consider the point x ■— 0.
ks
Yo

We have.
oo

lim /(x) = lim -x = 0 and, lim /(x) = lim x = 0


B

x -> 0 ;t- -> 0 .r —> 0+ x -» 0


re

lim /(x) = lim /(x) = /(0)


x -> 0_ x -» 0 -
ou
ad

Thus,/(x) is continuous at x = 0. Hence,/(x) is continuous on [-1,1].


Y

Now,
/(x) -m
nd

(LHD at x = 0) == lim
Re

X -> 0“ x-0
Fi

-x-0
=> (LHD at x = 0) == lim [•.• /(x) = - x for x < 0 and /(0) = 0]
x -> 0 x
-X
=> (LHDatx = 0) lim = lim -1 = -1
x -> 0 x x -> 0

and. (RHD at x = 0) = lim


fix) -/(0) lim
x-0
[-.• /(x) = x for x > 0]
x -> 0+ x-0 x -> 0 x

=> (RHD at x = 0) - lim - = lim 1=1


x -»0 x x -> 0
(LHD at x = 0) * RHD at x = 0.
This shows that /(x) is no t differentiable at x = 0 € (-1,1). Thus, the condition of derivability at
each point of (- 1,1) is not satisfied.
Hence, Rolle's theorem is not applicable to /(x) = | x | on [ - 1,1].
(ii) We have,/(x) = 3 + '(x - 2)2/ 3, x e [1, 3]
f'(x) = (2/3) (xr-2) -1/3

ReadYourFlow.COM
MEAN VALUE THEOREMS 15.3

Clearly, lim f '(x) = co. So, /(x) is not differentiable at x = 2 e (1, 3).
2+

Tence, Rolle's theorem is not applicable to /(x) = 3 + (x - 2) 2/ 3 on the interval [1, 3].
viii) We have, /(x) = tan x, x e [0, tt].
Since ^ e [0, tt] and/(x) is not continuous at x = So, the condition of continuity at each point
of [0, xt] is not satisfied.
Hence, Rolle's theorem is not applicable to / (x) = tan x on the interval [0, Tt].
EXAMPLE 2 Discuss the applicability of Rolle's theorem on the function
x2+l, when 0<x<l
fix) = [NCERT EXEMPLAR]
3 -x. ivhen 1 < x < 2
SOLUTION Since a polynomial function is everywhere continuous and differentiable.

w
Therefore, f (x) is continuous and differentiable at all points except possibly at x = 1.
Let us now discuss the differentiability of/(x) at x = 1.
fix) -/(l)
(LHD at x =1) = lim
x->r x —1

Flo
ree
(x2 +1)-(1 +1)
(LHD at x =1) = lim [v /(x) = x2 +1 for 0 < x < 1]

F
x —> 1 x—1
2-l or
ur
(LHD at x =1) = lim - = lim (x +1) = 2
x -> 1 X -1 x —> 1
sf

(RHD at x = 1) = lim
fix) -/(l) = lim (3 - x) - (1 + 1) lim
-(*-1)
= -1
k

and.
Yo

+ x —1 x —> 1 x —1 x —> 1 x -1
oo

x —> 1
(LHD at x = 1) ^ (RHD at x = 1).
B
re

So,/(x) is not differentiable at x = 1. Thus, the condition of differentiability at each point of the
given interval is not satisfied.
ou
ad

Hence, Rolle's theorem is not applicable to the given function on the interval [0, 2].
Y

Type II ON VERIFICATION OF ROLLE'S THEOREM FOR A GIVEN FUNCTION DEFINED ON A GIVEN


INTERVAL
nd
Re

EXAMPLE 3 Verify Rolle's theorem for the function f(x) = x2 -5x + 6 on the interval [2, 3].
Fi

(CBSE 2002C]
SOLUTION Since a polynomial function is everywhere differentiable and so continuous also.
Therefore, f(x) is continuous on [2, 3] and differentiable on (2, 3).
Also, f(2) = 22-5x2 + 6 = 0 and /(3) = 32 - 5 x 3 + 6 = 0
/(2) = /(3)
Thus, all the conditions of Rolle's theorem are satisfied. Now, we have to show that there exists
some c e (2, 3) such that / '(c) = 0.
For this we proceed as follows.
We have,
/ (x) = x2 -5x + 6 => fix) = 2x -5
fix) =0 ^ 2x-5 = 0 => x = 2.5
Thus, c = 2.5 e (2, 3) such that/ '(c) = 0. Hence, Rolle's theorem is verified.
EXAMPLE 4 Verify Rolle's theorem for the function fix) = x(x - 3)2, 0 < x < 3.

ReadYourFlow.COM
15.4 MATHEMATICS-XII

SOLUTION We have, f(x) =x3 -6x2 + 9x.


We know that a polynomial function is everywhere differentiable and hence continuous also.
So, f(x) being a polynomial function is continuous on [0, 3] and differentiable on (0, 3). Also,
/(0) =/(3) = 0. Thus, all the conditions of Rolle's theorem are satisfied. Now we have to show
that there exists c e (0, 3) such that / '(c) = 0.
We have.
f(x) = x3- 6x2 + 9x
=> f'(x) = 3x2-Ux + 9
f(x) = 0 => 3.r2 - 12a: + 9 = 0 => x2 - 4x + 3 = 0 => x = 1, 3
Thus, c = 1 e (0, 3) such that / '(c) = 0. Hence, Rolle's theorem is verified.
EXAMPLES Verify Rolle's theorem for the function /(x) = x3 - 6x2 + llx - 6 on the interval
[1,3],

w
SOLUTION Since a polynomial function is everywhere continuous and differentiable, therefore
/(x) is continuous on [1, 3] and differentiable on (1, 3).

Flo
Also, /(l) = 13 - 6 x l2 + 11 x 1 - 6 = 0 and /(3) = 33 - 6 x 32 + 11 x 3 - 6 = 0

ee
/(i) = m

Fr
Thus, all the three conditions of Rolle's theorem are satisfied. Now we have to show that there
exists c e (1, 3) such that / '(c) = 0. for
ur
We have.
f(x) = x3 - 6x2 + llx - 6
ks

=> /'(x) = 3x2 -12x + 11


Yo
oo

1
/'(x) = 0 => 3x2 -12x + 11 = 0 => x = 12 ±V144-13^ => x = 2 ± -==
B

6 V3
re

Clearly, both the values ofx lie in the interval (1,3). Thus, c = 2 ± ~ e(2, 3) such that/'(c) =0.
ou

V3
ad
Y

Hence, Rolle's theorem is verified.


EXAMPLE 6 Verify Rolle's theorem for the function f{x) =(x - a)m (x -b)n on the interval [a, b],
d
Re

where m, n are positive integers.


n
Fi

SOLUTION We have, /(x) = (x -a)m (x -h)" wherem, n eN


On expanding (x - a)m and (x -b)" by binomial theorem and then taking the product, we find
that /(x) is a polynomial of degree (m + n). Since a polynomial function is everywhere
differentiable and so continuous also. Therefore,/(x) is continuous on [a, b] and is derivable on
(a, b).
Also, f(a) = f(b) = 0.
Thus, all the three conditions of Rolle's theorem are satisfied.
Now, we have to show that there exists c e {a, b) such that / '(c) = 0.
We have.
/(x) = {x-a)m{x-b)n
=> /'(x) = m(x-a) m-1 {x-b)n + {x-a)m n{x-bf - 1
=> /'(x) = (x-a)m-l(x-b)n-1{m(x-b) + n{x-a)}

ReadYourFlow.COM
MEAN VALUE THEOREMS 15.5

=> f'(x) = (x-a) m-l (x-b) /j-1 [x(m + n) - (mb + no)}


f'(x)=0
=> (x-a) m-l (x-b) n-l (x(m + n) - (mb + tm)} = 0
(x-a) =0 or, (x-b) = 0 or, x(m + n)-(mb+na) = 0
mb + na
=> x - a or, x = b or, x =
m+n
mb + na mb + na
Since x = divides (a, b) into the ratio m : n. Therefore, e(a, b).
m+n m+n
mb + na
Thus, c = e (a, b) such that / '(c) = 0. Hence, Rolle's theorem is verified.
m+n

w
EXAMPLE 7 Verify Rolle's theorem for the function f(x) = /4 - x2 on [-2, 2].
SOLUTION Clearly, f(x) is defined for all x e [- 2, 2] and has a unique value for each x e [- 2, 2].

Flo
So, at each point of [ - 2,2], the limit of/(x) is equal to the value of the function. Therefore, f(x) is
continuous on [ - 2, 2].

ree
Now, f(x) =a/4-x2 => /'(x) = -x
V4-x2

F
Clearly,/'(x) = exists for all x e (- 2, 2). So, f(x) is differentiable on (- 2, 2).

or
ur
f
Also, /(- 2) =/(2) = 0. Thus, all the three conditions of Rolle's theorem are satisfied.
ks
Now we have to show that there exists c e (- 2, 2) such that / '(c) = 0.
Yo
oo

We have.
-x
eB

f(x) = ^-x2 => /'(x) =


A1?
r

-x
fix) = 0 => = 0 => x = 0
ou
ad
Y

Since c = 0 e (- 2, 2) such that / '(c) = 0. Hence, Rolle's theorem is verified.


nd
Re

x2 + ab
EXAMPLES Verify Rolle's theorem for thefunction f(x) =log • on[a,b], where0<a<b.
x(a + b)
Fi

SOLUTION We have,
x2 + ab
fix) = log ■ = log (x2 + ab) - log x - log (a + b).
x(a + b)

Since logarithmic function is differentiable and so continuous on its domain. Therefore, f(x) is
continuous on [a, b] and differential on (a, b).
a2 + ab b2 + ab
Also, f(a) = log = log 1 = 0, and f(b) = log = log 1 = 0.
a(a + b) b (a + b)

fia) = f(b)
Thus, all the three conditions of Rolle's theorem are satisfied.
Now, we have to show that there exists c e (a, b) such that / '(c) = 0.

ReadYourFlow.COM
15.6 MATHEMATICS-XII

We have,
f(x) = log (x + ab) - log x - log (a + b)
2x 1 x2 - ab
/'W =
x2 + ab X x (x2 + ab)
x2 - ab
f'(x) = 0 => = 0 => x2 = ab => x = yfab
x (x2 + ab)
Since a < Jab <b. Therefore, c = Jab e(a,b) is such that /'(c) =0. Hence, Rolle's theorem is
verified.
EXAMPLE 9 Verify Rolle's theorem for each of the following functions on indicated intervals :
(i) f(x) = sin2 x on 0 < x < Tt (ii) /(x) = sin x + cos x -1 on [0, tt/2]
(iii) f(x) = sin x - sin 2x on [0, re]
SOLUTION (i) Since sin x is everywhere continuous and differentiable and the product of

w
two continuous (differentiable) functions is continuous (differentiable). Therefore,
f(x) - sin x = sin x. sin x is continuous on [0, Tt] and differentiable on (0, Tt).

Flo
Also, /(0) = sin2 0 = 0 and /(Tt) = sin2 Tt = 0
/(0) = /(Tt)

ee
Thus, /(x) satisfies all the three conditions of Rolle's theorem.

Fr
Now, we have to show that there exists c e (0, Tt) such that f '(c) = 0.
We have. for
ur
/(x) = sin x => f'(x) = 2 sin x cos x = sin 2x
/'(x) = 0 => sin 2x = 0 => 2x = Tt=> x = Tt/2.
ks
Yo

Since c = Tt/2 e (0, Tt) such that/ '(c) = 0. Hence, Rolle's theorem is verified.
oo

(ii) Since sin x and cos x are everywhere continuous and differentiable.Therefore,
B

/(x) = sin x + cos x -1 is continuous on [0, Tt/2] and differentiable on (0, Tt/2).
re

Also, /(0) =sin 0 + cos 0-1=0 and /(Tt/2) = sin Tt/2 + cos Tt/2 -1=1-1=0
/(0) = / (Tt/2).
ou
ad

Thus,/(x) satisfies conditions of Rolle's theorem on [0, Tt/2]. Therefore, there exists c (0, Tt/2)
Y

such that / '(c) = 0.


Now,
d
Re
n

/(x) = sin x + cos x -1 => / '(x) = cos x - sin x


Fi

fix) = 0
=> cos x - sin x = 0 => sin x = cos x => tan x = 1 => x = Tt/4
Thus, c = Tt/4 g (0, Tt/2) such that / '(c) = 0. Hence, Rolle's theorem is verified.
(iii) Since sine function is everywhere continuous and differentiable, therefore so are sin x and
sin 2x. Consequently, /(x) = sin x - sin 2x is continuous on [0, tc] and differentiable on (0, Tt).
Also, /(0) = sin 0 - sin 0 = 0 and /(tt) = sin Tt - sin 2tt = 0
/(0) = /(Tt)
Thus,/(x) satisfies all the three conditions of Rolle's theorem on [0, tc]. Consequently there exists
c e(0, Tt) such that / '(c) = 0.
Now, /(x) = sin x - sin 2x => / '(x) = cos x - 2 cos 2x
fix) =0
=> cos x - 2 cos 2x = 0
>
=> cos x - 2 (2 cos2 x -1) = 0
2
=> 4 cos x - cos x - 2 = 0

ReadYourFlow.COM
MEAN VALUE THEOREMS 15.7

1 ± V33 = 0.8431 or, - 05931


=> COS X
8
-l (0.8431) or, cos-1 (- 05931)
=> .T = COS
-1 (0.8431) or, 180°-cos-1 (05931) [v COS 1 (- x) = 7t - COS 1 x]
=> X = COS

=> x = 32° 32' or, x = 126° 23'


Thus, c = 32° 32'; 126° 23' e (0, tc) such that f'(c) = 0. Hence, Rolle's theorem is verified.
EXAMPLE 10 Verify Rolle's theorem for each of the following functions on the indicated intervals:
(i) /(x) = x (x + 3) c - x/2 on [- 3, 0]. [NCERT EXEMPLAR]

(ii) /(x) = ex (sin x - cos x) on [n/ 4,57t/4].


SOLUTION (i) Since a polynomial function and an exponential function are everywhere

low
continuous and differentiable. Therefore, /(x), being product of these two, is continuous on
[ - 3, 0] and differentiable on (- 3, 0).
Also, /(- 3) =-3(-3 + 3)c3/2 = 0 and/(0) = 0

/(- 3) =/(0)

ee
rF
Thus, /(x) satisfies all the three conditions of Rolle's theorem on [ - 3, 0].

Fr
Consequently, there exists c e (- 3, 0) such that / '(c) = 0.

for
Now, /(x) = x(x + 3) c -x/2
u
- x2 + x + 6
=> / '(x) =(2x + 3) c -x/2 + (x2 + e~X/2 =e~x/2
ks
2
Yo
oo

/ '(x) = 0
eB

e-x/2 -x2 + x + 6
=> =0
2
r
ou
ad

=> -x2 + x + 6 = 0 => x2 - x - 6 = 0 => (x - 3) (x + 2) = 0 => x = -2,3


Y

Thus, c = - 2 e (- 3, 0) such that / '(c) = 0. Hence, Rolle's theorem is verified.


(ii) Since an exponential function and sine and cosine functions are everywhere continuous and
nd
Re

differentiable. Therefore,/"(x) is continuous on [ti/4, 57r/4] and differentiable on (n/4, 5tc/4) .


Fi

TC 1
Also, / = c1^4 J^sin --cos —
4 4
= c*/4fi
V2 V2
= 0

5k 5ti
and, /(f) = ^( sm------cos
4

4
= e5n/4 - sm
■ n

4
71
+ cos —1
4
= n0

43 - 4?)
Thus, f(x) satisfies all the three conditions of Rolle's theorem on [k/4, Sit/4]. Consequently,
there exists c e (k/4, 5k/4) such that f '(c) = 0.
Now, /(x) = ex (sin x - cos x)
=> / '(x) = ex (sin x - cos x) + ex (cos x + sin x) = 2ex sin x
f'(x) = 0 => 2ex sinx = 0=>sinx = 0=>x = K [v ex * 0]
Thus, c = k e (k/4, 5k/4) such that /'(c) =0. Hence, Rolle's theorem is verified.

ReadYourFlow.COM
15.8 MATHEMATICS-XI!

Type Ill MISCELLANEOUS EXERCISES


EXAMPLE 11 It is given that for the function f(x) = x - 6x + ax + b on [1, 3], Rolle's theorem holds
with c = 2 + j=. Find the values of a and b, if /(l) = f(3) = 0.
43
SOLUTION We are given that/(l) =/(3) = 0.
l^-6xl+a + b = 3^-6x3^ + 3a + b = 0 => a + b = 5 and 3a + b = 27
Solving these two equations for a and b, we get: a = 11 and b = -6.
We now verify whether for these values of a and b, f '(c) is zero or not.
We have,
f(x) = x3 - 6x2 + ax + b
=> f(x) = x3 - 6x2 + 11 x - 6 [v fl=ll/l7=-6]
/'(x) = 3x2-12X + 11

w
1 | + ll=12 + ^
/'(c) = 3c2-12c+ 11 = 3^2 +4= j -12^2 + ^ 12 + l-24--^
12 + ll =0
43 43 43

Flo
Hence, a = 11 and b = -6.
EXAMPLE 12 It is given that for the function f given by f (x) = x3 + bx2 + ax, x e[l, 3] . Rolle's

ee
theorem holds with c = 2 + -j= . Find the values of a and b.

Fr
v3
1
SOLUTION It is given that the Rolle's theorem holds for/ (x) defined on [1,3] with c = 2 +
for
ur
f (1) = / (3) and /' (c) = 0 43'
=> 1 + b + a = 27 + 9b + 3a and 3c2 + 2bc + a = 0
ks

1
Yo

=> 2a + 8b + 26 = 0 and 3 f 2 +-i=l + 2H 2 + + ^ =0


oo

43 43
B

=> a + 4b+13 = 0 and a + 4b + 13 +-^ (b + 6) = 0


re

V3
=> a + 4b + 13 = 0 and 0 + -3= (b + 6) = 0
ou

43
ad

=> a + 4b + 13 = 0 and b = - 6
Y

=> a = 11 and b = —6
d

EXAMPLE 13 Find the point on the curve y = cos x -1 , x e - , — at which the tangent is parallel to
Re
n

the x-axis. L2 2 .
Fi

SOLUTION Let f (x) = cos x -1. Clearly, / (x) is continuous on [7t/2 , 3 rc/2] and differentiable
on (n/2,3 tt/2) .
Also, f\j) cos -
2
. - / ?.
Thus, all the conditions of Rolle's theorem are satisfied. Consequently, there exists at least one
point c e (tc/2 , 3 7t/2) for which /' (c) = 0. But,
f(c)=0 =+ sin c = 0 => c = k.
f (c) = COS 71-1 = - 2
By the geometrical interpretation of Rolle's theorem (71, - 2) is the point on 1/ = cos x -1 where
tangent is parallel to x-axis.
EXERCISE 15.1
LEVEL-1
1. Discuss the applicability of Rolle's theorem for the following functions on the indicated
intervals:
(i) /(x) = 3 + (x - 2) 2/3 on [1, 3]

ReadYourFlow.COM
MEAN VALUE THEOREMS 15.9

(ii) f(x) =[x] for -1 <x <1, where [xj denotes the greatest integer not exceeding x
(iii) /(x) = sin - for -1 < x < 1 (iv) /(x) = 2x2 - 5x + 3 on [1, 3]
x
2/3 on [-1,1] - 4x + 5, 0 < x < 1
(v) f(x)=x (Vi) f(x) =
2x - 3, 1 <x<2
2. Verify Rolle's theorem for each of the following functions on the indicated intervals:
(i) /(x) = x2 - 8x + 12 on [2, 6]
(ii) /(x) = x2 - 4x +3 on [1, 3] [NCERT, CBSE2007]
(iii) f(x) = (x -1) (x - 2)2 on [1,2]
(iv) /(x) = x(x -1)2 on [0,1] [NCERT EXEMPLAR]
(v) /(x) =(x2-l)(x-2) on [-1,2]
(vi) /(x) = x (x —4)2 on [0, 4]

w
(vii) /(x) = x (x — 2)2 on [0, 2]
(viii) /(x) = x2 +5x+ 6 on [- 3, - 2]

Flo
3. Verify Rolle's theorem for each of the following functions on the indicated intervals:
(i) /(x) = cos 2 (x - jr/4) on [0,7t/2] (ii) f(x) = sin 2x on [0, 7t/2]

ree
(iii) /(x) = cos 2x on [- %/4, k/4] (iv) /(x) =cA sin x on [0, re]

F
(v) /(x) = ex cos x on [-7t/2, tc/2] (vi) /(x) = cos 2x on [0, tt]
sin x

or
ur
(vii) f(x) = on 0 < x < rc (viii) /(x) = sin 3x on [0, tt]

(ix) f(x)=e]~x2 on [-1,1] f


ks
Yo
(x) /(x) = log (x2 + 2) - log 3 on [-1,1] [NCERT EXEMPLAR]
oo

(xi) /(x) = sin x + cos x on [0, n/2] (xii) /(x) = 2 sin x + sin 2x on [0, tc]
eB

r kx
(xiv) /(x) = — - 4 sin2 x on 0,—
(xiii) /(x) =----sin---- on [-1, 0]
2 6 71 6_
(xv) f(x) = 4sin x on [0, ti] (xvi) /(x) = x2 - 5x + 4 on [1, 4] [CBSE 2007]
r
ou
ad

(xvii) / (x) = sin4 x + cos4 x on 0, ^ [NCERT EXEMPLAR]


Y

(xviii) / (x) = sin x - sin 2x on [0, tt]


nd

7. Using Rolle's theorem, find points on the curve y =16 - x2, x e [-1,1], where tangent is
Re

parallel to x-axis. [NCERT, CBSE 2000]


Fi

8. At what points on the following curves, is the tangent parallel to x-axis?


(i) y = x2 on [-2, 2] (ii) y = e1~x on [-1,1]
(iii) y = 12 (x + 1) (x - 2) on [-1, 2].
9, If /: [-5,5] -> R is differentiable and if /' (x) doesn't vanish anywhere, then prove that
/(-5)*/(5). [NCERT]
10. Examine if Rolle's theorem is applicable to any one of the following functions:
(i) / (x) = [x] for x e [5, 9] (ii) / (x) = [x] for x e [- 2, 2] [NCERT]
Can you say something about the converse of Rolle's Theorem from these functions?
11. It is given that the Rolle's theorem holds for the function/(x) = x3 + bx2 + cx, x e [1, 2] at the
4
point x = — .Find the values ofb and c.
ANSWERS
1. (i) Not applicable (ii) Not applicable (iii) Not applicable
(iv) Not applicable (v) Not applicable (vi) Not applicable.

ReadYourFlow.COM
15.10 MATHEMATICS-XII

7. (0,16) 8. (i) (0,0) (ii) (0,e) (iii) (1/2, -27) 11. b = 5, c = 8

HINTS TO NCERT& SELECTED PROBLEMS


1
1. (i) We have, / '(x) = => / '(.r) does not exist at x = 2
(x-2) 1/3

(ii) Since f(x) is not continuous at x = 0 e [-1,1]. So, Rolle's Theorem is not applicable.
(iii) Since /(x) is not continuous at x = 0 e [-1,1]. So, Rolle's Theorem is not applicable.
(iv) Since /(l) * /(3). So, Rolle's Theorem is not applicable
(v) Since / '(x) =(23) x -1/3 does not exist at x = 0. So, Rolle's theorem is not applicable.
5. (ii) We have, /(x) = x2 - 4x + 3, x e [1, 3].

w
Clearly, / (x), being a polynomial function, is continuous on [1, 3] and differentiable on
(1, 3). Also, / (1) =/ (3). Thus, / (x) satisfies all the conditions of Rolle's theorem.
Now, /(x) = x 2 - 4x + 3 => /' (x) = 2x - 4
/' (x) = 0 => x = 2.

Flo
ee
Clearly, c = 2 e (1, 3) such that / '(c) = 0. Hence, Rolle's theorem is verified.

Fr
7. The equation of the curve is y = 16 -x2. Let P (xj, iq) be a point on it where tangent is
or
ur
parallel to x-axis. Then, — = 0.
f
ydxjp
ks

dy
Yo

Now, y =16 - x2 => — = - 2x => = - 2xa


oo

dx dx Jp
B

= 0 => - 2x-i = 0 => Xj = 0


re

••• [dxjp
ou

Since P (xj , y1) lies on the curve y = 16 - x 2 . Therefore, yj = 16 - x~2


ad
Y

Whenxj = 0, i/j =16-Xj2 gives y^ = 16. Hence, (0,16) is the required point.
d
Re

9. If possible, let /(-5) =/(5). Then, by Rolle's theorem there exists c e(-5,5) such that
n

/'(c)=0. This is a contradiction to the fact that /'(x)*0 for any xe(-5,5). Hence,
Fi

/(-5)^/(5).
10. (i) /(x)=[x] is discontinuous at x=5, 6,7,8, 9 in [5, 9]. So, Rolle's theorem is not
applicable. The converse of Rolle's theorem does not hold good, because / '(x) = 0 for all
x e(5, 6) u(6, 7) u(7 8)u(8, 9). But,/(x) is neither continuous nor differentiable on [5,9],
(ii) Proceed as in (i)

15.2 LAGRANGE’S MEAN VALUE THEOREM


STATEMENT Let f(x) be a function defined on [a, b] such that
(i) it is continuous on [a, b], (ii) it is differentiable on (a, b).

Then, there exists a real number c e (a, b) such that f '(c)


m-f(a)
b -a
PROOF Consider a function <J)(x) =/(x) + Ax, where A is a constant to be chosen in such a way
that <t>(fl) =<b(b).

ReadYourFlow.COM
MEAN VALUETHEOREMS 15.11

Now, <|>(rt) = (j)(b)

=> f(a) + Aa = f(b) + Ab => f(b) -f(a) - -A(b-n) A =


m -/{a)
b -a
(i) Since f(x) is continuous on [a, b] and At, being a polynomial function, is everywhere
continuous. Therefore, <j>(A‘), being the sum of two continuous functions f(x) and Ax, is
continuous on [a, fr].
(ii) As f(x) is differentiable on (a, b) and Ax, being a polynomial function, is everywhere
differentiable. So, <|>(x), being the sum of two differentiable functions f(x) and Ax, is
differentiable on (n, b).
Also, <\)(a) = §(b).
Thus, all the three conditions of Rolle's theorem are satisfied by <j)(.v) on [a, b]. So, there must exist
some c e (a, b) such that ())'(c) = 0.
Now, §(x) = f(x) + A x

w
=> V(x) = f'(x) + A
=> V(c) = f '(c) + A
m-m
<l,'(c) = 0

Flo
/'(c) + A = 0 ^ /'(c) = -A => f'(c) =
b -a
[Using (i)]

ree
Q.E.D.

F
GEOMETRICAL INTERPRETATION Let f(x) be a function defined on [a, b], and let APB be the curve
represented by y =f(x). Then, coordinates of A and B are (a, f(a)) and (b, f(b)) respectively. Suppose the
or
ur
chord AB makes an angle i|/ with the axis ofx. Then, from the triangle ARB, we have
sf
BR
tan \\i = ---- => tan \\i = m -/(*)
k

AR b -a
Yo
oo

Y
B
re

B(b,m)
ou
ad
Y

j y = f(x)
nd
Re
Fi

O M N X

Fig. 15.3

By Lagrange's Mean Value theorem, we obtain


m - f(a)
f'(c) = b -a
tan \\i = f'(c)
=> Slope of the chord AB = Slope of the tangent at (c, /(c))
Thus, we arrive at the following geometrical interpretation of Lagrange's mean value theorem:
Let f(x) be a function defined on [a, bl such that the curve y =f(x) is a continuous curve between points
A(a, f(a)) and B(b, f(b)) and at every point on the curve, except at the end points, it is possible to draw a
unique tangent. Then there exists a point on the curve such that the tangent thereat is parallel to the chord
joining the end points of the curve.

ReadYourFlow.COM
15.12 MATHEMATICS-XII

ILLUSTRATIVE EXAMPLES

LEVEL-1

Type I VERIFICATION OF LAGRANGE’S MEAN VALUE THEOREM


EXAMPLE l Verify Lagrange's mean value theorem for the function
f{x) =(x -3)(x - 6) (x - 9) on the interval [3, 5].
SOLUTION We have,
f(x)=(x-3) (x-6)(x-9)=x3 -18x2 + 99X-162
Since a polynomial function is everywhere continuous and differentiable.
Therefore,/(x) is continuous on [3, 5] and differentiable on (3, 5). Thus, both the conditions of
Lagrange's mean value theorem are satisfied. So, there must exist at least one real number
c e (3,5) such that
= /(5) - /(3)
f(c)
5-3

w
Now, f(x) = x3 - 18x2 + 99x -162

Flo
=> f(x) = 3*2 - 36x + 99,/(5)=(5 -3)(5-6)(5-9)=8 and/(3)=0
/(5) ~/(3)
f(x) =

ee
5-3

Fr
=> 3x2 - 36x + 99 = 8-0
5-3 for
ur
=> 3x2 - 36x + 99 = 4
ks

3^-36, + 95 = 0 , = 36 ± #9631140 = 36 ±12.49


=> = 8.8, 4.8
Yo

6 6
oo

/(5) ~/(3)
B

Thus, c = 4.8 e (3,5) such that f '(c) = ■. Hence, Lagrange's mean value theorem is
5-3
re

verified.
ou
ad

EXAMPLE 2 Verify Lagrange's mean value theorem for the following functions on the indicated
Y

intervals. Also, find a point c in the indicated interval:


(i) f(x) = x(x - 2) on [1, 3] (ii) f(x) = x(x -1) (x - 2) on [0,1/2]
nd
Re

SOLUTION (i) We have, f(x) = x(x -2) =x2 - 2x.


Fi

We know that a polynomial function is everywhere continuous and differentiable. So,/(x) being
a polynomial, is continuous on [1, 3] and differentiable on (1, 3). Thus, f(x) satisfies both the
conditions of Lagrange's mean value theorem on [1, 3].
So, there must exist at least one real number c e (1, 3) such that / '(c) = /(3) ~/(l)
3-1
Now, f(x) =x2 -2x
=> f '(x) = 2x - 2,/(3) = 9 - 6 = 3 and /(l)=l-2=-l.
= /(3) ~/(l)
/'W 3-1
3 -(-1)
2x - 2 = =>2x-2 = 2=>x = 2
3-1
/(3)-/(D
Thus, c =2 e (1, 3) such that/'(c) =
3-1
Hence, Lagrange's mean value theorem is verified for/(x) on [1, 3].

ReadYourFlow.COM
MEAN VALUE THEOREMS 15.13

(ii) We have, J\x) = x(x -1) (x - 2) = x3 - 3x2 + 2x.


Since/(x) is a polynomial function and a polynomial function is everywhere continuous and
differentiable. Therefore,/(x) is continuous on [0,1/2] and differentiable on (0,1/2). Thus, both
the conditions of Lagrange's mean value theorem are satisfied. So, there must exist at least one
real number c e (0,1/2) such that
/(l/2)-/(0)
f'(c) =
1/2-0
Now, /(x) = x3 - 3x2 + 2x

=> / '(x) = 3x2 - 6x + 2,/(0) = 0 and /[i 1


8
/(1/2) ~/(0)
/'W = (1/2)-0

low
(3/8)-0
=> 3x2 - 6x + 2 =
(1/2) -0
24 ± 7336 721
3x2 - 6x + 2 = - => 12x2 - 24x + 5 = 0 => x = = 1±
4 24 6

ee
72T
rF
Fr
1 /d/2) -/(0)
Since c = 1 - s| 0/ “I such that/'(c) =
6 (1/2)-0 for
Hence, Lagrange's mean value theorem is verified.
ou
EXAMPLE 3 Using Lagrange's mean value theorem, find a point on the curve y = ^x - 2 defined on the
ks

interval 12, 3], where the tangent is parallel to the chord joining the end points of the curve.
oo

SOLUTION Let f(x) = yjx - 2. Since for each x e [2, 3], the function/(x) attains a unique definite
Y
B

value. So,/(x) is continuous on [2,3].


re

Also, / ' (x) - , ^ — exists for all x e (2, 3). So, /(x) is differentiable on (2,3).
2/x - 2
ou
ad

Thus, both the conditions of Lagrange's mean value theorem are satisfied. Consequently, there
Y

must exist some c e (2, 3) such that


nd
Re

= /(3)~/(2)
/'(c)
Fi

3-2
1
Now, f(x) = fx-2 => /'(x) = /(3) =1 and/(2) = 0
2^2'

fix) = /(3)-/(2)
3-2
1 1-0 1 1 9
=> — — = 1 => 4(x-2) =1 => x - 2 = — => x =
2/x-2 3-2 2/x - 2 4 4

Thus, r = - <= (2, such that / '(c) = ---^


4 3-2

Clearly,/(c) Thus, (c, /(c)) i.e. (9/4,1/2) is a point on the curve y = Jx -2 such
2
that the tangent at it is parallel to the chord joining the end points of the curve.

ReadYourFlow.COM
15.14 MATHEMATICS-XII

EXAMPLE 4 Verify Lagrange's mean value theorem for the foUoiving functions on indicated
intervals.
(i) f(x) = x - 2 sin x or [- k, tt] (ii) f(x) = 2 sin x + sin 2.r on [0, tt]
2 + x3 if x<l
(iii) /(x) =log,, x on [1, 2] (iv) /(*) = on [-1, 2]
3x if x > 1
SOLUTION (i) Since x and sin x are everywhere continuous and differentiable, therefore/(x) is
continuous on [- n, n] and differentiable on (- n, n). Thus, both the conditions of Lagrange's
mean value theorem are satisfied. So, there must exist at least one c e (- re, rc) such that
/(t) -/(- k)
/'(<-•) =
71-(-7T)

Now, /(x) = x - 2 sin x


=> /' (x) = 1 - 2 cos x, /(ti) = re - 2 sin tt = 7t and /(- it) = -n-2 sin (- n) = - n
f(n) -f(-n)
/'(*) =

w
n-(-n)
ti-(-k)
=> 1-2 cos x = =>1-2 cos x -1 cos x = 0 => x = ± n/2

Flo
n-(-n)

c = ± (rc/2) e (- ti, tc) such that f’(c) _f(n) -f(-n)

ee
Thus,
n-(-n)

Fr
Hence Lagrange's mean value theorem is verified.
for
(ii) Since sin x an -.1 sin 2x are everywhere continuous and differentiable, therefore f(x) is
ur
continuous on [0, tt] and differentiable on (0, ti). Thus, /(x) satisfies both the conditions of
Lagrange's mean value theorem. Consequently, there exists at least one c e (0, tc) such that
ks
Yo

/(7i)-/(0)
f(c)
oo

7C - 0
B

Now,
re

/ (x) = 2 sin x + sin 2x


=> / '(x) = 2 cos x + 2 cos 2x, f(0) = 0 and /(tc) = 2 sin + sin 2tc = 0
ou

tc
ad

= /(*) -/(Q)
Y

/'(x)
TC - 0
d
Re

0-0
2 cos x + 2 cos 2x =
n

TC - 0
Fi

=> 2 cos x + 2 cos 2x = 0


cos x + cos 2x = 0
=> cos 2x = - cos x => cos 2x - cos (tc - x) => 2x = tc - x => 3x = tc => x = tc/ 3
f(n) -f(0)
Thus,c = — e (0, tc) such that f '(c) = . Hence, Lagrange's mean value theorem is
3 TC - 0

verified.
(iii) Since/(x) = loge x is differentiable and so continuous for all x > 0. So,/(x) is continuous on
[1, 2] and differentiable on (1, 2). Thus, both the conditions of Lagrange's mean value theorem
are satisfied. Consequently, there must exist some c e (1, 2) such that
/(2) -/(l)
f'(c) =
2-1
1
Now, /(x) = log,, x => /'(x) = - , /(2) = loge 2 and /(l) =log£, 1=0
x

ReadYourFlow.COM
MEAN VALUE THEOREMS 15.15

/(2) -/(l)
/'(*) =
2-1
1 Jog, 2-0 1 , o 1 1
=> - = logt- 2 => x = -------- = log2 ^ ••• log;, =
X 2-1 x logt, 2 log* b
Now, 2<e <4. => log2 2 < log2 e < log2 4 => 1 < logo e <2.
/(2) -/(l)
Thus,c =log2 e £(1, 2) such that/'(c) = . Hence, Lagrange's mean value theorem
2-1
is verified.
(iv) Since 2 + x3and3xare polynomial functions. Therefore, /(x) is continuous and
differentiable for all values of x except possibly at x = 1.
Continuity at x = 1: We have,
lim f(x) = lim (2 + x3) = 2 + 13 = 3 and, lim /(x) = lim 3x = 3 x 1 = 3.
A' —> 1 +

w
A" —> 1~ “A' —>" 1 A" -> 1

Also, /(l) = 2 + 13 = 3.

Flo
lim /(x) = lim /(x) = /(l).
A' —> 1_ A -> 1+

ee
So, /(x) is continuous at x = 1.

Fr
Differentiability at x = 1: We have,
(LHD at x = 1) = lim
m ~/(i) _ lim
2 + x3 - 3
for lim
x3-l
ur
a -> r X —1 A —> 1 x —1 x —> 1 X-1

(x-l)(x2 + X + 1)
ks

=> (LHD at x = 1) = lim = lim x2 + x + l =12+1+1 = 3


Yo

x —1 A —> 1
oo

/(x) -/(l) = 3x - 3 3(x -1)


and. (RHD at x = 1) = lim lim = lim = 3
eB

A -» l+ x —1 A —> 1 X - 1 1 (x — 1)
(LHD at x = 1) = (RHD at x = 1)
r
ou

So,/(x) is differentiable at x = 1.
ad

Thus,/(x) is continuous and differentiable on [-1,2]. So, both the conditions of Lagrange's mean
Y

value theorem are satisfied. Consequently, there must exist some c e (- 1, 2) such that
nd

f'(c) = /(2)-/(-l)
Re

2-(-l)
Fi

Now,
2 + x3 /
f(x) = •
3x / ^>1
3x2 / x<l
=> /'J = 3 , x > 1' /(-l) = 2 + (-l)3 = 1 and /(2) = 3(2) =6
/(2)-/(-l)
/'(-v) =
2-(-l)
6-1 5
=> f'(x) =
2-(-D 3
Since f '(x) = 3 for x > 1, the value of x must be less than 1.
/'(x) =5/3
=> 3x2 =5/3 [v x < 1 and for x < 1, /' (x) = 3x2]
=^> x2 = 5/9 ^ x = ± y/d/3

ReadYourFlow.COM
15.16 MATHEMATICS-XII

Since
V5
c=± -—e (-1,2) such that/'(c) =
/(2)-/(-l)
3 2-(-l)
Hence, Lagrange's mean value theorem is verified.
LEVEL-2

Type II ON PROVING INEQUALITIES BY USING LAGRANGE'S MEAN VALUE THEOREM


EXAMPLE 5 Using Lagrange's mean value theorem, show that sin x < x for x > 0.
SOLUTION Consider the function / (x) = x - sin x defined on the interval [0, x], where x > 0.
Clearly, / (x) is everywhere continuous and differentiable. So, it is continuous on [0, x] and
differentiable on (0, x). Consequently, there exists c e (0, x) such that
/(*)-/(0)
f(c) = [By Lagrange's mean value theorem]
x-0
x - sm x

w
=> 1 - cos c =
x
x - sm x
=> >0 [v 1 - cos c > 0]

Flo
X
=> x - sin x > 0 [v x > 0]

ee
=> x > sin x

Fr
=> sin x < x for all x.
EXAMPLE 6 Using mean value theorem, prove that tan x > x for all x (0, k/2).
for
ur
SOLUTION Let x be any point in the interval (0 , k/2). Consider the function / given by
/(x) = tan x-x, where x e[0, x] c (0 , tc/2)
ks

Clearly, / (x) is continuous on [0, x] and differentiable on (0, x). So, there exists c e (0, x) such
Yo
oo

that
/(*)-/(0)
B

f'(c) =
x-0
re

(tan x - x) - 0
sec2 c -1 =
ou

X
ad

tan x - ^>0 v sec2 c > 1 for all c e (0, x) c j^O,


Y

=>
x
nd

=> tan x - x > 0


Re

[y x > 0]
=> tan x > x for all x e (0, 2).
Fi

b -a b—
EXAMPLE 7 Using Lagrange's mean value theorem, prove that < logf-I
a
< a
—, where

0 <a <b.
SOLUTION Consider the function/ given by / (x) = log,, x , x e [a, b],0<a<b.
Clearly, it is continuous on [a, b] and differentiable on (a, b). So, by Lagrange's mean value
theorem there exist c e (a, b) such that

f(c) =
b -a
1 _ log b - log a 1
=> ••• / (*) = log X => /' (x) = -
c b -a x
Now, c e {a, b)
=> a <c <b
1 1 1
=> [v 0 <a <b]
b c a

ReadYourFlow.COM
MEAN VALUE THEOREMS 15.17

I < log b ~ log a <1


=>
b b -a a
b -a
=>
f<jos(9 <
a
[•.• b - a>0]

Type III MISCELLANEOUS APPLICATIONS OF LAGRANGE'S MEAN VALUE THEOREM


EXAMPLE 8 Let f and g be differentiable on [0,1] such thatf(O) = 2, g (0) = 0, / (1) = 6 and g (1) = 2.
Show that there exists c e (0,1) such that f' (c) = 2 g' (c).
SOLUTION Consider the function § given by
<!>(*) = \g(l) -gm f (X) -{f (l) -f m g(x) for all x e [0,1]
or. <J) (x) = 2 f (x) - 4 g (x) for all x e [0,1].
Since f (x) and g (x) are differentiable on [0,1]. Therefore, <\> (x) is differentiable on [0,1]. As (j) (x)

w
is differentiable on [0,1]. So, it is also continuous on [0,1]. Consequently, by Lagrange's mean
value theorem there exists c e (0,1) such that
Hv-m
f (c) =
1-0

Flo
ee
=> 2/'(c)-4g'(c) = [2/(1) - 4 g (1)] - [2 / (0) - 4 g (0)]

Fr
2 /' (c) - 4 g' (c) = 4-4
=> 2/'(c) = 4g'(c)
for
ur
=> f'(c) = 2 g' (c)
EXAMPLE 9 Letf be a twice differentiable function such that f(a) = f(b) - 0 and /(c) >0for a <c <b.
ks

Prove that there exists at least one value y between a and bfor which f"(y) < 0.
Yo
oo

SOLUTION Let us consider the function / on [a, b].


B

It is given that f is twice differentiable. Therefore,/' and/both exist and are continuous on [a, fr].
re

Applying Lagrange's Mean value Theorem to / on the intervals [a, c] and [c, b] respectively, we
get
ou
ad

f(c)-f(a) m-m =/' (P), where P s (c, b)


Y

= /' (a), where a e (a, c) and.


c-a b-c
m = /'(a) -m =/' (P)
nd
Re

=> and [v/(*)=/(f>) = 0]


c-a b -c
Fi

Clearly, a < a <p <b. It is given that /' (x) is continuous on [a, b] and [a, P] c [a, b]. Therefore,
f (x) is continuous on [a, p] and differentiable on (a, P). Applying Lagrange’s Mean Value
Theorem to/' (x) and [a, p].
/' (P) -/' (a) _ /"(y), where a < y < p
p-a
f(c) f(c) /(c) ^ (b-a)
=> => f"(y) = - <o
P~a b-c c-a P-a (b-c)(c-a)

EXERCISE 15.2
LEVEL-1

1. Verify Lagrange's mean value theorem for the following functions on the indicated
intervals. In each case find a point ’c' in the indicated interval as stated by the Lagrange's
mean value theorem:
(i) /(x) =x2 -1 on [2, 3] (ii) /(x) = x3 - 2x2 - x + 3 on [0,1]
(iii) /(x) = x(x -1) on [1, 2] (iv) /(x) = x2 - 3x + 2 on [-1, 2]

ReadYourFlow.COM
15.18 MATHEMATICS-XII

(v) f(x) = 2x2 - 3x + 1 on [1, 3] (vi) f(x) = x2 - 2x + 4 on [1,5]


(vii) f(x) = 2x-x2 on [0,1] (viii) f(x) =(x - l)(.v - 2)(x - 3) on [0, 4]
(ix) f(x) =-J25 - x2 on [- 3, 4] -1 x on [0,1]
M f(x) = tan
(xi) f(x) = x + i on [1, 3] [CBSE 2000] (xii) f(x) =x(x + 4)2 on [0, 4]
x
(xiii) /(x) = tJx2 -4 on [2, 4] [CBSE 2002] (xiv) /(x) = x2 + x -1 on [0, 4] [CBSE 2002]

(xv) /(x) = sin x - sin 2x - x on [0,71] (xvi) /(x) = x3 -5x2 - 3x on [1, 3] [NCERT]
2. Discuss the applicability of Lagrange's mean value theorem for the function /(x) = | x| on
[-1,1].
3. Show that the lagrange's mean value theorem is not applicable to the function
1
f(x) = — on [-1,1],
x

w
4. Verify the hypothesis and conclusion of Lagrange's mean value theorem for the function
1
/(*) =- , 1 < x < 4.

Flo
4x -1
r\

5. Find a point on the parabola \j = (x - 4) , where the tangent is parallel to the chord joining

ee
(4,0) and (5,1).

Fr
6. Find a point on the curve y = x + x, where the tangent is parallel to the chord joining (0, 0)
and (1, 2). or
ur
7. Find a point on the parabola y = (x - 3) , where the tangent is parallel to the chord joining
f
(3, 0) and (4,1).
ks

o
8. Find the points on the curve y = x - 3x, where the tangent to the curve is parallel to the
Yo
oo

chord joining (1, - 2) and (2, 2).


B

9. Find a point on the curve y = x + 1 where the tangent is parallel to the chord joining (1, 2)
re

and (3, 28).


LEVEL-2
ou
ad

^ ^ 7T
10. Let C be a curve defined parametrically as x = a cos' 0, y = (7 sin 0, 0 < 0 < - . Determine a
Y

point P on C, where the tangent to C is parallel to the chord joining the points {a, 0) and
d
Re

(0, a). [CBSE 2014]


n
Fi

11. Using Lagrange's mean value theorem, prove that


9 9 71
[b - a) sec a < tan b - tan a <{b - a) sec b, where 0 <n <b < — .
2
ANSWERS
1. (i) c =5/2 (ii) c =1/3 (iii) c = 3/2 (iv) c = 1/2 (v) c = 2
(vi) c = 3 (vii) c = 1/2 (viii) c = 2 ± (ix) c = ± -j= (x) c =
yf3 ,12
- 8 + 4 V13
(xi) c = y/S (xii) c - (xiii) c = Vb (xiv) c = 2
3
!fl ±/33
(xv) c = cos (xvi) c = —
8 3
2. Not applicable 5. (9/2,1/4) 6. (1/2, 3/4) 7. (7/2,1/4)
13 13^/2 a a
9. +1 11.
3A3 2V2'2V2

ReadYourFlow.COM
MEAN VALUE THEOREMS 15.19

HINTS TO NCERT & SELECTED PROBLEMS


1. (xvi) Clearly, / (x), being a polynomial function, is continuous on [1, 3] and differentiable
on(l, 3).
Now,
/ (x) = x3 -5x2 - 3x
f'(x) = 3x2 -IO.y - 3,/(l) = -7 and /(3) =27 -45-9 = -27
/ (3) -/ (1)
•• fix) =
3-1
-27+7 lO.v + 7 = 0 => (3.y -7) (x -1) = 0 x=\,7-
=> 3.v2 -10y - 3 = => 3y2
2
/ (3) -/ (1)
Clearly, c = — e (1, 3) such that /' (c) = . Hence, LMVT is verified.

low
3 3-1

________________________________________ VERY SHORT ANSWER QUESTIONS (VSA)


Answer each of the following questions in one word or one sentence or as per exact requirement of the

ee
question:
rF
1. If /' (x) = Ax2 + Bx + C is such that/ (a) = f (b), then write the value of c in Rolle's theorem.

Fr
2. State Rolle's theorem. for
3. State Lagrange's mean value theorem.
4. If the value of c prescribed in Rolle's theorem for the function / (x) = 2x (x - 3)" on the
ou
2
ks

interval [0, 2^3] is — , write the value of n (a positive integer).


4
oo

5. Find the value of c prescribed by Lagrange's mean value theorem for the function
Y
B

/ (x) = -^x2 - 4 defined on [2, 3].


re

ANSWERS
ou

a+b
ad

1. 4. 3 5. V5
2
Y

MULTIPLE CHOICE QUESTIONS (MCQs)


nd
Re

Mark the correct alternative in each of the following:


x" -1 + “n-l x"-2 +. .. + a2X2 + fl-| x +
Fi

1. If the polynomial equation aQ xn + =0

n being a positive integer, has two different real roots a and [3, then between a and (3, the
equation n anxn - 1 + (n -1) _ -j x" “ 2 + ... + ^ = 0 has
(a) exactly one root (b) almost one root (c) at least one root (d) no root
2. If 4a + 2b + c = 0, then the equation 3ax2 + 2bx + c = 0 has at least one real root lying in the
interval
(a) (0,1) (b) (1, 2) (c) (0, 2) (d) none of these
3. For the function / (x) = x + - , x e [1, 3] , the value of c for the Lagrange's mean value
x
theorem is
(a) 1 (b) yf3 (c) 2 (d) none of these
f'(b)-f(a)
4. If from Lagrange's mean value theorem, we have / ' (xj) = , then
b -a
(a) a < Xi < b (b) a < x1 < b (c) a < xq < b (d) a < x-j < b
5. Rolle's theorem is applicable in case of <|> (x) = asm A, a > 0 in

ReadYourFlow.COM
15.20 MATHEMATICS-XII

(a) any interval (b) the interval [0, tt]


(c) the interval (0,n/2) (d) none of these
6. The value of c in Rolle's theorem when / (x) = 2x3 ~5x2 - 4x + 3, is x e [1/3, 3]
(a) 2 (b) -1/3 (c) -2 (d) 2/3
7. When the tangent to the curve y = x log x is parallel to the chord joining the points (1,0) and
(e, e), the value of x is
2c - 1
1/l-e (b) e(e-l)(2e-l) e -1
(a) c (C) e'- 1 (d)
e
8. The value of c in Rolle's theorem for the function f (x) = - defined on [-1,0] is
e
1 +V5 .. 1 — Vs

w
(a) 05 (b) C 2 (d) -05
2
9. The value of c in Lagrange's mean value theorem for the function / (x) = x (x - 2) when
x e [1, 2] is
(a) 1 (b) 1/2
Flo
(c) 2/3 (d) 3/2

ee
10. The value of c in Rolle's theorem for the function / (x) = x3 - 3x in the interval [0, v^3] is

Fr
(a) 1 (b) -1 (c) 3/2 for (d) 1/3
ur
11. Iff (x) = ex sin x in [0,7t], then c in Rolle's theorem is
(a) ti/6 (b) k/4 (c) n/2 (d) 3rc/4
ks
Yo

ANSWERS
oo
B

1. (c) 2. (c) 3. (b) 4. (c) 5. (b) 6. (a) 7. (a) 8. (c)


re

9. (d) 10. (a) 11. (d)


SUMMARY
ou
ad

1. Rolle's Theorem: Let / be a real value of function defined on the closed interval [a, b] such
Y

that (i) it is continuous on [o, fc] (ii) it is differentiable on (a, fr) and, (in) f (a) = f (b).
nd

Then, there exists a real number c e (ci, b) such that/' (c) =0.
Re

Geometrical Interpretation: Let / (x) be a real valued function defined on [a, fr] such that the
Fi

curve y = / (x) is a continuous curve between points A (a, f (a)) and B (b, f (b)) and the curve
has a unique tangent at every point between A and B. Also, the ordinates at the end points
of the interval [a, b] are equal. Then there exists at least one point (c, / (c)) between A and B
on the curve where tangent is parallel to x-axis.
Algebraic Interpretation: Between any two roots of a polynomial/ (x), there is always a root
of its derivative.
2. Largange's Mean Value Theorem: Let / (x) be a function defined on [a, b] such that it is
continuous on [a, 1’] and differentiable on (a, b). Then, there exists c e (a, b) such that

f(c) = f(b)-f(a)
b -a
Geometrical Interpretation: Let f (x) be a function defined on [a, b] such that the curve
i/ =f (x) is a continuous curve between points A (a, f (<?)) and B (b, f (b)) and at every point
on the curve, except at the end-points, it is possible to draw a unique tangent. Then there
exists a point on the curve such that tangent at it is parallel to the chord joining the end
points of the curve.

ReadYourFlow.COM
CHAPTER 16
TANGENTS AND NORMALS

16.1 PRELIMINARIES
SLOPE (GRADIENT) OF A LINE The trigonometrical tangent of the angle that a line makes with the
positive direction of x-axis in anticlockwise sense is called the slope or gradient of the line.

low
The slope of a line is generally denoted by m.
Y

ee
rF
Fr
0 for
X' O X
ou
ks
oo
Y
B

Y'
re

Fig. 16.1
ou

Thus, m = tan 0, where 0 is the angle which a line makes with the positive direction of x-axis in
ad

anticlockwise sense.
Y

Since a line parallel to x-axis makes an angle of 0° with x-axis. Therefore, its slope is tan 0° = 0.
d
Re

A line perpendicular to x-axis or parallel to i/-axis makes an angle of 90° with x-axis, so its slope
n

is tan rc/2 = co.


Fi

Also, the slope of a line equally inclined with axes is + 1 or, -1 as it makes either 45° or 135°
angle with x-axis.
Slope of a line in terms of coordinates of any two points on it: Let P (x1, y{) and Q (x2, y2) be two
points on a line. Then its slope m is given by
m = ^l = Difference of ordinates
x2-xi Difference of abscissae
4 -(-1)
For example, the slope of a line passing through (2, -1) and (3, 4) is m =----------= 5 .
3 2
Slope of a line when its equation is given: The slope of a line whose equation is ax + by + c = 0 is
given by
a Coefficient of x
m =
b Coefficient of y
-3 = 3
For example, the slope of the line 3x - 2y + 5 = 0 is m =
-2 2

ReadYourFlow.COM
16.2 MATHEMATICS-XII

Angle between two lines: The angle 0 between two lines having slopes m, and m2 is given by
tan 8 = + f —~
^ 1 + ni] m2
Condition of parallelism: If the lines are parallel, then 0 = 0°.
m2 -m-]
tan 0 = tan 0 = 0 => = 0 => m2 = W}
1 +- W} m2
Thus, when two lines are parallel, their slopes are equal.
Condition of perpendicularity: If two lines of slopes m^ and m2 are perpendicular, then
m1 m2 =-l.
Thus, when two lines are perpendicular, the product of their slopes is - 1. If m is the slope of a
line, then the slope of a line perpendicular to it is -
m

w
Equation of a straight line: The equation of a straight line passing through a point (iq, yf) and
having slope m is i/- i/| = m{x-xf).

16.2 SLOPES OF TANGENT AND NORMAL

Flo
ee
Slope of the tangent: Let y - f(x) be a continuous curve, and let P (aq, i/!) be a point on it. Then,

Fr
dy
as discussed in section 10.1, is the slope of the tangent to the curve y =f(x) at point P
dx 'P for
ur
y
i.e.. = tan \jy = Slope of the tangent at P, where v|/ is
dx JP
ks

9,.
the angle which the tangent at P(x^, y^) makes with the 9>
Yo

y=rn
oo

positive direction of x-axis.


B

If the tangent at P is parallel to x-axis, then


re

v|/ = 0 => tan v|/ = 0 => Slope = 0 => fy' =0


dx Jp P(xv yi)
ou
ad

If the tangent at P is perpendicular to x-axis, or parallel to O


Y

y-axis then \ X
' dx
nd

1
Re

\)/ = ^ => cot vy = 0 => = 0 => =0 Fig. 16.2


tan y dy yp
Fi

Slope of the normal: The normal to a curve at P (x-j, y-j) is a line perpendicular to the tangent at P
and passing through P.
1 1 dx
Slope of the normal at P = -
Slope of the tangent at P dy Jp

ILLUSTRATIVE EXAMPLES

LEVEL-1

Type I ON FINDING THE SLOPES OF THE TANGENT AND THE NORMAL AT A GIVEN POINT
EXAMPLE 1 Find the slopes of the tangent and the normal to the curve x2 + 3y + t/2 = 5 at (1,1).
SOLUTION The equation of the curve is x2 + 3y + y2 =5.
Differentiating with respect to x, we get
2x+ 3 — + 2y — = 0
dx dx

ReadYourFlow.COM
TANGENTS AND NORMALS 16.3

dy = 2x
dx 2y + 3
dy_ 2 2
dx A i, i) 2+3 5

dy 2
Slope of the tangent at (1,1) =
dx A i, i) 5

-1 -15
and. Slope of the normal at (1,1) =
-2 2
v dx (1,1) 5
o
EXAMPLE 2 S/70W that the tangents to the curve y = 2x - 3 at the points where x = 2 and x =-2 are
parallel. [CBSE1992C]

low
SOLUTION The equation of the curve is y = 2x - 3. -(i)
Differentiating with respect to x, we get
^ = 6x2
dx

ee
dy"
rF = 6x(2)2 =24

Fr
Now, = (Slope of the tangent at x = 2) = -j-
v dx Jx = 2

and. m2 = (Slope of the tangent at x = - 2) = — | = 6 (- 2)2 = 24.


for
Vdx)x = _2
ou
ks

Clearly, m1 = m2.
oo

Thus, the tangents to the given curve at the points where x = 2 and x = - 2 are parallel.
Y
B

EXAMPLE 3 Prove that the tangents to the curve y = x -5x + 6 at the points (2, 0) and (3, 0) are at
re

right angles. (CBSE1985,921


SOLUTION The equation of the curve is i/ = x2 - 5x + 6.
ou
ad

Differentiating with respect to x, we get


Y

dy = 2x -5
nd

dx
Re

Now, m-j = Slope of the tangent at (2, 0) Jdi = 2x 2-5 = -l


Fi

2(2,0)

and. m2 = Slope of the tangent at (3, 0) = — ] = 2 x 3-5=1


V J.(3,0)

Clearly, m17772 = -Ixl = -1-


Thus, the tangents to the given curve at (2, 0) and (3, 0) are at right angles.
EXAMPLE 4 The slope of the curve 2y2 = ax2 + bat (1,-1) is - 1. Find a, b.
SOLUTION The equation of the curve is
2y2 = ax2 + b ...(i)
Differentiating with respect to x, we get
4y = 2ax
dx
dy _ ax (dy ' -a
=> dx 2y ^
dx )(!,-■[) 2

ReadYourFlow.COM
16.4 MATHEMATICS-XII

It is given that the slope of the tangent at (1, - 1) is - 1. Therefore,


-- = -1 => a = 2
2
Since the point (1,-1) lies on (i). Therefore,
2(-l)2 =a(l)2 +b=> a + b =2
Putting a = 2ina + b = 2,we obtain b =0.
Hence, a = 2 and b =0.

EXAMPLE 5 Find the slope of the normal to the curve x = 1-asm Q ,y = b cos 0 at 0 = — .

[NCERT]
solution We have,
x = 1 - a sin 0 and y = b cos 0
dx = -2b cos 0 sin 0
=>

w
= -a cos 0 and
dQ dQ
dy
dy = d Q
dx
-2b sin 0 cos 0 2b

Flo
= — sin 0

ee
dx -a cos Q a
dQ

Fr
(dy 2b . k 2b
=> = — sin — = — or
ur
Kdx 0 = 7l/ 2 a 2 a
sf
Hence, ^ Slope of the normal at 0 = -^ j = 1 a
2b
k
Yo
oo

0=71/2
B

EXAMPLE 6 Find the slope of the normal to the curve x - a cos Q , y = a sin Qat Q = — .
re

SOLUTION We have, [NCERT]


ou
ad

x = a cos 0 , y = a sin' 0
Y

= - 3 a cos2 0 sin 0 , = 3 a sin2 0 cos 0


dQ ' dQ
nd
Re

dy _ dy/d 0
Fi

Now,
dx dx/d0
2
dy = 3a sin 0 cos 0
=> = - tan 0
dx - 3a cos2 0 sin 0
1 -1
Slope of the normal at any point on the curve = - = cot 0
dy - tan 0
dx
Hence, (Slope of the normal at 0 = tc/4) = cot - = 1.
4
Type II ON FINDING THE POINT(S) ON A GIVEN CURVE AT WHICH TANGENT(S) IS (ARE) PARALLEL OR
PERPENDICULAR TO A GIVEN LINE
o o
EXAMPLE 7 Find the points on the curve y-x - 2x - x at which the tangent lines are parallel to the
line y = 3x - 2.
SOLUTION Let P (x-j, yf) be the required point. The given curve is
y = x 3 -2x2 -x (i)

ReadYourFlow.COM
TANGENTS AND NORMALS 16.5

^ = 3*2 -Ax-1
dx
=> = 3x12-4x1-1
dx\x-i,n)
Since the tangent at (x^, y{) is parallel to the line y - 3x - 2.
Slope of the tangent at (x^, y{) = Slope of the line y = 3x - 2
dy = 3
=>
dx'(xi,yi)

=> 3x2 -4x^-1 =3 3x2 -4x-i -4 = 0 => (^ - 2) (3^ + 2) = 0 x-[ =2,- —


3
3 2
Since (X}, i/}) lies on curve (i). Therefore, yi = x^ -2x1 -Xi.

= 2 => yi = 23-2(2)2-2 = -2.

w
Now, *1
2 >.3 2f 2 = -14
and. -2

Flo
3 3 3 27
(-2 -14^
Thus, required points are (2, - 2) and I j

ee
Fr
j
EXAMPLE 8 Find the point on the curve y = 2x - 6x -4 at which the tangent is parallel to the x-axis.
SOLUTION Let the required point be P (xj, y{). The given curve is
for
ur
y = 2x2 - 6x - 4 -(i)
dy)
ks

=> dy = 4x - 6 => = 4x-l - 6


Yo

dx dxkxl,y1)
oo

Since the tangent at (xq, y{) is parallel to x-axis. Therefore,


B
re

= 0 => 4xa - 6 = 0 ^ x-i = -


dxAx-[,yi) 1 2
ou
ad

Since (xj, i/j) lies on curve (i). Therefore,


Y

y-t = 2x12-6x1-4
nd
Re

*1 = 3/2 => yi = 2(3/2)2 -6(3/2) -4 = -y


Fi

So, the required point is (3/2, -17/2).


EXAMPLE 9 At what points on the curve x +y -2x-4i/ + l =0, the tangents are parallel to the
y-axis? [NCERT EXEMPLAR]
SOLUTION Let P (Xi, y{) be the required point. As P(x1,y1) lies on the curve
x2+y2-2x-4y + l = 0.
xi2+yi2-2xi-4yi+1=° -(i)
If the tangent to the given curve at P is parallel to i/-axis, then
(dx)
=0
dy
The equation of the curve is
x2+ y2-2x-4y+ 1 =0
Differentiating both sides with respect to y, we obtain
2x — + 2y-2—-4 = 0
dy dy

ReadYourFlow.COM
16.6 MATHEMATICS-XII

2^(x-l) = 2(2-y)=> ^ = ' dx' _2~yi


=>
dy dy x-\ [dy Jp Xl-l
f dx
But, — =0. Therefore,
Uy)P
2-yi
= 0=>2-y1=0=>
ATi-1
Putting y| = 2 in (i), we obtain
x-j2+ 4-2*2-8 + 1 = 0 =?• *22 — — 3 = 0 =^> (*2-3) (*2 +1) => *2 =“1/ 3
Hence, the coordinates of required points (-1, 2) and (3, 2).
example io Find the points on the curve y = x3 flf which the slope of the tangent is equal to the
y-coordinate of the point. [NCERT, CBSE 2010]
SOLUTION Let the required point on the curve y = x3 be P (*2 , y2).

low
We have.
^ ^ ^ = 3x2 => ^ O
= Sxj 2
y = dx

ee
It is given that:
rF
Fr
Slope of the tangent at P (*2 , y2) = Ordinate of P (*2 , y2)
^ = yi for
dxAx\,yi)
u
=> = V\
ks

3 [••• (^i /]/i) Meson y = x3 3


Yo

=> yi =
oo

i *1
B

=> xl2 (*1 ~ 3) = 0 => *2 = 0, *2 = 3


re

3 3
Since (*2, y2) Mes on y = x . Therefore, y2 = x1
ou
ad

*2=0 => y2 = 0 and, *2 = 3 => y2 = 33 = 27


Y

Hence, required points are (0, 0) and (3, 27).


x2 y2
nd
Re

EXAMPLE 11 Find points on the curve — - ^ = 1 flt which the tangents are parallel to the (i) x-axis
Fi

(ii) y-axis.
2 2
SOLUTION Let P (*2 , y2) be a point on the curve = 1- Then,
2 2
xi yi = i
9 16
The equation of the curve is
2 2
* = i
9 16
Differentiating both sides with respect to x, we get
2x 2y dy _ dy _ 16s _ fdy} 16 *2
9 16 dx dx 9y [dx), 9yi
(i) If the tangent at P(xi, y^) is parallel to x-axis, then
'dy) 16 *2
= 0 => 0 => 16*2 = 0 => *2 = 0
<dxj 9yi

ReadYourFlow.COM
TANGENTS AND NORMALS 16.7
2
Putting = 0 in (i), we get y1 =-16, which is impossible as y1 is real. Hence, there is no point
on the curve where tangent is parallel to x-axis.
(ii) If the tangent at P (x-j, y{) is parallel to y-axis, then
' dx
= 0 =;> 91/1 = 0 => yi = 0.
Uy 16 Xj

Putting yx = 0 in (i), we get x1 = ± 3 .


Hence, required points are (3, 0) and (- 3,0).
EXAMPLE 12 Find a point on the curve y = (x - 3) , where the tangent is parallel to the line joining
(4,1) and(3, 0).
SOLUTION Let the required point be P(xi, y{). The equation of the given curve is
y = (x-3)2

low
dy =
=> 2(x - 3)
dx
=> = 2(x1-3)
Kdx (H-yi)

ee
rF
Fr
Since the tangent at P is parallel to the line joining (4,1) and (3, 0). Therefore,
Slope of the tangent at P = Slope of the line joining (4,1) and (3, 0) for
dy 0-1
=>
ou
3-4
ks

=> 2 (X! - 3) = 1 => Xj = 7/2


oo

Since the point P{xi, y{) lies on (i). Therefore,


Y
B

yi = (*i - 3)2
re

2
7 1
ou

*1 = 2 ^ yi = U
ad

4
Y

Thus, the required point is (7/2,1 /4 ).


EXAMPLE 13 Find the coordinates of the point on the curve fx+-fy=4at which tangent is equally
nd
Re

inclined to the axes. [NCERT EXEMPLAR]


Fi

SOLUTION Let the required point be P (x-j, y-j). The tangent to the curve Vx + fy = 4 at P is equally
inclined with the coordinate axes. Therefore, slope of the tangent to the curve at P is ± 1.
dy = ±1
i.e.
dxjp
The equation of the curve is Vx + ^/y = 4.
Differentiating with respect to x, we obtain
i 1 dy Vyi
2-Jx ijy dx dx Vx V. dx Jp V*i
dy
But, ^ = ± 1. Therefore,
dx Jp

&=±i => =±Vy^ ...a)


Vh
Clearly, P (xq, y-^) lies on the curve Vx + ^y = 4.

ReadYourFlow.COM
16.8 MATHEMATICS-XII

V^i+Vyi=4 ...(ii)
Now two cases arise:
CASE I When :
Putting in (ii), we obtain
2^/x7 = 4 => ^ = 2 => x-j = 4
So, the coordinates of P are (4, 4).
CASE II When Jx^ = - ^fy^ :
Putting ^ = --y/yi in (ii), we obtain
- Jyi + yfyl = 4 or 0 = 4, which is absurd.
So, ^ = ~ 7^1 is not possible.
Hence, the coordinates of the required point are (4,4).

low
EXAMPLE 14 Find the points on the curve 4x + 9y = 1, where the tangents are perpendicular to the
line 2y + x = 0.
SOLUTION Let the required point be P(xl7 y-j). The equation of the given curve is
4x2 + 9y2 =1

ee
=> 8x + ISy — = 0
rF [Differentiating with respect to x]

Fr
dx
dy _ 4x
=> for
dx 9y
ou
=> dy) - -4*i
ks

9yi
oo

Since tangent at (xj, y-^) is perpendicular to the line 2y + x = 0. Therefore,


Y
B

Slope of the tangent at (x^, y-^) x Slope of the line = - 1


re

i Coefficient of x
X-- = -1 Slope of a line = -
dxAxi,yi) 2 Coefficient of y
ou
ad

-4X! 1
Y

=> ----- - x---- = -1


9yi 2
nd
Re

-2xa
=> yi = ••(ii)
9
Fi

Since P(x1, y{) lies on the curve (i). Therefore,


4^12 + 9y12 = 1

=> 4x12 + 9 (-2xlf = 1 [Using (ii)]


9
4xf
=>
1
O
4x/ + —
9
‘iX.
= 1 => x2
1 =
9
40 ^1 = ±-4=
2 VTo
3 -2 3 1
Now, [Using (ii)]
" 2VlO ^ yi 9^2 VlO 3 Vio
3 -2 3 1
and. [Using (ii)]
11 2Vio =* yi ~ 9 2V10 3 VTo
3 -1 -3 1
Hence, the required points are —7 and
2 VTo' 3 VlO 2VTo' 3Vio

ReadYourFlow.COM
TANGENTS AND NORMALS 16.9
O
EXAMPLE 15 Find the point on the curve i/=x -ll.v + 5 at which the tangent has the equation
y=x-u. [CBSE 2012, NCERT]
SOLUTION Let the required point be P(xlr Since (xj, y{) lies on y = x -11 x + 5.
3 -llxj+5 -(i)
Now, 3 -llx + 5

=> -11
dx

=> dy = 3x^-1!
dx

Since the line y = x -11 is tangent at the point (x1, y{). Therefore,
Slope of the tangent at (xq, yj) = (Slope of the line y = x -11).

low
=> dy = (Slopeof thelinex-y-11 = 0)
dx

Coefficient of x
=> 3x,2 -11 = — v Slope = -
l

ee
1 Coefficient of y
rF
Fr
=> 3x-j2 = 12 => Xi = ± 2

Now, X1 = 2 ^ y, = 23 22 + 5 = -9 [Using (i)]


for
u
X1 = -2 => y = (- 2)3 -11 (-2) +5 =19 [Using (i)]
ks

So, two points are (2, - 9) and (- 2,19). Of these two points, (-2,19) does not lie on y = x -11.
Yo
oo

Therefore, the required point is (2, - 9).


B

EXAMPLE 16 Find the points on the curve 9y2 = x3 where normal to the curve makes equal intercepts
re

with the axes.


SOLUTION Let the required point be (xq, y-j). The equation of the curve is 9y = x .
ou
ad

Since (xq, y{) lies on the curve. Therefore,


Y

n 2 3
9yi =
nd
Re

2
x2 _*1
Fi

Now, 9y2 = x3 => — =


dx 6y I dx (x\>y\) 6yi
Since the normal to the curve at (xq, y{) make equal intercepts with the coordinate axes.
Slope of the normal = ± 1
1
= ±1
dy
dx'(*l,yi)
'dy
=> = ±1
v dx
(^U.'/l)
2
X1 = ±1
=>
6./i
=> xq2 = ± 6 yx
=> xl = 36 y i

ReadYourFlow.COM
16.10 MATHEMATICS-XII

3^
=> 4 = 36 i [Using (i)]
9

=> - 4x^ => (xi - 4) = 0 => Xj = 0, 4


Putting Xj = 0 in (i), we get
9Vl2 = 0 => yi = 0
Putting x^ = 4 in (i), we get

V 43 => i/i = ± —
3
But, the line making equal intercepts with the coordinate axes cannot pass through the origin.
Hence, the required points are (4, 8/ 3) and (4, - 8/3).
EXERCISE 16.1

low
LEVEL-1
1. Find the slopes of the tangent and the normal to the following curves at the indicated
points:
(i) y = V? at x = 4

ee
(ii) y = Vx at x = 9
rF
Fr
(iii) y = x3 - x at x = 2 [NCERT] (iv) y = 2x2 + 3 sin x at x = 0
(v) x = ci (0 — sin 0), y = 0 (1 + cos 0)at 0 = - n/2 for
(vi) x = a cos 3 0, y = fl sin 3 0 at 0 = tc/4
u
(vii) x = fl (0 - sin 0), y =a(l - cos 0) at 0 = tc/2 (viii) y = (sin 2x + cot x + 2)2 atx = n/2
ks
Yo

(ix) x2 + 3y + y2 =5 at (1,1) (x) xy = 6 at(l, 6)


oo

2. Find the values of a and b if the slope of the tangent to the curve xy + ax + by = 2 at (1,1) is 2.
B

3. If the tangent to the curve y = x3 + ax + b at (1, - 6) is parallel to the line x - y + 5 = 0, find


re

flandb. [CBSE 2005]


ou
ad

4. Find a point on the curve y = x3 -3x where the tangent is parallel to the chord joining
Y

(1,-2) and (2, 2).


5. Find the points on the curve y = x3 -2x2 -2x at which the tangent lines are parallel to the
nd
Re

line y -lx- 3.
Fi

6. Find the points on the curve y = 2x' at which the slope of the tangent is 3.
7. Find the points on the curve xy + 4 = 0 at which the tangents are inclined at an angle of 45°
with the x-axis.
8. Find the point on the curve y = x where the slope of the tangent is equal to the x-coordinate
of the point.
9. At what points on the circle x2 + y2 -2x -4y +1 = 0, the tangent is parallel to x-axis.
[CBSE 2002C]
10. At what point of the curve y = x2 does the tangent make an angle of 45° with the x-axis ?
11. Find the points on the curve y = 3x2 - 9x + 8 at which the tangents are equally inclined with
the axes.
12. At what points on the curve y = 2x2 - x +1 is the tangent parallel to the line y = 3x + 4 ?
13. Find the point on the curve y = 3x2 + 4 at which the tangent is perpendicular to the line
whose slope is - \

ReadYourFlow.COM
TANGENTS AND NORMALS 16.11
r\ r\

14. Find the points on the curve x + y -13, the tangent at each one of which is parallel to the
line 2x + 3t/ =7.
15. Find the points on the curve la1]! = x3 - 3ax1 where the tangent is parallel to x-axis.
16. At what points on the curve y = x - 4x + 5 is the tangent perpendicular to the line
2y + x = 7? 2 2
17. Find the points on the curve — = 1 at which the tangents are parallel to the
(i) x-axis (ii) y-axis. ^ [NCERT]
18. Find the points on the curve x + y - 2x - 3 = 0 at which the tangents are parallel to the
(i) x-axis. (ii) y-axis [NCERT, CBSE 2011]
19. Find the points on the curve — + — = 1 at which the tangents are (i) parallel to x-axis
(ii) parallel to y-axis. ^ ^ [NCERT]
20. Show that the tangents to the curve y = 7x 3 +11 at the points x = 2 and x = - 2 are parallel.

w
[NCERT]
21. Find the points on the curve y = x3 where the slope of the tangent is equal to x-coordinate

Flo
of the point. [CBSE 2008]
__ ANSWERS

ree
1. Slope of the Slope of the Slope of the Slope of the

F
tangent normal tangent normal
(i) 3 -1/3 (vi) or -1 1
ur
(ii) 1/6 -6 (vii) 1 -1
sf
(iii) 11 -1/11 (vhi) -12 1/12
(iv) 3 -1/3 (ix) -2/5 5/2
k
Yo
oo

(v) 1 -1 (x) -6 1/6


B

2. a = 5, = - 4 3. a = -2,b = -5 4. ±.1?+If-
3 3 V3
re

5. (2,-4); (-2/3,4/27) 6. (2,4) 7. (2, - 2) and (- 2, 2)


ou
ad

8- (0, 0) 9. (1,0), (1,4) 10. (1/2,1/4)


Y

11. (5/3,4/3) and (4/3,4/3) 12. (1,2)


13. (1,7) 14. (2,3); (-2,-3) 15. (0,0),(2*,-2fl)
nd
Re

16. (3, 2) 17. (i) (0,5), (0,-5) (ii) (2, 0), (- 2, 0)


Fi

18. (i) (1, ± 2) (ii) (-1,0), (3, 0)


19. (i) (0,4), (0,-4) (ii) (3, 0), (- 3, 0) 21. (0, 0), (1/3,1/27)
HINTS TO NCERT & SELECTED PROBLEMS
1. (iii) The equation of the curve is y = x - x.
.-. ^ = 3x2 -1
dx
Atx = 2, we get: ^ = 3x 22 -1 =11
dx
Hence, slope of the tangent at x = 2 is 11 and that of the normal is -

x2 y2
17. Let P (x-[, y{) be a point on the curve + ^ = 1- Then,
2 2
*1 +^ = 1 ...(i)
4 25

ReadYourFlow.COM
16.12 MATHEMATICS-XII

2 2
Now, ^_+r = i^ ^ = o ^ ^ =_^^i
4 25 2 25 dx dx 4 y U^Jp 4 yj
(i) If tangent at P is parallel to x-axis, then
dy = o => _“ 25 £l 0 => Xj = 0
4 yi =
dx Jp
Putting X} = 0 in (i), we get
2
h = 1 => yj = ± 5
25
Hence, required points are (0, 5) and (0, -5).
(ii) If the tangent at P is parallel to y-axis, then
= o=>-Ayi = 0 => y-L = 0

w
25 x1

Putting y-! = 0 in (i), we get


2

Flo
ee
*1 = 1 => X-i = ±2
4 1

Fr
Hence, required points are (± 2, 0).
18. Let P (x1, y-j) be a point on the curve x2 +y2 -2x - 3 = 0. Then,
for
ur
xa2 +y12 -2XJ -3 = 0
ks
Yo

We have, x2 + ij2 -2x-3 = 0


oo

Differentiating with respect to x, we get


B

2x + 2y -- - 2 = 0 ^ = 1 -x ^ [^1 =
re

dx dx y U-vJp yi
ou
ad

(i) If the tangent at P is parallel to x-axis, then


Y

dy = 0 => = 0 => X! = 1
dx Jp y\
nd
Re

Putting X] = 1 in (i), we get


Fi

l + yp-2-3 => yi=±2


Hence, required points are (1, ± 2).
(ii) If the tangent at P is parallel to y-axis, then
i - = 0=>-^-
= 0 => i/j = 0
dy 1-Xi
dx Jp
Putting y^ = 0 in (i), we get
Xj2 -2x1 - 3 = 0 => (x1 - 3) (xj +1) = 0 Xi = -1, 3

Hence, required points are (-1, 0) and (3, 0).


19. Proceed as in the solution of Q. No. 17.
20. The equation of the curve is y =7x + 11.

Differentiating with respect to x, we get — = 21 x2.


dx

ReadYourFlow.COM
TANGENTS AND NORMALS 16.13

Let and m2 be the slopes of tangents toi/=7x+llat x = - 2 and x = 2 respectively.


Then,

mi=®L-2=2l(-2)2=84 m2 x=2
= 21 (2)2 = 84

Clearly, = m2. Hence tangents at x = - 2 and x = 2 to the given curve are parallel.

16.3 EQUATIONS OF TANGENT AND NORMAL


We know that the equation of a line passing through a point (x1, iq) and having slope m is
y-y\ = m(x—xx)
As discussed in article 16.2 that the slopes of the tangent and the normal to the curve i/ = /(x) at a
point P(xl, y-]) are and - respectively. Therefore, the equation of the tangent at

w
ydxjp dy
dx Jp

Flo
P(x\r yi)to toe curve y = /(x) is
(dy)

ee
y-yi (x-x^
ydx 'P

Fr
Since the normal at P^, y{) passes through P and has slope - . Therefore, the equation of
dy
the normal at P(x1, y^) to the curve y =f(x) is
for
ur
dx Jp
y-yi = - ...(h)
ks

SI
Yo
oo
B

REMARK 1 // -2- = oo, then the tangent at (xq, is parallel to y-axis and its equation is x = xq.
re

\dxjp
ou

REMARK 2 /^ ( — | = 0, then the normal at (x-j, y^ is parallel to y-axis and its equation is x = xq.
ad
Y

In order to find the equations of tangent and normal to a given curve at a given point, we may
nd
Re

use the following algorithm.


Fi

ALGORITHM

STEP I Find — from the given equation y = /(x).


dx
dy at the given point P(xq, y{).
STEP II Find the value of-^-
dx
STEP III If is a non-zero finite number, then obtain the equations of tangent and
^Mxi,yi)
normal at (xq, yf) by using the formulae y-y\ = (“fM (x - x2) and
\ / (H'-'/l)
1
y-y\=- (dy (x - xq) respectively. Otherwise go to step IV.

dXJ(xVlA)

ReadYourFlow.COM
16.14 MATHEMATICS-XII

STEF IV //^)
= 0, then the equations of the tangent and normal at (xl, i/f) are y -1/^=0 and

x - Xj =0 respectively. If \ — = ± co, then the equations of the tangent and normal at


ydx)
(jq, y{) are x -xl=0 and y -yi=0 respectively.

ILLUSTRATIVE EXAMPLES
LEVEL-1
Type I ON FINDING THE EQUATIONS OF TANGENT AND NORMAL TO A CURVE AT A POINT
EXAMPLE i Find the equation of the tangent to the curve y = -5x2 + 6x + 7 at the point
(1/2, 35/4).
SOLUTION The equation of the given curve is

w
y = -5x2 + 6x + 7

=> dy = -10x + 6
dx
( dy 10

Flo
ee
=> -— + 6 = 1
v dx /(i/2, 35/4) 2

Fr
The required equation of the tangent at (1/2, 35/4) is
35 dy 1
for
ur
X-----
y-T dx (1/2, 35/4) 2
ks

35 1 33
=> y-T = X+—
Yo
oo

4
B

EXAMPLE 2 Find the equations of the tangent and normal to the parabola y2 =4ax at the point
re

(at2, lat). [NCERT, CBSE 2013]


SOLUTION The equation of the given curve is
ou
ad

y2 = 4ax ...(i)
Y

Differentiating (i) with respect to x, we get


d
Re

2y— = 4a=> — = — => f—1 2a_ = 1


n

dx dx y {dx^(at2,lat) lat t
Fi

So, the equation of the tangent at (at2, lat) is

y ~ 2at = dy (x-at2)
dx\at2,lat)
=> y-2at = — (x — at2) => ty = x + at2
t
The equation of the normal at (at2, lat) is
1 (x-at2)
y -lat = -

\dx )(at2'2at)
y - lat =-}^(x - at2)

t
y - lat = —t(x — at2) => y + tx = lat + at3

ReadYourFlow.COM
TANGENTS AND NORMALS 16.15

EXAMPLE 3 Find the equation of the normal to the curve y = 2x + 3 sin x at x = 0.


SOLUTION The equation of the given curve is
y = 2x2 + 3 sin x -(i)
Putting x = 0 in (i), we get y = 0.
So, the point of contact is (0, 0).
Now, y = 2x2 + 3 sin x

=> ^ 4x + 3 cos x [Differentiating with respect to x]


dx
=> = 4x0+3 cos 0 = 3
^dx\o,0)
So, the equation of the normal at (0, 0) is

low
y - 0 = - — (x - 0) or, x + 3y - 0
3
EXAMPLE 4 Find the equations of the tangent and the normal to 16x + 9y =144 at (xq, tq) where
x1=2 and y^ > 0.

ee
SOLUTION The equation of the given curve is
16x2 + 9y =144
rF
Fr
Since (X}, yf) lies on (i). Therefore,
99 o') 2 80 4 Vs
16x12+9y12 = 144^ 16(2)2 + 9y12=144=>y12= —=>y1= —
for
[••• y\ > o]
ou
4y/5)
ks

So, coordinates of the given point are 2,------.


oo
Y

Now, 16x2 + 9y2 = 144


B
re

=> 32x + 18y — = 0 [Differentiating with respect to x]


y dx
ou

dl = -16 x
ad

=>
dx 9y
Y

'dy), >=_ 16 x 2 _______


8
nd

<dx) ~
Re

n 4V5 ~ 3/5
9 x------
{ 3
Fi

3
4-S) .
The equation of the tangent at 2, ------- is
3
4V5 dy 4 V5 8
(x - 2) or, 8x + 3 V5y - 36 = 0
dx 2,f](^2)or'y“ 3 V5

4 V5 V
The equation of the normal at 2, —
------- is

4 VS 1
= (x - 2)
dy
ydx 4 fs'
' 3
4 VS 1 , 4 VS 3 VS (x - 2) or, 9 VS x - 24y + 14 VS =0.
or. y—3 -g-(x-2) or, y- — =
8
3 VS

ReadYourFlow.COM
16.16 MATHEMATICS-XII

\ 2 t/2
EXAMPLE 5 Find the equations of tangent and normal to the ellipse — 7T + ^r = 1 at (xl, y^.
b2
SOLUTION We have, a
2 2
= 1
a2 b2
Since P (xj , \j{) lies on the curve (i). Therefore,
2 2
+ h. = i ...(h)
«2 b2
Differentiating (i) with respect to x, we get

a2 b2 dx dx a2 y v dx
(A'Dyi)
The equation of the tangent at P (x-j, y-f is

low
dy
y-y\ = (x - xq)
dxj(n>y\)

=> y-yi (x - xj

ee
rF
Fr
2 2
yy\-y-j xx-j — x'1
=?>
b2 a2 for
2 2
ou
=>
XX 2
+ m -h. + h
ks

«2 b2 a2 b2
oo

=> +m = i [Using (ii)]


Y

«2 b2
B
re

The equation of the normal at P(x1, y^) is


1
y-yi (x — Xi)
ou
ad

dy
Y

dx
\x\>y\)
nd
Re

=> y-yi = (X - X|)


Fi

b1 (y-yi) a2 (t ~ *i)
=>
y\ x\
a x -n2
*/l *1

=> = a2-b2
xi yi
EXAMPLE 6 Find the equation of the tangent line to the curve x = 1 - cos 0, y = 6 - sin 0 at 0 = rr/4.
[CBSE 2004]
SOLUTION Putting 0 = ^ in x =1-cos0 and y = 0-sin 0, we get

- . K . K K 1
x = 1 - cos — = 1 —7= and y = — sin - = -
4 V2 J 4 4 4 vT

ReadYourFlow.COM
TANGENTS AND NORMALS 16.17

So, coordinates of the point of contact are V2'4 -v/2 J'


Now, x = 1 - cos 0 and y = 0 - sin 0

=> — = sin 0 and — = 1 - cos 0


dG dQ
dy _ dy/dQ _ 1 - cos 0
dx dx/dQ sin 0
At 0 = 7t/4, weget
1
dy _ 1 - cos k/ 4 ^ = V2-1
dx sin 7r/4 1
V2

low
So, the equation of the tangent line at 0 = ^ is

= (V2-l){x-[l--L]J or, (-Jl-Dx-y = 2(V2-l)-x/4

ee
EXAMPLE? Find the equations of the tangent and the normal at the point 't' on the curve
rF
Fr
x = a sin' t,y =b cos t. [NCERT, CBSE 2010,2014]
SOLUTION We have, for
x=asm3t and, y =b cos3 t
ou
ks

=> — = 3a sin2 t cos t and, — = - 3b cos2 t sin t


dt dt
oo

dy _ dy/dt _ - 3b cos2 t sin t -b cost


Y
B

dx dx/dt 3a sin2 t cost a sin t


re

So, the equation of the tangent at the point T is


ou
ad

y-bcos3t= --|(x-a sin3f)


Y

\dx)
b cos t
d

(x - a sin3 t) or, bx cos f + fly sin f = ab sin t cos t


Re

or. y -b cos'31 =
n

a sin t
Fi

The equation of the normal at the point T is


-1 (x-fl sin3f)
y -bcos3t =

5)
y -b cos3 t 1 (x - a sin3 t) or, ax sin t - by cos t = a2 sin4 t -b2 cos4 t
or.
- b cos t
a sin t
EXAMPLE 8 Show that the line — + — = 1 touches the curve y = be~ x^a at the point where it crosses the
y-axis. a b [CBSE 2005,2007, NCERT EXEMPLAR]
SOLUTION The equation of the given curve is
y = be - x/a ...G)
It crosses y-axis at the point, where x = 0. Putting x = 0 in (i), we get: y = be® = b
So, the point of contact is (0, fr).

ReadYourFlow.COM
16.18 MATHEMATICS-XII

Differentiating (i) with respect to x, we get


f - - e° b
dx dx\ a J dx a Kdx \0,b) a a
The equation of the tangent at (0, fr) is
dij^
y-b=\ ~ {x-a)
c1x\o,b)

y-b = --(x-0) => ay-ab = -b x => bx + ay = ab => £+y =1.


a a b
Hence, — + ^ = 1 touches the curve y = b e -x/a at the point where it crosses the axis of y.

x-7
EXAMPLE 9 Find the equations of the tangent and the normal to the curve y = at the
(x-2) (x-3)

w
point, where it cuts x-axis. [NCERT, CBSE 2010]
SOLUTION The equation of the given curve is

Flo
y(x-2)(x-3)-x + 7 = 0
This cuts the x-axis at the point, where y = 0. Putting y = 0 in (i), we get

ee
-x + 7 = 0 => x =7

Fr
So, the point of contact is (7, 0).
Differentiating (i) with respect to x, we get
for
ur
^ (x - 2) (x - 3) + y (2x -5) -1 =0
...(h)
dx
ks

Putting x =7 and y = 0 in (ii), we get


Yo
oo

dy i
(7-2) (7 - 3) -1 = 0 => dy
B

dry (jo) ydx (7,0) 20


re

So, the equation of the tangent at (7, 0) is


n (dx^ 1
ou
ad

y-o= v ~r (x-7) => y-0 = — (x-7) => x -20y-7 = 0


dx (7,0) 20
Y

The equation of the normal at (7, 0) is


nd

1
Re

y-o=- (x-7) => y - 0 = -20 (x -7) => 20x + y-140 = 0


dy)
Fi

V ^7(7,0)

EXAMPLE io Find the equation of the tangent to the curve y =(x3 -1) (x -2) at the points where the
curve cuts the x-axis.
SOLUTION The equation of the curve is
y = 1.x3-i) (.x-2)
It cuts x-axis at y = 0. So, putting y = 0 in (i), we get
(x3 -1) (x - 2) = 0
=> (x -1) (x - 2) (x2 + x + 1) = 0
x—1 = 0 , x - 2 = 0 [v x2 + x + 1 ^ 0]
=> x = 1, 2.
Thus, the points of intersection of curve (i) with x-axis are(l, 0) and (2, 0).

ReadYourFlow.COM
TANGENTS AND NORMALS 16.19

Now, y = (x3-1)(x-2)

=> ^ = 3x2{x-2)+ (x3-1)


dx
dy
=> = - 3 and. = 7.
7(1,0) ^ 7(2,0)
The equations of the tangents at (1, 0) and (2,0) are respectively

y-0 J dy (x-1) and y - 0 =


dy
(x-2)
7(1,0) dx

=> y-0 = - 3 (x -1) and y-0 = 7 (x - 2)


=> y + 3x - 3 = 0 and 7x-y-14 = 0.

Type II ON FINDING TANGENT AND NORMAL PARALLEL OR PERPENDICULAR TO A GIVEN LINE

w
EXAMPLE 11 Find the equation of the tangent line to the curve y = ^J5x - 3 - 2 which is parallel to the
line 4x - 2y + 3 = 0.
SOLUTION

Flo
Let the point of contact of the tangent line parallel to the given line be P(x1, y{).

ee
The equation of the curve is y = y'Sx - 3 - 2.

Fr
Differentiating both sides with respect to x, we get for
ur
dy = 5 5
=>
dx 2 ^5x^3 v dx 2^Sx! - 3
(xvyi)
ks
Yo

Since the tangent at (x1, y^ is parallel to the line 4x - 2y + 3 = 0. Therefore,


oo

dy
B

= (Slope of the line 4x - 2y + 3 = 0)


dx'/(xi'yi)
re

5 -4 73
=> => 4 ySx^ - 3 = 5 => 16 (5X| - 3) = 25 Xj = —
ou
ad

2 f5x1 - 3 -2
Y

Since (xlr y{) lies on y = ^/Sx - 3 - 2. Therefore,


nd
Re

yi = V5*!"3"2
3 73
Fi

=> y\ = -2 = -
80 4 80
(73 3^)
So, the coordinates of the point of contactare I ~ 'J I

Hence, the required equation of the tangent is

n-i (xi'yi)
(x-xj)

3 73 .. (dy\
= 2 x—^ =2
y- - 4 SO dx
Ul,i/l)
=> 80x - 40y -103 = 0
n
EXAMPLE 12 Find the equation of tangent line to y = 2x + 7 which is parallel to the line
4x - y + 3 = 0.
SOLUTION Let the point of contact of the required tangent line be (xq, y{).
The equation of the given curve is

ReadYourFlow.COM
16.20 MATHEMATICS-XII

y = lx2 + 7
Differentiating both sides with respect to x, we get
dy = 4x => dy
dx =
dxA*i,yi)
Since the line 4x - y + 3 = 0 is parallel to the tangent at (.tj, y^).
Slope of the tangent at (xj, y^) = (Slope of the line 4x - y + 3 = 0)
dy -4 Coefficient of x
=> v Slope = -
dx -1 Coefficient of y
=> 4X-L = 4 => xT = 1
Now, (xa/ y1) lies on y = 2x2 + 7.
yi = 2x12 + 7 yi = 2 + 7 = 9 [v *i=l]

w
So, the coordinates of the point of contact are (1, 9).
Hence, the required equation of the tangent line is
y - 9 = 4 (x -1) => 4x - y + 5 = 0

Flo
ee
EXAMPLE 13 Find the equation(s) of normal(s) to the curve 3x - y =8 zuhich is (are) parallel to the

Fr
linex + 3y = 4. [NCERT EXEMPLAR]
SOLUTION Let the required normal be drawn at the point (xq, y1). The equation of the given
curve is
for
ur
3x2 -y2 =8 -(i)
Differentiating both sides with respect to x, we get
ks
Yo

= 3^1
oo

dx dx y \dxj y\
(*uyi)
B

Since the normal at (x1, y1) is parallel to the line x + 3y = 4. Therefore,


re

Slope of the normal at (xq, yf) = (Slope of the line x + 3y = 4)


-1 -1
ou
ad

=> .••(ii)
dy 3
Y

dx\xvyi)
nd
Re

yi 1
=> - 3 =- yi = ...(hi)
3x-j
Fi

Since (xl7 y{) lies on (i). Therefore,


Sx^-yj2 = 8 ...(iv)
Eliminating y-| between (iii) and (iv), we get
SXj2 - xq2 = 8 =:> xq2 = 4 => xq = ± 2
1
Now, xj = 2 ^ yi = 2 [Using (iii)]
and. xq = - 2 => yl = - 2 [Using (hi)]
Thus, the coordinates of the point are (2,2) and (- 2, - 2). The equation of the normal at (2,2) is
1
y-2=- (x-2)
dy)
dx\x1/Vl)

or. y-2 = [Using (ii)]


or. x + 3y - 8 = 0

ReadYourFlow.COM
TANGENTS AND NORMALS 16.21

The equation of the normal at (- 2, - 2) is


1
y-(-2) = -^(*-(-2)) [Using (ii)]
or. x + 3y + 8 = 0
EXAMPLE 14 Find the eqmtion(s) of tangent(s) to the curve y = x + 2x + 6 which is perpendicular to
the line x + 14y + 4 = 0. [NCERT, CBSE 2010]
SOLUTION Let the coordinates of the point of contact be (x-j, y-j). As it lies on y = x3 + 2x + 6

yi = 3 + 2x: + 6
O
The equation of the curve is y = x + 2x + 6.
Differentiating both sides with respect to x, we get
^ = 3x2 + 2=> f—' = 3x^ + 2 •••(ii)
dx dx
Since the tangent at (x^, is perpendicular to the line x + 14y + 4 = 0. Therefore,

low
Slope of the tangent at (xj, y{) x Slope of the line = -1
=> dy x - i-=-l
dxj(x\,y\) 14

(3^2+2)(-y

ee
=> = -1 => 3x12 + 2 = 14 => X| = + 2
rF
Fr
Now, *1 = 2 => yx =23+ 2x2+6 =18 [Using (i)]
x-j = — 2 => yj = (-2)3 + 2(-2) + 6 = -6
for [Using (i)]
ou
So, the coordinates of the points of contact are (2,18) and (- 2, - 6).
ks

From (ii), we obtain


oo

dy = 3 (2)2 + 2 =14 and. dy = 3 (-2)2 + 2 =14


Y
B

dx;(2,18) dxJ(-2,-6)
re

The equation of the tangent at (2,18) is


dy"
ou

(x-2) => y -18 = 14 (x-2) 14x-y-10 = 0


ad

y-18 =
dx )(2,18)
Y

The equation of the tangent at (- 2, - 6) is


nd

Jii)
Re

y-(-6) (x-(-2)) => y -(- 6) = 14 (x -(- 2)) => 14x-y + 22 = 0


dx, (-2,-6)
Fi

Type III ON FINDING TANGENT OR NORMAL PASSING THROUGH A GIVEN POINT


EXAMPLE 15 Find the equations of the tangents drawn to the curve y - 2x - 4y + 8 = 0 from the
point (1, 2).
SOLUTION Suppose the tangent drawn from (1, 2) to the curve y - 2x - 4y + 8 = 0 touches
the curve at (h, k). Then, (h, k) lies on the curve.
k2-2h3-4k + 8 = 0 •••(i)
The equation of the curve is
y2 - 2x3 - 4y + 8 = 0
Differentiating with respect to x, we get
0^ ^ = ^ ('rfy' 3/j2
2y — ~ 6+ - 4 ~ =
dx dx dx y-2 dx)(h, k) k-2

ReadYourFlow.COM
16.22 MATHEMATICS-XII

So, the equation of the tangent at (h, k) is


3h2
y-k (x-h) ox, y-k = (x-h) ..•(ii)
Oi,k) k-2
It passes through (1, 2). Therefore,
3h2
2-k = (1-/7)
k-2
=> -(k-2)2 = 3h2 (1 -h)
=> 3h3 - 3h2 -k2 + 4k -4=0 ...(hi)
Adding (i) and (hi), we get
h3-3h2 + 4 = 0=> (h -2)2 (h +1) = 0 => h = -1,2.
Puttingh = 2in (hi), we get

low
24 -12 -Z:2 + 4k -4 = 0 =* k2-4k-8 = 0 => k = 2±2j3
Putting /? = -1 in (iii) we obtain imaginary values of k.
Thus, the points contact are (2, 2 ± 2V3).

ee
Putting the values of h and k in (h), we obtain the following equations of the tangent
rF
Fr
y-(2 + 2V3) = 2j3(x-2) and y-(2-V3) =-2V3 (x-2).
EXAMPLE 16 Find the equation of the normal to the curve x2 = 4 y which passes through the point (1, 2).
for
[NCERT, CBSE 2013]
ou
SOLUTION Suppose the normal at P (a^ , yf) on the parabola x2 = 4y passes through the point
ks

(1,2). Since P (xx , yf) lies on x2 = 4y.


oo

x* = 4y1
Y
B

The equation of the curve is a:2 = 4 y.


re

Differentiating with respect to, x we get


ou
ad

2 a: = 4^ ^ = £ => (fy) = £i
Y

dx dx 2 Kdx)p 2
The equation of the normal at P (x^, yf) is
nd
Re

1 2
y-y\ = - (AT-Aq) => y-y! =----- (x -at) ...(h)
Fi

It passes through (1, 2).


2-yi =-----(1 -Xj) => 2-ya = — + 2 => a:! yj = 2 ...(hi)
*1
Eliminating yj between (i) and (iii), we obtain
3
x1 o
~~ = 2 => A-T = 8 => ATj = 2

Putting at = 2 in (h), we get yx = 1.


Putting the values of Xj and y^ in (h), we get
y-1 = -1 (x - 2) => x + y - 3 = 0, which is the required equation of the normal.
EXAMPLE 17 Find the coordinates of the points on the curve y = a:2 + 3 a: + 4, the tangents at which pass
through the origin.

ReadYourFlow.COM
TANGENTS AND NORMALS 16.23

SOLUTION Let P (*! , y{) be a point on the given curve such that the tangent at P passes
through the origin. Since P (.r-j , lies on y = x2 + 3ar + 4.
\Jl = x12 + 3x-j + 4
The equation of the curve is
y = .t2 + 3x + 4
Differentiating with respect to x, we get
*1 = 2x + 3 => dy - 2xi + 3.
dx dx)p
The equation of the tangent at P (aq , y{) is

y-*/i (s-Xi) or, y- V! = (2x1 + 3) (x - x-^


[dxjp

w
It passes through the origin i.e. (0, 0).
0 -yi = (2xl + 3) (° -^l) => .Vl = 2xi2 + 3^i ...(ii)
Subtracting (ii) from (i), we get
2 + 4 = 0 => x1 = ±2.
Flo
ree
~xl
From (ii).

F
x-j = 2 => y-^ = 4 + 6 + 4=14 and, xq = - 2 => y^ = 4 - 6 + 4 = 2
or
ur
Hence, the required points are (2,14) and (- 2, 2.).
sf
Type JV MISCELLANEOUS EXAMPLES
k
Yo

EXAMPLE 18 For the curve y =4x3 - 2.x5 find all points at which the tangent passes through the
oo

origin. [NCERT, CBSE 2013]


B

SOLUTION Let (x1, y^ be the required point on y = 4x3 - 2.x5. Then,


re

^ 3 T 5 „.(i)
yi = 4*i ~2xi
ou
ad

The equation of the given curve is y = 4.x3 - 2x5.


Y

Differentiating with respect to x, we get


nd
Re

^ = 12.x2 -10.x4 ^ f ^ = 12x12 -lOjj4


Fi

dx dx',(xvn)
So, the equation of the tangent at (.xq, y-]) is
dy'' (x-x-i) => y-y1 = (12-Xj2 — lO.Xj4) (x — xf)
y-yi = -f:
dxkx\.y\)
This passes through the origin. Therefore,
0 -yT = (12x] 2 - lOXj4) (0 - Xj) => yi = Ux^-lOx/ ...(ii)

Subtracting (ii) from (i), we get


0 = -8x13 + 8x15 => 8x13(x12-1) = 0 =^> Xj = 0 or, x-[ = ± 1
When x-j = 0 => yi = 0 [Using (ii)]
When X! = 1 => y! = 12-10 = 2 [Using (ii)]
When Xi = -1 => y^ = -12 + 10 = -2 [Using (ii)]
Hence, the required points are (0, 0), (1, 2) and (- 1, - 2).

ReadYourFlow.COM
16.24 MATHEMATICS-XII

EXAMPLE 19 Find the equations of all lines of slope -1 that are tangents to the curve y = — , x * 1.
x-1
[NCERT]
SOLUTION Let (X| , yf) be the point of contact of a line of slope -1 which touches the curve
y= —. Then, f—1 = -l.
-1 [dxj
i
Now, y =
x—1
dy 1
=>
dx (x-D2
dy = -i
dxj(x\,y\)

w
i .. (dy
=> = -i = -i
(^i-D2

Flo
dxAxvvl)
=> (Xj-l)2 = 1 => x1-l = ± 1 => Xj = 0,x1 =2.

ee
Fr
Since (x1, y1) lies on the curve y = — . Therefore,
x-1
1
for
ur
yi = ...(i)
Xi -1
ks

Now, xl = 0 => y^ = ~ = -1 [Using (i)]


Yo
oo
B

and. = 2 => yi = = 1 [Using (i)]


2-1
re

Thus, the coordinates of the points of contact are (0, -1) and (2,1).
ou
ad

The equations of the tangents at (0, -1) and (2,1) are respectively.
Y

= (*»)
y+i (x-0) and y-1 (x-2)
dx. (0,-1)
nd
Re

(2,1)
Fi

=> y + 1 = -1 (x - 0) and (y -1) = -l(x-2)


=> x + y + 1 = 0 and x + y - 3 = 0
EXAMPLE 20 Prove that all normals to the curve x = a cos t + at sin t, y = a sin t - at cos t are at a
distance afrom the origin. [NCERT, CBSE 2013]
SOLUTION The equations of the curve are:
x = a cos t + at sin t and y = a sin t - at cos t
=> — = at cos t and — - at sin t
dt dt
dy
dy _ dt _ at s^n 1 = tan t
dx dx at cos t
dt
The equation of the normal at any point t is given by
y-(fl sin f-flf cosf) =--^-{x-(flcosf+ flfsinf))
dy
dx

ReadYourFlow.COM
TANGENTS AND NORMALS 16.25

=> y-(a sin t - at cos t) = - —— {x - (a cos t + at sin f)l


tan t
cos t
=> y - (a sin t - at cos t) = - {x - (a cos t + at sin t)}
sin t
=> 1/ sin t - (a sin ? t - at sin t cost) = -x cos t + a cos 2 t + at sm t cos t
x cos t + y sin t = a •~(i)
10 cos t + 0 sin t -a\ _ ^
Length of the perpendicular from the origin to (i) =
ycos2 t + sin2 t
Hence, all normals to the given curve are at a distance 'a' from the origin.

LEVEL-2

w
Type V ON FINDING THE EQUATIONS OF TANGENT AND NORMAL
EXAMPLE 21 Find the equation of the normal to the curve y =(1 + x)'7 + sin - 1 (sin2 x) at x = 0.

Flo
SOLUTION We have,
y = (1 + x)y + sin- 1 (sin2 x)

ee
Putting x = 0, we get

Fr
y = (1 + 0)^ + sin -1 (sin2 0) => y = 1. for
ur
Thus, we have to write the equation of the normal to (i) at P (0,1).
Differentiating (i) with respect to x, we get
ks

= eJ !°8 (1+*). A (y log (1+*)}.,. 1


Yo

(sin2*)
oo

dx dx - sm ix dx
B

^ = (l + x)* ^ . log (1 + x) + — 2 sin x cos x


re

=> +
dx dx 6 1+x cos x:\ V1 + sm2 x
ou
ad

Putting x = 0 and y = 1, we obtain


Y

dy dy dy
x o + n + o => = 1
nd
Re

dxjp dx Jp dx Jp
Fi

Hence, the equation of the normal at P (0,1) is

y-i =1A (x-0)=> y-1 = -1 (x - 0) => x + y = 1


dx Jp
EXAMPLE 22 Find all the tangents to the curve y = cos (x + y), - 2h < x < 2k that are parallel to the
linex + 2y = 0. [NCERT EXEMPLAR, CBSE 2016,2017]
SOLUTION Let the point of contact of one of the tangents be (xj, y1). Then, (xj, yf) lies on
y = cos (x + y)
y\ = co- (x-l + ya) ...(i)
Since the tangents are parallel to the line x + 2y = 0. Therefore,
Slope of the tangent at (xq, Vj) = (Slope of the line x + 2y = 0
dy 1
=>
dx (*i.yi) 2

The equation of the curve is y = cos (x + y).

ReadYourFlow.COM
16.26 MATHEMATICS-XII

Differentiating with respect to x, we get


dy = _ - sin (x + 1/) 1 + ^
dx dx

=> dy = - sin {x1 + i/!) U + dy)


dxhxvyi) dx\x1,y1)
1
=> sin (Xl + yi)
2
sin (x1 +y{) =1 ...(ii)
Squaring (i) and (ii) and then adding, we get
cos2 {x1 + yx) + sin2 + y{) = yj2 + 1 => 1 = y^2 + 1 => yj = 0

Putting yi = 0 in (i) and (ii), we get

w
- 3tz
cos = 0 and sin Xj = 1 => Xj = — [-.• - 2rc < Xi < 2tc]
2' 2

Flo
Hence, the points of contact are (rc/2, 0) and (- 3n/2,0).
The slope of the tangent is (-1/2). Therefore, equations of tangents at 0 j and ' 0j

ee
are

Fr
^ V + "T”) respectively
y-° = and y-° = for
ur
or. 2x + 4y - 7r = 0 and 2x + 4y + 3ji = 0 respectively.
ks

Type VT ON FINDING THE EQUATION OF THE CURVE


Yo

EXAMPLE 23 The curve y = ax3 + bx2 + cx + 5 touches the x-axis at P (- 2, 0) and cuts the y-axis at
oo

the point Q where its gradient is 3. Find the equation of the curve completely.
B

SOLUTION We have,
re

Q n
y = ax +bx + cx + 5
ou
ad

=> ^ = 3ax2 + 2bx + c


Y

dx
Since the curve y = ax3 + bx2 + cx + 5 touches the x-axis at P (- 2, 0). This means that the curve
nd
Re

passes through P (- 2, 0) and x-axis is the tangent at P (- 2, 0).


Fi

dy
0 = - 8a + 4b - 2c + 5 and. =0
dx Jp
=> 8a - 4b + 2c =5 and, 3a (- 2)2 + 2b x ( - 2) + c = 0
8a-4b + 2c =5 ...(i) and. 12a - 4b + c = 0 ...(ii)
The curve y = ax' + bx + cx + 5 meets y-axis at Q. Putting x = 0 in y = ax + bx + cx + 5, we
get: y =5. Thus, the coordinates of Q are (0, 5).
It is given that the gradient of the curve at Q is 3.
dy = 3 => 3AxO + 2bxO + c = 3 => c = 3
UxJq
Putting c = 3 in (i) and (ii), we get
8a - 4b = -1 and 12a - 4b = - 3
1 ,, 3
Solving these two equations, we get: a = --
— and b = —
4

ReadYourFlow.COM
TANGENTS AND NORMALS 16.27

Substituting the values of a, b and c in the equation of the curve, we obtain


1
— x 3 - — x2 + 3x + 5 as the equation of the curve.

EXAMPLE 24 Determine the quadratic curve y =f (x) if it touches the line y = x at the point x =1 and
passes through the point (-1, 0).
SOLUTION Let the required quadratic curve be
y = ax2 + bx + c
It passes through (-1, 0). Therefore,
0 = a-b + c ...(h)
Differentiating (i) with respect to x, we get
rdy
— = lax + b=> = 2a + b
dx dxjx=\

w
Since the line y - x touches curve (i) at x = 1. Therefore,
(Slope of the tangent at x = 1) = (Slope of the line y = x)

Flo
dy ...(hi)
=> = 1=> 2a+ b= l
\dxJx = 1

ee
Putting x = 1 in y = x, we get y = 1. Thus, the curve (i) passes through (1,1).

Fr
1 = a+b+c ...(iv)
Solving (ii), (hi) and (iv), we get
for
ur
1
a = — ,b = — and c = —
4 2 4
ks
Yo

x2 x 1
oo

Substituting these values in (i), we get y = —


^ + 2 + ^ 38 re<iu^re£^ S^dratic curve.
B
re

EXERCISE 16.2
LEVEL-1
ou
ad

1. Find the equation of the tangent to the curve Vx + ^/y = fl, at the point (fl2/4, a2/A).
Y

2. Find the equation of the normal to y = 2x3 - x2 + 3 at (1, 4).


nd
Re

3. Find the equations of the tangent and the normal to the following curves at the indicated
Fi

points:
(i) y = x4 - bx3 + 13x2 - lOx + 5 at (0,5) [NCERT]
(ii) y = x4 - 6x3 + 13x2-lOx + 5 at x =1 [NCERT, CBSE2011]
(hi) y = x2 at (0, 0) [NCERT] (iv) y = 2x2 - 3x -1 at (1,-2)
x3
(v) y2 = at (2,-2) (vi) y = x2 + 4x + 1 at x = 3 [CBSE 2004]
4 -x
x2 v2 2 2
(vii) — + ^ = 1 at (« cos 0, b sin 0) (viii) —y — = 1 at (fl sec 0, £> tan 0)
a2 b2 az bz
(x) c2 (x2 + y2) = x2 y2 atf—- c
(ix) y2 =\.ax at (a/w2, 2a/m)
^ cos 0 sm 0
2 2
(xi) xy = c2 at (ct, c/t) (xh) + \=1 at(x1,y1)
a b

ReadYourFlow.COM
16.28 MATHEMATICS-XII

2 2
(xiii) = 1 at (;r0/ y0) [NCERT] (xiv) x 2/3 + y2/3 =2 at(1,1) [NCERT]
a b
(xv) x2 = 4y at (2,1) (xvi) y2 = 4x at (1, 2) [NCERT]
(xvii) 4x2 + 9y2 = 36 at (3 cos 0, 2 sin 0) [CBSE 2011]
2 r2 v1
(xviii) y = 4ax at (xj, yj) [CBSE 2012] (xix) = 1 at (V2a, b) [CBSE 2014]
az b
4. Find the equation of the tangent to the curve x = 0 + sin0, y=l + cos 0 at 0 = ti/4.
5. Find the equations of the tangent and the normal to the following curves at the indicated
points:
(i) x = 0 + sin 0, y = 1 + cos 0 at 0 = k/ 2.
2 at2 2 at3 1
(ii) X = y = ----- yat t =1/2.
itt2' 1 +f2

low
(iii) x =flf2, y = 2flf at t=l.
(iv) x = fl sec f, y = & tan f at t.
(v) x = a(0 + sin 0), y =a(l-cos 0) at 0.

ee
(vi) x = 3 cos 0 - cos 3 0, y = 3 sin 0 - sin 3 0 [NCERT EXEMPLAR, CBSE 2016]
rF
Fr
6. Find the equation of the normal to the curve x + 2y - 4x - 6y + 8 = 0 at the point whose
abscissa is 2.
for
7. Find the equation of the normal to the curve ay2 = x 3 at the point (am2, am 3).
ou
ks

[CBSE 2012, NCERT]


8. The equation of the tangent at (2,3) on the curve y = ax +b is y = 4x - 5. Find the values of
oo
Y

aandb. [CBSE 2016]


B

9. Find the equation of the tangent line to the curve y = x + 4x -16 which is parallel to the
re

line 3x - y +1 = 0. o
10. Find an equation of normal line to the curve y = x + 2x + 6 which is parallel to the line
ou
ad

x + 14y + 4 = 0. [CBSE 2013]


Y

11. Determine the equation(s) of tangent(s) line to the curve y = 4x3 -3x + 5 which are
d

perpendicular to the line 9y + x + 3 = 0.


Re
n

12. Find the equation of a normal to the curve y = x log^ x which is parallel to the line
Fi

2x - 2y + 3 = 0.
13. Find the equation of the tangent line to the curve y = x - 2x + 7 which is
(i) parallel to the line 2x - y + 9 = 0
(ii) perpendicular to the line 5y - 15x = 13.
[NCERT, CBSE 2014]
14. Find the equations of all lines having slope 2 and that are tangent to the curve
1
y= , x * 3. [NCERT]
x-3
15. Find the equations of all lines of slope zero and that are tangent to the curve
1
y = [NCERT]
x2 - 2x + 3
16. Find the equation of the tangent to the curve y = ^J3x - 2 which is parallel to the line
4x - 2y + 5 = 0. [NCERT, CBSE 2005,2009]
17. Find the equation of the tangent to the curve x + 3y - 3 = 0, which is parallel to the line
y = 4x -5. [CBSE 2005]

ReadYourFlow.COM
TANGENTS AND NORMALS 16.29

18. Prove that j + ^j touches the straight line —+ ^ = 2 for all neN, at the

point(fl, b). [CBSE2007C1


19. Find the equation of the tangent to the curve * = sin 3f, y = cos 2f at f = ^. [CBSE2008]
o n
20. At what points will be tangents to the curve y = 2x - 15x + 36x - 21 be parallel to x-axis?
Also, find the equations of the tangents to the curve at these points. fCBSE 2011]
21. Find the equation of the tangents to the curve 3x - y =8, which passes through the point
(4/3, 0). [CBSE2013]
ANSWERS
1. x + y =cr/l 2. x + 4y =17
3. Tangent Normal

w
(i) y + 10.x - 5 = 0 x-lOy+ 50 = 0
(ii) 2x - y + 1 = 0 x + 2y - 7 = 0
x=0

Flo
(iii) y = 0
(iv) x - y - 3 = 0 x+y+1=0

ee
(v) 2x + y - 2 = 0 x - 2y - 6 = 0

Fr
(vi) 10x-y-8 = 0 x + lOy - 223 = 0
(vii) - cos 0 + — sin 0 = 1 ax sec Q - by cosec Q = a2 -b2
for
ur
a b
9 9
(viii) - sec 0-^ tan 0=1 ax cos Q + by cot Q = a +b
ks

a b
Yo
oo

(ix) m2x-my + fl = 0 m2 x + m3 y -2am2 - a = 0


B

(x) x cos3 0 + y sin 3 0 = c x sin 3 0 - y cos3 0 + 2c cot 20 = 0


re

(xi) x + yt2 =2ct x t3 -ty = ct4 - c


ou

m=1
ad

a2 x b2y
+ = a2 -b2
Y

b2 xi yi
a 2"
x b2y
d

xxo yyo_1
Re

+ = a2 +b2
n

b2 x0 yo
Fi

(xiv) x + y-2 =0 y-x =o


(xv) x - y -1 = 0 x+y-3=0
(xvi) x -y + 1 =0 x+y- 3 =0
(xvii) 2x cos 0 + 3y sin 0 = 6 3x sin 0 - 2y cos 0-5 sin 0 cos 0 = 0
(xviii) yy1=2fl(x + x1) -yi (X-Xi)
y-yi =
2a
V2x_y ax + by=az+bz
, 2 u2
(xix) =1 -^
a b

5. Tangent Normal
(i) 2x + 2y - it - 4 = 0 2x-2y = n
(ii) 13x -16 y - 2a = 0 16x + 13y-9fl = 0
(iii) x -y + a =0 x + y = 3a

ReadYourFlow.COM
16.30 MATHEMATICS-XII

(iv) b x sec t - ay tan t =ab ax cos t + by cot t= a2 + b2


(v) y =(x-a 0) tan (0/2) (y - 2a) tan (0/2) + .t - a 0 = 0
(vi) 4(ycos30-x sin30) = 3sin40
6. x = 2 7. 2x + 3m y - am2 (2 + 3m2) = 0
8. a = 2,b=-7 9. 12x - 4y - 65 = 0
10. x + 14y + 86 = 0, x + 14y - 254 = 0 11. 9x - y - 3 = 0, 9x - y +13 = 0
12. x -y = 3e -2 13. (i) 2x - y + 3 = 0 (ii) 12x + 36y - 227 = 0
14. There is no tangent to the curve that has slope 2.
15. y =1/2 16. 48x - 24y = 23 17. 4x - y + 13 = 0
19. 2V2x - 3y - 2 = 0 20. (2,7), (3, 6) 21. y = 3x-4

w
HINTS TO NCERT& SELECTED PROBLEMS
3. (i) The equation of the curve is y = x4 - frx3 + 13x2 - lOx + 5.

Flo
^ = 4x3 - 3frx2 + 26x -10 dy
= -10

ee
dx dx

Fr
The equation of tangent at (0,5) is
for
ur
y-5 = dy (x-0) y-5 = -10(x-0) => 10x + y-5 = 0
dx
ks

The equation of the normal at (0,5) is


Yo
oo

y -5 = - 1 1
(x -0) => y -5 = — (x - 0) => x -lOy + 50 = 0
B

10
re

dx 0,5)
(ii) The equation of the curve is y = x4 - 6x3 + 13x2 - lOx + 5
ou

-(i)
ad
Y

^ = 4x3-18x2+26X-10 ^ = 4-18 + 26-10 = 2


dx dxJx=l
nd
Re

Putting x = 1 in (i), we get y = 3.


Fi

The equation of the tangent at (1, 3) is


dy''
y-3 = (x-l)=^ y-3 = 2(x-l)=> 2x-y+ 1 = 0
dx)x=\
The equation of the normal at (1, 3) is
1 1
y - 3=- (x-l)=>y-3=--(x-l)=>x + 2y-7 = 0

\dx)x=\
(iii) The equation of the curve is y = x .
Differentiating with respect to x, we get
^ = 2x => dV = 0
dx dx '(0,0)
So, the tangent at (0,0) is parallel to x-axis and hence the normal there at is parallel to y-axis.
So, their equations are y = 0 and x = 0 respectively.

ReadYourFlow.COM
TANGENTS AND NORMALS 16.31

x2 y2
(xiii) The equation of the curve is = 1
b2
Point P(xq, y0) lies on (i). Therefore,
2 2
-ro Vo = 1 ...(ii)
«2 b2
Differentiating (i) with respect to x, we get
2x 2y dy
a2 b2 dx dx a2 y dx Jp
The equation of the tangent at P (x0, t/o)

y-yo

w
y-yo =
2 2

Flo
^ yyo-y0 -^o ^ yyo yo ^ =1 [Using (ii)]
b2 a2 «2 fo2 b2 a2 b2

ee
The equation of the normal at P (x0/ i/q) Is

Fr
y-yo =- dy—r (X — Xq)
for
ur
{dxjp
a2 a2 x b2y
(x-x0) => — (y-y0) = ------(x - x0) => = a2 +b2
ks

=> y-yo = -
yo x0 *o yo
Yo
oo

(xiv) We have,
B

1/3 dy
x2/3 + y2/3 = 1=>lx-V3 + 2 y3dy=0=>dy = -L- => = -1
re

3 3 “ dx dx x1/3 dx A h!)
ou

The equation of the tangent at (1,1) is


ad

rfy
Y

(x—1) => y-1 = -l(x-l) =>x + y-2 = 0


dx\l, 1)
nd
Re

The equation of the normal at (1,1) is


1 -1
Fi

y-i=- (x-1) => y -1 = —(x -1) => x = y


dy -1
ydx\\, i)
2
(xvi) The equation of the curve is y = 4x.
Differentiating with respect to x, we get
dy 2
2y = 4 => d JL = l^ = - = 1
dx dx y dxJ(l,2) 2
The equation of the tangent at (1, 2) is
dy
y-2 = (x -1) => y-2 = (x —1) => x-y + 1 = 0
dx'(l,2)
The equation of the normal at (1, 2) is
1 (x-1) => y-2 = -^-(x-l) => x + y-3 = 0
y-2=-

ReadYourFlow.COM
16.32 MATHEMATICS-XII

7. We have, ay2 -x3


Differentiating with respect to x, we get
3a2 m4 3m
dx dx lay dx J(cmi2 ,am3) la2 m3 2
The equation of the normal at (am2, am3) is
y-am3 = — (x - am2) or, 2x + 3my -am2 (2 + 3m2) = 0
3m
13. The equation of the curve is y = x2 - lx + 7.
Differentiating with respect to x, we get
— = 2x -2
dx
(i) Let P {x^, yj) be a point on y = .t2 - 2x + 7 such that tangent at P is parallel to the line

w
2x - y + 9 = 0. Then,
dy

Flo
= 2 => 2X| -2 = 2 => X] = 2
Vdxjp

ee
Since P (x^, y{) lies on y = x - 2x + 7. Therefore,

Fr
y\ = Xj2 -2xj +7 => y-j = 4- 4 + 7 = 7
Hence, required point is (2, 7).
for
ur
The equation of the tangent at (2, 7) is
dy''
ks

y-7 = (x - 2) => y - 7 = 2 (x - 2) => 2x - y + 3 = 0


Yo

dx)p
oo

(ii) If the tangent at P (xj, y-j) is perpendicular to the line 5y - 15x = 13. Then,
eB

dy_
x 3 = -1 => (2x1-2)x 3=-l=>x1= —
dx)p 6
r

9 —
ou

Since (x-j, y{) lies on y = x - 2x + 7.


ad

2 25 5+7 = 217
Y

yi = xa - 2xl +7 => y-j =


36 3 36
nd
Re

(5 217^
The equation of the tangent at P — ,---- is
V. 6 36 y
Fi

217 1 5 1
y- X-----
36 3 6 3
or, 12x + 36y - 227 =0
1
14. Let (xj, y-j) be the point of contact of a line of slope 2 which touches the curve y =
x * 3. x-3 '
1 => ^ = 1 dy 1
Now, y = ------- + =>
x-3 dx (x - 3)2 dx (^i - 3)2

But, dy = 2.
dx\x\,y\)
1
= 2
(^1 - 3)2
=> 2(xi~ 3) = -1, which is not possible as LHS is positive and RHS is negative.

ReadYourFlow.COM
TANGENTS AND NORMALS 16.33

Hence, there is no tangent line of slope 2 to the given curve.


15. Let P(x1/i/1) be the point of contact of a line of slope zero which touches the curve
1
y= at point P.
x2 - 2x + 3
1
The equation of the curve is y =
x2 - 2x + 3
Differentiating with respect to x, we get
dy _ -2(x-l)
=> dy
dx (x2 - 2x + 3)2 dx (^12-2x1 + 3)2
dy_
It is given that = 0.
dx
(*pyi)

w
= 0 => X! = 1
(Xj2 -2XJ + 3)2

Since (x^, y^) lies on y =


1
x2 - 2x + 3

Flo
ree
1 1 1

F
J/l = ^ yi =
x^2 - 2xi + 3 1-2+3 2
or
ur
Hence, the equation of the tangent is y - ^ = 0 (x -1) or, y = ^ .
sf

16. Let (xj, y|) be the point of contact of tangent to the curve y = ^j3x -2 which is parallel to
k
Yo
oo

the line 4x - 2y + 5 = 0. Then,


B

dy_ = (Slope of the line 4x - 2y + 5 = 0)


re

rfy
ou

=>
ad

= 2
Y

dy _ 3 dy 3
Now, y = J3x -2 => ...(ii)
nd

2px-2 ^ dxj(x\,y\)
Re

dx 2j3^2
Fi

From (i) and (ii), we get


3 41
= 2 => 9 = 16(3x1 -2) => X! = —
2A/3x1-2 48
Since (x-j, y^ lies on y = ^3x - 2. Therefore,

yi =
yi = JS'2 = !
f 41 3^1
So, the point of contact I ^ I•

f 41 3 s!
The equation of tangent at I ^^ I i

or, 48x - 24y = 23


48
18. We have, +^j
=2

ReadYourFlow.COM
16.34 MATHEMATICS-XII

Differentiating both sides with respect to x, we get


\n- 1
xY'-1! j/Y"1! rfy=0^ fy=Z_bfxY~1(b dy -b
n -----1-71 =>
a a b b dx dx a a y dxJ(a,b) a
The equation of the tangent at {a, b) is
—b x
y -b = — (x - a) => ay - ab =-bx + ab => bx + ay = lab => — + 1=2
a a b
Hence, — + — = 2 touches the given curve at (a, b) for all n e N.
a b

16.4 ANGLE OF INTERSECTION OF TWO CURVES


DEFINITION The angle of intersection of two curves is defined to be the angle between the tangents to the
two curves at their point of intersection.
LetC! and C2 be two curves having equations y =f(x) and y = g(x) respectively. Let P7\ and PT2

w
be tangents to the curves Cj and C2 respectively at their common point of intersection. Then, the
angle ^between PTj and PT2 is the angle of intersection of Cj and C2. Let \\i ^ and \\i2be the angles
made by PT-^ and PT2 with the positive direction of x-axis in anticlockwise sense. Then,
m1 = tan \\i j

Flo
ree
m-j = (Slope of the tangent to y =f(x) at P ) = j
==>
Cl

F
and. m2 = tan \\i2
or
ur
=^> m2 = (Slope of the tangent to y = ,y(x) at P) = [ —
sf
\dx)
C2
From Fig. 16.3, it is evident that
k
Yo

Y
oo

<l> = Vl “V2
B

=> tan (j) = tan (vj/j - \j/2)


re

tan v|/ j - tan \\i 2


=> tan (}) =
ou
ad

1 + tan vj/j tan \\i2


Y

'di) Jdy)
Jx)Cl {dx) C2
nd
Re

=> tan (j) =


df o
V
Fi

(dy
1+
dx)Cl Ux) Fig. 16.3

The other angle between the tangents is 180° - (jx Generally, the smaller of these two angles is
taken to be the angle of intersection.
ORTHOGONAL CURVES If the angle of intersection of two curves is a right angle, the two curves are said
to intersect orthogonally and the curves are called orthogonal curves.

If the curves are orthogonal, then ‘l’ = ~-

dy = -i
m\m2 ~ ~ 1 =>
dx, C2

REMARK If the angle of intersection of two curves is zero, then = dy at the point of
dxJcl ydx
C2
intersection and the two curve touch each other at the point of intersection.

ReadYourFlow.COM
TANGENTS AND NORMALS 16.35

ILLUSTRATIVE EXAMPLES

LEVEL-1
EXAMPLE 1 Find the angle of intersection of the folloxving curves:
(i) xy = 6 and x2y = 12 (ii) y2 = 4x and x2 = 4y
SOLUTION (i) The equations of the two curves are
^=6
and, x y = 12 ...(ii)
From (i), we obtain y =-. Putting this value of y in (ii), we obtain
x2(*
= 12 => 6x = 12 => x = 2
x
Putting x = 2 in (i) or (ii), we get y = 3. Thus, the two curves intersect at P (2, 3).

w
Differentiating (i) with respect to x, we get
x*I + v=0^?v=^l => W-j =
dv 3

Flo
dx dx x dxJ(2,3) 2
Differentiating (ii) with respect to x, we get

ee
x2^ + 2xy = 0 ^ = m2 = dV

Fr
= -3
dx dx x dxH2,3)
Let 0 be the angle of intersection of curves (i) and (ii) at point P, then
for
ur
W} -m2 -(3/2)+ 3 3
tan 0 = = — ^ 0 = tan -1
1 + m-j m2 l+(- 3/2) (-3) 11 11
ks
Yo

(ii) The equations of the two curves are


oo

y2 = 4x
B

and. x2 = 4y ...(ii)
re

2 2
From (i), we obtain x = V V in (ii), we get
Putting x = ^-
ou
ad
Y

y2
j =4y => y4 - 64y = 0 => y(y3 -64) = 0=>y = 0, y=4
nd
Re
Fi

From (i), when y = 0, we get x = 0 and when y = 4, we get x = 4. Thus the two curves intersect at
(0, 0) and (4, 4).
Differentiating (i) with respect to x, we get
dV dy _ 2
2y -f- = 4 ...(hi)
dx dx y
Differentiating (ii) with respect to x, we get
4dy ^ dy = x
2x = ...(iv)
dx dx 2
Angle of Intersection at (0, 0); From (iii), we get
= 00
dx y (0,0)
Therefore, the tangent to curve (i) at (0, 0) is parallel to y-axis.
From (iv), we get
=0
vdx 7(0,0)

ReadYourFlow.COM
16.36 MATHEMATICS-XII

Therefore, the tangent to curve (ii) at (0, 0) is parallel to x-axis.


Hence, the angle between the tangents to two curves at (0,0) is a right angle. Consequently, the
two curves intersect at right angle at (0, 0).
Angle of Intersection at (4, 4): From (iii), we obtain
dy 2 1
mi =
dxJ(4r4) 4 2
From (iv), we obtain
rdy ^ = 2
m2 =
dxJ(4,4) 2
Let 0 be the angle of intersection of the two curves. Then,
m2 - tn^ 2-(1/2) 3
tan 0 =
1 + m-! m2 1 + 2x(l/2) 4

w
EXAMPLE 2 Find the angle between the parabolas y = 4ax and x = 4by at their point of intersection
other than the origin. [CBSE 20161
SOLUTION

Flo
The equations of two parabolas are y = 4ax and x = 4by.

ree
x2
Now, x^2 =4by=>
Al
y=—.x

F
4b
Substituting this value of yiny2= 4ax, we get
or
ur
2^2
x
sf
= 4ax
4b
k
Yo

=> x4 - 64ab2 x = 0 => x(x3 -64ab2) =0 => x = 0, x3 = 64ab2 => x = 0, x = 4 3 b2^2,


oo

2
B

Putting x = 0 and x = 4fl1// 3 b2/ J successively in y = — , we get y = 0 and y = 4a2''3 b 1/3


re

respectively.
ou
ad

Thus, the two curves intersect at P(4a^ 3 b2^ 3,4fl2/ 3 bl/ 3) other than the origin O (0, 0).
Y

Now, y2 = 4ax and x2 = 4by


2y ty-=4a and 2x = 4b —
nd

=> [Differentiating both with respect to x]


Re

dx dx
Fi

=> ^= and ‘ty- = —


dx y dx 2b

=> ml =
dy
dx)p
2a
4a2/3b1/3 i tr Uy\
2 [dx 'P
4aV2b2'2 , aV/3
and m-y = — =--------------- = 2
2b
Let 0 be the angle between the tangents to the parabolas y2 = 4ax and x2 = 4by at P. Then,
b

-m2
tan 0 =
1 + m-! m2

1 (a
1/3
-2
«y/3 3(a
1/3
i
2Vb 3a1/3b1/3
2 yb b
=> tan 0 =

0 = tan 1
1 +
i cr X 2

3 (ab) 1/3
flV/3
b
i+
^

b
2(a2/3+b2/3)

=>
2(a2/3+b2/3)

ReadYourFlow.COM
TANGENTS AND NORMALS 16.37

EXAMPLE 3 Show that the curves x=y~ and xy = k cut at right angles, if 8k = 1.
fCBSE 2004, 2005, 2013]
SOLUTION The given curves are
x = y2
and, xy = k ...(h)
r\

From (i), we obtain x = y . Putting this value of x in (ii), we obtain


y3 = k => y = k1/3
Putting y = k 1/3 in (i), we get x = k2''3.
So, the two curves intersect at the point P (k 2/3 ji 1/3)
Differentiating (i) with respect to x, we get
_ dy dy 1 dV 1
1 = 2y — => — = — => mi = 2^1/3
dx dx 2y dx JP

w
Differentiating (ii) with respect to x, we get
*1/3 1
- — y =
1 -y + x — = 0 => => m2 =

Flo
dx dx x dxjp *2/3 *1/3

ree
For the curves (i) and (ii) to cut at right angles at P, we must have
, 1 1
mi m2 = -1 => 2*1/ 3 X ~ *!/ 3 = -l => 2k2/3=l => (2k2/ 3)3 =13 => 8k2 = 1.

F
EXAMPLE 4 Find the values of p for which the curves x = 9p (9 -y) and x =p(y +1) cut each other
or
ur
at right angles. fCBSE 2015]
sf
SOLUTION The equations of the given curves are
k
Yo

x2 = 9p (9 -y)
oo

and, x2 =p(y + l) ...(ii)


B

To find the coordinates of the point(s) of intersection of (i) and (ii), we solve the two equations
re

simultaneously. On eliminating x2, we obtain


ou
ad

9p (9-y) (y+ 1) => 81 -9y =y+ 1 => lOy = 80 => y = 8


Putting y = 8 in (i) or (ii), we obtain
Y

x2 = 9p => x = ± 3^Jp
nd
Re

Thus, curves (i) and (ii) intersect at P (S^Jp, 8) and Q (-S^fp, 8).
Fi

Differentiating (i) and respect to x, we obtain


n dy dy 2x 2x 3jp _ 2
2x = - 9p — => — =-----=> mi
H dx dx 9p ■(f Cl 9p 3VP
Differentiating (h) with respect to x, we obtain
„ dy dy 2x dy _2(3^) _ 6
2x = p —=> —= — => m2 =1 -f-
P dx dx p dx;c2 P fP
If curves (i) and (ii) cut each other at P at right angles, then
-2
m1m2 = -1 => —j= x -=
6
= -1i => p = 4^
3VP a/p
Similarly, by using the condition of orthogonality of the curves at Q, we obtain p = 4.
Hence, the two curves cut each other at right angles, if p = 4.
9 9 9 9
EXAMPLE 5 Show that the curves xy = a and x + y = 2a touch each other.
SOLUTION The given curves are [CBSE 2002]
xy = *2

ReadYourFlow.COM
16.38 MATHEMATICS-XII

X2 + y1 = 2a1 -(ii)
«2
From (i), we get y = —. Substituting this value of y in equation (ii), we get

.= la1 => a:4 -2flV +fl4 = 0 => (x2-a1)1 = Q => x = ±a


x
From (i), we get
y = a for x = a and, y = -a for x = - a.
Thus, the two curves intersect at P(a, a) and Q(- a, - a).
Differentiating both sides of curve (i) with respect to x, we get
= 0^ ^ = -y ...(in)
dx dx x
Differentiating both sides of curve (ii) with respect to x, we get

w
2x + 2y — = 0 => dy _ _ x ...(iv)
dx dx y

Flo
Angle of intersection at P (a, a): Substituting x = a, y = a in (iii) and (iv), we get

ree
dy
— = -1 and,
dx a dx a
'--2

F
Clearly, ^ dv at P. So, the two curves touch each other at P.
or
ur
dx C2
sf
Similarly, it can be seen that the two curve touch each other at Q.
k
Yo
oo

LEVEL-2
B

EXAMPLE 6 Show that the condition that the curves


re

ax2 + by2 = 1 ...(i) and a’ x2 +b’ y2 =1 ...(h)


ou

1111
ad

Shoidd intersect orthogonally is that —


a b a! b [NCERT EXEMPLAR]
Y

SOLUTION Let (xq, yf) be the point of intersection of the given curves. Then,
nd
Re

2 , 2
axl + m/1 = 1 ...(hi)
Fi

o'xl +b'y1 = 1 ...(iv)


Differentiating (i) with respect to x, we get
ax ax1
2 ax + 2by — = 0 => d± = ------ => m-j =
dy
...(v)
dx dx by by\
Differentiating (ii) with respect to x, we obtain
a'x dy
2a' x + 2b' y ^ = 0 ^ ^ = -------=> m2 = = a'xl
...(vi)
dx dx b'y dX',(xvy1) b'y\
The two curves will intersect orthogonally, if
i
;;q »72 = -1=> -SLx-^a. = -1 S. ao-x.2 = -bVy,2 ...(vh)
b’yi 1
Subti-acting (iv) from (hi), we obtain
(fl-fl'JXj2 = -(b-b')y^2 ...(vih)

ReadYourFlow.COM
TANGENTS AND NORMALS 16.39

Dividing (viii) by (vii), we get


a-a' b-V_ 1 _1 _ JL_ ^
aa’ bb' ^ a b ~ a' b''

x2 i/2
EXAMPLE 7 If the straight line xcos a + i/sin a = p touches the curve —y- + = 1, then prove that
a1 bl
a2 cos2 a+b2sm2a=p2. [NCERT EXEMPLAR]

x2 y2
SOLUTION Suppose the straight line xcosa + ysin a =tc touches the curve -y + ^ = 1 at
fc2
P (x1/ y1). Then it is the equation of tangent to the given curve at P {xlr y1). But, the equation of
x2 y 2
tangent to -y + — = 1 at P (xj, y-j) is

[See Example 5 on page 16.16]

w
-1

Thus, equations ^y +1 = 1 and xcos a + y sin a = p represent the same line.

Flo
a1 b2
xq/g2 ^ yi/b2 _ 1 2 b2 sin a
cos a

ee
a
- => Xq =
cos a sin a p

Fr
V V
2 2
x y
—r- +
The point P (xq, yq) lies on the curve Ty — 1. for
ur
a b2
r 2 ,/ 2
+yi_=i
ks

a hi
b
Yo

4
oo

fl'cos2a b4sin2a .
=> [Using (i)]
2„2
r_+_pP—=1
B

p a
re

=> a2 cos2a+i'2sin2a =p2


ou
ad

EXAMPLE 8 Show that the angle between the tangent at any point P and the line joining P to the origin
O is the same at all points on the curve log (x2 + y2) = k tan- 1 j
Y
nd

The equation of the curve is log x2 + y2) = k tan -i y


Re

SOLUTION
x
Fi

Differentiating with respect to x, we get


dy -y
x—
i i
2
* +y
2 {2x+2y%} = k dx
x2
Y

dy
2 i x + y -y- = k1 \ x —
dv -y
dx dx
dy
2x + ky = (kx - 2y) -j-
dx .■ a
dy _ 2x + ky
=>
dx ~ kx -2y 0
Let the coordinates of P be (xq, yq). Then, o T X
'di) = 2xl + ^1
\dxjp kx1-2y1
Fig. 16.4

ReadYourFlow.COM
16.40 MATHEMATICS-XII

If the tangent at P makes an angle 9 with x-axis, then


tan 9 = 2*1 + tyl
kx1 - 2y1
Suppose OP makes an angle <t> with .r-axis. Then, tan <() = Slope of OP = —
*1
Let a be the angle between OP and PT. Then,
9 = a + (J>
=> a = 9 - (J)
=> tan a = tan (9 - (j))
tan 9 - tan (j)
tan a =
1 + tan 9 tan (j)
2x1 + ky1_y1 2

low
foi-2yi xi 2xi +kxiyi-kxiyi + 2Vi = 2
=> tan a
k*!2-2x1y1 + 2xii,1 + ky12 k
1 + 2xi + kyi,, 3/1
kx1-2yl xj

ee
=> = Constant.
rF
Fr
k
for EXERCISE 16.3
LEVEL-1
ou
1- Find the angle of intersection of the following curves:
ks

(i) y2 =x and x2 = y [NCERT EXE-] (ii) y = x2 and x2 + y2 = 20


oo
Y
B

(iii) 2y2 = x3 and y2 = 32x (iv) x2 + y2 - 4x -1 = 0 and x2 + y2 - 2y - 9 = 0


re

(v) ^ + £=landx2 +y2


= ab (vi) x2 + 4y2 = 8 and x2 - 2y2 = 2
ou
ad
Y

(vii) x2 = 27y and y2 = 8x (viii) x2 + y2 = 2x and y2 = x


9 9
d

(ix) y = 4 - x and y = x [NCERT EXEMPLAR]


Re
n

2. Show that the following set of curves intersect orthogonally:


Fi

(i) y = x3 and 6y =7 - x2 (ii) x3 - 3xy2 = - 2 and 3x2 y -y3 = 2


(iii) x2 + 4y2 = 8 and x2 - 2y2 = 4.
3. Show that the following curves intersect orthogonally at the indicated points:
(i) x2 = 4y and 4y + x2 = 8 at (2,1) (ii) x2 = y and x3 + 6y = 7 at (1,1)
(iii) y2 = 8x and 2x2 + y2 = 10 at (1, 2 V2)
4. Show that the curves 4x = y2 and 4xy = k cut at right angles, if k2 -512.
5. Show that the curves 2x = y and 2xy = k cut at right angles, if k = 8.
[NCERT EXEMPLAR]
9 9
> 6. Prove that the curves xy = 4 and x + y = 8 touch each other. [NCERT EXEMPLAR]
9 9 9
7. Prove that the curves y = 4x and x + y - 6x +1 = 0 touch each other at the point (1, 2).
[NCERT EXEMPLAR]

ReadYourFlow.COM
TANGENTS AND NORMALS 16.41

LEVEL-2
8. Find the condition for the following set of curves to intersect orthogonally:
(i) ^--^-=1 and xy = c2 [NCERT EXEMPLAR]
*2 b

= 1 and —rr - vA= i.


(u)? b2 A2 B2
x2 x2
9. Show that the curves = 1 and 2 + = 1 intersect at right
a2 + fl + X2
angles.
x*~2 y2
10. If the straight line .rcosa + ysin a = p touches the curve then prove that
a2 b2

low
O'} O'} 9
a cos a-b sin a=p .
ANSWERS

1. (i) — and tan -1 3 (ii) tan 1 | (iii) —


^ and tan 1 -
2 4 2

ee
-1 a-b')
rF
(vii) tan -1 —
9 (viii) tan -ll -1 4V2

Fr
(v) tan (vi) tan-1 3
•Jab 13 7
8. (i) b2 = a2 (ii) a2-b2= A2+ B2
for
ou
_____________________________________ VERY SHORT ANSWER QUESTIONS (VSAOs)
ks

Answer each of the folloiving questions in one word or one sentence or as per exact requirement of the
oo

question:
Y
B

1. Find the point on the curve y = x -2x+ 3, where the tangent is parallel to x-axis.
re

2. Find the slope of the tangent to the curve x = f2 + 3t - 8, y = 2t2 - 2f - 5 at f = 2.


ou
ad

3. If the tangent line at a point (x, y) on the curve y =/ (x) is parallel to x-axis, then write the
Y

value of —.
dx
d
Re

4. Write the value of — dV , if the normal to the curve y =f (x) at (x, y) is parallel to y-axis.
n

dx
Fi

5. If the tangent to a curve at a point (x, y) is equally inclined to the coordinate axes, then write
the value of —.
dx
6. If the tangent line at a point (x, y) on the curve y =f (x) is parallel to y-axis, find the value of
dx
dV
7. Find the slope of the normal at the point Y on the curve x = - , y = f.
t
8. Write the coordinates of the point on the curve y =x where the tangent line makes an
angle ^ with x-axis.

9. Write the angle made by the tangent to the curve x = ef cos t,y = et sin f at f = —
^-----
with

the x-axis.

ReadYourFlow.COM
16.42 MATHEMATICS-XI!

7Z
10. Write the equation of the normal to the curve y = x + sin x cos xatx = —.

11. Find the coordinates of the point on the curve y = 3 - 4x where tangent is parallel to the
line 2x + y - 2 = 0.
12. Write the equation of the tangent to the curve y = x2 - x + 2 at the point where it crosses the
y-axis.
13. Write the angle between the curves y2 = 4x and x2 = 2y - 3 at the point (1, 2).
14. Write the angle between the curves y = e~x and y = ex at their point of intersection.
15. Write the slope of the normal to the curve y = — at the point ^ 3 , -^-1.

16. Write the coordinates of the point at which the tangent to the curve y = 2x2 - x + 1 is
parallel to the line y = 3x + 9.

low
17. Write the equation of the normal to the curve y = cos x at (0,1).
18. Write the equation of the tangent drawn to the curve y = sin x at the point (0, 0).
[CBSE2017]
ANSWERS

ee
2. * 1
1. (1,2) 3. 0
rF 4. 0 5. ± 1 6. 0 7. 4-

Fr
7 f2
1 1 1
8. 9. - 10. 2x = k 11. -,1
for 12. x + y - 2 = 0 13. 0
4'2 2 2
ou
14. 90° 15. 9 16- (1, 2) 17. x = 0 18. y=x
ks

MULTIPLE CHOICE QUESTIONS (MCQs)


oo

Mark the correct alternative in each of the folloioing:


Y
B

1. The equation to the normal to the curve y = sin x at (0, 0) is


re

(a) x = 0 (b) y = 0 (c) x + y = 0 (d) x - y = 0


2. The equation of the normal to the curve y = x + sin x cos x at x = tc/2 is
ou
ad

(a) x = 2 (b) X = K (c) x + 7t = 0 (d) 2 x = 7i


Y

3. The equation of the normal to the curve y = x (2 - x) at the point (2, 0) is


(a) x - 2 y = 2 (b) x - 2 y + 2 = 0 (c) 2 x + y = 4 (d) 2x + y-4 = 0
nd
Re

4. The point on the curve y = x where tangent makes 45° angle with x-axis is
Fi

(a) (1/2,1/4) (b) (1/4,1/2) (c) (4, 2) (d) (1,1)


5. If the tangent to the curve x = at2, y = 2at is perpendicular to x-axis, then its point of
contact is
(a) (a, a) (b) (0, a) (c) (0, 0) (d) (a, 0)
6. The point on the curve y = x - 3x + 2 where tangent is perpendicular to y = x is
(a) (0, 2) (b) (1,0) (c) (-1,6) (d) (2,-2)
7. The point on the curve y = x where tangent makes 45° angle with x-axis is
(a) (1/2,1/4) (b) (1/4,1/2) (c) (4, 2) (d) (1,1)
•y
8. The point on the curve y = 12x - x where the slope of the tangent is zero will be
(a) (0,0) (b) (2,16) (c) (3,9) (d) (6,36)
9. The angle between the curves y2 = x and x2 = y at (1,1) is
(a) tan 1 — (b) tan -1 3 (c) 90° (d) 45°
3 4

ReadYourFlow.COM
TANGENTS AND NORMALS 16.43

10. The equation of the normal to the curve 3x - y = 8 which is parallel to x + 3i/ = 8 is
(a) x - 3y = 8 (b) x - 3y + 8 = 0 (c) x + 3y + 8 = 0 (d) x + 3y = 0
11. The equation of tangent at those points where the curve y = x2 - 3x + 2 meets x-axis are
(a) x-y + 2 = 0=x-y-l (b) x + y-1 = 0= x- y- 2
(c) x-y-1 =0 = x-y (d) x-y = 0 =x + y
12. The slope of the tangent to the curve x =t2 + 3 f - 8, y = 2 f2 - 2 f - 5 at point (2, -1) is
(a) 22/7 (b) 6/7 (c)-6 (d) 7/6
13. At what points the slope of the tangent to the curve x2 + y2 - 2x - 3 = 0 is zero
(a) (3,0), (-1,0) (b) (3,0), (1,2) (c) (-1,0) ,(1,2) (d) (1,2), (1,-2)
7 9 ? 9 •
14. The angle of intersection of the curves xy = a and x - y =2a is
(a) 0° (b) 45° (c) 90° (d) 30°
15. If the curve ay + x2 =7 and x3 = y cut orthogonally at (1,1), then a is equal to

low
(a) 1 (b) -6 (c) 6 (d) 0
16. If the line y = x touches the curve y = x + bx + c at a point (1,1) then
(a) b =1, c = 2 (b) b=-l,c=l (c) b =2, c =1 (d) b=-2,c=l
17. The slope of the tangent to the curve x - 3t2 +l,y = t 3

ee
1 at x = 1 is
rF
Fr
(a) 1/2 (b) 0 (c) -2 (d) co
18. The curves y = aex and y = be~ x cut orthogonally, if for
(a) a=b (b) a = -b (c) ab = 1 (d) ab =2
ou
19. The equation of the normal to the curve x = cos 0, y = fl sin J 0 at the point 0 = ti/T is
ks

(a) x = 0 (b) y = 0 (c) x =y (d) x + y — a


oo

20. If the curves y = 2 cx and y = ae~ x intersect orthogonally, then a =


Y
B

(a) 1/2 (b) - 1/2 (c) 2 (d) 2t’2


re

21. The point on the curve y = 6x - x2 at which the tangent to the curve is inclined at re/4 to the
ou
ad

line x + y = 0 is
(c) 7/2,35/4)
Y

(a) (-3,-27) (b) (3,9) (d) (0, 0)


9 9
22. The angle of intersection of the parabolas y =4 ax and x = Aay at the origin is
d
Re

(a) k/6 (b) k/3 (c) rc/2 (d) ji/4


n
Fi

2 ft
23. The angle of intersection of the curves y = 2 sin x and y = cos 2 x at x = - is
6
(a) k/4 (b) k/2 (c) k/3 (d) k/6
24. Any tangent to the curve y = 2x7 + 3x + 5
(a) is parallel to x-axis (b) is parallel to y-axis
(c) makes an acute angle with x-axis (d) makes an obtuse angle with x-axis
25. The point on the curve 9y2 = x3, where the normal to the curve makes equal intercepts with
the axes is
(a) (4, ±8/3) (b) (-4,8/3) (c) (-4,-8/3) (d) (8/3,4)
26. The slope of the tangent to the curve x = f2 + 3f - 8, y = 2f2 - 2f - 5 at the point (2,-1) is
(a) 22/7 (b) 6/7 (c) 7/6 (d) -6/7
27. The line y =mx + 1 is a tangent to the curve y2 = 4x, if the value of m is
(a) 1 (b) 2 (c) 3 (d) 1/2

ReadYourFlow.COM
16.44 MATHEMATICS-XII

28. The normal at the point (1,1) on the curve 2y + .t2 = 3 is


(a) x + y = 0 (b) x - y = 0 (c) x + y + 1 = 0 (d) x - y = 1
29. The normal to the curve x = 4y passing through (1, 2) is
(a) x + y = 3 (b) x - y = 3 (c) x + y =1 (d) x-y = 1
ANSWERS
1. (c) 2. (d) 3. (a) 4. (b) 5. (c) 6. (b) 7. (b) 8. (d) 9. (b)
10. (c) 11. (b) 12. (b) 13. (d) 14. (c) 15. (c) 16. (b) 17. (b) 18. (c)
19- (c) 20. (a) 21. (b) 22. (c) 23. (c) 24. (c) 25. (a), (c) 26. (b) 27. (a)
28. (b) 29. (a)
SUMMARY
1- ^ y =/(*), then
' d\j
= Slope of the tangent to y = / (x) at point P.

low
\dxjp
-1
= Slope of the normal to y =/ (x) at point P.
d!/)
dxjp

ee
dy
rF
Fr
If the tangent is parallel to x-axis, then -2- = 0.
dx
dx for
If the tangent is parallel to y-axis, then — = 0
dy
ou
2. If P (x-j, y-]) is a point on the curve y = f (x), then
ks

dy
oo

y-.vi = (x - x^ is the equation of tangent at P.


dx Jp
Y
B

1
y-y\ = - (x - xq) is the equation of the normal at P.
re
ou
ad

3. The angle between the tangents to two given curves at their point of intersection is defined
Y

as the angle of intersection of two curves.


nd
Re

If C} and C2 are two curves having equations y = /(x) and y =g(x) respectively such that
they intersect at point P. The angle 0 of intersection of these two curves is given by
Fi

'dy) Jdy]
,dx)Cl v dx j C2
tan 0 =
(dy) (dy'
1 +
\ dx /C1 {dx)C2

If the angle of intersection of two curves is a right angle,, then the curves are said to intersect
orthogonally. The condition for orthogonality of two curves C} and C2 is
(4l) dx)c2
= -1
v dx Cl

4. Two curves ax2 + by2 = 1 and a' x2 +b' y2 =1 will intersect orthogonally, if

1 _I - l_i
a b ci' b'

ReadYourFlow.COM
CHAPTER
INCREASING AND
DECREASING FUNCTIONS
17.1 INTRODUCTION
In this chapter, we shall study monotonicity of functions. A function / (x) is said to be a mono-
tonically increasing function on[r,', b], if the values of / {x) increase or decrease with the increase
or decrease in x. If the values of f (x) decrease with the increase in the values of x, then / (x) is
said to be a monotonically decreasing function. The monotonicity of functions in [a,b] is

w
strongly connected to the sign of its derivative in [a, b]. The relation between the two will be
discused in section 17.4. In detern iining the intervals of monotonicity of a function in i ts domain,

Flo
we shall be solving the inequations /'(x)>0 and /'(x)<0. So, we shall first discuss the
procedure of solving inequations, in the following section.

ree
17.2 SOLUTION OF RATIONAL ALGEBRAIC INEQUATIONS

F
The following results are very useful in solving rational algebraic inequations:
(i) ab >0 =$ {a>0 and b > '0) or(a < 0 and i; < 0) or
ur
(ii) ab <0 => (fl > 0 and b < 0) or(« < 0 and b >0)
f
(hi) ab> 0 and a>0 => b >0
ks
Yo

(iv) ab <0 and a <0 => b >0


oo

PW>0 Pi*) ,Q p(*) > 0 and


If P(x)andQ(x) are polynomials, then the inequations
B

Q(x) ' Q(x) Q (x)


re

P(x)
< 0 are known as rational algebraic inequations. These inequations can be solved by using
QM
ou
ad

the following algorithm.


Y

ALGORITHM
nd
Re

STEP I Factorize P (x) and Q (x) into linearfactors.


Fi

STEP II Make coefficient ofx positive in all factors.


STEP III Equate all thefactors to zero and find the corresponding values ofx. These values are generally
known as critical points.
STEP IV Plot the critical points on the number line. Note that n critical points will divide the number line
in (« +1) regions.
STEP V In the right most region, the expression will be positive and in other regions it will be
alternatively negative ana' positive. So, mark positive sign in the right most region and than
mark alternatively negative and positive signs in the remaining regions.
STEP VI Obtain the solution set of .the given inequation by selecting the appropriate regions in step V.
Following illustrations will illustrate the above algorithm.
ILLUSTRATION l Solve: 4x3 - 24 x2 + 44x - 24 > 0.
SOLUTION We have,
4x3 - 24x2 + 44x - 24 > 0
=> 4 (x3 - 6x2 +11 x - 6) > 0
=> x3-6x2 + llx - 6 > 0 [v 4 > 0 and ab > 0, a > 0 => b >0]

ReadYourFlow.COM
17.2 MATHEMATICS-XII

=> (* -1) (x1 -5x + 6) > 0


=> (x-l)(x-2) (a:-3) >0 ...G)
On equating all factors, on LHS of the inequation, to zero, we obtain x=l, 2, 3 as critical points.
Let us plot these critical points on the number line as shown in Fig. 17.1. These points divide the
number line into four regions. In the right most region the expression on LHS of (i) bears
positive sign and then alternatively negative and positive signs as marked in Fig. 17.1. Since the
expression in (i) is positive. Therefore, solution set of inequation (i) is the union of the regions
marked with + signs. Hence from Fig. 17.1, we obtain (1, 2) u (3, oo) as the solution set.
i.e. *3-6x2+11x-6>0 => (x-1) (x-2) (*- 3) > 0 => * e(l, 2) u(3, <»).
Hence, the solution set of the given inequality is (1, 2) u (3, cc).

w
+ +
- 00
1 2 3 00

Fig. 17.1 Signs of (.v-l)(.r-2) (.v-3) for different values of.r

Flo
ee
ILLUSTRATION 2 Solve: -—- >0, x * -1, - 2.
*+1 (2 + x)

Fr
SOLUTION We have,
(2 + x)2 - 4 (x + 1)
for
ur
1 4 x2
x +1 (2 + x)2 (2 + x)2 (x + 1) (2 + x)2 (x + 1)
ks

1
Yo
oo

* + l (2 + x)2
B

x2
>0
re

(2 + x)2 (x + 1)
\2 /
ou
ad

X 1
=> >0
Y

2 + x /J x + 1 /
\2
nd
Re

1 X
=> > 0 and x * 0 > 0 and if fl > 0, then ab >0=> b >0
x+1 {2 + xj
Fi

=> x + 1 > 0 and x * 0 •/ - > 0 and a > 0, => >0


b
=> x > -1 and x ^ 0
x e (— 1, 0) w(0, co)
Hence, the solution set of the given inequality is (-1, 0) u (0, cc).
1-x2
ILLUSTRATION 3 Solve:
5x - 6 - x 2<°
SOLUTION We have,
1-x2
2 <0
5x - 6 - x

«> -(*2-u <0


-(x2 -5x + 6)

ReadYourFlow.COM
INCREASING AND DECREASING FUNCTIONS 17.3
+ + +
*2-l
<=> <0
- CO
-1 1 2 3 CO

x2 -5x + 6
Fig. 17.2 Signs of (*-!)(*+ 1) for different values of x
(x-2)(x-3)
^ (.t-1) (x + 1)
<0
(x-2) (x-3)
Equating all the factors to zero, we obtain x = 1, -1, 2, 3 as the critical points.
Now, we plot these points on the number line as show in Fig. 17.2. These points divide the
number line into 5 regions. In the right most region the expression in (i) bears '+' sign and in the
other regions the expression bears alternate negative and positive signs as shown in Fig. 17.2.
Since the expression in (i) is negative, so solution set of the given inequation is the union of
regions containing negative signs. Hence, from Fig. 17.2, we get x e (-1,1) u (2, 3)
1-x2

w
i.e. 2 < 0 => X€(-l,l)u(2, 3)
5x - 6 - x
8x2 +16X-51
ILLUSTRATION 4 Solve:
2x2 +5x-12

Flo>3.

ee
SOLUTION We have,

Fr
+ + +
8x2 +16x-51
>3
- CO
-4 -3 1 5 CO

2x2 +5x-12 2 2
or
ur
(x+ 3)(2x -5)
Fig. 17.3 Signs of for different values of x
8x2 + 16x -51
f
(x+ 4)(2x- 3)
<=> -3>0
ks

2x2 +5x -12


Yo
oo

8x2 + 16x -51 - 6x2 - 15x + 36


>0
B

2x2 +5x-12
re

2x2 + X-15
>0
ou

2x2 + 5x -12
ad
Y

2x2 + 6x -5x -15


>0
2x2 + 8x - 3x -12
nd
Re

(x+ 3) (2x -5)


>0 -(U
Fi

(x + 4) (2x - 3)
Equating all factors to zero, we obtain x = - 4, - 3, 3/2,5/2
Now, we plot these points on the number line as shown in Fig. 17.3. These points divide the
number line into five regions. In the right most region the expression in (i) bears positive sign
and in all other regions it bears alternate negative and positive signs as shown in Fig. 17.3.
Since the expression in (i) is positive, so the solution set of the given inequation is the union of
regions containing'+' signs. Hence, from Fig 17.3, we get x e (- oo, - 4) u (- 3, 3/2) u (5/2, <x>).
8x2 +16x-51
i.e. > 3 => xe(-oo/-4)u(-3, 3/2) w(5/2,oo).
2x2 +5x-12
2 - 2x + 5 1
ILLUSTRATIONS Solve: ——------------> -.
SOLUTION We have, 3x2 - 2x -5 2
x2 - 2x + 5 1
3x2 - 2x -5 2

ReadYourFlow.COM
17.4 MATHEMATICS-XII

x2 -2x + 5
->0
3x2 -2x-5 2
2(x2 -2x + 5) -(3x2 - 2x - 5)
<=> >0
2 (3x2 -2x-5)
-x2-2x + 15 + + +
<=> >0
2(3x2-2x-5) - CO
-5 -1 5 3 00

3
-(x2 + 2x -15)
<=> >0 (ar + 5)(A:- 3)
Fig. 17.4 Signs of for different values of x
2 (3a:2 -2a:-5) (a:+ l)(3x-5)
x2 + 2a:-15
o <0 ,-l>o=>-Z<o
2 (3a:2 - 2a: -5)

w
a:2 + 2a:-15
>0 v->0
3x2 -2x-5 2

Flo
(x + 5) (a: - 3)
<0 ...(i)
{x + 1) (3a: -5)

ee
On equating all factors to zero, we get a: = -5, -1,5/3, 3. Plotting these points on number line

Fr
and marking alternatively '+' and signs, we obtain as shown in Fig. 17.4.
Since the expression in (i) is negative, so the solution set of the given inequation is the union of
for
ur
regions marked with signs. Hence, from Fig. 17.4, we get x e (-5, -1) u (5/3, 3).
x2 -2x + 5 1
ks

i.e. —^------------ >-=> * e(-5, -1) u(5/3, 3).


3at - 2x -5 2
Yo
oo

2 - 2a: + 24
ILLUSTRATION 6 Solve: - <4.
B

a:2 - 3x + 4
SOLUTION We have,
re

a:2 - 2x + 24
<4
ou
ad

+ +
x2 -3x + 4
- CO 2
Y

4 CO

a:2 - 2a- + 24 3
<=> -4 <0
d

Fig. 17.5 Signs of (3x + 2) (x - 4) for different values of x.


Re

a2 - 3a + 4
n
Fi

(a2 - 2a + 24) - 4 (a2 - 3a + 4)


<=> <0
a2 - 3a + 4

- 3a2 + 10a + 8
4=> <0
A2 - 3x + 4

3a2 -10a-8
>0 v — < 0 => — > 0
a2 - 3a + 4
(3a + 2) (a - 4) ^ Q
<=>
(a2 - 3a + 4)
9 . 9
v Disc, of a - 3a + 4 is - ve and coeff. of az is + ve
<=> (3a+ 2) (a - 4) > 0
a2 - 3a + 4 > 0 for all a
2
<=> A <- — or a > 4 [See Fig. 17.5]
3

ReadYourFlow.COM
INCREASING AND DECREASING FUNCTIONS 17.5

<=> x e (- co, - 2/ 3] u [4, co)


2 - 2x + 24
Thus, — <4 X€(-co,-2/3u [4,00).
x2 - 3x + 4
_ , x 2 -4x + 7 ^2
ILLUSTRATION 7 Solve: —
x2 -7x + 12 ~ 3‘
SOLUTION We have, -
x2 - 4x + 7 ^2 + + +
- CO -3 1 3 4 CO

x2 -7x + 12 _ 3
(*+ 3)(x-l) for different values of x
x2 - 4x + 7 Fig. 17.6 Signs of
-<0
(*-3)(x-4)
x2 -7x + 12 3
3 (x2 - 4x + 7) - 2 (x2 - 7x + 12)
<=>
x2 -7x +12

w
x2 + 2x - 3 (x + 3) (x-1)
<=> <0 o <0
x2 -7x +12 (x - 3) (x - 4)

Flo
On equating all factors in (i) to zero, we get x = - 3,1, 3, 4 as critical points. Plotting these points

ee
on the number line and marking alternatively '+' and signs from the right most side, we

Fr
obtain that the inequation in (i) has the signs as shown in Fig. 17.6.
Since the expression in (i) is negative, so the solution set of the given in, equation is the union of
for
the regions marked with signs. Hence, from Fig 17.6, we get x € [- 3,1] u (3, 4).
ur
x2 - 4x + 7 <-| => xe[-3,l]e(3,4).
i.e.
ks

x2 -7x +12
Yo
oo

It should be noted that 3 and 4 are not included, because denominator becomes zero at x = 3 and
x = 4.
B
re

17.3 SOME DEFINITIONS


STRICTLY INCREASING FUNCTION A function f{x) is said to be a strictly increasingfunction on (a, b), if
ou
ad

x\<x2 => /(*l) < /(*2) for al1 XV x2 e (a' b)


Y

Thus, /(x) is strictly increasing on (a, b) if the values of f(x) increase with the increase in the
values of x.
nd
Re

Graphically,/(x) is increasing on (a, b) if the graph y =/(x) moves up as x moves to the right. The
Fi

graph in Fig. 17.7 is the graph of a strictly increasing function on {a, b).
Y

y =/(*)

M) i f{x2)

o a *1 x2 b X

Fig. 17.7 Graph of an increasing function

ReadYourFlow.COM
17.6 MATHEMATICS-XII

ILLUSTRATION 1 Show that thefunction f(x) =2x + 3 is strictly increasing function on R.


SOLUTION Let x^, x2 eR and let x^ <x2- Then,
X1 < x2 => 2xl < 2x2 ^ 2x1 + 3 < 2x2 + 3 fixf) < f(x2)
Thus, x1 <x2=> /(.T|) <f(x2) for all xlf x2 e R. So,/(x) is strictly increasing function on R.
This result is also evident from the graph of the function shown in Fig. 17.8.
Y

x"5
f(xi)
v:

w
X' X
o 1X 2^

Flo
ee
r

Fr
Fig. 17.8 Graph of f(x) = 2x+ 3
for
ur
ILLUSTRATION 2 Show that the function f(x) = x2 is strictly increasing function on [0, co).
SOLUTION Let x2, x2 e [0, co) such that x1<x2. Then,
ks

2
x-i<x2 => Xj < x2 x2 [Multiplying both sides by x-J
Yo
oo

again, x-i<x2 => xl x2 <x22 [Multiplying both sides by x2] ...(h)


B

From (i) and (ii), we get


re

X1 < x2 => x\2 <x2 => f(x\)<f(x2)


ou
ad

Thus, Xi < x2 => f(xi) <f(x2) for all x1, x2 e [0, oo).
Y

Hence,/(x) is strictly increasing function on [0, co) which is evident from the graph also.
d

Y
Re
n

fM = x2
Fi

o Axd
X' x\ x2 X
Y'
Fig. 17.9 Graph of/(;r) = x2,x>0

ILLUSTRATION 3 Show that the function /(x) = ax, a>l is strictly increasing on R.
SOLUTION Let xl7 x2 g R such that Xj < x2. Then,
xl < x2
=> ax' <ax2 [v a>\ .-. X2 <x2 => <ax2]

ReadYourFlow.COM
INCREASING AND DECREASING FUNCTIONS 17.7

=> /(xj) </(x2)


Thus, x1<x2=> fix-]) < f(x2) for all x-j, x2 e R.
Hence, /(x) is strictly increasing function on R. This fact is also exhibited in the graph of this
function as shown in Fig. 17.10.
Y

f(x) = ax,a > 1

/U'2)
/(^l)

X' o X2 X

low
r
Fig. 17.10 Graph of f(x) = ax,a>l

ee
rF
REMARK In the above example if we replace a by e (~ 2.71), then we find that f(x) =e: is also

Fr
increasing on R.
STRICTLY DECREASING FUNCTION A function f(x) is said to be a strictly decreasing function on
for
(a, b), if
ou
xl<x2 => /(X|) > /(x2) for all xlr x2 e (a, b)
ks

Thus, /(x) is strictly decreasing on (a, b) if the values of f(x) decrease with the increase in the
oo

values of x.
Y
B

Graphically it means that /(x) is a decreasing function on (a, b) if its graph moves down as x
re

moves to the right. The graph in Fig. 17.11 is the graph of a strictly decreasing function.
ou

Y
ad
Y
nd
Re
Fi

Axi)\ 'f(x2i
a O X
X' xi x2 b

Y'
Fig. 17.11 Graph of a decreasing function

ILLUSTRATION 4 Show that thefunction f(x) = - 3x + 12 is strictly decreasing function on R.


SOLUTION Let xlr x2 e R be such that xl < x2. Then,
Xi < x2 => - 3x1 > - 3x2 => - 3x1+12>- 3x2 + 12 /(x-j) >/(x2)
Thus, Xi<x2=> /(x-j) >/(x2) for all X!, x2 g R.
So, /(x) is strictly decreasing function on R.
This fact can also be observed from the graph of the function as shown in Fig. 17.12

ReadYourFlow.COM
17.8 MATHEMATICS-XII

Ax2)
f(h)
X' X
xl o x2

r
Fig. 17.12 Graph of/(A-) = -3.t+ 12

low
ILLUSTRATION 5 Show that thefunction f(x) = ax, 0 < a <1 is strictly decreasing on R.
SOLUTION Let X], „\'2 e R such that x-j < jv^- Then,
*l < x2

ee
=> ax\ >ax2 [ •.• 0 < a < 1 .\x1< x2 => a*1 > ax2]
rF
Fr
=> f{x\) > /(x2)
Thus,Xj < x2 => f{x{) > f{x2) for all x2, g R. for
Hence,/(x) is strictly decreasing function on R. This is also evident from the graph of f{x) as
ou
shown in Fig. 17.13.
ks

Y
oo
Y
B

/(.v) = ax, 0 : a < 1


re
ou
ad

Axt)
Ax\)
Y

O x2
d

X' xi X
Re
n
Fi

r
Fig. 17.13 Graph otf(x) = ax,0<a<l

REMARK Since 0 < e -1 = - < L therefore f(x) = {e~l)x = e~x is also a strictly decreasingfunction on R.
ILLUSTRATION 6 Shoiv that the function f(x) = x2 is a strictly decreasing function on(- oo, 0].
SOLUTION Let x^, x2 g (- co, 0] be such that x2 < x2. Then,
“ 2
Xl <X2 => Xl > Xl x2
2
and. xl < x2 => Xl x2 > x2 ...(ii)
From (i) and (ii), we obtain
X1 < x2 => xi 2 > x2 2 => f(xl)>f(x2)
Thus, Xi < x2 => f(xi) > f(x2) for all x2 g (- oo, 0].
Hence,/(x) is strictly decreasing on (- oo, 0]. See also Fig. 17.14

ReadYourFlow.COM
INCREASING AND DECREASING FUNCTIONS 17.9
r

X' *1 *2 o X

r
Fig. 17.14 Graph of f(x) = x2,x <0

w
Uptill now, we have been discussing about a strictly increasing or strictly decreasing functions.
But, it is possible that a function may neither be strictly increasing nor strictly decreasing on a

Flo
given interval. For example,/(x) in Fig. 17.15 is neither strictly increasing nor strictly decreasing
on (a, b). However, it is increasing on the sub-intervals (a, a-[), #3) and (a4, b) and decreasing

ee
on the intervals (fl1, fl2) and (a3, a4).

Fr
Y
for
ur
ks
Yo

X'
oo

a flj O a2 b
B

«3
re
ou
ad

Y
Y

Fig. 17.15
ILLUSTRATION 7 Show that thefunction f(x) = x2 is neither strictly increasing nor strictly decreasing
nd
Re

on R.
Fi

SOLUTION In illustrations 3 and 6 we have seen that/(x) =x is strictly increasing on [0, c©) and
strictly decreasing on (- 00, 0]. Hence, it is neither strictly increasing nor strictly decreasing on R
i.e. (- oo, 00).

o a a\ a2 b X
Fig. 17.16

ReadYourFlow.COM
17.10 MATHEMATICS-XII

Uptill now we were talking about strictly increasing and strictly decreasing functions. But, there
can be functions which are increasing (decreasing) but not strictly increasing (decreasing). For
example, consider the function whose graph is shown in Fig. 17.16. Clearly,/(x) is increasing on
(a, b) but it is strictly increasing only in the intervals (a, fl|) and (fl2/ b). In this chapter, we shall be
studying only strictly increasing and strictly decreasing function.
NOTE From now onwards, by an increasing or a decreasingfund ion we shall mean a strictly increasing
or a strictly decreasing function.
MONOTONIC FUNCTION A function f{x) is said to be monotonic on an interval (a, b) if it is either
increasing or decreasing on (a, b).
DEFINITION A function f(x) is said to be increasing (decreasing) at a point x0 if there is an interval
(x0 -h, Xq + h) containing x0 such that f(x) is increasing (decreasing) on (x0 - h, x0 + K).
DEFINITION A function f(x) is said to be increasing on [a, b] if it is increasing (decreasing) on (a, b) and

w
it is also increasing (decreasing) at x = a and x = b.
EXERCISE 17.1

Flo LEVEL-1

ee
1. Prove that the function/(x) = loge x is increasing on (0, oo).

Fr
2. Prove that the function/(x) = log,, x is increasing on (0, co) if a > 1 and decreasing on (0, oo),
if 0 < a < 1. for
ur
3. Prove that f(x) =ax + b, where a, b are constants and a > 0 is an increasing function on R.
4. Prove that f(x) = ax + b, where a, b are constants and a < 0 is a decreasing function on R.
ks
Yo

5. Show that/(x) = — is a decreasing function on (0, co).


oo
B

1
6. Show that/(x) = - decreases in the interval [0, oo) and increases in the interval (- oo, 0].
re

1+x
ou

7. Show that /(x) = r- is neither increasing nor decreasing on R.


ad

1 + x2
Y

8. Without using the derivative, show that the function / (x) = | x | is


nd
Re

(a) strictly increasing in (0, co) (b) strictly decreasing in ( - co, 0).
9. Without using the derivative show that the function / (x) = 7x - 3 is strictly increasing
Fi

function on R.
HINTS TO NCERT & SELECTED PROBLEMS
1. For any xl7 x2 e (0, co), we have
xa < x2 => log, X! < log,, x2 => /(*l) </(*2)-
So, / (x) is increasing on (0, °o).
2. CASE I When a>l: For any xl7 x2 e (0, co)
xa > x2 => logfl Xi > logfl x2 => fixf) >/(x2).
So,/(x) is increasing on (0, oo).
CASE II When fl < 1: For any Xj, x2 e (0, oo)
x1>x2=> logfl xt < logjj x2 => fix-i) </(x2).
So,/(x) is decreasing on (0, oo)

ReadYourFlow.COM
INCREASING AND DECREASING FUNCTIONS 17.11

17.4 NECESSARY AND SUFFICIENT CONDITIONS FOR MONOTONICITY


In this section, we intend to see how we can use derivative of a function to determine where it is
increasing and where it is decreasing.
THEOREM 1 (Necessary Condition) Let f (x) be continuous on [a, b] and differentiable on (a, b).
(i) Iff (x) is strictly increasing on (a, b), then f' (x) > Ofor all x e (a, b).
(ii) Iff (x) is strictly decreasing on (a, b), then f' (x) <0 for all x e(a, b).
PROOF Let x be an arbitrary point in (a, b). Since f(x) is differentiable on (a, b). So, it is
differentiable at x.
f(x + h)-f(x)
f'(x) = lim ,h>0 exists.
/f->0 h
(i) If / (x) is strictly increasing on (a, b), then
f (x + h) >f (x) for all h>0

w
f(x + h)-f(x) . ...
=> - > 0 for all /7 > 0
h

Flo
f(x + h)-f(x)
=> lim >0
h-+0 h

ee
/'(*)> o.

Fr
Since x is an arbitrary point of (a, b). Therefore, /' (x) > 0 for all x e (a,b).
(ii) If / (x) is strictly decreasing on (a, b), then
for
ur
f (x + h) <f (x) for all Iz > 0
ks

/ (x + h)-f (x)
< 0 for all /z > 0
Yo

h
oo

f(x + h)-f(x)
B

=> lim <0


h
re

h^O
=> /'(*)< 0
ou
ad

Since x is an arbitrary point of (a, b). Therefore, /' (x) < 0 for all x e (a, b).
Y

Q.E.D.
REMARK Iff (x) is an increasing function on (a, b), then as shozvn in Fig. 17.17, the tangent at every
nd
Re

point on the curve y = f (x) makes an acute angle 0 with the positive direction of x-axis.
Fi

Y Y

O / a b X O a b \ x

Fig. 17.17 Fig. 17.18


dy
tan 9 > 0 => >0 or, /'(x)>0 for all x e(a,b)
dx

ReadYourFlow.COM
17.12 MATHEMATICS-XII

Iff (x) is a decreasing function on (a, b), then as shown in Fig. 17.18, the tangent at every point on the
curve y = / (x) makes an obtuse angle 0 with x-axis.
dy
tan 0 < 0 => <0 or, f'(x)<0 for all x e (a, b).
dx
THEOREM 2 (Sufficient Condition) Let f be a differentiable real function defined on an open interval
(a,b).
(i) Iff'(x)> Ofor all x e (a, b), then f(x) is increasing on (a, b).
(ii) Iff' (x) < Ofor all x e (a, b), then f(x) is decreasing on (a, b).
PROOF Let x-j, x2 e.(a,b) such that Xj <x2. Consider the sub-interval [xlr x2] . Since /(x) is
differentiable on (a,b) and [xl7 x2] c (a, fc). Therefore, /(x) is continuous on [x1, x2] and
differentiable on (x-j, x2). By the Lagrange's mean value theorem, there exists c eCxj, x2) such

w
that
f'(c) = /(x2) -f(x{)

Flo
X2-Xi

(i) Since /' (x) > 0 for all x e (a, b), so in particular, f' (c) >0.

ee
Fr
Now, /'(c)>0

=>
/(x2) — f(xf) >0 for [Using (i)]
ur
X2-Xi

=> /(x2) ~f(xi) > 0 [•.• x2 - xx > 0 when Xj < x2]


ks
Yo

=> /(x2)>/(x1)^/(Xi)</(X2)
oo

Since Xj, x2 are arbitrary points in (a, b). Therefore,


B
re

x1 < x2 => /(x-j) < f{x2) ^or a" *1/ x2 e (a'


Hence,/(x) is increasing on (a, b).
ou
ad

(ii) Since / '(x) < 0 for all x e (a, b), so in particular, / '(c) < 0.
Y

Now, /'(c) < 0


nd
Re

/(x2) -/(X!)
=> < 0 [Using (i)]
Fi

x2~xl

=> f(x2) -f(x\) < 0 [•.• x2 - Xj > 0 when Xj < x2]


=> / (*2) < f(xl) => f(xl) > f(x2)
Since x1, x2 are arbitrary points in (a, b). Therefore,
Xi c x2 => /(x:) >/(x2) for all xl7 x2 e(a, b).
Hence, /(x) is decreasing on (a, b).
COROLLARY Let f(x) be a function defined on (a, b).
(i) If f '(x)> 0 for all x e (a, b) except for a finite number of points, where f'(x)= 0, then f(x) is
increasing on ((a, b).
(ii) If f'(x) <0 for all x e (a, b) except for a finite number of points, where / '(x) = 0, then f(x) is
decreasing on (a, b).
In order to find the interval in which a given function is increasing or decreasing, we may use
the following algorithm.

ReadYourFlow.COM
INCREASING AND DECREASING FUNCTIONS 17.13

ALGORITHM
STEP I Obtain the function and put it equal to f (x).
STEP H Findf'(x).
STEP HI Put f '(x) > 0 and solve this inequation.
For the values of x obtained in step III f(x) is increasing and for the remaining points in its
domain it is decreasing.

ILLUSTRATIVE EXAMPLES
LEVEL-1
Type I ON FINDING THE INTERVALS IN WHICH A FUNCTION IS INCREASING OR DECREASING
9 _
EXAMPLE l Find the intervals in which f(x) = - x-2x +15 is increasing or decreasing.

w
SOLUTION We have,

=>
f(x)=-x2 -2x + 15
f'(x) = -2x - 2 = - 2(x + 1)
Flo
ee
For f(x) to be increasing, we must have

Fr
/'(*) >0
=> -2(x + 1) > 0 or
ur
=> x + l <0 [v -2 <0 andab >0, a <0 => b <0]
sf
=> x < -1 => x e (- oo, -1)
Thus, /(x) is increasing on the interval (- co, -1).
k
Yo
oo

For /(x) to be decreasing, we must have


B

/'(x) < 0
-2(x + l) <0
re

=> x + l >0 [v - 2 < 0 and ab <0, a <0 => b >0]


ou
ad

=> x>-l=> xe(-l,co)


Y

So, /(x) is decreasing on (-1, oo).


EXAMPLE 2 Find the intervals in which the function /(x) = 2x3 - 9x2 + 12x + 15 is (i) increasing,
nd
Re

(ii) decreasing: [CBSE 2010,2011]


Fi

SOLUTION We have,
/(x)=2x3 -9x2 +12x + 15
/'(x) = 6x2 -18x + 12 = 6 (x2 - 3x + 2)
(i) For/(x) to be increasing, we must have
/'(*) >0
6 (x2 - 3x + 2) > 0
=> x2 - 3x + 2 > 0 [•.• 6 > 0 6(x2 - 3x + 2) > 0 => x2 - 3x + 2 > 0]
=> (x — 1) (x 2) > 0
x < 1 or x > 2 [See Fig. 17.19]
=> x e(-oo, 1) u(2, co).
So, /(x) is increasing on (- oo, 1) u (2, oo).
+
- CO
1 2 CO

Fig. 17.19 Signs of f'(x) for different values of .r

ReadYourFlow.COM
17.20 MATHEMATICS-XII

^ T X 4 + X 2 +1i
=> x2 -1 <0 < 0
x4
(x-l)(.r + l) <0
=> x e(-l, 0) u(0,1) [See Fig. 17.34] [v x ^ 0]
Hence, / (x) is decreasing an (-1, 0) u (0,1).
o
- 00 -1 0 1 00

Fig. 17.34 Signs of/'(*) for different values of x

X
EXAMPLE 14 For which values ofx, thefunction f(x) = —---- is increasing and for which values ofx,
it is decreasing. x2 +1
SOLUTION We have.
/W=-2^-

w
X +1
(x2 +1) X 1 - X (2x + 0) 1-x2
=> /'(*) =
(x2 + l)2

Flo (x2+l)2‘

ee
For/(x) to be increasing, we must have

Fr
/'(x) > 0
1-x2
for
ur
(X2 +1) 2>°
1 - x2 > 0 [v (x2 +1)2 > 0]
ks

=>
Yo
oo

-(x2 -1) >0 + +


B

=> x2 -1 <0 - 00 -l l oo
re

Fig. 17.35 Signs of/'(x) for different values ofx


=> (x-l)(x + l) <0
=> -1 < x < 1 [See Fig. 17.35]
ou
ad

=> xe(-l,l)
Y

So,/(x) is increasing on (-1,1).


nd

For/(x) to be decreasing, we must have


Re

/'(x) < 0
Fi

1-x2
=4>
(X2 +1) 2<°
1 - x2 < 0 [v (x2 +1)2 > 0]
=> -(x2 -1) < 0 + +
- oo oo
x2 -1 >0 -1
=> Fig. 17.36 Signs of/'(j:) for different values ofx
=> (x — 1) (x + 1) > 0
=> x < -1 or x > 1 [See Fig. 17.36]
So, /(x) is decreasing on (- oo, -1) u (1, co).
EXAMPLE 15 Find the intervals in which f (x) = 2 log (x - 2) - x2 + 4x +1 is increasing or
decreasing.
SOLUTION Clearly, / (x) is defined for all x > 2.
Now, / (x) =2 log (x - 2) - x2 + 4x + 1

ReadYourFlow.COM
INCREASING AND DECREASING FUNCTIONS 17.21

2-2x(x-2) + 4(x-2) -2x2 + 8x-6


=> fix) = — - 2x + 4 =
x-2 x-2 x-2
-2(x2 + 4x- 3) -2(x-l) (x-3)
=> f'(x) =
x-2 x-2
For / (x) to be increasing, we must have
f'(x) > 0
-2(x-l)(x-3)
=> > 0
x-2
(x-1) (x-3)
=> < 0
x-2
=> x - 3 <0 [v x e Domain (/)=> x>2=> x-l>0 and, x - 2 > 0]
=> x <3 => x e (2, 3) [v x > 2]

w
So, / (x) is increasing on (2, 3).
For / (x) to be decreasing, we must have
/'(*) < 0
-2(x-l) (x-3)

Flo
ee
< 0
x-2

Fr
(x-1) (x-3)
=> > 0
x-2
for
ur
=> x-3>0 [v For x>2,x-2>0 and x -1 > 0]
=> x>3
ks

=> x e (3, co)


Yo
oo

So,/ (x) is decreasing on (3, co).


B

EXAMPLE 16 Separate [0, iz/2] into subintervals in which f(x) = sin 3x is increasing or decreasing.
re

[NCERT]
SOLUTION We have,/(x) = sin 3x
ou
ad

/'(x) = 3cos3x
Y

Now, 0 <x <ti/2=> 0 < 3x < 37t/2.


Since cosine function is positive in first quadrant and negative in the second and third
nd
Re

quadrants. Therefore, we consider the following cases.


Fi

CASE I VWzen 0 < 3x <7t/2i.e. 0 <x < ti/6


In this case, we have
0 < 3x < ti/2 cos 3x > 0 => 3 cos 3x > 0 => / '(x) > 0
Thus,/ '(x) > 0 for 0 < 3x < n/2 i.e. 0 < x < ji/6.
So,/(x) is increasing on (0,71/6).
CASE II When n/2 <3x< 3n/2 i.e. n/6<x<n/2
In this case, we have
n/2 < 3x < 3n/2 ^ cos 3x < 0 => 3 cos 3x < 0 => / '(x) < 0
Thus, /'(x) < 0 for tt/2 < 3x < 3tc/2 i.e. n/6 <x <n/2.
So,/(x) is decreasing on(n/6, n/2).
Hence,/(x) is increasing on (0, n/6) and decreasing on (n/6, n/2).
EXAMPLE 17 Find the intervals in zvhich the function fgiven by
4 sin x - 2x - x cos x
fix) = , 0 < x < 2 71 [NCERT]
2 + cos x
is (i) increasing (ii) decreasing

ReadYourFlow.COM
17.22 MATHEMATICS-XII

SOLUTION We have.
4 sin x - 2x - x cos x
fix) =
2 + cos x
(2 + cos x) (4 cos x - 2 - cos x + x sin x) + (4 sin x - 2x - x cos x) sin x
=> /'(*) =
(2 + cos x)2
cos x (4 - cos x)
=> /'(*) =
(2 + cos x)2

(i) For / (x) to be increasing, we must have


/' (x) > 0
cos x (4 - cos x)
=> > 0
(2 + cos x)2
4 - cos x
=> cos x > 0 > 0
(2 + cos x)2

low
=> x e(0,7t/2) u(3Tt/2,2rc)
Hence, / (x) is increasing on (0 , tc/2) u (3Tt/2,2k).
(ii) For / (x) to be decreasing, we must have
/'(*)< 0

ee
cos x (4 - cos x)
=> < 0
rF
Fr
(2 + cos x)2
=> cos x < 0 for
x e (k/2, 3k/2)
ou
Hence, / (x) is decreasing on (k/2, 3k/2).
ks

EXAMPLE 18 Separate the interval [0, k/2] into sub-intervals in which f (x) = sin4 x + cos4 x is
oo

increasing or decreasing. [CBSE2000C]


Y
B

SOLUTION We have.
re

/ (x) = sin4 x + cos'*4 x


ou
ad

=> / '(x) = 4 sin x cos x - 4 cos x sin x


Y

=> f'(x) = - 4 sin x cos x (cos x - sin x)


=> / '(x) = - 2 (2 sin x cos x) (cos 2x)
nd
Re

=> f'(x) = - 2 sin 2x cos 2x


Fi

=> /'(x) = -sin 4x


We have , 0 < x < k/2 => 0 < 4x < 2k.
Since sine function is positive in the first and second quadrants and negative in the third and
fourth quadrants. So, we consider the following:
CASE I When 0 < 4x < k i.e. 0 < x < k/4
In this case, we have
sin 4x > 0 [v 0 < 4x < k]
- sin 4x < 0
=> fix) < 0
/'(x) < OforO <4x<Ki.e. 0<x<k/4.
So, / (x) is decreasing on [0, k/4].
CASE II When, k < 4x < 2k i.e. k/4 < x < k/2
In this case, we have
sin 4x <0

ReadYourFlow.COM
INCREASING AND DECREASING FUNCTIONS 17.23

=> - sin 4x > 0


=> /'(*) > 0
/' (x) > 0 for tc < 4>r < 2n i.e. tt/4 < x < n/2.
So, / (x) is increasing on [n/4, ti/2].
EXAMPLE 19 Find the intervals in which the function fgiven by f (x) = sin x + cos x , 0 <x <2 nis
increasing or decreasing. [NCERT, CBSE 2009,20171
SOLUTION We have,
/ (x) = sin x + cos x
=> /' (x) = cos x - sin x = V2( cos x sin - - sin x cosj = V2 sin ^-xj = -V2sin
4
7T „ n K k 7n
Now, 0<x<2tc=> 0--<x- - <271-- => — < x — < —
4 4 4 4 4 4
For / (x) to be increasing, we must have

w
/'(*)> 0
- V2 sin x - — > 0

Flo
4

ee
=> sin x - — < 0
4

Fr
K 7t _ n 7n
=> ---- < x- -<0or, ti:<x — < —
4 4 4 4 for
ur
=> 0 < x < - or, — < x < 271
4 4
ks

St:
=> or, x e —, 2k
Yo

4
oo

=> x e (0 , ti:/4) u (5k/4,2 tt)


B

Hence, / (x) is increasing on (0 , k/4) u (5k/4,2ti).


re

For / (x) to be decreasing, we must have


ou
ad

f'(x) < 0
Y

- -J2 sin x - — < 0


4
nd
Re

=> sin x - — > 0


Fi

4
r, K K 5 71 K 57t j
=> 0 < X-----< K => — <x<— => xe
4 4 4 4 ' 4 J'
Hence, / (x) is decreasing on (tt/4,5k/4).
EXAMPLE 20 Find the intervals in which f(x) = sin 3x - cos 3x, 0 < x < ti, is strictly increasing or
decreasing. [CBSE 2016]
SOLUTION We have,
/(x) = sin 3x - cos 3x
f (x) = 3 (cos 3x + sin 3x)
=> /' (x) = 3V2 ^ cos 3x + 4= sin 3xj = 3V2 ^sin-^cos3x+ cos^sin 3xj
V2
f'(x) = 3V2 sin [3x + ^)

It is given that

ReadYourFlow.COM
17.24 MATHEMATICS-XII

K - K 13ti
0<x<n:=> 0<3x<3ti=> — <3x + — <3k + —=> — < 3x + — <----
4 4 4 4 4 4
(i) For /(x) to be strictly increasing, we must have
/'(x) > 0
3V2sin^3x + ^j
=> >0

sin 3x + — > 0
4
=> — <3x + —<7t or, 2;: < 3x + — < 3ti
4 4 4
_ _ Sti 7n „ Htc
=> 0 < 3x < — or, — < 371 < —
4 4 4
r, 71 7 k 1 Iti
0 <x < - or, — < x <----

w
4 12 12
7k Uk
XG 0,-
'4 12' 12
So, /(x) is strictly increasing on

Flo 7k lire

ee
12' 12

Fr
(ii) For/(x) to be strictly decreasing we must have
/'(x) < 0 for
ur
3V2 sin 3x + ^ j <0
ks

sin 3x + -^ j < 0
=>
Yo
oo
B

=> 7i<3x + -<27i or, Stt < 3x + — < —


4 4 4
re

3ti: 7k 1171
=> — < 3x < — or. <3x <3k
4 4 4
ou
ad

K 7k Uk
=> — < x < — or. <X <K
Y

4 12 12
K 7k 117t
nd

u
Re

X G / 77
4'T 12
Fi

So,/(x) is strictly decreasing on

Type U ON PROVING THE MONOTONICITY OF A FUNCTION ON A GIVEN INTERVAL

EXAMPLE 21 Prove that thefunction f{x) =x3 - 3x2 + 3x -100 is increasing on R. [NCERT1
SOLUTION We have,
/(x) = x3 -3x2 + 3x -100
=>/'(x) = 3x2 - 6x + 3 = 3 (x -l)2
Now, x e R => (x -1)2 > 0 => / '(x) > 0.
Thus, /'(x) > 0 for all x g R.
Hence,/(x) is increasing on R.
EXAMPLE 22 Let I be an interval disjointed from [-1,1], Prove that the function /(x) = x + — is

increasing on I. [NCERT1

ReadYourFlow.COM
INCREASING AND DECREASING FUNCTIONS 17.25

SOLUTION We have,
/(*)=*+-*
x2-l
=>
X x2
Now, x el
=> t ^ [-1,1]
x < — 1 or x>l
=> X2 >1
=> x2 -1 >0

=> [•.• x2 > 1 > 0]

w
=> /'(x) >0

Flo
Thus, / '(x) > 0 for all x el. Hence,/(x) increasing on I.
3
EXAMPLE 23 Show that the function /(x) = — + 7 is decreasing for x e R -{0}.

ee
x

Fr
3
SOLUTION We have, /(x) = - + 7
x for
/'(*)=-Ax2
ur
1
ks

Now, x e R, x # 0 => 4<0^ / '(x) < 0.


7>0 ^ ^
Yo
oo

Hence,/(x) is decreasing for x e R, x ^ 0.


eB

EXAMPLE 24 S/iow that the function x + 1/x is increasingfor x > 1.


r

1
Then,
ou

SOLUTION Let f(x) = x +


ad

x
Y

x2-l
/'«=1-4
x2 x2
nd
Re

x2 — 1
Fi

Now,x>l => x2>l => 1:>0 => /'(x) >0.


x
Hence, /(x) is increasing for x > 1.
EXAMPLE 25 Shozu that f(x) = tan-1 (sin x + cos x) is an increasingfunction on the interval^, n/ 4).
[NCERT EXEMPLAR]
SOLUTION We have,
/(x) = tan-1 (sin x + cos x)
1 1
=>/'(*) = — (sin x + cos x) =
---------------------------------------- X - 2 x (cos x - sin x)
1 + (sin x + cos x)2 dx 1 + (sin + cos x)
cos x (1 - tan x)
=>/'(*) = 2
1 + (sin x + cos x)
Now, 0 < x < 71/4
1
=> cos x > 0, 2 and 1 - tan x > 0 [v 0 < tan x < 1 for 0 < x < ti/4]
1 + (sin x + cos x)

ReadYourFlow.COM
17.26 MATHEMATICS-XII

cos x (1 - tan x)
=> 2 >0
1 + (sin x + cos x)
=> /'(x) >0
Thus, / '(x) > 0 for all x e (0, re/4). Hence,/(x) is increasing on (0, tt/4).
4 sin 0 0 is an increasingfunction of 6 in 0 , ^
EXAMPLE 26 Prove that f (0) =
2 + cos 0
[NCERT, CBSE 2011]
SOLUTION We have,
4 sin 0
/(0) = 2 + cos 0 -0

(2 + cos 0) (4 cos 0) + 4 sin2 0


=>/'(0) = -1
(2 + cos 0)2

w
8 cos 0 + 4
=> f'(Q) = -1
(2 + cos 0)2
4 cos 0 - cos2 0

Flo
ree
=> /' (0) =
(2 + cos 0)2

F
cose(4^cose)>0forall9T0 ^
=> /' (0) = . [•.• cos 0 > 0, 4 - cos 0 > 0 and 2 + cos 0 > 0]
(2 + cos 0)2 V 2) or
ur
Hence, / (0) is increasing on [0,7r/2].
sf
EXAMPLE 27 Prove that the function f(x) - tan x - 4x is strictly decreasing on (-n/ 3, n/ 3).
k
Yo

[NCERT EXEMPLAR]
oo

SOLUTION We have,
B

/(x) = tan x - 4x
re

=> /'(x) =sec2 x - 4


ou
ad

1 -4cos2x 4 1 2 4 1 1
/'(X) = ----- COS X = - + cosx — cosx
Y

2 cos2 X 4 cos2x 2 2
COS X
Now,
nd
Re

x e(-n/3, n/3)
Fi

K n
=> — < x < —
3 3
=> — < cosx <1
2
1 < cosx and^ —
=> — 1 < 1 h COS X <-
1 +— 1 +l
2 2 2 2 2
1 3
=> -cosx < 0 and 1 < ----
- b COS X < —
2 2 2
I 1
=> —cosx < 0 and — + cosx > 0
2 2
1 - + cosx I <0
=> — cosx
2 2
4 1
=> — cosx - + cosx I <0
cos2 X 2 2
=> /' (x) < 0
Hence, / is strictly decreasing on (-re/ 3, tc/ 3).

ReadYourFlow.COM
INCREASING AND DECREASING FUNCTIONS 17.27

EXAMPLE 28 Show that f(x) = 2x + cot -1 x + log ^ 'll + x2 -x j is increasing on R.


[NCERT EXEMPLAR]
SOLUTION We have.
f(x) = 2x + cot -1 X + log +X2 -xj

1 X
=> f (x) = 2----- - 2 -1
l + x~ VI + x2 -x VI + x2
x-yjl+x2
=> /' (T) =
1 1
2-------- 7 +-?—T
=2_^____ L_
1+x2 Vl + x2 x v Vl +X2 1+X2 Vl+X2

1 1
=> + 1- > 0 for all x e R

low
/'(*)= 1
1 + x2 VTV7
f (x) > 0 for all x e R, x * 0.
Hence,/(x) is increasing on R.

ee
EXAMPLE 29 Test whether the function f(x) = x3 - Sis increasing on [1,2].
rF
Fr
SOLUTION We have,
f(x) = x3-8=>/'(x) = 3x2 => /' (x) > 0 for all x e [1, 2] for
So,/(x) is increasing on [1, 2].
u
EXAMPLE 30 Which of the following functions are decreasing on(0, tc/ 2) ?
ks
Yo

(i) cosx (ii) cos2x (iii) tanx (iv) cos3x [NCERT]


oo

SOLUTION (i) We have,


B

/(x)=cosx => /' (x) = - sin x


re

Now, x e(0,7r/2) => sinx>0=^> -sinx<0 => /'(x)<0


ou

So,/(x) is decreasing on (0, k/2).


ad
Y

(ii) Let /(x) = cos 2x. Then, / '(x) = - 2 sin 2x.


Now,
nd
Re

x e (0, k/2) => 0 < x < k/2 => 0 < 2x < tt => sin 2x > 0 => - 2 sin 2x < 0 => / '(x) < 0
Fi

So,/(x) is decreasing on (0, 7t/2).


(iii) Let /(x) = tan x. Then, /'(x) =sec2 x.
Now, xe(0, ti:/2) => sec2 x > 0 => /'(x)>0
So, /(x) is increasing on (0, k/2).
(iv) Let/(x) =cos 3x. Then,/'(x) = - 3 sin 3x
Now, xe(0, tc/2)
=> 0 < x < k/2
=> 0 < 3x < 3rc/2
sin 3x can be positive as well as negative
=> / '(x) = - 3 sin 3x can be positive as well as negative
So, /(x) is neither increasing nor decreasing on (0, ti/2).
9
EXAMPLE 31 Prove that thefunction f(x) = x - x + 1 is neither increasing nor decreasing on (-1,1).
[CBSE 2014, NCERT]

ReadYourFlow.COM
17.28 MATHEMATICS-XII

SOLUTION We have, f(x)=x2 -x + 1


f'(x) = 2x-l=:2(x-l/2).
Now, -1 <x <l/2=> (x -1/2) <0=> 2{x -1/2) < 0 => / '(*) < 0
and, 1/2<x<1=> x-l/2>0=> 2(;t-1/2) > 0 => /'(x) > 0.
Thus, / '(x) does not have the same sign throughout the interval (-1,1).
Hence,/(x) is neither increasing or decreasing on (-1,1).
EXAMPLE 32 On which of the following intervals is the function f(x) - x100 + sin x -1 increasing?
(i) (0, k/2) (ii) (tc/2, ti) (hi) (0,1) (iv) (-1,1). [NCERT]
100
SOLUTION We have, /(x) = x + sin x -1
/ '(x) =100x" + cos x

low
(i) x e(0, k/2)
0 <x <-
2
=> x" > 0 and cos x > 0 => 100x" + cos x > 0 => / '(x) > 0.

ee
Thus,/(x) is increasing on(0, k/2).
rF
Fr
(ii) x e(7r/2, k) for
k/2 <x <k
22 22
u
K
=> x">l •.•-<X<71=> ---- < X <----
ks

2 14 7
Yo

100x" > 100


oo

=> -(i)
B

Again, x e (k/2, k) => -1 < cos x <0 => 0 > cos x > -1 ...(ii)
re

From (i) and (ii), we obtain that for x e (k/2, rt)


100 X 99 >100 and cosx > -1
ou
ad
Y

=> 100x" + cos x> 100-1 = 99


100x" + cos x > 0
nd

=>
Re

=> /'(x) > 0


Fi

Thus,/(x) is increasing on (k/2, k).


(hi) x e (0,1) => x">0 => 100x" > 0
Again, x e (0,1)
=> x lies between 0 and 1 radian
=> x lies between 0° and 57° [v 1°~ 57°]
=> x lies in first quadrant
=> cos x > 0
x e (0,1) => 100 x" > 0 and cos x > 0 => 100x" + cos x > 0 => / '(x) > 0

Thus,/(x) is increasing on (0,1).


(iv) We have seen in (iii) that/ '(x) > 0 for 0 < x < 1. But, / '(x) can be positive as well as negative
when -1 < x < 0. So,/ '(x) can be positive as well as negative for x e (-1,1). Hence,/(x) is neither
increasing nor decreasing on (- 1,1).

ReadYourFlow.COM
INCREASING AND DECREASING FUNCTIONS 17.29

LEVEL-2
Type I ON FINDING THE INTERVAL IN WHICH A FUNCTION IS INCREASING OR DECREASING

EXAMPLE 33 Determine the values of x for which f (x) = xx, x > 0 is increasing or decreasing.
SOLUTION Clearly, f (x) = xx is defined for x > 0. So, domain / = (0, co).
Now, /(x) = x*
=> f{x) = e*10*1
=> f'(x) = eX log * A (x loge X)
dx
=> f'(x) = xx (1 + loge x)
For / (x) to be increasing, we must have

w
/' (x) > 0
=> xx (1 + loge x) > 0
=> 1 + loge x > 0

Flo [•.• xY > 0 for x > 0]

ee
loge x > -1

Fr
=> x > e -1 [•.• logfl x > N => x > a N for a>l. Here, e >1. So, loge x > -1 => x > <? l]
=> x e(l/c, qo)
for
ur
Thus, / (x) is increasing on (!/<?, oo).
For / (x) to be decreasing, we must have
ks
Yo

/ '(x) < 0
oo

=> (1 + loge x) < 0


B

=> 1 + loge x < 0 [•.• x v > 0 for x > 0]


re

=> loge x < -1


ou
ad

=> x < e -1
Y

=> x e(0,1/e)
nd
Re

Thus, / (x) is decreasing on (0,1/e).


Fi

Hence, / (x) is increasing on (1/e, oo) and decreasing on (0,1/e).


X
EXAMPLE 34 pjnci the intervals in which f (x) =------ is increasing or decreasing.
log x
SOLUTION Note that the domain of / (x) is the set of all positive real numbers other than unity
i.e. (0,1) u(l , oo).
Now, /(x) = -r^—
log x
log x -1
=> /'(*)
(log x)2
For / (x) to be increasing, we must have
/ '(x) > 0
=> lo-g^zl>0
(log X)

=> log x -1 > 0 [•.• (log x)2 > 0 for x > 0 , x ^ 1]

ReadYourFlow.COM
17.30 MATHEMATICS-XII

=> log x > 1


=> X > e1 [vlogflx>N => x for a >1. Here,e >1 logex>l x>e1]
=> x e(er oo).
So, / (x) is increasing on (e, oo).
For / (x) to be decreasing, we must have
/'(x) < 0
log X -1
=> < 0
(log x)2
=> log x -1 <0 [••• (log x)2 > 0 for x > 0 , x ^ 1]
log X < 1
=> x < e1

w
=> x e(0, e) -{1} [•.• / (x) is defined for x > 0 , x * 1]
So, / (x) is decreasing on (0, e) - {!}.

Flo
EXAMPLE 35 If a, b, c are real numbers, then find the intervals in which

ee
x + a2 ab ac

Fr
/ (x) = ab x + b2 be is increasing or decreasing.
ac be x + c2
for
ur
SOLUTION We have.
ks

x + a2 ab ac
Yo
oo

/(*) = ab x + b2 be
B

ac be x + c2
re

1 o 0 x+a ab ac x + a2 ab ac
ou
ad

=> /'(x) = ab x + b2 be + 0 1 0 + ab x + b be
Y

ac be x+ c 2 ac be x + c2 0 0 1
nd
Re

=> / '(x) = (x + b2) (x + c2) -b2 c2 + (x + a2) (x + c2) - a2 c2 +(x + a)2 (x + b)2 - a2 b2
Fi

=> f'(x) = 3x2 + 2x (a2 +b2 + c2).


For f (x) to be increasing, we must have
/'(*) >0 + +
=> 3x2 + 2x (a2 +b2 + c2) > 0 - 00
_|(a2 + fc2 + c2) 0 OO

Fig. 17.37 Signs otf'(x) for different values of x


=> x - 3x + 2 (a2 +b2 + c2) >0
t'
=> x < — (a2 + b2 + c2) or, x > 0 [See Fig. 17.37]
3
^ (a2 + b2 + c2) j u (0, oo).
=> x el - co, -

So, / (x) is increasing on|^- x>, -^(a2 +b2 + c2) j u(0, co)

For / (x) to be decreasing, we must have

ReadYourFlow.COM
INCREASING AND DECREASING FUNCTIONS 17.31

+
/'(*) < 0 o
- oo 2{a2 + b2 + c2) 0 CO
=> 3x2 + 2x(a2 +b2 + c2) < 0
Fig. 17.38 Signs of/'(.v) for different values of .v
=> .t < 3x + 2(a2 +b2 + c2) ■ < 0

=> — (a2+b2 + c2)< x <0 [See Fig. 17.38]


3
=> xe (-|(«2+^2 + c2)/0j
So, / (x) is decreasing on f- {a2 +b2 + c2), 0 j .

~ (a2 + b2 + c2) j u (0, co) and decreasing on


Hence, f(x) is increasing on -CO, -

w
(a2 +b2 + c2), oj.

Flo
Type II ON PROVING MONOTONICITY OF A FUNCTION ON A GIVEN INTERVAL

ee
EXAMPLE 36 Shozv that for a>l, f(x) = V3 sin x - cos x - 2ax + b is decreasing on R.

Fr
SOLUTION We have,
/(x) = v^sin x — cosx — 2ax + b
for
ur
=> f (x) = 43 cosx + sinx-2a
ks

=> f'(x)=2 ^y-cosx + -sinx -2a


Yo

2
oo

/' (x) =2^sin-^cosx + cos —sinx \-2a


B

=t>
3
re

=> /' (x) = 2sin x + — - 2a = 2 i sin x + — -a


ou
ad

3 3
Y

=> /' (x) < 0 for all x e R •.* sin x + — < 1 and r? > 1 for all x e R
3
nd
Re

Hence, f{x) is decreasing on R.


Fi

EXAMPLE 37 Show that f(x) = cos (2x + jt:/4) is an increasingfunction on (37t/8,7 n/ 8).
SOLUTION We have, /(x) = cos (2x + 7t/4)
/'(x) = - 2 sin (2x + tc/4)
Now,
x e(37t/8, 7ti/8)
=> 3tc/8 <x <7k/8
=> 3ti/4 < 2x <7k/4
=> k/4 + 371/4 < 2x + 7r/4 < 7ti/4 + 7t/4
k < 2x + ti/4 < 27:
=> sin (2x + ti:/4) < 0 [v sine function is negative in third and fourth quadrants]
=> - 2 sin (2x + 71/4) > 0
=> /'(*)>0
Hence,/(x) is increasing on (37r/8, 77t/8).

ReadYourFlow.COM
17.32 MATHEMATICS-XIt

EXAMPLE 38 Find the least value of 'a' such that thefunction f(x) - x2 + ax + l is increasing on [1, 2].
Also, find the greatest value of 'a'for zuhich f(x) is decreasing on [1, 2]. [NCERT]
SOLUTION We have, f(x)= x2 + ax+ \
f '(x) = 2x + a and / "(x) = 2 for all x.
Now, / " (x) = 2 for all x e (1, 2)
/ ''(x) > 0 for all x e [1, 2]
=> / '(x) is an increasing function on [1, 2]
=> / '(1) and/' (2) are the least and the greatest values of / '(x) on [1, 2].
As /(x) is increasing on [1, 2]
/'(x) >0forallx e[l, 2]
This is possible when least value of /' (x) i.e. /'(I) > 0.

w
Now, / '(1) > 0 => 2 + a > 0 => a> -2
Thus, the least value of a is -2.

Flo
If /(x) is decreasing on [1, 2], then
/'(x) < 0 for all x e [1, 2]

ree
Greatest value of / '(x) < 0 for x e [1, 2]

F
=> f (2) < 0 [••• f'(x) is increasing on [1, 2] /' (2) is the greatest value of/(x)]
=> 4 + a<0=>a<-4. or
ur
So, the greatest value of a is -4.
f
NOTE (i) ax2 + bx + c >0 for all x => a>0 and b2 - Aac < 0
ks
Yo
oo

(ii) +bx + c <0 for all x => a <0 and b2 - Aac < 0
B

(iii) If the least value off(x) defined on [a, b] is positive, then /(x) > Ofor all x e [a, b],
re

(iv) If the greatest value off(x) defined on [a, b] is negative, then f(x) < Ofor all x e [a, b).
EXAMPLE 39 Find the values 'a' for which the function f (x) = (a + 2) x3 - Sax2 + 9 ax -1 decreases
ou
ad

for all real values ofx.


Y

SOLUTION We have,
nd
Re

/ (x) = (a + 2) x3 - 3flx2 + 9ax -1


Fi

=> /' (x) = 3 (fl + 2) x2 - 6ax + 9a


Since / (x) is decreasing for all real values ofx. Therefore,
/' (x) < 0 for all x eR
=> 3 (a + 2) x2 - Sax + 9a <0 for all x e R
=> (a + 2) x2 - 2ax + 3a <0 for all x e K

+ 2 < 0 and Aa2 - 4 x (a + 2) x 3a <0 ax +bx + c <0 for all x e R


=>
=> < 0 and Disc < 0
=> a <-2 and a2 - 3a2 6a <0
=> a <-2 and - 2a2 -6a <0
=> a <-2 and -2a(a + 3) < 0 + +
Now, - 00
-3 0 CO

-2a(a + 3) < 0 Fig. 17.39 Signs of a (a + 3) for different values of x


=> a(a+ 3) > 0

ReadYourFlow.COM
INCREASING AND DECREASING FUNCTIONS 17.33

=> a < - 3 or, a > 0 [See Fig. 17.39]


=> a e ( - oo, - 3) u (0, oo)
a <-2 and -2a(a + 3) <0
a<-2 and a - 3) u(0/co)
=> a e (- oo, - 3).
Hence, / (x) decreases for all x e R, if a e(- <x>, - 3).
EXAMPLE 40 Find the values of k for which f (x) = kx'2 - 9kx2 + 9x + 3 is increasing on R.
SOLUTION It is given that f (x) is increasing on R. Therefore,
f(x) > 0 for all x <=R
=> 3kx2 - 18kx + 9 > 0 for all xeR
=> kx2 - 6kx +3 > 0 for all x e R

w
=> k>0 and 36k2-12k <0 [•.• ax2 +tx + c>0forallxeR=>a>0 and Disc < 0]
=> k >0 and 12A: (3A:-1) < 0
=>
=>
k>0 and k (3k -1) <0
3k-1 <0

Flo [•.• ft: > 0]

ee
=> k < — => /c e (0,1/3).

Fr
3
Hence, / (x) is increasing on R, if ft: e (0,1/ 3). for
ur
EXERCISE 17.2
LEVEL-1
ks
Yo
oo

1. Find the intervals in which the following functions are increasing or decreasing.
(i) /(x)=10-6x-2*2 [NCERT]
B
re

(ii) f(x) =x2 + 2x -5 [NCERT]


(hi) f{x) = 6 - 9x - x2 [NCERT]
ou
ad

(iv) f(x) =2x3 -12x2 + 18x + 15


Y

(v) f(x) = 5 + 36x + 3x2 - 2x3


nd
Re

(vi) f(x) = 8 + 36x + 3x2 - 2x3


Fi

(vii) f(x) =5x3 -15x2 -120x+ 3


(viii) /(x)=x3 -6x2 -36x + 2
(ix) /(x)=2x3 -15x2 + 36x + l [CBSE 2005,2010]
(x) f(x) =2x3 + 9x2 + 12x + 20 [CBSE 2011]
(xi) /(x)=2x3 -9x2 +12x-5
(xii) /(x) = 6 + 12x + 3x2 - 2x3
(xui) /(x)=2x3-24x + 107
(xiv) /(x) = - 2x3 - 9x2 - 12x + 1 [NCERT]
(xv) f(x) =(x -1) (x - 2)2
(xvi) /(x) = x3 - 12x2 + 36x +17 [CBSE 2001]

ReadYourFlow.COM
17.34 MATHEMATICS-XII

(xvii) f(x)=2x3 -24x + 7


, .... r, . 3 4 43 _ 2 36
(xvm) f{x) = — x — x - 3a: + — x + 11 [NCERT]
' lO 5 5
(xix) f(x) = x4 - 4x
x4
(xx) /(x) = ^- + -x3--x2-6x + 7
3 2
(xxi) f(x) = x4 - 4x3 + 4x2 + 15
(xxii) /(x) =5x 3/2 - 3x5/2, x > 0
(xxiii) /(x) = x8 + 6x2
(xxiv) / (x) = x3 - 6x2 + 9x + 15 ICBSE 2000,2004]
(xxv) / (x) = {x (x - 2)}2 [NCERT, CBSE 2010,2014]

low
(xxvi) / (x) = 3x4 -4x3 -12x2 +5 [CBSE2014]
— x4 -4x3 -45x2 +51
(xxvii) f(x)=^ [CBSE 2014]

ee
2x
(xxviii) / (x) = log (2 + x) -, x eR
rF [CBSE 2014]

Fr
2+x
2. Determine the values of x for which the function /(x) = x2 - 6x + 9
for is increasing or
decreasing. Also, find the coordinates of the point on the curve 1/ = x2 - 6x + 9 where the
ou
normal is parallel to the line y = x + 5.
ks

3. Find the intervals in which/(x) = sin x - cos x, where 0 < x < 27r is increasing or decreasing.
oo

4. Show that/(x) = e2x is increasing on R. [CBSE 2000,2010]


Y
B

5. Show that /(x) = e^x, x + 0 is a decreasing function for all x ^ 0.


re

6. Show that /(x) = logrt x, 0 < <7 < 1 is a decreasing function for all x > 0.
7. Show that /(x) = sin x is increasing on (0, jr/2) and decreasing on (rc/2, k) and neither
ou
ad

increasing nor decreasing in (0, 7t). [NCERT]


Y

8. Show that/(x) = log sin xis increasing on (0, k/2) and decreasing on (rc/2, rc). [NCERT]
9. Show that /(x) = x - sin x is increasing for all x e R.
d
Re
n

10. Show that/(x) = x3 - 15x2 + 75x -50 is an increasing function for all x e R.
Fi

11. Show that /(x) = cos x is a decreasing function on (0, rc/2).


12. Show that/(x) = sin x is an increasing function on (- rc/2, rc/2).
13. Show that /(x) = cos x is a decreasing function on (0, rc), increasing in (- rc , 0) and neither
increasing nor decreasing in (- rc, rc).
14. Show that /(x) = tan x is an increasing function on (- rc/2, rc/2).
15. Show that/(x) = tan-1 (sin x + cos x) is a decreasing function on the interval (rc/4, rc/2).
16. Show that the function /(x) = sin (2x + rc/4) is decreasing on (3rc/8,5rc/8).
17. Show that the function/(x) = cot_1(sin x + cosx) is decreasing on (0, rc/4) and increasing on
(rc/4, rc/2).
18. Show that/(x) = (x - \)ex + 1 is an increasing function for all x > 0.
19. Show that the function x - x + 1 is neither increasing nor decreasing on (0,1).
Q 7
20. Show that/(x) =x + 4x + 11 is an increasing function for all x e R.

ReadYourFlow.COM
INCREASING AND DECREASING FUNCTIONS 17.35

21. Prove that the function/(x) = x3 - 6x2 + 12x -18 is increasing on R. [CBSE2002C]
22. State when a function /(x) is said to be increasing on an interval [a, b]. Test whether the
function /(x) = x - 6x + 3 is increasing on the interval [4, 6].
23. Show that/(x) = sin x - cos x is an increasing function on (- tt/4, k/4).
24. Show that/(x) = tan-1 x - x is a decreasing function on R.
25. Determine whether /(x) = - x/2 + sin x is increasing or decreasing on (- n/ 3, k/ 3).
x
26. Find the intervals in which / (x) = log (1 + x)----1— is increasing or decreasing.
1+x
[CBSE 2000 C]
27. Find the intervals in which / (x) = (x + 2) e~ x is increasing or decreasing. [CBSE 2000 C]
28. Show that the function / given by / (x) = 10* is increasing for all x .

w
29. Prove that the function / given by / (x) = x - [x] is increasing in (0,1).
30. Prove that the following functions are increasing on R:

Flo
(i) /(x) = 3x5 +40x3 + 240x
(ii) /(x) = 4x3 -18x2 + 27x-27

ee
[CBSE 2017]

Fr
31. Prove that the function / given by / (x) = log cos x is strictly increasing on (- n/2, 0) and
strictly decreasing on (0,71/2). [NCERT]
Q 2
for
ur
32. Prove that the function/given by/(x) =x -3x + 4xis strictly increasing on P. [NCERT]
33. Prove that the function / (x) = cos x is:
ks

(i) strictly decreasing in (0, tc) (ii) strictly increasing in (n, 2n)
Yo
oo

(iii) neither increasing nor decreasing in (0, 2 7t) [NCERT]


eB

LEVEL-2

34. Show that/(x) = x - x sin x is an increasing function on (0, n/2).


r
ou
ad

35. Find the value(s) of a for which /(x) = x3 - ax is an increasing function on R.


Y

36. Find the values of b for which the function /(x) = sin x - bx + c is a decreasing function
nd

on R.
Re

37. Show that/(x) = x + cos x - a is an increasing function on R for all values of a.


Fi

38. Let / defined on [0, 1] be twice differentiable such that |/"(x) | <1 for all x £[0,1]. If
/(0) =/(l), then show that |/' (x) | <1 for all x e [0,1]. [NCERT]
39. Find the intervals in which/ (x) is increasing or decreasing: [CBSE 2014]
(i) f(x)=x\x\,xeR (ii) / (x) = sin x +1 sin x |, 0 < x < 2tc
(iii) /(x) = sin x (1 + cosx), 0 < x < -^

ANSWERS

1. Increasing Decreasing Increasing Decreasing


(i) (-00,-372) (- 3/2, co) (ii) (-1, co) (-00, -1)
(iii) (-oo, -9/2) (-9/2, a,) (iv) (-oo, 1) u(3, co) (1,3)
(v) (-2, 3) (- co, - 2) u (3, oo) (vi) (-2,3) (-co, -2) u(3, co)
(vii) (-<»,-2)u(4,«>) (-2,4) (viii) (- co, - 2) u (6, co) (- 2, 6)
(ix) (- oo, 2) u (3, oo) (2,3) (x) (-co,-2)u(-l,co) (-2,-1)
(xi) (- co, 1) u (2, oo) (1,2) (xii) (-1,2) ( co, -1) U (2, co)
(xiii) (- oo, - 2) u (2, oo) (-2,2) (xiv) (-2,-1) (- CO, - 2) U ( -1, oo)

ReadYourFlow.COM
17.36 MATHEMATICS-XII

(xv) (— co, 4/ 3) (2, co) (4/3,2) (xvi) (- co, 2) u (6, co) (2, 6)
(xvii) (-oo, -2) u(2, oo) (-2,2) (xviii) (- 2,1) u(3, co) (-oo,-2) o'(l, 3)
(xix) (1, oo) (-oo,l) (xx) (- 3, -1) u (2, c») (-00, - 3) u (-1, 2)
(xxi) (0,1) u (2, oo) (- do, 0) u (1, 2) (xxii) (0,1) (bco)
(xxiii) (0, oo) (- co, 0) (xxiv) (- co, -1) u (3, oo)
(1,3)
(xxv) (0,1) u(2 , oo) (- , 0) u (1, 2) (xxvi) (-1, 0) u(2, oo) (-oo,-l) u(0, 2)
(xxvii) (-3, 0) w(5, oo) (- co ,-3) u (0,5) (xviii) (2, oo) (-*>, 2)
2, Increasing on (3, oo) and decreasing on (- co, 3); (5/2,1/4)
3, Increasing on (0, 3n/4) u (7n/4, 2n) ; decreasing on (3n/4,7n/4)
22. Increasing 25. Increasing
26. Increasing on (0 , oo); decreasing on (-1, 0)
27. Increasing on (-co, -1); decreasing on (-1, oo)
31. Increasing on (-co,-1); decreasing on(-1, oo) 35_fl<0 36. ^ -1

w
39. (i) Increasing for all x e R
(ii) Increasing on (0, n/2), decreasing on (n/ 2, Jt), neither increasing nor decreasing on
(ti, 2n)

Flo
(iii) Increasing on (0, n/3), decreasing on (n/3, k/2)

ree
HINTS TO NCERT & SELECTED PROBLEMS

F
1. (i) We have, f (x) = 10 - 6x - 2x2
=> /' (x) = - 6 - 4a: = - 2 (2x + 3) or
ur
For / (x) to be increasing, we must have
sf
3
f'(x)>0 => -2(2x+3)>0 => 2x + 3 < 0 => x <----
k
Yo

2
oo

So, / (x) is increasing on (- oo, - 3/2.


B

For / (x) to be decreasing, we must have


re

/' (x) < 0 => - 2 (2x + 3) < 0 => 2x + 3 > 0 => x > --
2
ou
ad

So, / (x) is decreasing on (- 3 / 2, co)


Y

(ii) We have,/(x) = x2+2x-5


nd
Re

=> /'(x) = 2x + 2 = 2(x + l)


For / (x) to be increasing, we must have
Fi

/' (x) > 0 => 2(x + l)>0 => x + l>0=> x > — 1 => xe(-l,oo)
For / (x) to be decreasing, we must have
/' (x) < 0 => 2 (x +1) < 0 => x +1 < 0 => x < -1 => x e (-oo, -1)
Hence, / (x) is increasing on (-1, oo) and decreasing on (- oo, -1).
(iii) We have, / (x) = 6 - 9x - x2 => f '(x) = - 9 - 2x = - (2x + 9)
For / (x) to be increasing, we must have

/'(x)>0 => -(2x + 9)>0 2x + 9<0 => => xe(-oo, -9/2)

For / (x) to be decreasing, we must have


9
/'(x) < 0 -(2x + 9) < 0 => 2x + 9 > 0 => x > - -=> xe(-9/2, c»)

(xiv) We have, / (x) = - 2x3 - 9x2 - 12x +1


r^> /'(x) = -6x2 -18x -12 = - 6 (x2 + 3x + 2) = -6(x + l)(x + 2)

ReadYourFlow.COM
INCREASING AND DECREASING FUNCTIONS 17.37

For / (x) to be increasing, we must have


/'(x)>0=> -6(x + 1) (x + 2) > 0 => (x + 1) (x + 2) <0 => x e(-2,-1)
For / (x) to be decreasing, we must have
/'(x) <0 => -6 (x + 1) (x + 2) < 0
=?• (x + 1) (x + 2) > 0 => x < - 2 or x > -1 => x g (- co, - 2) u (-1, oo)
Hence, / (x) is increasing on (- 2, -1) and decreasing on (- co, - 2) u (-1, co).
(xviii) We have.
/(x) = — x4--x3-3x2+ —x + 11
10 5 5
36
— x 3 — — x2 - 6x + —
5 5

- 00 -2 1 3 CO

w
Fig. 17.40 Signs of/'(x) for different values of x

For / (x) to be increasing, we must have

Flo
/'(x) >0 => |(x-)(x-3) (x + 2)>0 => (x-l)(x-3) (x + 2) > 0 => x e(-2,1) u(3, oo)

ree
So, / (x) is increasing on (- 2,1) u (3, oo).

F
For / (x) to be decreasing, we must have
/ '(x) < 0 => — (x -1) (x - 3) (x + 2) < 0 => (x -1) (x - 3) (x + 2) < 0 => x e (- oo, - 2) u (1, 3)
or
ur
5
f
(xxii) We have, / (x) =5x3/2 - 3x5/2 => /'(x) = ^-Vx (1 -x)
ks
Yo
oo

Now, /' (x) > 0 => ^ Vx (1 - x) > 0 => 1 - x => x < 1


B

(xxiii) We have, / (x) = x8 + 6x2 => / '(x) = 4x (2x6 + 3)


re

/ '(x) > 0 => 4x (2x6 + 3) > 0 => x > 0 [v 2j:6 + 3>0)


ou
ad

(xxv) We have, / (x) = x2 (x - 2)2


Y

f'(x) = 2x (x - 2)2 + 2x2 (x - 2) = 2x (x - 2) (2x - 2) = 4(x-2)(x-l)x


nd
Re

For / (x) to be increasing, we must have


Fi

/' (x) > 0


=4> 4 (x - 2) (x -1) x > 0 - CO 0 12 CO

=> (x - 2) (x -1) x > 0 Fig. 17.41 Signs of/'(.r) for different values of x
x g(0, 1) u(2, co)
So, / (x) is increasing on (0,1) u (2, co).
For / (x) to be decreasing, we must have
/ (x) < 0
=> 4 (x - 2) (x -1) x < 0 => (x - 2) (x -1) x < C
=> x g(-oo, 0) u (l, 2)
So, / (x) is decreasing on (- co, 0) u (1, 2).
4. We have, /(x) = =3> /' (x) = e* > 0 for all .xeR.=> f(x) is increasing on R.
5. Since e1^x > 0 for all x * 0 and - 1/x2 <0 for all x ^ 0 .
/'(x) = — (1/x2) e1/* <0 for allx?t 0

ReadYourFlow.COM
17.38 MATHEMATICS-XII

6. We have, f(x) = logrt .v => f '(x) = —-—


x log a
Since 0 < < 1, therefore log a < 0.
Now, x > 0 => — > 0 => —-—<0 f'(x)<0.
x x log a
So, /(x) is decreasing for all x > 0
7. We have, /(x) = sin x => / '(x) = cos x.
So, / (x) is increasing on (0, re/2) and decreasing on (n/2, k).
8. We have,/(x) = log sin x => /'(x) = cot x
Clearly,/'(x) = cot x > 0 for x e (0, k/2) and /' (x) = cot x < 0 for x e (n/2, rc).
Hence, / (x) is increasing on (0, n/2) and decreasing on (n/2, tt).
9. We have.
f(x) = x - sin x => /' (x) = 1 - cos x.
=> /'(x) > 0 for all x e i?. [v cos x < 1]

low
So, f(x) is increasing for all x e R.
10. We have.
fix) = x3 -15x2 + 75 x -50

ee
fix) = 3x2 - 30x + 75 = 3 (x -5)2 > 0 for all x gR.
rF
Fr
So, / (x) is increasing function for all x e R.
11. We have.
r\

fix) = cos x => fix) = — sin 2x.


for
u
Now, 0 < x < jr/2 => 0 < 2x < 71 sin 2x > 0 => — sin 2x < 0 => / '(x) < 0.
ks

So, f(x) is decreasing on (0, n/2)


Yo

12. We have.
oo

fix) = sin x => /' (x) = cos x.


B

Now, n/2 <x <n/2=> cos x > 0 => /' (x) > 0.
re

So/ (x) is increasing on (- n/2, n/2).


13. We have.
ou
ad

fix) = cos x => / '(x) = - sin x.


Y

Now, 0 < x < 7r=> sin x > 0 => — sin x < 0 => / '(x) < 0.
So, fix) is decreasing on(0, n).
nd
Re

14. We have,
Fi

fix) = tan x => fix) = sec2 x.


Now, -n/2< x < n/2 => sec x > 0 => sec x > 0 => /'(x) > 0.
So, fix) is increasing on (-n/2, n/2).
16. We have.
f(x) = sin (2x + rc/4) => f(x) = 2 cos (2x + ji/4).
3tx
Now, x e (3ti/8, 7n/8) => 3n/8 < x <5ti/8 => 3n/4 < 2x <5rc/4 => 7t < 2x + ti/4 < —-
2
2 cos (2x + k/4) < 0 => f(x) < 0.
So, / (x) is decreasing on (37r/8, 7n/8).
18. We have.
f(x) = (x -1) ex +1=> f '(x) = x ex > 0 for all x > 0.
So, f(x) is increasing for all x > 0.
20. We have.
f(x) = x9 + 4x7 +11 => /' (x) = 9x8 + 28x6 > 0 for all x e R.
So, f(x) is increasing for all x e R.

ReadYourFlow.COM
INCREASING AND DECREASING FUNCTIONS 17.39

21. We have.
f(x) =x3-6x2+12x-l8 => f '(.*) = 3 (.y - 2)2 > 0 for all x e R.
So, f(x) is increasing for all on R.
22. We have.
f(x) =x2 -6x+ 3^ f'(x)=2 (x - 3).
Now, x e [4, 6] => x > 3 => 2 (x - 3) > 0 => / '(x) > 0
So, /(x) is increasing on [4, 6].
23. We have, /'(x) = cos x + sin x = ^ sin (x + tc/4).
Now, - n/4 < x < k/4 => 0 < x + n/4 <n/2=> sin (x + 71/4) > 0 => / ' (x) > 0.
Hence, f(x) is increasing on (-n/4, n/4).
31. We have, /(x) = log cos x => /'(x) = - tan x
Now, x e (- n/2, 0) => tan x < 0 => - tan x > 0 => f'(x) > 0
So, / (x) is strictly increasing on (- nJ2, 0)

w
x e (0, n/2) => tan x > 0 => - tan x < 0 => / '(x) < 0
So, / (x) is strictly decreasing on (0, n/2)
32. We have, / (x) = x3 - 3x2 + 4x

Flo
ree
=> / '(x) = 3x2 - 6x + 4 = 3 (x2 - 2x + 1) + 1 = 3 (x -1)2 + 1 > 0 for all x e R.
Hence, / (x) is strictly increasing on R.

F
33. We have, /(x) = cos x => /'(x) = -sin x
or
ur
(i) x e (0, rc) => sin x > 0 => - sin x <0 =i> / '(x) < 0
sf
So, / (x) is strictly decreasing on (0, n).
(ii) x e (n, 2n) => sin x < 0 - sin x > 0 => / '(x) > 0
k
Yo
oo

So, / (x) is strictly increasing on (n, 2n).


B

(hi) Asf'(x) < Oforx 6 (0,7i)and/' (x) > 0 for x e(0, 2k)
re

Hence, / (x) is neither increasing nor decreasing on (0, 2k).


35. If/(x) is increasing on R, then
ou
ad

/'(x) >0forallxeR => 3x2-fl>0 for all x e R => a < 3x2 for allxeR.
Y

But, the least value of 3x is 0 . Therefore rt < 0.


nd
Re

36. Since /(x) is decreasing on R


Fi

/' (x) < 0, for all x e R


=> cos x - b <0 for all X e R
=> cos x <b for all x e R
=> b >1

___ _________________________________ VERY SHORT ANSWER QUESTIONS (VSAQs)


Answer each of the following questions in one word or one sentence or as per exact requirement of the
question:
1. What are the values of 'a' for which / (x) = ax is increasing on R?
2. What are the values of 'a' for which / (x) = ax is decreasing on R?
3. Write tire set of values of 'a' for which / (x) = logfl x is increasing in its domain.
4. Write the set of values of 'a' for which / (x) = log„ x is decreasing in its domain.
5. Find 'a' for which / (x) = a (x + sin x) + a is increasing on R.
6. Find the values of 'a' for which the function/(x) = sin x - ax + 4 is increasing function on R.
7. Find the set of values of 'b' for which / (x) = b (x + cos x) + 4 is decreasing on R.

ReadYourFlow.COM
17.40 MATHEMATICS-XII

8. Find the set of values of 'a' for which / (x) - x + cos x + ax + b is increasing on R.
9. Write the set of values of k for which / (x) = kx - sin x is increasing on R.
10. If g(x) is a decreasing function on R and / (x) = tan- 1 \g (x)}. State whether / (x) is
increasing or decreasing on R.
11. Write the set of values of a for which the function f (x) = ax + b is decreasing for all x eR.
12. Write the interval in which / (x) = sin x + cos x, t e [0 , n/2] is increasing.
13. State whether / (x) = tan x - x is increasing or decreasing its domain.
14. Write the set of values of a for which / (x) = cos x + a2 x + b is strictly increasing on R.

ANSWERS
1. a >1 2. 0 <1 3. fl >1 4. 0 < a <1
5. e(0, cc) 6. <?e(-co,-1) 7. b e (- oo, 0) 8. a e (0, co)
9. k e (1, co) 10. Decreasing 11. a e (-co, 0) 12. [0,7i/4]

w
13. Increasing 14. fl e(-co,-1] u[l, co)

MULTIPLE CHOICE QUESTIONS (MCQs)

Flo
Mark the correct alternative in each of the folloiving:

ree
1. The interval of increase of the function / (x) = x - ex + tan (2tc/7) is

F
(a) (0, *>) (b) (-=o,0) (c) (1, co) (d) (-oo,l)
2. The function f (x) = cot 1 x + x increases in the interval
or
ur
(a) (1, co) (b) (-l,cx>) (c) (- co, co) (d) (0, a))
sf
3. The function / (x) = xv decreases on the interval
k
Yo

(a) (0, e) (b) (0,1) (c) (0,1/e) (d) (1 /e, e)


oo

4. The function / (x) = 2 log (x - 2) - x + 4x + 1 increases on the interval


B

(a) (L2) (b) (2,3) (c) (1, 3) (d) (2, 4)


re

5. If the function f (x) = 2x - kx + 5 is increasing on [1,2], then k lies in the interval


ou
ad

(a) (-oo, 4) (b) (4, co) (c) (-co, 8) (d) (8, co)
Y

6. Let f (x) = x + ax +bx + 5 sin x be an increasing function on the set R. Then, a and b
satisfy
nd
Re

(a) a2 - -15 > 0 (b) a2 -3b+ 15 >0 (c) a2 - 3/; + 15 < 0 (d) a>0andb> 0
Fi

7. The function/(x) =log£, ^x3 + -^/x^ + 1 | is of the following types:

(a) even and increasing (b) odd and increasing


(c) even and decreasing (d) odd and decreasing
8. If the function / (x) = 2 tan x + (2tf + 1) logt, | sec x \+(a -2) x is increasing on R, then
(a) a g(1 /2 , co) (b) rte(-l/2,l/2) (c) a=l/2 (d) agR
9. Let/ (x) = tan “ 1 {g (x)), whereg (x) is monotonically increasing for 0 < x < -^ .Tlien,/ (x) is

(a) increasing on (0, 7t/2) (b) decreasing on (0, k/2)


(c) increasing on (0, ti/4) and decreasing on (n:/4, x/2)
(d) none of these
10. Let / (x) = x3 - 6x2 + 15x + 3. Then,
(a) / (x) > 0 for all x e R (b) / (x) >/ (x + 1) for all x g R
(c) / (x) is invertible (d) /(x) < 0 for all x g R

ReadYourFlow.COM
INCREASING AND DECREASING FUNCTIONS 17.41

11. The function / (x) = x2 c x is monotonic increasing when


(a) xeR -[0,2] (b) 0 < x < 2 (c) 2 < x < oo (d) x < 0
12. Function / (x) = cos x - 2 X x is monotonic decreasing when
(a) X > 1/2 (b) X <1/2 (c) X < 2 (d) X>2
13. In the interval (1, 2), function / (x) = 21 x -1| + 31 x - 2| is
(a) monotonically increasing (b) monotonically decreasing
(c) not monotonic (d) constant
14. Function/ (x) = x3 - 27 x + 5 is monotonically increasing when
(a) x < - 3 (b) | x | > 3 (c) x < - 3 (d) | x | > 3
15. Function / (x) = 2 x3 - 9 x2 + 12 x + 29 is monotonically decreasing when
(a) x < 2 (b) x > 2 (c) x > 3 (d) 1 < x < 2
16. If the function /(x) = Ax3 - 9 x2 + 9 x + 3 is monotonically increasing in every interval.

w
then
(a) k <3 (b) k<3 (c) k >3 (d) k > 3
17. / (x) =2 x - tan x - log j x + -Jx2 +lj is monotonically increasing when

Flo
ee
(a) x > 0 (b) x < 0 (c) xeR (d) x eR - {0}

Fr
18. Function / (x) = | x | -1 x -11 is monotonically increasing when
(a) x < 0 (b) x > 1 (c) x<l for (d) 0 < x < 1
ur
19. Every invertible function is
(a) monotonic function (b) constant function
ks

(c) identity function (d) not necessarily monotonic function


Yo
oo

20. hi the interval (1, 2), function / (x) = 21 x -1| + 3 | x - 2| is


(a) increasing (b) decreasing (c) constant (d) none of these
eB

21. If the function /(x) - cos | x| - lax + b increases along the entire number scale, then
1 (d) fl>-|
(c) a <--
r

(a) a = b (b) fl = —
ou
ad

X
Y

22. The function / (x) = ---- -—- is


I +1 x |
nd

(b) strictly decreasing


Re

(a) strictly increasing


(c) neither increasing nor decreasing (d) none of these
Fi

X sin x + 2 cos x
23. The function / (x) =—; is increasing, if
sin x + cos x
(a) A,<1 (b) A > 1 (c) A. < 2 (d) A, > 2
24. Function / (x) = a* is increasing on R, if
(a) a >0 (b) a <0 (c) 0 <17 <1 (d) a >1
25. Function / (x) = logfl x is increasing on R, if
(a) 0 < ti < 1 (b) a>l (c) a < 1 (d) a > 0
26. Let (() (x) = / (x) + f {2a- x) and /'' (x) > 0 for all x e [0, a]. Then, <{> (x)
(a) increases on [0, a] (b) decreases on [0, a]
(c) increases on [ - 77, 0] (d) decreases on [a, 2a\
27. If the function / (x) =x2 -kx+5 is increasing on [2,4], then
(a) k e (2, co) (b) /c e (- co, 2) (c) k e (4, co) (d) k e(-cc, 4).
28. Tire function / (x) = - x/2 + sin x defined on [- iz/ 3, rc/ 3] is
(a) increasing (b) decreasing (c) constant (d) none of these

ReadYourFlow.COM
18.2 MATHEMATICS-XII

It follows from this expression that/ (1) = 10 is the maximum value of function/ and the point of
maximum value of / is x = 1. This fact is also evident from the graph of function / as shown in
Fig. 18.2.
y

(1,10)

X' x
o (1,0)

/(*)=-(*-1)2 +10

w
Y'

Flo
Fig. 18.2 Graph of/(ar) = -(.r- l)2 + 10

ree
MINIMUM Let f (x) be a realfunction defined on an interval [a, b]. Thenf (x) is said to have the minimum

F
value in interval [a, b], if there exists a point c e [a, b] such that f (x) >f(c) for all x e [a, b].
for
In such a case, the number / (c) is called the minimum value off (x) in the interval [a, b] and the
ur
point c is called a point of minimum value of / in the interval [a, b).
ks
Yo
oo
B
re
ou
ad
Y
nd
Re
Fi

X' a O c b X

r
Fig. 18.3 Minimum value off(x) atx = c

Consider the function / given by / (x) = x2 + 5. Clearly, domain (/) = R = (-co , oo).
We know that
x2 > 0 for all x e R

=> x2 + 5 > 5 for all x e R


=> / (x) > 5 for all x e R
=> / (x) >/ (0) for all x e R
It follows from this expression and the above definition that the minimum value of function
/ (x) = x2 +5 defined on R is 5 and the point of minimum value off is x = 0.
This observation is also evident from the graph off (x) = x2 + 5 as shown in Fig. 18.4.

ReadYourFlow.COM
MAXIMA AND MINIMA 18.3
Y
lf(x) = x2 + 5

(0, 5)

X' X
O

r
Fig. 18.4 Graph of f(x) = x2 + 5

low
In the above discussion, we have seen that the function / (x) =-(x -1) + 10, x eR has the
maximum value but it does not attain the minimum value, because - (x -l)2 + 10 can be made as
small as we please, which is also evident from the graph (Fig. 18.2). The function f(x) = x2 +5
attains the minimum value 5 at x = 0, but it does not attain the maximum value at any point in its
domain. In fact, / (x) can be made as large as we please. From the graph of / (x) (Fig. 18.4), we

ee
find that the values of / (x) are increasing rapidly. That is why it does not attain the maximum
rF
Fr
value.
Let us now consider the function / (x) = sin x defined on the interval [0,2 7t].
for
Clearly, -1 < sin x < 1 for all xe[0,27i]. So, -1 </(x) <1 for all x e [0, 27t].
ou
Also, -1.
ks
oo

for all x e [0, 2ti]


Y
B
re

Thus, / (x) attains both the maximum value 1 and the minimum value -1 in the interval [0, 2 re].
Points x = n/2 and x = 3rr/2 are respectively the points maximum and minimum values of /
ou
ad

in the interval [0, 2 n]. This is also evident from the graph off (x) as shown in Fig. 18.5.
Y

Y
nd
Re
Fi

(0,1)

X' o

Y'
Fig. 18.5 Graph of f(x) = sin.r, 0 <x<27t
'l
Now, consider the function / giver, by / (x) = x defined on (- 2, 2). Clearly, it is an increasing
function in the given interval. So, it should have the minimum value at a point closest to - 2 on
its right and the maximum value at a point closest to 2 on the left. In fact, it is not possible to
locate such points as shown in Fig. 18.6. Therefore, / (x) = x3 has neither the maximum value
nor the minimum value in the interval (- 2, 2).
It follows from the above discussion that a function / defined on an interval I.

ReadYourFlow.COM
18.4 MATHEMATICS-XII

X' o (2,0) x

Fig. 18.6 Graph off(x) = .r3

(i) may attain the maximum value at a point in I but not the minimum value at any point in /.

w
(ii) may attain the minimum at a point in I but not the maximum value at any point in I.
(iii) may attain both the maximum and minimum values at some points in /.
(iv) may not attain both the maximum and minimum values at any point in I.

Flo
Let us now discuss more examples on the maximum and minimum values of functions in their
domains.

ree
ILLUSTRATIVE EXAMPLES

F
LEVEL-1
or
ur
EXAMPLE 1 Find the maximum and the minimum values, if any, of thefollowing functions
sf
(i) f(x) = 3x2 + 6x + 8, x eR (ii) f(x) = -\ x - 1\ + 5 for all x g R
(iii) f(x) = sin 3x + 4,* e (- n/2 , k/2) (iv) f(x) = x3 + 1 for all x e R
k

[NCERT]
Yo
oo

(v) f(x) = sin (sin x) for all x g R (vi) f(x) =\ x + 3 \ for all x g R.
B

SOLUTION (i) We have,


f(x) = 3x2 + 6x + 8
re

or. f(x) = 3 (x2 + 2x + 1) + 5 = 3 (x + l)2 +5.


ou
ad

Clearly, 3 (x + l)2 > 0 for all x e R


Y

=> 3 (x + l)2 + 5 >5 for all xgR


=>
nd

/(x) > /(-l) for all xeR. [v /(-l) =5]


Re

Thus, 5 is the minimum value of/(x) which it attains at x = -1.


Fi

(fix) = 3(.r + l)2 + 5

X' X
(-L 0) O

r
Fig. 18.7 Graph of/(x)=3(x+ l)2 + 5

Since /(x) can be made as large as we please. Therefore, the maximum value does not exist
which can be observed from Fig. 18.7.

ReadYourFlow.COM
MAXIMA AND MINIMA

(ii) We have,
f(x) = -1 x -11 + 5 for all x eR
Clearly,
\x-l\ > 0 for all xeR
-1 x -11 < 0 for all x e R
=> -1 x -11 + 5 < 5 for all x e R
=> /(x) <5 for all x e R.
So, 5 is the maximum value of/(x).
Now,
f(x) =5 => -1 x - 11 + 5 =5=> | x - 11 = 0=> x =1.
Thus, /(x) attains the maximum value 5 at x = 1.
Since /(x) can be made as small as we please. Therefore the minimum value of /(x) does not
exist (see Fig. 18.8).

low
ee
rF
Fr
for
u
X' H/0) 0 (6,0)
ks

(1,0) X
Yo
oo

r
B
re

Fig. 18.8 Graph of /(x) = -|x-l| + 5

(iii) We have.
ou
ad

/(x) = sin 3x + 4 for all x e R


Y

Clearly, -1 < sin 3x < 1 for all x e R


=> -1 + 4 < sin3x + 4 <1 + 4 for all x e R
nd
Re

=> 3 < sin 3x + 4 < 5 for all x e R


Fi

=> 3 < /(x) < 5 for all x e R.


Thus, the maximum value of /(x) is 5 and the minimum value is 3.
Y

(rc/6, 5)
f(x) = sin 3x + 4
X-n/6,3) (0,4^

X' o X
— 21 7t n
2 3 6 6 3 2

Y"
Fig. 18.9 Graph of /(x) = sin 3x + 4

ReadYourFlow.COM
r 18.6 MATHEMATICS-XII

7t
Now, f(x) = 5 => sin 3x + 4 = 5 => sin 3x = 1 => 3x = ^ => X =
6
So, f(x) attains its maximum value 5 at x = —.
6
TC K
Also, f(x) = 3 => sin 3x + 4 = 3 => sin 3x = -1 => 3x = — => X = —
6
71
So, f(x) attains the minimum value 3 at x = - —.
(iv) We have, /(x) = x3 +1, x e R.
Here, we observe that the values of/(x) increase when the values of x are increased and/(x) can
be made as large as we please by giving large values to x. So, /(x) does not have the maximum
value. Similarly, /(x) can be made as small as we please by giving smaller values to x. So /(x)
does not have the minimum value also. (See Fig. 18.10).

f(x) = x3 + \

w
Flo
ee
Fr
X' O X
for
ur
ks

r
Yo

Fig. 18.10 Graph of/(x) = x3+ 1


oo
eB

(v) We have, /(x) = sin (sinx), x e R.


Clearly, -1 < sin x < 1 for all x e R
sin (-1) < sin (sin x) < sin 1 for all x e R [v sin x is an increasing function on [- 1,1]]
r
ou
ad

=> - sin 1 < /(x) < sin 1 for all x e R


This shows that the maximum value of /(x) is sin 1 and the minimum value is - sin 1.
Y

(vi) We have, /(x) = |x+3| for all x e R


nd

Clearly, | x + 31 > 0 for all x e R


Re

=> /(x) > 0 for all x e R.


Fi

So, the minimum value of /(x) is 0, which it attains at x = - 3.


Clearly,/(x) = | x + 31 does not have the maximum value. (See Fig. 18.11).
y

= |.r + 3|

X' (-3,0) O X

r
Fig. 18.11 Graph of/(x) = | x+ 3|

ReadYourFlow.COM
MAXIMA AND MINIMA 18.7

EXERCISE 18.1
LEVEL-1

Find the maximum and the minimum values, ifany, without using derivatives of the following functions:
1. f(x) = 4x2 - 4x + 4 on R 2. f(x) = - (x -1)2 + 2 on R [NCERT]
3. f(x) = | x + 21 on R 4. /(x) = sin 2x + 5 on R [NCERT]
5. f(x) =| sin 4x + 31 on R [NCERT] 6. /(x) = 2x3 + 5 on R
7. /(x) = — | x +11 + 3 on R [NCERT] 8. /(x) = 16x2 - 16x + 28 on R
9./(x) = x3 -Ion R
ANSWERS
1. Min. = 3, Max. dose not exist. 2. Max. = 2, Min. does not exist.
3. Min. = 0, Max. does not exist. 4. Max. = 6, Min. = 4.

w
5. Max. = 4, Min. = 2. 6. Max and Min. both do not exist.
7. Max. = 3, Min. does not exist. 8. Min. = 24, Max. does not exist
9. Max. and Min. both do not exist.

Flo
ee
HINTS TO NCERT & SELECTED PROBLEMS

Fr
2. We have.
/(x) =-(x—l)2, x e R for
ur
Clearly, - (x -1)2 < 0 for all x e R
=> - (x -1)2 + 2 < 2 for all x e R
ks
Yo

=> / (x) < 2 for all x e R


oo

So, f (x) attains maximum value 2 at x = l and the minimum value does not exist as
B

/ (x) can be made as small as we please.


re

4. We have.
ou

/(x) = sin 2x+5, xeR.


ad

Clearly, -1 < sin 2x < 1 for all x e R.


Y

=> 5 -1 < sin 2x 4- 5 < 1 4- 5 for all x e R


nd
Re

=> 4 </ (x) < 6 for all x e R


Fi

So, the minimum and the maximum values of / (x) are 4 and 6 respectively.
5. We have, / (x) = | sin 4x 4- 31, x e R.
We know that
-1 < sin 4x < 1 for all x e R.
3 -1 < sin 4x 4- 3 < 1 4- 3 for all x e R
=> 2 < sin 4x 4- 3 < 4 for all x e R
=> 2 < | sin 4x 4- 31 < 4 for all x e R
=> 2 < / (x) < 4 for all x e R
So, the minimum and the maximum values of / (x) are 2 and 4 respectively.
7. We have, / (x) = -1 x 4-11 4- 3, x g R
We know that
-1 x 4-1 | < 0 for all x g R => - |x 4-1| 4- 3 < 3 for all x g R => / (x) < 3 for all x g R.
So, the maximum value of / (x) is 3. As /(x) can be made as small as we please. So, the
minimum value of/(x) does not exist.

ReadYourFlow.COM
18.8 MATHEMATICS-XII

18.3 LOCAL MAXIMA AND LOCAL MINIMA


In the previous section, we have discussed about the greatest (maximum) and the least
(minimum) values of a function in its domain. But, there may be points in the domain of a
function where the function does not attain the greatest (or the least) value but the values at
these points are greater than or less than the values of the function at the neighbouring points.
Such points are known as the points of local minimum or local maximum and we will be mainly
discussing about the local maximum and local minimum values of a function.
LOCAL MAXIMUM A function f(x) is said to attain a local maximum at x = a if there exists a
neighbourhood (a - 5, a + 5) of a such that
/(*) <f(a) for all x &{a-8, a + 5), x * a.
or, f(x)-f(a) < Ofor all x e(a - 5, a + S), x * a.
In such a case, f(a) is called the local maximum value of f(x) at x = a.
LOCAL MINIMUM A function f(x) is said to attain a local minimum at x=a if there exists a

low
neighbourhood (a -§,a + 8) of a such that
f(x) > f(a) for all x e (a - 8, a + 8), x * a
or, f(x) - f(a) > 0 for all x e (a - 8, a + 8), x * a.

ee
The value of the function at x = a i.e. f(a) is called the local minimum value of/(*) at * = fl.
rF
Fr
The points at which a function attains either the local maximum values or local minimum values
are known as the extreme points or turning points and both local maximum and local minimum
values are called the extreme values of f(x). Thus, a function attains an extreme value at x = a if
for
f(a) is either a local maximum value or a local minimum value. Consequently, at an extreme
ou
point V, f(x) -f{a) keeps the same sign for all values of x in a deleted neighbourhood of a.
ks
oo

Y
Y
B
re
ou
ad
Y
nd
Re
Fi

X
O
!
Fig. 18.12

In Fig. 18.12 we observe that the x-coordinates of the points A, C and E are points of local
maximum and the values at these points i.e. their y-coordinates are the local maximum values of
/(x). The x-coordinates of points B and D are points of local minimum and their y-coordinates
are the local minimum values of /(x).
if
NOTE By a local maximum (or local minimum) value ofa function at a point x = a, toe mean the greatest ?
(or the least) value in the neighbourhood of point x = a a?id not the maximum (or the minimum) in the
domain of the function. In fact a function may have any number of points of local maximum (or local i
minimum) and even a local minimum value may be greater than a local maximum value. In Fig. 18.12 the f

minimum value at D is greater than the maximum value at A. Thus, a local maximum value may not be
the greatest value and a local minimum value may not be the least value of the function in its domain.

ReadYourFlow.COM
MAXIMA AND MINIMA 18.9

It follows from the above definition that if is a point of local maximum of a function/, then in
the neighbourhood of a the graph of / should be as shown in Fig. 18.13. Clearly, /(x) is
increasing in the left neighbourhood (a- h, a) of point a and decreasing in the right
neighbourhood oi x = a.
f' (x)>0 for x e (fl - 8, «) and, /' (x) < 0 for x e (a, a + 8)

w
O (7-5

Flo(7 (7 + 5 X

ree
Fig. 18.13

F
This suggests that /' (a) must be zero. or
ur
Similarly, if a is a point of localminimumof a function/, then in the neighbourhood of a the
sf
graph of / should be as shown in Fig. 18.14. Here, we observe that / (x) is decreasing in the left
neighbourhood (a - 8, a) of a and increasing in the right neighbourhood (a, n + 8) of a.
k
Yo
oo

/' (x) < 0 for x e (« - 5, fl) and, /' (x) > 0 for x e (a , a + 8).
B

Y
re
ou
ad
Y
nd
Re

/'(*)>0
Fi

O (7-5 (7 (7+ 5 X

Fig. 18.14

This also suggests that /' (a) must be zero.


In view of the above discussion we state the following theorem (without proof) which is known
as the necessary condition for points of local maximum or minimum.
THEOREM A necessary condition forf(a) to be an extreme value of a function f(x) is that f '(a) = 0,
in case it exists.

ReadYourFlow.COM
18.10 MATHEMATICS-XII

REMARK 1 This result states that if the derivative exists, it must be zero at the extreme points. A
function may however attain an extreme value at a point without being derivable thereat. For example, the
function f(x) =| x | attains the minimum value at the origin even though it is not derivable at x = 0.
Y

u = -x !/ = *,

X' O X

w
Fig. 18.15 Graph of/(x) = |x|
REMARK2 This condition is only a necessary condition for the point x = ato be an extreme point. It is

Flo
not sufficient i.e., /' (a) = 0 does not necessarily imply that x-ais an extreme point. There are functions
for which the derivatives vanish at a point but do not have an extreme value thereat. For example, for the

ree
function f(x) = x , /' (0) =0 but at x -0 thefunction does not attain an extreme value. In fact on the

F
left of x = 0, the curve is concave down and on its right the curve is concave up. That is, the concavity of
f(x) changes x as increases through O. Such points are called points of inflection. If x = c is a point of
or
ur
inflection of a fucntion f(x), then f"(c) = 0 and f"(x) change its sign as x increases through 'c'.
k sf
Yo
oo
B
re
ou
ad

X' O X
Y
nd
Re
Fi

Y'
Fig. 18.16 Graph of/(.t) = .v3

REMARK 3 Geometrically the above condition means that the tangent to the curve y =f(x) at a point
where the ordinate is maximum or minimum is parallel to the x-axis.
REMARK 4 As discussed in Remark 2 that all x,for -which f '{x) = 0, do not give us the extreme values.
The values of x for which f'(x) = 0 are called stationary points or turning points and the corresponding
values off(x) are called stationary or turning values off(x).
REMARK 5 The values of xfor which f'(x) = 0 or, f'(x) does not exist are known as critical points.

18.4 FIRST DERIVATIVE TEST FOR LOCAL MAXIMA AND MINIMA


In the previous section, we have seen that an extreme point (point of local maximum or
minimum) the derivative of the function either does not exist or in case it exists, it must be zero.
We have also seen that if r? is a point of local maximum value of a function /, then there exists a
neighbourhood (a -8, a + 8) of a such that
/' (x) >0 for all x e (rt - S, a) [See Fig. 18.13]

ReadYourFlow.COM
MAXIMA AND MINIMA 18.11

and, /' (x) <0 for all x e (fl , + 8).


In case, fl is a point of local minimum value of function /, then there exists a neighbourhood
(a - 5, fl + 5) of fl such that
/' (x) <0 for all x e (fl - 8, a)
and. f (x) >0 for all x e (fl , + 8) [See Fig. 18.14]
In the light of these observations, we state the following theorem (without proof) for finding the
points of local maxima or local minima.
THEOREM 1 (first derivative test) Let fbea differentiablefunction defined on an interval I and let a el.
Then,
(a) x =a is a point of local maximum value off, if
(i) /'
and, (ii) /' (x) changes sign from positive to negative as x increases through a

w
i.e. f (x) >0at every point sufficiently close to and to the left ofa, and f (x) < Ofli every point sufficiently
close to and to the right of a.

Flo
(b) x=a is a point of local minimum value off, if
(i) /»=0

ee
and, (ii) /'(x) changes sign from negative to positive as x increases through a

Fr
i.e. f (x) < 0 flf every point sufficiently close to and to the left of a, and f (x) > Oat every point sufficiently
close to and to the right of a.
for
ur
(c) Iff (a) = 0 and f (x) does not change sign as increases through a, that is, /' (x) has the same sign in
the complete neighbourhood of a, then a is neither a point of local maximum value nor a point of local
ks

minimum value. In fact, such a point is called a point of inflexion.


Yo
oo

The above theorem suggests the following algorithm to find the points to local maxima or local
B

minima of differentiable functions.


re

ALGORITHM
STEP I Put y = f{x)
ou
ad

STEP II find
Y

dx
STEPjH put ^ = 0 and solve this equationfor x. Let q, c2, c3,..., cn be the roots of this equation. Points
nd
Re

dx
Fi

C1 C2'C3' —' cn are critical points (stationary values ofx) and these are the possible points where
thefunction can attain a local maximum or a local minimum. So, we test thefunction at each one
of these poin ts.
STEPjy Consider x = q.

If — changes its sign from positive to negative as x increases through q, then thefunction attains
dx
a local maximum at x = q.
If -- changes its sign from negative to positive as x increases through q, then the function
' dx
attains a local minimum at x = q.
If — does not change sign as x increases through q, then x = q is neither a point of local
dx
maximum nor a point of local minimum. In this case x = q is a point of inflexion.
Similarly, we may deal with other values ofx.
Following examples will illustrate the above algorithm.

ReadYourFlow.COM
18.12 MATHEMATICS-XII

ILLUSTRATIVE EXAMPLES

LEVEL-1
EXAMPLE 1 Find all the points of local maxima and minima of thefunction f(x) - x3 - 6x2 + 9 x - 8.
[NCERT]
SOLUTION Let y = f(x) =x3 - 6x2 + 9 x- 8. Then,

= f'(x) = 3x2 - 12.r + 9 = 3 (a-2 - Ax + 3)


dx
dy = 0.
The critical points of f(x) are given by f'{x) = 0 or, —
dx
Now, ^- = 0 => 3 (a-2 - 4a- + 3) = 0 => x = 1, 3.
dx

low
We have to examine whether these points are points of local maxima or local minima or neither
of them.
dy
We have, — = 3 (x -1) (x - 3)
dx

ee
dy
The changes in signs of — for different values of x are shown in Fig. 18.17.
rF
Fr
dx
+ for +
- CO CO
1 3
f/ for different values of .v.
ou
Fig. 18.17 Signs of
dx
ks

dy
oo

Clearly, changes sign from positive to negative as increases through 1.


Y
B

So, x = 1 is a point of local maximum.


re

Also, ^ changes sign from negative to positive as x increases through 3.


ou
ad

So x = 3 is a point of local minimum.


Y

EXAMPLE 2 Find all the points of local maxima and local minima as well as the corresponding local
nd
Re

maximum and local minimum values for the function f(x) = (x -l)3 (x + l)2.
Fi

SOLUTION Let y = /(x) = (x -1)3 (x + l)2. Then,


dy = 3 (x -1)2 (x +1)2 + 2 (x +1) (x -1)3
dx
=> ^ = (x -l)2 (x + 1) {3 (x + 1) + 2 (x -1)}
dx
=> dy = (x -1)2 (x + 1) (5x + 1).
dx
At points of local maxima or local minima, we must have
it = 0 => (x-1)2 (x + 1) (5x + 1) =
0 =i> x = 1 or, x = -1 or, x = - -
dx 5
Now, we have to examine whether these points are points of local maximum or local minimum
or neither of them.
Since (x -1)2 is always positive, therefore the sign of — is same as that the (x + 1) (5x + 1).
dx

ReadYourFlow.COM
MAXIMA AND MINIMA 18.13

dy
The changes in signs of — for different values of x are shown in Fig. 18.18.
dx
+
-o -o
+ +
- CO oo
5
dy
Fig. 18.18 Signs of —for different values of x.
dx

Clearly, — does not change its sign as x passes through 1. So x =1 is neither a point of local
dx
maximum nor a point of local minimum. In fact, x = 1 is a point of inflexion.
Clearly, — changes sign from positive to negative as x passes through -1.
dx
So, x = -1 is a point of local maximum.
The local maximum value of/(x) at x = -1 is /(-1) = (- 2)3 (-1 + l)2 = 0.

w
It is evident from Fig. 18.18 that — changes sign from negative to positive as x passes through
dx
-1 /5. So, x = -1/5 is a point of local minimum.

Flo
ree
3456
The local minimum value of/(x) at x
3125

F
EXAMPLES Find all the points of local maxima and local minima of the function
or
ur
/(x)=x3 -6x2 + 12x-8.
sf
SOLUTION Let y =/(x) = x3 - 6x2 + 12x - 8. Then,
k
Yo

^ = 3x2 -12x + 12 = 3 (x - 2)2


oo

dx
B

The critical points of y = /(x) are given by ^ = 0.


dx
re

dy = 0 => 3 (x - 2)2 = 0 => x = 2. + +


Now
ou
ad

dx — CO 2 CO

We observe that du
Y

Fig. 18.19 Signs of — for different values of x.


— = 3 (x-2)2 > 0 for all x ^ 2. dx
nd

dx
Re

dy does not change sign as x increases through x = 2.


Fi

Thus,
dx
Hence, x = 2 is neither a point of local maximum nor a point of local minimum. In fact, it is a
point of inflexion.
EXAMPLE 4 Shoiv that the function /(x) =4x3-18x2+ 27x-7 has neither maxima nor
minima. INCERT EXEMPLAR]
SOLUTION We have,
y =/(x) =4x3 -18x2 + 27x-7
^ = 12x2 - 36x + 27 = 3 (4x2 -12x + 9) = 3 (2x - 3)2
dx
The critical points of y = /(x) are given by ^ = 0.

Now, —=0 3 (2x - 3)2 = 0=> 2x-3=0=> x = —


dx 2

ReadYourFlow.COM
18.14 MATHEMATICS-XM

Clearly, - = 3(2x-3)2 > 0 for all x* —


dx 2
+ +
- CO 3/2 CO

diJ
Fig. 18.20 Signs of — for different values of .r.
dx

Thus, — dy does not change its sign as x increases through x = 3/2 as shown in Fig. 18.20. Hence,
dx
x = 3/2 is neither a point of local maximum nor a point of local minimum. In fact, it is a point of
inflexion.
EXAMPLE 5 Find the points of local maxima, local minima and the points of inflection of the function
f(x) = .y5 -5.t4 +5x3 -1. Also, find the corresponding local maximum and local minimum values
[NCERT EXEMPLAR]

w
SOLUTION Let y =f(x) = x5 -5x4+5x3-1. Then,
^ =5x4 -20x3 + 15x2 =5x2 (x2 -4x + 3) =5x2 (x -1) (x - 3)

Flo
dx
The critical points of y = f(x) are given by — = 0.

ee
dx

Fr
Now, = 0=> 5x2 (x -1) (x - 3) = 0 => x = 0, x = 1, x - 3.
dx for
ur
Clearly, —dy does not change its sign as x increases through 0. So, x = 0 is a point of inflection.
dx
dy
ks

It is evident from Fig. 18.21 that — changes its sign from positive to negative as x increases
Yo

dx
oo

through 1. So, x = 1 is a point of local maximum.


eB

+ + +
-o-
- CO 0 1 3 CO
r

di> for different values of x.


ou
ad

Fig. 18.21 Signs of —


dx
Y

The local maximum value of/(x) is/(l) =1-5+ 5-1=0.


nd
Re

We observe, from Fig. 18.21, that — changes its sign from negative to positive as x increases
dx
Fi

through 3. So, x = 3 is a point of local minimum.


The local minimum value of/(x) is/(3) = 35 - 5 x 34 + 5 x 33 -1 = -28.
EXAMPLE 6 Find the local maxima or local minima, if any, of the function f(x) = sin x + cos x,
0 < x < -^ using the first derivative test. [NCERT]
SOLUTION We have,
y =/(x) = sin x + cos x => — = cos x - sin x
dx
The critical points of i/ = /(x) are given by — = 0.
dx
Now, — = 0 => cos x - sin x = 0 => tan x = 1 => x = — v 0<x<-
dx 4 L 2 J
Now, we will see whether x = ^ is a point of local maximum or a point of local minimum or none
of these.

ReadYourFlow.COM
MAXIMA AND MINIMA 18.15

In the left neighbourhood x = , we have

x < — => cos x > sin x => cos x - sin x > 0 => — > 0
4 dx
In the right neighbourhood of x = ^ , we have

7C . „ dy
x > — => cos x < sin x =i> cos x - sin x < 0 => <0
4 dx
Thus, — changes its sign from positive to negative as x increases through —. So,/(x) attains a
dx 4
local maximum at x = —.
4
EXAMPLE 7 Find the local maximum or local minimum, ifany, of thefunction f(x) = sin 4 x + cos4 x,

w
0<x<! using the first derivative test.

SOLUTION We have,
y = f(x) = sin4 x + cos4 x

Flo
ee
dy o o

Fr
=> ---- = 4 sin x cos x - 4 cos x sin x
dx
dy
= - 4 cos x sin x (cos2 x - sin 2 x) = - 2 sin 2x cos 2x = - sin 4x
for
ur
=>
dx
ks

The critical points of y -f(x) are given by = 0.


Yo
oo

Now, ^ = 0
B

dx
re

=> - sin 4x = 0
=> sin 4x = 0
ou
ad

=> 4x = re v 0<x<- 0 < 4x < 2rc


Y

2
n
nd
Re

=> x =
4
Fi

In the left neighbourhood of x = ^, we have

x < - => 4x < 3i => sin 4x > 0 => - sin 4x < 0 —<0
4 dx
In the right neighbourhood of x = ^, we have

x > — => 4x>7r=> sin4x<0=> -4sin4x>0=> —>0


4 dx
Thus, — changes sign from negative to positive as x increases through —. So, x = — is a point of
dx 4 4
local minimum.
The local minimum value of / (x) at x is / ^ j = ^sin + ^cos ^ j = ^—.
2

ReadYourFlow.COM
18.16 MATHEMAT1CS-XII

EXAMPLE 8 Find the points at which the function f given by f (x) = (x - 2)4 (x + 1)3 has
(i) local maxima (ii) local minima (iii) points of inflexion [NCERT1
SOLUTION We have,
/(x) = (x - 2)4 (x + l)3
=> /' (x) = 4 (x - 2)3 (x + l)3 + 3 (x - 2)4 (x + l)2
=> /'(x) = (x - 2)3 (x + l)2 (7x - 2)
/'(x) = (x-2)2(x + 1)2(x-2)(7x-2)
The critical points of/(x) are given by /' (x) = 0.
Now, /' (x) = 0 => x =

Since (x - 2) (x +1) is always positive. So, sign of /' (x) depends upon the sign of
(x - 2) (7x - 2). The changes in signs of /' (x) as x increases through 2/7 and 2 are shown in
Fig. 18.22.

w
— 00
-1 1 2 CO

Flo
Fig. 18.22 Signs otf'{x) for different values of .y.

ree
Clearly, /' (x) changes its sign from positive to negative as increases through 2/7.
2

F
So, x = - is a point of local maximum.
or
ur
We observe that/'(x) changes its sign from negative to positive as x increases through 2.
sf
So, x = 2 is a point of local minimum.
There is no change in the sign of /' (x) as increases through -1. So, x = -1 is a point of inflexion.
k
Yo
oo

EXERCISE 18.2
B

LEVEL-1
re

Find the points of local maxima or local minima, if any, of the following functions, using the first
ou
ad

derivative test. Also, find the local maximum or local minimum values, as the case may be:
l./(x) = (x-5)4 2. /(x) = x3 - 3x
Y

INCERT1
3. /(x)=x3 (x — l)2 4. /(x) =(x -1) (x + 2)2
nd
Re

1 6. /(x)=x3 - 6x2 + 9x + 15
5./(x) = [NCERT]
Fi

x2 + 2
7. /(x) = sin 2x, 0 < x < tt 8. /(x) = sin x - cos x, 0 < x < 271 [NCERT]
K
9. /(x) = cos x, 0 < x < Tt 10. /(x) = sin 2x - x, - ^ < x < -
2
n tc
11. /(x) = 2 sin x - x, - — < x < - 12. / (x) = x yjl -x , x > 0 [NCERT]
2
13./(x) = x3 (2x -l)3 14. / (x) = - + - , x > 0
J 2 x
ANSWERS
1. x = 5 is a point of local minimum, local minimum value = 0.
2. x = -1 is a point of local maximum, local maximum value = 2
x = 1 is a point of local minimum, local minimum value = - 2.
3. x = 1 is a point of local minimum, local minimum value = 0
3. 108
x = — is a point of local maximum, local maximum value =
3125

ReadYourFlow.COM
MAXIMA AND MINIMA 18.17

4- x = 0 is a point of local minimum, local minimum value = - 4


x = - 2 is a point of local maximum, local maximum value = 0.
5- Local maximum at x = 0, Local maximum value = —
2
6. x = 1 is a point of local maximum, local maximum value = 19
x = 3 is a point of local minimum, local minimum value = 15.
7. x = ^ is a point of local maximum local maximum value = 1

371
x = — is a point of local minimum local minimum value = -1.

8. x = is a point of local maximum, local maximum value = V2


*7 7C I—
x = — is a point of local minimum, local minimum value = - V2.

low
9. None in the interval (0,7t)
10. x = is a point of local maximum, local maximum value = ^

ee
7t ^
x = — - is a point of local minimum, local minimum value = ——
rF
Fr
6
11. x = — is a point of local maximum, local maximum value = \f3 - —
3 3
for
x = —— is a point of local minimum, local minimum value = -V3 + ^.
ou
3 3
ks

2 2 V3
12. Local maximum at x = — , Local Maximum value = —
oo

3 9
Y
B

1 1
13. Minimum at x = — , Local Minimum value = -
re

4' 512
14. Minimum at x - 2, Local Minimum value = 2
ou
ad

HINTS TO NCERT& SELECTED PROBLEMS


Y

2. We have.
d
Re

/(x) = x3 -3x
n
Fi

=> f'(x) = 3x2-3 = 3(x -1) (x + 1)


The critical points of/(x) are given by/' (x) = 0.
Now, /'(x) = 0 => 3(x -1) (x + 1) = 0 => x = l,-l.
Hie changes in signs of /' (x) for different value of x are as shown in Fig. 18.23.
+
- CO CO
-1 1
Fig. 18.23 Signs of /'(x) for different values of x.

Clearly, /' (x) changes its sign from positive to negative as x increases through -1. So,
x = -1 is a point of local maximum with the local maximum value given by
/(-l) = (-l)3-3(-l) = 2.
As /' (x) changes its sign from negative to positive as x increases through 1. So, x = 1 is a
point of local minimum with the local minimum value / (1) = 1 - 3 = - 2.

ReadYourFlow.COM
f

18.20 MATHEMATICS-XII

Iff IV (c-}) < 0, then x = Cj is a point of local maximum and iff (cf) > 0, then x = Cj is a
point of local minimum.
Iff IV (c-j) = 0, we mustfindf v (x), and so on. Similarly, the points c2, c3 may be tested.
POINT OF INFLECTION An arc ofa curve y -f(x) is called concave upward if, at each of its points, the arc
lies above the tangent at the point (see Fig. 18.26).
If y = f(x) is a concave upward curve, then as x increases, f '(x) either is of the same sign and
increasing (see Fig. 18.26) or changes sign from negative to positive (see Fig. 18.27). In either case
f '(x) is increasing and so f "(x) > 0. Thus, for a concave upward curve / "(x) > 0.
An arc of a curve y = f(x) is called concave downward if, at each of its points, the arc lies below the
tangent at the point.
y y

low
o x o x

ee
rF
Fr
Fig. 18.26 Fig. 18.27

If an arc of a curve y = f(x) is concave dozvmvard, then as x increases, / '(x) either is of the same
for
sign and decreasing (see Fig. 18.28) or changes sign from positive to negative (see Fig. 18.29). In
ou
either case/ '(x) is decreasing and so / " (x) < O.Thus, for a concave downward curve/ " (x) < 0.
ks

y, y,
oo
Y
B

\
re
ou
ad
Y
nd

o o
Re

X X
Fi

Fig. 18.28 Fig. 18.29

POINT OF INFLEXION A point of inflection is a point at which a curve is changing concave upward to
concave downward, or vice-versa.
y,

o x
Fig. 18.30

ReadYourFlow.COM
MAXIMA AND MINIMA 18.21

A curve y =f(x) has one of its points x = c as an inflection point, if


(i) / "(c) = 0 or is not defined and
(ii) / "(x) changes sign as x increases through x - c.
The later condition may be replaced by / '"(c) ^ 0 when/'"(c) exists.
Thus, x = c is a point of inflection iff "(c) = 0 and f '"(c) *0.
PROPERTIES OF MAXIMA AND MINIMA
(i) If f(x) is continuous function in its domain, then at least one maximum or one minimum
must lie between two equal values off(x).
(ii) Maxima and Minima occur alternately, that is, between two maxima there is one
minimum and vice-versa.
(iii) If f(x) co as x -> a or b and /' (x) =0 only for one value of x (say c) between a and b, then
/(c) is necessarily the minimum and the least value.
If f(x) - co as x -» fl or b, then f(c) is necessarily the maximum and the greatest value.

w
ILLUSTRATIVE EXAMPLES

LEVEL-1

Flo
EXAMPLE 1 Find all the points of local maxima and minima and the corresponding maximum and

ee
minimum values of thefunction f(x) = — x4 - 8 x3 - — x2 +105.

Fr
SOLUTION We have, for [NCERT EXEMPLAR]
ur
— x4 -8 x3
/(*)=-! x2 +105
2
ks

=> / '(x) = - 3x3 - 24x2 - 45x = - 3x (x2 + 8 x + 15)


Yo
oo

The cirtical points of/(x) are given by / ' (x) = 0.


B

Now, / '(x) = 0 => - 3x (x2 + 8 x + 15) = 0 => - 3 x (x + 3) (x + 5) = 0 => x = 0, - 3, -5


re

Thus, x = 0, x = - 3 and x = - 5 are the possible points of local maxima or minima.


Let us now test the function at each of these points.
ou
ad

Now, /' (x) = - 3x3 - 24x2 -45x


Y

=> /"(x) = - 9 x2 - 48 x - 45
nd
Re

At x = 0: We have,
Fi

/"(0) =-45 <0


So, x = 0 is a point of local maximum.
The local maximum value of /(x) at x = 0 is /(0) = 105.
At x = - 3: We have,
/ "(- 3) = - 9 (- 3)2 - 48 (- 3) - 45 = 18 > 0
So, x = - 3 is a point of local minimum.
The local minimum value of/(x) at x = - 3 is
/(-3)=-|(-3)4-8(-3)3-^(-3)2 + 105 = ^

Atx = - 5: We have,
/ "(- 5) = - 9 (- 5)2 - 48 (- 5) - 45 = - 30 < 0
So, x = - 5 is a point of local maximum.

ReadYourFlow.COM
18.22 MATHEMATICS-XII

The local maximum value of f(x) at x = - 5 is


/(-5)=-|(-5)4-8(-5)3-f(-5)2+ 105 = ^5
4 2 4
EXAMPLE 2 Find all the points of local maxima and minima and the corresponding maximum and
minimum values of thefunction f(x) = 2x -21 x + 36x - 20.
SOLUTION We have,
f(x) = 2x3 -21 x2 + 36x-20
=> f'(x) = 6x2 - 42x + 36
The critical points of f(x) are given by /' (x) = 0.
Now, f(x) = 0 => 6x2 -42x+ 36 = 0 => (x -1) (x - 6) = 0 => x = 1, 6.
Thus, x = 1 and x = 6 are the possible points of local maxima or minima.
Now, we test the function at each of these points.

low
We have,/"(x) = 12x-42
Atx = l: We have,
/"(l) = 12-42=-30 <0

ee
So, x = 1 is a point of local maximum.
rF
Fr
The local maximum value is /(l) = 2-21 + 36-20 = - 3
At x = 6: We have, for
/ "(6) =12 (6)-42 = 30 >0
ou
So x = 6 is a point of local minimum.
ks

The local minimum value is /(6) = 2 (6)3 - 21 (6)2 + 36 x 6 - 20 = -128.


oo

EXAMPLE 3 Find the points of local maxima and local minima, ifany, of each of thefollowing functions.
Y
B

Find also the local maximum and local minimum values, as the case may be:
re

(i) /(x) = sin 2x - x, zvhere ~^<x (ii) /(*) = sin x + ^ cos 2x, where 0 < x < ^
ou
ad

(iii) /(x) = sin4 x + cos4 x, 0 < x < -^


Y

SOLUTION (i) We have,


nd
Re

/(x) = sin 2x - x
Fi

=> /' (x) = 2 cos 2x -1


The critical points of/(x) are given by/'(x) = 0.
/'(*) = 0
=> 2 cos 2x -1 = 0
=> cos 2x = —
2
U K Inn
2x = ----or, 2x = — V----- < X < - - TC < 2x < 71
3 3 L 2 2
TC 7t
=> X — or, x =
6 6

Thus, x = —- and x = ^ are possible points of local maxima or minima.


6
Now, we test the function at each of these points.
Clearly, f"(x) = -4sin 2x.

ReadYourFlow.COM
MAXIMA AND MINIMA 18.23

Atx = -n/6: We have.


-Vs = 2 V3 > 0.
rtt) = -4 sin
K

3
= -4x
2

So, x = — ^ is a point of local minimum.

The local minimum value is = + ^ =: Vs K


"T’+6

At x = k/ 6: We have,
=- 4f = - 2 VS < 0.
3 I 2
So, x = is a point of local maximum.

w
n TC _ VS K
The local maximum value is / ^ j = sin----
3 6 ~~T~ 6
(ii) We have.

Flo
ee
/(*) = sin x + — cos 2x, where 0 < x < —

Fr
2 2
=> /'(x) = cos x - sin 2x. for
The critical points of /(x) are given by /' (x) = 0.
ur
f'(x) = 0
=> cos x - sin 2X = 0
ks
Yo

=> cos x - 2 sin x cos x = 0


oo

=> cos x (1 - 2 sin x) = 0


B

=> cos x = 0 or, 1 - 2 sin x = 0


re

TC TC
=> cos x = 0 or, sin x = — => x = — or x = — 0< x<-
2 2 6 2
ou
ad

Thus, x = — is a point of local maximum or local minimum.


Y

6
Now, we test the function at this point.
nd
Re

Clearly, / "(x) = - sin x - 2 cos 2x


Fi

n r, n
'■(3- - sin----2
6
cos — =
3
— -2x — =
2 2 2
<0

So, x - ^ is the point of local maximum.

The local maximum value is / n 1


j = sm - TC 1 1/1 3
+ — cos — = — + — | -
6 2 3 2 2v2 4
(iii) We have.
/(x) = sin4 x + cos4 x, where 0 < x < ^.

/'(x) = 4sin xcosx-4cos xsinx


=> / '(x) = - 4 cos x sin x (cos x - sin x) - - 2 sin 2x cos 2x = - sin 4x.
At points of local maximum and minimum, we must have
/'(x) = 0
=i> - sin 4x = 0

ReadYourFlow.COM
r-

18.24 MATHEMATICS-XII

4x = 71 v 0 <x <— 0 < 4x < 2tc


2
=> X = 7l/4
Now, /"(x) = -4 cos 4x

71
'"(3 = -4 cos 71 = (- 4) (-1) = 4 > 0

So, x = — is a point of local minimum and the local minimum value is

= sin — + cos 4 rc
. 4 71
4 4
if+fIV2J
W2j
if=l+l
4 4
= i.2
EXAMPLE 4 Find the points oflocal maxima or local minima, if any, of the followingfunctions. Find also
the local maximum or local minimum values, as the case may be:

low
(i) /(x) = sin x + cos x, where 0 < x < -^

(ii) f(x) = sin x - cos x,zuhere 0 <x <2n [CBSE2015]


(iii) /(x) = sin 2x, zuhere 0 <x <n

ee
(iv) /(x) = 2 cos x + x, where 0 < x < tt
rF
Fr
71
(v) f(x) = 2 sin x - x, -- < x < -.
2 for
SOLUTION (i) We have.
ou
/(x) = sin x + cos x, where 0 < x < -^
ks

/'(x) = cos x-sin x.


oo

The critical points of/(x) are given/'(x) = 0.


Y
B

/'(*) = 0
re

=> cos x - sin x = 0 => cos x = sin x => tan x = 1 => x = - .• 0 < x < -
4 2
ou
ad

Thus, x = -^ is a point of local maximum or minimum.


Y
nd

Now, f"(x) = - sin x - sin - - cos — = - V2 < 0.


Re

4 4
Fi

So, x = ^ is a point of local maximum.


The local maximum value is
n K 1 1
4=
sm - + cos - = -7= + -7= = —f= = V2.
4 4 V2 V2 V2
(ii) We have.
/(x) = sin x - cos x, where 0 < x < 2ti
=> / ' (x) = cos x + sin x
At points of local maximum and local minimum, we must have
fix) = 0
=> cos x + sin x = 0
3 7t 771
=> sin x = - cos x => tan x = -1 => x = ---- or, x = [v 0 < x < 2ti]
4 4
3 71 7k
Thus, x = —- and x = — are possible points of local maximum or minimum.
4
Now, we test the function at each of these points.

ReadYourFlow.COM
MAXIMA AND MINIMA 18.25

Clearly, f"{x) = - sin x + cos x.


At x = 3n/4: We have,
3K j___L = _
/
3 K
- sin---- + cos----- =
4 -Jl y[2~
5=<o.
\2
4 4
3n
So, x = is the point of local maximum.

The local maximum value is / j= 3 71 3 71 1 1 2


sin 1 - cos = -A= + = -r= = 4l.
4 4 >/2 V2 V2
At x=7rc/4: We have.
7k 7k 1 1 2
/" = - sin — + cos — = -?= + = —f= > 0.
4 4 V2 V2 V2
771
So, the function attains a local minimum at x = —.

w
4
. 7k 7k 1 1
The local minimum value is = sm------ cos — - yfl.
4 4 V2 V2
(iii) We have.

Flo
ree
f(x) = sin 2x, where 0 < x < ti.
/' (x) = 2 cos 2x.

F
At points of local maximum or local minimum, we must have
or
ur
/'(*) = o
sf
2 cos 2x = 0
=> cos 2x = 0
k
Yo

3 TI
oo

=> 2x = — [•.• 0 < x < k 0 < 2x < 27r]


2' 2
B

71 37t
=> X = - , ---- .
re

4 4
are possible points of local maximum or local minimum.
ou

Thus, x = — and x =
ad

4
Y

Now, we test the function at these points.


Clearly, /"(x) = -4sin 2x
nd
Re

At x = 7i/4: We have,
Fi

'"ti)
7C
= - 4 sin — = - 4 < 0.
2

So, x = — is a point of local maximum.

The local maximum value of/(x) is / = sin — = 1.


2
Af x = 3 tt/4: We have,
,/ 3 K 3 K
/ = - 4 sin---- = 4 > 0.
4 2
3 71
So, x = is a point of local minimum.

3k
The local minimum value of/(x) is = sin — = -1.
2
(iv) We have,
/(x) = 2 cos x + x, where 0 < x < 7t.
=> /' (x) = - 2 sin x + 1

ReadYourFlow.COM
18.26 MATHEMATICS-XII

At points of local maximum and minimum, we must have


/'(-v) = 0
I Tl 5 X
=> - 2 sin A' + 1 = 0 => sin x = — => x — [v 0 < A' < tc]
2 6 6
Thus, x = n and x - 5— n: are possible points of local maximum or minimum.
6 6
Now, we test the function at these points.
Clearly, f"(x) = -2 cos A'
Atx = n/6: We have,
= - 2 cos 71 = - V3 < t)
6

So, x = - is a point of local maximum. The local maximum value of f(x)is

low
6
o 7t 7t /^ 71
= 2 cos — + - = v3 +
6 6 6
At x =5n/6 : We have,

ee
/" = - 2 cos — = V3 > 0
rF
Fr
6
5n
So, a = - is a point of local minimum. for
ou
The local minimum value of/(a) is /1 j - 02 cos 5rt 571 rr- 571
— + — =-V3 + —.
ks

6 6 6
oo

(v) We have.
Y

f{x) = 2 sin x - x, where - n/2 < x < n/2.


B

f'(x) = 2 cos A 1
re

The critical points of/(a) are given by /' (a) = 0.


ou
ad

/'(a) - 0 2 cos a- I 0 => COS A - 1/2 => A = ±71/3 [■.• - 7t/2 < A < 7t/2]
Y

Thus, a = ± rt/ 3 are points of local maximum or minimum.


Clearly,/ "(a) = - 2sin a.
nd
Re

At a = - n/ 3: We have,
Fi

= -2 sin (-n/3) - 2 sin ti/3 = 2V3/2 = V3 > 0


So, a - n:/3 is a point of local minimum.
The local minimum value is / (- tx/3) = 2 sin ( tc/3) -( tt/3) = - V3 + ti/3.
At x ~n/3: We have,
f"(n/3) = -2 sin Tt/3 = -V3 <0
So, a = tt/3 is a point of local maximum.
The local maximum value is f(n/ 3) = 2 sin tt/3 - rt/ 3 = V3 - x/ 3.
example 5 Find the local maximum and local minimum values of
/(a) = sec A + log COS A, 0 < A < 27T. [NCERT EXEMPLAR]
SOLUTION We have.
/(a) = sec A + 2 log cos a
/'(a) = sec a tan a - 2 tan a = tan a (sec a - 2)
The critical points of/(a) are given by/' (a) = 0.
/'(*) =0
tan a (sec a - 2) = 0

ReadYourFlow.COM
MAXIMA AND MINIMA 18.27

=t> tan x = 0 or sec x = 2


I
=> tan x = 0 or cos x = -
2
n 5n
x = n, x = [v 0 < x < 2n]
3'"3
Thus, x = rt, a: = are possible points of local maximum and local minimum.
3 3
Now, /(x) - tan x (secx - 2)
=> /'' (x) = sec2 x (secx - 2) + tan 2 x secx
=t> /''(x) = sec2 x (secx - 2) + secx (sec2 x -1)
=> /" (x) = 2 sec 1 x - 2 sec2 x - sec x
Let us now investigate critical points for points of local maximum and local minimum.

low
At x = —: We have,
3
f"(x) -2 sec 3 x - 2 sec2 x - sec x

f(f) = 2 sec 3 —3 - 2 sec2 -3 -sec —3 - 2x8-2x4-2 = 6>0

ee
rF
Fr
Thus, x = — is a point of local minimum. The local minimum value of /(x) is given by
3
/ f—l = sec— + log cos2 — = 2 -+- log - = 2 - 2 log 2 for
\ 3/ 3 3 4
ou
Atx = n: We have,
ks

/" (x) = 2 sec x - 2 sec x -• sec x


oo

/" (n) = 2 sec3n - 2 sec27i - sec 7t = ~2 - 2 +1 = -3 < 0


Y
B

Thus, x = k is a point of local maximum. The local maximum value of /(x) is given by
re

/(7c) = sec tc+ log cos" re = - 1 + log I I.


ou
ad

At x = : We have,
3
Y

f" (x) = 2 sec3 x - 2 sec2 x - sec x


d

2 5tc
= 2 sec 35* - 2 sec
'•{f)
Re

sec 2(2)3 2 (2)2 + 2 =!()>()


n

3 3 3
Fi

5 71
Thus, x = — is a point of local minimum.
3
The local minimum value is given by
5 71 5• k<v . 2 571 „ . I
= sec 3 +log COS - --- 2 + log 4 2 2 log 2
At
EXAMPLE 6 Show that none of the following functions has a local maximum or a local minimum:
(i) x3+x2+ x + l (ii) ex P) logx (iv) cos x, 0 < x < 7t [NCERT1
SOLUTION (i) Let / (x) = x3 + x2 + x + 1. Then, /' (x) = 3x2 + 2x + 1.
At points of local maximum or minimum, we have
/'(*) = 0 => 3*2 + 2* + 1 = 0
But, this equation gives imaginary values of x. So,/' (x) * 0 for any real value of x.
Hence, / (x) does not have a maximum or minimum.
(ii) Let / (x) = ex. Then, /' (x) = ex. Clearly, /' (x) * 0 for any value of x.
So, / (x) = ex does not have a maximum or a minimum.

ReadYourFlow.COM
18.28 MATHEMATICS-XII

1
(iii) Let / (x) = log x. Then, /' (x) = —. Clearly, /' (x) ^ 0 for any value of x e Domain (/).
So, / (x) = log x does not have a maximum or a minimum.
(iv) Let/(x) = cos x. Then, / '(x) = - sin x. Clearly,/'(x) 0 for any x e (0, tc).
So,/(x) = cos x does not have a maximum or minimum on (0, tt).
EXAMPLE 7 Find the maximum profit that a company can make, if the profit function is riven
P(x) = 41 + 24x - 18x2. [NCERT]
SOLUTION We have,
P(x) = 41 + 24x -18x2
j2

i(PW) = 24-36 x and —; P (x) I = - 36


dx2
For maximum or minimum, we must have

low
Also,
£Pw) = 0 => 24 - 36x = 0 => x = -
3

2
= - 36 < 0. So, profit is maximum when x = —.

ee
dx2 3
J x = 2/3
rF
Fr
Maximum profit = (Value of P(x) at x = 2/3) =41 + 24x (2/3) -18 (2/3)2 =49
for
EXAMPLE 8 At what points, the slope of the curve y =-x3 + 3x2 + 9x - 27 is maximum ? Also,find the
maximum slope.
ou
[NCERT EXEMPLAR1
ks

SOLUTION The slope m of the curve y = -x3 + 3x2 + 9x - 27 at point (x, y) is given by
oo

m= = -3x2 + 6x + 9
dx
Y
B

dm d2m
re

= -6x + 6 and = -6
dx dx2
ou

For maximum or minimum values of w, we must have


ad
Y

= 0=> -6x+6 = 0=>x=l


dx
d2m
nd
Re

Clearly, — - 6 < 0 for all x.


dx2
Fi

So, m is maximum at x = 1. Putting xo = 1 ino (i), we obtain m = 12.


Putting x = 1 in the equation y =-x + 3x + 9x -27, we obtain y = -16.
Hence, the slope of the given curve is maximum at the point (1,-16) and the maximum value of
the slope is 12.
EXAMPLE 9 Iff (x) = a log | x | + bx + x has extreme values atx = -1 and at x = 2, then find a and b.
SOLUTION We observe that/(x) is defined for all x ^ 0.
Now,
/ (x) = fl log | x | + fox2 + x => /' (x) = - + 2bx + 1
x
It is given that f(x) has extreme values at x = -1 and x = 2.
/'(-I) =0 and /'(2) =0
=> -fl-2fo + l = 0 and — + 4fo + l =0=>fl + 2fo =1 and a + 8b = -2
2
Solving these equations, we get: a = 2 and fo = -1/2.

ReadYourFlow.COM
MAXIMA AND MINIMA 18.29

EXAMPLE 10 It is given that atx=l, thefunction x4 - 62x2 + ax + 9 attains its maximum value on the
interval [0,2]. Find the value ofa. [NCERT]
SOLUTION Let f(x) =x4 - 62x2 + ax + 9. Then,
f '(x) = 4x3 -124x + a.
It is given that/(x) attains its maximum at x = 1.
/ ' (1) = 0 => 4 -124 + a = 0 => a = 120
LEVEL-2

ax -b
EXAMPLE 11 Ify- has a turning point P (2, -1), find the values of a and b and show
(x-1) (x -4)

w
that y is maximum at P.
SOLUTION We have,
ax -b ax -b
y (x -1) (x - 4) x2-5x +A

Flo
ree
dy _ (x2 — 5x + 4) «-(ax-fr) (2x-5)
=> ...(ii)

F
dx (x2 -5x + 4)2
(4 -10 + 4) a -(2a -b) (4-5) b or
ur
=> dy)
sf
dx)P (4-10 + 4)2 4
k

Since P is a turning point of the curve (i). Therefore,


Yo
oo

dy
= 0 => - = 0 => b = 0 ...(hi)
B

dx Jp 4
re

ax -b
Since P(2, -1) lies on y = . Therefore,
(x -1) (x -4)
ou
ad

2a-b 2a-b
Y

-1 = => -1 => 2a -b =2 ...(iv)


(2-1) (2-4) -2
nd
Re

From (iii) and (iv), we get a = l,b =0.


Fi

Substituting the values of a and b in (ii), we get


dy _ (x2 -5x + 4) - x(2x -5) - x2 + 4
dx (x2 -5x + 4)2 (x2 -5x + 4)2
d2y _ (x2 -5x + 4)2(- 2x)-(-x2 +4) 2(x2 -5x + 4) (2x-5)
=>
dx2 (x2 -5x + 4)4
d2y _ - 2x (x2 - 5x + 4) + 2 (x2 - 4) (2x - 5)
dx2 (x2 -5x + 4)3

dl d y (-2) (-4)
Now, = 0 and. = -1 <0
d*2(2,-l) [dx2 2(2,-1) (-2)3

So, y is maximum at P when a = \ and b =0.


EXAMPLE 12 Show that the maximum value of j is e l/e

ReadYourFlow.COM
18.30 MATHEMATICS-XII

SOLUTION Let y = 1 = x A. Then,

log y = - x log x
\ dy _
=> = -(1 + log x) [Differentiating with respect to x]
y dx
dy =
=> - y (i + log x)
dx

=> ~~2 = -^a + logx)-^ = y (1 + log x)2 - — [Differentiating with respect to x]


dx dx x X

=> d-~ = X~x (1 + log x)2 - — = x x (l + log x)2 - x x 1


dx x

low
At points of local maximum and local minimum, we must have
dy = 0 => - y (1 + log x) = 0 => 1 + log x = 0 => log x = -1 => x = c-1 = i
uX g

Now,

ee
( ,2
d y if (iyl/e~l
rF
Fr
dx2
'x=\/e
for
= {e~l)~l/e (1-log e)2-{e-lrl/e-l = - e e_1+ 1 <0
[dx2 'x=\/e
ou
ks

So, x = 1/e is a point of local maximum. The local maximum value of y is obtained by putting
oo

x = l/e iny and is equal to


Y
B

EXAMPLE 13 Show that smf’ 0 cos'? 0 attains a maximum, when 0 = tan 1


re

<7
ou
ad

SOLUTION Let y = sin^’ 0 cos1? 0. Then,


rfy
Y

= p sin,?_1 0 cos 0 cos1? 0 + sin?’ Q q cos1?-1 0 (- sin 0)


nd
Re

=> = psmP-lQ cos ^ 1 0-^sinP+ 1 ©cos1?-1 0


dQ r
Fi

=> dy_ . in?’-1 0 cos1?-1 0 (p cos2 0 - g sin2 0)


d0
A = sinPe^ef^29-?^26
=> = sin?’ 0 cos1? 0 (p cot 0 - ^ tan 0)
dQ sin 0 cos 0
For maximum or minimum, we must have
=o
dQ

sin?’ 0 cos1? 0 (p cot Q - q tan 0) = 0


=> sin?’ 0 = 0 or, cos'? 0 = 0 or, p cot 0 - 9 tan 0 = 0

=> sin?’ 0 = 0 or, cos1? 0 = 0 or, tan 0 = |^

=> 0 = 0 or, 0 = tt/2 or, 0 = tan -1 Jp7q = a (say)

ReadYourFlow.COM
MAXIMA AND MINIMA 18.31

Now, — = sm.p 0 cos^ 0 (p cot 0 - ^ tan 0)


dQ

=> = y(p cot Q -c] tan 0)


d6

=> cot Q - q tan 0) + y(-p cosec2 6 - q sec2 0)

= ffy V 1 q — + sin^ 0 cos^ 0 - p cosec2 0 — ty sec2 0


trf02 / 0 = a Ue 0=a V p
( j2 \
=> dy = 0 - sin?’ 0 cos'^ 0 (p cosec2 0 + q sec2 0) < 0
[dO2 0=a

low
Hence, y is maximum when 0 = a = tan -1 P
<7

EXERCISE 18.3

ee
rF
Fr
LEVEL-1

1. Find the points of local maxima or local minima and corresponding local maximum and
for
local minimum values of each of the following functions. Also, find the points of inflection,
ou
if any:
ks

(i) f{x)=x* -62x2 +\20x +9 (ii) /(x)=x3 -6*2 + 9x + 15


oo

(hi) f(x)=(x-l)(x+2)2 (iv) f{x)=2/x-2/x1,x>0


Y
B
re

(v) f(x)=xex (vi) f(x) =x/2 + 2/xfx>0


ou

(vh) f(x)=(x + l)(x + 2) 1/3 ,x>-2


ad

(viii) f(x) = x ^32-x2, - 5 < ^ < 5


Y

a2
(ix) f(x) =x3 - 2ax2 + a2 x,a>0, x eR (x) f(x) = x + —, a>0, xjtQ
d

x
Re
n

(xi) f(x)=Xyl2-X2 -y[2 <X <y[2


Fi

(xh) f(x) = x + yjl -x, x < 1

2. Find the local extremum values of the following functions :


(i) /(x) = (x -1) (x - 2)2 (ii) /(x) = x jl-x, x < 1 (hi) /(x) =-(x-l)3 (x + 1)2

3. The function y = 0 log x + bx2 + x has extreme values at x = 1 and x = 2. Find a and b.
log*
4. Show that has a maximum value at x = e. fNCERT]
x
4
5. Find the maximum and minimum values of the function /(x) =------- + X.
x+2

6. Find the maximum and minimum values of/(x) = tan x - 2x.


7. If / (x) = x3 + ax2 +bx + c has a maximum at x = -1 and minimum at x = 3. Determine
a, b and c.
8. Prove that/(x) = sin x + V3 cos x has maximum value at x = -. INCERT EXEMPLAR]
6

ReadYourFlow.COM
18.32 MATHEMATICS-XII

ANSWERS
1. (i) Local Max. at x = 1, Local Max. value = 68
Local Min. at x = 5, - 6; Local Min. values are - 316 and - 1647.
(ii) Local Max. at x = 1, Local Max. value = 19
Local Min. at x = 3, Local Min. value = 15
(iii) Local Max. at x = - 2, Local Max. value = 0
Local Min at x = 0, Local Min value = - 4
(iv) Local Max. at x = 2, Local Max. value = 1/2
(v) Local Min. at x = - 1, Local Min. value = - 1/e
(vi) Local Min. at x = 2, Local Min. value = 2
3
(vii) Local Min. at x = - 7/4, Local Min. value = -

w
44/3

(viii) Local Max. at x = 4; Local Max. value = 16


Local Min. at x = - 4; Local Min. value = -16

Flo4«3

ee
(ix) Local Max. at x = a/3, Local Max. value =
27

Fr
Local Min. at x = a. Local Min. value = 0
(x) Local Max. at x = - a, Local Max value = - 2a for
ur
Local Min. at x = a, Local Min. value = 2a
(xi) Local Max. at x = 1, Local Max. value = 1
ks

Local Min. at x = - 1, Local Min. value = - 1


Yo
oo

(xii) Local Max. at x = 3/4, Local Max. value = 5/4


B

2. (i) Local Max. value = 4/27 at x = 4/ 3, Local Min. value = 0 at x = 2


re

(ii) Local Max. value =


JTs**"2'3
ou
ad

(iii) Local Max. value 3456/3125 at x = -1/5; Local Min. value = 0 at x = - 1


Y

3. a = -2/3,b =-1/6
5. Local Max. value = - 6 at x = - 4; Local Min. value = 2 at x = 0.
nd
Re

6. Local Max. value = -1 - 3n/2 at x = 3k/4; Local Min. value = 1 - n/2 at x = k/4.
Fi

7. a =-3,b =-a, c e R
HINTS TO NCERT& SELECTED PROBLEMS
4. We have,
log * 1 - log x
/(*) = =>/'(*)
X x2
At points of local maximum and minimum, we must have
1 ~ log * = 0=> log x = 1 => x = e
fix) = 0 =>
x2
Now, f' (x) = -—= x 2 (1 - log x)
xz
=> f"(x) = -2 x “ 3(1 - log x) -x -3 = -x 3 (3-2 log i)
f"(e) = -e - 3(3 - 2 log e) = -^<0.

Hence, / (x) has a local maximum value at x = c.

ReadYourFlow.COM
MAXIMA AND MiNIMA 18.33

18.6 MAXIMUM AND MINIMUM VALUES IN A CLOSED INTERVAL


Let y =f(x) be a function defined on [a, fc]. By a local maximum (or local minimum) value of a
function at a point c e [a, b] we mean the greatest (or the least) value in the immediate
neighbourhood of x - c. It does not mean the greatest or the maximum (or the least or the
minimum) of f(x) in the interval [n, b]. A function may have a number of local maxima or local
minima in a given interval and even a local minimum may be greater than a local maximum.
Thus, a local maximum value may not be the greatest (the maximum) value and a local
minimum value may not be the least (the minimum) value of the function in any given interval
as shown in Fig. 18.31.
Y

A Q

w
y=m

Flo
f(a) B

ee
m

Fr
o b X
Fig. 18.31
for
ur
However, if a function f(x) is differentiable and consequently continuous on a closed interval
[a, b], then it attains the absolute maximum (absolute minimum) at stationary points (points
ks
Yo

where /' (x) = 0) or at the end points of the interval [a, b]. Thus, to find the absolute maximum
oo

(absolute minimum) value of the function, we choose the largest and the smallest amongst the
B

numbers/(fl), /(q), /(c2), -,/(%), f(b) where x = q, c2,..., cn are the stationary points.
re

We may use the following algorithm for finding the maximum (absolute maximum) and the
minimum (absolute minimum) of a function/defined on a closed interval [a, b].
ou
ad

ALGORITHM
Y

STEP I Find f' (x)


STEP II Put f' (x) = 0 and find values ofx. Let cj, c2,.... q be the values ofx.
nd
Re

STEP III Puke the maximum and minimum values out of the values f(a), f(c/), f(c2),.... f(cn), f(b).
Fi

The maximum and minimum values obtained in step III are respectively the largest (or absolute
maximum) and the smallest (or absolute minimum) values of the function.
ILLUSTRATIVE EXAMPLES

LEVEL-1
O
EXAMPLE 1 Find the maximum and minimum values off(x) = 2x - 24x + 107 in the interval [1, 3].
INCERT]
SOLUTION We have,
/(x) = 2 x3 - 24 x + 107
=> /' (x) = 6 x2 - 24
Now, /' (x) = 0 => 6 x2 - 24 = 0 => x = ± 2
But, x = - 2 g [1, 3]. So x = 2 is the only stationary point.
Let us now compute the values of/(x) at x =1, 2, 3.
/(I) = 2-24 + 107 =85, /(2) = 2 (2)3 - 24 (2) + 107 =75

ReadYourFlow.COM
18.34 MATHEMATICS-XII

and. /(3) - 2 (3)3 -24 x 3 + 107 = 89


Clearly, largest of these values is 89 and the least is 75.
I leme, the maximum value of f(x) is 89 which it attains at r = 3 and Ihe minimum value is 75
which is attained at x = 2.
EXAMPLE 2 Find the maximum and minimum values of fix) =sin a in I In’ interval\n, 2n\.
SOLUTION We have, f(x) = sin .r
f'(a) = COS X
At stationary points, we must have
/ ' (a) - 0 => COS X =0 => A = --- |v a e[71,2 nil
2
3n:
Let us now compute the values of /(a) at a = n, ——, 2n:.

low
Now, f{n) - sin n = 0,/f~• j = sin -3n
— = - 1 and /(2n) = sin 2n ~ 0.

The greatest and the least of these values are 0 and -1 respectively.

ee
Hence, the maximum value of/(a) isO which it attains at a = nand 2n:,and the minimum value is
rF
Fr
- 1 which it attains at a = 3n/2.
I
example 3 Find the. maximum and minimum values of ffx) = sin for a + - cos 2.v in [0, tc/2|.
1
ou
SOLUTION We have, /(a) = sin a + — cos 2a
ks

/' (a) = cos a - sin 2.x


oo

At stationary points, we have


Y
B

/'(a) = 0
re

1 n , n v 0 <; * £ -71
cos a - 2 sin x cos a = 0 => cos x = 0 or, sin a = A = - and a
2 2 b 2
ou
ad

I .et us now calculate the values of /(a) at these points and al the end-points of the interval.
Y

1 1
Now, /(0) - sin 0 + — cos 0 = --
2 2'
nd
Re

. n 1 n 1 71 I 1 I
sin ~ - cos — = - + sin - + cos it - I
Fi

6 2 3 2 2 2 2 2
3 1
Of these values, the leagest value is — and the smallest value is ^.
3 |
Thus, the maximum and minimum values of/(a) are ^ and ^ respectively which it attains at

A= and a = 0, a = ^ respectively.

EXAMPLE 4 Find the maximum and minimum values off(x) = a50 -x 20 in the interval [0,1].
SOLUTION Let/(a) = a50 - a20. Then, / '(a) =50x4q - 20 a19.
At stationary points, we must have
/'(a) = 0
=> 50a49 - 20a19 = 0
1 / 30
x19 (50x 30 - 20) = 0 => x = 0 or, 50x30 = 20 => a = 0 or, a =

ReadYourFlow.COM
MAXIMA AND MINIMA 18.35

The values of f(x) at these points and at the end-points of the interval [0,1 ] are as given below.
/ n!/ 30 f r\ n50/ 30 ( r)\ 20/30 2 \2/ 3
2 3(2 ,2/3
Now, =(f) -(5) --1
5 5 5 15
and, /(l) =1-1=0.
3 ( 2 1/3
Of these values, the maximum value is 0 and the minimum value is - -
5 15
Thus, the maximum value of f{x) in [0,1] is 0 and the minimum value off(x) in [0,1] is
3(2 2/3
5 Is
EXAMPLE 5 Find the maximum and minimum values off(x) = x+ sin 2x in the interval [0, 2n\.
[NCERT]

low
SOLUTION We have, f(x) = x + sin 2x
f'(x) =1 + 2 cos 2x
At stationary points, we have

ee
/'(*) = 0
rF
Fr
=> 1 + 2 cos 2x = 0
1
cos 2 x = - for
2
2n An
[v 0 <,x<, 2n /. 0<2x<4n]
ou
2x = — or, 2x = —
3 3
ks

n 2n
x = or, x
oo

3 3
Y
B

Let us now compute the values of/(x) at these stationary points and at the end-points of the
re

interval [0, 2k].


• n n rf TO K . 2n K V3 ,-f2K'\ 2k 4k 2k V3
ou

= 0 + 9m0=0' /l3j = 3 + 8mT = 3 + T' /ItJ=T + sm —3 =


ad

Now, /(0)
3 2
Y

and, /(2k) = 2k + sin 4 k = 2k + 0 = 2k.


d

Of these values, the maximum value is 2k and the minimum value is 0.


Re
n

Thus, the maximum value of/(x) is 2k and the minimum value is 0.


Fi

EXAMPLE 6 Find the difference between the greatest and least values of the function f(x) = sin 2x-x
on [-k/2, k/2]. [NCERT EXEMPLAR]
SOLUTION We have,
/(x) = sin2x-x
f (x) = 2 cos 2x -1
At stationary points, we must have
/'(x)=0
K K K
=> 2cos2x~l=0=> cos2x = -- => 2x = -- — => x = -
2 3' 3 6' 6
Let us now compute the values of/(x) at these stationary points and also at the end-points of
the interval [-k/2, k/2].
Now, /(x) = sin 2x - x
rf k") , , . K K ,( K^ . / k3 K K
=>
6

ReadYourFlow.COM
18.36 MATHEMATICS-XII

n
’ ’ f-f
= sin------
3 6 /(f) = sin Ti —
7C

2 2
71 71
Of these values, the largest is — and the leastis ~ So, the greatest and the least values of
f{x) on [-7r/2, Tt/2] are n/2 and -n/2 respectively.
Hence, required difference “ ^ j = 7t.

EXAMPLE 7 Shoiu that f(x) = sin .r (1 + cos x) is maximum at x = — in the interval [0, TtJ.
3
SOLUTION We have,
f(x) = sin x (1 + cos x).
=> f' (x) = cos x (1 + cos x) - sin2 x

low
=> /' (x) = cos X + COS X - (1 - cos“ x)
«•>
=> /' (x) = 2 cos- x + cos x -1 = (2 cos x -1) (cos x + 1).
At stationary points, we have

ee
/'(*) = 0
rF
Fr
=> (2 cos x - 1) (cos x + 1) = 0 => cos x = — or, cos x=-l=>x = — or, x = Ti.
2 for 3
Le< us now compute the values of x at these stationary points and at the end-points of the
interval.
ou
ks

Now, /(0)=0,/(jJ = sin — f 1 + cos j= and f(n) = 0.


3
oo
Y
B

3 -J3 3
Of these values, the maximum value is —LJ1. Hence,/(x) attains the maximum value------ at
re

4 4
x = 7l/ 3.
ou
ad

EXAMPLES Find the absolute maximum value and the absolute minimum value of the follozving
Y

functions in the given intervals:


(i) /(x) =(1/2 -x)2 + x3 in [- 2, 25] (ii) /(x) = sin x + cos x in [0, ti] [NCERT]
nd
Re

SOLUTION (i) We have.


Fi

2
1 xj + x3, where x e [- 2, 25].
fix) = .2
=> f'(x) = - 2 (1/2 -x) + 3x2 = -1 + 2x + 3x2
At the points of local maximum and local maximum, we must have
/ '(x) = 0 => 3x2 + 2x -1 =0 => (3x -1) (x + 1) = 0 => x = 1/3, -1
The values of/(x) at these points and also at the end-points of the interval are computed as given
below.
= J_ 1 _ 7
/(-2) = 8 =- ~ 36 + 27 ~ 108'
/(-l) = (T + l)2+(-l)3 = ^ and,/(2.5)=^-2.sj 2 +(2.5)3 = 157
8
157 7
Of these values, the maximum value of / (x) is and the minimum value is
8 4

ReadYourFlow.COM
MAXIMA AND MINIMA 18.37

157 7
Thus, the absolute maximum = and, the absolute minimum = - —
8
(ii) We have.
f(x) = sin x + cos x, where x e [0, re]
=> f'(x) = cos x - sin x
The critical points of/(x) are given by
/ '(x) = 0 => cos x - sin x = 0 => cos x = sin x => tan x = 1 => x = ti/4
Let us now calculate the values of /(x) at the critical points and the end-points of the interval.
/(0) = sin 0 + cos 0 = 1, /(tc/4) = sin ti/4 + cos rc/4 = V2
and. f(n) = sin Tt + cos n = -1.
Of these values, the maximum and minimum values of / (x) are V2 and -1 respectively.

w
So, absolute maximum = V2 and, absolute minimum = - 1.
EXAMPLE 9 Find both the maximum and the minimum value of 3x4 - 8x3 + 12x2 - 48x + 1 on the
interval [1, 4].

Flo
ee
SOLUTION Let /(x) = 3x4 - 8x3 + 12x2 - 48x + 1. Then,

Fr
/ '(x) = 12x3 - 24x2 + 24x - 48 and / "(x) = 36x2 - 48x + 24
The critical points of/(x) are given by /'(x) = 0.
for
ur
Now, /'(x) = 0
ks

=> 12x3 - 24x2 + 24x - 48 = 0


Yo

=> x 3 - 2x2 + 2x -4 = 0=> x2(x - 2) + 2 (x - 2) = 0 => (x - 2)(x2 + 2)=0=>x = 2 [•/ x2 + 2 ^ 0]


oo
B

The values of/(x) at critical points and at the end-points of the interval are computed as follows:
re

/(2) = - 59, /(l) = - 40 and /(4) = 257.


Of these values the largest and the smallest values are /(4) = 257 and /(2) = -59.
ou
ad

So, the minimum and maximum values of /(x) on [1, 4] are -59 and 257 respectively.
Y

EXERCISE 18.4
d
Re

LEVEL-1
n
Fi

1. Find the absolute maximum and the absolute minimum values of the following functions
in the given intervals:
v2
(i) /(x) = 4x - y in [ - 2, 45] [NCERT] (ii) /(x) =(x -l)2 + 3 in [-3,1] [NCERT]

(hi) /(x) = 3x4 - 8x3 + 12x2 - 48x + 25 in [0, 3] [NCERT]


(iv) /(x) = (x -2) ^jx-\ in [1, 9]
o
2. Find the maximum value of 2x - 24x + 107 in the interval [1,3]. Find the maximum value
of the same function in [- 3, -1]. [NCERT]
3. Find the absolute maximum and minimum values of the function / given by
r\

f(x) = cos x + sin x,x e [0, tt]. [NCERT]


4. Find absolute maximum and minimum values of a function / given by
/(x) =12x4/3-6x1/3,x e[-l, 1]. [NCERT]
5. Find the absolute maximum and minimum values of a function / given by
/(x) = 2x3 - 15x2 + 36x + 1 on the interval [1,5]. [NCERT]

ReadYourFlow.COM
18.38 MATHEMATICS-XII

ANSWERS
1. (i) Absolute Maximum = 8 at x = 4, Absolute Minimum = -10 at x = - 2
(ii) Absolute Maximum = 19 at x = - 3, Absolute Minimum = 3 at x = 1
(iii) Absolute Maximum = 25 at x = 0, Absolute Minimum = - 39 at x = 2
2 4
(iv) Absolute Maximum = 14V2 at x = 9, Absolute Minimum = at x = —
3a/3 3
2. Maximum value = 89 at x = 3 in [1, 3], Maximum value = 139 at x = - 2 in [- 3, - 1]
3. Absolute Maximum = 5/4, Absolute Minimum = 1
9 1
4. Absolute Minimum, value = — at x = — Absolute Maximum value = 18 at x - -1
4 8'
5. Absolute Maximum value = 56 at x = 5, Absolute Minimum value = 24 at x = 1
HINTS TO NCERT& SELECTED PROBLEMS

low
x2
1. (i) We have, / (x) = 4 x - —

=> /' (x) = 4 - x

ee
/' (x) = 0 => x = 4
rF
Fr
Now,/(-2) = - 8-2 = -10,/(4.5) = 18-10.125 = 7.875 and,/(4) = 16-8 = 8
Absolute maximum is 8 at x = 4.
(ii) We have, / (x) = (x -1)2 + 3
for
ou
••• /'(*)= 2 (x-1)
ks

At critical points, we have/'(x) = 0


.’. /'(x) = 0 ^ 2 (x -1) = 0 => x = 1.
oo
Y

Now,/( - 3) = (-3-l)2+ 3 = 19 and/(1) = (l-l)2 + 3 = 3.


B
re

Absolute maximum = 19atx - -3, Absolute minimum = 3atx = 1.


(iii) We have,
ou
ad

/(x) = 3x4 -8x3 +12x2 -48x + 25 => /' (x) = 12x3 - 24x2 + 24x - 48
Y

At critical points, we have/'(x) = 0.


f'(x) =0
nd
Re

=> 12x3 - 24x2 + 24x - 48 = 0


Fi

=> 12 (x3 - 2x2 + 2x - 4) = 0 => x2 (x - 2) + 2 (x - 2) = 0 (x - 2) (x2 + 2) = 0 => x = 2.


The value of/(x) at critical points and at the end-points of interval are
/(0) = 25,/(2) = 48-64 + 48-96 + 25 = - 39
and,/(3) = 243-216 + 108-144 + 25 = 16
Of these values the greatest and the least are/(0) = 25 and/(2) = -39 respectively.
Absolute maximum = 25 at x = 0, Absolute minimum = - 39 at x = 2.
2. Let/(x) = 2x3 - 24x + 107.Then,
/' (x) = 6x2 - 24 = 6 (x - 2) (x + 2)
At critical points, we must have/'(x) = 0
.-. /'(x) = 0=> 6(x-2)(x + 2) = 0 => x = -2, 2
If/(x) is defined on [1, 3], Then/' (x) = Oatx = 2.
The values of/(x) at critical points and at the end-points of the interval are:
/(l) = -85,/(2) = 75 and/(3) = 89.
Absolute maximum = 89 at x = 3

ReadYourFlow.COM
MAXIMA AND MINIMA 18.39

If / (x) is defined on [- 3, -1], then /' (x) = 0 at x = - 2.


The values of/(x) at critical points and at the end-points if the interval are:
/ (- 3) = 125, / (- 2) = 139, and/(-l) = 129
So, the absolute maximum is 139 at x = - 2.
3. We have, / (x) = cos x + sin x, x e [0, tc]
/' (x) = - 2 sin x cos x + cos x = cosx (1 -2sin x)
At critical points, we must have /'(x) = 0.
rt / \
= n0 => cosx = r\0 or, sin
• ^ TITIStT
f'(x) x = - => x =—, —, —
y 2 2 6 6
The values of/(x) at critical points and at the end points of interval are:
/(0) =l,/(7r) =cos27i+sinK = l,/ 315 , .(Sk} 3 1 5
424 V. 6 J 42 4
71 571
Absolute maximum = — at x —; Absolute minimum = 1 at x = 0,1,^
4 6 6 2

w
4. We have,/(x) =12 x4 / 3 - 6 x17 3
- 2/ 3 _ 2 (8x -1)

Flo
/' (x) = 16x 1/3 - 2x
x2/3

ree
At critical points, we must have /'(x) = 0.
2^-1)=0^ X=I

F
••• /'(x)=0 => x2/3 V => x
8
or
ur
The values of/(x) at critical points and at the end points of interval are:
sf
/(-l) = 12 + 6 = ^ and / (1) =6
k
Yo

9 1
oo

Absolute maximum = 18 at x = -land. Absolute minimum = —at x


4 8
B

5. We have, / (x) = 2x3 -15x2 + 36x + 1


re

/'(x) = 6x2 - 30x + 36 = 6(x2 -5x + 6)


ou
ad

At critical points, we must have /' (x) = 0.


Y

/'(x) = 0 => 6(x2-5x + 6) = 0 => x =2, 3


The values of f(x) at these points and end points of interval are:
nd
Re

/(l) = 24, /(2) = 29,/(3) = 28 and/(5) = 56


Fi

Absolute maximum = 56 at x =5 and. Absolute minimum = 24 at x = 1

18.7 APPLIED PROBLEMS ON MAXIMA AND MINIMA


In this section, we will discuss some applied problems on maxima and minima for which
following results will be very useful.
(i) For a square of side x:
Area = x2, Perimeter = 4x.
(ii) For a rectangle of sides x and y.
Area = xy, Perimeter = 2 (x + y).
(iii) For a trapezium:
1
Area = - (Sum of parallel sides) x (Distance between them).

(iv) For a circle of radius r:


Area -nr , Circumference = 2n r.

ReadYourFlow.COM
18.40 MATHEMATICS-XII

(v) For a sphere of radius r:


4 ^ 9
Volume = — nr , Surface Area = 4rcr .
3
(vi) For a right circular cylinder of base radius r and height h:
9 9
Volume = nr h, Surface = 2nrh + 2nr , Curved surface = 2nrh.
(vii) For a right circular cone of height h, slant height / and radius of the base r
Volume = (1/ 3) Tt r2h, Curved surface = n rl, Total surface = nr2 + n rl.
(viii) For a cuboid of edges of lengths x, y and z:
Volume = xyz, Surface = 2(xy + yz + zx).
(ix) For a cube of edge length x:
O O
Volume = x , Surface Area = 6 x .
V3 2
(x) Area of an equilateral triangle = — (Side) .

w
REMARK Ifk is a positive constant, then afunction of theform kf(x),k + f(x), {f (x)}k, {f(x)}l^k, log f(x)
will be maximum or minimum according asf(x) is maximum or minimum provided that f(x) > 0.

Flo
ILLUSTRATIVE EXAMPLES

ee
LEVEL-1

Fr
EXAMPLE l Find two numbers whose sum is 24 and whose product is as large as possible.
SOLUTION Let the numbers be x and y. Then,
for [NCERT]
ur
x + y = 24 (given ) •••(i)
Let P be the product of these numbers. Then,
ks

P = xy - x(24 - x) [Using (i)]


Yo
oo

P = 24x - x2
B

dP d2P
— = 24 - 2x and
re

=- 2
dx dx2
ou
ad

The critical points of P are given by — = 0.


dx
Y

dP
— = 0=>24-2x = 0=>x=12
nd
Re

dx
d2P
Fi

Also, = - 2 < 0. So, P is maximum when x = 12.


vrfx2 dx = 12
Putting x = 12 in (i), we obtain y = 12. Hence, the required numbers are both equal to 12.
o
EXAMPLE 2 Find two positive numbers x and y such that x + y = 60 and xy is maximum. [NCERT]
SOLUTION Let P = xy3. It is given that x + y = 60. Therefore, x = 60 - y.
Now, P = xy3
=> P=(60-y)y3=60y3-y4
d2P
=> — = 180y2 - 4y3 and = 360y-12y2
dy dy2
dP
The critical points of P are given by — = 0.
dy

ReadYourFlow.COM
MAXIMA AND MINIMA 18.41

^ = 0.
dy
=> 180i/2 - 4y3 = 0 => 4y2 (45 -y)=0=> y = 0,y = 45=^y = 45 [■.• y = 0 is not possible]
d2P
Now, = 360 x 45 -12(45)2 = 12 x 45 (30 - 45) = - 8100 < 0
dy2;
y = 45

So, P is maximum when y = 45. Putting y = 45 in x + y = 60, we obtain x = 15.


Q _ _
Hence, xy is maximum when x = 15 and y = 45.
EXAMPLE 3 Find two positive numbers x and y such that their sum is 35 and the product x2 y5 is
maximum. [NCERT]
SOLUTION Let P = x2 y5. It is given that
x + y = 35 => x = 35-y

w
Putting x = 35 - y in P = x2y5, we get
P = (35 - y)2 y5

=>
dP
dy
2(35 -y) xf +5(35-y)2 y4

Flo
ee
dP i

Fr
=> = (35-y)y4 {-2y + 5(35-y)}
dy
rfP = y4 (35-y) (175-71/) = 7y4 (35-y) (25-y)
for
ur
=>
dy
dP
ks

The critical points of P are given by — = 0.


Yo

dy
oo

dP a
U1 = 0 => 7y4 (35-y) (25-y) = 0 y = 0, 25, 35
B

dy
re

But, y = 0 and y = 35 are not possible. So, y = 25.


d2P
ou
ad

Now, = 28y3 (35 -y) (25 -y) -7y4 (25-y) -7y4 (35 -y)
Y

d2P
- 7 (25)4 (35 - 20) = - 7 (25)4 (10) < 0
nd
Re

Uy2 Aj=25
Fi

Thus, P has maximum when y = 25. Putting y = 25 in (i), we obtain x = 10.


Hence, x2\p is maximum when x = 10, and y = 25.
EXAMPLE 4 Amongst all pairs ofpositive numbers with product 256, find those whose sum is the least.
SOLUTION Let the required numbers be x and y. Then,
xy = 256 (given)
Let S = x + y. Then,
256
S = x+ [Using (i)]
x
dS 256 d2 S 512
=> — = 1 and,
dx x dx2 x3
dS
The critical points of S are given by — = 0.
dx

ReadYourFlow.COM
18.46 MATHEMATICS-XII

=> P = 4x + 4 yja2 - x2 [Using (i)]


dP 4x D
=> = 4— C
dx -x2
O
dP 2y
The critical points of P are given by — = 0. y
dx
dP A x—-M B
— = 0 2x
dx
4x
=> 4—, - = 0
Fig. 18.33

4x a
4 = => ri -x 2 = x => a 2 -x 2 = x 2 => lx2 = a2 => x
V2
dP

low
Now, = 4—r—==
dx

.2 ^ (- *)

d2P
4 V-2 - x2 - 4fl2

ee
=>
rF («2-x2)3/2

Fr
rfx2

^2P^ -8V2
for
<0
ou
dx2 2 \3/2 a
= a/Jl a2- a
ks

S X

2
oo
Y
B

Thus, P is maximum when x = —


v2
re

Putting x = -JL in (i), we obtain y = -j=.


ou
ad
Y

x - y = a42 => lx = 2y=> AB = BC => ABCD is a square.


Hence, P is maximum when the rectangle is square of side 2x = =42 a.
nd
Re
Fi

EXAMPLE 13 AB is a diameter of a circle and C is any point on the circle. Show that the area of A ABC is
maximum, when it is isosceles. [NCERT EXEMPLAR]
SOLUTION Let AB = 2a, AC = x and CB = y. Since AB is a diameter of the circle having centre
TZ
O and C is a point on the semi-circle ACB. Therefore, ZACB = —.

Applying Pythagoras theorem in AACB, we obtain


AB2=AC2+CB2
(2a)2 =x2 +y2

=> y = yj4a2 -x2


Let A be the area of AACB. Then,
1
A=-ACxCB = 2^
2

ReadYourFlow.COM
MAXIMA AND MINIMA 18.47

=> A = -x 'l4a2 -x2 [Using (i)]


2
dA x2 2a2 -x2
=>
dx
=12 V4?I7- V4fl2 - .r2
V4fl2 -x2
dA
The stationary values of A are given by —— = 0.
dx
dA A B
—=0
dx
la2 -x2
=> = 0 => 2a2 =x2 => x = J2a

dA 2a2-x2
Now, Fig. 18.34

w
d* V4fl2 -x2
yl^a2 -x2 x -2x-(2a2 -x2) x
a/4^2 x(6a2-x2)

Flo
rf2A
=>
dx2 (4a2-x2)3/2

F ree
d2A
= -2 < 0
dx2 or
ur
Jx=4la
sf
Thus, A is maximum when x = -J2a and y = -J2a.
Hence, the area of AABC is maximum when it is isosceles.
k
Yo
oo

EXAMPLE 14 Tangent to the circle x2 + y2 = a2 at any point on it in thefirst quadrant makes intercepts
B

OA and OB on x and y axes respectively, O being the centre of the circle. Find the minimum value of
re

OA+OB. [CBSE2015]
0 0 0
SOLUTION Let P (a cos 0, a sin 0) be an arbitrary point on the circle x + y = fl . If P lies in the
ou
ad

first quadrant, then 0 < 0 < ti/2. The equation of tangent to x2 + y2 = a2 at P(flcos0, flsin 0) is
Y

xcos0 + ysin0 = a [The tangent to x2 + y2 = a2 at (xq, y^) is xx1 + yy-j = a2\


nd
Re

Y
Fi

B(0, a cosec 0)

P(a cos 0, a sinG)

O
X' A(a sec 0, 0)
m
X

x cos 0 + i/ sin 0 = fl
x2+y2 - a2

! Y'
Fig. 18.35

This cuts x and y-axis at A (aseed, 0) and B(0, a cosec 0) respectively.

ReadYourFlow.COM
18.48 MATHEMATICS-XII

OA = a sec 0 and OB = a cosec 0


Let S = OA + OB. Then,
S = a (sec 0 + cosec 0).
dS
— =a (sec 0 tan 0 - cosec 0 cot 0)
dQ
d2S
and. = a (sec3 0 + sec0tan2 0 + cosec3 0 + cosec 0 cot2 0)
dQ2
For maximum or minimum values of S, we must have
JQ
= 0=> a (sec0 tan 0 - cosec 0 cot 0) = 0 => tan 3 0 = 1 => tan 0 = 1 => 0 = ^
dQ
At 0 = —, we obtain
4
d2S

w
= a (2V2 + V2 + 2V2 + V2) = 6V2fl > 0
dQ2

Flo
Hence, S is minimum at 0 = ^ and the minimum value of S is given by

ree
S = a sec—+ cosec — = 242 a
4 4

F
EXAMPLE 15 If the sum of the lengths of the hypotenues and a side of a right angled triangle is given,
or
ur
show that the area of the triangle is maximum when the angle between them is n/ 3.
sf
[CBSE 2009,2014,2016,2017]
SOLUTION Let ABC be a right angled triangle with base BC = x and hypotenues AC = y such
k
Yo
oo

that x + y = k, where /c is a constant. Let 0 be the angle between the base and hypotenues. Let A
be the area of the triangle. Then,
B

A =-BCx AC =^x Jy2-x2


re

2
ou

x2
ad

A2 =
=>
t(*2-2)
Y

^{(k-x)2-x2} A
A2 =
nd

[•• x + y = k]
Re

4
Fi

k2 x2 -2kx3
=> A2 =
4
Differentiating with respect to x, we get
dA 2k2x - 6for2
2A — = ...(h)
dx 4
dA k2x - 3for2
=>
dx 4A B Fig. 18.36 C
dA
The critical numbers of A are given by — = 0.
dx
dA k2 x - 3kx2
Now, — = 0 => = 0 => x = -.
dx AA 3
Differentiating (ii) with respect to x, we get
d2 A 2k2-12kx
2f—)2
v dx
+ 2A
dx2 4
...(hi)

ReadYourFlow.COM
MAXIMA AND MINIMA 18.49

k dA dA
When x = —, —— = 0. Putting = 0 and x = ^ in (iii), we get
3 dx dx
d2A k2 n
— < 0.
dx2 4A
k k 2k
Thus, A is maximum when x = —. Putting x = — inx + y = k,we obtain y = —
3 3 3
BC x k/3
In AACB, cos 0 - ---- => cos 0 = — => cos 0 = = - => 0 = -.
AB y 2k/3 2 3

Thus, area of triangle ABC is maximum, when angle 0 between base BC and hypotenuse AB is
71/3.
EXAMPLE 16 Prove that the area of right-angled triangle of given hypotenues is maximum when the

w
triangle is isosceles.
SOLUTION Let h be the hypotenues of the right-angled triangle, and let .r be its altitude. Then,

Flo
Base of the triangle - fh2 -x2. -(i)

ree
Let A be the area of the triangle. Then, c

F
A -X2

or
ur
dA 2+xV-*2r1/2-(fc2-*2)j h
sf
=> x
dx 2 dx J
k
Yo
oo

dA x2 1 J h2 - 2x2
=>
B

dx
re

A B
dA Fig. 18.37
ou
ad

The critical numbers of x are given by —— = 0.


dx
Y

^ = 0 =>1. h2 - 2x2 h
nd

= 0=> h2 = 2x2 => x


Re

dx 2 h2-x2 V2
Fi

dA _\\ h2 -2x2
Now,

d2A = | (-4^ 1 = + (/t2 -2x2)j^-^(ft2 -x2) - 3/2 ^(/?2-x2)


=>
dx2 -x dx

d2A 1 -4x x(h2 — 2x2)


=>
dx2 2W^ -x
2 +(/72-X2)3/2

( d2 A
=> = - 2 < 0.
dx2 h
/ Y = ——-

V2

ReadYourFlow.COM
18.50 MATHEMATICS-XII

Thus, A is maximum when x = . Putting x = -^= in (i), we get: Base = h2- (h*)_A_
2 Jl '
AB = BC=4=
V2
Hence, A is maximum when the triangle is isosceles.
EXAMPLE 17 Show that the surface area of a closed cuboid with square base and given volume is
minimum, when it is a cube.
SOLUTION Let V be the fixed volume of a closed cuboid with length x, breadth x and height y.
Let S be the surface area of the cuboid. Then,
P = x2y
and. S = 2 (x2 + xy + xy) = 2x2 + Axy ...(ii)

w
Now, S = 2x2 + Axy

Flo
=> S = 2x2 +Ax ■■ V=x2V
V
x2 y=7

ee
=> S = 2x2 + AV

Fr
x
dS AV
— = Ax-^r ...(hi)
for
ur
dx x2
dS
The critical numbers of S are given by — = 0.
ks

dx
Yo
oo

dS
— = 0
B

dx
AV
re

=> Ax - —4- = 0
x2
ou
ad

=> V = X3
Y

=> x2 y = x3 V=x2y]
nd
Re

=> x = y.
Fi

Differentiating (iii) with respect to x, we get


d2S 8P
= 4 + = 4+ = 4 + 8y
dx2 x3 X

d2s)
=> = 12 > 0.
dx2
v 'y = x
Hence, S is minimum when length = x, breadth = x and height = x i.e., when it is a cube.
EXAMPLE 18 An open tank with a square base and vertical sides is to be constructed from a metal sheet
so as to hold a given quan tity of water. Shoiv that the cost of the material will be least when depth of the
tank is halfof its width. ICBSE 2007,2010]
SOLUTION Let the length, width and height of the open tank be x, x and y units respectively.
Then, its volume is x y and the total surface area is x + 4xy.
It is given that the tank can hold a given quantity of water. This means that its volume is
constant. Let it be V. Then,

ReadYourFlow.COM
MAXIMA AND MINIMA 18.51

V = x2y
The cost of the material will be least if the total surface area is
y
least. Let S denote the total surface area. Then,
S = x2 + 4xy ...(h) y
y
X
We have to minimize S subject to the condition that the y
volume V is constant.
Now,
X
S = x2 + Axy
Fig. 18.38
q 2 4V [Using (i)]
=> S = x +
x
dS 4V J d2S 0 8U
=> — = 2x- — and —y = 2 + —^
dx x2 dx2 x3

w
dS
The critical numbers of S are given by — = 0.
dx
dS

Flo
Now, — = 0
dx
4V

ee
=> 2x ——U = 0
x2

Fr
=> 2x3 = 4V
2x3 = 4x2 ij
for [••■ v = X2y]
ur
=>
=> x = 2ij
ks

d2 S 8V
Clearly, = 2 + —— > 0 for all x.
Yo
oo

dx2 x3
Hence, S is minimum when x = 2y i.e. the depth (height) of the tank is half of its width.
B
re

EXAMPLE 19 A metal box with a square base ami vertical sides is to contain 1024 cm' of water, the
material for the top and bottom costs ? 5 per cm and the materialfor the sides costs ? 2.50 per cm . Find
ou
ad

the least cost of the box. [NCERT EXEMPALR]


Y

SOLUTION Let the length, breadth and height of the metal box be x cm, x cm and y cm
respectively. It is given that the box can contain 1024 cm 3 of water.
nd
Re

1024 = x2i/=> y = 1024 ...(i)


Fi

x2
Let C be the total cost in ? of material used to construct the
box. Then, y
C =5x2 +5x2 + |x 4xy
y
y
C =10x2 +10xy x y
=>
We have to find the least value of C.
Now, x
C =10x2 + lOxy Fig. 18.39

=> C=10x2+10xx^ [Using (i)]


x2
C =10x2 + 10240
=>
x
dC 10240 d2C 20480
=> — = 20x - -r—and —- = 20 +
dx x2 dx2 x3

ReadYourFlow.COM
18.52 MATHEMATICS-XII

dd
The critical numbers for C are given by — = 0.
dx
Now,
dC 10240 = 0=> x3 =5012 ;=> x3 = 83 => x = 8
— = 0 ^ 20a: -
dx x2
d2C 20480
Also, = 20 + >0
U*2 '/x=S 83

Thus, the cost of the box is least when x = 8. Putting x = 8 in (i), we obtain y = 16. So, the
dimensions of the box are 8x8x16.
Putting x = 8 and y = 16 in C = 10x2 + lOxy, we obtain C = 1920.
Hence, the least cost of the box is ?1920.
EXAMPLE 20 An open box with a square base is to be made out ofa given quantity of card board of area c2

w
£^
square units. Show that the maximum volume of the box is —j= cubic units.
6v 3

Flo
[CBSE 2001C, 05,2012, NCERT EXEMPLAR]
SOLUTION Let the length, breadth and height of the box x, x and y units respectively. It is given

ee
that the area of the card board is c2 sq. units.

Fr
x2 + 4xy = c2
Let V be the volume of the box. Then, for
ur
V = x2y ...(h)
2 2^
ks

C -X
V = x2 [Using (i)]
Yo

4x
oo

2 X3
B

=> P = —x-
re

4 4
dV c2 3x2 d2V 3x
=> and
ou
ad

dx 4 4 dx2 2
Y

dV
The critical points of V are given by----= 0.
dx
nd
Re

dV n c2 3x2
Now, — = 0 =>----------- = 0 => x = ~^=
Fi

dx 4 4 73
(d2v) -3c Q
Clearly, < 0. Tlius, V is maximum when x = —?=
Ux2 'x = c/73 273 73
Putting x = c/a/3 in (i), we obtain y = c/273
c c
Putting x = -j^ and y = in (ii) the maximum volume of the box is given by
c2 c c3
V 3 X 273 673 cubic units

EXAMPLE 21 The sum of the surface areas of a rectangular parallelopiped with sides x, 2x and — and a
3
sphere is given to be constant. Prove that the sum of the volumes is minimum, ifx is equal to three times
the radius of the sphere. Also, find the minimum value of the sum of their volumes.
[NCERT EXEMPLAR, CBSE 2016]

ReadYourFlow.COM
MAXIMA AND MINIMA 18.53

SOLUTION Let y be the radius of the sphere and let S be the constant value of the sum of the
surface areas of the parallelepiped and the sphere. Then,
S = 2 f x x 2.t + 2x x ^ x x j + 4m/2

or. S = 6x2 + 4 m/2 ...(i)


Let V be the sum of the volumes of the sphere and the parallelepiped. Then,
4-3 x
V = — m/ +xx 2xx —
3 3
^ 323
=> -m/- t-x1*
3 v 3
3/2
4 { S - 6x2 2 X^3 S-6x2
=> V — —n H---- •/ S = 6x2 + Any2 => y2 =
3 An 3 An

w
=> y= L(S-6x2)3/2+2x3
6V7T 3
=>
dV 1 3

Flo
2\l/2 / n \ , -X
— = -7=x-(S-6x ) 7 (-12x) + —
dx 6yn 2
2
3
3x2

ee
dV p=(S-6x2)1/2x+2x2

Fr
...(h)
dx -Jn V
dV for
The critical numbers of 1/ are given by — = 0.
ur
dx
Now,
ks

^=0
Yo
oo

dx
B

=> (S-6x2)1/2x +2x2 =0


re

7C
3x (S-6x2)1/2 =2x2
=>
ou
ad
Y

=> L(S-6x2)1/2 = 2x
nd
Re

9 (S-6x2) =4jix2 [Squaring both sides]


Fi

9 (4m/2) = 4mx2 [Using (i)]


=> 9y2=x2
=> x = 3y
2
t 2 4tcx
Putting x = 3y or, y = —in (i), we obtain S - 6x + —
3 9
Differentiating (ii), we obtain
d2V = 3 (S^)!/2 18x2
+ Ax
dx2 Jn yjs~6x2

When x = 3t/ or, y = —


—, we obtain
3
1/2
d2V __ -3 4tix2 18x2 27 x 27 x
+ 4x = -2x + + 4x = + 2x > 0
rfx2 9 Vrc f-^Vrrxj 71 n

ReadYourFlow.COM
18.54 MATHEMATICS-XII

So, V is minimum when x = 3y.


x . .. 4 2
Putting x = 3y or, y = —
- mV= — TUT -x
+— , we obtain
*?32 3 3 3

-
2 3^
+—X15 = -x-3 1 + —
2k
3 3 3 27
Hence, the sum of the volume is minimum when x = 3y i.e. x is equal to three times the radius
of the sphere and the maximum value of the sum of the volumes is V =—x3|l + —|
3 V 27 J
EXAMPLE 22 Show that the triangle of maximum area that can be inscribed in a given circle is an
equilateral triangle.
SOLUTION Let ABC be a triangle inscribed in a given circle with centre O and radius r.
The area of the triangle will be maximum if its vertex A opposite to the base BC is at a maximum
distance from the base BC. This is possible only when A lies on the diameter perpendicular to

w
BC. Thus, AD _L BC. So, triangle ABC must be an isosceles triangle. Let OD = x.
Applying Pythagoras theorem in right triangle ODB, we get

Flo
OB2 = OD2 + BD2
r2 = x2 + BD2

ee
=>

Fr
=> BD
BC = 2BD = 2 yr2 -x2
for
ur
Also, AD = AO + OD = r + x.
ks

Let A denote the area of A ABC. Then,


Yo

A = ^ (BC x AD)
oo
B

=> A = - x 2 yjr2 - x2 x (r + x) Fig. 18.40


re

A = (r + x) yjr2 -x2
ou

=>
ad

dA 2 x (r + x)
Y

=> -x [Differentiating with respect to x]


dx
nd
Re

dA r2 -rx-2x2
=>
Fi

dx
dA
The critical numbers of A are given by — = 0.
dx
^ = 0
dx
r2 - rx - 2x2
=> = 0
V-*2
=> (r - 2x) (r + x) =0
=> r - 2x = 0 [v r + x * 0]
r
=> x
2
dA r2 - rx- 2x2
Now,
dx

ReadYourFlow.COM
MAXIMA AND MINIMA 18.55

d2 A _ (-r-ix) (r2-rx-2x2)x
=> [Differentiating both sides with respect to x]
dx1 ~ (r*-x2)3'2

(d2A)
=> 2 y[3 < 0
dx2
' x = r/2
r
Thus, A is maximum when x = —.
2
y[3r
BD Jr2 -x2 => BD =
2
In A ODB,
y[3r
BD
tan 0 = => tan 0 = 2_ = V3 => 6 = 60°
OD r

w
2
ZBAC = 0 = 60°

Flo
But, AB = AC. Therefore, ZB = ZC = 60°. Thus, we obtain ZA = ZB = ZC - 60°.

ee
Hence, A is maximum when A ABC is equilateral.

Fr
EXAMPLE 23 A wire of length 36 m is to be cut into two pieces. One of the pieces is to be made into a
square and the other into a circle. What should be the lengths of the two pieces, so that the combined area of
the square and the circle is minimum? for
ur
SOLUTION Let the length of a side of the square be x metres and the radius of the circle be
y metres. It is given that the length of the wire is 36 m.
ks

4x + 2jr y = 36
Yo
oo

=> 2x + mj = 18
B

Let A be the combined area of the square and the circle. Then,
re

^ =x2 + ny2 ...(h)


18-2x\2
ou
ad

=> A — x2 + % [Using (i)]


V tc y
Y

=> 21 = x2 + — (18 — 2x)2


n
nd
Re

s
— = 2x + — (18 -2x) (-2) = 2x-- (18 -2x) and,^-4 = 2--(-2) = 2 + —
Fi

=>
dx n n dx2 n it
dA
The critical numbers of A are given by = 0.

dA 36
— = 0 => 2x- — (18 - 2x) = 0 => x =
dx n 71+4

(d2A) 8
Clearly, = 2+— >0
dx2 n
d x= 36/71+4
36 36 18
Thus, A is minimum when x = . Putting x = in (i), we obtain y -
71+4 TC + 4 71+4

So, lengths of the two pieces of wire are


36 144 18 3671
4x = 4 x m and 27ty = 27tx m
7T+ 4 7C+ 4 7C + 4 71+4

ReadYourFlow.COM
18.56 MATHEMATICS-XII

Hence, the combined area of the square and the circle is minimum when the lengths of two
144 36k
pieces are metres and metres.
71+4 n+4
EXAMPLE 24 A figure consists of a semi-circle with a rectangle on its diameter. Given the perimeter of
thefigure, find its dimensions in order that the area may be maximum. [CBSE2002]
SOLUTION Let ABCD be a rectangle and let the semi-circle be described on side AB as diameter.
Let AB - 2x and AD = 2y. Let P be the perimeter and A be the area of the figure. Then,
P = 2x + 4y + kx
KX
2
and. A = (2x) (2y) + — -(ii)

Now A = 4xy +
* 2 /4 B
=> A = x(P -2x-nx) + — [Using (i)]

w
2 2y
2
2 KX
=> A = Px-2x2 -KX +

Flo
2 D 2x c
2 Fig. 18.41
71 X
A = Px- 2x2 -

ee
=>
2

Fr
dA d2 A
=> — = P - 4x - k x and = -4 -7t
dx dx2 for
ur
dA
The critical numbers of A are given by -— = 0.
dx
dA
ks

P
— = 0 => P -4x -kx = 0 => x =
Yo

dx 71+4
oo

d2A p
B

Clearly, = - 4 - 7c < 0 for all values of x. Thus, A is maximum when x =------


dx2 71+4
re

P P
Putting x = — in (i) we get y =---------- .
ou

71+4' 6 J 2(7r+4)
ad

So, area of the figure is maximum when dimensions of the figure are:
Y

2P P
Length = 2x = and Breadth = 2y =------
nd
Re

7T + 4 71+4
Fi

EXAMPLE 25 A square piece of tin of side 24 cm is to be made into a box without top by cutting a square
from each corner andfolding up the flaps to form a box. What should be the side of the square to be cut
off so that the volume of the box is maximum? Also, find this maximum volume.
SOLUTION Let x cm be the length of a side of the square which is cut-off from each comer of the
plate. Then, dimensions of the box as shown in Fig. 18.41 are Length = 24 - 2x, Breadth = 24 - 2x
and height = x.

3
i 24 cm
<N

“ii
24 cm
Fig. 18.42

ReadYourFlow.COM
MAXIMA AND MINIMA 18.57

Let V be the volume of the box. Then,


V = (24 - 2x)2 x = 4x3 - 96x2 + 576x
dV o d2V
— = Ux1 - 192x + 576 and = 24*-192
dx dx2
dV
The critical numbers of V are given by — = 0.
dx
dV
— = 0
dx
=> Ylx2 -192x + 576 = 0 x2 -16x + 48 = 0 {x-\2){x-4) =0 => x = 12, 4
But, x =12 is not possible. Therefore, x = 4.
d2v)
Clearly, = 24 x 4 -192 < 0. Thus, V is maximum when x = 4.
dx2

w
x=4
Hence, the volume of the box is maximum when the side of the square is 4 cm.

Flo
Putting x = 4 in V =(24-2x)2x, we obtain that the maximum volume of the box is given by
V = (24 - 8)2 x 4 = 1024 cm3.

ee
Fr
EXAMPLE 26 A rectangular sheet offix perimeter with sides having their lengths in the ratio 8 :15 is
converted into an open rectangular box by folding after removing squares of equal area from all four
corners. If the total area of removed square, is 100 square units, the resulting box has maximum volume.
for
ur
Find the length of the sides of the rectangular sheet.
SOLUTION Let the sides of rectangular sheet be Sa and 15a units respectively. Let the length of
ks

each side of the squares of same size removed from each comer of the sheet be x units. Then, the
Yo
oo

dimensions of the open box, formed by folding up the flaps, are:


Length = 15fl - 2x, breadth = Sa - 2x, height = x
B

Let V be the volume of the box formed. Then,


re

I—--------15a - 2x
ou
ad
Y

3
It*
8a
nd
Re

<2
Fi

-
15a - 2x
Fig. 18.43
V = (15a - 2x) (8a - 2x) x
=> V = 120a2 x-46ax2 +4x3
d2V
=> — = 120a2 - 92ax + 12x2 and = - 92a + 24x
dx dx2
dV
The critical numbers of V are given by----= 0.
dx
dV
— =0
dx
=> 120a2 -92ax + 12x2 = 0 30a2-23ax+3x2 = 0 => (5a - 3x) (6a-x) =0 => x = 6a, x = 5a

3

ReadYourFlow.COM
18.58 MATHEMATICS-XII

5/7
Butx = 6/7 is not possible as forx = 6/7 breadth = 8/7-12/7 = -4/?, which is not possible. So,x =—.
3
w, 5/7 d2V 5/7
When x = —, —- = -92/? + 40/7 = -52/7 < 0. Thus, V is maximum when x = —.
3 dx2 3
It is given that total area of four squares removed from each corner of the sheet is 100 sq. units.
4x2 = 100 x2 = 25 => 25fl2
= 25=>a2=9=>a-3
9
Hence, the dimensions of the sheet are 15/7 = 45 and 8/7 = 24.
EXAMPLE 27 Find the volume of the largest cylinder that can be inscribed in a sphere of radius r cm.
[CBSE 2009, 2012]
SOLUTION Let h be the height and R be the radius of the base of the inscribed cylinder. Let V be
the volume of the cylinder. Then,

w
L = nR2h
Applying Pythagoras Theorem in AOC4, we get

Flo
OA2 =OC2+CA2
hf + R2=> 2 h2 B'a C A'

ee
=> r2 = R2 = r
2 4

Fr
n ha
Substituting the value of R in (i), we get
O
2 ^1,
for
ur
V = K r----- h ha r
4
ks

y = Kr2h--h3 B li- 'A


=> c-
Yo

4
oo

dV 2 3 nh2 d2V 3 nh Fig. 18.44


B

nr---------- and
dh 4 dh2 2
re

The critical numbers of V are given by - 0.


ou

dh
ad

3nh2 4r2 2
Y

dv n 2 =0 h2 =
----- = 0 => 7W - => h= r
dh 4 3 V3
nd
Re

d2V 2r
Clearly, = - 43nr < 0. Thus, V is maximum when h
Fi

dh2)h* 43'

2r ? h2 [2
Putting h = —j= in R7 = r ----- , we obtain R =J— r.
43 4' 3
Substituting the values of R and h in (i), we find that the maximum volume of the cylinder is
given by
2r 4nr3
V = nR2 h = n -r2
3 43 343
EXAMPLE 28 Show that a cylinder ofa given volume which is open at the top, has minimum total surface
area, provided its height is equal to the radius of its base. [CBSE 2011,2014]
SOLUTION Let r be the radius and h be the height of a cylinder of given volume V. Then,
9 V
V = nr2h => h = —- ...(i)
nr2
Let S be the total surface area of the cylinder which is open at the top. Then,

ReadYourFlow.COM
MAXIMA AND MINIMA 18.59

S = 2%rh + nr2
V
=> S = 2n rx + nr 2 [Using (i)]
rcr2
2V
=> S = + nr 2
r
dS 2V
=> =-----2+lnr ...(ii)
dr
ds
The critical numbers of S are given by — = 0.
dr
dS 2V
— = 0 => - -y + 2nr = 0 => V = nr3 => nr2h = nr3 => h = r [v V =nr2h\
dr r
Differentiating (ii) with respect to r, we get
d2S 4V

low
= + 2k
dr2 r*
2r\
d^S 4V
=> + 2k > 0.
dr2 h3
2r=h

ee
rF
Hence, S is minimum when h = r i.e., when the height of the cylinder is equal to the radius of

Fr
the base.
EXAMPLE 29 Show that the height of the closed cylinder ofgiven surface and maximum volume, is equal
for
to the diameter of its base. [NCERT, CBSE 2012]
ou
SOLUTION Let r be the radius of the base and h be the height of a closed cylinder of given
ks

surface area S. Then,


oo

S = 2k r2 + 2Krh
Y
B

S - 2nr2
=> h = ...(i)
re

2k r
Let V be the volume of the cylinder. Then,
ou
ad

V = K r2h
Y

2 S - 2k r2
=> [Using (i)]
nd

V = Kr
Re

27tr
\
Fi

rS -2k r3 rS
=> V = -Kr3
2 2
dV = S-3* r2
•••(ii)
dr 2
dV
The critical numbers of V are given by — = 0.
dr
dV = 0 => - - 3k r2 = 0 => S = 6k r2 => 2k r2 + 2k rh = 6k r2 => h = 2r.
Now,
dr 2
d2V
Differentiating (ii) with respect to r, we obtain = - 6jrr < 0 for all r.
dr2
Hence, V is maximum when h = 2r i.e., when the height of the cylinder is equal to the diameter
of the base.
EXAMPLE 30 Show that the height ofa cylinder, which is open at the top, having a given surface area and
greatest volume, is equal to the radius of its base. [CBSE 2004,2010]

ReadYourFlow.COM
18.60 MATHEMATICS-XII

SOLUTION Let r be the radius and h be the height of a cylinder of given surface S. Then,
S = nr2 + 2nrh
S-nr2
h =
2n r
Let V be the volume of the cylinder. Then,
V = nr2 h
2 | S-nr 2
=> y = nr -------- [Using (i)]
2nr
V
Sr-nr3 Sr nr3
y =
2 2 2~
dV S 3 2
=> — nr ...(h)
dr 2 2

w
dV
The critical numbers of V are given by — = 0.
dr
dV S 3
dr 2 2

Flo
— = 0 =>------ nr 2 = 0 => S = 3nr2 => nr2 + 2nrh = 3nr2 => r = h.

ee
Differentiating (ii) with respect to r, we get

Fr
d2V
= - 3nr < 0.
dr2 for
ur
Hence, V is maximum when r = h i.e., when the height of the cylinder is equal to the radius of its
base.
EXAMPLE 31 Show that the height of the cylinder of maximum volume that can be inscribed in a sphere
ks
Yo

of radius a is —?= [NCERT, CBSE 2001,2012,2013,2014]


oo

V3‘
B

SOLUTION Let r be the radius of the base and h be the height of the cylinder ABCD which is
re

inscribed in a sphere of radius a. It is obvious that for maximum volume the axis of the cylinder
must be along the diameter of the sphere. Let O be the centre of the sphere such that OL = x. By
ou
ad

symmetry, O is the mid-points of LM. Applying Pythagoras Theorem in AALO, we get


Y

OA2 = OL2 + AL2


=> a2 = .Y2 + AL2
nd
Re

AL = -*2
Fi

M
D C
Let y be the volume of the cylinder. Then,
y = Ti (AL)2 x LM
=> V = Ti (AL)2 x 2(OL)
=> y = n (a2 -x2) x 2x
=> y = 2 7r(a2 x-x3)
A B
dV = 2n(a2 - 3x2) and d2 V L
=> = -12 n x
dx dx2 Fig. 18.45

The critical numbers of V are given by = 0.


dx
dV = 0=> 2 7c(fl^ - 3x2) = 0=> x = -4
dx V3

ReadYourFlow.COM
MAXIMA AND MINIMA 18.61

(d2v) = -i2 tix -4L < o.


Clearly,
dx2
'X = a/ -JH
Hence, V is maximum when x=-j^ and hence LM = 2x = -^=. In otherwords, the height of

the cyclic of maximum volume is la/ V3.


EXAMPLE 32 Show that the semi-vertical angle of a cone of maximum volume and given slant height is
tan-1 V2 or cos -1 1 [NCERT, CBSE 2011,2014]
VS'
SOLUTION Let a be the semi-vertical angle of a cone VAB of given slant height /.
JnAAOV,
VO OA
cos a = and sin a =
VA Wl
v

low
VO OA
=> cos a and sin a =
l /
a
=> VO = l cos a, OA = / sin a
Let V be the volume of the cone. Then, s
l

ee
ou
V = \n(.OA)2 (VO)
rF
Fr
<D

1 7t(/ sin a) 2 (/ cos a) / sin a


1/ = — for
3
A-- O
V = — rc/ sin 2 a cos a
Is
ou
=> Fig. 18.46
ks

3
dV = */3 a 0
oo

=> -sin a + 2 sin a cos a


da 3
Y
B

rfV 7l/3 . 2 2
re

=> sm a -sin a + 2 cos a


da 3
ou
ad

dV
The critical points of V are given by —— = 0.
Y

da
dV
nd

— = 0
Re

da
Fi

%l3 2 9
=> sin a - sin a + 2 cos a = 0
3
2 2
=> 2 cos a = sin a
tan2 a = 2 => tan a = 4l [■.• a is acute .\ sin a ^ 0]
1 1 [•.• tan a = 4l]
cos a =
-Jl + tan2 a “ V3

Differentiating (i) with respect to a, we get


d2V T^— (-3sin2 acosa + 2 cos3 a -4sin2 acosa) = ^—cos3 a (2-7 tan2 a)
da2 3 3
' d2V 1 -4nl3
= I^/3
3 /3
(2-7x2) =
3 V3
<0.
k da2
tan a = fl

ReadYourFlow.COM
18.62 MATHEMATICS-XII

Thus, V is maximum, when tan a = V2 or, a = tan ^ i.e. when the semi-vertical angle of the
cone is tan -1 -Jl.

EXAMPLE 33 Shoiv that the semi-vertical angle of a right circular cone of given surface area and
maximum volume is sin -if 1 [NCERT]
3
SOLUTION Let r be radius, / be the slant height and h be the height of the cone VAB of given
surface area S. Then,
S = nr2 + iirl
S - nr2
=> l =
nr

Let V be the volume of the cone. Then, a

w
V = — nr2h
3 /

Flo
1 1 r2)
=> V2 = n2 r4 h2 — 7t2r4 (l2 [v l2=r2 + h2] h
9 9

ee
2 S - nr2

Fr
=> K r4 <
V2 = — -r2 [Using (i)]
9 nr A r O B
for Fig. 18.47
ur
*2r4 {S-nr2)2-n2r*
=> V2 =
9 n2 r 2
ks
Yo
oo

=> V2 = — S (Sr2 - 2nr4)


9
B
re

Let Z = V . Tlaen, V is maximum or minimum according as Z is maximum or minimum.

Now, Z = — S (Sr2 - 2nr4)


ou
ad

9
Y

Since S is constant.
dZ
d

^ S(2Sr-8nr3)
Re

...(h)
n

dr
Fi

dZ
The critical numbers of Z are given by = 0.
dr
Now, — = 0 => 2Sr - 8nr3 = 0=> S = 4nr2 => r2 = — ...(hi)
dr 4n
Differentiating (ii) with respect to r, we get
d2Z
dr2
d2Z 4S2
=> -I 2S-24nx — <0
dr2 4n 9

2^2 2
Thus, Z is maximum when r = —- i.e. S = 4nr . Hence, V is maximum when 5 = 4;:
4n
Now,
S = 4nr2 => nrl + nr2 = 4nr2 => l = 3r.

ReadYourFlow.COM
MAXIMA AND MINIMA 18.63

r 1
sin a = -
l 3r 3
1
Hence, V is maximum when a - sin -1
3
EXAMPLE 34 Show that the volume of the largest cone that can be inscribed in a sphere of radius R is
8/27 of the volume of the sphere. [NCERT, CBSE 2008,2010 C, 2012,2013,2014,2016]
SOLUTION Let VAB be a cone of greatest volume inscribed in a sphere of radius R. It is obvious
that for maximum volume the axis of the cone must be along a diameter of the sphere. Let VC be
the axis of the cone and O be the centre of the sphere such that OC = x. Then,
VC = VO + OC = R + x = height of the cone.
Applying Pythagoras Theorem in A AGO, we get
OA2 = AC2+OC2
=> AC2 = OA2-OC2 =R2-x2

low
Let V be the volume of the cone. Then,
V = in (AC)2 (VC)

ee
=> V = i k(R2-x2)(R+x) •••(i)
3
rF
Fr
dV = in|r2 -x2 -2x(R + x)^
=> — Fig. 18.48
dx 3
for
dV = — n(R2 -2 Rx - 3x2) and d2 V 1
ou
= ^It(-2R -6x)
ks

dx 3 dx2
dV
oo

The critical numbers of V are given by — = 0.


Y
B

, — = 0
re

dx
ou

R2 - 2Rx - 3x2 = 0 => (R - 3x) (R + x) = 0 => R - 3x = 0 => x = — [v .R + x * 0]


ad

=>
3
Y

d2 V 1
Putting x = — m — n (- 2R - 6x), we get
nd

dx2 3
Re

d2V
Fi

--Rn < 0.
dx2 3
'x= R/3
Thus, V is maximum when x = i-. Putting x = i- in (i), we obtain

1 2 R2 32 nR3
V = Maximum volume of the cone = — n R - r+L =
3 9 3 81
ii^inR3j = (Volume of the sphere).

EXAMPLE 35 Prove that the radius of the right circular cylinder of greatest curved surface which can be
inscribed in a given cone is halfof that of the cone. [CBSE 2010 C, 2012,2013]
SOLUTION Let VAB be the cone of base radius r = 0 A and height h = VO. Let a cylinder of base
radius OC = x and height = OO' be inscribed in the cone.
Clearly, A VOB ~ A B'DB

ReadYourFlow.COM
18.64 MATHEMATICS-XII

VO _ OB
B'D ~ DB
h r
=> A' B'
B'D r -x
h(r-x) i
=> B'D =
r
Let S be the curved surface area of the cylinder. Then, x
S = 2 n(OC) (B'D) \
h(r-x) 2 nh A c....o.."D B
=> S = 2nx (rx-x2) r
r r
dS 2nh d2 S 4 nh Fig. 18.49
=> (r - 2x) and
dx r dx2 r
dS

w
The critical numbers of S are given by — = 0.
dx
dS 2nh

Flo
— = 0 => (r - 2 x) = 0=> x = -
dx r 2
d2 S -4nh

ee
Clearly, < 0 for all x.

Fr
dx2 r
Hence, S is maximum when * = ~ i-e- radius of the cylinder is half of the radius of the cone.
for
ur
EXAMPLE 36 Show that the volume of the greatest cylinder which can be inscribed in a cone of height h
ks

4 3 2 ^
and semi-vertical angle a is —nh tan a. Also, show that height of the cylinder is—.
Yo

217 3
oo

[NCERT, CBSE 2001C, 2007,2008,2010,20171


B

SOLUTION Let VAB be a given cone of height h, semi-vertical angle a and let x be the radius of
re

the base of the cylinder A' B' DC which is inscribed in the cone VAB.
ou

In AVO' A'
ad

O'A' x
Y

tan a =
VO' VO'
d
Re
n

VO' = x cot a
Fi

=> OO' = VO-VO' = h - x cot a


Let V be the volume of the cylinder. Then,
A' B'
V = (O' B')2(00')
■ x
=> V = nx2 (h — x cot a) . -(ii)
dV 2
— = 2;: xh - Sti x cot a
dx
dV
The critical numbers of V are given by — = 0. A C O D B
dx
dV Fig. 18.50
— = 0
dx
==> 2n xh - 3n x2 cot a = 0
2h w
=> x = — tan a [v x * 0]
3

ReadYourFlow.COM
MAXIMA AND MINIMA 18.65

dV i
Now, — = 2nx h — 3tc x cot a
dx
d2V
=> = 2nh - 6k x cot a
dx2
2h
When x = — tan a
3
d2V
= k {2h — 4/z) = -27i/z<0.
dx2
Hence, V is maximum when x = — tan a.
3
2h
Putting x = -— tan a in (ii), the maximum volume of the cylinder is given by
3
\2
2h 2h\ 4 ,32

w
V = k — tan a h - -— = — k h tan a.
3 3 27
2h
Putting x = — tan a in (i), we get
3
00' = /j -x cot a =
Flo
h_2h = h

ee
3 3

Fr
Hence, height of the cylinder = 00' = ^
for
ur
LEVEL-2
ks

EXAMPLE 37 Let AP and BQ be two vertical poles at points A and B respectively. If AP =16 m,
Yo

BQ = 22 in and AB = 20 m, then find the distance of a point R on AB from the point A such that
oo

RP + RQ is minimum. [NCERT, CBSE 2010]


B
re

SOLUTION Let R be a point on AB such that AR = x m. Then, RB = (20 - x) m.


Applying Pythagoras Theorem in A’s RAP and RBQ, we get
ou
ad

PR2 =x2 +162


Y

and. RQ2 =222 +(20 -x)2 ...(ii)


nd
Re

PR2 + RQ2 =x2 + 162 + 222 + (20 - x)2 = 2x2 - 40x + 1140
Fi

Let Z=RP2 + RQ2.Then,


Z = 2x2 - 40x + 1140. Q

dZ
=> — = 4x - 40 and
dx dx2 22 m
dZ
The critical numbers of Z are given by — = 0.
dx
dZ A—--- x —-R (20 - x) m B
— = 0 => 4x - 40 = 0 => x = 10
dx Fig. 18.51

d2Z
Clearly, = 4 > 0 for all x. So, Z is minimum when x = 10.
dx2
Thus, RP2 + RQ2 is minimum when, the distance of R from A is 10 m.

ReadYourFlow.COM
66 MATHEMATICS-XII

EXAMPLE 38 If the length of three sides of a trapezium other than base are equal to 10 cm, then find the
area of trapezium when it is maximum. [NCERT, CBSE 2010,2013]
SOLUTION Let ABCD be the given trapezium such that AD = DC = BC =10 cm. Draw
DP and CQ perpendiculars from D and C respectively on AB.
Clearly, A APD = A BQC.
Let AP -x cm. Then, BQ =x cm.
By applying Pythagoras Theorem in A APD and A BQC, we obtain
DP =QC =,]l00-x2

Let A be the area of trapezium ABCD. Then,


1
A = ^ (AB + CD) x DP
10 cm

w
=> A =| (10 +10 + 2x) x y]m-x2

Flo
=> A =(10 + x) -jm - x2

ree
dA x (10 + x) _ 100 -IQx - 2x2
=> = yjm -x2 - --- 10 cm —-------
dx ^100 - x2 -^100 - x2

F
A x cm P Q x cm B
dA or
ur
The critical numbers of A are given by — = 0. Fig. 18.52
dx
sf
, ^ =0
k
Yo

dx
oo

100 -lOx - 2x2


B

=> =0
•JlOO - x2
re

=> 100 -10x-2x2 =0


ou
ad
Y

=> x2 + 5x -50 = 0

=> (x + 10) (x -5) = 0


nd
Re

=> x =5 [v x > 0 x + 10 ^ 0]
Fi

dA _ 100-10x-2x2
Now,
dx VlOO-x2

-JlOO-x2 (-10 - 4x) + (100 -lOx - 2x2) x


d2A ■jm-x2 2x3 - 300x -1000
dx2 100-x2 (100-x2)3/2
d2A) -30
=> <0
dx2 -J75
'x = 5
Thus, the area of the trapezium is maximum when x = 5. Putting x = 5 in (i), the maximum area is
given by

A = ^ (10+5) 7100 -25 = cm2.

ReadYourFlow.COM
MAXIMA AND MINIMA 18.67

EXAMPLE 39 A telephone company in a town has 500 subscribers on its list and collects fixed charges of
? 300 per subscriber. The company proposes to increase the annual subscription and it is believed that
every increase oft 1 one subscriber will discontinue the service. Find what increase will bring maximum
revenue? [NCERT EXEMPLAR]
SOLUTION Let the increase of t x in annual subscription of t 300 maximize the profit of the
company. Due to this increase of t x, x subscribers will discontinue the service. Therefore,
Number of subscriber using the service =500 -x
Annual subscription of each subscriber = t (300 + x)
Let R be the total annual revenue of the company. Then,
R = (500 - x) (300 + x)
=> R = 150000+ 200.v-.v2
dR d2R
=> - = 200 - 2x and =-2
dx dx2

w
The stationary values of R are given by ^ = 0.
dx
, ®=0

=>
dx
200-2x = 0 => x =100
Flo
ee
d2R

Fr
Clearly, 2 < 0 for all x.
dx2
So, R is maximum when x =100.
for
ur
Thus, the total revenue received will be maximum if annual subscription is increased by ?100.
EXAMPLE 40 Find the point on the curve y2 = Ax which is nearest to the point (2,1).
ks
Yo

SOLUTION Let P(x, y) be a point on y2 = Ax and A{2,1) be the given point. Then,
oo
B

AP2 = (*-2)2 + (y-l)2


re

f 2 2
=> AP2 = -----2 +(i/-l)2 [v y1 = Ax :. x = y2/4]
ou

\ 4
ad

/
Y

Let Z = AP . Then, Z is maximum or minimum according as AP is maximum or minimum.


2 f
nd
Re

Now, Z = -— 2 + (y -l)2
4
Fi

dZ f 2 d2Z _ 3y2
— = 2 y
dV r-2 (f)+2(y-1)=T-2 and' dy2 A

The critical numbers of Z are given by ^ = 0.


dy
— = 0=>^--2 = 0=>y3=8=>y = 2
dy

Clearly,
d2z}\ 3 (2)2
—^— = 3 > 0. Thus, Z is minimum when y = 2.
dy2 j
y=2
Putting y = 2 in y = Ax, we obtain x = 1. So, the coordinates of P are (1, 2).
Hence, the point (1, 2) on y2 = 4x is nearest to the point (2,1).

ReadYourFlow.COM
18.68 MATHEMATICS-XII

EXAMPLE 41 A jet ofan enemy isflying along the curve y = x2 + 2.A soldier is placed at tl point(3,2).
What is the shortest distance between the soldier and the jet?
SOLUTION Let P(x, y) be the position of jet and the soldier is placed at A (3, 2). Then, the
distance between the soldier and jet is given by
AP = f(x - 3)2 + (i/ - 2)2 = ^(x-3)2+x4 [v y = x2 + 2]
Let Z = AP2. Then, Z = (x - 3)2 + x4
Clearly AP is maximum or minimum according as Z is maximum or minimum.
Now, Z = (x - 3)2 + x4
d7 d2Z
i
=> = 2 (x - 3) + 4x3 and = 12x2 + 2
dx dx2
dZ

w
The critical numbers of Z are given by -— = 0.

^=0
dx

Flo
ree
=> 2 (x - 3) + 4x3 =0
2x3 + x - 3 = 0

F
=> (x -1) (2x2 + 2x+ 3) = 0 or
ur
=> x=1 [•.• 2x2 + 2x + 3 = 0 gives imaginary values of x]
sf
d2Z
k
Yo

Clearly, = 12 + 2 = 14 > 0.
oo

dx2
?x= 1
B

Thus, Z is minimum when x = 2. Putting x = 1 in y = x2 + 2, we obtain y = 3. So, the coordinates


re

of P are (1,3).
ou
ad

Hence, AP is minimum when jet is at the point (1, 3) on the curve.


Y

Putting x =1 and y = 3 in AP =f(x-3)2+(y-2)2, we get


nd
Re

AP = -J(l - 3)2 + I2 = a/5. Hence, the shortest distance = Vs.


Fi

EXAMPLE 42 Find the shortest distance between the line y - x = 1 and the curve x -y2.
SOLUTION Let P (t2 ,t) be any point on the curve x = y2. The distance S of P from the given line is
t-t2-l t2-t + l t2-t + l
S = [•• t2 f + 1 > 0 for all f e R]
a/2 V2 a/2

dS _ 2t-l d2S
and
dt f2 it2 ^
dS
The critical numbers of S are given by — = 0.
dt
— = 0 ^ 2f-l = 0 => t=-
dt 2
d2S
Clearly, = a/2 > 0 for all t. So, S is minimum when t = — .
dt2 2

ReadYourFlow.COM
MAXIMA AND MINIMA 18.69

1 t2-t + l
Putting t=-mS = , the minimum value of S is given by
V2
if 1 + 1
2 2 3V2
S =
V2 8
EXAMPLE 43 Find the shortest distance of the point (0, c) from the parabola y = x , where
0 < c <5. [NCERT]
SOLUTION Let P (x, y) be any point on the parabola and Q (0, c) be the given point. Then,
PQ2 =x1 +(y-c)2

=> PQ2 =x2 + {X2 -C)2 [.■y = x2]

w
=> PQ2=x4-x2 (2c -l) + c2
Clearly, PQ will be minimum when PQ is minimum. Let Z = PQ . Then,
Z =x4 -x2 (2c -1) + c2

Flo ,Y

ree
\y = x1
d2Z 0(0, c)
=> — =4x3 - 2* (2c-1) and = 12x2 - 2 (2c -1)

F
dx dx2
or P(x,y)
dZ
ur
The critical numbers of Z are given by — = 0.
dx
sf
, ^=0 X' O X
k
Yo

dx
oo

=> 4x3 -2x (2c -1) = 0


B

Y'
re

=> 2x {2x2 -(2c -1)} = 0


Fig. 18.53
ou
ad

=> X = 0, x = ±
Y

=> x = 0, x = ± a, where a =
nd
Re
Fi

d2z) = 12a 9 -4a9 =8a2 > 0. So, Z is minimum at x = ± a.


Clearly,
dx2
Jx = ± a
9 9 9 9
Hence, PQ is minimum at x = ± a. Putting x = ± a in PQ = x + (x - c) , the minimum value
of PQ is given by
PQ2 = a2 + (a2 - c)2 = 1 + 2c-1 -c |2 _ 2c-1 1 _ 4c-1
2 I 2 +4~ 4

=> PQ
V4^T
2
V4c-1
Hence, the minimum distance is
2
EXAMPLE 44 Find the area of the greatest isosceles triangle that can be inscribed in a given ellipse
having its vertex coincident with one end of the major axis. [NCERT, CBSE 2010]

ReadYourFlow.COM
18.70 MATHEMATICS-XII

2 2
SOLUTION Let the equation of the ellipse be + ^ = 1.
a1 b2
Let APQ be an isosceles triangle having one vertex at A (a, 0). Let the coordinates of P be
(a cos Q ,b sin 0). Then the coordinates of Q are (a cos 0 , -fr sin 0).
Let A be the area of A APQ. Then,
A = | (PQ) (AM)
Y
=> A = ^ (2b sin Q) (a - a cos 0)
B(0, b)
P(a cos 0,b sin 0)
=> A = ab (sin 0 - sin 0 cos 0)
=> — = ab (cos 0 - cos2 0 + sin2 0) X' o X
dQ A'(-a, 0) M A(a, 0)
dA
=> — = ab (cos 0 - cos 20)

w
dQ B'(0,-b) Q(a cos 0, -b sin 0)
dA
The critical numbers of A are given by = 0.
r

Flo
dQ
dA Fig. 18.54
— = 0

ee
dQ

Fr
=> ab (cos 0 - cos 20) = 0
=> cos 0 = cos 2 0 for
ur
=> 0 = 271-20
2 71
=> 0 = —
ks

3
Yo

dA
oo

Now, — = ab (cos 0 - cos 2 0)


dQ
B

d2 A
re

= rtb (- sin 0 + 2 sin 20)


dQ2
ou
ad

In
For 0 = —, we obtain
Y

3
d2A 271 ^ . 4ti) . ^-2x^ <0
nd

= ab - sin — + 2 sin — = ab
Re

dQ2 3 3 2 2
Fi

Hence, A is maximum when 0 = 2n/ 3. The maximum area A is given by


2tc . 27t 27i) , V3 ^3 1) 3V3
A = ab sm----- sm — cos —\ = ab ---- +----- x — ab.
3 3 3 2 2 2 4

2 2
X If
EXAMPLE 45 Find the area of the greatest rectangle that can be inscribed in an ellipse -— + ^2— = 1.
a2 b2
[CBSE 2013]
2 2
SOLUTION Let PQRS be a rectangle inscribed in the ellipse + = 1. Let the coordinate of
a2 b2
P be (acos 0, fr sin 0). Then, the coordinates of Q, R and Sare (-rtcos0, frsin 0), (-acos0, bsin 0)
and (a cos 0, -b sin 0) respectively.
Let A be the area of rectangle PQRS. Then,
A = PQx PS
=> A = 2a cos 0 x 2b sin 0

ReadYourFlow.COM
MAXIMA AND MINIMA 18.71
y

Q(- a cos 0, b sin 0) B P(a cos 0, b sin 0)

A
X' A' X

R
(- a cos 0, - i> sin 0) B' S(a cos Q,-b sin 0)

Y'
Fig. 18.55

=> A = lab sin 20 -(i)

w
dA d2A
=> —- = lab cos 20 and = - &ab sin 20
dQ rf02

Flo
dA
The critical numbers of A are given by — = 0.

ree
^ = 0 => lab cos 20 = 0 => cos 20 = 0 => 20 = - or, — => 0 = - or, 0 = —

F
dQ 2 2 4 4
or
ur
Clearly, = - Sab sin ~ = “ < 0- So, A is maximum when 0 =
de2
sf
4
k
Yo

Putting 0 = ^ in (i), the maximum value of A is given by A = lab sin = lab.


oo
B

Hence, the area of the greatest triangle is lab sq. units.


re

EXAMPLE 46 A point on the hypotenuse of a right triangle is at distances a and bfrom the sides of the
triangle. Show that the minimum length of the hypotenuse is (a2''3 + b2^ 3)3//2.
ou
ad

[NCERT, CBSE 20081


Y

SOLUTION Let AOB be a right triangle with hypotenuse AB such that a point P on AB is at
distances a and b from OA and OB respectively, i.e. PL =a and PM = b.
nd
Re

Let ZOAB = 0. In A's ALP and PMB B


Fi

PL PM
sin 0 —- and cos 0 =
AP BP
a
=> sin 0 = ---- andj cos 0a = — b
AP BP
=> AP = a cosec 0 and BP = b sec 0 M
Let / be the length of the hypotenuse AB. Then,
l = AP + BP
=> l = a cosec Q + b sec 0
dl O L A
=> a cosec 0 cot 0 + fr sec 0 tan 0
dQ Fig. 18.56

d2l
and. = a cosec15 Q + a cosec 0 cot Q + b sec 0 + b sec 0 tan 0
dQ2
dl
The critical numbers of l are given by — = 0.
dQ

ReadYourFlow.COM
18.72 MATHEMATICS-XII

dl
---- = 0
dQ
=> - a cosec 0 cot 0 + fr sec 0 tan 0 = 0
a cos 0 b sin 0
=> --------- ?— +------ ?— = 0
sinz 0 cosz 0

=> tan3 0 = ^
b
a\1/3
=> tan 0 =
b
a1'3 b1/3
=> sin 0 = and, cos 0 =
>2/3^2/3 Ja2/3+b2/3

w
d2l fa\1/3 (a\1/3
Clearly, > 0 for tan 0 = I - J . Thus, / is minimum when tan ^ = |r I
rf02
The minimum value of / is given by

Flo a 2/ 3

ee
b'2/3
/ = flcosec 0 + b sec 0 = 0^1 + cot2 0 + -Jl + tan2 0 = fl Jl + + b ,ll +

Fr
a b
=> l = (n 2/3 + b213)312. for
ur
EXERCISE 18.5
ks

LEVEL-1
Yo
oo

1. Determine two positive numbers whose sum is 15 and the sum of whose squares is
eB

minimum.
2. Divide 64 into two parts such that the sum of the cubes of two parts is minimum.
r

3. How should we choose two numbers, each greater than or equal to - 2, whose sum is 1 /2 so
ou
ad

that the sum of the first and the cube of the second is minimum?
Y

4. Divide 15 into two parts such that the square of one multiplied with the cube of the other is
minimum.
nd
Re

5. Of all the closed cylindrical cans (right circular), which enclose a given volume of 100 cm ,
Fi

which has the minimum surface area? [NCERT, CBSE 2014]


6. A beam is supported at the two ends and is uniformly loaded. The bending moment M at a
distance .t from one end is given by
WL W 2
(ii) M
Wx _ w r3
(i) M=- .t------- X
2 3 ~ 3 L2
Find the point at which A4 is maximum in each case.
7. A wire of length 28 m is to be cut into two pieces. One of the pieces is to be made into a
square and the other into a circle. What should be the lengths of the two pieces so that the
combined area of the circle and the square is minimum? [NCERT, CBSE 2007,2010]
8. A wire of length 20 m is to be cut into two pieces. One of the pieces will be bent into shape of
a square and the other into shape of an equilateral triangle. Where the wire should be cut so
that the sum of the areas of the square and triangle is minimum? [CBSE 2005]
9. Given the sum of the perimeters of a square and a circle, show that the sum of their areas is
least when one side of the square is equal to diameter of the circle.
[NCERT, CBSE 2005,2011,2014]

ReadYourFlow.COM
MAXIMA AND MINIMA 18.73

10. Find the largest possible area of a right angled triangle whose hypotenuse is 5 cm long.
[CBSE2000]
11. Two sides of a triangle have lengths' a' and 'b' and the angle between them is 0. What value
of 0 will maximize the area of the triangle? Find the maximum area of the triangle also.
1CBSE2002C]
12. A square piece of tin of side 18 cm is to be made into a box without top by cutting a square
from each corner and folding up the flaps to form a box. What should be the side of the
square to be cut off so that the volume of the box is maximum? Also, find this maximum
volume. [NCERT]
13. A rectangular sheet of tin 45 cm by 24 cm is to be made into a box without top, by cutting off
squares from each corners and folding up the flaps. What should be the side of the square to
be cut off so that the volume of the box is maximum possible? [NCERT]
14. A tank with rectangular base and rectangular sides, open at the top is to be constructed so
that its depth is 2 m and volume is 8 m3. If building of tank costs ? 70 per square metre for

w
the base and ? 45 per square matre for sides, what is the cost of least expensive tank?
[NCERT, CBSE2009]
15.

Flo
A window in the form of a rectangle is surmounted by a semi-circular opening. The total
perimeter of the window is 10 m. Find the dimensions of the rectangular part of the

ree
window to admit maximum light through the whole opening.

F
[NCERT, CBSE 2000,2002,2011,2014]
16. A large window has the shape of a rectangle surmounted by an equilateral triangle. If the
or
ur
perimeter of the window is 12 metres find the dimensions of the rectangle that will produce
sf
the largest area of the window. [CBSE 2011]
17. Show that the height of the cylinder of maximum volume that can be inscribed in a sphere
k
Yo

2R
oo

of radius R is-F= [NCERT]


V3‘
B

18. A rectangle is inscribed in a semi-circle of radius r with one of its sides on diameter of
re

semi-circle. Find the dimensions of the rectangle so that its area is maximum. Find also the
area.
ou
ad

19. Prove that a conical tent of given capacity will require the least amount of canvas when the
Y

height is V2 times the radius of the base. [NCERT, CBSE 2007,2011,2013]


20. Show that the cone of the greatest volume which can be inscribed in a given sphere has an
nd
Re

altitude equal to 2/3 of the diameter of the sphere.


Fi

21. Prove that the semi-vertical angle of the right circular cone of given volume and least
curved surface is cot-1(\/2). [CBSE 2014]
22. An isosceles triangle of vertical angle 20 is inscribed in a circle of radius a. Show that the
area of the triangle is maximum when 6 = ~- [NCERT EXEMPLAR]

23. Prove that the least perimeter of an isosceles triangle in which a circle of radius r can be
inscribed is6V3r. [CBSE 2016]
24. Find the dimensions of the rectangle of perimeter 36 cm which will sweep out a volume as
large as possible when revolved about one of its sides. [NCERT EXEMPLAR]
25. Show that the height of the cone of maximum volume that can be inscribed in a sphere of
radius 12 cm is 16 cm. [CBSE 2005]
o
26. A closed cylinder has volume 2156 cm . What will be the radius of its base so that its total
surface area is minimum? [CBSE2000C]
27. Show that the maximum volume of the cylinder which can be inscribed in a sphere of
radiusSx^ cmis 500 7rcm3. [CBSE 2004]

ReadYourFlow.COM
18.74 MATHEMATiCS-XII

28. Show that among all positive numbers x and y with x1 + i/2 = r2, the sum x + 1/ is largest
when x=y = r/4l.
29. Determine the points on the curve x2 = 4i/ which are nearest to the point (0, 5).
30. Find the point on the curve y = 4x which is nearest to the point (28).
31. Find the point on the curve x = 81/ which is nearest to the point (2,4). [CBSE2007]
32. Find the point on the parabolas x = 2y which is closest to the point (0, 5).
33. Find the coordinates of a point on the parabola y = x2 + 7x+ 2 which is closest to the
straight line y = 3x - 3. [CBSE 2015]
34. Find the point on the curve y = 2x which is at a minimum distance from the point (1, 4).
[CBSE2011]

w
35. Find the maximum slope of the curve y = - x3 + 3x2 + 2x - 27.
0
X"
36. Tlie total cost of producing x radio sets per day is ? — + 35x + 25 and the price per set at

Flo
ree
which they may be sold is ? 150 - ^ j. Find the daily output to maximize the total profit.

F
37. Manufacturer can sell x items at a price of ? 15 - j each. The cost price is ? f ^ + 500 j
or
ur
Find the number of items he should sell to earn maximum profit. [NCERT, CBSE 2009]
sf
LEVEL-2
k
Yo
oo

38. An open tank is to be constructed with a square base and vertical sides so as to contain a
B

given quantity of water. Show that the expenses of lining with lead will be least, if depth is
re

made half of width.


39. A box of constant volume c is to be twice as long as it is wide. The material on the top and
ou
ad

four sides cost three times as much per square metre as that in the bottom. What are the
Y

most economic dimensions?


40. The sum of the surface areas of a sphere and a cube is given. Show that when the sum of
nd
Re

their volumes is least, the diameter of the sphere is equal to the edge of the cube.
Fi

41. A given quantity of metal is to be cast into a half cylinder with a rectangular base and
semicircular ends. Show that in order that the total surface area may be minimum, the ratio
of the length of the cylinder to the diameter of its semi-circular ends is tt :(rr + 2).
42. The strength of a beam varies as the product of its breadth and square of its depth. Find the
dimensions of the strongest beam which can be cut from a circular log of radius a.
43. A straight line is drawn through a given point P (1,4). Determine the least value of the sum
of the intercepts on the coordinate axes.
44. The total area of a page is 150 cm~. The combined width of the margin at the top and bottom
is 3 cm and the side 2 cm. What must be the dimensions of the page in order that the area of
the printed matter may be maximum?
45. The space s described in time t by a particle moving in a straight line is given by
s =t5 - 40t3 + 30 f2 + 80f - 250. Find the minimum value of acceleration.
46. A particle is moving in a straight line such that its distance s at any time t is given by
4
S= -2t + 4t~ -7. Find when its velocity is maximum and acceleration minimum.
4

ReadYourFlow.COM
MAXIMA AND MINIMA 18.75

ANSWERS

1. 15/2,15/2 2. 32, 32 3. i_ n j_
2 VsJ' V3
1/3
4. 6,9 5. The cylinder with radius j

L L 28n 112 8 80V3 180


6. (i) x = - (ii) x = 7. m. m
V3 71+4 7T + 4 9 + 4V3' 9 + 4V3

10. — cm2 11. —, Area = -ab 12. 3 cm, 432 cm3


4 2 2
20 10
13. 5 cm 14. ? 1000 15. Length = , Breadth =
7t + 4 71 + 4

w
, 12 18-6^3 18. -£=, -Jlr, Area = r2 24. 12 cm, 6 cm
•6-V3' 6-V3 V2
26. 7 cm 29. (± 2^37 3)

Flo 30. (4,-4)

ee
31- (4,2) 32. (± 2V2, 4) 33. (-2,-8)

Fr
34. (2, 2) 35. 5 at (1,-23) 36. 10 units 37. 240
1/3 1/3
39. Length = 2(^)
for
'Breadfc Ki)''Height Kir)
ur
38. a = - 3, b = - 9, c e R
ks

42. Breadth = -^L , Depth -2a — 43. 9


Yo

V3
oo

3
B

44. Length = 15 cm. Width = 10 cm 45. a = - 260 at f = 2


re

2
46. Velocity is max. at f = 2 - , Acceleration is min. at t = 2
V3
ou
ad

_____________________________________ HINTS TO NCERT& SELECTED PROBLEMS


Y

5. Let r be the radius and h be the height of the closed cylindrical cane of volume 100 cm .
nd
Re

Then,
Fi

nr2 h =100 => h = 100 -(i)


nr 2
Let S be the surface area of the can. Then,
S = 2nr h + 2nr2
200
=> S = + 2%r2 [Using (i)
r
dS 200 „ J d2 S 400
=> =---- — + 4 7T r and —— = —— + 4 n
dr rz dr1 r6
dS
The critical numbers of S are given by — = 0.
dr
dS 200 50 ^ 3
— = 0 => - + 4 71 r = 0 => 4 Ttr3 = 200 => r =
dr rL K
d2S 50n1/3
Clearly, —^ > 0 for all r. Hence, S is minimum when r = I —
dr2 \ 7c

ReadYourFlow.COM
18.76 MATHEMATICS-XII

7. Let r be the radius of the circle and x meter be the length of each side of the square. Then,
14 - 2x
2 nr + 4 x = 28=> nr + 2x = 14 => r =
n
Let A be the combined area of the circle and the square. Then,
A = nr2 + x2
U-2x'\2 2
=> A = n + .r [Using (i)]
n
=> A = — (14 -2x)2 + x2 = — (7 - x)2 + x2
n n
dA 8 d2A 8
=> — (7 - x) + 2x and
dx n dx2 n
The critical numbers of A are given by ^ = 0.

w
dx
28
— = 0 => --(7 -x) + 2x = 0 => x =
dx n 51+4

d2A 8

Flo 28

ee
Clearly, = — > 0 for all x. Hence, A is minimum when x =
dx2 71 71+4

Fr
112 112 28 Ti
The lengths of two partions are 4x = meter and, 28 - m respectively.
7t + 4
for 71+4 7T + 4
ur
9. Let x be the length of the each side of the square and y be the radius of the circle. Let S be the
sum of their perimeters. Then,
ks

S-4 x
Yo

S = 4x + 2ny => y =
oo

2tc
B

Let A be the sum of the areas of the square and the circle. Then,
A = x2 + Ti y2
re

=> A = x2 h——— (S — 4x)2 [Using (i)]


ou
ad

4n
Y

d2A 8
=> — = 2 x - — (S - 4 x) and = 2+
nd

dx 71 dx2 n
Re

dA
Fi

The critical numbers of A are given by = 0.


dx
dA
— = 0 => 2 x - -(S-ix) = 0 => tcx-S + 4x = 0 => x =
s
dx n 71 + 4
d2A 8
Clearly, = 2 + — > 0 for all x. So, A is minimum when x = —-— and for this value of
dx2 n 71+4
x the value of y is given by
T"— (S - 4 x) = -i- S 4S S
y =
2 7T 2 7t 7T + 4 2 (7t + 4)
Clearly, x = 2y i.e. side of the square is equal to the diameter of the circle.
Hence, A is minimum when side of the square is equal to the diameter of the circle.
12. Let the length of the each side of the square which is cut from each corner of the tin sheet be
x cm. By folding up the flaps, a cuboidal box is formed whose length, breadth and height
are 18 - 2x, 18 - 2x and x respectively. Then, its volume V is given by
V =(18-2x)(18-2x)x = 324 ix -72 x2 + 4 x3

ReadYourFlow.COM
MAXIMA AND MINIMA 18.77

dV o d2V
=> — = 324 -144 x + 12 xA and = -144 + 24x
dx dx2
The critical numbers of V are given by
— = 0 => 324 -144 x + 12 x2 = 0 => x2 -12x + 27 =0 => x = 3, 9.
dx
But, x = 9 is not possible. Therefore, x = 3.
d2V
Clearly, = -144 + 72 = -72 <0.
dx2
'X= 3

So, V is maximum when x = 3 i.e. the length of each side of the square to be cut is 3 cm.
13. Let the length of a side of the square be x cm and let V be the volume of the box. Then,
V = (45 - 2 x) (24 - 2x) x. Now, proceed as in Q. No. 10.

w
14. Let the length and breadth of the tank be x and t/ meters respectively. It is given that the
o
volume of the tank is 8mk and height is 2m.
, 4
2 xy = 8 => x y = 4 =>y= —
x

Flo ...(i)

ree
Let C be the cost of the tank. Then,
C = 70 xy + 45 (2 x 2y + 2 x 2x) = 70 xy + 180 y + 180 x

F
720
=> C = 280 + —+ 180x or [Using (i)]
x
ur
sf
dC 720 ,d2C 1440
=> — =----y- + 180 and —- = —r-
dx x2 dx2 x5
k
Yo

dC
oo

The critical numbers of C are given by — = 0.


dx
B

dC 720
re

— = 0 => -y + 180 = 0 => x = 2


dx x2
ou
ad

d2c)
Clearly, = 180 > 0. So, C is minimum when x = 2.
Y

dx2
Jx = 2
nd

720
Re

Putting x = 1 in C - 280 + + 180x, we get C = 1000.


x
Fi

Hence, the cost of least expensive tank is ? 1000.


15. Let the width and height of window be 2x m and y m respectively. It is given that the
perimeter of the window is 10 m.
.-. 2x + 2y + 7T x = 10 => y = 5 - ^ (tt + 2)

Let A be the area of the window. Then


x X
A = 2 xy + ^ x2

=> A = lOx-(7c+2) x2 + ^ x2 [Using (i)]

dA d2A
=> — = 10 - 2x (rc + 2) + rc x and, = -2(7t+2) + 7t = -7t-4
dx dx2
dA ► 2x -- ------
The critical numbers of A are given by — = 0.
dx Fig. 18.57

ReadYourFlow.COM
18.78 MATHEMATICS-XII

dA
— = 0 => 10 - 2.r (ji + 2) + ^ x = 0 => x =
dx
d2A
Clearly, = - 7i - 4 < 0 for all x.
dx2
10 5 (tt + 2)
So, A is maximum when x = and y = 5
71+4 (h + 4)
20
Hence, the dimensions of the window are 2x =
71+4

17. Let OC = OG = x. Then, AC = yfR2 -x2.

Let 1/ be the volume of the cylinder. Then.

= 7t j^R2 -x2)J

w
y 2x

=> V = 2 n(R2 x -x3)

dV d2V

Flo
ee
=> = 2n(R2 - 3x2) and = - 12 71 X
dx rfx2

Fr
dV
The critical numbers of V are given by — = 0. for
dx
ur
Now, — = 0 => R2 - 3 x2 = 0 => x = -5=
ks

dx V3
Yo
oo

Clearly,
U2v -12 k R
B

d x2 R 73
re

x~4l
R
ou

So, V is maximum when x = and height of the cylinder = 2x =


ad

73’
Y

19. Let r be the radius of the base h be the height and / be the slant height of the conical tent of
volume V and surface area S. Then,
nd
Re

v
Fi

V = — 7i r2 h and S = nr l
3

Now, S - nr 1
h

=> S2 = n2r2l2
A r B
O
=> Z = 7t2 r2(r2 + h2), where Z = S2
Fig. 18.59

2 3V
=> Z = TT2 r4 + 712 r2 v y = — nr2 h => h =
3 Tt2 r4

9 y2
Z = 7t2 r4 +
r2

ReadYourFlow.COM
MAXIMA AND MINIMA 18.79

dZ 18 V2 ,d2Z 54 V2
=> — = 4 jt2 r3 —and—T - 12 n2 r2 +
dr r3 dr2 r4
Clearly, Z is maximum or minimum according as S is maximum or minimum.
ciZ
The critical numbers of Z are given by — = 0.
dr

dZ 18V2
Now, — = 0 => 4 t:2 r3 = 0 => 4 7T2r6 =18 V2=> 2 re2 r6 = k2 r4 h2=>h=4lr
dr r3
d2Z V2
Clearly, r2 +54 —r > 0 for all values of V and r.
dr2 r4
So, Z and consequently S is minimum when h = r.
35. Slope m of the curve is given by m = ^ = - 3x2 + 6x + 2.
dx

w
Now, m - - 3x2 + 6x + 2
d2m
=> — = - 6x + 6 and

Flo
dx d?=_6

ree
The critical numbers of m are given by — = 0.
dx

F
dm
Now, = 0 => -6x + 6 = 0 => x = 1.
dx or
ur
d2m
f
Clearly, = - 6 < 0 for all x. So, m is maximum when x = 1. Putting x = 1 in the equation
dx2
ks
Yo

of the curve, we get y - -23. Thus, slope is maximum at the point (1, -23). The maximum
oo

value of slope is tn =5.


B

36. Profit P is given by


Y
re

X2
P = Revenue - Cost = ? 50 - — x - — + 35x + 25 ?[--x2+15x-25
ou

2 4 4
ad
Y

37. Suppose x items are sold to maximize the profit P. Then


P = Revenue - Cost
nd
Re

=> P =xfs—- - - + 500


Fi

100 5
24 x2
=> P = — x - -500
5 100
rfP _ 24 d2P 1
=> -— and
dx ~ 5 50 dx2 50
dP
The critical numbers of P are given by — = 0.
dx
dP 24 x
.-. — = 0 = 0 x = 240
dx 5 50
d2P 1
Clearly, < 0 for all x.
dx2 50
Hence, profit P is maximum when 240 items are sold.

ReadYourFlow.COM
18.80 MATHEMATICS-XII

43. The equation of a line passing through P (1,4) is y-4 = m(x-l), where m<0. Its
m -4
intercepts on the axes are------ and - (m - 4) respectively.
m
Let S be the sum of the intercepts. Then,
m-4
S = - (m - 4) = - m + 5 - —
w m
dS 4 d2 S 8
=> —- = -1 + — and
dm dm'" m3
The critical numbers of S are given by ^ = 0.
dx
dS
Now, = 0 => = 0 => m2 = 4 => f?? = - 2 [•.• w < 0]
dm
2,^|
For m = - = 1 > 0. So, S is minimum when m = -2.

w
dm2
For m = -2, The sum of the intercepts is given by S = 2+5 + 2 = 9.

Flo
_____________________________________VERY SHORT ANSWER QUESTIONS (VSAQs)
Answer each of the following questions in one word or one sentence or as per exact requirement of the

ee
question:

Fr
1. Write necessary condition for a point x = c to be an extreme point of the function / (x).
2. Write sufficient conditions for a point x = c to be a point of local maximum.
for
ur
3. If / (x) attains a local minimum at x = c, then write the values of /' (c) and / " (c).
1
ks

4. Write the minimum value of / (x) = x + — , x > 0.


Yo

x
oo

1
5. Write the maximum value of / (x) = x + - , x < 0.
eB

x
6. Write the point where / (x) = x loge x attains minimum value.
r

7. Find the least value of / (x) = ax + —, where a >QJb >0 and x > 0.
ou
ad

x
Y

8. Write the minimum value of f (x) = xx.


9. Write the maximum value of / (x) = x .
nd
Re

logx
10. Write the maximum value of f (x) = , if it exists.
Fi

x
________________ ANSWERS
1. f '(c) -0 2. /' (c) = 0 and / " (c) < 0 3. /' (c) = 0 and /"(c) >0
4. 2 5.-2
e e
1/r 9. c1/e 1
7. 2 Jab 8. e 10. -
e
MULTIPLE CHOICE QUESTIONS (MCQs)
Mark the correct alternative in each of the following:
1. The maximum value of x1/a, x > 0 is

(a) c 1/e (c) 1 (d) none of these


e

ReadYourFlow.COM
MAXIMA AND MINIMA 18.81

2. lfax + — > c for all positive .r where a,b,>0, then


x
c2 c2
(a) ab < —■ (b) ab > — (c) ab > — (d) none of these
4 4 4
X
3. The minimum value of......... is
loge *
(a) e (b) 1/e (c) 1 (d) none of these
1
4. For the function / (.r) = x +
-T
(a) x - 1 is a point of maximum (b) x = -1 is a point of minimum
(c) maximum value > minimum value (d) maximum value < minimum value
5. Let/ (x) =x3 + 3x2 -9x + 2. Then, / (x) has

w
(a) a maximum at x = 1 (b) a minimum at x = 1
(c) neither a maximum nor a minimum at x = — 3 (d) none of these

Flo
6. The minimum value of / (x) = x4 - x2 - 2x + 6 is
(a) 6 (b) 4 (c) 8 (d) none of these

ee
7. The number which exceeds its square by the greatest possible quantity is

Fr
1 1 , , 3
(a)- ^4 (d) none of these
4 for
ur
8. Let / (x) = (x - a)2 + (x -b)2 + (x - c)2. Then, / (x) has a minimum at x =
a +b + c 3
ks

(a) (b) 3'fabc (c) (d) none of these


Yo

3 1 1 1
oo

-+-+-
a b c
B

9. The sum of two non-zero numbers is 8, the minimum value of the sum of their reciprocals is
re

(a)
1
4 <4 1
(c) /
8
(d) none of these
ou
ad

5 ^
Y

10. The function / (x) = ^ (x -r)~ assumes minimum value at x


r=1
nd
Re

(a) 5 (b) 5/2 (c) 3 (d) 2


Fi

5 71
11. At x = — , / (x) = 2 sin 3x + 3 cos 3x is
6
(a) 0 (b) maximum (c) minimum (d) none of these
12. If x lies in the interval [0,1], then the least value of x + x + 1 is
(a) 3 (b) 3/4 (c)l (d) none of these
13. The least value of the function / (a-) = x3 - 18x2 + 96x in the interval [0, 9] is
(a) 126 (b) 135 (c) 160 (d) 0
14. The maximum value of / (x) = —----w on [-1,1] is
4 -x + x
1 1 1 1
<a) ~ 4 (b)"3 (C)6 (d) 5

15. The point on the curve i/ = 4x which is nearest to the point (2,1) is
(a) (1, 2/2) (b) (1,2) (c) (1,-2) (d) (-2,1)

ReadYourFlow.COM
18.82 MATHEMATICS-XII

16. If x + y = 8, then the maximum value of xy is


(a) 8 (b) 16 (c) 20 (d) 24
o ^
17. The least and greatest values of / (x) = x - 6x + 9x in [0, 6], are
(a) 3, 4 (b) 0,6 (c) 0, 3 (d) 3,6
18. / (x) = sin + cos x is maximum when x =

(a)f / \ n
6
(d) 0

19. If a cone of maximum volume is inscribed in a given sphere, then the ratio of the height of
the cone to the diameter of the sphere is
(a) 3/4 (b) 1/3 (c) 1/4 (d) 2/3
20. The minimum value of x2 + 250 Vis

w
x
(a) 75 (b) 50 (c) 25 (d) 55

Flo
1
21. If / (x) = x + — , x > 0, then its greatest value is
x

ee
(a) -2 (b) 0 (C) 3 (d) none of these

Fr
1
22. If/(x) = , then its maximum value is
4x2 + 2x + 1 for
ur
(a)f wf (c) 1 (dV4
ks
Yo

23. Let x, y be two variables and x > 0, xy = 1, then minimum value of x + y is


oo

1 (djal
B

(a) 1 (b) 2 (c)2-


re

o 2n
24. / (x) = 1 + 2 sin x + 3 cos^ x, 0 < x < — is
ou

3
ad
Y

(a) Minimum at x = n/2 (b) Maximum at x = sin 1 (1/^3)

(c) Minimum at x = Jt/6 (d) Maximum at sin - 1 (1/6)


nd
Re

25. The function / (x) = 2x3 - 15x2 + 36x + 4 is maximum at x =


Fi

(a) 3 (b) 0 (c) 4 (d) 2


26. The maximum value of / (x) = -—2 on hi. !]is
4 + x + xz
1
(a)-i (b)--
1
{c)l (d) -
5
27. Let / (x) = 2x3 - 3x2 - 12x + 5 on [- 2, 4]. The relative maximum occurs at x =

(a) -2 (b) -1 (c) 2 (d) 4


28. The minimum value of x logf x is equal to
(a) e (b) 1/e (c) -1/e (d) 2e (e) -e
29. The minimum value of the function / (x) = 2x 3 - 21 x2 + 36x - 20 is

(a) -128 (b) -126 (c) -120 (d) none of these

ReadYourFlow.COM
MAXIMA AND MINIMA 18.83

___ ANSWERS
1. (b) 2. (b) 3. (a) 4. (d) 5. (b) 6. (b) 7. (a) 8. (a) 9. (b)
10. (c) 11. (d) 12. (c) 13. (d) 14- (c) 15. (b) 16. (b) 17. (a) 18. (c)
19. (d) 20. (a) 21. (d) 22. (a) 23. (b) 24. (a) 25. (d) 26. (c) 27. (b)
28. (c) 29. (a)
SUMMARY
1. (i) Let/(x) be a function with domain D c R. Then, / (x) is said to attain the maximum
value at a point/? e D, if f(x) <f(a) for all x eD.
In such a case, a is called the point of maximum and/(fl) is known as the maximum value or
the greatest value or the absolute maximum value of f(x).
(ii) Let f(x) be a function with domain D c R. Then, f(x) is said to attain the minimum value
at a point a eD, if/(x) > f(a) for all x e D.

low
In such a case, the point a is called the point of minimum and/(a) is known as the minimum
value or the least value or the absolute minimum value of f(x).
(iii) A function f(x) is said to attain a local maximum at x - a if there exists a neighbour­
hood (a - 8, a + 8) of a such that /(x) <f(a) for all x e (a - 8, a + 8), x * a

ee
or, /(x) - f(a) < 0 for all x g (a - 5, a + 5), x * a.
rF
Fr
In such a case/(a) is called the local maximum value of/(x) at x = a.
(iv) A function/ (x) is said to attain a local minimum at x = a if there exists a neighbourhood
for
(a - 5, a + 8) of a such that /(x) > /(a) for all x g (a - 8, a + 5), x * a
ou
or, f(x) -f(a) > 0 for all x g (a - 6, a + 8), x * a.
ks

The value of the function at x = Ai.e.,/(A) is called the local minimum value of/(x) atx = a.
oo

The points at which a function attains either the local maximum values or local minimum
Y

values are known as the extreme points or turning points and both local maximum and
B

local minimum values are called the extreme values of /(x).


re

Thus, a function attains an extreme value at x = a if/(a) is either a local maximum value or a
ou
ad

local minimum value. Consequently, at an extreme point 'a', f(x) - /(a) keeps the same sign
for all values of x in a deleted neighbourhood of a.
Y

2. A necessary condition for /(a) to be an extreme value of a function/(x) is that / '(a) = 0, in


nd
Re

case it exists.
Above result states that if the derivative exists, it must be zero at the extreme points.
Fi

A function may however attain an extreme value at a point without being derivable there
at. For example, the function /(x) = | x | attains the minimum value at the origin even though
it is not derivable at x = 0.
This condition is only a necessary condition for the point x = a to be an extreme point. It is
not sufficient i.e., / '(a) = 0 does not necessarily imply that x = a is an extreme point. There
are functions for which the derivatives vanish at a point but do not have an extreme value
o
thereat. For example, for the function/(x) =x , / '(0) = Obut at x = 0 the function does not
attain an extreme value.
Geometrically the above condition means that the tangent to the curve y =/(x) at a point
where the ordinate is maximum or minimum is parallel to the x-axis.
As discussed in Remark 2 that all x, for which / '(x) = 0, do not give us the extreme values.
The values of x for which / '(x) = 0 are called stationary values or critical values of x and the
corresponding values of/(x) are called stationary or turning values of/(x).

ReadYourFlow.COM
18.84 MATHEMATICS-XII

3. (First derivative test for local maxima and minima) Letf(x) be a function differentiable at x = a.
Then,
(a) * = is a point of local maximum off(x), if
(i) f'(a) = 0 and,
(ii) f '(x) changes sign from positive to negative as x passes through a i.e., / '(x) > 0 at
every point in the left neighbourhood (a - 5, a) of a and /' (x) < 0 at every point in the
right neighbourhood (a, a + 8) of a.
(b) x - fl is a point of local minimum of /(x), if
(i) f'(a) = 0 and,
(ii) / '(x) changes sign from negative to positive as x passes through a i.e., / '(x) < 0 at
every point in the left neighbourhood (a - 5, a) of a and f '(x) > 0 at every point in the
right neighbourhood (a, a + 8) oi a.
(c) If / ■'(«) = 0, but / '(x) does not change sign, that is, f'(a) has the same sign in the

w
complete neighbourhood of a, then a is neither a point of local maximum nor a point
of local minimum.
4. (Higher order derivative test) Let / be a differentiable function on an interval I and let c be an
interior point of I such that

Flo
(i) /'(c) =/"(c)=/"'(c) =...=/" -1 (c) = 0, and

ee
Fr
(ii) / " (c) exists and is non-zero.
Then, for
ur
(a) if n is even and / ” (c) < 0 => x = c is a point of local maximum
(b) if nis even and/" (c) > 0 => x = c is a point of local minimum
ks
Yo

(c) if n is odd, x = c is neither a point of local maximum nor a point of local minimum.
oo

In order to find the points of local maximum/minimum of a function, we may use the
B

following steps:
re

STEP I Find f'(x)


ou

STEP II Put f'(x) = 0 and solve this equation for x. Let cl,c2,..., cn be the roots of this
ad

equation, cj ,c7..... cn are stationary values of x and these are the possible points zvhere the
Y

function can attain a local maximum or a local minimum. So, we test thefunction at each one of
these points.
d
Re
n

STEP HI Find f "(x). Consider x = c1.


Fi

If f"(c{)< 0, then x is a point of local maximum.


If / "(cj) > 0, then x = c1 is a point of local minimum.
If f "(C]) = 0, we must find f"'(x) and substitute in it c-j for x.
If / "'(c{) * 0, then x = c1 is neither a point of local maximum nor a point of local
minimum and is called the point of inflection.
If/ '"(cj) = 0, we must find fIV(x) and substitute in it q for x.
If/ IV (q) < 0, thenx = q is a point of local maximum and if/ IV(cf) > 0, thenx = q is
a point of local minimum.
If/ IV (q) =0,wemustfind/V/(x), and so on. Similarly, the values of c2)C3,..., maybe
tested.
5. Following are some properties of maxima and minima:
(i) If/(x) is continuous function in its domain, then at least one maxima and one minima
must lie between two equal values of x.

ReadYourFlow.COM
MAXIMA AND MINIMA 18.85

(ii) Maxima and Minima occur alternately, that is, between two maxima there is one
minimum and vice-versa.
(iii) If f(x) —> oo as x —» a or and /' (x) = 0 only for one value of x (say) between a and b,
then /(c) is necessarily the minimum and the least value.
If f(x) —^ — co as x —^ a or b, then /(c) is necessarily the maximum and the greatest
value.
6. The maximum and minimum values of a function defined on a closed interval may be
obtained by using the following steps.
Let i/ =/(x) be a function defined on [a, b].
STEP I Find ^ =/' (x)
dx
STEP II Put f' (x) -0 and find values ofx. Let Cj, c2,..., c„ be the values ofx.
STEP III Take the maximum and minimum values out of the values f(a),
f(c-l), /(^'-'/(Cn)'/^)-

w
The maximum and minimum values obtained in step III are respectively the largest or
absolute maximum and the smallest or absolute minimum values of the function.

Flo
ee
Fr
for
ur
ks
Yo
oo
B
re
ou
ad
Y
nd
Re
Fi

ReadYourFlow.COM
CHAPTER 19
INDEFINITE INTEGRALS

19.1 PRIMITIVE OR ANTIDERIVATIVE


DEFINITION A function 4>(x) is called a primitive (or an antiderivative or an integral) ofafunction f(x) if
<)>'(*) =f(x).

w
4 d x4 3
For example ,—^ is a primitive of x", because— — x .
4 dx 4

Flo
Let <t>(x) be a primitive of a function f(x) and let C be any constant. Then,

ree
-f-{(t>(x)+C} = <)>'(*) = /(*) [v <!>' (x) =f(x)]

F
dx
<j)(x) + C is also a primitive of/(x). or
ur
Thus, if a function/(x) possesses a primitive, then it possesses infinitely many primitives which
sf
are contained in the expression <|)(x) + C, where C is a constant.
4 4 4
k
Yo

xx 3
For example, , — + 2, — -1 etc. are primitives of x .
oo
B

19.2 INDEFINITE INTEGRAL


re

DEFINITION Let f(x) be a function. Then the family of all its primitives (or antiderivatives) is called the
ou
ad

indefinite integral off(x) and is denoted by J /(x) dx.


Y

The symbol J / (x) dx is read as the indefinite integral off (x) with respect to x.
nd
Re

Thus, (x)+cj=/(x) « J /(x) dx = $(x) + C


Fi

where (j) (x) is primitive of/(x) and C is an arbitrary constant known as the constant of integration.
Here, J is the integral sign, f(x) is the integrand, x is the variable of integration and dx is the
element of integration or differential of x.
DEFINITION The process offinding an indefinite integral of a given function is called integration of the
function.
It follows from the above discussion that integrating a function /(x) means finding a function
(j) (x) such that <t) W j = /(*)•

19.3 FUNDAMENTAL INTEGRATION FORMULAS


We know that
^ {<)>(*)} =/(*) « } fix) dx = <|>(x) + C,

ReadYourFlow.COM
* k

19.2 MATHEMATICS-XII

Based upon this and various standard differentiation formulae, we obtain the following
integration formulae:
d ( x" + 1 n+ 1
(i) -f = xn, n* - l => f xn dx = — + C, ?7 ^ -1
dx n + \ 77 + 1

1
(ii) ~ \oge X =- => [ — dx = log. I x I + C
dx x J x
d , x, x
(iii) — (e )=e => J ex dx = ex +C
dx
ax x
(iv)i = ax, a>0, a*l => f ax dx = —- +C
dx loge a J log,, a
. , d
(v) — (- cos x) = sin x => J sin x dx = - cos x + C

w
dx
(vi) — (sin x) = cos x
dx
=> 1 cos x dx - sin x + C

(vii) — (tan x) =sec2 x


dx
Flo
=> J sec2 x dx = tan x + C

ree
d 2
(viii) — (- cot x) = cosec x => J cosec x dx = - cot x + C

F
dx
(ix) — (sec x) = sec x tan x => J sec x tan x dx = sec x + C
or
ur
dx
sf
(x) — (-cosec x) = cosec x cot x => J cosec x cot x dx = - cosec x + C
dx
k
Yo
oo

(xi) — (log sin x) = cot x => J cot x dx = log | sin x | + C


dx
B

(xii) — (-log cos x) = tan x => J tan x dx = - log | cos x | + C


re

dx
ou
ad

(xiii) — {log (sec x + tan x)} = sec x => J sec x dx = log | sec x + tan x | + C
dx
Y

(xiv) — {log (cosec x - cot x)J = cosec x => J cosec x dx = log | cosec x - cot x | + C
dx
nd
Re

/
1 1 -1 x
(xv) l(sin_1f) => 1 dx = sin +C
Fi

2 2 2 2 a
a -x a -x
, d ( -1-t 1 1 -1 x
(xvi) — cos
dx a 2
a -x
2
=> I a -x
2 2
dx = cos
a
+C

/ \
(xvii) — f- tan -1 x 1 1 1 -1 X
2 2
=> S-2 — dx =— tan +C
dx y a a a +x a + xz a a
-1 X 1 dx = - cot -1 ^ + C
(xviii) — - cot
2 2
=> f- 1
dxya aj a +x J fl2 + x2 « a
... d fl -1 x = 1 1 , 1 -ix
(xix) — - sec
dx a a 2
x Jx -a
2
=>
J I 2
x x -a
2
dx = - sec
a a
+C

, , d 1 -l x 1 1 , 1 -l x
(xx) — -cosec dx = - cosec +C
dx l a a 2 2 xjx2-a2 a ya
xJx -a

Let us now discuss evaluation of some integrals based upon the above formulae.

ReadYourFlow.COM
INDEFINITE INTEGRALS 19.3

ILLUSTRATIVE EXAMPLES
LEVEL-1
EXAMPLE 1 Evaluate the following integrals:
(i) | x4 dx (ii) ^ Jx dx (iii) \-i=dx

(iv) \^3dx (V) j a 3 loga x fa r


4+ 1 5
SOLUTION (i) f x4 dx = - + c = —+ c [Using formula (i)]
J A4 + 1 5
1
++ 1
2
(ii) J Jx dx= \ x1/2 dx = + C = -x312 +C [Using formula (i)]
3
+1

low
2
1
+ l
-1/2 x 2
dx = + C =2x1/2 +C [Using formula (i)]
(iii) f jzdx= I x 1
+1

ee
2
rF
Fr
-3+1
(iv) { 4^ = 1 x-3dx=--3 + 1
X
+ C=-
2x Vc for [Using formula (i)]

4
(v) j a3 lo8« Xdx= l al°^ ^dx = j x3 dx = ^ 3+ 1
+ c =—+ c [•/ a logo * = x]
ou
3+1 4
ks

g51ogex _e41ogex
oo

EXAMPLE 2 Evaluate: J
e31ogex _ g21oge x
Y
B
re

SOLUTION Since ea logg x =x"


e5 loge x _ g4 loge x 5 4 3
ou

/ (^-i)
ad

1 ^3 loge x _ ^2 Ioge x x2(x-l)


dx = f X2 dx = — + C
J 3
Y

X -X

EXAMPLE 3 Evaluate:
nd
Re

2 2
«! dx (ii) } dx
Fi

1 + cos 2x 1 - cos 2x
SOLUTION (i) We know that 1 + cos 2x = 2 cos2 x.
2 dx = \ 2 f
see"^ x dx = tan x + C
1 1 + cos 2x 2 cos2 x
rfx =

2
(ii) We know that 1-cos2x = 2sin x.
2 2 f 2
1
1 - cos 2x
dx = j 2
2 sin x
rfx = I cosec x dx = - cot x + C

EXAMPLE 4 Evaluate:
2 2
cos 2x + 2 sin x 2 cos x - cos 2x
(i)j dx [NCERT] (ii) | dx
2 sin2 x
cos X
SOLUTION (i) We know that 1 - cos 2x = 2 sin " x.
2
cos 2x + 2 sin x
I dx
2
COS X

ReadYourFlow.COM
19.4 MATHEMATICS-XII

1-2 sin “ x + 2 sin “ .r dx = J —^— dx


=J 2 = rI sec2 x dx = tan x + C
COS X cos^ x
2
(ii) We know that cos2x = 2cos'" x -1.
2
2 cos x - cos 2x
dx
. 2 x
sin
. 2 cos2 x - (2 cos2 x -1)
. 2 dx = dx = fI cosec 2 x dx = - cot x + C
1 1
sin x sin2 x
EXAMPLE 5 lfa>0 and a evaluate thefollowing integrals:
e xloZe“dx (ii) | dx
(hi) J ex ax dx (iv) J 2 l°S‘Xdx

w
SOLUTION (i) We know that ^ =k.
ax
J ex'°^a dx = J elo^aX dx = J ax dx +C

Flo loge a

ee
(ii) We have.
a+ 1

Fr
| e“ [°ge x dx = J el°^ r“ dx=\x“ dx x
+C
a+1 for
ur
(hi) We have.
(ae)x
| ex ax rfx = | (ae)x dx +c
ks

log (ae)
Yo
oo

(iv) We know that xl08fl v x/°ZaX


B

x loge 2 + 1 xloge(2e)
\ 2 l°Zexdx = J x logt? 2 dx = +C = +C
re

log, 2 +1 loge (2c)


ou
ad

EXERCISE 19.1
Y

LEVEL-1
1. Evaluate each of the following integrals:
nd
Re

(i) J x4 dx (ii) J x5/4 dx 1


(hi) (iv) j x3/2 dx
^ 7dx
Fi

(v) J 3X dx («) J (vh) j 32log3Xdx (vih) j logr x dx

1 + cos 2x 1 - cos 2x
2. Evaluate:
(i) 1J 2
dx
e6\ogex _e5 log(> x
(ii)
2
dx

3. Evaluate: J dx [NCERT]
g4 loSe x _e31ogt,x

4. Evaluate: f —- dx
J X i x
a b
cos 2x + 2 sin 2 x 2
2 cos x - cos 2x
5. Evaluate: (i) J dx (ii) dx
. 2x 2
sin COS X

6. Evaluate: j dx
x

ReadYourFlow.COM
INDEFINITE INTEGRALS 19.5

ANSWERS
5

^ (i) y+c
- x9/4 + C (iii) -^J + C (iv) i+c
9 4x
(V) -3—
+C (vi) 3 x 1/3 + C (vii) ^- + C (viii) x + C
log 3
-X , -x
a b
2. (i) sin x + C (ii) - cos x + C 3. —+ C 4. +C
3 - l°ge (ab)
5. (i) - cot x + C (ii) tan x + C 6. 2 yfx + C
HINTS TO NCERT& SELECTED PROBLEMS
,6,5 6 .5 3
e6 loge x _ e5 \oSe x g'oge *
3. { dx = J ^7Z?dx=^2dx = T+c

low
dx =
e41°ge* _e3!°g<?* , 4 , 3
gl°ge* _ eloge ^

19.4 SOME STANDARD RESULTS ON INTEGRATION

ee
THEOREM (i) J f(x)dx =/(x)
rF
Fr
i.e., the differentiation of an integral is the integrand itself or differentiation and integration are inverse
for
operations.
ou
(ii) J A' f(x) dx = A' J /(x/ t/x, where k is a constant
ks

i.e., the integral of the product of a constant and a function = the constant x integral of the function.
oo

j {f(x) ± g(x)} dx = J f(x) dx ± J g(x) dx


Y

(iii)
B
re

i.e., the integral of the sum or difference of a finite number offunctions is equal to the sum or difference of
the integrals of the various functions.
ou
ad

PROOF (i) LetJ f(x)dx = <()(x). Then, by definition of an integral, we obtain


Y

^(<t> (*))=/(*)=> ^; \f(x)dx = f(x)


nd
Re
Fi

(ii) Let J f(x) dx = <|) (x). Then, by the definition of an integral, we get

fix)

4- $ <M*)} = k-4- (<\>(x)) = k f(x) [Using (i)]


dx dx
J k f(x) dx - k (j)(x) [By definition of an integral]
=> | k f(x) dx = k j f(x) dx [•■• j/(^) dx = <(>(x)]

COROLLARY If/(x) =1, then J k ■ dx-k ^ 1 • rfx = /c J x° dx = kx + C

Thus, integration of a constant k with respect to x is kx.


(iii) Let J /(x) dx = (j)(x) and J g(x) dx = vj/ (x) -(i)

Then, “-(<)) (x)) =/(x) and ^-(v|/(x)) =y(x)


dx dx

ReadYourFlow.COM
19.8 MATHEMATICS-XII

1
=> W ■T*
+ 2 y[x + dx
-1/2 dx + 2 ^ x^^2 dx + j x2'^2 dx
/
x3/2 //2 m+*x3,2 + 2//2
=> I = +2x + + C = 2x +c
1/2 3/2 5/2 3 5
3 - x2,+ x -1.
(vii) Let / = J — dx. Then,
x —1
x2(x-l) + (x-l) (x2 +l)(x-l) 3
x —1
dx
=1 X—1
— dx = f x2 + 1 dx = — 4- x + C
J 3
4 7
X +X +1
EXAMPLE 3 Evaluate: J —^ dx

w
X —X + 1
X4 + x2 + 1
SOLUTION Let / = dx. Then,
1-2
x -x+1

Flo
ee
/ = (x2;v2-*\x

Fr
X2 - X + 1
(x2 + 1 + x) (x2 + 1 - x) 3 2
for
ur
=> I =J (x2 - X + 1)
dx = J (x2 + x + 1) dx = + —+ x + C
2
ks
Yo

Type III INTEGRATION OF TRIGONOMETRIC FUNCTIONS


oo

EXAMPLE 4 Evaluate:
B

(i) J (3sin x-2cosx + 4sec2x-5cosec2x) rfx (ii) J + cos 2x dx


re

(iii) | - cos 2x dx (iv) J ^/l + sin 2x dx


ou
ad
Y

cos x - cos 2x
(v) J - sin 2x dx (vi) j
dx [NCERT EXEMPLAR]
1 - COS X
nd
Re

c 2 2
SOLUTION (i) Let I = 1 (3sinx-2cosx + 4sec x-5 cosec x) dx. Then,
Fi

I = 3 J sin x tfx - 2 J cos x dx + 4 J sec2 x dx - 5 J cosec2 x dx

=> I = - 3 cos x - 2 sin x + 4 tan x + 5 cot x + C


(ii) Let I = j JT+ cos 2x dx . Then,

/ = J ^2 cos2 x dx = ^2 J cos x dx = ^2 sin x + C


(iii) Let / = J ^1 - cos 2x dx. Then,

/ = J ^2 sin2 x dx = y/2 j sin x dx = - cos x + C

(iv) Let ^ = I + sin 2x dx. Then,

=> / =| 'Jsin2x + cos2x + 2sinxcosx dx

1 = | (sin x + cos x) dx = J sin x dx + J cos x dx = - cos x + sin x + C

ReadYourFlow.COM
INDEFINITE INTEGRALS 19.9

(v) Let I = J - sin 2x dx. Then,

I = J ^sin 2 x + cos2 x - 2 sin x cos x dx


2
=> I = \ Jsin1 X + cos x - 2 sin x cos x dx
=> I = J (sin x - cos x) dx = j sin x dx -1 cos x dx = - cos x - sin x + C
cos x - cos 2x
(vi) Let I=| dx. Then,
1 - cos x
COS X -(2 cos2 x -1) ,
I = I —-------------- - dx
1 - cos X
2
- (2 cos x - cos x -1) ^
=> I
=1

low
- (cos x -1)
(2 cos x + 1) (cos x -1) = J (2 cos x + l)dx = 2sinx + x + C
I =
(cos x -1)
EXAMPLE 5 Evaluate:

ee
(ii) J cot2 x dx
(i) J tan2 x dx
rF
Fr
1 cos 2x
<“>! 2— dx (iv) J . 2 2 dx
for
sin x cos x sin x cos x
2+3 cos x
ou
(V) I sin2 x dx (vi) J (2 tan x - 3 cot x)2 dx
ks
oo

cos 2x - cos 2 a
(vii) j dx [NCERT, CBSE2013]
Y

cos x - cos a
B
re

SOLUTION (i) Let / = | tan2 x dx. Then,


ou

/ = J (sec2 x -1) rfx = J sec2 x dx - J 1 • dx = tan x - x + C


ad
Y

(ii) Let J = J cot2 x dx. Then,


nd
Re

I = | (cosec2 x -1) dx = | cosec2 x dx - J 1 ■ dx = - cot x - x + C


Fi

1
(iii) Let f = J . 2 2
dx. Then,
sin x cos x
. 2 x + cos 2 x
sin
=> / =J . 2
sin x cos x
2
dx

1 1
=> 1 = 1 2
+ . 2 dx = fI sec2 x dx + fI cosec2 x dx = tan x - cot x + C
cos x sin x
cos 2x
(iv) Let / = J —^ -— dx. Then,
sin x cos x
2 • 2
cos x - sin x
I = I . 2
sin x cos x
2
dx

1 1 dx = J cosec2 x dx - J sec2 x dx = - cot x - tan x + C


=> 2
sin2 x COS X

ReadYourFlow.COM
19.10 MATHEMATICS-XII

2 + 3 cos x
(v) Let / = J dx. Then,
sin2 x
3 cos x , r 2
1 =1 tsinVx + sin—2—x dx = J (2 cosec x + 3 cot x cosec x) dx
2 f
=> I = 21 cosec x dx + 3 1 cosec x cot x dx =-2 cot x - 3 cosec x + C
(vi) Let J = J (2 tan x - 3 cot x)2 dx . Then,

I = J (4 tan 2 x + 9 cot2 x -12 tan x cot x) dx


=> Z = j {4 (sec2 x -1) + 9 (cosec2 x -1) -12} dx

=> I = J (4 sec2 x + 9 cosec2 x - 25) dx = 4 tan x - 9 cot x - 25x + C

w
cos 2x - cos 2 a
(vii) Let Z = J dx . Then,
cos x - cos a

Flo
(2 cos2 x -1) - (2 cos2 a -1) ,
Z
= J ------------------------------------
cos x - cos a
dx

ee
2 2

Fr
2 (cos x - cos a) ^
=> / =
J cos x - cos a for
ur
=> I = 2 J (cos x + cos a) dx - 2 J cos x dx + 2 j cos a dx
=> I = 2 | cos x dx + 2 cos a J 1 • dx = 2 sin x + 2x cos a + C
ks
Yo
oo

EXAMPLE 6 Evaluate:
1 1
wl
B

dx (ii) | dx
1 + sin x 1 + cos x
re

sin x sec x
(Hi) J 1 + sin x dx (iv) } dx
ou
ad

sec x + tan x
Y

SOLUTION (i) Let Z = | 1


dx. Then,
1 + sin x
nd
Re

1 1 - sin x 1 - sin x , r 1 - sin x


x dx =
—*=1 dx
Fi

1 + sin x 1 - sin x - sin x cos 2 x


sin x , r 2 , r
=> i =j 2— dx = J sec x dx - J tan x sec x dx = tan x - sec x + C
cos X COS X
1
(ii) Let / = J dx. Then,
1 + cos x
1 1 - cos x - COS X
I X dx j— dx
1 + COS X 1 - COS X 1 - COS X
1 - COS X
=> dx = J cosec2 x dx - J cot x cosec x dx = - cot x + cosec x + C
sin2 x
sin x
(iii) Let ^ = j" dx. Then,
1 + sin x
sin x (1 - sin x) sin x - sin 2 x
r Sin
z =1 dx =
J 1 . 2
dx
(1 + sin x) (1 - sin x) - sin x

ReadYourFlow.COM
INDEFINITE INTEGRALS 19.11

. 2 . 2
sin .t - sin sin x sin x
=> I
I 2
*dx = J 2 2
dx
COS X COS X COS X

=> I | tan x sec x dx - J tan2 x rfx = J tan x sec x dx - J (sec2 x -1) dx

=> 1 J sec x tan x dx - J sec 2 x dx + J 1 • dx = sec x - tan x + x + C


sec x
(iv) Let / = J dx. Then,
sec x + tan x
2
sec x (sec x - tan x) sec x - sec x tan x
l = I (sec x + tan x) (sec x - tan x) dx =
1 2 2
sec x - tan x
dx

=> I
= 1 sec 2 x - sec x tan x dx = J sec 2 x dx -1 sec x tan x dx = tan x - sec x + C
EXAMPLE 7 Evaluate:

w
. 6 6 1 + cos 4x
sin x + cos x (ii) |
(i)J • 2
sin x cos x
2
dx [CBSE 2014]
cot x - tan x
dx

(iii) j
1
dx

Flo
ee
tan x + cot x + secx + cosec x

Fr
. 6 6
sin x + cos x
SOLUTION (i) Let/ = J dx. Then,
. 2 2
sin x cos x
for
ur
2 2 3 3 sin ~ x cos x (sin x + cos x) ^ Using :fl3 +b3
9 2 2 2
(sin x + cos“ x)
I=
i
ks

. 2 2 = (a + b)3 -3ab (a+ b)


sin x cos x
Yo
oo

2 2
1-3 sin x cos x 1
I dx = J
B

=> / = -3 ■ dx
. 2 2 .2 2 .
sin x cos x sin x cos x
re

. 2 2
sm x + cos x - r\ r\
ou
ad

=> -3 dx = J (sec x + cosec“ x - 3) dx = tan x - cot x - 3x + C


. 2 2
sm x cos x
Y

1 + cos 4x
(ii) Let / = | dx. Then,
nd
Re

cot x - tan x
2
Fi

2 cos 2x cos x sin x


1
=J ? 2
cos x - sin x
dx = J cos 2x sin 2x dx = - f sin 4x dx = - — cos 4x + C
2 J 8

1
(iii) Let i = J dx. Then,
tan x + cot x + secx + cosec x
1
l =
I
sinx cosx
+ +
1
+
1
dx

cosx sinx cosx sinx


sin x cos x
=> I = .2L. 2 .
sm x + cos x + sm x + cos x
dx

2sinx cosx
/ = ---------------------dx
2 J 1 + sin x + cosx

=> 7 = if (1 + 2sinx cosx)-l dx


2J 1 + sin x +cosx
2 2
(sin x + cos x + 2 sin x cos x) -1
=> / = if
2 J 1 + sin x + cos x
dx

ReadYourFlow.COM
19.14 MATHEMATICS-XII

SOLUTION (i) Let x2 = t. Then,


I = J (x4 + x2 + 1) d(x2)

t3 t2 > r x6 4
=> I = J (t2 +t + l)dt — + — + t + C = — + — + x2 + C [V t = x2]
3 2 3 2
(ii) Lete^ =t. Then,
I = J sin (e^) rf(eA) = J sin t dt =-cost+ C =-cos(ex)+C

EXAMPLE 13 Iff (x) = 3x2 - and f (1) = 0, find f (x).


x
SOLUTION We have,
/ (*) = J f (*) dx
/

w
2 'l
=> m = J 3x2 T *
X
f -2>

Flo
3
/W = 3 - -2 i +C
3 -2

ree
1
/(x) = x3 + -2+C
=>

F
x or
=> /(l) = 1+1+C
ur
[Replacing x by 1]
sf
=> 0 = C+2 [••7(1) = o]
=> C = -2
k
Yo

/(x) = x3 + --2 [Putting C = - 2 in (i)]


oo

x
B

EXERCISE 19.2
re

LEVEL-1
ou
ad

Evaluate thefollowing integrals (1-44):


Y

1. | (3 x ^fx + 4 fx+5) dx 1
f 2X + —_____
2. dx
H ^ X1/3
nd
Re

J | Vx (ax2 +bx + c)| dx


Fi

3. 4. J (2 - 3x) (3 + 2x) (1 - 2x) dx

5. [ f — + — + mx + xm + mx \ dx 7
J ^x m
6.
Ik-i dx [NCERT]

<1+^3 ^ e
1.
I 8. J J x2 + e log* + • dx
2

9. J (xc + ex + ee) dx 10. dx

x6 +1
11.
I i(7) dx 12.
I x2 +1 dx

x-1,3 + ^+2 j (i + VJ)2 dx


13.
1 dx 14.
I
ReadYourFlow.COM
INDEFINITE INTEGRALS 19.15

16. j (* + 1)(*-2) .X
15. J (3 -5x) dx
4~x
x5 + x 2 + 2
17. j dx 18. J (3x + 4)2 rfx
x2
2x4 +7x3 + 6 x2 5 x4 + 12 x3 + 7x2
19-J x2 + 2x
dx 20. j 2
X +X
dx

sin2 x
21. dx 22. IC (sec2 x + cosec2 x) dx
j 1 + COS X
. 3
sin x - cos 3 5 cos 3 x + 6 sin 3 x
23. j .2
sin x cos 2 x
- dx [NCERT] 24-l 2 sin 2 x cos2 x
dx

1 - cos 2x

w
25. J (tan x + cot x)2 dx 26. { dx
1 + cos 2x
2 . 2
cos x cot x cos x - sin x
27j
1 - cos x
dx or
I cosec x - cot x dx
Flo28. J
+ cos 4x
dx

ree
1 1
29. J dx 3°. J 1 - sin x
dx

F
1 - cos x
tan x cosec x
31. J dx 32. j or dx
ur
sec x + tan x cosec x - cot x
sf
1 34. J 1
33. J dx dx
1 - cos 2x
k

1 + cos 2x
Yo
oo

sin 2x
35. \ tan -1 dx 36. J cos-1 (sin x) dx
B

1 + cos 2x
re

sin 2x 2 tan x
37. J cot -1 38. J sin -1 dx
ou

dx
ad

1 - cos 2x 1 + tan2 x
Y

39. | 40. | (a tan x + b cot x)2 dx


nd
Re

x - 2x + 4
Fi

* 3 - 3x2 + 5x - 7 + x2 ax dx
41. |r - 42. |
COS X
dx [NCERT]
lx2 1 + COS X
1 - COS X
43. J dx [NCERT]
1 + COS X

5 6 2 2
44. J 3 sin x - 4 cos x + + tan" x-cot x^ dx
2 - 2
cos x sin x

45. If /' (x) = x - V and /(l) = ^ , find / (x).


x 42
46. If /' (x) = x + =5, f(2) = 13, find / (x).
47. If /' (x) = 8 x3 - 2x,/(2) = 8, find/(x).
48. If f' (x) =a sin x + b cos x and /' (0) = 4,/(0) = 3, / — | = 5, find /(x).
2
1
49. Write the primitive or anti-derivative of / (x) = /x + -y= .

ReadYourFlow.COM
19.16 MATHEMATICS-XII

ANSWERS
6 5/2 8 3/2 2X
1. -x +—x + 5x + C 2. + 5 log x - — x2/ 3 + C
5 3 log 2 2
?lx7/2 + *bx5/2 + *cx3/2 + C 4. 3 x4 + — x3 17 .2
3. x +6x+C
7 5 3 3 2
2 a: m+ 1 2 2
m x mx
5. w log | x | + — + + +C 6. y-2x + log|x|+C
2 m log m m+1 2
3 2 X
3/2 + 6x5I2 + 2x7I2 8. ^ + ^- + 1 e
7. 2 V* + 2x +C +C
5 7 3 2 2
lo8 !
2 y
e+ 1
9. ^ + x+C 10. -x9/2-4Vx+C

w
e +1 9
5 3
x x _
11. 2yjx - +C 12.-------------h x + C

Flo
5 3
1/3 + ^x7/6 2/3 14. 2V^ + 2x + |x3/2 + C
13. 3x + 3x +C

ree
7
3/2 5/2 16. -x5/2-|x3/2-4V^ + C

F
15. 2x - 2x +C
5
4 -3 or
2
ur
1 (3x + 4)3 + C
18. —
17. -- + C
f
4 3 x 9
ks

19. — x3 + — x2 + C 20. —x3 +—x2 + C


Yo

3 2 3 2
oo

21. x-sinx + C 22. tan x - cot x + C


B

5
23. sec x + cosec x + C 24. — cosec x + 3 sec x + C
re

2
25. tan x - cot x + C 26. tan x - x + C
ou
ad

27. -cosec x - cot x - x + C 28. -4= + C


Y

V2
29. - cot x - cosec x + C 30. tan x + sec x + C
nd
Re

31. sec x - tan x + x + C 32. - cot x - cosec x + C


Fi

33. — tan x + C 34. - — cot x + C


2 2
2 2
K
35. —+C 36. -x- — + C
2 2 2
2
37. —+C 38. x2 + C
2
3 2
39. ±
— + — - 2x + C 40. fl2 tan x -b2 cot x - (a2 +b2 - 2ab) x + C
3 2
2 X
1 —— 3x + 5 log x + 7 a
41. —+ +C 42. - cosec x + cot x + x + C
2 2 x log fl

43. 2 (cosec x - cot x) - x + C 44. - 3 cos x - 4 sin x + 6 tan x + 7 cot x + C


x2 1 x2 13
45. — + - -1 46. — + — x-2
2 x 2 2
47. 2x4 -x 2 -20 48. /(x) = 2 cos x + 4 sin x+1 49. -x3/2 + 2x1/2 + C
3

ReadYourFlow.COM
INDEFINITE INTEGRALS 19.17

HINTS TO NCERT & SELECTED PROBLEMS


A2 2
1 X
j(^
6.

. 2
dx =
f(-*H 2
dx = — + loge x -2x + C

sin - COS X
21. — dx = \ - rfx = J (1 - cos x) dx = x - sin x + C
1 + COS X J 1 + COS X
. 3 3
sin x - cos ^ rfx = J (tan x sec x - cot x cosec x) dx = sec x + cosec x + C
23.
. 2 2
sin x cos x
2
COS X cos x (1 - cos x) cos x - COS X
42.
1 + COS X
dx
=1 2
1 - COS X
dx = . 2
sin x
dx

J (cot x cosec x - cot2 x) dx = J cosec x cot x dx - J (cosec 2 x -1) dx

w
- cosec x - ( - cot x - x) + C
2
- COS X (1 - cos x)2 1-2 cos x + cos x dx
43. dx =
J dx =
I dx
1 + COS X

Flo
(1 + cos x) (1 - cos x)
2
1 - cos2 X

ee
1-2 cos x + cos x 2 2
1 dx =
f<cosec x - 2 cot x cosec x + cot x) dx

Fr
sin2 x

J(2 cosec x - 2 cosec x cot x -1) dx = - 2 cot x + 2 cosec x - x + C


for
ur
19.5 GEOMETRICAL INTERPRETATION OF INDEFINITE INTEGRAL
ks

In order to understand the geometrical meaning of an indefinite integral, let us consider a


Yo
oo

function / given by / (x) = - 2x.


B

Clearly,
re

J / (x) dx - x2 + C, where C is the constant of integration.

Let us now consider the family of curves given by y = J / (x) dx or, y = -x2 +C.
ou
ad
Y

Clearly, y = -x2+C represents a family of parabolas having their common axis of symmetry
nd
Re

along i/-axis as shown in Fig. 19.1.


Y
Fi

i x = t?
\

P3>

X' \iyi
o
Pi
y = -x2 + 2
y = -x2 + 1
y = -x2 ^

T
Y'
Fig. 19.1

ReadYourFlow.COM
19.22 MATHEMATICS-XII

2 sin 2x cos 2x 2
/ =
I sin 2x
dx = 2 f cos 2x dx = — sin 2x + C = sin 2x + C
J 2
sin 4x
(vi) Let / = | dx. Then,
cos 2x
2 sin 2x cos 2x
/
=1 cos 2.r
rfx = 2 | sin 2x dx = - cos 2x + C

. 8 8
sin x - cos x
EXAMPLE 3 Evaluate:
J 2
1 - 2 sin x cos x
2
dx [NCERT]

• 8 8
sin x - cos x
SOLUTION Let / dx. Then,
2 2
- 2 sin “ x cos x
, ■ 4 4 w . 4 4 ,
(sin x + cos x) (sin x - cos x)
/
J2 2 2 ?
(sin x + cos x) - 2 sin " x cos x
2 dx

w
4 4 2 2 2 2
(sin x + cos x) (sin" x + cos" x) (sin x - cos x)
/
=1 . 4
sin x + cos x
4
dx

=> / J cos 2x dx
1

Flo
- - sin 2x + C

ee
2

Fr
EXAMPLE 4 Evaluate: J ^'l + sin x dx, 0 <x <k/2

SOLUTION Let ^ = J ^1 + sin x dx. Then, for


ur
. 2 x 2 x _ x x
/ = sin — + cos — + 2 sin — cos - dx
2 2 2 2
ks

\2
Yo

X- . X X _ r x
oo

I =
I cos - + sin — dx
2 2
cos — + sin
2
J dx = J cos -
2
dx +
J
sin - dx
2
B
re

=> / = 2 sin — ± 2 cos — + C = 2 sin — ± cos — + C


2 2 2 2
ou
ad

EXAMPLE 5 Evaluate:
Y

1 1
wj ^3x + 4 - ^3x + 1
dx (ii) J ^1 - 2x + ^3 - 2x
dx
nd
Re

1
SOLUTION (i) Let / = | dx. Then,
Fi

^3x + 4 -^3x + 1
^3 x + 4 + ^3x + 1
/ =
J x + 4 + ^Sx + l j^3x + 4 -^3x + l j
dx

^3x + 4 + ^3x + 1
=> /=
I (3x + 4) -(3x +1)
dx

=> / = : j i1/sx4+1/3^xi} &


=> / = — J '^3x + 4 dx + — J ^3x + 1 dx
3 3
3/2 3/2
(3x + 4) 1 (3 x +1) 2 ((3x + 4)3/2+(3x + l)3/2}+C
> +— < +C = —
3 3 3 3 27
3x 3x
2 2

ReadYourFlow.COM
INDEFINITE INTEGRALS 19.23

1
(ii) Let / = | dx. Then,

I = dx
</ -2x +

,1-27 -£_2X dx
=> I
=J -2x) -(3 -2x)

I
\ I / -2x dx + ^ ^3 - 2x dx

3/2
1 d-2x) 3/2 1 (3 - 2x) 1 3/2
=> / = - > +- < + C = ~ (1 “ 2 x) 3/2 1- (3 - 2x) +C
2 2x1 2 -2x1 6 b
2 2

w
1+ x 1-x
8 +4
EXAMPLE 6 Evaluate: J dx
2X

SOLUTION Let / = |
8
1+ X
+4
l-x

Flo
dx. Then,

ree
2X
3x+ 3 2 - 2x

F
+2
I = dx
2-r or
ur
2x+ 3 22 - 3x 2x+ 2 2- 3x
2x + 3 2 - 3x rf,= 2 2
l2
f
=> I +2 +C = - +C
2 log 2 (-3) log 2 log 2 3 log 2
ks
Yo

EXERCISE 19.3
oo

LEVEL-;
B

1 1
re

j/j (2x - 3)5 + ^3 x + 2 dx dx


(7i-5)3 fix-l
ou
ad

1 1 x+ 3
+ dx 4.
j T
Y

^2 4
2-3x - - (x + 1)
1 1
J
nd
Re

5. dx dx
yjx + 1 + yfx ^3.x + 3 + yj'Z.x — 3
Fi

2x 1
zJ (2x +1)
J dx 8.
1 + ^jx + b
dx

1 + COS X
9. | sin x + cos 2x rfx io. J dx [CBSE 2000]
1 - cos x
- COS X i
1 + cos X
dx
^•1 1 - sin -
dx

rn 1
1 + cos 3x
'4. (ex + l)2 / dx

\2
1 + cos 4x
dx
>1 cot x - tan x
dx

1
dx [CBSE 2002] [NCERT]

ReadYourFlow.COM
MATHEMATICS-XII

1
2 - dx [NCERT]
cos x (1 — tan .y)
.ANSWERS

L (2 y - 3)6 2 3/2 1
+ -(3y + 2) +c 2. —— (7 y - 5) 2+ — l5x-4+C
12 14 <5 v
3. - ilog|2-3AT|+|^3l 4. - 1 2
2 +C 3+C
2 (y + l)2 3 (y + 1)
2
5.
3
(x + l)3/2-I3/2 + C 6.±{(2,+ 3) 3/2 - (2y - 3) 3/2 +C

1 1 2 j(Y + fl) 3/2 -(x + b) 3/2


7. - log | 2y + 1 | + +C 8. +C
2 2 (2y +1) 3(a-b)
10. -2 cot f- I - y + C
9.------ cos 2y + C

w
lyjl 2

11. 2tan - -y + C 12. 2 tan — + sec — + C


2
1 - cos 3y

Flo 2 2
1 (ex + l)3 + C

ree
13. +C 14. —
3 sin 3y 3

F
1 e 2 Y + 2x--e 2x+C
15. — 16. - - cos 4y + C
2 2 8 or
ur
3/2 3/2 1
17 3{(a:+3)
f
17. - + (y + 2) +C 18. tan (2y - 3) - y + C
2
ks
Yo

19. — tan I — + y ) + C
oo

2 4
B

HINTS TO NCERT & SELECTED PROBLEMS


re

1 + COS Y
10. Let / = j
ou

dx. Then,
ad

1 - cos Y
Y

2 Y
2 cos
2 Y
I ^-rfY =
I cot.2 —
* j
1
nd

I = dx - cosec -1 rfY = -2 cot--Y + C


Re

2 Y 2 2 2
2 sin
Fi

2
18. Let / =J tan2 (2y - 3) rfY = J |sec2(2Y - 3)-l| ^y = -^ tan (2y - 3) - y + C

1
19. / = J 2 2 dx. Then,
cos y (1 - tan y)
1 1 1
I =
1 sin y
jdx = }
(cos y - sin y)
jdx = }
1 - sin 2y
dx
cos2 y 1 -
COS Y

1 1 1
=> I =
1 K
dx =
I 2 Tt
dx = —
2
J561:2 (i+x) dx
1 + cos - + 2 Y 2 cos +Y
2 V4
1 / \
r 1 . 71 + y + ^
=>/=-tan - C
2 4

ReadYourFlow.COM
INDEFINITE INTEGRALS 19.25

P(x)
19.8.2 EVALUATION OF INTEGRALS OF THE FORM -, n eN , WHERE p (x) IS A POLYNOMIAL
(ax + b)
In order to evaluate this type of Integrals, we may follow the following algorithm.
STEP 1 Check whether degree of P (x) > or <n.
STEP II If degree of P (x) < n, express P (x) in the form
A0 + Ai (ax + b) + A2 (ax + b)2 + ... + A,,(ax + bf 1

P(x) Ai a2 At-i
STEP III Write as + +... +
(ax + bf (ax + b)" (ax + b/1 ‘ 1 (ax + b)" 2 ax + b
STEP IV Evaluate
P(X) 1 dx + ... +A„_11 1
1 (ax + b)
— dx =Aq l ---- - dx + A-i J
(ax + b) (ax + b) ax + b
dx

P(x)

w
STEP V If degree of P (x) > n, then divide P (x) by (ax + bf and express as
(ax + bf
R(x)
Q(x) +
(ax + b)

Flo
- , where degree ofR (x) is less than n.

ee
Use step II and III to evaluate J ^Ldx

Fr
STEP VI
(ax + bf
for
ur
Following examples will illustrate the above procedure.
ILLUSTRATIVE EXAMPLES
ks
Yo

LEVEL-1
oo

EXAMPLE l Evaluate:
eB

3 <“>!( x-11 \4

dx dx
r

J (x + 2)4 x+1
ou
ad

3
Y

SOLUTION (i) Let 7 = f —-—-r dx. Then,


J (x + 2)4
nd
Re

^+2)-2'3*
I = 1
Fi

(* + 2)4
(x+2)3-6(^4-2)2+12(x+2)-8 a
=> J = 1 (* + 2)4

1 6 12 8
=> I =
H x+2 (x + 2)2 (x + 2)3 (x + 2)4
dx

6 8
7 = log | x + 21 h —----------- 2 T-------- 3+C
x + 2 (x + 2)2 3 (x + 2)
\4
(ii) Let J ^ x-1
dx. Then,
x+1
«x+D-2}\x
'=1 (x + 1)4

ReadYourFlow.COM
19.26 MATHEMATICS-XII

(x +1)4 -4 Ca (y +1)3 x 2 +4 C2 (x +1)2 x 22 -4 C3 (* +1) x 23 +4 C4 (2)4


=>
^=1 (x + l)4
(x + 1)4 - 8 (x + 1)3 + 24 (x + 1)2 - 32 (x + 1) +16
=> /
=1 U + D4
8 24 32 16
=> 1- dx
x+1 (x + 1)2 (x + 1)3 (x + 1)4
24 16 16
I =x-8 log| x + 11- 3+C
x+1 (x + 1)2 3 (x + 1)
EXAMPLE 2 Evaluate:
ax + b a r-\ f x + 2 2 + x + x2 2x -1
(i)j —+ dx (ll) I --------^ dx
(cx + rf)2 J (x + 1)2
P) J
-o--------- +---------9 dx
X- (2 + x) (x + l)2

w
ax + b
SOLUTION (i) Let / = J ------- y dx
(cx + d)
Let

Flo
ax + fr = X (cx + rf) + g. On equating coefficients of like powers of x, we get

ree
a be - ad
a = \c and b = Xd + \x => X = and g =
c c

F
ax + b
I = dx
(cd + d)2 or
ur
sf
X(cx + d) + u ,
=1 (cx + d)2
dx
k
Yo

1
oo

1
7 = dx + p J dx
cx + d (cx + d)2
B
re

=> I = - log | cx + d |---- - M 1


+C = log | cx + d | - X +C
c c (cx + d) c cx + d
ou
ad

ALITER Let
ax + b
Y

I
=1
(cx + d)
2 dx. Then,
nd
Re

x+-
Fi

I = a \ A dx [Making coefficient of x unity in the numerator]


(cx + d)2
be
cx +
=> i = -J -I- dx
c J (cx + d)2
[Making c as the coefficient of x in the numerator]

(cx + d) + — - d
=> 7 =
-I
c J
a
(CX + d)2
dx [Adding and subtracting d in the numerator]

1 (fee - ad)
7 = - J
1
=>
c J
— dx +
(cx + d) c I
(cx + d)2
dx [Separating the integrals]

(be - ad) 1
=> 7 ^ log | cx + d| +C
c c2 (cx + d)
(ii) Letx + 2 = A.(x + l) + ji.
On equating the coefficients of like powers of x on both sides, we get

ReadYourFlow.COM
INDEFINITE INTEGRALS 19.27

A. = 1 and 2 = A + |a => A = 1, (j. =1


x+2
I =} dx = j X(x + 1) + M dx = 1J — ■ dx
(x + 1)2 (* + l)2 x+1 (x + 1)2
1 1
I = dx = X log | x + 1|---- + C = log | x + 1| - +C
=>
^ r7T*+>iJ (x + 1)2 x +1 x+1

x+2 (x + l) + l dx =| 1 1
ALITER J=| dx = J • dx = log | x +11 - —+ C
x+1
(x + 1)2 (* + D2 x+1 (x + 1)2

2 + x + x2 2x -1
(iii) Let 7 = J • dx. Then,
x2 (2 + x) (x + ir
(2 + x) + x2 2 (x + 1) -3 2
= J V 2 +1 x 3

w
+ dx + • dx
x2 (2 + x) (x + 1)2 X x+1 (x + 1)2

Flo
7 = - - + log |2 + x| + 2 log|x + 1| + —^ + C
x

ee
EXAMPLE 3 Evaluate:

Fr
3 2
x
dx (ii) J (a + bx)2
dx
J t^+D2
for
ur
x2 +1
(iii) j - dx 1CBSE2006]
ks

(^ + D
Yo
oo

3
SOLUTION (i) Let 7 = f —- . dx.
B

1 (x + 1)2
re

Using long division method, we obtain


x3 - 3x + 2 3 (x + 1) - 3 + 2
ou
ad

= x-2+ = x-2+
(x + 1)2 (x + 1)2 (x + 1)2
Y

1
nd

=> —= x-2 + ——
Re

(x + 1)2 X+1 (x + 1)2


Fi

3 2
3 1 i x 1
dx = J • x-2 + dx --
^ - 2x + 3 log | x +11 +
x+1
+C
J (x + 1)2 x+1 (^ + D2
2
X
(ii) Let 7 =
I (a + bx)2
dx

Using Long division method, we get


2a a
2 —x-
x 1 b b2
(a + bx)2 b2 (bx + a)2
2
x 1 _ a (2bx + a)
(a + bx)2 b2 b2 (bx + a)2
2
x2 1 a 2(bx + a) - a 1 2a 1 a 1
(a + bx)2 b2 b2 (bx + a)2 b2 b2 bx + a b2 (bx + a)2

ReadYourFlow.COM
19.28 MATHEMATICS-XII

2
X
I
I (a + bx)2 dx

2
2a 1 a 1
=> I ------ + ~x (bx + a)2 • dx
1 ? b bx + n b
2
If.,, 2a r 1 1
=> I 2
b2 I l dx-^2
b2 1 bx + a dx+hb2 1 (bx + a)2
dx

x _2a| log \bx + a\~-2 1 1 ^


=> I + C = -x- \bx -2a log | bx + a\---- -— + C
b2 b bx + a b3 bx + a
ALITER We have.
2
X
/
1 dx

w
(a + bx)2
1 b12 x.2
=> /
J dx

Flo
b1 (a + bx)2
(b2 x2 + 2abx + a2) ~(2abx + a2)

ree
1
=> /
b2 I (bx + a)2
dx

F
1 (bx + a)2 - \2a (bx + a) - 2a1 + a2}
=> /
I dx or
ur
b2 (bx + a)2
sf
1 (bx + a)2 2a (bx + a) + a2
=> I I dx
k
Yo

b2 (bx + a)1
oo

2
1 2a
B

a
=> I
b2 J 1- +
bx + a (bx + a)2
• dx
re

2 2
ou

1 2a a
ad

=> I x- — log | bx + a\- + C = — \ bx -2a log\bx + a\---- -— + C


b2 b b (bx + a) b* bx + a
Y

x2 + l
nd

I
Re

(iii) Let I 2 dx. Then,


(x + 1)
Fi

x2 + 1 + 2x - 2x
I
1 (x + 1)2
dx

(x + l)2 - 2x
=> I
1 (x + 1)2
dx

2x
=> / f 1-
(x + 1)2
dx

x
=> / I l-dx-2 j
(a- + l)2
dx

=> I
1 l-dx-2 j U + l)-l dx
(x + 1)2

1 1
=> I 1 l-dx-2 J x+1 (x + 1)2
■ dx

ReadYourFlow.COM
INDEFINITE INTEGRALS 19.29

1 y dx = x - 2 log | x + 1| - ^ +C
=> / = \ \-dx-2 { rfx + 2 J
X+1 (x+1) x +1

EXAMPLE 4 Evaluate:
3 7 6
« jJ ^, dx (ii) | * dx (iu) r *
x+2 x +1 J X -1
SOLUTION (i) Using long division method, we obtain
3
x x2 - 2x + 4 - 8
x+2 x+2
3 3
8 x
=>
Jf-
x+2
dx =
Jb 2 - 2x + 4 -
x+2
dx = 1—
3
x2 + 4x - 8 log | x + 21 + C

3
ALITER Let / =| V dx. Then,
x+2

w
(x3 + 23) - 23 dx
x+2

I
(x + 2) (x2 - 2x + 4) 8

Flo
ee
=> I = dx
x+2 x+2

Fr
3
8
=> 1 = J < x2 - 2x + 4 - dx = — x2 + 4x - 8 log | x + 21 + C
for
ur
x+2 3

(ii) Using long division method, we have


ks

7
Yo

X 6 5 4 3 2 - 1
oo

X -X +X - x + x - x +1 -
x +1 X+1
B

7
6 5 4 1
I Xx+ 1 dx = J X - X + X - x3 + X2 - X +1 -
re

dx
X+1
ou
ad

7
7 6 5 4 3 2
, X X X X X X 11-11,-
=> 1- dx =-----------+------------+------------ + x - log I x +1| + C
Y

J X+1 7 6 5 4 3 2
7
nd

IJ -X + 1
Re

ALITER Let/ = dx. Then,


Fi

/ =J X+1
x7+l
=> I =
1 dx - J 1 dx
X+1 x +1
/ -(-l)7 1
=> 1 =
1 dx - J dx
x-(-l) X+1
>1 n
x -a
= xn-' + x’'-2a
=> i=K -X5 + X4 -x 3 + X2 — X + V)dx-\ x 1+1 dx x -a
-32 >7-2 >/-1
+ x” a + ... + x a +a
7 6 5 .4 .3 .2
x x X X X X
=> I = + + + x - log | x + 1| + C
7 6 5 4 3 2

ReadYourFlow.COM
19.30 MATHEMATICS-XII
6
(iii) Let/ = JJ -X-l dx
6_16+16
=> I
= 1- x-l dx
6 ^6
1
=> / =
J-x-l x-l
dx

xn-an n-1 . n-2 , n- 3 2


I = J • X5 + X4 + X3 + X2 + X + 1 + 1 =X +x a+x a
=> • dx ■ x-a
x-l
4- ... + X an-2+an~l
5
X
4 3 .2
/ =^ + ^- +----- h
6 5 4 y + y + ^ + logl^-ll + C

low
EXERCISE 19.4
LEVEL-1
x2 + 5x + 2 x3
M 2-l

ee
dx dx
x+2 x-2
rF
Fr
2
x + x+5 2x+ 3
3-J 3x + 2
dx
(x-l)2
for dx

x2 + 3x -1
ou
2x -1
5. | rfx 6.J dx
ks

(x +1)2 (^-D2
oo

ANSWERS
Y
B

2 3
re

1. — + 3x-41og|x +2|+C 2. — 4- X2 4- 4x 4- 8 log | X - 2 | 4- C


3
ou
ad

x2 1 43
3. — + - x + — log |3x + 21 + C 4. 2 log | x -11---- — + C
Y

6 9 27 61 1 x-l
3
5. x + log | x + 1| +------ + C
nd

6.---- + 2 log|x -1| 4-C


Re

X 4- 1
Fi

19.8.3 EVALUATION OF INTEGRALS OF THE FORM j (<?x + b) jcxTd dx AND J


ax + b
dx
■Jcx+d
In order to evaluate this type of integrals, we may use the following algorithm:
ALGORITHM
STEF I Write (ax + b) in terms of(cx 4- d) asfolloius:
(ax + b) = X(cx + d) + n
STEP II Find X and p by equating coefficients of like powers of x on both sides.
STEP HI Replace ax + b by X(cx + d) + \x in the given integral to obtain
J (ax + b) ^jcx 4- d rfx = J j X (cx 4- rf) 4- g J cx + d dx
= (cx + d)3/2 dx + \x J ^Jcx + d dx
= — (cx + d)5/2 4-^i (CX + d)3/1 + C
5c 3c

ReadYourFlow.COM
INDEFINITE INTEGRALS 19.31

ax+ b A, (cx + cf) + n


dx = J dx
tJcx
+d ^jcx + d
X J ^jcx + d dx + \x J 1
dx
,Jcx + d
2X 3/2 , 2^ 1/2
= — {cx + d) + (cx + d) +C
3c c
Following examples will illustrate the above procedure:
ILLUSTRATIVE EXAMPLES

LEVEL-1

EXAMPLE 1 Evaluate:
(i) J x tJx + 2 dx (ii) J (7 x - 2) px + 2 dx

w
SOLUTION (i) Let / = J x ^jx + 2 dx. Then,
I = | {{x + 2)-2) Jx + 2 dx [•.■ x = (x + 2) - 2]

=> I = } K* + 2) 3/2 -2{x + 2)


Flo
1/2 ) dx = | (x + 2)5/2-~ (x + 2) 3/2 + C

ree
(ii) Let / = J (7x - 2) yj3x + 2 dx

F
Let 7x - 2 = A. (3x + 2) + p
or
ur
On equating the coefficients of like powers of x on both sides, we get
sf
3A. = 7 and -2 = 2X + ^=>X = ^- and p = - —
J o
k
Yo

I = J (A,(3x + 2) + p) ^3x + 2 dx
oo

3/2 + p (3x + 2)1/2) dx


B

=> I = | {A.(3x + 2)
re

5/2 3/2
, (3x + 2) (3X+ 2) 14 5/2_40 (3x + 2) 3/2
ou

=> I = ^ + C= — (3x + 2) +C
ad

5 " r +F i 27
x 3
3x3 45
Y

2 2
ALITER Let/ = | (7x-2) ^3x + 2 rfx. Then,
nd
Re
Fi

I =1 x + 2 dx

'!!(
=> / 3x - ^ ^3x + 2 rfx

=>

=> ^
n( 3x + 2-2-^j ^3x + 2 dx

| J j(3*+2)-^j &
=> ' = | j {(3^ + 2) 3/2 f /sTTlj &
5/2 3/2
7 (3x + 2) 20 (3x + 2) — (3x + 2)5/2-— (3x + 2)3/2 + C
=> I = — <-------- = -------- X +C
3 3x 5 7 3 45 27
x3
2 2

ReadYourFlow.COM
19.32 MATHEMATICS-XII

x
EXAMPLE 2 Evaluate: J dx.
yjx + a + -\jx + ^
x
SOLUTION Let I
=J tJx + a + tJx + b
dx . Then,

xljxTa -J^b\
dx = j —^----------------------- >-dx
a + V^} {v^ -y[^] a-b

=> I = J jx ^x + fl-x^x + ^j dx

=> / = ^ J | (x + a - a) yjx + a -(x + b - b) ^x + fcj rfx

3/2 3/2 + b yjx + b^ dx


=> / =
^ J {(x+* a^Jx + a-(x + b)

w
=> / =
i
f (*+a) ^^{x + afn_2{x + bf/2 + 2i{x + b) 3/2
+c
a -b 3 5

Flo
3

ree
EXAMPLE 3 Evaluate:
8x +13 J X+1 X
(i)J , dx (ii) J dx (iii) J dx

F
V*+"2
8x +13
or
ur
SOLUTION Let/ = | dx.
sf

Let 8x +13 = X(4x + 7) + \x


k
Yo
oo

On equating the coefficients of like powers of x on both sides, we get


8 = 4X, 13 = 7X + [i => X = 2 and p = -1
B

Replacing 8x + 13 by A. (4x + 7) + p in the given integral, we get


re

A(4x + 7) + p
J = dx = J < X J4x + 7
ou
ad
Y

1
=> I = X J ^/4x + 7 dx + p | dx
^T7
nd
Re
Fi

3/2 1/2
(4x + 7) (4x + 7) | (4^ + 7) 3/2 1 1/2
=> I = X ■
3 r +h i 1
+c = - (4x + 7) +C
4x 4x -
2 2
8x + 13
ALITER Let / =
1 dx. Then,

13
X 4-
I = 8 j — dx
^7

4x 4-
13 (4*+7)+^-7j (4X + 7) - -
=> / -1
4 J V4^ +T7 *=2I dx =
2I #447
2 dx

1
=> / =
21 dx

ReadYourFlow.COM
INDEFINITE INTEGRALS 19.33

3/2 1/2
„ (4*+ 7)' 1 (4* + 7) 1 3/2 1/2
=> / = +C= (4*+ 7) “ (4a:+ 7) +C
1 .3
4x 2 4x 1
2 2
x+1
(ii) Let / = J dx. Then,

2x + 2 (2x-l)+ 3 3
/=!2 jJ rfx = — [
2 J ^ “ j ' V2:r_1 + V^T rfx

1
=> I 1 j *+| dx

3/2 1/2
(2X-1) 3 (2x-l) 1
=> > +— < + c = 7 (2x-l) 3/2 + | <2*-U 1/2 + c

w
2 3 2 1 D
2x 2x
2 2
x

Flo
(iii) Let / = J dx. Then,

ee
(x + 2)-2
=J
Fr
1 dx = • t?x
for
ur
3/2 1/2
(x + 2) > _ 2 \ (x + 2) 3/2 - 4 (x + 2)1/2 + C
=> / = + C = | (-2)
3 1
ks
Yo

2 2
oo

EXERCISE 19.5
B

LEVEL-1
re

x +1
M rfx 2. J x ^x + 2 dx
ou
ad

72IT3
Y

x —1 4. | (x + 2) ^3x+5 dx
3. dx
^/* + 4
nd
Re

2x +1 3x + 5
I 6.
J J7T9 tfx
Fi

5. dx
/3^+2
X 2 - 3x
7.
J dx 8.
I Tmr rfx
9. J (5x + 3) ^2x -1 dx
LEVEL-2
x
10.
j rfx
Vx+fl-jx+b
ANSWERS

1. 1(2*+3) 3/2 1 (2x + 3) 1/2 + C 2. |(I + 2)5/2-i(A: + 2) 3/2 + C

3. |(* + 4) 3/2 -10 (x + 4)1/2 + C 4. (9x + 20)(3a: + 5)3/2 +C

5. ^-(6* + l)/3?+2+C 6. n/7^T9 (7a: +17) + C

ReadYourFlow.COM
19.34 MATHEMATICS-XII

7. - (x - 8) ^x + 4 + C 8. | (8 - 3x) + 3x + C
3
9.1 (3x + 4) (2x -1) 3/2 + C
1 2 5/2 2fl .
10. -(x + a)' -—(x + a) 3/2 +, ^(x
2
+ b)
5/2
T(,+fc)
3/2
+C
a-b 5 3
19.8.4 EVALUATION OF INTEGRALS OF THE FORM J sin'" a dx, j cos"' a dx, WHERE m<,4,meN

To evaluate integrals of the form j sin?" x dx, J cos" x dx where m < 4, we express sin”' x and
cos" x in terms of sines and cosines of multiples of x by using the following trigonometrical
identities:
(i) sin2 x = 1 - cos 2x (ii) cos2 x = 1 + cos 2x

w
2 2
3 O
(hi) sin 3x = 3 sin x - 4 sin x (v) cos 3x = 4 cos x - 3 cos x.
Following examples will illustrate the procedure.

Flo
ee
ILLUSTRATIVE EXAMPLES

Fr
LEVEL-1
for
ur
EXAMPLE l Evaluate:
(i) j sin 2 x dx (ii) J cos2 x dx [NCERT]
ks
Yo

(hi) | sin2 x cos2 x dx


oo
B

SOLUTION (i) Let I = J sin2 x dx . Then,


re

1 - cos 2x 1 J (1 - cos 2x) dx = 1 j x sin 2x


1 =1 dx = - +C
ou
ad

2 2
Y

(ii) Let f = J cos2 x dx. Then,


d

1 + cos 2x sin 2x
Re

dx = 1 J (1 + cos 2x) dx = 1
I =J x+ +C
n

2 2
Fi

(iii) Let I = fI sin 2 x cos2 x dx. Then,


If 2
I = — J (2 sin x cos x) dx

1 - cos 4x 1 J (1 - cos 4x) dx = 1 j x sin 4x


=> I = 1 [ sin2 2x dx = 1 f dx = — +C
4J 4J 2 8 4
EXAMPLE 2 Evaluate:
(i) J sin 3 x dx [NCERT] (ii) j cos3 x dx

(iii) J sin3 x cos3 x dx [NCERT]

SOLUTION (i) Let 7 = J sin3 x dx. Then,


3 sin x - sin 3x dx = 1 J (3 sin x cos 3x
W 4
- sin 3x) dx = — - 3 cos x +
4 3
+C

ReadYourFlow.COM
INDEFINITE INTEGRALS 19.35

(ii) Let J = j" cos3 x dx Then,


cos 3x + 3 cos x If „ ^ , 1 f sin ^
I = I 4
dx = —
4J
cos 3x + 3 cos x ax = — <---
4 { 3
+ 3 sin x > + C

(ni) Let / = f sin3 x cos3 x dx. Then,

/ = — f (2 sin x cos x)3 dx = — [ sin 3 2x dx = f 3 sin 2x - sin 6x dx


8J 8J 8J 4
1 1 f 33 1
=> I — f (3 sin 2x - sin 6x) dx = — ■) ~ — cos 2x + — cos 6x > + C
32 J V 32 1 2 6
EXAMPLE 3 Evaluate:
(i) J sin4 x dx [NCERT, CBSE 2000,2004] (ii) J cos4 x dx [CBSE 2000,2003]

(iii) | sin4 x cos4 x dx

low
SOLUTION (i) Let/ = J sin4 x dx. Then,
n2
1 - cos 2x j
dx v sin 2 0 = - - cos 20

ee
2 2
rF
Fr
=> - [ 1 - 2 cos 2x + cos2 2x dx
4J
1 + cos 4x
=> / !- 2 cos 2x + dx
for
4J 2
ou
=> / = — f 2 - 4 cos 2 x +1 + cos 4x dx
ks

8J
- f 3 - 4 cos 2x + cos 4x dx = - i 3x - 2 sin 2x + sin 4x
oo

=> I = - +C
Y

8J 8l 4
B

(ii) Let / = | cos4 x dx. Then,


re

\2
1 + cos 2 x | cos2 0 = 1 + cos 20
ou
ad

/ = dx
2 2
Y

=> I = — [ 1 + 2 cos 2x + cos2 2x dx


d

4J
Re
n

=> i = lf 1 + 2 cos 2x + 1 + cos 4x


dx
Fi

4J 2
- f 3 + 4 cos 2x + cos 4x dx = - j 3x + 2 sin 2x + sin 4x + C
I = -
=>
8J 8l 4
(iii) Let I = j* sin4 x cos4 x dx. Then,
/ = — [ (2 sin x cos x)4 dx = — f (sin 2x)4 dx = ^~ f (sin2 2x)2 dx
16 J 16 J 16 J
1 - cos 4x ^l2
=> I = — dx
2
I = — J (1-2 cos 4x + cos2 4x) dx

=>
1=1 1-2 cos 4x +
1 + cos 8x
dx
64 2
1 f f 3 - 4 cos 4x + cos 8x1 dx = — 1
=> / = 3x - sin 4x + — sin 8x ^ + C
128 H J 128 8

ReadYourFlow.COM
19.38 MATHEMATICS-XII

sin 4x
SOLUTION Let I
=1 sin x
dx. Then,

2 sin 2x cos 2x 4 sin x cos x cos 2x


/
=1 sin x
dx
=J sin x
dx

2 | (cos 3 x + cos x) dx = 2 j sin 3x


=> I = 2 J 2 cos 2 x cos x dx + sin x ^ + C
3
LEVEL-2
cos 5x + cos 4x
EXAMPLE 4 Evaluate: J dx [NCERT EXEMPLAR]
1-2 cos 3x
SOLUTION Let
cos 5x + cos 4x
dx. Then,
1-2 cos 3x

w
sin 3x (cos 5x + cos 4x)
=> I =
J sin 3x - 2 sin 3x cos 3x
dx

Flo
3x 3x^ 9x x
2 sin cos — 2 cos — cos
2 2 2 2

ee
=> '= ! dx
sin 3x - sin 6x

Fr
3x 3x 9x x
sin cos cos cos
2 2
J 2 2 dx
=> I = 4
for
ur
9x 3x
- 2 cos sin
2 2
ks

cos ^ dx = - J (cos 2x + cos x) dx = - sin 2x + sin x + C


Yo

=> 1 = - J 2 cos
oo

2
B

EXERCISE 19.7
re

LEVEL-1
ou
ad

Integrate the following integrals:


Y

1. J sin 4x cos 7 x dx [CBSE 2007] 2. J cos 3x cos 4x dx


3. J cos nix cos nx dx, m * n 4. J sin mx cos nx dx, m*n
nd
Re

5. J sin 2x sin 4x sin 6x dx 6. J sin x cos 2x sin 3x dx


Fi

ANSWERS
1 1 1
1. ------cos 11 X + -1 cos 3x + C 2. — sin 7x + — sin x + C
22 6 14 2
1 sin (m + n) x sin (w -n) x 1 cos (m + n) x cos (m-ri) x
—----------— +C 4. +C
2 m+n m -n 2 m+n

5. —cos 8x + — cos!2x —— cos 4x + C 6. —+ —sin4x—^-sin6x-—sin2x + C


32 48 16 4 16 24 8

19.8.6 INTEGRALS OF THE FORM {


f(x)
/' (x)
THEOREM 1
fix) J
dx = log (/(x)) + C

f'(x)
PROOF Let/ = / dx.
fix)

ReadYourFlow.COM
INDEFINITE INTEGRALS 19.39

Putting / (x) = t and /' (x) dx = dt, we get


1 = J ^dt = \ogt + C= log |/ (x) | + C
Q.E.D.
REMARK Itfollows from the above theorem that if the numerator in integrand is exact differential of the
denominator then its integral is logarithm of the denominator.
SOME STANDARD RESULTS
THEOREM 2 Prove that: J tan x dx = - log | cos x | + C or, J tan x dx = log | sec x | + C
sin x
PROOF Let / = J tan x dx. Then, J = J dx
COS X

dt
Let cos x = t. Then, d (cos x) = => - sin x dx = dt => dx = -
sin x

low
Putting cos x = f, and dx = - dt/sin x, we get
sin x ^ = - J ^ dt = - log \t \ +C = - log | cos x | + C
X
cos x sin x
Hence, | tan x dx=- log | cos x | + C or, J tan x dx = log | sec x | + C

ee
Q.E.D.
rF
Fr
THEOREM 3 Prove that: J cot x dx = log | sin x | + C.

COS X
PROOF Let 7=| cot x dx. Then, J = j" dx.
for
sin x
ou
dt
ks

Let sin x=t. Then, d (sin x) = dt => cos x dx = dt=^> dx =


cos x
oo

Putting sin x = f, and dx = dt/cos x, we get


Y
B

cos x dt = J y df = log | f | + C = log | sin x | + C


=1
re

7
t COS X
ou
ad

Hence, J cot x dx = log|sinx|+C. Q.E.D.


Y

THEOREM 4 Prove that: J sec x dx = log | sec x + tan x | + C.


nd
Re

sec x (sec x + tan x) ^


PROOF Let 7 = J sec x dx. Then, 7 = J
Fi

(sec x + tan x)
Let sec x + tan x = t. Then,
2 dt
d (sec x + tan x) = dt => (sec x tan x + sec x) dx = dt => dx =
sec x (sec x + tan x)
dt
Putting sec x + tan x = t and dx = , we get
sec x (sec x + tan x)
sec x (sec x + tan x) ^ dt = J ^ di = log | f | + C = log | sec x + tan x | + C
t sec x (sec x + tan x)
Hence, J sec x dx = log | sec x + tan x | + C.
Q.E.D.
THEOREM 5 Prove that: J cosec x dx = log | cosec x - cot x | + C.
cosec x (cosec x - cot x) ^
PROOF Let 7 = j cosec x dx. Then, 7 = J
cosec x - cot x

ReadYourFlow.COM
19.40 MATHEMATICS-XII

Let cosec x - cot x - t. Then,


2 dt
d (cosec x - cot x) =dt => (- cosec x cot x + cosec“ x) dx = dt => dx =
cosec x (cosec x - cot x)
dt
Putting cosec x - cot .t = t and, dx = , we get
cosec x (cosec x - cot x)
cosec x (cosec x - cot x) dt
I
=1 cosec x - cot x (cosec x - cot x) cosec x

=> I = j ~ dt = log | f | + C = log | cosec x - cot x | + C

Hence, J cosec x dx = log | cosec x - cot x | + C Q.E.D.


ALTERATIVE FORMULAE FOR J cosec x dx AND [ sec x dx
1 1
=1

w
Let / = J cosec x dx = j — dx dx. Then,
sin x 2 sin x/2 cos x/ 2
sec2 x/2

Flo
2X
dx Divide both numerator and denumerator by cos —
2 tan x/2

ee
X
^ = t. Then, d tan ^ j 1 2 x 2dt

Fr
Let tan — = dt => — sec — dx = dt => dx =
2 2 2x
sec
2 or
ur
X 2dt
sf
Putting tan— = t and dx - we get
2 x '
sec
2
k
Yo
oo

] J Ut = log | f | + C =log tan ^ + C


B
re

Hence, J cosec x dx = log tan ^ + ^ •~(i)


ou
ad

\ / \
Y

f sec x dx = [ cosec — + x dx = log tan — + - +C [Using (i)]


J J UJ U 2)
nd
Re

Hence, | sec x dx = log tan f ^ ^ +C


Fi

ILLUSTRATIVE EXAMPLES
LEVEL-1
Type I PROBLEMS BASED ON | tanxdx, J cot x rf.v, J sec x da:, [ cosec x d.v
EXAMPLE l Evaluate:
1 1
a)/ + cos 2x
dx (ii) j
- COS X
dx

(m) ^ J 1 - cos 2x
1 + cos 2x

SOLUTION (i) We have.


(iv) f J
1 + COS X
1 - cos x
dx

1 1
dx = J dx = ~ f sec x dx -1= log | sec x + tan x| + C
+ cos 2x -^2 cos2 x V2 J V2

ReadYourFlow.COM
INDEFINITE INTEGRALS 19.41

(U) J 1
- COS X
=J 1
2 x
dx =
V2
cosec — dx = ^2 log cosec — - cot — + C
2
x
2 2
2 sin
2

1 - cos 2x 2 sin2 x
(Ui) 1J 1 + cos 2x
dx
=1 y 2 cos2 x
dx = J tan x dx - log | sec x | + C

2X
2 cos
1 + cos X r X X
(iv) J 1 - cos X
dx =J 2
2x
dx = J cot — dx = 2 log sin ^ + C

EXAMPLE 2 Evaluate:
1 2 sin
2

1 1
WJ
^1 + sin 2x
dx (U) |
- sin x
dx

low
SOLUTION (i) Let/ = J 1
dx. Then,
+ sin 2x
1
J = I dx

ee
1 - cos - + 2x
rF
Fr
2

1 dx - -i= [ cosec [ — + x I dx = 1 i [ TC X | „
1
/ =
for t= log tan — + — I + C
2 sin
2 f TC V2 J U J V2
^ 6 8 2
ou
+x
4
ks

1
(ii) Let / = J dx. Then,
oo

^ - sin x
Y
B

1 1 Tt X ,
/
=1 dx = J ^ =77? I sec — + — dx
re

71 2 ttx 4 2J
1 + cos - + X 2 cos 4+2
ou

2
ad
Y

Stt x
=> 7 = ^log tan (^+i+ij+c = ^21°8 tan T +4
+C
nd
Re

EXAMPLE 3 Evaluate:
Fi

sin x
(i)j SiniX-a)dx (ii) {
sin (x - a)
dx [NCERT, CBSE 2004]
sin x
1
P) J sin (x - a) sin (x - b)
dx

sin (x - a)
SOLUTION (i) Let / =| dx . Then,
sin x
sin x cos a - cos x sin a
I = 1 sin x
dx

:=> / = J cos a dx - J sin a cot x dx


=> 1 = cos fl J 1 • dx - sin a J cot x dx = x cos - sin a log | sin x | + C
sin x
(ii) Let/ = J dx. Then,
sin (x - a)
sin {(x -a) + a} ,
I = l -------------------dx
sin (x - a)

ReadYourFlow.COM
19.42 MATHEMATICS-XII

sin (x - a) cos a + cos (x - a) siix a ^


=> ^=J sin (x - a)
I = J (cos a + cot (x - a) sin d\ dx
=> I = cos fl | 1 • dx + sin a J cot (x - a) dx = x cos a + sin a log | sin (x - a) | + C
1
(iii) Let / = J dx. Then,
sin (x - a) sin (x-b)
1 sin {(x-fr)-(x-fl)} dx
I =
sin {a -b) J sin (x-fl) sin {x-b)
1 sin {x-b) cos {x-a) -cos{x -b) sin {x-a)
=> I = dx
sin {a -b) sin (x - a) sin (x -b)
1 | {cot {x - a) - cot (x -b)} dx
I =
sin {a -b)

low
1 | log | sin (x - fl) | - log | sin (x - b) | j + C
=> I =
sin {a -b)
sin (x - a)
=> I = cosec {a - b) log +C
sin {x-b)

ee
rF
Fr
EXAMPLE 4 Evaluate:
1 1
(i)J
sin (x - a) cos (x -b)
dx (ii) | for
cos (x - a) cos (x -b)
dx [NCERT]

1
ou
SOLUTION (i) Let/ = J dx . Then,
ks

sin (x - a) cos (x - b)
1 cos {a - b)
oo

I = dx
Y

cos {a-b) i sin (x - a) cos (x - b)


eB

1 cos {{x-b) -{x-a)} dx


=> I =
cos {a-b) I sin {x-a) cos (x -b)
r
ou
ad

1 cos (x - a) cos {x -b) + sin (x - a) sin (x - b)


=> I = dx
Y

cos {a-b) sin {x-a) cos (x -b)


1 J j cot (x - rt) + tan (x-b)jdx
d

=> I =
Re

cos {a - b)
n
Fi

1 | logt, | sin (x - a) | - loge | cos {x-b)\ j + C


=> I =
cos {a - b)
1 sin (x - a)
=> / = log +C
cos {a -b) 1 cos {x-b)
(ii) Let / = J 1
dx . Then,
cos(x-fl) cos (x-b)
1 sin {a - b)
I =
sin {a -b) I cos (x - a) cos (x -b)
dx

1 sin {(x-b) -(x-fl)}


=> / = 1
sin {a-b) J cos (x - a) cos (x - b)
1 sin (x - b) cos {x-a) - cos (x - b) sin (x - a) ^
=> / =
sin {a -b) cos (x - a) cos (x -b)
1 | / tan {x-b) - tan (x - fl) dx
=> I =
sin {a -b)

ReadYourFlow.COM
INDEFINITE INTEGRALS 19.43

1
=> 1 = {- logp | cos (x - b)\ + loge | cos (x - a)\) + C
sin (a -b)
1 cos (x - a)
=> I = log +C
sin (a -b) 1 cos(x-b)
sin (x + a)
EXAMPLES Evaluate: J dx
sin (x + b)
sin (x + a)
SOLUTION Let/ = J dx. Then,
sin (x + b)
sin (x + b + a - b)
I =
sin (x + b)
sin {(x + b) + (a -b)} ,
=> I = ---------------------------- dx
sin (x + b)

low
sin (x + b) cos (a -b) + cos (x + b) sin (a -b)
=> I =
1 sin (x + b)
=> I = J {cos (a -b) + cot (x + b) sin {a -b)} dx

ee
=> 1 = cos (a - fr) J 1 • dx + sin (a - b) J cot (x + b) dx
rF
Fr
=> I =x cos (a -b) + sin (a -b) log |sin (x + b) | + C
Type II EVALUATION OF INTEGRALS BASED UPON f fix) dx = log |/(x) | +C for
fix)
ou
In order to evaluate this type of integrals, we may use the following algorithm:
ks

ALGORITHM
oo

STEP I Obtain the integral, let it be! = j LW *


Y
B

/(*)
1
re

STEP II Put f (x) = t and replace f' (x) dx by dt to obtain J = j dt


ou
ad

STEP III Evaluate integral obtained in step II to obtain I = log |f| + C


Y

STEP IV Replace tbyf(x) in step III to get I = log \f (x) | + C


Following examples will illustrate the above procedure.
nd
Re

EXAMPLE 6 Evaluate:
Fi

2x + 5 , 1 - tan x
(i)| -Z------------ dx (ii) | dx
xz + 5x -7 1 + tan x
2 x -x
sec x -e
(iii) J
3 + tan x
dx (iv) J ^ -x
dx [NCERT]
e +e
(v) J e31°s*(x4 Tl)-1 dx [NCERT]
2x + 5
SOLUTION (i) Let / = J dx
x2 +5x-7

Let x2 + 5x - 7 = t. Then, d (x2 + 5x - 7) = dt=> (2x + 5) dx = dt => dx = dt


2x + 5
2 dt
Putting x + 5x -7 =t and dx = we get
2x + 5 '
2x + 5
/ =
I x2 + 5x -7
dx = J j dt = log | f | + C = log | x2 + 5x - 7 | + C

ReadYourFlow.COM
19.44 MATHEMATICS-XII

(ii) We have.
I = jJ -1 +- tan x
tan x
dx - J cos x - sin x
cos x + sin x
dx

Let cos x + sin x = t. Then,


dt
d (cos x + sin x) - dt => (- sin x + cos x) dx = dt => dx =
cos x - sin x
dt
Putting cos x + sin x = t and dx = we get
cos x - sin x 7
1 - tan x
I = I 1 + tan x
dx = J y df = log | f | + C = log | cos x + sin x | + C

2
sec
(iii) Let 7= J — dx
3 + tan x

w
Let 3 + tan x = t.Then, d (3 + tan x) = dt=> sec2 x dx = dt => dx = dt
2
sec x
dt
Putting tan x = t and dx = —^— , we get
Flo
ee
sec x

Fr
2
sec x dt
i = i 3 +1
X
2
sec x
| —— df = log | 3 + f | + C = log | 3 + tan x | + C
3 f
for
ur
x -e - X
(iv) Let/ = J ~ dx.
ks

-X
e +e
Yo
oo

dt
Letex + e x = /.Then, d(ex + e x) = dt=>(ex -e v) dx = dt=>dx =
e -e - x
x
B
re

dt
Putting ex + e x = t and dx = X
—, we get
ou
ad

e -e
x -x
Y

-e dx = J ^ = log111 + C = \og\ex + e 1 | + C
I =
e + e- x f
nd
Re

(v) We have.
Fi

elos*3 3
/ = f e31o^(x4-fir1dx = f ^
J x4 + 1
f—
dx = J x4 +1 dx

Let x4 + 1 = t. Then, d (x4 +1) = dt => Ax^ dx = dt => dx =


4x3
Putting x4 +1=7 and dx = ^ we get
4x3 '
/ = I f I dt = i log|f| + C = 7 log (x4 +1) + C
4J t 4 4
EXAMPLE 7 Evaluate:
sin 2x
(i) j dx (ii) | -j dx [CBSE 2005]
l+e x a sin x + b 2 cos2 x
2
tan x sinx-xcosx
(iii) | 2 dx (iv) J x (x + sin x)
dx [CBSE 2015]
a + b tan x

ReadYourFlow.COM
INDEFINITE INTEGRALS 19.45

1 - dx = f —- x
SOLUTION (i) Let / = J
1+e x J
1+ X
1
dx =
J ex +1
dx f-
e
Let ex +1 = t. Then, d (ex +1) =dt=> ex dx =dt=>dx = ~
e
x dt
Putting \ + e =t and dx = — , we get
e
X
ex dt
I = S -r dx = \ J y dt = log | f | + C = log 11 + e* | + C
e' +1 T7
sin 2x
(ii) Let / = J dx
a sin x + b 2 cos 2 x
2 2

w
2 2 2 2
Let a sin x + b cos x = t. Then,
2 2 2 2

Flo
d(a sin x + b cos x) = dt => (a -b ) sin 2x dx = dt => dx =
2 2 dt
(a2 -b2) sin 2x

ee
Fr
2 2 2 2 dt
Putting a sin x + b cos x = t and dx = , we get
(a2 -b2) sin 2x for
ur
sin 2x dt
/
1 t
X
2 sin x cos x(a 2 -b 2 )
ks
Yo

1 1 1 2 2 2
oo

=> I = dt = 2 “ 2~ loS U I + C = log \a sin x + b cos 2 x\+C


, 2 ,2, t (a2-b2) (a2-b2)
(« -b )
B

tan x
re

(iii) Let / =
J a + b tan x . Then, 2
ou
ad

sin xcos x , r sin x cos x sin 2x


/
= J sin-2-dx dx = — [
Y

= J 2 2 dx
x fl cos x +1 sin x 2 J a cos9 x + /? sin2 x
a+b
nd

2
Re

COS X
Fi

1 2 2
=> 7 = log | a cos“ x + & sin x | + C [See (ii)]
2(b-a)
sinx-xcosx
(iv) Let 7 = J dx. Then,
x (x + sinx)
(x + sin x) -x-xcosx
'=J x (x + sin x) dx
(x + sin x) - x (1 + cos x) ,
'=1 -----------------------------dx
x (x + sin x)
j _r f1 1 + cosx'|
=> dx
■’ [ x x + sin x
=> 7 = log x|-log(x + sinx)+C
x
=> 7= log +C
x + sin x

ReadYourFlow.COM
19.46 MATHEMATICS-XII

LEVEL-2
EXAMPLES Evaluate:
(i) J tan x tan 2x tan 3x dx (ii) | tan (x - 0) tan (x + 0) tan 2x dx
SOLUTION (i) We know that
tan 2x + tan x
tan 3 x = tan (2x + x) =
1 - tan 2x tan x
=> tan 3x (1 - tan 2x tan x) = tan 2x + tan x
=> tan 3x - tan 3x tan 2x tan x = tan 2x + tan x
=> tan 3x tan 2x tan x = tan 3x - tan 2x - tan x
'=1 tan x tan 2x tan 3x dx

=> 7=
J (tan 3x - tan 2x - tan x) dx = --~ loge | cos 3xj + ^ logt. | cos 2x| + loge | cos xj + C

low
(ii) We know that
2x = (x - 0) + (x + 0)
=> tan 2x = tan {(x - 0) + (x + 0))
tan (x - 0) + (x + 0)
tan 2x =

ee
1 - tan (x - 0) tan (x + 0)
rF
Fr
=> tan 2x - tan (x - 0) tan (x + 0) tan 2x = tan (x - 0) + tan (x + 0)
tan (x - 0) tan (x + 0) tan 2x = tan 2x - tan (x - 0) - tan (x + 0)
for
7 = J tan (x 0) tan (x + 0) tan 2x dx = J {tan 2x - tan (x - 0) - tan (x + 0)} dx
ou
ks

7 = - - log | cos 2x| + log | cos (x - 0) | + log | cos (x + 0) | + C


oo

1/2
Evaluate: J jl + 2 tan x (tan x + sec x)
Y

EXAMPLE 9 dx
eB

1/2
r

SOLUTION Let 7 = 1 + 2 tan x (tan x + sec x) dx. Then,


ou
ad

1/2
Y

1 + 2 tan 2 x + 2 tan x sec x


I
= J{ dx
d
Re

1/2
M 1 + tan 2 x + tan 2 x + 2 tan x sec x
n

=> I = dx
Fi

1/2
sec2 x + tan 2 x + 2 tan x sec x
=> I
-({ 1/2
dx

=> I = | |(sec x + tan x)2| dx

=> 7 = J (sec x + tan x) dx = log | sec x + tan x | + log | sec x| + C


sin 2x
EXAMPLE 10 Evaluate: J dx
n . k
sin x----sin x +
3 3
sin 2x
SOLUTION Let 7 = J dx. Then,
7C . It
sin x — sin x + —
3y 3

ReadYourFlow.COM
INDEFINITE INTEGRALS 19.47

n n
sin x---- + x + —
3y 3
I = dx
sin x -- sin x + ^
3 3y
it n in. n
sin x---- cos x + — + cos x---- sin x + —
3 3 3y 3
=> I =
I n 71
sin x — sin x + —
3 3;
/ \ / \ / \
=> / = I i cot x + — + cot x — rfx = loe sin x + — + log sin x - — +C
] \ { 3) {3) 6 3) 3

low
EXERCISE 19.8
LEVEL-1
Evaluate the following integrals:
1 1
dx 2.
1 dx

ee
^jl - cos 2x + cos X
rF
Fr
1 + cos 2x 1 - cos X
dx 4. dx
1 - cos 2x 1 + cos x
for
sec x cos 2x
u
5.J dx 6.
1 2 dx [NCERT]
ks

sec 2 x (cos x + sin x)


Yo
oo

sin (x -a) , sin (x - a)


7-l -------------- dx 8. dx [CBSE 2006,2013,2015]
B

sin (x -b) sin (x + a)


re

1 + tan x cos x
9-J 1 - tan x dx 10. |
cos (x - a)
dx
ou
ad

1 - sin 2x 3x
Y

11.
1 + sin 2x
dx 12.
14e +1
rfx
nd
Re

sec x tan x 1 - cot x


13.
I dx 14.
I 1 + cot X dx
Fi

3 sec x + 5
sec x cosec x 1
15.} dx 16. } dx
log (tan x) x (3 + log x)

17-I ^e<4+1
+x
dx 18. f —I—
J X log X
sin 2x cos X
19.}
a cos2 x + fr sin2 x
dx 20.
I 2 + 3 sin x dx
1 - sin x a
2!.} dx 22.
1 b + cex dx
x + cos x
cot x
24. j
23-J [CBSE 2003]
log sin x
dx

21-
2 cos x - 3 sin x
25J e -2
26.
1 6 cos x + 4 sin x
dx [NCERT]

ReadYourFlow.COM
19.48 MATHEMATICS-XII

cos 2* + x + 1 1
27. dx 28. dx [NCERT]
2 cos (x + a) cos (x + b)
x + sin 2x + 2x
- sin x + 2 cos x cos 4x - cos 2x
29.
2 sin x + cos x
dx 30.
I sin 4x - sin 2x
dx

sec x cosec x
31. J
log (sec x + tan x)
dx 32.
1 log tan ^
dx

2
1 cosec x
33.
1 X log X log (log x)
dx 34.
1 + cot x
dx

10 x9 + 10x loge 10 dx 1 - sin 2x


35.
10* + x10
36.
1 X + COS
2
X
dx [CBSE 2000]

1 + tan x sin 2x
J

w
37.
1 x + log sec x
dx [CBSE 2000] 38.
2,2.2
a + b sm x
dx

x +1 1
I
1
Flo
39. dx 40. dx
x (x + log x) ^/l - x2 (2+3 sin 1 x)

ee
2 2
sec x 2 cos 2x + sec x

Fr
41.
1 tan x + 2
dx 42. |
sin 2x + tan x - 5
dx

sin 2x 1 + cot x
or
ur
43. j dx 44. \ dx [CBSE 2000]
sin 5x sin 3x x + log sin x
f
ks

LEVEL-2
Yo
oo

1
45.
1 Vx +1)
dx 46. | tan 2x tan 3x tan 5x dx
B

sin 2x
re

47. | (1 + tan x tan (x + 0)} dx 48. J 7T dx


n . n
sm x — sm x + -
ou
ad

6y
Y

x— 1 e-1 1
+X
50. | dx
(5)1 e dx
sin x cos2 x
nd
Re

1
Fi

51. dx
cos 3x - cos x

ANSWERS

1. -Llog tan | +C 2. J2 log tan - + — +C


4 4

X
3. log | sin x | + C 4.-2 log cos ^ + ^

5. 2 sin x - log | sec x + tan x | + C 6. log | sin x + cos x | + C


7. x cos (b - a) + sin (b - a) log | sin (x - £>) | + C
8. x cos 2a - sin 2a log | sin (x + a) | + C 9. - log | cos x - sin x | + C
/ \
10. (x - a) cos a - sin a log | sec (x - a) | + C 11. log cos —-x + C
4

ReadYourFlow.COM
INDEFINITE INTEGRALS 19.49

12. — log | e3* + 11 + C 13. — log | 3 sec x + 51 + C


3 3
14. - log | cos x + sin x | + C 15. log (log tan x) + C
16. log | 3 + log x | + C 17. log| ex + x| +C
18. log | log x | + C 1 2 2
19. ------log | a cos x + b sin x | + C
b -a
20. — log | 2 + 3 sin x | + C 21. log | x + cos x | + C
3
22. ~ log\be~ x + c\+C 23. - log \l + e~x \ +C

24. log | log sin x | + C 25. ^ log | e2x - 21 + C

1 1 2
26. — log | 2 sin x + 3 cos x | + C 27. - log | x + sin 2x + 2x | + C

w
1 cos (x + b)
28. log +C 29. log | cos x + 2 sin x | + C
sin (a-b) cos (x + a)

30. — log | cos 3x | + C


Flo 31. log | log (sec x + tan x) | + C

ee
3

Fr
X
32. log log tan - +C 33. log {log (log x)} + C
for
ur
34. - log 11 + cot x | + C 35. log 110* + x10 | + C
2
ks

36. log | x + cos x | + C 37. log | x + log sec x | + C


Yo
oo

38. -z- log (a2 + b2 sin2 x) + C 39. log | x + log x | + C


B

b2
re

40. — log | 2 + 3 sin 1 x | + C 41. log | tan x + 21 + C


3
ou
ad

1 1
42. log | sin 2x + tan x - 51 + C 43. — log | sin 3x | — log | sin 5x | + C
Y

3 5
44. log | x + log sin x | + C 45. 21og|V*+l| + C
nd
Re

46. 1
log | sec 5x| - ^ log | sec 2x| - — log | sec 3x| + C
Fi

cos x 48. log sin2 x-i +C


47. cot 9 log +C
cos (x + 0) 4
x
49. - log | c* + x6, | + C 50. sec x + log tan ^ +
e
51. ^ jcosec x - log | sec x + tan x || + C

_HINTS TO NCERT& SELECTED PROBLEMS


rj 2 -
sec x cos 2x 2 cos x -1
5.J dx = j rfx = J dx
sec 2 x cos x cos x
J 2 cos x - sec x dx = 2 sin x - log tan ^ ^j + C

ReadYourFlow.COM
19.50 MATHEMATICS-XII

cos 2x cos 2x
6. Let 7 = J dx.Then, / = J dx
(cos x + sin x)2 1 + sin 2x

Let 1 + sin 2x = t. Then, d(l + sin 2x) = dt or, 2 cos 2x dx = dt

7=1 = ^ log | f | + C = ^ log 11 + sin 2x | + C


2 J t

cos 2x cos2 x - sin


■ 2
x cos x - sin x
AL1TER I
=J jdx = | rfx = J dx
(cos x + sin x) (cos x + sin x) cos x + sin x

I = log | cos x + sin x | + C

28. Let/ = | 1
dx. Then,
cos (x + a) cos (x + b)

w
1 sin {(x + b) - (x + a)} ^
7 =
sin (b - a) cos (x + a) cos (x + b)

7 =
1

Flo
sin (x + b) cos (x + a) - cos (x + b) sin (x + a) ^

ee
sin (b - a) cos (x + a) cos (x + b)

Fr
1 J | tan (x + b) - tan (x + a) j dx
=> 7 =
sin (b - a)
for
ur
1 j- log cos (x + b) + log cos (x + a) j +C
=> 7 =
ks

sin (b - a)
Yo
oo

1 cos (x + b)
=> 7 = log +C
eB

sin (a -b) cos (x + a)


. 2 2
1 sin x + cos x
'=J *=1 dx = j (sec x tan x + cosec x) dx
r

50.
ou

sin x cos2 x sin x cos2 x


ad
Y

x
= secx + log tan^ +C
nd
Re

. 2 2 • 2 , 2
Fi

51.
1
dx = J sin x + cos x dx = j sin x + cos
-dx
- 4 sin2 x cos x
cos 3x - cos x - 2 sin 2x sin x

= 4U sec x + cosec x cotx j rfx = -^ | log (secx + tan x) - cosec x j + C

19.8.7 INTEGRALS OF THE FORM f [f (x))n f '(x) dx

n+ 1
THEOREM J {f(x)}nf'(x)dx = ,n*-l

PROOF Let 7 = J {f{x)}n f' (x) dx.

Putting /(x) = t and /' (x) dx = dt, we get


M+l n+ 1
t
I = f {f(x))nf'{x)dx = jfdt + c = {fix)} + C,n*-l.
n+1 n+l Q.E.D.

ReadYourFlow.COM
INDEFINITE INTEGRALS 19.51

ILLUSTRATIVE EXAMPLES
LEVEL-1
EXAMPLE 1 Evaluate:
3x + l r 3
(i)J (3x2 + 2x + 1)
3dx (ii) I sin x cos x dx

(losx)\x
(iii) I tan x sec x dx (iv) j
X
3x +1
SOLUTION (i) Let / = J w dx. Let 3x2 + 2x + 1 = t. Then,
(3x2 + 2x +1)

d (3x2 + 2x + 1) = dt => (6x + 2) dx = dt => dx = dt


2 (3x +1)

w
Putting 3x2 + 2x + 1 = f and dx = ^ we get
6x + 2 '

Flo
3x +1 -2
dt
i
t3 X 2 (3x + 1) r3 dt = -1 t
+c

ree
2 -2

F
1 1
=> I = - 2+C = 2+C
41 4 (3x2 + 2x + l)
or
ur
f 3
sf
(ii) Let / = I sin x cos x dx.
k
Yo

dt
Let sin x = t. Then, d (sin x) = dt => cos x dx ~ dt => dx =
oo

cos x
B

dt
re

Putting sin x - t and dx = , we get


cos X
ou
ad

t4 . 4
J = J sin 3 x cos x dx = J t3 cos x x dt \ t3 dt = —+ sin
C = —+c
Y

COS X J 4 4
nd
Re

(iii) Let J = J tan 3 x sec2 x dx.


Fi

2 dt
Let tan x = t. Then, d (tan x) = dt=> sec x dx =dt=> dx =
2
sec x

Putting tan x = t and dx = dt sec2 x, we get

C 3 2 f 3 2 t4 . 4
tan x
/ = I tan x sec x dx = I f sec x x di2-=\t3dt
J =—
4 +C = +C
sec x 4

(log*)3
(iv) Let / = j" dx.
x

Let log x = f. Then, d (log x) = dt => - dx = dt => dx =x dt


x
Putting log x = t and dx = x dt, we get
f4 (log*)4
7 = I -xdt = Jt3rft = | —+
4 C = +C
4

ReadYourFlow.COM
19.52 MATHEMATICS-XII

EXAMPLE 2 Evaluate:
loe tan —
4{sin~lx)3 dx (ii)| . 81 2 sin x
(i)J dx (iii) | dx
sin x ^3 + 2 cos x

4 (sin-1 x)3
SOLUTION (i) Let I =j dx. Let sin 1 x = t. Then,

d (sin 1 x) = dt=> dx = dt=> dx = Jl-x2 dt

Putting sin 1 x = f and dx = Jl - x2 dt, we get

=1

low
/

(ii) Let / = J

ee
sin x
rF
Fr
x = t. Then, d ^log tan ^ j = 1 2 x 1a
Let log tan — dt => sec — x - dx = dt => dx = sin x dt
x 2 2
tan
for
2
x
ou
Putting log tan ^ z= * and dx = sin x dt, we get
ks

\2
oo

log tan ^
Y

f2
eB

t
/
=1 sin x
sin x dt = { tdt = —+ C
J 2 2
+C
r

sin x
(iii) Let J = J dx
ou
ad

^3 + 2 cos x
Y

dt
Let 3 + 2 cos x = t.Then, d(3 + 2 cos x) - dt 2 sin x dx = dt=> dx = -
2 sin x
d
Re
n

dt
Fi

Putting 3 + 2 cos x = t and dx = - we get


2 sin x 7
sin x dt 1 *1/2 f_________
-7=- X - =--[ r 1/2 dt = —^ x +^ + ^ = - ^3 + 2 cos x + C
V* 2 sin x 2J

EXAMPLE 3 Evaluate:
(1 + log x)2 sec2 (2 tan 1 x) ^
(i)J X
dx [NCERT, CBSE 2009] (ii) J
1 + x2
(1 + logx)^^
SOLUTION (i) Let J X

Let 1 + log x =t. Then, d(l + log x) = dt=> — dx = dt=z> dx =x dt


x
Putting 1 + log x = t and dx = = x dt, we get
f2 t3 (1 + log x)3
/ = [ —xxdt=[t2dt = — + C = +C
J x J 3 3

ReadYourFlow.COM
INDEFINITE INTEGRALS 19.53

sec2 (2 tan 1 x)
(ii) Let Z=| dx
l+x2

Let 2 tan 1 x =t. Then, d (2 tan 1 x) =dt=> 2 1 1. , l+x2 ,


dx = dt=> dx =------- dt
l + x2 2
l+x2
Putting 2 tan -1 x -t and dx = —^— dt, we get
2 2
sec t l+x ^1 J sec2 t dt = ^ tan f + C = -^ tan (2 tan-1 x) -C
dt = —
ITT3 2
EXAMPLE 4 Evaluate:
2
tan .r sec x
(ii) | sec3 x tan x dx
©J 2 2 dx
(a + b tan x)
2

w
tan x sec x 2
SOLUTION (i) Let / = J 2 2 dx. Let a + b tan x =t. Then,
(a + b tan x)
2 2

Flo
dt
d (a + b tan x) dt => 2 b tan x sec x dx = dt => dx = 2
2b tan x sec x

ree
Putting a + b tan 2 x = t, and dx = dt
2 , we get

F
2b tan x sec x
2 or
tan x sec x dt
ur
/
=J x---------------2
sf
t2 2b tan x sec x
1 t~2 dt=—— + C= - 1
k

=> +C
Yo

2b 2bt
oo

2b (a + b tan 2 x)
(ii) Let / = rI sec 3 x tan x dx =\r sec2 x (sec x tan x) dx
B
re

dt
Let sec x = f. Then, d(secx) = dt=> sec x tan x rfx = rff =^> dx =
ou

sec x tan x
ad

dt
Y

Putting sec x = t and dx = , we get


sec x tan x
nd
Re

f3 1 3 r.
/ = I f2 dt = —+ C — sec x + C
Fi

3 3
EXAMPLE 5 Evaluate:
(i) J x3 sin x4 dx (ii) f e * cosec2 (2 e x + 5) dx
-1 .3
(hi) f x 2 tan — dx
l+x6
SOLUTION (i) Let / = J x3 sin x4 dx.

Let x4 = f. Then, d(x4) = dt=> 4x3 dx = dt => dx = 1


3dt
4x3
r 3 . . dt 1 cos x4 + C
/ = I x sin t x sin t dt = - — cos t + C = —
4x3 ii 4 4
(ii) Let / = | e x cosec2 (2e x +5) dx.

Let2e~x +5=t Then, d(2e~x+5) = dt^> -2e~x dx = dt ^ dx = ——


2e x

ReadYourFlow.COM
19.54 MATHEMATICS-XII

r f -ST 2 , dt — f cosec2 t dt= ^ cot t + C = ^ cot (2c x + 5) + C


I = \ e cosec t -
2c x
\
-1 .3
2 tan dx. Let tan 1 x3 =t. Then,
(iii) Let J = J *
1+x6

d (tan 1 x3) =dt=> 1 - 3x2 dx = dt=> dx = (1+/) dt


1+x
O 3x2

f 1 +x6
; = J*2x dt = - [ tdt = -t2 +C = ^ {tan-1 x3}2 + C
r77x 3.t2 3 J 6

EXAMPLE 6 Evaluate:
(i) | ^tan x (1 + tan2 x) rfx (ii) J (f (ax + b)}n f' (ax + b) dx ,n * -1 [NCERT]

w
sin 2x
(iii) J - dx
(a + b cos x)
SOLUTION (ii) Let / = J ^jtanx (1 + tan2 x) dx = J ^jtanx sec2 x dx
2

Flo
ee
Putting tan x = t and sec x dx = dt, we get

Fr
3/2
t 3/2
I = \ ft dt = + C = | (tan x) +C
3/2 for
ur
(ii) Let / = | [f (ax + b))n f' (ax + b) dx
ks

Putting f(ax + b) = t and f '(ax + b). adx = dt, we get


n
Yo

n+ n+ 1
oo

1 t \f(ax + b)}
i = -! tn dt = - +C = + C,n*-1
B

a J a n+1 a(n+l)
re

sin 2x 2 sin x cos x


(iii) Let / =J 2 dx = J 2 dx
ou

(a + b cos x) (a + b cos x)
ad
Y

Putting a + b cos x -1 and- b sin x dx = dt or, dx = - dt/b sin x, we get


2 sin x cos x dt = - — f \ ■ cos x dt
x—
nd

1 =
Re

t2 b sin x b J fz
Fi

t -a
=> I = -
i(MT)
_2r l_a_
dt ■: a + b cos x = t cos x =
b

=> I = - dt
b2]{^'t2
2 a
=> I = - 4- i°g|ti +- + C = —- log \ a + b cos x | + +C
b2 t b2 a + b cos x

LEVEL-2
EXAMPLE 7 Evaluate:

2Z2X 22 1
(i) J 2 2X dx (ii) j dx, a*nK,n<=Z [NCERT]
Jsin3 x sin (x + a)

1/4
(x4-z)—dx
(iii) j 5
x

ReadYourFlow.COM
INDEFINITE INTEGRALS 19.55

2X
SOLUTION (i) Let / = jf 222^ 222X 2X dx. Let . 2
2Z
-t. Then,

22* 22 22X 2X (log 2)3 dx = dt


d 22 = dt^> 2

2Z2X 22x
22'V 2x (log 2)3 dx = dt, we get
Putting 2 = t and 2

1 1 2X
'=f (log 2)
3dl =
(log 2) (log 2)3
2 +c

(ii) We have.
in 3 x sin (x + a) = sin 3 ^ (sin x cos a + cos x sin a) = sin4 x (cos a + cot x sin a)
1
1

low
/ = dx
-Jsin3 x sin (x + a)
2
1 cosec x
=> I =
I-sin 2 i-----------------------------
x yjcos a + cot x sin a
dx = j
^jcos a + cot x sin a
dx

ee
rF
Fr
2
Putting cos a + cot x sin a = t and, - cosec x sin a dx = dt, we get

1 1 1 (tl/2
-1/2
for
I
=1 sin a ft
dt = -
sin a
dt = -
sin a 1/2
+C
u
ks

=> 1/2
I = -2 (cosec a) f + C = -2 cosec a (cos a + cot x sin a) +C
Yo
oo

\l/4
1
B

* 1-_3 / \1/4
re

* j r 1 1
(iii) We have, / = J
x
5 1 -jr—dx = J X V 1—J
t x y
dx
ou
ad

1 1 3 x4
Y

Let 1---- = t. Then, d 1-------- - = dt => --r dx = dt => dx = — dt


x1* x- x4 3
nd
Re

/ -I 1^ 4X
Putting 1---- - =t and, dx = — dt, we get
Fi

l x3 J 3
I = if 4 X f1/4 xx4 M = i f t
1/4 = —15/4 + C = i r4 + c
A i-L
3J x4 15 15l x3J
EXAMPLE 8 Evaluate:
4 9
sec x cos x
dx (ii) } dx
fan x sin x
4
sec x
SOLUTION (i) Let / = J = dx. Putting tan x = t and sec x dx = dt, we get

4 2 2
sec x dt sec x 1 + tan x 1 +t2
I
~Jt~X
V£ sec 2 x
1 f
dt
=1 f
dt
=1 f
dt

-1/2 + t3/2) dt = 2f1/2 + -t5/2 + C = 2 fan x + ^ tan 5/2


=> / = x +C
5

ReadYourFlow.COM
19.56 MATHEMATICS-XII

9
(ii) Let / =j * dx. Putting sin x-t and cos x dx = dt, we get
sin x
9 (1 - sin2 x)4 dt
COS X dt COS X
I
=1 t
X
COS X t =1
dt =
t I
2x4
d-n dt = 1 -4f2 + 6f4 -At6 +t8 dt = f --At + 6t3 -At5 +t7 dt
=> '=1 t I t J t
= log|t|-2f2 +|f4 -|f6 + 1 .8
1 r +c
8
3.4
2 sin x----2 sin
.6 1.8 -
=> / = log | sin x | - 2 sin ^ x + — x + — sin x + C
3 8
1
EXAMPLE 9 Evaluate: J dx
*l(x-l)3(x + 2f

w
1
SOLUTION Let
I dx. Then,
P~1)3(x + 2)5

Flo \— 3/4

ee
1 1 X —1 1
---- dx = j dx - j
It x — 1 A3
Fr
I= X 2 dx
\3/4 x+2 (x + 2)
8 x —1
(x + 2) (x + 2)2
for
ur
x+2 x+2

x —1 3
ks

Let = t or, 1 - = t. Then,


Yo

x+2 x+2
oo

3 3 1
B

d \- = dt=z> — dx = dt=^> 2dX= -


3 dt
x+2 (x + 2) (x + 2)
re

x —1 1
= t and -dx - —dt, we obtain
ou

Putting
ad

x+2 (x + 2) 2 3
Y

1/4
4 x—1
I =- \ t -3/4 dt = *tl/4 + C= -
nd

+C
Re

3 J 3 3 x+2
Fi

EXAMPLE 10 Evaluate:
(i) J xx (1 + log x) dx (ii) J x2a (1 + log x) dx

SOLUTION (i) Let x* = t. Then,


d (xx) =dt => d (exlogx) =dt=> g*logX(iogx +1) dx=dt^ xx(l + logx) dx = dt

/ = J xx (1 + log) ix = J dt=t + C=xx+C

(ii) Let 7 = J x2x (1 + log x) dx

Putting xx = t and xx(l + logx) dx = dt, we obtain


7 = J x'v xx (1 + log x) dx = J t dt = ^t2 +C = ^ (x'r)2 + ^ = -x2x
2 +C

x
EXAMPLE 11 Evaluate: J dx
x-V7 -1
ReadYourFlow.COM
INDEFINITE INTEGRALS 19.57

x
SOLUTION Let / =J dx. Then,
x-^x2 -1

x x+
idx
x- x + yjx2 — 1

X (x + V*2 -1) =J 2
=> 2,2^
X 4- X
x -(x -1)

/ = J x^c/x + | V^2 -1 x dx
.3 x
=> — + — j -Ji dt, where t = x

=> /=^+it3/2 +c

low
3 3

=i> /= 4+4(*2-
3 3
d 3/2
+c
3
cos x

ee
EXAMPLE 12 Evaluate: J dx
sin2x + sinx
rF
Fr
3
COS X
SOLUTION Let / = | dx. Then,
for
sin2 x + sinx
2
u
COS X (1 -sin2x)
W cosx dx = J cosx dx
ks

sin x (sin x 4-1) sin x (1 4- sin x)


Yo
oo

Let sin x =t. Then, d (sin x) = dt or, cos x dx = dt.


B

l-t2
dt=\ i-t
H dt = log | /1 -/ 4-C
re

t(l+t) t
=>
ou

I = log | sin x | - sin x 4- C


ad
Y

EXERCISE 19.9
LEVEL-1
nd
Re
Fi

Evaluate the following integrals:


1
log 1 +
log* ^dx
1. dx 2.
X x (1 + x)
(1 + v^)2
3. J dx 4. X
4- c c dx
X i

V?
X
5. J ^cos2 x sin x dx e
6.
I (1 + 0
Xv2
dx

sin -1 x>2
f 3 2 {e }
7. I cot x cosec x dx 8.
I dx

1 4- sin x 1
9- J V* - COS X dx 10.
•Jl - x2 (sin 1 x)2
dx

ReadYourFlow.COM
19.58 MATHEMATICS-XII

cot x tan x
11.
J ^jsm x
dx 12.
I tJcOSX
dx

3 . 3
cos x , sin ^ dx
j
13. = dx =
yjsinx cos .t

1 Jtan x
15. dx 16. —^--------- dx [NCERT]
tan
-l x (1 + x2) sin x cos x

17. f - (log x)2 dx 18. J sin5 x cos x dx


J X
3
19. | tan 3// 2 x sec2 x dx 20. *
2------- 3dX j

(x2+l)3
4x+ 3
21. \ (4 x + 2) ^x2 + x + 1 dx [NCERT]

low
22. dx
y 2 x2 + 3x + 1

1 f cos2
23.
1 + ^fx
dx 24. ie A sin 2x dx

ee
1 + COS X
rF cos x - sin x

Fr
25. 3* 26. dx [NCERT]
(x + sin x) 1 + sin 2x
for
sin 2x log *2
27. 2 dx 28.
1 dx
u
(a + b cos 2x) x
ks
Yo

sin x
I 30. J cot x log sin x dx
oo

29. 2 dx [NCERT]
(1 + cos x)
B

31. | sec x log (sec x + tan x) dx [NCERT] 32. J cosec x log (cosec x - cot x) dx
re

f 3 4 ,
34. | x3 sin x4 dx
ou

33. I x cos x dx
ad
Y

• -1 .2
x sin xJ ,
36. J x3 sin (x4 + 1) dx
35.
1 - dx
nd
Re
Fi

(* + 1)*'\x f 2 X3 r3
37.
J 2 /
cos (xe ) X\
38. J x e cos (e ) dx

x +1 2
39. J 2xsec3 (x2 + 3) tan (x2 + 3) dx 40.
I x
(x + log x) dx

[NCERT, CBSE 2002C]


Y sin {1 + (log x)2} dx
41. | tan x sec2 x - tan 2 x dx 42. j log
x
/1 \

43. \ Ar cos
2i 1 J sec4 x tan x dx
dx 44.
X xj
Jx , -Sx \ j cos X
5
cos(cv ) ,
45. 1 dx 46.
sin x
dx [CBSE 2005]

ReadYourFlow.COM
INDEFINITE INTEGRALS 19.59

sin 7^ (.r +1) ex


47. dx [NCERT, CBSE 2009] 48. dx
. 2/
sin (x e )
f 2 ^ m sin -1 x
tan -1 x * +2 so. J £
49. J 5 _r + dx dx
x2+l

cos V* dx sin (tan -1 x)


51. J [NCERT, CBSE 2009] 52. J dx [CBSE 2002]
V? 1 + x2
m tan -1 x
53.
sin (log x) dx
X
54
'r- 1+x2
dx [NCERT]

x x tan -1 x 2 ,
56. J — dx

w
55. dx
'x2 + a2 + 1x 2 -a 2 i + *4

(sin 1 x)3

Flo
sin (2 + 3 log x)
57. J dx 58. J dx
x

ee
2x

Fr
2
59. j xex dx 60' l T dx
l + ex for
ur
sec [CBSE 2009] 62. J tan 3 2x sec 2x dx [NCERT]
61. J
ks
Yo

LEVEL-2
oo
eB

x+ 64. | S5" 55X5xdx


63.
I x+2
r

1
ou
ad

65. dx [NCERT EXEMPLAR]


Y

X-V x
nd

1
Re

67.
1 (x + 1) (x2 + 2 x + 2)
dx
Fi

69. J 4 x3 ^5-x2 dx

1 dx [NCERT]
71.
2,4, 1N3/4
X (x + 1)

ANSWERS
2
1 (log x)2 2. - — J log 1+1 3. l(l + ^)3+C
+C +C
2 2 x 3
2 xx3/2 c 3 5/3 1
4. -(1 + e ) +C 5.---- cos x+C 6. - +C
3 5 (1 + ex)
-1 x,2
7. -Icot4 x + C 8. l{g sin r+c 9. 2 ^x - cos x + C
4 2

ReadYourFlow.COM
19.60 MATHEMATICS-XII

1 2 2
10. - +C 11. - +c 12. +c
sin 1 x ^jsm .v Jcosx
2 5/2
13. 2 ^/sin x _ ^ (sin x) +C 14. - 2 'cos x + - cos 7 x+C
v 5
15. 2 ^/tan -1 x + C 16. 2 ^/tan x + C 17. |(logx)3+C

18. — sin 6 x + C 19. - tan"’7 2 x + C 20. -


6 5 4 (a:2 +1)2
4 9 3/2 + C 22. 2 y 2x2 + 3x +1 + C
21. -(x2 + X + 1) 23. 2 V* - 2 log 11 + V* | + C
3
2 -1
cos 1
24. -e ^+c 25. 2+C 26. - +C
2 (x + sin x) (sin x + cos x)

w
1 + C 28. (log x)2 + C 1
27. 29. +C
2 b (a + b cos 2x) 1 + cos x

30.
^{log| sinx |j +C

Flo 31. ^ | log | sec x + tan x 11 +C

ree
1 2
1 4 ~
+C

F
32. cosec x- cot x 33. sin x + C
2 4
-1 2.2 ^ or
ur
34. - - cos x4 + C 35. — (sin X ) +C 36. ^ cos (x4 + 1) + C
4 4
sf
1 r3
37. tan (xex) + C 38. — sin (e' ) + C 39. sec3 (x2 + 3) +C
k
Yo
oo

40. — (x + log x)3 + C 41. --(1 - tan2 x) 3/2 +c 42. cos {1 +(logx)2} + C
B

3in
2 l x J 41 ”
re

1 2 1 4
43. - — sin - I+C 44. — tan x + — tan x + C
x 2 4
ou
ad

45. 2 sin (e^) + C 46. sin 4 Y - sin2 x + log | sin x | + C


Y

1 -1
nd
Re

47. - 2 cos yfx +C 48. - cot (xex) +C 49. (5Y+tan ^J+C


log5
Fi

1 nt sin-1 -1
50. c X +C 51. 2 sin Vx + C 52. - cos (tan x) + C
m
i -1
i in tan
53. - cos (log x) + C 54. -e ^ +c
m
1 ,, 2 2.3/2 , 2 2.3/2, , ^
55. ^-|(Y +fl ) 7 -(x -a )'} + C 56. — (tan -1 x2)2 + C
6a2 4
1 1 x2
57. — (sin 1 x)4 + C 58. - — cos (2+3 log x) + C 59. - c'r + C
4 3 2
1 1
60. - log (1 + ex) + C 61. 2 tan Jx + C 62. - sec' 2x - sec 2x + C
6 2
55’r

63. (x + 1) + 2 jx + 1 - 2 log | x + 21 - 2 tan 1 ^Jx + 1 + C 64.


s5 3+C
(\oge 5)
1 -1, 2. ~ 66. 2 yjex -1-2 tan-1 -Jex -1+C
65. — sec (x ) + C

ReadYourFlow.COM
INDEFINITE INTEGRALS 19.61

x+1
67. log +C 68. 2(i+*3)3/2 2(1 + *3)1/2 + C
-Jx2 + 2 x + 2 9 3V

69.*(5 -x2)5/2 20 2,3/2


+c 70. 2 log 11 + V* | + C
5
1/4
1 2 1
71. +C 72. - cos x - +C
cos x 3 cos 3 x

HINTS TO NCERT& SELECTED PROBLEMS


^tan x
16. Let / = J dx. Then,
sin x cos x

low
Vtan x 2
I = 1 sec x dx [Dividing numerator and denominator by cos9 x]
tan x
=j> I = § (tan x) 2 sec2 x dx

ee
2
Let tan x - t. Then, d (tan x) -dt => sec x dx = dt
rF
Fr
1/2 dt = 2 yfi + C =2 yjtan x + C
, i = Jr for
21. Let J = J (4x + 2) -Jx2 + x + 1 dx. Then, 1 = 2 J (2x + 1) -Jx2 + x + 1 dx
ou
ks

Let x2 + x + 1 = t. Then, d (x2 + x +1) = dt ■=> (2x + 1) dx = dt


oo

2 \ J at = 43 f3/2 + C = - 4 (x22 + X + 1) 3/2


/ +c
Y
eB

3
cos x - sin x cos x - sin x
26. Let 7 = J dx. Then, J = J 2 dx
1 + sin 2x
r

(cos x + sin x)
ou
ad

Let cos x + sin x = t. Then, d (cos x + sin x) =dt => (cos x - sin x) dx = dt
Y

1 1
I -- + C =- +c
d

t cos x + sin x
Re
n

30. Let I = J cot x log sin x dx. Let log sin x = f. Then, d (log sin x) =dt => cot x dx = dt
Fi

[ tdt = -t2 +C = — 1 2
(log sin x) + C
J 2
40. Let 7=|^ x + l] 2
(x + log x) dx. Let x + log x = t. Then,
x
x+1
d (x + log x) = dt => 1+ dx = dt =$ dx - dt
x x
7 = J t2 dt =—t3 + C = — (x + logx)3 + C
3 3
sin Jx dx. Let -Jx = t. Then, d (yx) = dt => -^=
47. Let 7 = J dx = dt => dx = 2 Jx dt
Jx 2jx
11 1
, z=j = — f sin t dt = — cos t + C - — cos Jx + C
2J 2 2

ReadYourFlow.COM
19.62 MATHEMATICS-XII

cos 1
51. Let ^ = J dx. Let = t. Then, d(^)=dt dx =dt => dx = 2y[x dt
y/x ijx
COS ■fx dx = ^ j cos t dt = ^ sin t + C = ^ sin x+C
.-. I
=1
m tan -1 x
-1
54. Let / dx. Let rn tan x = f. Then,
l+x2

d (w tan 1 x) - dt => ---- 2 dx = dt => dx = — dt


1+x 1 + x2 m
-1
1| e1t dt = - el + C = 1 m tan * +C
e
m J m m
f 3
62. Let I = I tan' 2x sec 2x dx. Then,

w
I = J tan2 2x sec 2x tan 2x dx = J (sec2 2x -1) sec 2x tan 2x dx

Flo
Let sec 2x = t. Then, d (sec 2x) =dt => 2 sec 2x tan 2x dx =dt
V------1

ee
1 +C = — sec3 2x - 1 sec 2x + C

Fr
2 3 6 2
/
2tdt
66. Let ex -1 = f2. Then, d (ex -1) = dt2 => ex dx = 21 dt => dx =
for
ur
t2+l
ks

t-2t f2 +1 -1
J ylex-l dx =j dt = 2 j dt = 2\ 1 -—^—dt =2(t- tan-1 t)+C
Yo
oo

t2 + l t2 + 1 J r +1
B

1 1
71. Let / = J dx = J dx. Let 1 + -K = t.
re

2,4 .,3/4 \3/4 .4


X (X + 1) X
i+i
ou
ad

X J
Y

1 1 1 4
Then, d 1 + —=dt => - — dx = dt => dx = — dt
X5 4
nd

V x J X
Re

1/4 ^
i r4 + C
Fi

1 -3/4 1 t
:. I dt = — t dt = - + C = ~ 1 + -4
t
3/4 ll 4 1/4 x y

P(x)
19.8.8 INTEGRALS OF THE FORM { (ax + bf P (x) dx, J - dx, WHERE P (x) IS A
(ax + b)
POLYNOMIAL AND n IS A POSITIVE RATIONAL NUMBER
In order to evaluate this type of integrals, we may follow the following algorithm.
ALGORITHM
t-b 1
STEP I Substitute ax + b = t or, x = and dx = — dt
a a
n+ 1
t
STEP II Simplify the integrand in terms oft and integrate with respect to t by using J tndt = + C.
n+1
STEP HI Replace t by ax + b
Following examples will illustrate the abo\ j procedure.

ReadYourFlow.COM
INDEFINITE INTEGRALS 19.63

ILLUSTRATIVE EXAMPLES

LEVEL-1
2
X
EXAMPLE 1 Evaluate: J dx

2
SOLUTION Let / dx. Substituting x + 2 = t and dx = dt, we get
V* + 2
f2 - 4f + 4 3/2 -4t1/2 +4f1/2) dt
1=J s *t=\
St
dt =

f = 2t5/2_8 (3/2 + gf 1/2 |<,+2) 5/2 8 3/2


=> +c = -(* + 2) + 8 tJx + 2 + C
5 3
EXAMPLE 2 Evaluate: j x2 + x dx

w
SOLUTION Let / = J x2 + x dx. Substituting 1 + x = t and dx = dt, we get

I = J (t-1)2 S dt = J {t2-2t + l)S dt = \ (t5/2 - 2t 3/2 +fl/2 ) dt


J = 2 ^/2_4f5/2+2f3/2+c =

Flo
I (1+*) 7/2 hi+x) 5/2 + |(l + x) 3/2 +c

ee
7 5 3

Fr
EXAMPLE 3 Evaluate: J x (1 - x)" dx
or
ur
SOLUTION Let / = J x (1 - x)'' dx. Substituting 1 - x = i and dx = - dt, we get
f
1 = | (1 -t) tn dt = - \ (tn-tn+ !) dt
ks
Yo

n +1 +2 1
t 1 n+ 1 j; + 2
oo

=> ------- 1------- + C = - (1-x) + (1-x) +C


H + 1 77 + 2 77 + 1 n+2
B

5
re

EXAMPLE 4 Evaluate: J — dx
x +1
ou
ad

5
Y

SOLUTION Let / = Jf X+l


dx. Substituting x + l = t and dx = dt, we get
nd
Re

(t-1)5 - 5C114 + 5C2 f3 - 5C3 t2 + 5C4 f - 5C5) dtj dt


1=1 dt = 1 7 (5c„f5
Fi

t
=> I = J i(f5-5f4+10f3-10f2+5f-l) dt

=> I = t4 -5f3 + 10f2 -lOf+ 5- 1 dt = L-^t* + ™t3 -5t2 +5t-\og\t\ + C


3

=> / =
S+if- — (x + l)4 + — (x + 1)3 -5 (x + l)2 + 5 (x + 1) - log | x + 11 + C
4
2
3
X
EXAMPLE 5 Evaluate: J dx
(a + bx)2
2
SOLUTION Let/ = f --------- r- dx. Substituting a + bx = t and d (a + bx) =dt or, b dx = dt,
(a + bx)2
we get

ReadYourFlow.COM
19.64 MATHEMATICS-XII

t2 - 2at + a2 2a a2
/ = I b2t2 b t2
*=4 j i -----t +
b3 J t2
dt

2 1 2
=> f - 2fl log | f | - — > + C = — (a + bx) - 2a log \a + bx\---- - +C
i3 * b3 a + bx

LEVEL-2
1
EXAMPLE 6 Evaluate: dx [NCERT]
x1'2 + X 1/3

SOLUTION Here, the exponents of x are — and — and the LCM of their denominators is 6.
2 3
So, to remove fractional exponents, we substitute x = f6 and dx = 6f5 dt.
6f5 f3 <f3 + 1)-1^

low
1
I
=1 x1/2 + x 1/3
dx =
1 f3+f
2* = 6f
f+ 1
dt =
t+1

=> / = 6 J f2 -f+ 1 - 1 dt
f+ 1

ee
\
rF
Fr
3 2
=> l = 6 L--L- + £_i0g|f + i|.+c = 2-Jx -3x 1/3 + 6 x1/6 - 6 log | x1/6 + 11 + C
for
1/2
X
u
EXAMPLE 7 Evaluate: J
3/4 dx
ks

1 +x
Yo
oo

1 3
SOLUTION Here, the LCM of the denominators 2 and 4 of the exponents - and — is 4. So, to
B

remove fractional exponents, we substitute x = t4 and dx = 4t3dt.


re

1/2 2 5 3
* = ! -JL^Jidx=j -^4t3df = 4j -2—d( = 4| 4—t2dt
ou
ad

1 l + x3'4 J 1+f3 J f3+l 1 t3+l


Y

Let t3 +1 = m. Then 3t2dt =du or, t2dt = —du.


nd

3
Re

^ f M-l
Fi

7 = 4 — x—du =
J u 3 3 l
=> / = |(u-log«) + C=||(t3+l)-log(t3 + l)|+C

=> +c

^ dx
EXAMPLE 8 Evaluate: J

xm
SOLUTION Let 7 = J dx.
x1/2-xV3
Clearly, the LCM of 2 and 3 is 6. So, by putting x = f6 and dx = 6f5 dt, we get
f3 t6 f6 -1 + 1 t6-!6 '-dt
I =
1 t3-t
^■6t5dt = 6j
t~l
dt = 6 \
t~l
dt = 6$
f-1 t-1

ReadYourFlow.COM
INDEFINITE INTEGRALS 19.65

7 = 6 f t5 +t4 +t2 +t + l + —5— dt


=>
J 7-1
76 75 3 2
=> I = 6 < — + — + ^- + y + 7 + log (7 -1) * + C
6 5
x5/6 x1/2 x1/3 1/6 1/6 -1) +C
=> I= 6^—+ + +X + log (x
6 5 3 2

1
EXAMPLE 9 Evaluate: J dx.
3Jx + l + t]x + 1
1 1 1
SOLUTION Let J dx. Here the exponents of (1 + x) are - and — and the LCM of
3JX + 1 + yjX + 1
their denominators is 6. So, we substitute x + 1 = 76 and dx = 675d7.

w
1 y 6i5 dt = 6 j 73 73 +1 -1
1 =1 t2 +t 7+1
dt =
7+1
dt

73 +1 1
Flo
ee
=> 7=6j d7-6j dt = 6 | (72 - 7 + 1) rf7 — 6 J
7+1 7+1

Fr
r3 r2 t - 6 log |7 + 1| + C = 273 - 372 + 67 - 6 log |7 + 1| + C
=> I= 6 -----------+ 7
for
ur
3 2

=> 1= 2(x + 1)1/2 - 3(x +1)1/3 + 6(x +1)1/6 -6 log| (x + l)16 + 11 + C


ks
Yo

EXERCISE 19.10
oo

LEVEL-1
eB

2 2
X
1. J X2 ^x + 2 dx
2-\-r
- V*-1
dx 3.
l y3x + 4
dx
r
ou
ad

x2 + 3x + 1
, f 2*-1 ^
1
Y

5.J(2x2 + 3) * 6. dx
J7 ^
- (x — 1) (* + l)2
nd
Re

2
dx 8. J x(l -x)23 dx
Fi

V1-X
LEVEL-2
1 1
■Jx + ^fx
dx 10.
1 x1/3(^/3-l)
rfx
___________________ ANSWERS

1. | (x + 2) 7/2 8 5/2 8 3/2 2. ^ (3x2 + 4x + 8) yjx^l + C


^(t + 2) + 3(X + 2) +C
5
3. (3J +4)5/2 (3x + 4)3^2 + (3x + 4) 1/2 + C 4. - 1
+ 2 log | x -11 + C
135 81 27 x —1
7/2 16 5/2 22 3/2
6. x + + log | x + 1| + C
+C
5. - (x + 2)
7 t(i+2) +
t(i+2) x +1
7. — (3x2 +4x +8)^/1 -X +C 8- - (1 ~ ^)24 (! + 24'y) + c
15 600
1/3
9. 2-v/x-4x1/4 +4 log 11 + x4 | + C 10. 3x 1/3 + 3 log (x -1)+C

ReadYourFlow.COM
19.66 MATHEMATICS-XII

19.8.9 INTEGRALS OF THE FORM J tanm x sec2” .t dx,j cotm x cosec2” x dx; m,neN

In order to evaluate this type of integrals. We may follow the following algorithm.
ALGORITHM
STEP I Write the given integral as / = J tan"' x (sec2 x)(" sec2 x dx

STEP II Put tan x = t and sec x dx = dt and write the integral as


I = J tan'" x (sec2 x)" ~ 1 sec2 x dx

or, ^ = J tan'" x (1 + tan2 x)n~ 1 sec2 x dx


2\H- 1 dt
or, / = J tm (1 + t2)
STEP III Expand (1 + t 2>) h- 1 by binomial theorem in step II and integrate.

w
STEP IV Replace t by tan x in step III.

Flo
Following examples will illustrate the above procedure.

ree
ILLUSTRATIVE EXAMPLES

F
LEVEL-1
EXAMPLE 1 Evaluate: or
ur
sf
(i) J tan" x sec2 x dx (ii) | tan 2 x sec4 x dx (iii) J sec4 x dx
k
Yo

SOLUTION (i) Let/ = J tan" xsec2 xdx


oo

2
B

Substituting tan x = t and sec x dx = dt, we get


n+ 1
re

t 1
/ = jtn dt = +C = tan n+ 1 x + C
n+1 n+1
ou
ad

(ii) Let / = J tan2 x sec4 x dx. Then,


Y

I = Jp tan 2 x sec2 x sec2 x dx =1("2 22


tan x (1 + tan x) sec x dx
nd
Re

2
Substituting tan x = t and sec x dx = dt, we get
Fi

3 5
I = \t2(l+t2)dt = \ (t2+t4)dt = L + L+c = I tan 3 x + - tan5 x + C
J J 3 5 3 5
(iii) Let J = J sec4 x dx. Then,
f 2 2 r 2 2
/ = j sec x sec x dx = I (1 + tan x) sec x dx
Putting tan x = t and sec 2 x dx = dt, we get
t3 1 tan 3 x + C
/ = J (1 +/2) dt = f + + C = tan x +
3 3
EXAMPLE 2 Evaluate:
(i) J cot2 x cosec4 x dx (ii) | cosec4 x dx

SOLUTION (i) Let J = J cot2 x cosec4 x dx. Then,

I = fI cot 2 x cosec 2 x cosec2 x dx

ReadYourFlow.COM
INDEFINITE INTEGRALS 19.67

=> 7=| cot2 x (cot2 x + 1) cosec2 x dx = j (cot4 x + cot2 x) cosec2 x dx


2
Substituting cot x = t and - cosec x dx = dt, we get
75 73 ^ 1 5 1.3 r
7 = - \(t4+t2) dt = --------- + C - — cot x----- cot x + C
5 3 5 3
(ii) Let 7 = J cosec4 x dx. Then,
f 2 2
7 = [ cosec x cosec x dx = J (1 + cot x) cosec x dx
2
Substituting cot x = 7 and-cosec x dx = df, we get
.5
I
7 = - f (1 + f2) dt = -t----- + C = - cot x - — .3 ^
cot x + C
J 3 3
EXAMPLE 3 Evaluate: J tan 8 x sec4 x dx [NCERT EXEMPLAR]

w
SOLUTION Let 7 =J tan8 x sec4x dx. Then,
. o 2 2 C 8 22

Flo
7 = [ tan x sec x sec x dx = j tan x (1 + tan x) sec x dx

Let tan x =t. Then, d (tan x) =dt=> sec x dx = dt.

ee
Fr
79
I =J t8 (l + f2)df=J 78+f10d7= —+ -—
^ + C = — tan9 x + — tan11 x + C
9 11 for
ur
2m +1 2>i+\
19.8.10 INTEGRALS OF THE FORM } tan x sec x dx, WHERE m, n ARE NON-NEGATIVE
INTEGERS
ks

In order to evaluate this type of integrals, we may follow the following algorithm.
Yo
oo

ALGORITHM
B

STEP I Write the given integral as 7 = j (tan2 x)m (sec x)2" sec x tan x dx
re

STEP II Substitute sec x = 7 and sec x tan x dx =dt and write the integrals as
ou
ad

7 = J (sec2 x -l)m (sec x)2” sec x tan x dx


Y

or, 7 =
d
Re
n

STEP III Expand (72 - l)m by binomial theorem in step II and integrate.
Fi

STEP IV Replace t by sec x in step 111.


Following examples will illustrate the above procedure.

ILLUSTRATIVE EXAMPLES

LEVEL-1

EXAMPLE 1 Evaluate: j tan3 x sec3 x dx


f 3 3
SOLUTION Let 7 I tan x sec x dx. Then,
f 2 2 f 2 2
7 = tan x sec x (sec x tan x) dx = I (sec x -1) sec x (sec x tan x) dx

Substituting sec x = t and sec x tan x dx = dt, we get

7 = J (72 -1) 72 d7 = J {t4-t2)dt = j-— + C = - sec5 x--sec3 x + C


5 3

ReadYourFlow.COM
19.68 MATHEMATICS-XII

EXAMPLE 2 Evaluate: J sec" x tan x dx


SOLUTION Let I | sec" x tan x dx. Then,
/ J sec" 1 x (sec x tan x) dx

Substituting sec x = f and sec x tan x dx = dt, we get


tn
t n — 1 dt = — + C = - sec" x + C
I
J =1 n n
19.8.11 INTEGRALS OF THE FORM J tan" x dx, J cot" x dx

ILLUSTRATIVE EXAMPLES

LEVEL-1
EXAMPLE 1 Evaluate:

w
(i) J tan 3 x dx (ii) J tan4 x dx [NCERT]
SOLUTION (i) Let ^ = J bm 3 x dx. Then,

Flo
ee
/ = J tan2 x tan x dx = J (sec2 x -1) tan x dx = j tan x sec2 x dx - j tan x dx

Fr
Putting tan x = t and sec x dx = dt in first integral, we get
f2 1 2
1 = lf *-J
for
ur
tan x dx = — + log | cos x| + C = — tan x + log | cos x| + C

(ii) Let/ = J tan4 x dx.Then,


ks
Yo

l = fI tan 2 x x tan 2 x dx
oo
B

=> I = J tan^ x (sec2 x-1) dx


re

=> I = J (tan 2 x sec2 x - tan 2 x) dx


ou
ad

=> / = J tan 2 x sec2 x dx - J tan 2 x dx


Y

I C 2 2 f 2
=> J tan x sec x dx - I (sec x -1) dx
nd
Re

Putting tan x = t and sec dx = dt in first integral, we get


Fi

3
/ Jf2rft-J (sec2 x-1) dt = ~ - (tan x - x) + C = tan x - tan x + x + C
3
EXAMPLE 2 Evaluate:
(i) J cot3 x dx (ii) J cot4 x dx
SOLUTION Let/ = j cot3 x dx. Then,

I = J cot2 x cot x dx = J (cosec2 x -1) cot x dx = j (cot x cosec2 x - cot x) dx

=> I = | cot x cosec2 x dx - J cot x dx


Substituting cot x = t and - cosec x dx - dt in first integral, we get
t2 1 2
/ = - j t dt -j cot x dx - — - log | sin x | + C = - cot“ x - log | sin x | + C
(ii) Let J = j cot4 x dx. Then,

ReadYourFlow.COM
INDEFINITE INTEGRALS 19.69

I J cot2 x cot2 x dx - j (cosec2 x -1) cot^ x dx

/ = r (cot2 xcosec “ x - cot x) dx


2 2
=>

=> I | cot2 x cosec2 x dx - J cot2 x dx

=> / = J cot2 x cosec2 x rfx-J (cosec2 x-1) dx


J
Substituting cot x = t and - cosec x dx = dt in the first integral, we get
f3 ] o
7 | f2 df — I (cosec2 x — 1) dx = ---- (- cot x -x) + C = — cot X + cot X + X + C
3 3
LEVEL-2
n-2
EXAMPLE 3 Prove that:\ tan” x dx = —— tan” -1 tan x dx
J n-1

w
SOLUTION Let ln = J tan” x dx. Then,

w tan n-2 x tan2 x dx

Flo
ee
tan n-2 x (sec2 x -1) dx
=> '»= J

Fr
n-2 n-2
x sec2 x dx - fI tan
= J tan x dx
for
ur
Substituting tan x = 7, sec2 x dx=dt in the first integral on the right hand side, we get
7” -1 n-2
at-\ tan n-2 x dx =
4=J n-2
I tan
ks

7 x dx
Yo

n-1
oo

1 n-1 x - J tan n-2


=> J tan” x dx = tan x dx
B

n -l
re

EXERCISE 19.11
LEVEL-1
ou
ad
Y

Evaluate the following integrals:


1. fI tan 3 x sec2 x dx 3. J tan5 x sec4 x dx
nd

2. | tan x sec4 x dx
Re
Fi

4. J sec6 x tan x dx 5. J tan5 x dx 6. J ^/tan x sec4 x dx

7. | sec4 2x dx 8. | cosec4 3x dx 9. j cot” x cosec2 x dx, /? * -1


10. | cot3 x cosec4 x dx 11. J cot5 x dx 12. | cot6 x dx
ANSWERS

1. — tan4 x + C ~ —
2. 1 ,tan 2 x + 1 tan 4 x + C
4 2 4
1tan 6 x + — tan 8 x + C 4. - sec6 x + C
3.
6 8 6
5. ^ tan4 x tan2 -r + l°g I sec *| + C 2
6. tan 3//2 x + — tan7//2 x + C
3 7
7. — tan 2x + — tan 3,2x + C 8. - — cot 3x - - cot3 3x + C
2 6 3 9
1 1 cot6 x - 1
9. - cot
n+ 1
x+C 10. - cot8 x + C
n+1 6 8

ReadYourFlow.COM
19.70 MATHEMATICS-XII

1 2 Is X + —
1 cot 3 X
11. —1 cot,4 X + — cot x + log | sin .r | + C 12. -cot - cot x - x + C
4 5 3

19.8.12 INTEGRALS OF THE FORM J sin'" r cos" x dx, m, neN

In order to evaluate the integrals of the form J sin'" x cos" x dx, we may use the following
algorithm.
ALGORITHM

STEP 1 Obtain the integral, say, j sin'" x cos" x dx.


STEP II Check the exponents o/sin x and cos x.
STEP HI If the exponen t o/sin x is an odd positive integer put cos x =t.
If the exponent of cos x is an odd positive integer put sin x =t.
Ifthe exponents o/sin xand cos x both are odd positive integers put either sin x = f or,cos x =t.

w
If the exponents o/sin xand cos x both are even positive integers, then express sin'" x cos" x in
terms o/sines and cosines of multiples of x by using trigonometric results or De' Moivere's
theorem.
STEP IV Evaluate the integral obtained in step III.
Flo
ee
Fr
Following examples will illustrate the procedure.

ILLUSTRATIVE EXAMPLES
for
ur
LEVEL-1
ks

EXAMPLE l Evaluate:
Yo
oo

wjsm 3 x cos4 x dx
, (ii) J sin2xcos5xrfx
B

(iii) J sin 3 x cos3 x dx


re

C O A
SOLUTION (i) Let / = j sin x cos x dx.
ou
ad
Y

Here, power of sin x is odd, so we substitute


dt
cos i = f => - sin x dx = dt => dx = -
nd
Re

sm x
Fi

r f • 3 ,4 dt
I = I sm x t -
V sin x
/ J sin2 xf4 rff = -J (1-t2) t4 dt =-j (t4-t6) dt
5 7
I. ^+c=-cos X cos
>
5 7 5
+
7
±+c
f 2 5
(ii) Let / = J sin “ x cos x dx.
Here, power of cos x is odd, so we substitute
sin x - t => cos x dx = dt => dx = —^
cos x
I = [ t2 cos5 x ^ = [ f2 (1 - sin2 x)2 dt
phi-*2)2*
J COS X J

. 3 . 7 x
sm T-2sinS* + sm
=> / = I (t2 -2t4 +t6) dt = ----- — f5 + — 4- C = +C
3 5 7 3 5 7

ReadYourFlow.COM
INDEFINITE INTEGRALS 19.71

(iii) Let / = J sin 3 x cos5 x dx


Here, powers of both sin x and cos x are odd. So we can substitute either sin x = f or, cos x = t
, , , dt
Putting cos x = t and - sin x dx = dt or, dx = - , we get
sin x
r . 3 ,5 dt = - J f5 sin2 x dt = - J t5 (1 -f2) dt | (f5-t7)dt
I = I sin xt x -
sin x
8 6 8
COS X COS X
=i> +C + +c
6 8 6 8
EXAMPLE 2 Evaluate: J cos 3xeloSsinxdx.

SOLUTION We have.
I \ cos 3 x e log sin .t dx = \ cos 3 x sin x dx

w
Putting cos x = f and - sin x dx = dt or, sin x dx = - dt, we get
4
t4

Flo
COS X
I = - j t3 dt = +C = - +C
4 4

ee
19.8.13 TO EVALUATE INTEGRALS OF THE FORM { sin"x cos" x dx, WHEREm,neQ SUCH

Fr
THAT m + n IS A NEGATIVE EVEN INTEGER
for
ur
ALGORITHM
STEP I Change the integrand in terms of tan x and sec x by, dividing numerator and denominator by
cosk x, where k = - (m + n).
ks
Yo
oo

STEP II Substitute tan x =t.


Following examples will illustrate the above procedure.
B
re

ILLUSTRATIVE EXAMPLES
ou
ad

LEVEL-1
Y

EXAMPLE 1 Evaluate:
nd

. 4 x
Re

sm 1
©J 8
dx (ii) J I . 3 5~
dx
Fi

COS X Jsm x cos x


. 4
sin
SOLUTION (i) Let/ = J
s — dx. Then,
COS X
. 4
sm x
4
cos x [Dividing numerator and denominator by cos4 x]
I = 1 COS
8
X
dx

4
COS X
f 4 4 .
=> / = tan xsec x dx
=> 1 = J tan4 x (1 + tan2 x) sec2 x dx J tan4 x (1 + tan2 x) sec2 x dx
2
Putting tan x = t and sec x dx = dt, we get
, 5 . 7
tan x tan x
/ = Jf4(l + t2)* = J + J + C = +C
5 7

ReadYourFlow.COM
19.72 MATHEMATICS-XII

1
(ii) Let / = J dx. Then,
3 5
sin x cos x
1
/ =1 sin
■ 3/2
x cos5/2 X
dx

4
sec x
=> I
=J , 3/2
tan x
dx [Dividing N1 and Dr by cos4 x]

(1 + tan2 x)
=> /
=1 tan
3/2
x
sec2 x dx

2
Putting tan x = f and sec x dx = dt, we get
i+(2 3/2
2 2
I
=1 3/2 * =J (f ^+ty2)dt = zl + L_ + C=- + — (tan x)
3/2
+C

w
t it 3/2 ^/tan x 3

EXAMPLE 2 Evaluate: j sec 4/3 x cosec 8/3 x dx

SOLUTION Let /
I sec 4/3
x cosec
8/3

Flo
x dx . Then,

ee
1
dx = J cos -4/3 x sin 8/3 x dx

Fr
4/3 • 8/3
cos x sin x
for
ur
c. 4 8
Since - — + — 4, which is an even integer. So, we divide both numerator and denominator
3 3
ks

, 4
by cos x.
Yo
oo

4 (1 + tan2 x)
sec x
I
=i dx =
1 sec2 x dx
B

tan8/ 3 x . 8/3 x
tan
re

2
Putting tan x = f and sec x dx = t/f, we get
ou
ad

i+t2
+ 3t 1/3 + C
8/3
=1 J<r + r 2/3 )dt 3 ,-5/3
Y

i dt = = --t
8/3 5
t
nd

3 -5/3 1/3
Re

=> I =---- tan x + 3 tan x+C


5
Fi

LEVEL-2

sin 2 x
EXAMPLE 3 Evaluate: J 31 dx
14
COS X

. 2
sm x . 2/3 -14/3
SOLUTION Let I
14
dx = J sm x cos x dx
COS X

Here, the sum of the exponents of sin x and cos x in - 4, which is a negative even integer. So, we
divide and multiply by cos4 x to get

sin 2^ 3 x cos - 14/3 x cos4 x sec4 x dx


,
I =
I
sin2'3 *
=> I = ! sec4 x dx = J tan 2/3 x (1 + tan 2 x) sec2 x dx
cos 2/3 x

ReadYourFlow.COM
INDEFINITE INTEGRALS 19.73
2
Putting tan x = t, and sec x dx = dt, we get
f r 2/3+(8/3 dt = 3 5/3+±tn/3 + C tan5/3* + -itan11/3* + C
j v ; 5 11 5 11
EXERCISE 19.12
LEVEL-1

Evaluate the following integrals:


1. J sin4 x cos3 x dx 2. J sin5 x dx 3. J cos5 x dx
c 7
4. J sin5 x cos x dx 5. Jf sin3 x cos6 x dx 6. j cos x dx
7. J x cos3 x2 sin x2 dx C 7 9. J sin 3 x cos5 x dx
8. I sin x dx
1 1 1
J

w
10. j dx dx 12. dx
. 4 2 . 3 5 . 3
sin x cos x
sin x cos x sin x cos x
1
13. {
3
sin x cos x
dx [NCERT]

Flo
ee
ANSWERS

Fr
1.5 1.7 , ^ 2 3 - cos5 x 1 + C
1. -sin x — sin x + C 2. COS X-----COS X+
5 7 3 5
for
ur
. 6
3. i sin xsin 3 x + - sin5 x+C sin x
4. +C
ks

3 5 6
Yo
oo

7 9
cos x cos x .3 x + —
3.5 1.7
5. - +C 6. sin x - sin sin x — sin x + ^
C
B

7 9 5 7
re

7. — cos4 x2 + C 3 3 5 1 7
8. - COS X + cos x - — COS X + — cos x + C
ou
ad

8 5 7
Y

1 6 1 8 ^ 1 3
9. — cos x + — cos x + C 10. ----cot x - 2 cot x + tan x + C
6 8 3
nd
Re

- (tan x)_ 2 + 3 log | tan x| + -^ tan2 x + — tan4 x + C


11.
Fi

4
1 1 2
12. log | tan x | - +C 13. - tan x + log | tan x | + C
2 tan2x 2

HINTS TO NCERT & SELECTED PROBLEMS

13. Let 1 = J 1
dx. Then,
3
sin x cos x
4
sec x
'=1 tan x
dx [Diving numerator and denominator by cos4 x]

(1 + tan2 x) i+f2 2
^ I = 1 sec 2 x dx = j dt, where t = tan x and dt = sec x dx
tan x t

= J + f j df = ^ + log * + C = i tan2 x + log tan x + C


=> /

ReadYourFlow.COM
19.74 MATHEMATICS-XH

19.9 EVALUATION OF INTEGRALS BY USING TRIGONOMETRIC SUBSTITUTIONS


In this section, we will discuss evaluation of integrals by using trigonometric substitutions.
Following are some substitutions useful in evaluating integrals.
Expression Substitution
2 2
a +x x = a tan 0 or rt cot 0
a 2 ~x 2 x = a sin 0 or a cos 0
x 2 -a 2 x = a sec 0 or a cosec 0
a-x a+x
or. x = a cos 2 0
a+x a -x
x -a 2 2
or. tJ{x — a) (x — p) x - a cos 0 + P sin 0.

w
P-*
Let us discuss some problems on evaluation of integrals by making above substitutions.

FloILLUSTRATIVE EXAMPLES

ee
LEVEL-1

Fr
Type I EVALUATION OF INTEGRALS BY MAKING SUBSTITUTION x = a sin0 or, * = a sin2 0
for
ur
EXAMPLE 1 Evaluate: J 1
dx.
(a2-x2)3/2
ks

1
Yo

SOLUTION Let / = J dx and .r = rt sin 0. Then, dx - d (a sin 0) => dx = a cos 0 dQ.


oo

(a2-x2)3/2
B

1 fl cos 0 d0 = J a cos 0 rf0 =


'=1 J sec2 0 dQ
re

,2 2 .
(a -a sin 0)2 m 3/2 3 3Q
a cos 0 a
sin 0 a: x
/ = ^2 tan 0 + C = -4- -=
ou
ad

+C = +C = +c
a n2 Vl -sin2 0 2 2 r~2 2
Y

a V«
a
nd
Re

2
EXAMPLE 2 Evaluate: J —jJL dx.
Fi

yjl-X2
2
SOLUTION Let / = f , X dx and x = sin 0. Then, dx = d (sin 0) = cos 0 c/0.
VI-X2
’ ^0 i
Sm = cos 0 dQ = J sin20c/0 = —J (1-cos 20) c/0
Vi -sin2 0 ^

=> I =-{ 0- —sin20| + C = -0--sin0cos0 + C


2 2 J___-__2 2
=> /=— • -1 x——x Vl -x2 +C
! sm
2 2
2 - 3x +1
EXAMPLE 3 Evaluate: | - dx. [CBSE2015]
VI-X2
2 _ 3r +1
SOLUTION Let 7 = J --7— - dx and x = sin 0. Then, dx = d (sin 0) = cos 0 c/0.
Vl-x2

ReadYourFlow.COM
INDEFINITE INTEGRALS 19.75

sin“0-3sin0 + l cos0 dQ = J (sin2 0 - 3 sin 0 + 1) t/0


VI -sin2 0
1 - cos 20 -3 sin 0 + 1 rf0 = ^|(3-6sin0 - cos 20) dQ
=> I=
2
1 1
=> /=— 30+ 6 cos 0 — sin 20 +C
2 2

=> I = ^ • 30 + 6 Vl -sin20-sin0 vl - sin2 0 +C

1
=> /= 3 sin -1 X + 6^1 -X2 -X l/l -AT2 +C
2
2
EXAMPLE 4 Evaluate: f *— rfx

w
VI — x
rt2 r x~2
SOLUTION Let / = ■ dx = . ----- ■_ rfx.
J J Jl-(Jx)

Flo
ee
Let Vx = sin 0 or, x = sin2 0. Then, dx = d (sin2 0) = 2sin 0cos0 rf0.

Fr
2 2
(sm_ 9)_ 2 sin 0 cos 0 d0 = 2 I sin5 0 rf0 = 2 J (1 - cos2 0)2 sin 0 dQ.
Vi
-sin2 0
for
ur
Let cos 0 = a. Then, d (cos 0) = du or, - sin 0 dQ = du.
I=-l\ (1-m2)2 du = -2\ (1 -2w2 + «4) du
ks
Yo
oo

=> I =-2 u--u3+ — +C = -— u (15-10u2 + 3i<4) + C


eB

3 5 15

=> / =-~: (15-10cos2 0+ 3cos4 0) cos0 + C


r
ou
ad

I =- ^ j 15-10 (1 - sin 2 0) + 3 (1 - sin 2 0)2 J VI sin2 0 + C


Y

=>

I =- — j8 + 4sin20 + 3 sin4 ©Wl -sin20 + C


nd
Re

=>
15
Fi

^ |8 + 4sin20 + 3 sin4 Q^'Jl -sin20 + C = —— (8 + 4x+ 3x2) -x +C


=> I=
15
Type II INTEGRALS BASED UPON THE SUBSTITUTION x = atan0 OR, x=atan20
EXAMPLE 5 Evaluate : J 1
dx
(a2+x2)
1 2
SOLUTION Let 7 = J dx and x = a tan 0. Then, dx = d (a tan Q) = a sec 0 dQ.
(a2+x2)
1 sec2 0 dQ
(a2 +rt2tan20)

=> I=±\ cos2 QdQ = 1 J (1 +cos20) t/0 =^jj^0 + -^sin20j + C


a 2a3

tan — + ^] +C
1 tan 0 1 -1 x
=> I= 0+ +C = 2 2
2a3 1 + tan 2 0 2a3 a a +x

ReadYourFlow.COM
r

19.76 MATHEMATICS-XII

EXAMPLE 6 Evaluate: I -77-6*


x +x
SOLUTION Let / = J — 1 7- dx. Then,
O
X 4- X
1
^777 dx = j x4(l+x2)
dx

Let x = tan 0. Then, dx = sec2 0 dQ.


1 1
'-I rfx = J sec 20 rf0 = J cot4 0 dQ
x4(l+x2) tan4 0 (1 + tan2 0)
i = J cot^ 0 (cosec2 0 -1) c/0 = J cot2 0 cosec2 0 rf0 - J cot2 0 dQ

=> 7 = J cot2 0 cosec2 0 rf0 - J (cosec2 0 -1) rf0

w
7 =- J t2dt - J (cosec2 0-1) dQ, where t = cot 0

=> 1 .3 -(-cot0-0)+C =-—cot30 + cot0 + 0 + C =


7 =-—f ^-r- + — + tan 1 x + C
3 3 3x

Flo x

ree
Type III EVALUATION OF INTEGRALS BY MAKING SUBSTITUTION x = flsec0 OR, x = asec20
EXAMPLE 7 Evaluate: f —4__ dx

F
J x3J7^ or
ur
SOLUTION Let 7 = f — 1 = dx and x = a sec 0. Then, dx = a sec 0 tan 0 dQ.
sf
J X 3 yx/ 2 -a
k

1
Yo

flsec0 tan 0 dQ
oo

(a sec 0)3 Va2 sec2 Q-a2


B

7 =^r- [ cos2 Q dQ = 1 J (1 + cos 20) dQ = 1 1


re

0 + —sin20 +C
~a J 2a5 T 3
2a 2
ou
ad

7=
-1 -
x + JLjx2-a2 +C
(0 + sin 0 cos 0) + C = — sec
Y

2a 2«3l a x
1
nd

EXAMPLE 8 Evaluate : J dx
Re

x Vx4 1
Fi

SOLUTION Let J 1
dx. Then,
-1 x V?
1
W
Xyj(x2)2-1 dx=\ ~r^
x2p2)2-l
x dx

2 2 secQ tan 0
Let x = sec 0. Then, d (x ) = d (sec 0) or, 2x dx = sec 0 tan 0 dQ or, dx = dQ.
2x
,1 1
x — sec0 tan0 dQ = — f 1-dQ =— 0 + C =4SeC_1x2 +C
i=l sec0 -\/sec2 0-1 2 2J 2 2

ALITER I =J 1 x dx =— [ 1 2
dt, where t = x
xp2)2-\ 2J

1
/=—sec 1f + C = -sec—12
x +C
2 2

ReadYourFlow.COM
INDEFINITE INTEGRALS 19.77

LEVEL-2
Type I INTEGRALS BASED ON THE SUBSTITUTION x = a sin2 0 or x = a sinG
x
EXAMPLE 9 Evaluate: |
4,3/2 dx.
d-O
X X
SOLUTION Let I =|
4,3/2
rfx = J 3/2
dx
(i-o {l-(*2)2}
Let x2 - sin 0. Then, d (x2) = d (sin 0) => 2x dx = cos Q dQ => dx = COS0 dx
2x
x COS0 dx = - \ 2 1
sec" 6 d6 = — tan 0 + C
(1-sin2 0)3/2 2x 2J 2
2
1 sin 0
=> I =— +C
^ Vl -sin2 0 2VT-x4

w
7
EXAMPLE 10 Evaluate: f —- dx.
2,5

Flo
J d-O
SOLUTION Let x = sin 0. Then, dx = d (sin 0) = cos 0 dO.

ee
7 sin7 0
x cos 0 dQ = j tan7 0 sec2 0 dQ
dx = j

Fr
2,5 (1 — sin 2 0)5
d-O
2 du for
ur
Let tan 0 -u. Then, sec 0 dO = du or, dO =
sec 0
8 ,c_l *8
ks

sin 0
I = f u du = — + C = —tan80 + C = - +C
Yo

J 8 8 8 (1 -sin2 0) 4 8 (l-*2)4
oo

1
B

EXAMPLE u Evaluate: j dx.


re

(1 + O) tJx-x2
2
ou
ad

SOLUTION Putting x = sin t and dx = 2 sin t cos t dt, we get


1 1
Y

'=! = dx = f 2 sin t cos t dt


,2 J (1 + sint) -\/sin2f -sin4t
nd
Re

1 1 - sin t
=> I= 21 dt = 2\ dt = 2 | (sec2 t - tan t sec t) dt
o
Fi

1 + sin t cos t
sint 1
=> 1=2 (tan t -sect) +C =2 +C
\/l-sin2t \/l-sin2t
x 2
=> I= 2 +C
1 —x
Type II INTEGRALS BASED ON THE SUBSTITUTION x =fltan0 OR x =fltan20
EXAMPLE 12 Evaluate: J 1
2 dX
(x2 + 2x+ 2)
SOLUTION Let / = J 1 - dx. Then,
(x2 + 2x + 2)
1
W {(j: + 1)2+12} 2 dx

ReadYourFlow.COM
19.78 MATHEMATICS-XII

Let x + 1 = tan 0. Then, d(x + l) =d (tan 0) => dx = sec2 0 dd.


1 sec2 0 t/0 = J cos2 0 rf0 = i | (1 + cos 20) dd
(tan2 0 + 1^

/=- 0 + -sin20) + C = 1 0 + tan 0


=> +C
2 2 2 1 + tan 2 0
1 x+1
=> 7 = — tan 1(x + l) + —^ +C
2 x + 2x + 2
1 + x2
EXAMPLE 13 Evaluate : J dx
(1 - x2) + x2 + x4

1 +x2
SOLUTION Let I
=1 = dx. Then,

w
(1 - X2) -Jl + X2 + X 4

1 +x2

7 = 1 x
2

Flo
dx [Dividing andD? by x2]

ee
i 1 + x2 + x4

Fr
-X
X 2
X
for
ur
1
1 + ~2
=> 7
= -J 1
X dx
ks

X—
x2 + ^- + 1
Yo

X
oo

1 1 11 d
B

=> 7 = -
1 1+ ~2 rfx
re

x ——
l
X-----
if + 3
ou

X X
ad
Y

1 ill 1
Let x — — =7.Then, rf x — =dt => 1+—^ dx = dt.
x \ x) X2J
nd
Re

dt u du
7
= -1 1
, where f + 3 = «2 and 2t dt = 2u du
Fi

t^Ts (u2 - 3) V?
1 U-yf3
=> 7 = - 1^3 du = -
2^3
log
u + -J3
+C

x + ^+l-V3
2

1 V? + 3-^3 1 X
7 = - log +C = - log +C
2^3 Vf + 3 + a/3 2 a/3 2
X + 2 +1 +
X

X —1
EXAMPLE 14 Evaluate: J dx.
(x + vj? + X2 + X
X —1
SOLUTION Let 7 = J dx.
(x-+ 1) V^3 + x_2 + x

ReadYourFlow.COM
INDEFINITE INTEGRALS 19.79

.t2-l
=> I = I (x + 1)2 tJx^ + X 2 +X
dx Multiplying the Nr and D1 by (x + 1)

(*2-l)
=> I = 1 (a-2 + 2x + 1) Jx^ + X 2 +X
dx

1
X
=> I = I x+
1
+2 x+
1
+1
dx [Dividing Nr and D) by a2]

X X

Let x + —+ 1 =f2.Then, x + - + l\ = d (t2)=> 1 1


— dx - It dt
X x x J

w
2t dt 1 1
=> I = 1-j 2I t2 + 1
dt = 2 tan 1 (t) + C = 2 tan 1 x+—+1 +C
x

Flo EXERCISE 19.13

ee
LEVEL-1

Fr
Evaluate thefollowing integrals:
2 7
for
ur
X X -1 1+x
(«2-^)3/2
dx
2-I (a/ 2 -x2y) dx 3. j cos { 2cot
1-x
dx
ks
Yo

1
M 5.J
oo

dx 2 dx
(x2 + 2x +10)
eB

ANSWERS
s 2
r

x . --[X 1 X
+c
ou

1. — sin 2. +C 3. —+ C
ad

8a (a -x2,)
o 2 , 2 2
Y

i (,2+d3/2 + C 1 -1 x +1 3 (x + 1)
nd
Re

4. 5. tan +C
3 3 54 3 x2 + 2x + 10
x
Fi

19.10 SOME SPECIAL INTEGRALS


Let us discuss problems on evaluation of integrals by making above substitutions.
THEOREM For any constant a, prove thefollowing:
1 1 -if x
y dx = - tan - +C (U) 1 2~ 2dX = V-]°8 +C
x + a1 a a J x2 _ 2a x +a

1 1 a+x 1 -1 x
(“> 1 ~2 2 dx =
2a
log
a-x
+C (iv) J 2 2
dx - sin
a)
+C
a -x a -x
1 = log | X + -Jfl2 + X 21 ~ dx = log | x + yjx2 - a2 \ + C
+ C (vOj-pi
(v) hia + x 2 dx x -a
1 2
PROOF (i) LetI=j -j 2 dx. Putting x = a tan 0 and dx = a sec Q d Q , we get
x +a

ReadYourFlow.COM

19.80 MATHEMATICS-XII

a sec2 0 rf 0
l = \ = - f 1-^0 = - 0 + C
a1 + a2 tan2 0 aJ a

=>
-1 ^
J = - tan - + C v tan 0 = — => 0 = tan -1 ^
a a a a
Similarly by making substitution x = a cot we get
1 -l x
I 2
a +x 1
dx = - cot
a a
+C

(ii) Clearly,
1 1 1 1 1
2
x -a 2 (x - a) (x + a) 2a x -a x+a

IV t*

w
I =
x -a

l = ijU
1
=>
2a J x - a x + fl
• dx

Flo
ee
— { J Ji-dx-l 1

Fr
=> / = dx •
2a ^ x -a •’ x + a
for
ur
1 x-a
=> I = ^-|log|x-fl|-log|x + fl|J + C = — log ------ + C
2a x+a
ks

(iii) Clearly,
Yo
oo

1 1 1 1 1
eB

2
a -x 2 (a - x) (fl + x) 2a a+x a-x
1
/ 2 dx
r

a -x
ou
ad
Y

=> 7 = —J f dx+\ -1 dx ■
2a J a+x J a -x
nd
Re

1 + + C = ^l0g a + x + C
Fi

=>
a -x

(iv) Let I =J -—- dx. Putting x = a sin 0 and dx = a cos 0 dQ , we get


P -x
a cos 0 0
/ = J (~2 2 • 2Q
Jfl -a sm 0
. -1 x v x = fl sin 0 => sin 0 = - => 0 = sin 1 -
=> I = J 1-tf 0 = 0 + C = sin +C
a a a
Similarly, by making substitution x = a cos 0, we get
/
1 -l x
1 2
a -x
2
dx = cos
yaj
+C

(v) Let J = J 1 2
- dx. Putting x = a tan 0 and dx=a sec 0 dQ, we get
a2 + x

ReadYourFlow.COM
INDEFINITE INTEGRALS 19.81

1
I = f Ja + a 2 tan
2 . 2 Q0
a sec2 0 rf0 = j sec 0 d0

=> I = log |sec 0 + tan 01 + C = log | tan 0 + -Jl + tan2 0 | + C

=> /r = 1log -
X
+ +C v tan 0=-
a a

=> I = log | x + yja2 + x 2 | - log fl + C


/ 2 2
=> I = log | x + + x | + Cj, where = C - log a
1
(vi) Let / = J = dx. Putting x = a sec 0 and dx = a sec 0 tan 0 d 0 , we get
-a
1

w
I =J yja2 sec2 0 - a2
a sec 0 tan d d 6

Flo
=> I = J sec 0 rf 0 = log | sec 0 + tan 01 + C

ree
=^> / = log | sec 0 + ,/sec2 0 -1 | + C = log — + v sec 0 = —
V n n a

F
=> I = log | x + -Jx2 - a2 \- log a + C or
ur
sf
I 2 2
=> I = log|x + Jx -a l+Cj^hereC! = C - log a.
k
Yo
oo

ILLUSTRATIVE EXAMPLES
B

LEVEL-1
re
ou
ad

EXAMPLE 1 Evaluate:
1
Y

(Oj 2 dx (ii) | —dx


9x2 -4
(iii) J ———2dx
4 + 9x 16-9x
nd
Re

SOLUTION (i) Let/ = J 1


- dx. Then,
Fi

4 + 9x
1 1
4
-dx= -9 Jf 2
2 (2/3)2+x
+X
9
1 1 -1 x 1 -if 3x^
=> I = -x tan + C = — tan +C
9 (2/3) 2/3 6 2
(ii) Let / = | 1 dx. Then,
9x2 -4
2
x----
/= 1
If__ 1 2 dx = — X-- ----- log 1 3x - 2
9 J x2 -(2/3) 9 2x - x+—
3
i+c=hlos
3
3x + 2
+C

(iii) Let / = J —-—y dx . Then,


16 - 9xz

ReadYourFlow.COM
19.82 MATHEMATICS-XII

1 dx = —\ 1
dx
2 9J 4 2 2
9 -x
1,3
4
—+x 4+ 3x
1
^K\x ios
=> —x 3 +c
4 +C = log
9 4 ------ A'
24 4-3a:
2
3

EXAMPLE 2 Evaluate:
1 1
(i)} rfx [NCERT] (ii) J dx
^9 - 25.r2 -Jl6?+25
1
(iii) j dx
I
-9

w
SOLUTION (i) Let / = J 1
dx. Then,
^9-25a2

Flo
ee
1 1 1 A 1 . -if5a'
sin -1
/
4i dx dx = +C - sin +C

Fr
9 2 3^2 2 5
\ 3/5 5 3
-----A
25 -A
5 for
ur
1
(ii) Let / = J dx . Then,
JlSx2 + 25
ks
Yo
oo

1 1 2 5^2 4a + yW2 + 25
i=if dx = — log A + JA + + C - 7 l°g +C
B

4J S'!2 4 4 4 4
2
re

A +
4
ou
ad

1
=> / =- log 4a + -Ji6a2 + 25 -—log4 + C =-log 4a + V16a2 + 25 +0^,
Y

4 4 4
1
nd

where C| = — log 4 + C
Re

4
Fi

(iii) Let 7 =| 1
dx. Then,
-9
1 1 1
7 = if
2J
A
2 9
dx
2 1 2 3l2
dx

4 X
2

=> I = ilog x + Jx2


3^ 2
+ C = i log a + lx2 - - + C = ^log 2a + \/4a2 + 9 + C
2 2 2 4 2 2

1
=^> 7 =- log 2a + V4A2 -9 - — log 2 + C = i log 2a + \/4a2 -9 + C 1'
2 2 2
1
where C1 = — log 2 + C
2

ReadYourFlow.COM
INDEFINITE INTEGRALS 19.83

LEVEL-2
4
EXAMPLE 3 Evaluate: f dx
x2 +1
4
SOLUTION Let /
=i * x2 + l
dx . Then,

4 -1 + 1
I
J- *2 + i dx

■4-i 3
1 1
=> I =
I-,2+i x2+l
dx = J (x2-l)dx+ j
x2 +1
dx = ----- x + tan 1 x + C
3
EXERCISE 19.14

w
LEVEL-1

Evaluate the following integrals:


1. f ------ ------ dx
J a2-b2x2
2-| 1
2 2, 2 dx

Flo 1
2 2, 2 dx

ee
a x -b a x +b

Fr
1 1
4. f ^-^-dx 5.J dx [NCERT] 6. | dx
x +4 ■Jl + 4x2 for ^a2+b2 x2
ur
1 1 1
7.
I ^77 dx 8.
J J(2-*)2 + l
dx
9-l fl-x)2-!
dx
ks
Yo
oo

LEVEL-2
eB

x4 + 1
10.
I x2+l
dx [CBSE2002C]
r
ou
ad

ANSWERS
Y

a + bx ax -b
1. V lo8 +c 2. Vlog +C
nd

lab a -bx 2 a£> ax +


Re
Fi

1 -1 ax A 5 f ~l X
3. — tan +C 4. x---- tan +C
ab b 2 2
5. log | 2x + ^4x2 + 1 | + C 6. - log bx + Ja2 + b2 x2 + C
b v
_7. -1 sin
. -i f fcx
— +C 8. - log (2 -x) + -J(2 -x)2 + 1 +C
b a
3
9. -log l-x + y/il-x)2 -1 +C -1
10. — - x + 2 tan x + C_
3
HINTS TO NCERT & SELECTED PROBLEMS

ReadYourFlow.COM
19.84 MATHEMATICS-XII

1 1 = d{l-x) =-log (2-x) + ^2-x)2+1 +C


8. j = -!
Jv-xf + l ^2-x)2 + 1

1 1
9-l rfx = - J = rf(2-x)=-log 2-x + ^(2-x)2 -1 +C
l/(2-x)2-l2 JV-x)2-!
1
19.10.1 EVALUATION OF INTEGRALS OF THE TYPE J
flX2 + fcx + c
2
To evaluate this type of integrals we express ax +bx + c as the sum or difference of two
squares by using the following algorithm.
ALGORITHM
2 ... .
STEP I Make the coefficient ofx unity, if it is not, by multiplying and dividing by it.

w
2
STEP II Add and subtract the square of the half of coefficient ofx to express ax +bx + c in theform a
b f Aac-b2
x+— +
2a 4a2

Flo
ee
STEP HI Use the suitable formula from thefollozuing formulas:

Fr
1 -1 1 1 a+x
+C
a + x1
7T dx = - tan
a aJ f +c' 1 V
a -x
-dx = - log
a -x
for
ur
r 1 , 1 . x-a
^ 2a l0g TTa +C'
ks
Yo
oo

ILLUSTRATIVE EXAMPLES
B
re

LEVEL-1
ou

EXAMPLE 1 Evaluate
ad

1
tt jJ -x2-x
y1 (ii) |
Y

dx dx
+l 2x2 + x-l
nd
Re

(iii) \ ------ r- dx
3 + 2x -x2
Fi

SOLUTION (i) Let 7 = J 1


dx. Then,
x -x +1
1
dx
1 1
X — X +------ + 1i
4 4
1
=> I
=1 (x -1/2)2 + 3/4
dx

1 -1 x -12 , _ 2 -if 2x-r


7 = 1 (x-1/2)2+(V3/2) 2
tt dx =
V3/2
tan
V3/2 /
+ C = -7= tan —r=— + C
73 ^3 ,
1
(ii) Let 7 = J dx . Then,
2x2 + x -1

7 = -2 Jf ---- ----T dx
2 x 1
x +------
2 2

ReadYourFlow.COM
INDEFINITE INTEGRALS 19.85

1
/ =if
2J
dx
x2 +x/2 + (l/4)2 ~{l/4)2-1/2
1
=> i = lf
2 J
2 dx
(at +1/4)2 -(3/4)

1 1 x +1/4-3/4 x -1/2 + C=ilog 2x -1


=> / —— x log + c = 4 log +C
2 2(3/4) X + 1/4+ 3/4 3 x +1 3 2 (x +1)
1
(iii) Let J = J 2 dx. Then,
3 + 2x - x
1
I =
1 (x2 - 2x - 3)
dx

1
=> I
=1 (x2 -2x + l -1 -3)
dx

w
1
=> I =
1 |(x-l)2 -22 j
dx

Flo
ee
1 2 + (x -1) 1 X+l
I
I 2 dx = 7^log + C = — log +C

Fr
22-(x-1) 2(2) 2 - (x -1) 4 3-x

EXAMPLE 2 Evaluate:
for
ur
1 1
(0 j 3x2 +13x-10
dx [NCERT] (ii) } 4x2 - 4x + 3
dx
ks
Yo

1 1
(iii) j [CBSE 2002, 2017] (iv) |
oo

x2 + 4x + 8 9x2 + 6x + 10
B

1
re

SOLUTION (i) Let / = J dx. Then,


3x2 +13X-10
ou
ad

1 1
dx
Y

3 13 10
x + — x-----
3 3
nd
Re

1 1
3 1 2 13 fl3)2 13 ^ 2
dx
Fi

10
X +— X+
3 6 6 3
1
=> / = —3 J[ 2 dx
13^12 17
x+
6 6
13 _17
x+
1 1 6 6 + C = 7^ log x -4/6 3x - 2
=> / = —x log
13 17
+ c = i log +c
3 x + ------ 1------- 17 x +5 17 3 (x + 5)
6 6
1
(ii) Let / = | dx. Then,
4x2 - 4x + 3
1 1 1
If _
I =
4J x2 -x +3/4
dx
1 1
X - X +------- +
3
dx -
if 1^
1
2 dx

4 4 4 x— +
2 V2

ReadYourFlow.COM
19.86 MATHEMATICS-XII

1 1 -if x-V2 1 -l 2x-l


=> tan +C = tail +c
4 X (1/V2) 1/V2 2V2 V2
1
(iii) Let 7 = J dx. Then,
x2 + 4x +8
1 1 -if x + 2
1=1 7
x + 4x + 4 + 4
dx
= f (x + 2)2 + 2 2 dx = —2 tan +C

1
(iv) Let 7 = J dx. Then,
9x2 + 6x + 10

7 = I9 Jr___i___
2 2 10
dx = -9 fJ ----
2 2
«--- -1 1 10
dx
x + x +-------+ —
3 9 3 9 9 9
1

w
x+—
7 =
1 ^ = —
dx 1 x - tan -1 3 + C = 1 tail -if 3x +1 +C
n2 +12 9 1 1 9 3

Flo
X+—
3
EXERCISE 19.15

ee
Fr
LEVEL-1

Evaluate the following integrals: for


ur
1 1
1.
J dx 2. f ___ 1___ dx 3.
I 2 dx
4x2 + 12x + 5 J x2 -lOx + 34 1 + x -x
ks

1
1
5.J
Yo

4.
I dx dx [NCERT]
oo

2x2 — x — 1 x2 + 6x + 13
eB

ANSWERS
1 2x +1 2. I tan"1 x -5 'l 1 V5 -1 + 2x
3.
r

1. log +C +C log +C
ou

8 2x + 5 3 3 J V5 \/5 + 1 - 2x
ad
Y

1 x —1 1 -1 f x + 3
4. - log +C 5. — tan ------- + C
3 2x +1 2 2
nd
Re

HINTS TO NCERT & SELECTED PROBLEMS


Fi

1 1 1 lf x + 3
5.
1 x + 6x + 13
dx =
I (x + 3)2 + 22
dx =
2
tan
2
+C

1
19.10.2 INTEGRALS REDUCIBLE TO THE FORM J dx
ax 2 + bx + c
Following examples will illustrate the procedure of evaluating the above type of integrals.

ILLUSTRATIVE EXAMPLES

LEVEL-1
EXAMPLE 1 Evaluate:
x
e
wJ x-4-2 . dx (ii) J e2x + 6ex + 5
dx
+x +1
sm x 2*3
(iii) | 2 dx (iv) J 8
dx
1 + cos X 4+x

ReadYourFlow.COM
INDEFINITE INTEGRALS 19.87

x X
SOLUTION (i) Let / = J rfx = j dx
x4 + x2 + 1 (x2)2 + x2 + 1
79 dt
Let x =t. Then, d (x ) = dt => 2x dx = dt=> dx = —
2x
x dt
I x----
t2 +t + l 2x
1
=> / = —2 Jf dt
t2 +t + l
1
=>
2J
t+ n2 + V3
2 2

1
t+ 1 2xz2 +1

w
i=l 1 -1 ? ^ 1 -i 2f +1 ^ 1
=> “ x —7=
tan ^ + C - -7= tan —^=— + C = —f= tan +C
2 ^3 V3 V3 v V3 , V3 V3
2 J

Flo
2
e eX

ee
(ii) Let f = J dx = f dx
e2x + 6ex + 5 (ex)2 + 6ex+5

Fr
Let ex = t.Then,d (ex) = dt ex dx = dt for
ur
I =
dt 1 ^log t+3-2 + C=ilog ex + 1 + C
2dt =
t2 + 6t +5 (t + 3)2 - 2 2x2 t+3 + 2 4 ex +5
ks
Yo
oo

sin x
(iii) Let / = J 2 dx
eB

1 + COS X
Let cos x = t Then, d (cos x) - dt=> - sin x dx = dt => dx = - dt sin x
sin x
r

dt =-f —^ rft =-tan-1 (0+C =-tan-1 (cos x) + C


ou
ad

sin x J i+r
Y

2x3 2.T3
(iv) 1 = J dx = I dx
nd
Re

8 22 +(x4)
4+x
Fi

Let x4 = t. Then, d (x4) = rff => 4 x3 rfx = dt=>dx = dt


4 x3
2x3 _1 ^ 4
1 1 1 x
= — x — tan -if t + C = tan -1
4 + t2 4x3 2 1 -
22 +.f 2 2 2 2J 4 2
+C

EXAMPLE 2 Evaluate:
-x
1 c
(i)j x {6 (log x)2 + 7 log x + 2}
dx (ii) j -2x
dx
16 + 9e
1
SOLUTION (i) Let 7 = J dx
x {6 (log x)2 + 7 log x + 2}
Let log x = t. Then, d (log x) = df => — dx = dt=> dx = x dt
x
1
I = dt
672 +77 + 2

ReadYourFlow.COM
19.88 MATHEMATICS-XII

=> / = -6 f ----
t1*7- 1
dt
t+
6 3
1
=> dt
7 4 +1. _ 49
t+—
12 J + 3 144

=> I
41 7V
t+—

12 12

1 2^ + 1 2 log x + 1
—x log ----——32. +c = log + C = log +C
6 7 3£ + 2 3 log x + 2
f+ +
12 12

w
-X -x
e e
(ii) Let / = J -2x
rfx = j* dx
16 + 9e 42 + (3e x)2

Let 3e_ ^ = t. Then, (3e_ ;'r) = dt => - 3e x dx = dt => dx = -

Flo
ee
3e x

Fr
—x dt
Putting 3e = t and dx = - —, we get
3e x for
ur
-X dt ) = _lr dt
I = = -\! i~2dt
16+ t2 3e x 3 16 +12 (4) +t
ks
Yo
oo

1 tan
. -if 34 + C =-----1 tan
. -l 3e ^
=> IT =----1 x -1 tan
. -iff + C =----- +c
4 12 4
B

3 4 12
re

EXAMPLE 3 Evaluate:
1
(i)J
ou

------ dx [CBSE2000C] (ii) | dx


ad

xix11 + 1) X (x5 + 1)
Y

SOLUTION We have.
n-1
nd
Re

1
W dx =
J f~ « / « , i\
dx
Fi

x (x" +1) x (x +1)


n-1 dt
Let x” + 1 = t. Then, d (xn +1) = dt=> nx dx = dt=> dx =
n-1
nx
1
I = Jf ^
nxnt
dt = -\
ni (t-l)t
dt [v x” + l=f xn =t-l]

dt 1
=> 1 = iJ -T-dt = -\ t2-t + l/A-l/A (t -1/2)2 -(1/2)
2dt
nJtA-t nJ

1 1 t -1/2 -1/2 + C = i log + C = - log — + C


I =-x log
n 2(1/2) t -1/2 +1/2 n &x%l
4
1 X
(ii) /
=J x (x5 +1)
dx =
1 x5 (x5 +1)
dx

dt
Let x5 +1 = t. Then, d (x5 +1) = dt=> 5x4 dx = dt=> dx =
5x4

ReadYourFlow.COM
INDEFINITE INTEGRALS 19.89

1
/ dt = -\ dt
5> tx5 5J f(f-l) 5J - t
i i
=>
;4i t2 -f+ 1/4-1/4 (£-l/2)2-(l/2)2

t -1/2 -1/2 + C=Ilog £ -1


=> / =-x "TT^ lo8 + C = 1r 1l08 ~ X +C
5 2(1/2) £ -1/2 + 1/2 £ 5 r3 +

LEVEL-2
sin.v + cos.v
EXAMPLE 4 Evaluate: J dx
9 + 16 sin 2x
SOLUTION We observe that sin .t + cos x occurs in the derivative of - cos x + sin x. So, we express
9 +16 sin 2x in terms of -cosx + sin x as follows.
2 2
(-cosx + sinx) =1 - sin2x or, sin 2x =1-(-cos x + sin x)

w
2
9+ 16 sin 2x = 9 + 16 |l -(cosx + sinx)2 j =25-|4 (-cosx +sinx)j

Flo
smx + cosx smx + cosx
Thus, / = J dx = J y dx
9+ 16 sin 2x
25- 14 (-cosx +sinx)|

ee
Fr
Let 4 (-cosx + sin x) = £. Then, d {4 (-cosx + sin x)} = dt or, 4 (sin x + cosx) dx = dt
dt
or. dx =
for
ur
4 (sin x + cosx)
sinx + cosx dt
X -------------------------------
ks

25-i2 4 (sin x +cosx)


Yo
oo

1 1 5+f 5 +(-cosx + sinx)


=> 2dt = x — log + C=TTlog +C
eB

4 J 25-£ 4 10 6 5 -£ 40 5-(-cosx + sinx)


1
EXAMPLES Evaluate: J dx
r

sinx + secx
ou
ad

1
SOLUTION Let / = J dx. Then,
Y

sin x + secx
cos x 2 cos x
dx = | dx
nd
Re

1 + sinx cosx 2 + 2sinxcosx


(cos x + sin x) + (cos x - sin x) ^
Fi

=>
2 + 2sinx cosx
cosx + sinx cos x - sm x
=> dx +J dx
2 + 2sinx cosx 2 + 2sinx cosx
cosx + sinx cosx-sinx
=> dx + J dx
3-(1 -2sinx cosx) 1 +(1 +2sinx cosx)
(cosx + sinx) cos x - sin x
=>
^=1 3-(sinx-cosx)
2 dx +J
1 + (sin x + cos x)
y dx

\-----2du + \ 1
=> 2 dv, where u = sin x - cos x and v = sin x + cos x
(V3)2 -u2 1 +u

=> I= ^ log j3+u + tan -1 u + C


2^3 y[3 -U

1 -/3 + (sinx-cosx) -l
=> /= log + tan (sin x + cos x) + C
2^3 y[3 - (sinx-cosx)

ReadYourFlow.COM
19.90 MATHEMATICS-XII

EXERCISE 19.16
LEVEL-1
Evaluate the following integrals:
2 x
sec e cos x
~dx
1 - tan2 x 2-l 1+e
2x
dx 3. | 2
sin x + 4 sin x + 5
dx

A' 3 .r
e
M e2x +5 ex +6
dx 5. |
4e
6 -v
-9
dx 6. [ ------------ dx
ex + e x
x 3x5
7‘1 dx 8‘ 1 dx 0Mr -6—6*
w
x4 + 2x2 + 3 12
1+x x -a
1
10-I ~ir~6 * 11.1 dx 4 *2 7 dx [CBSE 2007]
X + x(x6 +1)

w
x
x e
13. j dx 14. J dx [NCERT]
3x4 -18x2 +11 (1 + ex) (2 + ex)

Flo
LEVEL-2

ee
1

Fr
15. j dx
cos x + cosec x
for ANSWERS
ur
1 + tan x x
-1 e
1- l log -1
ks

+C 2. tan +C 3. tan (sin x + 2) + C


2 1 tan x 1
Yo

I
oo

3x
eB

e* + 2 ^7log 2e -3
4. log +C 5. +c 6. tan 1 (ex) + C
/+3 36 2c3;f + 3
r

2
ou

3
ad

1 l xz+l 1 . -1,6.^ 1 1 ^
7. tan +C 8. — tan (x ) + C 9. TT10^ “3
Y

2 V2 V2 2 6a x +
nd

3 6
Re

1 -1 -1 2x2 -1
10. 3 tan % +c 11- 7 log ^ +c 12. 4= tan +c
V3 V3
Fi

3 fl a 6 x6 +1

x2 - 3 - 4
13. ^ bg — V3 + C l+ex
14. log +C
48 6 2 4 2 + e*
x -3+
V3
1 V3 + sinx-cosx -1
15. log - tan (sin x + cos x) + C
2^3 •/3 - sin x + cos x
HINTS TO NCERT & SELECTED PROBLEMS
X
e
14. Let / = J dx. Let e* =f. Then, ex dx = dt
(l + ex) (2 + ex)
1
I (1 + 0 (2 + 0 1
(f2 + 3t + 2)
*=1 1
\2
(1
2*
3
t+
2 12

ReadYourFlow.COM
INDEFINITE INTEGRALS 19.91

3 1
t+ ex+l
==> I = lo§ =log + C = log +C
2x t H-----1--- ex + 2
2 2 2
1
19.10.3 INTEGRALS OF THE TYPE j dx
Jax2 + bx + c
In order to evaluate this type of integrals, we may use the following algorithm.
ALGORITHM
2
STEP I Make the coefficient ofx unity, if it is not.
STEP II Find half of the coefficient ofx.
(if2
STEP III Add and subtract ^—Coeff. of x inside the square root to express the quantity inside the

w
b ^|2 4ac -b2 4ac -b2 \2
b
square root in the form x + — + or, x+— .
2a 4«2 4fl2 2a;
STEP IV Use the suitableformula from thefollowing formulas:

Flo
ree
f , 1 dx = log x + yja2 + x2 + C, J I — = dx = log x + yjx2 -a2 +C
J ^

F
Jx2-a

j i
or
ur
dx = sin ^ +c
J 12
a -x 2 a
f
ks
Yo

ILLUSTRATIVE EXAMPLES
oo
B

LEVEL-1
re

EXAMPLE 1 Evaluate:
1 1
(i)J [NCERT] (ii) j dx
ou

dx
ad

p-l)(x-2) J9 + 8X- x2
Y

1 1
(iii) j dx (iv) J dx
nd

fx2 + 3x-2
Re

^jx (1 - 2x)
Fi

1
SOLUTION (i) Let Z = | dx. Then,
^-D (*-2)
1 1 1
^ -Jx2 -3x+ 2 dx =
In x - 3x +
9 9
+2
dx
=i 3V fi)2
dx

4 4 x—
2 U
3
=> / = log x— x — 3x + 2 + C
2

(ii) Let / = j 1
dx. Then,
j9 + 8x-x2
1
I = I x2 -8x-9
dx

ReadYourFlow.COM
19.92 MATHEMATICS-XII

1
=> I = I ^|-^2 -8a:+ 16-25 j
dx

1 1 'x-4 N
=> I = 1 dx = J dx = sin -1 + C.
J-ix-if-s2} ^52-(x-4)2

(iii) Let / = J 1
dx. Then,
Jx (1 - 2x)
1
'=1 yjx-lx2
dx

i 1
=> 1 dx = -4= f dx
V2 J
2 X
X -- + n2 n2 X-----
n2 n2

w
2 4 4j
1 4 4J
1 , 1 . _i x -1/4 + C = -4= sin

Flo
=> . -1
1 =V2'f dx = —?= sin (4x -1) + C.
O2 n2 V2 1/4 V2

ree
x—
4 4

F
(iv) Let J = J 1
dx. Then,
■\j2x2 + 3x - 2 or
ur
sf
1 1 1 1
V2 J V2 1
I = -~ rfx = rfx
2 3
k

312
Yo

X + - X-l
X+— -1
oo

2
4 16
B

1 3 ? 3
4r log
re

=> 7 =V2^ dx = x+— + x2+-x-l +C.


s^l2 fs^i2 V2 4 2
x+—
ou
ad

4
Y

EXAMPLE 2 Evaluate:
1 1
nd

(i)|
Re

dx [CBSE2001C] (ii) | dx
■j(x-d) (x -b) Jx2 - 4x + 2
Fi

SOLUTION (i) Let / = J 1


dx. Then,
J(x-a) (x-b)
1
I = J Jx2 -x(a + b) + ab
dx

1
=> / =1 (\2 / \2
dx
x2 - x(a + b) + a + b a+b
+ ab
2 2
1
/ = J 2
a-b)2
dx
a+b
x-
2 2

ReadYourFlow.COM
INDEFINITE INTEGRALS 19.93

2
a +b a+b a -b
=> I = log x- + J .r - +C
2 2 2

2x - a - b
=> I = log + yl{x-a){x-b) + C
2

(x - a) +(x -b) + 2 ^(x - a) (x - b)


=> I = log +C
2

=> I = lo§ -a + - log 2 + C

w
=> / = 2 log l^x - a + yjx -b + C, where C-f = C - log 2

(ii) Let / = J 1
dx. Then,
yx2 - 4x + 2

Flo
ree
1 1
I = 1 Jx2 -4x+ 4- 4+ 2
rfx = |
y/iX-I)2 -(V2)2
dx

F
log (x - 2) +-J(x - 2)2 -(y^)2 + C = log x-2 + f2 - 4x + 2 +C or
ur
=> / =
k sf

EXERCISE 19.17
Yo
oo

LEVEL-1
B

Evaluate the following integrals:


re

1 1
dx 2-l dx [NCERT]
ou
ad

^2x - x2 J8 + 3x - x2
Y

1 1
3. j = dx
2
[CBSE 2009] 4J dx
nd

•y 3x2 + 5x + 7
Re

^5 - 4x - 2x
Fi

1 1
5. J ^(x - a) (P - x)
dx, (P > a) 6.{ dx
^/7 - 3x - 2x2

1 1
7-l -Jl6 - 6x -x2
dx 8.}
■f? -6x - x
= dx [NCERT, CBSE 2002]

1
9-S ^5? - 2x
dx [NCERT]

ANSWERS
_l I 2x - 3
1. sin -l (x -1) + C 2. sin +C
V4i
1 . -1 - (x + 1) i + C 1 , 5 1 2 5 7 r
3. -r= sin 4. -t= log X + - + J x + —x + — + C
V2 7 # 6 6 \ 3 3

ReadYourFlow.COM
19.94 MATHEMATICS-XII

,6. 1 . _i 4x + 3
—sin +C
i V65
x+ 3
8. sin -1 +C
4

+C

H/A/7S 70 NCERT& SELECTED PROBLEMS


1 1 1
2. /
=1 \]8+ 3x-x2
clx = J
J- (x2-3x-8)
rfx = J
x2 - 3x + 9 9
dx
-8
4 4

w
1 1
=> 7=
J dx = f dx
\2
sf fViT N2
V4lf -

Flo
3
X— x—
2 2 2 2
I

F ree
3
x—
-1 -l 2x- 3
=> / = sin + C = sin or
ur
ViT
sf
2
1 1 1
=1 rfx = J =1
k

8. I dx dx
Yo
oo

^7 - 6x - x2 J-(x2 + 6x-7) J-{(x + 3)2-42I


B

1 . -\( x +3
- M dx = sin +C
re

■J42 ~(x+ 3)2 4


ou
ad

1 1 1 \ 1
9. I
= 1 -J5? - 2x dX ~ 1 dx = —?= dx
Y

X
2 2
-- X
V5 1
\2
if
5 x—
5 5J
nd
Re

1 1 2 2
Fi

=> 7 = -^= log x — + lx — x + C


5y 5
1
19.10.4 INTEGRALS REDUCIBLE TO THE FORM J dx
■J ax2 + bx+ c

Following examples will illustrate the procedure of evaluating this type of integrals:
ILLUSTRATIVE EXAMPLES
LEVEL-1
EXAMPLE 1 Evaluate:
ex
wj dx
2x
(ii) J dx

sec 2 x 1
(iii) J dx [NCERT] (iv) J dx
Jl6 + tan2 x xJilogxf-S

ReadYourFlow.COM
INDEFINITE INTEGRALS 19.95

.v x
e e
SOLUTION (i) Let / = J dx - j dx
2x
^2-(0
Let - t. Then, (e'v) - dt => ex dx = dt => dx = X
e
x \
_ | dt . -1 0+C = sin -1
e
I = 1 * = sin
2 2
+C

2 2
X x
(ii) Let / = J
yjl-x6
dx =
I dx

Let x3 = t. Then, rf(x3) = dt => 3x2 dx = dt^> dx = dt


3x2

w
1 r dt 1 . -1
/ = — sin (f) + C = ^ sin (x3) + C
3J 3

(iii) Let, I = \
2
sec x = dx = [ sec x
dx.

Flo
2

ee
2 J 1J42 + tan2 x
16 + tan x

Fr
2 dt
Let tan x = t. Then, d (tan x) =dt=> sec x dx = dt => dx = —^— for
ur
sec x
_ j- dt = log t + -J42 + t2 + C = log tan x + ^16 + tan 2 x + C
I = 1 Ju"+ t 2
ks

J
Yo
oo

1
(iv) Let I = J
B

dx
Xyjilogx)2 -5
re

1
Let log x = L Then, d (log x) = dt => — dx = dt dx = x dt
ou
ad

x
Y

1 ~ = log 11 + -\Jt2 - 5 | + C = log log x +-Joogx)2 -5 + C


d
Re
n
Fi

EXAMPLE 2 Evaluate:
x 2x
a
(i)J dx (ii) | dx [CBSE2005]
2x Jl-x2 -x4
x COS X
t’
(iii) | dx [CBSE 2009] (iv) J dx
]/5-4ex -e 2x Jsin2 x - 2 sin x - 3

x sin 2x cos 2x
(v) { 3 * [CBSE 2016] (vi) | dx
3 ■J9 - cos4 2x
a -x
X v
a a
SOLUTION (i) Let J = J
2x
dx = 1 dx.

dt
Let ax = LThen, d(rt'x) = dt=>ax loge a dx = dt=>dx -
ax logt, a

ReadYourFlow.COM
19.96 MATHEMATICS-XII

x
a dt 1_ f dt 1 1 sin 1 (ax) + C
x sin -1 (f) + C =
ax log a log a •' Jl2-t2 losa logfl

2x 2x
(ii) Let / = J dx = j dx.
4 ijl -X2 -(x2)
-X

Let x2 = f. Then, d (x2) = dt=^2xdx - dt=>dx =


2x
1 1 1
I
1 = dt= \
2 J ^-(t2+t-l)
dt =
1 t2 +t + 1 _2 -1
dt

4 4
1 1 1
=> i =
I Tdt=j
n2
dt
n2 s 5 V5 n2

w
t+ t+ t+-
4 2
1 2 4 2 2

Flo
. -i ft+ 1/2 f 2 ^
1 2f +1 . -i 2a- +1
I sin + C = sin + C = sin +C
v V5/2 l V5 V5

ree
.r .v
e e

F
(iii) Let 1 = J
t/5-4^ 2a
dx =
J dx
or
ur
f
Let <?A = t. Then, d (ex) - dt => ex dx = dt => dx = ~
X
ks

e
=^=1
Yo

1 1 1
M
oo

dt
2 J ^-(f2 + 4f-5)
f {(i + 2)2-32}
B
re

1 . -1 f+ 2 . -1 e* + 2
=> rff = sin + C = sin +C
^32-(f + 2)2 3 3
ou
ad
Y

COS X
(iv) Let / = J dx
Jsin2 x - 2 sin x - 3
nd
Re

dt
Let sin x = f. Then, rf (sin x) = dt=> cos x dx = dt => dx =
Fi

cos x
dt dt dt
I
=1 2t - 3
I -2f+ 1-1-3
1 -J((-l)2-22
=> 1 = log | (t -1) + ^jit -l)2 -22 | + C

=> / = log | f -1 + -Jt2 -2t - 3 | + C = log (sin x -1) + ^sin2 x - 2 sin x - 3 + C

y/x
(v) Let I
=1 dx = J dx.
»3/2)2H ^3/2f
3/2 3/2 2
Let x = t. Then, (x ) = dt=> -x1/2 dx = dt=> dx = dt
2 3y[x

ReadYourFlow.COM
INDEFINITE INTEGRALS 19.97

(r3/2)
1 r 2.-1
1
=!i dt - — sin 1 + C = — sin
7/2 + c
a3'2}2-? 3 U3/2
VM /
3

sin 2x cos 2x sin 2x cos 2x


(vi) Let J = J dx. Then, / = j" dx
-J9 - cos4 2x J32 - (cos2 2x)2
j
Putting cos 2x = t and - 4 sin 2* cos 2x dx = dt, we get

/ =
-u ^2-‘2
1 dt = -— sin 1
4

LEVEL-2
t
3
+ C = — sin
1
4
-1
2
cos 2x
3
+c

w
EXAMPLE 3 Evaluate:
1
(i) J Jsec x -1 dx (ii) j dx
ft 2x
r-e
SOLUTION (i) Let 7 = J Jsec x -1 dx =
Flo 1 - COS X
dx. Then,

ee
cos x

Fr
2
'(1 - cos x) ^ (1 + cos x) ^ _ r 1 - cos x sin x
=> I 2-dX = \
for dx
ur
cos x (1 + cos x) •'
dt
COS X + COS X
7 2
COS X + COS X

Let cos x = t. Then, (cos x) = df=>-sin xdx = dt => dx = -


ks

sin x
Yo
oo

-dt dt
I
=1 = -!
B

t+-
n2 if
re

2 y 2
ou
ad

+C
Y

=> I = - log
nd
Re

1
=> I = - log = - log COS x + — + y cos2 X + COS X +C
Fi

2 ,
-X -x
e e
(ii) Let 7 = J dx J -2x
dx - J dx
2x
1- e -1
-2x
e

Let e x =t.Then,d (e x) = dt => -e x dx = dt => dx - - -x


e
7 I rft = - log t + -Jt2 -1 + C = - log e x + -Je -2x -1 +C

Evaluate: J sin(x-a)rfx
EXAMPLE 4
sin (x + a)
sin (x - a) ^
SOLUTION Let 7
sin (x + a)

ReadYourFlow.COM
19.98 MATHEMAT1CS-XII

sin (x - a) sin (a- - a)


=> / = -----:------ - x----- ------- - dx
sin (x + a) sin (x - a)
sin (x - a) ^
=> /
=1 I 2
ysin x -sin a 2

sin x cos a - cos x sin a


'=1 . 2a
yjsin2 x - sm
dx

sm x cos x
=> / = cos a J == dx - sin a J dx
/ ? ? . 2
J1 - cos x -1 + cos" a -Jsin2 x - sm a
sm x cos x
=> / = cos a | == dx - sin a J dx
/ 2
Jcos a-cos x -^sin2 x - sm. 2
a

w
In the first integral we put cos x = t, so that - sin x dx = dt and in the second integral we put
sin x = it, so that cos x dx = du.
/ = - cos a J
dt
^cos9 a -f
= - sin a j

Flo
du
. 2 a
-Ju2 - sm

ee
Fr
. -1 t sin a log u + -Jit2 - sin9 a + C
=> I = - cos a sin
cos a for
ur
. -1 cos x sin a log sin x + Jsin2 x - sin2 a + C
=> I = - cos a sin
ks

cos a
Yo

Evaluate: J sm x + cosx dx
oo

EXAMPLE 5 [NCERT EXAMPLAR]


yjl + sin 2x
B

SOLUTION Let / = J sin x + cosx dx. Here, the integration of the numerator of the integrand is
re

Vl + sin 2x
ou
ad

- cosx + sin x. That is the numerator of the integrand occurs in the derivative of -cosx + sin x.
So, we express 1 + sin 2x in terms of -cosx + sin x. We observe that (-cosx + sin x)2 = 1 - sin 2x.
Y

Therefore, we write
nd
Re

1 + sin 2x = 2-(l - sin 2x) = 2-(sin x-cosx)2


Fi

sin x + cosx
W ^2-(sin x-cosx)2
dx

Let sin x - cos x = t. Then, d (sin x - cos x) = dt or, (cos x + sin x) dx = dt


1 . -1 t
'=1 2-r
1 J(j2)2 -t2
dt = sin
V2
+C

i
/ = sin 1 •! -4=(sinx-cosx) l +C = sin 1-| sin [ x--
=> +C
x/2 4

EXERCISE 19.18
LEVEL-1
Evaluate the following integrals:
x 2 X
sec x e
»
M 4
x +a
4
dx
2-l + tan2 x
dx [NCERT] 3. j
fe-e 2x
dx

ReadYourFlow.COM
INDEFINITE INTEGRALS 19.99

cos x sin x x
4. dx 5.J -J4 cos2 X -1
dx 6. | dx
■J4 + sin2 x

1 sin 8 x cos 2x
dx 8.J dx 9.
-Jsin 2 2x + 8
dx
x ^4-9 (log x)2 ^9 + sin4 4 x
sin 2x sin 2x COS X
10. 1 •^/sin4 x + 4 sin2 x - 2
dx 11. j
tJcos4 x - sin2 x + 2
dx 12.
-J4 - sin 2 x
dx

1 1
13. I x2/3^ -4
dx 14. J
-J(l -x2) {9 +(sin 1 x)2}
dx

LEVEL-2

low
COS X
15. 1 ^sin2 x - 2 sin x - 3
dx 16. | ^cosec x -1 dx

sin x - cos x cos x-sin x


rfx I dx

ee
17. | [CBSE 2011] 18.
^sin 2x x/8 - sin 2x
rF
Fr
for ANSWERS

1. ^ log x2 + -^x4 + fl4 + C 2. log tan x + y4 + tan2 x + C


ou
ks

~i e 4. log sin x + ^4 + sin2 x +C


oo

3. sin +C
4
Y
B

r 2'
re

5. - ^ log 2 cos x + ^ 4 cos 2 x-1 +C 1 .-ix


6. — sin - +C
2 2
ou
ad
Y

1 log sin2 4x + ^9 + sin4 4x + C


8. -
7. +C
3 2
nd
Re

9. log sin 2x + -^sin 2 2x + 8 + C 10. log sin2 x + 2 + ^sin4 x + 4sin2x-2 +C


Fi

cos2 ^ + ^ j + -^/cos4 X + cos2 X + 1 + C


11. - log

'sin x 1/3
12. sin -1 +C 13. 3 log x + -4 +C
2

14. log sin -1' x + -J9 + (sin 1 x)2 + C 15. log (sin x -1) + Jsm2 x - 2 sin x - 3 +C

16. log sin x + ~ j + ^sin2 x + sin x + C 17. - log (sin x + cos x) + .^/siiTlx | + C

-l —(sin
1 x + cosx) 1 + C
18. sin
3

ReadYourFlow.COM
19.104 MATHEMATICS-XII

Let x2 -1. Then, d (x2) = dt or, 2x dx = dt


t
/=if
2J
dt
t2 + 3t + 2
(2t + 3) - 3 2t+ 3 1
=> rff = dt
4 J f2 + 3f + 2 4i f2 + 3f + 2 r+ 3+2
2t+ 3 1
=> /=
il t2 + 3t + 2 4J 3f Tfdt
i+
2 2
3 1
f+
7 = — log | f2 + 3f + 21-— log 2 2 + C = — log | x4 + 3x2 + 21 - 3 log
4 4 3 1 4 4
t+ +-
2 2

w
EXERCISE 19.19
LEVEL-1
Evaluate the following integrals:

M
x
2-l
x+1
Flo 3.J x-3

ree
dx dx dx
x2 + 3x + 2 x2 + x + 3 x2 + 2 x - 4

F
2x -3 x —1 2x
x2 + 6x + 13
dx 5.J dx 6. Jor 2 dx
ur
3x2 - 4x + 3 2 + x -x
sf
1 -3x 2x + 5 C ax3 + bx
7.
1 dx
8-J dx 9- J Y dx
k
Yo

3x2 + 4x + 2 x -x -2 x +c
oo

(3 sin x - 2) cos x
n. j 2x2x++6x2 + 5 dx
B

10. J 2 dx [CBSE 2013,2016] [CBSE 20071


5 - cos x - 4 sin x
re

5x-2 x+5
12.
I 13- j 3x2 +13x-10 dx
ou

2 dx [CBSE 2013,2014] [CBSE 2017]


ad

1 + 2x + 3x
Y

(3 sin x - 2) cos x x+7


14. J dx [CBSE 2017] 45. j dx [CBSE 2017]
13 - cos2 x-7 sin x
nd

3x2 + 25x + 28
Re

LEVEL-2
Fi

3 x3 - 3x
x
16. J dx dx
x4+x2+l 17-J I
x4 + 2x2 -4
___ ANSWERS

1. Ilog|x2 + 3x + 2|-|log x + 1 + C I ? 1 -if 2x +1


2. — log | x + x + 31 + tan +C
x+2 2 ViT ViT
3. ^ log | x2 + 2x - 41 - ~ log x + 1 - Vs
+c
Vs x + 1 + Vs

4. log | x2 + 6x + 131 - ^ tan -1 f x + 3 + C


2

5. - log | 3x2 - 4x + 31 - — tan -1 3x - 2 + C


6 15 Vs

ReadYourFlow.COM
INDEFINITE INTEGRALS 19.105

6. - log | 2 + x - x2 | + | log 1 +x + C
2 -x

7. - ^ log | 3x2 + 4x + 21 + ~ tan -1 3x + 2 + C


V2
^ 2^
x
8. log | x2 - x - 21 + 2 log x - 2 + C « ,
9. - log iI x 4 +c21| + —
b
tan -1 +C
x +1 4 2c c

4
10. 3 log | 2 - sin x | + +C
2 - sin x

11. ^ log (2x2 + 6x + 5) + ^ tan 1 (2x + 3) + C

w
5 ? 11 -1 3x +1
12. -log 3x2 + 2x +1 - —t= tan r- +C
6 3^2 V2
13. — log 13x2 + 13x —10| + — log 3x-2 + C
6 6 3 (x + 5)

Flo
ee
Fr
14. 10 log (4-sinx) -7 log (3-sinx)+ C
15. —log | 3x + 4| + C for
ur
3
9
16. ilog|x4+x2+l|--^tan -1 2x^+1 + C
ks

V3
Yo
oo

17. ^log| x4 + 2x2 -4|—JL log x2 +1


B

+C
x2+1 + V5
re

P(x)
ou
ad

19.10.6 INTEGRALS OF THE FORM J 2 . ilv, WHERE P(x) IS A POLYNOMIAL OF


ax + /n + c
Y

DEGREE TWO OR MORE


To evaluate this type of integrals we divide the numerator by the denominator and express the
d
Re

integrand as
n

R(x)
Fi

Q(*) + , where R (x) is a linear function of x.


ax2 + bx + c
P(x) dx = J Q (x) rfx + J
2 2 .X
ax + fcx + c flX + frx + c
Now to evaluate the second integral on RHS apply the method discussed earlier.
Following examples will illustrate the procedure.
ILLUSTRATIVE EXAMPLES

LEVEL-1
EXAMPLE i Evaluate:
X3 +X + 1 x2 +5x + 3
(Dj x2-l dx (ii) j
x2 + 3x + 2
dx

ReadYourFlow.COM
19.106 MATHEMATICS-XII

x3 +x + l
SOLUTION (i) Let / = J dx. Then,
x2 -1
2x + l ,
I
= 1 x + ~2x -17dx
2
=> / = f x dx + f + f _1 dx = — + log | x2 -11 + — log +C
J J x2-l J .t2--1 2 2

x2 + 5x + 3
(ii) Let 7 = J dx. Then,
x2 + 3x + 2

2x +1
I
=1 1+
x2 + 3x + 2
dx

low
2x + 3 - 2
=> 7 = | 1 • t7x + J dx
x2 + 3x + 2
2x+ 3 1
=> 7 = J 1 • rfx + J dx-2\ dx

ee
x + 3x + 2 x2 + 3x + 2
rF
Fr
1
=> 7 = x + log j x2 + 3x + 21 - 2 J 2 dx
x+—
for
2) l2
u
3 1
ks

x +------
Yo

7 = x + log | x2 + 3x + 21 - 2 x 2 2 +C
oo

3 1
x+—+-
B

2 2
re

7 = x + log | x2 + 3x + 21 - 2 log x +1 + C
1/
ou
ad

x+2
Y

EXERCISE 19.20
nd

Evaluate the following integrals:


Re

x2 + x + 1 x2 + x -1 (1 -x2)
Fi

2
dx
X -x
2.
1 9
x" + x - 6
dx 3.
x (1 - 2 x)
dx [CBSE2010]

*2+l 2
X
4.
x - 5x + 6
dx [NCERT] 5.
1 x2 + 7 x + 10
dx
2
f X +X+1 x3 +x2 + 2x + l
6' J “2
X -X + 1
dx 7.
1 x2 + 2 x + 2
8.
1 x2 - x +1
dx

x\x4 + i)dx 2
9.
1 io. J X
dx [CBSE 2005]
x2 + 4 x2 + 6x + 12
ANSWERS
2 x—1
1. x + log | x - x | + 2 log +C
X

x-2
2. x + log +C
x+ 3

ReadYourFlow.COM
INDEFINITE INTEGRALS 19.107

3. ^ + log | x | log 11 - 2 x | + C

4. x-5 log |x - 21 + 10 log|x - 31 + C


7 ? 29 x+2
5. x- ~ log | x2 + 7 x + 101 + log +C
2 6 x +5

2 tan -1 2x -1
6. x + log | x 2 - x + 11 + -j= +C
V3 ^ V3
7. x - 2 log | x2 + 2x + 21 + 3 tan-1 (x + 1) + C

8. - x2 + 2x + — log | x2 - x + 11 + -1= tan -1 2x -1 1


+C
2 2 & V3 V3
4 ^
9. ^x5 -- x + 20 x-40 tan -i ^ Uc

w
5 3 2

10. x - 3 log | x2 + 6x +121 + 2^/3 tan -1 x + 3 + C

Flo
V3

ree
HINTS TO NCERT & SELECTED PROBLEMS

F
4. We have.
or
ur
x2 +1 5x -5 x —1
I = 1^x -5x + 6 dx = J 1 H-- ^ rfx = jl-rfx + 5j—^ rfx
sf
x - 5x + 6 x -5x + 6
k

2x - 2 5 r 2x - 5 + 3
Yo

r ^ r dx = x + — dx
oo

=* 1 = z
J ^
X - 5x + 6 - 5x + 6
B

2x -5 1
=* 1 = 3:+|z I
re

dx
x - 5x + 6
ou
ad
Y

5 1
x—
=> I = x + — log x2-5x + 6 + —x 2__2 +c
nd
Re

2 & 2 5 1
x—+-
Fi

2 2

5 2 _
=> / = x + - log x -5x + 6 +
2 b

1910.7 INTEGRALS OF THE FORM J Z^—dx


ijax2 + bx + c
In order to evaluate this type of integrals, we use the following algorithm:
ALGORITHM
STEF I Write the numerator px + q in thefollowing form:
px + q = A, ] —- (ax2 +bx + c) \ + p i.e. px + q = X (lax + b) + p
dx
STEP II Obtain the values ofX and p by equating the coefficients of like powers ofx on both sides.
STEP III Replace px + q by X (lax + b) + p in the given integral to get

ReadYourFlow.COM
19.108 MATHEMATICS-XII

Px + q lax + b 1
dx = X j dx + [X J dx
-Jax2 +bx + c ^Jax2 + bx + c yjax2 +bx + c

STEP IV Integrate RHS in step III and put the values of land ja obtained in step II.
Following examples will illustrate the above algorithm.

ILLUSTRATIVE EXAMPLES
LEVEL-1
EXAMPLE 1 Evaluate:
,.x r 2x + 3 x+2
dx (U) J ■Jx2 + 5x + 6
dx [CBSE 2010]

w
d ?
SOLUTION (i) Let 2x + 3 = X — (xz + 4x+ 1) + p. Then,

Flo
dx
2x + 3 = X (2x + 4) + g.

ee
Comparing the coefficients of like powers of x, we get

Fr
2X = 2 and 4 A. + g = A. = 1 and g = -1
2x + 3
I = I— ■ = dx for
ur
yjx2 + 4x + 1
(2x + 4) -1
=> /
=1 dx
ks

tJx2 + 4x + 1
Yo
oo

2x + 4 1
=> '= I dx - J
eB

dx
yjx2 + 4x + 1 Jx2 +4x + l
r

1 2
=> I = f ^_f
ou

dx, where t = x + 4x + 1
ad

J J J p + 2)2-(j3)2
Y

=> l = 2 Ji - log (x + 2) + -Jx2 + 4x + 1 + C


nd
Re
Fi

=> 7=2 Jx2 + 4x + 1 - log | x + 2 + ^x2 + 4x + 1 | + C

d ?
(ii) Let x + 2 = A. — (x + 5x + 6) + g. Then, x + 2 = A. (2x + 5) + g
dx
Comparing the coefficients of like powers of x, we get
1
1 = 2 A, and 5 A + g = 2 => X = -- and ^ g = — 1
2 2
x+2
■■■ I = S dx
yjx2 + 5x + 6
1 1
~ (2x + 5) -
=> Idx
ijx2 + 5x + 6
2x + 5 i
=> i dx - dx
■Jr2 + 5x + 6 ■Jx2 + 5x + 6

ReadYourFlow.COM
INDEFINITE INTEGRALS 19.109

1 r _1 1 2
=> I = 2 ' 2 J dx, where t - x +5x + 6
x+
5 2
n2
2 2

=> I = Ji -^log + ^ j + ^'2 + 5x + 6 +C

=>
1
/ = tJx2 + 5x + 6 - ^ log x + ^ j + -Jx2 + 5x + 6 + C

(iii) We have.
1+x 1 +X
I dx =
I X dx =|
yjx (1 + X)
dx = J
X
2
+ X
dx

w
Let x + 1 = A — (x + x) + (i. Then, x + 1 = X(2x + 1) + |i.
dx
Comparing the coefficients of like powers of x, we get
1
1 = 2 A. and A. + p = 1 => A. = 2' ^
1
2

Flo
ree
1+x
I = dx

F
- (2x + 1) + 1 or
ur
=>
w x +1 2 2 dx
sf
2
X + X
k
Yo

2x +1
/ =2f
oo

=^>
2J 2
X + X
B

= dx, where t = x 2 + x
re

=> / = -2 Jf 4=
Vt
+2
n2 1>2
ou
ad

x+-
2 u
Y

^x + —^ + -yx +x +C — -^x + x +
=> I = ^ + -log 4 log x+4 +
nd
Re

2 2 2y
Fi

EXAMPLE 2 Evaluate:
2 2
WJ dx
a +x
-x ^ “ X ,
SOLUTION (i) Let I = ~x dx
— j = x ^rfx= f ---— ■ dx. Then,
77 + X 77 + X (7 - X •' 2
77 - X
2

w 77

-X
= rfx - f
2 J
77
2
X

-X
2
dx

- 2x
=> I = a\J -rJ
T
= dx + - ( dx
- x 2 2 J 2
-X
2
\
V'7
. -l x - 2x
=> I - a sin
77
dx
41 2
7? -X
2

2 2
Putting 77 -x = f, and - 2x dx = dt, we get

ReadYourFlow.COM
19.110 MATHEMATICS-XII

^ ,1/2
-1 a: 1 f dt
I = rt sin + —2-1 ^ a sin +C
2 1/2

. -l x
/ = fl sin + y/t +C = a sin +C
a
2- 2
(ii) Let / = f x ----- 2 dx-
Va +x
2 dt
Putting x =t, and 2x dx = dt or, dx = —, we get

a2 -t dt
I
= ix 2
a +1

w
1 a 2-t
=> I

=> I
2
Flo
F ree
12.-1
=> I — a sin
2 or
ur
f
, 12.-1
=> I = - a sin
ks

2
Yo
oo

r 1 2 • -1
=> / = — a sin
B

2
re

EXERCISE 19.21
ou
ad
Y

Evaluate the following integrals:


x 2x +1
M dx 2.
I dx
nd
Re

■Jx2 + 6x + 10 y x2 + 2x -1
Fi

x +1 6x - 5
3.
I -J4 + 5x - x2
dx 4.
1 - 5x +1
dx

3x +1 x
2
dx 6.
f 8+x-x
2
dx

x+2

1
dx 8.
I dx [NCERT]

[NCERT] 10.
I yjx2 + X + 1
dx

2x + 5
12.
1 ■Jx2 + 2x + 5
dx

3x +1 1 -x
13.
I V5 " 2x ~ x2
dx 14.
1 +x
dx

ReadYourFlow.COM
INDEFINITE INTEGRALS 19.111

2x +1 2x+3
15. | dx [CBSE2000] 16. } r = dX [CBSE 2001]
■>jx + 4.v + 3 Jx2 + 4x + 5

17.} 5x + 3 x+2
dx [CBSE 2011,12] 18. } [CBSE 2013]
Jx2 + 4x + 10 •Jx2 + 2j+3
ANSWERS
1. yjx2 + 6x + 10 - 3 log | (x + 3) + Jx2 + 6x + 10 | + C

2. 2 Jx2 + 2x -1 - log | x + 1 + Jx2 + 2x -1 | + C

. -i(2x-5
3. - -J4 + 5x - x2 +^ sin +C
V41
4. 2 -Jsx2

w
5x + l+C

-1 x +1
5. -3j5-2x- x2 - 2 sin +C
V6

Flo
ree
I 2 1 . -if 2x -1
6. -yjo + x- x + — sin +C
V33

F
7. -Jx2 + 2x -1 + log | (x + 1) + Jx2 + 2x -11 + C
or
ur
8. -Jx2 —1 +2 log | x + Jx2 -1 |+C
k sf

9. -Jx2 +1 - log | x + Jx2 + 1 | + C


Yo
oo

2x +1 / 2 . ^
B

10. -Jx2 + X + 1 -y log —----- ■^■x + 1 -+C


re

11. Jx2 + 1 + log | x + -Jx2 + 1 | + C


ou
ad
Y

12. 2 Jx2 + 2x + 5 + 3 log | x + 1 + Jx2 + 2x + 5 | + C


nd

-1 f x + 1
Re

13. - 3 Js - 2x - x2 - 2 sin +C
l V6
Fi

-1 x + ^1 - x2 + C
14. sin
15. 2 -Jx2 + 4x + 3 - 3 log | x + 2 + -Jx2 + 4x + 3 | + C

16. 2-y x + 4x + 5 - log x + 2 + Jx2 + 4x + 5 + C

17. 5 Jx2 + 4x + 10 - 7 log x + 2 + Jx2 + 4x + 10 + C

18. \x + 2x + 3 + log (x +1) + ■yx + 2x +3 + C

H/NTS TO NCERT& SELECTED PROBLEMS


8. We have.
r x+ 2
I = dx = f , = dx + 2 \ 1 dx
j ^ J ^

ReadYourFlow.COM
19.112 MATHEMATICS-XII

9. We have.
I

2 2
X +1

= ^ x 2 yjx2 + 1 - log x + -Jx2 +1 + C yjx2 + 1 - log x + Jx2 + 1 +C

w
1 1
19.10.8 INTEGRALS OF THE FORM j .2 .2 dx, J dx,
a sin x + b cos x a + b sin^ .r

Flo
1 1 1
, 2
dx, J 2
dx,^
, • 2 2
dx
a + b cos x (a sin x + b cos x) a + b sin x + c cos x

ree
To evaluate this type of integrals we use the following algorithm.

F
ALGORITHM
2
Divide numerator and denominator both by cos x. or
ur
STEP I
2 2
sf
STEP II Replace sec x, if any, in denominator byl + tan x.
k

STEP III Put tan x=t so that sec xdx = dt. This substitution reduces the integral in the form
Yo
oo

1
dt.
B

at2 +bt + c
re

STEP IV Evaluate the integral obtained in step III by using the methods discussed earlier.
Following examples will illustrate the procedure.
ou
ad
Y

ILLUSTRATIVE EXAMPLES
nd

LEVEL-1
Re
Fi

EXAMPLE 1 Evaluate:
1 1
WJ -2 2 2 2
dx (ii) J 2 2
dx
a sin x + b cos x 1 + 3 sin x + 8 cos x
sm x 1
(iii) } dx (iv) J j dx
sin 3x (2 sin x + 3 cos x)
2
SOLUTION (i) Dividing the numerator and denominator of the given integrand by cos x,
we get
2
1 sec x
I =
2 1 2 2 2
a sin x + b cos x
dx = j 2 2
a tan x + b
2 dx

2
Putting tan x = t and sec x dx = dt, we get

= Jf dt =Tf dt 1_ J_ tan -1 t
+C
I a2t2 + b2 «2j a2 X b/a
t2+(b/a)2 b/a

1 -if at 1 -l l a tan x
=> I = — tan — + C = — tan +C
ab b ab b

ReadYourFlow.COM
INDEFINITE INTEGRALS 19.113
2
(ii) Dividing the numerator and denominator of the given integrand by cos x, we get
1
I =
J 2 2 dx
1 + 3 sin x + 8 cos x
2 2 sec2 x dx
sec x sec x
=> I =
sec2 x + 3 tan2 x + 8
dx =
J 1 + tan2 x + 3 tan2 x + 8
dx = j
4 tan2 x + 9
2
Putting tan x = t and sec x dx = dt, we get
dt 1 1 t
=> I = Jf 4f2 + 9 4 J f2 +(3/2)2 tan -l
—x +C
3/2 4 3/2
\
1 . -l(2t + C = — tan 1 2 tan x
=> 1 = — tan +C
6 3 6 3
(iii) We have.

low
sin x sin x 1
/
=1 dx = J 3 dx = j" dx
sin 3x 3 sin x - 4 sin x 3-4 sin2 x
2
sec x 2
=> dx [Dividing numerator and denominator by cos x]

ee
3 sec2 x - 4 tan 2 x
rF
Fr
2
Putting tan x = t and sec x dx = dt, we get
dt 1
for
/ =
3(1 +f2) -4f2 =J * = 1
3-f2 (/3)2-f
2^
u
ks

^3+f 1 -/3 + tan x


I = log
Yo

=> io8 +C = +C
oo

2^3 j3-t 2^3 -JS - tan x


B

(iv) Dividing the numerator and denominator by cos 2 x, we get


re

2
1 sec x
I = J 2dx = I 2 dX
ou
ad

(2 sin x + 3 cos x) (2 tan x + 3)


Y

2
Putting tan x = t and sec x dx = dtr we get
nd
Re

1 1
I = f
J (2f + 3)2 ^ 2 (2f + 3)
+C = -
2 (2 tan x + 3)
+C
Fi

EXAMPLE 2 Evaluate:
1 (ii) |
1
(i)J dx dx
3 + sin 2x 2-3 cos 2x
SOLUTION (i) Let
1 1
I =1 dx = J 2 2 dx
3 + sin 2x 3 (sin x + cos x) + 2 sin x cos x
2
sec x 2
=> / = dx [Dividing numerator and denominator by cos x]
3 tan2 x + 2 tan x + 3
2
Putting tan x = t and sec x dx = dt, we get
dt 1 r __ & dt
/ =1 3f2 + 2t+3
3J
t+1
3
Ii t+
if + 2V2f
3 3

ReadYourFlow.COM
19.114 MATHEMATICS-XII

1
t+ 3 1 -1 3t + l 1 -1 3 tan x +1
+C = tan +C = tan +c
2 V2 lyjl 2 yfl 2yf2 2V2
3

(ii) Let 1 = \ dx. Then,


2-3 cos 2x
1
=>
W 2 2
2-3 (cos x - sin x)
dx

2 2
sec x
=> I =
f 2 sec2 x - 3 + 3 tan2 x
dx [Dividing numerator and denominator by cos x]

w
2 2
sec x sec x
=> I =
I dx = J dx

Flo
2(1 + tan2 x) - 3 + 3 tan 2 x 5 tan2 x -1
2

ee
Putting tan x = t and sec x dx = dt, we get

Fr
1
t-
1 dt 1 1 ■v/5
I
= 1 5f2'-1 1 \2 5X log
for +C
ur
5 1 1
t2- 1 2 t+
V5J V5
U/5
ks

V v /
Yo

Vst-i yj5 tan x -1


oo

=> I VloS
2 VS yjst + 1
+c = V loS
2^/5 V5 tan x + 1
+c
eB

EXERCISE 19.22
r

LEVEL-1
ou
ad
Y

Evaluate the following integrals:


1 1 2
M 2 dx
2.
I dx 3.J dx
nd
Re

2 4 sin2 x + 5 cos2 x 2 + sin 2x


4 cos x + 9 sin x
Fi

cos x 1 1
4.
I cos 3x
dx 5.
1 1 + 3 sin2 x
dx 6.
1 3 + 2 cos" x
dx

1 sin 2x
7.
I (sin x - 2 cos x) (2 sin x + cos x)
dx 8.
1 . 4
sm x + cos x
4
dx

1 1 1
9.
I cos x (sin x + 2 cos x)
dx m \
sin2 x + sin 2x
dx 11.
1 2 dx
cos 2x + 3 sin" x

ANSWERS

-1 3 tan x 1 -1 2 tan x
1. — tan +C 2. tan +C
6 2 2 V5 Vs
2 1 2 tan x + 1 1 1 + V3 tan x
3. -= tan +C 4. log +C
V3 V3 2 V3 1-^3 tan x

ReadYourFlow.COM
INDEFINITE INTEGRALS 19.115

1 -l 1 -1 y[3 tan x
5. — tan (2 tan x) + C 6. tan +C
Vl5
7. I log tan x - 2
+c 8. tan 1 (tan2 x) + C
2 tan x + 1

10. i log tan x


9. log | tan x + 2| + C +C
tan x + 2
11. -j= tan-1 (V2 tan x) + C
V2
1 1 1
19.10.9 INTEGRALS OF THE FORM dx, J dx, f dx,
a sin x + b cos x a + b sin x a + b cos x
1
dx
a sin x + b cos x + c

w
To evaluate this type of integrals we use the following algorithm.
ALGORITHM

STEP I Put sin x =


2 tan x/2
and, cos x =

Flo
1 - tan2 x/2
and simplify.

ee
1 + tan 2 x/ 2 1 + tan2 x/2

Fr
2 X . 2 X
STEP II Replace 1 + tan — in the numerator by sec — . for
ur
1 2 X
STEP III Put tan ~=t so that - sec -dx = dt. This substitution reduces the integral in the form
2 2
ks

1
1
Yo

dt.
oo

at2 +bt + c
eB

STEP IV Evaluate the integral obtained in step III by using methods discussed earlier.
Following examples will illustrate the above procedure.
r
ou
ad

ILLUSTRATIVE EXAMPLES
Y

[ LEVEL-1
nd

EXAMPLE 1 Evaluate:
Re

1 1
wj (ii) j
Fi

dx dx
1 + sin x + cos x 2 + cos x
1 + sin x 1
(iii) J dx (iv) { dx
sin x (1 + cos x) 1-2 sin x

2 tan x/2 1 - tan 2 x/ 2


SOLUTION (i) Putting sin x = and cos x = , we get
1 + tan2 x/2 1 + tan2 x/2

7 = I 1 + sin x1 + cos x dx = J 2 tan x/2


1
1 - tan2 x/2
dx
1 + +
1 + tan2 x/2 1 + tan2 x/2

1 + tan2 x/2 sec2 x/2


=> 7
I 1 + tan2 x/2 + 2 tan x/2 + 1 - tan” x/2
dx =
I 2 + 2 tan x/2
dx

X 1
Putting tan — = t and — sec 2 - dx = dt or, sec2 — dx = 2dt, we get
2 2

ReadYourFlow.COM
19.116 MATHEMATICS-XII

2dt 1 rff = log | f + 11 + C = log tan ^ + 1 + C


i = \
2 + 21 =1 t+1
(ii) We have.

1 1 1 - tan2 x/2
l = dx = | dx V COS X =
2 + cos x 1 - tan 2 x/ 2 1 + tan2 x/2
2+
1 + tan2 x/2
1 + tan2 x/2 sec2 x/2
=> I
= 1 2 (1 + tan" x/2) + 1 - tan2 x/2 dx =|
tan" x/2 + 3
dx

Putting tan x/2 =t and (1/2 sec" (x/2) = dt or, sec2 (x/2) dx = 2dt, we get

2dt -1 dt 2 t _l tan x/2


'=1 f2 + 3 2I 12+(V3)2
„ _ = —p= tan ~r +C=~T= tan V3
+c

w
1 + sin x
(iii) Let / = J dx

Flo
sin x (1 + cos x)
1 - tan 2 x/ 2

ee
2 tan x/2
Putting sin x = and cos x = , we get

Fr
1 + tan2 x/2 1 + tan2 x/2
2 tan x/2
1 +
for
ur
1 + tan2 x/2
I
=1 2 tan x/2 1 - tan 2 x
dx
ks

1 +
Yo

1 + tan“ x/2 1 + tan2 x/2


oo
eB

(1 + tan2 x/2 + 2 tan x/2) (1 + tan“ x/2) (1 + tan x/2)2 sec2 x/2
=> I = dx = J dx
2 tan x/2 (1 + tan2 x/2 + 1 - tan x/2) 4 tan x/2
r
ou
ad

Putting tan x/2 =1 and (1/2) sec2 (x/2) dx = dt or, sec2 (x/2) dx = 2 dt, we get
Y

(1+f)2
I = 1 it 2dt
nd
Re

l+t2 + 2t
* = 1} (l + f + 2) dt
Fi

=> I
t

1 f2 tan2 x/2
=> / = + 2t + C = ^ • log | tan x/21 + + 2 tan x/2 + C
2 r°8^+2 2
1 2 tan x/2
(iv) Let / = J dx. Putting sin x = , we get
1 - 2sin x 1 + tan" x/2

1 1 + tan2 x/2 sec2 x/2


4 tan x/2
dx =J dx = j dx
1 + tan2 x/2 -4 tan x/2 1 + tan 2 x/2 - 4 tan x/2
1
1 + tan" x/2
Putting tan x/2 = t and ^ sec 2 (x/2) dx = dt or, sec" (x/2) dx = 2dt, we get

2 1 1
W 1 + l2 - 41
rft = 2}
£2 _ 4f + 4 _ 4 + 1
dt =
2J (t-2)2-(j3)
2dt

ReadYourFlow.COM
INDEFINITE INTEGRALS 19.117

I = 2x
1 f-2-V3 1 tan x/2 - 2-^3
=> log +C
2^3 t-2 + j3 +C = W'°S tan x/2 - 2 + -J3
EXERCISE 19.23
LEVEL-1

Evaluate thefollozving integrals:


1 1 1
5 + 4 cos x
[CBSE 2003]
2-l 5 - 4 sin x
dx 3. j
1-2 sin x
dx

1 1
dx 5. | dx
4 cos x -1 1 - sin x + cos x
1 1
6.J 3 + 2 sin x + cos x
dx [CBSE 2004] 7'l dx
13+3 cos x + 4 sin x

low
1
8. J cos x - sin x
dx
9-1J -—1— dx
sin x + cos x
1 1
io. } dx 11. J dx
5-4 cos x 2 + sin x + cos x

ee
1
rF
Fr
1
12. J dx 13. j dx
sin x + -J3 cos x yf3 sin X + COS X
for
1 1
14'1 ^ sin x -V3 cos x
dx 15. J
5 + 7 cos x + sin x
dx
u
ks

ANSWERS
Yo
oo

2 . -if tan x/2 „ 2 , -i(5 tan x/2-4


1. tan +C 2. — tan ---------------- + C
B

3 3 3 3
re

tan — - 2 - J3
2 1 y[3 + •JE tan x/2
ou

3.
ad

lo8 +C 4. log +C
V3 tan — - 2 + ^3 Vl5 y[3 -^5 tan x/2
Y

2
nd
Re

5. - log 6. tan 1
6 1 - tan —
2 +C 1 + tan — + C
2
Fi

1 -1 / 5 tan x/2 + 2 1 V2 + tan x/2 + 1


7. — tan +C 8. log +C
6 6 V2 \/2 - tan x/2 -1
1 ^2 + tan x/2 -1 -1
9- —Tjr log +c 10. — tan 3 tan — + C
V2 ^2 - tan x/ 2 + 1 3 2

-1 tan (x/2) + 1 12. ^ log 1 + V3 tan x/2 + C


11. V2 tan +C
V2 3 - V3 tan x/2
13. — log tan f- + — 14.1 log +C
+c
2 6 ^2 12

1 tan x/2+ 2
15. - log. -------------- + C
5 tan x/2 - 3

ReadYourFlow.COM
19.118 MATHEMATICS-XII

1
19.10.10 ALTERNATIVE METHOD TO EVALUATE INTEGRALS OF THE FORM \ dx
a sin x + b cos x
To evaluate this type of integrals, we substitute

a=r cos Q,b =r sin 0 so thatr - -Ja2 +b2 and, 0 = tan 1

a sin x + b cos x = r cos 0 sin x + r sin 0 cos x = r sin {x + 0)


1 1
So,
^=J a sin x + b cos x
dx = - \
r J sin (x + 0)
dx

=> I = — f cosec (x + 0) dx
rJ
i, , 0 "i 1 -1 b
=> I = - log tan — + — +C = . . ilog .tan *— + —
1 ,
tan +C
r 5 {22) ^2+b. ,2 6 2 2 a

w
Following examples will illustrate the above algorithm.
ILLUSTRATIVE EXAMPLES

Flo
LEVEL-1

ee
Fr
EXAMPLE 1 Evaluate:
1 1
dx (ii) f - dx
*J3 sin J sm x + V3 COS X
X + COS X
for
ur
SOLUTION (i) Let ^3 = r sin 0 and 1 = r cos 0. Then
ks

r = ^/3)2+l2 and tan 0 = — => r = 2 and 0 = -


Yo

1 3
oo

1
1
B

I = dx
*J~3 sin x + cos x
re

1
=> I =
1 r sin 0 sin x + r cos 0 cos x dx
ou
ad
Y

1 dx = - j sec (x - 0) dx = - log tan ^ ^ ^


=> r = -J
r J cos (x - 0)
+C
nd
Re

1 , . n 1 XT!
Fi

=> I = -log tan - + + C = — log tan — h----- +C


2 6J 2 l 2 12

(ii) Let 1 = r cos 0 and /3 = r sin 0. Then,


r = Jl2 + (V3)2 and tan 0 = — => r = 2 and 0 = —
1 3
i i
I =
isin x + a/3 cos x
dx = -
sin x cos 0 + cos x sin 0
dx

1
dx = - \ cosec (x + 0) dx
r J sin (x + 0) r J

1 A f* 0 1 + C„ = —
I = - log tan — + —
1 . L (x n
log tan — + — +C
r 6 [2 2) 2 6 v2 6

a sin x + b cos x
19.10.11 INTEGRALS OF THE FORM j dx
c sinx + d cos x
To evaluate this type of integrals, we use the following algorithm.

ReadYourFlow.COM
INDEFINITE INTEGRALS 19.119

ALGORITHM
STEP I Write
Numerator = X (Diff. of denominator) + (.1 (Denominator)
i.e. a sin x + b cos .r = X(c cos x - d sin x) + (.1 (c sin x + d cos x)
STEP II Obtain the values of X and fi by equating the coefficients o/sin x and cos x on both the sides.
STEP HI Replace numerator in the integrand by X (c cos x -d sin x) + (a (c sin x + d cos x) to obtain
a sin x + b cos x c cos x -d sin x c sin x + d cos x
dx = Xj dx + (.1 J
c sin x + d cos .r c sin x + d cos x c sin x + d cos x dx
= X log | c sin a: + cos xj + g x + C
Following examples illustrate the above procedure.

ILLUSTRATIVE EXAMPLES

low
LEVEL-1
EXAMPLE 1 Evaluate:
3 sin x + 2 cos x 1
wj dx (ii) J dx

ee
3 cos x + 2 sin x 1 + tan x
rF
Fr
3 sin x + 2 cos x
SOLUTION (i) We have, I = J dx
3 cos x + 2 sin x for
Let 3 sin x + 2 cos x = X-^-(3 cos x + 2 sin x) + p (3 cos x + 2 sin x)
u
dx
ks

i.e. 3 sin x + 2 cos x = X (- 3 sin x + 2 cos x) + p (3 cos x + 2 sin x)


Yo
oo

Comparing the coefficients of sin x and cos x on both sides, we get


B

5
-3A.+ 2p = 3 and 2X+3p = 2 => p = -— and X = —
re

13 13
A. (- 3 sin x + 2 cos x) + p (3 cos x + 2 sin x) ,
I = ------------------------------------------------- ----- dx
ou
ad

3 cos x + 2 sin x
Y

- 3 sin x + 2 cos x
=> I = p J 1 ■ dx + X j dx
3 cos x + 2 sin x
nd
Re

=> I = p x + A, J y-, where f = 3 cos x + 2 sin x


Fi

=> 1 = p x + A. log | f | + C = ^ x - — log | 3 cos x + 2 sin x | + C


13 13
1 cos X
(ii) /
=11 + tan x
dx = I
cos x + sin x
dx

Let cos x = X (cos x + sin x) + p (cos x + sin x). Then,


dx
cos x = A. (- sin x + cos x) + p (sin x + cos x)
Comparing the coefficients of sin x and cos x on both sides, we get
- A, + p =0 and A. + p = 1 => A, = p = -
cos x
1
=1 cos x + sin x
dx

1/2 (- sin x + cos x) + 1/2 (cos x + sin x) ^


=>
cos x + sm x

ReadYourFlow.COM
19.120 MATHEMATICS-XII

- sin x + cos x dx + — \ cos x + sin x


=> I
= 1/ cos x + sin x 2 J cos x + sin x
dx

=> /=!}
2J
dtt + —2 Jf
— 1 ■ dx, where t = cos x + sin x

1 1 11
=> I = — log I f I+ — x + C = — x + — log I sin x + cos x I + C
2 5i i 2 2 2 61
1
EXAMPLE 2 Evaluate: J dx
1 + COt X
SOLUTION We have.
1 1 sin x
W dx = j cos x
dx = J dx
1 + cot x sin x + cos x
1+
sin x

w
Let sin x = X — (sin x + cos x) + n (sin x + cos x)
dx
i.e. sin x = X (cos x - sin x) + \x (sin x + cos x)

Flo
Comparing the coefficients of sin x and cos x on both sides, we get

ee
0 = A. + p and 1 = - X + g => X = -1/2, p = 1/2

Fr
A. (cos x - sin x) + p (sin x + cos x) ^
W sin x + cos x for
ur
cos x - sm x sin x + cos x
=> 1 = dx + p J dx
sin x + cos x sin x + cos x
ks
Yo

=> l = A.J y- + p | 1 • dx, where t = sin x + cos x


oo
B

=> I = \ log | f [ + p x + C
re

=> / = - (1/2) log | sin x + cos x| + (1/2) x + C


(? sin x + b cos x + c ,
ou

---------------------- dx
ad

19.10.12 INTEGRALS OF THE FORM j


p sin x + q cos x + r
Y

To evaluate this type of integrals, we use the following algorithm.


nd
Re

ALGORITHM
Fi

STEP 1 Write: Numerator = X (Diff. of denominator) + p (Denominator) + v


i.e. a sin x + b cos x + c = A. (p cos x - g sin x) + p (p sin x + 9 cos x + r) + v
STEP II Obtain the values ofX and p by equating the coefficients of sm x and cos x and the constant
terms on both the sides.
STEP HI Replace the numerator in the integrand by A. (p cos x - g sin x) + p (p sin x + ^ cos x + r) + v
to obtain
77 sin x + b cos x + c ,
'=1------------------------ dx
p sin x + 77 cos x + r
^ r p cos x - 7/ sin x , r p Sin X + 77 cos x + r 1
=> / = dx + p --------1----------- + v dx
•' p sin x + (j cos x + r J p sm x + 77 cos x + r p sin x + 17 cos x + r
=> I = A, log | p sin x + 77 cos x + r| + px + v J 1
dx
p sin x + 77 cos x + r
STEP IV Evaluate the integral on RHS in step III by using the method discussed earlier.
The following example illustrates the procedure.

ReadYourFlow.COM
1

INDEFINITE INTEGRALS 19.121

ILLUSTRATIVE EXAMPLES

LEVEL-1

3 cos x + 2
EXAMPLE Evaluate: J dx
sin x + 2 cos j + 3
SOLUTION We have.
3 cos x + 2
dx
sin x + 2 cos x + 3
Let 3 cos x + 2 = X (sin x + 2 cos x + 3) + ji (cos x - 2 sin x) + v
Comparing the coefficients of sin x, cos x and constant term on both sides, we get
?.-2p=0, 2X + p = 3, 3 A. + v = 2
X = ^ 8
=> 5 ' ^ = — and v =

low
5 5
X (sin x + 2 cos x + 3) + p (cos x - 2 sin x) + v
I
I sin x + 2 cos x + 3
dx

cos x - 2 sin x 1

ee
=> l = X | dx + p j* dx + v J dx
rF
sin x + 2 cos x + 3 sin x + 2 cos x + 3

Fr
1
=> I = X x + p log | sin x + 2 cos x + 31 + v fj, where = J dx
sin x + 2 cos x + 3
for
ou
2 tan x/2 1 - tan2 x/2
Putting sin x = and cos x = , we get
ks

1 + tan"- x/2 1 + tan2 x/2


oo

1
dx
Y
B

2 tan x/2 2(1 - tan2 x/2)


+ +3
re

1 + tan2 x/2 1 + tan2 x/2


ou
ad

1 + tan"" x/2
Il
I dx
Y

2 tan x/2 + 2-2 tan2 x/2 + 3 (1 + tan2 x/2)


nd

sec2 x/2
Re

=> dx
Fi

tan2 x/2 + 2 tan x/2 + 5


X 1 2 X 2 X
Putting tan -=t and - sec dx = dt or, sec — dx = 2 dt, we get
2 2 2 2 6
x 1
tan +1
2dt dt 2 -lff + 1 -1 2
/i = j = 2{ = — tan ----- = tan
t2 +2t+ 5 (t + l)2 + 22 2 2 2

tan A +1
-1 2
7 = A. x + p log | sin x + 2 cos x + 31 + v tan +C
2

tan 'r +1i


or. 1 — x + — log I sin x + 2cosx + 31 -- tan -1 2 +C
5 5 6| ! 5 2

ReadYourFlow.COM
r

19.122 MATHEMATICS-XII

EXERCISE 19.24
LEVEL-1
Evaluate the following integrals:
1 3 + 2 cos x + 4 sin x
1. J --- ---- dx
1 - cot x 2-J 1 - tan x
dx 3.J 2 sin x + cos x + 3
dx

1 5 cos x + 6 2 sin x + 3 cos x


p + cj tan x
dx 5.J 2 cos x + sin x + 3
dx 6.}
3 sin x + 4 cos x
dx

1 2 tan x + 3 1
7-1 3 + 4 cot x
dx 8.J 3 tan x + 4
dx
4+3 tan x
dx

8 cot x + 1 4 sin x + 5 cos x


10.
3 cot x + 2
dx M 5 sin x + 4 cos x
dx

ANSWERS

low
1. ^ x + ^ log | sin x - cos x | + C 2. — x - — log I sin x - cos x I + C
2 2 61
-1 tan — + 1 I + C x+-. ^ log | p cos x + q sin x \ + C
3. 2x - 3 tan
2 p +q

ee
V +1
rF 18 1

Fr
5. 2 x + log | 2 cos x + sinx+3|+C 6. ^ * + ^ I 3 sin x + 4 cos x | + C
for
7. — x - — log | 3 sin x + 4 cos x | + C 8. — x+ — log | 3 sin x +4 cos x | + C
25 25 25 25
u
4 3
9. + I 4 cos x + 3 sin x | + C 10. 2 x + log | 2 sin x + 3 cos x | + C
ks
Yo

40 9
oo

11. — x + — log | 5 sin x + 4 cos x | + C


B
re

19.11 INTEGRATION BY PARTS


THEOREM Ifu and v are two functions ofx, then
ou
ad

j uv dx = u J v dx -J dx j dx
Y
nd

i.e. The integral of the product of two functions = (First function) x (Integral of second function)
Re

Integral of j (Diff. offirst function) x (integral of second function) j


Fi

PROOF For any two functions / (x) and g (x), we have


y-VM-sM)
dx
= /M-f dx
IsMI + gM-f
dx
(/Ml
1 {/ M ■ £ Is Ml +?(*) — (/ M) jix=fix)g (X)
=> [ |/W~(gW) } <ia: + J {sW-|(/W}|^=/WgW
j (x)~{g(x)}^ dx=f(x)-g(x)-j |g(x)
-(i)

Let / (x) = u and (x)} = V so that g (x) = | v dx. Substituting these in (i), we get
dx
^ uv dx = u ■ j v dx ■ - j | ' j 11 dx

ReadYourFlow.COM
INDEFINITE INTEGRALS 19.123

NOTE 1 Proper choice offirst and secondfunction —


Integration with the help of the above rule is called theintegration by parts. In the above rule there are two
terms on RHS and in both the terms the integral of the second function is involved. Therefore in the
product of two functions if one of the tzvo functions is not directly integrable (e.g., log x, shi x, cos -l x,
tan~l xetc.), ive take it as thefirst function and the remaining function is taken as the second function. If
there is no otherfunction, then unity is taken as the second function. If in the integral both the functions
are easily integrable, then thefirst function is chosen in such a way that the derivative of thefunction is a
simple function and the function thus obtained under the integral sign is easily integrable than the
original function.
NOTE 2 We can also choose thefirstfunction as thefunction which comesfirst in the word ILATE, where
I - stands for the inverse trigonometric function (sin 1 x, cos 1 x, tan 1 x etc.)
L - stands for the logarithmic functions

w
/l - stands for the algebraic functions
T - stands for the trigonometric functions
E - stands for the exponential functions

Flo
ILLUSTRATIVE EXAMPLES

ee
Fr
LEVEL-1

EXAMPLE 1 Evaluate:
for
ur
(i) J x sin 3x dx [NCERT] (ii) J x sec2 x dx [NCERT]
ks

(iii) | x log x dx [NCERTJ (iv) J x sin2 x dx


Yo
oo

SOLUTION (i) Here both the functions viz. x and sin 3 x are easily integrable and the derivative
B

of a: is one, a less complicated function. Therefore, we take x as the first function and sin 3 .r as the
re

second function.
I = [ x sin 3x dx
ou
ad

J I II
Y

=> I = x • J sin 3x dx ■ - sin 3x dx ^ dx


d
Re
n

1
Fi

=> I = xx —cos 3x - —cos 3x V dx


3
11 111
=> I = - — x cos 3x + — [ cos 3x dx = — x cos 3x + — sin 3x + C.
3 3J 3 9
(ii) Let 7 = I* x sec2 x dx . Then,
J I II

7 =
x 1 sec2 x dx ■ - !(£«-) sec2 x dx • dx
=> I = x tan x - J lx tan x dx = x tan x + log | cos x | + C

(iii) Let 7 = f x log x dx. Then,


J II I
| (log x) x J x dxj dx
I = log x ■ J x dx ■ -

ReadYourFlow.COM
r

19.124 MATHEMATICS-XII

2 2
/ = aog*)y-J ix —
2
rfx = — log x - - f x dx
2 2J
x2, 1 X: 2 *2 , 1 2 r
I --- log X - - +C — log x — x + C
2 2 2 2 b 4

r 2
(iv) Let / = | x sin x dx . Then,

/ =
H
1
2
x2
1 - cos 2x
2

1
dx = — f x dx-— [ x cos 2x dx
2J 2J i H

=> x j cos 2x dx • - cos 2x dx > dx


2 2 2
2
1 2 1 (X r sin 2x J sin 2x rfxj
=> I = -x — <^ - sin 2x - dxl= — - -sin 2x - —

w
4 2(2 J 2 4 2 2 2
2
x 1 x . _ 1 1 2 X 1
=> I — — sin 2x — - cos 2x + C = - x — sin 2x — cos 2x + C
4 2(2 2 2

Flo 4 4 8

ree
EXAMPLE 2 Evaluate:
r 2
(i) lx sin x dx (ii) J x2 ex dx [NCERT]

F
SOLUTION (i) Let / = f x2 sin x dx . Then, or
ur
J I II
f
1 }-J | f
ks

I = sin x dx dx
Yo
oo

2
=> / = - X cos x - f 2x (- cos x) dx = - x2 cos x + 2 [ x cos x dx
B

J J I II
re

=> I = - x2 cos x + 2 x • J cos x dx cos x dx > dx


ou
ad
Y

2 2
=> / = - x cos x + 2 • x sin x - J sin x dx ■ x cos x + 2 (x sin x + cos x) + C
nd
Re

(ii) Let / = f x2 ex dx. Then,


Fi

J I II

I = dx
dx J
=> Ij = x 2 e x | 2x ex dx = x2 ex 2 f x ex dx
J I II

=> I = x2 ex -2 ^ (x) x J ex dx ■ dx
dx

=> t 2 x ~
I = x e -2 {x eX~SeXdx} 2 x
- x e -2 (x ex-ex) + C

EXAMPLE 3 Evaluate:
(i) J log x dx [NCERT] (ii) | (log x)2 dx

SOLUTION (i) Let / = f log x • 1 dx


J I n

k ReadYourFlow.COM
INDEFINITE INTEGRALS 19.125

I = log x • {j |£(logx)-J 1*|*


=> I = (log x) x - | —■ x dx = x (log x) - J 1 • tfx = x (log x) - x + C

(ii) Let ^ = [ (l°g *)2 ' 1 dx. Then,


I 11
1 • dx • - j" | (log x)2 • J 1 • dx^j dx
/ = (log x)2 ■ j

=> I = (log x)2 x - J 2 log x • - • x dx = x (log x)2 - 2 f log x • 1 dx


I 11

=> I x(logx)2-2 (log x) < J 1 • dx


1 {f(los*)J l'*|*

w
=> I = x (log x)2 - 2 | (log x) x — J ^ ■ x dx j = x (log x)2 - 2 (x log x - x) + C

Flo
ee
EXAMPLE 4 Evaluate:
-1

Fr
sin x dx [NCERT] (ii) j tan 1 x dx [NCERT]
-1 [NCERT]
(iii) j sec x dx
for
ur
SOLUTION (i) Let sin - 1 x = f. Then, x = sin t => dx = d (sin t) = cos t dt
ks

/ = J sin-1 x dx
Yo
oo

=> I = j t cos t dt = t sin f - J 1 • (sin t) dt = t sin f - J sin t dt = t sin t + cos t + C


B

1 II
re

=> I = x sin -1 x + -Jl - sin2 t + C = x sin 1 x + yl -x2 + C


ou
ad

(ii) Let tan —1 x=f. Then, x = tan f and dx = sec2 t dt


Y

I = J tan-1 x dx
d
Re

/ = J f sec2 t dt = t (tan 0 ~ J 1 •tari t dt = t tan t + log | cos t \ +C


n

=>
Fi

1 1 1
=> I = x tan -l x + log +C v tan t - x=> cos t =
^iTx2 \/l + tan2f ^1 + x2

=> I - x tan -1 x - | log 11 + X2 I + C

(iii) Let sec -1 x = t then, x = sec f => dx = sec t tan t dt


-1
1
I = sec x dx
=> / = f f (sec t tan t) dt
J I II
I = t (sec f) - J 1 • sec t dt
=> I = t sec t - log | (sec t + tan I) I + (T
=> I = t sec t - log | sec f + ^sec2 t -1 | + C = x (sec 1 x) - log | x + yx2 -1 | + C

ReadYourFlow.COM
r 19.130 MATHEMATICS-XII

LEVEL-2

EXAMPLE 8 Evaluate: J sin 2x tan 1 (sin x) dx

SOLUTION Let 7=| sin lx tan 1 (sin x) dx. Putting sin x = t and cos x dx = dt, we get
2 t2
1
sin 2x tan 1 (sin x) dx = 2 { t tan- 11 dt = 2- (tan- 11)----- [ ------2X~odt
J II I 2 J 1 +r 2
(t2+l)-l
=> I = t2 (tan 11) -1 dt = t2 tan lt-\ 1------ dt
1+t2 l+t2)

=> I = t2 tan ^ t - t + tan- 17 + C


2—1 —1
=> 7 = sin x tan (sin x) - sin x + tan (sin x) +C

low
EXAMPLE 9 Evaluate: J cot- 1 (1 - x + x2) dx

SOLUTION Let 7 = J cot 1 (1 - x + x2) dx. Then,

ee
7 = J cot-1 | l-x(l-x)j dx = j tan-1 • 1
• dx
rF
Fr
1 — x (1 — x)

w tan -l
X + (1 -x)
■ dx=\ { tan -1 x + tan 1 (1 - x) j dx
=>
for
1 -x(l -x)
u
-1
I = \ tan- 1 x dx + J tan
ks

=> (1 -x) dx= Ii + 72 ...(i)


Yo
oo

where 7J = J tan -1 x dx and 72 = | tan 1 (1 - x) dx.


B

Now,
re

tan -1 x dx = | tan -1 x 1 dx = x tan- 1 x - f


W I II
~~2 dx
J 1+x
ou
ad

Il= x tan-1 ,-If


Y

2 2x dx = x tan 1 x - ~ log (1 + x2) ...(ii)


2 J 1+x
nd
Re

and.
Fi

'2=1 tan 1 (1 - x) dx = - J tan 17 dt, where 7=1 - x.


7 tan -1x7-ilog(l+72)j
=> h - [Using (ii)]

72= - (1 -x) tan- 1 (1 - x) -log jl + (1 - x)2

Substituting the values of 7] and 72 in (i), we get


Jcot !(1 -x + x2) dx = x tan- ax-^ log (1 + x2) -(1 -x) tan-1(1 -x) +~ log jl + (1 -x)2 j
+C

EXAMPLE 10 Evaluate : J ^ 3x2 1 2


tan---- x sec dx
x x)

SOLUTION Let 7=1 { 3x2 tan — -xsec 2 1 dx


x x

=> 7 = [ 3x2 tan — dx - f x sec2 -


1
dx
J II I x J x

ReadYourFlow.COM
INDEFINITE INTEGRALS 19.131

7 = x3 tan (j^-J [sec^jx-i-xx^-j x sec2 ^ dx

2 1 1
I = x3 tan — + [ x sec -dx-\ x sec 2 — dx + C = .\'3 tan +C
x •’ x J x X

log (1 + x2)
EXAMPLE 11 Evaluate: J - dx
3
X
log (1 + x2)
SOLUTION Let I =
1 X
3
dx. Then,

I = \ log (1 + X2) X 3 dx = - -iy log (1 + X2) + f x dx


J -I -II 2xl x2 (1 + X2)

w
1 2 1 r dt where 7 = x2
=> I = - — lo8 (1 + *2) + - |
2x2 2 J 7(1+0'

=> I= -
1
2x2
log (1 + a:2) + t}

Flo
1
7+1
dt = -
2x2
log (l+X2) + i logt-log (7+ 1) j- +C
2

ee
\

Fr
=> I = - —2 log (1 + X ) + - log +C
2x2 2 or
ur
f
. -1 -1
sin Vx - cos £dx
EXAMPLE 12 Evaluate: J [NCERT]
ks

sin 1 Vx + cos -1 Vx
Yo
oo

7t
SOLUTION We know that sin -1 X + cos -1 X = —
eB

sin Vx - - - sin 1 Vx
sin 1 Vx - cos -l
r

Vx dx= j 2
ou

=1
ad

/ dx
sin 1 Vx + cos 1 Vx 71
Y

2
nd

-1
f1 1j
Re

=> 7 = sin Vx rfx - J 1 • rfx


Fi

4 r . -1 Jxdx-x + C=- I-i-x + C, where 72 = [ sin 1 Vx dx


=> 7 = — sm
K J K J

Putting x = sin 2 0 and dx = 2 sin 0 cos 0 t/0 = sin 20 <70, we get


cos 2 0 0 1
7l =1 0 sin 20 <70 = - 0
I H 2
+ [ — cos 20 dQ
J 2
— cos 20 + — sin 20
2 4
=> - ^ 0 (1 - 2 sin2 0) + -^ sin 0 yl - sin2 0
'l =
1 -1 Vx +i Vx 71
=> 72 = — (1 - 2x) sin -(ii)
2
From (i) and (ii), we get
4 ~ (1 ~ 2x) sin 1 Vx + ^-Jx -x 2 -x + C
7 = -
71

2 -(1 -2x) sin-1 Vxj


=> -X -x + C

ReadYourFlow.COM
19.132 MATHEMATICS-XII

^■2 + 1 | log (a-2 + 1) - 2 log xj


EXAMPLE 13 Evaluate: J dx [CBSE 2012, NCERT]
4
X
SOLUTION Let
■Jx2 +1 j log (x2 + 1) - 2 log xl
/ = 1-------1-----4-----------
X

=> • log (x2 + 1) - log x2 dx

if. 1 1 ,
=> log 1 + -=- rfx
x x'3

w
1 1 2 , , 1 1
Let 1 + — = L Tlien, d 1 + — = dt => - —j dx = dt =>—^- dx = — dt
x x y X X 3 2

/ = -
2 II
logtrft
I
Flo
ee
=> J = -l{|(i°sf)f 3/2
is i
-x t 3/2 * i l(logt)t3/2-*,3/2
+c

Fr
f 2 3 9
n3/2
/ = -It3/2 {l0g(-|} + c
for
1 1
ur
1
=> =-
3
— 1 H—-r-
3 X2, l08 l + 7 -3- +c.
ks

2
Yo

X
Evaluate: J
oo

EXAMPLE 14 2 dx
(x sin x + cos x)
B

2
re

x X COS X
SOLUTION Let I = \ - dx. Then, I = J (x sec x) x 2 dx
(x sin x + cos x) I (x sin x + cos x)
ou
ad

II
Y

Let t = x sin x + cos x. Then,


xcosx 2 dx = J \ dt = 1 1
J
nd
Re

(xsinx + cosx) t t x sin x + cos x


Fi

Using this as a result and integrating by parts, we obtain


-1 -1
I - (x sec x) x —; J (sec x + x sec x tan x) x dx
x sin x + cos x x sin x + cos x
x sec x
=> I = - + fI sec 2 x dx
x sin x + cos x
-x (sin x - x cos x)
=> I = + tan x + C = +C
cos x (x sin x + cos x) x sin x + cos x
EXAMPLE 15 Find an anti-derivative of the function f (x) g" {x) -f " (x) g (x)
SOLUTION Required anti-derivative of /(x) g" (x) - f " (x) g (x) is given by
J (/ (x) g" (x) -f" (x) g (x)} dx

j / (x) g " (x) dx - J f"(x)g(x) dx


I II II I
{/(*)£'(*)- J /' (x) #' (x) dx j - j # (x) /' (x) - J /'(*)£' (x) dx J + C

= f (x) g' (x) -g(x) f (x) +C

ReadYourFlow.COM
1

INDEFINITE INTEGRALS 19.133

Evaluate : J sin 1 2x + 2
EXAMPLE 16 • dx
^ 4x2 + 8x + 13

SOLUTION Let
2a + 2 -1 2x+ 2
I = J sin -1 dx = sin
J (2a + 2)2 + 32
• dx
yJ4x2 + 8a +13

3 2
Substituting 2a + 2 = 3 tan 0 and dx = - sec 0 c/0, we get

| sin 1 3 tan 0 3 2 0 jq 3
- f 0 sec2 0 c/0
7 x — sec 0 c/0 = —
3 sec 0 2 2 J > „

w
/= ! J tan 0 c/0j = -| | 0 tan 0 - log | sec 01 j
0 tan 0 -
2

=>
3
I= -
f 2a + 2 tan 1 2a + 2

Flo
- log . 1 +
2i' + 2f
+C

ee
2 3 3 3

Fr
7=1 2a+ 2 2a + 2^ log ^4a2 + 8a + 13 • + C
=> tan -1 I
3
2 3
for
ur
-1 2x+2) | log (4a2 + 8a + 13) + C
=> 7 = (a + 1) tan
ks

3
Yo
oo

EXERCISE 19.25
B

LEVEL-1
re

Evaluate the following integrals:


ou

2. J log (a + 1) dx 3. | A3 log A dx
ad

1. J a cos a dx
Y

4. J xe* dx [NCERT] 5. J xe2x dx 6. \x2e x dx


nd
Re

f 2 9. J a sin 2a dx
7. j A COS A c/A 8. I a cos 2a dx
Fi

bg (log A) ^
10.
J 1 A
11. J A COS A dx 12. 1 a cosec 2 x dx

13. J A COS2 A dx 14. } A ' log A dx 15. Jf a"


dx

f 2 2 r 3 v2 f 3 2
16. J a sin a dx 17. j 2a e dx 18. A COS A dx

21. | (log a)2 dx


19. J x sin a cos a dx 20. | sin
23. r los^i*
a log (cos a) dx
x + sin x
a

22. J dx 24. J dx
1 (x+2)2 1 + cos A
-1
A COS
25. J log10 xc/x 26. | cos fx dx 27. j - dx [CBSE 2014]
Vl-X2
-1
28. J (xlogy
+ 1)2
dx [CBSE 2015] 29. J cosec3 a dx 30.
1 sec fx dx

ReadYourFlow.COM
19.134 MATHEMATICS-XII

sec 2x -1
31. J sin 1 -Jx dx 32. Jf x tan 2“ x dx 33. j * dx
sec 2x +1

34. J (x + 1) ex log (.wA) dx 35. J sin ^(3 a — 4 a2) dx 36. J sin -1 2x


2 [NCERT]
1+A
( . -1
-1 2>x-x
37. { tan dx 38. J a2 sin 1 a dx 39. J sin Y^dx [CBSE 2004]
1 - 3x2 A
A 2 tan -1 -1 1 - A2
40. j -dx 41. J cos 1 (4 a3 - 3a) dx 42. J cos dx
1+A2 l7?
43. J tan -1 2A
2 dx 44. J (a + 1) log a dx [CBSE 2002C]

low
1 -A
45. J a2 tan-1 x dx [CBSE 2012] 46. J (elog A + sin a) cos a dx
(a tan 1 a)
49. J *3 tan -1
1 48. | tan 1 (yfx) dx

ee
47. dx a dx
(1 + a2)3/2
rF
Fr
50. J sin a cos 2a dx
a 51. J (tan-1 a2) a dx
. -1
for
a sin
52. J ^ dx [CBSE 2012, 2016, NCERT]
u
Jl-x2
ks
Yo

LEVEL-2
oo

54. Jr sin 3
B

53. | sin 3 y/x dx a a dx 55. J cos3 dx


re

1 -
56. fI A cos 3 a dx 57. J tan -1 - dx [NCERT]
ou

1+A
ad

3.-12 2 . -1
Y

58. J sin -1 dx 59. J


A sin A A sin idx
=-— dx 60. J
2,3/2
(1 — A )
nd
Re

ANSWERS
Fi

1. sin a + cos a + C
a 2. A log (a + 1) - A + log (a + 1) + C
4 4
3. — log a — ——i- C 4. (a — 1) ex +C
4 ° 16
5. {—~ — c +C 2.v
6. -e x (a2 + 2a + 2) + C
U 4
2 x2 x sin 2a
7. a sin a + 2 a cos a - 2 sin a + C 8. :— sin 2a + - cos 2a - +C
2 2 4
A 1
9. - - cos 2a + - sin 2a + C 10. log a {log (log a) -1) + C
2 4
2 .
11. A sin a + 2 a cos a - 2 sin a + C 12. - a cot a + log | sin a | + C
2 H+l n+1
a sin 2a cos 2a
13. — + + +C 14. - log A - — 2+C
4 4 8 n+1 (« + D
l- » \-n
15. - log a - — 2+C 16. ~ x2, - — a" sin 2a - ~ a cos 2a + — sin 2a + C
1 -n (1-n) 6 4 4 8

ReadYourFlow.COM
INDEFINITE INTEGRALS 19.135

r2 2
17. ex (x -1)+C 18. - a:2 sin x2 + - cos x2 +C
2 2
19. x cos 2x + — sin +C 20. cos x (1 - log cos x) + C
4 8
21- Y|(iog^)2-log^ + |J + C 22. 2e^ (Vx-1)+C

1 log (x + 2)
23. - +C 24. x tan — + C
(x + 2) (x + 2) 2
1
25. - jx(logx-l)j +C 26. 2 x x + cos Jxl +C
log 10

27. - 1 -x2 cos 1x + xl + C 28. -i^ + log ^ +C


X +1 x +1

w
29. - — cosec x cot x + — log tan — + C 30. x sec-1 yfx - yjx -1 + C
2 2 2

Flo
2
31. ^ (2x -1) sin 1 V*" + -^ -J* - x2 + C 32. x tan x - log | sec x|----- + C
2

ee
2

Fr
33. x tan x - log | sec x|----- + C 34. x e* | log (x e*) -lj +C
2
-1 -1 x - log 11 + x2 | + C
for I
ur
35. 3 x sin x + 3 36. 2 x tan
3
37. 3 x tan-1 x log | x2 + 11 + C 1
38. — sin 1 x + —
ks

+C
3 3
Yo
oo

sin 1 * i 2
+x -1
- + log i + Vi x - — log 11 + x2 | - — (tan 1 x)2 + C
B

39. - +C 40. x tan


x x 2 2
re

41. 3 x cos -l x - 3 -Jl -x2 + C x - log 11 + x2 | + C


ou

42. 2 x tan
ad
Y

2^ 2
-1 x - log 11 + x2 | + C 44. x + —— logx- x +
X
+C
43. 2 x tan
2 4
nd
Re

3 x2 1 2
Fi

45. — tan 1 x - ~g- + ^ log | x2 +1| + C 1 2


46. x sin x + cos x + — sin x + C
3 2
tan -1 x x 48. (x + 1) tan 1 Jx -Jx +C
47. - +C
i1 2
+X +X
2

49. —tan ^--x2+-log+|x2+1|+C 1 x cos 3x sin 3x


50. - + x cos x - sin x > + C
3 6 6 6 ' ' 2 3 9
51. - x2 tan 1 x2 — T log (! +x4) + C 52. -Jl-x2 sin -1 x + x + C
2 4
3/2
53. - 3x cos 3 V* + 6 x1^ 3 sm3 yfx + 6 cos3 -Jx +C
3x cos x 3 sin x x cos 3x sin 3 x
54. - + +C
4 4 12 36
55. 3x3/2 sin3 V^ + 6x1/3 cos 3 + 6 cos3 sfx +C

ReadYourFlow.COM
19.136 MATHEMATICS-XII

x sin 3x cos 3x 3 3
56. + + — x sin x + — cos x + C
12 36 4 4
1 x (cos 1 x) - i - x2 + C
57. 58. x tan -1 — - -Jax + a tan -1
2 i? a
-l
1 . -1 2 x sin * x 1 , . -i . 2 1 2
59. - sin x +C 60. - (sin x) + - log (1 -x2) + C
2 2 2

HINTS TO NCERT& SELECTED PROBLEMS


4. / = [ x ex dx = x ex lx ex dx = x ex - ex + C
J I II J
x sin x 2 X
24. I =J rfx +J dx =— f x sec — rfx + f tan — dx
1 + COS X 1 + COS X 2 J I II 2 J 2

w
=> / = — i 2x tan — - f 1x2 tan — dx
tan - dx
2 1 2 2 2
X c X r X X

Flo
=> I - x tan----tan - dx + tan - rfx + C = x tan - + C
2 J 2 J 2 2
2 r

ee
32. J x tan2 x dx = | x (sec2 x-1) dx = J x sec x dx - I x dx
I II J

Fr
r x2 x2
= x tan x - tan x dx----- = x tan x - log sec x - -— + C
I j 2
for & 2
ur
r (1 - COS 2x) r 2 sin 2 x
33. I = x ------------- - dx = X dx = j x tan2 x dx
J (1 + cos 2x) J T2 cos 2 x
ks
Yo

2
oo

= J x (sec x -1) dx = j x sec2 x dx -J x dx = x tan x - J tan x dx - X^


B

2
re

X
= x tan x - log secx —+C
2
ou
ad

-1 2x
36. / = J sin dx = J 2 tan 1 x dx = 2 | tan - 1 x 1 dx
Y

1 + *2 I II
nd
Re

= 2 x tan 1 x - J 2 rfx ■ = 2 j x tan 1 x - ^log|l +x2|J + C


Fi

1+x
-1
x sin X
39. Let/ =
J dx and let sin x = t or, x = sin t. Then, dx = cos t dt
V1 - *2
1 =
It 1
sin t dt = -1 cos f - J (- cos t) dt = -t cos t + sin t + C
II

=>/=-/ V1 - sin 2 f + sin f + C = -x -x2 + x + C


50. / = J x sin x cos 2x dx = ^ J x (2 sin x cos 2x) dx = ^ j x (sin 3x - sin x) dx

=> / = — f x sin 3x rfx - — f x sin x dx


2J 2J
1 1
=> I = \ -—cos 3x4- f cos 3x dx —X COS X 4- J COS X rfxj
2 [ 3 J 2

ReadYourFlow.COM
INDEFINITE INTEGRALS 19.137

=> 1=1
2 3
cos 3x + — sin 3x
3 -H -xcos.r + sin a:| +C

57. Let I = } tan^J 11 +-Xx and x = cos 0. Then, dx = - sin 0 rf0

0^
I I tan 1 tan — (- sin 0) dQ = - f 0 sin 0 rf0
2J 1 II

1 1
=> I= - 0 cos 0 - J - cos 0 dQ ■ 0 cos 0 + sin 0
2 2
i | - 0 cos 0 + -^/l - cos2 0 l + C = -
I= -

low
19.11.1 INTEGRALS OF THE FORM J e'j/(x) +/'(,r)| J.v

THEOREM Prove that: J ex |/(x) + /' (x)| dx = ex f(x) + C

ee
PROOF We have,
rF
Fr
J ex [f(x) + f' (*)} dx = J ex f(x) dx +1 ex f (x) dx

i
=>
for
J ex [f(x) + f' (x)} dx = f(x) ■ ex -1 /' (x) ex dx + j ex f' (x) dx + C
u
=> J ex (/(x) + /' (x)} dx = ex f(x) + C Q.E.D.
ks
Yo

This theorem suggests the following algorithm to evaluate integrals of the form
oo

l **{/(*) +/■«} dx.


B
re

ALGORITHM
ou
ad

STEP I Express the integral as the sum of two integrals, one consisting of f (x) and other containing
Y

f'(x).
J ex |/(x) + /' (x) j dx =j ex f (x) dx + J ex f (x) dx
nd

i.e.
Re
Fi

STEP II Evaluate the first integi'al on RHS using integration by parts by taking ex as the second
function. The second integral on RHS will cancel out from the second term obtained by
evaluating thefirst integral.
NOTE The above theorem is also true if we have ekx in place of ex.

i.e.
J f(x)+f'(x)}dx = eto/(i) + C

ILLUSTRATIVE EXAMPLES

LEVEL-1
EXAMPLE l Evaluate:
\
(i) | <?* dx [NCERT] (ii) ^ ex (sin x + cos x) dx [NCERT1

(iii) | {sin (log x) + cos (log x)} dx

ReadYourFlow.COM
19.138 MATHEMATICS-XII

SOLUTION (i) Let / dx. Then, I - [ ex — + dx


J X r2
U /■
=> J = J e* i dx-\ ex \dx
n I x

=> I = — ex ex dx-\ ex dx+ C


x J x2 J x2
=> I = -i- ■ ex dx + C = — ex + C
x
(ii) Let

w
I = J ex (sin x + cos x) dx
f f

Flo
=> I = I ex sin x dx + | ex cos x dx
J II I J

ee
=> I = (sin x) ex - J (cos x) ex dx + J ex cos x dx + C = sin x + C

Fr
(iii) Let / = J {sin (log x) + cos (log x)} dx. Let log x=t. Then, x = e => dx = d(e) = e dt
) or
ur
I = j e1 (sm t + cos t) dt
f
ks

=> I = \ el sin t dt + [ el cos t dt


Yo

J II I J
oo

=> I = sin f • - J cos t -e dt + j cost ■ e dt + C


B

I = ^ • sin f + C = clog Y sin (log x) + C = x sin (log x) + C


re

=>
EXAMPLE 2 Evaluate:
ou
ad

(ii) J c x 2 + sin 2x
Y

(i) J ex (tan x + log sec x) dx dx [NCERT]


1 + cos 2x
nd
Re

1 + sin x cos x
(iii) J e1 dx (iv)K;i-sin*
Fi

2 dx [NCERT]
COS X - COS X

SOLUTION (i) Let

I = j ex (tan x + log sec x) dx = j ex log secx + tan x dx


< * f j

=> / = j ex log sec x dx + { ex tan x dx


n i
1
=> I = (log sec x) ex - J X sec x tan x ex dx + j ex tan x dx + C
sec x
=> / = ex log sec x - J ex tan x rfx + J ex tan x dx + C = ex ■ log sec x + C

(ii) Let J ex 2 + sin 2x dx. Then,


1 + cos 2x

ReadYourFlow.COM
INDEFINITE INTEGRALS 19.139

x 2 + 2 sin x cos x
I dx
2 cos2 x

=> I = J ex (sec2 x + tan x) dx = f ex tan x dx + f ex sec2 x dx


f' f J IT I J
x r 2
=> l = (tan x) e - I sec x e x dx + ^ ex sec2 x dx + C - ex tan x + C

1 + sin x cos x
(iii) Let / = J £■* dx
2
cos x

1 sin x cos x
=> I = K COS
2
X
+
COS
2
X
dx

w
=> I = J (sec2 x + tan x) dx
f f

Flo x
I = [ ex tan x dx + \ e sec x dx
2

ee
J II I J
x c 2

Fr
=> / = (tan x) e' - I sec x e x dx + | ex sec2 xdx + C = ex tan x + C
/ \
1 - sin x
for
ur
(iv) Let I = j ex dx. Then,
1 - cos X
ks

1 - 2 sin * cos -X
Yo

2x x
dx = Jr e xfI —
1 cosec
=K
oo

I 2 2 ----cot - dx
2x 2 2
2 sin
B

2
re
ou

x 1 2 x
ad

=> I
=K - cot — + — cosec
2 2 2
• dx
Y

/ /'
nd
Re

=> / = - f ex cot - dx + — f ex cosec2 - dx


J II I 2 2 J 2
Fi

^ x r 2*1 x . 1 r x 2x .
=> I =- cot — ■ e - - cosec ----- e dx > + — e cosec — dx
2 J 2 2 J 2 J 2
x X l r x 2 *
rfx + — [ cosec2 — dx + C
=> I = - e cot------ e cosec —
2 2 J 2 2 J 2
=> I = - ex cot — + C
2
EXAMPLE 3 Evaluate:
(i) f e* —dx [NCERT] (ii) f ^ X X- 3
3 dx [NCERT] (iii) { ^og *
2 dx
J (x + D2 (x-1) (1 + log x)

SOLUTION (i) Let / = f ex-----dx


j (* + D2
x x + 1-1
=> I 2 dx
(x +1)

ReadYourFlow.COM
19.140 MATHEMATICS-XII

1 -1
=> ,-Se* x + l +--------2
dx
(* + l)2
f /'
1 dx + \ ex (-1)
=> I = \ ex 2 dx
n x+l (x + l)
I
-1
I 1 X f eX dx + \ e x (-D T dx + C = —- ex +C
x+l J (x + l)2 (x + l) 2 x+l
(ii) Let / = J x-3
j dx. Then,
(x-1)

w
= cex(x^2dx = cx 1 (-2)
I dx
(x-1)3 1 (x-1)2 (x-1)3

=K Flo 1
/
-2
/'

ee
=> I c/x + J x 3dx
j ii (x-1)2 (^-D

Fr
I
1 -2
x exdx + J ex x
=> / =
for
ur
(^-D2 (x-1)3 (x-1)
x X
<?
ks

=> / = +2| -j rfx - 2 J —^—dx + C


Yo

(x-1)2 J (x-1)3 J (x-1)3


oo
B

=> I = —~2 +C
(*-D2
re

logx
(iii) I=\ dx. Let log x = t. Then, x = e => dx = d (e ) = e dt
ou
ad

(1 + log x)2
Y

t
te (f + l)-l eJ dt
I
=1 (f + 1)2 L
nd

(t+1)2
Re
Fi

1-1 t
=> I = 1 f+1
(f + 1)2
> e dt
/
/
1 e dt + j Kredt
I = 1
f+1 (f + 1)2 ~
I II
1 t f -1 e1 dt + J -1 t
=> I = e dt + C
f+1 J (t + D2 (f + 1)2
t
x
=> I = —+ C = +C
f+1 (log X + 1)
EXAMPLE 4 Evaluate:
1 1 1
(i)J !og* (logx)2
• dx (ii) J log (log x)
(log x)2
■ dx [CBSE 2010]

ReadYourFlow.COM
'I

INDEFINITE INTEGRALS 19.141

1
SOLUTION (i) Let / = | -—^ • dx- Putting logex=t or x = e and dx = e dt,
log* (log x)
we obtain

I =
1 +. (-D
- e‘dt=\
j t t2
n i
V r)
=> I = -
t
V_J t
e x \dt + C
t
=> I = I + C = ~^ + C
t logx
1 t t t

low
(ii) Let / = J ■ log (log x) + > dx. Let log x = t. Then, x = e => dx = d(e ) = e dt.
(log x)2

1 = J ■ logt + ~ ■ e dt

ee
rF
Fr
1 1 1
=> / = \ l°g ^ +-------+ —> e t dtj,
t t t2 for
1 i i t
u
=> I = | log£ + - e dt + j t + t2 e dt
ks

t
f f l f f)
Yo
oo

=> l = [ e log t dt + \ e* ■- dt + l el (-1/t) dt + [ e dt


B

J n i t ii i r
re

I = (log f) -1 y ■ / rft + J e1 ■- dt + je .e1 dt + j e dt + C


=>
ou
ad

x „
Y

=> I = . log t - - / + C = x log (log x) - -------+ C


t logx
nd
Re

LEVEL-2
Fi

EXAMPLE 5 Evaluate:

(i) j *
x x2 +1 .
^dx (ii) \ e* SLZ?)2 dx
(x + 1) (1 + x2)2

a: X2 +1
SOLUTION (i) Let I = j e 2 dx
(x + 1)
/
X
1 = 1^1- 21 dx = \ ex dx-2 j ex J
---- 2 «X
(x +1)2 ^ (x + 1)
x + 1 -1
■=>
l e1 -2 J ex • y dx
(X + 1)

1 -1
=> I =ex-2 jex dx
x+1 (x + l)2
/ /'

ReadYourFlow.COM
'

19.142 MATHEMATICS-XII

=> I = ex-2
hx- x +1 1 dx-j ex (x +~—2
1)
dx
II I

1 1
Z = ex - 2 < jexdx-fex -—T dx ■
x+1 (x + 1) (x + -r
1)

1 1
=>
x+1 (x + 1)2
^x-J • ~—2 dx +C
(x + 1)
x 2 e*
=> I = (? +C
x+1
x (l-^)2

low
(ii) Let / = | (? dx. Then,
2,2
(1 + x")
^ 1 - 2x + x2
I = je (1 + x2)2
dx

ee
x (1 + x2) + (- 2x)
rF
Fr
=> I = je 2,2
I (1 + * ) for
u
1 (-2x)
=> / = ■ dx
ks

1 + x2 2,2
(1 + x")
Yo
oo

/ /'
B

1
=> i-l + (1+^2)
dx + ^ e ^ (-2^
2,2
.X
re

d+^)
II I
ou

(-2x) ex±^L dx + C
ad

1 x r
=> / =
2,2
exrfx +J
Y

(1 + X ) (1 + x2)2
x
nd

/ = - %+C
Re

=>
1+X
Fi

/
2^ 1 + sin 2x
EXAMPLE 6 Evaluate: \e dx [CBSE 2010]
1 + cos 2x

2x 1 + sin 2x
SOLUTION Let I = \e dx
1 + cos 2x

1 + 2 sin x cos x
=> I7 -\e
f 2x dx
2 cos2 x

2 2x 2 -tanx +- sec2 x
=> / = sec x + tan x dx
2 / 2 /'
1 C 2v
=> I - \ e2x • tan x dx + — \ e ' ■ sec2 x dx
II I 2J

ReadYourFlow.COM
1

INDEFINITE INTEGRALS 19.143

2x 2x 2
2 e 1 f 2x
=> 1 = sec x ■ - dx + ~ e sec x dx + C
2 2J
i 1 2x . 1 2x 1 2x
I = —e tan x — sec2x dx + — [ e2x sec2 x dx + C = — e tan x + C
2 2 2J 2

EXERCISE 19.26
LEVEL-1

Evaluate the following integrals:

1. J ex (cos x - sin x) dx 2. T ix
1.x x
1 + sin x r x 2
3. dx [NCERT] 4. I c (cot x - cosec x) dx

w
1 + cos X

X —1
K
Flo
5. dx 6. J ex sec x (1 + tan x) dx
2x2

ree
7. J ex (tan x - log cos x) dx 8. J ex [sec x + log (sec x + tan x)] dx

F
x —1
9. J ex (cot x + log sin x) dx 10. J e* 3dx
or 1
(x + 1)
ur
f
sin 4x - 4 ^ 2-x
11. j ^ dx [CBSE 2010] 12. | 2 e* dx
ks

1 - cos 4x (1 - x)
Yo
oo

1+x - sin x
13. } eA' 14.} e x/l 2 dx
B

2dx [CBSE 2013]


(2 + x) 1 + COS X
re

15. } ex log x + - j dx 16. } ex log x + ^ dx


ou
ad

x
Y

.v - x2 sin -1J x + 1
17. J — |x (log x)2 + 2 log xj dx 18. }
nd

dx
Re

tJi-x2
Fi

1
19. } e2x (- sm x + 2 cos x) dx 20. } cA‘ tan 1" x + dx
l+x2

sin x cos x -1
21. J ex . 2
dx 22. J [tan (log x) + sec2 (log x)} dx
sm x

.v (^-4) 2* (1 - sin 2x
23. } c jdx [CBSE 2009] 24. } e dx [CBSE 2013]
(x - 2) 1 -cos2xy

ANSWERS
X
1. ex cos x + C 2. —2 + C 3. ex tan — + C
x 2
A
4. ex cot x + C 5. —+ C 6. ex sec x + C
2x

ReadYourFlow.COM
19.144 MATHEMATICS-XII

7. ex log secx + C 8. ex log (sec x + tan x) + C 9. ex log sin x + C


x
ex 12. -?— + C
10. _1_- + C 11. ex cot 2x + C
(x + D2 1-x
X
c 14. -e~x/2 sec {x/2) + C
+C 15. ex log x + C
x+2
,
16. ex log a: —
1 +C 17. e* (log x)2 +C 18. ex sin -l1 x + C
x
19. e2x cos x + C 20. ex tan- 1 x + C 21. ex cot x + C
gX
x tan (log x) + C 23. +C 24. -—e2xcotx + C
(x-2)2 2
HINTS TO NCERT & SELECTED PROBLEMS

low
x 1 + sin x x 1 + 2 sin x/2 cos x/2 x 1 2 x
3. 1 = j e dx=j e dx = \ ex tan — + — sec — dx
1 + cos x 2 cos2 x/2 2 2 2
/ /'

ee
2 x
rF
Fr
=> I = [ ex tan x/2 dx + j ex\ - sec dx
II I V2 2

=> 7 = tan^ gX [I560 2 ^ 1 e x sec 2 - rfx + C = ^ tan - + C


for
ex dx + \ —
2 J 2 2 2
u
ks

/ \
Yo

x —1 | f X 1 (-D l
h)dx+leX
oo

2x 2-^ = 1 ^
5. dx = dx
2x2 J I2* 2x2
B

/ /' n i
re

1 1 1 X ^
^ / = Jf —e / - ^ +{ O — dx+C = — e + C
ou

2x 2xZj 2x2 2x
ad
Y

x—1 1 -2
dx= r\ e x ---------
(^ ■*■ 1) —rdx=\
2 j r ex
nd
Re

10. / = J / dx
(x + 1)3 J (x + D3 J (x + 1)2 (x + 1)3
Fi

/ /'
1= f e*_I_rfx+f e*-t^x
=>
J (x + i)2 •’ (x + iy
II
x
(-2) (-2)
=> I = ex dx + j ex 3dx + C = ~ 2 +C
(x + l)2 (x + 1)3 (x + 1) (x + 1)2

sin 4x 4 > dx = ^ ex ■ cot 2x + (-2 cosec2 2x) • dx


n. 1=1 ex- 1 - cos 4x 1 - cos 4x / /

X X+2-1
13. I=\ e rfx = J ex ■ ■ dx
(x + 2)2

ReadYourFlow.COM
1

INDEFINITE INTEGRALS 19.145

1 1 1 ^
16. / = f ex log x + — - — H—— dx = f log x + — dx + [ ex
xx X 2 j x
f /J
j
J
f f)

17. I = \ ex ■ (log x)2 + — (log x) ■ dx 19. I = j e2x 2cosx + (- sin x) dx


f‘ X f _ f f
ax
19.11.2 INTEGRALS OF THE FORM J eax sm bx dx AND } e cos bx dx

In this section, we will discuss problems based upon integrals of the form J eax sin bx dx

and | eax cos bx dx. In order to evaluate this type of integrals, we may use the following
formulae:

w
THEOREM Prove that:
ax

Flo
ax g
(i) J e sin bx dx - -—^ (a sinbx -b cos bx) + C
a + b2
[CBSE2002]

ee
ax
e ax cos bx dx = c----- {a cos bx + b sin bx) + C

Fr
(ii) J
a2 + b2 for
ur
PROOF (i) Let / = J eax sin bx dx. Then,

I = f eax sin bx dx
ks

J I II
Yo
oo

ax cos bx f ax - cos bx
=> I = -e J ae dx
B

b b
re

=> / = - - eax cos bx + — [ eax cos bx dx


ou

b b J l ll
ad

sin bx
Y

=> I = - aeax dx
b b{ b i b
nd
Re

=> I = -- eax cos bx + eax sin bx -[ e ax sin bx dx


Fi

b b2 b2 J
i 2
=> / = -- eax cos bx + Ar eax sin bx -
b b2 P’
2 g
ax
T T a
=> = —— (a sinbx-b cos bx)
I+,T> b2
( 2 u2 eax
I a +b
=> —r- (a sinbx-b cos bx)
b2 b2
/
eax
=> I = 0(a sinbx -b cos bx) + C
a2 + b2
ax
e
(ii) Similarly, we can prove that J eax cos bx dx (a cos bx + b sin bx) + C Q.E.D.
a2+b2

ReadYourFlow.COM
19.146 MATHEMATICS-XII

ILLUSTRATIVE EXAMPLES

LEVEL-1
EXAMPLE 1 Evaluate:
(i) j ^ 2x sin 3 x dx (ii) J e x cos x dx

SOLUTION (i) Let / = I e 2a sin 3x dx. Then,

I = \ e 2x sin 3x dx
I II
2x cos 3x \ 1e2x cos 3x
=> I = e dx
3 3

=> I --e2x cos 3x + 2 \ e 2x cos 3x dx


3 3 I II

low
=> I =- — e2x cos 3x + — \ e sin 3x j2e 2x sin 3x dx
2x
3 3 3 3
1 4.2xA 4
=>
2 2X .
e cos 3x + — e sin 3x -
9
^

9 I* 2x sin 3x dx

ee
3
2 2x 4
rF
Fr
=> cos 3x + - e sin 3.t — /
3 9 9
2x
4 e
for
=> I + I = ---- (2 sin 3x - 3 cos 3x)
9 9
u
2x
ks

13 e
=> —I = (2 sin 3x - 3 cos 3x)
Yo

9 9
oo

2x
B

=> 1 = -— (2 sin 3x - 3 cos 3x) + C


13
re

(ii) Let I = j e X cos x dx. Then,


ou
ad

I = \ e x cos x dx
Y

J 1 II
nd
Re

=> I = e A sin x - J - e A sin x dx


Fi

=> I = e~ x sin x + [ e x sin x dx


J I II
=> 1 = A sin x + A (- cos x) - J (- e~ A) (- cos x) dx

=> 1 = e x sin x - e - X cos x - J e~ x cos x dx

=> I = e A sin x - e A cos x - /

=> 21 = e~ x (sin x - cos x)


-x
(sin x - cos x) + C
2
EXAMPLE 2 Evaluate:
ax cos (bx + c) dx (ii) | sin (log x) dx
(i) J c
(hi) J ex cos2 x dx

ReadYourFlow.COM
INDEFINITE INTEGRALS 19.147

SOLUTION (i) Let J = J e ax cos (bx + c) dx


I II
Integrating by parts, we get
ax sin (to + c) ^
I sin (bx + c) - J ae ax
b
ax
=> I sin (bx + c) - ^ eax sin (bx + c) dx

ax
eax cos (bx + c) _ r ax cos (to + c)
=> I = -— sin (to + c) - - ae
b b b •> b
ax ax
=> I = -— sin (bx +c) -— cos (bx + c) + - [ e ax cos (bx + c) dx ■
b ' b b b*

low
ax ax
, e . . a e ,, s a r
=> I = ----sin (bx + c) —------- cos (bx + c) + - I ■
b b b b
ax 2

ee
C . \ Cl ax \ ci T
=> / = — sin (bx + c) + —t- e cos (bx + c) —T I
b b2 b2
rF
Fr
2
, a , bax ( 1
=> ~Y sin (bx + c) + a cos (bx +c)|
l + 7i] “72"
for
b b
u
I (2,2
a +b ax , ,
ks

=> = r J a cos (bx + c) + b sin (bx + c) \


Yo

b1 b2 t J
oo
B

ax
e |a cos (bx + c) + sin (to + c )|
=> I
re

2 i2
a +b
ou
ad

ax
Hence, f eax cos (to + c) dx = j a cos (to + c) + b sin (to + c) j +
Y

J
aZ +b
, i z
nd
Re

(ii) Let 7 = J sin (log x) dx. Let log x = t. Then,* = e => dx = d(e) = e dt
Fi

I = f sin t e dt
J II I

=> 7 = - e cos t - J e (- cos f) dt [Integrating by parts]


I = -e cos t + [ e* cos t dt
J I II

=> I = -e cos f + 1sin 7 - J e t sin t dt

t t
=> I = -e cos t + e sin 7 - 7
=> 27 = e (sin 7 - cos 7)
t
=> 7 = ~ (sin 7 - cos 7) + C

Hence, J sin (log x) dx = ^ {sin (log x) - cos (log x)} + C.

ReadYourFlow.COM
19.148 MATHEMATICS-XII

(iii) Let / = | ex cos2 x dx. Then,


x(l + cos 2x
I=le 2
dx

=> I = ex (1 + cos 2x) dx

=> I = - \ ex dx + — [ ex cos 2x dx
2J 2J
I = - ex + -1^+0
2 2 1
where !;[ = J ex cos 2x dx.
Now,
U = [ ex cos 2x dx

w
J I II
x sin 2x r * sin 2x
h = e 2 J
e --------dx
2
[Integrating by parts]

=> II = — ex sin 2x - — [ ex sin 2x dx

Flo
ree
1 2 2J I II
x cos 2x

F
h = -ex sin 2x -— < - e dx •
2 2 2 or
ur
lx lx 1
h = - e sin 2x + — e' cos 2x - j ex cos 2x dx
f
2 4 4
ks

1 1 1
Yo

h ” 2 ex sin 2x + - ex cos 2x - 1
oo

4 4 1
1 ex (cos 2a: + 2 sin 2x)
B

=> /l + 1 7l = ^
re

5 1 X
=> / - e' (cos 2a: + 2 sin 2x)
4 l -
ou
ad

4
a:
Y

II - — (cos 2a: + 2 sin 2x) ...(h)


5
nd
Re

1 X ex
/ = —ex + — (cos 2x + 2 sin 2a:) + C
Fi

2 10
LEVEL-2
ax
EXAMPLE 3 cos b x dx and /2 = J eax sin bx dx, prove that
r. \
(i) («2+fo2)(/12 + /22) = c lax (ii) tan 1 h + tan 1 - = fo.
h) a
SOLUTION (i) We have,
ax
Il \ eax cos bx dx = T (a cos bx + b sin bx)
a2+b2
and.
ax e ax
h ~\ e sin bx dx = 0^ {a sinbx -b cos bx)
a2+b2

ReadYourFlow.COM
INDEFINITE INTEGRALS 19.149

lax
e j (a cos bx + b sin bx)2 + (a sin bx -b cos bx)21
h2+h2= (a2+b2)
lax lax
e (a2+b2) e
h2+h2 {a2 +b2) a2+b2
(a2 + b2) (/12 + /22) = c lax

h, = a sin bx -b cos bx
(ii)
h a cos bx + b sin bx
tan bx - -
=> h _ A [Diving Nr and Dr by cos bx]
h b
1 + - tan bx
a
12 tan bx - tan {tan 1 (b/a)}
=>

low
I1 1 + tan bx . tan - 1 (b/a)

h -ifb
=> = tan bx - tan
a

ee
-1 h -1 b
rF -1 b \ = ta.
-

Fr
=> tan - bx - tan => tan + tan
h a a
for EXERCISE 19.27
u
LEVEL-1
ks

Evaluate the following integrals:


Yo
oo

!• J eax cos bx dx [CBSE 2002] 2- | eax sin (bx + c) dx


B

3. J cos (log x) dx 4. J e2x cos (3x + 4) dx


re

r 2x 6.
ie 2x sin x dx [NCERT]
ou

5- I c sin x cos x dx
ad
Y

7. J e2x sin (3x + 1) dx [CBSE 2015] 8. J ex sin2 x dx


nd
Re

LEVEL-2
Fi

9. J ^3 sin (log x) dx 10. J e2x cos2 x dx


x
3 3
11. je -lx sin x dx 12.
1 x2 e* cos x dx
_____________________ ANS WERS
g ax e ax
1- T (a cos bx + b sin bx) + C 2. | a sin (bx + c) -b cos (bx + c) j + Cj
a2 +b2 a2+b2

^ | cos (log x) + sin (log x) j + C


3. - 4. £— 12 cos (3x + 4) + 3 sin (3x + 4) j + C
lx lx
5. £ (sin 2x - cos 2x) + C 6. £— (2 sin x - cos x) + C
8
2x x
7. £ 1 ex
12 sin (3x + l) - 3 cos (3x + l)j +C 8. — — (cos 2x + 2 sin 2x) + C
13 2 10

ReadYourFlow.COM J
19.150 MATHEMATICS-XII

2x e 2x
9. _ 1 | cos (log x) + 2 sin (log x) j + C io. £ + ----(cos 2x + sin 2x) + C
Sx2 4 8
-2x x 3
11. £ (- 2 sin x - cos x) + C 12. £ — (sin x 3 + cos x 3 ) + C
5 6
HINTS TO NCERT & SELECTED PROBLEMS
5- ^ = ^ { e2X (2 x cos x)rfx = i| e2x sin 2x dx
II
6- Let I = f e2* sin x dx. Then,
1 II
I = - e2'T cos x + 2 je2x cos x dx
I II

w
=> I = -e2x cos x + 2 {e2x sin x - 2 J e2x sin x dx)
2x 2x
=> I = -e cos x + 2e sin x - 4/
=>

Flo
51 = - e2x cos x + 2 e2x sin x

ee
2x
1 2x 2 2x £

Fr
I = - e cos x + — e sin x + C =---- (2sinx-cosx)+C
5 5 5
8. I = jex —c°s ^ dx = ^-j ex rfx - ^ J ex cos 2x dx for
ur
10. I = \ e2x<1+COs2x) dx = 2\ e21 dx + 2\ e2* cos 2x dx
ks

J 2 2J 2J
Yo
oo

19.12 SOME IMPORTANT INTEGRALS


B

In this section, we will prove three formulae which will be used in evaluating integrals of the
re

form J Vax2 +bx + c dx and J (px + q) V «x2 +bx + c dx.


ou
ad

THEOREM Prove that:


Y

(i) J yja2-x2 rfx = ^ x -Jo2 -x 2 + —


1 2 - -ifx
a sm - +C
nd

2 {aj
Re
Fi

^ a 2 log x + yja2 + x 2 + C
(ii) J -Ji?2 + x2 dx = ^ x .Ja2 + x2 + -.

(iii) | -Jx2 - a2 dx = -^ x -Jx2 -a 2 ^ a2 log x + Jx2 - a2 + C

PROOF (i) Let/=J ^/fl2- x2 dx. Then, / = J Ja2 - x2 1 • dx


II
Integrating by parts, we get
1/2
I = x-j ~(a2-x2) (0 - 2x) x dx
2
-x
=> I = x I r~2 2
yja -x
dx

,2 2, 2
(a -x)-a ix
=> I = X
1
ft -X
2

ReadYourFlow.COM
INDEFINITE INTEGRALS 19.151

I = xja2-x2 - { Ja2-x2dx + a2 f 1
=> dx
2
a - x2

1 =xft~7-I + + c

=> 2/ = x Ja2 -x2 + a2 sin 1 - + c


V /7

1 2 2 ,1 2 . -1 [ —
*
=> I — x a -x + — a sin + C
2 2
(ii) Let / = | ija2 + x2 dx. Then, I = J ^ja2 + x2 1 ■ dx
j n

Integrating by parts, we obtain


Hz 2
I = a +x x- J|(«2+x2r1/2 (0 + 2x) x dx

low
2
I = x ^ja2 + x2 - J x
=> dx
2 2
a +x

ee
(a2 + x2)-a2
I = x ija2 + x2 - J
=>
rF
dx

Fr
2
a2 + x
I - x -Ja2 + x 2 1
for
=> I + + a2 l 2 2
dx
u
a +x
ks

2 2 / 2 2
Yo

=> I = x Ja + x - I + a log |;t +Ja 4-x |


oo

21 = x -yja2 + x2 + a2 log | x 4- -Ja2 4- x2 |


B

=>
re

1 2 2 I 2 I 2 2
=> I = -x a 4- x + 2 1 x + V+ x I +
2
ou
ad

(iii) LetJ=Jx2-a2 Jx. Then, I = f Vx2-a2 1 - dx


Y

v J ! II
nd

Integrating by parts, we obtain


Re

2 2 f 1 / 2 2,-1/2
Fi

I = x -a •x-j -(x -a ) (2x) x dx


2
X
=> / = x
J2-fl2 J I 2
x -a
2
dx

(x2-a2) + a2 dx
=> 7 = x aH-J-J 2 2
x -a
1
=> 7 = x-J? - a 2 - J yjx2 -a2 dx-a2 \
2
dx
-a
2 2
7 = x Jx -a 2 - 7 - a2 log | x 4- Jx2 -a

=> 27 = x -Jx2 - a2 - a2 log | x 4- -Jx2 -a 2


1 x2 - a2 - — a2 log | x 4- -Jx2 - a2 | 4- C
7 = -x
2 2

ReadYourFlow.COM
19.152 MATHEMATICS-XII

19.12.1 INTEGRALS OF THE FORM [ Ja x2 + bx + c dx

In order to evaluate the above type of integrals, we use the following algorithm.
ALGORITHM
2 • ib c
STEP I Make coefficient of x as one by taking 'a' common to obtain x + - X + -.
a a

STEP II
bf 2 b c . .
Add and subtract — inx + - x + - to obtain x + — +
bb 4ac -b2
a a 2a
After applying these two steps the integral reduces to one of the following threeforms:
J f2+x2 dx, J f2 -X2 dx, \ fi2 -a2 dx.

STEP HI Use the appropriate formula.

ILLUSTRATIVE EXAMPLES

low
LEVEL-1
EXAMPLE 1 Evaluate:
(i) J j4x2 + 9 dx (ii) J -Jx2 + 2x + 5 dx

ee
[NCERT]
rF
Fr
SOLUTION (i) Let / = J ^4x2 + 9 dx. Then,

3^2
for
dx
u
ks
Yo

1 1(3)2. [~2 9
oo

-XJx2 + ~4 + 2{2J
7 = 2
2 bJlos x+r +4 +C
B
re

=> I = ^ ^4x2 + 9 + ^ log | 2x + ^4x2 + 9 | + C


ou
ad

(ii) ^ = J '\Jx2 + 2x + 5 dx
Y

=> I = j Jx2 + 2x + 1 + 4 dx = J -J(x + 1)2 + 22 dx


nd
Re
Fi

=> I = I(x + 1) J(x + l)2 + 22 + ^ (2)2 log (x + 1) + -^(x + l)2 + 22 + C

=> 7 = i (x + 1) Jx2 + 2x + 5 + 2 log (x + 1) + tJx2 + 2x + 5 + C

EXAMPLE 2 Evaluate:
(i) J J? x -10 -x2 dx (u) J J(x-3)(5-x) dx
SOLUTION (i) Let 7 = | ^-(x2 -7 x + 10) dx.Then,

49 49 i
I
IH*’ -7 x + ——— + 10 rfx
4 4

=> I ■ dx dx
1
ReadYourFlow.COM
INDEFINITE INTEGRALS 19.153

1 f 7 3^ 2 7 2 1 . -if x-7/2
=> / — x— x— +— sin +C
2f 2 v 2/ 2J 2l 2 J 3/2

=> 9 sin -1 2x-7^ + C.


/ = i (2x -7) J? x-10-x2 + —
8 3 J
(ii) Let J = J ^(x - 3) (5 - x) dx. Then,
I =| J- x2 + 8 x -15 dx

I J ^-jx2 - 8 x +16 -16 + isj dx


=>

=> / = (T-4)2 l2j dx = \ yjl2 -(x -4)2 dx

low
=> / =
|(x-4)^l2-(x-4)2+I(l)2sin -1 f x - 4 + C
1

=>
I = 1 (x - 4) ^(x - 3) (5-x) + ^ sin-1 (x-4)+C

ee
rF
Fr
1 +x
EXAMPLE 3 Evaluate: rfx
1 -x for
1 +x
SOLUTION Let / = dx. Then,
u
1 -x
ks

2
Yo

X (1 + x) x
oo

'= 1 Jl-x2
dx = J i=
-x2
dx + Jf
^77
rfx
B
re

- 2x
=> / = - = ^x - f
ou

2 J yjl-x2
ad

V1
Y

1
=> I = yjl-X2 - j yJl-X2 dx + j dx
nd
Re

sin -1 x + sm "1 x + C
Fi

I = Jl-x2 - ^Xy/l -x2

/= -
ijl -x2 - ^ x - x2 + ^ sin 1 x + C

Evaluate: J J
EXAMPLE 4 dx [NCERT]
1 + Jx
1 -yfx dx. Putting x = f2 and dx = 2t dt, we get
SOLUTION Let / =
1 + Vx
1 -t
I = .tdt = 2\
1 +t

-t2
=> /.} 2f dt + 2! dt

ReadYourFlow.COM
19.154 MATHEMATICS-XII

==> z = 2fjdt + 2| (1-f2)-1df

-2Z =~ dt + 2 j ^?dt-2\ 1

=> I = - 2^? + 2x||tA/1 -f2 +sin 1 f l -2 sin 1Z + C

=> I = - -sin 1 f + C = (1 -x) (Vx - 2) - sin 1 +C


2
X
EXAMPLE 5 Evaluate: J dx
^-2x-x2
2
X
Let / =J = dx . Then,

low
SOLUTION
-Jl -2x -x 2
2
-x (1 -2x-x2) + (2x-1)
dx = - j dx
A/l-2x-x2 yjl-2x-x2

ee
2x -1
rF
Fr
=> / = -1 -^1 - 2x - x2 rfx - J rfx
yjl-2x-x2 for
^ / = - J -^1 -2x -x2 rfx + J
-2x-2+3
dx
ou
-2x-x2
ks

- 2x - 2
=> J = -1 Jl - 2x - x2 rfx + | = dx + 3 J 1
oo

dx
Jl-2x-x Jl-2x-x2
Y
B

1 1
re

=> 7 = -J J(j2)2-(x + l)2 dx + f = rf(l -2x-x2) + 3 J rfx


-Jl -2x-x -J(V2)2 -(x + 1)2
ou
ad

- 2x - x2 +2 sin -1 X+1 + 2 yjl-2x- x2 + 3 sin" 1 (


Y

=> 7 = - +C
V2 V2,
nd
Re

EXERCISE 19.28
Fi

LEVEL-1
Evaluate the following integrals:
1. J J 3 + 2x - x2 dx [NCERT] 2. J tJx2 + x + 1 dx

3. 1 v^'-^2 dx 4. J + x - 2x2 dx
5. J cos x y 4 - sin 2 x dx 6. j Je 2x + 1 dx
7. J ^9 -x2 dx 8. J Jl6 x2 + 25 dx
9. -5 dx 10. J ^2x2 + 3x + 4 dx

11. j f-2x-2x2 dx 12. J xf*7ldx


fe+dogx)2
13. J i2 f6-x6 dx 14. j [CBSE 2005]
x

L
ReadYourFlow.COM
INDEFINITE INTEGRALS 19.155

15. | ijlax - x2 dx 16. I 1J3 -x2 dx


17. J Jx2-2xdx [CBSE2017] 18. J Jlx-x2 dx [CBSE 2017]

ANSWERS

1.^2) ■J3 + 2x -x2 + 2 sin 1 x-l} + C


2 2
2x +1 ^
2. Jx2 + x + 1 + 3 log (2x + 1) + ^Jx2 + x + 1 + C
4 J 8
3. i (2x -1) tJx - x2 + -^ sin 1 (2x -1) + C

4. I(4x-1) yll + x-2x2 + 9V2 sin


. -if 4x -1
1 +C

w
32 3
1 _1 ( sin x
5. - sin x -^4 - sin2 x + 2 sin +C
2 2

Flo
ree
6- ^ yle2X +1 + ^ log ^ + Ve 2x + 1 +C

F
7. -
1
+C or
ur
2 3J
f
o n 12 25 25 . 2 25 „
8. 2xjx + — + — log x + Jx +— + C
ks

V 16 8 6 V 16
Yo
oo

2 _5 _5 log x + jx 2 5
9. x lx +C
B

4 4 4
re

23 V2
10. ^ ^2x2 + 3x + 4 + ^log(x + |U x 2 +
+-x+2 +C
ou
ad

l 8 y 2
Y

11. i (2x + 1) ^3-2x-2x2 + sin _l f 2x +1 + C


nd
Re

6 3
Fi

12. I x2 jx4 + 1 + ^ log x2 + }/x4+l + C 61 x 3 16


13. - 6 A . -1 x
a -x + — sin

73 +C
6 a
14. ~ log x -J(log x)2 + 16 + 8 log | log x + -J(log x)2 + 16 | + C

2
15. ^ (x - a) J2ax ~x2 + sin
-1 f x - -Uc 16.
1 [ 2 3
x J 3 - x + — si
-l x „
-?= + C
a 2 M 2 {J3)
17. l(x-l) Vx2-2x-|log (x-l) + yjx2 -2x +C 18. i(x-l) ^2x-x2 +|sin_1(x-l)+C

______________________________________ HINTS TO NCERT& SELECTED PROBLEMS


1. We have.
7 = J ^3 + 2x -x2 dx = j J- (x2 - 2x - 3) dx = j j-j(x-l)2-22j rfx
7 =J ^22 -(x-1)2 rfx = i(x-l) V3 + 2x-x2 +2sin'1^1
=>

ReadYourFlow.COM
19.156 MATHEMATICS-XII

19.12.2 INTEGRALS OF THE FORM | (px + q) ^jax2 + bx+cdx

In order to evaluate this type of integrals, we use the following algorithm.


ALGORITHM
STEP I Express px + c] as
px + cj = X —d (ax 2 +bx + c) +p i.e. px + q = X (lax + b) + p

STEP II Obtain the values ofX and p bp equating the coefficients ofx and constant terms on both sides.
STEP HI Replace px + q by X (lax + b) + p in the integral to obtain
J(px + q) fax2 +bx + c dx-X j(lax + b) fax2 + bx + c dx + p Jfax2 +bx + c dx
n+ 1
STEP IV To evaluate first integral on RHS, use the formula J (/ (x))" f' (x) dx = - ^ ^

low
Evaluate second integral on RHS by the method discussed in the previous section.

ILLUSTRATIVE EXAMPLES

ee
rF LEVEL-1

Fr
EXAMPLE 1 Evaluate: for
(i) J (x -5) yjx2 + x dx (ii) | (3x - 2) fx2 + x + 1 dx [CBSE2014]
u
d ?
ks

SOLUTION Let (x -5) = A. — (x + x) + p i.e. x -5 = X (2x + 1) + p


Yo

dx
oo

Comparing coefficients of like powers of x, we get


B

1 A 11
1 = 2 A. and A + p= -5 => A = — and p = ----
re

2 2
1 (x~5)f2 + xdx
ou

I =
ad
Y

=> I = I( 2 (2x + 1) - "Vx2 + x dx


nd
Re

=> I = J ^ (2x + 1) ^x2 + x dx — J ^Jx2 + x dx


Fi

=> I = ^ J (2x + 1) -Jx2 + x dx j -Jx2 + x dx

2
=> dx, where t = x + x

=> —x
t 3/2 11 1
X+- J X+—
n2 n2
2 3/2 2 2 2 J Vv 2 2

i ri^i2
21 2
i
log I x + — + " n2
x+-
2
n2
2
+C

=> J = lt3/2 11 J^f2 + x -^log ^ + ^) + A/Ar2 + x +C


3 2

ReadYourFlow.COM
INDEFINITE INTEGRALS 19.157

3/2 11 2x + i rj
=> I = 3<l2 + l)
2 HrV^+x- ijQg + +%

(ii) Let3x-2 = X, — (x2 + x + 1) + n i.e. 3x-2 = X(2x + l) + n


dx
Comparing the coefficients of like powers of x, we get
2 X= 3 and ^, + p = — 2=>‘k = — 3 and ^ u = —7
2 2
I = J {3x-2)^jx2 + x + l dx

=> I = | • (2x +1) - ^ • tJx2 + x + 1 dx

f = -^ J (2x + 1) ijx2 + x + 1 dx - ^ J tJx2 + x + 1 dx


=>

low
r-\2
=> I = +
V3 2
dx, where t = x + x + 1
2

ee
rF
/ ,— n2

Fr
3/2 7 log x + ^ + ijx2 + X + 1 • +C
=> / = f x +x+1 +
4 2 for
I = (x2 +X + 1) 3/2 7
x + ij^pTTTl+|l°g
u
=> X+ + yjx2 + X + 1 • +C
ks

2
v
Yo
oo

EXAMPLE 2 Evaluate:
B

(i) J x -^/l + x - x2 dx (ii) J (x + 1) yjl - x - x2 dx


re

SOLUTION Let x = X — (1 + x - x2) + p i.e. x = X (1 - 2x) + p


ou
ad

dx
Y

Comparing the coefficients of like powers of x, we get


1 =-2X and A.+ u - 0=>X = — and p = —
nd
Re

2 2
Fi

I = J x^jl + x-x2 dx
| ^1 + x - x2 dx
I = — (1 -2x) + —
2 2

=> I \1 + x - x2 rfx + ^ J -Jl + x-x2 dx

=> / = - — x + -— —— l] dx
4 4 J
3/ 2 ^
=> I = -
1 t
2 3/2 kijfi'-B)' dx, where f =1 + x -x
2

=> / =-If3/2+I< x—
1
r-N2
V5 x— if + -ifvsf
— sm
. _1 x -1/2
+c
3 2 2 2 2y 2 2
<
V5/2 /

ReadYourFlow.COM
19.158 MATHEMATICS-XII

1 2x -1
=> / = —~ (1 + A' — x 2>3/2
) + 1, + x - x2 -h - sin 1 +C
3 2 8

(ii) Let x + 1 = A. — (1 - x - x2) + i.e. x + 1 = A (-1 - 2x) + ja


cix
Comparing the coefficients of like powers of x, we get
- 2 A = 1 and p-A, = 1 => A = - — and p = -
2 2
J (x + 1) \/l -x-x2 rfx = J | 1 1
I --(-l-2x) + f } t[^x-x2 dx
2 2
1 J (-1 -2x) ijl - x-x2 dx + ^ | -Jl - x -x2 dx
=> I = -
2

low
/ = ^ J (-1 - 2x) -x -x2 dx + 2 1 1
X 4- X +------- dx
4 4

A2 /
=> I = x+ -
if dx, where f = 1 - x - x
2

ee
2 2
rF
Fr
l(t^) 1
1 1 C 2 1 5 . -1 +1/2
=> / = - +— x+— 1 -x-x + — x — sin for +C
23/2 22 2 2 4 ■J5/2
V /
u
-i(1-x-*V'2 + l(2X + l)firx - x 2 + —165 sin. -1 2x +1
ks

I = +C
Yo

l V5
oo

LEVEL-2
B
re

Evaluate: J
x
EXAMPLE 3 dx
x-yfx2 -1
ou
ad

SOLUTION Let,
Y

x + ^[x2~-l
nd
Re

X
'=\ dx = J x ( -r- dx
x—^x2 — 1
[x-tf -1
Fi

X + xjx2 -1

x| x+yjx2 -1
2 dx x2 dx + | x Jx2 -1 dx
=> ^dx = { X
x2-(x2-l)

=> / = | x2 dx + ~ J -ft dt, where f = x2 -1

/=i! + I(3/2+c = *! + I(I2_1)3/2+c


3 3 3 3
EXERCISE 19.29
LEVEL-1
Evaluate the following integrals:

1. J (x + Vyfx2 - x +1 dx 2. J (x +1) yj2x2 + 3 dx

ReadYourFlow.COM
1

INDEFINITE INTEGRALS 19.159

3. J (2x-5) -Jl + 3x-x2 dx 4. J (x + 2) yjx2 + x + 1 dx

5. J (4j + 1) -J? -x - 2 dx 6. J (x-2) ^2? - 6 x + 5 dx


7. | (x + 1) ijx2 + x + 1 dx 8. J (2x+ 3) yx2 + 4x + 3 dx

9. J (2x -5) Jx2 -4x + 3 dx 10. |x tJx2 + x dx

11. j (x - 3) \x2 + 3x-18 dx [CBSE2014] 12. | (x+ 3) V3-4x-x2 dx [CBSE14,2015]

13. J (3x +1) V 4 - 3x - 2x2 dx [CBSE2016] 14. | (2x+5) V10-4x-3x2 dx [CBSE2016]


_________________________________ ANSWERS
(x2 - X + l) 3/2 |(2x-l) ^x2 -x + l +^log ^
4 - 4 ijx2 4 X 4 1 4 C
^ ^ 2y

w
V2 x 4 ^2x2 4 3
2. i(2x2 + 3) 3/2 4 - J2x2 4 3 4 3 V2 4C
— log

Flo
6 2^ V3

ee
2x - 3 J2 4 3x -x2 - — sin _ 1 r 2x - 3' 4 C
3. - — (2 + 3x -x2)3/2

Fr
3 2 M 4

4. ^(x2+x + l) 3/2 4 3 (2x 4 1) ^x2 4 x 4 1 4 ^ log ^ + ^ j + ^ + 1 +C


for
ur
8

+ ^(2x-l)Jx2 -x-2 - fiog|(-i)+^ -x-2


ks

4 . 2 3/2 4C
5- 3(X -x-2)
Yo
oo
B

2x - 3 2 0 5 1 . 2x - 3
6. — (2x2 - 6x + 5) 3/2 1 x - 3x 4 - 4 - log--------- 4 x2 - 3x 4 - Uc
re

6 V2 4 2 8 6 2 2
ou
ad

7. i(x2+i + l) 3/2 1 2x 41 ("2 7 3 . 11 r~i


4 - -------- Jx 4 x 41 4 — log XH--- 4-i/x 4X4l >• 4C
4 V 8 & 2/ V
Y

v!? 2
nd

8. - (x2 + 4x + 3) 3/2 -■ ^ (x 4 2) -Jx2 4 4x 4 3 ^ log (x 4 2) 4 Jx24 4x 4 3 • 4C


Re

3
Fi

9. ~(x2-4x+3) 3/2 -< ^(X-2) yjx2 -4x4 3 - logx-24i/x2-4x43 -40

10. ~ (x 4- X^)
2,3/2 - ^ (2x 4 1) -Jx2 4X4 1 1 f 1 j 4 -Jx2 4 X 4 C
O 16 { 2
11. ^(x2 + 3x-18) 3/2 ^ (2x 4 3) Jx24 3x-18 4 243 1log
8 V
3
X 4 —-
z
]W x'~ 4 3x-18 4 C

“ 2 7 . -ifX42
12. (3-4x-x2)3/2 + ^ (x 4 2) V 3 -4x-x 4-sin —f-- 4 C
3 2 V7 >
2,3/2 -—(4x4 3) V4-3x-2x2 - 205 . -i 4x 4 3 4 C
13. -l(4-3x-2x") -----= sin —p=^-
32 64^2 V41
„ „ 2 187 . -if 3x42
-(10-4x-3x2)3/2-f —(3x4 2) VlO -4x-3x
14. - 4—i=sm —7=— 4 C
9 18 9-/3 V34

ReadYourFlow.COM
19.162 MATHEMATICS-XII

Now, to determine constants we equate numerators on both sides. Some of the constants are
determined by substitution as in case I and remaining are obtained by comparing coefficients of
equal powers of x on both sides.
Following illustration illustrates the procedure.
3*-2
ILLUSTRATION 3 Resolve into partial fractions.
(x-1)2(x + 1)(x + 2)
3x - 2 A2 A3 , A4
SOLUTION Let +
(x -l)2(x + l)(x + l) x-1 (x-1)2 x+1 x+2
=> 3x-2 = A { (x-1) (x + 1) (x + 2) + A2 (x + 1) (x + 2)
+ A3 (x -l)2 (x + 2) + A4 (x -l)2 (x + 1) ...(i)
Putting x -1 = 0 or, x = 1 in (i), we get
1 = A2 (1 +11) (1 + 2) => A2 = -
6

w
Putting x + 1 = 0 or, x = -1 in (i), we get
-5 = A3(-2)2(-l+2)=> A3 =

Putting x + 2 = 0 or, x = - 2 in (i), we get

Flo
ree
-8= A4 (-;3)2(-1) => A4 = |

F
Now, equating coefficient of x 3 on both sides, we get
0 - A1 + A3i + A4 => A1 -~A3-A4 = ^ ^ or 13
ur
36
sf
3x-2 13 1 5 8
+ +
k

(x-1)2 (x +1) (x + 2) 36 (x-1) 6 (x-1)2 4 (x + 1) 9 (x + 2)


Yo
oo

CASE HI When some of the factors of denominator g(x) are quadratic but non-repeating.
B

Corresponding to each quadratic factor ax + bx + c. we assume partial fraction of the type


re

Ax + B
2 , where A and B are constants to be determined by comparing coefficients of similar
ou
ad

ax +bx + c
Y

powers of x in the numerator of both sides. In practice it is advisable to assume partial fractions
. . f A(2ax + b) B
of the type 2 -- + 2
nd
Re

ax + bx + c ax +bx + c
Fi

Following illustration illustrates the procedure.


2x -1
ILLUSTRATION 4 Resolve into partial fractions.
(x + 1)(x2+2)
2x-l /l Bx + C
SOLUTION Let + . Then,
(2 +1) (x2 + 2) X + 1 x2 + 2
2x -1 A (x2 + 2) + (Bx + C) (x + 1)
(x +1) (x2 + 2) (x + 1) (x2 + 2)
=> 2x -1 = A (x2 + 2) + (Bx + C) (x +1)
Putting x + 1 = 0 or, x •-= -1 in (i), we get
- 3 = /l (3) => 24 =-l.
Comparing coefficients of like powers of x on both sides of (i), we get
A + B = 0, C + 2 A =-l and C + B = 2
-1 + B = 0, C-2 = -1 [Putting 24 = - 1]
=> B = 1, C = 1.

ReadYourFlow.COM
INDEFINITE INTEGRALS 19.163

2x -1 1 x+\
+
(x + 1) (x2 + 2) x + 1 x2 + 2
CASE IV When some of the factors of the denominator g (x) are quadratic and repeating.
2 k
or every quadratic repeating factor of the type (ax + bx + c) , we assume 2 k partial fractions of
the form
Aq (2ax + b) A1 A-y (2ax + b) A2
+ + +-
ax2 + bx + c ax2 +bx + c (ax2 + bx + c)2 2 2
(ax + bx + c)

Aik - 1 (2ax + b) Alk


+ ... + • +
(ax2 + bx + c)k (ax2 + bx + c)k

Following illustrations will illustrate the procedure.


2*-3
ILLUSTRATION 5 Resolve - into partial fractions.

w
(x-i) Or+ i)
2x - 3 A Bx + C Dx + E
SOLUTION Let 2 • Tl-ien,
(x-l)(x2 +1)2 x-1

Flo
x2 +1 (x2 + 1):
2x -3 = A (x2+ l)2 + (Bx + C) (x -1) (x2 +1) + (Dx + £) (x-1)

ee
-(i)

Fr
Putting x = 1 in (i), we get
-1 =A(l+l)2=> A=-~
4
for
ur
Equating coefficients of like powers of x, we get
A + B = 0, C-B = 0, 2A + B-C + D=0, C+E-B-D =2and A-C-E = -3
ks

Putting A = -- and solving these equations, we get


Yo
oo

B = — = C, D = — and E = —
B

4 2 2
x +5
re

2x - 3 -1 x+1 +
(x-l)(x2+l)2 4 (x-1) 4 (x2 + 1) 2 (x2 + l)2
ou
ad

ILLUSTRATION 6 Resolve into partial fractions.


Y

x -1
2x 2x
nd
Re

SOLUTION We have, —^
x-1 (x-1) (x2 + x + 1)
Fi

2x A Bx + C
So, let + . Then,
(x-1) (x2 + x + 1) x-1 x2 + x + 1
2x = A (x2 + x +1) + (Bx + C) (x -1) -(i)
Putting x -1 = 0 or, x = 1 in (i), we get
2 = 3 A=> A = —
3
Putting x = 0 in (i), we get
A -C = 0=>C =
3
Putting x = -1 in (i), we get
2 4 2
- 2 = A + 2B-2C => - 2 = — - 2 B - — => B = -
3 3 3
2x 2 1 2/3x+ 2/3 2x _ 2 1 2 x+1
+
x3-l 3 x-1 x2 + x +1 x3 -1 3 x-1 3 *2 + x + i

We shall now use partial fractions in evaluating integrals containing rational algebraic
functions.

ReadYourFlow.COM
19.164 MATHEMATICS-XII

ILLUSTRATIVE EXAMPLES
LEVEL-1

Type I WHEN THE DENOMINATOR IS EXPRESSIBLE AS A PRODUCT OF DISTINCT LINEAR FACTORS


EXAMPLE 1 Evaluate:
x-1
(x +1) (x-2)
dx (u) J (x-l)(x2x-l
+ 2)(x-3)
dx [CBSE2005]

x3
(iii) | dx
(x-1) (x-2)
x-1 A B
SOLUTION (i) Let +
(x + l)(x-2) x + 1 x-2
=> x-1 = A (x-2)+ B (x + 1) ...(ii)

low
Putting x - 2 = 0 or, x = 2 in (ii), we get
1 =3B ^ B =1/3
Putting x + 1 = 0 or, x = -1 in (ii), we get
-2 =- 3A=> A- 2/3

ee
Substituting the values of A and B in (i), we get
rF
Fr
x-1 2 1 1_ 1
(x + 1) (x-2) 3 ’ x+1 + 3 ’ x-2
x-1
or
/ = dx
f
ou
(x + 1) (x-2)
ks

1 dx + —\ 1
=> 1 dx = |log|x + l| + |log|x-2|+C
oo

3 J x+1 3 J x-2
Y
B

2x -1 A B C
(ii) Let + ...(i)
re

(x-1) (x + 2) (x - 3) x-1 x + 2 x -3
ou

=> 2x -1 = A (x + 2) (x - 3) + B (x -1) (x - 3) + C (x -1) (x + 2) ...(ii)


ad

Putting x + 2 = 0or, x = -2in (ii), we get


Y

-5 = B (- 3) (-5) => B = -1/3


nd
Re

Putting x - 3 = 0 or, x = 3 in (ii), we get


Fi

5 = C (2) (5) => C =1/2


Putting x -1 = 0 or, x = 1 in (ii), we get
1 = A (3) (-2)=> A =-1/6
Substituting the values of A, B and C in (i), we obtain
2x -1 1111 1 1
(x -1) (x + 2) (x - 3) 6 x-1 3x + 2 2x-3
2x -1
/ = dx
(x-1) (x + 2) (x - 3)
1 1 1
=> / = -
6Jt I1
x -1
dx — [ —— dx + - [
3Jx + 2 2 •’ x - 3
dx

=> I = - t log I x —1| - log | x + 2| + ^- log | x - 3| + C


o o z
(iii) Here, the degree of numerator is greater than that of denominator. So, we divide the
numerator by denominator to obtain

ReadYourFlow.COM
INDEFINITE INTEGRALS 19.165

3 7x-6
= x+ 3+ -(i)
-1) (* - 2) (^--D (^-2)
7x - 6 A B
Now, let + .••(ii)
(x-l)(x-2) x —1 x -2
=> 7x -6 = A(x-2) + B(x-l) ...(iii)
Putting x - 2 = 0 or, x = 2 in (iii), we get: B = 8
Puttingx -1 = 0 or, x = 1 in (iii), we get: A=-l
Substituting the values of A and B in (ii), we obtain
7x - 6 1 8
+
(x -1) (x - 2) x —1 x-2
3
x 8
= x+ 3-— [From (i)]
x —1 x-2

w
(x-l)(x-2)
3
x
I = J (x -1) (x - 2) dx

Flo
2

ee
1 8
=> I
=J x+ 3- +
x —1 x -2
dx = ^ + 3x - log | x -1| + 8 log | x - 2| + C

Fr
EXAMPLE 2 Evaluate: or
ur
cos 0
wj (x2 + 1)2x(x2 + 2) dx [NCERT] (ii) J dQ
f
(2 + sin 0) (3 + 4 sin 0)
ks

1 1 - cos x
Yo

(iii) | dx [CBSE 2010] (iv) J dx


oo

sin x - sin 2x cos x (1 + cos x)


B

2x 2
SOLUTION (i) Let / = J dx. Putting x =t and 2x dx = dt, we get
re

(x2 + 1) (x2 + 2)
dt
ou
ad

'=1 (t + 1) (f + 2)
Y

1 A B
Let + ...(i)
nd
Re

(f + 1) (f + 2) f +1 t + 2
1 = A (t + 2)+ B(t + 1) ...(ii)
Fi

=>
Putting f = - 2 in (ii), we obtain: B --1
Putting f = -1 in (ii), we obtain: A = 1.
Substituting the values of A and B in (i), we get
1 1 1
(t + l)(t + 2) t +1 t + 2
1
I =J (t + l)(t + 2)
dt

1 1
I = 1 t+1 t+2
dt

=> I = log | f +11 - log | f + 21 + C = log | x2 + 11 - log | x2 + 21 + C


cos 0
(ii) Let 7 = J dQ.
(2 + sin 0) (3 + 4 sin 0)
Putting sin 0 = f, and cos Q dQ = dt, we get

ReadYourFlow.COM
19.166 MATHEMATICS-XII

dt
1
I (2 + 0(3 + 40
1 A B
Let
(2 + 0(3 + 40 2 +t 3 + 4t
1 =21(3 + 40 + 6(2 + 0 ...(ii)
3
Putting 3 + 4f = 0 i.e. t = - - in (ii), we get

1=b(2 - —B = -
4 5
Putting 2 + t - 0 i.e. f = - 2 in (ii), we get
l=2l(3-8)=>A=-i

Substituting the values of A and B in (i), we obtain

low
1 1 1 4 1
+—•
(2 + 0(3 + 40 5 2 + f 5 3 + 4f
1
W (2 + 0(3 + 40
dt

ee
1 dt + -\ —dt
rF
Fr
=> 1
si
2+f 5 J 3 + 4f
-log|2 + f|+|-ilog|3 + 4f|+C= log|2 + sin 6) + ^ log|3 + 4 sin 0|+C
for
u
(iii) We have.
ks

1
Yo

1 ! dx
oo

sin x - sin 2x
B

1 1 sin x
=> ' = \ dx = | dx = J dx
re

(sm x - 2 sin x cos x) sin x (1 - 2 cos x) 2


sin x (1 - 2 cos x)
sin x
ou
ad

=> I
1
(1 - cos2 x) (1-2 cos x)
dx
Y

Putting cos = t, and - sin x dx = dt or, sin x dx = - dt, we get


nd
Re

- dt -1
/
=1 J dt
Fi

a-t2) (1-20 (1-0 (1+0(1-20


-1 A B C
Let - + . Then,
(l-o (1+0 (1-20 1 -t 1+f 1 - 2f
-1 = A (1 + 0 (1 - 20 + B (1 - 0 (1 - 20 + C (1 - 0 (1 + 0 ...(i)
Putting f + 1 = 0 or, f = -1 in (i), we get
1
-1 =6B=> B =
6
Putting 1 - f = 0 or, f = 1 in (i), we get
-1 =-2A=> A =-
2
Putting 1 - 2f = 0 or, f = i in (i), we get

fl
-1=C
2 2 3

ReadYourFlow.COM
INDEFINITE INTEGRALS 19.167

-1 1 JL___ 1 1 4 1
(1-0(1 +0(1-20 2 1-t 6 1 +1 31-2t
-dt
l = I (l-o (i + o (1-20
dt - - r — - f -^—dt
dt--
2 J 1- 6 J 1+ 3 J l-2t
=> / = - ~ log 11 - f | - -^ log 11 + f | - ^ x - ilog|l-2f|+C
112
=> I = ~ — log 11 - cos x\ — log 11 + cos x\+ — log 11 - 2 cos x| + C
2 6 3
1 - cos x
(iv) Let / = | dx and let cos x = y. Then,
cos x (1 + cos x)
1 - cos x = !-y

w
cos x (1 + cos x) y(l + y)
i -y a B
Let ------- — = — +
y (1 + y) y 1+y

Flo
ee
=> 1 - y = A (1 + y) + By ...(h)

Fr
Putting y = 0 in (ii), we get A = 1. Putting y = -1 in (ii), we get B = - 2.
Substituting the values of A and B in (i), we obtain for
ur
1-y = 1___ 2_
y (1 + y) y 1+y
ks

1 - COS X 1 2
Yo

=> [v y = cos x]
oo

cos x(l + cos x) COS X 1 + COS X


B

1 - COS X
I
=1 dx = J 1 dx - [---------- dx
re

COS X (1 + cos x) cos x J 1 + cos x


2 dx - j sec ^ - J sec2 x/2 dx
I = J secxdx-J
ou

=>
ad

2 cos2 x/2
Y

=> I = log | sec x + tan x | - 2 tan x/2 + C


nd

(x -1) (x - 2) (x - 3)
Re

EXAMPLE 3 Evaluate: | dx.


(x -4) (x -5) (x - 6)
Fi

(x-1) (x - 2) (x - 3) A B C
SOLUTION Let = 1+ +
(x - 4) (x -5) (x - 6) (x-4) x -5 x - 6
Then, (x-1) (x - 2) (x - 3) = (x - 4) (x-5) (x - 6) + A(x-5) (x - 6)
+ B (x - 4) (x - 6) + C (x - 4) (x -5) ...(ii)
Putting x = 4,5 and 6 successively in (ii), we obtain
A = 3, B = -24 and C = 30
Substituting values of A, B and C in (i), we obtain
(x -1) (x -2) (x - 3) 3 24 30
= 1+ +
(x - 4) (x -5) (x - 6) x-4 x-5 x-6
(x-l)(x-2)(x-3)dY
1 -i (x - 4) (x -5) (x - 6)
1 dx + 30\ 1
=> / = J 1 • rfx + 3 J dx - 24 J dx
x -5 x-6
=> I = x + 3 log | x - 41 - 24 log | x - 51 + 30 log | x - 61 + C

1 ReadYourFlow.COM
19.168 MATHEMATICS-XII

Type II WHEN DENOMINATOR CONTAINS SOME REPEATING LINEAR FACTORS

EXAMPLE 4 Evaluate:
3x + l x2 + 1
<‘)J (x-2)2 (x + 2) dx [CBSE 2007] (ii) J (x-1)2(x + 3)
dx [CBSE 2012]

3x +1 A B C
SOLUTION (i) Let +
(x-2)2 (x + 2) .r - 2 (x ~ 2)2 x+2

=> 3x + 1 = A (x - 2) (x + 2) + B (x + 2) + C (x - 2)1 •••(ii)


Putting a: - 2 = 0 i.e. x = 2 in (ii), we get
7 =4B=> B = —
4
Putting a: + 2 = 0 i.e. a = - 2 in (ii), we get
-5 =16C => C=~ —

w
16
Comparing coefficients of a2 on both sides of the identity (ii), we get

Flo
A+C = 0=> A = = —
16

ree
Substituting the values of A, B and C in (i), we get
3a+ 1 5 1 7 1 5

F
+—
(x-2)2(x + 2) " 16 ’ a-2 4 (x - 2)2 16 (a+ 2)
or
ur
3a +1
/
= 1 (x-2)2 (x + 2) dx
sf
1 1
k

dx - — [
Yo

=> / dx + dx
ii (
oo

16 J a- a-2)2 16 J A + 2
B

I = ~7 i°g I A ~ 2 | - —— - log I a + 21 + C
re

16 4 (a-2) 16
(ii) We have.
ou
ad

a2 +1
Y

dx
(A-l)2 (A+3)
nd
Re

A2 +1 A B C
Let + +
Fi

(a-l)2 (a + 3) A —1 (A-l)2 A+ 3

=> a2 + 1 = A(a-1)(a+3) + B(a+3)+C(a-1)2 ...(ii)


Putting a -1 = 0 i.e. a = 1 in (ii), we get
2 = 4B=> B =-
2
Putting a + 3 = 0 i.e. a = - 3 in (ii), we get
10=16C=>C =-
8
2
Equating the coefficients of a on both sides of the identity (ii), we get
1 = A + C => A = 1 -C = 1-5 = 3
8 8
Substituting the values of A, B and C in (i), we get
x2 +1 3 1 1
---------- + — • -—~ + — (a + 3)
(x-1)2 (x+3) 8 A —1 2 (A-l)2 8

ReadYourFlow.COM
INDEFINITE INTEGRALS 19.169

A.'2 + 1
dx = * \ 1 dx + - \ —-—x- dx + —[ 1 - dx
=> I = 1(x-1)2(x+3) 8 J x —1 Z^x-!)2 8Jx+3

=> / = - log I x -11------ ----- + - log | x + 31 + C


8", 61 1 2(x-l) 8 61
2
X +X+1
EXAMPLE 5 Evaluate: | dx.
(x-D3
X2 + X + 1
SOLUTION We have, 1 = J — dx. Putting x — 1 = f and dx = dt, we get
(x-D
(t + 1)2 +(t + l)+l f2 + 3f + 3 i 3 3 ) ^
'=j t3
dt =
I t3 ^=1 - + —+ -j dt
t t r

low
=> 7 = log111 -1 ^ + C = log|x-l|---- ^
t 2r ^ -1 2(x-l)
NOTE This sum can also be done by using partial fractions. We write
2
B C

ee
X +X+1 A
rF
Fr
(x-1)3 x-1 (x-1)2 (x-1)3
2
X
EXAMPLE 6 Evaluate: J dx
for
u
ks

SOLUTION We have.
Yo

2
oo

X
dx
B

(X-1)3(I + 1)
re

2
x B C D
Let (x-1)2 + (x-1)3 + x + 1 -(i)
ou

(x-1)3 (x + 1) x-1
ad
Y

=> x2 = A (x -l)2 (x + 1) + B (x -1) (x + 1) + C (x + 1) + D (x -l)3 ...(h)


nd

Putting x-1 = 0 i.e. x = 1 in (ii), we get


Re
Fi

1 = 2C ^ C = -
2
Putting x + 1 = 0 i.e. x = -1 in (ii), we get
1=-8D => D =- —
8
Putting x = 0 in (iii), we get
0 = A-B + C-D => A - B = ~-
8
Putting x = 2 in (ii), we get
4 = 3A + 3B+3C+D=> 4 = 3(A+ B + C) + D=>A + B = ^
8
7
Now, A - B = - — and A + B = — => A = - and B = -
8 8 8 4
1 1
Thus, we have A = -, B = 2 c-I D —
4,C'2'
8 8
Substituting the values of A, B, C and D in (i), we get

ReadYourFlow.COM
19.170 MATHEMATICS-XII

2
x 1 3 1 1
(x-l)3(x + l) 8(^-l) 4 (x -l)2 2 (x -l)3 8(x + l)
2
X
/ = dx
(x-l)3(x + l)
Ifl, 3r 1 , Ir 1 Irl ,
=> I
8 J x-1 4 J (x_1)2 2 J (*-i)3 8 J x+1
1
=> I = ^log|x-l| ^ -^l°g|x + l|+C
8 4 (x-1) 4 (x-1)2
x-1 3
=> / = l8 l°g x+1
-—- + C
4(^-l) 4(x -l)2
Type 111 THE DENOMINATOR CONTAINS IRREDUCIBLE QUADRATIC FACTORS

w
EXAMPLE 7 Evaluate:
8 X
dx [CBSE2013] (ii) J (x-1)(x2+4) dx
©J(x + 2) (x2 + 4)

Flo
ee
8 A Bx + C
SOLUTION (i) Let

Fr
(x + 2) (x2 + 4) x+2 x2 +4
Then, 8 = A (x2 + 4) + (Bx + C) (x + 2) for (ii)
ur
Putting x + 2 = 0 i.e. x = - 2 in (ii), we get
ks

8 = 8A^>A =1
Yo
oo

Putting x = 0 and 1 respectively in (ii), we get


eB

8 =4A + 2C and 8 = 5A + 3B + 3C
Solving these equations, we obtain
r
ou

A = 1, C = 2 and B = -l.
ad

Substituting the values of A, B and C in (i), we obtain


Y

8 1 -x+ 2
+
nd
Re

(x + 2) (x2 + 4) x+2 x2 +4
Fi

/ =
I (x + 2) 8(x2 + 4) dx
1 dx + j
-x + 2
=> I dx
x+2 x2 + 4

=> / = f —-— dx - f — dx + 2 f —^— dx

=> I = log | x + 21 - i J ydt + 2x^-tan —1 - X 9


+ C, where t = x +4
2
I = log | x + 21 - ^ log t + tan -1 ^ + C = bg | x + 21 - ^ log (x2 + 4) + tan _1 ^ + C

x A Bx + C
(ii) Let -(i)
(x -1) (x2 + 4) x-1 x2 + 4
x = A (x2 + 4) + (Bx + C) (x -1) ...(ii)

ReadYourFlow.COM
INDEFINITE INTEGRALS 19.171

Putting x = 1 in (ii), we get: l =5A


Putting x = 0 in (ii), we get: 0 = 4A - C
Putting x = -1 in (ii), we get: -1 = 5/1 + 26 - 2C
Solving these equations, we obtain
A =—, B = — — and C = —
5 5 5
Substituting the values of A, B and C in (i), we obtain
1 4
x+-
x 1 5 5
+
(x -1) (x2 + 4) • 5 (x -1) x2 + 4
x 1 1 (*~4)
=>
(x-\) (/ + 4) 5 (x -1) 5 (*2 + 4)

1 1 x-4 ■ dx = - [ 1 dx

w
x
/
= 1 (x -1) (x2 + 4) dx = j 5(-v-l) 5'x2 + 4 5 J x-1 5J
\ dx

Flo
1 2* dx 4 r z-----dx
1
=> /
41 x-1
dx-— [
10 J x2 + 4
j +-
5 J x2 + 4
j

ree
=> / = ilog|x-l|-^log(x2 + 4) + |xItan-I| + C

F
or
ur
=> / = i log | x -11 log (x2 + 4) + | ^ -1 X + C
f
2
ks

IMPORTANT NOTE If a rational function contains only even powers of x in both the numerator and
Yo
oo

denominator, then to resolve it into partial fractions, we proceed as follows:


B

STEP I Put x =yin the given rational function.


re

STEP II Resolve the rational function obtained in step I into partial fractions.
2
ou

STEP III Replace y by x .


ad
Y

EXAMPLE 8 Evaluate:
2 x2+l
X
[CBSE 2013, 2014] (ii) j
nd

Wj (x2 + 1) (x2 + 4) dx dx
Re

(x2 + 2) (2x2 + 1)
Fi

SOLUTION Let x2 = y. Then,


2
x y
(x2 + 1) (x2 + 4) (y +1) (y + 4)
A B
Let y + ...(D
(y +1) (y + 4) y+1 y+4
=> y = /I (y + 4) + 6 (y + 1) ...(ii)
Putting y = -1 and y = - 4 successively in (ii), we get
A = -— and 6 = —.
3 3
Substituting the values of /I and 6 in (i), we obtain
y 1 4
(y +1) (y + 4) 3 (y + 1) 3 (y + 4)

ReadYourFlow.COM
19.172 MATHEMATICS-XII

2
Replacing yby x , we obtain
2
x 1 4
+
(x2 + 1) (.v2 + 4) 3 (.t2 +1) 3 (a2 + 4)
2
A 1 4 1
1 (A2 + 1) (A2 + 4) dx=\ 3 (a2 +1) 3 (a2+ 4)
dx
A2 + 1
dx + — [ — dx
3 J r2 + 4

1 -1 4 1 -1 x ^ 1-1 2 —l f A
=>/=-— tan a + — x — tan + C =---- tan a + — tan — +C
3 3 2 2 3 3 2
(ii) Let a2 = y. Then,
a2 +1 y+i
(a2 + 2) (2a2 + 1) (]/ + 2) (2y + 1)

low
y+i A B
Let + ...(i)
(y + 2) (2y + 1) y+2 2y + l

=> y + 1 = A (2y + 1) + B (y + 2) -(ii)


Putting y + 2 = 0 i.e. y = - 2 in (ii), we get

ee
1
rF
Fr
-1=-3,4=>A=-
3
Putting 2y + 1 = 0 i.e. y = - - in (ii), we get
for
u
1
=Bf22 1
ks

=> B = -
2 3
Yo
oo

Substituting the values of A and B in (i), we obtain


B

y+1 1111
+—
re

(y + 2) (2y + 1) 3 y+2 3 (2y + 1)


2
ou
ad

Replacing y by a , we get
Y

A2 + 1 1 1 1
+
(a2 + 2) (2a2 +1) 3 x2 + 2 3(2a2 + 1)
nd
Re

A2 + 1 1
Fi

I \ dx=2r i ^a + — f dx
(a2 + 2) (2a2 + 1) 3 jr2 + 2 3 J (V2a)2 +1

1
=> / = — x —= tan -l
A -1 A + tan 1 V'2 a l + C
—7= tan-1 (V2^) + C = — tan -7=
3 -J2 V2j 3^2 3^/2 V2
LEVEL-2
A+1
EXAMPLE 9 Evaluate: J dx
a\2
a (1 + x e )

A+1 (a +1)
SOLUTION Let I =
J a (1 + ae )
A\2
dx =|
a (1 + a ex)2
dx

Let xex =f.Then d (xex) =dt or, (a + 1) exdx = dt.


I
I =
J t(l ' t)
2dt' where t=xex.

ReadYourFlow.COM
INDEFINITE INTEGRALS 19.173

1 A B C
Let — +----- +
t 1+t (1+02

=> 1 = A(l+t)2 + Bt (l+ t)+Ct ...(ii)


Putting t = 0 and f = -1 respectively in (ii) we get
A =l,C = -1
Now, putting f = 1 in (ii) we get
1 = 4A + 2B + C => 1=4+25-1 => B=-l
Substituting the values of A, B and C in (i), we get
1 11 1
i(i+o2 t 1+t (1+02
1 1 1
I =1 t(l+t) 7 1+t (1 + t)2
dt

low
=> 1 = log I f I - log 11 + f I + — + C = log (xex) - log (1 + x ex) + —1-— + C
1+t 1 + xe

Evaluate: J 1
EXAMPLE 10 dx

ee
X+ ■yjx2 - X + 1
rF
Fr
1
SOLUTION Let I = J dx
X + ■yjx2 - X + 1
for
u
Let x + -Jx2 - x + 1 = t. Then,
ks
Yo

t2-l
Jx2 - x + 1 = f-x => x2 -x +1 = (f-x)2 => -2x + 1 = f2 -2tx => x =
oo

2t-l
B

(2t -1) 2t - 2 (f2 -1) 2t2 -2t + 2


re

dx = dt = dt
(2t -l)2 (2f -l)2
ou
ad

Substituting these values, we get


Y

2f2 - 2t + 2 t2-f + l
I =
Ir dt = 21 f (2t-l)2
dt
nd

(2f-l)2
Re

t2 -t + 1 A
Fi

B C
Let t + 2t-l
t (2t -l)2 (2t -1)2
3
Using cover-up method, we obtain A = 1 and C = — •
From (i), we obtain
f2 -t + 1 = A (2t -1)2 + B(2t-l)t+ Q
2
On equating the coefficient of t on both sides, we get
1 = 4A ^ 2B => B = - -
2
Substituting the \'alues of A, B, C in (i), we get
t2-t + l 1 3 3 1
f(2t-l)2 1 2(2t-l) 2 (2t -l)2

I = 2 f - dt-3 \ dt + -\ ---- 1—T dt


J t J 2t-l 2J (2t -l)2

ReadYourFlow.COM
19.174 MATHEMATICS-XII

r = 21ogf-|log(2f-l)-| 1
+C
4 (2f -1)

=> I = 2 log X + -X+1 -1^1(2*-!)+ 2^/7 — x +1


3
+C
4 |(2x -1) + 2 -Jx2 — x + 1J

sin x
EXAMPLE 11 Evaluate: | dx
sin 4x
SOLUTION We have,
sin x sin x sin x
/ =
J
sin 4x
dx
=1
2 sin 2x cos 2x
dx = I —
4 sin x cos x cos 2x
J 2l
dx

low
1 1 cos X
=> 1 =
4 J cos x cos 2x
dx = -
4 J
cos2 x cos 2x
dx

cos X
=> I
iJ (1 - sin2 x) (1-2 sin2 x) dx

ee
Putting sin x = t and cos x dx = dt, we get
rF
Fr
i = i f dt
4 J (1 -t2) (1 - 2f2)
for
u
Let t2 =y. Then,
ks

1 1
Yo
oo

(1 -t2)(l -2f2) (l-y)(l-2y)


B

1 24 B
Let . Then,
re

(1-y) (1 - 2y) 1-y l-2y


1 = 24(l-2y) + B(l-y)
ou

-(i)
ad

1
Putting y = 1 and y = - respectively in (i), we get: A =-l and B =2.
Y

2
1 -1 2
nd
Re

+
(l-y)(l-2y) 1-y l-2y
Fi

1 1 2
=>
(1 -t2)(l -2f2) 1-t2 1 - 212
dt i
=^> / =14 JJ (1 -f2)(l -2f2)
+ 2 ^dt
-
i-r i -2t
1
=> / = -
il 1 -t 2dt + l\ l-(V2t)
1 1 i 1+f 1 X ---1 7= .log 1 + V2 t + C
=> l = -- x- log +-
4 2 6 i-t 2 2 V2 6 \-Jit
1 1+f 1 l+Jlt
=> 1 = --^og log +C
8 1-f 4^2 1 -V2f
1 + sin x 1 + -J2 sin x
=> I = Tlog T^log +C
8 1 - sin x 4 a/2 1 -y2 sin x

ReadYourFlow.COM
INDEFINITE INTEGRALS 19.175

tan 0 + tan 3 0
EXAMPLE 12 Evaluate: J dQ
1 + tan 3 0
SOLUTION We have.
tan 0 + tan 3 0 tan 0 (1 + tan2 0) tan 0 sec2 0
I =
I rf0 = J rf0 = J dQ
1 + tan 3 0 1 + tan 3 0 (1 + tan 3 0)
2
Putting tan 0 = t and sec 0 rf0 = dt, we get
t dt t
W (i + r3) J (i + 0(f2-f + i)
dt

t A Bt + C
Let Then,
(i+f)(i-t+f2) 1+t 1-t + t2

t = A(1 -t + t2)+(Bt + C)(t + l)

low
Putting l+t = 0or,£ = -lin (i), we get: A =----
3
2
Comparing the coefficients of t on both sides of (i), we get

ee
A + B = 0=> B = - A = —
3
rF
Fr
Comparing, constant terms on both sides of (i), we get
-A=I
A + C = 0=> C =
for
3
u
1 1
-f+ -
ks

t 1 3_ _ _ 1 1 t+1
+ 3
Yo

3(t + l) 3 (i_f + f2)


oo

l + t3 3(l+f) 1 -t + t2
B

f+ 1
=> I =- dt + dt
re

3 Jl + t2-t + 1
1 r 2t-1 + 3
ou
ad

=> I =- dt + dt
3 J1 + 6 f2 -f+ 1
Y

1 2f -1 3 f 1
=> dt + - \ dt
nd

I = - dt + s) 2
Re

1+t 6 J t2-t + l 6J r-f+ i


Fi

|log|l+f| + ^log|/2 -f + l| + | J 1
=> 2dt
ip + V3
t~-
2 2

1
t-
| log 11 + 11 + -^ log | f2 - f + 11 + | x -1 2 +C
=> tan
V3
2 2
-if 2f -1
I = - | log 11 + f I + ^ log | f2 - f + 11 + -^ tan +C
V3 J
_1 f 2 tan 0-1
=> / = - — log 11 + tan 01 + - log | tan 2 0 - tan 0 + 11 + -^= tan +C
3 6 *J3 V3
1
EXAMPLE 13 Evaluate: | 2 dx
sin x (2cos x-1)

ReadYourFlow.COM
19.176 MATHEMATICS-XII

SOLUTION Putting cos x = t and d (cosx) = dt or, -sin x dx = dt, we get


1 1 dt
dx = j x-
sin .v (2 cos2 x -1) sin x (It2 -1) sin x

'=~\ 1
=> dt
(1 -t2) (It2 -1)

1 2 1 2 dt - 2 j -J— it
dt =-J
1-t2 2f2 -1 1 -f 2t2 -1
1
=> I 2dt
i1-0-/42)
yflt -1 1 1 + COS X 1 V2 cos x -1
=> 1 +C = -2 108 I - cos x +c
-Jit +1 V2 Jl cos x + 1

low
____ EXERCISE 19.30
LEVEL-1

ee
Evaluate the following integrals:
2x +1
rF 1

Fr
M (x + 1) (x - 2)
dx 2.
I x(x - 2) (x - 4) dx
for
x2 + x -1 3 + 4x-x2
3. dx 4.
I (x + 2) (x -1) dx
u
x +x-6
ks

2
Yo

r x2 +1 X
I (x-1) (x - 2) (x - 3) dx
oo

5. dx 6.
J “2
x -1
B
re

5x x2 +1
7.
1 (* + U (*2-4) dx [NCERT] 8.
J x(a:2-1)
dx
ou
ad

3
2x - 3
Y

X
9.
(x2 -1) (2x + 3)
dx 10.
I (x-1) (x - 2) (x - 3) dx
nd
Re

sin 2 x 2x
M (1 + sin x) (2 + sin x) dx [CBSE 2004] 12.
I (x2 + 1) (x2 + 3) dx [CBSE 2004,11]
Fi

X2 + X + 1
13.
I x log x (21 + log x) dx 14.
1 —x----------------
(X2 + 1) (x + 2)
dx [CBSE 2015]

2
ax +bx + c
15.
1 (x - a) (x -b) (x - c)- dx, where a, fr, c are distinct.

x 1
16.
J (x2+l)(x-l) dx [CBSE 2013] 17.
1 (x-1) (x + 1) (x + 2) dx
2 5x2 -1
X
dx [CBSE 2013] 19. J dx
(x2 + 4) (x2 + 9) x (x -1) (x + 1)

x2 + 6x - 8 , x2+l
20.
1 -----5--------- dx
x- - 4x
21.
I (2x +1) (^2^1) dx

ReadYourFlow.COM
INDEFINITE INTEGRALS 19.177

1 23. f —^----- dx
22. j dx [NCERT]
x 16 (log x)2 + 7 log x + 2 j J x(xn + l)

x x2+l
24. j
/(x 2 -a2\)(x
/ 2 -fcu2\)
dx & { (x2 + 4) (x2 + 25)
dx [CBSE 2013]

X3 + X + 1 , 3x - 2
26. J -----=--------dx 27. dx [CBSE 2013]
x2 -1 (x + l)2 (x 4- 3)
2x +1 x2 +1
28. j 2 d* 29. dx
(x + 2) (x - 3) (x - 2)2 (x 4- 3)
2
X X
30. j dx [NCERT] 31. 2 dx
(x-1)2 (x + 2) (x-1) (x + 1)

low
X2 +X-1 2x2 + 7x - 3
32. dx 33. dx
(x + 1)2 (x + 2) x2 (2x +1)
5x2 + 20x + 6 18
34. J dx 35. dx [CBSE 2013]
x3 + 2x2 + x

ee
(x + 2) (x2 + 4)
5
rF
Fr
x
36. dx 37. dx [CBSE 2002,05]
(x2+\)(x + 2) (x + 1) (x2 +1)
for
1 1
38. 3dx 39. dx
1 + X + X2 + X
i (x + 1)2 (x2 + 1)
u
ks

\ 1
^>7 -1 dx dx
Yo

(x2 + 1) (x2 + 4)
oo

2
B

x 3x + 5
el (x2 +l)(3x2 + 4) dx 43.
L3 dx [NCERT, CBSE 2013]
re

- X 2-x +l
3-l 2
ou

X +X+1
ad

44. S ~~3 dx 45.


(x + 1)2 (x + 2)
dx [NCERT, CBSE 2014]
Y

X + X
1 1
46. j dx 47. dx [CBSE 2013]
nd
Re

x (x4 +1) x (x3 + 8)


Fi

3 COS X
48. J 2~ dx [CBSE 2012] 49. dx
(1-x) (1 + x ) (1 - sin x)3 (2 + sin x)
2x2 +1 cos x
50. dx [CBSE 2013] 51. J dx [CBSE 2007]
x2(x2 + 4) (1 - sin x) (2 - sin x)
2x +1 1
52. dx [CBSE 2007] 53. j dx [CBSE 2010]
(x - 2) (x - 3) (x2 + 1) (x2 + 2)
1 1
54. J dx [NCERT] 55. dx [NCERT]
x -1
2
2x x
56. J 2 dx [CBSE 2017] 57. j ix [CBSE 2017]
(x2 + 1)(x2 + 2) (x-1) (x2 +1)
LEVEL-2
2
X 1
dx 59. J dx
(x2 +a2)(x2 +b2) cos x (5 - 4 sin x)

ReadYourFlow.COM
19.178 MATHEMATICS-XH

1 1
60. dx 61. dx [CBSE2015]
sin x (3 + 2 cos x) sin * + sin 2x
(x2 + 1) {x1 + 2) ^
62. J
x+l
x(l + x ex)
dx J {x2 + 3) (x2 + 4)
4x4 + 3 4
X
64. J dx 65. dx
(x2 + 2) (x2 + 3) (x2 + 4) (*-l)(x2 +1)
2 f ^ ^
dx 67. J ------jdx
2 -12 1 -x
2 (x2 + l) (ar2 + 4) ^
dx [NCERT, CBSE 2016] ®( (x2 + 3) (x2 -5)
[CBSE2016]

w
ANSWERS
1. log I x +11 +1 log I x - 21 + C

1 . x (x - 4)

Flo
ee
2. - log +C
8 (x-2)2

Fr
3. x - log | x + 31 + log | x - 21 + C for
ur
4. - x + 3 log | x + 21 + 2 log | x -11 + C
x —1
5. x + log +C
ks

x+l
Yo
oo

6. “ log | x -11 - 4 log | x - 21 + ^ log | x - 31 + C


B
re

7.|log (x + l)2 (x-2) + C


ou
ad

(x + 2)3
Y

2-l
8. log - +C
nd
Re

x
Fi

9. | log | x + 11 - ^ log | x -11 - y log | 2x + 31 + C

10. x + ^- log | x -11 - 8 log | x - 2| + y- log | x - 31 + C

(2 + sin x)4
11. log | +C
| (1 + sin x)2
I
12. | log x2+l + C
x2 + 3

13. | log log^ + C


2 + log x
14. — log | x + 21 + - log (x2 +1) + - tan-1 x + C
5 5 5

ReadYourFlow.COM
INDEFINITE INTEGRALS 19.179

+ ab + c ab2 +b2 + c 2 + be + c
15. log \x -a\ + \og\x-b\ + -^ log |x - c| + K
(a-b) (a-c) (b - a) (b - c) (c - a) (c-b)
16. i log | x -11 - ^ log (x2 +1) + ^ tan _1 x + C

(x-l) (x + 2)2
17. Ilog +c
6 (x + 1)3
2 -ix 3 -ix
18. — tan — + — tan — + C
5 2 5 3
19. log |x (x2 -l)2 | + C
x2(x - 2)
20. log | +C
(x + 2)2

low
21. - | log | 2x +11 + - log | x -11 + log | x + 11 + C
o o
22. log | 2 log x + 11 - log | 3 log x + 21 + C
i a

ee
23- -log — +C
rF
Fr
W | x" 4

2 2
1 -a
for
24. i
log * 22 , 2 + C
2(a2-b2) X -fr
u
ks

1 -l X 8 -IX
25.---- tan ‘-4- —tan 4C
Yo

14 2 35 5
oo

x-l
B

26. y + lOg | X2 - 1 | + ^ lOg +C


x+1
re

11 5 11
27- ^ log | x + 11 + — log | x + 31 4 C
ou
ad

2 (x + 1) 4
Y

3 7
28. - — log | x 4- 21 + — log I x - 31------------ + C
25 25 b ' 1 5 (x - 3)
nd
Re

29. | log | x + 31 +1 log | x - 21 - — +C


Fi

5 5 x-
x-l 1
30. - log +C
9 6 x+2 3 (x-l)
31. ilog|x-l| + |log|x + l| + 1
+C
2 (x +1)
32. -1—+ log|x + 2|+C
x+1

33. - + 13 log | x| -12 log |2x +1| + C


x
34. 6 log | x | - log | x + 11 - +C

35. ^ | log | x + 21 - ^ log | x2 + 41 + tan -1 x


+C
2

ReadYourFlow.COM
r
19.180 MATHEMATICS-XII

36. 2 tan ^ x | + 11 + log | x + 21 + C

21 tan -1' x + C
3?. - i log | x + 11 + ^- log | x2 + 11 + —

3S. | log | x + 11 - ^ log 11 + x2 | + ^ tan -1 x + C

1 —-^log|x2 + l|+C
39. - logjx + 1| -
2 2 (x + 1) 4
40. - log | x —1| — — log | x2 + x +1| H—'j= tan -1 2x +1
+C
3 3 V3 V3 J
41. — tan 1 x - — tan -1 - + C 2
42. —= tail -1
f Jlx - tan
-1
x+C
3 6 2 2

low
x +1 4 -1 x + - log (x2 + 1) + C
43. ^log +C 44. x - log | x | - tan
2 x —1 x —1 2
4 "l
45. - 2 log | x + 11------1— 3 log | x + 2| + C 46. 1 log x
+C

ee
x +1 4 x4 +1
rF
Fr
3 x2 + 1
47. llog|x|—^-log x3+8 +C 48. 2+2 tan 1 x +C
• log
8 24 4
for (1 - x)
u
1 1 1 1
49.------ log 11 - sin x | + + log | 2 + sin x | + C
ks

27 9 (1 - sin x) 6 (1 - sin x)2 27


Yo
oo

1 7 -l X 2 - sin x
50. —— + — tan - +C 51. log +c
B

4x 8 1 - sin x
re

|(*-3)7
52. log +C 53. —]= tan 1 + tan 1 x + C
ou
ad

|(*-2)5 V2 V2
Y

1 , x4-1 x—1 1 -1
+C 55. T log — tan 'x + C
nd
Re

4 x +1 2
Fi

x2 + l 1
56. log +C
x2 +2 x2 + 2

57. — log|x-l|+ —log(x2 + l) + —tan ^x + C 58. a -ix b -ix


"2---- 2tan __ a2-b 2^ - + C
2 4 2 a2-b2 «
1 4
59. log 11 + sin x| - - log 11 - sin x | + - log | 5 - 4 sin x | + C
18 2
112
60. - - log 11 + cos x | + — log 11 - cos x| + — log | 3 + 2 cos x | + C
112
61. — log 11 - cos x | + — log 11 + cos x| — log 11 + 2 cos x | + C
6 2 3
.r
xe 2 -1 -Stan'1 ^ + C
62. log +C 63. x + -7= tan
1 + x/ 73 V3 2
N
19 -1 x 39 -1 x 67 . _ix
64. tan tan + tan +C
2 72 v"2 73 73 4 UJ

ReadYourFlow.COM
INDEFINITE INTEGRALS 19.181

65. ^ log | .v -11 - i log (x2 + 1) - ^ tan 1 x + C

66. ~ log x-2 + V3 tan 1 -¥= + C 67. T1o8 1 —x — tan 1 x + C


7 x+2 7 4 1 +x 2

68. 2^tan 1^= + -log


1 x—1
+C
1
69. x + —7= tan
-1 x 27
-7= + —7= log
x-V5 +c
3 V2 6 x+1 4^ V3 8-75 x + 75
HINTS TO NCERT & SELECTED PROBLEMS

7- We have.
5x 5x
I = 1 (x + 1) (x2 -4) dx = J dx
(x + 1) (x + 2) (x-2)
71 B

low
5x C
Let +
(x + 1) (x + 2) (x - 2) x+1 x+2 x-2
Using Cover-up method, we get
5 -10 -5 10 5
71 = ,B = and C = —

ee
3 4 2 12 6
rF
Fr
5x 5 5 5
M (x + 1) (x + 2) (x-2) rfx = — log | x +11 — log | x + 21 + - log | x - 21 + C
3 2 for 6
23. We have.
n-l
u
1
1 dx = f—— dx
ks

I = J Xn (x
, n +1)
,
X (x" + 1)
Yo
oo

Let x” = t. Then, n xn ~ 1 dx - dt
B

1 1 ^ = ^|log t-log (1 + 1)}+C


ll
re

1(1 + 1) n J [t 1+1
ou
ad

1 X
=> / = -log + C =- log +C
Y

n 1+1 n x” + l
nd
Re

30. Let / = J 1
dx
(x-1)2(x + 2)
Fi

1 71 B C
Let + . Then,
(x-1)2 (x + 2) x-1 (x-1)2 x+2

1 = 7l(x-l)(x + 2) + B(x + 2)+C(x-l)2 -(i)


Putting x =1,-2 and 0 successively in (i), we get
1 1 1
B = ,C = — and 1 = -27l + 26 + C => B = - ,C = ,A = -
3 9 3 9 9
1 1 1 1
(x-1)2 (x + 2) 9(*-l) 3 (x -1)2 9 (x + 2)
1 1 1 1 1
=> / =
I (x-1)2 (x + 2) dx =
I
9 J X —1
dx + —
3 I (x-1)2 dx + -9 Jx
f—+ 2 dx
1 1 1 x+2 1
=> 7 iogU-1!- + - log | x + 21 + C = - log +C
9 3 (x-1) 9 9 x-1 3 (x-1)

ReadYourFlow.COM
19.182 MATHEMATICS-XII

3x + 5
43. Let I = J ^ dx. Then,
X -X 2-x + l

3x + 5 3x + 5 3x + 5
W x1(x-l)-(,x-l)
dx =
1 (x ”1) (x2 -1) dx = 1 (x-l)2 (x + 1) dx

3x + 5 A B
+
c . Then,
Let
(x-1)2 (x + 1) x-1 (x-D2 x + 1

3x + 5 = A (x -1) (x + 1) + B (x + 1) + C (x -l)2

Putting x = 1, -1 and 0 successively in (i), we get


1 1
8 = 2B, 2 = 4C and5 = -A + B + C => B = 4, C = -, A =
2 2

w
3x + 5 1 4 1
+ +
(i-l)2 {x + 1) 2(x-l) {x-if 2 {x + 1)

=>
3x + 5
i (l:-l)2(a: + l)
Flo 1
dx = - Mog|i-l|- +! iog i *+11+c

ee
Fr
X2 + X + 1
45. Let / = J dx.
(x + 1)2 {x + 2)
for
ur
2
X +X+1 A B C
ks

Let
Yo

(x + 1)2 (x + 2) x+1 (* + D2 x+2


oo

x2 + x + 1 = A (x + 1) (x + 2) + B (x + 2) + C (x + l)2 •~(i)
B

Putting x = -1,-2 and 0 successively in (i), we get


re

1 = B, 3 = C and 1 = 2A + 2B + C => A = - 2, B = 1, C = 3
ou
ad

x2 + x + 1 -2 1 3
Y

(x +1)2 (x + 2) ' x + 1 (x + 1)2 x + 2


nd

X2 + X + 1
Re

dx = — 2 f —dx + [ —-—^ dx + 3 [ —-— dx


(x + 1)2 (x + 2) J x+1 J (x + 1)2 J x+ 2
Fi

1
= - 2 log | x + 11 - + 3 log | x + 21 + C
x+1
COS X
51. Let I = { dx and let sin x = t. Then, d (sin x) =dt => cos x dx = dt
(1 - sin x) (2 - sin x)
1 1 1
I = J (1-f) (2-t) 1-! 2-t
dt

2-t 2 - sin x
7 = - log (1 - 0 + log (2 - 0 + C = log + C = log j +C
1 -t - sin x
1
54. Let dx. Then,
X{x4-1)
3 4
x 1
W X4 (x4-!) dx = - [
4 J t(t-l)
dt, where f = x

ReadYourFlow.COM
INDEFINITE INTEGRALS 19.183

=> / = if -i -i
4 1 t-l t
= i{log(t-l)-logfl+C

1 x4-l
=> ' = J log + c = 4l0S ~x* +c
t

55. Let 7 = J -j1 tfa: = j"


- * -1 (I2-1)(X2+1)
Let x2 = y. Then, 1 1
(x2-l)(x2+l) (y-l)(y + l)
1 A B
Let
(y-i) (y+i) y-i y+i
Using cover-up method, we obtain

low
A = - and, B = - —
2 2
1 1 1
(y ~ 1) (y +1) 2(y-l) 2 (y + 1)

ee
1 1 1
rF
Fr
(x2-l) (x2 + l) 2 (x2 -1) 2 (x2 +1)
1
(x2-l) (x2+l)
dx = -
2 ■'
M —-1 dx -
x2
1
for
x2 + 1
dx = IIo8 —tan 1x + C
2
u
ks

19.14 INTEGRALS OF THE FORM


Yo

*2+l *2-l
oo

dx,\-t dx' \ —““2— dx,where XeR


x4 + Xx2 +1 x4 + Xx2 +1
B
re

To evaluate this tjq^e of integrals, we use the following algorithm.


ou

ALGORITHM
ad

2
step I Divide numerator and denominator by x .
Y

( 1
STEP II Express the denominator of integrand in the form \ x + — ±k2.
nd
Re

XJ
Fi

1 1
STEP HI Introduce d x + — \ or,d x — or both in the numerator.
I x) V xJ
STEP IV Substitute x + - = t or, x - — =t as the case may be.
x x
This substitution reduces the integral in one of the followingformsj — 1 2‘M^r 2 dx.
x +a x -a
STEP V Use the appropriate formula.

ILLUSTRATIVE EXAMPLES
LEVEL-1
EXAMPLE l Evaluate:
r\ f x2 +1 2-l
W J -4 + 1 dx [CBSE 2007,11] (ii) j 2
J x4+x2+l
dx

x2 + 4
(m) f -j dx [CBSE 2007]
x +16

ReadYourFlow.COM
19.184 MATHEMATICS-XII

*2+l
SOLUTION (i) Let / = | dx. Then,
*4+l

2
=> I
=J [Dividing the numerator and denominator by x~]

X
1 + J2
=> I = I if + 2
dx
x-
a:

1 1 1
Let x — = t => d x —- = rff => 1+— dx = dt.
X * x-

low
f2+(V2)2
1 -1 t 1 1 x -1/a: 1 -1 x 2-l
+C = tan +C

ee
=> I= tan + C = -t= tan
V2 IV2 V2 l V2 ■J2 V2 x
rF
Fr
2^1
(ii) Let I = J dx. Then, for
x4 + x2 + 1

-4
u
ks

2
I
=H X dx [Dividing the numerator and denominator by x ]
Yo

1
oo

X + 1 + -y
X
B

1
re

=> /
=J X c/x
ou

V2
ad

X+- ~12
Y

1 1
nd

Let x + - = u. Then, d x + — = du => 1 - y dx = du


Re

x x x
Fi

r dw
l ~ J 2
2 ,2
U -1
1
X + —— 1 2-x + l
1 u-1 1 x 1 1 x
=> 1 = log + C = - log +C = log - +C
2(1) w+1 2 1 2 X2 + X + 1
X+-+1
X

x2 + 4
(iii) Let 1 = J -4 dx. Then,
x +16
4 4
i+4 1 +-4
2 1 + ~2
X
I = 1^6
X +
dx =
1 2
X

4f -8 + 8
dx.
2 x + X----- +8
X
u X

4 4^ 4
Let x — =t. Then, d x — = dt => 1 + ~2 dx = dt
x x x 7

ReadYourFlow.COM
INDEFINITE INTEGRALS 19.185

/ =
t2 + (2 V2)2
4
x— x2 - 4
1 -l t 1 1
=> I = tan + C = tan -l x +C = tan -1 +C
2 V2 2 -Jl 2j2 2 V2 2 V2 2x -Jl

EXAMPLE 2 Evaluate:
2
(i) rfx rfx
x4+l x +1
(iii) J ^tan 0 rf0

SOLUTION (i) Let / = { -4 1 rfx. Then,

low
+1

=>

ee
rF
Fr
=> for
X + —r
2
u
X
1
ks

1 f 1+ 2
Yo
oo

=> I = -
2 J 2 1
X +^r
B

2
X
re

1
1+^
1
ou
ad

=> I = -
2 hi
X +
h- dx-~
1 2
Y

X + —=-
2 2
X X
1 1
nd
Re

1+^
1 x x
=> I = -
1 dx - — \ rfx
Fi

2 if 2 J if
x— +2 X + - -2
xj XJ
1 1
Putting x — = w in first integral and x + — = in second integral, we get
x x
1 r du 1 du
I = -
2 J 2
«-+(V2)2 I
2>v2-(j2)2
1 u ) 1 1 . u-V2
=> 7 = tan -1 V2 J 2 X 2 V2 °8 + C
2 V2 u + V2
1 1 x-l/x 1 x + l/x - V2
=> 7 = tan log + C
2 V2 V2 x J 4 V2 x + l/x + V2
^ 2
1 _l x -1 1 x2 -V2 x + 1
=> 7 = tan log + C
2 ^2 42 X 4V2 X2 + -y/2 X + 1

ReadYourFlow.COMJ
19.186 MATHEMATICS-XII

2
(ii) I = dx
x +1

I =
x2 +l/x

=> I =
X2 +l/x:
(1 +l/x2) + (1 -1/x2) l+l/x2 1 -1/x2
=> / = dx = - f dx+ - \
li x2 +1/x2 2 J x2 +l/x2 2 J X2 + 1/X
2 ^

l + l/x2 1 -1/x2 dx
=> / = !2 Jf dx + - f
(x-l/x)2 + 2 2 J (x + l/x)2 -2

low
Putting x — — = w in first integral and x + - = z; in second integral, we get
X x
1 f___ du 1 <■ dv
I = -
2 J «2+(V2)2
2 2 J v2~(V2)2

ee
rF V-y/2

Fr
=> lj = —x
1 1
—f= tan -1 \X2l2IOS + C
2 V2 V2j y+ V2
1 lf x-l/x x + 1/x - V2
or
J = tan T]2los x + l/x +V2 + C
f
ou
2 V2 V2
ks

-1 x2-l '2-xV2+1
oo

1
=> I = tan + + C
Y

2 V2 4 V2
B

X -y/2 x +x V2+1
re

(iii) Let J = J ^tan 0 d0.


ou
ad

Let tan 0 = x2. Then,


Y

d (tan 0) = d (x2) => sec2 0 d 0 = 2x dx => d0 = 2x dx 2x dx 2x dx


nd

sec2 0 1 + tan 2 0 1 + x4
Re
Fi

2x dx 2x2 1 +l/x2 +1-1/X2


I =
1+x
4 -= Jf-4
x’ +1
dx = |
x2 + 1/x' x2 + 1/x2
dx

1 + 1/x2 1-1/x2 1 + 1/x2 1 -1/x2 dx


=> 7
= 1 x2 +1/x2 dx + J x2 +l/x2dx = J (x-l/x)2+ 2 dx + J (x + l/x)2-2
Putting x - — = w in first integral and x + — = y in second integral, we get
j du
I =
«2+(V2)2 »2-<V2)2

1 1 y-V2
I = tan -l + log + C
V2 IV2 2 V2 y + V2
r 1 t -if x + 1/x - V2
7 = -t= tan —f=— log +C
V2 V2 2y[2 x + 1/x + -J2

ReadYourFlow.COM
INDEFINITE INTEGRALS 19.187

=> / = 4= tan-1 V-l 1 . x2-J2x +1


+C
^los
V2 V2 x 2 s/l

1 tan 0-1 1 tan 0 - ^2 tan 0 + 1


=> 7 = —?= tan -1 J2 tan 0 + 2 V2 log tan 0 + ^2 tan 0 + 1 + C
V2
EXAMPLE 3 Evaluate: tan 0 + ^cot 0J o'© [CBSE 2010, 2013, 2014]

SOLUTION Let I = an 0 + ^cot 0 j dQ . Then,

1 tan 0 + 1
I = | • ^/tan 0 +
^tan 0
dQ => I =
J ^/tan 0
dQ.

w
Let tan 0 = x2. Then,

d (tan Q) = d (x2) => sec2 B d Q = 2x dx => dQ = 2x rfx 2x c7x 2x rfx

Flo
sec2 0 1 + tan2 0 1 + x4

ee
x2+l 2x dx x2 + l 1 +l/x2
I
=1 2J x4 +1 =21 x2 +l/x2 dx

Fr
1 +l/x2 1 + l/x2
for
2I (x-l/x)2 + 2 dx = 2 j (
ur
=> I = 2 dx
x-1/x)2+(V2)
ks

2 -1 4= + C, where x - -
/ = 2j— ----- +
Yo

=> = —?= tan = u


oo

w + (V2)2 ^ V2, X
eB

-1 f x — 1/x ^ -1 x2-l tan 0-1


=> I = -y/2 tan +C = tan + C = V2 tan -1 +C
V2 a/2 x ■J2tan 0
r
ou
ad

EXAMPLE 4Evaluate: j---- ^ 1 1— [CBSE 2014]


4 dx.
Y

sin x + cos x
nd

1
Re

SOLUTION Let 7 = J ----j--------- 2— dx. Then,


sin x + cos x
Fi

1/cos4 x
7 =
. 4 4 rfx
sin x + cos x
4
cos x
2
sec4 x 2 2
sec x • sec x 1 + tan x • sec2 x dx
=> I
=J 4
tan x +1
dx = \
tan4 x +1
dx - j ■
1 + tan4 x
2
Putting tan x = f and sec x dx = dt, we get
i+t2 tan -d<2- -
7
= J 1+t T4 dt = 4=
V2 yf2t
+C [Proceed as in Example 3]

' 2
-1 tan x -1
=> —7= tan + C.
V2 V2 tan x

ReadYourFlow.COM
19.188 MATHEMATICS-XII

UEVEL-2
1
EXAMPLE 5 Evaluate: J dx
6. 6
cos x + sin x
1
SOLUTION Let / = | dx. Then,
6. 6
cos x + sin x
1
W (cos2 x + sin2 x) (cos4 x + sin4 x - cos2 x sin2 x)
dx

1
=> I =
J 2 22 2
(cos x + sin x) - 3 sin x cos x
2
dx

4 Dividing Nr and
1 sec x
dx = | dx
io-2 2 4 2 Dr by
1-3 sin x cos x sec x - 3 tan x COS 4 X

w
(1 + tan2x) sec2 x f2+i 2
=>
W (1 + tan2 x)2 - 3 tan2 x *== I rA - r2 +1 df, where f = tan x

1 1

Flo
ee
1 +T 1+^
=> t j du , where u = t —
1

Fr
if + 1 u2 +1 t
f2 t-
t for
ur
=> I = tan 1 u + C = tan 1 (tan x - cot x) + C
ks

x4+l
Yo

EXAMPLE 6 Evaluate: J dx
oo

x6 +1
eB

x4+l
SOLUTION Let / = J dx. Then,
x6 +1
r
ou
ad

(x2+1)2-2x2
'=1 dx
Y

(x2 +1) (x4-x2+l)


nd

2 2
Re

X +1
=> dx -2 f -7-— dx
Fi

X- -X2 +1 J x6+l

=> I=
I
-4 x dx-- f
3x2 dx
dx
x2-l + -^ 3 J (x3)2+l
X
1+J2
2 1
=> x
if + 12 3 1 (x3)2+l2
3x2 dx

x—
X

=> I = f 7 1 - du- - ( 0 1 . dv, whereu=x - — andr; = x3


1 u2+l2 3J p2+l2 x

1
I = tan 1 « - — tan 1+ C = tan -1 x—----
2‘tan -1x3 +C
=>
3 x 3

ReadYourFlow.COM
INDEFINITE INTEGRALS 19.189

3
LxAMPi.E 7 Evaluate : [ — dx
a:1” + 4

3 3
X
SOLUTION Let I = J —^
x +4
dx
=1 (xY+4
dx

Putting x4 and 4x3 dx = dt, we get


1

/=-4 Jf t4 + 4
1
dt= -
4 ivr +
^— dt
4
[Dividing Nr and Dr by ]

t2

4 1 + 21 1-
2
f2 r

low
t2 1
=> /=— f -
16 J *2 4
r + -^r
dt = —
16 I t2+4
dt

t2 2
t
2 2
1 +-=- 1

ee
1 t2 1 l2- dt
/=
I-i dt- —
rF
Fr
16 4 16 4
r+ f +
t2 t2 for
2 2
1 + ,2 1-
u
^-----dt
=> /=— f I dt-— f
ks

16 J 2f 16 J 2f -4
Yo

t-- + 4 t+-
oo

t t
B

du 1 2 2
=> J I^ , where u-t---- andz? = t + —
re

16 J u2 + 22 16 v2 - 22 f t
ou
ad

1 .tan -1 f -
« 1 1 v-2
+C
=> I = — 16 X 2x 2 log y + 2
Y

32 x2

f^2 -2
nd

t2 -2t +2
Re

-1 1
tan 7T lo8 +C
32 2t 64 f2 + 2f + 2
Fi

8 8 - 2x4 + 2
-1 xu -2 1
=> tan log - +C
32 2x4 64 x8 + 2x4 + 2

x 2-l
EXAMPLE 8 Evaluate: J dx
(x2+l) Vx4+1

X 2-l
SOLUTION Pet / = J dx. Then,
(x2+l) Vx4+1

x; dx [Dividing andDr byx2]


1 2 1
x+— X +
X 2
X

ReadYourFlow.COM
p«*

19.190 MATHEMATICS-XII

=> x' dx

x+
1 if
x + — -2
X

1 1 1
Letx + —=1.Then,dx + —= i/for, 1—^ dx = dt.
x \ *) lx2
1
1 1 1 t 1
x-
M rff=j dt = -7= sec -i + C =-7=sec
-i __x + C
t Vf-2 V2 IV2 V2 V2

EXERCISE 19.31

w
LEVEL-1
Evaluate thefolloiving integrals:

1-1
x2+l
rfx
Flo
2. | ^/cot 0

ee
x4 + x2 + 1

Fr
x2 + 9 , 1
3. j -4-------dx dx
for
x4 + 81 x4 + x2 + 1
ur
2 -3x + l x2+l
5,Jf A"
z4 + *2+l dx 6- j -4 dx
ks

x4 - x2 +1
Yo
oo

f x2_1 x2 +1
B

x +1
dx [CBSE2007] 8J-4
x4 + 7x2 + 1
dx
re

1
10J^x4 + 3x2
ou
ad

4 9 dx
X +X +1 +1
Y

1
11. 1 dx [CBSE 2014]
nd
Re

sin4 x + sin2 x cos2 x + cos4 x


Fi

ANSWERS
1 f 2 ^
-1
J-^tan +c
-J3x
1 -l cot 0-1 cot 0 + 1 - V2 cot 0
^2 cot e 2 V2 108 +C
2--V2tan cot 0 + 1 + 72 cot 0

1 -1 x2 -9 1 -1 x2-l 1 2 -x + 1
3- 3J2 tan +C tan -tog -X +C
3 a/2 x 2y[3 X 73 4 x2 + x + 1
2
1 -1 1 2x2 +1
5- 73tan + 73 tan +C
J3x 73
2 2 -72 X + 1
-l 1
6. tan +c 1 *
log - +C
X 2 72 x2 + 72 x + 1

ReadYourFlow.COM
INDEFINITE INTEGRALS 19.191

r 2 2
9. —)= tan -l 2 -1 2r +1
8. — tan -1 — ---- f= tan +C
3 3x V3 V3 x V3 V3
/
^ 2 1^ .
1 -1 X -1 1 -1 x2+l 1 -1 tan x-cot x
10. tan ---- tan +C 11- —f= tan +C
2 V5 fix 2 x V3 V3
19.15 INTEGRATION OF SOME SPECIAL IRRATIONAL ALGEBRAIC FUNCTIONS
<t>(x)
In this section, we shall discuss four integrals of the form J dx, where P and Q are
pVQ
polynomial functions of x.
19.15.1 INTEGRALS OF THE FORM ( <!>(*) dx, WHERE P AND Q BOTH ARE LINEAR
FUNCTIONS OF v p Vq

w
To evaluate this type of integrals we put Q = r i.e., to evaluate integrals of the form
1 2
dx, put cx + d = t .
(ax + b) Jcx + d

Following examples illustrate the procedure.

Flo
ee
ILLUSTRATIVE EXAMPLES

Fr
LEVEL-1 for
ur
1
EXAMPLE 1 Evaluate: J dx
(x - 3) Jx + 1
ks

1
Yo

SOLUTION Let / = J dx
oo

(x - 3) ^jx + 1
B

2 • 2
Here, P and Q both are linear, so we substitute Q = t i.e. x + 1 = t and dx - 2t dt.
re

1 2t
I
=1 dt
ou
ad

l^n-2
Y

1 t-2 1
' = 2J A t-2
= 2x
2(2)
log
f+ 2
+c = ^ los! 77TT + 2 + C.
nd
Re

EXAMPLE 2 Evaluate J
Fi

dx.
x +1
x 2
SOLUTION Let / = | V
dx = J dx. Putting x —t and dx -2t dt, we get
X+1 Vx (X + 1)
t2 2t dt t2 t2+l-l
l 2 I f2+l dt =
2I f2+l
dt

1 -1 -1
=> 1 2 Jl f2+l
dt - 2 f - tan - tan

19.15.2 INTEGRALS OF THE FORM } ^ dx, WHERE P IS A QUADRATIC EXPRESSION


P v' Q
AND Q IS A LINEAR EXPRESSION
2
To evaluate this type of integrals we put Q=t i.e., to evaluate integrals of the form
1 2
dx, we put px + q = t .
2 i-----------
(ax + bx + c) Vpx + q
Following examples will illustrate the procedure.

ReadYourFlow.COM
r
19.192 MATHEMATICS-XII

ILLUSTRATIVE EXAMPLES

LEVEL-1
1
ex a mpu: i Evaluate: | dx
(x2-4)^/?TT
1
SOLUTION Let 7 = dx.
(x2 - 4)

Putting x + l= t and dx = 2t dt, we get


2t dt dt
7
=1 (72-l)2-4 Vf2 2J -1-2) (72 -1 + 2) 2 J (t2-3)dt(t2 +1)
Let t2 - y. Then,

w
1 1
(f2 - 3) (t2 + 1) (y-3) (y +1)
1 B
Let
(y-3) (y +1) y-3 y +1

Flo
ee
=> 1 = A(y + l) + B(y-3) ...(ii)

Fr
Putting y = -1, 3 respectively in (ii), we get
1 1
B =---- and A = —
4 4 or
ur
Substituting the values of A and B in (i), we obtain
f
1 1 1
ks

(y “ 3) (y + 1) 4 (y - 3) 4(y + l)
Yo
oo

1 1 1
[v y=f2]
B

(f2 “ 3) (f2 + l) 4 (f2 - 3) 4 (72 + 1)


re

1 1
7
21) 4 (f2 - 3) dt
ou
ad

4 (f2 + 1)
Y

=> 7= , * ‘ft-l f
f2 -(v3)2 2 J f2+l2
nd
Re

1 1 , t-y/3
Fi

=> 7 = 2 X 2 x/3 °8 t + y[3 — tan 1 (7) + C


2

1 . 1
=^> 7 = —= log V ---- t== — tan -1 +C
4V3 6 + 1 + V3 2 1

LEVEL-2
x+2
EXAMPLE 2 Evaluate: J dx
(x2 + 3x + 3) ^x + 1
x+2
SOLUTION Let 7
= J (x2 + 3x + 3) yjx + l dx. Putting x + 1 = f2, and dx = 2t dt, we get
(t2 +1) 27
7 =
J j(72 -l)2 + 3 (72 -1)+ 3j x/t2 dt

ReadYourFlow.COM
'NDEFINITE INTE< 19.193

1
(t2+l) 1+
=> I = 2
hit4+t2+l dt = 2 i-2t2 + \+i dt [Dividing Nr and D1 by t2]

t
1
1+
t2
=> I = 2
I if + (V3)2
dt = 2
h2+(V3)2
, where t - - = u
t
t-
t
1
t-~
2 -1 -1
=> I = -7= tan 4= +c = —j=
2 tan —
t +C
V3 ^3 73

w
( f2
2 -1 t 1 2 -1 x
t = tan + C = —f= tan +C
73 73 f 73 p(x + l)
'9.15.3 INTEGRALS OF TH
A QUADRATIC EXF

Flo fiSSION AND Q

ree
To evaluate ^ this type of integrals we put P = 1/t i.e. to evaluate integrals of the form

F
1
I (ax + b) Jpx2 + qx + r
dx, we put ax + b = - .
t or
ur
sf
Following examples will illustrate the procedure.
k
Yo

ILLUSTRATIVE EXAMPLES
oo

LEVEL-1
B

1 1
re

EXAMPLE 1 Evaluate (i) | dx (ii) j dx


(z + l) (x-l)f2 + 4
ou
ad

1 1
SOLUTION (i) Let / = J
Y

dx. Putting x + 1 = - and dx = - dt, we get


(x + 1) ^x2 -1 t t2
nd
Re

1 1 dt =-{ dt 1/2 d-20 1/2


1=
I-1 -j (1-20 dt =- +C
Fi

2 t2) JT^Tt
t
1
t
-1 -1
-ii
2 X —1
1= jr^2i+c= +C = +C
x+1 x+1
1 1 1
(ii) Let / = J dx. Putting x -1 = -, and dx = -— dt, we get
(x-l) f2 + 4 1 t
1
-^2dt

"i
t dt dt
I =
11 1
\2
!
yjBt2 + 2f + 1 't2 + 2t + 1
+1 +4 5 5
t t
1 dt 1 1
=>
I t2 + 2 t + 1 ^ 1 _ 1
=
75 Itt + H2 2^ 2
dt
+ +
5 25 5 25 5 15

ReadYourFlow.COM J
wr

19.194 MATHEMATICS-XII

=> i -4 t+
i
+ t+-
n2 + 21 2
+C
V5 5 5 5

1 log (t + ')+Jt2 + ^t + ?: +C
=>
V5 5

1 f 1 1 1 2
=> / !og +— + + - +C
V5 x -1 5 (A'-l)2 5 (.y -1) 5

1 i 1 1 x2 + 4
=> I ^ log - — + - + +C
V5 x -1 5 5 (a--l)2

w
19.15.4 INTEGRALS OF THE FORM r) 4>(a1 dx, WHERE P AND Q BOTH ARE PURE QUADRATIC
1 p v'Q

Flo
EXPRESSION IN v i.e. P = nx2 + b AND Q = cx2 + d

1 2 2

ee
To evaluate this type of integrals we put x = - and then c + dt = u i.e., to evaluate integrals of theform

Fr
-tdt
I (ax2 + b)1yjcx2 + d dx, we put x = jto obtain J (a + bt2) Jc + dt2 and then substitute c + dt 2 = u2 .
for
ur
Following examples will illustrate the procedure.
ks
Yo

ILLUSTRATIVE EXAMPLES
oo

LEVEL-1
B
re

Evaluate: f----- ^ dx
EXAMPLE 1
J X 2 yjl+X
/7 2
ou
ad
Y

SOLUTION Let / = f — 1 111 ?


= dx. Putting x = - and —j dx = dt or, dx = -x dt, we get
J x22 +x t X
nd
Re

-dt tdt u du 2 2
=1 1
Fi

I
I 11
2
, where + 1 =»

J1 + A2
=> I = J -1 ■ du = - u + C = - ft2 + 1 + C = - +c = +c
x

LEVEL-2
Jl + x2
EXAMPLE 2 Evaluate: J —----- y- “A
1 -x2
SOLUTION We have.

I = 1 dx

ReadYourFlow.COM
INDEFINITE INTEGRALS 19.195

=> I = r v1 + x2 dx
1 -X
1+X
=>
2
+X
2
1
=> I = 1 +X
= dx + {
2 J (l-x2)^l + x2
dx

2
1 -X
=> I = J (1 - x2) tJi + x2 dx - j
(1 -X2) yjl + X2
dx

1 (l-x2)-l
/ = 1 = dx - f dx

w
2 J (1 -X2) + x2
+X

Flo
1 1 -x2 1
=> I
= 1 (1 -X2) yjl+X2 dx- j - (1 - x2) Jl + x2 (1-x2) Jl+x2
■ dx

ee
Fr
1 1 1
=> I
= 1 (1 -x2)^l + x2 dx- j yi+ x Tdx+ 1 (1 -x2) xjl+x2 dx for
ur
1 1
=> / = 2I I ^ + x2 dx
ks

+X
Yo
oo

/ = 2 /-j - log | x + -^1 + x2 | + C, where = J 1


=> dx
eB

(l-x2)^ + x2
1
Putting x = - and dx = — dt in /j, we get
r
ou

t t2
ad
Y

1
dt
nd

'1=1
Re

1 1
1 1 + .2
Fi

t2 t
tdt
h = l-2 (f2-i)^2+i
u du 2 2
h = l~2 , where t +1 = u and t dt -u du
(u2 - 2) V?
^ log u - V2 1 ~V2
h 2 V2 u + 42 2V2l0g ^t2 + 1+42
«2-(V2)2
1
2 +1-V2 ■Jl + x2 -42 x
1 X
/1 - - log ^ log
242 \ + l + j2 242 yjl + X2 + 42 X
X

ReadYourFlow.COM
r
19.196 MATHEMATICS-XII

1 + x2 -V2 x
I log - log I x + Jl + X2 I + C
V2 + x2 + J2 x

Evaluate: J 1
PI E 3 dx
x yjax - x2
1 11
SOLUTION Let I r dx. Putting x = - and dx = --j dt, we get
-'-i ax - x2 t t
1/2
1 1
i =11 0 1 t2 0
+c
t t f2 2
-2 -2
=> --1 +C = +C

w
a 0 x 0
19.32

Flo
LEVEL-1

ee
Evaluate the following integrals:
1 1

Fr
1. 1
(x — 1) ^Jx + 2
dx 2- I (x-1) ^2x^3 dx
2
for
ur
x +1 X
1 (x-1) tJx + 2
dx 4.
1 (x-1) ^x + 2 dx
ks

x 1
dx 6. dx
Yo

5.
(x - 3) ^/xTT
oo

(xz +1)
eB

X 1
7 1 (x2 + 2x + 2) ^/x + 1 dx 8. {
(x-1) Jx2 +1
dx
r

1
ou

9 f (x + 1) Jx2 + X + 1 dx f___ v.....


ad

10. dx
(x2 -1) Jx2 +1
Y

X 1
dx 12. 1 (1 + x2) ^1 -x2 dx
nd

11.
Re

(x2 + 4) Jx2 +1
Fi

1 X
13. J
2x2 + 3)
dx 14.
1 (x2 + 4) ^x2 +9 dx
ANSWERS
1 1 ,
l - f log +c 2. log , ---- 7= + c
V3 ^/Fr2+V3 V5 b ^T3+Vs

3. 2 ^x + 2 +
2 , ^2-43 +C
V3 8 7JT2+V3
_____ i A/x + 2 - -/3 ------- 3
4. |(x + 2) 3/2 +C
2^ + 73 108 ^T2 + V3 +C5- 2J*TT+2l°S ^n + 2

1 -l x-1 x - V2x + 1 1 (x + 2) -^2 (x + 1)


b. -7= tan lo8 +C 7- -7^log +C
Vz V2?J 2^2 x + Jlx+l Vz (x +2)+ 72^ + 1)

ReadYourFlow.COM
19.197

i'ogHJ+jHl + —+ C, where t = ——
4 x-

2 -Jx2 + x + 1 ^2 X + ^X2 + 1
1 1
log +C log +C
X X+1 2V2 »/2 x - -Jx2 + 1
1 1 -1
tan -1 +C -f= tan +C
V2
Vll x + V 3x2 -12 1 Jx2 + 9 - V5
log +c log +C
2 V33 VTT x - -J3x2 -12 2V5 -Jx2 + 9 + V5

w
Anszver each of the following questions in one word or, one sentence or, as per exact requirement of the

Flo
question:
1 + cot x Write a value of J e 3 log Y x4 rfx.
Write a value of J dx.

ee
x + log sin x

Fr
Write a value of J x2 sin x3 dx . Write a value of J tan 3 x sec2 x dx.
Write a value of | ex (sin x + cos x) dx . Write a value of f tan x sec x rfx.
for
ur
cos x Write a value of J ex sec x (1 + tan x) dx.
Write a value of J dx.
3 + 2 sin x
ks
Yo

Write a value of J ^ * dx.


oo

Write a value of J X- dx.


eB

Write a value of J elog sm A cos x dx. Write a value of J sin 3 x cos x dx.
Write a value of J cos 4 x sin x dx. Write a value of J tan x sec3 x dx.
r
ou
ad

1 Write a value of f---- ----- dx.


Write a value of J dx.
Y

\+ex J 1 + 2 e*
2
(tan^3 dx.
nd

sec x
Re

Write a value of J Write a value of J dx.


1 + x2 (5 + tan x)4
Fi

sin x + cos x Write a value of J loge x dx.


Write a value of J dx.
Vl + sin 2x
2x2 + In x
21 Write a value of j ax ex dx. Write a value of J c dx.
Write a value of J (ex loge a + e a logt, x ) dx.
COS X
Write a value of J dx.
sin x log sin x
sin 2x
...i. Write a value of J dx. Write a value of J —-—- dx.
2 22 2 3+a
a sin x + b cos x
7. Write a value of f ^ + ^ ^ dx. Write a value of [ Sm^-Y dx.
J 3 + x log x J cos'3 x
sin x - cos x , Write a value of [----- —
29. Write a value of J , . -- dx. dx.
^1 + sin 2x J x (log x)"
Write a value of J eax sin bx dx. 32 Write a value of J eax cosbx dx.

ReadYourFlow.COM
W'
19.198 MATHEMATICS-XII

Write a value of J ex f— 1
33. 2 dx. 34. Write a value of J eax [a f (x) + f' (x)} dx.
X
35. Write a value of J ^4 - x2 dx. 36. Write a value of J ^9 + x2 dx.
2J
37. Write a value of J ^x2 -9 dx. 38. Evaluate: J ------- ^ dx (C BSE 20081

x2 + 4x sec'
39. Evaluate:J dx fCBSE 20081 40. Evaluate: ! dx [CBS E 2009]
x3 + 6x2 + 5

41. Evaluate: j
sin V* cos Jx ,
dx [CBSE2009] 42. Evaluate: ! dx ICBSE20091

(1 + log x)1
43. Evaluate: J dx iCBSE 20091 44. Evaluate: J sec2 (7 — 4x) dx 1CBSE2009]

45. Evaluate: f ^ l0^ 1 1CBSE2010] 46. Evaluate:! 2X dx [CBSE 2010J

low
J x
1 - sin x x3 -1
47. Evaluate:| dx 1CBSE 2010J 48. Evaluate:! —-— dx ICBSE20101
2
COS X X

3-x2+x-l tan -1 x

ee
49. Evaluate:!— dx 1CBSE2011] 50. Evaluate: !- dx ICBSE 20111
x—1
rF 1 + x2

Fr
51. Evaluate: f —= dx [CBSE20111 52. Evaluate:!secx (secx + tan x) dx [CBSE 2011]
J ^ for
53. Evaluate: [ ——^---- dx 1CBSE2011I 54. Evaluate:! (1 - x) Jx dx
u
[CBSE 2012]
ks

J x2 +16
Yo

55. Evaluate^ * + COs6* dx


oo

2 [CBSE 20121
3x + sin 6x
B

x —1
re

56. If! —ex dx =f (x) cA + C, then write the value of f (x). [CBSE 2012]
*
ou
ad

57. If! ex (tan x + 1) sec x dx = ex f (x) + C, then write the value/ (x). [CBSE 20121
rs
Y

58. Evaluate:! dx ICBSE2012]


1 - cos 2x
nd
Re

1
59. Write the anti derivative of 3 V* + -7= [CBSE 2014]
Fi

1
60. Evaluate:! cos -1 (sinx) dx ICBSE2014] 61. Evaluate! dx ICBSE 20141
. 2 2
sin xcos x
1
62. Evaluate :! dx ICBSE 2017|
x (1 + log x)
ANSWERS
8
1. log |x + log sin x| + C 2. — + c 1 3 n
---- cos x + C
8 3
tan x . 7
, tan x
4. --------- + C
_ 5. ex sin x + C (> +c
-------------- , „
1 4 7
7. - log 13 + 2 sin x | + C 8. ex sec x + C 9. ^ (log x)2 + C
n+ 1 . 4
, sin x
10. +C 11. +C I ?. +C
n+1 2 4
5 3
COS X sec
13. - +C 14. ±+c 15. - log (1 + e A) + C
5 3

ReadYourFlow.COM
INDEFINITE INTEGRALS 19.199

(tan 1 A')4 1
16. -log|2 + e X\+C 17. +c 18. 3+C
4 - 3 (5 + tan x)
(^)A
19. a + C 20. a (log^ a -1) + C 21. +C
log (ae)
A a+ 1
1 .2.v2 a x
22. -e +C 23. + +C 24. log (log sin x) + C
4 log^ a a+\
1 2 log (fl2 sin2 a + cos2 a) + C
25. 26. —log(3 + flA)+C
a2-b logfl
l 2 r
27. log (3 + a log a) + C 28. - sec a + C 29. - log | sin a + cos x\ + C
2
fl.t
(log £
30. +C 31. ----- - (a sin bx - b cos bx) + C
1 -n a + b2
ax x

w
c c
32. ----- ^ (a cos bx + b sin bx) +C 33. — + C 34. <?flA / (a) + C
a2 4-
+bh2- x

Flo
1 I 2 9
35. — x J4 - a2 + 2 sin 1 — + C 36. — x yj9 + x + — log A + ^9 + A2 + C
2 ’ 2

ee
1 ^ log A + ^ 2 -9 +C 38. — log 11 + a3 | + C

Fr
37. - a
2 3
39. ^ log | a3 + 6a2 + 51 + C 40. 2 tan +C 41. - 2 cos yfx +C
for
ur
(1 + log a)3
42. 2 sin +C 43. +c 44. - ~ tan (7 - 4a) + C
ks

3
Yo

2X
oo

45. ^(log a)2 +C 46. +C 47. tan a - sec a + C


log, 2
B

48. —+ I + C
a3 tan - 1
re

49. — + A + C 50. e A +c
2 A 3
ou

1 -1 A
ad

51. sin- 1 a + C 52. sec a + tan a + C 53. — tan +c


4 4
Y

54. — a3/2 - — AT5/2 + C 1 ? 1


55. — log | 3a + sin 6a | + C 56. —
nd

3 5 6 A
Re

57. sec a 58. - cot a + C 59. 2 (a 3/2 + x1/2) + C


Fi

2
n x~ n
60. 61. -2cot2A + C 62. log (1 + log a) + C
2 2
MULTIPLE CHOICE QUESTIONS (MCQs)
Mark the coned alternative in each of the following:
1. f———— dx is equal to
J 4 + a4
2^ 2^
/ \ 1 , -1 A
(a) — tan " 1 a2 + C (b) - tan 1 — (c) - tan — (d) none of these
4 4 2 2 2
V
1
2- \ dx is equal to
cos x + -fs sin a
(a) logtan^| + ^j +C (b) log tan ^ - +C
3y
(c) | log tan^ + |j + C
(d) none of these

ReadYourFlow.COM
19.202 MATHEMATICS-XII

1 tan2 x + C
(a) — 1 tan2 x + C
(b) — — tan 3 x + C
W3 (d) none of these
3 2
( 1
24. The primitive of the function / (x) = 1----^ a -Y , a > 0 is
x J
x+ -
l l 1
x+
1 X+ x+ -
(I x
(b) log{, a- a x loge a (cl) X-------
logt> a x l°ge a
1
25. The value of J dx is
x + x log x
(a) 1 + log x (b) x + log x (c) x log (1 + log x) (d) log (1 + log x)
26. | eclua^

w
(a) sin 1 -v/x + C (b) sin - 1 [Jx - tJx (1 -x)} + C
(c) sin 1 yx (1-x)) + C (d) sin - 1 Vx - - x) + C
27. { ex[f(x)+f(x)} dx =

Flo
ee
(a) ex f (x) +C (b) ex + / (x) + C (c) 2ex f (x) + C (d) ex -f (x) + C

Fr
sin x + cos x
28. The value of J dx is equal to
y - sin 2x
for
ur
(a) ^jsm 2x + C (b) ^cos 2x + C (c) ± (sin x - cos x) + C (d) ± log (sin x - cos x) + C
29. If J x sin x dx = -x cos x + a , then a is equal to
ks
Yo

(a) sin x + C (b) cos x + C


oo

(c)C (d) none of these


cos 2x -1
30. J
eB

dx =
cos 2x +1
(a) tan x - x + C (b) x + tan x + C (c) x - tan x + C (d) - x - cot x 4- C
r

cos 2x - cos 20
ou
ad

31. J dx is equal to
cos x - cos 0
Y

(a) 2 (sin x 4- x cos 0) 4- C (b) 2 (sinx-xcos0) 4-C


nd

(c) 2 (sin x 4- 2x cos 0) 4- C (d) 2 (sin x - 2x cos 0) 4-C


Re

9
x
Fi

32. J dx is equal to
(4x2 +1)6
\-5 -5 V5 -5
1 + C (b) i 4 + T 1 1
(a) +C
5x
4+^
x X
+ c (C) ~
10xU
T+4
+C(<M7+4
33 . J * dx = (14- X2)3/2+b Vl+x2 + C, then
yjl+x2
1 1
(a)rt=3 ,b=l (b) a = —,b =1 (c) a = - --,b=-l (d) a = —, b — —1
3 3 3
r x3
34. J ---- - dx is qual to
2 3 2 .3
, . x-
(a) * + — + ^-log|l-x|4-C (b) x+y y-log|l-x|+C
2 r3 2 .3
i(c)\ x ~r log 114- x 14- C (d) + y-log|l4-x| + C

k
ReadYourFlow.COM
INDEFINITE INTEGRALS 19.203

1 2 -1 1
35. dx = a log 11 + x | + b tan ~ x + -log | x + 21 + C, then
(x + 2) (x2+l) 5

(a) a =
10'
- (b) a =—, b =
5 10
1 ,2
— ,b=— (d)
10 5 I {c)a=~
1,2
—,b =-
105
_____ANSWERS
1. (b) 2. (c) 3. (a) 4. (a) 5. (a) 6- (b) 7. (c) 8. (a) 9. (c)
10. (a) 11. (d) 12. (d) 13. (d) 14. (b) 15. (b) 16. (a) 17. (c) 18. (a)
19. (c) 20. (b) 21. (a) 22. (c) 23. (c) 24. (a) 25. (d) 26. (d) 27. (a)
28. (d) 29. (a) 3<>. (c) 31. (a) 32. (d) 33. (d) 34. (d) 35. (c)
REVISION EXERCISE
Evaluate the following integrals :
1 c i1 - x 4 , x+2
Vx + ^x + 1
dx 2. -------- dx
J 1 -x
3.J 3dx
(x + 1)

low
8x +13 1 + X + X2
i (2X + 3X)2
4-l dx
X2 (1 + X)
dx 6. j
6X
dx

4
sin x X + X f 2 2

ee
7. ------------ dx 9. I sec x cos 2x dx
1 + sin x x2 +1
rF
Fr
f 2 2 ! I. J sin4 2x dx 12. | cos3 3x dx
10. J cosec x cos 2x dx
sin 2x 1
for (sin x)
13. | II. dx 15. J dx
2 ,2 . 2
u
a +b sin x (sm 1 x) Jl-x2
ks
Yo

1 r ? •v -1 1
t7-!-x 18-H
oo

16. dx dx dx
ex +1 e +1 ■ ex + e 1
B

7
j COS X
re

19. dx : | sin x sin 2x sin 3x dx 21. | cos x cos 2x cos 3x dx


sin x
ou
ad

sin x + cos x sin x - cos x 1


22. J dx 23. j dx 24. j dx
Y

yjsm 2x Jsin 2x sin (x - a) sin (x - b)


sin x
25. j
1
dx 26. J dx 27J cossin 2xx dx
nd
Re

cos (x - a) cos (x - b) ^1 + sin x


Fi

28. J tan 3 x dx 29. | tan4 x dx 30. J tan5 x dx


2
31. | cot4 x dx 32. | cot5 x dx 33. f * , dx
J (*-D3
3
X .5 2 2 ,
34. J x ^2x + 3 dx 35. j dx 36. J x sin x cos x dx
(1 + x2)2
37. J sin 3 x cos4 x dx 38. J sin5 x dx 39. | cos5 x dx
sin 2x 42. J ■ , 1 — dx
40. J ^/sin x cos3 x dx
41J—4 i—
4 dx
sm x + cos x J 2 2
X - fl

43. f , 1 dx 1 1
44. j ? dx 45. J 2 dx
dx + a 4x + 4x + 5 x + 4x - 5
1 1 sm x
46. j 2 dx 47. J dx 48. J dx
1 - x - 4x 3x2 +13x-10 y cos2 x - 2 cos x - 3

ReadYourFlow.COMA
19.204 lEMATtCS-XII

1 x+1
J cosec x -1 dx dx ' I x2 + 4x + 5 dx
5x + 7
dx 5.1. J dx
p-5)(x-4)
Ja --Jx 1
1 -yfax
dx
1 (sin x - 2 cos x) (2 sin x + cos x) dx
1
57. dx
4 sin x + 4 sin x cos x + 5 cos“2 x
2
1 1 sin x + 2 cos x
58.
a + b tan x
dx 59 I
2
sin x + sin 2x
dx 60. J
2 sin x + cosx
dx

3
x 1 cos X
61.
1 dx 62. J
2-3 cos 2x
dx 63. It
2
dx

low
- COS X
4
64. J 1 65. J"
1 66. J 1
dx dx dx
1+2 cos x 1 - 2 sin x sin x (2 + 3 cos x)
1 69. J 1

ee
67. dx 7—
sin x + sin 2x 1 4 4_ dx 5 - 4 sin x
dx
rF
sin x + cos x

Fr
70. | sec4 x dx ~1. J cosec4 2x dx 1 + sin x
72. \ dx
for sin x (1 + cos x)
1 6x + 5
dx 74. J dx 75. J dx
u
2 + cos x y 6 + x - 2x2
ks
Yo

. 5 5 ■ 6
oo

sin x r COS X , sin x


76. J r:rfx 77. --------dx 78. J dx
B

COS X J sin x COS X


re

■ 2
sin x ^ 80. J sec6 x dx 81. J tan5 x sec3 x dx
79. J t— dx
b
ou
ad

COS X
1
84. | ■Ja2 + x2 dx
Y

82. | tan 3 x sec4 x dx 83. J dx


sec x + cosec x
nd
Re

85. J ,Jx2 -a2 dx 86. j i/«2 "*2 dx 87. J yj3.x2 + 4x + 1 dx


Fi

88. | yjl + 2x - 3x2 dx 89. + x -x2 dx 90. | (2x + 3) ^ 4x2 +5x + 6 dx


9 J (1 + x2) cos 2x dx log (log x) dx
l>2.{ log10xrfx 93. J
J X
C 2 95. J x sin 3 x dx 96. J (x + 1)2 ex dx
94. J x sec 2x dx
97. J log f x + J? + a2 dx
98. J Tsidx 99. J bgd-x)^
3 2
X X
2
100. J x3 (log x)2 dx 101. J 1 102. J
dx dx
X + x”
5 1 + x2
103. J X 104. J
dx dx 105. dx
3/1+*3
1 r sin x + cos x
108. J x2 tan 1 x dx
106. dx 107. j 7—
4 dx
X yll + X3 sin x + cos x

k ReadYourFlow.COM
INDEFINITE INTEGRALS 19.205

109. J tan 1 yfx dx 110. J sin 1 Jx dx 111. J sec -1 ■Jx dx


1-x 113. f sin 1 114. J sin-1 (3x-4x3) dx
112. dx dx
1 +x J ]ja + x
-1
x sin
115. I (sin 1 x)3 dx 116. J cos 1 (1 - 2x2) dx 117.
1 2,3/2
(1-X )
* j

2x fl + sin 2x^ ll^le-*ndx


118. I* 1 + cos 2x
dx 119. J
1 + COS X
120. J ex
(1 + x )
2x2
.X

m tan -i x 2
X
121. dx 122. J dx 123. f dx
(l + x2)3/2 (x-l)3(x + l) J x3-l
1 1
124. |
1 +X+X +X
2 3dx 125.
i ( 2 + 2)( 2+5) dx
x x
126.
X -x
dx

low
x2 + X + 1 sin 4x - 2 2x ,
127.
1 + -s/x
128. J
(* + l)2 (ar + 2)
dx 129.
I 1 - cos 4x e dx

3
{cotx + cot } X fa
130. J

ee
1 + cot3 X
rF
Fr
__ ANSWERS
2 3 4
2
{(*+i) 3/2 _xV2 i 1 1-2+C
+C 2. x h--------1-------- 1--------1- C
for 3. -
1. -
3 2 3 4 x+1 2 (x +1)2
u
1
ks

4. - (4x + 7)3/2 - - ^4x + 7 + C 5. - - + log | x + 11 + C


Yo

3 2 x
oo

\X
2 1 3 1
+ 2x + C
B

6. 7. x - tan x + sec x + C
3J log 2J
1o8(I)
re

3
ou
ad

x -1
8. 11-----tan ‘ x + C 9. sin 2x + tan x - 2x + C 10. - cot x - sin 2x - 2x + C
Y

3
3 sin 8x sin 4x sin 3x sin 3x 13. P log (a2 + b2 sin2 x) + C
11. — x + + C 12. +C
nd
Re

8 64 8 3 9 o
Fi

14. log | sin 1 x | + C 15. — (sin 1 x)4 + C 16. x - log {ex + 1) + C


4
17. 2 log (e* + 1) - x + C 18. tan-1 (ex) + C
sin6 x 3 sin2 x 3 sin4 x
19. log | sin x | - +C
6 2 4
cos 4 x cos 2x cos 6 x sin 6x sin 4x sin 2x
20. + +C 21. -
4+ 24 + 16 + 8
+C
16 8 24
22. sin 1 (sin x - cos x) + C 23. - log | (sin x + cos x) + ^sin 2x | + C

1 sin (x - a) 1 cos (x - a)
24. log +c 25. log +C
sin (fl -b) sin (x -fr) sin (fl -fr) cos (x - b)

26. 2 [ sin - - cos — - a/2 log tan — + — 1 + V2 cos x


+C
+C 27. —7= log r-
2 2) 6 U 8J 2V2 6 I-a/2 COS X

1 2 1 3
28. — tan x - log | sec x | + C 29. — tan x - tan x + x + C
2 3

ReadYourFlow.COM
r 19.206 MATHEMATICS-XII

30. i tan4 x - - tan2 x + log | sec x | + C 31. - — cot3 x + cot x + x + C


4 2 3
32. - ^ cot4 x + ~ cot2 x + log | sin x | + C 33. log | x -11 - ^ ^72 +C
x — 1 2 (x -1)

1 ; (2x+ 3)5/2 - —(2x + 3)3/2 +C 35. I logd + xV-^ + C


34.
10 2 2 1 +x2
5 7
36. sin6 x2 + C cos x cos x
37. +C
12 5 7
2 1 5 ~
38. - cos x + — cos 3 x — cos x + C 39. sin x + ^ sin5 x- — sin3 x + C
3 5 3
40. — sin3^2 x — 2 . 7/2 41. tan 1(tan2x)+C
sin x+C
3 7
42. log | x + Jx2 - a2 | + C 43. log | x + -Jx2 + a2 | + C

w
1 1> x —1
44. — tan -1 x + - + C 1
45. — log +C
4
1 ,
27
1 )+^ - x - 4x2 +C
Flo 6 x+5

3x - 2

ee
46. x log
2
^
8
47. ilog
17 3x +15
+C

Fr
48. - log (1 - cos x) + -Jcos2 x - 2 cos x - 3 + C for
ur
f
1^ / . ? _1 T x + 1N
49. log sin x + - + Jsin x + sin x + C 50. sin +C
ks

2/ 2
Yo
oo

51. ^ log | x2 + 4x + 51 + tan 1 (x + 2) + C


B

52. 6 -Jx2 -9x + 20 + 34 log J x - 9^1 + -Jx2 - 9x + 20 + C


re

2J
ou
ad

53. -Jx2 + x + -^ log + ^ + -Jx2 + x +C54. ^|x -x2 +^sin 1 (2x -1) + C
Y
nd
Re

2 j (a -1) log 11 - -Jax | + (2 - a) (1 - -/ax) 12(i-7^)2|+c


55. -
a -Ja
Fi

tan x - 2 1
56. - log +C 57. — tan -1 tan x + - + C
5 2 tan x + 1 4 2

a & 1 tan x
58. x+ log | a cos x + sin x | + C 59. — log +C
a2+b2 ~ a2+b2 2 tan x + 2
4 3 41 log x8 + yx8 + 4 + C
60. — x + — log | 2 sin x + cos x | + C 61. -
5 3
1 ^5 tan x -1 2 sin x - ^3
62. log +C 63. log +C
-J5 tan x + 1 y[3 2 sin x + -^3

y/3 + tan — tan - - 2-y[3


1 2 2
64. -=
23 8 V3-tan^=
log |------------ +c 65. -7^ log +C
V3 tan — - 2 + y[3
2 2

ReadYourFlow.COM
INDEFINITE INTEGRALS 19.207

113
66. — log| cos x -11 + — log | cos x + 11---- log| 3 cos x + 2\ +C
10 2 5
112
67. - log | cos a: -11 + - log | cos x + 11 — log | 2 cos x +11 + C
6 2 3

5 tan - - 4
1 -1 2 -i 2
68. -7= tan -X= tan 2x + C 69. - tan +C
s/2 V2 3 3

1 cot3 2x + C
70. tan x + — tan 3 x + C 71. - — cot 2x -
3 2 6
1 2 X X1 . , X „ 2 -1
72. - tan — +tan — + — log tan — + C 73. -7= tan tan - + C
4 2 22 2 V3 V3 2J
74. -Jx2 + ax +-^ log x + -^ + -Jx2 + ax + C 75 - 3 -Js + x -2x2 + _1 ( 4x -1
+C

low
7
2 1 77. - sin4 x - sin2 x + log | sin x | + C
76. - cos x - + 3 +C
cos X 3 cos x 4

ee
1.5 1.3 . ,1 , 1 + sin x
78. - - sin x---- sin x - sin x + — log
rF +C

Fr
5j 3 2 1 - sin x
2
79. — tan 3 x + - tan5 x + C 80. tan x h— tan 3 x + - tan5 x + C
for
3 5 3 5
u
13^
81. — sec7 x - — sec5 x + — sec x + C 82. — tan4 x + — tan6 x + C
ks

7 5 3 4 6
Yo
oo

1 1 .
83. — cos x + — sin x -
1 , f f -X* + -71 +C
^
B

2 2 8
re

1 x -v/fl2 + x2 + -^ tf2 log | x + -Jx2 + a2 \ +C


84. —
2
ou
ad

85. ^xjx2 - a 2 ^ fl2 log | x + -Jx2 - fl2| + C 86. x Ja2 - x 2 + —


1 2 . -1 ( X
a sin — +C
Y

2
87. ^ (3x + 2) -Jsx2 + 4x + 1 ~ log + |] + J 2 4x 1
nd
Re

x +—+ +C
3 3
Fi

88.
"T2
- 3x
2V3 .
+------- sin
-1 3x -1
+C
9 2
2 5 . -l^x-l)
89. — (8x2 - 2x - 11) Vi + x - x + — sin +C
24 16 \ V5
v /
1 (128x2 + 328x + 297) ^4x2 + 5x + 6 + 497 log fx + -1 + lx2 + - x + — +C
90.
192 256 8j V 4 2
1 2 x sin 2x
91. — (1 + x ) sin 2x + - cos 2x - +C 92. x (log x — 1) • log10 6 + C
2 2 4
93. log {log (log x)} - log x + C 94. ^ x tan 2x - ^ log | sec 2x | + C

1 f x sin 3x +C 96. ex (x2 + 1) + C


95. -M - 3x cos x + 3 sin x + —cos 3x -
4 1 3 9
97. x log | x + yjx2 + a 21 1
- V*2 + fl2 +C 98. - (2 log x + 1) + C
4x2

ReadYourFlow.COM
r
19.208 MATHEMATICS'XII

4
99. -f— log (1 - x) - log x + C 100. (log x)2 - log a: + ^ x4 + C

1 ^ yjl -x 3x2 + 4x 4- 8 j + C
101. - log +C 102. - —
n JT+x” +i
103. ^1 + x3(x3-2)+C 104. - sin-1 x--Jl-x2 + C
2 2 v

105. [--1 ,/l~? 1 -1


--sin x + C 106.
1
log
-1•Ji + *3
+C
i2 Jv 2 3 ^]l+x3 +1

107. J= 1
log
VVf+T 1
tan
-1 t
, where t = sin x - cos
72 2^/7171 772 + 1 -f 775^ TT^IJ

w
^2 1 2
108. i-tan-11 A:--j,+ -log|x2 + l|+C 109, (x + 1) tan 1 +C
3
HO* - ^ (1 - 2x)2 sin-1 Vx + i -yjx -x2 + C

Flo -1

ee
111- ^ sec yfx -^jx-1 + C

Fr
112. 1 -l x-Jl-x2 +C
x cos 113. (x + a) tan -1 - -slax +C
2 for a
ur
’ > 3 7 .r sin +C
ks

115. x sin -1 x | (sin 1 x)2 - oi + 3 j (sin 1 x)2


2}ii -x2 +C
Yo
oo

. -1
B

sin x 1 1 +x
116. 2 x sin -l x + Jl - x2 + C 117, +C
re

7i-7"208 1 -x
ou

x
ad

118. — e2x tan x + C 119. -e~x/2 sec -Uc


X 120. '-I*0
Y

2 ,2 1 +x
m tan -1 x
d
Re

-1 -1 x —1 3
121. _ tan * x - cot m } + C 122. I log
n

cos x+1 4 (x -1) 4(x-1)2+C


Fi

+1
1 1 2 1 -if 2x +1
123. -log|x-l|--log|x +x + l| + -^=tan +C
v V3
v y

1 1 -1 -1
124. _ log + — tan 1 x + C
2
125.
3 V2
tan
V2J 3 -y/5
tan 4^ + c

126. 2 log | x | - — log | x2 -11 - — log | x2 + 11 + C


4'* 4
-l
127. -2^1 - X + cos 4x + Jx 4- + 3 log | x + 21 + C
-x + C 128. - 2 log 11 + x I

129. i e2x cot 2x + C


2
1 2 1 1 -if 2 cot x -1 ^
130. - - log | cot X - cot X + 1| + — log | cot X + 11- —j= tan +C
6 3 73 73

ReadYourFlow.COM

You might also like